july 2014 mee questions and analyses - ncbe...this publication includes the questions and analyses...

44
July 2014 MEE Questions and Analyses National Conference of Bar Examiners 302 South Bedford Street | Madison, WI 53703-3622 Phone: 608-280-8550 | Fax: 608-280-8552 | TDD: 608-661-1275 www.ncbex.org e-mail: [email protected]

Upload: others

Post on 26-Feb-2020

14 views

Category:

Documents


0 download

TRANSCRIPT

Page 1: July 2014 MEE Questions and Analyses - NCBE...This publication includes the questions and analyses from the July 2014 MEE. (In the actual test, the questions are simply numbered rather

July 2014 MEE Questions and Analyses

National Conference of Bar Examiners 302 South Bedford Street | Madison WI 53703-3622 Phone 608-280-8550 | Fax 608-280-8552 | TDD 608-661-1275

wwwncbexorg e-mail contactncbexorg

Copyright copy 2014 by the National Conference of Bar Examiners All rights reserved

Contents

Prefaceii

Description of the MEE ii

Instructionsiii

July 2014 Q uestions

Criminal Law and Procedure Question 3

Contracts Question 5

Family Law Question 6

Federal Civil Procedure Question 7

Evidence Question 8

Corporations Question 9

July 2014 A nalyses

Criminal Law and Procedure Analysis 13

Contracts Analysis 16

Family Law Analysis 19

Federal Civil Procedure Analysis 22

Evidence Analysis 27

Corporations Analysis 32

i

Preface

The Multistate Essay Examination (MEE) is developed by the National Conference of Bar Examiners (NCBE) This publication includes the questions and analyses from the July 2014 MEE (In the actual test the questions are simply numbered rather than being identified by area of law) The instructions for the test appear on page iii

The model analyses for the MEE are illustrative of the discussions that might appear in excellent answers to the questions They are provided to the user jurisdictions to assist graders in grading the examination They address all the legal and factual issues the drafters intended to raise in the questions

The subjects covered by each question are listed on the first page of its accompanying analysis identified by roman numerals that refer to the MEE subject matter outline for that subject For example the Federal Civil Procedure question on the July 2014 MEE tested the following areas from the Federal Civil Procedure outline III Injunctions and provisional remedies and IVC Pretrial proceduresmdashJoinder of parties and claims (including class actions)

For more information about the MEE including subject matter outlines visit the NCBE website at wwwncbexorg

Description of the MEE

The MEE consists of six 30-minute questions and is a component of the Uniform Bar Examination (UBE) It is administered by user jurisdictions as part of the bar examination on the Tuesday before the last Wednesday in February and July of each year Areas of law that may be covered on the MEE include the following Business Associations (Agency and Partnership Corporations and Limited Liability Companies) Conflict of Laws Constitutional Law Contracts Criminal Law and Procedure Evidence Family Law Federal Civil Procedure Real Property Torts Trusts and Estates (Decedentsrsquo Estates Trusts and Future Interests) and Uniform Commercial Code (Negotiable Instruments and Bank Deposits and Collections Secured Transactions) (Effective with the February 2015 MEE administration Uniform Commercial Code Articles 3 and 4 [Negotiable Instruments and Bank Deposits and Collections] is no longer being tested) Some questions may include issues in more than one area of law The particular areas covered vary from exam to exam

The purpose of the MEE is to test the examineersquos ability to (1) identify legal issues raised by a hypothetical factual situation (2) separate material which is relevant from that which is not (3) present a reasoned analysis of the relevant issues in a clear concise and well-organized composition and (4) demonstrate an understanding of the fundamental legal principles relevant to the probable solution of the issues raised by the factual situation The primary distinction between the MEE and the Multistate Bar Examination (MBE) is that the MEE requires the examinee to demonstrate an ability to communicate effectively in writing

ii

Instructions The back cover of each test booklet contains the following instructions

You will be instructed when to begin and when to stop this test Do not break the seal on this booklet until you are told to begin

You may answer the questions in any order you wish Do not answer more than one question in each answer booklet If you make a mistake or wish to revise your answer simply draw a line through the material you wish to delete

If you are using a laptop computer to answer the questions your jurisdiction will provide you with specific instructions

Read each fact situation very carefully and do not assume facts that are not given in the question Do not assume that each question covers only a single area of the law some of the questions may cover more than one of the areas you are responsible for knowing

Demonstrate your ability to reason and analyze Each of your answers should show an understanding of the facts a recognition of the issues included a knowledge of the applicable principles of law and the reasoning by which you arrive at your conclusions The value of your answer depends not as much upon your conclusions as upon the presence and quality of the elements mentioned above

Clarity and conciseness are important but make your answer complete Do not volunteer irrelevant or immaterial information

Answer all questions according to generally accepted fundamental legal principles unless your testing jurisdiction has instructed you to answer according to local case or statutory law

NOTE Examinees testing in UBE jurisdictions must answer according to generally accepted fundamental legal principles rather than local case or statutory law

iii

July 2014 MEE

QUESTIONS

Contracts Criminal Law and Procedure

Family L aw Federal Civil Procedure

Evidence Corporations

CRIMINAL LAW AND PROCEDURE QUESTION

While on routine patrol a police officer observed a suspect driving erratically and pulled the suspectrsquos car over to investigate When he approached the suspectrsquos car the officer detected a strong odor of marijuana The officer immediately arrested the suspect for driving under the influence of an intoxicant (DUI) While the officer was standing near the suspectrsquos car placing handcuffs on the suspect the officer observed burglary tools on the backseat

The officer seized the burglary tools He then took the suspect to the county jail booked him for the DUI and placed him in a holding cell Later that day the officer gave the tools he had found in the suspectrsquos car to a detective who was investigating a number of recent burglaries in the neighborhood where the suspect had been arrested

At the time of his DUI arrest the suspect had a six-month-old aggravated assault charge pending against him and was being represented on the assault charge by a lawyer

Early the next morning upon learning of her clientrsquos arrest the lawyer went to the jail She arrived at 900 am immediately identified herself to the jailer as the suspectrsquos attorney and demanded to speak with the suspect The lawyer also told the jailer that she did not want the suspect questioned unless she was present The jailer told the lawyer that she would need to wait one hour to see the suspect After speaking with the lawyer the jailer did not inform anyone of the lawyerrsquos presence or her demands

The detective who had also arrived at the jail at 900 am overheard the lawyerrsquos conversation with the jailer The detective then entered the windowless interview room in the jail where the suspect had been taken 30 minutes earlier Without informing the suspect of the lawyerrsquos presence or her demands the detective read to the suspect full and accurate Miranda warnings The detective then informed the suspect that he wanted to ask about the burglary tools found in his car and the recent burglaries in the neighborhood where he had been arrested The suspect replied ldquoI think I want my lawyer here before I talk to yourdquo The detective responded ldquoThatrsquos up to yourdquo

After a few minutes of silence the suspect said ldquoWell unless there is anything else I need to know letrsquos not waste any time waiting for someone to call my attorney and having her drive here I probably should keep my mouth shut but Irsquom willing to talk to you for a whilerdquo The suspect then signed a Miranda waiver form and after interrogation by the detective made incriminating statements regarding five burglaries The interview lasted from 915 am to 1000 am

In addition to the DUI the suspect has been charged with five counts of burglary

The lawyer has filed a motion to suppress all statements made by the suspect to the detective in connection with the five burglaries

The state supreme court follows federal constitutional principles in all cases interpreting a criminal defendantrsquos rights

3

Criminal Law and Procedure Question

1 Did the detective violate the suspectrsquos Sixth Amendment right to counsel when he questioned the suspect in the absence of the lawyer Explain

2 Under Miranda did the suspect effectively invoke his right to counsel Explain

3 Was the suspectrsquos waiver of his Miranda rights valid Explain

4

CONTRACTS QUESTION

A music conservatory has two concert halls One concert hall had a pipe organ that was in poor repair and the other had no organ The conservatory decided to repair the existing organ and buy a new organ for the other concert hall After some negotiation the conservatory entered into two contracts with a business that both repairs and sells organs Under one contract the business agreed to repair the existing pipe organ for the conservatory for $100000 The business would usually charge a higher price for a project of this magnitude but the business agreed to this price because the conservatory agreed to prepay the entire amount Under the other contract the business agreed to sell a new organ to the conservatory for the other concert hall for $225000 As with the repair contract the business agreed to a low sales price because the conservatory agreed to prepay the entire amount Both contracts were signed on January 3 and the conservatory paid the business a total of $325000 that day

Two weeks later before the business had commenced repair of the existing organ the business suffered serious and unanticipated financial reversals The chief financial officer for the business contacted the conservatory and said

Bad news We had an unexpected liability and as a result are in a real cash crunch In fact even though we havenrsquot acquired the new organ from our supplier or started repair of your existing organ wersquove already spent the cash you gave us and we have no free cash on hand Wersquore really sorry but wersquore in a fix I think that we can find a way to perform both contracts but not at the original prices If you agree to pay $60000 more for the repair and $40000 more for the new organ we can probably find financing to finish everything If you donrsquot agree to pay us the extra money I doubt that we will ever be able to perform either contract and yoursquoll be out the money you already paid us

After receiving this unwelcome news the conservatory agreed to pay the extra amounts provided that the extra amount on each contract would be paid only upon completion of the businessrsquos obligations under that contract The business agreed to this arrangement and the parties quickly signed documents reflecting these changes to each contract The business then repaired the existing organ delivered the new organ and demanded payment of the additional $100000

The conservatory now has refused to pay the business the additional amounts for the repair and the new organ

1 Must the conservatory pay the additional $60000 for the organ repair Explain

2 Must the conservatory pay the additional $40000 for the new organ Explain

5

FAMILY LAW QUESTION

In 1994 a man and a woman were married in State A

In 1998 their daughter was born in State A

In 2010 the family moved to State B

In 2012 the husband and wife divorced in State B Under the terms of the divorce decree

(a) the husband and wife share legal and physical custody of their daughter (b) the husband must pay the wife $1000 per month in child support until their daughter reaches age 18 (c) the marital residence was awarded to the wife with the proviso that if it is sold before the daughter reaches age 18 the husband will receive 25 of the net sale proceeds remaining after satisfaction of the mortgage on the residence and (d) the remaining marital assets were divided between the husband and the wife equally

Six months ago the husband was offered a job in State A that pays significantly less than his job in State B but provides him with more responsibilities and much better promotion opportunities The husband accepted the job in State A and moved from State B back to State A

Since returning to State A the husband has not paid child support because due to his lower salary he has had insufficient funds to meet all his obligations

One month ago the wife sold the marital home netting $10000 after paying off the mortgage She then moved to a smaller residence The husband believes that he should receive more than 25 of the net sale proceeds given his financial difficulties

Last week when the wife brought the daughter to the husbandrsquos State A home for a weekend visit the husband served the wife with a summons in a State A action to modify the support and marital-residence-sale-proceeds provisions of the State B divorce decree The husband brought the action in the State A court that adjudicates all domestic relations issues

1 Does the State A court have jurisdiction to modify (a) the child support provision of the State B divorce decree Explain (b) the marital-residence-sale-proceeds provision of the State B divorce decree

Explain

2 On the merits could the husband obtain (a) retroactive modification of his child support obligation to the daughter Explain (b) prospective modification of his child support obligation to the daughter Explain (c) modification of the marital-residence-sale-proceeds provision of the State B

divorce decree Explain

6

FEDERAL CIVIL PROCEDURE QUESTION

The United States Forest Service (USFS) manages public lands in national forests including the Scenic National Forest Without conducting an environmental evaluation or preparing an environmental impact statement the USFS approved a development project in the Scenic National Forest that required the clearing of 5000 acres of old-growth forest The trees in the forest are hundreds of years old and the forest is home to a higher concentration of wildlife than can be found anywhere else in the western United States

The USFS solicited bids from logging companies to harvest the trees on the 5000 acres of forest targeted for clearing and it ultimately awarded the logging contract to the company that had submitted the highest bid for the trees However the USFS has not yet issued the company a logging permit Once it does so the company intends to begin cutting down trees immediately

A nonprofit organization whose mission is the preservation of natural resources has filed suit in federal district court against the USFS The nonprofit alleges that the USFS violated the National Environmental Policy Act (NEPA) by failing to prepare an environmental impact statement for the proposed logging project Among other remedies the nonprofit seeks a permanent injunction barring the USFS from issuing a logging permit to the logging company until an adequate environmental impact statement is completed The nonprofit believes that the logging project would destroy important wildlife habitat and thereby cause serious harm to wildlife in the Scenic National Forest including some endangered species

Assume that federal subject-matter jurisdiction is available that the nonprofit has standing to bring this action and that venue is proper

1 If the logging company seeks to join the litigation as a party must the federal district court allow it to do so as a matter of right Explain

2 What types of relief could the nonprofit seek to stop the USFS from issuing a logging permit during the pendency of the action what must the nonprofit demonstrate to obtain that relief and is the federal district court likely to grant that relief Explain

7

EVIDENCE QUESTION

A prison inmate has filed a civil rights lawsuit against a guard at the prison alleging that the guard violated the inmatersquos constitutional rights during an altercation The inmate and the guard are the only witnesses to this altercation They have provided contradictory reports about what occurred

The trial will be before a jury The inmate plans to testify at trial The guardrsquos counsel has moved for leave to impeach the inmate with the following

(a) Twelve years ago the inmate was convicted of felony distribution of marijuana He served a three-year prison sentence which began immediately after he was convicted He served his full sentence and was released from prison nine years ago (b) Eight years ago the inmate pleaded guilty to perjury a misdemeanor punishable by up to one year in jail He paid a $5000 fine (c) Seven years ago the inmate was convicted of felony sexual assault of a child and is currently serving a 10-year prison sentence for the crime The victim was the inmatersquos daughter who was 13 years old at the time of the assault

The inmatersquos counsel objects to the admission of any evidence related to these three convictions and to any cross-examination based on this evidence

The guard also plans to testify at trial The inmatersquos counsel has moved for leave to impeach the guard with the following

Last year the guard applied for a promotion to prison supervisor The guard submitted a reacutesumeacute to the state that indicated that he had been awarded a BA in Criminal Justice from a local college An official copy of the guardrsquos academic transcript from that college indicates that the guard dropped out after his first semester and did not receive a degree

The guardrsquos counsel objects to the admission of this evidence and to any cross-examination based on this evidence

The transcript and the reacutesumeacute have been properly authenticated The trial will be held in a jurisdiction that has adopted all of the Federal Rules of Evidence

1 What evidence if any proffered by the guard to impeach the inmate should be admitted Explain

2 What evidence if any proffered by the inmate to impeach the guard should be admitted Explain

8

CORPORATIONS QUESTION

Mega Inc is a publicly traded corporation incorporated in a state whose corporate statute is modeled on the Model Business Corporation Act (MBCA) Megarsquos articles of incorporation do not address the election of directors or amendment of the bylaws by shareholders

Well within the deadline for the submission of shareholder proposals for the upcoming annual shareholdersrsquo meeting an investor who was a large and long-standing shareholder of Mega submitted a proposed amendment to Megarsquos bylaws The proposal which the investor asked to be included in the corporationrsquos proxy materials and voted on at the upcoming shareholdersrsquo meeting read as follows

Section 20 The Corporation shall include in its proxy materials (including the proxy ballot) for a shareholdersrsquo meeting at which directors are to be elected the name of a person nominated for election to the Board of Directors by a shareholder or group of shareholders that beneficially have owned 3 or more of the Corporationrsquos outstanding common stock for at least one year

This Section shall supersede any inconsistent provision in these Bylaws and may not be amended or repealed by the Board of Directors without shareholder approval

Megarsquos management decided to exclude the investorrsquos proposal from the corporationrsquos proxy materials and explained its reasons in a letter to the investor

The investorrsquos proposed bylaw provision would be inconsistent with relevant state law because the Board of Directors has the authority to manage the business and affairs of the Corporation Generally shareholders lack the authority to interfere with corporate management by seeking to create a method for the nomination and election of directors inconsistent with the method chosen by the Board of Directors

Furthermore at its most recent meeting the Board of Directors unanimously approved an amendment to the Corporationrsquos bylaws that provides for proxy access for director nominations by a shareholder or a group of shareholders holding at least 10 of the Corporationrsquos voting shares for at least three years This procedure takes precedence over any nomination methods that might be sought or approved by shareholders

The investor is considering bringing a suit challenging managementrsquos refusal to include the investorrsquos proposed bylaw provision and challenging the boardrsquos amendment of the bylaws at its recent meeting

1 Is the investorrsquos proposed bylaw provision inconsistent with state law Explain

2 If the investorrsquos proposed bylaw provision were approved by the shareholders would the bylaw amendment previously approved by the board take precedence over the investorrsquos proposed bylaw provision Explain

3 Must the investor make a demand on Megarsquos board of directors before bringing suit Explain

9

July 2014 MEE

ANALYSES

Contracts Family Law

Criminal Law and Procedure

Federal Civil Procedure Evidence

Corporations

CRIMINAL LAW AND PROCEDURE ANALYSIS (Criminal Law and Procedure VA B D)

ANALYSIS

Legal Problems

(1) Did the detective violate the suspectrsquos Sixth Amendment right to counsel when he questioned the suspect about the burglaries without the lawyer present given that the lawyer represented the suspect in an unrelated criminal matter

(2) Under Miranda did the suspect effectively invoke his right to counsel when he said ldquoI think I want my lawyer here before I talk to yourdquo

(3) Was the suspectrsquos waiver of his right to remain silent under Miranda valid

DISCUSSION

Summary

The Sixth Amendment right to counsel as applied to states through the Fourteenth Amendment is offense-specific Although the suspect had an attorney representing him on his pending assault charge he had no Sixth Amendment right to the assistance of counsel with respect to the five uncharged burglaries because formal adversarial proceedings had not yet commenced on those charges The suspectrsquos Sixth Amendment right to counsel was not violated by the detectiversquos failure to inform him that the lawyer was present or of the lawyerrsquos demands

However a person undergoing custodial interrogation also has an independent constitutional right to counsel during custodial interrogation under Miranda When a suspect invokes his right to counsel under Miranda custodial interrogation must immediately cease for a period of at least 14 days However the invocation of the right to counsel must be unambiguous and clearly convey that the suspect has requested counsel Here because the suspectrsquos statement ldquoI think I want my lawyer here before I talk to yourdquo was ambiguous he did not invoke his Miranda right to counsel

A waiver of rights must be knowing intelligent and voluntary Here the suspect waived his right to remain silent under Miranda when he signed the waiver form The fact that the detective did not correct the suspectrsquos assumption that the lawyer would need to drive to the jailmdashby telling him that the lawyer was in the waiting room and was demanding to see himmdashdid not affect the validity of the suspectrsquos waiver

Point One (35) The suspectrsquos Sixth Amendment right to counsel was not violated because the right does not attach on new charges until formal adversarial judicial proceedings have commenced on those charges

The Sixth Amendment as applied to the states through the Fourteenth Amendment provides that ldquo[i]n all criminal prosecutions the accused shall enjoy the right to have the Assistance of Counsel for his defenserdquo The right to counsel does not attach with respect to particular charges until formal adversarial judicial proceedings have commenced (ie ldquoat or after the initiation of

13

Criminal Law and Procedure Analysis

adversary judicial criminal proceedingsmdashwhether by way of formal charge preliminary hearing indictment information or arraignment [or in some states arrest warrant]rdquo McNeil v Wisconsin 501 US 171 175 (1991) (internal quotations omitted)) Once a suspectrsquos Sixth Amendment right to counsel has attached any attempts to ldquodeliberately elicitrdquo statements from him in the absence of his attorney violate the Sixth Amendment See Massiah v United States 377 US 201 (1964) Brewer v Williams 430 US 387 (1977)

The Sixth Amendment right to counsel is charge- or offense -specific Representation by counsel in one prosecution does not in itself guarantee counsel for uncharged offenses See McNeil 501 US at 175 Texas v Cobb 532 US 162 (2001) Here the suspectrsquos Sixth Amendment right to counsel had attached only for the pending aggravated assault charge The suspectrsquos right to counsel for the aggravated assault case did not guarantee counsel for the five unrelated and uncharged burglaries that were the subject of the detectiversquos interrogation Thus because formal adversarial judicial proceedings against the suspect for the uncharged burglaries had not begun he had no Sixth Amendment right to counsel

Finally the detectiversquos failure to inform the suspect of the lawyerrsquos presence and demands to speak with him does not implicate the suspectrsquos Sixth Amendment right to counsel which had not yet attached See id Moran v Burbine 475 US 412 428ndash31 (1986)

Point Two (30) The suspect did not effectively invoke his right to counsel under Miranda because his statement was not unambiguous

A suspect subject to custodial interrogation has a right to consult with counsel and to have an attorney present during questioning Miranda v Arizona 384 US 436 (1966) When a suspect invokes his right to counsel during an interrogation law enforcement must immediately cease all questioning See Edwards v Arizona 451 US 477 484ndash85 (1981) Custodial interrogation cannot be reinitiated unless and until the suspect has been re-advised of his Miranda rights has provided a knowing and voluntary waiver and (1) counsel is present and (2) the suspect himself initiated further communication with the police see id at 484 or (3) (if the suspect was released from custody after the initial interrogation) at least 14 days have passed Maryland v Shatzer 559 US 98 110 (2010)

To invoke the right to counsel a suspectrsquos request must be ldquounambiguousrdquo This means that the suspect must articulate the desire for counsel sufficiently clearly that a reasonable officer would understand the statement to be a request for counsel Davis v United States 512 US 452 459 (1994) If the request is ambiguous the police are not required to stop the interrogation

In this case the suspectrsquos statement ldquoI think I want my lawyer here before I talk to yourdquo was not an unambiguous request for counsel The most reasonable interpretation of this statement is that the suspect might be invoking his right to counsel Id at 461 (ldquomaybe I should talk to a lawyerrdquo is not an unequivocal request for counsel) See also Burket v Angelone 208 F3d 172 197ndash98 (4th Cir 2000) (ldquoI think I need a lawyerrdquo is not an unambiguous request for an attorney) Soffar v Cockrell 300 F3d 588 594ndash95 (5th Cir 2002) (discussion of various statements that did not constitute unequivocal requests for counsel)

Under these circumstances the detective was not required to cease the custodial interrogation of the suspect Nor was the detective required to clarify or ask follow-up questions to determine whether the suspect in fact wanted an attorney Davis 512 US at 459ndash60

14

Criminal Law and Procedure Analysis

Point Three (35) The suspectrsquos waiver of his Miranda rights was knowing intelligent and voluntary despite the fact that he was never told of the lawyerrsquos presence in the jail or of the lawyerrsquos demands

A valid waiver of Miranda rights must be ldquovoluntaryrdquomdashie the product of a free or deliberate choice rather than intimidation coercion or deception Berghuis v Thompkins 560 US 370 382ndash83 (2010) In addition the waiver must be knowing and intelligent That is it ldquomust have been made with a full awareness of both the nature of the right being abandoned and the consequences of the decision to abandon itrdquo Moran v Burbine 475 US 421 (1986)

In this case the suspect signed a Miranda waiver form after receiving proper warnings There is no evidence ldquothat the police resorted to physical or psychological pressure to elicit the statementsrdquo Id The entire interview lasted only 45 minutes The only issue is whether the suspect knowingly and intelligently waived his Miranda rights despite the fact that the detective did not tell the suspect about the lawyerrsquos presence and her demands

The Supreme Court has said that ldquo[e]vents occurring outside of the presence of the suspect and entirely unknown to him surely can have no bearing on the capacity to comprehend and knowingly relinquish a constitutional rightrdquo Id at 422 If the suspect ldquoknew that he could stand mute and request a lawyer and was aware of the Statersquos intention to use his statements to secure a convictionrdquo then the waiver is valid regardless of the information withheld Id at 422ndash23

Here the suspect was correctly informed of his rights Miranda v Arizona 384 US at 467ndash73 His comments demonstrate that he understood that he could have a lawyer present if he desired (ie wondering whether he should call his attorney) and that he understood that there might be consequences to speaking with the detective (ldquoI probably should keep my mouth shut but Irsquom willing to talk to you for a whilerdquo) His comment ldquo[L]etrsquos not waste any time waiting for someone to call my attorney and having her drive hererdquo along with his signature on the Miranda waiver form show that his waiver was valid under the constitutional standard

The fact that the detective did not tell the suspect about the lawyerrsquos presence and demands has no bearing on the validity of the suspectrsquos waiver because ldquosuch conduct is only relevant to the constitutional validity of a waiver if it deprives a defendant of knowledge essential to his ability to understand the nature of his rights and the consequences of abandoning themrdquo Moran at 424 The Supreme Court has specifically declined to adopt a rule requiring that law enforcement tell a suspect of an attorneyrsquos efforts to contact him id at 425 (ldquoNor are we prepared to adopt a rule requiring that the police inform a suspect of an attorneyrsquos efforts to reach himrdquo)

[NOTE An examinee might also recognize that this general rule is further supported by the Supreme Courtrsquos decision in Florida v Powell 559 US 50 (2010) approving state Miranda warnings that do not explicitly warn suspects that they have a right to have counsel present during custodial interrogation]

15

CONTRACTS ANALYSIS (Contracts IB2 IIB IVA3 amp A5)

ANALYSIS

Legal Problems

(1) In the case of a service contract (governed by the common law of contracts) is a modification enforceable when a party agrees to pay more for the same performance than was originally promised

(2) In the case of a contract for the sale of goods (governed by Article 2 of the UCC) is a modification enforceable when a party agrees to pay more for the same goods than was originally promised

(3) May a party avoid an agreement on the basis of economic duress

DISCUSSION

Summary

There are two arguments that the conservatory can make to support the claim that it is not bound to pay the higher prices lack of consideration and economic duress

The organ repair contract is governed by the common law of contracts Under the common law the business would have difficulty recovering the additional $60000 for the organ repair because under the ldquopreexisting duty rulerdquo the agreement of the conservatory to pay the extra price was not supported by consideration However the business might argue that the modification is enforceable under an exception to the preexisting duty rule for fair and equitable modifications made in light of unanticipated circumstances

The organ sale contract is governed by Article 2 of the Uniform Commercial Code The business would likely recover the additional amount under that contract because Article 2 provides that consideration is not required for a modification to be binding

In both cases the conservatory could seek to avoid its agreement on the grounds of economic duress but that argument is not likely to succeed

Point One (45) The business probably cannot recover the additional $60000 for the organ repair because the conservatoryrsquos promise to pay more money was not supported by consideration

The general rule is that to be enforceable a promise must be supported by consideration Under RESTATEMENT (SECOND) OF CONTRACTS sect 71 a promise is supported by consideration if it is bargained for in exchange for a return promise or performance However under the ldquopreexisting duty rulerdquo (exemplified in RESTATEMENT (SECOND) OF CONTRACTS sect 73 and Alaska Packersrsquo Assrsquon v Domenico 117 F 99 (9th Cir 1902)) promise of performance of a legal duty already owed to a promisor which is neither doubtful nor the subject of honest dispute is not consideration

If the business had promised the conservatory anything new or different in exchange for the agreement to pay the additional $60000 (such as for example repairing the pipe organ more

16

Contracts Analysis

quickly or using better parts) that would constitute consideration especially in light of the principle that courts do not inquire into the adequacy of consideration Here however the business already had a legal duty under the original contract and did not agree to do anything else in exchange for the conservatoryrsquos promise to pay $60000 more

However an exception to the preexisting duty rule is sometimes applied in situations of unanticipated changed circumstances Under RESTATEMENT (SECOND) OF CONTRACTS sect 89 followed in many jurisdictions a promise modifying a duty under a contract not fully performed on either side is binding even if not supported by consideration if the modification is fair and equitable in view of circumstances not anticipated by the parties when the contract was made

If a court applies the rule in Restatement sect 89 the critical issues will be whether the modification was in fact ldquofair and equitablerdquo and whether it can be justified in light of unanticipated circumstances In many cases in which modifications have been upheld a party encountered difficulties or burdens in performing far beyond what was knowingly bargained for in the original contract with the result bordering on impracticability such as having to excavate solid rock instead of soft dirt or having to remove garbage far in excess of the amounts contemplated The conservatory would argue that the businessrsquos performance difficulties were not of this sort at allmdashnothing about repairing the pipe organ itself was any different from or more difficult than originally contemplated except that the business itself encountered financial distress unrelated to its burdens in performing its obligations under these contracts

Even if the business satisfies that element of the rule in Restatement sect 89 the business must also demonstrate that the circumstances that gave rise to the need to modify the contract were ldquounanticipatedrdquo at the time the original contract was made Here the facts suggest that when the business entered into the original contract it expected that the price paid by the conservatory would enable it to perform However any evidence that the business knew or had reason to know at the time of execution that it would need more money from the conservatory to be able to perform would mean that the request to modify was not ldquounanticipatedrdquo

[NOTE Some cases such as Schwartzreich v Bauman-Basch Inc 231 NY 196 131 NE 887 (1921) find that if the parties mutually agreed to rescind the original contract and then after rescission entered into an entirely new contract for a higher price the new contract is supported by consideration There is no evidence that such a rescission followed by a new contract took place here]

Point Two (45) The business can recover the additional $40000 for the new organ because no consideration is required under Article 2 of the UCC for good-faith contract modifications

The contract to buy a new organ is a contract for the sale of goods and therefore is governed by Article 2 of the Uniform Commercial Code UCC sect 2-102 Under Article 2 unlike the common law an agreement modifying a contract needs no consideration to be binding UCC sect 2-209(1) Section 2-209(1) thus obviates the preexisting duty rule entirely in contracts for the sale of goods

Even though consideration is not required modifications governed by sect 2-209 must satisfy the obligation of good faith imposed by the UCC UCC sect 1-304 See also Official Comment 2 to UCC sect 2-209 Good faith means ldquohonesty in fact and the observance of reasonable commercial standards of fair dealingrdquo UCC sect 1-201(b)(20) In this context the obligation of good faith means that ldquo[t]he effective use of bad faith to escape performance on the original contract terms is barred and the extortion of a lsquomodificationrsquo without legitimate commercial reason is ineffective as a violation of the duty of good faithrdquo Official Comment 2 to

17

Contracts Analysis

UCC sect 2-209 Here because the businessrsquos financial reversals were serious and apparently unanticipated at the time that the business entered into the contract with the conservatory and commitment of the extra money was needed to enable the business to perform a court would likely find that the business acted in good faith Thus a court would likely uphold the enforceability of the conservatoryrsquos promise to pay the additional $40000

Point Three (10) The conservatory is unlikely to be able to defend against enforcement of its promises to pay additional money under the theory of economic duress because the business probably did not make an improper threat

Under the common law of contracts parties may raise the defense of duress This common law defense also applies to contracts governed by UCC Article 2 See UCC sect 1-103(b)

A contract is voidable on the ground of economic duress by threat when it is established that a partyrsquos manifestation of assent is induced by an improper threat that leaves the party no reasonable alternative See RESTATEMENT (SECOND) OF CONTRACTS sect 175 See also eg Austin Instrument Inc v Loral Corp 272 NE2d 533 (NY 1971) (a threat to withhold essential goods can constitute duress) In order to void its agreement to pay the additional sum because of economic duress the conservatory must demonstrate that (1) the business made a threat to the conservatory (2) the threat was ldquoimproperrdquo or ldquowrongfulrdquo (3) the threat induced the conservatoryrsquos manifestation of assent to the modification and (4) the threat was sufficiently grave to justify the conservatoryrsquos assent

Here it appears that three of the four elements are likely satisfied The business plainly made a threat Moreover the threat induced the conservatoryrsquos assent to the modification and the threat was sufficiently grave to justify that assent If the conservatory had not agreed to pay the business the extra amounts the conservatory would have lost its entire $325000 investment In light of this potential loss a court could easily conclude that the conservatory had no reasonable alternative

However the business has a strong argument that its threat (indicating that it would breach the contracts unless the prices were increased) was not wrongful or improper but was instead nothing more than a communication of the reality of its own perilous situation to the conservatory

A mere threat to breach a contract is not in and of itself improper so as to support an action of economic duress or business compulsion Something more is required such as a breach of the duty of good faith and fair dealing as was present in Austin Instrument Inc supra Because the business could not perform the original contract without the requested modification the economic duress claim for the conservatory would likely fail for much the same reason that the business would be able to enforce the modification At the time the modification was requested the business was not trying to extort a price increase because of the conservatoryrsquos vulnerability but instead was simply stating the reality that the business could not perform without more money

18

FAMILY LAW ANALYSIS (Family Law IIIB D amp G)

ANALYSIS

Legal Problems

(1)(a) Does the State A court have jurisdiction to modify the State B child support order

(1)(b) Does the State A court have jurisdiction to modify the marital-residence-saleshyproceeds provision of the State B property-division decree

(2)(a) May a child support order be modified retroactively

(2)(b) May a child support order be modified prospectively based on a change of employment with a lower salary

(2)(c) May a property-division order be modified after entry of a divorce decree

DISCUSSION

Summary

The State A court may exercise personal jurisdiction over the wife because she was personally served in State A However subject-matter jurisdiction over the interstate modification of child support is governed by the Uniform Interstate Family Support Act (UIFSA) Under UIFSA State A does not have jurisdiction to modify the order for the daughterrsquos support because the wife is still a resident of State B UIFSA on the other hand does not govern property distributions and thus a State A court is not precluded from hearing the husbandrsquos petition to modify the marital-residence-sale-proceeds provision of the divorce decree

A child support order may not be modified retroactively A child support order may be modified prospectively based on a substantial change in circumstances Courts agree that a significant decrease in income is a substantial change in circumstances All states treat voluntary income reductions differently than involuntary reductions but employ different approaches for evaluating the impact of a voluntary reduction Whether the husband could obtain prospective modification of the child support order depends on which approach is applied

A property-division order is not subject to post-divorce modification based on a change in circumstances Thus the husband may in some states obtain prospective modification of the order for the daughterrsquos support but he may not obtain modification of the marital-residenceshysale-proceeds provision

Point One(a) (25) Personal jurisdiction over a nonresident respondent does not confer subject-matter jurisdiction over child support modification Under UIFSA a State A court may not modify a child support order issued by a State B court when as here the child or either parent continues to reside in State B the jurisdiction that issued the child support order

The State A court may exercise personal jurisdiction over the wife The wife was personally served in State A and a state may exercise jurisdiction based on in-state personal service See

19

Family Law Analysis

Burnham v Superior Court 495 US 604 (1990) But personal jurisdiction over the wife is not enough to give a State A court jurisdiction to modify the State B support order

The interstate enforcement and modification of child support is governed by the Uniform Interstate Family Support Act (UIFSA) which has been adopted by all states Under UIFSA the state that originally issued a child support order (here State B) has continuing exclusive jurisdiction to modify the order if that state remains the residence of the obligee the child or the obligor and all parties do not consent to the jurisdiction of another forum See UIFSA sect 205 See also UIFSA sect 603 (ldquoA tribunal of this State shall recognize and enforce but may not modify a registered order if the issuing tribunal had jurisdictionrdquo) The wife and daughter continue to reside in State B and the wife has not consented to the jurisdiction of another forum Thus a State A court does not have jurisdiction to modify the State B child support order

[NOTE Examinees who do not discuss personal jurisdiction but fully discuss UIFSA may receive full credit]

Point One(b) (15) UIFSA does not apply to disputes over property division Thus the State A court may exercise jurisdiction over the husbandrsquos petition to modify the marital-residence-sale-proceeds provision of the State B divorce decree because it has personal jurisdiction over the wife

The State A court in which the husband brought his action has jurisdiction to adjudicate domestic relations issues The husbandrsquos petition to modify the property settlement is a domestic relations issue The courts of State A may exercise personal jurisdiction over the wife because she was personally served in State A See Burnham v Superior Court 495 US 604 (1990) see Point One(a)

UIFSA does not apply to divorce property-division disputes Thus although a State A court may not adjudicate the husbandrsquos petition to modify his child support obligations it may adjudicate his property-division claims (Even though the court has jurisdiction it may not modify the property-division award on the merits See Point Two(c))

Point Two(a) (20) A child support order may not be modified retroactively

State courts have long held that obligations to pay child support ordinarily may not be modified retroactively ldquoIf the hardship is particularly severe the courts sometimes devised a way to protect the obligor but in most instances the courts hold that retroactive modification of this kind is beyond their power and indeed the governing statute may so providerdquo HOMER H CLARK THE LAW OF DOMESTIC RELATIONSHIPS IN THE UNITED STATES 725 (2d ed 1987)

Federal law now goes further and requires the states as a condition of federal child-support funding to adopt rules that absolutely forbid retroactive modification of the support obligation See 42 USC sect 666(a)(9)(C) The states have adopted rules consistent with the federal requirements

Point Two(b) (25) It is unclear whether the husband could obtain prospective downward modification of his child support based on his voluntary acceptance of a job with a lower salary

Prospective modification of a child support order is typically available only when the petitioner can show a substantial change in circumstances See ROBERT E OLIPHANT amp NANCY VER

20

Family Law Analysis

STEEGH FAMILY LAW 213ndash15 (3d ed 2010) A significant decrease in income is typically viewed as a substantial change

However when a parent seeks to modify a child support obligation because he has voluntarily reduced his income a court will not modify the obligation based solely on the income loss Some courts refuse to modify whenever the income shift was voluntary See eg Aguiar v Aguiar 127 P3d 234 (Idaho Ct App 2005) Others look primarily to the petitionerrsquos intentions and permit downward modification if he has acted in good faith See eg In re Marriage of Horn 650 NE2d 1103 (Ill App Ct 1995) Many courts use a multifactor approach See OLIPHANT amp VER STEEGH supra 217ndash18

Here there is no question that the husbandrsquos loss of income was voluntary In a jurisdiction in which voluntary income reduction bars support modification the husbandrsquos petition would be denied

In a jurisdiction employing a good-faith or multifactor approach it is possible but not certain that the husband could obtain downward modification The evidence supports the husbandrsquos good faith his change in employment appears to be based on his new jobrsquos greater responsibilities and better promotion possibilities In a jurisdiction using a multifactor approach the court would likely also consider the impact of such a shift on the daughter the likely duration of the husbandrsquos income loss and the likelihood of a promotion that would ultimately inure to the daughterrsquos benefit Thus on these facts it is possible but by no means certain that the husband could prospectively obtain downward modification of his child support obligation to his daughter

Point Two(c)(15) A divorce property-division award is not subject to modification

A support order is aimed at meeting the post-divorce needs of the supported individual Because the future is unpredictable courts are empowered to modify a support award to take account of changed circumstances that may occur during the period in which support is paid

By contrast a property-distribution award divides assets of the marriage based on the equities at the time of divorce Because the past can be ascertained a property-division award is not subject to post-divorce modification See HARRY A KRAUSE ET AL FAMILY LAW CASES COMMENTS AND QUESTIONS 691 (6th ed 2007)

Here the husband is seeking modification of a property-division award with respect to an asset owned by the parties at the time of divorce Thus the husband may not obtain a modification of the marital-residence-sale-proceeds provision of the divorce decree based on his reduced income

21

FEDERAL CIVIL PROCEDURE ANALYSIS (Federal Civil Procedure III IVC)

ANALYSIS

Legal Problems

(1) Is the logging company entitled to join this action as a matter of right

(2)(a) May the nonprofit organization obtain a temporary restraining order to stop the USFS from issuing a logging permit

(2)(b) May the nonprofit organization obtain a preliminary injunction to stop the USFS from issuing a logging permit during the pendency of the action

DISCUSSION

Summary

The logging company is entitled to intervene in this action as a matter of right because it has an interest in the property or transaction that is the subject of the action and is so situated that its interest may be impaired or impeded as a practical matter if the action goes forward without it The logging companyrsquos interest is not adequately represented by the USFSrsquos presence in the lawsuit

The nonprofit organization may seek a temporary restraining order (TRO) followed by a preliminary injunction to prevent the USFS from issuing a logging permit pending the outcome of the action The nonprofit is likely to obtain a TRO if it can demonstrate a risk of immediate and irreparable injury The nonprofit is also likely to obtain a preliminary injunction if it can demonstrate a significant threat of irreparable harm and a likelihood of success on the merits of its National Environmental Policy Act (NEPA) claim

Point One (50) Rule 24(a) of the Federal Rules of Civil Procedure requires federal courts to allow a person to intervene in an action as a matter of right if the person a) is interested in the property or transaction that is the subject of the action b) is so situated that its interest may be impaired or impeded if the litigation goes forward without it and c) is not adequately represented by existing parties Here the logging company likely meets all three requirements and should be allowed to intervene as a matter of right

Rule 24 of the Federal Rules of Civil Procedure governs intervention the process by which a non-party to an action may join the litigation Under Rule 24(a) (intervention of right) a person must be permitted to intervene if three conditions are met (1) the movant ldquoclaims an interest relating to the property or transaction that is the subject of the actionrdquo (2) the movant ldquois so situated that disposition of the action may as a practical matter impair or impede the movantrsquos ability to protect its interestrdquo and (3) ldquoexisting partiesrdquo do not ldquoadequately represent [the movantrsquos] interestrdquo FED R CIV P 24(a) The three requirements for intervention of right are often ldquovery interrelatedrdquo 7C CHARLES ALAN WRIGHT ET AL FEDERAL PRACTICE AND PROCEDURE sect 1908 at 297 (2007 amp 2011 Supp)

22

Federal Civil Procedure Analysis

Here the court should find that the logging company meets this test First the logging company has a strong interest in the property or transaction that is the subject of this action The USFS has accepted the logging companyrsquos bid and the logging company is merely awaiting issuance of a logging permit to begin logging The nonprofit organization is seeking to prevent this logging The logging company therefore has a strong direct and substantial interest in the subject matter of the lawsuit and in having its winning bid honored and a logging permit issued See eg Kleissler v US Forest Serv 157 F3d 964 972 (3d Cir 1998) (stating that ldquo[t]imber companies have direct and substantial interests in a lawsuit aimed at halting loggingrdquo) see also Natural Resources Defense Council v US Nuclear Regulatory Commrsquon 578 F2d 1341 1343ndash 44 (10th Cir 1978) (holding that applicants whose license renewals were pending had Rule 24(a)(2) interests where the lawsuit sought to halt the license-issuing process pending preparation of environmental impact statements) See generally 7C WRIGHT ET AL supra sect 19081 at 309 (ldquoIf there is a direct substantial legally protectable interest in the proceedings it is clear that this requirement of the rule is satisfiedrdquo) Second the logging companyrsquos interest in receiving a logging permit may well be impaired as a practical matter by the outcome of the lawsuit If the USFS loses the lawsuit it will have to prepare an environmental impact statement before issuing the logging companyrsquos permit This will at a minimum delay the logging companyrsquos ability to exercise its rights and may in the long r un mean that no logging permit is ever issued Intervention of right is not limited to those that would be legally bound as a matter of preclusion doctrine Id sect 19082 at 368 Rather ldquo[t]he rule is satisfied whenever disposition of the present action would put the movant at a practical disadvantage in protecting its interestrdquo Id sect 19082 at 369 Here that condition is easily satisfied See Kleissler 157 F3d at 972 (ldquoTimber companies have direct and substantial interests in a lawsuit aimed at halting logging rdquo)

Given that the logging company has an interest that may be impaired by disposition of the action it should be allowed to intervene unless the court is persuaded that the USFS adequately represents the logging companyrsquos interest See Rule 24(a)(2) 7C WRIGHT ET AL supra sect 1909 Here it could be argued that the USFS adequately represents the logging companyrsquos interest because the USFS presumably wants the court to uphold its development plan and allow it to proceed with issuance of the logging permit which is the same relief that the logging company would seek However whether representation is truly adequate depends upon ldquo[a] discriminating appraisal of the circumstancesrdquo 7C WRIGHT ET AL supra sect 1909 at 440 Although both the government and the logging company wish to avoid the preparation of an environmental impact statement their interests are distinct The USFSrsquos interest is proper management of the national forest system while the logging companyrsquos interest is making a profit from logging the 5000-acre tract The USFSrsquos handling of the litigation is likely to be affected by a variety of policy concerns and political considerations that have nothing to do with the logging companyrsquos purely economic interest in securing the right to cut trees in the Scenic National Forest See eg Kleissler 157 F3d at 973ndash74 (ldquo[T]he government represents numerous complex and conflicting interests in matters of this nature The straightforward business interests asserted by intervenors here may become lost in the thicket of sometimes inconsistent governmental policiesrdquo)

[NOTES (1) Examinees who mistakenly analyze the logging companyrsquos case for joinder under the related but incorrect Rule 19 ldquoRequired Joinder of Partiesrdquo may receive credit Rule 19 allows existing parties to demand joinder of non-parties (or seek dismissal of the case if they canrsquot get it) There is a close relationship between Rule 24 and Rule 19 and both contain a similar standard for determining when ldquointerestedrdquo third parties are ldquoentitledrdquo or ldquorequiredrdquo to be in the lawsuit Indeed the two prongs of the Rule 24 intervention test that are discussed above

23

Federal Civil Procedure Analysis

are nearly identical to the two prongs of the Rule 19(a) required joinder test Examinees who discuss and apply the test should receive credit even if they cite Rule 19 rather than Rule 24

(2) Examinees may discuss permissive joinder Although permissive joinder is a possibility here the question asks only whether the logging company can join the action as a matter of right and a permissive joinder analysis is not responsive to the question To the extent an examinee discusses permissive joinder the analysis will focus on whether the logging company ldquohas a claim or defense that shares with the main action a common question of law or factrdquo FED R CIV P 24(b)(1)(B) The district court also ldquomust consider whether the intervention will unduly delay or prejudice the adjudication of the original partiesrsquo rightsrdquo FED R CIV P 24(b)(3) On our facts the logging companyrsquos claim for the issuance of a logging permit would certainly share common questions of law and fact with the USFSrsquos defense against the nonprofitrsquos claim There are no facts suggesting that the logging companyrsquos presence would unduly delay or otherwise prejudice adjudication of the original action Thus the district court would have discretion to permit the logging company to intervene even if it denied intervention of right]

Point Two(a) (25) The nonprofit organization could seek and would likely obtain a temporary restraining order to stop the USFS from issuing a logging permit pending a hearing on an application for a preliminary injunction

The first type of interim relief the nonprofit could seek to stop the USFS from issuing a logging permit to the logging company is a temporary restraining order (TRO) prohibiting the USFS from issuing the logging permit A TRO can be issued without notice to the adverse party but only in limited circumstances and only for a limited time FED R CIV P 65(b) To secure a TRO without notice the nonprofit would need to submit an affidavit containing specific facts that demonstrate a risk of ldquoimmediate and irreparable injuryrdquo if a permit is issued FED R CIV P 65(b)(1) In deciding whether to grant a TRO courts will also consider the same factors that are relevant in deciding whether to grant a preliminary injunction (eg the moving partyrsquos likelihood of success on the merits the balance of hardships and the public interest) See Point Two(b) infra The TRO would last only long enough for the court to consider and resolve a request by the nonprofit for a preliminary injunction but no longer than 14 days (unless the court extends it for good cause or the adverse party consents to an extension) In addition bond is required

Here the court is likely to grant the nonprofitrsquos request The nonprofit could plausibly claim that cutting down 5000 acres of old-growth forest in an area that is home to the highest concentration of wildlife in the western United States would have ldquoan immediate and irreparablerdquo adverse impact on the environment and cause irreparable harm to the nonprofitrsquos interest in preserving and protecting natural resources including wildlife habitat

Point Two(b) (25) The nonprofit could also seek and would likely obtain a preliminary injunction to stop the USFS which is likely to be granted if the nonprofitrsquos claim that the USFS violated NEPA has a strong basis in fact and law

Because the TRO would be temporary the nonprofit would need to move for a preliminary injunction to prevent the USFS from issuing a logging permit throughout the pendency of the litigation Preliminary injunctions are injunctions that seek to ldquoprotect [the] plaintiff from

24

Federal Civil Procedure Analysis

irreparable injury and to preserve the courtrsquos power to render a meaningful decision after a trial on the meritsrdquo 11A CHARLES ALAN WRIGHT ET AL FEDERAL PRACTICE AND PROCEDURE sect 2947 at 112 (2013) Rule 65 of the Federal Rules of Civil Procedure sets out the procedural requirements for preliminary injunctions Preliminary injunctions may be granted only upon notice to the adverse party FED R CIV P 65(a)(1) and only if the movant ldquogives security in an amount that the court considers proper to pay the costs and damages sustained by any party found to have been wrongfully enjoined or restrainedrdquo FED R CIV P 65(c)

While Rule 65 sets out the procedural requirements for preliminary injunctive relief it does not specify the substantive grounds upon which it may be granted The courtrsquos discretion in ruling upon a motion for a preliminary injunction ldquois exercised in conformity with historic federal equity practicerdquo 11A WRIGHT ET AL supra sect 2947 at 114 The court typically considers four factors

(1) the significance of the threat of irreparable harm to the plaintiff if the injunction is not granted (2) the balance between this harm and the injury that granting the injunction would inflict on the defendant (3) the probability that the plaintiff will succeed on the merits and (4) the public interest

Id sect 2948 at 122ndash24 accord Habitat Educ Center v Bosworth 363 F Supp 2d 1070 1088 (ED Wis 2005) The most important of these factors is the risk of irreparable harm to the plaintiff 11A WRIGHT ET AL supra sect 29481 at 129 If the plaintiff has an adequate remedy at law (eg if money damages can compensate the plaintiff for its loss) then a preliminary injunction will be denied Id sect 29481

Here a court would likely conclude that the potential for environmental damage to the forest creates a significant threat of irreparable harm ldquo[E]nvironmental injury is often irreparable Courts have recognized that logging such as would occur [here] can have longshyterm environmental consequences and thus satisfy the irreparable injury criterionrdquo Habitat Educ Center 363 F Supp 2d at 1089 (citing Idaho Sporting Congress Inc v Alexander 222 F3d 562 569 (9th Cir 2000) (noting that the imminent and continuing logging activities presented ldquoevidence of environmental harm sufficient to tip the balance in favor of injunctive reliefrdquo)) Neighbors of Cuddy Mountain v US Forest Service 137 F3d 1372 1382 (9th Cir 1998) (stating that ldquo[t]he old growth forests plaintiffs seek to protect would if cut take hundreds of years to reproducerdquo) (internal citation omitted)) see also 11C WRIGHT ET AL supra sect 29481 at 151 (noting that ldquoa preliminary injunction has been issued to prevent harm to the environmentrdquo)

The second factor the balance between the harm to the plaintiff and the harm the defendant will suffer if the injunction is issued also appears to support issuance of a preliminary injunction here The USFS will have to wait before it can develop the Scenic National Forest and the logging company may lose money if the delay is prolonged These economic harms could be compensated monetarily if an injunction is issued inappropriately Where ldquoan injunction bond can compensate [the] defendant for any harm the injunction is likely to inflict the balance should be struck in favor of [the] plaintiffrdquo Id sect 29482 at 192 See also Habitat Educ Center 363 F Supp 2d at 1089 (stating that ldquothe relative absence of harmful effects on the Forest Service weighs in favor of granting the injunctionrdquo)

The third factor is the likelihood that the plaintiff will prevail on the merits Although there is limited information concerning the merits of the action the nonprofit alleges that the federal statute (NEPA) requires an environmental impact statement and further states that the USFS created no environmental impact analysis or statement at all Assuming that those

25

Federal Civil Procedure Analysis

allegations are correct it seems plausible to conclude that the nonprofit will be able to show a likelihood of success on the merits

Finally courts deciding whether or not to issue preliminary injunctive relief are to consider the public interest ldquoFocusing on this factor is another way of inquiring whether there are policy considerations that bear on whether the order should issuerdquo 11C WRIGHT ET AL supra sect 29484 at 214 If the court concludes that the nonprofit is likely to succeed on its NEPA claim because the USFS wrongfully failed to conduct an environmental impact assessment it is likely to find that the public interest would be served by restraining the USFS from proceeding with logging in a national forest See Heartwood Inc v US Forest Service 73 F Supp 2d 962 979 (SD Ill 1999) affrsquod on other grounds 230 F3d 947 (7th Cir 2000) (ldquoviolations by federal agencies of NEPArsquos provisions as established by Congress harm the public as well as the environmentrdquo)

Thus a court is very likely to grant a preliminary injunction if it concludes that the nonprofit has a significant likelihood of success on the merits

26

EVIDENCE ANALYSIS (Evidence ID IIA amp C)

ANALYSIS

Legal Problems

(1) Under what circumstances can evidence of prior convictions be used to impeach a witnessrsquos credibility in a civil case

(1)(a) May the inmatersquos credibility be impeached by evidence of a 12-year-old felony drug conviction if he was released from prison 9 years ago

(1)(b) May the inmatersquos credibility be impeached by evidence of an 8-year-old misdemeanor perjury conviction that was punishable by 1 year in jail if he pleaded guilty and was sentenced only to pay a $5000 fine

(1)(c) May the inmatersquos credibility be impeached by evidence of a 7-year-old sexual assault conviction if the inmate is still serving a 10-year prison sentence and the victim was his 13-year-old daughter

(2)(a) May the guardrsquos credibility be impeached by cross-examination regarding specific instances of misconduct (ie lying on his reacutesumeacute) relevant to credibility

(2)(b) May the guardrsquos credibility be impeached by admission of extrinsic evidence (his reacutesumeacute and academic transcript) offered to prove specific instances of misconduct relevant to credibility

DISCUSSION

Summary

Under the Federal Rules of Evidence witnesses can be impeached with evidence of prior convictions andor specific instances of misconduct Whether evidence of prior convictions should be admitted to impeach generally depends on the nature of the crime the amount of time that has passed and (only in criminal cases) whether the ldquowitnessrdquo is the defendant FED R EVID 609(a)

In this civil case evidence of the inmatersquos conviction for distribution of marijuana should be admitted to impeach the inmate because he was convicted of a felony and was released from prison fewer than 10 years ago FED R EVID 609(a)(1) Credibility is critically important in this case because the jury will hear conflicting testimony from the two disputing parties and there were no other eyewitnesses to the altercation Under Rule 609(a)(1) the inmatersquos conviction should be admitted because it has some bearing on his credibility and its probative value is not substantially outweighed by concerns of unfair prejudice confusion or delay Id

Evidence of the inmatersquos misdemeanor conviction for perjury must be admitted because the crime ldquorequired provingmdashor the witnessrsquos admittingmdasha dishonest act or false statementrdquo by the inmate FED R EVID 609(a)(2)

27

Evidence Analysis

Evidence of the inmatersquos felony conviction for sexual assault should be excluded because its probative value is substantially outweighed by the danger of unfair prejudice to the inmate based on the heinous nature of the crime FED R EVID 609(a)(1) In the alternative the judge could limit the evidence relating to this conviction by excluding details of the inmatersquos crime

In all civil (and criminal) cases witnesses can also be impeached with evidence of specific instances of prior misconduct that did not result in a conviction FED R EVID 608(b) Pursuant to Rule 608(b) misconduct probative of untruthfulness can be inquired into on cross-examination but cannot be proved through extrinsic evidence Id Thus the inmatersquos counsel should be permitted to cross-examine the guard regarding the false statement in the guardrsquos reacutesumeacute However extrinsic evidence of the guardrsquos misconduct (ie the guardrsquos authenticated reacutesumeacute and transcript from the local college) should not be admitted even if the guard denies wrongdoing or refuses to answer cross-examination questions about these matters Id

Point One (10) The Federal Rules of Evidence permit impeachment of witnesses with evidence of prior convictions

Whether convictions should be admitted to impeach generally depends on the nature of the crime the amount of time that has passed and (only in criminal cases) whether the ldquowitnessrdquo is the defendant FED R EVID 609(a) Under Rule 609(a) evidence of prior convictions may be admitted for the purpose of ldquoattacking a witnessrsquos character for truthfulnessrdquo Id

There are two basic types of convictions that can be admitted for the purpose of impeachment

(1) convictions for crimes ldquopunishable by death or by imprisonment for more than one yearrdquo (which generally correlates to ldquofeloniesrdquo) FED R EVID 609(a)(1) and (2) convictions ldquofor any crimes regardless of the punishment if the court can readily determine that establishing the elements of the crime required provingmdashor the witnessrsquos admittingmdasha dishonest act or false statementrdquo FED R EVID 609(a)(2)

Pursuant to Rule 609(a)(1) in civil cases the admission of evidence of a felony conviction is ldquosubject to Rule 403 [which says that a court may exclude relevant evidence if its probative value is substantially outweighed by other factors]rdquo FED R EVID 609(a)(1) However Rule 403 does not protect the witness against admission of prior convictions involving dishonestymdashwhich must be admitted by the court FED R EVID 609(a)(2)

Finally Federal Rule of Evidence 609(b) contains the presumption that a conviction that is more than 10 years old or where more than 10 years has passed since the witnessrsquos release from confinement (whichever is later) should not be admitted unless ldquoits probative value supported by specific facts and circumstances substantially outweighs its prejudicial effectrdquo and the proponent has provided the adverse party with reasonable written notice FED R EVID 609(b)

Point One(a) (25) The court should admit evidence of the inmatersquos 12-year-old felony marijuana distribution conviction

The inmatersquos conviction for marijuana distribution was for a felony punishable by imprisonment for more than one year See FED R EVID 609(a)(1) Moreover although the conviction was 12 years ago the 10-year time limit of Rule 609(b) is not exceeded because that time limit runs

28

Evidence Analysis

from the date of either ldquothe witnessrsquos conviction or release from confinement for it whichever is laterrdquo FED R EVID 609(b) Because the inmate served three years in prison he was released from confinement nine years ago

However pursuant to Rule 609(a)(1) the admission of felony convictions to impeach a witness in a civil case is ldquosubject to Rule 403rdquo FED R EVID 609(a)(1) Neither Rule 609(a) nor the advisory committee notes specify which factors courts should consider when balancing the probative value of a conviction against the dangers identified in Rule 403 (which include (1) unfair prejudice (2) confusion of the issues (3) misleading the jury (4) waste of time or undue delay and (5) needless presentation of cumulative evidence) FED R EVID 403

In this case credibility is very important because the evidence consists primarily of the testimony of the disputing parties and there were no other eyewitnesses to the altercation This enhances the probative value of any evidence bearing on the inmatersquos credibility A court is likely to conclude that the inmatersquos prior felony drug conviction is relevant to his credibility See eg United States v Brito 427 F3d 53 64 (1st Cir 2005) (ldquoPrior drug-trafficking crimes are generally viewed as having some bearing on veracityrdquo) Although the probative value of any conviction diminishes with age see eg United States v Brewer 451 F Supp 50 53 (ED Tenn 1978) the inmatersquos ongoing problems with the law suggest that he has continued (and even escalated) his criminal behavior over the past nine years The court should admit this evidence because its probative value is not substantially outweighed by any Rule 403 concerns Specifically any prejudice to the inmate would be slight because the conviction is unrelated to the altercation at issue and the conviction was not for a heinous crime that might inflame the jury

[NOTE Whether an examinee identifies the jury instruction as containing a ldquoconclusiverdquo or ldquomandatoryrdquo presumption is less important than the examineersquos analysis of the constitutional infirmities]

Point One(b) (15) The court must admit evidence of the inmatersquos eight-year-old misdemeanor conviction because perjury is a crime of dishonesty

Rule 609(a)(2) provides that evidence of a criminal conviction ldquomust be admitted if the court can readily determine that establishing the elements of the crime required provingmdashor the witnessrsquos admittingmdasha dishonest act or false statementrdquo FED R EVID 609(a)(2) The inmatersquos conviction for perjury would have necessarily required proving that the inmate engaged in an act of dishonesty This conviction occurred within the past 10 years so it ldquomust be admittedrdquo because in contrast to Rule 609(a)(1) (discussed in Point One(a)) admission under Rule 609(a)(2) is mandatory and not subject to Rule 403

Point One(c) (20) The court should exclude evidence of the inmatersquos seven-year-old felony sexual assault conviction because the probative value of this evidence is substantially outweighed by the danger of unfair prejudice In the alternative the details of the prior conviction could be excluded

The inmatersquos conviction for felony sexual assault was seven years ago and he has not yet been released from incarceration so Rule 609(a) but not 609(b) is applicable here FED R EVID 609(a) This conviction is therefore admissible to impeach the inmate unless its probative value is substantially outweighed by the danger of unfair prejudice or any other Rule 403 concern Id

29

Evidence Analysis

Sex crimes are generally not considered relevant to credibility see Hopkins v State 639 So 2d 1247 1254 (Miss 1993) so the probative value of this conviction is relatively low Moreover the heinous nature of the inmatersquos crime (sexual assault on his daughter) makes the danger of unfair prejudice to the inmate very high Thus the court should exclude evidence of the conviction because it was for a heinous offense that is likely to inflame the jury and it has little bearing on credibility See eg United States v Beahm 664 F2d 414 419 (4th Cir 1981)

As an alternative to excluding this evidence the judge could minimize the unfair prejudice to the inmate by permitting limited cross-examination but refusing to allow specific questions about the nature of the inmatersquos conviction For example a court could limit cross-examination to the fact that the inmate was convicted of a ldquofelonyrdquo or perhaps that he was convicted of a ldquosexual assaultrdquo without identifying the victim However because evidence of the inmatersquos prior convictions can be admitted solely for the purpose of enabling the jury to assess his credibility and because his two earlier convictions should have already been admitted the court should exclude all evidence of the felony sexual assault conviction

Point Two(a) (15) The court should permit the inmatersquos counsel to cross-examine the guard regarding the false statement in his reacutesumeacute because the guardrsquos misconduct bears on his truthfulness

The inmate wishes to cross-examine the guard about his prior dishonest behaviormdashlying on his reacutesumeacutemdashthat did not involve a criminal conviction Rule 608(b) allows witnesses to be cross-examined about specific instances of prior non-conviction misconduct probative of untruthfulness ldquoin order to attack the witnessrsquos character for truthfulnessrdquo FED R EVID 608(b)

The courtrsquos decision to allow cross-examination about the guardrsquos prior dishonest behavior depends on the probative value of such evidence balanced against the danger of unfair prejudice to the guard or any other Rule 403 concern FED R EVID 403 Here the guardrsquos false statement on his reacutesumeacute that he obtained a degree in Criminal Justice is highly probative of his untruthfulness because it grossly misrepresents his actual academic record was made recently and was made with the intent to deceive Because the probative value of this evidence is very strong and is not substantially outweighed by any Rule 403 concerns cross-examination of the guard on this topic should be permitted The court may also consider it fair to permit this cross-examination of the guard on these matters assuming that one or more of the inmatersquos prior convictions have been admitted to impeach his credibility

Point Two(b) (15) The court should exclude extrinsic evidence of the guardrsquos non-conviction misconduct even if the guard denies wrongdoing or refuses to answer questions about the matter

Although Rule 608(b) allows cross-examination about specific instances of prior misconduct probative of untruthfulness ldquoextrinsic evidencerdquo offered to prove such misconduct is not admissible FED R EVID 608(b) The rationale for this rule is that allowing the introduction of extrinsic evidence of prior misconduct by witnesses when these acts are relevant only to the witnessesrsquo truthfulness and not to the main issues in the case would create too great a risk of confusing the jury and unduly delaying the trial The court does not have discretion to admit this extrinsic evidence See eg United States v Elliot 89 F3d 1360 1368 (8th Cir 1996)

30

Evidence Analysis

Here the inmatersquos counsel may cross-examine the guard about the false statement on his reacutesumeacute However the inmatersquos counsel must accept the guardrsquos response Even if the guard denies wrongdoing or refuses to answer questions about the matter the inmatersquos counsel cannot introduce the guardrsquos reacutesumeacute or the transcript from the local college to prove the guardrsquos misconduct

31

CORPORATIONS ANALYSIS (Corporations VA2 IX)

ANALYSIS

Legal Problems

(1) Do shareholders have the authority to amend a corporationrsquos bylaws with respect to director nominations

(2) Do board-approved bylaws on a particular subject here nomination of directors preempt subsequent conflicting bylaw amendments by shareholders

(3) Is a suit challenging both managementrsquos refusal to include the proposed bylaw amendment in Megarsquos proxy statement and the boardrsquos amendment of the bylaws dealing with nomination of directors a direct or derivative suit

DISCUSSION

Summary

The voting and litigation rights of the shareholders of Mega are subject to the provisions of the Model Business Corporations Act (MBCA)

The investorrsquos proposed bylaw provision is not inconsistent with state law Under the MBCA shareholders may amend the bylaws when the amendment deals with a proper matter for the corporationrsquos bylaws such as procedures for nominating directors

The Mega boardrsquos bylaw amendment does not preempt the investorrsquos proposed bylaw provision or the Mega shareholdersrsquo power to approve it While shareholders can limit the boardrsquos power to amend or repeal the bylaws the board cannot limit the shareholdersrsquo power

Whether the investor must make a demand on Megarsquos board depends on how the investor frames its claim If the investor claims a violation of shareholder voting rights the claim is direct and pre-suit demand on the board is not required If on the other hand the investor claims that the directors violated their fiduciary duties by amending the bylaws to entrench themselves the claim is derivative and a pre-suit demand is required

Point One (30) Shareholders may amend the corporationrsquos bylaws where the proposed bylaw provision relates to procedural matters typically included in the bylaws such as the nomination of directors

Internal affairs of the corporation such as the conduct of shareholder meetings and election of directors are subject to the corporate law of the state of incorporation See McDermott Inc v Lewis 531 A2d 206 (Del 1987) (applying law of jurisdiction where corporation was incorporated in case involving voting rights) This statersquos corporate statute is modeled on the MBCA

Under the MBCA ldquoshareholders may amend the corporationrsquos bylawsrdquo MBCA sect 1020(a) Thus the only question is whether the bylaws can specify the procedures for shareholder nomination of directors

32

Corporations Analysis

The MBCA states that the bylaws ldquomay contain any provision that is not inconsistent with law or the articles of incorporationrdquo MBCA sect 206(b) In addition the MBCA was revised in 2009 to address shareholder nomination of directors in public corporations (known as ldquoproxy accessrdquo) and specifies that the bylaws ldquomay contain a requirement that the corporation include in its [proxy materials] one or more individuals nominated by a shareholderrdquo MBCA sect 206(c)(1) see Committee on Corporate Laws ABA Section of Business Law Report on the Roles of Boards of Directors and Shareholders of Publicly Owned Corporations and Changes to the Model Business Corporations ActmdashAdoption of Shareholder Proxy Access Amendments to Chapters 2 and 10 65 BUS LAWYER 1105 (2010)

The inclusion of director-nomination procedures in the bylaws is consistent with practice and is recognized by the Delaware courts whose views on corporate law carry significant weight Typically the procedures for nomination of directors are found in the bylaws See 1 COX amp HAZEN TREATISE ON THE LAW OF CORPORATIONS sect 312 (3d ed 2011) see also 4 FLETCHER CORP FORMS ANN PART III ch 21 (2013) (including sample bylaws that permit nomination of directors by shareholders) The Delaware Supreme Court has confirmed that the bylaws may ldquodefine the process and proceduresrdquo for director elections See CA Inc v AFSCME Employees Pension Plan 953 A2d 227 (Del 2008) (concluding that bylaw amendment requiring reimbursement of election expenses to certain successful shareholder nominators is ldquoproper subjectrdquo under Delaware law)

[NOTE The question of the proper scope of the bylaws can be answered using the more general MBCA sect 206(b) or the 2009 MBCA revision adding sect 206(c)(1) (adopted in CT ME VA) In addition some examinees might raise the point that shareholder proposals may not compel the board to take action such as by including shareholder nominations in the companyrsquos proxy materials on the theory that the ldquobusiness and affairsrdquo of the corporation are to be managed by the board See MBCA sect 801(b) Although shareholders are generally limited to adopting precatory resolutions that recommend or encourage board action this limitation does not apply when shareholders have specific authority to take binding action on their ownmdashsuch as to amend the bylaws]

Point Two (30) Shareholders can amend (or repeal) board-approved bylaws Further shareholders can limit the boardrsquos power to later amend and repeal a shareholder-approved bylaw

Under the MBCA shareholders have the power to amend the bylaws See Point One The board shares this power with the shareholders unless (1) the corporationrsquos articles ldquoreserve that power exclusively to the shareholdersrdquo or (2) ldquothe shareholders in amending repealing or adopting a bylaw expressly provide that the board of directors may not amend repeal or reinstate that bylawrdquo See MBCA sect 1020(b)

Shareholder-approved bylaw provisions can amend or repeal existing bylaw provisions whether originally approved by the board or by shareholders See ALAN R PALMITER CORPORATIONS EXAMPLES AND EXPLANATIONS sect 713 (7th ed 2012) Thus the Mega boardrsquos bylaw amendmentmdashwhich set more demanding thresholds for shareholder nomination of directors than the investorrsquos proposed bylaw provisionmdashwould be superseded (repealed) if Megarsquos shareholders were to approve the investorrsquos proposal

Further a shareholder-approved bylaw generally can limit the power of the board to later amend or repeal it See MBCA sect 1020(b)(2) Thus if Megarsquos shareholders approved the bylaw

33

Corporations Analysis

provision proposed by the investor Megarsquos board could not repeal the provision because it includes a ldquono board repealrdquo clause

The revision to the MBCA in 2009 dealing with shareholder proxy access does not change this conclusion That revision specifies that a shareholder-approved bylaw dealing with director nominations may not limit the boardrsquos power to amend add or repeal ldquoany procedure or condition to such a bylaw in order to provide for a reasonable practicable and orderly processrdquo MBCA sect 206(d) Thus according to the revision if shareholders approve a bylaw amendment that limits further board changes the board would nonetheless retain the power to ldquotinkerrdquo with the bylaw to safeguard the voting process but could not repeal the shareholder-approved bylaw The Official Comment to MBCA sect 206(d) makes clear that the revision is ldquonot intended to allow the board of directors to frustrate the purpose of the shareholder-adopted proxy access provisionrdquo Thus if Megarsquos shareholders were to approve the bylaw provision proposed by the investor Megarsquos board could only amend the provision regarding its procedures or conditions in a manner consistent with its purpose of permitting proxy access for Megarsquos shareholders

[NOTE The boardrsquos attempted interference with a shareholder voting initiative may also have been a violation of the directorsrsquo fiduciary duties See Blasius Indus Inc v Atlas Corp 564 A2d 651 (Del Ch 1988) (finding that directors breached their fiduciary duties by amending bylaws and expanding size of board to thwart insurgentrsquos plan to amend bylaws and seat a majority of new directors) The call however asks examinees to consider whether shareholders or the board have ldquoprecedencerdquo over amending the corporate bylaws Thus an examineersquos answer should be framed in terms of ldquopowerrdquo and not ldquodutyrdquo]

Point Three (40) The investor need not make a demand on the board if the investor states a direct claim such as an allegation that the board interfered with the investorrsquos right to amend the bylaws But the investor must make a demand on the board if the investor states a derivative claim (on behalf of the corporation) such as an allegation that the directors sought to entrench themselves by interfering with the proposed proxy access

The MBCA generally requires that shareholders make a demand on the board of directors before initiation of a derivative suit MBCA sect 742 (shareholder may not bring derivative proceeding until written demand has been made on corporation and 90 days have expired) A derivative suit is essentially two suits in one where the plaintiff-shareholder seeks to bring on behalf of the corporation a claim that vindicates corporate rights usually based on violation of fiduciary duties PALMITER supra sect 1811 (6th ed 2009) The demand permits the board to investigate the situation identified by the shareholder and take suitable action No demand on the board is required however if the shareholder brings a direct suit to vindicate the shareholderrsquos own rights not those of the corporation

Is the suit brought by the investor derivative or direct The MBCA defines a ldquoderivative proceedingrdquo as one brought ldquoin the right of a domestic corporationrdquo MBCA sect 740(1) Thus the answer to how the investorrsquos suit should be characterized turns on what rights the investor seeks to vindicate If the investor frames its claim as one of fiduciary breach by directorsmdashfor example for failing to become adequately informed about voting procedures or for seeking to entrench themselves in office by manipulating the voting structure to avoid a shareholder insurgencymdashthen the suit is ldquoderivativerdquo and the investor must make a demand on the board See MBCA Ch 7 Subch D Introductory Comment (ldquothe derivative suit has historically been the principal method of challenging allegedly illegal action by managementrdquo)

34

Corporations Analysis

If however the investor frames its claim as one to vindicate shareholder rights the suit is direct and no demand is required For many courts the direct-derivative question turns on who is injured and who is to receive the relief sought by the plaintiff-shareholders See Tooley v Donaldson Lufkin amp Jenrette Inc 845 A2d 1031 (Del 2004) (characterizing a merger-delay claim as direct because delay of merger only harmed shareholders not corporation) Thus if the investor claims that managementrsquos refusal to include its proposed bylaw amendment in the corporationrsquos proxy materials violates its shareholder rights to initiate corporate governance reforms the suit will be direct Courts have not questioned the ability of shareholders to bring direct suits challenging board action to exclude their proposed bylaw amendments from the corporationrsquos proxy materials See JANA Master Fund Ltd v CNET Networks Inc 954 A2d 335 (Del Ch 2008) (upholding shareholderrsquos direct challenge to boardrsquos interpretation of advance-notice bylaw) Chesapeake Corp v Shore 771 A2d 293 (Del Ch 2000) (upholding shareholderrsquos direct challenge to actions by board that effectively prevented it from proposing bylaw amendments in contest for control)

Is the way that the investor frames its claim conclusive Courts have permitted shareholder-plaintiffs to challenge a transaction in a direct suit even though the same transaction could also be challenged as a fiduciary breach See Eisenberg v Flying Tiger Line Inc 451 F2d 267 (2d Cir 1971) (permitting direct suit challenging a corporate reorganization as a dilution of shareholder voting power even though reorganization may have involved conflicts of interest and thus constituted a fiduciary breach) Thus the investorrsquos choice to pursue a claim challenging the legality of managementrsquos decision to exclude the investorrsquos proposal from the corporationrsquos proxy materialsmdashrather than a possible breach of fiduciary dutymdashis likely to be respected See 3 COX amp HAZEN supra sect 153 (describing situations in which a claim can be framed as derivative or direct)

[NOTE Some issues under Delaware corporate law regarding pre-suit demand are not relevant here For example whether the Mega directors are independent and disinterested is not relevant to the MBCA requirement of a pre-suit demand As the Official Comment to MBCA sect 742 points out the MBCArsquos requirement of ldquouniversal demandrdquo gives the board ldquothe opportunity to reexamine the act complained of in the light of a potential lawsuit and take corrective actionrdquo even when the directors might be non-independent or have conflicts of interest

Nor is it relevant to the MBCA pre-suit demand requirement that the statutory 90-day waiting period may be onerous The first paragraph of MBCA sect 742 requires a pre-suit demand without exception the second paragraph of the section imposes a 90-day waiting period before a derivative suit may be brought which can be shortened if the board rejects the demand or ldquoirreparable injury to the corporation would result by waiting for the expiration of the 90-day periodrdquo The call as written asks only whether a pre-suit demand should be made and does not ask examinees to address whether the post-demand waiting period should be shortened under the ldquoirreparable injuryrdquo standard]

35

National Conference of Bar Examiners 302 South Bedford Street | Madison WI 53703-3622 Phone 608-280-8550 | Fax 608-280-8552 | TDD 608-661-1275

wwwncbexorg e-mail contactncbexorg

  • Preface
  • Description of the MEE
  • Instructions
  • July 2014 Questions
    • CRIMINAL LAW AND PROCEDURE QUESTION
    • CONTRACTS QUESTION
    • FAMILY LAW QUESTION
    • FEDERAL CIVIL PROCEDURE QUESTION
    • EVIDENCE QUESTION
    • CORPORATIONS QUESTION
      • July 2014 Analyses
        • CRIMINAL LAW AND PROCEDURE ANALYSIS
        • CONTRACTS ANALYSIS
        • FAMILY LAW ANALYSIS
        • FEDERAL CIVIL PROCEDURE ANALYSIS
        • EVIDENCE ANALYSIS
        • CORPORATIONS ANALYSIS
            • ltlt13 ASCII85EncodePages false13 AllowTransparency false13 AutoPositionEPSFiles true13 AutoRotatePages None13 Binding Left13 CalGrayProfile (Dot Gain 20)13 CalRGBProfile (sRGB IEC61966-21)13 CalCMYKProfile (US Web Coated 050SWOP051 v2)13 sRGBProfile (sRGB IEC61966-21)13 CannotEmbedFontPolicy Error13 CompatibilityLevel 1413 CompressObjects Tags13 CompressPages true13 ConvertImagesToIndexed true13 PassThroughJPEGImages true13 CreateJobTicket false13 DefaultRenderingIntent Default13 DetectBlends true13 DetectCurves 0000013 ColorConversionStrategy CMYK13 DoThumbnails false13 EmbedAllFonts true13 EmbedOpenType false13 ParseICCProfilesInComments true13 EmbedJobOptions true13 DSCReportingLevel 013 EmitDSCWarnings false13 EndPage -113 ImageMemory 104857613 LockDistillerParams false13 MaxSubsetPct 10013 Optimize true13 OPM 113 ParseDSCComments true13 ParseDSCCommentsForDocInfo true13 PreserveCopyPage true13 PreserveDICMYKValues true13 PreserveEPSInfo true13 PreserveFlatness true13 PreserveHalftoneInfo false13 PreserveOPIComments true13 PreserveOverprintSettings true13 StartPage 113 SubsetFonts true13 TransferFunctionInfo Apply13 UCRandBGInfo Preserve13 UsePrologue false13 ColorSettingsFile ()13 AlwaysEmbed [ true13 ]13 NeverEmbed [ true13 ]13 AntiAliasColorImages false13 CropColorImages true13 ColorImageMinResolution 30013 ColorImageMinResolutionPolicy OK13 DownsampleColorImages true13 ColorImageDownsampleType Bicubic13 ColorImageResolution 30013 ColorImageDepth -113 ColorImageMinDownsampleDepth 113 ColorImageDownsampleThreshold 15000013 EncodeColorImages true13 ColorImageFilter DCTEncode13 AutoFilterColorImages true13 ColorImageAutoFilterStrategy JPEG13 ColorACSImageDict ltlt13 QFactor 01513 HSamples [1 1 1 1] VSamples [1 1 1 1]13 gtgt13 ColorImageDict ltlt13 QFactor 01513 HSamples [1 1 1 1] VSamples [1 1 1 1]13 gtgt13 JPEG2000ColorACSImageDict ltlt13 TileWidth 25613 TileHeight 25613 Quality 3013 gtgt13 JPEG2000ColorImageDict ltlt13 TileWidth 25613 TileHeight 25613 Quality 3013 gtgt13 AntiAliasGrayImages false13 CropGrayImages true13 GrayImageMinResolution 30013 GrayImageMinResolutionPolicy OK13 DownsampleGrayImages true13 GrayImageDownsampleType Bicubic13 GrayImageResolution 30013 GrayImageDepth -113 GrayImageMinDownsampleDepth 213 GrayImageDownsampleThreshold 15000013 EncodeGrayImages true13 GrayImageFilter DCTEncode13 AutoFilterGrayImages true13 GrayImageAutoFilterStrategy JPEG13 GrayACSImageDict ltlt13 QFactor 01513 HSamples [1 1 1 1] VSamples [1 1 1 1]13 gtgt13 GrayImageDict ltlt13 QFactor 01513 HSamples [1 1 1 1] VSamples [1 1 1 1]13 gtgt13 JPEG2000GrayACSImageDict ltlt13 TileWidth 25613 TileHeight 25613 Quality 3013 gtgt13 JPEG2000GrayImageDict ltlt13 TileWidth 25613 TileHeight 25613 Quality 3013 gtgt13 AntiAliasMonoImages false13 CropMonoImages true13 MonoImageMinResolution 120013 MonoImageMinResolutionPolicy OK13 DownsampleMonoImages true13 MonoImageDownsampleType Bicubic13 MonoImageResolution 120013 MonoImageDepth -113 MonoImageDownsampleThreshold 15000013 EncodeMonoImages true13 MonoImageFilter CCITTFaxEncode13 MonoImageDict ltlt13 K -113 gtgt13 AllowPSXObjects false13 CheckCompliance [13 None13 ]13 PDFX1aCheck false13 PDFX3Check false13 PDFXCompliantPDFOnly false13 PDFXNoTrimBoxError true13 PDFXTrimBoxToMediaBoxOffset [13 00000013 00000013 00000013 00000013 ]13 PDFXSetBleedBoxToMediaBox true13 PDFXBleedBoxToTrimBoxOffset [13 00000013 00000013 00000013 00000013 ]13 PDFXOutputIntentProfile ()13 PDFXOutputConditionIdentifier ()13 PDFXOutputCondition ()13 PDFXRegistryName ()13 PDFXTrapped False1313 CreateJDFFile false13 Description ltlt13 ARA 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 BGR ltFEFF04180437043f043e043b043704320430043904420435002004420435043704380020043d0430044104420440043e0439043a0438002c00200437043000200434043000200441044a0437043404300432043004420435002000410064006f00620065002000500044004600200434043e043a0443043c0435043d04420438002c0020043c0430043a04410438043c0430043b043d043e0020043f044004380433043e04340435043d04380020043704300020043204380441043e043a043e043a0430044704350441044204320435043d0020043f04350447043004420020043704300020043f044004350434043f0435044704300442043d04300020043f043e04340433043e0442043e0432043a0430002e002000200421044a04370434043004340435043d043804420435002000500044004600200434043e043a0443043c0435043d044204380020043c043e0433043004420020043404300020044104350020043e0442043204300440044f0442002004410020004100630072006f00620061007400200438002000410064006f00620065002000520065006100640065007200200035002e00300020043800200441043b0435043404320430044904380020043204350440044104380438002egt13 CHS ltFEFF4f7f75288fd94e9b8bbe5b9a521b5efa7684002000410064006f006200650020005000440046002065876863900275284e8e9ad88d2891cf76845370524d53705237300260a853ef4ee54f7f75280020004100630072006f0062006100740020548c002000410064006f00620065002000520065006100640065007200200035002e003000204ee553ca66f49ad87248672c676562535f00521b5efa768400200050004400460020658768633002gt13 CHT ltFEFF4f7f752890194e9b8a2d7f6e5efa7acb7684002000410064006f006200650020005000440046002065874ef69069752865bc9ad854c18cea76845370524d5370523786557406300260a853ef4ee54f7f75280020004100630072006f0062006100740020548c002000410064006f00620065002000520065006100640065007200200035002e003000204ee553ca66f49ad87248672c4f86958b555f5df25efa7acb76840020005000440046002065874ef63002gt13 CZE 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 DAN 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 DEU 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 ESP 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 ETI 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 FRA 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 GRE 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 HEB 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 HRV (Za stvaranje Adobe PDF dokumenata najpogodnijih za visokokvalitetni ispis prije tiskanja koristite ove postavke Stvoreni PDF dokumenti mogu se otvoriti Acrobat i Adobe Reader 50 i kasnijim verzijama)13 HUN 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 ITA 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 JPN ltFEFF9ad854c18cea306a30d730ea30d730ec30b951fa529b7528002000410064006f0062006500200050004400460020658766f8306e4f5c6210306b4f7f75283057307e305930023053306e8a2d5b9a30674f5c62103055308c305f0020005000440046002030d530a130a430eb306f3001004100630072006f0062006100740020304a30883073002000410064006f00620065002000520065006100640065007200200035002e003000204ee5964d3067958b304f30533068304c3067304d307e305930023053306e8a2d5b9a306b306f30d530a930f330c8306e57cb30818fbc307f304c5fc59808306730593002gt13 KOR ltFEFFc7740020c124c815c7440020c0acc6a9d558c5ec0020ace0d488c9c80020c2dcd5d80020c778c1c4c5d00020ac00c7a50020c801d569d55c002000410064006f0062006500200050004400460020bb38c11cb97c0020c791c131d569b2c8b2e4002e0020c774b807ac8c0020c791c131b41c00200050004400460020bb38c11cb2940020004100630072006f0062006100740020bc0f002000410064006f00620065002000520065006100640065007200200035002e00300020c774c0c1c5d0c11c0020c5f40020c2180020c788c2b5b2c8b2e4002egt13 LTH 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 LVI 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 NLD (Gebruik deze instellingen om Adobe PDF-documenten te maken die zijn geoptimaliseerd voor prepress-afdrukken van hoge kwaliteit De gemaakte PDF-documenten kunnen worden geopend met Acrobat en Adobe Reader 50 en hoger)13 NOR 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 POL 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 PTB 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 RUM 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 RUS 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 SKY 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 SLV 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 SUO 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 SVE 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 TUR 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 UKR ltFEFF04120438043a043e0440043804410442043e043204430439044204350020044604560020043f043004400430043c043504420440043800200434043b044f0020044104420432043e04400435043d043d044f00200434043e043a0443043c0435043d044204560432002000410064006f006200650020005000440046002c0020044f043a04560020043d04300439043a04400430044904350020043f045604340445043e0434044f0442044c00200434043b044f0020043204380441043e043a043e044f043a04560441043d043e0433043e0020043f0435044004350434043404400443043a043e0432043e0433043e0020043404400443043a0443002e00200020042104420432043e04400435043d045600200434043e043a0443043c0435043d0442043800200050004400460020043c043e0436043d04300020043204560434043a0440043804420438002004430020004100630072006f006200610074002004420430002000410064006f00620065002000520065006100640065007200200035002e0030002004300431043e0020043f04560437043d04560448043e04570020043204350440044104560457002egt13 ENU (Use these settings to create Adobe PDF documents best suited for high-quality prepress printing Created PDF documents can be opened with Acrobat and Adobe Reader 50 and later)13 gtgt13 Namespace [13 (Adobe)13 (Common)13 (10)13 ]13 OtherNamespaces [13 ltlt13 AsReaderSpreads false13 CropImagesToFrames true13 ErrorControl WarnAndContinue13 FlattenerIgnoreSpreadOverrides false13 IncludeGuidesGrids false13 IncludeNonPrinting false13 IncludeSlug false13 Namespace [13 (Adobe)13 (InDesign)13 (40)13 ]13 OmitPlacedBitmaps false13 OmitPlacedEPS false13 OmitPlacedPDF false13 SimulateOverprint Legacy13 gtgt13 ltlt13 AddBleedMarks false13 AddColorBars false13 AddCropMarks false13 AddPageInfo false13 AddRegMarks false13 ConvertColors ConvertToCMYK13 DestinationProfileName ()13 DestinationProfileSelector DocumentCMYK13 Downsample16BitImages true13 FlattenerPreset ltlt13 PresetSelector MediumResolution13 gtgt13 FormElements false13 GenerateStructure false13 IncludeBookmarks false13 IncludeHyperlinks false13 IncludeInteractive false13 IncludeLayers false13 IncludeProfiles false13 MultimediaHandling UseObjectSettings13 Namespace [13 (Adobe)13 (CreativeSuite)13 (20)13 ]13 PDFXOutputIntentProfileSelector DocumentCMYK13 PreserveEditing true13 UntaggedCMYKHandling LeaveUntagged13 UntaggedRGBHandling UseDocumentProfile13 UseDocumentBleed false13 gtgt13 ]13gtgt setdistillerparams13ltlt13 HWResolution [2400 2400]13 PageSize [612000 792000]13gtgt setpagedevice13

Page 2: July 2014 MEE Questions and Analyses - NCBE...This publication includes the questions and analyses from the July 2014 MEE. (In the actual test, the questions are simply numbered rather

Copyright copy 2014 by the National Conference of Bar Examiners All rights reserved

Contents

Prefaceii

Description of the MEE ii

Instructionsiii

July 2014 Q uestions

Criminal Law and Procedure Question 3

Contracts Question 5

Family Law Question 6

Federal Civil Procedure Question 7

Evidence Question 8

Corporations Question 9

July 2014 A nalyses

Criminal Law and Procedure Analysis 13

Contracts Analysis 16

Family Law Analysis 19

Federal Civil Procedure Analysis 22

Evidence Analysis 27

Corporations Analysis 32

i

Preface

The Multistate Essay Examination (MEE) is developed by the National Conference of Bar Examiners (NCBE) This publication includes the questions and analyses from the July 2014 MEE (In the actual test the questions are simply numbered rather than being identified by area of law) The instructions for the test appear on page iii

The model analyses for the MEE are illustrative of the discussions that might appear in excellent answers to the questions They are provided to the user jurisdictions to assist graders in grading the examination They address all the legal and factual issues the drafters intended to raise in the questions

The subjects covered by each question are listed on the first page of its accompanying analysis identified by roman numerals that refer to the MEE subject matter outline for that subject For example the Federal Civil Procedure question on the July 2014 MEE tested the following areas from the Federal Civil Procedure outline III Injunctions and provisional remedies and IVC Pretrial proceduresmdashJoinder of parties and claims (including class actions)

For more information about the MEE including subject matter outlines visit the NCBE website at wwwncbexorg

Description of the MEE

The MEE consists of six 30-minute questions and is a component of the Uniform Bar Examination (UBE) It is administered by user jurisdictions as part of the bar examination on the Tuesday before the last Wednesday in February and July of each year Areas of law that may be covered on the MEE include the following Business Associations (Agency and Partnership Corporations and Limited Liability Companies) Conflict of Laws Constitutional Law Contracts Criminal Law and Procedure Evidence Family Law Federal Civil Procedure Real Property Torts Trusts and Estates (Decedentsrsquo Estates Trusts and Future Interests) and Uniform Commercial Code (Negotiable Instruments and Bank Deposits and Collections Secured Transactions) (Effective with the February 2015 MEE administration Uniform Commercial Code Articles 3 and 4 [Negotiable Instruments and Bank Deposits and Collections] is no longer being tested) Some questions may include issues in more than one area of law The particular areas covered vary from exam to exam

The purpose of the MEE is to test the examineersquos ability to (1) identify legal issues raised by a hypothetical factual situation (2) separate material which is relevant from that which is not (3) present a reasoned analysis of the relevant issues in a clear concise and well-organized composition and (4) demonstrate an understanding of the fundamental legal principles relevant to the probable solution of the issues raised by the factual situation The primary distinction between the MEE and the Multistate Bar Examination (MBE) is that the MEE requires the examinee to demonstrate an ability to communicate effectively in writing

ii

Instructions The back cover of each test booklet contains the following instructions

You will be instructed when to begin and when to stop this test Do not break the seal on this booklet until you are told to begin

You may answer the questions in any order you wish Do not answer more than one question in each answer booklet If you make a mistake or wish to revise your answer simply draw a line through the material you wish to delete

If you are using a laptop computer to answer the questions your jurisdiction will provide you with specific instructions

Read each fact situation very carefully and do not assume facts that are not given in the question Do not assume that each question covers only a single area of the law some of the questions may cover more than one of the areas you are responsible for knowing

Demonstrate your ability to reason and analyze Each of your answers should show an understanding of the facts a recognition of the issues included a knowledge of the applicable principles of law and the reasoning by which you arrive at your conclusions The value of your answer depends not as much upon your conclusions as upon the presence and quality of the elements mentioned above

Clarity and conciseness are important but make your answer complete Do not volunteer irrelevant or immaterial information

Answer all questions according to generally accepted fundamental legal principles unless your testing jurisdiction has instructed you to answer according to local case or statutory law

NOTE Examinees testing in UBE jurisdictions must answer according to generally accepted fundamental legal principles rather than local case or statutory law

iii

July 2014 MEE

QUESTIONS

Contracts Criminal Law and Procedure

Family L aw Federal Civil Procedure

Evidence Corporations

CRIMINAL LAW AND PROCEDURE QUESTION

While on routine patrol a police officer observed a suspect driving erratically and pulled the suspectrsquos car over to investigate When he approached the suspectrsquos car the officer detected a strong odor of marijuana The officer immediately arrested the suspect for driving under the influence of an intoxicant (DUI) While the officer was standing near the suspectrsquos car placing handcuffs on the suspect the officer observed burglary tools on the backseat

The officer seized the burglary tools He then took the suspect to the county jail booked him for the DUI and placed him in a holding cell Later that day the officer gave the tools he had found in the suspectrsquos car to a detective who was investigating a number of recent burglaries in the neighborhood where the suspect had been arrested

At the time of his DUI arrest the suspect had a six-month-old aggravated assault charge pending against him and was being represented on the assault charge by a lawyer

Early the next morning upon learning of her clientrsquos arrest the lawyer went to the jail She arrived at 900 am immediately identified herself to the jailer as the suspectrsquos attorney and demanded to speak with the suspect The lawyer also told the jailer that she did not want the suspect questioned unless she was present The jailer told the lawyer that she would need to wait one hour to see the suspect After speaking with the lawyer the jailer did not inform anyone of the lawyerrsquos presence or her demands

The detective who had also arrived at the jail at 900 am overheard the lawyerrsquos conversation with the jailer The detective then entered the windowless interview room in the jail where the suspect had been taken 30 minutes earlier Without informing the suspect of the lawyerrsquos presence or her demands the detective read to the suspect full and accurate Miranda warnings The detective then informed the suspect that he wanted to ask about the burglary tools found in his car and the recent burglaries in the neighborhood where he had been arrested The suspect replied ldquoI think I want my lawyer here before I talk to yourdquo The detective responded ldquoThatrsquos up to yourdquo

After a few minutes of silence the suspect said ldquoWell unless there is anything else I need to know letrsquos not waste any time waiting for someone to call my attorney and having her drive here I probably should keep my mouth shut but Irsquom willing to talk to you for a whilerdquo The suspect then signed a Miranda waiver form and after interrogation by the detective made incriminating statements regarding five burglaries The interview lasted from 915 am to 1000 am

In addition to the DUI the suspect has been charged with five counts of burglary

The lawyer has filed a motion to suppress all statements made by the suspect to the detective in connection with the five burglaries

The state supreme court follows federal constitutional principles in all cases interpreting a criminal defendantrsquos rights

3

Criminal Law and Procedure Question

1 Did the detective violate the suspectrsquos Sixth Amendment right to counsel when he questioned the suspect in the absence of the lawyer Explain

2 Under Miranda did the suspect effectively invoke his right to counsel Explain

3 Was the suspectrsquos waiver of his Miranda rights valid Explain

4

CONTRACTS QUESTION

A music conservatory has two concert halls One concert hall had a pipe organ that was in poor repair and the other had no organ The conservatory decided to repair the existing organ and buy a new organ for the other concert hall After some negotiation the conservatory entered into two contracts with a business that both repairs and sells organs Under one contract the business agreed to repair the existing pipe organ for the conservatory for $100000 The business would usually charge a higher price for a project of this magnitude but the business agreed to this price because the conservatory agreed to prepay the entire amount Under the other contract the business agreed to sell a new organ to the conservatory for the other concert hall for $225000 As with the repair contract the business agreed to a low sales price because the conservatory agreed to prepay the entire amount Both contracts were signed on January 3 and the conservatory paid the business a total of $325000 that day

Two weeks later before the business had commenced repair of the existing organ the business suffered serious and unanticipated financial reversals The chief financial officer for the business contacted the conservatory and said

Bad news We had an unexpected liability and as a result are in a real cash crunch In fact even though we havenrsquot acquired the new organ from our supplier or started repair of your existing organ wersquove already spent the cash you gave us and we have no free cash on hand Wersquore really sorry but wersquore in a fix I think that we can find a way to perform both contracts but not at the original prices If you agree to pay $60000 more for the repair and $40000 more for the new organ we can probably find financing to finish everything If you donrsquot agree to pay us the extra money I doubt that we will ever be able to perform either contract and yoursquoll be out the money you already paid us

After receiving this unwelcome news the conservatory agreed to pay the extra amounts provided that the extra amount on each contract would be paid only upon completion of the businessrsquos obligations under that contract The business agreed to this arrangement and the parties quickly signed documents reflecting these changes to each contract The business then repaired the existing organ delivered the new organ and demanded payment of the additional $100000

The conservatory now has refused to pay the business the additional amounts for the repair and the new organ

1 Must the conservatory pay the additional $60000 for the organ repair Explain

2 Must the conservatory pay the additional $40000 for the new organ Explain

5

FAMILY LAW QUESTION

In 1994 a man and a woman were married in State A

In 1998 their daughter was born in State A

In 2010 the family moved to State B

In 2012 the husband and wife divorced in State B Under the terms of the divorce decree

(a) the husband and wife share legal and physical custody of their daughter (b) the husband must pay the wife $1000 per month in child support until their daughter reaches age 18 (c) the marital residence was awarded to the wife with the proviso that if it is sold before the daughter reaches age 18 the husband will receive 25 of the net sale proceeds remaining after satisfaction of the mortgage on the residence and (d) the remaining marital assets were divided between the husband and the wife equally

Six months ago the husband was offered a job in State A that pays significantly less than his job in State B but provides him with more responsibilities and much better promotion opportunities The husband accepted the job in State A and moved from State B back to State A

Since returning to State A the husband has not paid child support because due to his lower salary he has had insufficient funds to meet all his obligations

One month ago the wife sold the marital home netting $10000 after paying off the mortgage She then moved to a smaller residence The husband believes that he should receive more than 25 of the net sale proceeds given his financial difficulties

Last week when the wife brought the daughter to the husbandrsquos State A home for a weekend visit the husband served the wife with a summons in a State A action to modify the support and marital-residence-sale-proceeds provisions of the State B divorce decree The husband brought the action in the State A court that adjudicates all domestic relations issues

1 Does the State A court have jurisdiction to modify (a) the child support provision of the State B divorce decree Explain (b) the marital-residence-sale-proceeds provision of the State B divorce decree

Explain

2 On the merits could the husband obtain (a) retroactive modification of his child support obligation to the daughter Explain (b) prospective modification of his child support obligation to the daughter Explain (c) modification of the marital-residence-sale-proceeds provision of the State B

divorce decree Explain

6

FEDERAL CIVIL PROCEDURE QUESTION

The United States Forest Service (USFS) manages public lands in national forests including the Scenic National Forest Without conducting an environmental evaluation or preparing an environmental impact statement the USFS approved a development project in the Scenic National Forest that required the clearing of 5000 acres of old-growth forest The trees in the forest are hundreds of years old and the forest is home to a higher concentration of wildlife than can be found anywhere else in the western United States

The USFS solicited bids from logging companies to harvest the trees on the 5000 acres of forest targeted for clearing and it ultimately awarded the logging contract to the company that had submitted the highest bid for the trees However the USFS has not yet issued the company a logging permit Once it does so the company intends to begin cutting down trees immediately

A nonprofit organization whose mission is the preservation of natural resources has filed suit in federal district court against the USFS The nonprofit alleges that the USFS violated the National Environmental Policy Act (NEPA) by failing to prepare an environmental impact statement for the proposed logging project Among other remedies the nonprofit seeks a permanent injunction barring the USFS from issuing a logging permit to the logging company until an adequate environmental impact statement is completed The nonprofit believes that the logging project would destroy important wildlife habitat and thereby cause serious harm to wildlife in the Scenic National Forest including some endangered species

Assume that federal subject-matter jurisdiction is available that the nonprofit has standing to bring this action and that venue is proper

1 If the logging company seeks to join the litigation as a party must the federal district court allow it to do so as a matter of right Explain

2 What types of relief could the nonprofit seek to stop the USFS from issuing a logging permit during the pendency of the action what must the nonprofit demonstrate to obtain that relief and is the federal district court likely to grant that relief Explain

7

EVIDENCE QUESTION

A prison inmate has filed a civil rights lawsuit against a guard at the prison alleging that the guard violated the inmatersquos constitutional rights during an altercation The inmate and the guard are the only witnesses to this altercation They have provided contradictory reports about what occurred

The trial will be before a jury The inmate plans to testify at trial The guardrsquos counsel has moved for leave to impeach the inmate with the following

(a) Twelve years ago the inmate was convicted of felony distribution of marijuana He served a three-year prison sentence which began immediately after he was convicted He served his full sentence and was released from prison nine years ago (b) Eight years ago the inmate pleaded guilty to perjury a misdemeanor punishable by up to one year in jail He paid a $5000 fine (c) Seven years ago the inmate was convicted of felony sexual assault of a child and is currently serving a 10-year prison sentence for the crime The victim was the inmatersquos daughter who was 13 years old at the time of the assault

The inmatersquos counsel objects to the admission of any evidence related to these three convictions and to any cross-examination based on this evidence

The guard also plans to testify at trial The inmatersquos counsel has moved for leave to impeach the guard with the following

Last year the guard applied for a promotion to prison supervisor The guard submitted a reacutesumeacute to the state that indicated that he had been awarded a BA in Criminal Justice from a local college An official copy of the guardrsquos academic transcript from that college indicates that the guard dropped out after his first semester and did not receive a degree

The guardrsquos counsel objects to the admission of this evidence and to any cross-examination based on this evidence

The transcript and the reacutesumeacute have been properly authenticated The trial will be held in a jurisdiction that has adopted all of the Federal Rules of Evidence

1 What evidence if any proffered by the guard to impeach the inmate should be admitted Explain

2 What evidence if any proffered by the inmate to impeach the guard should be admitted Explain

8

CORPORATIONS QUESTION

Mega Inc is a publicly traded corporation incorporated in a state whose corporate statute is modeled on the Model Business Corporation Act (MBCA) Megarsquos articles of incorporation do not address the election of directors or amendment of the bylaws by shareholders

Well within the deadline for the submission of shareholder proposals for the upcoming annual shareholdersrsquo meeting an investor who was a large and long-standing shareholder of Mega submitted a proposed amendment to Megarsquos bylaws The proposal which the investor asked to be included in the corporationrsquos proxy materials and voted on at the upcoming shareholdersrsquo meeting read as follows

Section 20 The Corporation shall include in its proxy materials (including the proxy ballot) for a shareholdersrsquo meeting at which directors are to be elected the name of a person nominated for election to the Board of Directors by a shareholder or group of shareholders that beneficially have owned 3 or more of the Corporationrsquos outstanding common stock for at least one year

This Section shall supersede any inconsistent provision in these Bylaws and may not be amended or repealed by the Board of Directors without shareholder approval

Megarsquos management decided to exclude the investorrsquos proposal from the corporationrsquos proxy materials and explained its reasons in a letter to the investor

The investorrsquos proposed bylaw provision would be inconsistent with relevant state law because the Board of Directors has the authority to manage the business and affairs of the Corporation Generally shareholders lack the authority to interfere with corporate management by seeking to create a method for the nomination and election of directors inconsistent with the method chosen by the Board of Directors

Furthermore at its most recent meeting the Board of Directors unanimously approved an amendment to the Corporationrsquos bylaws that provides for proxy access for director nominations by a shareholder or a group of shareholders holding at least 10 of the Corporationrsquos voting shares for at least three years This procedure takes precedence over any nomination methods that might be sought or approved by shareholders

The investor is considering bringing a suit challenging managementrsquos refusal to include the investorrsquos proposed bylaw provision and challenging the boardrsquos amendment of the bylaws at its recent meeting

1 Is the investorrsquos proposed bylaw provision inconsistent with state law Explain

2 If the investorrsquos proposed bylaw provision were approved by the shareholders would the bylaw amendment previously approved by the board take precedence over the investorrsquos proposed bylaw provision Explain

3 Must the investor make a demand on Megarsquos board of directors before bringing suit Explain

9

July 2014 MEE

ANALYSES

Contracts Family Law

Criminal Law and Procedure

Federal Civil Procedure Evidence

Corporations

CRIMINAL LAW AND PROCEDURE ANALYSIS (Criminal Law and Procedure VA B D)

ANALYSIS

Legal Problems

(1) Did the detective violate the suspectrsquos Sixth Amendment right to counsel when he questioned the suspect about the burglaries without the lawyer present given that the lawyer represented the suspect in an unrelated criminal matter

(2) Under Miranda did the suspect effectively invoke his right to counsel when he said ldquoI think I want my lawyer here before I talk to yourdquo

(3) Was the suspectrsquos waiver of his right to remain silent under Miranda valid

DISCUSSION

Summary

The Sixth Amendment right to counsel as applied to states through the Fourteenth Amendment is offense-specific Although the suspect had an attorney representing him on his pending assault charge he had no Sixth Amendment right to the assistance of counsel with respect to the five uncharged burglaries because formal adversarial proceedings had not yet commenced on those charges The suspectrsquos Sixth Amendment right to counsel was not violated by the detectiversquos failure to inform him that the lawyer was present or of the lawyerrsquos demands

However a person undergoing custodial interrogation also has an independent constitutional right to counsel during custodial interrogation under Miranda When a suspect invokes his right to counsel under Miranda custodial interrogation must immediately cease for a period of at least 14 days However the invocation of the right to counsel must be unambiguous and clearly convey that the suspect has requested counsel Here because the suspectrsquos statement ldquoI think I want my lawyer here before I talk to yourdquo was ambiguous he did not invoke his Miranda right to counsel

A waiver of rights must be knowing intelligent and voluntary Here the suspect waived his right to remain silent under Miranda when he signed the waiver form The fact that the detective did not correct the suspectrsquos assumption that the lawyer would need to drive to the jailmdashby telling him that the lawyer was in the waiting room and was demanding to see himmdashdid not affect the validity of the suspectrsquos waiver

Point One (35) The suspectrsquos Sixth Amendment right to counsel was not violated because the right does not attach on new charges until formal adversarial judicial proceedings have commenced on those charges

The Sixth Amendment as applied to the states through the Fourteenth Amendment provides that ldquo[i]n all criminal prosecutions the accused shall enjoy the right to have the Assistance of Counsel for his defenserdquo The right to counsel does not attach with respect to particular charges until formal adversarial judicial proceedings have commenced (ie ldquoat or after the initiation of

13

Criminal Law and Procedure Analysis

adversary judicial criminal proceedingsmdashwhether by way of formal charge preliminary hearing indictment information or arraignment [or in some states arrest warrant]rdquo McNeil v Wisconsin 501 US 171 175 (1991) (internal quotations omitted)) Once a suspectrsquos Sixth Amendment right to counsel has attached any attempts to ldquodeliberately elicitrdquo statements from him in the absence of his attorney violate the Sixth Amendment See Massiah v United States 377 US 201 (1964) Brewer v Williams 430 US 387 (1977)

The Sixth Amendment right to counsel is charge- or offense -specific Representation by counsel in one prosecution does not in itself guarantee counsel for uncharged offenses See McNeil 501 US at 175 Texas v Cobb 532 US 162 (2001) Here the suspectrsquos Sixth Amendment right to counsel had attached only for the pending aggravated assault charge The suspectrsquos right to counsel for the aggravated assault case did not guarantee counsel for the five unrelated and uncharged burglaries that were the subject of the detectiversquos interrogation Thus because formal adversarial judicial proceedings against the suspect for the uncharged burglaries had not begun he had no Sixth Amendment right to counsel

Finally the detectiversquos failure to inform the suspect of the lawyerrsquos presence and demands to speak with him does not implicate the suspectrsquos Sixth Amendment right to counsel which had not yet attached See id Moran v Burbine 475 US 412 428ndash31 (1986)

Point Two (30) The suspect did not effectively invoke his right to counsel under Miranda because his statement was not unambiguous

A suspect subject to custodial interrogation has a right to consult with counsel and to have an attorney present during questioning Miranda v Arizona 384 US 436 (1966) When a suspect invokes his right to counsel during an interrogation law enforcement must immediately cease all questioning See Edwards v Arizona 451 US 477 484ndash85 (1981) Custodial interrogation cannot be reinitiated unless and until the suspect has been re-advised of his Miranda rights has provided a knowing and voluntary waiver and (1) counsel is present and (2) the suspect himself initiated further communication with the police see id at 484 or (3) (if the suspect was released from custody after the initial interrogation) at least 14 days have passed Maryland v Shatzer 559 US 98 110 (2010)

To invoke the right to counsel a suspectrsquos request must be ldquounambiguousrdquo This means that the suspect must articulate the desire for counsel sufficiently clearly that a reasonable officer would understand the statement to be a request for counsel Davis v United States 512 US 452 459 (1994) If the request is ambiguous the police are not required to stop the interrogation

In this case the suspectrsquos statement ldquoI think I want my lawyer here before I talk to yourdquo was not an unambiguous request for counsel The most reasonable interpretation of this statement is that the suspect might be invoking his right to counsel Id at 461 (ldquomaybe I should talk to a lawyerrdquo is not an unequivocal request for counsel) See also Burket v Angelone 208 F3d 172 197ndash98 (4th Cir 2000) (ldquoI think I need a lawyerrdquo is not an unambiguous request for an attorney) Soffar v Cockrell 300 F3d 588 594ndash95 (5th Cir 2002) (discussion of various statements that did not constitute unequivocal requests for counsel)

Under these circumstances the detective was not required to cease the custodial interrogation of the suspect Nor was the detective required to clarify or ask follow-up questions to determine whether the suspect in fact wanted an attorney Davis 512 US at 459ndash60

14

Criminal Law and Procedure Analysis

Point Three (35) The suspectrsquos waiver of his Miranda rights was knowing intelligent and voluntary despite the fact that he was never told of the lawyerrsquos presence in the jail or of the lawyerrsquos demands

A valid waiver of Miranda rights must be ldquovoluntaryrdquomdashie the product of a free or deliberate choice rather than intimidation coercion or deception Berghuis v Thompkins 560 US 370 382ndash83 (2010) In addition the waiver must be knowing and intelligent That is it ldquomust have been made with a full awareness of both the nature of the right being abandoned and the consequences of the decision to abandon itrdquo Moran v Burbine 475 US 421 (1986)

In this case the suspect signed a Miranda waiver form after receiving proper warnings There is no evidence ldquothat the police resorted to physical or psychological pressure to elicit the statementsrdquo Id The entire interview lasted only 45 minutes The only issue is whether the suspect knowingly and intelligently waived his Miranda rights despite the fact that the detective did not tell the suspect about the lawyerrsquos presence and her demands

The Supreme Court has said that ldquo[e]vents occurring outside of the presence of the suspect and entirely unknown to him surely can have no bearing on the capacity to comprehend and knowingly relinquish a constitutional rightrdquo Id at 422 If the suspect ldquoknew that he could stand mute and request a lawyer and was aware of the Statersquos intention to use his statements to secure a convictionrdquo then the waiver is valid regardless of the information withheld Id at 422ndash23

Here the suspect was correctly informed of his rights Miranda v Arizona 384 US at 467ndash73 His comments demonstrate that he understood that he could have a lawyer present if he desired (ie wondering whether he should call his attorney) and that he understood that there might be consequences to speaking with the detective (ldquoI probably should keep my mouth shut but Irsquom willing to talk to you for a whilerdquo) His comment ldquo[L]etrsquos not waste any time waiting for someone to call my attorney and having her drive hererdquo along with his signature on the Miranda waiver form show that his waiver was valid under the constitutional standard

The fact that the detective did not tell the suspect about the lawyerrsquos presence and demands has no bearing on the validity of the suspectrsquos waiver because ldquosuch conduct is only relevant to the constitutional validity of a waiver if it deprives a defendant of knowledge essential to his ability to understand the nature of his rights and the consequences of abandoning themrdquo Moran at 424 The Supreme Court has specifically declined to adopt a rule requiring that law enforcement tell a suspect of an attorneyrsquos efforts to contact him id at 425 (ldquoNor are we prepared to adopt a rule requiring that the police inform a suspect of an attorneyrsquos efforts to reach himrdquo)

[NOTE An examinee might also recognize that this general rule is further supported by the Supreme Courtrsquos decision in Florida v Powell 559 US 50 (2010) approving state Miranda warnings that do not explicitly warn suspects that they have a right to have counsel present during custodial interrogation]

15

CONTRACTS ANALYSIS (Contracts IB2 IIB IVA3 amp A5)

ANALYSIS

Legal Problems

(1) In the case of a service contract (governed by the common law of contracts) is a modification enforceable when a party agrees to pay more for the same performance than was originally promised

(2) In the case of a contract for the sale of goods (governed by Article 2 of the UCC) is a modification enforceable when a party agrees to pay more for the same goods than was originally promised

(3) May a party avoid an agreement on the basis of economic duress

DISCUSSION

Summary

There are two arguments that the conservatory can make to support the claim that it is not bound to pay the higher prices lack of consideration and economic duress

The organ repair contract is governed by the common law of contracts Under the common law the business would have difficulty recovering the additional $60000 for the organ repair because under the ldquopreexisting duty rulerdquo the agreement of the conservatory to pay the extra price was not supported by consideration However the business might argue that the modification is enforceable under an exception to the preexisting duty rule for fair and equitable modifications made in light of unanticipated circumstances

The organ sale contract is governed by Article 2 of the Uniform Commercial Code The business would likely recover the additional amount under that contract because Article 2 provides that consideration is not required for a modification to be binding

In both cases the conservatory could seek to avoid its agreement on the grounds of economic duress but that argument is not likely to succeed

Point One (45) The business probably cannot recover the additional $60000 for the organ repair because the conservatoryrsquos promise to pay more money was not supported by consideration

The general rule is that to be enforceable a promise must be supported by consideration Under RESTATEMENT (SECOND) OF CONTRACTS sect 71 a promise is supported by consideration if it is bargained for in exchange for a return promise or performance However under the ldquopreexisting duty rulerdquo (exemplified in RESTATEMENT (SECOND) OF CONTRACTS sect 73 and Alaska Packersrsquo Assrsquon v Domenico 117 F 99 (9th Cir 1902)) promise of performance of a legal duty already owed to a promisor which is neither doubtful nor the subject of honest dispute is not consideration

If the business had promised the conservatory anything new or different in exchange for the agreement to pay the additional $60000 (such as for example repairing the pipe organ more

16

Contracts Analysis

quickly or using better parts) that would constitute consideration especially in light of the principle that courts do not inquire into the adequacy of consideration Here however the business already had a legal duty under the original contract and did not agree to do anything else in exchange for the conservatoryrsquos promise to pay $60000 more

However an exception to the preexisting duty rule is sometimes applied in situations of unanticipated changed circumstances Under RESTATEMENT (SECOND) OF CONTRACTS sect 89 followed in many jurisdictions a promise modifying a duty under a contract not fully performed on either side is binding even if not supported by consideration if the modification is fair and equitable in view of circumstances not anticipated by the parties when the contract was made

If a court applies the rule in Restatement sect 89 the critical issues will be whether the modification was in fact ldquofair and equitablerdquo and whether it can be justified in light of unanticipated circumstances In many cases in which modifications have been upheld a party encountered difficulties or burdens in performing far beyond what was knowingly bargained for in the original contract with the result bordering on impracticability such as having to excavate solid rock instead of soft dirt or having to remove garbage far in excess of the amounts contemplated The conservatory would argue that the businessrsquos performance difficulties were not of this sort at allmdashnothing about repairing the pipe organ itself was any different from or more difficult than originally contemplated except that the business itself encountered financial distress unrelated to its burdens in performing its obligations under these contracts

Even if the business satisfies that element of the rule in Restatement sect 89 the business must also demonstrate that the circumstances that gave rise to the need to modify the contract were ldquounanticipatedrdquo at the time the original contract was made Here the facts suggest that when the business entered into the original contract it expected that the price paid by the conservatory would enable it to perform However any evidence that the business knew or had reason to know at the time of execution that it would need more money from the conservatory to be able to perform would mean that the request to modify was not ldquounanticipatedrdquo

[NOTE Some cases such as Schwartzreich v Bauman-Basch Inc 231 NY 196 131 NE 887 (1921) find that if the parties mutually agreed to rescind the original contract and then after rescission entered into an entirely new contract for a higher price the new contract is supported by consideration There is no evidence that such a rescission followed by a new contract took place here]

Point Two (45) The business can recover the additional $40000 for the new organ because no consideration is required under Article 2 of the UCC for good-faith contract modifications

The contract to buy a new organ is a contract for the sale of goods and therefore is governed by Article 2 of the Uniform Commercial Code UCC sect 2-102 Under Article 2 unlike the common law an agreement modifying a contract needs no consideration to be binding UCC sect 2-209(1) Section 2-209(1) thus obviates the preexisting duty rule entirely in contracts for the sale of goods

Even though consideration is not required modifications governed by sect 2-209 must satisfy the obligation of good faith imposed by the UCC UCC sect 1-304 See also Official Comment 2 to UCC sect 2-209 Good faith means ldquohonesty in fact and the observance of reasonable commercial standards of fair dealingrdquo UCC sect 1-201(b)(20) In this context the obligation of good faith means that ldquo[t]he effective use of bad faith to escape performance on the original contract terms is barred and the extortion of a lsquomodificationrsquo without legitimate commercial reason is ineffective as a violation of the duty of good faithrdquo Official Comment 2 to

17

Contracts Analysis

UCC sect 2-209 Here because the businessrsquos financial reversals were serious and apparently unanticipated at the time that the business entered into the contract with the conservatory and commitment of the extra money was needed to enable the business to perform a court would likely find that the business acted in good faith Thus a court would likely uphold the enforceability of the conservatoryrsquos promise to pay the additional $40000

Point Three (10) The conservatory is unlikely to be able to defend against enforcement of its promises to pay additional money under the theory of economic duress because the business probably did not make an improper threat

Under the common law of contracts parties may raise the defense of duress This common law defense also applies to contracts governed by UCC Article 2 See UCC sect 1-103(b)

A contract is voidable on the ground of economic duress by threat when it is established that a partyrsquos manifestation of assent is induced by an improper threat that leaves the party no reasonable alternative See RESTATEMENT (SECOND) OF CONTRACTS sect 175 See also eg Austin Instrument Inc v Loral Corp 272 NE2d 533 (NY 1971) (a threat to withhold essential goods can constitute duress) In order to void its agreement to pay the additional sum because of economic duress the conservatory must demonstrate that (1) the business made a threat to the conservatory (2) the threat was ldquoimproperrdquo or ldquowrongfulrdquo (3) the threat induced the conservatoryrsquos manifestation of assent to the modification and (4) the threat was sufficiently grave to justify the conservatoryrsquos assent

Here it appears that three of the four elements are likely satisfied The business plainly made a threat Moreover the threat induced the conservatoryrsquos assent to the modification and the threat was sufficiently grave to justify that assent If the conservatory had not agreed to pay the business the extra amounts the conservatory would have lost its entire $325000 investment In light of this potential loss a court could easily conclude that the conservatory had no reasonable alternative

However the business has a strong argument that its threat (indicating that it would breach the contracts unless the prices were increased) was not wrongful or improper but was instead nothing more than a communication of the reality of its own perilous situation to the conservatory

A mere threat to breach a contract is not in and of itself improper so as to support an action of economic duress or business compulsion Something more is required such as a breach of the duty of good faith and fair dealing as was present in Austin Instrument Inc supra Because the business could not perform the original contract without the requested modification the economic duress claim for the conservatory would likely fail for much the same reason that the business would be able to enforce the modification At the time the modification was requested the business was not trying to extort a price increase because of the conservatoryrsquos vulnerability but instead was simply stating the reality that the business could not perform without more money

18

FAMILY LAW ANALYSIS (Family Law IIIB D amp G)

ANALYSIS

Legal Problems

(1)(a) Does the State A court have jurisdiction to modify the State B child support order

(1)(b) Does the State A court have jurisdiction to modify the marital-residence-saleshyproceeds provision of the State B property-division decree

(2)(a) May a child support order be modified retroactively

(2)(b) May a child support order be modified prospectively based on a change of employment with a lower salary

(2)(c) May a property-division order be modified after entry of a divorce decree

DISCUSSION

Summary

The State A court may exercise personal jurisdiction over the wife because she was personally served in State A However subject-matter jurisdiction over the interstate modification of child support is governed by the Uniform Interstate Family Support Act (UIFSA) Under UIFSA State A does not have jurisdiction to modify the order for the daughterrsquos support because the wife is still a resident of State B UIFSA on the other hand does not govern property distributions and thus a State A court is not precluded from hearing the husbandrsquos petition to modify the marital-residence-sale-proceeds provision of the divorce decree

A child support order may not be modified retroactively A child support order may be modified prospectively based on a substantial change in circumstances Courts agree that a significant decrease in income is a substantial change in circumstances All states treat voluntary income reductions differently than involuntary reductions but employ different approaches for evaluating the impact of a voluntary reduction Whether the husband could obtain prospective modification of the child support order depends on which approach is applied

A property-division order is not subject to post-divorce modification based on a change in circumstances Thus the husband may in some states obtain prospective modification of the order for the daughterrsquos support but he may not obtain modification of the marital-residenceshysale-proceeds provision

Point One(a) (25) Personal jurisdiction over a nonresident respondent does not confer subject-matter jurisdiction over child support modification Under UIFSA a State A court may not modify a child support order issued by a State B court when as here the child or either parent continues to reside in State B the jurisdiction that issued the child support order

The State A court may exercise personal jurisdiction over the wife The wife was personally served in State A and a state may exercise jurisdiction based on in-state personal service See

19

Family Law Analysis

Burnham v Superior Court 495 US 604 (1990) But personal jurisdiction over the wife is not enough to give a State A court jurisdiction to modify the State B support order

The interstate enforcement and modification of child support is governed by the Uniform Interstate Family Support Act (UIFSA) which has been adopted by all states Under UIFSA the state that originally issued a child support order (here State B) has continuing exclusive jurisdiction to modify the order if that state remains the residence of the obligee the child or the obligor and all parties do not consent to the jurisdiction of another forum See UIFSA sect 205 See also UIFSA sect 603 (ldquoA tribunal of this State shall recognize and enforce but may not modify a registered order if the issuing tribunal had jurisdictionrdquo) The wife and daughter continue to reside in State B and the wife has not consented to the jurisdiction of another forum Thus a State A court does not have jurisdiction to modify the State B child support order

[NOTE Examinees who do not discuss personal jurisdiction but fully discuss UIFSA may receive full credit]

Point One(b) (15) UIFSA does not apply to disputes over property division Thus the State A court may exercise jurisdiction over the husbandrsquos petition to modify the marital-residence-sale-proceeds provision of the State B divorce decree because it has personal jurisdiction over the wife

The State A court in which the husband brought his action has jurisdiction to adjudicate domestic relations issues The husbandrsquos petition to modify the property settlement is a domestic relations issue The courts of State A may exercise personal jurisdiction over the wife because she was personally served in State A See Burnham v Superior Court 495 US 604 (1990) see Point One(a)

UIFSA does not apply to divorce property-division disputes Thus although a State A court may not adjudicate the husbandrsquos petition to modify his child support obligations it may adjudicate his property-division claims (Even though the court has jurisdiction it may not modify the property-division award on the merits See Point Two(c))

Point Two(a) (20) A child support order may not be modified retroactively

State courts have long held that obligations to pay child support ordinarily may not be modified retroactively ldquoIf the hardship is particularly severe the courts sometimes devised a way to protect the obligor but in most instances the courts hold that retroactive modification of this kind is beyond their power and indeed the governing statute may so providerdquo HOMER H CLARK THE LAW OF DOMESTIC RELATIONSHIPS IN THE UNITED STATES 725 (2d ed 1987)

Federal law now goes further and requires the states as a condition of federal child-support funding to adopt rules that absolutely forbid retroactive modification of the support obligation See 42 USC sect 666(a)(9)(C) The states have adopted rules consistent with the federal requirements

Point Two(b) (25) It is unclear whether the husband could obtain prospective downward modification of his child support based on his voluntary acceptance of a job with a lower salary

Prospective modification of a child support order is typically available only when the petitioner can show a substantial change in circumstances See ROBERT E OLIPHANT amp NANCY VER

20

Family Law Analysis

STEEGH FAMILY LAW 213ndash15 (3d ed 2010) A significant decrease in income is typically viewed as a substantial change

However when a parent seeks to modify a child support obligation because he has voluntarily reduced his income a court will not modify the obligation based solely on the income loss Some courts refuse to modify whenever the income shift was voluntary See eg Aguiar v Aguiar 127 P3d 234 (Idaho Ct App 2005) Others look primarily to the petitionerrsquos intentions and permit downward modification if he has acted in good faith See eg In re Marriage of Horn 650 NE2d 1103 (Ill App Ct 1995) Many courts use a multifactor approach See OLIPHANT amp VER STEEGH supra 217ndash18

Here there is no question that the husbandrsquos loss of income was voluntary In a jurisdiction in which voluntary income reduction bars support modification the husbandrsquos petition would be denied

In a jurisdiction employing a good-faith or multifactor approach it is possible but not certain that the husband could obtain downward modification The evidence supports the husbandrsquos good faith his change in employment appears to be based on his new jobrsquos greater responsibilities and better promotion possibilities In a jurisdiction using a multifactor approach the court would likely also consider the impact of such a shift on the daughter the likely duration of the husbandrsquos income loss and the likelihood of a promotion that would ultimately inure to the daughterrsquos benefit Thus on these facts it is possible but by no means certain that the husband could prospectively obtain downward modification of his child support obligation to his daughter

Point Two(c)(15) A divorce property-division award is not subject to modification

A support order is aimed at meeting the post-divorce needs of the supported individual Because the future is unpredictable courts are empowered to modify a support award to take account of changed circumstances that may occur during the period in which support is paid

By contrast a property-distribution award divides assets of the marriage based on the equities at the time of divorce Because the past can be ascertained a property-division award is not subject to post-divorce modification See HARRY A KRAUSE ET AL FAMILY LAW CASES COMMENTS AND QUESTIONS 691 (6th ed 2007)

Here the husband is seeking modification of a property-division award with respect to an asset owned by the parties at the time of divorce Thus the husband may not obtain a modification of the marital-residence-sale-proceeds provision of the divorce decree based on his reduced income

21

FEDERAL CIVIL PROCEDURE ANALYSIS (Federal Civil Procedure III IVC)

ANALYSIS

Legal Problems

(1) Is the logging company entitled to join this action as a matter of right

(2)(a) May the nonprofit organization obtain a temporary restraining order to stop the USFS from issuing a logging permit

(2)(b) May the nonprofit organization obtain a preliminary injunction to stop the USFS from issuing a logging permit during the pendency of the action

DISCUSSION

Summary

The logging company is entitled to intervene in this action as a matter of right because it has an interest in the property or transaction that is the subject of the action and is so situated that its interest may be impaired or impeded as a practical matter if the action goes forward without it The logging companyrsquos interest is not adequately represented by the USFSrsquos presence in the lawsuit

The nonprofit organization may seek a temporary restraining order (TRO) followed by a preliminary injunction to prevent the USFS from issuing a logging permit pending the outcome of the action The nonprofit is likely to obtain a TRO if it can demonstrate a risk of immediate and irreparable injury The nonprofit is also likely to obtain a preliminary injunction if it can demonstrate a significant threat of irreparable harm and a likelihood of success on the merits of its National Environmental Policy Act (NEPA) claim

Point One (50) Rule 24(a) of the Federal Rules of Civil Procedure requires federal courts to allow a person to intervene in an action as a matter of right if the person a) is interested in the property or transaction that is the subject of the action b) is so situated that its interest may be impaired or impeded if the litigation goes forward without it and c) is not adequately represented by existing parties Here the logging company likely meets all three requirements and should be allowed to intervene as a matter of right

Rule 24 of the Federal Rules of Civil Procedure governs intervention the process by which a non-party to an action may join the litigation Under Rule 24(a) (intervention of right) a person must be permitted to intervene if three conditions are met (1) the movant ldquoclaims an interest relating to the property or transaction that is the subject of the actionrdquo (2) the movant ldquois so situated that disposition of the action may as a practical matter impair or impede the movantrsquos ability to protect its interestrdquo and (3) ldquoexisting partiesrdquo do not ldquoadequately represent [the movantrsquos] interestrdquo FED R CIV P 24(a) The three requirements for intervention of right are often ldquovery interrelatedrdquo 7C CHARLES ALAN WRIGHT ET AL FEDERAL PRACTICE AND PROCEDURE sect 1908 at 297 (2007 amp 2011 Supp)

22

Federal Civil Procedure Analysis

Here the court should find that the logging company meets this test First the logging company has a strong interest in the property or transaction that is the subject of this action The USFS has accepted the logging companyrsquos bid and the logging company is merely awaiting issuance of a logging permit to begin logging The nonprofit organization is seeking to prevent this logging The logging company therefore has a strong direct and substantial interest in the subject matter of the lawsuit and in having its winning bid honored and a logging permit issued See eg Kleissler v US Forest Serv 157 F3d 964 972 (3d Cir 1998) (stating that ldquo[t]imber companies have direct and substantial interests in a lawsuit aimed at halting loggingrdquo) see also Natural Resources Defense Council v US Nuclear Regulatory Commrsquon 578 F2d 1341 1343ndash 44 (10th Cir 1978) (holding that applicants whose license renewals were pending had Rule 24(a)(2) interests where the lawsuit sought to halt the license-issuing process pending preparation of environmental impact statements) See generally 7C WRIGHT ET AL supra sect 19081 at 309 (ldquoIf there is a direct substantial legally protectable interest in the proceedings it is clear that this requirement of the rule is satisfiedrdquo) Second the logging companyrsquos interest in receiving a logging permit may well be impaired as a practical matter by the outcome of the lawsuit If the USFS loses the lawsuit it will have to prepare an environmental impact statement before issuing the logging companyrsquos permit This will at a minimum delay the logging companyrsquos ability to exercise its rights and may in the long r un mean that no logging permit is ever issued Intervention of right is not limited to those that would be legally bound as a matter of preclusion doctrine Id sect 19082 at 368 Rather ldquo[t]he rule is satisfied whenever disposition of the present action would put the movant at a practical disadvantage in protecting its interestrdquo Id sect 19082 at 369 Here that condition is easily satisfied See Kleissler 157 F3d at 972 (ldquoTimber companies have direct and substantial interests in a lawsuit aimed at halting logging rdquo)

Given that the logging company has an interest that may be impaired by disposition of the action it should be allowed to intervene unless the court is persuaded that the USFS adequately represents the logging companyrsquos interest See Rule 24(a)(2) 7C WRIGHT ET AL supra sect 1909 Here it could be argued that the USFS adequately represents the logging companyrsquos interest because the USFS presumably wants the court to uphold its development plan and allow it to proceed with issuance of the logging permit which is the same relief that the logging company would seek However whether representation is truly adequate depends upon ldquo[a] discriminating appraisal of the circumstancesrdquo 7C WRIGHT ET AL supra sect 1909 at 440 Although both the government and the logging company wish to avoid the preparation of an environmental impact statement their interests are distinct The USFSrsquos interest is proper management of the national forest system while the logging companyrsquos interest is making a profit from logging the 5000-acre tract The USFSrsquos handling of the litigation is likely to be affected by a variety of policy concerns and political considerations that have nothing to do with the logging companyrsquos purely economic interest in securing the right to cut trees in the Scenic National Forest See eg Kleissler 157 F3d at 973ndash74 (ldquo[T]he government represents numerous complex and conflicting interests in matters of this nature The straightforward business interests asserted by intervenors here may become lost in the thicket of sometimes inconsistent governmental policiesrdquo)

[NOTES (1) Examinees who mistakenly analyze the logging companyrsquos case for joinder under the related but incorrect Rule 19 ldquoRequired Joinder of Partiesrdquo may receive credit Rule 19 allows existing parties to demand joinder of non-parties (or seek dismissal of the case if they canrsquot get it) There is a close relationship between Rule 24 and Rule 19 and both contain a similar standard for determining when ldquointerestedrdquo third parties are ldquoentitledrdquo or ldquorequiredrdquo to be in the lawsuit Indeed the two prongs of the Rule 24 intervention test that are discussed above

23

Federal Civil Procedure Analysis

are nearly identical to the two prongs of the Rule 19(a) required joinder test Examinees who discuss and apply the test should receive credit even if they cite Rule 19 rather than Rule 24

(2) Examinees may discuss permissive joinder Although permissive joinder is a possibility here the question asks only whether the logging company can join the action as a matter of right and a permissive joinder analysis is not responsive to the question To the extent an examinee discusses permissive joinder the analysis will focus on whether the logging company ldquohas a claim or defense that shares with the main action a common question of law or factrdquo FED R CIV P 24(b)(1)(B) The district court also ldquomust consider whether the intervention will unduly delay or prejudice the adjudication of the original partiesrsquo rightsrdquo FED R CIV P 24(b)(3) On our facts the logging companyrsquos claim for the issuance of a logging permit would certainly share common questions of law and fact with the USFSrsquos defense against the nonprofitrsquos claim There are no facts suggesting that the logging companyrsquos presence would unduly delay or otherwise prejudice adjudication of the original action Thus the district court would have discretion to permit the logging company to intervene even if it denied intervention of right]

Point Two(a) (25) The nonprofit organization could seek and would likely obtain a temporary restraining order to stop the USFS from issuing a logging permit pending a hearing on an application for a preliminary injunction

The first type of interim relief the nonprofit could seek to stop the USFS from issuing a logging permit to the logging company is a temporary restraining order (TRO) prohibiting the USFS from issuing the logging permit A TRO can be issued without notice to the adverse party but only in limited circumstances and only for a limited time FED R CIV P 65(b) To secure a TRO without notice the nonprofit would need to submit an affidavit containing specific facts that demonstrate a risk of ldquoimmediate and irreparable injuryrdquo if a permit is issued FED R CIV P 65(b)(1) In deciding whether to grant a TRO courts will also consider the same factors that are relevant in deciding whether to grant a preliminary injunction (eg the moving partyrsquos likelihood of success on the merits the balance of hardships and the public interest) See Point Two(b) infra The TRO would last only long enough for the court to consider and resolve a request by the nonprofit for a preliminary injunction but no longer than 14 days (unless the court extends it for good cause or the adverse party consents to an extension) In addition bond is required

Here the court is likely to grant the nonprofitrsquos request The nonprofit could plausibly claim that cutting down 5000 acres of old-growth forest in an area that is home to the highest concentration of wildlife in the western United States would have ldquoan immediate and irreparablerdquo adverse impact on the environment and cause irreparable harm to the nonprofitrsquos interest in preserving and protecting natural resources including wildlife habitat

Point Two(b) (25) The nonprofit could also seek and would likely obtain a preliminary injunction to stop the USFS which is likely to be granted if the nonprofitrsquos claim that the USFS violated NEPA has a strong basis in fact and law

Because the TRO would be temporary the nonprofit would need to move for a preliminary injunction to prevent the USFS from issuing a logging permit throughout the pendency of the litigation Preliminary injunctions are injunctions that seek to ldquoprotect [the] plaintiff from

24

Federal Civil Procedure Analysis

irreparable injury and to preserve the courtrsquos power to render a meaningful decision after a trial on the meritsrdquo 11A CHARLES ALAN WRIGHT ET AL FEDERAL PRACTICE AND PROCEDURE sect 2947 at 112 (2013) Rule 65 of the Federal Rules of Civil Procedure sets out the procedural requirements for preliminary injunctions Preliminary injunctions may be granted only upon notice to the adverse party FED R CIV P 65(a)(1) and only if the movant ldquogives security in an amount that the court considers proper to pay the costs and damages sustained by any party found to have been wrongfully enjoined or restrainedrdquo FED R CIV P 65(c)

While Rule 65 sets out the procedural requirements for preliminary injunctive relief it does not specify the substantive grounds upon which it may be granted The courtrsquos discretion in ruling upon a motion for a preliminary injunction ldquois exercised in conformity with historic federal equity practicerdquo 11A WRIGHT ET AL supra sect 2947 at 114 The court typically considers four factors

(1) the significance of the threat of irreparable harm to the plaintiff if the injunction is not granted (2) the balance between this harm and the injury that granting the injunction would inflict on the defendant (3) the probability that the plaintiff will succeed on the merits and (4) the public interest

Id sect 2948 at 122ndash24 accord Habitat Educ Center v Bosworth 363 F Supp 2d 1070 1088 (ED Wis 2005) The most important of these factors is the risk of irreparable harm to the plaintiff 11A WRIGHT ET AL supra sect 29481 at 129 If the plaintiff has an adequate remedy at law (eg if money damages can compensate the plaintiff for its loss) then a preliminary injunction will be denied Id sect 29481

Here a court would likely conclude that the potential for environmental damage to the forest creates a significant threat of irreparable harm ldquo[E]nvironmental injury is often irreparable Courts have recognized that logging such as would occur [here] can have longshyterm environmental consequences and thus satisfy the irreparable injury criterionrdquo Habitat Educ Center 363 F Supp 2d at 1089 (citing Idaho Sporting Congress Inc v Alexander 222 F3d 562 569 (9th Cir 2000) (noting that the imminent and continuing logging activities presented ldquoevidence of environmental harm sufficient to tip the balance in favor of injunctive reliefrdquo)) Neighbors of Cuddy Mountain v US Forest Service 137 F3d 1372 1382 (9th Cir 1998) (stating that ldquo[t]he old growth forests plaintiffs seek to protect would if cut take hundreds of years to reproducerdquo) (internal citation omitted)) see also 11C WRIGHT ET AL supra sect 29481 at 151 (noting that ldquoa preliminary injunction has been issued to prevent harm to the environmentrdquo)

The second factor the balance between the harm to the plaintiff and the harm the defendant will suffer if the injunction is issued also appears to support issuance of a preliminary injunction here The USFS will have to wait before it can develop the Scenic National Forest and the logging company may lose money if the delay is prolonged These economic harms could be compensated monetarily if an injunction is issued inappropriately Where ldquoan injunction bond can compensate [the] defendant for any harm the injunction is likely to inflict the balance should be struck in favor of [the] plaintiffrdquo Id sect 29482 at 192 See also Habitat Educ Center 363 F Supp 2d at 1089 (stating that ldquothe relative absence of harmful effects on the Forest Service weighs in favor of granting the injunctionrdquo)

The third factor is the likelihood that the plaintiff will prevail on the merits Although there is limited information concerning the merits of the action the nonprofit alleges that the federal statute (NEPA) requires an environmental impact statement and further states that the USFS created no environmental impact analysis or statement at all Assuming that those

25

Federal Civil Procedure Analysis

allegations are correct it seems plausible to conclude that the nonprofit will be able to show a likelihood of success on the merits

Finally courts deciding whether or not to issue preliminary injunctive relief are to consider the public interest ldquoFocusing on this factor is another way of inquiring whether there are policy considerations that bear on whether the order should issuerdquo 11C WRIGHT ET AL supra sect 29484 at 214 If the court concludes that the nonprofit is likely to succeed on its NEPA claim because the USFS wrongfully failed to conduct an environmental impact assessment it is likely to find that the public interest would be served by restraining the USFS from proceeding with logging in a national forest See Heartwood Inc v US Forest Service 73 F Supp 2d 962 979 (SD Ill 1999) affrsquod on other grounds 230 F3d 947 (7th Cir 2000) (ldquoviolations by federal agencies of NEPArsquos provisions as established by Congress harm the public as well as the environmentrdquo)

Thus a court is very likely to grant a preliminary injunction if it concludes that the nonprofit has a significant likelihood of success on the merits

26

EVIDENCE ANALYSIS (Evidence ID IIA amp C)

ANALYSIS

Legal Problems

(1) Under what circumstances can evidence of prior convictions be used to impeach a witnessrsquos credibility in a civil case

(1)(a) May the inmatersquos credibility be impeached by evidence of a 12-year-old felony drug conviction if he was released from prison 9 years ago

(1)(b) May the inmatersquos credibility be impeached by evidence of an 8-year-old misdemeanor perjury conviction that was punishable by 1 year in jail if he pleaded guilty and was sentenced only to pay a $5000 fine

(1)(c) May the inmatersquos credibility be impeached by evidence of a 7-year-old sexual assault conviction if the inmate is still serving a 10-year prison sentence and the victim was his 13-year-old daughter

(2)(a) May the guardrsquos credibility be impeached by cross-examination regarding specific instances of misconduct (ie lying on his reacutesumeacute) relevant to credibility

(2)(b) May the guardrsquos credibility be impeached by admission of extrinsic evidence (his reacutesumeacute and academic transcript) offered to prove specific instances of misconduct relevant to credibility

DISCUSSION

Summary

Under the Federal Rules of Evidence witnesses can be impeached with evidence of prior convictions andor specific instances of misconduct Whether evidence of prior convictions should be admitted to impeach generally depends on the nature of the crime the amount of time that has passed and (only in criminal cases) whether the ldquowitnessrdquo is the defendant FED R EVID 609(a)

In this civil case evidence of the inmatersquos conviction for distribution of marijuana should be admitted to impeach the inmate because he was convicted of a felony and was released from prison fewer than 10 years ago FED R EVID 609(a)(1) Credibility is critically important in this case because the jury will hear conflicting testimony from the two disputing parties and there were no other eyewitnesses to the altercation Under Rule 609(a)(1) the inmatersquos conviction should be admitted because it has some bearing on his credibility and its probative value is not substantially outweighed by concerns of unfair prejudice confusion or delay Id

Evidence of the inmatersquos misdemeanor conviction for perjury must be admitted because the crime ldquorequired provingmdashor the witnessrsquos admittingmdasha dishonest act or false statementrdquo by the inmate FED R EVID 609(a)(2)

27

Evidence Analysis

Evidence of the inmatersquos felony conviction for sexual assault should be excluded because its probative value is substantially outweighed by the danger of unfair prejudice to the inmate based on the heinous nature of the crime FED R EVID 609(a)(1) In the alternative the judge could limit the evidence relating to this conviction by excluding details of the inmatersquos crime

In all civil (and criminal) cases witnesses can also be impeached with evidence of specific instances of prior misconduct that did not result in a conviction FED R EVID 608(b) Pursuant to Rule 608(b) misconduct probative of untruthfulness can be inquired into on cross-examination but cannot be proved through extrinsic evidence Id Thus the inmatersquos counsel should be permitted to cross-examine the guard regarding the false statement in the guardrsquos reacutesumeacute However extrinsic evidence of the guardrsquos misconduct (ie the guardrsquos authenticated reacutesumeacute and transcript from the local college) should not be admitted even if the guard denies wrongdoing or refuses to answer cross-examination questions about these matters Id

Point One (10) The Federal Rules of Evidence permit impeachment of witnesses with evidence of prior convictions

Whether convictions should be admitted to impeach generally depends on the nature of the crime the amount of time that has passed and (only in criminal cases) whether the ldquowitnessrdquo is the defendant FED R EVID 609(a) Under Rule 609(a) evidence of prior convictions may be admitted for the purpose of ldquoattacking a witnessrsquos character for truthfulnessrdquo Id

There are two basic types of convictions that can be admitted for the purpose of impeachment

(1) convictions for crimes ldquopunishable by death or by imprisonment for more than one yearrdquo (which generally correlates to ldquofeloniesrdquo) FED R EVID 609(a)(1) and (2) convictions ldquofor any crimes regardless of the punishment if the court can readily determine that establishing the elements of the crime required provingmdashor the witnessrsquos admittingmdasha dishonest act or false statementrdquo FED R EVID 609(a)(2)

Pursuant to Rule 609(a)(1) in civil cases the admission of evidence of a felony conviction is ldquosubject to Rule 403 [which says that a court may exclude relevant evidence if its probative value is substantially outweighed by other factors]rdquo FED R EVID 609(a)(1) However Rule 403 does not protect the witness against admission of prior convictions involving dishonestymdashwhich must be admitted by the court FED R EVID 609(a)(2)

Finally Federal Rule of Evidence 609(b) contains the presumption that a conviction that is more than 10 years old or where more than 10 years has passed since the witnessrsquos release from confinement (whichever is later) should not be admitted unless ldquoits probative value supported by specific facts and circumstances substantially outweighs its prejudicial effectrdquo and the proponent has provided the adverse party with reasonable written notice FED R EVID 609(b)

Point One(a) (25) The court should admit evidence of the inmatersquos 12-year-old felony marijuana distribution conviction

The inmatersquos conviction for marijuana distribution was for a felony punishable by imprisonment for more than one year See FED R EVID 609(a)(1) Moreover although the conviction was 12 years ago the 10-year time limit of Rule 609(b) is not exceeded because that time limit runs

28

Evidence Analysis

from the date of either ldquothe witnessrsquos conviction or release from confinement for it whichever is laterrdquo FED R EVID 609(b) Because the inmate served three years in prison he was released from confinement nine years ago

However pursuant to Rule 609(a)(1) the admission of felony convictions to impeach a witness in a civil case is ldquosubject to Rule 403rdquo FED R EVID 609(a)(1) Neither Rule 609(a) nor the advisory committee notes specify which factors courts should consider when balancing the probative value of a conviction against the dangers identified in Rule 403 (which include (1) unfair prejudice (2) confusion of the issues (3) misleading the jury (4) waste of time or undue delay and (5) needless presentation of cumulative evidence) FED R EVID 403

In this case credibility is very important because the evidence consists primarily of the testimony of the disputing parties and there were no other eyewitnesses to the altercation This enhances the probative value of any evidence bearing on the inmatersquos credibility A court is likely to conclude that the inmatersquos prior felony drug conviction is relevant to his credibility See eg United States v Brito 427 F3d 53 64 (1st Cir 2005) (ldquoPrior drug-trafficking crimes are generally viewed as having some bearing on veracityrdquo) Although the probative value of any conviction diminishes with age see eg United States v Brewer 451 F Supp 50 53 (ED Tenn 1978) the inmatersquos ongoing problems with the law suggest that he has continued (and even escalated) his criminal behavior over the past nine years The court should admit this evidence because its probative value is not substantially outweighed by any Rule 403 concerns Specifically any prejudice to the inmate would be slight because the conviction is unrelated to the altercation at issue and the conviction was not for a heinous crime that might inflame the jury

[NOTE Whether an examinee identifies the jury instruction as containing a ldquoconclusiverdquo or ldquomandatoryrdquo presumption is less important than the examineersquos analysis of the constitutional infirmities]

Point One(b) (15) The court must admit evidence of the inmatersquos eight-year-old misdemeanor conviction because perjury is a crime of dishonesty

Rule 609(a)(2) provides that evidence of a criminal conviction ldquomust be admitted if the court can readily determine that establishing the elements of the crime required provingmdashor the witnessrsquos admittingmdasha dishonest act or false statementrdquo FED R EVID 609(a)(2) The inmatersquos conviction for perjury would have necessarily required proving that the inmate engaged in an act of dishonesty This conviction occurred within the past 10 years so it ldquomust be admittedrdquo because in contrast to Rule 609(a)(1) (discussed in Point One(a)) admission under Rule 609(a)(2) is mandatory and not subject to Rule 403

Point One(c) (20) The court should exclude evidence of the inmatersquos seven-year-old felony sexual assault conviction because the probative value of this evidence is substantially outweighed by the danger of unfair prejudice In the alternative the details of the prior conviction could be excluded

The inmatersquos conviction for felony sexual assault was seven years ago and he has not yet been released from incarceration so Rule 609(a) but not 609(b) is applicable here FED R EVID 609(a) This conviction is therefore admissible to impeach the inmate unless its probative value is substantially outweighed by the danger of unfair prejudice or any other Rule 403 concern Id

29

Evidence Analysis

Sex crimes are generally not considered relevant to credibility see Hopkins v State 639 So 2d 1247 1254 (Miss 1993) so the probative value of this conviction is relatively low Moreover the heinous nature of the inmatersquos crime (sexual assault on his daughter) makes the danger of unfair prejudice to the inmate very high Thus the court should exclude evidence of the conviction because it was for a heinous offense that is likely to inflame the jury and it has little bearing on credibility See eg United States v Beahm 664 F2d 414 419 (4th Cir 1981)

As an alternative to excluding this evidence the judge could minimize the unfair prejudice to the inmate by permitting limited cross-examination but refusing to allow specific questions about the nature of the inmatersquos conviction For example a court could limit cross-examination to the fact that the inmate was convicted of a ldquofelonyrdquo or perhaps that he was convicted of a ldquosexual assaultrdquo without identifying the victim However because evidence of the inmatersquos prior convictions can be admitted solely for the purpose of enabling the jury to assess his credibility and because his two earlier convictions should have already been admitted the court should exclude all evidence of the felony sexual assault conviction

Point Two(a) (15) The court should permit the inmatersquos counsel to cross-examine the guard regarding the false statement in his reacutesumeacute because the guardrsquos misconduct bears on his truthfulness

The inmate wishes to cross-examine the guard about his prior dishonest behaviormdashlying on his reacutesumeacutemdashthat did not involve a criminal conviction Rule 608(b) allows witnesses to be cross-examined about specific instances of prior non-conviction misconduct probative of untruthfulness ldquoin order to attack the witnessrsquos character for truthfulnessrdquo FED R EVID 608(b)

The courtrsquos decision to allow cross-examination about the guardrsquos prior dishonest behavior depends on the probative value of such evidence balanced against the danger of unfair prejudice to the guard or any other Rule 403 concern FED R EVID 403 Here the guardrsquos false statement on his reacutesumeacute that he obtained a degree in Criminal Justice is highly probative of his untruthfulness because it grossly misrepresents his actual academic record was made recently and was made with the intent to deceive Because the probative value of this evidence is very strong and is not substantially outweighed by any Rule 403 concerns cross-examination of the guard on this topic should be permitted The court may also consider it fair to permit this cross-examination of the guard on these matters assuming that one or more of the inmatersquos prior convictions have been admitted to impeach his credibility

Point Two(b) (15) The court should exclude extrinsic evidence of the guardrsquos non-conviction misconduct even if the guard denies wrongdoing or refuses to answer questions about the matter

Although Rule 608(b) allows cross-examination about specific instances of prior misconduct probative of untruthfulness ldquoextrinsic evidencerdquo offered to prove such misconduct is not admissible FED R EVID 608(b) The rationale for this rule is that allowing the introduction of extrinsic evidence of prior misconduct by witnesses when these acts are relevant only to the witnessesrsquo truthfulness and not to the main issues in the case would create too great a risk of confusing the jury and unduly delaying the trial The court does not have discretion to admit this extrinsic evidence See eg United States v Elliot 89 F3d 1360 1368 (8th Cir 1996)

30

Evidence Analysis

Here the inmatersquos counsel may cross-examine the guard about the false statement on his reacutesumeacute However the inmatersquos counsel must accept the guardrsquos response Even if the guard denies wrongdoing or refuses to answer questions about the matter the inmatersquos counsel cannot introduce the guardrsquos reacutesumeacute or the transcript from the local college to prove the guardrsquos misconduct

31

CORPORATIONS ANALYSIS (Corporations VA2 IX)

ANALYSIS

Legal Problems

(1) Do shareholders have the authority to amend a corporationrsquos bylaws with respect to director nominations

(2) Do board-approved bylaws on a particular subject here nomination of directors preempt subsequent conflicting bylaw amendments by shareholders

(3) Is a suit challenging both managementrsquos refusal to include the proposed bylaw amendment in Megarsquos proxy statement and the boardrsquos amendment of the bylaws dealing with nomination of directors a direct or derivative suit

DISCUSSION

Summary

The voting and litigation rights of the shareholders of Mega are subject to the provisions of the Model Business Corporations Act (MBCA)

The investorrsquos proposed bylaw provision is not inconsistent with state law Under the MBCA shareholders may amend the bylaws when the amendment deals with a proper matter for the corporationrsquos bylaws such as procedures for nominating directors

The Mega boardrsquos bylaw amendment does not preempt the investorrsquos proposed bylaw provision or the Mega shareholdersrsquo power to approve it While shareholders can limit the boardrsquos power to amend or repeal the bylaws the board cannot limit the shareholdersrsquo power

Whether the investor must make a demand on Megarsquos board depends on how the investor frames its claim If the investor claims a violation of shareholder voting rights the claim is direct and pre-suit demand on the board is not required If on the other hand the investor claims that the directors violated their fiduciary duties by amending the bylaws to entrench themselves the claim is derivative and a pre-suit demand is required

Point One (30) Shareholders may amend the corporationrsquos bylaws where the proposed bylaw provision relates to procedural matters typically included in the bylaws such as the nomination of directors

Internal affairs of the corporation such as the conduct of shareholder meetings and election of directors are subject to the corporate law of the state of incorporation See McDermott Inc v Lewis 531 A2d 206 (Del 1987) (applying law of jurisdiction where corporation was incorporated in case involving voting rights) This statersquos corporate statute is modeled on the MBCA

Under the MBCA ldquoshareholders may amend the corporationrsquos bylawsrdquo MBCA sect 1020(a) Thus the only question is whether the bylaws can specify the procedures for shareholder nomination of directors

32

Corporations Analysis

The MBCA states that the bylaws ldquomay contain any provision that is not inconsistent with law or the articles of incorporationrdquo MBCA sect 206(b) In addition the MBCA was revised in 2009 to address shareholder nomination of directors in public corporations (known as ldquoproxy accessrdquo) and specifies that the bylaws ldquomay contain a requirement that the corporation include in its [proxy materials] one or more individuals nominated by a shareholderrdquo MBCA sect 206(c)(1) see Committee on Corporate Laws ABA Section of Business Law Report on the Roles of Boards of Directors and Shareholders of Publicly Owned Corporations and Changes to the Model Business Corporations ActmdashAdoption of Shareholder Proxy Access Amendments to Chapters 2 and 10 65 BUS LAWYER 1105 (2010)

The inclusion of director-nomination procedures in the bylaws is consistent with practice and is recognized by the Delaware courts whose views on corporate law carry significant weight Typically the procedures for nomination of directors are found in the bylaws See 1 COX amp HAZEN TREATISE ON THE LAW OF CORPORATIONS sect 312 (3d ed 2011) see also 4 FLETCHER CORP FORMS ANN PART III ch 21 (2013) (including sample bylaws that permit nomination of directors by shareholders) The Delaware Supreme Court has confirmed that the bylaws may ldquodefine the process and proceduresrdquo for director elections See CA Inc v AFSCME Employees Pension Plan 953 A2d 227 (Del 2008) (concluding that bylaw amendment requiring reimbursement of election expenses to certain successful shareholder nominators is ldquoproper subjectrdquo under Delaware law)

[NOTE The question of the proper scope of the bylaws can be answered using the more general MBCA sect 206(b) or the 2009 MBCA revision adding sect 206(c)(1) (adopted in CT ME VA) In addition some examinees might raise the point that shareholder proposals may not compel the board to take action such as by including shareholder nominations in the companyrsquos proxy materials on the theory that the ldquobusiness and affairsrdquo of the corporation are to be managed by the board See MBCA sect 801(b) Although shareholders are generally limited to adopting precatory resolutions that recommend or encourage board action this limitation does not apply when shareholders have specific authority to take binding action on their ownmdashsuch as to amend the bylaws]

Point Two (30) Shareholders can amend (or repeal) board-approved bylaws Further shareholders can limit the boardrsquos power to later amend and repeal a shareholder-approved bylaw

Under the MBCA shareholders have the power to amend the bylaws See Point One The board shares this power with the shareholders unless (1) the corporationrsquos articles ldquoreserve that power exclusively to the shareholdersrdquo or (2) ldquothe shareholders in amending repealing or adopting a bylaw expressly provide that the board of directors may not amend repeal or reinstate that bylawrdquo See MBCA sect 1020(b)

Shareholder-approved bylaw provisions can amend or repeal existing bylaw provisions whether originally approved by the board or by shareholders See ALAN R PALMITER CORPORATIONS EXAMPLES AND EXPLANATIONS sect 713 (7th ed 2012) Thus the Mega boardrsquos bylaw amendmentmdashwhich set more demanding thresholds for shareholder nomination of directors than the investorrsquos proposed bylaw provisionmdashwould be superseded (repealed) if Megarsquos shareholders were to approve the investorrsquos proposal

Further a shareholder-approved bylaw generally can limit the power of the board to later amend or repeal it See MBCA sect 1020(b)(2) Thus if Megarsquos shareholders approved the bylaw

33

Corporations Analysis

provision proposed by the investor Megarsquos board could not repeal the provision because it includes a ldquono board repealrdquo clause

The revision to the MBCA in 2009 dealing with shareholder proxy access does not change this conclusion That revision specifies that a shareholder-approved bylaw dealing with director nominations may not limit the boardrsquos power to amend add or repeal ldquoany procedure or condition to such a bylaw in order to provide for a reasonable practicable and orderly processrdquo MBCA sect 206(d) Thus according to the revision if shareholders approve a bylaw amendment that limits further board changes the board would nonetheless retain the power to ldquotinkerrdquo with the bylaw to safeguard the voting process but could not repeal the shareholder-approved bylaw The Official Comment to MBCA sect 206(d) makes clear that the revision is ldquonot intended to allow the board of directors to frustrate the purpose of the shareholder-adopted proxy access provisionrdquo Thus if Megarsquos shareholders were to approve the bylaw provision proposed by the investor Megarsquos board could only amend the provision regarding its procedures or conditions in a manner consistent with its purpose of permitting proxy access for Megarsquos shareholders

[NOTE The boardrsquos attempted interference with a shareholder voting initiative may also have been a violation of the directorsrsquo fiduciary duties See Blasius Indus Inc v Atlas Corp 564 A2d 651 (Del Ch 1988) (finding that directors breached their fiduciary duties by amending bylaws and expanding size of board to thwart insurgentrsquos plan to amend bylaws and seat a majority of new directors) The call however asks examinees to consider whether shareholders or the board have ldquoprecedencerdquo over amending the corporate bylaws Thus an examineersquos answer should be framed in terms of ldquopowerrdquo and not ldquodutyrdquo]

Point Three (40) The investor need not make a demand on the board if the investor states a direct claim such as an allegation that the board interfered with the investorrsquos right to amend the bylaws But the investor must make a demand on the board if the investor states a derivative claim (on behalf of the corporation) such as an allegation that the directors sought to entrench themselves by interfering with the proposed proxy access

The MBCA generally requires that shareholders make a demand on the board of directors before initiation of a derivative suit MBCA sect 742 (shareholder may not bring derivative proceeding until written demand has been made on corporation and 90 days have expired) A derivative suit is essentially two suits in one where the plaintiff-shareholder seeks to bring on behalf of the corporation a claim that vindicates corporate rights usually based on violation of fiduciary duties PALMITER supra sect 1811 (6th ed 2009) The demand permits the board to investigate the situation identified by the shareholder and take suitable action No demand on the board is required however if the shareholder brings a direct suit to vindicate the shareholderrsquos own rights not those of the corporation

Is the suit brought by the investor derivative or direct The MBCA defines a ldquoderivative proceedingrdquo as one brought ldquoin the right of a domestic corporationrdquo MBCA sect 740(1) Thus the answer to how the investorrsquos suit should be characterized turns on what rights the investor seeks to vindicate If the investor frames its claim as one of fiduciary breach by directorsmdashfor example for failing to become adequately informed about voting procedures or for seeking to entrench themselves in office by manipulating the voting structure to avoid a shareholder insurgencymdashthen the suit is ldquoderivativerdquo and the investor must make a demand on the board See MBCA Ch 7 Subch D Introductory Comment (ldquothe derivative suit has historically been the principal method of challenging allegedly illegal action by managementrdquo)

34

Corporations Analysis

If however the investor frames its claim as one to vindicate shareholder rights the suit is direct and no demand is required For many courts the direct-derivative question turns on who is injured and who is to receive the relief sought by the plaintiff-shareholders See Tooley v Donaldson Lufkin amp Jenrette Inc 845 A2d 1031 (Del 2004) (characterizing a merger-delay claim as direct because delay of merger only harmed shareholders not corporation) Thus if the investor claims that managementrsquos refusal to include its proposed bylaw amendment in the corporationrsquos proxy materials violates its shareholder rights to initiate corporate governance reforms the suit will be direct Courts have not questioned the ability of shareholders to bring direct suits challenging board action to exclude their proposed bylaw amendments from the corporationrsquos proxy materials See JANA Master Fund Ltd v CNET Networks Inc 954 A2d 335 (Del Ch 2008) (upholding shareholderrsquos direct challenge to boardrsquos interpretation of advance-notice bylaw) Chesapeake Corp v Shore 771 A2d 293 (Del Ch 2000) (upholding shareholderrsquos direct challenge to actions by board that effectively prevented it from proposing bylaw amendments in contest for control)

Is the way that the investor frames its claim conclusive Courts have permitted shareholder-plaintiffs to challenge a transaction in a direct suit even though the same transaction could also be challenged as a fiduciary breach See Eisenberg v Flying Tiger Line Inc 451 F2d 267 (2d Cir 1971) (permitting direct suit challenging a corporate reorganization as a dilution of shareholder voting power even though reorganization may have involved conflicts of interest and thus constituted a fiduciary breach) Thus the investorrsquos choice to pursue a claim challenging the legality of managementrsquos decision to exclude the investorrsquos proposal from the corporationrsquos proxy materialsmdashrather than a possible breach of fiduciary dutymdashis likely to be respected See 3 COX amp HAZEN supra sect 153 (describing situations in which a claim can be framed as derivative or direct)

[NOTE Some issues under Delaware corporate law regarding pre-suit demand are not relevant here For example whether the Mega directors are independent and disinterested is not relevant to the MBCA requirement of a pre-suit demand As the Official Comment to MBCA sect 742 points out the MBCArsquos requirement of ldquouniversal demandrdquo gives the board ldquothe opportunity to reexamine the act complained of in the light of a potential lawsuit and take corrective actionrdquo even when the directors might be non-independent or have conflicts of interest

Nor is it relevant to the MBCA pre-suit demand requirement that the statutory 90-day waiting period may be onerous The first paragraph of MBCA sect 742 requires a pre-suit demand without exception the second paragraph of the section imposes a 90-day waiting period before a derivative suit may be brought which can be shortened if the board rejects the demand or ldquoirreparable injury to the corporation would result by waiting for the expiration of the 90-day periodrdquo The call as written asks only whether a pre-suit demand should be made and does not ask examinees to address whether the post-demand waiting period should be shortened under the ldquoirreparable injuryrdquo standard]

35

National Conference of Bar Examiners 302 South Bedford Street | Madison WI 53703-3622 Phone 608-280-8550 | Fax 608-280-8552 | TDD 608-661-1275

wwwncbexorg e-mail contactncbexorg

  • Preface
  • Description of the MEE
  • Instructions
  • July 2014 Questions
    • CRIMINAL LAW AND PROCEDURE QUESTION
    • CONTRACTS QUESTION
    • FAMILY LAW QUESTION
    • FEDERAL CIVIL PROCEDURE QUESTION
    • EVIDENCE QUESTION
    • CORPORATIONS QUESTION
      • July 2014 Analyses
        • CRIMINAL LAW AND PROCEDURE ANALYSIS
        • CONTRACTS ANALYSIS
        • FAMILY LAW ANALYSIS
        • FEDERAL CIVIL PROCEDURE ANALYSIS
        • EVIDENCE ANALYSIS
        • CORPORATIONS ANALYSIS
            • ltlt13 ASCII85EncodePages false13 AllowTransparency false13 AutoPositionEPSFiles true13 AutoRotatePages None13 Binding Left13 CalGrayProfile (Dot Gain 20)13 CalRGBProfile (sRGB IEC61966-21)13 CalCMYKProfile (US Web Coated 050SWOP051 v2)13 sRGBProfile (sRGB IEC61966-21)13 CannotEmbedFontPolicy Error13 CompatibilityLevel 1413 CompressObjects Tags13 CompressPages true13 ConvertImagesToIndexed true13 PassThroughJPEGImages true13 CreateJobTicket false13 DefaultRenderingIntent Default13 DetectBlends true13 DetectCurves 0000013 ColorConversionStrategy CMYK13 DoThumbnails false13 EmbedAllFonts true13 EmbedOpenType false13 ParseICCProfilesInComments true13 EmbedJobOptions true13 DSCReportingLevel 013 EmitDSCWarnings false13 EndPage -113 ImageMemory 104857613 LockDistillerParams false13 MaxSubsetPct 10013 Optimize true13 OPM 113 ParseDSCComments true13 ParseDSCCommentsForDocInfo true13 PreserveCopyPage true13 PreserveDICMYKValues true13 PreserveEPSInfo true13 PreserveFlatness true13 PreserveHalftoneInfo false13 PreserveOPIComments true13 PreserveOverprintSettings true13 StartPage 113 SubsetFonts true13 TransferFunctionInfo Apply13 UCRandBGInfo Preserve13 UsePrologue false13 ColorSettingsFile ()13 AlwaysEmbed [ true13 ]13 NeverEmbed [ true13 ]13 AntiAliasColorImages false13 CropColorImages true13 ColorImageMinResolution 30013 ColorImageMinResolutionPolicy OK13 DownsampleColorImages true13 ColorImageDownsampleType Bicubic13 ColorImageResolution 30013 ColorImageDepth -113 ColorImageMinDownsampleDepth 113 ColorImageDownsampleThreshold 15000013 EncodeColorImages true13 ColorImageFilter DCTEncode13 AutoFilterColorImages true13 ColorImageAutoFilterStrategy JPEG13 ColorACSImageDict ltlt13 QFactor 01513 HSamples [1 1 1 1] VSamples [1 1 1 1]13 gtgt13 ColorImageDict ltlt13 QFactor 01513 HSamples [1 1 1 1] VSamples [1 1 1 1]13 gtgt13 JPEG2000ColorACSImageDict ltlt13 TileWidth 25613 TileHeight 25613 Quality 3013 gtgt13 JPEG2000ColorImageDict ltlt13 TileWidth 25613 TileHeight 25613 Quality 3013 gtgt13 AntiAliasGrayImages false13 CropGrayImages true13 GrayImageMinResolution 30013 GrayImageMinResolutionPolicy OK13 DownsampleGrayImages true13 GrayImageDownsampleType Bicubic13 GrayImageResolution 30013 GrayImageDepth -113 GrayImageMinDownsampleDepth 213 GrayImageDownsampleThreshold 15000013 EncodeGrayImages true13 GrayImageFilter DCTEncode13 AutoFilterGrayImages true13 GrayImageAutoFilterStrategy JPEG13 GrayACSImageDict ltlt13 QFactor 01513 HSamples [1 1 1 1] VSamples [1 1 1 1]13 gtgt13 GrayImageDict ltlt13 QFactor 01513 HSamples [1 1 1 1] VSamples [1 1 1 1]13 gtgt13 JPEG2000GrayACSImageDict ltlt13 TileWidth 25613 TileHeight 25613 Quality 3013 gtgt13 JPEG2000GrayImageDict ltlt13 TileWidth 25613 TileHeight 25613 Quality 3013 gtgt13 AntiAliasMonoImages false13 CropMonoImages true13 MonoImageMinResolution 120013 MonoImageMinResolutionPolicy OK13 DownsampleMonoImages true13 MonoImageDownsampleType Bicubic13 MonoImageResolution 120013 MonoImageDepth -113 MonoImageDownsampleThreshold 15000013 EncodeMonoImages true13 MonoImageFilter CCITTFaxEncode13 MonoImageDict ltlt13 K -113 gtgt13 AllowPSXObjects false13 CheckCompliance [13 None13 ]13 PDFX1aCheck false13 PDFX3Check false13 PDFXCompliantPDFOnly false13 PDFXNoTrimBoxError true13 PDFXTrimBoxToMediaBoxOffset [13 00000013 00000013 00000013 00000013 ]13 PDFXSetBleedBoxToMediaBox true13 PDFXBleedBoxToTrimBoxOffset [13 00000013 00000013 00000013 00000013 ]13 PDFXOutputIntentProfile ()13 PDFXOutputConditionIdentifier ()13 PDFXOutputCondition ()13 PDFXRegistryName ()13 PDFXTrapped False1313 CreateJDFFile false13 Description ltlt13 ARA 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 BGR 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 CHS ltFEFF4f7f75288fd94e9b8bbe5b9a521b5efa7684002000410064006f006200650020005000440046002065876863900275284e8e9ad88d2891cf76845370524d53705237300260a853ef4ee54f7f75280020004100630072006f0062006100740020548c002000410064006f00620065002000520065006100640065007200200035002e003000204ee553ca66f49ad87248672c676562535f00521b5efa768400200050004400460020658768633002gt13 CHT ltFEFF4f7f752890194e9b8a2d7f6e5efa7acb7684002000410064006f006200650020005000440046002065874ef69069752865bc9ad854c18cea76845370524d5370523786557406300260a853ef4ee54f7f75280020004100630072006f0062006100740020548c002000410064006f00620065002000520065006100640065007200200035002e003000204ee553ca66f49ad87248672c4f86958b555f5df25efa7acb76840020005000440046002065874ef63002gt13 CZE 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 DAN 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 DEU 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 ESP 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 ETI 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 FRA 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 GRE ltFEFF03a703c103b703c303b903bc03bf03c003bf03b903ae03c303c403b5002003b103c503c403ad03c2002003c403b903c2002003c103c503b803bc03af03c303b503b903c2002003b303b903b1002003bd03b1002003b403b703bc03b903bf03c503c103b303ae03c303b503c403b5002003ad03b303b303c103b103c603b1002000410064006f006200650020005000440046002003c003bf03c5002003b503af03bd03b103b9002003ba03b103c42019002003b503be03bf03c703ae03bd002003ba03b103c403ac03bb03bb03b703bb03b1002003b303b903b1002003c003c103bf002d03b503ba03c403c503c003c903c403b903ba03ad03c2002003b503c103b303b103c303af03b503c2002003c503c803b703bb03ae03c2002003c003bf03b903cc03c403b703c403b103c2002e0020002003a403b10020005000440046002003ad03b303b303c103b103c603b1002003c003bf03c5002003ad03c703b503c403b5002003b403b703bc03b903bf03c503c103b303ae03c303b503b9002003bc03c003bf03c103bf03cd03bd002003bd03b1002003b103bd03bf03b903c703c403bf03cd03bd002003bc03b5002003c403bf0020004100630072006f006200610074002c002003c403bf002000410064006f00620065002000520065006100640065007200200035002e0030002003ba03b103b9002003bc03b503c403b103b303b503bd03ad03c303c403b503c103b503c2002003b503ba03b403cc03c303b503b903c2002egt13 HEB 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 HRV (Za stvaranje Adobe PDF dokumenata najpogodnijih za visokokvalitetni ispis prije tiskanja koristite ove postavke Stvoreni PDF dokumenti mogu se otvoriti Acrobat i Adobe Reader 50 i kasnijim verzijama)13 HUN 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 ITA 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 JPN ltFEFF9ad854c18cea306a30d730ea30d730ec30b951fa529b7528002000410064006f0062006500200050004400460020658766f8306e4f5c6210306b4f7f75283057307e305930023053306e8a2d5b9a30674f5c62103055308c305f0020005000440046002030d530a130a430eb306f3001004100630072006f0062006100740020304a30883073002000410064006f00620065002000520065006100640065007200200035002e003000204ee5964d3067958b304f30533068304c3067304d307e305930023053306e8a2d5b9a306b306f30d530a930f330c8306e57cb30818fbc307f304c5fc59808306730593002gt13 KOR ltFEFFc7740020c124c815c7440020c0acc6a9d558c5ec0020ace0d488c9c80020c2dcd5d80020c778c1c4c5d00020ac00c7a50020c801d569d55c002000410064006f0062006500200050004400460020bb38c11cb97c0020c791c131d569b2c8b2e4002e0020c774b807ac8c0020c791c131b41c00200050004400460020bb38c11cb2940020004100630072006f0062006100740020bc0f002000410064006f00620065002000520065006100640065007200200035002e00300020c774c0c1c5d0c11c0020c5f40020c2180020c788c2b5b2c8b2e4002egt13 LTH 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 LVI 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 NLD (Gebruik deze instellingen om Adobe PDF-documenten te maken die zijn geoptimaliseerd voor prepress-afdrukken van hoge kwaliteit De gemaakte PDF-documenten kunnen worden geopend met Acrobat en Adobe Reader 50 en hoger)13 NOR 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 POL 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 PTB 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 RUM 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 RUS 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 SKY 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 SLV 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 SUO 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 SVE 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 TUR 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 UKR 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 ENU (Use these settings to create Adobe PDF documents best suited for high-quality prepress printing Created PDF documents can be opened with Acrobat and Adobe Reader 50 and later)13 gtgt13 Namespace [13 (Adobe)13 (Common)13 (10)13 ]13 OtherNamespaces [13 ltlt13 AsReaderSpreads false13 CropImagesToFrames true13 ErrorControl WarnAndContinue13 FlattenerIgnoreSpreadOverrides false13 IncludeGuidesGrids false13 IncludeNonPrinting false13 IncludeSlug false13 Namespace [13 (Adobe)13 (InDesign)13 (40)13 ]13 OmitPlacedBitmaps false13 OmitPlacedEPS false13 OmitPlacedPDF false13 SimulateOverprint Legacy13 gtgt13 ltlt13 AddBleedMarks false13 AddColorBars false13 AddCropMarks false13 AddPageInfo false13 AddRegMarks false13 ConvertColors ConvertToCMYK13 DestinationProfileName ()13 DestinationProfileSelector DocumentCMYK13 Downsample16BitImages true13 FlattenerPreset ltlt13 PresetSelector MediumResolution13 gtgt13 FormElements false13 GenerateStructure false13 IncludeBookmarks false13 IncludeHyperlinks false13 IncludeInteractive false13 IncludeLayers false13 IncludeProfiles false13 MultimediaHandling UseObjectSettings13 Namespace [13 (Adobe)13 (CreativeSuite)13 (20)13 ]13 PDFXOutputIntentProfileSelector DocumentCMYK13 PreserveEditing true13 UntaggedCMYKHandling LeaveUntagged13 UntaggedRGBHandling UseDocumentProfile13 UseDocumentBleed false13 gtgt13 ]13gtgt setdistillerparams13ltlt13 HWResolution [2400 2400]13 PageSize [612000 792000]13gtgt setpagedevice13

Page 3: July 2014 MEE Questions and Analyses - NCBE...This publication includes the questions and analyses from the July 2014 MEE. (In the actual test, the questions are simply numbered rather

Contents

Prefaceii

Description of the MEE ii

Instructionsiii

July 2014 Q uestions

Criminal Law and Procedure Question 3

Contracts Question 5

Family Law Question 6

Federal Civil Procedure Question 7

Evidence Question 8

Corporations Question 9

July 2014 A nalyses

Criminal Law and Procedure Analysis 13

Contracts Analysis 16

Family Law Analysis 19

Federal Civil Procedure Analysis 22

Evidence Analysis 27

Corporations Analysis 32

i

Preface

The Multistate Essay Examination (MEE) is developed by the National Conference of Bar Examiners (NCBE) This publication includes the questions and analyses from the July 2014 MEE (In the actual test the questions are simply numbered rather than being identified by area of law) The instructions for the test appear on page iii

The model analyses for the MEE are illustrative of the discussions that might appear in excellent answers to the questions They are provided to the user jurisdictions to assist graders in grading the examination They address all the legal and factual issues the drafters intended to raise in the questions

The subjects covered by each question are listed on the first page of its accompanying analysis identified by roman numerals that refer to the MEE subject matter outline for that subject For example the Federal Civil Procedure question on the July 2014 MEE tested the following areas from the Federal Civil Procedure outline III Injunctions and provisional remedies and IVC Pretrial proceduresmdashJoinder of parties and claims (including class actions)

For more information about the MEE including subject matter outlines visit the NCBE website at wwwncbexorg

Description of the MEE

The MEE consists of six 30-minute questions and is a component of the Uniform Bar Examination (UBE) It is administered by user jurisdictions as part of the bar examination on the Tuesday before the last Wednesday in February and July of each year Areas of law that may be covered on the MEE include the following Business Associations (Agency and Partnership Corporations and Limited Liability Companies) Conflict of Laws Constitutional Law Contracts Criminal Law and Procedure Evidence Family Law Federal Civil Procedure Real Property Torts Trusts and Estates (Decedentsrsquo Estates Trusts and Future Interests) and Uniform Commercial Code (Negotiable Instruments and Bank Deposits and Collections Secured Transactions) (Effective with the February 2015 MEE administration Uniform Commercial Code Articles 3 and 4 [Negotiable Instruments and Bank Deposits and Collections] is no longer being tested) Some questions may include issues in more than one area of law The particular areas covered vary from exam to exam

The purpose of the MEE is to test the examineersquos ability to (1) identify legal issues raised by a hypothetical factual situation (2) separate material which is relevant from that which is not (3) present a reasoned analysis of the relevant issues in a clear concise and well-organized composition and (4) demonstrate an understanding of the fundamental legal principles relevant to the probable solution of the issues raised by the factual situation The primary distinction between the MEE and the Multistate Bar Examination (MBE) is that the MEE requires the examinee to demonstrate an ability to communicate effectively in writing

ii

Instructions The back cover of each test booklet contains the following instructions

You will be instructed when to begin and when to stop this test Do not break the seal on this booklet until you are told to begin

You may answer the questions in any order you wish Do not answer more than one question in each answer booklet If you make a mistake or wish to revise your answer simply draw a line through the material you wish to delete

If you are using a laptop computer to answer the questions your jurisdiction will provide you with specific instructions

Read each fact situation very carefully and do not assume facts that are not given in the question Do not assume that each question covers only a single area of the law some of the questions may cover more than one of the areas you are responsible for knowing

Demonstrate your ability to reason and analyze Each of your answers should show an understanding of the facts a recognition of the issues included a knowledge of the applicable principles of law and the reasoning by which you arrive at your conclusions The value of your answer depends not as much upon your conclusions as upon the presence and quality of the elements mentioned above

Clarity and conciseness are important but make your answer complete Do not volunteer irrelevant or immaterial information

Answer all questions according to generally accepted fundamental legal principles unless your testing jurisdiction has instructed you to answer according to local case or statutory law

NOTE Examinees testing in UBE jurisdictions must answer according to generally accepted fundamental legal principles rather than local case or statutory law

iii

July 2014 MEE

QUESTIONS

Contracts Criminal Law and Procedure

Family L aw Federal Civil Procedure

Evidence Corporations

CRIMINAL LAW AND PROCEDURE QUESTION

While on routine patrol a police officer observed a suspect driving erratically and pulled the suspectrsquos car over to investigate When he approached the suspectrsquos car the officer detected a strong odor of marijuana The officer immediately arrested the suspect for driving under the influence of an intoxicant (DUI) While the officer was standing near the suspectrsquos car placing handcuffs on the suspect the officer observed burglary tools on the backseat

The officer seized the burglary tools He then took the suspect to the county jail booked him for the DUI and placed him in a holding cell Later that day the officer gave the tools he had found in the suspectrsquos car to a detective who was investigating a number of recent burglaries in the neighborhood where the suspect had been arrested

At the time of his DUI arrest the suspect had a six-month-old aggravated assault charge pending against him and was being represented on the assault charge by a lawyer

Early the next morning upon learning of her clientrsquos arrest the lawyer went to the jail She arrived at 900 am immediately identified herself to the jailer as the suspectrsquos attorney and demanded to speak with the suspect The lawyer also told the jailer that she did not want the suspect questioned unless she was present The jailer told the lawyer that she would need to wait one hour to see the suspect After speaking with the lawyer the jailer did not inform anyone of the lawyerrsquos presence or her demands

The detective who had also arrived at the jail at 900 am overheard the lawyerrsquos conversation with the jailer The detective then entered the windowless interview room in the jail where the suspect had been taken 30 minutes earlier Without informing the suspect of the lawyerrsquos presence or her demands the detective read to the suspect full and accurate Miranda warnings The detective then informed the suspect that he wanted to ask about the burglary tools found in his car and the recent burglaries in the neighborhood where he had been arrested The suspect replied ldquoI think I want my lawyer here before I talk to yourdquo The detective responded ldquoThatrsquos up to yourdquo

After a few minutes of silence the suspect said ldquoWell unless there is anything else I need to know letrsquos not waste any time waiting for someone to call my attorney and having her drive here I probably should keep my mouth shut but Irsquom willing to talk to you for a whilerdquo The suspect then signed a Miranda waiver form and after interrogation by the detective made incriminating statements regarding five burglaries The interview lasted from 915 am to 1000 am

In addition to the DUI the suspect has been charged with five counts of burglary

The lawyer has filed a motion to suppress all statements made by the suspect to the detective in connection with the five burglaries

The state supreme court follows federal constitutional principles in all cases interpreting a criminal defendantrsquos rights

3

Criminal Law and Procedure Question

1 Did the detective violate the suspectrsquos Sixth Amendment right to counsel when he questioned the suspect in the absence of the lawyer Explain

2 Under Miranda did the suspect effectively invoke his right to counsel Explain

3 Was the suspectrsquos waiver of his Miranda rights valid Explain

4

CONTRACTS QUESTION

A music conservatory has two concert halls One concert hall had a pipe organ that was in poor repair and the other had no organ The conservatory decided to repair the existing organ and buy a new organ for the other concert hall After some negotiation the conservatory entered into two contracts with a business that both repairs and sells organs Under one contract the business agreed to repair the existing pipe organ for the conservatory for $100000 The business would usually charge a higher price for a project of this magnitude but the business agreed to this price because the conservatory agreed to prepay the entire amount Under the other contract the business agreed to sell a new organ to the conservatory for the other concert hall for $225000 As with the repair contract the business agreed to a low sales price because the conservatory agreed to prepay the entire amount Both contracts were signed on January 3 and the conservatory paid the business a total of $325000 that day

Two weeks later before the business had commenced repair of the existing organ the business suffered serious and unanticipated financial reversals The chief financial officer for the business contacted the conservatory and said

Bad news We had an unexpected liability and as a result are in a real cash crunch In fact even though we havenrsquot acquired the new organ from our supplier or started repair of your existing organ wersquove already spent the cash you gave us and we have no free cash on hand Wersquore really sorry but wersquore in a fix I think that we can find a way to perform both contracts but not at the original prices If you agree to pay $60000 more for the repair and $40000 more for the new organ we can probably find financing to finish everything If you donrsquot agree to pay us the extra money I doubt that we will ever be able to perform either contract and yoursquoll be out the money you already paid us

After receiving this unwelcome news the conservatory agreed to pay the extra amounts provided that the extra amount on each contract would be paid only upon completion of the businessrsquos obligations under that contract The business agreed to this arrangement and the parties quickly signed documents reflecting these changes to each contract The business then repaired the existing organ delivered the new organ and demanded payment of the additional $100000

The conservatory now has refused to pay the business the additional amounts for the repair and the new organ

1 Must the conservatory pay the additional $60000 for the organ repair Explain

2 Must the conservatory pay the additional $40000 for the new organ Explain

5

FAMILY LAW QUESTION

In 1994 a man and a woman were married in State A

In 1998 their daughter was born in State A

In 2010 the family moved to State B

In 2012 the husband and wife divorced in State B Under the terms of the divorce decree

(a) the husband and wife share legal and physical custody of their daughter (b) the husband must pay the wife $1000 per month in child support until their daughter reaches age 18 (c) the marital residence was awarded to the wife with the proviso that if it is sold before the daughter reaches age 18 the husband will receive 25 of the net sale proceeds remaining after satisfaction of the mortgage on the residence and (d) the remaining marital assets were divided between the husband and the wife equally

Six months ago the husband was offered a job in State A that pays significantly less than his job in State B but provides him with more responsibilities and much better promotion opportunities The husband accepted the job in State A and moved from State B back to State A

Since returning to State A the husband has not paid child support because due to his lower salary he has had insufficient funds to meet all his obligations

One month ago the wife sold the marital home netting $10000 after paying off the mortgage She then moved to a smaller residence The husband believes that he should receive more than 25 of the net sale proceeds given his financial difficulties

Last week when the wife brought the daughter to the husbandrsquos State A home for a weekend visit the husband served the wife with a summons in a State A action to modify the support and marital-residence-sale-proceeds provisions of the State B divorce decree The husband brought the action in the State A court that adjudicates all domestic relations issues

1 Does the State A court have jurisdiction to modify (a) the child support provision of the State B divorce decree Explain (b) the marital-residence-sale-proceeds provision of the State B divorce decree

Explain

2 On the merits could the husband obtain (a) retroactive modification of his child support obligation to the daughter Explain (b) prospective modification of his child support obligation to the daughter Explain (c) modification of the marital-residence-sale-proceeds provision of the State B

divorce decree Explain

6

FEDERAL CIVIL PROCEDURE QUESTION

The United States Forest Service (USFS) manages public lands in national forests including the Scenic National Forest Without conducting an environmental evaluation or preparing an environmental impact statement the USFS approved a development project in the Scenic National Forest that required the clearing of 5000 acres of old-growth forest The trees in the forest are hundreds of years old and the forest is home to a higher concentration of wildlife than can be found anywhere else in the western United States

The USFS solicited bids from logging companies to harvest the trees on the 5000 acres of forest targeted for clearing and it ultimately awarded the logging contract to the company that had submitted the highest bid for the trees However the USFS has not yet issued the company a logging permit Once it does so the company intends to begin cutting down trees immediately

A nonprofit organization whose mission is the preservation of natural resources has filed suit in federal district court against the USFS The nonprofit alleges that the USFS violated the National Environmental Policy Act (NEPA) by failing to prepare an environmental impact statement for the proposed logging project Among other remedies the nonprofit seeks a permanent injunction barring the USFS from issuing a logging permit to the logging company until an adequate environmental impact statement is completed The nonprofit believes that the logging project would destroy important wildlife habitat and thereby cause serious harm to wildlife in the Scenic National Forest including some endangered species

Assume that federal subject-matter jurisdiction is available that the nonprofit has standing to bring this action and that venue is proper

1 If the logging company seeks to join the litigation as a party must the federal district court allow it to do so as a matter of right Explain

2 What types of relief could the nonprofit seek to stop the USFS from issuing a logging permit during the pendency of the action what must the nonprofit demonstrate to obtain that relief and is the federal district court likely to grant that relief Explain

7

EVIDENCE QUESTION

A prison inmate has filed a civil rights lawsuit against a guard at the prison alleging that the guard violated the inmatersquos constitutional rights during an altercation The inmate and the guard are the only witnesses to this altercation They have provided contradictory reports about what occurred

The trial will be before a jury The inmate plans to testify at trial The guardrsquos counsel has moved for leave to impeach the inmate with the following

(a) Twelve years ago the inmate was convicted of felony distribution of marijuana He served a three-year prison sentence which began immediately after he was convicted He served his full sentence and was released from prison nine years ago (b) Eight years ago the inmate pleaded guilty to perjury a misdemeanor punishable by up to one year in jail He paid a $5000 fine (c) Seven years ago the inmate was convicted of felony sexual assault of a child and is currently serving a 10-year prison sentence for the crime The victim was the inmatersquos daughter who was 13 years old at the time of the assault

The inmatersquos counsel objects to the admission of any evidence related to these three convictions and to any cross-examination based on this evidence

The guard also plans to testify at trial The inmatersquos counsel has moved for leave to impeach the guard with the following

Last year the guard applied for a promotion to prison supervisor The guard submitted a reacutesumeacute to the state that indicated that he had been awarded a BA in Criminal Justice from a local college An official copy of the guardrsquos academic transcript from that college indicates that the guard dropped out after his first semester and did not receive a degree

The guardrsquos counsel objects to the admission of this evidence and to any cross-examination based on this evidence

The transcript and the reacutesumeacute have been properly authenticated The trial will be held in a jurisdiction that has adopted all of the Federal Rules of Evidence

1 What evidence if any proffered by the guard to impeach the inmate should be admitted Explain

2 What evidence if any proffered by the inmate to impeach the guard should be admitted Explain

8

CORPORATIONS QUESTION

Mega Inc is a publicly traded corporation incorporated in a state whose corporate statute is modeled on the Model Business Corporation Act (MBCA) Megarsquos articles of incorporation do not address the election of directors or amendment of the bylaws by shareholders

Well within the deadline for the submission of shareholder proposals for the upcoming annual shareholdersrsquo meeting an investor who was a large and long-standing shareholder of Mega submitted a proposed amendment to Megarsquos bylaws The proposal which the investor asked to be included in the corporationrsquos proxy materials and voted on at the upcoming shareholdersrsquo meeting read as follows

Section 20 The Corporation shall include in its proxy materials (including the proxy ballot) for a shareholdersrsquo meeting at which directors are to be elected the name of a person nominated for election to the Board of Directors by a shareholder or group of shareholders that beneficially have owned 3 or more of the Corporationrsquos outstanding common stock for at least one year

This Section shall supersede any inconsistent provision in these Bylaws and may not be amended or repealed by the Board of Directors without shareholder approval

Megarsquos management decided to exclude the investorrsquos proposal from the corporationrsquos proxy materials and explained its reasons in a letter to the investor

The investorrsquos proposed bylaw provision would be inconsistent with relevant state law because the Board of Directors has the authority to manage the business and affairs of the Corporation Generally shareholders lack the authority to interfere with corporate management by seeking to create a method for the nomination and election of directors inconsistent with the method chosen by the Board of Directors

Furthermore at its most recent meeting the Board of Directors unanimously approved an amendment to the Corporationrsquos bylaws that provides for proxy access for director nominations by a shareholder or a group of shareholders holding at least 10 of the Corporationrsquos voting shares for at least three years This procedure takes precedence over any nomination methods that might be sought or approved by shareholders

The investor is considering bringing a suit challenging managementrsquos refusal to include the investorrsquos proposed bylaw provision and challenging the boardrsquos amendment of the bylaws at its recent meeting

1 Is the investorrsquos proposed bylaw provision inconsistent with state law Explain

2 If the investorrsquos proposed bylaw provision were approved by the shareholders would the bylaw amendment previously approved by the board take precedence over the investorrsquos proposed bylaw provision Explain

3 Must the investor make a demand on Megarsquos board of directors before bringing suit Explain

9

July 2014 MEE

ANALYSES

Contracts Family Law

Criminal Law and Procedure

Federal Civil Procedure Evidence

Corporations

CRIMINAL LAW AND PROCEDURE ANALYSIS (Criminal Law and Procedure VA B D)

ANALYSIS

Legal Problems

(1) Did the detective violate the suspectrsquos Sixth Amendment right to counsel when he questioned the suspect about the burglaries without the lawyer present given that the lawyer represented the suspect in an unrelated criminal matter

(2) Under Miranda did the suspect effectively invoke his right to counsel when he said ldquoI think I want my lawyer here before I talk to yourdquo

(3) Was the suspectrsquos waiver of his right to remain silent under Miranda valid

DISCUSSION

Summary

The Sixth Amendment right to counsel as applied to states through the Fourteenth Amendment is offense-specific Although the suspect had an attorney representing him on his pending assault charge he had no Sixth Amendment right to the assistance of counsel with respect to the five uncharged burglaries because formal adversarial proceedings had not yet commenced on those charges The suspectrsquos Sixth Amendment right to counsel was not violated by the detectiversquos failure to inform him that the lawyer was present or of the lawyerrsquos demands

However a person undergoing custodial interrogation also has an independent constitutional right to counsel during custodial interrogation under Miranda When a suspect invokes his right to counsel under Miranda custodial interrogation must immediately cease for a period of at least 14 days However the invocation of the right to counsel must be unambiguous and clearly convey that the suspect has requested counsel Here because the suspectrsquos statement ldquoI think I want my lawyer here before I talk to yourdquo was ambiguous he did not invoke his Miranda right to counsel

A waiver of rights must be knowing intelligent and voluntary Here the suspect waived his right to remain silent under Miranda when he signed the waiver form The fact that the detective did not correct the suspectrsquos assumption that the lawyer would need to drive to the jailmdashby telling him that the lawyer was in the waiting room and was demanding to see himmdashdid not affect the validity of the suspectrsquos waiver

Point One (35) The suspectrsquos Sixth Amendment right to counsel was not violated because the right does not attach on new charges until formal adversarial judicial proceedings have commenced on those charges

The Sixth Amendment as applied to the states through the Fourteenth Amendment provides that ldquo[i]n all criminal prosecutions the accused shall enjoy the right to have the Assistance of Counsel for his defenserdquo The right to counsel does not attach with respect to particular charges until formal adversarial judicial proceedings have commenced (ie ldquoat or after the initiation of

13

Criminal Law and Procedure Analysis

adversary judicial criminal proceedingsmdashwhether by way of formal charge preliminary hearing indictment information or arraignment [or in some states arrest warrant]rdquo McNeil v Wisconsin 501 US 171 175 (1991) (internal quotations omitted)) Once a suspectrsquos Sixth Amendment right to counsel has attached any attempts to ldquodeliberately elicitrdquo statements from him in the absence of his attorney violate the Sixth Amendment See Massiah v United States 377 US 201 (1964) Brewer v Williams 430 US 387 (1977)

The Sixth Amendment right to counsel is charge- or offense -specific Representation by counsel in one prosecution does not in itself guarantee counsel for uncharged offenses See McNeil 501 US at 175 Texas v Cobb 532 US 162 (2001) Here the suspectrsquos Sixth Amendment right to counsel had attached only for the pending aggravated assault charge The suspectrsquos right to counsel for the aggravated assault case did not guarantee counsel for the five unrelated and uncharged burglaries that were the subject of the detectiversquos interrogation Thus because formal adversarial judicial proceedings against the suspect for the uncharged burglaries had not begun he had no Sixth Amendment right to counsel

Finally the detectiversquos failure to inform the suspect of the lawyerrsquos presence and demands to speak with him does not implicate the suspectrsquos Sixth Amendment right to counsel which had not yet attached See id Moran v Burbine 475 US 412 428ndash31 (1986)

Point Two (30) The suspect did not effectively invoke his right to counsel under Miranda because his statement was not unambiguous

A suspect subject to custodial interrogation has a right to consult with counsel and to have an attorney present during questioning Miranda v Arizona 384 US 436 (1966) When a suspect invokes his right to counsel during an interrogation law enforcement must immediately cease all questioning See Edwards v Arizona 451 US 477 484ndash85 (1981) Custodial interrogation cannot be reinitiated unless and until the suspect has been re-advised of his Miranda rights has provided a knowing and voluntary waiver and (1) counsel is present and (2) the suspect himself initiated further communication with the police see id at 484 or (3) (if the suspect was released from custody after the initial interrogation) at least 14 days have passed Maryland v Shatzer 559 US 98 110 (2010)

To invoke the right to counsel a suspectrsquos request must be ldquounambiguousrdquo This means that the suspect must articulate the desire for counsel sufficiently clearly that a reasonable officer would understand the statement to be a request for counsel Davis v United States 512 US 452 459 (1994) If the request is ambiguous the police are not required to stop the interrogation

In this case the suspectrsquos statement ldquoI think I want my lawyer here before I talk to yourdquo was not an unambiguous request for counsel The most reasonable interpretation of this statement is that the suspect might be invoking his right to counsel Id at 461 (ldquomaybe I should talk to a lawyerrdquo is not an unequivocal request for counsel) See also Burket v Angelone 208 F3d 172 197ndash98 (4th Cir 2000) (ldquoI think I need a lawyerrdquo is not an unambiguous request for an attorney) Soffar v Cockrell 300 F3d 588 594ndash95 (5th Cir 2002) (discussion of various statements that did not constitute unequivocal requests for counsel)

Under these circumstances the detective was not required to cease the custodial interrogation of the suspect Nor was the detective required to clarify or ask follow-up questions to determine whether the suspect in fact wanted an attorney Davis 512 US at 459ndash60

14

Criminal Law and Procedure Analysis

Point Three (35) The suspectrsquos waiver of his Miranda rights was knowing intelligent and voluntary despite the fact that he was never told of the lawyerrsquos presence in the jail or of the lawyerrsquos demands

A valid waiver of Miranda rights must be ldquovoluntaryrdquomdashie the product of a free or deliberate choice rather than intimidation coercion or deception Berghuis v Thompkins 560 US 370 382ndash83 (2010) In addition the waiver must be knowing and intelligent That is it ldquomust have been made with a full awareness of both the nature of the right being abandoned and the consequences of the decision to abandon itrdquo Moran v Burbine 475 US 421 (1986)

In this case the suspect signed a Miranda waiver form after receiving proper warnings There is no evidence ldquothat the police resorted to physical or psychological pressure to elicit the statementsrdquo Id The entire interview lasted only 45 minutes The only issue is whether the suspect knowingly and intelligently waived his Miranda rights despite the fact that the detective did not tell the suspect about the lawyerrsquos presence and her demands

The Supreme Court has said that ldquo[e]vents occurring outside of the presence of the suspect and entirely unknown to him surely can have no bearing on the capacity to comprehend and knowingly relinquish a constitutional rightrdquo Id at 422 If the suspect ldquoknew that he could stand mute and request a lawyer and was aware of the Statersquos intention to use his statements to secure a convictionrdquo then the waiver is valid regardless of the information withheld Id at 422ndash23

Here the suspect was correctly informed of his rights Miranda v Arizona 384 US at 467ndash73 His comments demonstrate that he understood that he could have a lawyer present if he desired (ie wondering whether he should call his attorney) and that he understood that there might be consequences to speaking with the detective (ldquoI probably should keep my mouth shut but Irsquom willing to talk to you for a whilerdquo) His comment ldquo[L]etrsquos not waste any time waiting for someone to call my attorney and having her drive hererdquo along with his signature on the Miranda waiver form show that his waiver was valid under the constitutional standard

The fact that the detective did not tell the suspect about the lawyerrsquos presence and demands has no bearing on the validity of the suspectrsquos waiver because ldquosuch conduct is only relevant to the constitutional validity of a waiver if it deprives a defendant of knowledge essential to his ability to understand the nature of his rights and the consequences of abandoning themrdquo Moran at 424 The Supreme Court has specifically declined to adopt a rule requiring that law enforcement tell a suspect of an attorneyrsquos efforts to contact him id at 425 (ldquoNor are we prepared to adopt a rule requiring that the police inform a suspect of an attorneyrsquos efforts to reach himrdquo)

[NOTE An examinee might also recognize that this general rule is further supported by the Supreme Courtrsquos decision in Florida v Powell 559 US 50 (2010) approving state Miranda warnings that do not explicitly warn suspects that they have a right to have counsel present during custodial interrogation]

15

CONTRACTS ANALYSIS (Contracts IB2 IIB IVA3 amp A5)

ANALYSIS

Legal Problems

(1) In the case of a service contract (governed by the common law of contracts) is a modification enforceable when a party agrees to pay more for the same performance than was originally promised

(2) In the case of a contract for the sale of goods (governed by Article 2 of the UCC) is a modification enforceable when a party agrees to pay more for the same goods than was originally promised

(3) May a party avoid an agreement on the basis of economic duress

DISCUSSION

Summary

There are two arguments that the conservatory can make to support the claim that it is not bound to pay the higher prices lack of consideration and economic duress

The organ repair contract is governed by the common law of contracts Under the common law the business would have difficulty recovering the additional $60000 for the organ repair because under the ldquopreexisting duty rulerdquo the agreement of the conservatory to pay the extra price was not supported by consideration However the business might argue that the modification is enforceable under an exception to the preexisting duty rule for fair and equitable modifications made in light of unanticipated circumstances

The organ sale contract is governed by Article 2 of the Uniform Commercial Code The business would likely recover the additional amount under that contract because Article 2 provides that consideration is not required for a modification to be binding

In both cases the conservatory could seek to avoid its agreement on the grounds of economic duress but that argument is not likely to succeed

Point One (45) The business probably cannot recover the additional $60000 for the organ repair because the conservatoryrsquos promise to pay more money was not supported by consideration

The general rule is that to be enforceable a promise must be supported by consideration Under RESTATEMENT (SECOND) OF CONTRACTS sect 71 a promise is supported by consideration if it is bargained for in exchange for a return promise or performance However under the ldquopreexisting duty rulerdquo (exemplified in RESTATEMENT (SECOND) OF CONTRACTS sect 73 and Alaska Packersrsquo Assrsquon v Domenico 117 F 99 (9th Cir 1902)) promise of performance of a legal duty already owed to a promisor which is neither doubtful nor the subject of honest dispute is not consideration

If the business had promised the conservatory anything new or different in exchange for the agreement to pay the additional $60000 (such as for example repairing the pipe organ more

16

Contracts Analysis

quickly or using better parts) that would constitute consideration especially in light of the principle that courts do not inquire into the adequacy of consideration Here however the business already had a legal duty under the original contract and did not agree to do anything else in exchange for the conservatoryrsquos promise to pay $60000 more

However an exception to the preexisting duty rule is sometimes applied in situations of unanticipated changed circumstances Under RESTATEMENT (SECOND) OF CONTRACTS sect 89 followed in many jurisdictions a promise modifying a duty under a contract not fully performed on either side is binding even if not supported by consideration if the modification is fair and equitable in view of circumstances not anticipated by the parties when the contract was made

If a court applies the rule in Restatement sect 89 the critical issues will be whether the modification was in fact ldquofair and equitablerdquo and whether it can be justified in light of unanticipated circumstances In many cases in which modifications have been upheld a party encountered difficulties or burdens in performing far beyond what was knowingly bargained for in the original contract with the result bordering on impracticability such as having to excavate solid rock instead of soft dirt or having to remove garbage far in excess of the amounts contemplated The conservatory would argue that the businessrsquos performance difficulties were not of this sort at allmdashnothing about repairing the pipe organ itself was any different from or more difficult than originally contemplated except that the business itself encountered financial distress unrelated to its burdens in performing its obligations under these contracts

Even if the business satisfies that element of the rule in Restatement sect 89 the business must also demonstrate that the circumstances that gave rise to the need to modify the contract were ldquounanticipatedrdquo at the time the original contract was made Here the facts suggest that when the business entered into the original contract it expected that the price paid by the conservatory would enable it to perform However any evidence that the business knew or had reason to know at the time of execution that it would need more money from the conservatory to be able to perform would mean that the request to modify was not ldquounanticipatedrdquo

[NOTE Some cases such as Schwartzreich v Bauman-Basch Inc 231 NY 196 131 NE 887 (1921) find that if the parties mutually agreed to rescind the original contract and then after rescission entered into an entirely new contract for a higher price the new contract is supported by consideration There is no evidence that such a rescission followed by a new contract took place here]

Point Two (45) The business can recover the additional $40000 for the new organ because no consideration is required under Article 2 of the UCC for good-faith contract modifications

The contract to buy a new organ is a contract for the sale of goods and therefore is governed by Article 2 of the Uniform Commercial Code UCC sect 2-102 Under Article 2 unlike the common law an agreement modifying a contract needs no consideration to be binding UCC sect 2-209(1) Section 2-209(1) thus obviates the preexisting duty rule entirely in contracts for the sale of goods

Even though consideration is not required modifications governed by sect 2-209 must satisfy the obligation of good faith imposed by the UCC UCC sect 1-304 See also Official Comment 2 to UCC sect 2-209 Good faith means ldquohonesty in fact and the observance of reasonable commercial standards of fair dealingrdquo UCC sect 1-201(b)(20) In this context the obligation of good faith means that ldquo[t]he effective use of bad faith to escape performance on the original contract terms is barred and the extortion of a lsquomodificationrsquo without legitimate commercial reason is ineffective as a violation of the duty of good faithrdquo Official Comment 2 to

17

Contracts Analysis

UCC sect 2-209 Here because the businessrsquos financial reversals were serious and apparently unanticipated at the time that the business entered into the contract with the conservatory and commitment of the extra money was needed to enable the business to perform a court would likely find that the business acted in good faith Thus a court would likely uphold the enforceability of the conservatoryrsquos promise to pay the additional $40000

Point Three (10) The conservatory is unlikely to be able to defend against enforcement of its promises to pay additional money under the theory of economic duress because the business probably did not make an improper threat

Under the common law of contracts parties may raise the defense of duress This common law defense also applies to contracts governed by UCC Article 2 See UCC sect 1-103(b)

A contract is voidable on the ground of economic duress by threat when it is established that a partyrsquos manifestation of assent is induced by an improper threat that leaves the party no reasonable alternative See RESTATEMENT (SECOND) OF CONTRACTS sect 175 See also eg Austin Instrument Inc v Loral Corp 272 NE2d 533 (NY 1971) (a threat to withhold essential goods can constitute duress) In order to void its agreement to pay the additional sum because of economic duress the conservatory must demonstrate that (1) the business made a threat to the conservatory (2) the threat was ldquoimproperrdquo or ldquowrongfulrdquo (3) the threat induced the conservatoryrsquos manifestation of assent to the modification and (4) the threat was sufficiently grave to justify the conservatoryrsquos assent

Here it appears that three of the four elements are likely satisfied The business plainly made a threat Moreover the threat induced the conservatoryrsquos assent to the modification and the threat was sufficiently grave to justify that assent If the conservatory had not agreed to pay the business the extra amounts the conservatory would have lost its entire $325000 investment In light of this potential loss a court could easily conclude that the conservatory had no reasonable alternative

However the business has a strong argument that its threat (indicating that it would breach the contracts unless the prices were increased) was not wrongful or improper but was instead nothing more than a communication of the reality of its own perilous situation to the conservatory

A mere threat to breach a contract is not in and of itself improper so as to support an action of economic duress or business compulsion Something more is required such as a breach of the duty of good faith and fair dealing as was present in Austin Instrument Inc supra Because the business could not perform the original contract without the requested modification the economic duress claim for the conservatory would likely fail for much the same reason that the business would be able to enforce the modification At the time the modification was requested the business was not trying to extort a price increase because of the conservatoryrsquos vulnerability but instead was simply stating the reality that the business could not perform without more money

18

FAMILY LAW ANALYSIS (Family Law IIIB D amp G)

ANALYSIS

Legal Problems

(1)(a) Does the State A court have jurisdiction to modify the State B child support order

(1)(b) Does the State A court have jurisdiction to modify the marital-residence-saleshyproceeds provision of the State B property-division decree

(2)(a) May a child support order be modified retroactively

(2)(b) May a child support order be modified prospectively based on a change of employment with a lower salary

(2)(c) May a property-division order be modified after entry of a divorce decree

DISCUSSION

Summary

The State A court may exercise personal jurisdiction over the wife because she was personally served in State A However subject-matter jurisdiction over the interstate modification of child support is governed by the Uniform Interstate Family Support Act (UIFSA) Under UIFSA State A does not have jurisdiction to modify the order for the daughterrsquos support because the wife is still a resident of State B UIFSA on the other hand does not govern property distributions and thus a State A court is not precluded from hearing the husbandrsquos petition to modify the marital-residence-sale-proceeds provision of the divorce decree

A child support order may not be modified retroactively A child support order may be modified prospectively based on a substantial change in circumstances Courts agree that a significant decrease in income is a substantial change in circumstances All states treat voluntary income reductions differently than involuntary reductions but employ different approaches for evaluating the impact of a voluntary reduction Whether the husband could obtain prospective modification of the child support order depends on which approach is applied

A property-division order is not subject to post-divorce modification based on a change in circumstances Thus the husband may in some states obtain prospective modification of the order for the daughterrsquos support but he may not obtain modification of the marital-residenceshysale-proceeds provision

Point One(a) (25) Personal jurisdiction over a nonresident respondent does not confer subject-matter jurisdiction over child support modification Under UIFSA a State A court may not modify a child support order issued by a State B court when as here the child or either parent continues to reside in State B the jurisdiction that issued the child support order

The State A court may exercise personal jurisdiction over the wife The wife was personally served in State A and a state may exercise jurisdiction based on in-state personal service See

19

Family Law Analysis

Burnham v Superior Court 495 US 604 (1990) But personal jurisdiction over the wife is not enough to give a State A court jurisdiction to modify the State B support order

The interstate enforcement and modification of child support is governed by the Uniform Interstate Family Support Act (UIFSA) which has been adopted by all states Under UIFSA the state that originally issued a child support order (here State B) has continuing exclusive jurisdiction to modify the order if that state remains the residence of the obligee the child or the obligor and all parties do not consent to the jurisdiction of another forum See UIFSA sect 205 See also UIFSA sect 603 (ldquoA tribunal of this State shall recognize and enforce but may not modify a registered order if the issuing tribunal had jurisdictionrdquo) The wife and daughter continue to reside in State B and the wife has not consented to the jurisdiction of another forum Thus a State A court does not have jurisdiction to modify the State B child support order

[NOTE Examinees who do not discuss personal jurisdiction but fully discuss UIFSA may receive full credit]

Point One(b) (15) UIFSA does not apply to disputes over property division Thus the State A court may exercise jurisdiction over the husbandrsquos petition to modify the marital-residence-sale-proceeds provision of the State B divorce decree because it has personal jurisdiction over the wife

The State A court in which the husband brought his action has jurisdiction to adjudicate domestic relations issues The husbandrsquos petition to modify the property settlement is a domestic relations issue The courts of State A may exercise personal jurisdiction over the wife because she was personally served in State A See Burnham v Superior Court 495 US 604 (1990) see Point One(a)

UIFSA does not apply to divorce property-division disputes Thus although a State A court may not adjudicate the husbandrsquos petition to modify his child support obligations it may adjudicate his property-division claims (Even though the court has jurisdiction it may not modify the property-division award on the merits See Point Two(c))

Point Two(a) (20) A child support order may not be modified retroactively

State courts have long held that obligations to pay child support ordinarily may not be modified retroactively ldquoIf the hardship is particularly severe the courts sometimes devised a way to protect the obligor but in most instances the courts hold that retroactive modification of this kind is beyond their power and indeed the governing statute may so providerdquo HOMER H CLARK THE LAW OF DOMESTIC RELATIONSHIPS IN THE UNITED STATES 725 (2d ed 1987)

Federal law now goes further and requires the states as a condition of federal child-support funding to adopt rules that absolutely forbid retroactive modification of the support obligation See 42 USC sect 666(a)(9)(C) The states have adopted rules consistent with the federal requirements

Point Two(b) (25) It is unclear whether the husband could obtain prospective downward modification of his child support based on his voluntary acceptance of a job with a lower salary

Prospective modification of a child support order is typically available only when the petitioner can show a substantial change in circumstances See ROBERT E OLIPHANT amp NANCY VER

20

Family Law Analysis

STEEGH FAMILY LAW 213ndash15 (3d ed 2010) A significant decrease in income is typically viewed as a substantial change

However when a parent seeks to modify a child support obligation because he has voluntarily reduced his income a court will not modify the obligation based solely on the income loss Some courts refuse to modify whenever the income shift was voluntary See eg Aguiar v Aguiar 127 P3d 234 (Idaho Ct App 2005) Others look primarily to the petitionerrsquos intentions and permit downward modification if he has acted in good faith See eg In re Marriage of Horn 650 NE2d 1103 (Ill App Ct 1995) Many courts use a multifactor approach See OLIPHANT amp VER STEEGH supra 217ndash18

Here there is no question that the husbandrsquos loss of income was voluntary In a jurisdiction in which voluntary income reduction bars support modification the husbandrsquos petition would be denied

In a jurisdiction employing a good-faith or multifactor approach it is possible but not certain that the husband could obtain downward modification The evidence supports the husbandrsquos good faith his change in employment appears to be based on his new jobrsquos greater responsibilities and better promotion possibilities In a jurisdiction using a multifactor approach the court would likely also consider the impact of such a shift on the daughter the likely duration of the husbandrsquos income loss and the likelihood of a promotion that would ultimately inure to the daughterrsquos benefit Thus on these facts it is possible but by no means certain that the husband could prospectively obtain downward modification of his child support obligation to his daughter

Point Two(c)(15) A divorce property-division award is not subject to modification

A support order is aimed at meeting the post-divorce needs of the supported individual Because the future is unpredictable courts are empowered to modify a support award to take account of changed circumstances that may occur during the period in which support is paid

By contrast a property-distribution award divides assets of the marriage based on the equities at the time of divorce Because the past can be ascertained a property-division award is not subject to post-divorce modification See HARRY A KRAUSE ET AL FAMILY LAW CASES COMMENTS AND QUESTIONS 691 (6th ed 2007)

Here the husband is seeking modification of a property-division award with respect to an asset owned by the parties at the time of divorce Thus the husband may not obtain a modification of the marital-residence-sale-proceeds provision of the divorce decree based on his reduced income

21

FEDERAL CIVIL PROCEDURE ANALYSIS (Federal Civil Procedure III IVC)

ANALYSIS

Legal Problems

(1) Is the logging company entitled to join this action as a matter of right

(2)(a) May the nonprofit organization obtain a temporary restraining order to stop the USFS from issuing a logging permit

(2)(b) May the nonprofit organization obtain a preliminary injunction to stop the USFS from issuing a logging permit during the pendency of the action

DISCUSSION

Summary

The logging company is entitled to intervene in this action as a matter of right because it has an interest in the property or transaction that is the subject of the action and is so situated that its interest may be impaired or impeded as a practical matter if the action goes forward without it The logging companyrsquos interest is not adequately represented by the USFSrsquos presence in the lawsuit

The nonprofit organization may seek a temporary restraining order (TRO) followed by a preliminary injunction to prevent the USFS from issuing a logging permit pending the outcome of the action The nonprofit is likely to obtain a TRO if it can demonstrate a risk of immediate and irreparable injury The nonprofit is also likely to obtain a preliminary injunction if it can demonstrate a significant threat of irreparable harm and a likelihood of success on the merits of its National Environmental Policy Act (NEPA) claim

Point One (50) Rule 24(a) of the Federal Rules of Civil Procedure requires federal courts to allow a person to intervene in an action as a matter of right if the person a) is interested in the property or transaction that is the subject of the action b) is so situated that its interest may be impaired or impeded if the litigation goes forward without it and c) is not adequately represented by existing parties Here the logging company likely meets all three requirements and should be allowed to intervene as a matter of right

Rule 24 of the Federal Rules of Civil Procedure governs intervention the process by which a non-party to an action may join the litigation Under Rule 24(a) (intervention of right) a person must be permitted to intervene if three conditions are met (1) the movant ldquoclaims an interest relating to the property or transaction that is the subject of the actionrdquo (2) the movant ldquois so situated that disposition of the action may as a practical matter impair or impede the movantrsquos ability to protect its interestrdquo and (3) ldquoexisting partiesrdquo do not ldquoadequately represent [the movantrsquos] interestrdquo FED R CIV P 24(a) The three requirements for intervention of right are often ldquovery interrelatedrdquo 7C CHARLES ALAN WRIGHT ET AL FEDERAL PRACTICE AND PROCEDURE sect 1908 at 297 (2007 amp 2011 Supp)

22

Federal Civil Procedure Analysis

Here the court should find that the logging company meets this test First the logging company has a strong interest in the property or transaction that is the subject of this action The USFS has accepted the logging companyrsquos bid and the logging company is merely awaiting issuance of a logging permit to begin logging The nonprofit organization is seeking to prevent this logging The logging company therefore has a strong direct and substantial interest in the subject matter of the lawsuit and in having its winning bid honored and a logging permit issued See eg Kleissler v US Forest Serv 157 F3d 964 972 (3d Cir 1998) (stating that ldquo[t]imber companies have direct and substantial interests in a lawsuit aimed at halting loggingrdquo) see also Natural Resources Defense Council v US Nuclear Regulatory Commrsquon 578 F2d 1341 1343ndash 44 (10th Cir 1978) (holding that applicants whose license renewals were pending had Rule 24(a)(2) interests where the lawsuit sought to halt the license-issuing process pending preparation of environmental impact statements) See generally 7C WRIGHT ET AL supra sect 19081 at 309 (ldquoIf there is a direct substantial legally protectable interest in the proceedings it is clear that this requirement of the rule is satisfiedrdquo) Second the logging companyrsquos interest in receiving a logging permit may well be impaired as a practical matter by the outcome of the lawsuit If the USFS loses the lawsuit it will have to prepare an environmental impact statement before issuing the logging companyrsquos permit This will at a minimum delay the logging companyrsquos ability to exercise its rights and may in the long r un mean that no logging permit is ever issued Intervention of right is not limited to those that would be legally bound as a matter of preclusion doctrine Id sect 19082 at 368 Rather ldquo[t]he rule is satisfied whenever disposition of the present action would put the movant at a practical disadvantage in protecting its interestrdquo Id sect 19082 at 369 Here that condition is easily satisfied See Kleissler 157 F3d at 972 (ldquoTimber companies have direct and substantial interests in a lawsuit aimed at halting logging rdquo)

Given that the logging company has an interest that may be impaired by disposition of the action it should be allowed to intervene unless the court is persuaded that the USFS adequately represents the logging companyrsquos interest See Rule 24(a)(2) 7C WRIGHT ET AL supra sect 1909 Here it could be argued that the USFS adequately represents the logging companyrsquos interest because the USFS presumably wants the court to uphold its development plan and allow it to proceed with issuance of the logging permit which is the same relief that the logging company would seek However whether representation is truly adequate depends upon ldquo[a] discriminating appraisal of the circumstancesrdquo 7C WRIGHT ET AL supra sect 1909 at 440 Although both the government and the logging company wish to avoid the preparation of an environmental impact statement their interests are distinct The USFSrsquos interest is proper management of the national forest system while the logging companyrsquos interest is making a profit from logging the 5000-acre tract The USFSrsquos handling of the litigation is likely to be affected by a variety of policy concerns and political considerations that have nothing to do with the logging companyrsquos purely economic interest in securing the right to cut trees in the Scenic National Forest See eg Kleissler 157 F3d at 973ndash74 (ldquo[T]he government represents numerous complex and conflicting interests in matters of this nature The straightforward business interests asserted by intervenors here may become lost in the thicket of sometimes inconsistent governmental policiesrdquo)

[NOTES (1) Examinees who mistakenly analyze the logging companyrsquos case for joinder under the related but incorrect Rule 19 ldquoRequired Joinder of Partiesrdquo may receive credit Rule 19 allows existing parties to demand joinder of non-parties (or seek dismissal of the case if they canrsquot get it) There is a close relationship between Rule 24 and Rule 19 and both contain a similar standard for determining when ldquointerestedrdquo third parties are ldquoentitledrdquo or ldquorequiredrdquo to be in the lawsuit Indeed the two prongs of the Rule 24 intervention test that are discussed above

23

Federal Civil Procedure Analysis

are nearly identical to the two prongs of the Rule 19(a) required joinder test Examinees who discuss and apply the test should receive credit even if they cite Rule 19 rather than Rule 24

(2) Examinees may discuss permissive joinder Although permissive joinder is a possibility here the question asks only whether the logging company can join the action as a matter of right and a permissive joinder analysis is not responsive to the question To the extent an examinee discusses permissive joinder the analysis will focus on whether the logging company ldquohas a claim or defense that shares with the main action a common question of law or factrdquo FED R CIV P 24(b)(1)(B) The district court also ldquomust consider whether the intervention will unduly delay or prejudice the adjudication of the original partiesrsquo rightsrdquo FED R CIV P 24(b)(3) On our facts the logging companyrsquos claim for the issuance of a logging permit would certainly share common questions of law and fact with the USFSrsquos defense against the nonprofitrsquos claim There are no facts suggesting that the logging companyrsquos presence would unduly delay or otherwise prejudice adjudication of the original action Thus the district court would have discretion to permit the logging company to intervene even if it denied intervention of right]

Point Two(a) (25) The nonprofit organization could seek and would likely obtain a temporary restraining order to stop the USFS from issuing a logging permit pending a hearing on an application for a preliminary injunction

The first type of interim relief the nonprofit could seek to stop the USFS from issuing a logging permit to the logging company is a temporary restraining order (TRO) prohibiting the USFS from issuing the logging permit A TRO can be issued without notice to the adverse party but only in limited circumstances and only for a limited time FED R CIV P 65(b) To secure a TRO without notice the nonprofit would need to submit an affidavit containing specific facts that demonstrate a risk of ldquoimmediate and irreparable injuryrdquo if a permit is issued FED R CIV P 65(b)(1) In deciding whether to grant a TRO courts will also consider the same factors that are relevant in deciding whether to grant a preliminary injunction (eg the moving partyrsquos likelihood of success on the merits the balance of hardships and the public interest) See Point Two(b) infra The TRO would last only long enough for the court to consider and resolve a request by the nonprofit for a preliminary injunction but no longer than 14 days (unless the court extends it for good cause or the adverse party consents to an extension) In addition bond is required

Here the court is likely to grant the nonprofitrsquos request The nonprofit could plausibly claim that cutting down 5000 acres of old-growth forest in an area that is home to the highest concentration of wildlife in the western United States would have ldquoan immediate and irreparablerdquo adverse impact on the environment and cause irreparable harm to the nonprofitrsquos interest in preserving and protecting natural resources including wildlife habitat

Point Two(b) (25) The nonprofit could also seek and would likely obtain a preliminary injunction to stop the USFS which is likely to be granted if the nonprofitrsquos claim that the USFS violated NEPA has a strong basis in fact and law

Because the TRO would be temporary the nonprofit would need to move for a preliminary injunction to prevent the USFS from issuing a logging permit throughout the pendency of the litigation Preliminary injunctions are injunctions that seek to ldquoprotect [the] plaintiff from

24

Federal Civil Procedure Analysis

irreparable injury and to preserve the courtrsquos power to render a meaningful decision after a trial on the meritsrdquo 11A CHARLES ALAN WRIGHT ET AL FEDERAL PRACTICE AND PROCEDURE sect 2947 at 112 (2013) Rule 65 of the Federal Rules of Civil Procedure sets out the procedural requirements for preliminary injunctions Preliminary injunctions may be granted only upon notice to the adverse party FED R CIV P 65(a)(1) and only if the movant ldquogives security in an amount that the court considers proper to pay the costs and damages sustained by any party found to have been wrongfully enjoined or restrainedrdquo FED R CIV P 65(c)

While Rule 65 sets out the procedural requirements for preliminary injunctive relief it does not specify the substantive grounds upon which it may be granted The courtrsquos discretion in ruling upon a motion for a preliminary injunction ldquois exercised in conformity with historic federal equity practicerdquo 11A WRIGHT ET AL supra sect 2947 at 114 The court typically considers four factors

(1) the significance of the threat of irreparable harm to the plaintiff if the injunction is not granted (2) the balance between this harm and the injury that granting the injunction would inflict on the defendant (3) the probability that the plaintiff will succeed on the merits and (4) the public interest

Id sect 2948 at 122ndash24 accord Habitat Educ Center v Bosworth 363 F Supp 2d 1070 1088 (ED Wis 2005) The most important of these factors is the risk of irreparable harm to the plaintiff 11A WRIGHT ET AL supra sect 29481 at 129 If the plaintiff has an adequate remedy at law (eg if money damages can compensate the plaintiff for its loss) then a preliminary injunction will be denied Id sect 29481

Here a court would likely conclude that the potential for environmental damage to the forest creates a significant threat of irreparable harm ldquo[E]nvironmental injury is often irreparable Courts have recognized that logging such as would occur [here] can have longshyterm environmental consequences and thus satisfy the irreparable injury criterionrdquo Habitat Educ Center 363 F Supp 2d at 1089 (citing Idaho Sporting Congress Inc v Alexander 222 F3d 562 569 (9th Cir 2000) (noting that the imminent and continuing logging activities presented ldquoevidence of environmental harm sufficient to tip the balance in favor of injunctive reliefrdquo)) Neighbors of Cuddy Mountain v US Forest Service 137 F3d 1372 1382 (9th Cir 1998) (stating that ldquo[t]he old growth forests plaintiffs seek to protect would if cut take hundreds of years to reproducerdquo) (internal citation omitted)) see also 11C WRIGHT ET AL supra sect 29481 at 151 (noting that ldquoa preliminary injunction has been issued to prevent harm to the environmentrdquo)

The second factor the balance between the harm to the plaintiff and the harm the defendant will suffer if the injunction is issued also appears to support issuance of a preliminary injunction here The USFS will have to wait before it can develop the Scenic National Forest and the logging company may lose money if the delay is prolonged These economic harms could be compensated monetarily if an injunction is issued inappropriately Where ldquoan injunction bond can compensate [the] defendant for any harm the injunction is likely to inflict the balance should be struck in favor of [the] plaintiffrdquo Id sect 29482 at 192 See also Habitat Educ Center 363 F Supp 2d at 1089 (stating that ldquothe relative absence of harmful effects on the Forest Service weighs in favor of granting the injunctionrdquo)

The third factor is the likelihood that the plaintiff will prevail on the merits Although there is limited information concerning the merits of the action the nonprofit alleges that the federal statute (NEPA) requires an environmental impact statement and further states that the USFS created no environmental impact analysis or statement at all Assuming that those

25

Federal Civil Procedure Analysis

allegations are correct it seems plausible to conclude that the nonprofit will be able to show a likelihood of success on the merits

Finally courts deciding whether or not to issue preliminary injunctive relief are to consider the public interest ldquoFocusing on this factor is another way of inquiring whether there are policy considerations that bear on whether the order should issuerdquo 11C WRIGHT ET AL supra sect 29484 at 214 If the court concludes that the nonprofit is likely to succeed on its NEPA claim because the USFS wrongfully failed to conduct an environmental impact assessment it is likely to find that the public interest would be served by restraining the USFS from proceeding with logging in a national forest See Heartwood Inc v US Forest Service 73 F Supp 2d 962 979 (SD Ill 1999) affrsquod on other grounds 230 F3d 947 (7th Cir 2000) (ldquoviolations by federal agencies of NEPArsquos provisions as established by Congress harm the public as well as the environmentrdquo)

Thus a court is very likely to grant a preliminary injunction if it concludes that the nonprofit has a significant likelihood of success on the merits

26

EVIDENCE ANALYSIS (Evidence ID IIA amp C)

ANALYSIS

Legal Problems

(1) Under what circumstances can evidence of prior convictions be used to impeach a witnessrsquos credibility in a civil case

(1)(a) May the inmatersquos credibility be impeached by evidence of a 12-year-old felony drug conviction if he was released from prison 9 years ago

(1)(b) May the inmatersquos credibility be impeached by evidence of an 8-year-old misdemeanor perjury conviction that was punishable by 1 year in jail if he pleaded guilty and was sentenced only to pay a $5000 fine

(1)(c) May the inmatersquos credibility be impeached by evidence of a 7-year-old sexual assault conviction if the inmate is still serving a 10-year prison sentence and the victim was his 13-year-old daughter

(2)(a) May the guardrsquos credibility be impeached by cross-examination regarding specific instances of misconduct (ie lying on his reacutesumeacute) relevant to credibility

(2)(b) May the guardrsquos credibility be impeached by admission of extrinsic evidence (his reacutesumeacute and academic transcript) offered to prove specific instances of misconduct relevant to credibility

DISCUSSION

Summary

Under the Federal Rules of Evidence witnesses can be impeached with evidence of prior convictions andor specific instances of misconduct Whether evidence of prior convictions should be admitted to impeach generally depends on the nature of the crime the amount of time that has passed and (only in criminal cases) whether the ldquowitnessrdquo is the defendant FED R EVID 609(a)

In this civil case evidence of the inmatersquos conviction for distribution of marijuana should be admitted to impeach the inmate because he was convicted of a felony and was released from prison fewer than 10 years ago FED R EVID 609(a)(1) Credibility is critically important in this case because the jury will hear conflicting testimony from the two disputing parties and there were no other eyewitnesses to the altercation Under Rule 609(a)(1) the inmatersquos conviction should be admitted because it has some bearing on his credibility and its probative value is not substantially outweighed by concerns of unfair prejudice confusion or delay Id

Evidence of the inmatersquos misdemeanor conviction for perjury must be admitted because the crime ldquorequired provingmdashor the witnessrsquos admittingmdasha dishonest act or false statementrdquo by the inmate FED R EVID 609(a)(2)

27

Evidence Analysis

Evidence of the inmatersquos felony conviction for sexual assault should be excluded because its probative value is substantially outweighed by the danger of unfair prejudice to the inmate based on the heinous nature of the crime FED R EVID 609(a)(1) In the alternative the judge could limit the evidence relating to this conviction by excluding details of the inmatersquos crime

In all civil (and criminal) cases witnesses can also be impeached with evidence of specific instances of prior misconduct that did not result in a conviction FED R EVID 608(b) Pursuant to Rule 608(b) misconduct probative of untruthfulness can be inquired into on cross-examination but cannot be proved through extrinsic evidence Id Thus the inmatersquos counsel should be permitted to cross-examine the guard regarding the false statement in the guardrsquos reacutesumeacute However extrinsic evidence of the guardrsquos misconduct (ie the guardrsquos authenticated reacutesumeacute and transcript from the local college) should not be admitted even if the guard denies wrongdoing or refuses to answer cross-examination questions about these matters Id

Point One (10) The Federal Rules of Evidence permit impeachment of witnesses with evidence of prior convictions

Whether convictions should be admitted to impeach generally depends on the nature of the crime the amount of time that has passed and (only in criminal cases) whether the ldquowitnessrdquo is the defendant FED R EVID 609(a) Under Rule 609(a) evidence of prior convictions may be admitted for the purpose of ldquoattacking a witnessrsquos character for truthfulnessrdquo Id

There are two basic types of convictions that can be admitted for the purpose of impeachment

(1) convictions for crimes ldquopunishable by death or by imprisonment for more than one yearrdquo (which generally correlates to ldquofeloniesrdquo) FED R EVID 609(a)(1) and (2) convictions ldquofor any crimes regardless of the punishment if the court can readily determine that establishing the elements of the crime required provingmdashor the witnessrsquos admittingmdasha dishonest act or false statementrdquo FED R EVID 609(a)(2)

Pursuant to Rule 609(a)(1) in civil cases the admission of evidence of a felony conviction is ldquosubject to Rule 403 [which says that a court may exclude relevant evidence if its probative value is substantially outweighed by other factors]rdquo FED R EVID 609(a)(1) However Rule 403 does not protect the witness against admission of prior convictions involving dishonestymdashwhich must be admitted by the court FED R EVID 609(a)(2)

Finally Federal Rule of Evidence 609(b) contains the presumption that a conviction that is more than 10 years old or where more than 10 years has passed since the witnessrsquos release from confinement (whichever is later) should not be admitted unless ldquoits probative value supported by specific facts and circumstances substantially outweighs its prejudicial effectrdquo and the proponent has provided the adverse party with reasonable written notice FED R EVID 609(b)

Point One(a) (25) The court should admit evidence of the inmatersquos 12-year-old felony marijuana distribution conviction

The inmatersquos conviction for marijuana distribution was for a felony punishable by imprisonment for more than one year See FED R EVID 609(a)(1) Moreover although the conviction was 12 years ago the 10-year time limit of Rule 609(b) is not exceeded because that time limit runs

28

Evidence Analysis

from the date of either ldquothe witnessrsquos conviction or release from confinement for it whichever is laterrdquo FED R EVID 609(b) Because the inmate served three years in prison he was released from confinement nine years ago

However pursuant to Rule 609(a)(1) the admission of felony convictions to impeach a witness in a civil case is ldquosubject to Rule 403rdquo FED R EVID 609(a)(1) Neither Rule 609(a) nor the advisory committee notes specify which factors courts should consider when balancing the probative value of a conviction against the dangers identified in Rule 403 (which include (1) unfair prejudice (2) confusion of the issues (3) misleading the jury (4) waste of time or undue delay and (5) needless presentation of cumulative evidence) FED R EVID 403

In this case credibility is very important because the evidence consists primarily of the testimony of the disputing parties and there were no other eyewitnesses to the altercation This enhances the probative value of any evidence bearing on the inmatersquos credibility A court is likely to conclude that the inmatersquos prior felony drug conviction is relevant to his credibility See eg United States v Brito 427 F3d 53 64 (1st Cir 2005) (ldquoPrior drug-trafficking crimes are generally viewed as having some bearing on veracityrdquo) Although the probative value of any conviction diminishes with age see eg United States v Brewer 451 F Supp 50 53 (ED Tenn 1978) the inmatersquos ongoing problems with the law suggest that he has continued (and even escalated) his criminal behavior over the past nine years The court should admit this evidence because its probative value is not substantially outweighed by any Rule 403 concerns Specifically any prejudice to the inmate would be slight because the conviction is unrelated to the altercation at issue and the conviction was not for a heinous crime that might inflame the jury

[NOTE Whether an examinee identifies the jury instruction as containing a ldquoconclusiverdquo or ldquomandatoryrdquo presumption is less important than the examineersquos analysis of the constitutional infirmities]

Point One(b) (15) The court must admit evidence of the inmatersquos eight-year-old misdemeanor conviction because perjury is a crime of dishonesty

Rule 609(a)(2) provides that evidence of a criminal conviction ldquomust be admitted if the court can readily determine that establishing the elements of the crime required provingmdashor the witnessrsquos admittingmdasha dishonest act or false statementrdquo FED R EVID 609(a)(2) The inmatersquos conviction for perjury would have necessarily required proving that the inmate engaged in an act of dishonesty This conviction occurred within the past 10 years so it ldquomust be admittedrdquo because in contrast to Rule 609(a)(1) (discussed in Point One(a)) admission under Rule 609(a)(2) is mandatory and not subject to Rule 403

Point One(c) (20) The court should exclude evidence of the inmatersquos seven-year-old felony sexual assault conviction because the probative value of this evidence is substantially outweighed by the danger of unfair prejudice In the alternative the details of the prior conviction could be excluded

The inmatersquos conviction for felony sexual assault was seven years ago and he has not yet been released from incarceration so Rule 609(a) but not 609(b) is applicable here FED R EVID 609(a) This conviction is therefore admissible to impeach the inmate unless its probative value is substantially outweighed by the danger of unfair prejudice or any other Rule 403 concern Id

29

Evidence Analysis

Sex crimes are generally not considered relevant to credibility see Hopkins v State 639 So 2d 1247 1254 (Miss 1993) so the probative value of this conviction is relatively low Moreover the heinous nature of the inmatersquos crime (sexual assault on his daughter) makes the danger of unfair prejudice to the inmate very high Thus the court should exclude evidence of the conviction because it was for a heinous offense that is likely to inflame the jury and it has little bearing on credibility See eg United States v Beahm 664 F2d 414 419 (4th Cir 1981)

As an alternative to excluding this evidence the judge could minimize the unfair prejudice to the inmate by permitting limited cross-examination but refusing to allow specific questions about the nature of the inmatersquos conviction For example a court could limit cross-examination to the fact that the inmate was convicted of a ldquofelonyrdquo or perhaps that he was convicted of a ldquosexual assaultrdquo without identifying the victim However because evidence of the inmatersquos prior convictions can be admitted solely for the purpose of enabling the jury to assess his credibility and because his two earlier convictions should have already been admitted the court should exclude all evidence of the felony sexual assault conviction

Point Two(a) (15) The court should permit the inmatersquos counsel to cross-examine the guard regarding the false statement in his reacutesumeacute because the guardrsquos misconduct bears on his truthfulness

The inmate wishes to cross-examine the guard about his prior dishonest behaviormdashlying on his reacutesumeacutemdashthat did not involve a criminal conviction Rule 608(b) allows witnesses to be cross-examined about specific instances of prior non-conviction misconduct probative of untruthfulness ldquoin order to attack the witnessrsquos character for truthfulnessrdquo FED R EVID 608(b)

The courtrsquos decision to allow cross-examination about the guardrsquos prior dishonest behavior depends on the probative value of such evidence balanced against the danger of unfair prejudice to the guard or any other Rule 403 concern FED R EVID 403 Here the guardrsquos false statement on his reacutesumeacute that he obtained a degree in Criminal Justice is highly probative of his untruthfulness because it grossly misrepresents his actual academic record was made recently and was made with the intent to deceive Because the probative value of this evidence is very strong and is not substantially outweighed by any Rule 403 concerns cross-examination of the guard on this topic should be permitted The court may also consider it fair to permit this cross-examination of the guard on these matters assuming that one or more of the inmatersquos prior convictions have been admitted to impeach his credibility

Point Two(b) (15) The court should exclude extrinsic evidence of the guardrsquos non-conviction misconduct even if the guard denies wrongdoing or refuses to answer questions about the matter

Although Rule 608(b) allows cross-examination about specific instances of prior misconduct probative of untruthfulness ldquoextrinsic evidencerdquo offered to prove such misconduct is not admissible FED R EVID 608(b) The rationale for this rule is that allowing the introduction of extrinsic evidence of prior misconduct by witnesses when these acts are relevant only to the witnessesrsquo truthfulness and not to the main issues in the case would create too great a risk of confusing the jury and unduly delaying the trial The court does not have discretion to admit this extrinsic evidence See eg United States v Elliot 89 F3d 1360 1368 (8th Cir 1996)

30

Evidence Analysis

Here the inmatersquos counsel may cross-examine the guard about the false statement on his reacutesumeacute However the inmatersquos counsel must accept the guardrsquos response Even if the guard denies wrongdoing or refuses to answer questions about the matter the inmatersquos counsel cannot introduce the guardrsquos reacutesumeacute or the transcript from the local college to prove the guardrsquos misconduct

31

CORPORATIONS ANALYSIS (Corporations VA2 IX)

ANALYSIS

Legal Problems

(1) Do shareholders have the authority to amend a corporationrsquos bylaws with respect to director nominations

(2) Do board-approved bylaws on a particular subject here nomination of directors preempt subsequent conflicting bylaw amendments by shareholders

(3) Is a suit challenging both managementrsquos refusal to include the proposed bylaw amendment in Megarsquos proxy statement and the boardrsquos amendment of the bylaws dealing with nomination of directors a direct or derivative suit

DISCUSSION

Summary

The voting and litigation rights of the shareholders of Mega are subject to the provisions of the Model Business Corporations Act (MBCA)

The investorrsquos proposed bylaw provision is not inconsistent with state law Under the MBCA shareholders may amend the bylaws when the amendment deals with a proper matter for the corporationrsquos bylaws such as procedures for nominating directors

The Mega boardrsquos bylaw amendment does not preempt the investorrsquos proposed bylaw provision or the Mega shareholdersrsquo power to approve it While shareholders can limit the boardrsquos power to amend or repeal the bylaws the board cannot limit the shareholdersrsquo power

Whether the investor must make a demand on Megarsquos board depends on how the investor frames its claim If the investor claims a violation of shareholder voting rights the claim is direct and pre-suit demand on the board is not required If on the other hand the investor claims that the directors violated their fiduciary duties by amending the bylaws to entrench themselves the claim is derivative and a pre-suit demand is required

Point One (30) Shareholders may amend the corporationrsquos bylaws where the proposed bylaw provision relates to procedural matters typically included in the bylaws such as the nomination of directors

Internal affairs of the corporation such as the conduct of shareholder meetings and election of directors are subject to the corporate law of the state of incorporation See McDermott Inc v Lewis 531 A2d 206 (Del 1987) (applying law of jurisdiction where corporation was incorporated in case involving voting rights) This statersquos corporate statute is modeled on the MBCA

Under the MBCA ldquoshareholders may amend the corporationrsquos bylawsrdquo MBCA sect 1020(a) Thus the only question is whether the bylaws can specify the procedures for shareholder nomination of directors

32

Corporations Analysis

The MBCA states that the bylaws ldquomay contain any provision that is not inconsistent with law or the articles of incorporationrdquo MBCA sect 206(b) In addition the MBCA was revised in 2009 to address shareholder nomination of directors in public corporations (known as ldquoproxy accessrdquo) and specifies that the bylaws ldquomay contain a requirement that the corporation include in its [proxy materials] one or more individuals nominated by a shareholderrdquo MBCA sect 206(c)(1) see Committee on Corporate Laws ABA Section of Business Law Report on the Roles of Boards of Directors and Shareholders of Publicly Owned Corporations and Changes to the Model Business Corporations ActmdashAdoption of Shareholder Proxy Access Amendments to Chapters 2 and 10 65 BUS LAWYER 1105 (2010)

The inclusion of director-nomination procedures in the bylaws is consistent with practice and is recognized by the Delaware courts whose views on corporate law carry significant weight Typically the procedures for nomination of directors are found in the bylaws See 1 COX amp HAZEN TREATISE ON THE LAW OF CORPORATIONS sect 312 (3d ed 2011) see also 4 FLETCHER CORP FORMS ANN PART III ch 21 (2013) (including sample bylaws that permit nomination of directors by shareholders) The Delaware Supreme Court has confirmed that the bylaws may ldquodefine the process and proceduresrdquo for director elections See CA Inc v AFSCME Employees Pension Plan 953 A2d 227 (Del 2008) (concluding that bylaw amendment requiring reimbursement of election expenses to certain successful shareholder nominators is ldquoproper subjectrdquo under Delaware law)

[NOTE The question of the proper scope of the bylaws can be answered using the more general MBCA sect 206(b) or the 2009 MBCA revision adding sect 206(c)(1) (adopted in CT ME VA) In addition some examinees might raise the point that shareholder proposals may not compel the board to take action such as by including shareholder nominations in the companyrsquos proxy materials on the theory that the ldquobusiness and affairsrdquo of the corporation are to be managed by the board See MBCA sect 801(b) Although shareholders are generally limited to adopting precatory resolutions that recommend or encourage board action this limitation does not apply when shareholders have specific authority to take binding action on their ownmdashsuch as to amend the bylaws]

Point Two (30) Shareholders can amend (or repeal) board-approved bylaws Further shareholders can limit the boardrsquos power to later amend and repeal a shareholder-approved bylaw

Under the MBCA shareholders have the power to amend the bylaws See Point One The board shares this power with the shareholders unless (1) the corporationrsquos articles ldquoreserve that power exclusively to the shareholdersrdquo or (2) ldquothe shareholders in amending repealing or adopting a bylaw expressly provide that the board of directors may not amend repeal or reinstate that bylawrdquo See MBCA sect 1020(b)

Shareholder-approved bylaw provisions can amend or repeal existing bylaw provisions whether originally approved by the board or by shareholders See ALAN R PALMITER CORPORATIONS EXAMPLES AND EXPLANATIONS sect 713 (7th ed 2012) Thus the Mega boardrsquos bylaw amendmentmdashwhich set more demanding thresholds for shareholder nomination of directors than the investorrsquos proposed bylaw provisionmdashwould be superseded (repealed) if Megarsquos shareholders were to approve the investorrsquos proposal

Further a shareholder-approved bylaw generally can limit the power of the board to later amend or repeal it See MBCA sect 1020(b)(2) Thus if Megarsquos shareholders approved the bylaw

33

Corporations Analysis

provision proposed by the investor Megarsquos board could not repeal the provision because it includes a ldquono board repealrdquo clause

The revision to the MBCA in 2009 dealing with shareholder proxy access does not change this conclusion That revision specifies that a shareholder-approved bylaw dealing with director nominations may not limit the boardrsquos power to amend add or repeal ldquoany procedure or condition to such a bylaw in order to provide for a reasonable practicable and orderly processrdquo MBCA sect 206(d) Thus according to the revision if shareholders approve a bylaw amendment that limits further board changes the board would nonetheless retain the power to ldquotinkerrdquo with the bylaw to safeguard the voting process but could not repeal the shareholder-approved bylaw The Official Comment to MBCA sect 206(d) makes clear that the revision is ldquonot intended to allow the board of directors to frustrate the purpose of the shareholder-adopted proxy access provisionrdquo Thus if Megarsquos shareholders were to approve the bylaw provision proposed by the investor Megarsquos board could only amend the provision regarding its procedures or conditions in a manner consistent with its purpose of permitting proxy access for Megarsquos shareholders

[NOTE The boardrsquos attempted interference with a shareholder voting initiative may also have been a violation of the directorsrsquo fiduciary duties See Blasius Indus Inc v Atlas Corp 564 A2d 651 (Del Ch 1988) (finding that directors breached their fiduciary duties by amending bylaws and expanding size of board to thwart insurgentrsquos plan to amend bylaws and seat a majority of new directors) The call however asks examinees to consider whether shareholders or the board have ldquoprecedencerdquo over amending the corporate bylaws Thus an examineersquos answer should be framed in terms of ldquopowerrdquo and not ldquodutyrdquo]

Point Three (40) The investor need not make a demand on the board if the investor states a direct claim such as an allegation that the board interfered with the investorrsquos right to amend the bylaws But the investor must make a demand on the board if the investor states a derivative claim (on behalf of the corporation) such as an allegation that the directors sought to entrench themselves by interfering with the proposed proxy access

The MBCA generally requires that shareholders make a demand on the board of directors before initiation of a derivative suit MBCA sect 742 (shareholder may not bring derivative proceeding until written demand has been made on corporation and 90 days have expired) A derivative suit is essentially two suits in one where the plaintiff-shareholder seeks to bring on behalf of the corporation a claim that vindicates corporate rights usually based on violation of fiduciary duties PALMITER supra sect 1811 (6th ed 2009) The demand permits the board to investigate the situation identified by the shareholder and take suitable action No demand on the board is required however if the shareholder brings a direct suit to vindicate the shareholderrsquos own rights not those of the corporation

Is the suit brought by the investor derivative or direct The MBCA defines a ldquoderivative proceedingrdquo as one brought ldquoin the right of a domestic corporationrdquo MBCA sect 740(1) Thus the answer to how the investorrsquos suit should be characterized turns on what rights the investor seeks to vindicate If the investor frames its claim as one of fiduciary breach by directorsmdashfor example for failing to become adequately informed about voting procedures or for seeking to entrench themselves in office by manipulating the voting structure to avoid a shareholder insurgencymdashthen the suit is ldquoderivativerdquo and the investor must make a demand on the board See MBCA Ch 7 Subch D Introductory Comment (ldquothe derivative suit has historically been the principal method of challenging allegedly illegal action by managementrdquo)

34

Corporations Analysis

If however the investor frames its claim as one to vindicate shareholder rights the suit is direct and no demand is required For many courts the direct-derivative question turns on who is injured and who is to receive the relief sought by the plaintiff-shareholders See Tooley v Donaldson Lufkin amp Jenrette Inc 845 A2d 1031 (Del 2004) (characterizing a merger-delay claim as direct because delay of merger only harmed shareholders not corporation) Thus if the investor claims that managementrsquos refusal to include its proposed bylaw amendment in the corporationrsquos proxy materials violates its shareholder rights to initiate corporate governance reforms the suit will be direct Courts have not questioned the ability of shareholders to bring direct suits challenging board action to exclude their proposed bylaw amendments from the corporationrsquos proxy materials See JANA Master Fund Ltd v CNET Networks Inc 954 A2d 335 (Del Ch 2008) (upholding shareholderrsquos direct challenge to boardrsquos interpretation of advance-notice bylaw) Chesapeake Corp v Shore 771 A2d 293 (Del Ch 2000) (upholding shareholderrsquos direct challenge to actions by board that effectively prevented it from proposing bylaw amendments in contest for control)

Is the way that the investor frames its claim conclusive Courts have permitted shareholder-plaintiffs to challenge a transaction in a direct suit even though the same transaction could also be challenged as a fiduciary breach See Eisenberg v Flying Tiger Line Inc 451 F2d 267 (2d Cir 1971) (permitting direct suit challenging a corporate reorganization as a dilution of shareholder voting power even though reorganization may have involved conflicts of interest and thus constituted a fiduciary breach) Thus the investorrsquos choice to pursue a claim challenging the legality of managementrsquos decision to exclude the investorrsquos proposal from the corporationrsquos proxy materialsmdashrather than a possible breach of fiduciary dutymdashis likely to be respected See 3 COX amp HAZEN supra sect 153 (describing situations in which a claim can be framed as derivative or direct)

[NOTE Some issues under Delaware corporate law regarding pre-suit demand are not relevant here For example whether the Mega directors are independent and disinterested is not relevant to the MBCA requirement of a pre-suit demand As the Official Comment to MBCA sect 742 points out the MBCArsquos requirement of ldquouniversal demandrdquo gives the board ldquothe opportunity to reexamine the act complained of in the light of a potential lawsuit and take corrective actionrdquo even when the directors might be non-independent or have conflicts of interest

Nor is it relevant to the MBCA pre-suit demand requirement that the statutory 90-day waiting period may be onerous The first paragraph of MBCA sect 742 requires a pre-suit demand without exception the second paragraph of the section imposes a 90-day waiting period before a derivative suit may be brought which can be shortened if the board rejects the demand or ldquoirreparable injury to the corporation would result by waiting for the expiration of the 90-day periodrdquo The call as written asks only whether a pre-suit demand should be made and does not ask examinees to address whether the post-demand waiting period should be shortened under the ldquoirreparable injuryrdquo standard]

35

National Conference of Bar Examiners 302 South Bedford Street | Madison WI 53703-3622 Phone 608-280-8550 | Fax 608-280-8552 | TDD 608-661-1275

wwwncbexorg e-mail contactncbexorg

  • Preface
  • Description of the MEE
  • Instructions
  • July 2014 Questions
    • CRIMINAL LAW AND PROCEDURE QUESTION
    • CONTRACTS QUESTION
    • FAMILY LAW QUESTION
    • FEDERAL CIVIL PROCEDURE QUESTION
    • EVIDENCE QUESTION
    • CORPORATIONS QUESTION
      • July 2014 Analyses
        • CRIMINAL LAW AND PROCEDURE ANALYSIS
        • CONTRACTS ANALYSIS
        • FAMILY LAW ANALYSIS
        • FEDERAL CIVIL PROCEDURE ANALYSIS
        • EVIDENCE ANALYSIS
        • CORPORATIONS ANALYSIS
            • ltlt13 ASCII85EncodePages false13 AllowTransparency false13 AutoPositionEPSFiles true13 AutoRotatePages None13 Binding Left13 CalGrayProfile (Dot Gain 20)13 CalRGBProfile (sRGB IEC61966-21)13 CalCMYKProfile (US Web Coated 050SWOP051 v2)13 sRGBProfile (sRGB IEC61966-21)13 CannotEmbedFontPolicy Error13 CompatibilityLevel 1413 CompressObjects Tags13 CompressPages true13 ConvertImagesToIndexed true13 PassThroughJPEGImages true13 CreateJobTicket false13 DefaultRenderingIntent Default13 DetectBlends true13 DetectCurves 0000013 ColorConversionStrategy CMYK13 DoThumbnails false13 EmbedAllFonts true13 EmbedOpenType false13 ParseICCProfilesInComments true13 EmbedJobOptions true13 DSCReportingLevel 013 EmitDSCWarnings false13 EndPage -113 ImageMemory 104857613 LockDistillerParams false13 MaxSubsetPct 10013 Optimize true13 OPM 113 ParseDSCComments true13 ParseDSCCommentsForDocInfo true13 PreserveCopyPage true13 PreserveDICMYKValues true13 PreserveEPSInfo true13 PreserveFlatness true13 PreserveHalftoneInfo false13 PreserveOPIComments true13 PreserveOverprintSettings true13 StartPage 113 SubsetFonts true13 TransferFunctionInfo Apply13 UCRandBGInfo Preserve13 UsePrologue false13 ColorSettingsFile ()13 AlwaysEmbed [ true13 ]13 NeverEmbed [ true13 ]13 AntiAliasColorImages false13 CropColorImages true13 ColorImageMinResolution 30013 ColorImageMinResolutionPolicy OK13 DownsampleColorImages true13 ColorImageDownsampleType Bicubic13 ColorImageResolution 30013 ColorImageDepth -113 ColorImageMinDownsampleDepth 113 ColorImageDownsampleThreshold 15000013 EncodeColorImages true13 ColorImageFilter DCTEncode13 AutoFilterColorImages true13 ColorImageAutoFilterStrategy JPEG13 ColorACSImageDict ltlt13 QFactor 01513 HSamples [1 1 1 1] VSamples [1 1 1 1]13 gtgt13 ColorImageDict ltlt13 QFactor 01513 HSamples [1 1 1 1] VSamples [1 1 1 1]13 gtgt13 JPEG2000ColorACSImageDict ltlt13 TileWidth 25613 TileHeight 25613 Quality 3013 gtgt13 JPEG2000ColorImageDict ltlt13 TileWidth 25613 TileHeight 25613 Quality 3013 gtgt13 AntiAliasGrayImages false13 CropGrayImages true13 GrayImageMinResolution 30013 GrayImageMinResolutionPolicy OK13 DownsampleGrayImages true13 GrayImageDownsampleType Bicubic13 GrayImageResolution 30013 GrayImageDepth -113 GrayImageMinDownsampleDepth 213 GrayImageDownsampleThreshold 15000013 EncodeGrayImages true13 GrayImageFilter DCTEncode13 AutoFilterGrayImages true13 GrayImageAutoFilterStrategy JPEG13 GrayACSImageDict ltlt13 QFactor 01513 HSamples [1 1 1 1] VSamples [1 1 1 1]13 gtgt13 GrayImageDict ltlt13 QFactor 01513 HSamples [1 1 1 1] VSamples [1 1 1 1]13 gtgt13 JPEG2000GrayACSImageDict ltlt13 TileWidth 25613 TileHeight 25613 Quality 3013 gtgt13 JPEG2000GrayImageDict ltlt13 TileWidth 25613 TileHeight 25613 Quality 3013 gtgt13 AntiAliasMonoImages false13 CropMonoImages true13 MonoImageMinResolution 120013 MonoImageMinResolutionPolicy OK13 DownsampleMonoImages true13 MonoImageDownsampleType Bicubic13 MonoImageResolution 120013 MonoImageDepth -113 MonoImageDownsampleThreshold 15000013 EncodeMonoImages true13 MonoImageFilter CCITTFaxEncode13 MonoImageDict ltlt13 K -113 gtgt13 AllowPSXObjects false13 CheckCompliance [13 None13 ]13 PDFX1aCheck false13 PDFX3Check false13 PDFXCompliantPDFOnly false13 PDFXNoTrimBoxError true13 PDFXTrimBoxToMediaBoxOffset [13 00000013 00000013 00000013 00000013 ]13 PDFXSetBleedBoxToMediaBox true13 PDFXBleedBoxToTrimBoxOffset [13 00000013 00000013 00000013 00000013 ]13 PDFXOutputIntentProfile ()13 PDFXOutputConditionIdentifier ()13 PDFXOutputCondition ()13 PDFXRegistryName ()13 PDFXTrapped False1313 CreateJDFFile false13 Description ltlt13 ARA 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 BGR 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 CHS ltFEFF4f7f75288fd94e9b8bbe5b9a521b5efa7684002000410064006f006200650020005000440046002065876863900275284e8e9ad88d2891cf76845370524d53705237300260a853ef4ee54f7f75280020004100630072006f0062006100740020548c002000410064006f00620065002000520065006100640065007200200035002e003000204ee553ca66f49ad87248672c676562535f00521b5efa768400200050004400460020658768633002gt13 CHT ltFEFF4f7f752890194e9b8a2d7f6e5efa7acb7684002000410064006f006200650020005000440046002065874ef69069752865bc9ad854c18cea76845370524d5370523786557406300260a853ef4ee54f7f75280020004100630072006f0062006100740020548c002000410064006f00620065002000520065006100640065007200200035002e003000204ee553ca66f49ad87248672c4f86958b555f5df25efa7acb76840020005000440046002065874ef63002gt13 CZE 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 DAN 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 DEU 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 ESP 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 ETI 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 FRA 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 GRE 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 HEB ltFEFF05D405E905EA05DE05E905D5002005D105D405D205D305E805D505EA002005D005DC05D4002005DB05D305D9002005DC05D905E605D505E8002005DE05E105DE05DB05D9002000410064006F006200650020005000440046002005D405DE05D505EA05D005DE05D905DD002005DC05D405D305E405E105EA002005E705D305DD002D05D305E405D505E1002005D005D905DB05D505EA05D905EA002E002005DE05E105DE05DB05D90020005000440046002005E905E005D505E605E805D5002005E005D905EA05E005D905DD002005DC05E405EA05D905D705D4002005D105D005DE05E605E205D505EA0020004100630072006F006200610074002005D5002D00410064006F00620065002000520065006100640065007200200035002E0030002005D505D205E805E105D005D505EA002005DE05EA05E705D305DE05D505EA002005D905D505EA05E8002E05D005DE05D905DD002005DC002D005000440046002F0058002D0033002C002005E205D905D905E005D5002005D105DE05D305E805D905DA002005DC05DE05E905EA05DE05E9002005E905DC0020004100630072006F006200610074002E002005DE05E105DE05DB05D90020005000440046002005E905E005D505E605E805D5002005E005D905EA05E005D905DD002005DC05E405EA05D905D705D4002005D105D005DE05E605E205D505EA0020004100630072006F006200610074002005D5002D00410064006F00620065002000520065006100640065007200200035002E0030002005D505D205E805E105D005D505EA002005DE05EA05E705D305DE05D505EA002005D905D505EA05E8002Egt13 HRV (Za stvaranje Adobe PDF dokumenata najpogodnijih za visokokvalitetni ispis prije tiskanja koristite ove postavke Stvoreni PDF dokumenti mogu se otvoriti Acrobat i Adobe Reader 50 i kasnijim verzijama)13 HUN 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 ITA 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 JPN ltFEFF9ad854c18cea306a30d730ea30d730ec30b951fa529b7528002000410064006f0062006500200050004400460020658766f8306e4f5c6210306b4f7f75283057307e305930023053306e8a2d5b9a30674f5c62103055308c305f0020005000440046002030d530a130a430eb306f3001004100630072006f0062006100740020304a30883073002000410064006f00620065002000520065006100640065007200200035002e003000204ee5964d3067958b304f30533068304c3067304d307e305930023053306e8a2d5b9a306b306f30d530a930f330c8306e57cb30818fbc307f304c5fc59808306730593002gt13 KOR ltFEFFc7740020c124c815c7440020c0acc6a9d558c5ec0020ace0d488c9c80020c2dcd5d80020c778c1c4c5d00020ac00c7a50020c801d569d55c002000410064006f0062006500200050004400460020bb38c11cb97c0020c791c131d569b2c8b2e4002e0020c774b807ac8c0020c791c131b41c00200050004400460020bb38c11cb2940020004100630072006f0062006100740020bc0f002000410064006f00620065002000520065006100640065007200200035002e00300020c774c0c1c5d0c11c0020c5f40020c2180020c788c2b5b2c8b2e4002egt13 LTH 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 LVI 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 NLD (Gebruik deze instellingen om Adobe PDF-documenten te maken die zijn geoptimaliseerd voor prepress-afdrukken van hoge kwaliteit De gemaakte PDF-documenten kunnen worden geopend met Acrobat en Adobe Reader 50 en hoger)13 NOR 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 POL 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 PTB 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 RUM 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 RUS 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 SKY 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 SLV 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 SUO 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 SVE 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 TUR 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 UKR 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 ENU (Use these settings to create Adobe PDF documents best suited for high-quality prepress printing Created PDF documents can be opened with Acrobat and Adobe Reader 50 and later)13 gtgt13 Namespace [13 (Adobe)13 (Common)13 (10)13 ]13 OtherNamespaces [13 ltlt13 AsReaderSpreads false13 CropImagesToFrames true13 ErrorControl WarnAndContinue13 FlattenerIgnoreSpreadOverrides false13 IncludeGuidesGrids false13 IncludeNonPrinting false13 IncludeSlug false13 Namespace [13 (Adobe)13 (InDesign)13 (40)13 ]13 OmitPlacedBitmaps false13 OmitPlacedEPS false13 OmitPlacedPDF false13 SimulateOverprint Legacy13 gtgt13 ltlt13 AddBleedMarks false13 AddColorBars false13 AddCropMarks false13 AddPageInfo false13 AddRegMarks false13 ConvertColors ConvertToCMYK13 DestinationProfileName ()13 DestinationProfileSelector DocumentCMYK13 Downsample16BitImages true13 FlattenerPreset ltlt13 PresetSelector MediumResolution13 gtgt13 FormElements false13 GenerateStructure false13 IncludeBookmarks false13 IncludeHyperlinks false13 IncludeInteractive false13 IncludeLayers false13 IncludeProfiles false13 MultimediaHandling UseObjectSettings13 Namespace [13 (Adobe)13 (CreativeSuite)13 (20)13 ]13 PDFXOutputIntentProfileSelector DocumentCMYK13 PreserveEditing true13 UntaggedCMYKHandling LeaveUntagged13 UntaggedRGBHandling UseDocumentProfile13 UseDocumentBleed false13 gtgt13 ]13gtgt setdistillerparams13ltlt13 HWResolution [2400 2400]13 PageSize [612000 792000]13gtgt setpagedevice13

Page 4: July 2014 MEE Questions and Analyses - NCBE...This publication includes the questions and analyses from the July 2014 MEE. (In the actual test, the questions are simply numbered rather

Preface

The Multistate Essay Examination (MEE) is developed by the National Conference of Bar Examiners (NCBE) This publication includes the questions and analyses from the July 2014 MEE (In the actual test the questions are simply numbered rather than being identified by area of law) The instructions for the test appear on page iii

The model analyses for the MEE are illustrative of the discussions that might appear in excellent answers to the questions They are provided to the user jurisdictions to assist graders in grading the examination They address all the legal and factual issues the drafters intended to raise in the questions

The subjects covered by each question are listed on the first page of its accompanying analysis identified by roman numerals that refer to the MEE subject matter outline for that subject For example the Federal Civil Procedure question on the July 2014 MEE tested the following areas from the Federal Civil Procedure outline III Injunctions and provisional remedies and IVC Pretrial proceduresmdashJoinder of parties and claims (including class actions)

For more information about the MEE including subject matter outlines visit the NCBE website at wwwncbexorg

Description of the MEE

The MEE consists of six 30-minute questions and is a component of the Uniform Bar Examination (UBE) It is administered by user jurisdictions as part of the bar examination on the Tuesday before the last Wednesday in February and July of each year Areas of law that may be covered on the MEE include the following Business Associations (Agency and Partnership Corporations and Limited Liability Companies) Conflict of Laws Constitutional Law Contracts Criminal Law and Procedure Evidence Family Law Federal Civil Procedure Real Property Torts Trusts and Estates (Decedentsrsquo Estates Trusts and Future Interests) and Uniform Commercial Code (Negotiable Instruments and Bank Deposits and Collections Secured Transactions) (Effective with the February 2015 MEE administration Uniform Commercial Code Articles 3 and 4 [Negotiable Instruments and Bank Deposits and Collections] is no longer being tested) Some questions may include issues in more than one area of law The particular areas covered vary from exam to exam

The purpose of the MEE is to test the examineersquos ability to (1) identify legal issues raised by a hypothetical factual situation (2) separate material which is relevant from that which is not (3) present a reasoned analysis of the relevant issues in a clear concise and well-organized composition and (4) demonstrate an understanding of the fundamental legal principles relevant to the probable solution of the issues raised by the factual situation The primary distinction between the MEE and the Multistate Bar Examination (MBE) is that the MEE requires the examinee to demonstrate an ability to communicate effectively in writing

ii

Instructions The back cover of each test booklet contains the following instructions

You will be instructed when to begin and when to stop this test Do not break the seal on this booklet until you are told to begin

You may answer the questions in any order you wish Do not answer more than one question in each answer booklet If you make a mistake or wish to revise your answer simply draw a line through the material you wish to delete

If you are using a laptop computer to answer the questions your jurisdiction will provide you with specific instructions

Read each fact situation very carefully and do not assume facts that are not given in the question Do not assume that each question covers only a single area of the law some of the questions may cover more than one of the areas you are responsible for knowing

Demonstrate your ability to reason and analyze Each of your answers should show an understanding of the facts a recognition of the issues included a knowledge of the applicable principles of law and the reasoning by which you arrive at your conclusions The value of your answer depends not as much upon your conclusions as upon the presence and quality of the elements mentioned above

Clarity and conciseness are important but make your answer complete Do not volunteer irrelevant or immaterial information

Answer all questions according to generally accepted fundamental legal principles unless your testing jurisdiction has instructed you to answer according to local case or statutory law

NOTE Examinees testing in UBE jurisdictions must answer according to generally accepted fundamental legal principles rather than local case or statutory law

iii

July 2014 MEE

QUESTIONS

Contracts Criminal Law and Procedure

Family L aw Federal Civil Procedure

Evidence Corporations

CRIMINAL LAW AND PROCEDURE QUESTION

While on routine patrol a police officer observed a suspect driving erratically and pulled the suspectrsquos car over to investigate When he approached the suspectrsquos car the officer detected a strong odor of marijuana The officer immediately arrested the suspect for driving under the influence of an intoxicant (DUI) While the officer was standing near the suspectrsquos car placing handcuffs on the suspect the officer observed burglary tools on the backseat

The officer seized the burglary tools He then took the suspect to the county jail booked him for the DUI and placed him in a holding cell Later that day the officer gave the tools he had found in the suspectrsquos car to a detective who was investigating a number of recent burglaries in the neighborhood where the suspect had been arrested

At the time of his DUI arrest the suspect had a six-month-old aggravated assault charge pending against him and was being represented on the assault charge by a lawyer

Early the next morning upon learning of her clientrsquos arrest the lawyer went to the jail She arrived at 900 am immediately identified herself to the jailer as the suspectrsquos attorney and demanded to speak with the suspect The lawyer also told the jailer that she did not want the suspect questioned unless she was present The jailer told the lawyer that she would need to wait one hour to see the suspect After speaking with the lawyer the jailer did not inform anyone of the lawyerrsquos presence or her demands

The detective who had also arrived at the jail at 900 am overheard the lawyerrsquos conversation with the jailer The detective then entered the windowless interview room in the jail where the suspect had been taken 30 minutes earlier Without informing the suspect of the lawyerrsquos presence or her demands the detective read to the suspect full and accurate Miranda warnings The detective then informed the suspect that he wanted to ask about the burglary tools found in his car and the recent burglaries in the neighborhood where he had been arrested The suspect replied ldquoI think I want my lawyer here before I talk to yourdquo The detective responded ldquoThatrsquos up to yourdquo

After a few minutes of silence the suspect said ldquoWell unless there is anything else I need to know letrsquos not waste any time waiting for someone to call my attorney and having her drive here I probably should keep my mouth shut but Irsquom willing to talk to you for a whilerdquo The suspect then signed a Miranda waiver form and after interrogation by the detective made incriminating statements regarding five burglaries The interview lasted from 915 am to 1000 am

In addition to the DUI the suspect has been charged with five counts of burglary

The lawyer has filed a motion to suppress all statements made by the suspect to the detective in connection with the five burglaries

The state supreme court follows federal constitutional principles in all cases interpreting a criminal defendantrsquos rights

3

Criminal Law and Procedure Question

1 Did the detective violate the suspectrsquos Sixth Amendment right to counsel when he questioned the suspect in the absence of the lawyer Explain

2 Under Miranda did the suspect effectively invoke his right to counsel Explain

3 Was the suspectrsquos waiver of his Miranda rights valid Explain

4

CONTRACTS QUESTION

A music conservatory has two concert halls One concert hall had a pipe organ that was in poor repair and the other had no organ The conservatory decided to repair the existing organ and buy a new organ for the other concert hall After some negotiation the conservatory entered into two contracts with a business that both repairs and sells organs Under one contract the business agreed to repair the existing pipe organ for the conservatory for $100000 The business would usually charge a higher price for a project of this magnitude but the business agreed to this price because the conservatory agreed to prepay the entire amount Under the other contract the business agreed to sell a new organ to the conservatory for the other concert hall for $225000 As with the repair contract the business agreed to a low sales price because the conservatory agreed to prepay the entire amount Both contracts were signed on January 3 and the conservatory paid the business a total of $325000 that day

Two weeks later before the business had commenced repair of the existing organ the business suffered serious and unanticipated financial reversals The chief financial officer for the business contacted the conservatory and said

Bad news We had an unexpected liability and as a result are in a real cash crunch In fact even though we havenrsquot acquired the new organ from our supplier or started repair of your existing organ wersquove already spent the cash you gave us and we have no free cash on hand Wersquore really sorry but wersquore in a fix I think that we can find a way to perform both contracts but not at the original prices If you agree to pay $60000 more for the repair and $40000 more for the new organ we can probably find financing to finish everything If you donrsquot agree to pay us the extra money I doubt that we will ever be able to perform either contract and yoursquoll be out the money you already paid us

After receiving this unwelcome news the conservatory agreed to pay the extra amounts provided that the extra amount on each contract would be paid only upon completion of the businessrsquos obligations under that contract The business agreed to this arrangement and the parties quickly signed documents reflecting these changes to each contract The business then repaired the existing organ delivered the new organ and demanded payment of the additional $100000

The conservatory now has refused to pay the business the additional amounts for the repair and the new organ

1 Must the conservatory pay the additional $60000 for the organ repair Explain

2 Must the conservatory pay the additional $40000 for the new organ Explain

5

FAMILY LAW QUESTION

In 1994 a man and a woman were married in State A

In 1998 their daughter was born in State A

In 2010 the family moved to State B

In 2012 the husband and wife divorced in State B Under the terms of the divorce decree

(a) the husband and wife share legal and physical custody of their daughter (b) the husband must pay the wife $1000 per month in child support until their daughter reaches age 18 (c) the marital residence was awarded to the wife with the proviso that if it is sold before the daughter reaches age 18 the husband will receive 25 of the net sale proceeds remaining after satisfaction of the mortgage on the residence and (d) the remaining marital assets were divided between the husband and the wife equally

Six months ago the husband was offered a job in State A that pays significantly less than his job in State B but provides him with more responsibilities and much better promotion opportunities The husband accepted the job in State A and moved from State B back to State A

Since returning to State A the husband has not paid child support because due to his lower salary he has had insufficient funds to meet all his obligations

One month ago the wife sold the marital home netting $10000 after paying off the mortgage She then moved to a smaller residence The husband believes that he should receive more than 25 of the net sale proceeds given his financial difficulties

Last week when the wife brought the daughter to the husbandrsquos State A home for a weekend visit the husband served the wife with a summons in a State A action to modify the support and marital-residence-sale-proceeds provisions of the State B divorce decree The husband brought the action in the State A court that adjudicates all domestic relations issues

1 Does the State A court have jurisdiction to modify (a) the child support provision of the State B divorce decree Explain (b) the marital-residence-sale-proceeds provision of the State B divorce decree

Explain

2 On the merits could the husband obtain (a) retroactive modification of his child support obligation to the daughter Explain (b) prospective modification of his child support obligation to the daughter Explain (c) modification of the marital-residence-sale-proceeds provision of the State B

divorce decree Explain

6

FEDERAL CIVIL PROCEDURE QUESTION

The United States Forest Service (USFS) manages public lands in national forests including the Scenic National Forest Without conducting an environmental evaluation or preparing an environmental impact statement the USFS approved a development project in the Scenic National Forest that required the clearing of 5000 acres of old-growth forest The trees in the forest are hundreds of years old and the forest is home to a higher concentration of wildlife than can be found anywhere else in the western United States

The USFS solicited bids from logging companies to harvest the trees on the 5000 acres of forest targeted for clearing and it ultimately awarded the logging contract to the company that had submitted the highest bid for the trees However the USFS has not yet issued the company a logging permit Once it does so the company intends to begin cutting down trees immediately

A nonprofit organization whose mission is the preservation of natural resources has filed suit in federal district court against the USFS The nonprofit alleges that the USFS violated the National Environmental Policy Act (NEPA) by failing to prepare an environmental impact statement for the proposed logging project Among other remedies the nonprofit seeks a permanent injunction barring the USFS from issuing a logging permit to the logging company until an adequate environmental impact statement is completed The nonprofit believes that the logging project would destroy important wildlife habitat and thereby cause serious harm to wildlife in the Scenic National Forest including some endangered species

Assume that federal subject-matter jurisdiction is available that the nonprofit has standing to bring this action and that venue is proper

1 If the logging company seeks to join the litigation as a party must the federal district court allow it to do so as a matter of right Explain

2 What types of relief could the nonprofit seek to stop the USFS from issuing a logging permit during the pendency of the action what must the nonprofit demonstrate to obtain that relief and is the federal district court likely to grant that relief Explain

7

EVIDENCE QUESTION

A prison inmate has filed a civil rights lawsuit against a guard at the prison alleging that the guard violated the inmatersquos constitutional rights during an altercation The inmate and the guard are the only witnesses to this altercation They have provided contradictory reports about what occurred

The trial will be before a jury The inmate plans to testify at trial The guardrsquos counsel has moved for leave to impeach the inmate with the following

(a) Twelve years ago the inmate was convicted of felony distribution of marijuana He served a three-year prison sentence which began immediately after he was convicted He served his full sentence and was released from prison nine years ago (b) Eight years ago the inmate pleaded guilty to perjury a misdemeanor punishable by up to one year in jail He paid a $5000 fine (c) Seven years ago the inmate was convicted of felony sexual assault of a child and is currently serving a 10-year prison sentence for the crime The victim was the inmatersquos daughter who was 13 years old at the time of the assault

The inmatersquos counsel objects to the admission of any evidence related to these three convictions and to any cross-examination based on this evidence

The guard also plans to testify at trial The inmatersquos counsel has moved for leave to impeach the guard with the following

Last year the guard applied for a promotion to prison supervisor The guard submitted a reacutesumeacute to the state that indicated that he had been awarded a BA in Criminal Justice from a local college An official copy of the guardrsquos academic transcript from that college indicates that the guard dropped out after his first semester and did not receive a degree

The guardrsquos counsel objects to the admission of this evidence and to any cross-examination based on this evidence

The transcript and the reacutesumeacute have been properly authenticated The trial will be held in a jurisdiction that has adopted all of the Federal Rules of Evidence

1 What evidence if any proffered by the guard to impeach the inmate should be admitted Explain

2 What evidence if any proffered by the inmate to impeach the guard should be admitted Explain

8

CORPORATIONS QUESTION

Mega Inc is a publicly traded corporation incorporated in a state whose corporate statute is modeled on the Model Business Corporation Act (MBCA) Megarsquos articles of incorporation do not address the election of directors or amendment of the bylaws by shareholders

Well within the deadline for the submission of shareholder proposals for the upcoming annual shareholdersrsquo meeting an investor who was a large and long-standing shareholder of Mega submitted a proposed amendment to Megarsquos bylaws The proposal which the investor asked to be included in the corporationrsquos proxy materials and voted on at the upcoming shareholdersrsquo meeting read as follows

Section 20 The Corporation shall include in its proxy materials (including the proxy ballot) for a shareholdersrsquo meeting at which directors are to be elected the name of a person nominated for election to the Board of Directors by a shareholder or group of shareholders that beneficially have owned 3 or more of the Corporationrsquos outstanding common stock for at least one year

This Section shall supersede any inconsistent provision in these Bylaws and may not be amended or repealed by the Board of Directors without shareholder approval

Megarsquos management decided to exclude the investorrsquos proposal from the corporationrsquos proxy materials and explained its reasons in a letter to the investor

The investorrsquos proposed bylaw provision would be inconsistent with relevant state law because the Board of Directors has the authority to manage the business and affairs of the Corporation Generally shareholders lack the authority to interfere with corporate management by seeking to create a method for the nomination and election of directors inconsistent with the method chosen by the Board of Directors

Furthermore at its most recent meeting the Board of Directors unanimously approved an amendment to the Corporationrsquos bylaws that provides for proxy access for director nominations by a shareholder or a group of shareholders holding at least 10 of the Corporationrsquos voting shares for at least three years This procedure takes precedence over any nomination methods that might be sought or approved by shareholders

The investor is considering bringing a suit challenging managementrsquos refusal to include the investorrsquos proposed bylaw provision and challenging the boardrsquos amendment of the bylaws at its recent meeting

1 Is the investorrsquos proposed bylaw provision inconsistent with state law Explain

2 If the investorrsquos proposed bylaw provision were approved by the shareholders would the bylaw amendment previously approved by the board take precedence over the investorrsquos proposed bylaw provision Explain

3 Must the investor make a demand on Megarsquos board of directors before bringing suit Explain

9

July 2014 MEE

ANALYSES

Contracts Family Law

Criminal Law and Procedure

Federal Civil Procedure Evidence

Corporations

CRIMINAL LAW AND PROCEDURE ANALYSIS (Criminal Law and Procedure VA B D)

ANALYSIS

Legal Problems

(1) Did the detective violate the suspectrsquos Sixth Amendment right to counsel when he questioned the suspect about the burglaries without the lawyer present given that the lawyer represented the suspect in an unrelated criminal matter

(2) Under Miranda did the suspect effectively invoke his right to counsel when he said ldquoI think I want my lawyer here before I talk to yourdquo

(3) Was the suspectrsquos waiver of his right to remain silent under Miranda valid

DISCUSSION

Summary

The Sixth Amendment right to counsel as applied to states through the Fourteenth Amendment is offense-specific Although the suspect had an attorney representing him on his pending assault charge he had no Sixth Amendment right to the assistance of counsel with respect to the five uncharged burglaries because formal adversarial proceedings had not yet commenced on those charges The suspectrsquos Sixth Amendment right to counsel was not violated by the detectiversquos failure to inform him that the lawyer was present or of the lawyerrsquos demands

However a person undergoing custodial interrogation also has an independent constitutional right to counsel during custodial interrogation under Miranda When a suspect invokes his right to counsel under Miranda custodial interrogation must immediately cease for a period of at least 14 days However the invocation of the right to counsel must be unambiguous and clearly convey that the suspect has requested counsel Here because the suspectrsquos statement ldquoI think I want my lawyer here before I talk to yourdquo was ambiguous he did not invoke his Miranda right to counsel

A waiver of rights must be knowing intelligent and voluntary Here the suspect waived his right to remain silent under Miranda when he signed the waiver form The fact that the detective did not correct the suspectrsquos assumption that the lawyer would need to drive to the jailmdashby telling him that the lawyer was in the waiting room and was demanding to see himmdashdid not affect the validity of the suspectrsquos waiver

Point One (35) The suspectrsquos Sixth Amendment right to counsel was not violated because the right does not attach on new charges until formal adversarial judicial proceedings have commenced on those charges

The Sixth Amendment as applied to the states through the Fourteenth Amendment provides that ldquo[i]n all criminal prosecutions the accused shall enjoy the right to have the Assistance of Counsel for his defenserdquo The right to counsel does not attach with respect to particular charges until formal adversarial judicial proceedings have commenced (ie ldquoat or after the initiation of

13

Criminal Law and Procedure Analysis

adversary judicial criminal proceedingsmdashwhether by way of formal charge preliminary hearing indictment information or arraignment [or in some states arrest warrant]rdquo McNeil v Wisconsin 501 US 171 175 (1991) (internal quotations omitted)) Once a suspectrsquos Sixth Amendment right to counsel has attached any attempts to ldquodeliberately elicitrdquo statements from him in the absence of his attorney violate the Sixth Amendment See Massiah v United States 377 US 201 (1964) Brewer v Williams 430 US 387 (1977)

The Sixth Amendment right to counsel is charge- or offense -specific Representation by counsel in one prosecution does not in itself guarantee counsel for uncharged offenses See McNeil 501 US at 175 Texas v Cobb 532 US 162 (2001) Here the suspectrsquos Sixth Amendment right to counsel had attached only for the pending aggravated assault charge The suspectrsquos right to counsel for the aggravated assault case did not guarantee counsel for the five unrelated and uncharged burglaries that were the subject of the detectiversquos interrogation Thus because formal adversarial judicial proceedings against the suspect for the uncharged burglaries had not begun he had no Sixth Amendment right to counsel

Finally the detectiversquos failure to inform the suspect of the lawyerrsquos presence and demands to speak with him does not implicate the suspectrsquos Sixth Amendment right to counsel which had not yet attached See id Moran v Burbine 475 US 412 428ndash31 (1986)

Point Two (30) The suspect did not effectively invoke his right to counsel under Miranda because his statement was not unambiguous

A suspect subject to custodial interrogation has a right to consult with counsel and to have an attorney present during questioning Miranda v Arizona 384 US 436 (1966) When a suspect invokes his right to counsel during an interrogation law enforcement must immediately cease all questioning See Edwards v Arizona 451 US 477 484ndash85 (1981) Custodial interrogation cannot be reinitiated unless and until the suspect has been re-advised of his Miranda rights has provided a knowing and voluntary waiver and (1) counsel is present and (2) the suspect himself initiated further communication with the police see id at 484 or (3) (if the suspect was released from custody after the initial interrogation) at least 14 days have passed Maryland v Shatzer 559 US 98 110 (2010)

To invoke the right to counsel a suspectrsquos request must be ldquounambiguousrdquo This means that the suspect must articulate the desire for counsel sufficiently clearly that a reasonable officer would understand the statement to be a request for counsel Davis v United States 512 US 452 459 (1994) If the request is ambiguous the police are not required to stop the interrogation

In this case the suspectrsquos statement ldquoI think I want my lawyer here before I talk to yourdquo was not an unambiguous request for counsel The most reasonable interpretation of this statement is that the suspect might be invoking his right to counsel Id at 461 (ldquomaybe I should talk to a lawyerrdquo is not an unequivocal request for counsel) See also Burket v Angelone 208 F3d 172 197ndash98 (4th Cir 2000) (ldquoI think I need a lawyerrdquo is not an unambiguous request for an attorney) Soffar v Cockrell 300 F3d 588 594ndash95 (5th Cir 2002) (discussion of various statements that did not constitute unequivocal requests for counsel)

Under these circumstances the detective was not required to cease the custodial interrogation of the suspect Nor was the detective required to clarify or ask follow-up questions to determine whether the suspect in fact wanted an attorney Davis 512 US at 459ndash60

14

Criminal Law and Procedure Analysis

Point Three (35) The suspectrsquos waiver of his Miranda rights was knowing intelligent and voluntary despite the fact that he was never told of the lawyerrsquos presence in the jail or of the lawyerrsquos demands

A valid waiver of Miranda rights must be ldquovoluntaryrdquomdashie the product of a free or deliberate choice rather than intimidation coercion or deception Berghuis v Thompkins 560 US 370 382ndash83 (2010) In addition the waiver must be knowing and intelligent That is it ldquomust have been made with a full awareness of both the nature of the right being abandoned and the consequences of the decision to abandon itrdquo Moran v Burbine 475 US 421 (1986)

In this case the suspect signed a Miranda waiver form after receiving proper warnings There is no evidence ldquothat the police resorted to physical or psychological pressure to elicit the statementsrdquo Id The entire interview lasted only 45 minutes The only issue is whether the suspect knowingly and intelligently waived his Miranda rights despite the fact that the detective did not tell the suspect about the lawyerrsquos presence and her demands

The Supreme Court has said that ldquo[e]vents occurring outside of the presence of the suspect and entirely unknown to him surely can have no bearing on the capacity to comprehend and knowingly relinquish a constitutional rightrdquo Id at 422 If the suspect ldquoknew that he could stand mute and request a lawyer and was aware of the Statersquos intention to use his statements to secure a convictionrdquo then the waiver is valid regardless of the information withheld Id at 422ndash23

Here the suspect was correctly informed of his rights Miranda v Arizona 384 US at 467ndash73 His comments demonstrate that he understood that he could have a lawyer present if he desired (ie wondering whether he should call his attorney) and that he understood that there might be consequences to speaking with the detective (ldquoI probably should keep my mouth shut but Irsquom willing to talk to you for a whilerdquo) His comment ldquo[L]etrsquos not waste any time waiting for someone to call my attorney and having her drive hererdquo along with his signature on the Miranda waiver form show that his waiver was valid under the constitutional standard

The fact that the detective did not tell the suspect about the lawyerrsquos presence and demands has no bearing on the validity of the suspectrsquos waiver because ldquosuch conduct is only relevant to the constitutional validity of a waiver if it deprives a defendant of knowledge essential to his ability to understand the nature of his rights and the consequences of abandoning themrdquo Moran at 424 The Supreme Court has specifically declined to adopt a rule requiring that law enforcement tell a suspect of an attorneyrsquos efforts to contact him id at 425 (ldquoNor are we prepared to adopt a rule requiring that the police inform a suspect of an attorneyrsquos efforts to reach himrdquo)

[NOTE An examinee might also recognize that this general rule is further supported by the Supreme Courtrsquos decision in Florida v Powell 559 US 50 (2010) approving state Miranda warnings that do not explicitly warn suspects that they have a right to have counsel present during custodial interrogation]

15

CONTRACTS ANALYSIS (Contracts IB2 IIB IVA3 amp A5)

ANALYSIS

Legal Problems

(1) In the case of a service contract (governed by the common law of contracts) is a modification enforceable when a party agrees to pay more for the same performance than was originally promised

(2) In the case of a contract for the sale of goods (governed by Article 2 of the UCC) is a modification enforceable when a party agrees to pay more for the same goods than was originally promised

(3) May a party avoid an agreement on the basis of economic duress

DISCUSSION

Summary

There are two arguments that the conservatory can make to support the claim that it is not bound to pay the higher prices lack of consideration and economic duress

The organ repair contract is governed by the common law of contracts Under the common law the business would have difficulty recovering the additional $60000 for the organ repair because under the ldquopreexisting duty rulerdquo the agreement of the conservatory to pay the extra price was not supported by consideration However the business might argue that the modification is enforceable under an exception to the preexisting duty rule for fair and equitable modifications made in light of unanticipated circumstances

The organ sale contract is governed by Article 2 of the Uniform Commercial Code The business would likely recover the additional amount under that contract because Article 2 provides that consideration is not required for a modification to be binding

In both cases the conservatory could seek to avoid its agreement on the grounds of economic duress but that argument is not likely to succeed

Point One (45) The business probably cannot recover the additional $60000 for the organ repair because the conservatoryrsquos promise to pay more money was not supported by consideration

The general rule is that to be enforceable a promise must be supported by consideration Under RESTATEMENT (SECOND) OF CONTRACTS sect 71 a promise is supported by consideration if it is bargained for in exchange for a return promise or performance However under the ldquopreexisting duty rulerdquo (exemplified in RESTATEMENT (SECOND) OF CONTRACTS sect 73 and Alaska Packersrsquo Assrsquon v Domenico 117 F 99 (9th Cir 1902)) promise of performance of a legal duty already owed to a promisor which is neither doubtful nor the subject of honest dispute is not consideration

If the business had promised the conservatory anything new or different in exchange for the agreement to pay the additional $60000 (such as for example repairing the pipe organ more

16

Contracts Analysis

quickly or using better parts) that would constitute consideration especially in light of the principle that courts do not inquire into the adequacy of consideration Here however the business already had a legal duty under the original contract and did not agree to do anything else in exchange for the conservatoryrsquos promise to pay $60000 more

However an exception to the preexisting duty rule is sometimes applied in situations of unanticipated changed circumstances Under RESTATEMENT (SECOND) OF CONTRACTS sect 89 followed in many jurisdictions a promise modifying a duty under a contract not fully performed on either side is binding even if not supported by consideration if the modification is fair and equitable in view of circumstances not anticipated by the parties when the contract was made

If a court applies the rule in Restatement sect 89 the critical issues will be whether the modification was in fact ldquofair and equitablerdquo and whether it can be justified in light of unanticipated circumstances In many cases in which modifications have been upheld a party encountered difficulties or burdens in performing far beyond what was knowingly bargained for in the original contract with the result bordering on impracticability such as having to excavate solid rock instead of soft dirt or having to remove garbage far in excess of the amounts contemplated The conservatory would argue that the businessrsquos performance difficulties were not of this sort at allmdashnothing about repairing the pipe organ itself was any different from or more difficult than originally contemplated except that the business itself encountered financial distress unrelated to its burdens in performing its obligations under these contracts

Even if the business satisfies that element of the rule in Restatement sect 89 the business must also demonstrate that the circumstances that gave rise to the need to modify the contract were ldquounanticipatedrdquo at the time the original contract was made Here the facts suggest that when the business entered into the original contract it expected that the price paid by the conservatory would enable it to perform However any evidence that the business knew or had reason to know at the time of execution that it would need more money from the conservatory to be able to perform would mean that the request to modify was not ldquounanticipatedrdquo

[NOTE Some cases such as Schwartzreich v Bauman-Basch Inc 231 NY 196 131 NE 887 (1921) find that if the parties mutually agreed to rescind the original contract and then after rescission entered into an entirely new contract for a higher price the new contract is supported by consideration There is no evidence that such a rescission followed by a new contract took place here]

Point Two (45) The business can recover the additional $40000 for the new organ because no consideration is required under Article 2 of the UCC for good-faith contract modifications

The contract to buy a new organ is a contract for the sale of goods and therefore is governed by Article 2 of the Uniform Commercial Code UCC sect 2-102 Under Article 2 unlike the common law an agreement modifying a contract needs no consideration to be binding UCC sect 2-209(1) Section 2-209(1) thus obviates the preexisting duty rule entirely in contracts for the sale of goods

Even though consideration is not required modifications governed by sect 2-209 must satisfy the obligation of good faith imposed by the UCC UCC sect 1-304 See also Official Comment 2 to UCC sect 2-209 Good faith means ldquohonesty in fact and the observance of reasonable commercial standards of fair dealingrdquo UCC sect 1-201(b)(20) In this context the obligation of good faith means that ldquo[t]he effective use of bad faith to escape performance on the original contract terms is barred and the extortion of a lsquomodificationrsquo without legitimate commercial reason is ineffective as a violation of the duty of good faithrdquo Official Comment 2 to

17

Contracts Analysis

UCC sect 2-209 Here because the businessrsquos financial reversals were serious and apparently unanticipated at the time that the business entered into the contract with the conservatory and commitment of the extra money was needed to enable the business to perform a court would likely find that the business acted in good faith Thus a court would likely uphold the enforceability of the conservatoryrsquos promise to pay the additional $40000

Point Three (10) The conservatory is unlikely to be able to defend against enforcement of its promises to pay additional money under the theory of economic duress because the business probably did not make an improper threat

Under the common law of contracts parties may raise the defense of duress This common law defense also applies to contracts governed by UCC Article 2 See UCC sect 1-103(b)

A contract is voidable on the ground of economic duress by threat when it is established that a partyrsquos manifestation of assent is induced by an improper threat that leaves the party no reasonable alternative See RESTATEMENT (SECOND) OF CONTRACTS sect 175 See also eg Austin Instrument Inc v Loral Corp 272 NE2d 533 (NY 1971) (a threat to withhold essential goods can constitute duress) In order to void its agreement to pay the additional sum because of economic duress the conservatory must demonstrate that (1) the business made a threat to the conservatory (2) the threat was ldquoimproperrdquo or ldquowrongfulrdquo (3) the threat induced the conservatoryrsquos manifestation of assent to the modification and (4) the threat was sufficiently grave to justify the conservatoryrsquos assent

Here it appears that three of the four elements are likely satisfied The business plainly made a threat Moreover the threat induced the conservatoryrsquos assent to the modification and the threat was sufficiently grave to justify that assent If the conservatory had not agreed to pay the business the extra amounts the conservatory would have lost its entire $325000 investment In light of this potential loss a court could easily conclude that the conservatory had no reasonable alternative

However the business has a strong argument that its threat (indicating that it would breach the contracts unless the prices were increased) was not wrongful or improper but was instead nothing more than a communication of the reality of its own perilous situation to the conservatory

A mere threat to breach a contract is not in and of itself improper so as to support an action of economic duress or business compulsion Something more is required such as a breach of the duty of good faith and fair dealing as was present in Austin Instrument Inc supra Because the business could not perform the original contract without the requested modification the economic duress claim for the conservatory would likely fail for much the same reason that the business would be able to enforce the modification At the time the modification was requested the business was not trying to extort a price increase because of the conservatoryrsquos vulnerability but instead was simply stating the reality that the business could not perform without more money

18

FAMILY LAW ANALYSIS (Family Law IIIB D amp G)

ANALYSIS

Legal Problems

(1)(a) Does the State A court have jurisdiction to modify the State B child support order

(1)(b) Does the State A court have jurisdiction to modify the marital-residence-saleshyproceeds provision of the State B property-division decree

(2)(a) May a child support order be modified retroactively

(2)(b) May a child support order be modified prospectively based on a change of employment with a lower salary

(2)(c) May a property-division order be modified after entry of a divorce decree

DISCUSSION

Summary

The State A court may exercise personal jurisdiction over the wife because she was personally served in State A However subject-matter jurisdiction over the interstate modification of child support is governed by the Uniform Interstate Family Support Act (UIFSA) Under UIFSA State A does not have jurisdiction to modify the order for the daughterrsquos support because the wife is still a resident of State B UIFSA on the other hand does not govern property distributions and thus a State A court is not precluded from hearing the husbandrsquos petition to modify the marital-residence-sale-proceeds provision of the divorce decree

A child support order may not be modified retroactively A child support order may be modified prospectively based on a substantial change in circumstances Courts agree that a significant decrease in income is a substantial change in circumstances All states treat voluntary income reductions differently than involuntary reductions but employ different approaches for evaluating the impact of a voluntary reduction Whether the husband could obtain prospective modification of the child support order depends on which approach is applied

A property-division order is not subject to post-divorce modification based on a change in circumstances Thus the husband may in some states obtain prospective modification of the order for the daughterrsquos support but he may not obtain modification of the marital-residenceshysale-proceeds provision

Point One(a) (25) Personal jurisdiction over a nonresident respondent does not confer subject-matter jurisdiction over child support modification Under UIFSA a State A court may not modify a child support order issued by a State B court when as here the child or either parent continues to reside in State B the jurisdiction that issued the child support order

The State A court may exercise personal jurisdiction over the wife The wife was personally served in State A and a state may exercise jurisdiction based on in-state personal service See

19

Family Law Analysis

Burnham v Superior Court 495 US 604 (1990) But personal jurisdiction over the wife is not enough to give a State A court jurisdiction to modify the State B support order

The interstate enforcement and modification of child support is governed by the Uniform Interstate Family Support Act (UIFSA) which has been adopted by all states Under UIFSA the state that originally issued a child support order (here State B) has continuing exclusive jurisdiction to modify the order if that state remains the residence of the obligee the child or the obligor and all parties do not consent to the jurisdiction of another forum See UIFSA sect 205 See also UIFSA sect 603 (ldquoA tribunal of this State shall recognize and enforce but may not modify a registered order if the issuing tribunal had jurisdictionrdquo) The wife and daughter continue to reside in State B and the wife has not consented to the jurisdiction of another forum Thus a State A court does not have jurisdiction to modify the State B child support order

[NOTE Examinees who do not discuss personal jurisdiction but fully discuss UIFSA may receive full credit]

Point One(b) (15) UIFSA does not apply to disputes over property division Thus the State A court may exercise jurisdiction over the husbandrsquos petition to modify the marital-residence-sale-proceeds provision of the State B divorce decree because it has personal jurisdiction over the wife

The State A court in which the husband brought his action has jurisdiction to adjudicate domestic relations issues The husbandrsquos petition to modify the property settlement is a domestic relations issue The courts of State A may exercise personal jurisdiction over the wife because she was personally served in State A See Burnham v Superior Court 495 US 604 (1990) see Point One(a)

UIFSA does not apply to divorce property-division disputes Thus although a State A court may not adjudicate the husbandrsquos petition to modify his child support obligations it may adjudicate his property-division claims (Even though the court has jurisdiction it may not modify the property-division award on the merits See Point Two(c))

Point Two(a) (20) A child support order may not be modified retroactively

State courts have long held that obligations to pay child support ordinarily may not be modified retroactively ldquoIf the hardship is particularly severe the courts sometimes devised a way to protect the obligor but in most instances the courts hold that retroactive modification of this kind is beyond their power and indeed the governing statute may so providerdquo HOMER H CLARK THE LAW OF DOMESTIC RELATIONSHIPS IN THE UNITED STATES 725 (2d ed 1987)

Federal law now goes further and requires the states as a condition of federal child-support funding to adopt rules that absolutely forbid retroactive modification of the support obligation See 42 USC sect 666(a)(9)(C) The states have adopted rules consistent with the federal requirements

Point Two(b) (25) It is unclear whether the husband could obtain prospective downward modification of his child support based on his voluntary acceptance of a job with a lower salary

Prospective modification of a child support order is typically available only when the petitioner can show a substantial change in circumstances See ROBERT E OLIPHANT amp NANCY VER

20

Family Law Analysis

STEEGH FAMILY LAW 213ndash15 (3d ed 2010) A significant decrease in income is typically viewed as a substantial change

However when a parent seeks to modify a child support obligation because he has voluntarily reduced his income a court will not modify the obligation based solely on the income loss Some courts refuse to modify whenever the income shift was voluntary See eg Aguiar v Aguiar 127 P3d 234 (Idaho Ct App 2005) Others look primarily to the petitionerrsquos intentions and permit downward modification if he has acted in good faith See eg In re Marriage of Horn 650 NE2d 1103 (Ill App Ct 1995) Many courts use a multifactor approach See OLIPHANT amp VER STEEGH supra 217ndash18

Here there is no question that the husbandrsquos loss of income was voluntary In a jurisdiction in which voluntary income reduction bars support modification the husbandrsquos petition would be denied

In a jurisdiction employing a good-faith or multifactor approach it is possible but not certain that the husband could obtain downward modification The evidence supports the husbandrsquos good faith his change in employment appears to be based on his new jobrsquos greater responsibilities and better promotion possibilities In a jurisdiction using a multifactor approach the court would likely also consider the impact of such a shift on the daughter the likely duration of the husbandrsquos income loss and the likelihood of a promotion that would ultimately inure to the daughterrsquos benefit Thus on these facts it is possible but by no means certain that the husband could prospectively obtain downward modification of his child support obligation to his daughter

Point Two(c)(15) A divorce property-division award is not subject to modification

A support order is aimed at meeting the post-divorce needs of the supported individual Because the future is unpredictable courts are empowered to modify a support award to take account of changed circumstances that may occur during the period in which support is paid

By contrast a property-distribution award divides assets of the marriage based on the equities at the time of divorce Because the past can be ascertained a property-division award is not subject to post-divorce modification See HARRY A KRAUSE ET AL FAMILY LAW CASES COMMENTS AND QUESTIONS 691 (6th ed 2007)

Here the husband is seeking modification of a property-division award with respect to an asset owned by the parties at the time of divorce Thus the husband may not obtain a modification of the marital-residence-sale-proceeds provision of the divorce decree based on his reduced income

21

FEDERAL CIVIL PROCEDURE ANALYSIS (Federal Civil Procedure III IVC)

ANALYSIS

Legal Problems

(1) Is the logging company entitled to join this action as a matter of right

(2)(a) May the nonprofit organization obtain a temporary restraining order to stop the USFS from issuing a logging permit

(2)(b) May the nonprofit organization obtain a preliminary injunction to stop the USFS from issuing a logging permit during the pendency of the action

DISCUSSION

Summary

The logging company is entitled to intervene in this action as a matter of right because it has an interest in the property or transaction that is the subject of the action and is so situated that its interest may be impaired or impeded as a practical matter if the action goes forward without it The logging companyrsquos interest is not adequately represented by the USFSrsquos presence in the lawsuit

The nonprofit organization may seek a temporary restraining order (TRO) followed by a preliminary injunction to prevent the USFS from issuing a logging permit pending the outcome of the action The nonprofit is likely to obtain a TRO if it can demonstrate a risk of immediate and irreparable injury The nonprofit is also likely to obtain a preliminary injunction if it can demonstrate a significant threat of irreparable harm and a likelihood of success on the merits of its National Environmental Policy Act (NEPA) claim

Point One (50) Rule 24(a) of the Federal Rules of Civil Procedure requires federal courts to allow a person to intervene in an action as a matter of right if the person a) is interested in the property or transaction that is the subject of the action b) is so situated that its interest may be impaired or impeded if the litigation goes forward without it and c) is not adequately represented by existing parties Here the logging company likely meets all three requirements and should be allowed to intervene as a matter of right

Rule 24 of the Federal Rules of Civil Procedure governs intervention the process by which a non-party to an action may join the litigation Under Rule 24(a) (intervention of right) a person must be permitted to intervene if three conditions are met (1) the movant ldquoclaims an interest relating to the property or transaction that is the subject of the actionrdquo (2) the movant ldquois so situated that disposition of the action may as a practical matter impair or impede the movantrsquos ability to protect its interestrdquo and (3) ldquoexisting partiesrdquo do not ldquoadequately represent [the movantrsquos] interestrdquo FED R CIV P 24(a) The three requirements for intervention of right are often ldquovery interrelatedrdquo 7C CHARLES ALAN WRIGHT ET AL FEDERAL PRACTICE AND PROCEDURE sect 1908 at 297 (2007 amp 2011 Supp)

22

Federal Civil Procedure Analysis

Here the court should find that the logging company meets this test First the logging company has a strong interest in the property or transaction that is the subject of this action The USFS has accepted the logging companyrsquos bid and the logging company is merely awaiting issuance of a logging permit to begin logging The nonprofit organization is seeking to prevent this logging The logging company therefore has a strong direct and substantial interest in the subject matter of the lawsuit and in having its winning bid honored and a logging permit issued See eg Kleissler v US Forest Serv 157 F3d 964 972 (3d Cir 1998) (stating that ldquo[t]imber companies have direct and substantial interests in a lawsuit aimed at halting loggingrdquo) see also Natural Resources Defense Council v US Nuclear Regulatory Commrsquon 578 F2d 1341 1343ndash 44 (10th Cir 1978) (holding that applicants whose license renewals were pending had Rule 24(a)(2) interests where the lawsuit sought to halt the license-issuing process pending preparation of environmental impact statements) See generally 7C WRIGHT ET AL supra sect 19081 at 309 (ldquoIf there is a direct substantial legally protectable interest in the proceedings it is clear that this requirement of the rule is satisfiedrdquo) Second the logging companyrsquos interest in receiving a logging permit may well be impaired as a practical matter by the outcome of the lawsuit If the USFS loses the lawsuit it will have to prepare an environmental impact statement before issuing the logging companyrsquos permit This will at a minimum delay the logging companyrsquos ability to exercise its rights and may in the long r un mean that no logging permit is ever issued Intervention of right is not limited to those that would be legally bound as a matter of preclusion doctrine Id sect 19082 at 368 Rather ldquo[t]he rule is satisfied whenever disposition of the present action would put the movant at a practical disadvantage in protecting its interestrdquo Id sect 19082 at 369 Here that condition is easily satisfied See Kleissler 157 F3d at 972 (ldquoTimber companies have direct and substantial interests in a lawsuit aimed at halting logging rdquo)

Given that the logging company has an interest that may be impaired by disposition of the action it should be allowed to intervene unless the court is persuaded that the USFS adequately represents the logging companyrsquos interest See Rule 24(a)(2) 7C WRIGHT ET AL supra sect 1909 Here it could be argued that the USFS adequately represents the logging companyrsquos interest because the USFS presumably wants the court to uphold its development plan and allow it to proceed with issuance of the logging permit which is the same relief that the logging company would seek However whether representation is truly adequate depends upon ldquo[a] discriminating appraisal of the circumstancesrdquo 7C WRIGHT ET AL supra sect 1909 at 440 Although both the government and the logging company wish to avoid the preparation of an environmental impact statement their interests are distinct The USFSrsquos interest is proper management of the national forest system while the logging companyrsquos interest is making a profit from logging the 5000-acre tract The USFSrsquos handling of the litigation is likely to be affected by a variety of policy concerns and political considerations that have nothing to do with the logging companyrsquos purely economic interest in securing the right to cut trees in the Scenic National Forest See eg Kleissler 157 F3d at 973ndash74 (ldquo[T]he government represents numerous complex and conflicting interests in matters of this nature The straightforward business interests asserted by intervenors here may become lost in the thicket of sometimes inconsistent governmental policiesrdquo)

[NOTES (1) Examinees who mistakenly analyze the logging companyrsquos case for joinder under the related but incorrect Rule 19 ldquoRequired Joinder of Partiesrdquo may receive credit Rule 19 allows existing parties to demand joinder of non-parties (or seek dismissal of the case if they canrsquot get it) There is a close relationship between Rule 24 and Rule 19 and both contain a similar standard for determining when ldquointerestedrdquo third parties are ldquoentitledrdquo or ldquorequiredrdquo to be in the lawsuit Indeed the two prongs of the Rule 24 intervention test that are discussed above

23

Federal Civil Procedure Analysis

are nearly identical to the two prongs of the Rule 19(a) required joinder test Examinees who discuss and apply the test should receive credit even if they cite Rule 19 rather than Rule 24

(2) Examinees may discuss permissive joinder Although permissive joinder is a possibility here the question asks only whether the logging company can join the action as a matter of right and a permissive joinder analysis is not responsive to the question To the extent an examinee discusses permissive joinder the analysis will focus on whether the logging company ldquohas a claim or defense that shares with the main action a common question of law or factrdquo FED R CIV P 24(b)(1)(B) The district court also ldquomust consider whether the intervention will unduly delay or prejudice the adjudication of the original partiesrsquo rightsrdquo FED R CIV P 24(b)(3) On our facts the logging companyrsquos claim for the issuance of a logging permit would certainly share common questions of law and fact with the USFSrsquos defense against the nonprofitrsquos claim There are no facts suggesting that the logging companyrsquos presence would unduly delay or otherwise prejudice adjudication of the original action Thus the district court would have discretion to permit the logging company to intervene even if it denied intervention of right]

Point Two(a) (25) The nonprofit organization could seek and would likely obtain a temporary restraining order to stop the USFS from issuing a logging permit pending a hearing on an application for a preliminary injunction

The first type of interim relief the nonprofit could seek to stop the USFS from issuing a logging permit to the logging company is a temporary restraining order (TRO) prohibiting the USFS from issuing the logging permit A TRO can be issued without notice to the adverse party but only in limited circumstances and only for a limited time FED R CIV P 65(b) To secure a TRO without notice the nonprofit would need to submit an affidavit containing specific facts that demonstrate a risk of ldquoimmediate and irreparable injuryrdquo if a permit is issued FED R CIV P 65(b)(1) In deciding whether to grant a TRO courts will also consider the same factors that are relevant in deciding whether to grant a preliminary injunction (eg the moving partyrsquos likelihood of success on the merits the balance of hardships and the public interest) See Point Two(b) infra The TRO would last only long enough for the court to consider and resolve a request by the nonprofit for a preliminary injunction but no longer than 14 days (unless the court extends it for good cause or the adverse party consents to an extension) In addition bond is required

Here the court is likely to grant the nonprofitrsquos request The nonprofit could plausibly claim that cutting down 5000 acres of old-growth forest in an area that is home to the highest concentration of wildlife in the western United States would have ldquoan immediate and irreparablerdquo adverse impact on the environment and cause irreparable harm to the nonprofitrsquos interest in preserving and protecting natural resources including wildlife habitat

Point Two(b) (25) The nonprofit could also seek and would likely obtain a preliminary injunction to stop the USFS which is likely to be granted if the nonprofitrsquos claim that the USFS violated NEPA has a strong basis in fact and law

Because the TRO would be temporary the nonprofit would need to move for a preliminary injunction to prevent the USFS from issuing a logging permit throughout the pendency of the litigation Preliminary injunctions are injunctions that seek to ldquoprotect [the] plaintiff from

24

Federal Civil Procedure Analysis

irreparable injury and to preserve the courtrsquos power to render a meaningful decision after a trial on the meritsrdquo 11A CHARLES ALAN WRIGHT ET AL FEDERAL PRACTICE AND PROCEDURE sect 2947 at 112 (2013) Rule 65 of the Federal Rules of Civil Procedure sets out the procedural requirements for preliminary injunctions Preliminary injunctions may be granted only upon notice to the adverse party FED R CIV P 65(a)(1) and only if the movant ldquogives security in an amount that the court considers proper to pay the costs and damages sustained by any party found to have been wrongfully enjoined or restrainedrdquo FED R CIV P 65(c)

While Rule 65 sets out the procedural requirements for preliminary injunctive relief it does not specify the substantive grounds upon which it may be granted The courtrsquos discretion in ruling upon a motion for a preliminary injunction ldquois exercised in conformity with historic federal equity practicerdquo 11A WRIGHT ET AL supra sect 2947 at 114 The court typically considers four factors

(1) the significance of the threat of irreparable harm to the plaintiff if the injunction is not granted (2) the balance between this harm and the injury that granting the injunction would inflict on the defendant (3) the probability that the plaintiff will succeed on the merits and (4) the public interest

Id sect 2948 at 122ndash24 accord Habitat Educ Center v Bosworth 363 F Supp 2d 1070 1088 (ED Wis 2005) The most important of these factors is the risk of irreparable harm to the plaintiff 11A WRIGHT ET AL supra sect 29481 at 129 If the plaintiff has an adequate remedy at law (eg if money damages can compensate the plaintiff for its loss) then a preliminary injunction will be denied Id sect 29481

Here a court would likely conclude that the potential for environmental damage to the forest creates a significant threat of irreparable harm ldquo[E]nvironmental injury is often irreparable Courts have recognized that logging such as would occur [here] can have longshyterm environmental consequences and thus satisfy the irreparable injury criterionrdquo Habitat Educ Center 363 F Supp 2d at 1089 (citing Idaho Sporting Congress Inc v Alexander 222 F3d 562 569 (9th Cir 2000) (noting that the imminent and continuing logging activities presented ldquoevidence of environmental harm sufficient to tip the balance in favor of injunctive reliefrdquo)) Neighbors of Cuddy Mountain v US Forest Service 137 F3d 1372 1382 (9th Cir 1998) (stating that ldquo[t]he old growth forests plaintiffs seek to protect would if cut take hundreds of years to reproducerdquo) (internal citation omitted)) see also 11C WRIGHT ET AL supra sect 29481 at 151 (noting that ldquoa preliminary injunction has been issued to prevent harm to the environmentrdquo)

The second factor the balance between the harm to the plaintiff and the harm the defendant will suffer if the injunction is issued also appears to support issuance of a preliminary injunction here The USFS will have to wait before it can develop the Scenic National Forest and the logging company may lose money if the delay is prolonged These economic harms could be compensated monetarily if an injunction is issued inappropriately Where ldquoan injunction bond can compensate [the] defendant for any harm the injunction is likely to inflict the balance should be struck in favor of [the] plaintiffrdquo Id sect 29482 at 192 See also Habitat Educ Center 363 F Supp 2d at 1089 (stating that ldquothe relative absence of harmful effects on the Forest Service weighs in favor of granting the injunctionrdquo)

The third factor is the likelihood that the plaintiff will prevail on the merits Although there is limited information concerning the merits of the action the nonprofit alleges that the federal statute (NEPA) requires an environmental impact statement and further states that the USFS created no environmental impact analysis or statement at all Assuming that those

25

Federal Civil Procedure Analysis

allegations are correct it seems plausible to conclude that the nonprofit will be able to show a likelihood of success on the merits

Finally courts deciding whether or not to issue preliminary injunctive relief are to consider the public interest ldquoFocusing on this factor is another way of inquiring whether there are policy considerations that bear on whether the order should issuerdquo 11C WRIGHT ET AL supra sect 29484 at 214 If the court concludes that the nonprofit is likely to succeed on its NEPA claim because the USFS wrongfully failed to conduct an environmental impact assessment it is likely to find that the public interest would be served by restraining the USFS from proceeding with logging in a national forest See Heartwood Inc v US Forest Service 73 F Supp 2d 962 979 (SD Ill 1999) affrsquod on other grounds 230 F3d 947 (7th Cir 2000) (ldquoviolations by federal agencies of NEPArsquos provisions as established by Congress harm the public as well as the environmentrdquo)

Thus a court is very likely to grant a preliminary injunction if it concludes that the nonprofit has a significant likelihood of success on the merits

26

EVIDENCE ANALYSIS (Evidence ID IIA amp C)

ANALYSIS

Legal Problems

(1) Under what circumstances can evidence of prior convictions be used to impeach a witnessrsquos credibility in a civil case

(1)(a) May the inmatersquos credibility be impeached by evidence of a 12-year-old felony drug conviction if he was released from prison 9 years ago

(1)(b) May the inmatersquos credibility be impeached by evidence of an 8-year-old misdemeanor perjury conviction that was punishable by 1 year in jail if he pleaded guilty and was sentenced only to pay a $5000 fine

(1)(c) May the inmatersquos credibility be impeached by evidence of a 7-year-old sexual assault conviction if the inmate is still serving a 10-year prison sentence and the victim was his 13-year-old daughter

(2)(a) May the guardrsquos credibility be impeached by cross-examination regarding specific instances of misconduct (ie lying on his reacutesumeacute) relevant to credibility

(2)(b) May the guardrsquos credibility be impeached by admission of extrinsic evidence (his reacutesumeacute and academic transcript) offered to prove specific instances of misconduct relevant to credibility

DISCUSSION

Summary

Under the Federal Rules of Evidence witnesses can be impeached with evidence of prior convictions andor specific instances of misconduct Whether evidence of prior convictions should be admitted to impeach generally depends on the nature of the crime the amount of time that has passed and (only in criminal cases) whether the ldquowitnessrdquo is the defendant FED R EVID 609(a)

In this civil case evidence of the inmatersquos conviction for distribution of marijuana should be admitted to impeach the inmate because he was convicted of a felony and was released from prison fewer than 10 years ago FED R EVID 609(a)(1) Credibility is critically important in this case because the jury will hear conflicting testimony from the two disputing parties and there were no other eyewitnesses to the altercation Under Rule 609(a)(1) the inmatersquos conviction should be admitted because it has some bearing on his credibility and its probative value is not substantially outweighed by concerns of unfair prejudice confusion or delay Id

Evidence of the inmatersquos misdemeanor conviction for perjury must be admitted because the crime ldquorequired provingmdashor the witnessrsquos admittingmdasha dishonest act or false statementrdquo by the inmate FED R EVID 609(a)(2)

27

Evidence Analysis

Evidence of the inmatersquos felony conviction for sexual assault should be excluded because its probative value is substantially outweighed by the danger of unfair prejudice to the inmate based on the heinous nature of the crime FED R EVID 609(a)(1) In the alternative the judge could limit the evidence relating to this conviction by excluding details of the inmatersquos crime

In all civil (and criminal) cases witnesses can also be impeached with evidence of specific instances of prior misconduct that did not result in a conviction FED R EVID 608(b) Pursuant to Rule 608(b) misconduct probative of untruthfulness can be inquired into on cross-examination but cannot be proved through extrinsic evidence Id Thus the inmatersquos counsel should be permitted to cross-examine the guard regarding the false statement in the guardrsquos reacutesumeacute However extrinsic evidence of the guardrsquos misconduct (ie the guardrsquos authenticated reacutesumeacute and transcript from the local college) should not be admitted even if the guard denies wrongdoing or refuses to answer cross-examination questions about these matters Id

Point One (10) The Federal Rules of Evidence permit impeachment of witnesses with evidence of prior convictions

Whether convictions should be admitted to impeach generally depends on the nature of the crime the amount of time that has passed and (only in criminal cases) whether the ldquowitnessrdquo is the defendant FED R EVID 609(a) Under Rule 609(a) evidence of prior convictions may be admitted for the purpose of ldquoattacking a witnessrsquos character for truthfulnessrdquo Id

There are two basic types of convictions that can be admitted for the purpose of impeachment

(1) convictions for crimes ldquopunishable by death or by imprisonment for more than one yearrdquo (which generally correlates to ldquofeloniesrdquo) FED R EVID 609(a)(1) and (2) convictions ldquofor any crimes regardless of the punishment if the court can readily determine that establishing the elements of the crime required provingmdashor the witnessrsquos admittingmdasha dishonest act or false statementrdquo FED R EVID 609(a)(2)

Pursuant to Rule 609(a)(1) in civil cases the admission of evidence of a felony conviction is ldquosubject to Rule 403 [which says that a court may exclude relevant evidence if its probative value is substantially outweighed by other factors]rdquo FED R EVID 609(a)(1) However Rule 403 does not protect the witness against admission of prior convictions involving dishonestymdashwhich must be admitted by the court FED R EVID 609(a)(2)

Finally Federal Rule of Evidence 609(b) contains the presumption that a conviction that is more than 10 years old or where more than 10 years has passed since the witnessrsquos release from confinement (whichever is later) should not be admitted unless ldquoits probative value supported by specific facts and circumstances substantially outweighs its prejudicial effectrdquo and the proponent has provided the adverse party with reasonable written notice FED R EVID 609(b)

Point One(a) (25) The court should admit evidence of the inmatersquos 12-year-old felony marijuana distribution conviction

The inmatersquos conviction for marijuana distribution was for a felony punishable by imprisonment for more than one year See FED R EVID 609(a)(1) Moreover although the conviction was 12 years ago the 10-year time limit of Rule 609(b) is not exceeded because that time limit runs

28

Evidence Analysis

from the date of either ldquothe witnessrsquos conviction or release from confinement for it whichever is laterrdquo FED R EVID 609(b) Because the inmate served three years in prison he was released from confinement nine years ago

However pursuant to Rule 609(a)(1) the admission of felony convictions to impeach a witness in a civil case is ldquosubject to Rule 403rdquo FED R EVID 609(a)(1) Neither Rule 609(a) nor the advisory committee notes specify which factors courts should consider when balancing the probative value of a conviction against the dangers identified in Rule 403 (which include (1) unfair prejudice (2) confusion of the issues (3) misleading the jury (4) waste of time or undue delay and (5) needless presentation of cumulative evidence) FED R EVID 403

In this case credibility is very important because the evidence consists primarily of the testimony of the disputing parties and there were no other eyewitnesses to the altercation This enhances the probative value of any evidence bearing on the inmatersquos credibility A court is likely to conclude that the inmatersquos prior felony drug conviction is relevant to his credibility See eg United States v Brito 427 F3d 53 64 (1st Cir 2005) (ldquoPrior drug-trafficking crimes are generally viewed as having some bearing on veracityrdquo) Although the probative value of any conviction diminishes with age see eg United States v Brewer 451 F Supp 50 53 (ED Tenn 1978) the inmatersquos ongoing problems with the law suggest that he has continued (and even escalated) his criminal behavior over the past nine years The court should admit this evidence because its probative value is not substantially outweighed by any Rule 403 concerns Specifically any prejudice to the inmate would be slight because the conviction is unrelated to the altercation at issue and the conviction was not for a heinous crime that might inflame the jury

[NOTE Whether an examinee identifies the jury instruction as containing a ldquoconclusiverdquo or ldquomandatoryrdquo presumption is less important than the examineersquos analysis of the constitutional infirmities]

Point One(b) (15) The court must admit evidence of the inmatersquos eight-year-old misdemeanor conviction because perjury is a crime of dishonesty

Rule 609(a)(2) provides that evidence of a criminal conviction ldquomust be admitted if the court can readily determine that establishing the elements of the crime required provingmdashor the witnessrsquos admittingmdasha dishonest act or false statementrdquo FED R EVID 609(a)(2) The inmatersquos conviction for perjury would have necessarily required proving that the inmate engaged in an act of dishonesty This conviction occurred within the past 10 years so it ldquomust be admittedrdquo because in contrast to Rule 609(a)(1) (discussed in Point One(a)) admission under Rule 609(a)(2) is mandatory and not subject to Rule 403

Point One(c) (20) The court should exclude evidence of the inmatersquos seven-year-old felony sexual assault conviction because the probative value of this evidence is substantially outweighed by the danger of unfair prejudice In the alternative the details of the prior conviction could be excluded

The inmatersquos conviction for felony sexual assault was seven years ago and he has not yet been released from incarceration so Rule 609(a) but not 609(b) is applicable here FED R EVID 609(a) This conviction is therefore admissible to impeach the inmate unless its probative value is substantially outweighed by the danger of unfair prejudice or any other Rule 403 concern Id

29

Evidence Analysis

Sex crimes are generally not considered relevant to credibility see Hopkins v State 639 So 2d 1247 1254 (Miss 1993) so the probative value of this conviction is relatively low Moreover the heinous nature of the inmatersquos crime (sexual assault on his daughter) makes the danger of unfair prejudice to the inmate very high Thus the court should exclude evidence of the conviction because it was for a heinous offense that is likely to inflame the jury and it has little bearing on credibility See eg United States v Beahm 664 F2d 414 419 (4th Cir 1981)

As an alternative to excluding this evidence the judge could minimize the unfair prejudice to the inmate by permitting limited cross-examination but refusing to allow specific questions about the nature of the inmatersquos conviction For example a court could limit cross-examination to the fact that the inmate was convicted of a ldquofelonyrdquo or perhaps that he was convicted of a ldquosexual assaultrdquo without identifying the victim However because evidence of the inmatersquos prior convictions can be admitted solely for the purpose of enabling the jury to assess his credibility and because his two earlier convictions should have already been admitted the court should exclude all evidence of the felony sexual assault conviction

Point Two(a) (15) The court should permit the inmatersquos counsel to cross-examine the guard regarding the false statement in his reacutesumeacute because the guardrsquos misconduct bears on his truthfulness

The inmate wishes to cross-examine the guard about his prior dishonest behaviormdashlying on his reacutesumeacutemdashthat did not involve a criminal conviction Rule 608(b) allows witnesses to be cross-examined about specific instances of prior non-conviction misconduct probative of untruthfulness ldquoin order to attack the witnessrsquos character for truthfulnessrdquo FED R EVID 608(b)

The courtrsquos decision to allow cross-examination about the guardrsquos prior dishonest behavior depends on the probative value of such evidence balanced against the danger of unfair prejudice to the guard or any other Rule 403 concern FED R EVID 403 Here the guardrsquos false statement on his reacutesumeacute that he obtained a degree in Criminal Justice is highly probative of his untruthfulness because it grossly misrepresents his actual academic record was made recently and was made with the intent to deceive Because the probative value of this evidence is very strong and is not substantially outweighed by any Rule 403 concerns cross-examination of the guard on this topic should be permitted The court may also consider it fair to permit this cross-examination of the guard on these matters assuming that one or more of the inmatersquos prior convictions have been admitted to impeach his credibility

Point Two(b) (15) The court should exclude extrinsic evidence of the guardrsquos non-conviction misconduct even if the guard denies wrongdoing or refuses to answer questions about the matter

Although Rule 608(b) allows cross-examination about specific instances of prior misconduct probative of untruthfulness ldquoextrinsic evidencerdquo offered to prove such misconduct is not admissible FED R EVID 608(b) The rationale for this rule is that allowing the introduction of extrinsic evidence of prior misconduct by witnesses when these acts are relevant only to the witnessesrsquo truthfulness and not to the main issues in the case would create too great a risk of confusing the jury and unduly delaying the trial The court does not have discretion to admit this extrinsic evidence See eg United States v Elliot 89 F3d 1360 1368 (8th Cir 1996)

30

Evidence Analysis

Here the inmatersquos counsel may cross-examine the guard about the false statement on his reacutesumeacute However the inmatersquos counsel must accept the guardrsquos response Even if the guard denies wrongdoing or refuses to answer questions about the matter the inmatersquos counsel cannot introduce the guardrsquos reacutesumeacute or the transcript from the local college to prove the guardrsquos misconduct

31

CORPORATIONS ANALYSIS (Corporations VA2 IX)

ANALYSIS

Legal Problems

(1) Do shareholders have the authority to amend a corporationrsquos bylaws with respect to director nominations

(2) Do board-approved bylaws on a particular subject here nomination of directors preempt subsequent conflicting bylaw amendments by shareholders

(3) Is a suit challenging both managementrsquos refusal to include the proposed bylaw amendment in Megarsquos proxy statement and the boardrsquos amendment of the bylaws dealing with nomination of directors a direct or derivative suit

DISCUSSION

Summary

The voting and litigation rights of the shareholders of Mega are subject to the provisions of the Model Business Corporations Act (MBCA)

The investorrsquos proposed bylaw provision is not inconsistent with state law Under the MBCA shareholders may amend the bylaws when the amendment deals with a proper matter for the corporationrsquos bylaws such as procedures for nominating directors

The Mega boardrsquos bylaw amendment does not preempt the investorrsquos proposed bylaw provision or the Mega shareholdersrsquo power to approve it While shareholders can limit the boardrsquos power to amend or repeal the bylaws the board cannot limit the shareholdersrsquo power

Whether the investor must make a demand on Megarsquos board depends on how the investor frames its claim If the investor claims a violation of shareholder voting rights the claim is direct and pre-suit demand on the board is not required If on the other hand the investor claims that the directors violated their fiduciary duties by amending the bylaws to entrench themselves the claim is derivative and a pre-suit demand is required

Point One (30) Shareholders may amend the corporationrsquos bylaws where the proposed bylaw provision relates to procedural matters typically included in the bylaws such as the nomination of directors

Internal affairs of the corporation such as the conduct of shareholder meetings and election of directors are subject to the corporate law of the state of incorporation See McDermott Inc v Lewis 531 A2d 206 (Del 1987) (applying law of jurisdiction where corporation was incorporated in case involving voting rights) This statersquos corporate statute is modeled on the MBCA

Under the MBCA ldquoshareholders may amend the corporationrsquos bylawsrdquo MBCA sect 1020(a) Thus the only question is whether the bylaws can specify the procedures for shareholder nomination of directors

32

Corporations Analysis

The MBCA states that the bylaws ldquomay contain any provision that is not inconsistent with law or the articles of incorporationrdquo MBCA sect 206(b) In addition the MBCA was revised in 2009 to address shareholder nomination of directors in public corporations (known as ldquoproxy accessrdquo) and specifies that the bylaws ldquomay contain a requirement that the corporation include in its [proxy materials] one or more individuals nominated by a shareholderrdquo MBCA sect 206(c)(1) see Committee on Corporate Laws ABA Section of Business Law Report on the Roles of Boards of Directors and Shareholders of Publicly Owned Corporations and Changes to the Model Business Corporations ActmdashAdoption of Shareholder Proxy Access Amendments to Chapters 2 and 10 65 BUS LAWYER 1105 (2010)

The inclusion of director-nomination procedures in the bylaws is consistent with practice and is recognized by the Delaware courts whose views on corporate law carry significant weight Typically the procedures for nomination of directors are found in the bylaws See 1 COX amp HAZEN TREATISE ON THE LAW OF CORPORATIONS sect 312 (3d ed 2011) see also 4 FLETCHER CORP FORMS ANN PART III ch 21 (2013) (including sample bylaws that permit nomination of directors by shareholders) The Delaware Supreme Court has confirmed that the bylaws may ldquodefine the process and proceduresrdquo for director elections See CA Inc v AFSCME Employees Pension Plan 953 A2d 227 (Del 2008) (concluding that bylaw amendment requiring reimbursement of election expenses to certain successful shareholder nominators is ldquoproper subjectrdquo under Delaware law)

[NOTE The question of the proper scope of the bylaws can be answered using the more general MBCA sect 206(b) or the 2009 MBCA revision adding sect 206(c)(1) (adopted in CT ME VA) In addition some examinees might raise the point that shareholder proposals may not compel the board to take action such as by including shareholder nominations in the companyrsquos proxy materials on the theory that the ldquobusiness and affairsrdquo of the corporation are to be managed by the board See MBCA sect 801(b) Although shareholders are generally limited to adopting precatory resolutions that recommend or encourage board action this limitation does not apply when shareholders have specific authority to take binding action on their ownmdashsuch as to amend the bylaws]

Point Two (30) Shareholders can amend (or repeal) board-approved bylaws Further shareholders can limit the boardrsquos power to later amend and repeal a shareholder-approved bylaw

Under the MBCA shareholders have the power to amend the bylaws See Point One The board shares this power with the shareholders unless (1) the corporationrsquos articles ldquoreserve that power exclusively to the shareholdersrdquo or (2) ldquothe shareholders in amending repealing or adopting a bylaw expressly provide that the board of directors may not amend repeal or reinstate that bylawrdquo See MBCA sect 1020(b)

Shareholder-approved bylaw provisions can amend or repeal existing bylaw provisions whether originally approved by the board or by shareholders See ALAN R PALMITER CORPORATIONS EXAMPLES AND EXPLANATIONS sect 713 (7th ed 2012) Thus the Mega boardrsquos bylaw amendmentmdashwhich set more demanding thresholds for shareholder nomination of directors than the investorrsquos proposed bylaw provisionmdashwould be superseded (repealed) if Megarsquos shareholders were to approve the investorrsquos proposal

Further a shareholder-approved bylaw generally can limit the power of the board to later amend or repeal it See MBCA sect 1020(b)(2) Thus if Megarsquos shareholders approved the bylaw

33

Corporations Analysis

provision proposed by the investor Megarsquos board could not repeal the provision because it includes a ldquono board repealrdquo clause

The revision to the MBCA in 2009 dealing with shareholder proxy access does not change this conclusion That revision specifies that a shareholder-approved bylaw dealing with director nominations may not limit the boardrsquos power to amend add or repeal ldquoany procedure or condition to such a bylaw in order to provide for a reasonable practicable and orderly processrdquo MBCA sect 206(d) Thus according to the revision if shareholders approve a bylaw amendment that limits further board changes the board would nonetheless retain the power to ldquotinkerrdquo with the bylaw to safeguard the voting process but could not repeal the shareholder-approved bylaw The Official Comment to MBCA sect 206(d) makes clear that the revision is ldquonot intended to allow the board of directors to frustrate the purpose of the shareholder-adopted proxy access provisionrdquo Thus if Megarsquos shareholders were to approve the bylaw provision proposed by the investor Megarsquos board could only amend the provision regarding its procedures or conditions in a manner consistent with its purpose of permitting proxy access for Megarsquos shareholders

[NOTE The boardrsquos attempted interference with a shareholder voting initiative may also have been a violation of the directorsrsquo fiduciary duties See Blasius Indus Inc v Atlas Corp 564 A2d 651 (Del Ch 1988) (finding that directors breached their fiduciary duties by amending bylaws and expanding size of board to thwart insurgentrsquos plan to amend bylaws and seat a majority of new directors) The call however asks examinees to consider whether shareholders or the board have ldquoprecedencerdquo over amending the corporate bylaws Thus an examineersquos answer should be framed in terms of ldquopowerrdquo and not ldquodutyrdquo]

Point Three (40) The investor need not make a demand on the board if the investor states a direct claim such as an allegation that the board interfered with the investorrsquos right to amend the bylaws But the investor must make a demand on the board if the investor states a derivative claim (on behalf of the corporation) such as an allegation that the directors sought to entrench themselves by interfering with the proposed proxy access

The MBCA generally requires that shareholders make a demand on the board of directors before initiation of a derivative suit MBCA sect 742 (shareholder may not bring derivative proceeding until written demand has been made on corporation and 90 days have expired) A derivative suit is essentially two suits in one where the plaintiff-shareholder seeks to bring on behalf of the corporation a claim that vindicates corporate rights usually based on violation of fiduciary duties PALMITER supra sect 1811 (6th ed 2009) The demand permits the board to investigate the situation identified by the shareholder and take suitable action No demand on the board is required however if the shareholder brings a direct suit to vindicate the shareholderrsquos own rights not those of the corporation

Is the suit brought by the investor derivative or direct The MBCA defines a ldquoderivative proceedingrdquo as one brought ldquoin the right of a domestic corporationrdquo MBCA sect 740(1) Thus the answer to how the investorrsquos suit should be characterized turns on what rights the investor seeks to vindicate If the investor frames its claim as one of fiduciary breach by directorsmdashfor example for failing to become adequately informed about voting procedures or for seeking to entrench themselves in office by manipulating the voting structure to avoid a shareholder insurgencymdashthen the suit is ldquoderivativerdquo and the investor must make a demand on the board See MBCA Ch 7 Subch D Introductory Comment (ldquothe derivative suit has historically been the principal method of challenging allegedly illegal action by managementrdquo)

34

Corporations Analysis

If however the investor frames its claim as one to vindicate shareholder rights the suit is direct and no demand is required For many courts the direct-derivative question turns on who is injured and who is to receive the relief sought by the plaintiff-shareholders See Tooley v Donaldson Lufkin amp Jenrette Inc 845 A2d 1031 (Del 2004) (characterizing a merger-delay claim as direct because delay of merger only harmed shareholders not corporation) Thus if the investor claims that managementrsquos refusal to include its proposed bylaw amendment in the corporationrsquos proxy materials violates its shareholder rights to initiate corporate governance reforms the suit will be direct Courts have not questioned the ability of shareholders to bring direct suits challenging board action to exclude their proposed bylaw amendments from the corporationrsquos proxy materials See JANA Master Fund Ltd v CNET Networks Inc 954 A2d 335 (Del Ch 2008) (upholding shareholderrsquos direct challenge to boardrsquos interpretation of advance-notice bylaw) Chesapeake Corp v Shore 771 A2d 293 (Del Ch 2000) (upholding shareholderrsquos direct challenge to actions by board that effectively prevented it from proposing bylaw amendments in contest for control)

Is the way that the investor frames its claim conclusive Courts have permitted shareholder-plaintiffs to challenge a transaction in a direct suit even though the same transaction could also be challenged as a fiduciary breach See Eisenberg v Flying Tiger Line Inc 451 F2d 267 (2d Cir 1971) (permitting direct suit challenging a corporate reorganization as a dilution of shareholder voting power even though reorganization may have involved conflicts of interest and thus constituted a fiduciary breach) Thus the investorrsquos choice to pursue a claim challenging the legality of managementrsquos decision to exclude the investorrsquos proposal from the corporationrsquos proxy materialsmdashrather than a possible breach of fiduciary dutymdashis likely to be respected See 3 COX amp HAZEN supra sect 153 (describing situations in which a claim can be framed as derivative or direct)

[NOTE Some issues under Delaware corporate law regarding pre-suit demand are not relevant here For example whether the Mega directors are independent and disinterested is not relevant to the MBCA requirement of a pre-suit demand As the Official Comment to MBCA sect 742 points out the MBCArsquos requirement of ldquouniversal demandrdquo gives the board ldquothe opportunity to reexamine the act complained of in the light of a potential lawsuit and take corrective actionrdquo even when the directors might be non-independent or have conflicts of interest

Nor is it relevant to the MBCA pre-suit demand requirement that the statutory 90-day waiting period may be onerous The first paragraph of MBCA sect 742 requires a pre-suit demand without exception the second paragraph of the section imposes a 90-day waiting period before a derivative suit may be brought which can be shortened if the board rejects the demand or ldquoirreparable injury to the corporation would result by waiting for the expiration of the 90-day periodrdquo The call as written asks only whether a pre-suit demand should be made and does not ask examinees to address whether the post-demand waiting period should be shortened under the ldquoirreparable injuryrdquo standard]

35

National Conference of Bar Examiners 302 South Bedford Street | Madison WI 53703-3622 Phone 608-280-8550 | Fax 608-280-8552 | TDD 608-661-1275

wwwncbexorg e-mail contactncbexorg

  • Preface
  • Description of the MEE
  • Instructions
  • July 2014 Questions
    • CRIMINAL LAW AND PROCEDURE QUESTION
    • CONTRACTS QUESTION
    • FAMILY LAW QUESTION
    • FEDERAL CIVIL PROCEDURE QUESTION
    • EVIDENCE QUESTION
    • CORPORATIONS QUESTION
      • July 2014 Analyses
        • CRIMINAL LAW AND PROCEDURE ANALYSIS
        • CONTRACTS ANALYSIS
        • FAMILY LAW ANALYSIS
        • FEDERAL CIVIL PROCEDURE ANALYSIS
        • EVIDENCE ANALYSIS
        • CORPORATIONS ANALYSIS
            • ltlt13 ASCII85EncodePages false13 AllowTransparency false13 AutoPositionEPSFiles true13 AutoRotatePages None13 Binding Left13 CalGrayProfile (Dot Gain 20)13 CalRGBProfile (sRGB IEC61966-21)13 CalCMYKProfile (US Web Coated 050SWOP051 v2)13 sRGBProfile (sRGB IEC61966-21)13 CannotEmbedFontPolicy Error13 CompatibilityLevel 1413 CompressObjects Tags13 CompressPages true13 ConvertImagesToIndexed true13 PassThroughJPEGImages true13 CreateJobTicket false13 DefaultRenderingIntent Default13 DetectBlends true13 DetectCurves 0000013 ColorConversionStrategy CMYK13 DoThumbnails false13 EmbedAllFonts true13 EmbedOpenType false13 ParseICCProfilesInComments true13 EmbedJobOptions true13 DSCReportingLevel 013 EmitDSCWarnings false13 EndPage -113 ImageMemory 104857613 LockDistillerParams false13 MaxSubsetPct 10013 Optimize true13 OPM 113 ParseDSCComments true13 ParseDSCCommentsForDocInfo true13 PreserveCopyPage true13 PreserveDICMYKValues true13 PreserveEPSInfo true13 PreserveFlatness true13 PreserveHalftoneInfo false13 PreserveOPIComments true13 PreserveOverprintSettings true13 StartPage 113 SubsetFonts true13 TransferFunctionInfo Apply13 UCRandBGInfo Preserve13 UsePrologue false13 ColorSettingsFile ()13 AlwaysEmbed [ true13 ]13 NeverEmbed [ true13 ]13 AntiAliasColorImages false13 CropColorImages true13 ColorImageMinResolution 30013 ColorImageMinResolutionPolicy OK13 DownsampleColorImages true13 ColorImageDownsampleType Bicubic13 ColorImageResolution 30013 ColorImageDepth -113 ColorImageMinDownsampleDepth 113 ColorImageDownsampleThreshold 15000013 EncodeColorImages true13 ColorImageFilter DCTEncode13 AutoFilterColorImages true13 ColorImageAutoFilterStrategy JPEG13 ColorACSImageDict ltlt13 QFactor 01513 HSamples [1 1 1 1] VSamples [1 1 1 1]13 gtgt13 ColorImageDict ltlt13 QFactor 01513 HSamples [1 1 1 1] VSamples [1 1 1 1]13 gtgt13 JPEG2000ColorACSImageDict ltlt13 TileWidth 25613 TileHeight 25613 Quality 3013 gtgt13 JPEG2000ColorImageDict ltlt13 TileWidth 25613 TileHeight 25613 Quality 3013 gtgt13 AntiAliasGrayImages false13 CropGrayImages true13 GrayImageMinResolution 30013 GrayImageMinResolutionPolicy OK13 DownsampleGrayImages true13 GrayImageDownsampleType Bicubic13 GrayImageResolution 30013 GrayImageDepth -113 GrayImageMinDownsampleDepth 213 GrayImageDownsampleThreshold 15000013 EncodeGrayImages true13 GrayImageFilter DCTEncode13 AutoFilterGrayImages true13 GrayImageAutoFilterStrategy JPEG13 GrayACSImageDict ltlt13 QFactor 01513 HSamples [1 1 1 1] VSamples [1 1 1 1]13 gtgt13 GrayImageDict ltlt13 QFactor 01513 HSamples [1 1 1 1] VSamples [1 1 1 1]13 gtgt13 JPEG2000GrayACSImageDict ltlt13 TileWidth 25613 TileHeight 25613 Quality 3013 gtgt13 JPEG2000GrayImageDict ltlt13 TileWidth 25613 TileHeight 25613 Quality 3013 gtgt13 AntiAliasMonoImages false13 CropMonoImages true13 MonoImageMinResolution 120013 MonoImageMinResolutionPolicy OK13 DownsampleMonoImages true13 MonoImageDownsampleType Bicubic13 MonoImageResolution 120013 MonoImageDepth -113 MonoImageDownsampleThreshold 15000013 EncodeMonoImages true13 MonoImageFilter CCITTFaxEncode13 MonoImageDict ltlt13 K -113 gtgt13 AllowPSXObjects false13 CheckCompliance [13 None13 ]13 PDFX1aCheck false13 PDFX3Check false13 PDFXCompliantPDFOnly false13 PDFXNoTrimBoxError true13 PDFXTrimBoxToMediaBoxOffset [13 00000013 00000013 00000013 00000013 ]13 PDFXSetBleedBoxToMediaBox true13 PDFXBleedBoxToTrimBoxOffset [13 00000013 00000013 00000013 00000013 ]13 PDFXOutputIntentProfile ()13 PDFXOutputConditionIdentifier ()13 PDFXOutputCondition ()13 PDFXRegistryName ()13 PDFXTrapped False1313 CreateJDFFile false13 Description ltlt13 ARA 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 BGR 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 CHS ltFEFF4f7f75288fd94e9b8bbe5b9a521b5efa7684002000410064006f006200650020005000440046002065876863900275284e8e9ad88d2891cf76845370524d53705237300260a853ef4ee54f7f75280020004100630072006f0062006100740020548c002000410064006f00620065002000520065006100640065007200200035002e003000204ee553ca66f49ad87248672c676562535f00521b5efa768400200050004400460020658768633002gt13 CHT ltFEFF4f7f752890194e9b8a2d7f6e5efa7acb7684002000410064006f006200650020005000440046002065874ef69069752865bc9ad854c18cea76845370524d5370523786557406300260a853ef4ee54f7f75280020004100630072006f0062006100740020548c002000410064006f00620065002000520065006100640065007200200035002e003000204ee553ca66f49ad87248672c4f86958b555f5df25efa7acb76840020005000440046002065874ef63002gt13 CZE 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 DAN ltFEFF004200720075006700200069006e0064007300740069006c006c0069006e006700650072006e0065002000740069006c0020006100740020006f007000720065007400740065002000410064006f006200650020005000440046002d0064006f006b0075006d0065006e007400650072002c0020006400650072002000620065006400730074002000650067006e006500720020007300690067002000740069006c002000700072006500700072006500730073002d007500640073006b007200690076006e0069006e00670020006100660020006800f8006a0020006b00760061006c0069007400650074002e0020004400650020006f007000720065007400740065006400650020005000440046002d0064006f006b0075006d0065006e0074006500720020006b0061006e002000e50062006e00650073002000690020004100630072006f00620061007400200065006c006c006500720020004100630072006f006200610074002000520065006100640065007200200035002e00300020006f00670020006e0079006500720065002egt13 DEU 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 ESP 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 ETI 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 FRA 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 GRE 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 HEB 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 HRV (Za stvaranje Adobe PDF dokumenata najpogodnijih za visokokvalitetni ispis prije tiskanja koristite ove postavke Stvoreni PDF dokumenti mogu se otvoriti Acrobat i Adobe Reader 50 i kasnijim verzijama)13 HUN 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 ITA ltFEFF005500740069006c0069007a007a006100720065002000710075006500730074006500200069006d0070006f007300740061007a0069006f006e00690020007000650072002000630072006500610072006500200064006f00630075006d0065006e00740069002000410064006f00620065002000500044004600200070006900f900200061006400610074007400690020006100200075006e00610020007000720065007300740061006d0070006100200064006900200061006c007400610020007100750061006c0069007400e0002e0020004900200064006f00630075006d0065006e007400690020005000440046002000630072006500610074006900200070006f00730073006f006e006f0020006500730073006500720065002000610070006500720074006900200063006f006e0020004100630072006f00620061007400200065002000410064006f00620065002000520065006100640065007200200035002e003000200065002000760065007200730069006f006e006900200073007500630063006500730073006900760065002egt13 JPN ltFEFF9ad854c18cea306a30d730ea30d730ec30b951fa529b7528002000410064006f0062006500200050004400460020658766f8306e4f5c6210306b4f7f75283057307e305930023053306e8a2d5b9a30674f5c62103055308c305f0020005000440046002030d530a130a430eb306f3001004100630072006f0062006100740020304a30883073002000410064006f00620065002000520065006100640065007200200035002e003000204ee5964d3067958b304f30533068304c3067304d307e305930023053306e8a2d5b9a306b306f30d530a930f330c8306e57cb30818fbc307f304c5fc59808306730593002gt13 KOR ltFEFFc7740020c124c815c7440020c0acc6a9d558c5ec0020ace0d488c9c80020c2dcd5d80020c778c1c4c5d00020ac00c7a50020c801d569d55c002000410064006f0062006500200050004400460020bb38c11cb97c0020c791c131d569b2c8b2e4002e0020c774b807ac8c0020c791c131b41c00200050004400460020bb38c11cb2940020004100630072006f0062006100740020bc0f002000410064006f00620065002000520065006100640065007200200035002e00300020c774c0c1c5d0c11c0020c5f40020c2180020c788c2b5b2c8b2e4002egt13 LTH 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 LVI 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 NLD (Gebruik deze instellingen om Adobe PDF-documenten te maken die zijn geoptimaliseerd voor prepress-afdrukken van hoge kwaliteit De gemaakte PDF-documenten kunnen worden geopend met Acrobat en Adobe Reader 50 en hoger)13 NOR 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 POL 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 PTB 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 RUM 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 RUS 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 SKY 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 SLV 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 SUO 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 SVE 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 TUR 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 UKR ltFEFF04120438043a043e0440043804410442043e043204430439044204350020044604560020043f043004400430043c043504420440043800200434043b044f0020044104420432043e04400435043d043d044f00200434043e043a0443043c0435043d044204560432002000410064006f006200650020005000440046002c0020044f043a04560020043d04300439043a04400430044904350020043f045604340445043e0434044f0442044c00200434043b044f0020043204380441043e043a043e044f043a04560441043d043e0433043e0020043f0435044004350434043404400443043a043e0432043e0433043e0020043404400443043a0443002e00200020042104420432043e04400435043d045600200434043e043a0443043c0435043d0442043800200050004400460020043c043e0436043d04300020043204560434043a0440043804420438002004430020004100630072006f006200610074002004420430002000410064006f00620065002000520065006100640065007200200035002e0030002004300431043e0020043f04560437043d04560448043e04570020043204350440044104560457002egt13 ENU (Use these settings to create Adobe PDF documents best suited for high-quality prepress printing Created PDF documents can be opened with Acrobat and Adobe Reader 50 and later)13 gtgt13 Namespace [13 (Adobe)13 (Common)13 (10)13 ]13 OtherNamespaces [13 ltlt13 AsReaderSpreads false13 CropImagesToFrames true13 ErrorControl WarnAndContinue13 FlattenerIgnoreSpreadOverrides false13 IncludeGuidesGrids false13 IncludeNonPrinting false13 IncludeSlug false13 Namespace [13 (Adobe)13 (InDesign)13 (40)13 ]13 OmitPlacedBitmaps false13 OmitPlacedEPS false13 OmitPlacedPDF false13 SimulateOverprint Legacy13 gtgt13 ltlt13 AddBleedMarks false13 AddColorBars false13 AddCropMarks false13 AddPageInfo false13 AddRegMarks false13 ConvertColors ConvertToCMYK13 DestinationProfileName ()13 DestinationProfileSelector DocumentCMYK13 Downsample16BitImages true13 FlattenerPreset ltlt13 PresetSelector MediumResolution13 gtgt13 FormElements false13 GenerateStructure false13 IncludeBookmarks false13 IncludeHyperlinks false13 IncludeInteractive false13 IncludeLayers false13 IncludeProfiles false13 MultimediaHandling UseObjectSettings13 Namespace [13 (Adobe)13 (CreativeSuite)13 (20)13 ]13 PDFXOutputIntentProfileSelector DocumentCMYK13 PreserveEditing true13 UntaggedCMYKHandling LeaveUntagged13 UntaggedRGBHandling UseDocumentProfile13 UseDocumentBleed false13 gtgt13 ]13gtgt setdistillerparams13ltlt13 HWResolution [2400 2400]13 PageSize [612000 792000]13gtgt setpagedevice13

Page 5: July 2014 MEE Questions and Analyses - NCBE...This publication includes the questions and analyses from the July 2014 MEE. (In the actual test, the questions are simply numbered rather

Instructions The back cover of each test booklet contains the following instructions

You will be instructed when to begin and when to stop this test Do not break the seal on this booklet until you are told to begin

You may answer the questions in any order you wish Do not answer more than one question in each answer booklet If you make a mistake or wish to revise your answer simply draw a line through the material you wish to delete

If you are using a laptop computer to answer the questions your jurisdiction will provide you with specific instructions

Read each fact situation very carefully and do not assume facts that are not given in the question Do not assume that each question covers only a single area of the law some of the questions may cover more than one of the areas you are responsible for knowing

Demonstrate your ability to reason and analyze Each of your answers should show an understanding of the facts a recognition of the issues included a knowledge of the applicable principles of law and the reasoning by which you arrive at your conclusions The value of your answer depends not as much upon your conclusions as upon the presence and quality of the elements mentioned above

Clarity and conciseness are important but make your answer complete Do not volunteer irrelevant or immaterial information

Answer all questions according to generally accepted fundamental legal principles unless your testing jurisdiction has instructed you to answer according to local case or statutory law

NOTE Examinees testing in UBE jurisdictions must answer according to generally accepted fundamental legal principles rather than local case or statutory law

iii

July 2014 MEE

QUESTIONS

Contracts Criminal Law and Procedure

Family L aw Federal Civil Procedure

Evidence Corporations

CRIMINAL LAW AND PROCEDURE QUESTION

While on routine patrol a police officer observed a suspect driving erratically and pulled the suspectrsquos car over to investigate When he approached the suspectrsquos car the officer detected a strong odor of marijuana The officer immediately arrested the suspect for driving under the influence of an intoxicant (DUI) While the officer was standing near the suspectrsquos car placing handcuffs on the suspect the officer observed burglary tools on the backseat

The officer seized the burglary tools He then took the suspect to the county jail booked him for the DUI and placed him in a holding cell Later that day the officer gave the tools he had found in the suspectrsquos car to a detective who was investigating a number of recent burglaries in the neighborhood where the suspect had been arrested

At the time of his DUI arrest the suspect had a six-month-old aggravated assault charge pending against him and was being represented on the assault charge by a lawyer

Early the next morning upon learning of her clientrsquos arrest the lawyer went to the jail She arrived at 900 am immediately identified herself to the jailer as the suspectrsquos attorney and demanded to speak with the suspect The lawyer also told the jailer that she did not want the suspect questioned unless she was present The jailer told the lawyer that she would need to wait one hour to see the suspect After speaking with the lawyer the jailer did not inform anyone of the lawyerrsquos presence or her demands

The detective who had also arrived at the jail at 900 am overheard the lawyerrsquos conversation with the jailer The detective then entered the windowless interview room in the jail where the suspect had been taken 30 minutes earlier Without informing the suspect of the lawyerrsquos presence or her demands the detective read to the suspect full and accurate Miranda warnings The detective then informed the suspect that he wanted to ask about the burglary tools found in his car and the recent burglaries in the neighborhood where he had been arrested The suspect replied ldquoI think I want my lawyer here before I talk to yourdquo The detective responded ldquoThatrsquos up to yourdquo

After a few minutes of silence the suspect said ldquoWell unless there is anything else I need to know letrsquos not waste any time waiting for someone to call my attorney and having her drive here I probably should keep my mouth shut but Irsquom willing to talk to you for a whilerdquo The suspect then signed a Miranda waiver form and after interrogation by the detective made incriminating statements regarding five burglaries The interview lasted from 915 am to 1000 am

In addition to the DUI the suspect has been charged with five counts of burglary

The lawyer has filed a motion to suppress all statements made by the suspect to the detective in connection with the five burglaries

The state supreme court follows federal constitutional principles in all cases interpreting a criminal defendantrsquos rights

3

Criminal Law and Procedure Question

1 Did the detective violate the suspectrsquos Sixth Amendment right to counsel when he questioned the suspect in the absence of the lawyer Explain

2 Under Miranda did the suspect effectively invoke his right to counsel Explain

3 Was the suspectrsquos waiver of his Miranda rights valid Explain

4

CONTRACTS QUESTION

A music conservatory has two concert halls One concert hall had a pipe organ that was in poor repair and the other had no organ The conservatory decided to repair the existing organ and buy a new organ for the other concert hall After some negotiation the conservatory entered into two contracts with a business that both repairs and sells organs Under one contract the business agreed to repair the existing pipe organ for the conservatory for $100000 The business would usually charge a higher price for a project of this magnitude but the business agreed to this price because the conservatory agreed to prepay the entire amount Under the other contract the business agreed to sell a new organ to the conservatory for the other concert hall for $225000 As with the repair contract the business agreed to a low sales price because the conservatory agreed to prepay the entire amount Both contracts were signed on January 3 and the conservatory paid the business a total of $325000 that day

Two weeks later before the business had commenced repair of the existing organ the business suffered serious and unanticipated financial reversals The chief financial officer for the business contacted the conservatory and said

Bad news We had an unexpected liability and as a result are in a real cash crunch In fact even though we havenrsquot acquired the new organ from our supplier or started repair of your existing organ wersquove already spent the cash you gave us and we have no free cash on hand Wersquore really sorry but wersquore in a fix I think that we can find a way to perform both contracts but not at the original prices If you agree to pay $60000 more for the repair and $40000 more for the new organ we can probably find financing to finish everything If you donrsquot agree to pay us the extra money I doubt that we will ever be able to perform either contract and yoursquoll be out the money you already paid us

After receiving this unwelcome news the conservatory agreed to pay the extra amounts provided that the extra amount on each contract would be paid only upon completion of the businessrsquos obligations under that contract The business agreed to this arrangement and the parties quickly signed documents reflecting these changes to each contract The business then repaired the existing organ delivered the new organ and demanded payment of the additional $100000

The conservatory now has refused to pay the business the additional amounts for the repair and the new organ

1 Must the conservatory pay the additional $60000 for the organ repair Explain

2 Must the conservatory pay the additional $40000 for the new organ Explain

5

FAMILY LAW QUESTION

In 1994 a man and a woman were married in State A

In 1998 their daughter was born in State A

In 2010 the family moved to State B

In 2012 the husband and wife divorced in State B Under the terms of the divorce decree

(a) the husband and wife share legal and physical custody of their daughter (b) the husband must pay the wife $1000 per month in child support until their daughter reaches age 18 (c) the marital residence was awarded to the wife with the proviso that if it is sold before the daughter reaches age 18 the husband will receive 25 of the net sale proceeds remaining after satisfaction of the mortgage on the residence and (d) the remaining marital assets were divided between the husband and the wife equally

Six months ago the husband was offered a job in State A that pays significantly less than his job in State B but provides him with more responsibilities and much better promotion opportunities The husband accepted the job in State A and moved from State B back to State A

Since returning to State A the husband has not paid child support because due to his lower salary he has had insufficient funds to meet all his obligations

One month ago the wife sold the marital home netting $10000 after paying off the mortgage She then moved to a smaller residence The husband believes that he should receive more than 25 of the net sale proceeds given his financial difficulties

Last week when the wife brought the daughter to the husbandrsquos State A home for a weekend visit the husband served the wife with a summons in a State A action to modify the support and marital-residence-sale-proceeds provisions of the State B divorce decree The husband brought the action in the State A court that adjudicates all domestic relations issues

1 Does the State A court have jurisdiction to modify (a) the child support provision of the State B divorce decree Explain (b) the marital-residence-sale-proceeds provision of the State B divorce decree

Explain

2 On the merits could the husband obtain (a) retroactive modification of his child support obligation to the daughter Explain (b) prospective modification of his child support obligation to the daughter Explain (c) modification of the marital-residence-sale-proceeds provision of the State B

divorce decree Explain

6

FEDERAL CIVIL PROCEDURE QUESTION

The United States Forest Service (USFS) manages public lands in national forests including the Scenic National Forest Without conducting an environmental evaluation or preparing an environmental impact statement the USFS approved a development project in the Scenic National Forest that required the clearing of 5000 acres of old-growth forest The trees in the forest are hundreds of years old and the forest is home to a higher concentration of wildlife than can be found anywhere else in the western United States

The USFS solicited bids from logging companies to harvest the trees on the 5000 acres of forest targeted for clearing and it ultimately awarded the logging contract to the company that had submitted the highest bid for the trees However the USFS has not yet issued the company a logging permit Once it does so the company intends to begin cutting down trees immediately

A nonprofit organization whose mission is the preservation of natural resources has filed suit in federal district court against the USFS The nonprofit alleges that the USFS violated the National Environmental Policy Act (NEPA) by failing to prepare an environmental impact statement for the proposed logging project Among other remedies the nonprofit seeks a permanent injunction barring the USFS from issuing a logging permit to the logging company until an adequate environmental impact statement is completed The nonprofit believes that the logging project would destroy important wildlife habitat and thereby cause serious harm to wildlife in the Scenic National Forest including some endangered species

Assume that federal subject-matter jurisdiction is available that the nonprofit has standing to bring this action and that venue is proper

1 If the logging company seeks to join the litigation as a party must the federal district court allow it to do so as a matter of right Explain

2 What types of relief could the nonprofit seek to stop the USFS from issuing a logging permit during the pendency of the action what must the nonprofit demonstrate to obtain that relief and is the federal district court likely to grant that relief Explain

7

EVIDENCE QUESTION

A prison inmate has filed a civil rights lawsuit against a guard at the prison alleging that the guard violated the inmatersquos constitutional rights during an altercation The inmate and the guard are the only witnesses to this altercation They have provided contradictory reports about what occurred

The trial will be before a jury The inmate plans to testify at trial The guardrsquos counsel has moved for leave to impeach the inmate with the following

(a) Twelve years ago the inmate was convicted of felony distribution of marijuana He served a three-year prison sentence which began immediately after he was convicted He served his full sentence and was released from prison nine years ago (b) Eight years ago the inmate pleaded guilty to perjury a misdemeanor punishable by up to one year in jail He paid a $5000 fine (c) Seven years ago the inmate was convicted of felony sexual assault of a child and is currently serving a 10-year prison sentence for the crime The victim was the inmatersquos daughter who was 13 years old at the time of the assault

The inmatersquos counsel objects to the admission of any evidence related to these three convictions and to any cross-examination based on this evidence

The guard also plans to testify at trial The inmatersquos counsel has moved for leave to impeach the guard with the following

Last year the guard applied for a promotion to prison supervisor The guard submitted a reacutesumeacute to the state that indicated that he had been awarded a BA in Criminal Justice from a local college An official copy of the guardrsquos academic transcript from that college indicates that the guard dropped out after his first semester and did not receive a degree

The guardrsquos counsel objects to the admission of this evidence and to any cross-examination based on this evidence

The transcript and the reacutesumeacute have been properly authenticated The trial will be held in a jurisdiction that has adopted all of the Federal Rules of Evidence

1 What evidence if any proffered by the guard to impeach the inmate should be admitted Explain

2 What evidence if any proffered by the inmate to impeach the guard should be admitted Explain

8

CORPORATIONS QUESTION

Mega Inc is a publicly traded corporation incorporated in a state whose corporate statute is modeled on the Model Business Corporation Act (MBCA) Megarsquos articles of incorporation do not address the election of directors or amendment of the bylaws by shareholders

Well within the deadline for the submission of shareholder proposals for the upcoming annual shareholdersrsquo meeting an investor who was a large and long-standing shareholder of Mega submitted a proposed amendment to Megarsquos bylaws The proposal which the investor asked to be included in the corporationrsquos proxy materials and voted on at the upcoming shareholdersrsquo meeting read as follows

Section 20 The Corporation shall include in its proxy materials (including the proxy ballot) for a shareholdersrsquo meeting at which directors are to be elected the name of a person nominated for election to the Board of Directors by a shareholder or group of shareholders that beneficially have owned 3 or more of the Corporationrsquos outstanding common stock for at least one year

This Section shall supersede any inconsistent provision in these Bylaws and may not be amended or repealed by the Board of Directors without shareholder approval

Megarsquos management decided to exclude the investorrsquos proposal from the corporationrsquos proxy materials and explained its reasons in a letter to the investor

The investorrsquos proposed bylaw provision would be inconsistent with relevant state law because the Board of Directors has the authority to manage the business and affairs of the Corporation Generally shareholders lack the authority to interfere with corporate management by seeking to create a method for the nomination and election of directors inconsistent with the method chosen by the Board of Directors

Furthermore at its most recent meeting the Board of Directors unanimously approved an amendment to the Corporationrsquos bylaws that provides for proxy access for director nominations by a shareholder or a group of shareholders holding at least 10 of the Corporationrsquos voting shares for at least three years This procedure takes precedence over any nomination methods that might be sought or approved by shareholders

The investor is considering bringing a suit challenging managementrsquos refusal to include the investorrsquos proposed bylaw provision and challenging the boardrsquos amendment of the bylaws at its recent meeting

1 Is the investorrsquos proposed bylaw provision inconsistent with state law Explain

2 If the investorrsquos proposed bylaw provision were approved by the shareholders would the bylaw amendment previously approved by the board take precedence over the investorrsquos proposed bylaw provision Explain

3 Must the investor make a demand on Megarsquos board of directors before bringing suit Explain

9

July 2014 MEE

ANALYSES

Contracts Family Law

Criminal Law and Procedure

Federal Civil Procedure Evidence

Corporations

CRIMINAL LAW AND PROCEDURE ANALYSIS (Criminal Law and Procedure VA B D)

ANALYSIS

Legal Problems

(1) Did the detective violate the suspectrsquos Sixth Amendment right to counsel when he questioned the suspect about the burglaries without the lawyer present given that the lawyer represented the suspect in an unrelated criminal matter

(2) Under Miranda did the suspect effectively invoke his right to counsel when he said ldquoI think I want my lawyer here before I talk to yourdquo

(3) Was the suspectrsquos waiver of his right to remain silent under Miranda valid

DISCUSSION

Summary

The Sixth Amendment right to counsel as applied to states through the Fourteenth Amendment is offense-specific Although the suspect had an attorney representing him on his pending assault charge he had no Sixth Amendment right to the assistance of counsel with respect to the five uncharged burglaries because formal adversarial proceedings had not yet commenced on those charges The suspectrsquos Sixth Amendment right to counsel was not violated by the detectiversquos failure to inform him that the lawyer was present or of the lawyerrsquos demands

However a person undergoing custodial interrogation also has an independent constitutional right to counsel during custodial interrogation under Miranda When a suspect invokes his right to counsel under Miranda custodial interrogation must immediately cease for a period of at least 14 days However the invocation of the right to counsel must be unambiguous and clearly convey that the suspect has requested counsel Here because the suspectrsquos statement ldquoI think I want my lawyer here before I talk to yourdquo was ambiguous he did not invoke his Miranda right to counsel

A waiver of rights must be knowing intelligent and voluntary Here the suspect waived his right to remain silent under Miranda when he signed the waiver form The fact that the detective did not correct the suspectrsquos assumption that the lawyer would need to drive to the jailmdashby telling him that the lawyer was in the waiting room and was demanding to see himmdashdid not affect the validity of the suspectrsquos waiver

Point One (35) The suspectrsquos Sixth Amendment right to counsel was not violated because the right does not attach on new charges until formal adversarial judicial proceedings have commenced on those charges

The Sixth Amendment as applied to the states through the Fourteenth Amendment provides that ldquo[i]n all criminal prosecutions the accused shall enjoy the right to have the Assistance of Counsel for his defenserdquo The right to counsel does not attach with respect to particular charges until formal adversarial judicial proceedings have commenced (ie ldquoat or after the initiation of

13

Criminal Law and Procedure Analysis

adversary judicial criminal proceedingsmdashwhether by way of formal charge preliminary hearing indictment information or arraignment [or in some states arrest warrant]rdquo McNeil v Wisconsin 501 US 171 175 (1991) (internal quotations omitted)) Once a suspectrsquos Sixth Amendment right to counsel has attached any attempts to ldquodeliberately elicitrdquo statements from him in the absence of his attorney violate the Sixth Amendment See Massiah v United States 377 US 201 (1964) Brewer v Williams 430 US 387 (1977)

The Sixth Amendment right to counsel is charge- or offense -specific Representation by counsel in one prosecution does not in itself guarantee counsel for uncharged offenses See McNeil 501 US at 175 Texas v Cobb 532 US 162 (2001) Here the suspectrsquos Sixth Amendment right to counsel had attached only for the pending aggravated assault charge The suspectrsquos right to counsel for the aggravated assault case did not guarantee counsel for the five unrelated and uncharged burglaries that were the subject of the detectiversquos interrogation Thus because formal adversarial judicial proceedings against the suspect for the uncharged burglaries had not begun he had no Sixth Amendment right to counsel

Finally the detectiversquos failure to inform the suspect of the lawyerrsquos presence and demands to speak with him does not implicate the suspectrsquos Sixth Amendment right to counsel which had not yet attached See id Moran v Burbine 475 US 412 428ndash31 (1986)

Point Two (30) The suspect did not effectively invoke his right to counsel under Miranda because his statement was not unambiguous

A suspect subject to custodial interrogation has a right to consult with counsel and to have an attorney present during questioning Miranda v Arizona 384 US 436 (1966) When a suspect invokes his right to counsel during an interrogation law enforcement must immediately cease all questioning See Edwards v Arizona 451 US 477 484ndash85 (1981) Custodial interrogation cannot be reinitiated unless and until the suspect has been re-advised of his Miranda rights has provided a knowing and voluntary waiver and (1) counsel is present and (2) the suspect himself initiated further communication with the police see id at 484 or (3) (if the suspect was released from custody after the initial interrogation) at least 14 days have passed Maryland v Shatzer 559 US 98 110 (2010)

To invoke the right to counsel a suspectrsquos request must be ldquounambiguousrdquo This means that the suspect must articulate the desire for counsel sufficiently clearly that a reasonable officer would understand the statement to be a request for counsel Davis v United States 512 US 452 459 (1994) If the request is ambiguous the police are not required to stop the interrogation

In this case the suspectrsquos statement ldquoI think I want my lawyer here before I talk to yourdquo was not an unambiguous request for counsel The most reasonable interpretation of this statement is that the suspect might be invoking his right to counsel Id at 461 (ldquomaybe I should talk to a lawyerrdquo is not an unequivocal request for counsel) See also Burket v Angelone 208 F3d 172 197ndash98 (4th Cir 2000) (ldquoI think I need a lawyerrdquo is not an unambiguous request for an attorney) Soffar v Cockrell 300 F3d 588 594ndash95 (5th Cir 2002) (discussion of various statements that did not constitute unequivocal requests for counsel)

Under these circumstances the detective was not required to cease the custodial interrogation of the suspect Nor was the detective required to clarify or ask follow-up questions to determine whether the suspect in fact wanted an attorney Davis 512 US at 459ndash60

14

Criminal Law and Procedure Analysis

Point Three (35) The suspectrsquos waiver of his Miranda rights was knowing intelligent and voluntary despite the fact that he was never told of the lawyerrsquos presence in the jail or of the lawyerrsquos demands

A valid waiver of Miranda rights must be ldquovoluntaryrdquomdashie the product of a free or deliberate choice rather than intimidation coercion or deception Berghuis v Thompkins 560 US 370 382ndash83 (2010) In addition the waiver must be knowing and intelligent That is it ldquomust have been made with a full awareness of both the nature of the right being abandoned and the consequences of the decision to abandon itrdquo Moran v Burbine 475 US 421 (1986)

In this case the suspect signed a Miranda waiver form after receiving proper warnings There is no evidence ldquothat the police resorted to physical or psychological pressure to elicit the statementsrdquo Id The entire interview lasted only 45 minutes The only issue is whether the suspect knowingly and intelligently waived his Miranda rights despite the fact that the detective did not tell the suspect about the lawyerrsquos presence and her demands

The Supreme Court has said that ldquo[e]vents occurring outside of the presence of the suspect and entirely unknown to him surely can have no bearing on the capacity to comprehend and knowingly relinquish a constitutional rightrdquo Id at 422 If the suspect ldquoknew that he could stand mute and request a lawyer and was aware of the Statersquos intention to use his statements to secure a convictionrdquo then the waiver is valid regardless of the information withheld Id at 422ndash23

Here the suspect was correctly informed of his rights Miranda v Arizona 384 US at 467ndash73 His comments demonstrate that he understood that he could have a lawyer present if he desired (ie wondering whether he should call his attorney) and that he understood that there might be consequences to speaking with the detective (ldquoI probably should keep my mouth shut but Irsquom willing to talk to you for a whilerdquo) His comment ldquo[L]etrsquos not waste any time waiting for someone to call my attorney and having her drive hererdquo along with his signature on the Miranda waiver form show that his waiver was valid under the constitutional standard

The fact that the detective did not tell the suspect about the lawyerrsquos presence and demands has no bearing on the validity of the suspectrsquos waiver because ldquosuch conduct is only relevant to the constitutional validity of a waiver if it deprives a defendant of knowledge essential to his ability to understand the nature of his rights and the consequences of abandoning themrdquo Moran at 424 The Supreme Court has specifically declined to adopt a rule requiring that law enforcement tell a suspect of an attorneyrsquos efforts to contact him id at 425 (ldquoNor are we prepared to adopt a rule requiring that the police inform a suspect of an attorneyrsquos efforts to reach himrdquo)

[NOTE An examinee might also recognize that this general rule is further supported by the Supreme Courtrsquos decision in Florida v Powell 559 US 50 (2010) approving state Miranda warnings that do not explicitly warn suspects that they have a right to have counsel present during custodial interrogation]

15

CONTRACTS ANALYSIS (Contracts IB2 IIB IVA3 amp A5)

ANALYSIS

Legal Problems

(1) In the case of a service contract (governed by the common law of contracts) is a modification enforceable when a party agrees to pay more for the same performance than was originally promised

(2) In the case of a contract for the sale of goods (governed by Article 2 of the UCC) is a modification enforceable when a party agrees to pay more for the same goods than was originally promised

(3) May a party avoid an agreement on the basis of economic duress

DISCUSSION

Summary

There are two arguments that the conservatory can make to support the claim that it is not bound to pay the higher prices lack of consideration and economic duress

The organ repair contract is governed by the common law of contracts Under the common law the business would have difficulty recovering the additional $60000 for the organ repair because under the ldquopreexisting duty rulerdquo the agreement of the conservatory to pay the extra price was not supported by consideration However the business might argue that the modification is enforceable under an exception to the preexisting duty rule for fair and equitable modifications made in light of unanticipated circumstances

The organ sale contract is governed by Article 2 of the Uniform Commercial Code The business would likely recover the additional amount under that contract because Article 2 provides that consideration is not required for a modification to be binding

In both cases the conservatory could seek to avoid its agreement on the grounds of economic duress but that argument is not likely to succeed

Point One (45) The business probably cannot recover the additional $60000 for the organ repair because the conservatoryrsquos promise to pay more money was not supported by consideration

The general rule is that to be enforceable a promise must be supported by consideration Under RESTATEMENT (SECOND) OF CONTRACTS sect 71 a promise is supported by consideration if it is bargained for in exchange for a return promise or performance However under the ldquopreexisting duty rulerdquo (exemplified in RESTATEMENT (SECOND) OF CONTRACTS sect 73 and Alaska Packersrsquo Assrsquon v Domenico 117 F 99 (9th Cir 1902)) promise of performance of a legal duty already owed to a promisor which is neither doubtful nor the subject of honest dispute is not consideration

If the business had promised the conservatory anything new or different in exchange for the agreement to pay the additional $60000 (such as for example repairing the pipe organ more

16

Contracts Analysis

quickly or using better parts) that would constitute consideration especially in light of the principle that courts do not inquire into the adequacy of consideration Here however the business already had a legal duty under the original contract and did not agree to do anything else in exchange for the conservatoryrsquos promise to pay $60000 more

However an exception to the preexisting duty rule is sometimes applied in situations of unanticipated changed circumstances Under RESTATEMENT (SECOND) OF CONTRACTS sect 89 followed in many jurisdictions a promise modifying a duty under a contract not fully performed on either side is binding even if not supported by consideration if the modification is fair and equitable in view of circumstances not anticipated by the parties when the contract was made

If a court applies the rule in Restatement sect 89 the critical issues will be whether the modification was in fact ldquofair and equitablerdquo and whether it can be justified in light of unanticipated circumstances In many cases in which modifications have been upheld a party encountered difficulties or burdens in performing far beyond what was knowingly bargained for in the original contract with the result bordering on impracticability such as having to excavate solid rock instead of soft dirt or having to remove garbage far in excess of the amounts contemplated The conservatory would argue that the businessrsquos performance difficulties were not of this sort at allmdashnothing about repairing the pipe organ itself was any different from or more difficult than originally contemplated except that the business itself encountered financial distress unrelated to its burdens in performing its obligations under these contracts

Even if the business satisfies that element of the rule in Restatement sect 89 the business must also demonstrate that the circumstances that gave rise to the need to modify the contract were ldquounanticipatedrdquo at the time the original contract was made Here the facts suggest that when the business entered into the original contract it expected that the price paid by the conservatory would enable it to perform However any evidence that the business knew or had reason to know at the time of execution that it would need more money from the conservatory to be able to perform would mean that the request to modify was not ldquounanticipatedrdquo

[NOTE Some cases such as Schwartzreich v Bauman-Basch Inc 231 NY 196 131 NE 887 (1921) find that if the parties mutually agreed to rescind the original contract and then after rescission entered into an entirely new contract for a higher price the new contract is supported by consideration There is no evidence that such a rescission followed by a new contract took place here]

Point Two (45) The business can recover the additional $40000 for the new organ because no consideration is required under Article 2 of the UCC for good-faith contract modifications

The contract to buy a new organ is a contract for the sale of goods and therefore is governed by Article 2 of the Uniform Commercial Code UCC sect 2-102 Under Article 2 unlike the common law an agreement modifying a contract needs no consideration to be binding UCC sect 2-209(1) Section 2-209(1) thus obviates the preexisting duty rule entirely in contracts for the sale of goods

Even though consideration is not required modifications governed by sect 2-209 must satisfy the obligation of good faith imposed by the UCC UCC sect 1-304 See also Official Comment 2 to UCC sect 2-209 Good faith means ldquohonesty in fact and the observance of reasonable commercial standards of fair dealingrdquo UCC sect 1-201(b)(20) In this context the obligation of good faith means that ldquo[t]he effective use of bad faith to escape performance on the original contract terms is barred and the extortion of a lsquomodificationrsquo without legitimate commercial reason is ineffective as a violation of the duty of good faithrdquo Official Comment 2 to

17

Contracts Analysis

UCC sect 2-209 Here because the businessrsquos financial reversals were serious and apparently unanticipated at the time that the business entered into the contract with the conservatory and commitment of the extra money was needed to enable the business to perform a court would likely find that the business acted in good faith Thus a court would likely uphold the enforceability of the conservatoryrsquos promise to pay the additional $40000

Point Three (10) The conservatory is unlikely to be able to defend against enforcement of its promises to pay additional money under the theory of economic duress because the business probably did not make an improper threat

Under the common law of contracts parties may raise the defense of duress This common law defense also applies to contracts governed by UCC Article 2 See UCC sect 1-103(b)

A contract is voidable on the ground of economic duress by threat when it is established that a partyrsquos manifestation of assent is induced by an improper threat that leaves the party no reasonable alternative See RESTATEMENT (SECOND) OF CONTRACTS sect 175 See also eg Austin Instrument Inc v Loral Corp 272 NE2d 533 (NY 1971) (a threat to withhold essential goods can constitute duress) In order to void its agreement to pay the additional sum because of economic duress the conservatory must demonstrate that (1) the business made a threat to the conservatory (2) the threat was ldquoimproperrdquo or ldquowrongfulrdquo (3) the threat induced the conservatoryrsquos manifestation of assent to the modification and (4) the threat was sufficiently grave to justify the conservatoryrsquos assent

Here it appears that three of the four elements are likely satisfied The business plainly made a threat Moreover the threat induced the conservatoryrsquos assent to the modification and the threat was sufficiently grave to justify that assent If the conservatory had not agreed to pay the business the extra amounts the conservatory would have lost its entire $325000 investment In light of this potential loss a court could easily conclude that the conservatory had no reasonable alternative

However the business has a strong argument that its threat (indicating that it would breach the contracts unless the prices were increased) was not wrongful or improper but was instead nothing more than a communication of the reality of its own perilous situation to the conservatory

A mere threat to breach a contract is not in and of itself improper so as to support an action of economic duress or business compulsion Something more is required such as a breach of the duty of good faith and fair dealing as was present in Austin Instrument Inc supra Because the business could not perform the original contract without the requested modification the economic duress claim for the conservatory would likely fail for much the same reason that the business would be able to enforce the modification At the time the modification was requested the business was not trying to extort a price increase because of the conservatoryrsquos vulnerability but instead was simply stating the reality that the business could not perform without more money

18

FAMILY LAW ANALYSIS (Family Law IIIB D amp G)

ANALYSIS

Legal Problems

(1)(a) Does the State A court have jurisdiction to modify the State B child support order

(1)(b) Does the State A court have jurisdiction to modify the marital-residence-saleshyproceeds provision of the State B property-division decree

(2)(a) May a child support order be modified retroactively

(2)(b) May a child support order be modified prospectively based on a change of employment with a lower salary

(2)(c) May a property-division order be modified after entry of a divorce decree

DISCUSSION

Summary

The State A court may exercise personal jurisdiction over the wife because she was personally served in State A However subject-matter jurisdiction over the interstate modification of child support is governed by the Uniform Interstate Family Support Act (UIFSA) Under UIFSA State A does not have jurisdiction to modify the order for the daughterrsquos support because the wife is still a resident of State B UIFSA on the other hand does not govern property distributions and thus a State A court is not precluded from hearing the husbandrsquos petition to modify the marital-residence-sale-proceeds provision of the divorce decree

A child support order may not be modified retroactively A child support order may be modified prospectively based on a substantial change in circumstances Courts agree that a significant decrease in income is a substantial change in circumstances All states treat voluntary income reductions differently than involuntary reductions but employ different approaches for evaluating the impact of a voluntary reduction Whether the husband could obtain prospective modification of the child support order depends on which approach is applied

A property-division order is not subject to post-divorce modification based on a change in circumstances Thus the husband may in some states obtain prospective modification of the order for the daughterrsquos support but he may not obtain modification of the marital-residenceshysale-proceeds provision

Point One(a) (25) Personal jurisdiction over a nonresident respondent does not confer subject-matter jurisdiction over child support modification Under UIFSA a State A court may not modify a child support order issued by a State B court when as here the child or either parent continues to reside in State B the jurisdiction that issued the child support order

The State A court may exercise personal jurisdiction over the wife The wife was personally served in State A and a state may exercise jurisdiction based on in-state personal service See

19

Family Law Analysis

Burnham v Superior Court 495 US 604 (1990) But personal jurisdiction over the wife is not enough to give a State A court jurisdiction to modify the State B support order

The interstate enforcement and modification of child support is governed by the Uniform Interstate Family Support Act (UIFSA) which has been adopted by all states Under UIFSA the state that originally issued a child support order (here State B) has continuing exclusive jurisdiction to modify the order if that state remains the residence of the obligee the child or the obligor and all parties do not consent to the jurisdiction of another forum See UIFSA sect 205 See also UIFSA sect 603 (ldquoA tribunal of this State shall recognize and enforce but may not modify a registered order if the issuing tribunal had jurisdictionrdquo) The wife and daughter continue to reside in State B and the wife has not consented to the jurisdiction of another forum Thus a State A court does not have jurisdiction to modify the State B child support order

[NOTE Examinees who do not discuss personal jurisdiction but fully discuss UIFSA may receive full credit]

Point One(b) (15) UIFSA does not apply to disputes over property division Thus the State A court may exercise jurisdiction over the husbandrsquos petition to modify the marital-residence-sale-proceeds provision of the State B divorce decree because it has personal jurisdiction over the wife

The State A court in which the husband brought his action has jurisdiction to adjudicate domestic relations issues The husbandrsquos petition to modify the property settlement is a domestic relations issue The courts of State A may exercise personal jurisdiction over the wife because she was personally served in State A See Burnham v Superior Court 495 US 604 (1990) see Point One(a)

UIFSA does not apply to divorce property-division disputes Thus although a State A court may not adjudicate the husbandrsquos petition to modify his child support obligations it may adjudicate his property-division claims (Even though the court has jurisdiction it may not modify the property-division award on the merits See Point Two(c))

Point Two(a) (20) A child support order may not be modified retroactively

State courts have long held that obligations to pay child support ordinarily may not be modified retroactively ldquoIf the hardship is particularly severe the courts sometimes devised a way to protect the obligor but in most instances the courts hold that retroactive modification of this kind is beyond their power and indeed the governing statute may so providerdquo HOMER H CLARK THE LAW OF DOMESTIC RELATIONSHIPS IN THE UNITED STATES 725 (2d ed 1987)

Federal law now goes further and requires the states as a condition of federal child-support funding to adopt rules that absolutely forbid retroactive modification of the support obligation See 42 USC sect 666(a)(9)(C) The states have adopted rules consistent with the federal requirements

Point Two(b) (25) It is unclear whether the husband could obtain prospective downward modification of his child support based on his voluntary acceptance of a job with a lower salary

Prospective modification of a child support order is typically available only when the petitioner can show a substantial change in circumstances See ROBERT E OLIPHANT amp NANCY VER

20

Family Law Analysis

STEEGH FAMILY LAW 213ndash15 (3d ed 2010) A significant decrease in income is typically viewed as a substantial change

However when a parent seeks to modify a child support obligation because he has voluntarily reduced his income a court will not modify the obligation based solely on the income loss Some courts refuse to modify whenever the income shift was voluntary See eg Aguiar v Aguiar 127 P3d 234 (Idaho Ct App 2005) Others look primarily to the petitionerrsquos intentions and permit downward modification if he has acted in good faith See eg In re Marriage of Horn 650 NE2d 1103 (Ill App Ct 1995) Many courts use a multifactor approach See OLIPHANT amp VER STEEGH supra 217ndash18

Here there is no question that the husbandrsquos loss of income was voluntary In a jurisdiction in which voluntary income reduction bars support modification the husbandrsquos petition would be denied

In a jurisdiction employing a good-faith or multifactor approach it is possible but not certain that the husband could obtain downward modification The evidence supports the husbandrsquos good faith his change in employment appears to be based on his new jobrsquos greater responsibilities and better promotion possibilities In a jurisdiction using a multifactor approach the court would likely also consider the impact of such a shift on the daughter the likely duration of the husbandrsquos income loss and the likelihood of a promotion that would ultimately inure to the daughterrsquos benefit Thus on these facts it is possible but by no means certain that the husband could prospectively obtain downward modification of his child support obligation to his daughter

Point Two(c)(15) A divorce property-division award is not subject to modification

A support order is aimed at meeting the post-divorce needs of the supported individual Because the future is unpredictable courts are empowered to modify a support award to take account of changed circumstances that may occur during the period in which support is paid

By contrast a property-distribution award divides assets of the marriage based on the equities at the time of divorce Because the past can be ascertained a property-division award is not subject to post-divorce modification See HARRY A KRAUSE ET AL FAMILY LAW CASES COMMENTS AND QUESTIONS 691 (6th ed 2007)

Here the husband is seeking modification of a property-division award with respect to an asset owned by the parties at the time of divorce Thus the husband may not obtain a modification of the marital-residence-sale-proceeds provision of the divorce decree based on his reduced income

21

FEDERAL CIVIL PROCEDURE ANALYSIS (Federal Civil Procedure III IVC)

ANALYSIS

Legal Problems

(1) Is the logging company entitled to join this action as a matter of right

(2)(a) May the nonprofit organization obtain a temporary restraining order to stop the USFS from issuing a logging permit

(2)(b) May the nonprofit organization obtain a preliminary injunction to stop the USFS from issuing a logging permit during the pendency of the action

DISCUSSION

Summary

The logging company is entitled to intervene in this action as a matter of right because it has an interest in the property or transaction that is the subject of the action and is so situated that its interest may be impaired or impeded as a practical matter if the action goes forward without it The logging companyrsquos interest is not adequately represented by the USFSrsquos presence in the lawsuit

The nonprofit organization may seek a temporary restraining order (TRO) followed by a preliminary injunction to prevent the USFS from issuing a logging permit pending the outcome of the action The nonprofit is likely to obtain a TRO if it can demonstrate a risk of immediate and irreparable injury The nonprofit is also likely to obtain a preliminary injunction if it can demonstrate a significant threat of irreparable harm and a likelihood of success on the merits of its National Environmental Policy Act (NEPA) claim

Point One (50) Rule 24(a) of the Federal Rules of Civil Procedure requires federal courts to allow a person to intervene in an action as a matter of right if the person a) is interested in the property or transaction that is the subject of the action b) is so situated that its interest may be impaired or impeded if the litigation goes forward without it and c) is not adequately represented by existing parties Here the logging company likely meets all three requirements and should be allowed to intervene as a matter of right

Rule 24 of the Federal Rules of Civil Procedure governs intervention the process by which a non-party to an action may join the litigation Under Rule 24(a) (intervention of right) a person must be permitted to intervene if three conditions are met (1) the movant ldquoclaims an interest relating to the property or transaction that is the subject of the actionrdquo (2) the movant ldquois so situated that disposition of the action may as a practical matter impair or impede the movantrsquos ability to protect its interestrdquo and (3) ldquoexisting partiesrdquo do not ldquoadequately represent [the movantrsquos] interestrdquo FED R CIV P 24(a) The three requirements for intervention of right are often ldquovery interrelatedrdquo 7C CHARLES ALAN WRIGHT ET AL FEDERAL PRACTICE AND PROCEDURE sect 1908 at 297 (2007 amp 2011 Supp)

22

Federal Civil Procedure Analysis

Here the court should find that the logging company meets this test First the logging company has a strong interest in the property or transaction that is the subject of this action The USFS has accepted the logging companyrsquos bid and the logging company is merely awaiting issuance of a logging permit to begin logging The nonprofit organization is seeking to prevent this logging The logging company therefore has a strong direct and substantial interest in the subject matter of the lawsuit and in having its winning bid honored and a logging permit issued See eg Kleissler v US Forest Serv 157 F3d 964 972 (3d Cir 1998) (stating that ldquo[t]imber companies have direct and substantial interests in a lawsuit aimed at halting loggingrdquo) see also Natural Resources Defense Council v US Nuclear Regulatory Commrsquon 578 F2d 1341 1343ndash 44 (10th Cir 1978) (holding that applicants whose license renewals were pending had Rule 24(a)(2) interests where the lawsuit sought to halt the license-issuing process pending preparation of environmental impact statements) See generally 7C WRIGHT ET AL supra sect 19081 at 309 (ldquoIf there is a direct substantial legally protectable interest in the proceedings it is clear that this requirement of the rule is satisfiedrdquo) Second the logging companyrsquos interest in receiving a logging permit may well be impaired as a practical matter by the outcome of the lawsuit If the USFS loses the lawsuit it will have to prepare an environmental impact statement before issuing the logging companyrsquos permit This will at a minimum delay the logging companyrsquos ability to exercise its rights and may in the long r un mean that no logging permit is ever issued Intervention of right is not limited to those that would be legally bound as a matter of preclusion doctrine Id sect 19082 at 368 Rather ldquo[t]he rule is satisfied whenever disposition of the present action would put the movant at a practical disadvantage in protecting its interestrdquo Id sect 19082 at 369 Here that condition is easily satisfied See Kleissler 157 F3d at 972 (ldquoTimber companies have direct and substantial interests in a lawsuit aimed at halting logging rdquo)

Given that the logging company has an interest that may be impaired by disposition of the action it should be allowed to intervene unless the court is persuaded that the USFS adequately represents the logging companyrsquos interest See Rule 24(a)(2) 7C WRIGHT ET AL supra sect 1909 Here it could be argued that the USFS adequately represents the logging companyrsquos interest because the USFS presumably wants the court to uphold its development plan and allow it to proceed with issuance of the logging permit which is the same relief that the logging company would seek However whether representation is truly adequate depends upon ldquo[a] discriminating appraisal of the circumstancesrdquo 7C WRIGHT ET AL supra sect 1909 at 440 Although both the government and the logging company wish to avoid the preparation of an environmental impact statement their interests are distinct The USFSrsquos interest is proper management of the national forest system while the logging companyrsquos interest is making a profit from logging the 5000-acre tract The USFSrsquos handling of the litigation is likely to be affected by a variety of policy concerns and political considerations that have nothing to do with the logging companyrsquos purely economic interest in securing the right to cut trees in the Scenic National Forest See eg Kleissler 157 F3d at 973ndash74 (ldquo[T]he government represents numerous complex and conflicting interests in matters of this nature The straightforward business interests asserted by intervenors here may become lost in the thicket of sometimes inconsistent governmental policiesrdquo)

[NOTES (1) Examinees who mistakenly analyze the logging companyrsquos case for joinder under the related but incorrect Rule 19 ldquoRequired Joinder of Partiesrdquo may receive credit Rule 19 allows existing parties to demand joinder of non-parties (or seek dismissal of the case if they canrsquot get it) There is a close relationship between Rule 24 and Rule 19 and both contain a similar standard for determining when ldquointerestedrdquo third parties are ldquoentitledrdquo or ldquorequiredrdquo to be in the lawsuit Indeed the two prongs of the Rule 24 intervention test that are discussed above

23

Federal Civil Procedure Analysis

are nearly identical to the two prongs of the Rule 19(a) required joinder test Examinees who discuss and apply the test should receive credit even if they cite Rule 19 rather than Rule 24

(2) Examinees may discuss permissive joinder Although permissive joinder is a possibility here the question asks only whether the logging company can join the action as a matter of right and a permissive joinder analysis is not responsive to the question To the extent an examinee discusses permissive joinder the analysis will focus on whether the logging company ldquohas a claim or defense that shares with the main action a common question of law or factrdquo FED R CIV P 24(b)(1)(B) The district court also ldquomust consider whether the intervention will unduly delay or prejudice the adjudication of the original partiesrsquo rightsrdquo FED R CIV P 24(b)(3) On our facts the logging companyrsquos claim for the issuance of a logging permit would certainly share common questions of law and fact with the USFSrsquos defense against the nonprofitrsquos claim There are no facts suggesting that the logging companyrsquos presence would unduly delay or otherwise prejudice adjudication of the original action Thus the district court would have discretion to permit the logging company to intervene even if it denied intervention of right]

Point Two(a) (25) The nonprofit organization could seek and would likely obtain a temporary restraining order to stop the USFS from issuing a logging permit pending a hearing on an application for a preliminary injunction

The first type of interim relief the nonprofit could seek to stop the USFS from issuing a logging permit to the logging company is a temporary restraining order (TRO) prohibiting the USFS from issuing the logging permit A TRO can be issued without notice to the adverse party but only in limited circumstances and only for a limited time FED R CIV P 65(b) To secure a TRO without notice the nonprofit would need to submit an affidavit containing specific facts that demonstrate a risk of ldquoimmediate and irreparable injuryrdquo if a permit is issued FED R CIV P 65(b)(1) In deciding whether to grant a TRO courts will also consider the same factors that are relevant in deciding whether to grant a preliminary injunction (eg the moving partyrsquos likelihood of success on the merits the balance of hardships and the public interest) See Point Two(b) infra The TRO would last only long enough for the court to consider and resolve a request by the nonprofit for a preliminary injunction but no longer than 14 days (unless the court extends it for good cause or the adverse party consents to an extension) In addition bond is required

Here the court is likely to grant the nonprofitrsquos request The nonprofit could plausibly claim that cutting down 5000 acres of old-growth forest in an area that is home to the highest concentration of wildlife in the western United States would have ldquoan immediate and irreparablerdquo adverse impact on the environment and cause irreparable harm to the nonprofitrsquos interest in preserving and protecting natural resources including wildlife habitat

Point Two(b) (25) The nonprofit could also seek and would likely obtain a preliminary injunction to stop the USFS which is likely to be granted if the nonprofitrsquos claim that the USFS violated NEPA has a strong basis in fact and law

Because the TRO would be temporary the nonprofit would need to move for a preliminary injunction to prevent the USFS from issuing a logging permit throughout the pendency of the litigation Preliminary injunctions are injunctions that seek to ldquoprotect [the] plaintiff from

24

Federal Civil Procedure Analysis

irreparable injury and to preserve the courtrsquos power to render a meaningful decision after a trial on the meritsrdquo 11A CHARLES ALAN WRIGHT ET AL FEDERAL PRACTICE AND PROCEDURE sect 2947 at 112 (2013) Rule 65 of the Federal Rules of Civil Procedure sets out the procedural requirements for preliminary injunctions Preliminary injunctions may be granted only upon notice to the adverse party FED R CIV P 65(a)(1) and only if the movant ldquogives security in an amount that the court considers proper to pay the costs and damages sustained by any party found to have been wrongfully enjoined or restrainedrdquo FED R CIV P 65(c)

While Rule 65 sets out the procedural requirements for preliminary injunctive relief it does not specify the substantive grounds upon which it may be granted The courtrsquos discretion in ruling upon a motion for a preliminary injunction ldquois exercised in conformity with historic federal equity practicerdquo 11A WRIGHT ET AL supra sect 2947 at 114 The court typically considers four factors

(1) the significance of the threat of irreparable harm to the plaintiff if the injunction is not granted (2) the balance between this harm and the injury that granting the injunction would inflict on the defendant (3) the probability that the plaintiff will succeed on the merits and (4) the public interest

Id sect 2948 at 122ndash24 accord Habitat Educ Center v Bosworth 363 F Supp 2d 1070 1088 (ED Wis 2005) The most important of these factors is the risk of irreparable harm to the plaintiff 11A WRIGHT ET AL supra sect 29481 at 129 If the plaintiff has an adequate remedy at law (eg if money damages can compensate the plaintiff for its loss) then a preliminary injunction will be denied Id sect 29481

Here a court would likely conclude that the potential for environmental damage to the forest creates a significant threat of irreparable harm ldquo[E]nvironmental injury is often irreparable Courts have recognized that logging such as would occur [here] can have longshyterm environmental consequences and thus satisfy the irreparable injury criterionrdquo Habitat Educ Center 363 F Supp 2d at 1089 (citing Idaho Sporting Congress Inc v Alexander 222 F3d 562 569 (9th Cir 2000) (noting that the imminent and continuing logging activities presented ldquoevidence of environmental harm sufficient to tip the balance in favor of injunctive reliefrdquo)) Neighbors of Cuddy Mountain v US Forest Service 137 F3d 1372 1382 (9th Cir 1998) (stating that ldquo[t]he old growth forests plaintiffs seek to protect would if cut take hundreds of years to reproducerdquo) (internal citation omitted)) see also 11C WRIGHT ET AL supra sect 29481 at 151 (noting that ldquoa preliminary injunction has been issued to prevent harm to the environmentrdquo)

The second factor the balance between the harm to the plaintiff and the harm the defendant will suffer if the injunction is issued also appears to support issuance of a preliminary injunction here The USFS will have to wait before it can develop the Scenic National Forest and the logging company may lose money if the delay is prolonged These economic harms could be compensated monetarily if an injunction is issued inappropriately Where ldquoan injunction bond can compensate [the] defendant for any harm the injunction is likely to inflict the balance should be struck in favor of [the] plaintiffrdquo Id sect 29482 at 192 See also Habitat Educ Center 363 F Supp 2d at 1089 (stating that ldquothe relative absence of harmful effects on the Forest Service weighs in favor of granting the injunctionrdquo)

The third factor is the likelihood that the plaintiff will prevail on the merits Although there is limited information concerning the merits of the action the nonprofit alleges that the federal statute (NEPA) requires an environmental impact statement and further states that the USFS created no environmental impact analysis or statement at all Assuming that those

25

Federal Civil Procedure Analysis

allegations are correct it seems plausible to conclude that the nonprofit will be able to show a likelihood of success on the merits

Finally courts deciding whether or not to issue preliminary injunctive relief are to consider the public interest ldquoFocusing on this factor is another way of inquiring whether there are policy considerations that bear on whether the order should issuerdquo 11C WRIGHT ET AL supra sect 29484 at 214 If the court concludes that the nonprofit is likely to succeed on its NEPA claim because the USFS wrongfully failed to conduct an environmental impact assessment it is likely to find that the public interest would be served by restraining the USFS from proceeding with logging in a national forest See Heartwood Inc v US Forest Service 73 F Supp 2d 962 979 (SD Ill 1999) affrsquod on other grounds 230 F3d 947 (7th Cir 2000) (ldquoviolations by federal agencies of NEPArsquos provisions as established by Congress harm the public as well as the environmentrdquo)

Thus a court is very likely to grant a preliminary injunction if it concludes that the nonprofit has a significant likelihood of success on the merits

26

EVIDENCE ANALYSIS (Evidence ID IIA amp C)

ANALYSIS

Legal Problems

(1) Under what circumstances can evidence of prior convictions be used to impeach a witnessrsquos credibility in a civil case

(1)(a) May the inmatersquos credibility be impeached by evidence of a 12-year-old felony drug conviction if he was released from prison 9 years ago

(1)(b) May the inmatersquos credibility be impeached by evidence of an 8-year-old misdemeanor perjury conviction that was punishable by 1 year in jail if he pleaded guilty and was sentenced only to pay a $5000 fine

(1)(c) May the inmatersquos credibility be impeached by evidence of a 7-year-old sexual assault conviction if the inmate is still serving a 10-year prison sentence and the victim was his 13-year-old daughter

(2)(a) May the guardrsquos credibility be impeached by cross-examination regarding specific instances of misconduct (ie lying on his reacutesumeacute) relevant to credibility

(2)(b) May the guardrsquos credibility be impeached by admission of extrinsic evidence (his reacutesumeacute and academic transcript) offered to prove specific instances of misconduct relevant to credibility

DISCUSSION

Summary

Under the Federal Rules of Evidence witnesses can be impeached with evidence of prior convictions andor specific instances of misconduct Whether evidence of prior convictions should be admitted to impeach generally depends on the nature of the crime the amount of time that has passed and (only in criminal cases) whether the ldquowitnessrdquo is the defendant FED R EVID 609(a)

In this civil case evidence of the inmatersquos conviction for distribution of marijuana should be admitted to impeach the inmate because he was convicted of a felony and was released from prison fewer than 10 years ago FED R EVID 609(a)(1) Credibility is critically important in this case because the jury will hear conflicting testimony from the two disputing parties and there were no other eyewitnesses to the altercation Under Rule 609(a)(1) the inmatersquos conviction should be admitted because it has some bearing on his credibility and its probative value is not substantially outweighed by concerns of unfair prejudice confusion or delay Id

Evidence of the inmatersquos misdemeanor conviction for perjury must be admitted because the crime ldquorequired provingmdashor the witnessrsquos admittingmdasha dishonest act or false statementrdquo by the inmate FED R EVID 609(a)(2)

27

Evidence Analysis

Evidence of the inmatersquos felony conviction for sexual assault should be excluded because its probative value is substantially outweighed by the danger of unfair prejudice to the inmate based on the heinous nature of the crime FED R EVID 609(a)(1) In the alternative the judge could limit the evidence relating to this conviction by excluding details of the inmatersquos crime

In all civil (and criminal) cases witnesses can also be impeached with evidence of specific instances of prior misconduct that did not result in a conviction FED R EVID 608(b) Pursuant to Rule 608(b) misconduct probative of untruthfulness can be inquired into on cross-examination but cannot be proved through extrinsic evidence Id Thus the inmatersquos counsel should be permitted to cross-examine the guard regarding the false statement in the guardrsquos reacutesumeacute However extrinsic evidence of the guardrsquos misconduct (ie the guardrsquos authenticated reacutesumeacute and transcript from the local college) should not be admitted even if the guard denies wrongdoing or refuses to answer cross-examination questions about these matters Id

Point One (10) The Federal Rules of Evidence permit impeachment of witnesses with evidence of prior convictions

Whether convictions should be admitted to impeach generally depends on the nature of the crime the amount of time that has passed and (only in criminal cases) whether the ldquowitnessrdquo is the defendant FED R EVID 609(a) Under Rule 609(a) evidence of prior convictions may be admitted for the purpose of ldquoattacking a witnessrsquos character for truthfulnessrdquo Id

There are two basic types of convictions that can be admitted for the purpose of impeachment

(1) convictions for crimes ldquopunishable by death or by imprisonment for more than one yearrdquo (which generally correlates to ldquofeloniesrdquo) FED R EVID 609(a)(1) and (2) convictions ldquofor any crimes regardless of the punishment if the court can readily determine that establishing the elements of the crime required provingmdashor the witnessrsquos admittingmdasha dishonest act or false statementrdquo FED R EVID 609(a)(2)

Pursuant to Rule 609(a)(1) in civil cases the admission of evidence of a felony conviction is ldquosubject to Rule 403 [which says that a court may exclude relevant evidence if its probative value is substantially outweighed by other factors]rdquo FED R EVID 609(a)(1) However Rule 403 does not protect the witness against admission of prior convictions involving dishonestymdashwhich must be admitted by the court FED R EVID 609(a)(2)

Finally Federal Rule of Evidence 609(b) contains the presumption that a conviction that is more than 10 years old or where more than 10 years has passed since the witnessrsquos release from confinement (whichever is later) should not be admitted unless ldquoits probative value supported by specific facts and circumstances substantially outweighs its prejudicial effectrdquo and the proponent has provided the adverse party with reasonable written notice FED R EVID 609(b)

Point One(a) (25) The court should admit evidence of the inmatersquos 12-year-old felony marijuana distribution conviction

The inmatersquos conviction for marijuana distribution was for a felony punishable by imprisonment for more than one year See FED R EVID 609(a)(1) Moreover although the conviction was 12 years ago the 10-year time limit of Rule 609(b) is not exceeded because that time limit runs

28

Evidence Analysis

from the date of either ldquothe witnessrsquos conviction or release from confinement for it whichever is laterrdquo FED R EVID 609(b) Because the inmate served three years in prison he was released from confinement nine years ago

However pursuant to Rule 609(a)(1) the admission of felony convictions to impeach a witness in a civil case is ldquosubject to Rule 403rdquo FED R EVID 609(a)(1) Neither Rule 609(a) nor the advisory committee notes specify which factors courts should consider when balancing the probative value of a conviction against the dangers identified in Rule 403 (which include (1) unfair prejudice (2) confusion of the issues (3) misleading the jury (4) waste of time or undue delay and (5) needless presentation of cumulative evidence) FED R EVID 403

In this case credibility is very important because the evidence consists primarily of the testimony of the disputing parties and there were no other eyewitnesses to the altercation This enhances the probative value of any evidence bearing on the inmatersquos credibility A court is likely to conclude that the inmatersquos prior felony drug conviction is relevant to his credibility See eg United States v Brito 427 F3d 53 64 (1st Cir 2005) (ldquoPrior drug-trafficking crimes are generally viewed as having some bearing on veracityrdquo) Although the probative value of any conviction diminishes with age see eg United States v Brewer 451 F Supp 50 53 (ED Tenn 1978) the inmatersquos ongoing problems with the law suggest that he has continued (and even escalated) his criminal behavior over the past nine years The court should admit this evidence because its probative value is not substantially outweighed by any Rule 403 concerns Specifically any prejudice to the inmate would be slight because the conviction is unrelated to the altercation at issue and the conviction was not for a heinous crime that might inflame the jury

[NOTE Whether an examinee identifies the jury instruction as containing a ldquoconclusiverdquo or ldquomandatoryrdquo presumption is less important than the examineersquos analysis of the constitutional infirmities]

Point One(b) (15) The court must admit evidence of the inmatersquos eight-year-old misdemeanor conviction because perjury is a crime of dishonesty

Rule 609(a)(2) provides that evidence of a criminal conviction ldquomust be admitted if the court can readily determine that establishing the elements of the crime required provingmdashor the witnessrsquos admittingmdasha dishonest act or false statementrdquo FED R EVID 609(a)(2) The inmatersquos conviction for perjury would have necessarily required proving that the inmate engaged in an act of dishonesty This conviction occurred within the past 10 years so it ldquomust be admittedrdquo because in contrast to Rule 609(a)(1) (discussed in Point One(a)) admission under Rule 609(a)(2) is mandatory and not subject to Rule 403

Point One(c) (20) The court should exclude evidence of the inmatersquos seven-year-old felony sexual assault conviction because the probative value of this evidence is substantially outweighed by the danger of unfair prejudice In the alternative the details of the prior conviction could be excluded

The inmatersquos conviction for felony sexual assault was seven years ago and he has not yet been released from incarceration so Rule 609(a) but not 609(b) is applicable here FED R EVID 609(a) This conviction is therefore admissible to impeach the inmate unless its probative value is substantially outweighed by the danger of unfair prejudice or any other Rule 403 concern Id

29

Evidence Analysis

Sex crimes are generally not considered relevant to credibility see Hopkins v State 639 So 2d 1247 1254 (Miss 1993) so the probative value of this conviction is relatively low Moreover the heinous nature of the inmatersquos crime (sexual assault on his daughter) makes the danger of unfair prejudice to the inmate very high Thus the court should exclude evidence of the conviction because it was for a heinous offense that is likely to inflame the jury and it has little bearing on credibility See eg United States v Beahm 664 F2d 414 419 (4th Cir 1981)

As an alternative to excluding this evidence the judge could minimize the unfair prejudice to the inmate by permitting limited cross-examination but refusing to allow specific questions about the nature of the inmatersquos conviction For example a court could limit cross-examination to the fact that the inmate was convicted of a ldquofelonyrdquo or perhaps that he was convicted of a ldquosexual assaultrdquo without identifying the victim However because evidence of the inmatersquos prior convictions can be admitted solely for the purpose of enabling the jury to assess his credibility and because his two earlier convictions should have already been admitted the court should exclude all evidence of the felony sexual assault conviction

Point Two(a) (15) The court should permit the inmatersquos counsel to cross-examine the guard regarding the false statement in his reacutesumeacute because the guardrsquos misconduct bears on his truthfulness

The inmate wishes to cross-examine the guard about his prior dishonest behaviormdashlying on his reacutesumeacutemdashthat did not involve a criminal conviction Rule 608(b) allows witnesses to be cross-examined about specific instances of prior non-conviction misconduct probative of untruthfulness ldquoin order to attack the witnessrsquos character for truthfulnessrdquo FED R EVID 608(b)

The courtrsquos decision to allow cross-examination about the guardrsquos prior dishonest behavior depends on the probative value of such evidence balanced against the danger of unfair prejudice to the guard or any other Rule 403 concern FED R EVID 403 Here the guardrsquos false statement on his reacutesumeacute that he obtained a degree in Criminal Justice is highly probative of his untruthfulness because it grossly misrepresents his actual academic record was made recently and was made with the intent to deceive Because the probative value of this evidence is very strong and is not substantially outweighed by any Rule 403 concerns cross-examination of the guard on this topic should be permitted The court may also consider it fair to permit this cross-examination of the guard on these matters assuming that one or more of the inmatersquos prior convictions have been admitted to impeach his credibility

Point Two(b) (15) The court should exclude extrinsic evidence of the guardrsquos non-conviction misconduct even if the guard denies wrongdoing or refuses to answer questions about the matter

Although Rule 608(b) allows cross-examination about specific instances of prior misconduct probative of untruthfulness ldquoextrinsic evidencerdquo offered to prove such misconduct is not admissible FED R EVID 608(b) The rationale for this rule is that allowing the introduction of extrinsic evidence of prior misconduct by witnesses when these acts are relevant only to the witnessesrsquo truthfulness and not to the main issues in the case would create too great a risk of confusing the jury and unduly delaying the trial The court does not have discretion to admit this extrinsic evidence See eg United States v Elliot 89 F3d 1360 1368 (8th Cir 1996)

30

Evidence Analysis

Here the inmatersquos counsel may cross-examine the guard about the false statement on his reacutesumeacute However the inmatersquos counsel must accept the guardrsquos response Even if the guard denies wrongdoing or refuses to answer questions about the matter the inmatersquos counsel cannot introduce the guardrsquos reacutesumeacute or the transcript from the local college to prove the guardrsquos misconduct

31

CORPORATIONS ANALYSIS (Corporations VA2 IX)

ANALYSIS

Legal Problems

(1) Do shareholders have the authority to amend a corporationrsquos bylaws with respect to director nominations

(2) Do board-approved bylaws on a particular subject here nomination of directors preempt subsequent conflicting bylaw amendments by shareholders

(3) Is a suit challenging both managementrsquos refusal to include the proposed bylaw amendment in Megarsquos proxy statement and the boardrsquos amendment of the bylaws dealing with nomination of directors a direct or derivative suit

DISCUSSION

Summary

The voting and litigation rights of the shareholders of Mega are subject to the provisions of the Model Business Corporations Act (MBCA)

The investorrsquos proposed bylaw provision is not inconsistent with state law Under the MBCA shareholders may amend the bylaws when the amendment deals with a proper matter for the corporationrsquos bylaws such as procedures for nominating directors

The Mega boardrsquos bylaw amendment does not preempt the investorrsquos proposed bylaw provision or the Mega shareholdersrsquo power to approve it While shareholders can limit the boardrsquos power to amend or repeal the bylaws the board cannot limit the shareholdersrsquo power

Whether the investor must make a demand on Megarsquos board depends on how the investor frames its claim If the investor claims a violation of shareholder voting rights the claim is direct and pre-suit demand on the board is not required If on the other hand the investor claims that the directors violated their fiduciary duties by amending the bylaws to entrench themselves the claim is derivative and a pre-suit demand is required

Point One (30) Shareholders may amend the corporationrsquos bylaws where the proposed bylaw provision relates to procedural matters typically included in the bylaws such as the nomination of directors

Internal affairs of the corporation such as the conduct of shareholder meetings and election of directors are subject to the corporate law of the state of incorporation See McDermott Inc v Lewis 531 A2d 206 (Del 1987) (applying law of jurisdiction where corporation was incorporated in case involving voting rights) This statersquos corporate statute is modeled on the MBCA

Under the MBCA ldquoshareholders may amend the corporationrsquos bylawsrdquo MBCA sect 1020(a) Thus the only question is whether the bylaws can specify the procedures for shareholder nomination of directors

32

Corporations Analysis

The MBCA states that the bylaws ldquomay contain any provision that is not inconsistent with law or the articles of incorporationrdquo MBCA sect 206(b) In addition the MBCA was revised in 2009 to address shareholder nomination of directors in public corporations (known as ldquoproxy accessrdquo) and specifies that the bylaws ldquomay contain a requirement that the corporation include in its [proxy materials] one or more individuals nominated by a shareholderrdquo MBCA sect 206(c)(1) see Committee on Corporate Laws ABA Section of Business Law Report on the Roles of Boards of Directors and Shareholders of Publicly Owned Corporations and Changes to the Model Business Corporations ActmdashAdoption of Shareholder Proxy Access Amendments to Chapters 2 and 10 65 BUS LAWYER 1105 (2010)

The inclusion of director-nomination procedures in the bylaws is consistent with practice and is recognized by the Delaware courts whose views on corporate law carry significant weight Typically the procedures for nomination of directors are found in the bylaws See 1 COX amp HAZEN TREATISE ON THE LAW OF CORPORATIONS sect 312 (3d ed 2011) see also 4 FLETCHER CORP FORMS ANN PART III ch 21 (2013) (including sample bylaws that permit nomination of directors by shareholders) The Delaware Supreme Court has confirmed that the bylaws may ldquodefine the process and proceduresrdquo for director elections See CA Inc v AFSCME Employees Pension Plan 953 A2d 227 (Del 2008) (concluding that bylaw amendment requiring reimbursement of election expenses to certain successful shareholder nominators is ldquoproper subjectrdquo under Delaware law)

[NOTE The question of the proper scope of the bylaws can be answered using the more general MBCA sect 206(b) or the 2009 MBCA revision adding sect 206(c)(1) (adopted in CT ME VA) In addition some examinees might raise the point that shareholder proposals may not compel the board to take action such as by including shareholder nominations in the companyrsquos proxy materials on the theory that the ldquobusiness and affairsrdquo of the corporation are to be managed by the board See MBCA sect 801(b) Although shareholders are generally limited to adopting precatory resolutions that recommend or encourage board action this limitation does not apply when shareholders have specific authority to take binding action on their ownmdashsuch as to amend the bylaws]

Point Two (30) Shareholders can amend (or repeal) board-approved bylaws Further shareholders can limit the boardrsquos power to later amend and repeal a shareholder-approved bylaw

Under the MBCA shareholders have the power to amend the bylaws See Point One The board shares this power with the shareholders unless (1) the corporationrsquos articles ldquoreserve that power exclusively to the shareholdersrdquo or (2) ldquothe shareholders in amending repealing or adopting a bylaw expressly provide that the board of directors may not amend repeal or reinstate that bylawrdquo See MBCA sect 1020(b)

Shareholder-approved bylaw provisions can amend or repeal existing bylaw provisions whether originally approved by the board or by shareholders See ALAN R PALMITER CORPORATIONS EXAMPLES AND EXPLANATIONS sect 713 (7th ed 2012) Thus the Mega boardrsquos bylaw amendmentmdashwhich set more demanding thresholds for shareholder nomination of directors than the investorrsquos proposed bylaw provisionmdashwould be superseded (repealed) if Megarsquos shareholders were to approve the investorrsquos proposal

Further a shareholder-approved bylaw generally can limit the power of the board to later amend or repeal it See MBCA sect 1020(b)(2) Thus if Megarsquos shareholders approved the bylaw

33

Corporations Analysis

provision proposed by the investor Megarsquos board could not repeal the provision because it includes a ldquono board repealrdquo clause

The revision to the MBCA in 2009 dealing with shareholder proxy access does not change this conclusion That revision specifies that a shareholder-approved bylaw dealing with director nominations may not limit the boardrsquos power to amend add or repeal ldquoany procedure or condition to such a bylaw in order to provide for a reasonable practicable and orderly processrdquo MBCA sect 206(d) Thus according to the revision if shareholders approve a bylaw amendment that limits further board changes the board would nonetheless retain the power to ldquotinkerrdquo with the bylaw to safeguard the voting process but could not repeal the shareholder-approved bylaw The Official Comment to MBCA sect 206(d) makes clear that the revision is ldquonot intended to allow the board of directors to frustrate the purpose of the shareholder-adopted proxy access provisionrdquo Thus if Megarsquos shareholders were to approve the bylaw provision proposed by the investor Megarsquos board could only amend the provision regarding its procedures or conditions in a manner consistent with its purpose of permitting proxy access for Megarsquos shareholders

[NOTE The boardrsquos attempted interference with a shareholder voting initiative may also have been a violation of the directorsrsquo fiduciary duties See Blasius Indus Inc v Atlas Corp 564 A2d 651 (Del Ch 1988) (finding that directors breached their fiduciary duties by amending bylaws and expanding size of board to thwart insurgentrsquos plan to amend bylaws and seat a majority of new directors) The call however asks examinees to consider whether shareholders or the board have ldquoprecedencerdquo over amending the corporate bylaws Thus an examineersquos answer should be framed in terms of ldquopowerrdquo and not ldquodutyrdquo]

Point Three (40) The investor need not make a demand on the board if the investor states a direct claim such as an allegation that the board interfered with the investorrsquos right to amend the bylaws But the investor must make a demand on the board if the investor states a derivative claim (on behalf of the corporation) such as an allegation that the directors sought to entrench themselves by interfering with the proposed proxy access

The MBCA generally requires that shareholders make a demand on the board of directors before initiation of a derivative suit MBCA sect 742 (shareholder may not bring derivative proceeding until written demand has been made on corporation and 90 days have expired) A derivative suit is essentially two suits in one where the plaintiff-shareholder seeks to bring on behalf of the corporation a claim that vindicates corporate rights usually based on violation of fiduciary duties PALMITER supra sect 1811 (6th ed 2009) The demand permits the board to investigate the situation identified by the shareholder and take suitable action No demand on the board is required however if the shareholder brings a direct suit to vindicate the shareholderrsquos own rights not those of the corporation

Is the suit brought by the investor derivative or direct The MBCA defines a ldquoderivative proceedingrdquo as one brought ldquoin the right of a domestic corporationrdquo MBCA sect 740(1) Thus the answer to how the investorrsquos suit should be characterized turns on what rights the investor seeks to vindicate If the investor frames its claim as one of fiduciary breach by directorsmdashfor example for failing to become adequately informed about voting procedures or for seeking to entrench themselves in office by manipulating the voting structure to avoid a shareholder insurgencymdashthen the suit is ldquoderivativerdquo and the investor must make a demand on the board See MBCA Ch 7 Subch D Introductory Comment (ldquothe derivative suit has historically been the principal method of challenging allegedly illegal action by managementrdquo)

34

Corporations Analysis

If however the investor frames its claim as one to vindicate shareholder rights the suit is direct and no demand is required For many courts the direct-derivative question turns on who is injured and who is to receive the relief sought by the plaintiff-shareholders See Tooley v Donaldson Lufkin amp Jenrette Inc 845 A2d 1031 (Del 2004) (characterizing a merger-delay claim as direct because delay of merger only harmed shareholders not corporation) Thus if the investor claims that managementrsquos refusal to include its proposed bylaw amendment in the corporationrsquos proxy materials violates its shareholder rights to initiate corporate governance reforms the suit will be direct Courts have not questioned the ability of shareholders to bring direct suits challenging board action to exclude their proposed bylaw amendments from the corporationrsquos proxy materials See JANA Master Fund Ltd v CNET Networks Inc 954 A2d 335 (Del Ch 2008) (upholding shareholderrsquos direct challenge to boardrsquos interpretation of advance-notice bylaw) Chesapeake Corp v Shore 771 A2d 293 (Del Ch 2000) (upholding shareholderrsquos direct challenge to actions by board that effectively prevented it from proposing bylaw amendments in contest for control)

Is the way that the investor frames its claim conclusive Courts have permitted shareholder-plaintiffs to challenge a transaction in a direct suit even though the same transaction could also be challenged as a fiduciary breach See Eisenberg v Flying Tiger Line Inc 451 F2d 267 (2d Cir 1971) (permitting direct suit challenging a corporate reorganization as a dilution of shareholder voting power even though reorganization may have involved conflicts of interest and thus constituted a fiduciary breach) Thus the investorrsquos choice to pursue a claim challenging the legality of managementrsquos decision to exclude the investorrsquos proposal from the corporationrsquos proxy materialsmdashrather than a possible breach of fiduciary dutymdashis likely to be respected See 3 COX amp HAZEN supra sect 153 (describing situations in which a claim can be framed as derivative or direct)

[NOTE Some issues under Delaware corporate law regarding pre-suit demand are not relevant here For example whether the Mega directors are independent and disinterested is not relevant to the MBCA requirement of a pre-suit demand As the Official Comment to MBCA sect 742 points out the MBCArsquos requirement of ldquouniversal demandrdquo gives the board ldquothe opportunity to reexamine the act complained of in the light of a potential lawsuit and take corrective actionrdquo even when the directors might be non-independent or have conflicts of interest

Nor is it relevant to the MBCA pre-suit demand requirement that the statutory 90-day waiting period may be onerous The first paragraph of MBCA sect 742 requires a pre-suit demand without exception the second paragraph of the section imposes a 90-day waiting period before a derivative suit may be brought which can be shortened if the board rejects the demand or ldquoirreparable injury to the corporation would result by waiting for the expiration of the 90-day periodrdquo The call as written asks only whether a pre-suit demand should be made and does not ask examinees to address whether the post-demand waiting period should be shortened under the ldquoirreparable injuryrdquo standard]

35

National Conference of Bar Examiners 302 South Bedford Street | Madison WI 53703-3622 Phone 608-280-8550 | Fax 608-280-8552 | TDD 608-661-1275

wwwncbexorg e-mail contactncbexorg

  • Preface
  • Description of the MEE
  • Instructions
  • July 2014 Questions
    • CRIMINAL LAW AND PROCEDURE QUESTION
    • CONTRACTS QUESTION
    • FAMILY LAW QUESTION
    • FEDERAL CIVIL PROCEDURE QUESTION
    • EVIDENCE QUESTION
    • CORPORATIONS QUESTION
      • July 2014 Analyses
        • CRIMINAL LAW AND PROCEDURE ANALYSIS
        • CONTRACTS ANALYSIS
        • FAMILY LAW ANALYSIS
        • FEDERAL CIVIL PROCEDURE ANALYSIS
        • EVIDENCE ANALYSIS
        • CORPORATIONS ANALYSIS
            • ltlt13 ASCII85EncodePages false13 AllowTransparency false13 AutoPositionEPSFiles true13 AutoRotatePages None13 Binding Left13 CalGrayProfile (Dot Gain 20)13 CalRGBProfile (sRGB IEC61966-21)13 CalCMYKProfile (US Web Coated 050SWOP051 v2)13 sRGBProfile (sRGB IEC61966-21)13 CannotEmbedFontPolicy Error13 CompatibilityLevel 1413 CompressObjects Tags13 CompressPages true13 ConvertImagesToIndexed true13 PassThroughJPEGImages true13 CreateJobTicket false13 DefaultRenderingIntent Default13 DetectBlends true13 DetectCurves 0000013 ColorConversionStrategy CMYK13 DoThumbnails false13 EmbedAllFonts true13 EmbedOpenType false13 ParseICCProfilesInComments true13 EmbedJobOptions true13 DSCReportingLevel 013 EmitDSCWarnings false13 EndPage -113 ImageMemory 104857613 LockDistillerParams false13 MaxSubsetPct 10013 Optimize true13 OPM 113 ParseDSCComments true13 ParseDSCCommentsForDocInfo true13 PreserveCopyPage true13 PreserveDICMYKValues true13 PreserveEPSInfo true13 PreserveFlatness true13 PreserveHalftoneInfo false13 PreserveOPIComments true13 PreserveOverprintSettings true13 StartPage 113 SubsetFonts true13 TransferFunctionInfo Apply13 UCRandBGInfo Preserve13 UsePrologue false13 ColorSettingsFile ()13 AlwaysEmbed [ true13 ]13 NeverEmbed [ true13 ]13 AntiAliasColorImages false13 CropColorImages true13 ColorImageMinResolution 30013 ColorImageMinResolutionPolicy OK13 DownsampleColorImages true13 ColorImageDownsampleType Bicubic13 ColorImageResolution 30013 ColorImageDepth -113 ColorImageMinDownsampleDepth 113 ColorImageDownsampleThreshold 15000013 EncodeColorImages true13 ColorImageFilter DCTEncode13 AutoFilterColorImages true13 ColorImageAutoFilterStrategy JPEG13 ColorACSImageDict ltlt13 QFactor 01513 HSamples [1 1 1 1] VSamples [1 1 1 1]13 gtgt13 ColorImageDict ltlt13 QFactor 01513 HSamples [1 1 1 1] VSamples [1 1 1 1]13 gtgt13 JPEG2000ColorACSImageDict ltlt13 TileWidth 25613 TileHeight 25613 Quality 3013 gtgt13 JPEG2000ColorImageDict ltlt13 TileWidth 25613 TileHeight 25613 Quality 3013 gtgt13 AntiAliasGrayImages false13 CropGrayImages true13 GrayImageMinResolution 30013 GrayImageMinResolutionPolicy OK13 DownsampleGrayImages true13 GrayImageDownsampleType Bicubic13 GrayImageResolution 30013 GrayImageDepth -113 GrayImageMinDownsampleDepth 213 GrayImageDownsampleThreshold 15000013 EncodeGrayImages true13 GrayImageFilter DCTEncode13 AutoFilterGrayImages true13 GrayImageAutoFilterStrategy JPEG13 GrayACSImageDict ltlt13 QFactor 01513 HSamples [1 1 1 1] VSamples [1 1 1 1]13 gtgt13 GrayImageDict ltlt13 QFactor 01513 HSamples [1 1 1 1] VSamples [1 1 1 1]13 gtgt13 JPEG2000GrayACSImageDict ltlt13 TileWidth 25613 TileHeight 25613 Quality 3013 gtgt13 JPEG2000GrayImageDict ltlt13 TileWidth 25613 TileHeight 25613 Quality 3013 gtgt13 AntiAliasMonoImages false13 CropMonoImages true13 MonoImageMinResolution 120013 MonoImageMinResolutionPolicy OK13 DownsampleMonoImages true13 MonoImageDownsampleType Bicubic13 MonoImageResolution 120013 MonoImageDepth -113 MonoImageDownsampleThreshold 15000013 EncodeMonoImages true13 MonoImageFilter CCITTFaxEncode13 MonoImageDict ltlt13 K -113 gtgt13 AllowPSXObjects false13 CheckCompliance [13 None13 ]13 PDFX1aCheck false13 PDFX3Check false13 PDFXCompliantPDFOnly false13 PDFXNoTrimBoxError true13 PDFXTrimBoxToMediaBoxOffset [13 00000013 00000013 00000013 00000013 ]13 PDFXSetBleedBoxToMediaBox true13 PDFXBleedBoxToTrimBoxOffset [13 00000013 00000013 00000013 00000013 ]13 PDFXOutputIntentProfile ()13 PDFXOutputConditionIdentifier ()13 PDFXOutputCondition ()13 PDFXRegistryName ()13 PDFXTrapped False1313 CreateJDFFile false13 Description ltlt13 ARA 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 BGR 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 CHS ltFEFF4f7f75288fd94e9b8bbe5b9a521b5efa7684002000410064006f006200650020005000440046002065876863900275284e8e9ad88d2891cf76845370524d53705237300260a853ef4ee54f7f75280020004100630072006f0062006100740020548c002000410064006f00620065002000520065006100640065007200200035002e003000204ee553ca66f49ad87248672c676562535f00521b5efa768400200050004400460020658768633002gt13 CHT ltFEFF4f7f752890194e9b8a2d7f6e5efa7acb7684002000410064006f006200650020005000440046002065874ef69069752865bc9ad854c18cea76845370524d5370523786557406300260a853ef4ee54f7f75280020004100630072006f0062006100740020548c002000410064006f00620065002000520065006100640065007200200035002e003000204ee553ca66f49ad87248672c4f86958b555f5df25efa7acb76840020005000440046002065874ef63002gt13 CZE 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 DAN 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 DEU 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 ESP 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 ETI ltFEFF004b00610073007500740061006700650020006e0065006900640020007300e4007400740065006900640020006b00760061006c006900740065006500740073006500200074007200fc006b006900650065006c007300650020007000720069006e00740069006d0069007300650020006a0061006f006b007300200073006f00620069006c0069006b0065002000410064006f006200650020005000440046002d0064006f006b0075006d0065006e00740069006400650020006c006f006f006d006900730065006b0073002e00200020004c006f006f0064007500640020005000440046002d0064006f006b0075006d0065006e00740065002000730061006100740065002000610076006100640061002000700072006f006700720061006d006d006900640065006700610020004100630072006f0062006100740020006e0069006e0067002000410064006f00620065002000520065006100640065007200200035002e00300020006a00610020007500750065006d006100740065002000760065007200730069006f006f006e00690064006500670061002e000d000agt13 FRA 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 GRE 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 HEB ltFEFF05D405E905EA05DE05E905D5002005D105D405D205D305E805D505EA002005D005DC05D4002005DB05D305D9002005DC05D905E605D505E8002005DE05E105DE05DB05D9002000410064006F006200650020005000440046002005D405DE05D505EA05D005DE05D905DD002005DC05D405D305E405E105EA002005E705D305DD002D05D305E405D505E1002005D005D905DB05D505EA05D905EA002E002005DE05E105DE05DB05D90020005000440046002005E905E005D505E605E805D5002005E005D905EA05E005D905DD002005DC05E405EA05D905D705D4002005D105D005DE05E605E205D505EA0020004100630072006F006200610074002005D5002D00410064006F00620065002000520065006100640065007200200035002E0030002005D505D205E805E105D005D505EA002005DE05EA05E705D305DE05D505EA002005D905D505EA05E8002E05D005DE05D905DD002005DC002D005000440046002F0058002D0033002C002005E205D905D905E005D5002005D105DE05D305E805D905DA002005DC05DE05E905EA05DE05E9002005E905DC0020004100630072006F006200610074002E002005DE05E105DE05DB05D90020005000440046002005E905E005D505E605E805D5002005E005D905EA05E005D905DD002005DC05E405EA05D905D705D4002005D105D005DE05E605E205D505EA0020004100630072006F006200610074002005D5002D00410064006F00620065002000520065006100640065007200200035002E0030002005D505D205E805E105D005D505EA002005DE05EA05E705D305DE05D505EA002005D905D505EA05E8002Egt13 HRV (Za stvaranje Adobe PDF dokumenata najpogodnijih za visokokvalitetni ispis prije tiskanja koristite ove postavke Stvoreni PDF dokumenti mogu se otvoriti Acrobat i Adobe Reader 50 i kasnijim verzijama)13 HUN 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 ITA 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 JPN ltFEFF9ad854c18cea306a30d730ea30d730ec30b951fa529b7528002000410064006f0062006500200050004400460020658766f8306e4f5c6210306b4f7f75283057307e305930023053306e8a2d5b9a30674f5c62103055308c305f0020005000440046002030d530a130a430eb306f3001004100630072006f0062006100740020304a30883073002000410064006f00620065002000520065006100640065007200200035002e003000204ee5964d3067958b304f30533068304c3067304d307e305930023053306e8a2d5b9a306b306f30d530a930f330c8306e57cb30818fbc307f304c5fc59808306730593002gt13 KOR ltFEFFc7740020c124c815c7440020c0acc6a9d558c5ec0020ace0d488c9c80020c2dcd5d80020c778c1c4c5d00020ac00c7a50020c801d569d55c002000410064006f0062006500200050004400460020bb38c11cb97c0020c791c131d569b2c8b2e4002e0020c774b807ac8c0020c791c131b41c00200050004400460020bb38c11cb2940020004100630072006f0062006100740020bc0f002000410064006f00620065002000520065006100640065007200200035002e00300020c774c0c1c5d0c11c0020c5f40020c2180020c788c2b5b2c8b2e4002egt13 LTH 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 LVI 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 NLD (Gebruik deze instellingen om Adobe PDF-documenten te maken die zijn geoptimaliseerd voor prepress-afdrukken van hoge kwaliteit De gemaakte PDF-documenten kunnen worden geopend met Acrobat en Adobe Reader 50 en hoger)13 NOR 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 POL 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 PTB ltFEFF005500740069006c0069007a006500200065007300730061007300200063006f006e00660069006700750072006100e700f50065007300200064006500200066006f0072006d00610020006100200063007200690061007200200064006f00630075006d0065006e0074006f0073002000410064006f0062006500200050004400460020006d00610069007300200061006400650071007500610064006f00730020007000610072006100200070007200e9002d0069006d0070007200650073007300f50065007300200064006500200061006c007400610020007100750061006c00690064006100640065002e0020004f007300200064006f00630075006d0065006e0074006f00730020005000440046002000630072006900610064006f007300200070006f00640065006d0020007300650072002000610062006500720074006f007300200063006f006d0020006f0020004100630072006f006200610074002000650020006f002000410064006f00620065002000520065006100640065007200200035002e0030002000650020007600650072007300f50065007300200070006f00730074006500720069006f007200650073002egt13 RUM 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 RUS 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 SKY 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 SLV 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 SUO ltFEFF004b00e40079007400e40020006e00e40069007400e4002000610073006500740075006b007300690061002c0020006b0075006e0020006c0075006f00740020006c00e400680069006e006e00e4002000760061006100740069007600610061006e0020007000610069006e006100740075006b00730065006e002000760061006c006d0069007300740065006c00750074007900f6006800f6006e00200073006f00700069007600690061002000410064006f0062006500200050004400460020002d0064006f006b0075006d0065006e007400740065006a0061002e0020004c0075006f0064007500740020005000440046002d0064006f006b0075006d0065006e00740069007400200076006f0069006400610061006e0020006100760061007400610020004100630072006f0062006100740069006c006c00610020006a0061002000410064006f00620065002000520065006100640065007200200035002e0030003a006c006c00610020006a006100200075007500640065006d006d0069006c006c0061002egt13 SVE 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 TUR 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 UKR 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 ENU (Use these settings to create Adobe PDF documents best suited for high-quality prepress printing Created PDF documents can be opened with Acrobat and Adobe Reader 50 and later)13 gtgt13 Namespace [13 (Adobe)13 (Common)13 (10)13 ]13 OtherNamespaces [13 ltlt13 AsReaderSpreads false13 CropImagesToFrames true13 ErrorControl WarnAndContinue13 FlattenerIgnoreSpreadOverrides false13 IncludeGuidesGrids false13 IncludeNonPrinting false13 IncludeSlug false13 Namespace [13 (Adobe)13 (InDesign)13 (40)13 ]13 OmitPlacedBitmaps false13 OmitPlacedEPS false13 OmitPlacedPDF false13 SimulateOverprint Legacy13 gtgt13 ltlt13 AddBleedMarks false13 AddColorBars false13 AddCropMarks false13 AddPageInfo false13 AddRegMarks false13 ConvertColors ConvertToCMYK13 DestinationProfileName ()13 DestinationProfileSelector DocumentCMYK13 Downsample16BitImages true13 FlattenerPreset ltlt13 PresetSelector MediumResolution13 gtgt13 FormElements false13 GenerateStructure false13 IncludeBookmarks false13 IncludeHyperlinks false13 IncludeInteractive false13 IncludeLayers false13 IncludeProfiles false13 MultimediaHandling UseObjectSettings13 Namespace [13 (Adobe)13 (CreativeSuite)13 (20)13 ]13 PDFXOutputIntentProfileSelector DocumentCMYK13 PreserveEditing true13 UntaggedCMYKHandling LeaveUntagged13 UntaggedRGBHandling UseDocumentProfile13 UseDocumentBleed false13 gtgt13 ]13gtgt setdistillerparams13ltlt13 HWResolution [2400 2400]13 PageSize [612000 792000]13gtgt setpagedevice13

Page 6: July 2014 MEE Questions and Analyses - NCBE...This publication includes the questions and analyses from the July 2014 MEE. (In the actual test, the questions are simply numbered rather

July 2014 MEE

QUESTIONS

Contracts Criminal Law and Procedure

Family L aw Federal Civil Procedure

Evidence Corporations

CRIMINAL LAW AND PROCEDURE QUESTION

While on routine patrol a police officer observed a suspect driving erratically and pulled the suspectrsquos car over to investigate When he approached the suspectrsquos car the officer detected a strong odor of marijuana The officer immediately arrested the suspect for driving under the influence of an intoxicant (DUI) While the officer was standing near the suspectrsquos car placing handcuffs on the suspect the officer observed burglary tools on the backseat

The officer seized the burglary tools He then took the suspect to the county jail booked him for the DUI and placed him in a holding cell Later that day the officer gave the tools he had found in the suspectrsquos car to a detective who was investigating a number of recent burglaries in the neighborhood where the suspect had been arrested

At the time of his DUI arrest the suspect had a six-month-old aggravated assault charge pending against him and was being represented on the assault charge by a lawyer

Early the next morning upon learning of her clientrsquos arrest the lawyer went to the jail She arrived at 900 am immediately identified herself to the jailer as the suspectrsquos attorney and demanded to speak with the suspect The lawyer also told the jailer that she did not want the suspect questioned unless she was present The jailer told the lawyer that she would need to wait one hour to see the suspect After speaking with the lawyer the jailer did not inform anyone of the lawyerrsquos presence or her demands

The detective who had also arrived at the jail at 900 am overheard the lawyerrsquos conversation with the jailer The detective then entered the windowless interview room in the jail where the suspect had been taken 30 minutes earlier Without informing the suspect of the lawyerrsquos presence or her demands the detective read to the suspect full and accurate Miranda warnings The detective then informed the suspect that he wanted to ask about the burglary tools found in his car and the recent burglaries in the neighborhood where he had been arrested The suspect replied ldquoI think I want my lawyer here before I talk to yourdquo The detective responded ldquoThatrsquos up to yourdquo

After a few minutes of silence the suspect said ldquoWell unless there is anything else I need to know letrsquos not waste any time waiting for someone to call my attorney and having her drive here I probably should keep my mouth shut but Irsquom willing to talk to you for a whilerdquo The suspect then signed a Miranda waiver form and after interrogation by the detective made incriminating statements regarding five burglaries The interview lasted from 915 am to 1000 am

In addition to the DUI the suspect has been charged with five counts of burglary

The lawyer has filed a motion to suppress all statements made by the suspect to the detective in connection with the five burglaries

The state supreme court follows federal constitutional principles in all cases interpreting a criminal defendantrsquos rights

3

Criminal Law and Procedure Question

1 Did the detective violate the suspectrsquos Sixth Amendment right to counsel when he questioned the suspect in the absence of the lawyer Explain

2 Under Miranda did the suspect effectively invoke his right to counsel Explain

3 Was the suspectrsquos waiver of his Miranda rights valid Explain

4

CONTRACTS QUESTION

A music conservatory has two concert halls One concert hall had a pipe organ that was in poor repair and the other had no organ The conservatory decided to repair the existing organ and buy a new organ for the other concert hall After some negotiation the conservatory entered into two contracts with a business that both repairs and sells organs Under one contract the business agreed to repair the existing pipe organ for the conservatory for $100000 The business would usually charge a higher price for a project of this magnitude but the business agreed to this price because the conservatory agreed to prepay the entire amount Under the other contract the business agreed to sell a new organ to the conservatory for the other concert hall for $225000 As with the repair contract the business agreed to a low sales price because the conservatory agreed to prepay the entire amount Both contracts were signed on January 3 and the conservatory paid the business a total of $325000 that day

Two weeks later before the business had commenced repair of the existing organ the business suffered serious and unanticipated financial reversals The chief financial officer for the business contacted the conservatory and said

Bad news We had an unexpected liability and as a result are in a real cash crunch In fact even though we havenrsquot acquired the new organ from our supplier or started repair of your existing organ wersquove already spent the cash you gave us and we have no free cash on hand Wersquore really sorry but wersquore in a fix I think that we can find a way to perform both contracts but not at the original prices If you agree to pay $60000 more for the repair and $40000 more for the new organ we can probably find financing to finish everything If you donrsquot agree to pay us the extra money I doubt that we will ever be able to perform either contract and yoursquoll be out the money you already paid us

After receiving this unwelcome news the conservatory agreed to pay the extra amounts provided that the extra amount on each contract would be paid only upon completion of the businessrsquos obligations under that contract The business agreed to this arrangement and the parties quickly signed documents reflecting these changes to each contract The business then repaired the existing organ delivered the new organ and demanded payment of the additional $100000

The conservatory now has refused to pay the business the additional amounts for the repair and the new organ

1 Must the conservatory pay the additional $60000 for the organ repair Explain

2 Must the conservatory pay the additional $40000 for the new organ Explain

5

FAMILY LAW QUESTION

In 1994 a man and a woman were married in State A

In 1998 their daughter was born in State A

In 2010 the family moved to State B

In 2012 the husband and wife divorced in State B Under the terms of the divorce decree

(a) the husband and wife share legal and physical custody of their daughter (b) the husband must pay the wife $1000 per month in child support until their daughter reaches age 18 (c) the marital residence was awarded to the wife with the proviso that if it is sold before the daughter reaches age 18 the husband will receive 25 of the net sale proceeds remaining after satisfaction of the mortgage on the residence and (d) the remaining marital assets were divided between the husband and the wife equally

Six months ago the husband was offered a job in State A that pays significantly less than his job in State B but provides him with more responsibilities and much better promotion opportunities The husband accepted the job in State A and moved from State B back to State A

Since returning to State A the husband has not paid child support because due to his lower salary he has had insufficient funds to meet all his obligations

One month ago the wife sold the marital home netting $10000 after paying off the mortgage She then moved to a smaller residence The husband believes that he should receive more than 25 of the net sale proceeds given his financial difficulties

Last week when the wife brought the daughter to the husbandrsquos State A home for a weekend visit the husband served the wife with a summons in a State A action to modify the support and marital-residence-sale-proceeds provisions of the State B divorce decree The husband brought the action in the State A court that adjudicates all domestic relations issues

1 Does the State A court have jurisdiction to modify (a) the child support provision of the State B divorce decree Explain (b) the marital-residence-sale-proceeds provision of the State B divorce decree

Explain

2 On the merits could the husband obtain (a) retroactive modification of his child support obligation to the daughter Explain (b) prospective modification of his child support obligation to the daughter Explain (c) modification of the marital-residence-sale-proceeds provision of the State B

divorce decree Explain

6

FEDERAL CIVIL PROCEDURE QUESTION

The United States Forest Service (USFS) manages public lands in national forests including the Scenic National Forest Without conducting an environmental evaluation or preparing an environmental impact statement the USFS approved a development project in the Scenic National Forest that required the clearing of 5000 acres of old-growth forest The trees in the forest are hundreds of years old and the forest is home to a higher concentration of wildlife than can be found anywhere else in the western United States

The USFS solicited bids from logging companies to harvest the trees on the 5000 acres of forest targeted for clearing and it ultimately awarded the logging contract to the company that had submitted the highest bid for the trees However the USFS has not yet issued the company a logging permit Once it does so the company intends to begin cutting down trees immediately

A nonprofit organization whose mission is the preservation of natural resources has filed suit in federal district court against the USFS The nonprofit alleges that the USFS violated the National Environmental Policy Act (NEPA) by failing to prepare an environmental impact statement for the proposed logging project Among other remedies the nonprofit seeks a permanent injunction barring the USFS from issuing a logging permit to the logging company until an adequate environmental impact statement is completed The nonprofit believes that the logging project would destroy important wildlife habitat and thereby cause serious harm to wildlife in the Scenic National Forest including some endangered species

Assume that federal subject-matter jurisdiction is available that the nonprofit has standing to bring this action and that venue is proper

1 If the logging company seeks to join the litigation as a party must the federal district court allow it to do so as a matter of right Explain

2 What types of relief could the nonprofit seek to stop the USFS from issuing a logging permit during the pendency of the action what must the nonprofit demonstrate to obtain that relief and is the federal district court likely to grant that relief Explain

7

EVIDENCE QUESTION

A prison inmate has filed a civil rights lawsuit against a guard at the prison alleging that the guard violated the inmatersquos constitutional rights during an altercation The inmate and the guard are the only witnesses to this altercation They have provided contradictory reports about what occurred

The trial will be before a jury The inmate plans to testify at trial The guardrsquos counsel has moved for leave to impeach the inmate with the following

(a) Twelve years ago the inmate was convicted of felony distribution of marijuana He served a three-year prison sentence which began immediately after he was convicted He served his full sentence and was released from prison nine years ago (b) Eight years ago the inmate pleaded guilty to perjury a misdemeanor punishable by up to one year in jail He paid a $5000 fine (c) Seven years ago the inmate was convicted of felony sexual assault of a child and is currently serving a 10-year prison sentence for the crime The victim was the inmatersquos daughter who was 13 years old at the time of the assault

The inmatersquos counsel objects to the admission of any evidence related to these three convictions and to any cross-examination based on this evidence

The guard also plans to testify at trial The inmatersquos counsel has moved for leave to impeach the guard with the following

Last year the guard applied for a promotion to prison supervisor The guard submitted a reacutesumeacute to the state that indicated that he had been awarded a BA in Criminal Justice from a local college An official copy of the guardrsquos academic transcript from that college indicates that the guard dropped out after his first semester and did not receive a degree

The guardrsquos counsel objects to the admission of this evidence and to any cross-examination based on this evidence

The transcript and the reacutesumeacute have been properly authenticated The trial will be held in a jurisdiction that has adopted all of the Federal Rules of Evidence

1 What evidence if any proffered by the guard to impeach the inmate should be admitted Explain

2 What evidence if any proffered by the inmate to impeach the guard should be admitted Explain

8

CORPORATIONS QUESTION

Mega Inc is a publicly traded corporation incorporated in a state whose corporate statute is modeled on the Model Business Corporation Act (MBCA) Megarsquos articles of incorporation do not address the election of directors or amendment of the bylaws by shareholders

Well within the deadline for the submission of shareholder proposals for the upcoming annual shareholdersrsquo meeting an investor who was a large and long-standing shareholder of Mega submitted a proposed amendment to Megarsquos bylaws The proposal which the investor asked to be included in the corporationrsquos proxy materials and voted on at the upcoming shareholdersrsquo meeting read as follows

Section 20 The Corporation shall include in its proxy materials (including the proxy ballot) for a shareholdersrsquo meeting at which directors are to be elected the name of a person nominated for election to the Board of Directors by a shareholder or group of shareholders that beneficially have owned 3 or more of the Corporationrsquos outstanding common stock for at least one year

This Section shall supersede any inconsistent provision in these Bylaws and may not be amended or repealed by the Board of Directors without shareholder approval

Megarsquos management decided to exclude the investorrsquos proposal from the corporationrsquos proxy materials and explained its reasons in a letter to the investor

The investorrsquos proposed bylaw provision would be inconsistent with relevant state law because the Board of Directors has the authority to manage the business and affairs of the Corporation Generally shareholders lack the authority to interfere with corporate management by seeking to create a method for the nomination and election of directors inconsistent with the method chosen by the Board of Directors

Furthermore at its most recent meeting the Board of Directors unanimously approved an amendment to the Corporationrsquos bylaws that provides for proxy access for director nominations by a shareholder or a group of shareholders holding at least 10 of the Corporationrsquos voting shares for at least three years This procedure takes precedence over any nomination methods that might be sought or approved by shareholders

The investor is considering bringing a suit challenging managementrsquos refusal to include the investorrsquos proposed bylaw provision and challenging the boardrsquos amendment of the bylaws at its recent meeting

1 Is the investorrsquos proposed bylaw provision inconsistent with state law Explain

2 If the investorrsquos proposed bylaw provision were approved by the shareholders would the bylaw amendment previously approved by the board take precedence over the investorrsquos proposed bylaw provision Explain

3 Must the investor make a demand on Megarsquos board of directors before bringing suit Explain

9

July 2014 MEE

ANALYSES

Contracts Family Law

Criminal Law and Procedure

Federal Civil Procedure Evidence

Corporations

CRIMINAL LAW AND PROCEDURE ANALYSIS (Criminal Law and Procedure VA B D)

ANALYSIS

Legal Problems

(1) Did the detective violate the suspectrsquos Sixth Amendment right to counsel when he questioned the suspect about the burglaries without the lawyer present given that the lawyer represented the suspect in an unrelated criminal matter

(2) Under Miranda did the suspect effectively invoke his right to counsel when he said ldquoI think I want my lawyer here before I talk to yourdquo

(3) Was the suspectrsquos waiver of his right to remain silent under Miranda valid

DISCUSSION

Summary

The Sixth Amendment right to counsel as applied to states through the Fourteenth Amendment is offense-specific Although the suspect had an attorney representing him on his pending assault charge he had no Sixth Amendment right to the assistance of counsel with respect to the five uncharged burglaries because formal adversarial proceedings had not yet commenced on those charges The suspectrsquos Sixth Amendment right to counsel was not violated by the detectiversquos failure to inform him that the lawyer was present or of the lawyerrsquos demands

However a person undergoing custodial interrogation also has an independent constitutional right to counsel during custodial interrogation under Miranda When a suspect invokes his right to counsel under Miranda custodial interrogation must immediately cease for a period of at least 14 days However the invocation of the right to counsel must be unambiguous and clearly convey that the suspect has requested counsel Here because the suspectrsquos statement ldquoI think I want my lawyer here before I talk to yourdquo was ambiguous he did not invoke his Miranda right to counsel

A waiver of rights must be knowing intelligent and voluntary Here the suspect waived his right to remain silent under Miranda when he signed the waiver form The fact that the detective did not correct the suspectrsquos assumption that the lawyer would need to drive to the jailmdashby telling him that the lawyer was in the waiting room and was demanding to see himmdashdid not affect the validity of the suspectrsquos waiver

Point One (35) The suspectrsquos Sixth Amendment right to counsel was not violated because the right does not attach on new charges until formal adversarial judicial proceedings have commenced on those charges

The Sixth Amendment as applied to the states through the Fourteenth Amendment provides that ldquo[i]n all criminal prosecutions the accused shall enjoy the right to have the Assistance of Counsel for his defenserdquo The right to counsel does not attach with respect to particular charges until formal adversarial judicial proceedings have commenced (ie ldquoat or after the initiation of

13

Criminal Law and Procedure Analysis

adversary judicial criminal proceedingsmdashwhether by way of formal charge preliminary hearing indictment information or arraignment [or in some states arrest warrant]rdquo McNeil v Wisconsin 501 US 171 175 (1991) (internal quotations omitted)) Once a suspectrsquos Sixth Amendment right to counsel has attached any attempts to ldquodeliberately elicitrdquo statements from him in the absence of his attorney violate the Sixth Amendment See Massiah v United States 377 US 201 (1964) Brewer v Williams 430 US 387 (1977)

The Sixth Amendment right to counsel is charge- or offense -specific Representation by counsel in one prosecution does not in itself guarantee counsel for uncharged offenses See McNeil 501 US at 175 Texas v Cobb 532 US 162 (2001) Here the suspectrsquos Sixth Amendment right to counsel had attached only for the pending aggravated assault charge The suspectrsquos right to counsel for the aggravated assault case did not guarantee counsel for the five unrelated and uncharged burglaries that were the subject of the detectiversquos interrogation Thus because formal adversarial judicial proceedings against the suspect for the uncharged burglaries had not begun he had no Sixth Amendment right to counsel

Finally the detectiversquos failure to inform the suspect of the lawyerrsquos presence and demands to speak with him does not implicate the suspectrsquos Sixth Amendment right to counsel which had not yet attached See id Moran v Burbine 475 US 412 428ndash31 (1986)

Point Two (30) The suspect did not effectively invoke his right to counsel under Miranda because his statement was not unambiguous

A suspect subject to custodial interrogation has a right to consult with counsel and to have an attorney present during questioning Miranda v Arizona 384 US 436 (1966) When a suspect invokes his right to counsel during an interrogation law enforcement must immediately cease all questioning See Edwards v Arizona 451 US 477 484ndash85 (1981) Custodial interrogation cannot be reinitiated unless and until the suspect has been re-advised of his Miranda rights has provided a knowing and voluntary waiver and (1) counsel is present and (2) the suspect himself initiated further communication with the police see id at 484 or (3) (if the suspect was released from custody after the initial interrogation) at least 14 days have passed Maryland v Shatzer 559 US 98 110 (2010)

To invoke the right to counsel a suspectrsquos request must be ldquounambiguousrdquo This means that the suspect must articulate the desire for counsel sufficiently clearly that a reasonable officer would understand the statement to be a request for counsel Davis v United States 512 US 452 459 (1994) If the request is ambiguous the police are not required to stop the interrogation

In this case the suspectrsquos statement ldquoI think I want my lawyer here before I talk to yourdquo was not an unambiguous request for counsel The most reasonable interpretation of this statement is that the suspect might be invoking his right to counsel Id at 461 (ldquomaybe I should talk to a lawyerrdquo is not an unequivocal request for counsel) See also Burket v Angelone 208 F3d 172 197ndash98 (4th Cir 2000) (ldquoI think I need a lawyerrdquo is not an unambiguous request for an attorney) Soffar v Cockrell 300 F3d 588 594ndash95 (5th Cir 2002) (discussion of various statements that did not constitute unequivocal requests for counsel)

Under these circumstances the detective was not required to cease the custodial interrogation of the suspect Nor was the detective required to clarify or ask follow-up questions to determine whether the suspect in fact wanted an attorney Davis 512 US at 459ndash60

14

Criminal Law and Procedure Analysis

Point Three (35) The suspectrsquos waiver of his Miranda rights was knowing intelligent and voluntary despite the fact that he was never told of the lawyerrsquos presence in the jail or of the lawyerrsquos demands

A valid waiver of Miranda rights must be ldquovoluntaryrdquomdashie the product of a free or deliberate choice rather than intimidation coercion or deception Berghuis v Thompkins 560 US 370 382ndash83 (2010) In addition the waiver must be knowing and intelligent That is it ldquomust have been made with a full awareness of both the nature of the right being abandoned and the consequences of the decision to abandon itrdquo Moran v Burbine 475 US 421 (1986)

In this case the suspect signed a Miranda waiver form after receiving proper warnings There is no evidence ldquothat the police resorted to physical or psychological pressure to elicit the statementsrdquo Id The entire interview lasted only 45 minutes The only issue is whether the suspect knowingly and intelligently waived his Miranda rights despite the fact that the detective did not tell the suspect about the lawyerrsquos presence and her demands

The Supreme Court has said that ldquo[e]vents occurring outside of the presence of the suspect and entirely unknown to him surely can have no bearing on the capacity to comprehend and knowingly relinquish a constitutional rightrdquo Id at 422 If the suspect ldquoknew that he could stand mute and request a lawyer and was aware of the Statersquos intention to use his statements to secure a convictionrdquo then the waiver is valid regardless of the information withheld Id at 422ndash23

Here the suspect was correctly informed of his rights Miranda v Arizona 384 US at 467ndash73 His comments demonstrate that he understood that he could have a lawyer present if he desired (ie wondering whether he should call his attorney) and that he understood that there might be consequences to speaking with the detective (ldquoI probably should keep my mouth shut but Irsquom willing to talk to you for a whilerdquo) His comment ldquo[L]etrsquos not waste any time waiting for someone to call my attorney and having her drive hererdquo along with his signature on the Miranda waiver form show that his waiver was valid under the constitutional standard

The fact that the detective did not tell the suspect about the lawyerrsquos presence and demands has no bearing on the validity of the suspectrsquos waiver because ldquosuch conduct is only relevant to the constitutional validity of a waiver if it deprives a defendant of knowledge essential to his ability to understand the nature of his rights and the consequences of abandoning themrdquo Moran at 424 The Supreme Court has specifically declined to adopt a rule requiring that law enforcement tell a suspect of an attorneyrsquos efforts to contact him id at 425 (ldquoNor are we prepared to adopt a rule requiring that the police inform a suspect of an attorneyrsquos efforts to reach himrdquo)

[NOTE An examinee might also recognize that this general rule is further supported by the Supreme Courtrsquos decision in Florida v Powell 559 US 50 (2010) approving state Miranda warnings that do not explicitly warn suspects that they have a right to have counsel present during custodial interrogation]

15

CONTRACTS ANALYSIS (Contracts IB2 IIB IVA3 amp A5)

ANALYSIS

Legal Problems

(1) In the case of a service contract (governed by the common law of contracts) is a modification enforceable when a party agrees to pay more for the same performance than was originally promised

(2) In the case of a contract for the sale of goods (governed by Article 2 of the UCC) is a modification enforceable when a party agrees to pay more for the same goods than was originally promised

(3) May a party avoid an agreement on the basis of economic duress

DISCUSSION

Summary

There are two arguments that the conservatory can make to support the claim that it is not bound to pay the higher prices lack of consideration and economic duress

The organ repair contract is governed by the common law of contracts Under the common law the business would have difficulty recovering the additional $60000 for the organ repair because under the ldquopreexisting duty rulerdquo the agreement of the conservatory to pay the extra price was not supported by consideration However the business might argue that the modification is enforceable under an exception to the preexisting duty rule for fair and equitable modifications made in light of unanticipated circumstances

The organ sale contract is governed by Article 2 of the Uniform Commercial Code The business would likely recover the additional amount under that contract because Article 2 provides that consideration is not required for a modification to be binding

In both cases the conservatory could seek to avoid its agreement on the grounds of economic duress but that argument is not likely to succeed

Point One (45) The business probably cannot recover the additional $60000 for the organ repair because the conservatoryrsquos promise to pay more money was not supported by consideration

The general rule is that to be enforceable a promise must be supported by consideration Under RESTATEMENT (SECOND) OF CONTRACTS sect 71 a promise is supported by consideration if it is bargained for in exchange for a return promise or performance However under the ldquopreexisting duty rulerdquo (exemplified in RESTATEMENT (SECOND) OF CONTRACTS sect 73 and Alaska Packersrsquo Assrsquon v Domenico 117 F 99 (9th Cir 1902)) promise of performance of a legal duty already owed to a promisor which is neither doubtful nor the subject of honest dispute is not consideration

If the business had promised the conservatory anything new or different in exchange for the agreement to pay the additional $60000 (such as for example repairing the pipe organ more

16

Contracts Analysis

quickly or using better parts) that would constitute consideration especially in light of the principle that courts do not inquire into the adequacy of consideration Here however the business already had a legal duty under the original contract and did not agree to do anything else in exchange for the conservatoryrsquos promise to pay $60000 more

However an exception to the preexisting duty rule is sometimes applied in situations of unanticipated changed circumstances Under RESTATEMENT (SECOND) OF CONTRACTS sect 89 followed in many jurisdictions a promise modifying a duty under a contract not fully performed on either side is binding even if not supported by consideration if the modification is fair and equitable in view of circumstances not anticipated by the parties when the contract was made

If a court applies the rule in Restatement sect 89 the critical issues will be whether the modification was in fact ldquofair and equitablerdquo and whether it can be justified in light of unanticipated circumstances In many cases in which modifications have been upheld a party encountered difficulties or burdens in performing far beyond what was knowingly bargained for in the original contract with the result bordering on impracticability such as having to excavate solid rock instead of soft dirt or having to remove garbage far in excess of the amounts contemplated The conservatory would argue that the businessrsquos performance difficulties were not of this sort at allmdashnothing about repairing the pipe organ itself was any different from or more difficult than originally contemplated except that the business itself encountered financial distress unrelated to its burdens in performing its obligations under these contracts

Even if the business satisfies that element of the rule in Restatement sect 89 the business must also demonstrate that the circumstances that gave rise to the need to modify the contract were ldquounanticipatedrdquo at the time the original contract was made Here the facts suggest that when the business entered into the original contract it expected that the price paid by the conservatory would enable it to perform However any evidence that the business knew or had reason to know at the time of execution that it would need more money from the conservatory to be able to perform would mean that the request to modify was not ldquounanticipatedrdquo

[NOTE Some cases such as Schwartzreich v Bauman-Basch Inc 231 NY 196 131 NE 887 (1921) find that if the parties mutually agreed to rescind the original contract and then after rescission entered into an entirely new contract for a higher price the new contract is supported by consideration There is no evidence that such a rescission followed by a new contract took place here]

Point Two (45) The business can recover the additional $40000 for the new organ because no consideration is required under Article 2 of the UCC for good-faith contract modifications

The contract to buy a new organ is a contract for the sale of goods and therefore is governed by Article 2 of the Uniform Commercial Code UCC sect 2-102 Under Article 2 unlike the common law an agreement modifying a contract needs no consideration to be binding UCC sect 2-209(1) Section 2-209(1) thus obviates the preexisting duty rule entirely in contracts for the sale of goods

Even though consideration is not required modifications governed by sect 2-209 must satisfy the obligation of good faith imposed by the UCC UCC sect 1-304 See also Official Comment 2 to UCC sect 2-209 Good faith means ldquohonesty in fact and the observance of reasonable commercial standards of fair dealingrdquo UCC sect 1-201(b)(20) In this context the obligation of good faith means that ldquo[t]he effective use of bad faith to escape performance on the original contract terms is barred and the extortion of a lsquomodificationrsquo without legitimate commercial reason is ineffective as a violation of the duty of good faithrdquo Official Comment 2 to

17

Contracts Analysis

UCC sect 2-209 Here because the businessrsquos financial reversals were serious and apparently unanticipated at the time that the business entered into the contract with the conservatory and commitment of the extra money was needed to enable the business to perform a court would likely find that the business acted in good faith Thus a court would likely uphold the enforceability of the conservatoryrsquos promise to pay the additional $40000

Point Three (10) The conservatory is unlikely to be able to defend against enforcement of its promises to pay additional money under the theory of economic duress because the business probably did not make an improper threat

Under the common law of contracts parties may raise the defense of duress This common law defense also applies to contracts governed by UCC Article 2 See UCC sect 1-103(b)

A contract is voidable on the ground of economic duress by threat when it is established that a partyrsquos manifestation of assent is induced by an improper threat that leaves the party no reasonable alternative See RESTATEMENT (SECOND) OF CONTRACTS sect 175 See also eg Austin Instrument Inc v Loral Corp 272 NE2d 533 (NY 1971) (a threat to withhold essential goods can constitute duress) In order to void its agreement to pay the additional sum because of economic duress the conservatory must demonstrate that (1) the business made a threat to the conservatory (2) the threat was ldquoimproperrdquo or ldquowrongfulrdquo (3) the threat induced the conservatoryrsquos manifestation of assent to the modification and (4) the threat was sufficiently grave to justify the conservatoryrsquos assent

Here it appears that three of the four elements are likely satisfied The business plainly made a threat Moreover the threat induced the conservatoryrsquos assent to the modification and the threat was sufficiently grave to justify that assent If the conservatory had not agreed to pay the business the extra amounts the conservatory would have lost its entire $325000 investment In light of this potential loss a court could easily conclude that the conservatory had no reasonable alternative

However the business has a strong argument that its threat (indicating that it would breach the contracts unless the prices were increased) was not wrongful or improper but was instead nothing more than a communication of the reality of its own perilous situation to the conservatory

A mere threat to breach a contract is not in and of itself improper so as to support an action of economic duress or business compulsion Something more is required such as a breach of the duty of good faith and fair dealing as was present in Austin Instrument Inc supra Because the business could not perform the original contract without the requested modification the economic duress claim for the conservatory would likely fail for much the same reason that the business would be able to enforce the modification At the time the modification was requested the business was not trying to extort a price increase because of the conservatoryrsquos vulnerability but instead was simply stating the reality that the business could not perform without more money

18

FAMILY LAW ANALYSIS (Family Law IIIB D amp G)

ANALYSIS

Legal Problems

(1)(a) Does the State A court have jurisdiction to modify the State B child support order

(1)(b) Does the State A court have jurisdiction to modify the marital-residence-saleshyproceeds provision of the State B property-division decree

(2)(a) May a child support order be modified retroactively

(2)(b) May a child support order be modified prospectively based on a change of employment with a lower salary

(2)(c) May a property-division order be modified after entry of a divorce decree

DISCUSSION

Summary

The State A court may exercise personal jurisdiction over the wife because she was personally served in State A However subject-matter jurisdiction over the interstate modification of child support is governed by the Uniform Interstate Family Support Act (UIFSA) Under UIFSA State A does not have jurisdiction to modify the order for the daughterrsquos support because the wife is still a resident of State B UIFSA on the other hand does not govern property distributions and thus a State A court is not precluded from hearing the husbandrsquos petition to modify the marital-residence-sale-proceeds provision of the divorce decree

A child support order may not be modified retroactively A child support order may be modified prospectively based on a substantial change in circumstances Courts agree that a significant decrease in income is a substantial change in circumstances All states treat voluntary income reductions differently than involuntary reductions but employ different approaches for evaluating the impact of a voluntary reduction Whether the husband could obtain prospective modification of the child support order depends on which approach is applied

A property-division order is not subject to post-divorce modification based on a change in circumstances Thus the husband may in some states obtain prospective modification of the order for the daughterrsquos support but he may not obtain modification of the marital-residenceshysale-proceeds provision

Point One(a) (25) Personal jurisdiction over a nonresident respondent does not confer subject-matter jurisdiction over child support modification Under UIFSA a State A court may not modify a child support order issued by a State B court when as here the child or either parent continues to reside in State B the jurisdiction that issued the child support order

The State A court may exercise personal jurisdiction over the wife The wife was personally served in State A and a state may exercise jurisdiction based on in-state personal service See

19

Family Law Analysis

Burnham v Superior Court 495 US 604 (1990) But personal jurisdiction over the wife is not enough to give a State A court jurisdiction to modify the State B support order

The interstate enforcement and modification of child support is governed by the Uniform Interstate Family Support Act (UIFSA) which has been adopted by all states Under UIFSA the state that originally issued a child support order (here State B) has continuing exclusive jurisdiction to modify the order if that state remains the residence of the obligee the child or the obligor and all parties do not consent to the jurisdiction of another forum See UIFSA sect 205 See also UIFSA sect 603 (ldquoA tribunal of this State shall recognize and enforce but may not modify a registered order if the issuing tribunal had jurisdictionrdquo) The wife and daughter continue to reside in State B and the wife has not consented to the jurisdiction of another forum Thus a State A court does not have jurisdiction to modify the State B child support order

[NOTE Examinees who do not discuss personal jurisdiction but fully discuss UIFSA may receive full credit]

Point One(b) (15) UIFSA does not apply to disputes over property division Thus the State A court may exercise jurisdiction over the husbandrsquos petition to modify the marital-residence-sale-proceeds provision of the State B divorce decree because it has personal jurisdiction over the wife

The State A court in which the husband brought his action has jurisdiction to adjudicate domestic relations issues The husbandrsquos petition to modify the property settlement is a domestic relations issue The courts of State A may exercise personal jurisdiction over the wife because she was personally served in State A See Burnham v Superior Court 495 US 604 (1990) see Point One(a)

UIFSA does not apply to divorce property-division disputes Thus although a State A court may not adjudicate the husbandrsquos petition to modify his child support obligations it may adjudicate his property-division claims (Even though the court has jurisdiction it may not modify the property-division award on the merits See Point Two(c))

Point Two(a) (20) A child support order may not be modified retroactively

State courts have long held that obligations to pay child support ordinarily may not be modified retroactively ldquoIf the hardship is particularly severe the courts sometimes devised a way to protect the obligor but in most instances the courts hold that retroactive modification of this kind is beyond their power and indeed the governing statute may so providerdquo HOMER H CLARK THE LAW OF DOMESTIC RELATIONSHIPS IN THE UNITED STATES 725 (2d ed 1987)

Federal law now goes further and requires the states as a condition of federal child-support funding to adopt rules that absolutely forbid retroactive modification of the support obligation See 42 USC sect 666(a)(9)(C) The states have adopted rules consistent with the federal requirements

Point Two(b) (25) It is unclear whether the husband could obtain prospective downward modification of his child support based on his voluntary acceptance of a job with a lower salary

Prospective modification of a child support order is typically available only when the petitioner can show a substantial change in circumstances See ROBERT E OLIPHANT amp NANCY VER

20

Family Law Analysis

STEEGH FAMILY LAW 213ndash15 (3d ed 2010) A significant decrease in income is typically viewed as a substantial change

However when a parent seeks to modify a child support obligation because he has voluntarily reduced his income a court will not modify the obligation based solely on the income loss Some courts refuse to modify whenever the income shift was voluntary See eg Aguiar v Aguiar 127 P3d 234 (Idaho Ct App 2005) Others look primarily to the petitionerrsquos intentions and permit downward modification if he has acted in good faith See eg In re Marriage of Horn 650 NE2d 1103 (Ill App Ct 1995) Many courts use a multifactor approach See OLIPHANT amp VER STEEGH supra 217ndash18

Here there is no question that the husbandrsquos loss of income was voluntary In a jurisdiction in which voluntary income reduction bars support modification the husbandrsquos petition would be denied

In a jurisdiction employing a good-faith or multifactor approach it is possible but not certain that the husband could obtain downward modification The evidence supports the husbandrsquos good faith his change in employment appears to be based on his new jobrsquos greater responsibilities and better promotion possibilities In a jurisdiction using a multifactor approach the court would likely also consider the impact of such a shift on the daughter the likely duration of the husbandrsquos income loss and the likelihood of a promotion that would ultimately inure to the daughterrsquos benefit Thus on these facts it is possible but by no means certain that the husband could prospectively obtain downward modification of his child support obligation to his daughter

Point Two(c)(15) A divorce property-division award is not subject to modification

A support order is aimed at meeting the post-divorce needs of the supported individual Because the future is unpredictable courts are empowered to modify a support award to take account of changed circumstances that may occur during the period in which support is paid

By contrast a property-distribution award divides assets of the marriage based on the equities at the time of divorce Because the past can be ascertained a property-division award is not subject to post-divorce modification See HARRY A KRAUSE ET AL FAMILY LAW CASES COMMENTS AND QUESTIONS 691 (6th ed 2007)

Here the husband is seeking modification of a property-division award with respect to an asset owned by the parties at the time of divorce Thus the husband may not obtain a modification of the marital-residence-sale-proceeds provision of the divorce decree based on his reduced income

21

FEDERAL CIVIL PROCEDURE ANALYSIS (Federal Civil Procedure III IVC)

ANALYSIS

Legal Problems

(1) Is the logging company entitled to join this action as a matter of right

(2)(a) May the nonprofit organization obtain a temporary restraining order to stop the USFS from issuing a logging permit

(2)(b) May the nonprofit organization obtain a preliminary injunction to stop the USFS from issuing a logging permit during the pendency of the action

DISCUSSION

Summary

The logging company is entitled to intervene in this action as a matter of right because it has an interest in the property or transaction that is the subject of the action and is so situated that its interest may be impaired or impeded as a practical matter if the action goes forward without it The logging companyrsquos interest is not adequately represented by the USFSrsquos presence in the lawsuit

The nonprofit organization may seek a temporary restraining order (TRO) followed by a preliminary injunction to prevent the USFS from issuing a logging permit pending the outcome of the action The nonprofit is likely to obtain a TRO if it can demonstrate a risk of immediate and irreparable injury The nonprofit is also likely to obtain a preliminary injunction if it can demonstrate a significant threat of irreparable harm and a likelihood of success on the merits of its National Environmental Policy Act (NEPA) claim

Point One (50) Rule 24(a) of the Federal Rules of Civil Procedure requires federal courts to allow a person to intervene in an action as a matter of right if the person a) is interested in the property or transaction that is the subject of the action b) is so situated that its interest may be impaired or impeded if the litigation goes forward without it and c) is not adequately represented by existing parties Here the logging company likely meets all three requirements and should be allowed to intervene as a matter of right

Rule 24 of the Federal Rules of Civil Procedure governs intervention the process by which a non-party to an action may join the litigation Under Rule 24(a) (intervention of right) a person must be permitted to intervene if three conditions are met (1) the movant ldquoclaims an interest relating to the property or transaction that is the subject of the actionrdquo (2) the movant ldquois so situated that disposition of the action may as a practical matter impair or impede the movantrsquos ability to protect its interestrdquo and (3) ldquoexisting partiesrdquo do not ldquoadequately represent [the movantrsquos] interestrdquo FED R CIV P 24(a) The three requirements for intervention of right are often ldquovery interrelatedrdquo 7C CHARLES ALAN WRIGHT ET AL FEDERAL PRACTICE AND PROCEDURE sect 1908 at 297 (2007 amp 2011 Supp)

22

Federal Civil Procedure Analysis

Here the court should find that the logging company meets this test First the logging company has a strong interest in the property or transaction that is the subject of this action The USFS has accepted the logging companyrsquos bid and the logging company is merely awaiting issuance of a logging permit to begin logging The nonprofit organization is seeking to prevent this logging The logging company therefore has a strong direct and substantial interest in the subject matter of the lawsuit and in having its winning bid honored and a logging permit issued See eg Kleissler v US Forest Serv 157 F3d 964 972 (3d Cir 1998) (stating that ldquo[t]imber companies have direct and substantial interests in a lawsuit aimed at halting loggingrdquo) see also Natural Resources Defense Council v US Nuclear Regulatory Commrsquon 578 F2d 1341 1343ndash 44 (10th Cir 1978) (holding that applicants whose license renewals were pending had Rule 24(a)(2) interests where the lawsuit sought to halt the license-issuing process pending preparation of environmental impact statements) See generally 7C WRIGHT ET AL supra sect 19081 at 309 (ldquoIf there is a direct substantial legally protectable interest in the proceedings it is clear that this requirement of the rule is satisfiedrdquo) Second the logging companyrsquos interest in receiving a logging permit may well be impaired as a practical matter by the outcome of the lawsuit If the USFS loses the lawsuit it will have to prepare an environmental impact statement before issuing the logging companyrsquos permit This will at a minimum delay the logging companyrsquos ability to exercise its rights and may in the long r un mean that no logging permit is ever issued Intervention of right is not limited to those that would be legally bound as a matter of preclusion doctrine Id sect 19082 at 368 Rather ldquo[t]he rule is satisfied whenever disposition of the present action would put the movant at a practical disadvantage in protecting its interestrdquo Id sect 19082 at 369 Here that condition is easily satisfied See Kleissler 157 F3d at 972 (ldquoTimber companies have direct and substantial interests in a lawsuit aimed at halting logging rdquo)

Given that the logging company has an interest that may be impaired by disposition of the action it should be allowed to intervene unless the court is persuaded that the USFS adequately represents the logging companyrsquos interest See Rule 24(a)(2) 7C WRIGHT ET AL supra sect 1909 Here it could be argued that the USFS adequately represents the logging companyrsquos interest because the USFS presumably wants the court to uphold its development plan and allow it to proceed with issuance of the logging permit which is the same relief that the logging company would seek However whether representation is truly adequate depends upon ldquo[a] discriminating appraisal of the circumstancesrdquo 7C WRIGHT ET AL supra sect 1909 at 440 Although both the government and the logging company wish to avoid the preparation of an environmental impact statement their interests are distinct The USFSrsquos interest is proper management of the national forest system while the logging companyrsquos interest is making a profit from logging the 5000-acre tract The USFSrsquos handling of the litigation is likely to be affected by a variety of policy concerns and political considerations that have nothing to do with the logging companyrsquos purely economic interest in securing the right to cut trees in the Scenic National Forest See eg Kleissler 157 F3d at 973ndash74 (ldquo[T]he government represents numerous complex and conflicting interests in matters of this nature The straightforward business interests asserted by intervenors here may become lost in the thicket of sometimes inconsistent governmental policiesrdquo)

[NOTES (1) Examinees who mistakenly analyze the logging companyrsquos case for joinder under the related but incorrect Rule 19 ldquoRequired Joinder of Partiesrdquo may receive credit Rule 19 allows existing parties to demand joinder of non-parties (or seek dismissal of the case if they canrsquot get it) There is a close relationship between Rule 24 and Rule 19 and both contain a similar standard for determining when ldquointerestedrdquo third parties are ldquoentitledrdquo or ldquorequiredrdquo to be in the lawsuit Indeed the two prongs of the Rule 24 intervention test that are discussed above

23

Federal Civil Procedure Analysis

are nearly identical to the two prongs of the Rule 19(a) required joinder test Examinees who discuss and apply the test should receive credit even if they cite Rule 19 rather than Rule 24

(2) Examinees may discuss permissive joinder Although permissive joinder is a possibility here the question asks only whether the logging company can join the action as a matter of right and a permissive joinder analysis is not responsive to the question To the extent an examinee discusses permissive joinder the analysis will focus on whether the logging company ldquohas a claim or defense that shares with the main action a common question of law or factrdquo FED R CIV P 24(b)(1)(B) The district court also ldquomust consider whether the intervention will unduly delay or prejudice the adjudication of the original partiesrsquo rightsrdquo FED R CIV P 24(b)(3) On our facts the logging companyrsquos claim for the issuance of a logging permit would certainly share common questions of law and fact with the USFSrsquos defense against the nonprofitrsquos claim There are no facts suggesting that the logging companyrsquos presence would unduly delay or otherwise prejudice adjudication of the original action Thus the district court would have discretion to permit the logging company to intervene even if it denied intervention of right]

Point Two(a) (25) The nonprofit organization could seek and would likely obtain a temporary restraining order to stop the USFS from issuing a logging permit pending a hearing on an application for a preliminary injunction

The first type of interim relief the nonprofit could seek to stop the USFS from issuing a logging permit to the logging company is a temporary restraining order (TRO) prohibiting the USFS from issuing the logging permit A TRO can be issued without notice to the adverse party but only in limited circumstances and only for a limited time FED R CIV P 65(b) To secure a TRO without notice the nonprofit would need to submit an affidavit containing specific facts that demonstrate a risk of ldquoimmediate and irreparable injuryrdquo if a permit is issued FED R CIV P 65(b)(1) In deciding whether to grant a TRO courts will also consider the same factors that are relevant in deciding whether to grant a preliminary injunction (eg the moving partyrsquos likelihood of success on the merits the balance of hardships and the public interest) See Point Two(b) infra The TRO would last only long enough for the court to consider and resolve a request by the nonprofit for a preliminary injunction but no longer than 14 days (unless the court extends it for good cause or the adverse party consents to an extension) In addition bond is required

Here the court is likely to grant the nonprofitrsquos request The nonprofit could plausibly claim that cutting down 5000 acres of old-growth forest in an area that is home to the highest concentration of wildlife in the western United States would have ldquoan immediate and irreparablerdquo adverse impact on the environment and cause irreparable harm to the nonprofitrsquos interest in preserving and protecting natural resources including wildlife habitat

Point Two(b) (25) The nonprofit could also seek and would likely obtain a preliminary injunction to stop the USFS which is likely to be granted if the nonprofitrsquos claim that the USFS violated NEPA has a strong basis in fact and law

Because the TRO would be temporary the nonprofit would need to move for a preliminary injunction to prevent the USFS from issuing a logging permit throughout the pendency of the litigation Preliminary injunctions are injunctions that seek to ldquoprotect [the] plaintiff from

24

Federal Civil Procedure Analysis

irreparable injury and to preserve the courtrsquos power to render a meaningful decision after a trial on the meritsrdquo 11A CHARLES ALAN WRIGHT ET AL FEDERAL PRACTICE AND PROCEDURE sect 2947 at 112 (2013) Rule 65 of the Federal Rules of Civil Procedure sets out the procedural requirements for preliminary injunctions Preliminary injunctions may be granted only upon notice to the adverse party FED R CIV P 65(a)(1) and only if the movant ldquogives security in an amount that the court considers proper to pay the costs and damages sustained by any party found to have been wrongfully enjoined or restrainedrdquo FED R CIV P 65(c)

While Rule 65 sets out the procedural requirements for preliminary injunctive relief it does not specify the substantive grounds upon which it may be granted The courtrsquos discretion in ruling upon a motion for a preliminary injunction ldquois exercised in conformity with historic federal equity practicerdquo 11A WRIGHT ET AL supra sect 2947 at 114 The court typically considers four factors

(1) the significance of the threat of irreparable harm to the plaintiff if the injunction is not granted (2) the balance between this harm and the injury that granting the injunction would inflict on the defendant (3) the probability that the plaintiff will succeed on the merits and (4) the public interest

Id sect 2948 at 122ndash24 accord Habitat Educ Center v Bosworth 363 F Supp 2d 1070 1088 (ED Wis 2005) The most important of these factors is the risk of irreparable harm to the plaintiff 11A WRIGHT ET AL supra sect 29481 at 129 If the plaintiff has an adequate remedy at law (eg if money damages can compensate the plaintiff for its loss) then a preliminary injunction will be denied Id sect 29481

Here a court would likely conclude that the potential for environmental damage to the forest creates a significant threat of irreparable harm ldquo[E]nvironmental injury is often irreparable Courts have recognized that logging such as would occur [here] can have longshyterm environmental consequences and thus satisfy the irreparable injury criterionrdquo Habitat Educ Center 363 F Supp 2d at 1089 (citing Idaho Sporting Congress Inc v Alexander 222 F3d 562 569 (9th Cir 2000) (noting that the imminent and continuing logging activities presented ldquoevidence of environmental harm sufficient to tip the balance in favor of injunctive reliefrdquo)) Neighbors of Cuddy Mountain v US Forest Service 137 F3d 1372 1382 (9th Cir 1998) (stating that ldquo[t]he old growth forests plaintiffs seek to protect would if cut take hundreds of years to reproducerdquo) (internal citation omitted)) see also 11C WRIGHT ET AL supra sect 29481 at 151 (noting that ldquoa preliminary injunction has been issued to prevent harm to the environmentrdquo)

The second factor the balance between the harm to the plaintiff and the harm the defendant will suffer if the injunction is issued also appears to support issuance of a preliminary injunction here The USFS will have to wait before it can develop the Scenic National Forest and the logging company may lose money if the delay is prolonged These economic harms could be compensated monetarily if an injunction is issued inappropriately Where ldquoan injunction bond can compensate [the] defendant for any harm the injunction is likely to inflict the balance should be struck in favor of [the] plaintiffrdquo Id sect 29482 at 192 See also Habitat Educ Center 363 F Supp 2d at 1089 (stating that ldquothe relative absence of harmful effects on the Forest Service weighs in favor of granting the injunctionrdquo)

The third factor is the likelihood that the plaintiff will prevail on the merits Although there is limited information concerning the merits of the action the nonprofit alleges that the federal statute (NEPA) requires an environmental impact statement and further states that the USFS created no environmental impact analysis or statement at all Assuming that those

25

Federal Civil Procedure Analysis

allegations are correct it seems plausible to conclude that the nonprofit will be able to show a likelihood of success on the merits

Finally courts deciding whether or not to issue preliminary injunctive relief are to consider the public interest ldquoFocusing on this factor is another way of inquiring whether there are policy considerations that bear on whether the order should issuerdquo 11C WRIGHT ET AL supra sect 29484 at 214 If the court concludes that the nonprofit is likely to succeed on its NEPA claim because the USFS wrongfully failed to conduct an environmental impact assessment it is likely to find that the public interest would be served by restraining the USFS from proceeding with logging in a national forest See Heartwood Inc v US Forest Service 73 F Supp 2d 962 979 (SD Ill 1999) affrsquod on other grounds 230 F3d 947 (7th Cir 2000) (ldquoviolations by federal agencies of NEPArsquos provisions as established by Congress harm the public as well as the environmentrdquo)

Thus a court is very likely to grant a preliminary injunction if it concludes that the nonprofit has a significant likelihood of success on the merits

26

EVIDENCE ANALYSIS (Evidence ID IIA amp C)

ANALYSIS

Legal Problems

(1) Under what circumstances can evidence of prior convictions be used to impeach a witnessrsquos credibility in a civil case

(1)(a) May the inmatersquos credibility be impeached by evidence of a 12-year-old felony drug conviction if he was released from prison 9 years ago

(1)(b) May the inmatersquos credibility be impeached by evidence of an 8-year-old misdemeanor perjury conviction that was punishable by 1 year in jail if he pleaded guilty and was sentenced only to pay a $5000 fine

(1)(c) May the inmatersquos credibility be impeached by evidence of a 7-year-old sexual assault conviction if the inmate is still serving a 10-year prison sentence and the victim was his 13-year-old daughter

(2)(a) May the guardrsquos credibility be impeached by cross-examination regarding specific instances of misconduct (ie lying on his reacutesumeacute) relevant to credibility

(2)(b) May the guardrsquos credibility be impeached by admission of extrinsic evidence (his reacutesumeacute and academic transcript) offered to prove specific instances of misconduct relevant to credibility

DISCUSSION

Summary

Under the Federal Rules of Evidence witnesses can be impeached with evidence of prior convictions andor specific instances of misconduct Whether evidence of prior convictions should be admitted to impeach generally depends on the nature of the crime the amount of time that has passed and (only in criminal cases) whether the ldquowitnessrdquo is the defendant FED R EVID 609(a)

In this civil case evidence of the inmatersquos conviction for distribution of marijuana should be admitted to impeach the inmate because he was convicted of a felony and was released from prison fewer than 10 years ago FED R EVID 609(a)(1) Credibility is critically important in this case because the jury will hear conflicting testimony from the two disputing parties and there were no other eyewitnesses to the altercation Under Rule 609(a)(1) the inmatersquos conviction should be admitted because it has some bearing on his credibility and its probative value is not substantially outweighed by concerns of unfair prejudice confusion or delay Id

Evidence of the inmatersquos misdemeanor conviction for perjury must be admitted because the crime ldquorequired provingmdashor the witnessrsquos admittingmdasha dishonest act or false statementrdquo by the inmate FED R EVID 609(a)(2)

27

Evidence Analysis

Evidence of the inmatersquos felony conviction for sexual assault should be excluded because its probative value is substantially outweighed by the danger of unfair prejudice to the inmate based on the heinous nature of the crime FED R EVID 609(a)(1) In the alternative the judge could limit the evidence relating to this conviction by excluding details of the inmatersquos crime

In all civil (and criminal) cases witnesses can also be impeached with evidence of specific instances of prior misconduct that did not result in a conviction FED R EVID 608(b) Pursuant to Rule 608(b) misconduct probative of untruthfulness can be inquired into on cross-examination but cannot be proved through extrinsic evidence Id Thus the inmatersquos counsel should be permitted to cross-examine the guard regarding the false statement in the guardrsquos reacutesumeacute However extrinsic evidence of the guardrsquos misconduct (ie the guardrsquos authenticated reacutesumeacute and transcript from the local college) should not be admitted even if the guard denies wrongdoing or refuses to answer cross-examination questions about these matters Id

Point One (10) The Federal Rules of Evidence permit impeachment of witnesses with evidence of prior convictions

Whether convictions should be admitted to impeach generally depends on the nature of the crime the amount of time that has passed and (only in criminal cases) whether the ldquowitnessrdquo is the defendant FED R EVID 609(a) Under Rule 609(a) evidence of prior convictions may be admitted for the purpose of ldquoattacking a witnessrsquos character for truthfulnessrdquo Id

There are two basic types of convictions that can be admitted for the purpose of impeachment

(1) convictions for crimes ldquopunishable by death or by imprisonment for more than one yearrdquo (which generally correlates to ldquofeloniesrdquo) FED R EVID 609(a)(1) and (2) convictions ldquofor any crimes regardless of the punishment if the court can readily determine that establishing the elements of the crime required provingmdashor the witnessrsquos admittingmdasha dishonest act or false statementrdquo FED R EVID 609(a)(2)

Pursuant to Rule 609(a)(1) in civil cases the admission of evidence of a felony conviction is ldquosubject to Rule 403 [which says that a court may exclude relevant evidence if its probative value is substantially outweighed by other factors]rdquo FED R EVID 609(a)(1) However Rule 403 does not protect the witness against admission of prior convictions involving dishonestymdashwhich must be admitted by the court FED R EVID 609(a)(2)

Finally Federal Rule of Evidence 609(b) contains the presumption that a conviction that is more than 10 years old or where more than 10 years has passed since the witnessrsquos release from confinement (whichever is later) should not be admitted unless ldquoits probative value supported by specific facts and circumstances substantially outweighs its prejudicial effectrdquo and the proponent has provided the adverse party with reasonable written notice FED R EVID 609(b)

Point One(a) (25) The court should admit evidence of the inmatersquos 12-year-old felony marijuana distribution conviction

The inmatersquos conviction for marijuana distribution was for a felony punishable by imprisonment for more than one year See FED R EVID 609(a)(1) Moreover although the conviction was 12 years ago the 10-year time limit of Rule 609(b) is not exceeded because that time limit runs

28

Evidence Analysis

from the date of either ldquothe witnessrsquos conviction or release from confinement for it whichever is laterrdquo FED R EVID 609(b) Because the inmate served three years in prison he was released from confinement nine years ago

However pursuant to Rule 609(a)(1) the admission of felony convictions to impeach a witness in a civil case is ldquosubject to Rule 403rdquo FED R EVID 609(a)(1) Neither Rule 609(a) nor the advisory committee notes specify which factors courts should consider when balancing the probative value of a conviction against the dangers identified in Rule 403 (which include (1) unfair prejudice (2) confusion of the issues (3) misleading the jury (4) waste of time or undue delay and (5) needless presentation of cumulative evidence) FED R EVID 403

In this case credibility is very important because the evidence consists primarily of the testimony of the disputing parties and there were no other eyewitnesses to the altercation This enhances the probative value of any evidence bearing on the inmatersquos credibility A court is likely to conclude that the inmatersquos prior felony drug conviction is relevant to his credibility See eg United States v Brito 427 F3d 53 64 (1st Cir 2005) (ldquoPrior drug-trafficking crimes are generally viewed as having some bearing on veracityrdquo) Although the probative value of any conviction diminishes with age see eg United States v Brewer 451 F Supp 50 53 (ED Tenn 1978) the inmatersquos ongoing problems with the law suggest that he has continued (and even escalated) his criminal behavior over the past nine years The court should admit this evidence because its probative value is not substantially outweighed by any Rule 403 concerns Specifically any prejudice to the inmate would be slight because the conviction is unrelated to the altercation at issue and the conviction was not for a heinous crime that might inflame the jury

[NOTE Whether an examinee identifies the jury instruction as containing a ldquoconclusiverdquo or ldquomandatoryrdquo presumption is less important than the examineersquos analysis of the constitutional infirmities]

Point One(b) (15) The court must admit evidence of the inmatersquos eight-year-old misdemeanor conviction because perjury is a crime of dishonesty

Rule 609(a)(2) provides that evidence of a criminal conviction ldquomust be admitted if the court can readily determine that establishing the elements of the crime required provingmdashor the witnessrsquos admittingmdasha dishonest act or false statementrdquo FED R EVID 609(a)(2) The inmatersquos conviction for perjury would have necessarily required proving that the inmate engaged in an act of dishonesty This conviction occurred within the past 10 years so it ldquomust be admittedrdquo because in contrast to Rule 609(a)(1) (discussed in Point One(a)) admission under Rule 609(a)(2) is mandatory and not subject to Rule 403

Point One(c) (20) The court should exclude evidence of the inmatersquos seven-year-old felony sexual assault conviction because the probative value of this evidence is substantially outweighed by the danger of unfair prejudice In the alternative the details of the prior conviction could be excluded

The inmatersquos conviction for felony sexual assault was seven years ago and he has not yet been released from incarceration so Rule 609(a) but not 609(b) is applicable here FED R EVID 609(a) This conviction is therefore admissible to impeach the inmate unless its probative value is substantially outweighed by the danger of unfair prejudice or any other Rule 403 concern Id

29

Evidence Analysis

Sex crimes are generally not considered relevant to credibility see Hopkins v State 639 So 2d 1247 1254 (Miss 1993) so the probative value of this conviction is relatively low Moreover the heinous nature of the inmatersquos crime (sexual assault on his daughter) makes the danger of unfair prejudice to the inmate very high Thus the court should exclude evidence of the conviction because it was for a heinous offense that is likely to inflame the jury and it has little bearing on credibility See eg United States v Beahm 664 F2d 414 419 (4th Cir 1981)

As an alternative to excluding this evidence the judge could minimize the unfair prejudice to the inmate by permitting limited cross-examination but refusing to allow specific questions about the nature of the inmatersquos conviction For example a court could limit cross-examination to the fact that the inmate was convicted of a ldquofelonyrdquo or perhaps that he was convicted of a ldquosexual assaultrdquo without identifying the victim However because evidence of the inmatersquos prior convictions can be admitted solely for the purpose of enabling the jury to assess his credibility and because his two earlier convictions should have already been admitted the court should exclude all evidence of the felony sexual assault conviction

Point Two(a) (15) The court should permit the inmatersquos counsel to cross-examine the guard regarding the false statement in his reacutesumeacute because the guardrsquos misconduct bears on his truthfulness

The inmate wishes to cross-examine the guard about his prior dishonest behaviormdashlying on his reacutesumeacutemdashthat did not involve a criminal conviction Rule 608(b) allows witnesses to be cross-examined about specific instances of prior non-conviction misconduct probative of untruthfulness ldquoin order to attack the witnessrsquos character for truthfulnessrdquo FED R EVID 608(b)

The courtrsquos decision to allow cross-examination about the guardrsquos prior dishonest behavior depends on the probative value of such evidence balanced against the danger of unfair prejudice to the guard or any other Rule 403 concern FED R EVID 403 Here the guardrsquos false statement on his reacutesumeacute that he obtained a degree in Criminal Justice is highly probative of his untruthfulness because it grossly misrepresents his actual academic record was made recently and was made with the intent to deceive Because the probative value of this evidence is very strong and is not substantially outweighed by any Rule 403 concerns cross-examination of the guard on this topic should be permitted The court may also consider it fair to permit this cross-examination of the guard on these matters assuming that one or more of the inmatersquos prior convictions have been admitted to impeach his credibility

Point Two(b) (15) The court should exclude extrinsic evidence of the guardrsquos non-conviction misconduct even if the guard denies wrongdoing or refuses to answer questions about the matter

Although Rule 608(b) allows cross-examination about specific instances of prior misconduct probative of untruthfulness ldquoextrinsic evidencerdquo offered to prove such misconduct is not admissible FED R EVID 608(b) The rationale for this rule is that allowing the introduction of extrinsic evidence of prior misconduct by witnesses when these acts are relevant only to the witnessesrsquo truthfulness and not to the main issues in the case would create too great a risk of confusing the jury and unduly delaying the trial The court does not have discretion to admit this extrinsic evidence See eg United States v Elliot 89 F3d 1360 1368 (8th Cir 1996)

30

Evidence Analysis

Here the inmatersquos counsel may cross-examine the guard about the false statement on his reacutesumeacute However the inmatersquos counsel must accept the guardrsquos response Even if the guard denies wrongdoing or refuses to answer questions about the matter the inmatersquos counsel cannot introduce the guardrsquos reacutesumeacute or the transcript from the local college to prove the guardrsquos misconduct

31

CORPORATIONS ANALYSIS (Corporations VA2 IX)

ANALYSIS

Legal Problems

(1) Do shareholders have the authority to amend a corporationrsquos bylaws with respect to director nominations

(2) Do board-approved bylaws on a particular subject here nomination of directors preempt subsequent conflicting bylaw amendments by shareholders

(3) Is a suit challenging both managementrsquos refusal to include the proposed bylaw amendment in Megarsquos proxy statement and the boardrsquos amendment of the bylaws dealing with nomination of directors a direct or derivative suit

DISCUSSION

Summary

The voting and litigation rights of the shareholders of Mega are subject to the provisions of the Model Business Corporations Act (MBCA)

The investorrsquos proposed bylaw provision is not inconsistent with state law Under the MBCA shareholders may amend the bylaws when the amendment deals with a proper matter for the corporationrsquos bylaws such as procedures for nominating directors

The Mega boardrsquos bylaw amendment does not preempt the investorrsquos proposed bylaw provision or the Mega shareholdersrsquo power to approve it While shareholders can limit the boardrsquos power to amend or repeal the bylaws the board cannot limit the shareholdersrsquo power

Whether the investor must make a demand on Megarsquos board depends on how the investor frames its claim If the investor claims a violation of shareholder voting rights the claim is direct and pre-suit demand on the board is not required If on the other hand the investor claims that the directors violated their fiduciary duties by amending the bylaws to entrench themselves the claim is derivative and a pre-suit demand is required

Point One (30) Shareholders may amend the corporationrsquos bylaws where the proposed bylaw provision relates to procedural matters typically included in the bylaws such as the nomination of directors

Internal affairs of the corporation such as the conduct of shareholder meetings and election of directors are subject to the corporate law of the state of incorporation See McDermott Inc v Lewis 531 A2d 206 (Del 1987) (applying law of jurisdiction where corporation was incorporated in case involving voting rights) This statersquos corporate statute is modeled on the MBCA

Under the MBCA ldquoshareholders may amend the corporationrsquos bylawsrdquo MBCA sect 1020(a) Thus the only question is whether the bylaws can specify the procedures for shareholder nomination of directors

32

Corporations Analysis

The MBCA states that the bylaws ldquomay contain any provision that is not inconsistent with law or the articles of incorporationrdquo MBCA sect 206(b) In addition the MBCA was revised in 2009 to address shareholder nomination of directors in public corporations (known as ldquoproxy accessrdquo) and specifies that the bylaws ldquomay contain a requirement that the corporation include in its [proxy materials] one or more individuals nominated by a shareholderrdquo MBCA sect 206(c)(1) see Committee on Corporate Laws ABA Section of Business Law Report on the Roles of Boards of Directors and Shareholders of Publicly Owned Corporations and Changes to the Model Business Corporations ActmdashAdoption of Shareholder Proxy Access Amendments to Chapters 2 and 10 65 BUS LAWYER 1105 (2010)

The inclusion of director-nomination procedures in the bylaws is consistent with practice and is recognized by the Delaware courts whose views on corporate law carry significant weight Typically the procedures for nomination of directors are found in the bylaws See 1 COX amp HAZEN TREATISE ON THE LAW OF CORPORATIONS sect 312 (3d ed 2011) see also 4 FLETCHER CORP FORMS ANN PART III ch 21 (2013) (including sample bylaws that permit nomination of directors by shareholders) The Delaware Supreme Court has confirmed that the bylaws may ldquodefine the process and proceduresrdquo for director elections See CA Inc v AFSCME Employees Pension Plan 953 A2d 227 (Del 2008) (concluding that bylaw amendment requiring reimbursement of election expenses to certain successful shareholder nominators is ldquoproper subjectrdquo under Delaware law)

[NOTE The question of the proper scope of the bylaws can be answered using the more general MBCA sect 206(b) or the 2009 MBCA revision adding sect 206(c)(1) (adopted in CT ME VA) In addition some examinees might raise the point that shareholder proposals may not compel the board to take action such as by including shareholder nominations in the companyrsquos proxy materials on the theory that the ldquobusiness and affairsrdquo of the corporation are to be managed by the board See MBCA sect 801(b) Although shareholders are generally limited to adopting precatory resolutions that recommend or encourage board action this limitation does not apply when shareholders have specific authority to take binding action on their ownmdashsuch as to amend the bylaws]

Point Two (30) Shareholders can amend (or repeal) board-approved bylaws Further shareholders can limit the boardrsquos power to later amend and repeal a shareholder-approved bylaw

Under the MBCA shareholders have the power to amend the bylaws See Point One The board shares this power with the shareholders unless (1) the corporationrsquos articles ldquoreserve that power exclusively to the shareholdersrdquo or (2) ldquothe shareholders in amending repealing or adopting a bylaw expressly provide that the board of directors may not amend repeal or reinstate that bylawrdquo See MBCA sect 1020(b)

Shareholder-approved bylaw provisions can amend or repeal existing bylaw provisions whether originally approved by the board or by shareholders See ALAN R PALMITER CORPORATIONS EXAMPLES AND EXPLANATIONS sect 713 (7th ed 2012) Thus the Mega boardrsquos bylaw amendmentmdashwhich set more demanding thresholds for shareholder nomination of directors than the investorrsquos proposed bylaw provisionmdashwould be superseded (repealed) if Megarsquos shareholders were to approve the investorrsquos proposal

Further a shareholder-approved bylaw generally can limit the power of the board to later amend or repeal it See MBCA sect 1020(b)(2) Thus if Megarsquos shareholders approved the bylaw

33

Corporations Analysis

provision proposed by the investor Megarsquos board could not repeal the provision because it includes a ldquono board repealrdquo clause

The revision to the MBCA in 2009 dealing with shareholder proxy access does not change this conclusion That revision specifies that a shareholder-approved bylaw dealing with director nominations may not limit the boardrsquos power to amend add or repeal ldquoany procedure or condition to such a bylaw in order to provide for a reasonable practicable and orderly processrdquo MBCA sect 206(d) Thus according to the revision if shareholders approve a bylaw amendment that limits further board changes the board would nonetheless retain the power to ldquotinkerrdquo with the bylaw to safeguard the voting process but could not repeal the shareholder-approved bylaw The Official Comment to MBCA sect 206(d) makes clear that the revision is ldquonot intended to allow the board of directors to frustrate the purpose of the shareholder-adopted proxy access provisionrdquo Thus if Megarsquos shareholders were to approve the bylaw provision proposed by the investor Megarsquos board could only amend the provision regarding its procedures or conditions in a manner consistent with its purpose of permitting proxy access for Megarsquos shareholders

[NOTE The boardrsquos attempted interference with a shareholder voting initiative may also have been a violation of the directorsrsquo fiduciary duties See Blasius Indus Inc v Atlas Corp 564 A2d 651 (Del Ch 1988) (finding that directors breached their fiduciary duties by amending bylaws and expanding size of board to thwart insurgentrsquos plan to amend bylaws and seat a majority of new directors) The call however asks examinees to consider whether shareholders or the board have ldquoprecedencerdquo over amending the corporate bylaws Thus an examineersquos answer should be framed in terms of ldquopowerrdquo and not ldquodutyrdquo]

Point Three (40) The investor need not make a demand on the board if the investor states a direct claim such as an allegation that the board interfered with the investorrsquos right to amend the bylaws But the investor must make a demand on the board if the investor states a derivative claim (on behalf of the corporation) such as an allegation that the directors sought to entrench themselves by interfering with the proposed proxy access

The MBCA generally requires that shareholders make a demand on the board of directors before initiation of a derivative suit MBCA sect 742 (shareholder may not bring derivative proceeding until written demand has been made on corporation and 90 days have expired) A derivative suit is essentially two suits in one where the plaintiff-shareholder seeks to bring on behalf of the corporation a claim that vindicates corporate rights usually based on violation of fiduciary duties PALMITER supra sect 1811 (6th ed 2009) The demand permits the board to investigate the situation identified by the shareholder and take suitable action No demand on the board is required however if the shareholder brings a direct suit to vindicate the shareholderrsquos own rights not those of the corporation

Is the suit brought by the investor derivative or direct The MBCA defines a ldquoderivative proceedingrdquo as one brought ldquoin the right of a domestic corporationrdquo MBCA sect 740(1) Thus the answer to how the investorrsquos suit should be characterized turns on what rights the investor seeks to vindicate If the investor frames its claim as one of fiduciary breach by directorsmdashfor example for failing to become adequately informed about voting procedures or for seeking to entrench themselves in office by manipulating the voting structure to avoid a shareholder insurgencymdashthen the suit is ldquoderivativerdquo and the investor must make a demand on the board See MBCA Ch 7 Subch D Introductory Comment (ldquothe derivative suit has historically been the principal method of challenging allegedly illegal action by managementrdquo)

34

Corporations Analysis

If however the investor frames its claim as one to vindicate shareholder rights the suit is direct and no demand is required For many courts the direct-derivative question turns on who is injured and who is to receive the relief sought by the plaintiff-shareholders See Tooley v Donaldson Lufkin amp Jenrette Inc 845 A2d 1031 (Del 2004) (characterizing a merger-delay claim as direct because delay of merger only harmed shareholders not corporation) Thus if the investor claims that managementrsquos refusal to include its proposed bylaw amendment in the corporationrsquos proxy materials violates its shareholder rights to initiate corporate governance reforms the suit will be direct Courts have not questioned the ability of shareholders to bring direct suits challenging board action to exclude their proposed bylaw amendments from the corporationrsquos proxy materials See JANA Master Fund Ltd v CNET Networks Inc 954 A2d 335 (Del Ch 2008) (upholding shareholderrsquos direct challenge to boardrsquos interpretation of advance-notice bylaw) Chesapeake Corp v Shore 771 A2d 293 (Del Ch 2000) (upholding shareholderrsquos direct challenge to actions by board that effectively prevented it from proposing bylaw amendments in contest for control)

Is the way that the investor frames its claim conclusive Courts have permitted shareholder-plaintiffs to challenge a transaction in a direct suit even though the same transaction could also be challenged as a fiduciary breach See Eisenberg v Flying Tiger Line Inc 451 F2d 267 (2d Cir 1971) (permitting direct suit challenging a corporate reorganization as a dilution of shareholder voting power even though reorganization may have involved conflicts of interest and thus constituted a fiduciary breach) Thus the investorrsquos choice to pursue a claim challenging the legality of managementrsquos decision to exclude the investorrsquos proposal from the corporationrsquos proxy materialsmdashrather than a possible breach of fiduciary dutymdashis likely to be respected See 3 COX amp HAZEN supra sect 153 (describing situations in which a claim can be framed as derivative or direct)

[NOTE Some issues under Delaware corporate law regarding pre-suit demand are not relevant here For example whether the Mega directors are independent and disinterested is not relevant to the MBCA requirement of a pre-suit demand As the Official Comment to MBCA sect 742 points out the MBCArsquos requirement of ldquouniversal demandrdquo gives the board ldquothe opportunity to reexamine the act complained of in the light of a potential lawsuit and take corrective actionrdquo even when the directors might be non-independent or have conflicts of interest

Nor is it relevant to the MBCA pre-suit demand requirement that the statutory 90-day waiting period may be onerous The first paragraph of MBCA sect 742 requires a pre-suit demand without exception the second paragraph of the section imposes a 90-day waiting period before a derivative suit may be brought which can be shortened if the board rejects the demand or ldquoirreparable injury to the corporation would result by waiting for the expiration of the 90-day periodrdquo The call as written asks only whether a pre-suit demand should be made and does not ask examinees to address whether the post-demand waiting period should be shortened under the ldquoirreparable injuryrdquo standard]

35

National Conference of Bar Examiners 302 South Bedford Street | Madison WI 53703-3622 Phone 608-280-8550 | Fax 608-280-8552 | TDD 608-661-1275

wwwncbexorg e-mail contactncbexorg

  • Preface
  • Description of the MEE
  • Instructions
  • July 2014 Questions
    • CRIMINAL LAW AND PROCEDURE QUESTION
    • CONTRACTS QUESTION
    • FAMILY LAW QUESTION
    • FEDERAL CIVIL PROCEDURE QUESTION
    • EVIDENCE QUESTION
    • CORPORATIONS QUESTION
      • July 2014 Analyses
        • CRIMINAL LAW AND PROCEDURE ANALYSIS
        • CONTRACTS ANALYSIS
        • FAMILY LAW ANALYSIS
        • FEDERAL CIVIL PROCEDURE ANALYSIS
        • EVIDENCE ANALYSIS
        • CORPORATIONS ANALYSIS
            • ltlt13 ASCII85EncodePages false13 AllowTransparency false13 AutoPositionEPSFiles true13 AutoRotatePages None13 Binding Left13 CalGrayProfile (Dot Gain 20)13 CalRGBProfile (sRGB IEC61966-21)13 CalCMYKProfile (US Web Coated 050SWOP051 v2)13 sRGBProfile (sRGB IEC61966-21)13 CannotEmbedFontPolicy Error13 CompatibilityLevel 1413 CompressObjects Tags13 CompressPages true13 ConvertImagesToIndexed true13 PassThroughJPEGImages true13 CreateJobTicket false13 DefaultRenderingIntent Default13 DetectBlends true13 DetectCurves 0000013 ColorConversionStrategy CMYK13 DoThumbnails false13 EmbedAllFonts true13 EmbedOpenType false13 ParseICCProfilesInComments true13 EmbedJobOptions true13 DSCReportingLevel 013 EmitDSCWarnings false13 EndPage -113 ImageMemory 104857613 LockDistillerParams false13 MaxSubsetPct 10013 Optimize true13 OPM 113 ParseDSCComments true13 ParseDSCCommentsForDocInfo true13 PreserveCopyPage true13 PreserveDICMYKValues true13 PreserveEPSInfo true13 PreserveFlatness true13 PreserveHalftoneInfo false13 PreserveOPIComments true13 PreserveOverprintSettings true13 StartPage 113 SubsetFonts true13 TransferFunctionInfo Apply13 UCRandBGInfo Preserve13 UsePrologue false13 ColorSettingsFile ()13 AlwaysEmbed [ true13 ]13 NeverEmbed [ true13 ]13 AntiAliasColorImages false13 CropColorImages true13 ColorImageMinResolution 30013 ColorImageMinResolutionPolicy OK13 DownsampleColorImages true13 ColorImageDownsampleType Bicubic13 ColorImageResolution 30013 ColorImageDepth -113 ColorImageMinDownsampleDepth 113 ColorImageDownsampleThreshold 15000013 EncodeColorImages true13 ColorImageFilter DCTEncode13 AutoFilterColorImages true13 ColorImageAutoFilterStrategy JPEG13 ColorACSImageDict ltlt13 QFactor 01513 HSamples [1 1 1 1] VSamples [1 1 1 1]13 gtgt13 ColorImageDict ltlt13 QFactor 01513 HSamples [1 1 1 1] VSamples [1 1 1 1]13 gtgt13 JPEG2000ColorACSImageDict ltlt13 TileWidth 25613 TileHeight 25613 Quality 3013 gtgt13 JPEG2000ColorImageDict ltlt13 TileWidth 25613 TileHeight 25613 Quality 3013 gtgt13 AntiAliasGrayImages false13 CropGrayImages true13 GrayImageMinResolution 30013 GrayImageMinResolutionPolicy OK13 DownsampleGrayImages true13 GrayImageDownsampleType Bicubic13 GrayImageResolution 30013 GrayImageDepth -113 GrayImageMinDownsampleDepth 213 GrayImageDownsampleThreshold 15000013 EncodeGrayImages true13 GrayImageFilter DCTEncode13 AutoFilterGrayImages true13 GrayImageAutoFilterStrategy JPEG13 GrayACSImageDict ltlt13 QFactor 01513 HSamples [1 1 1 1] VSamples [1 1 1 1]13 gtgt13 GrayImageDict ltlt13 QFactor 01513 HSamples [1 1 1 1] VSamples [1 1 1 1]13 gtgt13 JPEG2000GrayACSImageDict ltlt13 TileWidth 25613 TileHeight 25613 Quality 3013 gtgt13 JPEG2000GrayImageDict ltlt13 TileWidth 25613 TileHeight 25613 Quality 3013 gtgt13 AntiAliasMonoImages false13 CropMonoImages true13 MonoImageMinResolution 120013 MonoImageMinResolutionPolicy OK13 DownsampleMonoImages true13 MonoImageDownsampleType Bicubic13 MonoImageResolution 120013 MonoImageDepth -113 MonoImageDownsampleThreshold 15000013 EncodeMonoImages true13 MonoImageFilter CCITTFaxEncode13 MonoImageDict ltlt13 K -113 gtgt13 AllowPSXObjects false13 CheckCompliance [13 None13 ]13 PDFX1aCheck false13 PDFX3Check false13 PDFXCompliantPDFOnly false13 PDFXNoTrimBoxError true13 PDFXTrimBoxToMediaBoxOffset [13 00000013 00000013 00000013 00000013 ]13 PDFXSetBleedBoxToMediaBox true13 PDFXBleedBoxToTrimBoxOffset [13 00000013 00000013 00000013 00000013 ]13 PDFXOutputIntentProfile ()13 PDFXOutputConditionIdentifier ()13 PDFXOutputCondition ()13 PDFXRegistryName ()13 PDFXTrapped False1313 CreateJDFFile false13 Description ltlt13 ARA 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 BGR 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 CHS ltFEFF4f7f75288fd94e9b8bbe5b9a521b5efa7684002000410064006f006200650020005000440046002065876863900275284e8e9ad88d2891cf76845370524d53705237300260a853ef4ee54f7f75280020004100630072006f0062006100740020548c002000410064006f00620065002000520065006100640065007200200035002e003000204ee553ca66f49ad87248672c676562535f00521b5efa768400200050004400460020658768633002gt13 CHT ltFEFF4f7f752890194e9b8a2d7f6e5efa7acb7684002000410064006f006200650020005000440046002065874ef69069752865bc9ad854c18cea76845370524d5370523786557406300260a853ef4ee54f7f75280020004100630072006f0062006100740020548c002000410064006f00620065002000520065006100640065007200200035002e003000204ee553ca66f49ad87248672c4f86958b555f5df25efa7acb76840020005000440046002065874ef63002gt13 CZE ltFEFF005400610074006f0020006e006100730074006100760065006e00ed00200070006f0075017e0069006a007400650020006b0020007600790074007600e101590065006e00ed00200064006f006b0075006d0065006e0074016f002000410064006f006200650020005000440046002c0020006b00740065007200e90020007300650020006e0065006a006c00e90070006500200068006f006400ed002000700072006f0020006b00760061006c00690074006e00ed0020007400690073006b00200061002000700072006500700072006500730073002e002000200056007900740076006f01590065006e00e900200064006f006b0075006d0065006e007400790020005000440046002000620075006400650020006d006f017e006e00e90020006f007400650076015900ed007400200076002000700072006f006700720061006d0065006300680020004100630072006f00620061007400200061002000410064006f00620065002000520065006100640065007200200035002e0030002000610020006e006f0076011b006a016100ed00630068002egt13 DAN 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 DEU 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 ESP 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 ETI ltFEFF004b00610073007500740061006700650020006e0065006900640020007300e4007400740065006900640020006b00760061006c006900740065006500740073006500200074007200fc006b006900650065006c007300650020007000720069006e00740069006d0069007300650020006a0061006f006b007300200073006f00620069006c0069006b0065002000410064006f006200650020005000440046002d0064006f006b0075006d0065006e00740069006400650020006c006f006f006d006900730065006b0073002e00200020004c006f006f0064007500640020005000440046002d0064006f006b0075006d0065006e00740065002000730061006100740065002000610076006100640061002000700072006f006700720061006d006d006900640065006700610020004100630072006f0062006100740020006e0069006e0067002000410064006f00620065002000520065006100640065007200200035002e00300020006a00610020007500750065006d006100740065002000760065007200730069006f006f006e00690064006500670061002e000d000agt13 FRA ltFEFF005500740069006c006900730065007a00200063006500730020006f007000740069006f006e00730020006100660069006e00200064006500200063007200e900650072002000640065007300200064006f00630075006d0065006e00740073002000410064006f00620065002000500044004600200070006f0075007200200075006e00650020007100750061006c0069007400e90020006400270069006d007000720065007300730069006f006e00200070007200e9007000720065007300730065002e0020004c0065007300200064006f00630075006d0065006e00740073002000500044004600200063007200e900e90073002000700065007500760065006e0074002000ea0074007200650020006f007500760065007200740073002000640061006e00730020004100630072006f006200610074002c002000610069006e00730069002000710075002700410064006f00620065002000520065006100640065007200200035002e0030002000650074002000760065007200730069006f006e007300200075006c007400e90072006900650075007200650073002egt13 GRE 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 HEB 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 HRV (Za stvaranje Adobe PDF dokumenata najpogodnijih za visokokvalitetni ispis prije tiskanja koristite ove postavke Stvoreni PDF dokumenti mogu se otvoriti Acrobat i Adobe Reader 50 i kasnijim verzijama)13 HUN 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 ITA 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 JPN ltFEFF9ad854c18cea306a30d730ea30d730ec30b951fa529b7528002000410064006f0062006500200050004400460020658766f8306e4f5c6210306b4f7f75283057307e305930023053306e8a2d5b9a30674f5c62103055308c305f0020005000440046002030d530a130a430eb306f3001004100630072006f0062006100740020304a30883073002000410064006f00620065002000520065006100640065007200200035002e003000204ee5964d3067958b304f30533068304c3067304d307e305930023053306e8a2d5b9a306b306f30d530a930f330c8306e57cb30818fbc307f304c5fc59808306730593002gt13 KOR ltFEFFc7740020c124c815c7440020c0acc6a9d558c5ec0020ace0d488c9c80020c2dcd5d80020c778c1c4c5d00020ac00c7a50020c801d569d55c002000410064006f0062006500200050004400460020bb38c11cb97c0020c791c131d569b2c8b2e4002e0020c774b807ac8c0020c791c131b41c00200050004400460020bb38c11cb2940020004100630072006f0062006100740020bc0f002000410064006f00620065002000520065006100640065007200200035002e00300020c774c0c1c5d0c11c0020c5f40020c2180020c788c2b5b2c8b2e4002egt13 LTH 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 LVI 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 NLD (Gebruik deze instellingen om Adobe PDF-documenten te maken die zijn geoptimaliseerd voor prepress-afdrukken van hoge kwaliteit De gemaakte PDF-documenten kunnen worden geopend met Acrobat en Adobe Reader 50 en hoger)13 NOR ltFEFF004200720075006b00200064006900730073006500200069006e006e007300740069006c006c0069006e00670065006e0065002000740069006c002000e50020006f0070007000720065007400740065002000410064006f006200650020005000440046002d0064006f006b0075006d0065006e00740065007200200073006f006d00200065007200200062006500730074002000650067006e0065007400200066006f00720020006600f80072007400720079006b006b0073007500740073006b00720069006600740020006100760020006800f800790020006b00760061006c0069007400650074002e0020005000440046002d0064006f006b0075006d0065006e00740065006e00650020006b0061006e002000e50070006e00650073002000690020004100630072006f00620061007400200065006c006c00650072002000410064006f00620065002000520065006100640065007200200035002e003000200065006c006c00650072002000730065006e006500720065002egt13 POL 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 PTB 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 RUM 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 RUS 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 SKY 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 SLV 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 SUO 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 SVE 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 TUR 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 UKR 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 ENU (Use these settings to create Adobe PDF documents best suited for high-quality prepress printing Created PDF documents can be opened with Acrobat and Adobe Reader 50 and later)13 gtgt13 Namespace [13 (Adobe)13 (Common)13 (10)13 ]13 OtherNamespaces [13 ltlt13 AsReaderSpreads false13 CropImagesToFrames true13 ErrorControl WarnAndContinue13 FlattenerIgnoreSpreadOverrides false13 IncludeGuidesGrids false13 IncludeNonPrinting false13 IncludeSlug false13 Namespace [13 (Adobe)13 (InDesign)13 (40)13 ]13 OmitPlacedBitmaps false13 OmitPlacedEPS false13 OmitPlacedPDF false13 SimulateOverprint Legacy13 gtgt13 ltlt13 AddBleedMarks false13 AddColorBars false13 AddCropMarks false13 AddPageInfo false13 AddRegMarks false13 ConvertColors ConvertToCMYK13 DestinationProfileName ()13 DestinationProfileSelector DocumentCMYK13 Downsample16BitImages true13 FlattenerPreset ltlt13 PresetSelector MediumResolution13 gtgt13 FormElements false13 GenerateStructure false13 IncludeBookmarks false13 IncludeHyperlinks false13 IncludeInteractive false13 IncludeLayers false13 IncludeProfiles false13 MultimediaHandling UseObjectSettings13 Namespace [13 (Adobe)13 (CreativeSuite)13 (20)13 ]13 PDFXOutputIntentProfileSelector DocumentCMYK13 PreserveEditing true13 UntaggedCMYKHandling LeaveUntagged13 UntaggedRGBHandling UseDocumentProfile13 UseDocumentBleed false13 gtgt13 ]13gtgt setdistillerparams13ltlt13 HWResolution [2400 2400]13 PageSize [612000 792000]13gtgt setpagedevice13

Page 7: July 2014 MEE Questions and Analyses - NCBE...This publication includes the questions and analyses from the July 2014 MEE. (In the actual test, the questions are simply numbered rather

CRIMINAL LAW AND PROCEDURE QUESTION

While on routine patrol a police officer observed a suspect driving erratically and pulled the suspectrsquos car over to investigate When he approached the suspectrsquos car the officer detected a strong odor of marijuana The officer immediately arrested the suspect for driving under the influence of an intoxicant (DUI) While the officer was standing near the suspectrsquos car placing handcuffs on the suspect the officer observed burglary tools on the backseat

The officer seized the burglary tools He then took the suspect to the county jail booked him for the DUI and placed him in a holding cell Later that day the officer gave the tools he had found in the suspectrsquos car to a detective who was investigating a number of recent burglaries in the neighborhood where the suspect had been arrested

At the time of his DUI arrest the suspect had a six-month-old aggravated assault charge pending against him and was being represented on the assault charge by a lawyer

Early the next morning upon learning of her clientrsquos arrest the lawyer went to the jail She arrived at 900 am immediately identified herself to the jailer as the suspectrsquos attorney and demanded to speak with the suspect The lawyer also told the jailer that she did not want the suspect questioned unless she was present The jailer told the lawyer that she would need to wait one hour to see the suspect After speaking with the lawyer the jailer did not inform anyone of the lawyerrsquos presence or her demands

The detective who had also arrived at the jail at 900 am overheard the lawyerrsquos conversation with the jailer The detective then entered the windowless interview room in the jail where the suspect had been taken 30 minutes earlier Without informing the suspect of the lawyerrsquos presence or her demands the detective read to the suspect full and accurate Miranda warnings The detective then informed the suspect that he wanted to ask about the burglary tools found in his car and the recent burglaries in the neighborhood where he had been arrested The suspect replied ldquoI think I want my lawyer here before I talk to yourdquo The detective responded ldquoThatrsquos up to yourdquo

After a few minutes of silence the suspect said ldquoWell unless there is anything else I need to know letrsquos not waste any time waiting for someone to call my attorney and having her drive here I probably should keep my mouth shut but Irsquom willing to talk to you for a whilerdquo The suspect then signed a Miranda waiver form and after interrogation by the detective made incriminating statements regarding five burglaries The interview lasted from 915 am to 1000 am

In addition to the DUI the suspect has been charged with five counts of burglary

The lawyer has filed a motion to suppress all statements made by the suspect to the detective in connection with the five burglaries

The state supreme court follows federal constitutional principles in all cases interpreting a criminal defendantrsquos rights

3

Criminal Law and Procedure Question

1 Did the detective violate the suspectrsquos Sixth Amendment right to counsel when he questioned the suspect in the absence of the lawyer Explain

2 Under Miranda did the suspect effectively invoke his right to counsel Explain

3 Was the suspectrsquos waiver of his Miranda rights valid Explain

4

CONTRACTS QUESTION

A music conservatory has two concert halls One concert hall had a pipe organ that was in poor repair and the other had no organ The conservatory decided to repair the existing organ and buy a new organ for the other concert hall After some negotiation the conservatory entered into two contracts with a business that both repairs and sells organs Under one contract the business agreed to repair the existing pipe organ for the conservatory for $100000 The business would usually charge a higher price for a project of this magnitude but the business agreed to this price because the conservatory agreed to prepay the entire amount Under the other contract the business agreed to sell a new organ to the conservatory for the other concert hall for $225000 As with the repair contract the business agreed to a low sales price because the conservatory agreed to prepay the entire amount Both contracts were signed on January 3 and the conservatory paid the business a total of $325000 that day

Two weeks later before the business had commenced repair of the existing organ the business suffered serious and unanticipated financial reversals The chief financial officer for the business contacted the conservatory and said

Bad news We had an unexpected liability and as a result are in a real cash crunch In fact even though we havenrsquot acquired the new organ from our supplier or started repair of your existing organ wersquove already spent the cash you gave us and we have no free cash on hand Wersquore really sorry but wersquore in a fix I think that we can find a way to perform both contracts but not at the original prices If you agree to pay $60000 more for the repair and $40000 more for the new organ we can probably find financing to finish everything If you donrsquot agree to pay us the extra money I doubt that we will ever be able to perform either contract and yoursquoll be out the money you already paid us

After receiving this unwelcome news the conservatory agreed to pay the extra amounts provided that the extra amount on each contract would be paid only upon completion of the businessrsquos obligations under that contract The business agreed to this arrangement and the parties quickly signed documents reflecting these changes to each contract The business then repaired the existing organ delivered the new organ and demanded payment of the additional $100000

The conservatory now has refused to pay the business the additional amounts for the repair and the new organ

1 Must the conservatory pay the additional $60000 for the organ repair Explain

2 Must the conservatory pay the additional $40000 for the new organ Explain

5

FAMILY LAW QUESTION

In 1994 a man and a woman were married in State A

In 1998 their daughter was born in State A

In 2010 the family moved to State B

In 2012 the husband and wife divorced in State B Under the terms of the divorce decree

(a) the husband and wife share legal and physical custody of their daughter (b) the husband must pay the wife $1000 per month in child support until their daughter reaches age 18 (c) the marital residence was awarded to the wife with the proviso that if it is sold before the daughter reaches age 18 the husband will receive 25 of the net sale proceeds remaining after satisfaction of the mortgage on the residence and (d) the remaining marital assets were divided between the husband and the wife equally

Six months ago the husband was offered a job in State A that pays significantly less than his job in State B but provides him with more responsibilities and much better promotion opportunities The husband accepted the job in State A and moved from State B back to State A

Since returning to State A the husband has not paid child support because due to his lower salary he has had insufficient funds to meet all his obligations

One month ago the wife sold the marital home netting $10000 after paying off the mortgage She then moved to a smaller residence The husband believes that he should receive more than 25 of the net sale proceeds given his financial difficulties

Last week when the wife brought the daughter to the husbandrsquos State A home for a weekend visit the husband served the wife with a summons in a State A action to modify the support and marital-residence-sale-proceeds provisions of the State B divorce decree The husband brought the action in the State A court that adjudicates all domestic relations issues

1 Does the State A court have jurisdiction to modify (a) the child support provision of the State B divorce decree Explain (b) the marital-residence-sale-proceeds provision of the State B divorce decree

Explain

2 On the merits could the husband obtain (a) retroactive modification of his child support obligation to the daughter Explain (b) prospective modification of his child support obligation to the daughter Explain (c) modification of the marital-residence-sale-proceeds provision of the State B

divorce decree Explain

6

FEDERAL CIVIL PROCEDURE QUESTION

The United States Forest Service (USFS) manages public lands in national forests including the Scenic National Forest Without conducting an environmental evaluation or preparing an environmental impact statement the USFS approved a development project in the Scenic National Forest that required the clearing of 5000 acres of old-growth forest The trees in the forest are hundreds of years old and the forest is home to a higher concentration of wildlife than can be found anywhere else in the western United States

The USFS solicited bids from logging companies to harvest the trees on the 5000 acres of forest targeted for clearing and it ultimately awarded the logging contract to the company that had submitted the highest bid for the trees However the USFS has not yet issued the company a logging permit Once it does so the company intends to begin cutting down trees immediately

A nonprofit organization whose mission is the preservation of natural resources has filed suit in federal district court against the USFS The nonprofit alleges that the USFS violated the National Environmental Policy Act (NEPA) by failing to prepare an environmental impact statement for the proposed logging project Among other remedies the nonprofit seeks a permanent injunction barring the USFS from issuing a logging permit to the logging company until an adequate environmental impact statement is completed The nonprofit believes that the logging project would destroy important wildlife habitat and thereby cause serious harm to wildlife in the Scenic National Forest including some endangered species

Assume that federal subject-matter jurisdiction is available that the nonprofit has standing to bring this action and that venue is proper

1 If the logging company seeks to join the litigation as a party must the federal district court allow it to do so as a matter of right Explain

2 What types of relief could the nonprofit seek to stop the USFS from issuing a logging permit during the pendency of the action what must the nonprofit demonstrate to obtain that relief and is the federal district court likely to grant that relief Explain

7

EVIDENCE QUESTION

A prison inmate has filed a civil rights lawsuit against a guard at the prison alleging that the guard violated the inmatersquos constitutional rights during an altercation The inmate and the guard are the only witnesses to this altercation They have provided contradictory reports about what occurred

The trial will be before a jury The inmate plans to testify at trial The guardrsquos counsel has moved for leave to impeach the inmate with the following

(a) Twelve years ago the inmate was convicted of felony distribution of marijuana He served a three-year prison sentence which began immediately after he was convicted He served his full sentence and was released from prison nine years ago (b) Eight years ago the inmate pleaded guilty to perjury a misdemeanor punishable by up to one year in jail He paid a $5000 fine (c) Seven years ago the inmate was convicted of felony sexual assault of a child and is currently serving a 10-year prison sentence for the crime The victim was the inmatersquos daughter who was 13 years old at the time of the assault

The inmatersquos counsel objects to the admission of any evidence related to these three convictions and to any cross-examination based on this evidence

The guard also plans to testify at trial The inmatersquos counsel has moved for leave to impeach the guard with the following

Last year the guard applied for a promotion to prison supervisor The guard submitted a reacutesumeacute to the state that indicated that he had been awarded a BA in Criminal Justice from a local college An official copy of the guardrsquos academic transcript from that college indicates that the guard dropped out after his first semester and did not receive a degree

The guardrsquos counsel objects to the admission of this evidence and to any cross-examination based on this evidence

The transcript and the reacutesumeacute have been properly authenticated The trial will be held in a jurisdiction that has adopted all of the Federal Rules of Evidence

1 What evidence if any proffered by the guard to impeach the inmate should be admitted Explain

2 What evidence if any proffered by the inmate to impeach the guard should be admitted Explain

8

CORPORATIONS QUESTION

Mega Inc is a publicly traded corporation incorporated in a state whose corporate statute is modeled on the Model Business Corporation Act (MBCA) Megarsquos articles of incorporation do not address the election of directors or amendment of the bylaws by shareholders

Well within the deadline for the submission of shareholder proposals for the upcoming annual shareholdersrsquo meeting an investor who was a large and long-standing shareholder of Mega submitted a proposed amendment to Megarsquos bylaws The proposal which the investor asked to be included in the corporationrsquos proxy materials and voted on at the upcoming shareholdersrsquo meeting read as follows

Section 20 The Corporation shall include in its proxy materials (including the proxy ballot) for a shareholdersrsquo meeting at which directors are to be elected the name of a person nominated for election to the Board of Directors by a shareholder or group of shareholders that beneficially have owned 3 or more of the Corporationrsquos outstanding common stock for at least one year

This Section shall supersede any inconsistent provision in these Bylaws and may not be amended or repealed by the Board of Directors without shareholder approval

Megarsquos management decided to exclude the investorrsquos proposal from the corporationrsquos proxy materials and explained its reasons in a letter to the investor

The investorrsquos proposed bylaw provision would be inconsistent with relevant state law because the Board of Directors has the authority to manage the business and affairs of the Corporation Generally shareholders lack the authority to interfere with corporate management by seeking to create a method for the nomination and election of directors inconsistent with the method chosen by the Board of Directors

Furthermore at its most recent meeting the Board of Directors unanimously approved an amendment to the Corporationrsquos bylaws that provides for proxy access for director nominations by a shareholder or a group of shareholders holding at least 10 of the Corporationrsquos voting shares for at least three years This procedure takes precedence over any nomination methods that might be sought or approved by shareholders

The investor is considering bringing a suit challenging managementrsquos refusal to include the investorrsquos proposed bylaw provision and challenging the boardrsquos amendment of the bylaws at its recent meeting

1 Is the investorrsquos proposed bylaw provision inconsistent with state law Explain

2 If the investorrsquos proposed bylaw provision were approved by the shareholders would the bylaw amendment previously approved by the board take precedence over the investorrsquos proposed bylaw provision Explain

3 Must the investor make a demand on Megarsquos board of directors before bringing suit Explain

9

July 2014 MEE

ANALYSES

Contracts Family Law

Criminal Law and Procedure

Federal Civil Procedure Evidence

Corporations

CRIMINAL LAW AND PROCEDURE ANALYSIS (Criminal Law and Procedure VA B D)

ANALYSIS

Legal Problems

(1) Did the detective violate the suspectrsquos Sixth Amendment right to counsel when he questioned the suspect about the burglaries without the lawyer present given that the lawyer represented the suspect in an unrelated criminal matter

(2) Under Miranda did the suspect effectively invoke his right to counsel when he said ldquoI think I want my lawyer here before I talk to yourdquo

(3) Was the suspectrsquos waiver of his right to remain silent under Miranda valid

DISCUSSION

Summary

The Sixth Amendment right to counsel as applied to states through the Fourteenth Amendment is offense-specific Although the suspect had an attorney representing him on his pending assault charge he had no Sixth Amendment right to the assistance of counsel with respect to the five uncharged burglaries because formal adversarial proceedings had not yet commenced on those charges The suspectrsquos Sixth Amendment right to counsel was not violated by the detectiversquos failure to inform him that the lawyer was present or of the lawyerrsquos demands

However a person undergoing custodial interrogation also has an independent constitutional right to counsel during custodial interrogation under Miranda When a suspect invokes his right to counsel under Miranda custodial interrogation must immediately cease for a period of at least 14 days However the invocation of the right to counsel must be unambiguous and clearly convey that the suspect has requested counsel Here because the suspectrsquos statement ldquoI think I want my lawyer here before I talk to yourdquo was ambiguous he did not invoke his Miranda right to counsel

A waiver of rights must be knowing intelligent and voluntary Here the suspect waived his right to remain silent under Miranda when he signed the waiver form The fact that the detective did not correct the suspectrsquos assumption that the lawyer would need to drive to the jailmdashby telling him that the lawyer was in the waiting room and was demanding to see himmdashdid not affect the validity of the suspectrsquos waiver

Point One (35) The suspectrsquos Sixth Amendment right to counsel was not violated because the right does not attach on new charges until formal adversarial judicial proceedings have commenced on those charges

The Sixth Amendment as applied to the states through the Fourteenth Amendment provides that ldquo[i]n all criminal prosecutions the accused shall enjoy the right to have the Assistance of Counsel for his defenserdquo The right to counsel does not attach with respect to particular charges until formal adversarial judicial proceedings have commenced (ie ldquoat or after the initiation of

13

Criminal Law and Procedure Analysis

adversary judicial criminal proceedingsmdashwhether by way of formal charge preliminary hearing indictment information or arraignment [or in some states arrest warrant]rdquo McNeil v Wisconsin 501 US 171 175 (1991) (internal quotations omitted)) Once a suspectrsquos Sixth Amendment right to counsel has attached any attempts to ldquodeliberately elicitrdquo statements from him in the absence of his attorney violate the Sixth Amendment See Massiah v United States 377 US 201 (1964) Brewer v Williams 430 US 387 (1977)

The Sixth Amendment right to counsel is charge- or offense -specific Representation by counsel in one prosecution does not in itself guarantee counsel for uncharged offenses See McNeil 501 US at 175 Texas v Cobb 532 US 162 (2001) Here the suspectrsquos Sixth Amendment right to counsel had attached only for the pending aggravated assault charge The suspectrsquos right to counsel for the aggravated assault case did not guarantee counsel for the five unrelated and uncharged burglaries that were the subject of the detectiversquos interrogation Thus because formal adversarial judicial proceedings against the suspect for the uncharged burglaries had not begun he had no Sixth Amendment right to counsel

Finally the detectiversquos failure to inform the suspect of the lawyerrsquos presence and demands to speak with him does not implicate the suspectrsquos Sixth Amendment right to counsel which had not yet attached See id Moran v Burbine 475 US 412 428ndash31 (1986)

Point Two (30) The suspect did not effectively invoke his right to counsel under Miranda because his statement was not unambiguous

A suspect subject to custodial interrogation has a right to consult with counsel and to have an attorney present during questioning Miranda v Arizona 384 US 436 (1966) When a suspect invokes his right to counsel during an interrogation law enforcement must immediately cease all questioning See Edwards v Arizona 451 US 477 484ndash85 (1981) Custodial interrogation cannot be reinitiated unless and until the suspect has been re-advised of his Miranda rights has provided a knowing and voluntary waiver and (1) counsel is present and (2) the suspect himself initiated further communication with the police see id at 484 or (3) (if the suspect was released from custody after the initial interrogation) at least 14 days have passed Maryland v Shatzer 559 US 98 110 (2010)

To invoke the right to counsel a suspectrsquos request must be ldquounambiguousrdquo This means that the suspect must articulate the desire for counsel sufficiently clearly that a reasonable officer would understand the statement to be a request for counsel Davis v United States 512 US 452 459 (1994) If the request is ambiguous the police are not required to stop the interrogation

In this case the suspectrsquos statement ldquoI think I want my lawyer here before I talk to yourdquo was not an unambiguous request for counsel The most reasonable interpretation of this statement is that the suspect might be invoking his right to counsel Id at 461 (ldquomaybe I should talk to a lawyerrdquo is not an unequivocal request for counsel) See also Burket v Angelone 208 F3d 172 197ndash98 (4th Cir 2000) (ldquoI think I need a lawyerrdquo is not an unambiguous request for an attorney) Soffar v Cockrell 300 F3d 588 594ndash95 (5th Cir 2002) (discussion of various statements that did not constitute unequivocal requests for counsel)

Under these circumstances the detective was not required to cease the custodial interrogation of the suspect Nor was the detective required to clarify or ask follow-up questions to determine whether the suspect in fact wanted an attorney Davis 512 US at 459ndash60

14

Criminal Law and Procedure Analysis

Point Three (35) The suspectrsquos waiver of his Miranda rights was knowing intelligent and voluntary despite the fact that he was never told of the lawyerrsquos presence in the jail or of the lawyerrsquos demands

A valid waiver of Miranda rights must be ldquovoluntaryrdquomdashie the product of a free or deliberate choice rather than intimidation coercion or deception Berghuis v Thompkins 560 US 370 382ndash83 (2010) In addition the waiver must be knowing and intelligent That is it ldquomust have been made with a full awareness of both the nature of the right being abandoned and the consequences of the decision to abandon itrdquo Moran v Burbine 475 US 421 (1986)

In this case the suspect signed a Miranda waiver form after receiving proper warnings There is no evidence ldquothat the police resorted to physical or psychological pressure to elicit the statementsrdquo Id The entire interview lasted only 45 minutes The only issue is whether the suspect knowingly and intelligently waived his Miranda rights despite the fact that the detective did not tell the suspect about the lawyerrsquos presence and her demands

The Supreme Court has said that ldquo[e]vents occurring outside of the presence of the suspect and entirely unknown to him surely can have no bearing on the capacity to comprehend and knowingly relinquish a constitutional rightrdquo Id at 422 If the suspect ldquoknew that he could stand mute and request a lawyer and was aware of the Statersquos intention to use his statements to secure a convictionrdquo then the waiver is valid regardless of the information withheld Id at 422ndash23

Here the suspect was correctly informed of his rights Miranda v Arizona 384 US at 467ndash73 His comments demonstrate that he understood that he could have a lawyer present if he desired (ie wondering whether he should call his attorney) and that he understood that there might be consequences to speaking with the detective (ldquoI probably should keep my mouth shut but Irsquom willing to talk to you for a whilerdquo) His comment ldquo[L]etrsquos not waste any time waiting for someone to call my attorney and having her drive hererdquo along with his signature on the Miranda waiver form show that his waiver was valid under the constitutional standard

The fact that the detective did not tell the suspect about the lawyerrsquos presence and demands has no bearing on the validity of the suspectrsquos waiver because ldquosuch conduct is only relevant to the constitutional validity of a waiver if it deprives a defendant of knowledge essential to his ability to understand the nature of his rights and the consequences of abandoning themrdquo Moran at 424 The Supreme Court has specifically declined to adopt a rule requiring that law enforcement tell a suspect of an attorneyrsquos efforts to contact him id at 425 (ldquoNor are we prepared to adopt a rule requiring that the police inform a suspect of an attorneyrsquos efforts to reach himrdquo)

[NOTE An examinee might also recognize that this general rule is further supported by the Supreme Courtrsquos decision in Florida v Powell 559 US 50 (2010) approving state Miranda warnings that do not explicitly warn suspects that they have a right to have counsel present during custodial interrogation]

15

CONTRACTS ANALYSIS (Contracts IB2 IIB IVA3 amp A5)

ANALYSIS

Legal Problems

(1) In the case of a service contract (governed by the common law of contracts) is a modification enforceable when a party agrees to pay more for the same performance than was originally promised

(2) In the case of a contract for the sale of goods (governed by Article 2 of the UCC) is a modification enforceable when a party agrees to pay more for the same goods than was originally promised

(3) May a party avoid an agreement on the basis of economic duress

DISCUSSION

Summary

There are two arguments that the conservatory can make to support the claim that it is not bound to pay the higher prices lack of consideration and economic duress

The organ repair contract is governed by the common law of contracts Under the common law the business would have difficulty recovering the additional $60000 for the organ repair because under the ldquopreexisting duty rulerdquo the agreement of the conservatory to pay the extra price was not supported by consideration However the business might argue that the modification is enforceable under an exception to the preexisting duty rule for fair and equitable modifications made in light of unanticipated circumstances

The organ sale contract is governed by Article 2 of the Uniform Commercial Code The business would likely recover the additional amount under that contract because Article 2 provides that consideration is not required for a modification to be binding

In both cases the conservatory could seek to avoid its agreement on the grounds of economic duress but that argument is not likely to succeed

Point One (45) The business probably cannot recover the additional $60000 for the organ repair because the conservatoryrsquos promise to pay more money was not supported by consideration

The general rule is that to be enforceable a promise must be supported by consideration Under RESTATEMENT (SECOND) OF CONTRACTS sect 71 a promise is supported by consideration if it is bargained for in exchange for a return promise or performance However under the ldquopreexisting duty rulerdquo (exemplified in RESTATEMENT (SECOND) OF CONTRACTS sect 73 and Alaska Packersrsquo Assrsquon v Domenico 117 F 99 (9th Cir 1902)) promise of performance of a legal duty already owed to a promisor which is neither doubtful nor the subject of honest dispute is not consideration

If the business had promised the conservatory anything new or different in exchange for the agreement to pay the additional $60000 (such as for example repairing the pipe organ more

16

Contracts Analysis

quickly or using better parts) that would constitute consideration especially in light of the principle that courts do not inquire into the adequacy of consideration Here however the business already had a legal duty under the original contract and did not agree to do anything else in exchange for the conservatoryrsquos promise to pay $60000 more

However an exception to the preexisting duty rule is sometimes applied in situations of unanticipated changed circumstances Under RESTATEMENT (SECOND) OF CONTRACTS sect 89 followed in many jurisdictions a promise modifying a duty under a contract not fully performed on either side is binding even if not supported by consideration if the modification is fair and equitable in view of circumstances not anticipated by the parties when the contract was made

If a court applies the rule in Restatement sect 89 the critical issues will be whether the modification was in fact ldquofair and equitablerdquo and whether it can be justified in light of unanticipated circumstances In many cases in which modifications have been upheld a party encountered difficulties or burdens in performing far beyond what was knowingly bargained for in the original contract with the result bordering on impracticability such as having to excavate solid rock instead of soft dirt or having to remove garbage far in excess of the amounts contemplated The conservatory would argue that the businessrsquos performance difficulties were not of this sort at allmdashnothing about repairing the pipe organ itself was any different from or more difficult than originally contemplated except that the business itself encountered financial distress unrelated to its burdens in performing its obligations under these contracts

Even if the business satisfies that element of the rule in Restatement sect 89 the business must also demonstrate that the circumstances that gave rise to the need to modify the contract were ldquounanticipatedrdquo at the time the original contract was made Here the facts suggest that when the business entered into the original contract it expected that the price paid by the conservatory would enable it to perform However any evidence that the business knew or had reason to know at the time of execution that it would need more money from the conservatory to be able to perform would mean that the request to modify was not ldquounanticipatedrdquo

[NOTE Some cases such as Schwartzreich v Bauman-Basch Inc 231 NY 196 131 NE 887 (1921) find that if the parties mutually agreed to rescind the original contract and then after rescission entered into an entirely new contract for a higher price the new contract is supported by consideration There is no evidence that such a rescission followed by a new contract took place here]

Point Two (45) The business can recover the additional $40000 for the new organ because no consideration is required under Article 2 of the UCC for good-faith contract modifications

The contract to buy a new organ is a contract for the sale of goods and therefore is governed by Article 2 of the Uniform Commercial Code UCC sect 2-102 Under Article 2 unlike the common law an agreement modifying a contract needs no consideration to be binding UCC sect 2-209(1) Section 2-209(1) thus obviates the preexisting duty rule entirely in contracts for the sale of goods

Even though consideration is not required modifications governed by sect 2-209 must satisfy the obligation of good faith imposed by the UCC UCC sect 1-304 See also Official Comment 2 to UCC sect 2-209 Good faith means ldquohonesty in fact and the observance of reasonable commercial standards of fair dealingrdquo UCC sect 1-201(b)(20) In this context the obligation of good faith means that ldquo[t]he effective use of bad faith to escape performance on the original contract terms is barred and the extortion of a lsquomodificationrsquo without legitimate commercial reason is ineffective as a violation of the duty of good faithrdquo Official Comment 2 to

17

Contracts Analysis

UCC sect 2-209 Here because the businessrsquos financial reversals were serious and apparently unanticipated at the time that the business entered into the contract with the conservatory and commitment of the extra money was needed to enable the business to perform a court would likely find that the business acted in good faith Thus a court would likely uphold the enforceability of the conservatoryrsquos promise to pay the additional $40000

Point Three (10) The conservatory is unlikely to be able to defend against enforcement of its promises to pay additional money under the theory of economic duress because the business probably did not make an improper threat

Under the common law of contracts parties may raise the defense of duress This common law defense also applies to contracts governed by UCC Article 2 See UCC sect 1-103(b)

A contract is voidable on the ground of economic duress by threat when it is established that a partyrsquos manifestation of assent is induced by an improper threat that leaves the party no reasonable alternative See RESTATEMENT (SECOND) OF CONTRACTS sect 175 See also eg Austin Instrument Inc v Loral Corp 272 NE2d 533 (NY 1971) (a threat to withhold essential goods can constitute duress) In order to void its agreement to pay the additional sum because of economic duress the conservatory must demonstrate that (1) the business made a threat to the conservatory (2) the threat was ldquoimproperrdquo or ldquowrongfulrdquo (3) the threat induced the conservatoryrsquos manifestation of assent to the modification and (4) the threat was sufficiently grave to justify the conservatoryrsquos assent

Here it appears that three of the four elements are likely satisfied The business plainly made a threat Moreover the threat induced the conservatoryrsquos assent to the modification and the threat was sufficiently grave to justify that assent If the conservatory had not agreed to pay the business the extra amounts the conservatory would have lost its entire $325000 investment In light of this potential loss a court could easily conclude that the conservatory had no reasonable alternative

However the business has a strong argument that its threat (indicating that it would breach the contracts unless the prices were increased) was not wrongful or improper but was instead nothing more than a communication of the reality of its own perilous situation to the conservatory

A mere threat to breach a contract is not in and of itself improper so as to support an action of economic duress or business compulsion Something more is required such as a breach of the duty of good faith and fair dealing as was present in Austin Instrument Inc supra Because the business could not perform the original contract without the requested modification the economic duress claim for the conservatory would likely fail for much the same reason that the business would be able to enforce the modification At the time the modification was requested the business was not trying to extort a price increase because of the conservatoryrsquos vulnerability but instead was simply stating the reality that the business could not perform without more money

18

FAMILY LAW ANALYSIS (Family Law IIIB D amp G)

ANALYSIS

Legal Problems

(1)(a) Does the State A court have jurisdiction to modify the State B child support order

(1)(b) Does the State A court have jurisdiction to modify the marital-residence-saleshyproceeds provision of the State B property-division decree

(2)(a) May a child support order be modified retroactively

(2)(b) May a child support order be modified prospectively based on a change of employment with a lower salary

(2)(c) May a property-division order be modified after entry of a divorce decree

DISCUSSION

Summary

The State A court may exercise personal jurisdiction over the wife because she was personally served in State A However subject-matter jurisdiction over the interstate modification of child support is governed by the Uniform Interstate Family Support Act (UIFSA) Under UIFSA State A does not have jurisdiction to modify the order for the daughterrsquos support because the wife is still a resident of State B UIFSA on the other hand does not govern property distributions and thus a State A court is not precluded from hearing the husbandrsquos petition to modify the marital-residence-sale-proceeds provision of the divorce decree

A child support order may not be modified retroactively A child support order may be modified prospectively based on a substantial change in circumstances Courts agree that a significant decrease in income is a substantial change in circumstances All states treat voluntary income reductions differently than involuntary reductions but employ different approaches for evaluating the impact of a voluntary reduction Whether the husband could obtain prospective modification of the child support order depends on which approach is applied

A property-division order is not subject to post-divorce modification based on a change in circumstances Thus the husband may in some states obtain prospective modification of the order for the daughterrsquos support but he may not obtain modification of the marital-residenceshysale-proceeds provision

Point One(a) (25) Personal jurisdiction over a nonresident respondent does not confer subject-matter jurisdiction over child support modification Under UIFSA a State A court may not modify a child support order issued by a State B court when as here the child or either parent continues to reside in State B the jurisdiction that issued the child support order

The State A court may exercise personal jurisdiction over the wife The wife was personally served in State A and a state may exercise jurisdiction based on in-state personal service See

19

Family Law Analysis

Burnham v Superior Court 495 US 604 (1990) But personal jurisdiction over the wife is not enough to give a State A court jurisdiction to modify the State B support order

The interstate enforcement and modification of child support is governed by the Uniform Interstate Family Support Act (UIFSA) which has been adopted by all states Under UIFSA the state that originally issued a child support order (here State B) has continuing exclusive jurisdiction to modify the order if that state remains the residence of the obligee the child or the obligor and all parties do not consent to the jurisdiction of another forum See UIFSA sect 205 See also UIFSA sect 603 (ldquoA tribunal of this State shall recognize and enforce but may not modify a registered order if the issuing tribunal had jurisdictionrdquo) The wife and daughter continue to reside in State B and the wife has not consented to the jurisdiction of another forum Thus a State A court does not have jurisdiction to modify the State B child support order

[NOTE Examinees who do not discuss personal jurisdiction but fully discuss UIFSA may receive full credit]

Point One(b) (15) UIFSA does not apply to disputes over property division Thus the State A court may exercise jurisdiction over the husbandrsquos petition to modify the marital-residence-sale-proceeds provision of the State B divorce decree because it has personal jurisdiction over the wife

The State A court in which the husband brought his action has jurisdiction to adjudicate domestic relations issues The husbandrsquos petition to modify the property settlement is a domestic relations issue The courts of State A may exercise personal jurisdiction over the wife because she was personally served in State A See Burnham v Superior Court 495 US 604 (1990) see Point One(a)

UIFSA does not apply to divorce property-division disputes Thus although a State A court may not adjudicate the husbandrsquos petition to modify his child support obligations it may adjudicate his property-division claims (Even though the court has jurisdiction it may not modify the property-division award on the merits See Point Two(c))

Point Two(a) (20) A child support order may not be modified retroactively

State courts have long held that obligations to pay child support ordinarily may not be modified retroactively ldquoIf the hardship is particularly severe the courts sometimes devised a way to protect the obligor but in most instances the courts hold that retroactive modification of this kind is beyond their power and indeed the governing statute may so providerdquo HOMER H CLARK THE LAW OF DOMESTIC RELATIONSHIPS IN THE UNITED STATES 725 (2d ed 1987)

Federal law now goes further and requires the states as a condition of federal child-support funding to adopt rules that absolutely forbid retroactive modification of the support obligation See 42 USC sect 666(a)(9)(C) The states have adopted rules consistent with the federal requirements

Point Two(b) (25) It is unclear whether the husband could obtain prospective downward modification of his child support based on his voluntary acceptance of a job with a lower salary

Prospective modification of a child support order is typically available only when the petitioner can show a substantial change in circumstances See ROBERT E OLIPHANT amp NANCY VER

20

Family Law Analysis

STEEGH FAMILY LAW 213ndash15 (3d ed 2010) A significant decrease in income is typically viewed as a substantial change

However when a parent seeks to modify a child support obligation because he has voluntarily reduced his income a court will not modify the obligation based solely on the income loss Some courts refuse to modify whenever the income shift was voluntary See eg Aguiar v Aguiar 127 P3d 234 (Idaho Ct App 2005) Others look primarily to the petitionerrsquos intentions and permit downward modification if he has acted in good faith See eg In re Marriage of Horn 650 NE2d 1103 (Ill App Ct 1995) Many courts use a multifactor approach See OLIPHANT amp VER STEEGH supra 217ndash18

Here there is no question that the husbandrsquos loss of income was voluntary In a jurisdiction in which voluntary income reduction bars support modification the husbandrsquos petition would be denied

In a jurisdiction employing a good-faith or multifactor approach it is possible but not certain that the husband could obtain downward modification The evidence supports the husbandrsquos good faith his change in employment appears to be based on his new jobrsquos greater responsibilities and better promotion possibilities In a jurisdiction using a multifactor approach the court would likely also consider the impact of such a shift on the daughter the likely duration of the husbandrsquos income loss and the likelihood of a promotion that would ultimately inure to the daughterrsquos benefit Thus on these facts it is possible but by no means certain that the husband could prospectively obtain downward modification of his child support obligation to his daughter

Point Two(c)(15) A divorce property-division award is not subject to modification

A support order is aimed at meeting the post-divorce needs of the supported individual Because the future is unpredictable courts are empowered to modify a support award to take account of changed circumstances that may occur during the period in which support is paid

By contrast a property-distribution award divides assets of the marriage based on the equities at the time of divorce Because the past can be ascertained a property-division award is not subject to post-divorce modification See HARRY A KRAUSE ET AL FAMILY LAW CASES COMMENTS AND QUESTIONS 691 (6th ed 2007)

Here the husband is seeking modification of a property-division award with respect to an asset owned by the parties at the time of divorce Thus the husband may not obtain a modification of the marital-residence-sale-proceeds provision of the divorce decree based on his reduced income

21

FEDERAL CIVIL PROCEDURE ANALYSIS (Federal Civil Procedure III IVC)

ANALYSIS

Legal Problems

(1) Is the logging company entitled to join this action as a matter of right

(2)(a) May the nonprofit organization obtain a temporary restraining order to stop the USFS from issuing a logging permit

(2)(b) May the nonprofit organization obtain a preliminary injunction to stop the USFS from issuing a logging permit during the pendency of the action

DISCUSSION

Summary

The logging company is entitled to intervene in this action as a matter of right because it has an interest in the property or transaction that is the subject of the action and is so situated that its interest may be impaired or impeded as a practical matter if the action goes forward without it The logging companyrsquos interest is not adequately represented by the USFSrsquos presence in the lawsuit

The nonprofit organization may seek a temporary restraining order (TRO) followed by a preliminary injunction to prevent the USFS from issuing a logging permit pending the outcome of the action The nonprofit is likely to obtain a TRO if it can demonstrate a risk of immediate and irreparable injury The nonprofit is also likely to obtain a preliminary injunction if it can demonstrate a significant threat of irreparable harm and a likelihood of success on the merits of its National Environmental Policy Act (NEPA) claim

Point One (50) Rule 24(a) of the Federal Rules of Civil Procedure requires federal courts to allow a person to intervene in an action as a matter of right if the person a) is interested in the property or transaction that is the subject of the action b) is so situated that its interest may be impaired or impeded if the litigation goes forward without it and c) is not adequately represented by existing parties Here the logging company likely meets all three requirements and should be allowed to intervene as a matter of right

Rule 24 of the Federal Rules of Civil Procedure governs intervention the process by which a non-party to an action may join the litigation Under Rule 24(a) (intervention of right) a person must be permitted to intervene if three conditions are met (1) the movant ldquoclaims an interest relating to the property or transaction that is the subject of the actionrdquo (2) the movant ldquois so situated that disposition of the action may as a practical matter impair or impede the movantrsquos ability to protect its interestrdquo and (3) ldquoexisting partiesrdquo do not ldquoadequately represent [the movantrsquos] interestrdquo FED R CIV P 24(a) The three requirements for intervention of right are often ldquovery interrelatedrdquo 7C CHARLES ALAN WRIGHT ET AL FEDERAL PRACTICE AND PROCEDURE sect 1908 at 297 (2007 amp 2011 Supp)

22

Federal Civil Procedure Analysis

Here the court should find that the logging company meets this test First the logging company has a strong interest in the property or transaction that is the subject of this action The USFS has accepted the logging companyrsquos bid and the logging company is merely awaiting issuance of a logging permit to begin logging The nonprofit organization is seeking to prevent this logging The logging company therefore has a strong direct and substantial interest in the subject matter of the lawsuit and in having its winning bid honored and a logging permit issued See eg Kleissler v US Forest Serv 157 F3d 964 972 (3d Cir 1998) (stating that ldquo[t]imber companies have direct and substantial interests in a lawsuit aimed at halting loggingrdquo) see also Natural Resources Defense Council v US Nuclear Regulatory Commrsquon 578 F2d 1341 1343ndash 44 (10th Cir 1978) (holding that applicants whose license renewals were pending had Rule 24(a)(2) interests where the lawsuit sought to halt the license-issuing process pending preparation of environmental impact statements) See generally 7C WRIGHT ET AL supra sect 19081 at 309 (ldquoIf there is a direct substantial legally protectable interest in the proceedings it is clear that this requirement of the rule is satisfiedrdquo) Second the logging companyrsquos interest in receiving a logging permit may well be impaired as a practical matter by the outcome of the lawsuit If the USFS loses the lawsuit it will have to prepare an environmental impact statement before issuing the logging companyrsquos permit This will at a minimum delay the logging companyrsquos ability to exercise its rights and may in the long r un mean that no logging permit is ever issued Intervention of right is not limited to those that would be legally bound as a matter of preclusion doctrine Id sect 19082 at 368 Rather ldquo[t]he rule is satisfied whenever disposition of the present action would put the movant at a practical disadvantage in protecting its interestrdquo Id sect 19082 at 369 Here that condition is easily satisfied See Kleissler 157 F3d at 972 (ldquoTimber companies have direct and substantial interests in a lawsuit aimed at halting logging rdquo)

Given that the logging company has an interest that may be impaired by disposition of the action it should be allowed to intervene unless the court is persuaded that the USFS adequately represents the logging companyrsquos interest See Rule 24(a)(2) 7C WRIGHT ET AL supra sect 1909 Here it could be argued that the USFS adequately represents the logging companyrsquos interest because the USFS presumably wants the court to uphold its development plan and allow it to proceed with issuance of the logging permit which is the same relief that the logging company would seek However whether representation is truly adequate depends upon ldquo[a] discriminating appraisal of the circumstancesrdquo 7C WRIGHT ET AL supra sect 1909 at 440 Although both the government and the logging company wish to avoid the preparation of an environmental impact statement their interests are distinct The USFSrsquos interest is proper management of the national forest system while the logging companyrsquos interest is making a profit from logging the 5000-acre tract The USFSrsquos handling of the litigation is likely to be affected by a variety of policy concerns and political considerations that have nothing to do with the logging companyrsquos purely economic interest in securing the right to cut trees in the Scenic National Forest See eg Kleissler 157 F3d at 973ndash74 (ldquo[T]he government represents numerous complex and conflicting interests in matters of this nature The straightforward business interests asserted by intervenors here may become lost in the thicket of sometimes inconsistent governmental policiesrdquo)

[NOTES (1) Examinees who mistakenly analyze the logging companyrsquos case for joinder under the related but incorrect Rule 19 ldquoRequired Joinder of Partiesrdquo may receive credit Rule 19 allows existing parties to demand joinder of non-parties (or seek dismissal of the case if they canrsquot get it) There is a close relationship between Rule 24 and Rule 19 and both contain a similar standard for determining when ldquointerestedrdquo third parties are ldquoentitledrdquo or ldquorequiredrdquo to be in the lawsuit Indeed the two prongs of the Rule 24 intervention test that are discussed above

23

Federal Civil Procedure Analysis

are nearly identical to the two prongs of the Rule 19(a) required joinder test Examinees who discuss and apply the test should receive credit even if they cite Rule 19 rather than Rule 24

(2) Examinees may discuss permissive joinder Although permissive joinder is a possibility here the question asks only whether the logging company can join the action as a matter of right and a permissive joinder analysis is not responsive to the question To the extent an examinee discusses permissive joinder the analysis will focus on whether the logging company ldquohas a claim or defense that shares with the main action a common question of law or factrdquo FED R CIV P 24(b)(1)(B) The district court also ldquomust consider whether the intervention will unduly delay or prejudice the adjudication of the original partiesrsquo rightsrdquo FED R CIV P 24(b)(3) On our facts the logging companyrsquos claim for the issuance of a logging permit would certainly share common questions of law and fact with the USFSrsquos defense against the nonprofitrsquos claim There are no facts suggesting that the logging companyrsquos presence would unduly delay or otherwise prejudice adjudication of the original action Thus the district court would have discretion to permit the logging company to intervene even if it denied intervention of right]

Point Two(a) (25) The nonprofit organization could seek and would likely obtain a temporary restraining order to stop the USFS from issuing a logging permit pending a hearing on an application for a preliminary injunction

The first type of interim relief the nonprofit could seek to stop the USFS from issuing a logging permit to the logging company is a temporary restraining order (TRO) prohibiting the USFS from issuing the logging permit A TRO can be issued without notice to the adverse party but only in limited circumstances and only for a limited time FED R CIV P 65(b) To secure a TRO without notice the nonprofit would need to submit an affidavit containing specific facts that demonstrate a risk of ldquoimmediate and irreparable injuryrdquo if a permit is issued FED R CIV P 65(b)(1) In deciding whether to grant a TRO courts will also consider the same factors that are relevant in deciding whether to grant a preliminary injunction (eg the moving partyrsquos likelihood of success on the merits the balance of hardships and the public interest) See Point Two(b) infra The TRO would last only long enough for the court to consider and resolve a request by the nonprofit for a preliminary injunction but no longer than 14 days (unless the court extends it for good cause or the adverse party consents to an extension) In addition bond is required

Here the court is likely to grant the nonprofitrsquos request The nonprofit could plausibly claim that cutting down 5000 acres of old-growth forest in an area that is home to the highest concentration of wildlife in the western United States would have ldquoan immediate and irreparablerdquo adverse impact on the environment and cause irreparable harm to the nonprofitrsquos interest in preserving and protecting natural resources including wildlife habitat

Point Two(b) (25) The nonprofit could also seek and would likely obtain a preliminary injunction to stop the USFS which is likely to be granted if the nonprofitrsquos claim that the USFS violated NEPA has a strong basis in fact and law

Because the TRO would be temporary the nonprofit would need to move for a preliminary injunction to prevent the USFS from issuing a logging permit throughout the pendency of the litigation Preliminary injunctions are injunctions that seek to ldquoprotect [the] plaintiff from

24

Federal Civil Procedure Analysis

irreparable injury and to preserve the courtrsquos power to render a meaningful decision after a trial on the meritsrdquo 11A CHARLES ALAN WRIGHT ET AL FEDERAL PRACTICE AND PROCEDURE sect 2947 at 112 (2013) Rule 65 of the Federal Rules of Civil Procedure sets out the procedural requirements for preliminary injunctions Preliminary injunctions may be granted only upon notice to the adverse party FED R CIV P 65(a)(1) and only if the movant ldquogives security in an amount that the court considers proper to pay the costs and damages sustained by any party found to have been wrongfully enjoined or restrainedrdquo FED R CIV P 65(c)

While Rule 65 sets out the procedural requirements for preliminary injunctive relief it does not specify the substantive grounds upon which it may be granted The courtrsquos discretion in ruling upon a motion for a preliminary injunction ldquois exercised in conformity with historic federal equity practicerdquo 11A WRIGHT ET AL supra sect 2947 at 114 The court typically considers four factors

(1) the significance of the threat of irreparable harm to the plaintiff if the injunction is not granted (2) the balance between this harm and the injury that granting the injunction would inflict on the defendant (3) the probability that the plaintiff will succeed on the merits and (4) the public interest

Id sect 2948 at 122ndash24 accord Habitat Educ Center v Bosworth 363 F Supp 2d 1070 1088 (ED Wis 2005) The most important of these factors is the risk of irreparable harm to the plaintiff 11A WRIGHT ET AL supra sect 29481 at 129 If the plaintiff has an adequate remedy at law (eg if money damages can compensate the plaintiff for its loss) then a preliminary injunction will be denied Id sect 29481

Here a court would likely conclude that the potential for environmental damage to the forest creates a significant threat of irreparable harm ldquo[E]nvironmental injury is often irreparable Courts have recognized that logging such as would occur [here] can have longshyterm environmental consequences and thus satisfy the irreparable injury criterionrdquo Habitat Educ Center 363 F Supp 2d at 1089 (citing Idaho Sporting Congress Inc v Alexander 222 F3d 562 569 (9th Cir 2000) (noting that the imminent and continuing logging activities presented ldquoevidence of environmental harm sufficient to tip the balance in favor of injunctive reliefrdquo)) Neighbors of Cuddy Mountain v US Forest Service 137 F3d 1372 1382 (9th Cir 1998) (stating that ldquo[t]he old growth forests plaintiffs seek to protect would if cut take hundreds of years to reproducerdquo) (internal citation omitted)) see also 11C WRIGHT ET AL supra sect 29481 at 151 (noting that ldquoa preliminary injunction has been issued to prevent harm to the environmentrdquo)

The second factor the balance between the harm to the plaintiff and the harm the defendant will suffer if the injunction is issued also appears to support issuance of a preliminary injunction here The USFS will have to wait before it can develop the Scenic National Forest and the logging company may lose money if the delay is prolonged These economic harms could be compensated monetarily if an injunction is issued inappropriately Where ldquoan injunction bond can compensate [the] defendant for any harm the injunction is likely to inflict the balance should be struck in favor of [the] plaintiffrdquo Id sect 29482 at 192 See also Habitat Educ Center 363 F Supp 2d at 1089 (stating that ldquothe relative absence of harmful effects on the Forest Service weighs in favor of granting the injunctionrdquo)

The third factor is the likelihood that the plaintiff will prevail on the merits Although there is limited information concerning the merits of the action the nonprofit alleges that the federal statute (NEPA) requires an environmental impact statement and further states that the USFS created no environmental impact analysis or statement at all Assuming that those

25

Federal Civil Procedure Analysis

allegations are correct it seems plausible to conclude that the nonprofit will be able to show a likelihood of success on the merits

Finally courts deciding whether or not to issue preliminary injunctive relief are to consider the public interest ldquoFocusing on this factor is another way of inquiring whether there are policy considerations that bear on whether the order should issuerdquo 11C WRIGHT ET AL supra sect 29484 at 214 If the court concludes that the nonprofit is likely to succeed on its NEPA claim because the USFS wrongfully failed to conduct an environmental impact assessment it is likely to find that the public interest would be served by restraining the USFS from proceeding with logging in a national forest See Heartwood Inc v US Forest Service 73 F Supp 2d 962 979 (SD Ill 1999) affrsquod on other grounds 230 F3d 947 (7th Cir 2000) (ldquoviolations by federal agencies of NEPArsquos provisions as established by Congress harm the public as well as the environmentrdquo)

Thus a court is very likely to grant a preliminary injunction if it concludes that the nonprofit has a significant likelihood of success on the merits

26

EVIDENCE ANALYSIS (Evidence ID IIA amp C)

ANALYSIS

Legal Problems

(1) Under what circumstances can evidence of prior convictions be used to impeach a witnessrsquos credibility in a civil case

(1)(a) May the inmatersquos credibility be impeached by evidence of a 12-year-old felony drug conviction if he was released from prison 9 years ago

(1)(b) May the inmatersquos credibility be impeached by evidence of an 8-year-old misdemeanor perjury conviction that was punishable by 1 year in jail if he pleaded guilty and was sentenced only to pay a $5000 fine

(1)(c) May the inmatersquos credibility be impeached by evidence of a 7-year-old sexual assault conviction if the inmate is still serving a 10-year prison sentence and the victim was his 13-year-old daughter

(2)(a) May the guardrsquos credibility be impeached by cross-examination regarding specific instances of misconduct (ie lying on his reacutesumeacute) relevant to credibility

(2)(b) May the guardrsquos credibility be impeached by admission of extrinsic evidence (his reacutesumeacute and academic transcript) offered to prove specific instances of misconduct relevant to credibility

DISCUSSION

Summary

Under the Federal Rules of Evidence witnesses can be impeached with evidence of prior convictions andor specific instances of misconduct Whether evidence of prior convictions should be admitted to impeach generally depends on the nature of the crime the amount of time that has passed and (only in criminal cases) whether the ldquowitnessrdquo is the defendant FED R EVID 609(a)

In this civil case evidence of the inmatersquos conviction for distribution of marijuana should be admitted to impeach the inmate because he was convicted of a felony and was released from prison fewer than 10 years ago FED R EVID 609(a)(1) Credibility is critically important in this case because the jury will hear conflicting testimony from the two disputing parties and there were no other eyewitnesses to the altercation Under Rule 609(a)(1) the inmatersquos conviction should be admitted because it has some bearing on his credibility and its probative value is not substantially outweighed by concerns of unfair prejudice confusion or delay Id

Evidence of the inmatersquos misdemeanor conviction for perjury must be admitted because the crime ldquorequired provingmdashor the witnessrsquos admittingmdasha dishonest act or false statementrdquo by the inmate FED R EVID 609(a)(2)

27

Evidence Analysis

Evidence of the inmatersquos felony conviction for sexual assault should be excluded because its probative value is substantially outweighed by the danger of unfair prejudice to the inmate based on the heinous nature of the crime FED R EVID 609(a)(1) In the alternative the judge could limit the evidence relating to this conviction by excluding details of the inmatersquos crime

In all civil (and criminal) cases witnesses can also be impeached with evidence of specific instances of prior misconduct that did not result in a conviction FED R EVID 608(b) Pursuant to Rule 608(b) misconduct probative of untruthfulness can be inquired into on cross-examination but cannot be proved through extrinsic evidence Id Thus the inmatersquos counsel should be permitted to cross-examine the guard regarding the false statement in the guardrsquos reacutesumeacute However extrinsic evidence of the guardrsquos misconduct (ie the guardrsquos authenticated reacutesumeacute and transcript from the local college) should not be admitted even if the guard denies wrongdoing or refuses to answer cross-examination questions about these matters Id

Point One (10) The Federal Rules of Evidence permit impeachment of witnesses with evidence of prior convictions

Whether convictions should be admitted to impeach generally depends on the nature of the crime the amount of time that has passed and (only in criminal cases) whether the ldquowitnessrdquo is the defendant FED R EVID 609(a) Under Rule 609(a) evidence of prior convictions may be admitted for the purpose of ldquoattacking a witnessrsquos character for truthfulnessrdquo Id

There are two basic types of convictions that can be admitted for the purpose of impeachment

(1) convictions for crimes ldquopunishable by death or by imprisonment for more than one yearrdquo (which generally correlates to ldquofeloniesrdquo) FED R EVID 609(a)(1) and (2) convictions ldquofor any crimes regardless of the punishment if the court can readily determine that establishing the elements of the crime required provingmdashor the witnessrsquos admittingmdasha dishonest act or false statementrdquo FED R EVID 609(a)(2)

Pursuant to Rule 609(a)(1) in civil cases the admission of evidence of a felony conviction is ldquosubject to Rule 403 [which says that a court may exclude relevant evidence if its probative value is substantially outweighed by other factors]rdquo FED R EVID 609(a)(1) However Rule 403 does not protect the witness against admission of prior convictions involving dishonestymdashwhich must be admitted by the court FED R EVID 609(a)(2)

Finally Federal Rule of Evidence 609(b) contains the presumption that a conviction that is more than 10 years old or where more than 10 years has passed since the witnessrsquos release from confinement (whichever is later) should not be admitted unless ldquoits probative value supported by specific facts and circumstances substantially outweighs its prejudicial effectrdquo and the proponent has provided the adverse party with reasonable written notice FED R EVID 609(b)

Point One(a) (25) The court should admit evidence of the inmatersquos 12-year-old felony marijuana distribution conviction

The inmatersquos conviction for marijuana distribution was for a felony punishable by imprisonment for more than one year See FED R EVID 609(a)(1) Moreover although the conviction was 12 years ago the 10-year time limit of Rule 609(b) is not exceeded because that time limit runs

28

Evidence Analysis

from the date of either ldquothe witnessrsquos conviction or release from confinement for it whichever is laterrdquo FED R EVID 609(b) Because the inmate served three years in prison he was released from confinement nine years ago

However pursuant to Rule 609(a)(1) the admission of felony convictions to impeach a witness in a civil case is ldquosubject to Rule 403rdquo FED R EVID 609(a)(1) Neither Rule 609(a) nor the advisory committee notes specify which factors courts should consider when balancing the probative value of a conviction against the dangers identified in Rule 403 (which include (1) unfair prejudice (2) confusion of the issues (3) misleading the jury (4) waste of time or undue delay and (5) needless presentation of cumulative evidence) FED R EVID 403

In this case credibility is very important because the evidence consists primarily of the testimony of the disputing parties and there were no other eyewitnesses to the altercation This enhances the probative value of any evidence bearing on the inmatersquos credibility A court is likely to conclude that the inmatersquos prior felony drug conviction is relevant to his credibility See eg United States v Brito 427 F3d 53 64 (1st Cir 2005) (ldquoPrior drug-trafficking crimes are generally viewed as having some bearing on veracityrdquo) Although the probative value of any conviction diminishes with age see eg United States v Brewer 451 F Supp 50 53 (ED Tenn 1978) the inmatersquos ongoing problems with the law suggest that he has continued (and even escalated) his criminal behavior over the past nine years The court should admit this evidence because its probative value is not substantially outweighed by any Rule 403 concerns Specifically any prejudice to the inmate would be slight because the conviction is unrelated to the altercation at issue and the conviction was not for a heinous crime that might inflame the jury

[NOTE Whether an examinee identifies the jury instruction as containing a ldquoconclusiverdquo or ldquomandatoryrdquo presumption is less important than the examineersquos analysis of the constitutional infirmities]

Point One(b) (15) The court must admit evidence of the inmatersquos eight-year-old misdemeanor conviction because perjury is a crime of dishonesty

Rule 609(a)(2) provides that evidence of a criminal conviction ldquomust be admitted if the court can readily determine that establishing the elements of the crime required provingmdashor the witnessrsquos admittingmdasha dishonest act or false statementrdquo FED R EVID 609(a)(2) The inmatersquos conviction for perjury would have necessarily required proving that the inmate engaged in an act of dishonesty This conviction occurred within the past 10 years so it ldquomust be admittedrdquo because in contrast to Rule 609(a)(1) (discussed in Point One(a)) admission under Rule 609(a)(2) is mandatory and not subject to Rule 403

Point One(c) (20) The court should exclude evidence of the inmatersquos seven-year-old felony sexual assault conviction because the probative value of this evidence is substantially outweighed by the danger of unfair prejudice In the alternative the details of the prior conviction could be excluded

The inmatersquos conviction for felony sexual assault was seven years ago and he has not yet been released from incarceration so Rule 609(a) but not 609(b) is applicable here FED R EVID 609(a) This conviction is therefore admissible to impeach the inmate unless its probative value is substantially outweighed by the danger of unfair prejudice or any other Rule 403 concern Id

29

Evidence Analysis

Sex crimes are generally not considered relevant to credibility see Hopkins v State 639 So 2d 1247 1254 (Miss 1993) so the probative value of this conviction is relatively low Moreover the heinous nature of the inmatersquos crime (sexual assault on his daughter) makes the danger of unfair prejudice to the inmate very high Thus the court should exclude evidence of the conviction because it was for a heinous offense that is likely to inflame the jury and it has little bearing on credibility See eg United States v Beahm 664 F2d 414 419 (4th Cir 1981)

As an alternative to excluding this evidence the judge could minimize the unfair prejudice to the inmate by permitting limited cross-examination but refusing to allow specific questions about the nature of the inmatersquos conviction For example a court could limit cross-examination to the fact that the inmate was convicted of a ldquofelonyrdquo or perhaps that he was convicted of a ldquosexual assaultrdquo without identifying the victim However because evidence of the inmatersquos prior convictions can be admitted solely for the purpose of enabling the jury to assess his credibility and because his two earlier convictions should have already been admitted the court should exclude all evidence of the felony sexual assault conviction

Point Two(a) (15) The court should permit the inmatersquos counsel to cross-examine the guard regarding the false statement in his reacutesumeacute because the guardrsquos misconduct bears on his truthfulness

The inmate wishes to cross-examine the guard about his prior dishonest behaviormdashlying on his reacutesumeacutemdashthat did not involve a criminal conviction Rule 608(b) allows witnesses to be cross-examined about specific instances of prior non-conviction misconduct probative of untruthfulness ldquoin order to attack the witnessrsquos character for truthfulnessrdquo FED R EVID 608(b)

The courtrsquos decision to allow cross-examination about the guardrsquos prior dishonest behavior depends on the probative value of such evidence balanced against the danger of unfair prejudice to the guard or any other Rule 403 concern FED R EVID 403 Here the guardrsquos false statement on his reacutesumeacute that he obtained a degree in Criminal Justice is highly probative of his untruthfulness because it grossly misrepresents his actual academic record was made recently and was made with the intent to deceive Because the probative value of this evidence is very strong and is not substantially outweighed by any Rule 403 concerns cross-examination of the guard on this topic should be permitted The court may also consider it fair to permit this cross-examination of the guard on these matters assuming that one or more of the inmatersquos prior convictions have been admitted to impeach his credibility

Point Two(b) (15) The court should exclude extrinsic evidence of the guardrsquos non-conviction misconduct even if the guard denies wrongdoing or refuses to answer questions about the matter

Although Rule 608(b) allows cross-examination about specific instances of prior misconduct probative of untruthfulness ldquoextrinsic evidencerdquo offered to prove such misconduct is not admissible FED R EVID 608(b) The rationale for this rule is that allowing the introduction of extrinsic evidence of prior misconduct by witnesses when these acts are relevant only to the witnessesrsquo truthfulness and not to the main issues in the case would create too great a risk of confusing the jury and unduly delaying the trial The court does not have discretion to admit this extrinsic evidence See eg United States v Elliot 89 F3d 1360 1368 (8th Cir 1996)

30

Evidence Analysis

Here the inmatersquos counsel may cross-examine the guard about the false statement on his reacutesumeacute However the inmatersquos counsel must accept the guardrsquos response Even if the guard denies wrongdoing or refuses to answer questions about the matter the inmatersquos counsel cannot introduce the guardrsquos reacutesumeacute or the transcript from the local college to prove the guardrsquos misconduct

31

CORPORATIONS ANALYSIS (Corporations VA2 IX)

ANALYSIS

Legal Problems

(1) Do shareholders have the authority to amend a corporationrsquos bylaws with respect to director nominations

(2) Do board-approved bylaws on a particular subject here nomination of directors preempt subsequent conflicting bylaw amendments by shareholders

(3) Is a suit challenging both managementrsquos refusal to include the proposed bylaw amendment in Megarsquos proxy statement and the boardrsquos amendment of the bylaws dealing with nomination of directors a direct or derivative suit

DISCUSSION

Summary

The voting and litigation rights of the shareholders of Mega are subject to the provisions of the Model Business Corporations Act (MBCA)

The investorrsquos proposed bylaw provision is not inconsistent with state law Under the MBCA shareholders may amend the bylaws when the amendment deals with a proper matter for the corporationrsquos bylaws such as procedures for nominating directors

The Mega boardrsquos bylaw amendment does not preempt the investorrsquos proposed bylaw provision or the Mega shareholdersrsquo power to approve it While shareholders can limit the boardrsquos power to amend or repeal the bylaws the board cannot limit the shareholdersrsquo power

Whether the investor must make a demand on Megarsquos board depends on how the investor frames its claim If the investor claims a violation of shareholder voting rights the claim is direct and pre-suit demand on the board is not required If on the other hand the investor claims that the directors violated their fiduciary duties by amending the bylaws to entrench themselves the claim is derivative and a pre-suit demand is required

Point One (30) Shareholders may amend the corporationrsquos bylaws where the proposed bylaw provision relates to procedural matters typically included in the bylaws such as the nomination of directors

Internal affairs of the corporation such as the conduct of shareholder meetings and election of directors are subject to the corporate law of the state of incorporation See McDermott Inc v Lewis 531 A2d 206 (Del 1987) (applying law of jurisdiction where corporation was incorporated in case involving voting rights) This statersquos corporate statute is modeled on the MBCA

Under the MBCA ldquoshareholders may amend the corporationrsquos bylawsrdquo MBCA sect 1020(a) Thus the only question is whether the bylaws can specify the procedures for shareholder nomination of directors

32

Corporations Analysis

The MBCA states that the bylaws ldquomay contain any provision that is not inconsistent with law or the articles of incorporationrdquo MBCA sect 206(b) In addition the MBCA was revised in 2009 to address shareholder nomination of directors in public corporations (known as ldquoproxy accessrdquo) and specifies that the bylaws ldquomay contain a requirement that the corporation include in its [proxy materials] one or more individuals nominated by a shareholderrdquo MBCA sect 206(c)(1) see Committee on Corporate Laws ABA Section of Business Law Report on the Roles of Boards of Directors and Shareholders of Publicly Owned Corporations and Changes to the Model Business Corporations ActmdashAdoption of Shareholder Proxy Access Amendments to Chapters 2 and 10 65 BUS LAWYER 1105 (2010)

The inclusion of director-nomination procedures in the bylaws is consistent with practice and is recognized by the Delaware courts whose views on corporate law carry significant weight Typically the procedures for nomination of directors are found in the bylaws See 1 COX amp HAZEN TREATISE ON THE LAW OF CORPORATIONS sect 312 (3d ed 2011) see also 4 FLETCHER CORP FORMS ANN PART III ch 21 (2013) (including sample bylaws that permit nomination of directors by shareholders) The Delaware Supreme Court has confirmed that the bylaws may ldquodefine the process and proceduresrdquo for director elections See CA Inc v AFSCME Employees Pension Plan 953 A2d 227 (Del 2008) (concluding that bylaw amendment requiring reimbursement of election expenses to certain successful shareholder nominators is ldquoproper subjectrdquo under Delaware law)

[NOTE The question of the proper scope of the bylaws can be answered using the more general MBCA sect 206(b) or the 2009 MBCA revision adding sect 206(c)(1) (adopted in CT ME VA) In addition some examinees might raise the point that shareholder proposals may not compel the board to take action such as by including shareholder nominations in the companyrsquos proxy materials on the theory that the ldquobusiness and affairsrdquo of the corporation are to be managed by the board See MBCA sect 801(b) Although shareholders are generally limited to adopting precatory resolutions that recommend or encourage board action this limitation does not apply when shareholders have specific authority to take binding action on their ownmdashsuch as to amend the bylaws]

Point Two (30) Shareholders can amend (or repeal) board-approved bylaws Further shareholders can limit the boardrsquos power to later amend and repeal a shareholder-approved bylaw

Under the MBCA shareholders have the power to amend the bylaws See Point One The board shares this power with the shareholders unless (1) the corporationrsquos articles ldquoreserve that power exclusively to the shareholdersrdquo or (2) ldquothe shareholders in amending repealing or adopting a bylaw expressly provide that the board of directors may not amend repeal or reinstate that bylawrdquo See MBCA sect 1020(b)

Shareholder-approved bylaw provisions can amend or repeal existing bylaw provisions whether originally approved by the board or by shareholders See ALAN R PALMITER CORPORATIONS EXAMPLES AND EXPLANATIONS sect 713 (7th ed 2012) Thus the Mega boardrsquos bylaw amendmentmdashwhich set more demanding thresholds for shareholder nomination of directors than the investorrsquos proposed bylaw provisionmdashwould be superseded (repealed) if Megarsquos shareholders were to approve the investorrsquos proposal

Further a shareholder-approved bylaw generally can limit the power of the board to later amend or repeal it See MBCA sect 1020(b)(2) Thus if Megarsquos shareholders approved the bylaw

33

Corporations Analysis

provision proposed by the investor Megarsquos board could not repeal the provision because it includes a ldquono board repealrdquo clause

The revision to the MBCA in 2009 dealing with shareholder proxy access does not change this conclusion That revision specifies that a shareholder-approved bylaw dealing with director nominations may not limit the boardrsquos power to amend add or repeal ldquoany procedure or condition to such a bylaw in order to provide for a reasonable practicable and orderly processrdquo MBCA sect 206(d) Thus according to the revision if shareholders approve a bylaw amendment that limits further board changes the board would nonetheless retain the power to ldquotinkerrdquo with the bylaw to safeguard the voting process but could not repeal the shareholder-approved bylaw The Official Comment to MBCA sect 206(d) makes clear that the revision is ldquonot intended to allow the board of directors to frustrate the purpose of the shareholder-adopted proxy access provisionrdquo Thus if Megarsquos shareholders were to approve the bylaw provision proposed by the investor Megarsquos board could only amend the provision regarding its procedures or conditions in a manner consistent with its purpose of permitting proxy access for Megarsquos shareholders

[NOTE The boardrsquos attempted interference with a shareholder voting initiative may also have been a violation of the directorsrsquo fiduciary duties See Blasius Indus Inc v Atlas Corp 564 A2d 651 (Del Ch 1988) (finding that directors breached their fiduciary duties by amending bylaws and expanding size of board to thwart insurgentrsquos plan to amend bylaws and seat a majority of new directors) The call however asks examinees to consider whether shareholders or the board have ldquoprecedencerdquo over amending the corporate bylaws Thus an examineersquos answer should be framed in terms of ldquopowerrdquo and not ldquodutyrdquo]

Point Three (40) The investor need not make a demand on the board if the investor states a direct claim such as an allegation that the board interfered with the investorrsquos right to amend the bylaws But the investor must make a demand on the board if the investor states a derivative claim (on behalf of the corporation) such as an allegation that the directors sought to entrench themselves by interfering with the proposed proxy access

The MBCA generally requires that shareholders make a demand on the board of directors before initiation of a derivative suit MBCA sect 742 (shareholder may not bring derivative proceeding until written demand has been made on corporation and 90 days have expired) A derivative suit is essentially two suits in one where the plaintiff-shareholder seeks to bring on behalf of the corporation a claim that vindicates corporate rights usually based on violation of fiduciary duties PALMITER supra sect 1811 (6th ed 2009) The demand permits the board to investigate the situation identified by the shareholder and take suitable action No demand on the board is required however if the shareholder brings a direct suit to vindicate the shareholderrsquos own rights not those of the corporation

Is the suit brought by the investor derivative or direct The MBCA defines a ldquoderivative proceedingrdquo as one brought ldquoin the right of a domestic corporationrdquo MBCA sect 740(1) Thus the answer to how the investorrsquos suit should be characterized turns on what rights the investor seeks to vindicate If the investor frames its claim as one of fiduciary breach by directorsmdashfor example for failing to become adequately informed about voting procedures or for seeking to entrench themselves in office by manipulating the voting structure to avoid a shareholder insurgencymdashthen the suit is ldquoderivativerdquo and the investor must make a demand on the board See MBCA Ch 7 Subch D Introductory Comment (ldquothe derivative suit has historically been the principal method of challenging allegedly illegal action by managementrdquo)

34

Corporations Analysis

If however the investor frames its claim as one to vindicate shareholder rights the suit is direct and no demand is required For many courts the direct-derivative question turns on who is injured and who is to receive the relief sought by the plaintiff-shareholders See Tooley v Donaldson Lufkin amp Jenrette Inc 845 A2d 1031 (Del 2004) (characterizing a merger-delay claim as direct because delay of merger only harmed shareholders not corporation) Thus if the investor claims that managementrsquos refusal to include its proposed bylaw amendment in the corporationrsquos proxy materials violates its shareholder rights to initiate corporate governance reforms the suit will be direct Courts have not questioned the ability of shareholders to bring direct suits challenging board action to exclude their proposed bylaw amendments from the corporationrsquos proxy materials See JANA Master Fund Ltd v CNET Networks Inc 954 A2d 335 (Del Ch 2008) (upholding shareholderrsquos direct challenge to boardrsquos interpretation of advance-notice bylaw) Chesapeake Corp v Shore 771 A2d 293 (Del Ch 2000) (upholding shareholderrsquos direct challenge to actions by board that effectively prevented it from proposing bylaw amendments in contest for control)

Is the way that the investor frames its claim conclusive Courts have permitted shareholder-plaintiffs to challenge a transaction in a direct suit even though the same transaction could also be challenged as a fiduciary breach See Eisenberg v Flying Tiger Line Inc 451 F2d 267 (2d Cir 1971) (permitting direct suit challenging a corporate reorganization as a dilution of shareholder voting power even though reorganization may have involved conflicts of interest and thus constituted a fiduciary breach) Thus the investorrsquos choice to pursue a claim challenging the legality of managementrsquos decision to exclude the investorrsquos proposal from the corporationrsquos proxy materialsmdashrather than a possible breach of fiduciary dutymdashis likely to be respected See 3 COX amp HAZEN supra sect 153 (describing situations in which a claim can be framed as derivative or direct)

[NOTE Some issues under Delaware corporate law regarding pre-suit demand are not relevant here For example whether the Mega directors are independent and disinterested is not relevant to the MBCA requirement of a pre-suit demand As the Official Comment to MBCA sect 742 points out the MBCArsquos requirement of ldquouniversal demandrdquo gives the board ldquothe opportunity to reexamine the act complained of in the light of a potential lawsuit and take corrective actionrdquo even when the directors might be non-independent or have conflicts of interest

Nor is it relevant to the MBCA pre-suit demand requirement that the statutory 90-day waiting period may be onerous The first paragraph of MBCA sect 742 requires a pre-suit demand without exception the second paragraph of the section imposes a 90-day waiting period before a derivative suit may be brought which can be shortened if the board rejects the demand or ldquoirreparable injury to the corporation would result by waiting for the expiration of the 90-day periodrdquo The call as written asks only whether a pre-suit demand should be made and does not ask examinees to address whether the post-demand waiting period should be shortened under the ldquoirreparable injuryrdquo standard]

35

National Conference of Bar Examiners 302 South Bedford Street | Madison WI 53703-3622 Phone 608-280-8550 | Fax 608-280-8552 | TDD 608-661-1275

wwwncbexorg e-mail contactncbexorg

  • Preface
  • Description of the MEE
  • Instructions
  • July 2014 Questions
    • CRIMINAL LAW AND PROCEDURE QUESTION
    • CONTRACTS QUESTION
    • FAMILY LAW QUESTION
    • FEDERAL CIVIL PROCEDURE QUESTION
    • EVIDENCE QUESTION
    • CORPORATIONS QUESTION
      • July 2014 Analyses
        • CRIMINAL LAW AND PROCEDURE ANALYSIS
        • CONTRACTS ANALYSIS
        • FAMILY LAW ANALYSIS
        • FEDERAL CIVIL PROCEDURE ANALYSIS
        • EVIDENCE ANALYSIS
        • CORPORATIONS ANALYSIS
            • ltlt13 ASCII85EncodePages false13 AllowTransparency false13 AutoPositionEPSFiles true13 AutoRotatePages None13 Binding Left13 CalGrayProfile (Dot Gain 20)13 CalRGBProfile (sRGB IEC61966-21)13 CalCMYKProfile (US Web Coated 050SWOP051 v2)13 sRGBProfile (sRGB IEC61966-21)13 CannotEmbedFontPolicy Error13 CompatibilityLevel 1413 CompressObjects Tags13 CompressPages true13 ConvertImagesToIndexed true13 PassThroughJPEGImages true13 CreateJobTicket false13 DefaultRenderingIntent Default13 DetectBlends true13 DetectCurves 0000013 ColorConversionStrategy CMYK13 DoThumbnails false13 EmbedAllFonts true13 EmbedOpenType false13 ParseICCProfilesInComments true13 EmbedJobOptions true13 DSCReportingLevel 013 EmitDSCWarnings false13 EndPage -113 ImageMemory 104857613 LockDistillerParams false13 MaxSubsetPct 10013 Optimize true13 OPM 113 ParseDSCComments true13 ParseDSCCommentsForDocInfo true13 PreserveCopyPage true13 PreserveDICMYKValues true13 PreserveEPSInfo true13 PreserveFlatness true13 PreserveHalftoneInfo false13 PreserveOPIComments true13 PreserveOverprintSettings true13 StartPage 113 SubsetFonts true13 TransferFunctionInfo Apply13 UCRandBGInfo Preserve13 UsePrologue false13 ColorSettingsFile ()13 AlwaysEmbed [ true13 ]13 NeverEmbed [ true13 ]13 AntiAliasColorImages false13 CropColorImages true13 ColorImageMinResolution 30013 ColorImageMinResolutionPolicy OK13 DownsampleColorImages true13 ColorImageDownsampleType Bicubic13 ColorImageResolution 30013 ColorImageDepth -113 ColorImageMinDownsampleDepth 113 ColorImageDownsampleThreshold 15000013 EncodeColorImages true13 ColorImageFilter DCTEncode13 AutoFilterColorImages true13 ColorImageAutoFilterStrategy JPEG13 ColorACSImageDict ltlt13 QFactor 01513 HSamples [1 1 1 1] VSamples [1 1 1 1]13 gtgt13 ColorImageDict ltlt13 QFactor 01513 HSamples [1 1 1 1] VSamples [1 1 1 1]13 gtgt13 JPEG2000ColorACSImageDict ltlt13 TileWidth 25613 TileHeight 25613 Quality 3013 gtgt13 JPEG2000ColorImageDict ltlt13 TileWidth 25613 TileHeight 25613 Quality 3013 gtgt13 AntiAliasGrayImages false13 CropGrayImages true13 GrayImageMinResolution 30013 GrayImageMinResolutionPolicy OK13 DownsampleGrayImages true13 GrayImageDownsampleType Bicubic13 GrayImageResolution 30013 GrayImageDepth -113 GrayImageMinDownsampleDepth 213 GrayImageDownsampleThreshold 15000013 EncodeGrayImages true13 GrayImageFilter DCTEncode13 AutoFilterGrayImages true13 GrayImageAutoFilterStrategy JPEG13 GrayACSImageDict ltlt13 QFactor 01513 HSamples [1 1 1 1] VSamples [1 1 1 1]13 gtgt13 GrayImageDict ltlt13 QFactor 01513 HSamples [1 1 1 1] VSamples [1 1 1 1]13 gtgt13 JPEG2000GrayACSImageDict ltlt13 TileWidth 25613 TileHeight 25613 Quality 3013 gtgt13 JPEG2000GrayImageDict ltlt13 TileWidth 25613 TileHeight 25613 Quality 3013 gtgt13 AntiAliasMonoImages false13 CropMonoImages true13 MonoImageMinResolution 120013 MonoImageMinResolutionPolicy OK13 DownsampleMonoImages true13 MonoImageDownsampleType Bicubic13 MonoImageResolution 120013 MonoImageDepth -113 MonoImageDownsampleThreshold 15000013 EncodeMonoImages true13 MonoImageFilter CCITTFaxEncode13 MonoImageDict ltlt13 K -113 gtgt13 AllowPSXObjects false13 CheckCompliance [13 None13 ]13 PDFX1aCheck false13 PDFX3Check false13 PDFXCompliantPDFOnly false13 PDFXNoTrimBoxError true13 PDFXTrimBoxToMediaBoxOffset [13 00000013 00000013 00000013 00000013 ]13 PDFXSetBleedBoxToMediaBox true13 PDFXBleedBoxToTrimBoxOffset [13 00000013 00000013 00000013 00000013 ]13 PDFXOutputIntentProfile ()13 PDFXOutputConditionIdentifier ()13 PDFXOutputCondition ()13 PDFXRegistryName ()13 PDFXTrapped False1313 CreateJDFFile false13 Description ltlt13 ARA ltFEFF06270633062A062E062F0645002006470630064700200627064406250639062F0627062F0627062A002006440625064606340627062100200648062B062706260642002000410064006F00620065002000500044004600200645062A064806270641064206290020064406440637062806270639062900200641064A00200627064406450637062706280639002006300627062A0020062F0631062C0627062A002006270644062C0648062F0629002006270644063906270644064A0629061B0020064A06450643064600200641062A062D00200648062B0627062606420020005000440046002006270644064506460634062306290020062806270633062A062E062F062706450020004100630072006F0062006100740020064800410064006F006200650020005200650061006400650072002006250635062F0627063100200035002E0030002006480627064406250635062F062706310627062A0020062706440623062D062F062B002E0635062F0627063100200035002E0030002006480627064406250635062F062706310627062A0020062706440623062D062F062B002Egt13 BGR 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 CHS ltFEFF4f7f75288fd94e9b8bbe5b9a521b5efa7684002000410064006f006200650020005000440046002065876863900275284e8e9ad88d2891cf76845370524d53705237300260a853ef4ee54f7f75280020004100630072006f0062006100740020548c002000410064006f00620065002000520065006100640065007200200035002e003000204ee553ca66f49ad87248672c676562535f00521b5efa768400200050004400460020658768633002gt13 CHT ltFEFF4f7f752890194e9b8a2d7f6e5efa7acb7684002000410064006f006200650020005000440046002065874ef69069752865bc9ad854c18cea76845370524d5370523786557406300260a853ef4ee54f7f75280020004100630072006f0062006100740020548c002000410064006f00620065002000520065006100640065007200200035002e003000204ee553ca66f49ad87248672c4f86958b555f5df25efa7acb76840020005000440046002065874ef63002gt13 CZE 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 DAN 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 DEU ltFEFF00560065007200770065006e00640065006e0020005300690065002000640069006500730065002000450069006e007300740065006c006c0075006e00670065006e0020007a0075006d002000450072007300740065006c006c0065006e00200076006f006e002000410064006f006200650020005000440046002d0044006f006b0075006d0065006e00740065006e002c00200076006f006e002000640065006e0065006e002000530069006500200068006f006300680077006500720074006900670065002000500072006500700072006500730073002d0044007200750063006b0065002000650072007a0065007500670065006e0020006d00f60063006800740065006e002e002000450072007300740065006c006c007400650020005000440046002d0044006f006b0075006d0065006e007400650020006b00f6006e006e0065006e0020006d006900740020004100630072006f00620061007400200075006e0064002000410064006f00620065002000520065006100640065007200200035002e00300020006f0064006500720020006800f600680065007200200067006500f600660066006e00650074002000770065007200640065006e002egt13 ESP 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 ETI 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 FRA 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 GRE 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 HEB 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 HRV (Za stvaranje Adobe PDF dokumenata najpogodnijih za visokokvalitetni ispis prije tiskanja koristite ove postavke Stvoreni PDF dokumenti mogu se otvoriti Acrobat i Adobe Reader 50 i kasnijim verzijama)13 HUN 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 ITA 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 JPN ltFEFF9ad854c18cea306a30d730ea30d730ec30b951fa529b7528002000410064006f0062006500200050004400460020658766f8306e4f5c6210306b4f7f75283057307e305930023053306e8a2d5b9a30674f5c62103055308c305f0020005000440046002030d530a130a430eb306f3001004100630072006f0062006100740020304a30883073002000410064006f00620065002000520065006100640065007200200035002e003000204ee5964d3067958b304f30533068304c3067304d307e305930023053306e8a2d5b9a306b306f30d530a930f330c8306e57cb30818fbc307f304c5fc59808306730593002gt13 KOR ltFEFFc7740020c124c815c7440020c0acc6a9d558c5ec0020ace0d488c9c80020c2dcd5d80020c778c1c4c5d00020ac00c7a50020c801d569d55c002000410064006f0062006500200050004400460020bb38c11cb97c0020c791c131d569b2c8b2e4002e0020c774b807ac8c0020c791c131b41c00200050004400460020bb38c11cb2940020004100630072006f0062006100740020bc0f002000410064006f00620065002000520065006100640065007200200035002e00300020c774c0c1c5d0c11c0020c5f40020c2180020c788c2b5b2c8b2e4002egt13 LTH 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 LVI 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 NLD (Gebruik deze instellingen om Adobe PDF-documenten te maken die zijn geoptimaliseerd voor prepress-afdrukken van hoge kwaliteit De gemaakte PDF-documenten kunnen worden geopend met Acrobat en Adobe Reader 50 en hoger)13 NOR 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 POL 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 PTB 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 RUM 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 RUS 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 SKY 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 SLV 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 SUO 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 SVE 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 TUR ltFEFF005900fc006b00730065006b0020006b0061006c006900740065006c0069002000f6006e002000790061007a006401310072006d00610020006200610073006b013100730131006e006100200065006e0020006900790069002000750079006100620069006c006500630065006b002000410064006f006200650020005000440046002000620065006c00670065006c0065007200690020006f006c0075015f007400750072006d0061006b0020006900e70069006e00200062007500200061007900610072006c0061007201310020006b0075006c006c0061006e0131006e002e00200020004f006c0075015f0074007500720075006c0061006e0020005000440046002000620065006c00670065006c0065007200690020004100630072006f006200610074002000760065002000410064006f00620065002000520065006100640065007200200035002e003000200076006500200073006f006e0072006100730131006e00640061006b00690020007300fc007200fc006d006c00650072006c00650020006100e70131006c006100620069006c00690072002egt13 UKR 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 ENU (Use these settings to create Adobe PDF documents best suited for high-quality prepress printing Created PDF documents can be opened with Acrobat and Adobe Reader 50 and later)13 gtgt13 Namespace [13 (Adobe)13 (Common)13 (10)13 ]13 OtherNamespaces [13 ltlt13 AsReaderSpreads false13 CropImagesToFrames true13 ErrorControl WarnAndContinue13 FlattenerIgnoreSpreadOverrides false13 IncludeGuidesGrids false13 IncludeNonPrinting false13 IncludeSlug false13 Namespace [13 (Adobe)13 (InDesign)13 (40)13 ]13 OmitPlacedBitmaps false13 OmitPlacedEPS false13 OmitPlacedPDF false13 SimulateOverprint Legacy13 gtgt13 ltlt13 AddBleedMarks false13 AddColorBars false13 AddCropMarks false13 AddPageInfo false13 AddRegMarks false13 ConvertColors ConvertToCMYK13 DestinationProfileName ()13 DestinationProfileSelector DocumentCMYK13 Downsample16BitImages true13 FlattenerPreset ltlt13 PresetSelector MediumResolution13 gtgt13 FormElements false13 GenerateStructure false13 IncludeBookmarks false13 IncludeHyperlinks false13 IncludeInteractive false13 IncludeLayers false13 IncludeProfiles false13 MultimediaHandling UseObjectSettings13 Namespace [13 (Adobe)13 (CreativeSuite)13 (20)13 ]13 PDFXOutputIntentProfileSelector DocumentCMYK13 PreserveEditing true13 UntaggedCMYKHandling LeaveUntagged13 UntaggedRGBHandling UseDocumentProfile13 UseDocumentBleed false13 gtgt13 ]13gtgt setdistillerparams13ltlt13 HWResolution [2400 2400]13 PageSize [612000 792000]13gtgt setpagedevice13

Page 8: July 2014 MEE Questions and Analyses - NCBE...This publication includes the questions and analyses from the July 2014 MEE. (In the actual test, the questions are simply numbered rather

Criminal Law and Procedure Question

1 Did the detective violate the suspectrsquos Sixth Amendment right to counsel when he questioned the suspect in the absence of the lawyer Explain

2 Under Miranda did the suspect effectively invoke his right to counsel Explain

3 Was the suspectrsquos waiver of his Miranda rights valid Explain

4

CONTRACTS QUESTION

A music conservatory has two concert halls One concert hall had a pipe organ that was in poor repair and the other had no organ The conservatory decided to repair the existing organ and buy a new organ for the other concert hall After some negotiation the conservatory entered into two contracts with a business that both repairs and sells organs Under one contract the business agreed to repair the existing pipe organ for the conservatory for $100000 The business would usually charge a higher price for a project of this magnitude but the business agreed to this price because the conservatory agreed to prepay the entire amount Under the other contract the business agreed to sell a new organ to the conservatory for the other concert hall for $225000 As with the repair contract the business agreed to a low sales price because the conservatory agreed to prepay the entire amount Both contracts were signed on January 3 and the conservatory paid the business a total of $325000 that day

Two weeks later before the business had commenced repair of the existing organ the business suffered serious and unanticipated financial reversals The chief financial officer for the business contacted the conservatory and said

Bad news We had an unexpected liability and as a result are in a real cash crunch In fact even though we havenrsquot acquired the new organ from our supplier or started repair of your existing organ wersquove already spent the cash you gave us and we have no free cash on hand Wersquore really sorry but wersquore in a fix I think that we can find a way to perform both contracts but not at the original prices If you agree to pay $60000 more for the repair and $40000 more for the new organ we can probably find financing to finish everything If you donrsquot agree to pay us the extra money I doubt that we will ever be able to perform either contract and yoursquoll be out the money you already paid us

After receiving this unwelcome news the conservatory agreed to pay the extra amounts provided that the extra amount on each contract would be paid only upon completion of the businessrsquos obligations under that contract The business agreed to this arrangement and the parties quickly signed documents reflecting these changes to each contract The business then repaired the existing organ delivered the new organ and demanded payment of the additional $100000

The conservatory now has refused to pay the business the additional amounts for the repair and the new organ

1 Must the conservatory pay the additional $60000 for the organ repair Explain

2 Must the conservatory pay the additional $40000 for the new organ Explain

5

FAMILY LAW QUESTION

In 1994 a man and a woman were married in State A

In 1998 their daughter was born in State A

In 2010 the family moved to State B

In 2012 the husband and wife divorced in State B Under the terms of the divorce decree

(a) the husband and wife share legal and physical custody of their daughter (b) the husband must pay the wife $1000 per month in child support until their daughter reaches age 18 (c) the marital residence was awarded to the wife with the proviso that if it is sold before the daughter reaches age 18 the husband will receive 25 of the net sale proceeds remaining after satisfaction of the mortgage on the residence and (d) the remaining marital assets were divided between the husband and the wife equally

Six months ago the husband was offered a job in State A that pays significantly less than his job in State B but provides him with more responsibilities and much better promotion opportunities The husband accepted the job in State A and moved from State B back to State A

Since returning to State A the husband has not paid child support because due to his lower salary he has had insufficient funds to meet all his obligations

One month ago the wife sold the marital home netting $10000 after paying off the mortgage She then moved to a smaller residence The husband believes that he should receive more than 25 of the net sale proceeds given his financial difficulties

Last week when the wife brought the daughter to the husbandrsquos State A home for a weekend visit the husband served the wife with a summons in a State A action to modify the support and marital-residence-sale-proceeds provisions of the State B divorce decree The husband brought the action in the State A court that adjudicates all domestic relations issues

1 Does the State A court have jurisdiction to modify (a) the child support provision of the State B divorce decree Explain (b) the marital-residence-sale-proceeds provision of the State B divorce decree

Explain

2 On the merits could the husband obtain (a) retroactive modification of his child support obligation to the daughter Explain (b) prospective modification of his child support obligation to the daughter Explain (c) modification of the marital-residence-sale-proceeds provision of the State B

divorce decree Explain

6

FEDERAL CIVIL PROCEDURE QUESTION

The United States Forest Service (USFS) manages public lands in national forests including the Scenic National Forest Without conducting an environmental evaluation or preparing an environmental impact statement the USFS approved a development project in the Scenic National Forest that required the clearing of 5000 acres of old-growth forest The trees in the forest are hundreds of years old and the forest is home to a higher concentration of wildlife than can be found anywhere else in the western United States

The USFS solicited bids from logging companies to harvest the trees on the 5000 acres of forest targeted for clearing and it ultimately awarded the logging contract to the company that had submitted the highest bid for the trees However the USFS has not yet issued the company a logging permit Once it does so the company intends to begin cutting down trees immediately

A nonprofit organization whose mission is the preservation of natural resources has filed suit in federal district court against the USFS The nonprofit alleges that the USFS violated the National Environmental Policy Act (NEPA) by failing to prepare an environmental impact statement for the proposed logging project Among other remedies the nonprofit seeks a permanent injunction barring the USFS from issuing a logging permit to the logging company until an adequate environmental impact statement is completed The nonprofit believes that the logging project would destroy important wildlife habitat and thereby cause serious harm to wildlife in the Scenic National Forest including some endangered species

Assume that federal subject-matter jurisdiction is available that the nonprofit has standing to bring this action and that venue is proper

1 If the logging company seeks to join the litigation as a party must the federal district court allow it to do so as a matter of right Explain

2 What types of relief could the nonprofit seek to stop the USFS from issuing a logging permit during the pendency of the action what must the nonprofit demonstrate to obtain that relief and is the federal district court likely to grant that relief Explain

7

EVIDENCE QUESTION

A prison inmate has filed a civil rights lawsuit against a guard at the prison alleging that the guard violated the inmatersquos constitutional rights during an altercation The inmate and the guard are the only witnesses to this altercation They have provided contradictory reports about what occurred

The trial will be before a jury The inmate plans to testify at trial The guardrsquos counsel has moved for leave to impeach the inmate with the following

(a) Twelve years ago the inmate was convicted of felony distribution of marijuana He served a three-year prison sentence which began immediately after he was convicted He served his full sentence and was released from prison nine years ago (b) Eight years ago the inmate pleaded guilty to perjury a misdemeanor punishable by up to one year in jail He paid a $5000 fine (c) Seven years ago the inmate was convicted of felony sexual assault of a child and is currently serving a 10-year prison sentence for the crime The victim was the inmatersquos daughter who was 13 years old at the time of the assault

The inmatersquos counsel objects to the admission of any evidence related to these three convictions and to any cross-examination based on this evidence

The guard also plans to testify at trial The inmatersquos counsel has moved for leave to impeach the guard with the following

Last year the guard applied for a promotion to prison supervisor The guard submitted a reacutesumeacute to the state that indicated that he had been awarded a BA in Criminal Justice from a local college An official copy of the guardrsquos academic transcript from that college indicates that the guard dropped out after his first semester and did not receive a degree

The guardrsquos counsel objects to the admission of this evidence and to any cross-examination based on this evidence

The transcript and the reacutesumeacute have been properly authenticated The trial will be held in a jurisdiction that has adopted all of the Federal Rules of Evidence

1 What evidence if any proffered by the guard to impeach the inmate should be admitted Explain

2 What evidence if any proffered by the inmate to impeach the guard should be admitted Explain

8

CORPORATIONS QUESTION

Mega Inc is a publicly traded corporation incorporated in a state whose corporate statute is modeled on the Model Business Corporation Act (MBCA) Megarsquos articles of incorporation do not address the election of directors or amendment of the bylaws by shareholders

Well within the deadline for the submission of shareholder proposals for the upcoming annual shareholdersrsquo meeting an investor who was a large and long-standing shareholder of Mega submitted a proposed amendment to Megarsquos bylaws The proposal which the investor asked to be included in the corporationrsquos proxy materials and voted on at the upcoming shareholdersrsquo meeting read as follows

Section 20 The Corporation shall include in its proxy materials (including the proxy ballot) for a shareholdersrsquo meeting at which directors are to be elected the name of a person nominated for election to the Board of Directors by a shareholder or group of shareholders that beneficially have owned 3 or more of the Corporationrsquos outstanding common stock for at least one year

This Section shall supersede any inconsistent provision in these Bylaws and may not be amended or repealed by the Board of Directors without shareholder approval

Megarsquos management decided to exclude the investorrsquos proposal from the corporationrsquos proxy materials and explained its reasons in a letter to the investor

The investorrsquos proposed bylaw provision would be inconsistent with relevant state law because the Board of Directors has the authority to manage the business and affairs of the Corporation Generally shareholders lack the authority to interfere with corporate management by seeking to create a method for the nomination and election of directors inconsistent with the method chosen by the Board of Directors

Furthermore at its most recent meeting the Board of Directors unanimously approved an amendment to the Corporationrsquos bylaws that provides for proxy access for director nominations by a shareholder or a group of shareholders holding at least 10 of the Corporationrsquos voting shares for at least three years This procedure takes precedence over any nomination methods that might be sought or approved by shareholders

The investor is considering bringing a suit challenging managementrsquos refusal to include the investorrsquos proposed bylaw provision and challenging the boardrsquos amendment of the bylaws at its recent meeting

1 Is the investorrsquos proposed bylaw provision inconsistent with state law Explain

2 If the investorrsquos proposed bylaw provision were approved by the shareholders would the bylaw amendment previously approved by the board take precedence over the investorrsquos proposed bylaw provision Explain

3 Must the investor make a demand on Megarsquos board of directors before bringing suit Explain

9

July 2014 MEE

ANALYSES

Contracts Family Law

Criminal Law and Procedure

Federal Civil Procedure Evidence

Corporations

CRIMINAL LAW AND PROCEDURE ANALYSIS (Criminal Law and Procedure VA B D)

ANALYSIS

Legal Problems

(1) Did the detective violate the suspectrsquos Sixth Amendment right to counsel when he questioned the suspect about the burglaries without the lawyer present given that the lawyer represented the suspect in an unrelated criminal matter

(2) Under Miranda did the suspect effectively invoke his right to counsel when he said ldquoI think I want my lawyer here before I talk to yourdquo

(3) Was the suspectrsquos waiver of his right to remain silent under Miranda valid

DISCUSSION

Summary

The Sixth Amendment right to counsel as applied to states through the Fourteenth Amendment is offense-specific Although the suspect had an attorney representing him on his pending assault charge he had no Sixth Amendment right to the assistance of counsel with respect to the five uncharged burglaries because formal adversarial proceedings had not yet commenced on those charges The suspectrsquos Sixth Amendment right to counsel was not violated by the detectiversquos failure to inform him that the lawyer was present or of the lawyerrsquos demands

However a person undergoing custodial interrogation also has an independent constitutional right to counsel during custodial interrogation under Miranda When a suspect invokes his right to counsel under Miranda custodial interrogation must immediately cease for a period of at least 14 days However the invocation of the right to counsel must be unambiguous and clearly convey that the suspect has requested counsel Here because the suspectrsquos statement ldquoI think I want my lawyer here before I talk to yourdquo was ambiguous he did not invoke his Miranda right to counsel

A waiver of rights must be knowing intelligent and voluntary Here the suspect waived his right to remain silent under Miranda when he signed the waiver form The fact that the detective did not correct the suspectrsquos assumption that the lawyer would need to drive to the jailmdashby telling him that the lawyer was in the waiting room and was demanding to see himmdashdid not affect the validity of the suspectrsquos waiver

Point One (35) The suspectrsquos Sixth Amendment right to counsel was not violated because the right does not attach on new charges until formal adversarial judicial proceedings have commenced on those charges

The Sixth Amendment as applied to the states through the Fourteenth Amendment provides that ldquo[i]n all criminal prosecutions the accused shall enjoy the right to have the Assistance of Counsel for his defenserdquo The right to counsel does not attach with respect to particular charges until formal adversarial judicial proceedings have commenced (ie ldquoat or after the initiation of

13

Criminal Law and Procedure Analysis

adversary judicial criminal proceedingsmdashwhether by way of formal charge preliminary hearing indictment information or arraignment [or in some states arrest warrant]rdquo McNeil v Wisconsin 501 US 171 175 (1991) (internal quotations omitted)) Once a suspectrsquos Sixth Amendment right to counsel has attached any attempts to ldquodeliberately elicitrdquo statements from him in the absence of his attorney violate the Sixth Amendment See Massiah v United States 377 US 201 (1964) Brewer v Williams 430 US 387 (1977)

The Sixth Amendment right to counsel is charge- or offense -specific Representation by counsel in one prosecution does not in itself guarantee counsel for uncharged offenses See McNeil 501 US at 175 Texas v Cobb 532 US 162 (2001) Here the suspectrsquos Sixth Amendment right to counsel had attached only for the pending aggravated assault charge The suspectrsquos right to counsel for the aggravated assault case did not guarantee counsel for the five unrelated and uncharged burglaries that were the subject of the detectiversquos interrogation Thus because formal adversarial judicial proceedings against the suspect for the uncharged burglaries had not begun he had no Sixth Amendment right to counsel

Finally the detectiversquos failure to inform the suspect of the lawyerrsquos presence and demands to speak with him does not implicate the suspectrsquos Sixth Amendment right to counsel which had not yet attached See id Moran v Burbine 475 US 412 428ndash31 (1986)

Point Two (30) The suspect did not effectively invoke his right to counsel under Miranda because his statement was not unambiguous

A suspect subject to custodial interrogation has a right to consult with counsel and to have an attorney present during questioning Miranda v Arizona 384 US 436 (1966) When a suspect invokes his right to counsel during an interrogation law enforcement must immediately cease all questioning See Edwards v Arizona 451 US 477 484ndash85 (1981) Custodial interrogation cannot be reinitiated unless and until the suspect has been re-advised of his Miranda rights has provided a knowing and voluntary waiver and (1) counsel is present and (2) the suspect himself initiated further communication with the police see id at 484 or (3) (if the suspect was released from custody after the initial interrogation) at least 14 days have passed Maryland v Shatzer 559 US 98 110 (2010)

To invoke the right to counsel a suspectrsquos request must be ldquounambiguousrdquo This means that the suspect must articulate the desire for counsel sufficiently clearly that a reasonable officer would understand the statement to be a request for counsel Davis v United States 512 US 452 459 (1994) If the request is ambiguous the police are not required to stop the interrogation

In this case the suspectrsquos statement ldquoI think I want my lawyer here before I talk to yourdquo was not an unambiguous request for counsel The most reasonable interpretation of this statement is that the suspect might be invoking his right to counsel Id at 461 (ldquomaybe I should talk to a lawyerrdquo is not an unequivocal request for counsel) See also Burket v Angelone 208 F3d 172 197ndash98 (4th Cir 2000) (ldquoI think I need a lawyerrdquo is not an unambiguous request for an attorney) Soffar v Cockrell 300 F3d 588 594ndash95 (5th Cir 2002) (discussion of various statements that did not constitute unequivocal requests for counsel)

Under these circumstances the detective was not required to cease the custodial interrogation of the suspect Nor was the detective required to clarify or ask follow-up questions to determine whether the suspect in fact wanted an attorney Davis 512 US at 459ndash60

14

Criminal Law and Procedure Analysis

Point Three (35) The suspectrsquos waiver of his Miranda rights was knowing intelligent and voluntary despite the fact that he was never told of the lawyerrsquos presence in the jail or of the lawyerrsquos demands

A valid waiver of Miranda rights must be ldquovoluntaryrdquomdashie the product of a free or deliberate choice rather than intimidation coercion or deception Berghuis v Thompkins 560 US 370 382ndash83 (2010) In addition the waiver must be knowing and intelligent That is it ldquomust have been made with a full awareness of both the nature of the right being abandoned and the consequences of the decision to abandon itrdquo Moran v Burbine 475 US 421 (1986)

In this case the suspect signed a Miranda waiver form after receiving proper warnings There is no evidence ldquothat the police resorted to physical or psychological pressure to elicit the statementsrdquo Id The entire interview lasted only 45 minutes The only issue is whether the suspect knowingly and intelligently waived his Miranda rights despite the fact that the detective did not tell the suspect about the lawyerrsquos presence and her demands

The Supreme Court has said that ldquo[e]vents occurring outside of the presence of the suspect and entirely unknown to him surely can have no bearing on the capacity to comprehend and knowingly relinquish a constitutional rightrdquo Id at 422 If the suspect ldquoknew that he could stand mute and request a lawyer and was aware of the Statersquos intention to use his statements to secure a convictionrdquo then the waiver is valid regardless of the information withheld Id at 422ndash23

Here the suspect was correctly informed of his rights Miranda v Arizona 384 US at 467ndash73 His comments demonstrate that he understood that he could have a lawyer present if he desired (ie wondering whether he should call his attorney) and that he understood that there might be consequences to speaking with the detective (ldquoI probably should keep my mouth shut but Irsquom willing to talk to you for a whilerdquo) His comment ldquo[L]etrsquos not waste any time waiting for someone to call my attorney and having her drive hererdquo along with his signature on the Miranda waiver form show that his waiver was valid under the constitutional standard

The fact that the detective did not tell the suspect about the lawyerrsquos presence and demands has no bearing on the validity of the suspectrsquos waiver because ldquosuch conduct is only relevant to the constitutional validity of a waiver if it deprives a defendant of knowledge essential to his ability to understand the nature of his rights and the consequences of abandoning themrdquo Moran at 424 The Supreme Court has specifically declined to adopt a rule requiring that law enforcement tell a suspect of an attorneyrsquos efforts to contact him id at 425 (ldquoNor are we prepared to adopt a rule requiring that the police inform a suspect of an attorneyrsquos efforts to reach himrdquo)

[NOTE An examinee might also recognize that this general rule is further supported by the Supreme Courtrsquos decision in Florida v Powell 559 US 50 (2010) approving state Miranda warnings that do not explicitly warn suspects that they have a right to have counsel present during custodial interrogation]

15

CONTRACTS ANALYSIS (Contracts IB2 IIB IVA3 amp A5)

ANALYSIS

Legal Problems

(1) In the case of a service contract (governed by the common law of contracts) is a modification enforceable when a party agrees to pay more for the same performance than was originally promised

(2) In the case of a contract for the sale of goods (governed by Article 2 of the UCC) is a modification enforceable when a party agrees to pay more for the same goods than was originally promised

(3) May a party avoid an agreement on the basis of economic duress

DISCUSSION

Summary

There are two arguments that the conservatory can make to support the claim that it is not bound to pay the higher prices lack of consideration and economic duress

The organ repair contract is governed by the common law of contracts Under the common law the business would have difficulty recovering the additional $60000 for the organ repair because under the ldquopreexisting duty rulerdquo the agreement of the conservatory to pay the extra price was not supported by consideration However the business might argue that the modification is enforceable under an exception to the preexisting duty rule for fair and equitable modifications made in light of unanticipated circumstances

The organ sale contract is governed by Article 2 of the Uniform Commercial Code The business would likely recover the additional amount under that contract because Article 2 provides that consideration is not required for a modification to be binding

In both cases the conservatory could seek to avoid its agreement on the grounds of economic duress but that argument is not likely to succeed

Point One (45) The business probably cannot recover the additional $60000 for the organ repair because the conservatoryrsquos promise to pay more money was not supported by consideration

The general rule is that to be enforceable a promise must be supported by consideration Under RESTATEMENT (SECOND) OF CONTRACTS sect 71 a promise is supported by consideration if it is bargained for in exchange for a return promise or performance However under the ldquopreexisting duty rulerdquo (exemplified in RESTATEMENT (SECOND) OF CONTRACTS sect 73 and Alaska Packersrsquo Assrsquon v Domenico 117 F 99 (9th Cir 1902)) promise of performance of a legal duty already owed to a promisor which is neither doubtful nor the subject of honest dispute is not consideration

If the business had promised the conservatory anything new or different in exchange for the agreement to pay the additional $60000 (such as for example repairing the pipe organ more

16

Contracts Analysis

quickly or using better parts) that would constitute consideration especially in light of the principle that courts do not inquire into the adequacy of consideration Here however the business already had a legal duty under the original contract and did not agree to do anything else in exchange for the conservatoryrsquos promise to pay $60000 more

However an exception to the preexisting duty rule is sometimes applied in situations of unanticipated changed circumstances Under RESTATEMENT (SECOND) OF CONTRACTS sect 89 followed in many jurisdictions a promise modifying a duty under a contract not fully performed on either side is binding even if not supported by consideration if the modification is fair and equitable in view of circumstances not anticipated by the parties when the contract was made

If a court applies the rule in Restatement sect 89 the critical issues will be whether the modification was in fact ldquofair and equitablerdquo and whether it can be justified in light of unanticipated circumstances In many cases in which modifications have been upheld a party encountered difficulties or burdens in performing far beyond what was knowingly bargained for in the original contract with the result bordering on impracticability such as having to excavate solid rock instead of soft dirt or having to remove garbage far in excess of the amounts contemplated The conservatory would argue that the businessrsquos performance difficulties were not of this sort at allmdashnothing about repairing the pipe organ itself was any different from or more difficult than originally contemplated except that the business itself encountered financial distress unrelated to its burdens in performing its obligations under these contracts

Even if the business satisfies that element of the rule in Restatement sect 89 the business must also demonstrate that the circumstances that gave rise to the need to modify the contract were ldquounanticipatedrdquo at the time the original contract was made Here the facts suggest that when the business entered into the original contract it expected that the price paid by the conservatory would enable it to perform However any evidence that the business knew or had reason to know at the time of execution that it would need more money from the conservatory to be able to perform would mean that the request to modify was not ldquounanticipatedrdquo

[NOTE Some cases such as Schwartzreich v Bauman-Basch Inc 231 NY 196 131 NE 887 (1921) find that if the parties mutually agreed to rescind the original contract and then after rescission entered into an entirely new contract for a higher price the new contract is supported by consideration There is no evidence that such a rescission followed by a new contract took place here]

Point Two (45) The business can recover the additional $40000 for the new organ because no consideration is required under Article 2 of the UCC for good-faith contract modifications

The contract to buy a new organ is a contract for the sale of goods and therefore is governed by Article 2 of the Uniform Commercial Code UCC sect 2-102 Under Article 2 unlike the common law an agreement modifying a contract needs no consideration to be binding UCC sect 2-209(1) Section 2-209(1) thus obviates the preexisting duty rule entirely in contracts for the sale of goods

Even though consideration is not required modifications governed by sect 2-209 must satisfy the obligation of good faith imposed by the UCC UCC sect 1-304 See also Official Comment 2 to UCC sect 2-209 Good faith means ldquohonesty in fact and the observance of reasonable commercial standards of fair dealingrdquo UCC sect 1-201(b)(20) In this context the obligation of good faith means that ldquo[t]he effective use of bad faith to escape performance on the original contract terms is barred and the extortion of a lsquomodificationrsquo without legitimate commercial reason is ineffective as a violation of the duty of good faithrdquo Official Comment 2 to

17

Contracts Analysis

UCC sect 2-209 Here because the businessrsquos financial reversals were serious and apparently unanticipated at the time that the business entered into the contract with the conservatory and commitment of the extra money was needed to enable the business to perform a court would likely find that the business acted in good faith Thus a court would likely uphold the enforceability of the conservatoryrsquos promise to pay the additional $40000

Point Three (10) The conservatory is unlikely to be able to defend against enforcement of its promises to pay additional money under the theory of economic duress because the business probably did not make an improper threat

Under the common law of contracts parties may raise the defense of duress This common law defense also applies to contracts governed by UCC Article 2 See UCC sect 1-103(b)

A contract is voidable on the ground of economic duress by threat when it is established that a partyrsquos manifestation of assent is induced by an improper threat that leaves the party no reasonable alternative See RESTATEMENT (SECOND) OF CONTRACTS sect 175 See also eg Austin Instrument Inc v Loral Corp 272 NE2d 533 (NY 1971) (a threat to withhold essential goods can constitute duress) In order to void its agreement to pay the additional sum because of economic duress the conservatory must demonstrate that (1) the business made a threat to the conservatory (2) the threat was ldquoimproperrdquo or ldquowrongfulrdquo (3) the threat induced the conservatoryrsquos manifestation of assent to the modification and (4) the threat was sufficiently grave to justify the conservatoryrsquos assent

Here it appears that three of the four elements are likely satisfied The business plainly made a threat Moreover the threat induced the conservatoryrsquos assent to the modification and the threat was sufficiently grave to justify that assent If the conservatory had not agreed to pay the business the extra amounts the conservatory would have lost its entire $325000 investment In light of this potential loss a court could easily conclude that the conservatory had no reasonable alternative

However the business has a strong argument that its threat (indicating that it would breach the contracts unless the prices were increased) was not wrongful or improper but was instead nothing more than a communication of the reality of its own perilous situation to the conservatory

A mere threat to breach a contract is not in and of itself improper so as to support an action of economic duress or business compulsion Something more is required such as a breach of the duty of good faith and fair dealing as was present in Austin Instrument Inc supra Because the business could not perform the original contract without the requested modification the economic duress claim for the conservatory would likely fail for much the same reason that the business would be able to enforce the modification At the time the modification was requested the business was not trying to extort a price increase because of the conservatoryrsquos vulnerability but instead was simply stating the reality that the business could not perform without more money

18

FAMILY LAW ANALYSIS (Family Law IIIB D amp G)

ANALYSIS

Legal Problems

(1)(a) Does the State A court have jurisdiction to modify the State B child support order

(1)(b) Does the State A court have jurisdiction to modify the marital-residence-saleshyproceeds provision of the State B property-division decree

(2)(a) May a child support order be modified retroactively

(2)(b) May a child support order be modified prospectively based on a change of employment with a lower salary

(2)(c) May a property-division order be modified after entry of a divorce decree

DISCUSSION

Summary

The State A court may exercise personal jurisdiction over the wife because she was personally served in State A However subject-matter jurisdiction over the interstate modification of child support is governed by the Uniform Interstate Family Support Act (UIFSA) Under UIFSA State A does not have jurisdiction to modify the order for the daughterrsquos support because the wife is still a resident of State B UIFSA on the other hand does not govern property distributions and thus a State A court is not precluded from hearing the husbandrsquos petition to modify the marital-residence-sale-proceeds provision of the divorce decree

A child support order may not be modified retroactively A child support order may be modified prospectively based on a substantial change in circumstances Courts agree that a significant decrease in income is a substantial change in circumstances All states treat voluntary income reductions differently than involuntary reductions but employ different approaches for evaluating the impact of a voluntary reduction Whether the husband could obtain prospective modification of the child support order depends on which approach is applied

A property-division order is not subject to post-divorce modification based on a change in circumstances Thus the husband may in some states obtain prospective modification of the order for the daughterrsquos support but he may not obtain modification of the marital-residenceshysale-proceeds provision

Point One(a) (25) Personal jurisdiction over a nonresident respondent does not confer subject-matter jurisdiction over child support modification Under UIFSA a State A court may not modify a child support order issued by a State B court when as here the child or either parent continues to reside in State B the jurisdiction that issued the child support order

The State A court may exercise personal jurisdiction over the wife The wife was personally served in State A and a state may exercise jurisdiction based on in-state personal service See

19

Family Law Analysis

Burnham v Superior Court 495 US 604 (1990) But personal jurisdiction over the wife is not enough to give a State A court jurisdiction to modify the State B support order

The interstate enforcement and modification of child support is governed by the Uniform Interstate Family Support Act (UIFSA) which has been adopted by all states Under UIFSA the state that originally issued a child support order (here State B) has continuing exclusive jurisdiction to modify the order if that state remains the residence of the obligee the child or the obligor and all parties do not consent to the jurisdiction of another forum See UIFSA sect 205 See also UIFSA sect 603 (ldquoA tribunal of this State shall recognize and enforce but may not modify a registered order if the issuing tribunal had jurisdictionrdquo) The wife and daughter continue to reside in State B and the wife has not consented to the jurisdiction of another forum Thus a State A court does not have jurisdiction to modify the State B child support order

[NOTE Examinees who do not discuss personal jurisdiction but fully discuss UIFSA may receive full credit]

Point One(b) (15) UIFSA does not apply to disputes over property division Thus the State A court may exercise jurisdiction over the husbandrsquos petition to modify the marital-residence-sale-proceeds provision of the State B divorce decree because it has personal jurisdiction over the wife

The State A court in which the husband brought his action has jurisdiction to adjudicate domestic relations issues The husbandrsquos petition to modify the property settlement is a domestic relations issue The courts of State A may exercise personal jurisdiction over the wife because she was personally served in State A See Burnham v Superior Court 495 US 604 (1990) see Point One(a)

UIFSA does not apply to divorce property-division disputes Thus although a State A court may not adjudicate the husbandrsquos petition to modify his child support obligations it may adjudicate his property-division claims (Even though the court has jurisdiction it may not modify the property-division award on the merits See Point Two(c))

Point Two(a) (20) A child support order may not be modified retroactively

State courts have long held that obligations to pay child support ordinarily may not be modified retroactively ldquoIf the hardship is particularly severe the courts sometimes devised a way to protect the obligor but in most instances the courts hold that retroactive modification of this kind is beyond their power and indeed the governing statute may so providerdquo HOMER H CLARK THE LAW OF DOMESTIC RELATIONSHIPS IN THE UNITED STATES 725 (2d ed 1987)

Federal law now goes further and requires the states as a condition of federal child-support funding to adopt rules that absolutely forbid retroactive modification of the support obligation See 42 USC sect 666(a)(9)(C) The states have adopted rules consistent with the federal requirements

Point Two(b) (25) It is unclear whether the husband could obtain prospective downward modification of his child support based on his voluntary acceptance of a job with a lower salary

Prospective modification of a child support order is typically available only when the petitioner can show a substantial change in circumstances See ROBERT E OLIPHANT amp NANCY VER

20

Family Law Analysis

STEEGH FAMILY LAW 213ndash15 (3d ed 2010) A significant decrease in income is typically viewed as a substantial change

However when a parent seeks to modify a child support obligation because he has voluntarily reduced his income a court will not modify the obligation based solely on the income loss Some courts refuse to modify whenever the income shift was voluntary See eg Aguiar v Aguiar 127 P3d 234 (Idaho Ct App 2005) Others look primarily to the petitionerrsquos intentions and permit downward modification if he has acted in good faith See eg In re Marriage of Horn 650 NE2d 1103 (Ill App Ct 1995) Many courts use a multifactor approach See OLIPHANT amp VER STEEGH supra 217ndash18

Here there is no question that the husbandrsquos loss of income was voluntary In a jurisdiction in which voluntary income reduction bars support modification the husbandrsquos petition would be denied

In a jurisdiction employing a good-faith or multifactor approach it is possible but not certain that the husband could obtain downward modification The evidence supports the husbandrsquos good faith his change in employment appears to be based on his new jobrsquos greater responsibilities and better promotion possibilities In a jurisdiction using a multifactor approach the court would likely also consider the impact of such a shift on the daughter the likely duration of the husbandrsquos income loss and the likelihood of a promotion that would ultimately inure to the daughterrsquos benefit Thus on these facts it is possible but by no means certain that the husband could prospectively obtain downward modification of his child support obligation to his daughter

Point Two(c)(15) A divorce property-division award is not subject to modification

A support order is aimed at meeting the post-divorce needs of the supported individual Because the future is unpredictable courts are empowered to modify a support award to take account of changed circumstances that may occur during the period in which support is paid

By contrast a property-distribution award divides assets of the marriage based on the equities at the time of divorce Because the past can be ascertained a property-division award is not subject to post-divorce modification See HARRY A KRAUSE ET AL FAMILY LAW CASES COMMENTS AND QUESTIONS 691 (6th ed 2007)

Here the husband is seeking modification of a property-division award with respect to an asset owned by the parties at the time of divorce Thus the husband may not obtain a modification of the marital-residence-sale-proceeds provision of the divorce decree based on his reduced income

21

FEDERAL CIVIL PROCEDURE ANALYSIS (Federal Civil Procedure III IVC)

ANALYSIS

Legal Problems

(1) Is the logging company entitled to join this action as a matter of right

(2)(a) May the nonprofit organization obtain a temporary restraining order to stop the USFS from issuing a logging permit

(2)(b) May the nonprofit organization obtain a preliminary injunction to stop the USFS from issuing a logging permit during the pendency of the action

DISCUSSION

Summary

The logging company is entitled to intervene in this action as a matter of right because it has an interest in the property or transaction that is the subject of the action and is so situated that its interest may be impaired or impeded as a practical matter if the action goes forward without it The logging companyrsquos interest is not adequately represented by the USFSrsquos presence in the lawsuit

The nonprofit organization may seek a temporary restraining order (TRO) followed by a preliminary injunction to prevent the USFS from issuing a logging permit pending the outcome of the action The nonprofit is likely to obtain a TRO if it can demonstrate a risk of immediate and irreparable injury The nonprofit is also likely to obtain a preliminary injunction if it can demonstrate a significant threat of irreparable harm and a likelihood of success on the merits of its National Environmental Policy Act (NEPA) claim

Point One (50) Rule 24(a) of the Federal Rules of Civil Procedure requires federal courts to allow a person to intervene in an action as a matter of right if the person a) is interested in the property or transaction that is the subject of the action b) is so situated that its interest may be impaired or impeded if the litigation goes forward without it and c) is not adequately represented by existing parties Here the logging company likely meets all three requirements and should be allowed to intervene as a matter of right

Rule 24 of the Federal Rules of Civil Procedure governs intervention the process by which a non-party to an action may join the litigation Under Rule 24(a) (intervention of right) a person must be permitted to intervene if three conditions are met (1) the movant ldquoclaims an interest relating to the property or transaction that is the subject of the actionrdquo (2) the movant ldquois so situated that disposition of the action may as a practical matter impair or impede the movantrsquos ability to protect its interestrdquo and (3) ldquoexisting partiesrdquo do not ldquoadequately represent [the movantrsquos] interestrdquo FED R CIV P 24(a) The three requirements for intervention of right are often ldquovery interrelatedrdquo 7C CHARLES ALAN WRIGHT ET AL FEDERAL PRACTICE AND PROCEDURE sect 1908 at 297 (2007 amp 2011 Supp)

22

Federal Civil Procedure Analysis

Here the court should find that the logging company meets this test First the logging company has a strong interest in the property or transaction that is the subject of this action The USFS has accepted the logging companyrsquos bid and the logging company is merely awaiting issuance of a logging permit to begin logging The nonprofit organization is seeking to prevent this logging The logging company therefore has a strong direct and substantial interest in the subject matter of the lawsuit and in having its winning bid honored and a logging permit issued See eg Kleissler v US Forest Serv 157 F3d 964 972 (3d Cir 1998) (stating that ldquo[t]imber companies have direct and substantial interests in a lawsuit aimed at halting loggingrdquo) see also Natural Resources Defense Council v US Nuclear Regulatory Commrsquon 578 F2d 1341 1343ndash 44 (10th Cir 1978) (holding that applicants whose license renewals were pending had Rule 24(a)(2) interests where the lawsuit sought to halt the license-issuing process pending preparation of environmental impact statements) See generally 7C WRIGHT ET AL supra sect 19081 at 309 (ldquoIf there is a direct substantial legally protectable interest in the proceedings it is clear that this requirement of the rule is satisfiedrdquo) Second the logging companyrsquos interest in receiving a logging permit may well be impaired as a practical matter by the outcome of the lawsuit If the USFS loses the lawsuit it will have to prepare an environmental impact statement before issuing the logging companyrsquos permit This will at a minimum delay the logging companyrsquos ability to exercise its rights and may in the long r un mean that no logging permit is ever issued Intervention of right is not limited to those that would be legally bound as a matter of preclusion doctrine Id sect 19082 at 368 Rather ldquo[t]he rule is satisfied whenever disposition of the present action would put the movant at a practical disadvantage in protecting its interestrdquo Id sect 19082 at 369 Here that condition is easily satisfied See Kleissler 157 F3d at 972 (ldquoTimber companies have direct and substantial interests in a lawsuit aimed at halting logging rdquo)

Given that the logging company has an interest that may be impaired by disposition of the action it should be allowed to intervene unless the court is persuaded that the USFS adequately represents the logging companyrsquos interest See Rule 24(a)(2) 7C WRIGHT ET AL supra sect 1909 Here it could be argued that the USFS adequately represents the logging companyrsquos interest because the USFS presumably wants the court to uphold its development plan and allow it to proceed with issuance of the logging permit which is the same relief that the logging company would seek However whether representation is truly adequate depends upon ldquo[a] discriminating appraisal of the circumstancesrdquo 7C WRIGHT ET AL supra sect 1909 at 440 Although both the government and the logging company wish to avoid the preparation of an environmental impact statement their interests are distinct The USFSrsquos interest is proper management of the national forest system while the logging companyrsquos interest is making a profit from logging the 5000-acre tract The USFSrsquos handling of the litigation is likely to be affected by a variety of policy concerns and political considerations that have nothing to do with the logging companyrsquos purely economic interest in securing the right to cut trees in the Scenic National Forest See eg Kleissler 157 F3d at 973ndash74 (ldquo[T]he government represents numerous complex and conflicting interests in matters of this nature The straightforward business interests asserted by intervenors here may become lost in the thicket of sometimes inconsistent governmental policiesrdquo)

[NOTES (1) Examinees who mistakenly analyze the logging companyrsquos case for joinder under the related but incorrect Rule 19 ldquoRequired Joinder of Partiesrdquo may receive credit Rule 19 allows existing parties to demand joinder of non-parties (or seek dismissal of the case if they canrsquot get it) There is a close relationship between Rule 24 and Rule 19 and both contain a similar standard for determining when ldquointerestedrdquo third parties are ldquoentitledrdquo or ldquorequiredrdquo to be in the lawsuit Indeed the two prongs of the Rule 24 intervention test that are discussed above

23

Federal Civil Procedure Analysis

are nearly identical to the two prongs of the Rule 19(a) required joinder test Examinees who discuss and apply the test should receive credit even if they cite Rule 19 rather than Rule 24

(2) Examinees may discuss permissive joinder Although permissive joinder is a possibility here the question asks only whether the logging company can join the action as a matter of right and a permissive joinder analysis is not responsive to the question To the extent an examinee discusses permissive joinder the analysis will focus on whether the logging company ldquohas a claim or defense that shares with the main action a common question of law or factrdquo FED R CIV P 24(b)(1)(B) The district court also ldquomust consider whether the intervention will unduly delay or prejudice the adjudication of the original partiesrsquo rightsrdquo FED R CIV P 24(b)(3) On our facts the logging companyrsquos claim for the issuance of a logging permit would certainly share common questions of law and fact with the USFSrsquos defense against the nonprofitrsquos claim There are no facts suggesting that the logging companyrsquos presence would unduly delay or otherwise prejudice adjudication of the original action Thus the district court would have discretion to permit the logging company to intervene even if it denied intervention of right]

Point Two(a) (25) The nonprofit organization could seek and would likely obtain a temporary restraining order to stop the USFS from issuing a logging permit pending a hearing on an application for a preliminary injunction

The first type of interim relief the nonprofit could seek to stop the USFS from issuing a logging permit to the logging company is a temporary restraining order (TRO) prohibiting the USFS from issuing the logging permit A TRO can be issued without notice to the adverse party but only in limited circumstances and only for a limited time FED R CIV P 65(b) To secure a TRO without notice the nonprofit would need to submit an affidavit containing specific facts that demonstrate a risk of ldquoimmediate and irreparable injuryrdquo if a permit is issued FED R CIV P 65(b)(1) In deciding whether to grant a TRO courts will also consider the same factors that are relevant in deciding whether to grant a preliminary injunction (eg the moving partyrsquos likelihood of success on the merits the balance of hardships and the public interest) See Point Two(b) infra The TRO would last only long enough for the court to consider and resolve a request by the nonprofit for a preliminary injunction but no longer than 14 days (unless the court extends it for good cause or the adverse party consents to an extension) In addition bond is required

Here the court is likely to grant the nonprofitrsquos request The nonprofit could plausibly claim that cutting down 5000 acres of old-growth forest in an area that is home to the highest concentration of wildlife in the western United States would have ldquoan immediate and irreparablerdquo adverse impact on the environment and cause irreparable harm to the nonprofitrsquos interest in preserving and protecting natural resources including wildlife habitat

Point Two(b) (25) The nonprofit could also seek and would likely obtain a preliminary injunction to stop the USFS which is likely to be granted if the nonprofitrsquos claim that the USFS violated NEPA has a strong basis in fact and law

Because the TRO would be temporary the nonprofit would need to move for a preliminary injunction to prevent the USFS from issuing a logging permit throughout the pendency of the litigation Preliminary injunctions are injunctions that seek to ldquoprotect [the] plaintiff from

24

Federal Civil Procedure Analysis

irreparable injury and to preserve the courtrsquos power to render a meaningful decision after a trial on the meritsrdquo 11A CHARLES ALAN WRIGHT ET AL FEDERAL PRACTICE AND PROCEDURE sect 2947 at 112 (2013) Rule 65 of the Federal Rules of Civil Procedure sets out the procedural requirements for preliminary injunctions Preliminary injunctions may be granted only upon notice to the adverse party FED R CIV P 65(a)(1) and only if the movant ldquogives security in an amount that the court considers proper to pay the costs and damages sustained by any party found to have been wrongfully enjoined or restrainedrdquo FED R CIV P 65(c)

While Rule 65 sets out the procedural requirements for preliminary injunctive relief it does not specify the substantive grounds upon which it may be granted The courtrsquos discretion in ruling upon a motion for a preliminary injunction ldquois exercised in conformity with historic federal equity practicerdquo 11A WRIGHT ET AL supra sect 2947 at 114 The court typically considers four factors

(1) the significance of the threat of irreparable harm to the plaintiff if the injunction is not granted (2) the balance between this harm and the injury that granting the injunction would inflict on the defendant (3) the probability that the plaintiff will succeed on the merits and (4) the public interest

Id sect 2948 at 122ndash24 accord Habitat Educ Center v Bosworth 363 F Supp 2d 1070 1088 (ED Wis 2005) The most important of these factors is the risk of irreparable harm to the plaintiff 11A WRIGHT ET AL supra sect 29481 at 129 If the plaintiff has an adequate remedy at law (eg if money damages can compensate the plaintiff for its loss) then a preliminary injunction will be denied Id sect 29481

Here a court would likely conclude that the potential for environmental damage to the forest creates a significant threat of irreparable harm ldquo[E]nvironmental injury is often irreparable Courts have recognized that logging such as would occur [here] can have longshyterm environmental consequences and thus satisfy the irreparable injury criterionrdquo Habitat Educ Center 363 F Supp 2d at 1089 (citing Idaho Sporting Congress Inc v Alexander 222 F3d 562 569 (9th Cir 2000) (noting that the imminent and continuing logging activities presented ldquoevidence of environmental harm sufficient to tip the balance in favor of injunctive reliefrdquo)) Neighbors of Cuddy Mountain v US Forest Service 137 F3d 1372 1382 (9th Cir 1998) (stating that ldquo[t]he old growth forests plaintiffs seek to protect would if cut take hundreds of years to reproducerdquo) (internal citation omitted)) see also 11C WRIGHT ET AL supra sect 29481 at 151 (noting that ldquoa preliminary injunction has been issued to prevent harm to the environmentrdquo)

The second factor the balance between the harm to the plaintiff and the harm the defendant will suffer if the injunction is issued also appears to support issuance of a preliminary injunction here The USFS will have to wait before it can develop the Scenic National Forest and the logging company may lose money if the delay is prolonged These economic harms could be compensated monetarily if an injunction is issued inappropriately Where ldquoan injunction bond can compensate [the] defendant for any harm the injunction is likely to inflict the balance should be struck in favor of [the] plaintiffrdquo Id sect 29482 at 192 See also Habitat Educ Center 363 F Supp 2d at 1089 (stating that ldquothe relative absence of harmful effects on the Forest Service weighs in favor of granting the injunctionrdquo)

The third factor is the likelihood that the plaintiff will prevail on the merits Although there is limited information concerning the merits of the action the nonprofit alleges that the federal statute (NEPA) requires an environmental impact statement and further states that the USFS created no environmental impact analysis or statement at all Assuming that those

25

Federal Civil Procedure Analysis

allegations are correct it seems plausible to conclude that the nonprofit will be able to show a likelihood of success on the merits

Finally courts deciding whether or not to issue preliminary injunctive relief are to consider the public interest ldquoFocusing on this factor is another way of inquiring whether there are policy considerations that bear on whether the order should issuerdquo 11C WRIGHT ET AL supra sect 29484 at 214 If the court concludes that the nonprofit is likely to succeed on its NEPA claim because the USFS wrongfully failed to conduct an environmental impact assessment it is likely to find that the public interest would be served by restraining the USFS from proceeding with logging in a national forest See Heartwood Inc v US Forest Service 73 F Supp 2d 962 979 (SD Ill 1999) affrsquod on other grounds 230 F3d 947 (7th Cir 2000) (ldquoviolations by federal agencies of NEPArsquos provisions as established by Congress harm the public as well as the environmentrdquo)

Thus a court is very likely to grant a preliminary injunction if it concludes that the nonprofit has a significant likelihood of success on the merits

26

EVIDENCE ANALYSIS (Evidence ID IIA amp C)

ANALYSIS

Legal Problems

(1) Under what circumstances can evidence of prior convictions be used to impeach a witnessrsquos credibility in a civil case

(1)(a) May the inmatersquos credibility be impeached by evidence of a 12-year-old felony drug conviction if he was released from prison 9 years ago

(1)(b) May the inmatersquos credibility be impeached by evidence of an 8-year-old misdemeanor perjury conviction that was punishable by 1 year in jail if he pleaded guilty and was sentenced only to pay a $5000 fine

(1)(c) May the inmatersquos credibility be impeached by evidence of a 7-year-old sexual assault conviction if the inmate is still serving a 10-year prison sentence and the victim was his 13-year-old daughter

(2)(a) May the guardrsquos credibility be impeached by cross-examination regarding specific instances of misconduct (ie lying on his reacutesumeacute) relevant to credibility

(2)(b) May the guardrsquos credibility be impeached by admission of extrinsic evidence (his reacutesumeacute and academic transcript) offered to prove specific instances of misconduct relevant to credibility

DISCUSSION

Summary

Under the Federal Rules of Evidence witnesses can be impeached with evidence of prior convictions andor specific instances of misconduct Whether evidence of prior convictions should be admitted to impeach generally depends on the nature of the crime the amount of time that has passed and (only in criminal cases) whether the ldquowitnessrdquo is the defendant FED R EVID 609(a)

In this civil case evidence of the inmatersquos conviction for distribution of marijuana should be admitted to impeach the inmate because he was convicted of a felony and was released from prison fewer than 10 years ago FED R EVID 609(a)(1) Credibility is critically important in this case because the jury will hear conflicting testimony from the two disputing parties and there were no other eyewitnesses to the altercation Under Rule 609(a)(1) the inmatersquos conviction should be admitted because it has some bearing on his credibility and its probative value is not substantially outweighed by concerns of unfair prejudice confusion or delay Id

Evidence of the inmatersquos misdemeanor conviction for perjury must be admitted because the crime ldquorequired provingmdashor the witnessrsquos admittingmdasha dishonest act or false statementrdquo by the inmate FED R EVID 609(a)(2)

27

Evidence Analysis

Evidence of the inmatersquos felony conviction for sexual assault should be excluded because its probative value is substantially outweighed by the danger of unfair prejudice to the inmate based on the heinous nature of the crime FED R EVID 609(a)(1) In the alternative the judge could limit the evidence relating to this conviction by excluding details of the inmatersquos crime

In all civil (and criminal) cases witnesses can also be impeached with evidence of specific instances of prior misconduct that did not result in a conviction FED R EVID 608(b) Pursuant to Rule 608(b) misconduct probative of untruthfulness can be inquired into on cross-examination but cannot be proved through extrinsic evidence Id Thus the inmatersquos counsel should be permitted to cross-examine the guard regarding the false statement in the guardrsquos reacutesumeacute However extrinsic evidence of the guardrsquos misconduct (ie the guardrsquos authenticated reacutesumeacute and transcript from the local college) should not be admitted even if the guard denies wrongdoing or refuses to answer cross-examination questions about these matters Id

Point One (10) The Federal Rules of Evidence permit impeachment of witnesses with evidence of prior convictions

Whether convictions should be admitted to impeach generally depends on the nature of the crime the amount of time that has passed and (only in criminal cases) whether the ldquowitnessrdquo is the defendant FED R EVID 609(a) Under Rule 609(a) evidence of prior convictions may be admitted for the purpose of ldquoattacking a witnessrsquos character for truthfulnessrdquo Id

There are two basic types of convictions that can be admitted for the purpose of impeachment

(1) convictions for crimes ldquopunishable by death or by imprisonment for more than one yearrdquo (which generally correlates to ldquofeloniesrdquo) FED R EVID 609(a)(1) and (2) convictions ldquofor any crimes regardless of the punishment if the court can readily determine that establishing the elements of the crime required provingmdashor the witnessrsquos admittingmdasha dishonest act or false statementrdquo FED R EVID 609(a)(2)

Pursuant to Rule 609(a)(1) in civil cases the admission of evidence of a felony conviction is ldquosubject to Rule 403 [which says that a court may exclude relevant evidence if its probative value is substantially outweighed by other factors]rdquo FED R EVID 609(a)(1) However Rule 403 does not protect the witness against admission of prior convictions involving dishonestymdashwhich must be admitted by the court FED R EVID 609(a)(2)

Finally Federal Rule of Evidence 609(b) contains the presumption that a conviction that is more than 10 years old or where more than 10 years has passed since the witnessrsquos release from confinement (whichever is later) should not be admitted unless ldquoits probative value supported by specific facts and circumstances substantially outweighs its prejudicial effectrdquo and the proponent has provided the adverse party with reasonable written notice FED R EVID 609(b)

Point One(a) (25) The court should admit evidence of the inmatersquos 12-year-old felony marijuana distribution conviction

The inmatersquos conviction for marijuana distribution was for a felony punishable by imprisonment for more than one year See FED R EVID 609(a)(1) Moreover although the conviction was 12 years ago the 10-year time limit of Rule 609(b) is not exceeded because that time limit runs

28

Evidence Analysis

from the date of either ldquothe witnessrsquos conviction or release from confinement for it whichever is laterrdquo FED R EVID 609(b) Because the inmate served three years in prison he was released from confinement nine years ago

However pursuant to Rule 609(a)(1) the admission of felony convictions to impeach a witness in a civil case is ldquosubject to Rule 403rdquo FED R EVID 609(a)(1) Neither Rule 609(a) nor the advisory committee notes specify which factors courts should consider when balancing the probative value of a conviction against the dangers identified in Rule 403 (which include (1) unfair prejudice (2) confusion of the issues (3) misleading the jury (4) waste of time or undue delay and (5) needless presentation of cumulative evidence) FED R EVID 403

In this case credibility is very important because the evidence consists primarily of the testimony of the disputing parties and there were no other eyewitnesses to the altercation This enhances the probative value of any evidence bearing on the inmatersquos credibility A court is likely to conclude that the inmatersquos prior felony drug conviction is relevant to his credibility See eg United States v Brito 427 F3d 53 64 (1st Cir 2005) (ldquoPrior drug-trafficking crimes are generally viewed as having some bearing on veracityrdquo) Although the probative value of any conviction diminishes with age see eg United States v Brewer 451 F Supp 50 53 (ED Tenn 1978) the inmatersquos ongoing problems with the law suggest that he has continued (and even escalated) his criminal behavior over the past nine years The court should admit this evidence because its probative value is not substantially outweighed by any Rule 403 concerns Specifically any prejudice to the inmate would be slight because the conviction is unrelated to the altercation at issue and the conviction was not for a heinous crime that might inflame the jury

[NOTE Whether an examinee identifies the jury instruction as containing a ldquoconclusiverdquo or ldquomandatoryrdquo presumption is less important than the examineersquos analysis of the constitutional infirmities]

Point One(b) (15) The court must admit evidence of the inmatersquos eight-year-old misdemeanor conviction because perjury is a crime of dishonesty

Rule 609(a)(2) provides that evidence of a criminal conviction ldquomust be admitted if the court can readily determine that establishing the elements of the crime required provingmdashor the witnessrsquos admittingmdasha dishonest act or false statementrdquo FED R EVID 609(a)(2) The inmatersquos conviction for perjury would have necessarily required proving that the inmate engaged in an act of dishonesty This conviction occurred within the past 10 years so it ldquomust be admittedrdquo because in contrast to Rule 609(a)(1) (discussed in Point One(a)) admission under Rule 609(a)(2) is mandatory and not subject to Rule 403

Point One(c) (20) The court should exclude evidence of the inmatersquos seven-year-old felony sexual assault conviction because the probative value of this evidence is substantially outweighed by the danger of unfair prejudice In the alternative the details of the prior conviction could be excluded

The inmatersquos conviction for felony sexual assault was seven years ago and he has not yet been released from incarceration so Rule 609(a) but not 609(b) is applicable here FED R EVID 609(a) This conviction is therefore admissible to impeach the inmate unless its probative value is substantially outweighed by the danger of unfair prejudice or any other Rule 403 concern Id

29

Evidence Analysis

Sex crimes are generally not considered relevant to credibility see Hopkins v State 639 So 2d 1247 1254 (Miss 1993) so the probative value of this conviction is relatively low Moreover the heinous nature of the inmatersquos crime (sexual assault on his daughter) makes the danger of unfair prejudice to the inmate very high Thus the court should exclude evidence of the conviction because it was for a heinous offense that is likely to inflame the jury and it has little bearing on credibility See eg United States v Beahm 664 F2d 414 419 (4th Cir 1981)

As an alternative to excluding this evidence the judge could minimize the unfair prejudice to the inmate by permitting limited cross-examination but refusing to allow specific questions about the nature of the inmatersquos conviction For example a court could limit cross-examination to the fact that the inmate was convicted of a ldquofelonyrdquo or perhaps that he was convicted of a ldquosexual assaultrdquo without identifying the victim However because evidence of the inmatersquos prior convictions can be admitted solely for the purpose of enabling the jury to assess his credibility and because his two earlier convictions should have already been admitted the court should exclude all evidence of the felony sexual assault conviction

Point Two(a) (15) The court should permit the inmatersquos counsel to cross-examine the guard regarding the false statement in his reacutesumeacute because the guardrsquos misconduct bears on his truthfulness

The inmate wishes to cross-examine the guard about his prior dishonest behaviormdashlying on his reacutesumeacutemdashthat did not involve a criminal conviction Rule 608(b) allows witnesses to be cross-examined about specific instances of prior non-conviction misconduct probative of untruthfulness ldquoin order to attack the witnessrsquos character for truthfulnessrdquo FED R EVID 608(b)

The courtrsquos decision to allow cross-examination about the guardrsquos prior dishonest behavior depends on the probative value of such evidence balanced against the danger of unfair prejudice to the guard or any other Rule 403 concern FED R EVID 403 Here the guardrsquos false statement on his reacutesumeacute that he obtained a degree in Criminal Justice is highly probative of his untruthfulness because it grossly misrepresents his actual academic record was made recently and was made with the intent to deceive Because the probative value of this evidence is very strong and is not substantially outweighed by any Rule 403 concerns cross-examination of the guard on this topic should be permitted The court may also consider it fair to permit this cross-examination of the guard on these matters assuming that one or more of the inmatersquos prior convictions have been admitted to impeach his credibility

Point Two(b) (15) The court should exclude extrinsic evidence of the guardrsquos non-conviction misconduct even if the guard denies wrongdoing or refuses to answer questions about the matter

Although Rule 608(b) allows cross-examination about specific instances of prior misconduct probative of untruthfulness ldquoextrinsic evidencerdquo offered to prove such misconduct is not admissible FED R EVID 608(b) The rationale for this rule is that allowing the introduction of extrinsic evidence of prior misconduct by witnesses when these acts are relevant only to the witnessesrsquo truthfulness and not to the main issues in the case would create too great a risk of confusing the jury and unduly delaying the trial The court does not have discretion to admit this extrinsic evidence See eg United States v Elliot 89 F3d 1360 1368 (8th Cir 1996)

30

Evidence Analysis

Here the inmatersquos counsel may cross-examine the guard about the false statement on his reacutesumeacute However the inmatersquos counsel must accept the guardrsquos response Even if the guard denies wrongdoing or refuses to answer questions about the matter the inmatersquos counsel cannot introduce the guardrsquos reacutesumeacute or the transcript from the local college to prove the guardrsquos misconduct

31

CORPORATIONS ANALYSIS (Corporations VA2 IX)

ANALYSIS

Legal Problems

(1) Do shareholders have the authority to amend a corporationrsquos bylaws with respect to director nominations

(2) Do board-approved bylaws on a particular subject here nomination of directors preempt subsequent conflicting bylaw amendments by shareholders

(3) Is a suit challenging both managementrsquos refusal to include the proposed bylaw amendment in Megarsquos proxy statement and the boardrsquos amendment of the bylaws dealing with nomination of directors a direct or derivative suit

DISCUSSION

Summary

The voting and litigation rights of the shareholders of Mega are subject to the provisions of the Model Business Corporations Act (MBCA)

The investorrsquos proposed bylaw provision is not inconsistent with state law Under the MBCA shareholders may amend the bylaws when the amendment deals with a proper matter for the corporationrsquos bylaws such as procedures for nominating directors

The Mega boardrsquos bylaw amendment does not preempt the investorrsquos proposed bylaw provision or the Mega shareholdersrsquo power to approve it While shareholders can limit the boardrsquos power to amend or repeal the bylaws the board cannot limit the shareholdersrsquo power

Whether the investor must make a demand on Megarsquos board depends on how the investor frames its claim If the investor claims a violation of shareholder voting rights the claim is direct and pre-suit demand on the board is not required If on the other hand the investor claims that the directors violated their fiduciary duties by amending the bylaws to entrench themselves the claim is derivative and a pre-suit demand is required

Point One (30) Shareholders may amend the corporationrsquos bylaws where the proposed bylaw provision relates to procedural matters typically included in the bylaws such as the nomination of directors

Internal affairs of the corporation such as the conduct of shareholder meetings and election of directors are subject to the corporate law of the state of incorporation See McDermott Inc v Lewis 531 A2d 206 (Del 1987) (applying law of jurisdiction where corporation was incorporated in case involving voting rights) This statersquos corporate statute is modeled on the MBCA

Under the MBCA ldquoshareholders may amend the corporationrsquos bylawsrdquo MBCA sect 1020(a) Thus the only question is whether the bylaws can specify the procedures for shareholder nomination of directors

32

Corporations Analysis

The MBCA states that the bylaws ldquomay contain any provision that is not inconsistent with law or the articles of incorporationrdquo MBCA sect 206(b) In addition the MBCA was revised in 2009 to address shareholder nomination of directors in public corporations (known as ldquoproxy accessrdquo) and specifies that the bylaws ldquomay contain a requirement that the corporation include in its [proxy materials] one or more individuals nominated by a shareholderrdquo MBCA sect 206(c)(1) see Committee on Corporate Laws ABA Section of Business Law Report on the Roles of Boards of Directors and Shareholders of Publicly Owned Corporations and Changes to the Model Business Corporations ActmdashAdoption of Shareholder Proxy Access Amendments to Chapters 2 and 10 65 BUS LAWYER 1105 (2010)

The inclusion of director-nomination procedures in the bylaws is consistent with practice and is recognized by the Delaware courts whose views on corporate law carry significant weight Typically the procedures for nomination of directors are found in the bylaws See 1 COX amp HAZEN TREATISE ON THE LAW OF CORPORATIONS sect 312 (3d ed 2011) see also 4 FLETCHER CORP FORMS ANN PART III ch 21 (2013) (including sample bylaws that permit nomination of directors by shareholders) The Delaware Supreme Court has confirmed that the bylaws may ldquodefine the process and proceduresrdquo for director elections See CA Inc v AFSCME Employees Pension Plan 953 A2d 227 (Del 2008) (concluding that bylaw amendment requiring reimbursement of election expenses to certain successful shareholder nominators is ldquoproper subjectrdquo under Delaware law)

[NOTE The question of the proper scope of the bylaws can be answered using the more general MBCA sect 206(b) or the 2009 MBCA revision adding sect 206(c)(1) (adopted in CT ME VA) In addition some examinees might raise the point that shareholder proposals may not compel the board to take action such as by including shareholder nominations in the companyrsquos proxy materials on the theory that the ldquobusiness and affairsrdquo of the corporation are to be managed by the board See MBCA sect 801(b) Although shareholders are generally limited to adopting precatory resolutions that recommend or encourage board action this limitation does not apply when shareholders have specific authority to take binding action on their ownmdashsuch as to amend the bylaws]

Point Two (30) Shareholders can amend (or repeal) board-approved bylaws Further shareholders can limit the boardrsquos power to later amend and repeal a shareholder-approved bylaw

Under the MBCA shareholders have the power to amend the bylaws See Point One The board shares this power with the shareholders unless (1) the corporationrsquos articles ldquoreserve that power exclusively to the shareholdersrdquo or (2) ldquothe shareholders in amending repealing or adopting a bylaw expressly provide that the board of directors may not amend repeal or reinstate that bylawrdquo See MBCA sect 1020(b)

Shareholder-approved bylaw provisions can amend or repeal existing bylaw provisions whether originally approved by the board or by shareholders See ALAN R PALMITER CORPORATIONS EXAMPLES AND EXPLANATIONS sect 713 (7th ed 2012) Thus the Mega boardrsquos bylaw amendmentmdashwhich set more demanding thresholds for shareholder nomination of directors than the investorrsquos proposed bylaw provisionmdashwould be superseded (repealed) if Megarsquos shareholders were to approve the investorrsquos proposal

Further a shareholder-approved bylaw generally can limit the power of the board to later amend or repeal it See MBCA sect 1020(b)(2) Thus if Megarsquos shareholders approved the bylaw

33

Corporations Analysis

provision proposed by the investor Megarsquos board could not repeal the provision because it includes a ldquono board repealrdquo clause

The revision to the MBCA in 2009 dealing with shareholder proxy access does not change this conclusion That revision specifies that a shareholder-approved bylaw dealing with director nominations may not limit the boardrsquos power to amend add or repeal ldquoany procedure or condition to such a bylaw in order to provide for a reasonable practicable and orderly processrdquo MBCA sect 206(d) Thus according to the revision if shareholders approve a bylaw amendment that limits further board changes the board would nonetheless retain the power to ldquotinkerrdquo with the bylaw to safeguard the voting process but could not repeal the shareholder-approved bylaw The Official Comment to MBCA sect 206(d) makes clear that the revision is ldquonot intended to allow the board of directors to frustrate the purpose of the shareholder-adopted proxy access provisionrdquo Thus if Megarsquos shareholders were to approve the bylaw provision proposed by the investor Megarsquos board could only amend the provision regarding its procedures or conditions in a manner consistent with its purpose of permitting proxy access for Megarsquos shareholders

[NOTE The boardrsquos attempted interference with a shareholder voting initiative may also have been a violation of the directorsrsquo fiduciary duties See Blasius Indus Inc v Atlas Corp 564 A2d 651 (Del Ch 1988) (finding that directors breached their fiduciary duties by amending bylaws and expanding size of board to thwart insurgentrsquos plan to amend bylaws and seat a majority of new directors) The call however asks examinees to consider whether shareholders or the board have ldquoprecedencerdquo over amending the corporate bylaws Thus an examineersquos answer should be framed in terms of ldquopowerrdquo and not ldquodutyrdquo]

Point Three (40) The investor need not make a demand on the board if the investor states a direct claim such as an allegation that the board interfered with the investorrsquos right to amend the bylaws But the investor must make a demand on the board if the investor states a derivative claim (on behalf of the corporation) such as an allegation that the directors sought to entrench themselves by interfering with the proposed proxy access

The MBCA generally requires that shareholders make a demand on the board of directors before initiation of a derivative suit MBCA sect 742 (shareholder may not bring derivative proceeding until written demand has been made on corporation and 90 days have expired) A derivative suit is essentially two suits in one where the plaintiff-shareholder seeks to bring on behalf of the corporation a claim that vindicates corporate rights usually based on violation of fiduciary duties PALMITER supra sect 1811 (6th ed 2009) The demand permits the board to investigate the situation identified by the shareholder and take suitable action No demand on the board is required however if the shareholder brings a direct suit to vindicate the shareholderrsquos own rights not those of the corporation

Is the suit brought by the investor derivative or direct The MBCA defines a ldquoderivative proceedingrdquo as one brought ldquoin the right of a domestic corporationrdquo MBCA sect 740(1) Thus the answer to how the investorrsquos suit should be characterized turns on what rights the investor seeks to vindicate If the investor frames its claim as one of fiduciary breach by directorsmdashfor example for failing to become adequately informed about voting procedures or for seeking to entrench themselves in office by manipulating the voting structure to avoid a shareholder insurgencymdashthen the suit is ldquoderivativerdquo and the investor must make a demand on the board See MBCA Ch 7 Subch D Introductory Comment (ldquothe derivative suit has historically been the principal method of challenging allegedly illegal action by managementrdquo)

34

Corporations Analysis

If however the investor frames its claim as one to vindicate shareholder rights the suit is direct and no demand is required For many courts the direct-derivative question turns on who is injured and who is to receive the relief sought by the plaintiff-shareholders See Tooley v Donaldson Lufkin amp Jenrette Inc 845 A2d 1031 (Del 2004) (characterizing a merger-delay claim as direct because delay of merger only harmed shareholders not corporation) Thus if the investor claims that managementrsquos refusal to include its proposed bylaw amendment in the corporationrsquos proxy materials violates its shareholder rights to initiate corporate governance reforms the suit will be direct Courts have not questioned the ability of shareholders to bring direct suits challenging board action to exclude their proposed bylaw amendments from the corporationrsquos proxy materials See JANA Master Fund Ltd v CNET Networks Inc 954 A2d 335 (Del Ch 2008) (upholding shareholderrsquos direct challenge to boardrsquos interpretation of advance-notice bylaw) Chesapeake Corp v Shore 771 A2d 293 (Del Ch 2000) (upholding shareholderrsquos direct challenge to actions by board that effectively prevented it from proposing bylaw amendments in contest for control)

Is the way that the investor frames its claim conclusive Courts have permitted shareholder-plaintiffs to challenge a transaction in a direct suit even though the same transaction could also be challenged as a fiduciary breach See Eisenberg v Flying Tiger Line Inc 451 F2d 267 (2d Cir 1971) (permitting direct suit challenging a corporate reorganization as a dilution of shareholder voting power even though reorganization may have involved conflicts of interest and thus constituted a fiduciary breach) Thus the investorrsquos choice to pursue a claim challenging the legality of managementrsquos decision to exclude the investorrsquos proposal from the corporationrsquos proxy materialsmdashrather than a possible breach of fiduciary dutymdashis likely to be respected See 3 COX amp HAZEN supra sect 153 (describing situations in which a claim can be framed as derivative or direct)

[NOTE Some issues under Delaware corporate law regarding pre-suit demand are not relevant here For example whether the Mega directors are independent and disinterested is not relevant to the MBCA requirement of a pre-suit demand As the Official Comment to MBCA sect 742 points out the MBCArsquos requirement of ldquouniversal demandrdquo gives the board ldquothe opportunity to reexamine the act complained of in the light of a potential lawsuit and take corrective actionrdquo even when the directors might be non-independent or have conflicts of interest

Nor is it relevant to the MBCA pre-suit demand requirement that the statutory 90-day waiting period may be onerous The first paragraph of MBCA sect 742 requires a pre-suit demand without exception the second paragraph of the section imposes a 90-day waiting period before a derivative suit may be brought which can be shortened if the board rejects the demand or ldquoirreparable injury to the corporation would result by waiting for the expiration of the 90-day periodrdquo The call as written asks only whether a pre-suit demand should be made and does not ask examinees to address whether the post-demand waiting period should be shortened under the ldquoirreparable injuryrdquo standard]

35

National Conference of Bar Examiners 302 South Bedford Street | Madison WI 53703-3622 Phone 608-280-8550 | Fax 608-280-8552 | TDD 608-661-1275

wwwncbexorg e-mail contactncbexorg

  • Preface
  • Description of the MEE
  • Instructions
  • July 2014 Questions
    • CRIMINAL LAW AND PROCEDURE QUESTION
    • CONTRACTS QUESTION
    • FAMILY LAW QUESTION
    • FEDERAL CIVIL PROCEDURE QUESTION
    • EVIDENCE QUESTION
    • CORPORATIONS QUESTION
      • July 2014 Analyses
        • CRIMINAL LAW AND PROCEDURE ANALYSIS
        • CONTRACTS ANALYSIS
        • FAMILY LAW ANALYSIS
        • FEDERAL CIVIL PROCEDURE ANALYSIS
        • EVIDENCE ANALYSIS
        • CORPORATIONS ANALYSIS
            • ltlt13 ASCII85EncodePages false13 AllowTransparency false13 AutoPositionEPSFiles true13 AutoRotatePages None13 Binding Left13 CalGrayProfile (Dot Gain 20)13 CalRGBProfile (sRGB IEC61966-21)13 CalCMYKProfile (US Web Coated 050SWOP051 v2)13 sRGBProfile (sRGB IEC61966-21)13 CannotEmbedFontPolicy Error13 CompatibilityLevel 1413 CompressObjects Tags13 CompressPages true13 ConvertImagesToIndexed true13 PassThroughJPEGImages true13 CreateJobTicket false13 DefaultRenderingIntent Default13 DetectBlends true13 DetectCurves 0000013 ColorConversionStrategy CMYK13 DoThumbnails false13 EmbedAllFonts true13 EmbedOpenType false13 ParseICCProfilesInComments true13 EmbedJobOptions true13 DSCReportingLevel 013 EmitDSCWarnings false13 EndPage -113 ImageMemory 104857613 LockDistillerParams false13 MaxSubsetPct 10013 Optimize true13 OPM 113 ParseDSCComments true13 ParseDSCCommentsForDocInfo true13 PreserveCopyPage true13 PreserveDICMYKValues true13 PreserveEPSInfo true13 PreserveFlatness true13 PreserveHalftoneInfo false13 PreserveOPIComments true13 PreserveOverprintSettings true13 StartPage 113 SubsetFonts true13 TransferFunctionInfo Apply13 UCRandBGInfo Preserve13 UsePrologue false13 ColorSettingsFile ()13 AlwaysEmbed [ true13 ]13 NeverEmbed [ true13 ]13 AntiAliasColorImages false13 CropColorImages true13 ColorImageMinResolution 30013 ColorImageMinResolutionPolicy OK13 DownsampleColorImages true13 ColorImageDownsampleType Bicubic13 ColorImageResolution 30013 ColorImageDepth -113 ColorImageMinDownsampleDepth 113 ColorImageDownsampleThreshold 15000013 EncodeColorImages true13 ColorImageFilter DCTEncode13 AutoFilterColorImages true13 ColorImageAutoFilterStrategy JPEG13 ColorACSImageDict ltlt13 QFactor 01513 HSamples [1 1 1 1] VSamples [1 1 1 1]13 gtgt13 ColorImageDict ltlt13 QFactor 01513 HSamples [1 1 1 1] VSamples [1 1 1 1]13 gtgt13 JPEG2000ColorACSImageDict ltlt13 TileWidth 25613 TileHeight 25613 Quality 3013 gtgt13 JPEG2000ColorImageDict ltlt13 TileWidth 25613 TileHeight 25613 Quality 3013 gtgt13 AntiAliasGrayImages false13 CropGrayImages true13 GrayImageMinResolution 30013 GrayImageMinResolutionPolicy OK13 DownsampleGrayImages true13 GrayImageDownsampleType Bicubic13 GrayImageResolution 30013 GrayImageDepth -113 GrayImageMinDownsampleDepth 213 GrayImageDownsampleThreshold 15000013 EncodeGrayImages true13 GrayImageFilter DCTEncode13 AutoFilterGrayImages true13 GrayImageAutoFilterStrategy JPEG13 GrayACSImageDict ltlt13 QFactor 01513 HSamples [1 1 1 1] VSamples [1 1 1 1]13 gtgt13 GrayImageDict ltlt13 QFactor 01513 HSamples [1 1 1 1] VSamples [1 1 1 1]13 gtgt13 JPEG2000GrayACSImageDict ltlt13 TileWidth 25613 TileHeight 25613 Quality 3013 gtgt13 JPEG2000GrayImageDict ltlt13 TileWidth 25613 TileHeight 25613 Quality 3013 gtgt13 AntiAliasMonoImages false13 CropMonoImages true13 MonoImageMinResolution 120013 MonoImageMinResolutionPolicy OK13 DownsampleMonoImages true13 MonoImageDownsampleType Bicubic13 MonoImageResolution 120013 MonoImageDepth -113 MonoImageDownsampleThreshold 15000013 EncodeMonoImages true13 MonoImageFilter CCITTFaxEncode13 MonoImageDict ltlt13 K -113 gtgt13 AllowPSXObjects false13 CheckCompliance [13 None13 ]13 PDFX1aCheck false13 PDFX3Check false13 PDFXCompliantPDFOnly false13 PDFXNoTrimBoxError true13 PDFXTrimBoxToMediaBoxOffset [13 00000013 00000013 00000013 00000013 ]13 PDFXSetBleedBoxToMediaBox true13 PDFXBleedBoxToTrimBoxOffset [13 00000013 00000013 00000013 00000013 ]13 PDFXOutputIntentProfile ()13 PDFXOutputConditionIdentifier ()13 PDFXOutputCondition ()13 PDFXRegistryName ()13 PDFXTrapped False1313 CreateJDFFile false13 Description ltlt13 ARA 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 BGR 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 CHS ltFEFF4f7f75288fd94e9b8bbe5b9a521b5efa7684002000410064006f006200650020005000440046002065876863900275284e8e9ad88d2891cf76845370524d53705237300260a853ef4ee54f7f75280020004100630072006f0062006100740020548c002000410064006f00620065002000520065006100640065007200200035002e003000204ee553ca66f49ad87248672c676562535f00521b5efa768400200050004400460020658768633002gt13 CHT ltFEFF4f7f752890194e9b8a2d7f6e5efa7acb7684002000410064006f006200650020005000440046002065874ef69069752865bc9ad854c18cea76845370524d5370523786557406300260a853ef4ee54f7f75280020004100630072006f0062006100740020548c002000410064006f00620065002000520065006100640065007200200035002e003000204ee553ca66f49ad87248672c4f86958b555f5df25efa7acb76840020005000440046002065874ef63002gt13 CZE 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 DAN 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 DEU ltFEFF00560065007200770065006e00640065006e0020005300690065002000640069006500730065002000450069006e007300740065006c006c0075006e00670065006e0020007a0075006d002000450072007300740065006c006c0065006e00200076006f006e002000410064006f006200650020005000440046002d0044006f006b0075006d0065006e00740065006e002c00200076006f006e002000640065006e0065006e002000530069006500200068006f006300680077006500720074006900670065002000500072006500700072006500730073002d0044007200750063006b0065002000650072007a0065007500670065006e0020006d00f60063006800740065006e002e002000450072007300740065006c006c007400650020005000440046002d0044006f006b0075006d0065006e007400650020006b00f6006e006e0065006e0020006d006900740020004100630072006f00620061007400200075006e0064002000410064006f00620065002000520065006100640065007200200035002e00300020006f0064006500720020006800f600680065007200200067006500f600660066006e00650074002000770065007200640065006e002egt13 ESP 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 ETI 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 FRA 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 GRE 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 HEB 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 HRV (Za stvaranje Adobe PDF dokumenata najpogodnijih za visokokvalitetni ispis prije tiskanja koristite ove postavke Stvoreni PDF dokumenti mogu se otvoriti Acrobat i Adobe Reader 50 i kasnijim verzijama)13 HUN 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 ITA 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 JPN ltFEFF9ad854c18cea306a30d730ea30d730ec30b951fa529b7528002000410064006f0062006500200050004400460020658766f8306e4f5c6210306b4f7f75283057307e305930023053306e8a2d5b9a30674f5c62103055308c305f0020005000440046002030d530a130a430eb306f3001004100630072006f0062006100740020304a30883073002000410064006f00620065002000520065006100640065007200200035002e003000204ee5964d3067958b304f30533068304c3067304d307e305930023053306e8a2d5b9a306b306f30d530a930f330c8306e57cb30818fbc307f304c5fc59808306730593002gt13 KOR ltFEFFc7740020c124c815c7440020c0acc6a9d558c5ec0020ace0d488c9c80020c2dcd5d80020c778c1c4c5d00020ac00c7a50020c801d569d55c002000410064006f0062006500200050004400460020bb38c11cb97c0020c791c131d569b2c8b2e4002e0020c774b807ac8c0020c791c131b41c00200050004400460020bb38c11cb2940020004100630072006f0062006100740020bc0f002000410064006f00620065002000520065006100640065007200200035002e00300020c774c0c1c5d0c11c0020c5f40020c2180020c788c2b5b2c8b2e4002egt13 LTH 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 LVI 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 NLD (Gebruik deze instellingen om Adobe PDF-documenten te maken die zijn geoptimaliseerd voor prepress-afdrukken van hoge kwaliteit De gemaakte PDF-documenten kunnen worden geopend met Acrobat en Adobe Reader 50 en hoger)13 NOR ltFEFF004200720075006b00200064006900730073006500200069006e006e007300740069006c006c0069006e00670065006e0065002000740069006c002000e50020006f0070007000720065007400740065002000410064006f006200650020005000440046002d0064006f006b0075006d0065006e00740065007200200073006f006d00200065007200200062006500730074002000650067006e0065007400200066006f00720020006600f80072007400720079006b006b0073007500740073006b00720069006600740020006100760020006800f800790020006b00760061006c0069007400650074002e0020005000440046002d0064006f006b0075006d0065006e00740065006e00650020006b0061006e002000e50070006e00650073002000690020004100630072006f00620061007400200065006c006c00650072002000410064006f00620065002000520065006100640065007200200035002e003000200065006c006c00650072002000730065006e006500720065002egt13 POL 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 PTB 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 RUM ltFEFF005500740069006c0069007a00610163006900200061006300650073007400650020007300650074010300720069002000700065006e007400720075002000610020006300720065006100200064006f00630075006d0065006e00740065002000410064006f006200650020005000440046002000610064006500630076006100740065002000700065006e0074007200750020007400690070010300720069007200650061002000700072006500700072006500730073002000640065002000630061006c006900740061007400650020007300750070006500720069006f006100720103002e002000200044006f00630075006d0065006e00740065006c00650020005000440046002000630072006500610074006500200070006f00740020006600690020006400650073006300680069007300650020006300750020004100630072006f006200610074002c002000410064006f00620065002000520065006100640065007200200035002e00300020015f00690020007600650072007300690075006e0069006c006500200075006c0074006500720069006f006100720065002egt13 RUS 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 SKY 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 SLV 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 SUO ltFEFF004b00e40079007400e40020006e00e40069007400e4002000610073006500740075006b007300690061002c0020006b0075006e0020006c0075006f00740020006c00e400680069006e006e00e4002000760061006100740069007600610061006e0020007000610069006e006100740075006b00730065006e002000760061006c006d0069007300740065006c00750074007900f6006800f6006e00200073006f00700069007600690061002000410064006f0062006500200050004400460020002d0064006f006b0075006d0065006e007400740065006a0061002e0020004c0075006f0064007500740020005000440046002d0064006f006b0075006d0065006e00740069007400200076006f0069006400610061006e0020006100760061007400610020004100630072006f0062006100740069006c006c00610020006a0061002000410064006f00620065002000520065006100640065007200200035002e0030003a006c006c00610020006a006100200075007500640065006d006d0069006c006c0061002egt13 SVE 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 TUR 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 UKR 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 ENU (Use these settings to create Adobe PDF documents best suited for high-quality prepress printing Created PDF documents can be opened with Acrobat and Adobe Reader 50 and later)13 gtgt13 Namespace [13 (Adobe)13 (Common)13 (10)13 ]13 OtherNamespaces [13 ltlt13 AsReaderSpreads false13 CropImagesToFrames true13 ErrorControl WarnAndContinue13 FlattenerIgnoreSpreadOverrides false13 IncludeGuidesGrids false13 IncludeNonPrinting false13 IncludeSlug false13 Namespace [13 (Adobe)13 (InDesign)13 (40)13 ]13 OmitPlacedBitmaps false13 OmitPlacedEPS false13 OmitPlacedPDF false13 SimulateOverprint Legacy13 gtgt13 ltlt13 AddBleedMarks false13 AddColorBars false13 AddCropMarks false13 AddPageInfo false13 AddRegMarks false13 ConvertColors ConvertToCMYK13 DestinationProfileName ()13 DestinationProfileSelector DocumentCMYK13 Downsample16BitImages true13 FlattenerPreset ltlt13 PresetSelector MediumResolution13 gtgt13 FormElements false13 GenerateStructure false13 IncludeBookmarks false13 IncludeHyperlinks false13 IncludeInteractive false13 IncludeLayers false13 IncludeProfiles false13 MultimediaHandling UseObjectSettings13 Namespace [13 (Adobe)13 (CreativeSuite)13 (20)13 ]13 PDFXOutputIntentProfileSelector DocumentCMYK13 PreserveEditing true13 UntaggedCMYKHandling LeaveUntagged13 UntaggedRGBHandling UseDocumentProfile13 UseDocumentBleed false13 gtgt13 ]13gtgt setdistillerparams13ltlt13 HWResolution [2400 2400]13 PageSize [612000 792000]13gtgt setpagedevice13

Page 9: July 2014 MEE Questions and Analyses - NCBE...This publication includes the questions and analyses from the July 2014 MEE. (In the actual test, the questions are simply numbered rather

CONTRACTS QUESTION

A music conservatory has two concert halls One concert hall had a pipe organ that was in poor repair and the other had no organ The conservatory decided to repair the existing organ and buy a new organ for the other concert hall After some negotiation the conservatory entered into two contracts with a business that both repairs and sells organs Under one contract the business agreed to repair the existing pipe organ for the conservatory for $100000 The business would usually charge a higher price for a project of this magnitude but the business agreed to this price because the conservatory agreed to prepay the entire amount Under the other contract the business agreed to sell a new organ to the conservatory for the other concert hall for $225000 As with the repair contract the business agreed to a low sales price because the conservatory agreed to prepay the entire amount Both contracts were signed on January 3 and the conservatory paid the business a total of $325000 that day

Two weeks later before the business had commenced repair of the existing organ the business suffered serious and unanticipated financial reversals The chief financial officer for the business contacted the conservatory and said

Bad news We had an unexpected liability and as a result are in a real cash crunch In fact even though we havenrsquot acquired the new organ from our supplier or started repair of your existing organ wersquove already spent the cash you gave us and we have no free cash on hand Wersquore really sorry but wersquore in a fix I think that we can find a way to perform both contracts but not at the original prices If you agree to pay $60000 more for the repair and $40000 more for the new organ we can probably find financing to finish everything If you donrsquot agree to pay us the extra money I doubt that we will ever be able to perform either contract and yoursquoll be out the money you already paid us

After receiving this unwelcome news the conservatory agreed to pay the extra amounts provided that the extra amount on each contract would be paid only upon completion of the businessrsquos obligations under that contract The business agreed to this arrangement and the parties quickly signed documents reflecting these changes to each contract The business then repaired the existing organ delivered the new organ and demanded payment of the additional $100000

The conservatory now has refused to pay the business the additional amounts for the repair and the new organ

1 Must the conservatory pay the additional $60000 for the organ repair Explain

2 Must the conservatory pay the additional $40000 for the new organ Explain

5

FAMILY LAW QUESTION

In 1994 a man and a woman were married in State A

In 1998 their daughter was born in State A

In 2010 the family moved to State B

In 2012 the husband and wife divorced in State B Under the terms of the divorce decree

(a) the husband and wife share legal and physical custody of their daughter (b) the husband must pay the wife $1000 per month in child support until their daughter reaches age 18 (c) the marital residence was awarded to the wife with the proviso that if it is sold before the daughter reaches age 18 the husband will receive 25 of the net sale proceeds remaining after satisfaction of the mortgage on the residence and (d) the remaining marital assets were divided between the husband and the wife equally

Six months ago the husband was offered a job in State A that pays significantly less than his job in State B but provides him with more responsibilities and much better promotion opportunities The husband accepted the job in State A and moved from State B back to State A

Since returning to State A the husband has not paid child support because due to his lower salary he has had insufficient funds to meet all his obligations

One month ago the wife sold the marital home netting $10000 after paying off the mortgage She then moved to a smaller residence The husband believes that he should receive more than 25 of the net sale proceeds given his financial difficulties

Last week when the wife brought the daughter to the husbandrsquos State A home for a weekend visit the husband served the wife with a summons in a State A action to modify the support and marital-residence-sale-proceeds provisions of the State B divorce decree The husband brought the action in the State A court that adjudicates all domestic relations issues

1 Does the State A court have jurisdiction to modify (a) the child support provision of the State B divorce decree Explain (b) the marital-residence-sale-proceeds provision of the State B divorce decree

Explain

2 On the merits could the husband obtain (a) retroactive modification of his child support obligation to the daughter Explain (b) prospective modification of his child support obligation to the daughter Explain (c) modification of the marital-residence-sale-proceeds provision of the State B

divorce decree Explain

6

FEDERAL CIVIL PROCEDURE QUESTION

The United States Forest Service (USFS) manages public lands in national forests including the Scenic National Forest Without conducting an environmental evaluation or preparing an environmental impact statement the USFS approved a development project in the Scenic National Forest that required the clearing of 5000 acres of old-growth forest The trees in the forest are hundreds of years old and the forest is home to a higher concentration of wildlife than can be found anywhere else in the western United States

The USFS solicited bids from logging companies to harvest the trees on the 5000 acres of forest targeted for clearing and it ultimately awarded the logging contract to the company that had submitted the highest bid for the trees However the USFS has not yet issued the company a logging permit Once it does so the company intends to begin cutting down trees immediately

A nonprofit organization whose mission is the preservation of natural resources has filed suit in federal district court against the USFS The nonprofit alleges that the USFS violated the National Environmental Policy Act (NEPA) by failing to prepare an environmental impact statement for the proposed logging project Among other remedies the nonprofit seeks a permanent injunction barring the USFS from issuing a logging permit to the logging company until an adequate environmental impact statement is completed The nonprofit believes that the logging project would destroy important wildlife habitat and thereby cause serious harm to wildlife in the Scenic National Forest including some endangered species

Assume that federal subject-matter jurisdiction is available that the nonprofit has standing to bring this action and that venue is proper

1 If the logging company seeks to join the litigation as a party must the federal district court allow it to do so as a matter of right Explain

2 What types of relief could the nonprofit seek to stop the USFS from issuing a logging permit during the pendency of the action what must the nonprofit demonstrate to obtain that relief and is the federal district court likely to grant that relief Explain

7

EVIDENCE QUESTION

A prison inmate has filed a civil rights lawsuit against a guard at the prison alleging that the guard violated the inmatersquos constitutional rights during an altercation The inmate and the guard are the only witnesses to this altercation They have provided contradictory reports about what occurred

The trial will be before a jury The inmate plans to testify at trial The guardrsquos counsel has moved for leave to impeach the inmate with the following

(a) Twelve years ago the inmate was convicted of felony distribution of marijuana He served a three-year prison sentence which began immediately after he was convicted He served his full sentence and was released from prison nine years ago (b) Eight years ago the inmate pleaded guilty to perjury a misdemeanor punishable by up to one year in jail He paid a $5000 fine (c) Seven years ago the inmate was convicted of felony sexual assault of a child and is currently serving a 10-year prison sentence for the crime The victim was the inmatersquos daughter who was 13 years old at the time of the assault

The inmatersquos counsel objects to the admission of any evidence related to these three convictions and to any cross-examination based on this evidence

The guard also plans to testify at trial The inmatersquos counsel has moved for leave to impeach the guard with the following

Last year the guard applied for a promotion to prison supervisor The guard submitted a reacutesumeacute to the state that indicated that he had been awarded a BA in Criminal Justice from a local college An official copy of the guardrsquos academic transcript from that college indicates that the guard dropped out after his first semester and did not receive a degree

The guardrsquos counsel objects to the admission of this evidence and to any cross-examination based on this evidence

The transcript and the reacutesumeacute have been properly authenticated The trial will be held in a jurisdiction that has adopted all of the Federal Rules of Evidence

1 What evidence if any proffered by the guard to impeach the inmate should be admitted Explain

2 What evidence if any proffered by the inmate to impeach the guard should be admitted Explain

8

CORPORATIONS QUESTION

Mega Inc is a publicly traded corporation incorporated in a state whose corporate statute is modeled on the Model Business Corporation Act (MBCA) Megarsquos articles of incorporation do not address the election of directors or amendment of the bylaws by shareholders

Well within the deadline for the submission of shareholder proposals for the upcoming annual shareholdersrsquo meeting an investor who was a large and long-standing shareholder of Mega submitted a proposed amendment to Megarsquos bylaws The proposal which the investor asked to be included in the corporationrsquos proxy materials and voted on at the upcoming shareholdersrsquo meeting read as follows

Section 20 The Corporation shall include in its proxy materials (including the proxy ballot) for a shareholdersrsquo meeting at which directors are to be elected the name of a person nominated for election to the Board of Directors by a shareholder or group of shareholders that beneficially have owned 3 or more of the Corporationrsquos outstanding common stock for at least one year

This Section shall supersede any inconsistent provision in these Bylaws and may not be amended or repealed by the Board of Directors without shareholder approval

Megarsquos management decided to exclude the investorrsquos proposal from the corporationrsquos proxy materials and explained its reasons in a letter to the investor

The investorrsquos proposed bylaw provision would be inconsistent with relevant state law because the Board of Directors has the authority to manage the business and affairs of the Corporation Generally shareholders lack the authority to interfere with corporate management by seeking to create a method for the nomination and election of directors inconsistent with the method chosen by the Board of Directors

Furthermore at its most recent meeting the Board of Directors unanimously approved an amendment to the Corporationrsquos bylaws that provides for proxy access for director nominations by a shareholder or a group of shareholders holding at least 10 of the Corporationrsquos voting shares for at least three years This procedure takes precedence over any nomination methods that might be sought or approved by shareholders

The investor is considering bringing a suit challenging managementrsquos refusal to include the investorrsquos proposed bylaw provision and challenging the boardrsquos amendment of the bylaws at its recent meeting

1 Is the investorrsquos proposed bylaw provision inconsistent with state law Explain

2 If the investorrsquos proposed bylaw provision were approved by the shareholders would the bylaw amendment previously approved by the board take precedence over the investorrsquos proposed bylaw provision Explain

3 Must the investor make a demand on Megarsquos board of directors before bringing suit Explain

9

July 2014 MEE

ANALYSES

Contracts Family Law

Criminal Law and Procedure

Federal Civil Procedure Evidence

Corporations

CRIMINAL LAW AND PROCEDURE ANALYSIS (Criminal Law and Procedure VA B D)

ANALYSIS

Legal Problems

(1) Did the detective violate the suspectrsquos Sixth Amendment right to counsel when he questioned the suspect about the burglaries without the lawyer present given that the lawyer represented the suspect in an unrelated criminal matter

(2) Under Miranda did the suspect effectively invoke his right to counsel when he said ldquoI think I want my lawyer here before I talk to yourdquo

(3) Was the suspectrsquos waiver of his right to remain silent under Miranda valid

DISCUSSION

Summary

The Sixth Amendment right to counsel as applied to states through the Fourteenth Amendment is offense-specific Although the suspect had an attorney representing him on his pending assault charge he had no Sixth Amendment right to the assistance of counsel with respect to the five uncharged burglaries because formal adversarial proceedings had not yet commenced on those charges The suspectrsquos Sixth Amendment right to counsel was not violated by the detectiversquos failure to inform him that the lawyer was present or of the lawyerrsquos demands

However a person undergoing custodial interrogation also has an independent constitutional right to counsel during custodial interrogation under Miranda When a suspect invokes his right to counsel under Miranda custodial interrogation must immediately cease for a period of at least 14 days However the invocation of the right to counsel must be unambiguous and clearly convey that the suspect has requested counsel Here because the suspectrsquos statement ldquoI think I want my lawyer here before I talk to yourdquo was ambiguous he did not invoke his Miranda right to counsel

A waiver of rights must be knowing intelligent and voluntary Here the suspect waived his right to remain silent under Miranda when he signed the waiver form The fact that the detective did not correct the suspectrsquos assumption that the lawyer would need to drive to the jailmdashby telling him that the lawyer was in the waiting room and was demanding to see himmdashdid not affect the validity of the suspectrsquos waiver

Point One (35) The suspectrsquos Sixth Amendment right to counsel was not violated because the right does not attach on new charges until formal adversarial judicial proceedings have commenced on those charges

The Sixth Amendment as applied to the states through the Fourteenth Amendment provides that ldquo[i]n all criminal prosecutions the accused shall enjoy the right to have the Assistance of Counsel for his defenserdquo The right to counsel does not attach with respect to particular charges until formal adversarial judicial proceedings have commenced (ie ldquoat or after the initiation of

13

Criminal Law and Procedure Analysis

adversary judicial criminal proceedingsmdashwhether by way of formal charge preliminary hearing indictment information or arraignment [or in some states arrest warrant]rdquo McNeil v Wisconsin 501 US 171 175 (1991) (internal quotations omitted)) Once a suspectrsquos Sixth Amendment right to counsel has attached any attempts to ldquodeliberately elicitrdquo statements from him in the absence of his attorney violate the Sixth Amendment See Massiah v United States 377 US 201 (1964) Brewer v Williams 430 US 387 (1977)

The Sixth Amendment right to counsel is charge- or offense -specific Representation by counsel in one prosecution does not in itself guarantee counsel for uncharged offenses See McNeil 501 US at 175 Texas v Cobb 532 US 162 (2001) Here the suspectrsquos Sixth Amendment right to counsel had attached only for the pending aggravated assault charge The suspectrsquos right to counsel for the aggravated assault case did not guarantee counsel for the five unrelated and uncharged burglaries that were the subject of the detectiversquos interrogation Thus because formal adversarial judicial proceedings against the suspect for the uncharged burglaries had not begun he had no Sixth Amendment right to counsel

Finally the detectiversquos failure to inform the suspect of the lawyerrsquos presence and demands to speak with him does not implicate the suspectrsquos Sixth Amendment right to counsel which had not yet attached See id Moran v Burbine 475 US 412 428ndash31 (1986)

Point Two (30) The suspect did not effectively invoke his right to counsel under Miranda because his statement was not unambiguous

A suspect subject to custodial interrogation has a right to consult with counsel and to have an attorney present during questioning Miranda v Arizona 384 US 436 (1966) When a suspect invokes his right to counsel during an interrogation law enforcement must immediately cease all questioning See Edwards v Arizona 451 US 477 484ndash85 (1981) Custodial interrogation cannot be reinitiated unless and until the suspect has been re-advised of his Miranda rights has provided a knowing and voluntary waiver and (1) counsel is present and (2) the suspect himself initiated further communication with the police see id at 484 or (3) (if the suspect was released from custody after the initial interrogation) at least 14 days have passed Maryland v Shatzer 559 US 98 110 (2010)

To invoke the right to counsel a suspectrsquos request must be ldquounambiguousrdquo This means that the suspect must articulate the desire for counsel sufficiently clearly that a reasonable officer would understand the statement to be a request for counsel Davis v United States 512 US 452 459 (1994) If the request is ambiguous the police are not required to stop the interrogation

In this case the suspectrsquos statement ldquoI think I want my lawyer here before I talk to yourdquo was not an unambiguous request for counsel The most reasonable interpretation of this statement is that the suspect might be invoking his right to counsel Id at 461 (ldquomaybe I should talk to a lawyerrdquo is not an unequivocal request for counsel) See also Burket v Angelone 208 F3d 172 197ndash98 (4th Cir 2000) (ldquoI think I need a lawyerrdquo is not an unambiguous request for an attorney) Soffar v Cockrell 300 F3d 588 594ndash95 (5th Cir 2002) (discussion of various statements that did not constitute unequivocal requests for counsel)

Under these circumstances the detective was not required to cease the custodial interrogation of the suspect Nor was the detective required to clarify or ask follow-up questions to determine whether the suspect in fact wanted an attorney Davis 512 US at 459ndash60

14

Criminal Law and Procedure Analysis

Point Three (35) The suspectrsquos waiver of his Miranda rights was knowing intelligent and voluntary despite the fact that he was never told of the lawyerrsquos presence in the jail or of the lawyerrsquos demands

A valid waiver of Miranda rights must be ldquovoluntaryrdquomdashie the product of a free or deliberate choice rather than intimidation coercion or deception Berghuis v Thompkins 560 US 370 382ndash83 (2010) In addition the waiver must be knowing and intelligent That is it ldquomust have been made with a full awareness of both the nature of the right being abandoned and the consequences of the decision to abandon itrdquo Moran v Burbine 475 US 421 (1986)

In this case the suspect signed a Miranda waiver form after receiving proper warnings There is no evidence ldquothat the police resorted to physical or psychological pressure to elicit the statementsrdquo Id The entire interview lasted only 45 minutes The only issue is whether the suspect knowingly and intelligently waived his Miranda rights despite the fact that the detective did not tell the suspect about the lawyerrsquos presence and her demands

The Supreme Court has said that ldquo[e]vents occurring outside of the presence of the suspect and entirely unknown to him surely can have no bearing on the capacity to comprehend and knowingly relinquish a constitutional rightrdquo Id at 422 If the suspect ldquoknew that he could stand mute and request a lawyer and was aware of the Statersquos intention to use his statements to secure a convictionrdquo then the waiver is valid regardless of the information withheld Id at 422ndash23

Here the suspect was correctly informed of his rights Miranda v Arizona 384 US at 467ndash73 His comments demonstrate that he understood that he could have a lawyer present if he desired (ie wondering whether he should call his attorney) and that he understood that there might be consequences to speaking with the detective (ldquoI probably should keep my mouth shut but Irsquom willing to talk to you for a whilerdquo) His comment ldquo[L]etrsquos not waste any time waiting for someone to call my attorney and having her drive hererdquo along with his signature on the Miranda waiver form show that his waiver was valid under the constitutional standard

The fact that the detective did not tell the suspect about the lawyerrsquos presence and demands has no bearing on the validity of the suspectrsquos waiver because ldquosuch conduct is only relevant to the constitutional validity of a waiver if it deprives a defendant of knowledge essential to his ability to understand the nature of his rights and the consequences of abandoning themrdquo Moran at 424 The Supreme Court has specifically declined to adopt a rule requiring that law enforcement tell a suspect of an attorneyrsquos efforts to contact him id at 425 (ldquoNor are we prepared to adopt a rule requiring that the police inform a suspect of an attorneyrsquos efforts to reach himrdquo)

[NOTE An examinee might also recognize that this general rule is further supported by the Supreme Courtrsquos decision in Florida v Powell 559 US 50 (2010) approving state Miranda warnings that do not explicitly warn suspects that they have a right to have counsel present during custodial interrogation]

15

CONTRACTS ANALYSIS (Contracts IB2 IIB IVA3 amp A5)

ANALYSIS

Legal Problems

(1) In the case of a service contract (governed by the common law of contracts) is a modification enforceable when a party agrees to pay more for the same performance than was originally promised

(2) In the case of a contract for the sale of goods (governed by Article 2 of the UCC) is a modification enforceable when a party agrees to pay more for the same goods than was originally promised

(3) May a party avoid an agreement on the basis of economic duress

DISCUSSION

Summary

There are two arguments that the conservatory can make to support the claim that it is not bound to pay the higher prices lack of consideration and economic duress

The organ repair contract is governed by the common law of contracts Under the common law the business would have difficulty recovering the additional $60000 for the organ repair because under the ldquopreexisting duty rulerdquo the agreement of the conservatory to pay the extra price was not supported by consideration However the business might argue that the modification is enforceable under an exception to the preexisting duty rule for fair and equitable modifications made in light of unanticipated circumstances

The organ sale contract is governed by Article 2 of the Uniform Commercial Code The business would likely recover the additional amount under that contract because Article 2 provides that consideration is not required for a modification to be binding

In both cases the conservatory could seek to avoid its agreement on the grounds of economic duress but that argument is not likely to succeed

Point One (45) The business probably cannot recover the additional $60000 for the organ repair because the conservatoryrsquos promise to pay more money was not supported by consideration

The general rule is that to be enforceable a promise must be supported by consideration Under RESTATEMENT (SECOND) OF CONTRACTS sect 71 a promise is supported by consideration if it is bargained for in exchange for a return promise or performance However under the ldquopreexisting duty rulerdquo (exemplified in RESTATEMENT (SECOND) OF CONTRACTS sect 73 and Alaska Packersrsquo Assrsquon v Domenico 117 F 99 (9th Cir 1902)) promise of performance of a legal duty already owed to a promisor which is neither doubtful nor the subject of honest dispute is not consideration

If the business had promised the conservatory anything new or different in exchange for the agreement to pay the additional $60000 (such as for example repairing the pipe organ more

16

Contracts Analysis

quickly or using better parts) that would constitute consideration especially in light of the principle that courts do not inquire into the adequacy of consideration Here however the business already had a legal duty under the original contract and did not agree to do anything else in exchange for the conservatoryrsquos promise to pay $60000 more

However an exception to the preexisting duty rule is sometimes applied in situations of unanticipated changed circumstances Under RESTATEMENT (SECOND) OF CONTRACTS sect 89 followed in many jurisdictions a promise modifying a duty under a contract not fully performed on either side is binding even if not supported by consideration if the modification is fair and equitable in view of circumstances not anticipated by the parties when the contract was made

If a court applies the rule in Restatement sect 89 the critical issues will be whether the modification was in fact ldquofair and equitablerdquo and whether it can be justified in light of unanticipated circumstances In many cases in which modifications have been upheld a party encountered difficulties or burdens in performing far beyond what was knowingly bargained for in the original contract with the result bordering on impracticability such as having to excavate solid rock instead of soft dirt or having to remove garbage far in excess of the amounts contemplated The conservatory would argue that the businessrsquos performance difficulties were not of this sort at allmdashnothing about repairing the pipe organ itself was any different from or more difficult than originally contemplated except that the business itself encountered financial distress unrelated to its burdens in performing its obligations under these contracts

Even if the business satisfies that element of the rule in Restatement sect 89 the business must also demonstrate that the circumstances that gave rise to the need to modify the contract were ldquounanticipatedrdquo at the time the original contract was made Here the facts suggest that when the business entered into the original contract it expected that the price paid by the conservatory would enable it to perform However any evidence that the business knew or had reason to know at the time of execution that it would need more money from the conservatory to be able to perform would mean that the request to modify was not ldquounanticipatedrdquo

[NOTE Some cases such as Schwartzreich v Bauman-Basch Inc 231 NY 196 131 NE 887 (1921) find that if the parties mutually agreed to rescind the original contract and then after rescission entered into an entirely new contract for a higher price the new contract is supported by consideration There is no evidence that such a rescission followed by a new contract took place here]

Point Two (45) The business can recover the additional $40000 for the new organ because no consideration is required under Article 2 of the UCC for good-faith contract modifications

The contract to buy a new organ is a contract for the sale of goods and therefore is governed by Article 2 of the Uniform Commercial Code UCC sect 2-102 Under Article 2 unlike the common law an agreement modifying a contract needs no consideration to be binding UCC sect 2-209(1) Section 2-209(1) thus obviates the preexisting duty rule entirely in contracts for the sale of goods

Even though consideration is not required modifications governed by sect 2-209 must satisfy the obligation of good faith imposed by the UCC UCC sect 1-304 See also Official Comment 2 to UCC sect 2-209 Good faith means ldquohonesty in fact and the observance of reasonable commercial standards of fair dealingrdquo UCC sect 1-201(b)(20) In this context the obligation of good faith means that ldquo[t]he effective use of bad faith to escape performance on the original contract terms is barred and the extortion of a lsquomodificationrsquo without legitimate commercial reason is ineffective as a violation of the duty of good faithrdquo Official Comment 2 to

17

Contracts Analysis

UCC sect 2-209 Here because the businessrsquos financial reversals were serious and apparently unanticipated at the time that the business entered into the contract with the conservatory and commitment of the extra money was needed to enable the business to perform a court would likely find that the business acted in good faith Thus a court would likely uphold the enforceability of the conservatoryrsquos promise to pay the additional $40000

Point Three (10) The conservatory is unlikely to be able to defend against enforcement of its promises to pay additional money under the theory of economic duress because the business probably did not make an improper threat

Under the common law of contracts parties may raise the defense of duress This common law defense also applies to contracts governed by UCC Article 2 See UCC sect 1-103(b)

A contract is voidable on the ground of economic duress by threat when it is established that a partyrsquos manifestation of assent is induced by an improper threat that leaves the party no reasonable alternative See RESTATEMENT (SECOND) OF CONTRACTS sect 175 See also eg Austin Instrument Inc v Loral Corp 272 NE2d 533 (NY 1971) (a threat to withhold essential goods can constitute duress) In order to void its agreement to pay the additional sum because of economic duress the conservatory must demonstrate that (1) the business made a threat to the conservatory (2) the threat was ldquoimproperrdquo or ldquowrongfulrdquo (3) the threat induced the conservatoryrsquos manifestation of assent to the modification and (4) the threat was sufficiently grave to justify the conservatoryrsquos assent

Here it appears that three of the four elements are likely satisfied The business plainly made a threat Moreover the threat induced the conservatoryrsquos assent to the modification and the threat was sufficiently grave to justify that assent If the conservatory had not agreed to pay the business the extra amounts the conservatory would have lost its entire $325000 investment In light of this potential loss a court could easily conclude that the conservatory had no reasonable alternative

However the business has a strong argument that its threat (indicating that it would breach the contracts unless the prices were increased) was not wrongful or improper but was instead nothing more than a communication of the reality of its own perilous situation to the conservatory

A mere threat to breach a contract is not in and of itself improper so as to support an action of economic duress or business compulsion Something more is required such as a breach of the duty of good faith and fair dealing as was present in Austin Instrument Inc supra Because the business could not perform the original contract without the requested modification the economic duress claim for the conservatory would likely fail for much the same reason that the business would be able to enforce the modification At the time the modification was requested the business was not trying to extort a price increase because of the conservatoryrsquos vulnerability but instead was simply stating the reality that the business could not perform without more money

18

FAMILY LAW ANALYSIS (Family Law IIIB D amp G)

ANALYSIS

Legal Problems

(1)(a) Does the State A court have jurisdiction to modify the State B child support order

(1)(b) Does the State A court have jurisdiction to modify the marital-residence-saleshyproceeds provision of the State B property-division decree

(2)(a) May a child support order be modified retroactively

(2)(b) May a child support order be modified prospectively based on a change of employment with a lower salary

(2)(c) May a property-division order be modified after entry of a divorce decree

DISCUSSION

Summary

The State A court may exercise personal jurisdiction over the wife because she was personally served in State A However subject-matter jurisdiction over the interstate modification of child support is governed by the Uniform Interstate Family Support Act (UIFSA) Under UIFSA State A does not have jurisdiction to modify the order for the daughterrsquos support because the wife is still a resident of State B UIFSA on the other hand does not govern property distributions and thus a State A court is not precluded from hearing the husbandrsquos petition to modify the marital-residence-sale-proceeds provision of the divorce decree

A child support order may not be modified retroactively A child support order may be modified prospectively based on a substantial change in circumstances Courts agree that a significant decrease in income is a substantial change in circumstances All states treat voluntary income reductions differently than involuntary reductions but employ different approaches for evaluating the impact of a voluntary reduction Whether the husband could obtain prospective modification of the child support order depends on which approach is applied

A property-division order is not subject to post-divorce modification based on a change in circumstances Thus the husband may in some states obtain prospective modification of the order for the daughterrsquos support but he may not obtain modification of the marital-residenceshysale-proceeds provision

Point One(a) (25) Personal jurisdiction over a nonresident respondent does not confer subject-matter jurisdiction over child support modification Under UIFSA a State A court may not modify a child support order issued by a State B court when as here the child or either parent continues to reside in State B the jurisdiction that issued the child support order

The State A court may exercise personal jurisdiction over the wife The wife was personally served in State A and a state may exercise jurisdiction based on in-state personal service See

19

Family Law Analysis

Burnham v Superior Court 495 US 604 (1990) But personal jurisdiction over the wife is not enough to give a State A court jurisdiction to modify the State B support order

The interstate enforcement and modification of child support is governed by the Uniform Interstate Family Support Act (UIFSA) which has been adopted by all states Under UIFSA the state that originally issued a child support order (here State B) has continuing exclusive jurisdiction to modify the order if that state remains the residence of the obligee the child or the obligor and all parties do not consent to the jurisdiction of another forum See UIFSA sect 205 See also UIFSA sect 603 (ldquoA tribunal of this State shall recognize and enforce but may not modify a registered order if the issuing tribunal had jurisdictionrdquo) The wife and daughter continue to reside in State B and the wife has not consented to the jurisdiction of another forum Thus a State A court does not have jurisdiction to modify the State B child support order

[NOTE Examinees who do not discuss personal jurisdiction but fully discuss UIFSA may receive full credit]

Point One(b) (15) UIFSA does not apply to disputes over property division Thus the State A court may exercise jurisdiction over the husbandrsquos petition to modify the marital-residence-sale-proceeds provision of the State B divorce decree because it has personal jurisdiction over the wife

The State A court in which the husband brought his action has jurisdiction to adjudicate domestic relations issues The husbandrsquos petition to modify the property settlement is a domestic relations issue The courts of State A may exercise personal jurisdiction over the wife because she was personally served in State A See Burnham v Superior Court 495 US 604 (1990) see Point One(a)

UIFSA does not apply to divorce property-division disputes Thus although a State A court may not adjudicate the husbandrsquos petition to modify his child support obligations it may adjudicate his property-division claims (Even though the court has jurisdiction it may not modify the property-division award on the merits See Point Two(c))

Point Two(a) (20) A child support order may not be modified retroactively

State courts have long held that obligations to pay child support ordinarily may not be modified retroactively ldquoIf the hardship is particularly severe the courts sometimes devised a way to protect the obligor but in most instances the courts hold that retroactive modification of this kind is beyond their power and indeed the governing statute may so providerdquo HOMER H CLARK THE LAW OF DOMESTIC RELATIONSHIPS IN THE UNITED STATES 725 (2d ed 1987)

Federal law now goes further and requires the states as a condition of federal child-support funding to adopt rules that absolutely forbid retroactive modification of the support obligation See 42 USC sect 666(a)(9)(C) The states have adopted rules consistent with the federal requirements

Point Two(b) (25) It is unclear whether the husband could obtain prospective downward modification of his child support based on his voluntary acceptance of a job with a lower salary

Prospective modification of a child support order is typically available only when the petitioner can show a substantial change in circumstances See ROBERT E OLIPHANT amp NANCY VER

20

Family Law Analysis

STEEGH FAMILY LAW 213ndash15 (3d ed 2010) A significant decrease in income is typically viewed as a substantial change

However when a parent seeks to modify a child support obligation because he has voluntarily reduced his income a court will not modify the obligation based solely on the income loss Some courts refuse to modify whenever the income shift was voluntary See eg Aguiar v Aguiar 127 P3d 234 (Idaho Ct App 2005) Others look primarily to the petitionerrsquos intentions and permit downward modification if he has acted in good faith See eg In re Marriage of Horn 650 NE2d 1103 (Ill App Ct 1995) Many courts use a multifactor approach See OLIPHANT amp VER STEEGH supra 217ndash18

Here there is no question that the husbandrsquos loss of income was voluntary In a jurisdiction in which voluntary income reduction bars support modification the husbandrsquos petition would be denied

In a jurisdiction employing a good-faith or multifactor approach it is possible but not certain that the husband could obtain downward modification The evidence supports the husbandrsquos good faith his change in employment appears to be based on his new jobrsquos greater responsibilities and better promotion possibilities In a jurisdiction using a multifactor approach the court would likely also consider the impact of such a shift on the daughter the likely duration of the husbandrsquos income loss and the likelihood of a promotion that would ultimately inure to the daughterrsquos benefit Thus on these facts it is possible but by no means certain that the husband could prospectively obtain downward modification of his child support obligation to his daughter

Point Two(c)(15) A divorce property-division award is not subject to modification

A support order is aimed at meeting the post-divorce needs of the supported individual Because the future is unpredictable courts are empowered to modify a support award to take account of changed circumstances that may occur during the period in which support is paid

By contrast a property-distribution award divides assets of the marriage based on the equities at the time of divorce Because the past can be ascertained a property-division award is not subject to post-divorce modification See HARRY A KRAUSE ET AL FAMILY LAW CASES COMMENTS AND QUESTIONS 691 (6th ed 2007)

Here the husband is seeking modification of a property-division award with respect to an asset owned by the parties at the time of divorce Thus the husband may not obtain a modification of the marital-residence-sale-proceeds provision of the divorce decree based on his reduced income

21

FEDERAL CIVIL PROCEDURE ANALYSIS (Federal Civil Procedure III IVC)

ANALYSIS

Legal Problems

(1) Is the logging company entitled to join this action as a matter of right

(2)(a) May the nonprofit organization obtain a temporary restraining order to stop the USFS from issuing a logging permit

(2)(b) May the nonprofit organization obtain a preliminary injunction to stop the USFS from issuing a logging permit during the pendency of the action

DISCUSSION

Summary

The logging company is entitled to intervene in this action as a matter of right because it has an interest in the property or transaction that is the subject of the action and is so situated that its interest may be impaired or impeded as a practical matter if the action goes forward without it The logging companyrsquos interest is not adequately represented by the USFSrsquos presence in the lawsuit

The nonprofit organization may seek a temporary restraining order (TRO) followed by a preliminary injunction to prevent the USFS from issuing a logging permit pending the outcome of the action The nonprofit is likely to obtain a TRO if it can demonstrate a risk of immediate and irreparable injury The nonprofit is also likely to obtain a preliminary injunction if it can demonstrate a significant threat of irreparable harm and a likelihood of success on the merits of its National Environmental Policy Act (NEPA) claim

Point One (50) Rule 24(a) of the Federal Rules of Civil Procedure requires federal courts to allow a person to intervene in an action as a matter of right if the person a) is interested in the property or transaction that is the subject of the action b) is so situated that its interest may be impaired or impeded if the litigation goes forward without it and c) is not adequately represented by existing parties Here the logging company likely meets all three requirements and should be allowed to intervene as a matter of right

Rule 24 of the Federal Rules of Civil Procedure governs intervention the process by which a non-party to an action may join the litigation Under Rule 24(a) (intervention of right) a person must be permitted to intervene if three conditions are met (1) the movant ldquoclaims an interest relating to the property or transaction that is the subject of the actionrdquo (2) the movant ldquois so situated that disposition of the action may as a practical matter impair or impede the movantrsquos ability to protect its interestrdquo and (3) ldquoexisting partiesrdquo do not ldquoadequately represent [the movantrsquos] interestrdquo FED R CIV P 24(a) The three requirements for intervention of right are often ldquovery interrelatedrdquo 7C CHARLES ALAN WRIGHT ET AL FEDERAL PRACTICE AND PROCEDURE sect 1908 at 297 (2007 amp 2011 Supp)

22

Federal Civil Procedure Analysis

Here the court should find that the logging company meets this test First the logging company has a strong interest in the property or transaction that is the subject of this action The USFS has accepted the logging companyrsquos bid and the logging company is merely awaiting issuance of a logging permit to begin logging The nonprofit organization is seeking to prevent this logging The logging company therefore has a strong direct and substantial interest in the subject matter of the lawsuit and in having its winning bid honored and a logging permit issued See eg Kleissler v US Forest Serv 157 F3d 964 972 (3d Cir 1998) (stating that ldquo[t]imber companies have direct and substantial interests in a lawsuit aimed at halting loggingrdquo) see also Natural Resources Defense Council v US Nuclear Regulatory Commrsquon 578 F2d 1341 1343ndash 44 (10th Cir 1978) (holding that applicants whose license renewals were pending had Rule 24(a)(2) interests where the lawsuit sought to halt the license-issuing process pending preparation of environmental impact statements) See generally 7C WRIGHT ET AL supra sect 19081 at 309 (ldquoIf there is a direct substantial legally protectable interest in the proceedings it is clear that this requirement of the rule is satisfiedrdquo) Second the logging companyrsquos interest in receiving a logging permit may well be impaired as a practical matter by the outcome of the lawsuit If the USFS loses the lawsuit it will have to prepare an environmental impact statement before issuing the logging companyrsquos permit This will at a minimum delay the logging companyrsquos ability to exercise its rights and may in the long r un mean that no logging permit is ever issued Intervention of right is not limited to those that would be legally bound as a matter of preclusion doctrine Id sect 19082 at 368 Rather ldquo[t]he rule is satisfied whenever disposition of the present action would put the movant at a practical disadvantage in protecting its interestrdquo Id sect 19082 at 369 Here that condition is easily satisfied See Kleissler 157 F3d at 972 (ldquoTimber companies have direct and substantial interests in a lawsuit aimed at halting logging rdquo)

Given that the logging company has an interest that may be impaired by disposition of the action it should be allowed to intervene unless the court is persuaded that the USFS adequately represents the logging companyrsquos interest See Rule 24(a)(2) 7C WRIGHT ET AL supra sect 1909 Here it could be argued that the USFS adequately represents the logging companyrsquos interest because the USFS presumably wants the court to uphold its development plan and allow it to proceed with issuance of the logging permit which is the same relief that the logging company would seek However whether representation is truly adequate depends upon ldquo[a] discriminating appraisal of the circumstancesrdquo 7C WRIGHT ET AL supra sect 1909 at 440 Although both the government and the logging company wish to avoid the preparation of an environmental impact statement their interests are distinct The USFSrsquos interest is proper management of the national forest system while the logging companyrsquos interest is making a profit from logging the 5000-acre tract The USFSrsquos handling of the litigation is likely to be affected by a variety of policy concerns and political considerations that have nothing to do with the logging companyrsquos purely economic interest in securing the right to cut trees in the Scenic National Forest See eg Kleissler 157 F3d at 973ndash74 (ldquo[T]he government represents numerous complex and conflicting interests in matters of this nature The straightforward business interests asserted by intervenors here may become lost in the thicket of sometimes inconsistent governmental policiesrdquo)

[NOTES (1) Examinees who mistakenly analyze the logging companyrsquos case for joinder under the related but incorrect Rule 19 ldquoRequired Joinder of Partiesrdquo may receive credit Rule 19 allows existing parties to demand joinder of non-parties (or seek dismissal of the case if they canrsquot get it) There is a close relationship between Rule 24 and Rule 19 and both contain a similar standard for determining when ldquointerestedrdquo third parties are ldquoentitledrdquo or ldquorequiredrdquo to be in the lawsuit Indeed the two prongs of the Rule 24 intervention test that are discussed above

23

Federal Civil Procedure Analysis

are nearly identical to the two prongs of the Rule 19(a) required joinder test Examinees who discuss and apply the test should receive credit even if they cite Rule 19 rather than Rule 24

(2) Examinees may discuss permissive joinder Although permissive joinder is a possibility here the question asks only whether the logging company can join the action as a matter of right and a permissive joinder analysis is not responsive to the question To the extent an examinee discusses permissive joinder the analysis will focus on whether the logging company ldquohas a claim or defense that shares with the main action a common question of law or factrdquo FED R CIV P 24(b)(1)(B) The district court also ldquomust consider whether the intervention will unduly delay or prejudice the adjudication of the original partiesrsquo rightsrdquo FED R CIV P 24(b)(3) On our facts the logging companyrsquos claim for the issuance of a logging permit would certainly share common questions of law and fact with the USFSrsquos defense against the nonprofitrsquos claim There are no facts suggesting that the logging companyrsquos presence would unduly delay or otherwise prejudice adjudication of the original action Thus the district court would have discretion to permit the logging company to intervene even if it denied intervention of right]

Point Two(a) (25) The nonprofit organization could seek and would likely obtain a temporary restraining order to stop the USFS from issuing a logging permit pending a hearing on an application for a preliminary injunction

The first type of interim relief the nonprofit could seek to stop the USFS from issuing a logging permit to the logging company is a temporary restraining order (TRO) prohibiting the USFS from issuing the logging permit A TRO can be issued without notice to the adverse party but only in limited circumstances and only for a limited time FED R CIV P 65(b) To secure a TRO without notice the nonprofit would need to submit an affidavit containing specific facts that demonstrate a risk of ldquoimmediate and irreparable injuryrdquo if a permit is issued FED R CIV P 65(b)(1) In deciding whether to grant a TRO courts will also consider the same factors that are relevant in deciding whether to grant a preliminary injunction (eg the moving partyrsquos likelihood of success on the merits the balance of hardships and the public interest) See Point Two(b) infra The TRO would last only long enough for the court to consider and resolve a request by the nonprofit for a preliminary injunction but no longer than 14 days (unless the court extends it for good cause or the adverse party consents to an extension) In addition bond is required

Here the court is likely to grant the nonprofitrsquos request The nonprofit could plausibly claim that cutting down 5000 acres of old-growth forest in an area that is home to the highest concentration of wildlife in the western United States would have ldquoan immediate and irreparablerdquo adverse impact on the environment and cause irreparable harm to the nonprofitrsquos interest in preserving and protecting natural resources including wildlife habitat

Point Two(b) (25) The nonprofit could also seek and would likely obtain a preliminary injunction to stop the USFS which is likely to be granted if the nonprofitrsquos claim that the USFS violated NEPA has a strong basis in fact and law

Because the TRO would be temporary the nonprofit would need to move for a preliminary injunction to prevent the USFS from issuing a logging permit throughout the pendency of the litigation Preliminary injunctions are injunctions that seek to ldquoprotect [the] plaintiff from

24

Federal Civil Procedure Analysis

irreparable injury and to preserve the courtrsquos power to render a meaningful decision after a trial on the meritsrdquo 11A CHARLES ALAN WRIGHT ET AL FEDERAL PRACTICE AND PROCEDURE sect 2947 at 112 (2013) Rule 65 of the Federal Rules of Civil Procedure sets out the procedural requirements for preliminary injunctions Preliminary injunctions may be granted only upon notice to the adverse party FED R CIV P 65(a)(1) and only if the movant ldquogives security in an amount that the court considers proper to pay the costs and damages sustained by any party found to have been wrongfully enjoined or restrainedrdquo FED R CIV P 65(c)

While Rule 65 sets out the procedural requirements for preliminary injunctive relief it does not specify the substantive grounds upon which it may be granted The courtrsquos discretion in ruling upon a motion for a preliminary injunction ldquois exercised in conformity with historic federal equity practicerdquo 11A WRIGHT ET AL supra sect 2947 at 114 The court typically considers four factors

(1) the significance of the threat of irreparable harm to the plaintiff if the injunction is not granted (2) the balance between this harm and the injury that granting the injunction would inflict on the defendant (3) the probability that the plaintiff will succeed on the merits and (4) the public interest

Id sect 2948 at 122ndash24 accord Habitat Educ Center v Bosworth 363 F Supp 2d 1070 1088 (ED Wis 2005) The most important of these factors is the risk of irreparable harm to the plaintiff 11A WRIGHT ET AL supra sect 29481 at 129 If the plaintiff has an adequate remedy at law (eg if money damages can compensate the plaintiff for its loss) then a preliminary injunction will be denied Id sect 29481

Here a court would likely conclude that the potential for environmental damage to the forest creates a significant threat of irreparable harm ldquo[E]nvironmental injury is often irreparable Courts have recognized that logging such as would occur [here] can have longshyterm environmental consequences and thus satisfy the irreparable injury criterionrdquo Habitat Educ Center 363 F Supp 2d at 1089 (citing Idaho Sporting Congress Inc v Alexander 222 F3d 562 569 (9th Cir 2000) (noting that the imminent and continuing logging activities presented ldquoevidence of environmental harm sufficient to tip the balance in favor of injunctive reliefrdquo)) Neighbors of Cuddy Mountain v US Forest Service 137 F3d 1372 1382 (9th Cir 1998) (stating that ldquo[t]he old growth forests plaintiffs seek to protect would if cut take hundreds of years to reproducerdquo) (internal citation omitted)) see also 11C WRIGHT ET AL supra sect 29481 at 151 (noting that ldquoa preliminary injunction has been issued to prevent harm to the environmentrdquo)

The second factor the balance between the harm to the plaintiff and the harm the defendant will suffer if the injunction is issued also appears to support issuance of a preliminary injunction here The USFS will have to wait before it can develop the Scenic National Forest and the logging company may lose money if the delay is prolonged These economic harms could be compensated monetarily if an injunction is issued inappropriately Where ldquoan injunction bond can compensate [the] defendant for any harm the injunction is likely to inflict the balance should be struck in favor of [the] plaintiffrdquo Id sect 29482 at 192 See also Habitat Educ Center 363 F Supp 2d at 1089 (stating that ldquothe relative absence of harmful effects on the Forest Service weighs in favor of granting the injunctionrdquo)

The third factor is the likelihood that the plaintiff will prevail on the merits Although there is limited information concerning the merits of the action the nonprofit alleges that the federal statute (NEPA) requires an environmental impact statement and further states that the USFS created no environmental impact analysis or statement at all Assuming that those

25

Federal Civil Procedure Analysis

allegations are correct it seems plausible to conclude that the nonprofit will be able to show a likelihood of success on the merits

Finally courts deciding whether or not to issue preliminary injunctive relief are to consider the public interest ldquoFocusing on this factor is another way of inquiring whether there are policy considerations that bear on whether the order should issuerdquo 11C WRIGHT ET AL supra sect 29484 at 214 If the court concludes that the nonprofit is likely to succeed on its NEPA claim because the USFS wrongfully failed to conduct an environmental impact assessment it is likely to find that the public interest would be served by restraining the USFS from proceeding with logging in a national forest See Heartwood Inc v US Forest Service 73 F Supp 2d 962 979 (SD Ill 1999) affrsquod on other grounds 230 F3d 947 (7th Cir 2000) (ldquoviolations by federal agencies of NEPArsquos provisions as established by Congress harm the public as well as the environmentrdquo)

Thus a court is very likely to grant a preliminary injunction if it concludes that the nonprofit has a significant likelihood of success on the merits

26

EVIDENCE ANALYSIS (Evidence ID IIA amp C)

ANALYSIS

Legal Problems

(1) Under what circumstances can evidence of prior convictions be used to impeach a witnessrsquos credibility in a civil case

(1)(a) May the inmatersquos credibility be impeached by evidence of a 12-year-old felony drug conviction if he was released from prison 9 years ago

(1)(b) May the inmatersquos credibility be impeached by evidence of an 8-year-old misdemeanor perjury conviction that was punishable by 1 year in jail if he pleaded guilty and was sentenced only to pay a $5000 fine

(1)(c) May the inmatersquos credibility be impeached by evidence of a 7-year-old sexual assault conviction if the inmate is still serving a 10-year prison sentence and the victim was his 13-year-old daughter

(2)(a) May the guardrsquos credibility be impeached by cross-examination regarding specific instances of misconduct (ie lying on his reacutesumeacute) relevant to credibility

(2)(b) May the guardrsquos credibility be impeached by admission of extrinsic evidence (his reacutesumeacute and academic transcript) offered to prove specific instances of misconduct relevant to credibility

DISCUSSION

Summary

Under the Federal Rules of Evidence witnesses can be impeached with evidence of prior convictions andor specific instances of misconduct Whether evidence of prior convictions should be admitted to impeach generally depends on the nature of the crime the amount of time that has passed and (only in criminal cases) whether the ldquowitnessrdquo is the defendant FED R EVID 609(a)

In this civil case evidence of the inmatersquos conviction for distribution of marijuana should be admitted to impeach the inmate because he was convicted of a felony and was released from prison fewer than 10 years ago FED R EVID 609(a)(1) Credibility is critically important in this case because the jury will hear conflicting testimony from the two disputing parties and there were no other eyewitnesses to the altercation Under Rule 609(a)(1) the inmatersquos conviction should be admitted because it has some bearing on his credibility and its probative value is not substantially outweighed by concerns of unfair prejudice confusion or delay Id

Evidence of the inmatersquos misdemeanor conviction for perjury must be admitted because the crime ldquorequired provingmdashor the witnessrsquos admittingmdasha dishonest act or false statementrdquo by the inmate FED R EVID 609(a)(2)

27

Evidence Analysis

Evidence of the inmatersquos felony conviction for sexual assault should be excluded because its probative value is substantially outweighed by the danger of unfair prejudice to the inmate based on the heinous nature of the crime FED R EVID 609(a)(1) In the alternative the judge could limit the evidence relating to this conviction by excluding details of the inmatersquos crime

In all civil (and criminal) cases witnesses can also be impeached with evidence of specific instances of prior misconduct that did not result in a conviction FED R EVID 608(b) Pursuant to Rule 608(b) misconduct probative of untruthfulness can be inquired into on cross-examination but cannot be proved through extrinsic evidence Id Thus the inmatersquos counsel should be permitted to cross-examine the guard regarding the false statement in the guardrsquos reacutesumeacute However extrinsic evidence of the guardrsquos misconduct (ie the guardrsquos authenticated reacutesumeacute and transcript from the local college) should not be admitted even if the guard denies wrongdoing or refuses to answer cross-examination questions about these matters Id

Point One (10) The Federal Rules of Evidence permit impeachment of witnesses with evidence of prior convictions

Whether convictions should be admitted to impeach generally depends on the nature of the crime the amount of time that has passed and (only in criminal cases) whether the ldquowitnessrdquo is the defendant FED R EVID 609(a) Under Rule 609(a) evidence of prior convictions may be admitted for the purpose of ldquoattacking a witnessrsquos character for truthfulnessrdquo Id

There are two basic types of convictions that can be admitted for the purpose of impeachment

(1) convictions for crimes ldquopunishable by death or by imprisonment for more than one yearrdquo (which generally correlates to ldquofeloniesrdquo) FED R EVID 609(a)(1) and (2) convictions ldquofor any crimes regardless of the punishment if the court can readily determine that establishing the elements of the crime required provingmdashor the witnessrsquos admittingmdasha dishonest act or false statementrdquo FED R EVID 609(a)(2)

Pursuant to Rule 609(a)(1) in civil cases the admission of evidence of a felony conviction is ldquosubject to Rule 403 [which says that a court may exclude relevant evidence if its probative value is substantially outweighed by other factors]rdquo FED R EVID 609(a)(1) However Rule 403 does not protect the witness against admission of prior convictions involving dishonestymdashwhich must be admitted by the court FED R EVID 609(a)(2)

Finally Federal Rule of Evidence 609(b) contains the presumption that a conviction that is more than 10 years old or where more than 10 years has passed since the witnessrsquos release from confinement (whichever is later) should not be admitted unless ldquoits probative value supported by specific facts and circumstances substantially outweighs its prejudicial effectrdquo and the proponent has provided the adverse party with reasonable written notice FED R EVID 609(b)

Point One(a) (25) The court should admit evidence of the inmatersquos 12-year-old felony marijuana distribution conviction

The inmatersquos conviction for marijuana distribution was for a felony punishable by imprisonment for more than one year See FED R EVID 609(a)(1) Moreover although the conviction was 12 years ago the 10-year time limit of Rule 609(b) is not exceeded because that time limit runs

28

Evidence Analysis

from the date of either ldquothe witnessrsquos conviction or release from confinement for it whichever is laterrdquo FED R EVID 609(b) Because the inmate served three years in prison he was released from confinement nine years ago

However pursuant to Rule 609(a)(1) the admission of felony convictions to impeach a witness in a civil case is ldquosubject to Rule 403rdquo FED R EVID 609(a)(1) Neither Rule 609(a) nor the advisory committee notes specify which factors courts should consider when balancing the probative value of a conviction against the dangers identified in Rule 403 (which include (1) unfair prejudice (2) confusion of the issues (3) misleading the jury (4) waste of time or undue delay and (5) needless presentation of cumulative evidence) FED R EVID 403

In this case credibility is very important because the evidence consists primarily of the testimony of the disputing parties and there were no other eyewitnesses to the altercation This enhances the probative value of any evidence bearing on the inmatersquos credibility A court is likely to conclude that the inmatersquos prior felony drug conviction is relevant to his credibility See eg United States v Brito 427 F3d 53 64 (1st Cir 2005) (ldquoPrior drug-trafficking crimes are generally viewed as having some bearing on veracityrdquo) Although the probative value of any conviction diminishes with age see eg United States v Brewer 451 F Supp 50 53 (ED Tenn 1978) the inmatersquos ongoing problems with the law suggest that he has continued (and even escalated) his criminal behavior over the past nine years The court should admit this evidence because its probative value is not substantially outweighed by any Rule 403 concerns Specifically any prejudice to the inmate would be slight because the conviction is unrelated to the altercation at issue and the conviction was not for a heinous crime that might inflame the jury

[NOTE Whether an examinee identifies the jury instruction as containing a ldquoconclusiverdquo or ldquomandatoryrdquo presumption is less important than the examineersquos analysis of the constitutional infirmities]

Point One(b) (15) The court must admit evidence of the inmatersquos eight-year-old misdemeanor conviction because perjury is a crime of dishonesty

Rule 609(a)(2) provides that evidence of a criminal conviction ldquomust be admitted if the court can readily determine that establishing the elements of the crime required provingmdashor the witnessrsquos admittingmdasha dishonest act or false statementrdquo FED R EVID 609(a)(2) The inmatersquos conviction for perjury would have necessarily required proving that the inmate engaged in an act of dishonesty This conviction occurred within the past 10 years so it ldquomust be admittedrdquo because in contrast to Rule 609(a)(1) (discussed in Point One(a)) admission under Rule 609(a)(2) is mandatory and not subject to Rule 403

Point One(c) (20) The court should exclude evidence of the inmatersquos seven-year-old felony sexual assault conviction because the probative value of this evidence is substantially outweighed by the danger of unfair prejudice In the alternative the details of the prior conviction could be excluded

The inmatersquos conviction for felony sexual assault was seven years ago and he has not yet been released from incarceration so Rule 609(a) but not 609(b) is applicable here FED R EVID 609(a) This conviction is therefore admissible to impeach the inmate unless its probative value is substantially outweighed by the danger of unfair prejudice or any other Rule 403 concern Id

29

Evidence Analysis

Sex crimes are generally not considered relevant to credibility see Hopkins v State 639 So 2d 1247 1254 (Miss 1993) so the probative value of this conviction is relatively low Moreover the heinous nature of the inmatersquos crime (sexual assault on his daughter) makes the danger of unfair prejudice to the inmate very high Thus the court should exclude evidence of the conviction because it was for a heinous offense that is likely to inflame the jury and it has little bearing on credibility See eg United States v Beahm 664 F2d 414 419 (4th Cir 1981)

As an alternative to excluding this evidence the judge could minimize the unfair prejudice to the inmate by permitting limited cross-examination but refusing to allow specific questions about the nature of the inmatersquos conviction For example a court could limit cross-examination to the fact that the inmate was convicted of a ldquofelonyrdquo or perhaps that he was convicted of a ldquosexual assaultrdquo without identifying the victim However because evidence of the inmatersquos prior convictions can be admitted solely for the purpose of enabling the jury to assess his credibility and because his two earlier convictions should have already been admitted the court should exclude all evidence of the felony sexual assault conviction

Point Two(a) (15) The court should permit the inmatersquos counsel to cross-examine the guard regarding the false statement in his reacutesumeacute because the guardrsquos misconduct bears on his truthfulness

The inmate wishes to cross-examine the guard about his prior dishonest behaviormdashlying on his reacutesumeacutemdashthat did not involve a criminal conviction Rule 608(b) allows witnesses to be cross-examined about specific instances of prior non-conviction misconduct probative of untruthfulness ldquoin order to attack the witnessrsquos character for truthfulnessrdquo FED R EVID 608(b)

The courtrsquos decision to allow cross-examination about the guardrsquos prior dishonest behavior depends on the probative value of such evidence balanced against the danger of unfair prejudice to the guard or any other Rule 403 concern FED R EVID 403 Here the guardrsquos false statement on his reacutesumeacute that he obtained a degree in Criminal Justice is highly probative of his untruthfulness because it grossly misrepresents his actual academic record was made recently and was made with the intent to deceive Because the probative value of this evidence is very strong and is not substantially outweighed by any Rule 403 concerns cross-examination of the guard on this topic should be permitted The court may also consider it fair to permit this cross-examination of the guard on these matters assuming that one or more of the inmatersquos prior convictions have been admitted to impeach his credibility

Point Two(b) (15) The court should exclude extrinsic evidence of the guardrsquos non-conviction misconduct even if the guard denies wrongdoing or refuses to answer questions about the matter

Although Rule 608(b) allows cross-examination about specific instances of prior misconduct probative of untruthfulness ldquoextrinsic evidencerdquo offered to prove such misconduct is not admissible FED R EVID 608(b) The rationale for this rule is that allowing the introduction of extrinsic evidence of prior misconduct by witnesses when these acts are relevant only to the witnessesrsquo truthfulness and not to the main issues in the case would create too great a risk of confusing the jury and unduly delaying the trial The court does not have discretion to admit this extrinsic evidence See eg United States v Elliot 89 F3d 1360 1368 (8th Cir 1996)

30

Evidence Analysis

Here the inmatersquos counsel may cross-examine the guard about the false statement on his reacutesumeacute However the inmatersquos counsel must accept the guardrsquos response Even if the guard denies wrongdoing or refuses to answer questions about the matter the inmatersquos counsel cannot introduce the guardrsquos reacutesumeacute or the transcript from the local college to prove the guardrsquos misconduct

31

CORPORATIONS ANALYSIS (Corporations VA2 IX)

ANALYSIS

Legal Problems

(1) Do shareholders have the authority to amend a corporationrsquos bylaws with respect to director nominations

(2) Do board-approved bylaws on a particular subject here nomination of directors preempt subsequent conflicting bylaw amendments by shareholders

(3) Is a suit challenging both managementrsquos refusal to include the proposed bylaw amendment in Megarsquos proxy statement and the boardrsquos amendment of the bylaws dealing with nomination of directors a direct or derivative suit

DISCUSSION

Summary

The voting and litigation rights of the shareholders of Mega are subject to the provisions of the Model Business Corporations Act (MBCA)

The investorrsquos proposed bylaw provision is not inconsistent with state law Under the MBCA shareholders may amend the bylaws when the amendment deals with a proper matter for the corporationrsquos bylaws such as procedures for nominating directors

The Mega boardrsquos bylaw amendment does not preempt the investorrsquos proposed bylaw provision or the Mega shareholdersrsquo power to approve it While shareholders can limit the boardrsquos power to amend or repeal the bylaws the board cannot limit the shareholdersrsquo power

Whether the investor must make a demand on Megarsquos board depends on how the investor frames its claim If the investor claims a violation of shareholder voting rights the claim is direct and pre-suit demand on the board is not required If on the other hand the investor claims that the directors violated their fiduciary duties by amending the bylaws to entrench themselves the claim is derivative and a pre-suit demand is required

Point One (30) Shareholders may amend the corporationrsquos bylaws where the proposed bylaw provision relates to procedural matters typically included in the bylaws such as the nomination of directors

Internal affairs of the corporation such as the conduct of shareholder meetings and election of directors are subject to the corporate law of the state of incorporation See McDermott Inc v Lewis 531 A2d 206 (Del 1987) (applying law of jurisdiction where corporation was incorporated in case involving voting rights) This statersquos corporate statute is modeled on the MBCA

Under the MBCA ldquoshareholders may amend the corporationrsquos bylawsrdquo MBCA sect 1020(a) Thus the only question is whether the bylaws can specify the procedures for shareholder nomination of directors

32

Corporations Analysis

The MBCA states that the bylaws ldquomay contain any provision that is not inconsistent with law or the articles of incorporationrdquo MBCA sect 206(b) In addition the MBCA was revised in 2009 to address shareholder nomination of directors in public corporations (known as ldquoproxy accessrdquo) and specifies that the bylaws ldquomay contain a requirement that the corporation include in its [proxy materials] one or more individuals nominated by a shareholderrdquo MBCA sect 206(c)(1) see Committee on Corporate Laws ABA Section of Business Law Report on the Roles of Boards of Directors and Shareholders of Publicly Owned Corporations and Changes to the Model Business Corporations ActmdashAdoption of Shareholder Proxy Access Amendments to Chapters 2 and 10 65 BUS LAWYER 1105 (2010)

The inclusion of director-nomination procedures in the bylaws is consistent with practice and is recognized by the Delaware courts whose views on corporate law carry significant weight Typically the procedures for nomination of directors are found in the bylaws See 1 COX amp HAZEN TREATISE ON THE LAW OF CORPORATIONS sect 312 (3d ed 2011) see also 4 FLETCHER CORP FORMS ANN PART III ch 21 (2013) (including sample bylaws that permit nomination of directors by shareholders) The Delaware Supreme Court has confirmed that the bylaws may ldquodefine the process and proceduresrdquo for director elections See CA Inc v AFSCME Employees Pension Plan 953 A2d 227 (Del 2008) (concluding that bylaw amendment requiring reimbursement of election expenses to certain successful shareholder nominators is ldquoproper subjectrdquo under Delaware law)

[NOTE The question of the proper scope of the bylaws can be answered using the more general MBCA sect 206(b) or the 2009 MBCA revision adding sect 206(c)(1) (adopted in CT ME VA) In addition some examinees might raise the point that shareholder proposals may not compel the board to take action such as by including shareholder nominations in the companyrsquos proxy materials on the theory that the ldquobusiness and affairsrdquo of the corporation are to be managed by the board See MBCA sect 801(b) Although shareholders are generally limited to adopting precatory resolutions that recommend or encourage board action this limitation does not apply when shareholders have specific authority to take binding action on their ownmdashsuch as to amend the bylaws]

Point Two (30) Shareholders can amend (or repeal) board-approved bylaws Further shareholders can limit the boardrsquos power to later amend and repeal a shareholder-approved bylaw

Under the MBCA shareholders have the power to amend the bylaws See Point One The board shares this power with the shareholders unless (1) the corporationrsquos articles ldquoreserve that power exclusively to the shareholdersrdquo or (2) ldquothe shareholders in amending repealing or adopting a bylaw expressly provide that the board of directors may not amend repeal or reinstate that bylawrdquo See MBCA sect 1020(b)

Shareholder-approved bylaw provisions can amend or repeal existing bylaw provisions whether originally approved by the board or by shareholders See ALAN R PALMITER CORPORATIONS EXAMPLES AND EXPLANATIONS sect 713 (7th ed 2012) Thus the Mega boardrsquos bylaw amendmentmdashwhich set more demanding thresholds for shareholder nomination of directors than the investorrsquos proposed bylaw provisionmdashwould be superseded (repealed) if Megarsquos shareholders were to approve the investorrsquos proposal

Further a shareholder-approved bylaw generally can limit the power of the board to later amend or repeal it See MBCA sect 1020(b)(2) Thus if Megarsquos shareholders approved the bylaw

33

Corporations Analysis

provision proposed by the investor Megarsquos board could not repeal the provision because it includes a ldquono board repealrdquo clause

The revision to the MBCA in 2009 dealing with shareholder proxy access does not change this conclusion That revision specifies that a shareholder-approved bylaw dealing with director nominations may not limit the boardrsquos power to amend add or repeal ldquoany procedure or condition to such a bylaw in order to provide for a reasonable practicable and orderly processrdquo MBCA sect 206(d) Thus according to the revision if shareholders approve a bylaw amendment that limits further board changes the board would nonetheless retain the power to ldquotinkerrdquo with the bylaw to safeguard the voting process but could not repeal the shareholder-approved bylaw The Official Comment to MBCA sect 206(d) makes clear that the revision is ldquonot intended to allow the board of directors to frustrate the purpose of the shareholder-adopted proxy access provisionrdquo Thus if Megarsquos shareholders were to approve the bylaw provision proposed by the investor Megarsquos board could only amend the provision regarding its procedures or conditions in a manner consistent with its purpose of permitting proxy access for Megarsquos shareholders

[NOTE The boardrsquos attempted interference with a shareholder voting initiative may also have been a violation of the directorsrsquo fiduciary duties See Blasius Indus Inc v Atlas Corp 564 A2d 651 (Del Ch 1988) (finding that directors breached their fiduciary duties by amending bylaws and expanding size of board to thwart insurgentrsquos plan to amend bylaws and seat a majority of new directors) The call however asks examinees to consider whether shareholders or the board have ldquoprecedencerdquo over amending the corporate bylaws Thus an examineersquos answer should be framed in terms of ldquopowerrdquo and not ldquodutyrdquo]

Point Three (40) The investor need not make a demand on the board if the investor states a direct claim such as an allegation that the board interfered with the investorrsquos right to amend the bylaws But the investor must make a demand on the board if the investor states a derivative claim (on behalf of the corporation) such as an allegation that the directors sought to entrench themselves by interfering with the proposed proxy access

The MBCA generally requires that shareholders make a demand on the board of directors before initiation of a derivative suit MBCA sect 742 (shareholder may not bring derivative proceeding until written demand has been made on corporation and 90 days have expired) A derivative suit is essentially two suits in one where the plaintiff-shareholder seeks to bring on behalf of the corporation a claim that vindicates corporate rights usually based on violation of fiduciary duties PALMITER supra sect 1811 (6th ed 2009) The demand permits the board to investigate the situation identified by the shareholder and take suitable action No demand on the board is required however if the shareholder brings a direct suit to vindicate the shareholderrsquos own rights not those of the corporation

Is the suit brought by the investor derivative or direct The MBCA defines a ldquoderivative proceedingrdquo as one brought ldquoin the right of a domestic corporationrdquo MBCA sect 740(1) Thus the answer to how the investorrsquos suit should be characterized turns on what rights the investor seeks to vindicate If the investor frames its claim as one of fiduciary breach by directorsmdashfor example for failing to become adequately informed about voting procedures or for seeking to entrench themselves in office by manipulating the voting structure to avoid a shareholder insurgencymdashthen the suit is ldquoderivativerdquo and the investor must make a demand on the board See MBCA Ch 7 Subch D Introductory Comment (ldquothe derivative suit has historically been the principal method of challenging allegedly illegal action by managementrdquo)

34

Corporations Analysis

If however the investor frames its claim as one to vindicate shareholder rights the suit is direct and no demand is required For many courts the direct-derivative question turns on who is injured and who is to receive the relief sought by the plaintiff-shareholders See Tooley v Donaldson Lufkin amp Jenrette Inc 845 A2d 1031 (Del 2004) (characterizing a merger-delay claim as direct because delay of merger only harmed shareholders not corporation) Thus if the investor claims that managementrsquos refusal to include its proposed bylaw amendment in the corporationrsquos proxy materials violates its shareholder rights to initiate corporate governance reforms the suit will be direct Courts have not questioned the ability of shareholders to bring direct suits challenging board action to exclude their proposed bylaw amendments from the corporationrsquos proxy materials See JANA Master Fund Ltd v CNET Networks Inc 954 A2d 335 (Del Ch 2008) (upholding shareholderrsquos direct challenge to boardrsquos interpretation of advance-notice bylaw) Chesapeake Corp v Shore 771 A2d 293 (Del Ch 2000) (upholding shareholderrsquos direct challenge to actions by board that effectively prevented it from proposing bylaw amendments in contest for control)

Is the way that the investor frames its claim conclusive Courts have permitted shareholder-plaintiffs to challenge a transaction in a direct suit even though the same transaction could also be challenged as a fiduciary breach See Eisenberg v Flying Tiger Line Inc 451 F2d 267 (2d Cir 1971) (permitting direct suit challenging a corporate reorganization as a dilution of shareholder voting power even though reorganization may have involved conflicts of interest and thus constituted a fiduciary breach) Thus the investorrsquos choice to pursue a claim challenging the legality of managementrsquos decision to exclude the investorrsquos proposal from the corporationrsquos proxy materialsmdashrather than a possible breach of fiduciary dutymdashis likely to be respected See 3 COX amp HAZEN supra sect 153 (describing situations in which a claim can be framed as derivative or direct)

[NOTE Some issues under Delaware corporate law regarding pre-suit demand are not relevant here For example whether the Mega directors are independent and disinterested is not relevant to the MBCA requirement of a pre-suit demand As the Official Comment to MBCA sect 742 points out the MBCArsquos requirement of ldquouniversal demandrdquo gives the board ldquothe opportunity to reexamine the act complained of in the light of a potential lawsuit and take corrective actionrdquo even when the directors might be non-independent or have conflicts of interest

Nor is it relevant to the MBCA pre-suit demand requirement that the statutory 90-day waiting period may be onerous The first paragraph of MBCA sect 742 requires a pre-suit demand without exception the second paragraph of the section imposes a 90-day waiting period before a derivative suit may be brought which can be shortened if the board rejects the demand or ldquoirreparable injury to the corporation would result by waiting for the expiration of the 90-day periodrdquo The call as written asks only whether a pre-suit demand should be made and does not ask examinees to address whether the post-demand waiting period should be shortened under the ldquoirreparable injuryrdquo standard]

35

National Conference of Bar Examiners 302 South Bedford Street | Madison WI 53703-3622 Phone 608-280-8550 | Fax 608-280-8552 | TDD 608-661-1275

wwwncbexorg e-mail contactncbexorg

  • Preface
  • Description of the MEE
  • Instructions
  • July 2014 Questions
    • CRIMINAL LAW AND PROCEDURE QUESTION
    • CONTRACTS QUESTION
    • FAMILY LAW QUESTION
    • FEDERAL CIVIL PROCEDURE QUESTION
    • EVIDENCE QUESTION
    • CORPORATIONS QUESTION
      • July 2014 Analyses
        • CRIMINAL LAW AND PROCEDURE ANALYSIS
        • CONTRACTS ANALYSIS
        • FAMILY LAW ANALYSIS
        • FEDERAL CIVIL PROCEDURE ANALYSIS
        • EVIDENCE ANALYSIS
        • CORPORATIONS ANALYSIS
            • ltlt13 ASCII85EncodePages false13 AllowTransparency false13 AutoPositionEPSFiles true13 AutoRotatePages None13 Binding Left13 CalGrayProfile (Dot Gain 20)13 CalRGBProfile (sRGB IEC61966-21)13 CalCMYKProfile (US Web Coated 050SWOP051 v2)13 sRGBProfile (sRGB IEC61966-21)13 CannotEmbedFontPolicy Error13 CompatibilityLevel 1413 CompressObjects Tags13 CompressPages true13 ConvertImagesToIndexed true13 PassThroughJPEGImages true13 CreateJobTicket false13 DefaultRenderingIntent Default13 DetectBlends true13 DetectCurves 0000013 ColorConversionStrategy CMYK13 DoThumbnails false13 EmbedAllFonts true13 EmbedOpenType false13 ParseICCProfilesInComments true13 EmbedJobOptions true13 DSCReportingLevel 013 EmitDSCWarnings false13 EndPage -113 ImageMemory 104857613 LockDistillerParams false13 MaxSubsetPct 10013 Optimize true13 OPM 113 ParseDSCComments true13 ParseDSCCommentsForDocInfo true13 PreserveCopyPage true13 PreserveDICMYKValues true13 PreserveEPSInfo true13 PreserveFlatness true13 PreserveHalftoneInfo false13 PreserveOPIComments true13 PreserveOverprintSettings true13 StartPage 113 SubsetFonts true13 TransferFunctionInfo Apply13 UCRandBGInfo Preserve13 UsePrologue false13 ColorSettingsFile ()13 AlwaysEmbed [ true13 ]13 NeverEmbed [ true13 ]13 AntiAliasColorImages false13 CropColorImages true13 ColorImageMinResolution 30013 ColorImageMinResolutionPolicy OK13 DownsampleColorImages true13 ColorImageDownsampleType Bicubic13 ColorImageResolution 30013 ColorImageDepth -113 ColorImageMinDownsampleDepth 113 ColorImageDownsampleThreshold 15000013 EncodeColorImages true13 ColorImageFilter DCTEncode13 AutoFilterColorImages true13 ColorImageAutoFilterStrategy JPEG13 ColorACSImageDict ltlt13 QFactor 01513 HSamples [1 1 1 1] VSamples [1 1 1 1]13 gtgt13 ColorImageDict ltlt13 QFactor 01513 HSamples [1 1 1 1] VSamples [1 1 1 1]13 gtgt13 JPEG2000ColorACSImageDict ltlt13 TileWidth 25613 TileHeight 25613 Quality 3013 gtgt13 JPEG2000ColorImageDict ltlt13 TileWidth 25613 TileHeight 25613 Quality 3013 gtgt13 AntiAliasGrayImages false13 CropGrayImages true13 GrayImageMinResolution 30013 GrayImageMinResolutionPolicy OK13 DownsampleGrayImages true13 GrayImageDownsampleType Bicubic13 GrayImageResolution 30013 GrayImageDepth -113 GrayImageMinDownsampleDepth 213 GrayImageDownsampleThreshold 15000013 EncodeGrayImages true13 GrayImageFilter DCTEncode13 AutoFilterGrayImages true13 GrayImageAutoFilterStrategy JPEG13 GrayACSImageDict ltlt13 QFactor 01513 HSamples [1 1 1 1] VSamples [1 1 1 1]13 gtgt13 GrayImageDict ltlt13 QFactor 01513 HSamples [1 1 1 1] VSamples [1 1 1 1]13 gtgt13 JPEG2000GrayACSImageDict ltlt13 TileWidth 25613 TileHeight 25613 Quality 3013 gtgt13 JPEG2000GrayImageDict ltlt13 TileWidth 25613 TileHeight 25613 Quality 3013 gtgt13 AntiAliasMonoImages false13 CropMonoImages true13 MonoImageMinResolution 120013 MonoImageMinResolutionPolicy OK13 DownsampleMonoImages true13 MonoImageDownsampleType Bicubic13 MonoImageResolution 120013 MonoImageDepth -113 MonoImageDownsampleThreshold 15000013 EncodeMonoImages true13 MonoImageFilter CCITTFaxEncode13 MonoImageDict ltlt13 K -113 gtgt13 AllowPSXObjects false13 CheckCompliance [13 None13 ]13 PDFX1aCheck false13 PDFX3Check false13 PDFXCompliantPDFOnly false13 PDFXNoTrimBoxError true13 PDFXTrimBoxToMediaBoxOffset [13 00000013 00000013 00000013 00000013 ]13 PDFXSetBleedBoxToMediaBox true13 PDFXBleedBoxToTrimBoxOffset [13 00000013 00000013 00000013 00000013 ]13 PDFXOutputIntentProfile ()13 PDFXOutputConditionIdentifier ()13 PDFXOutputCondition ()13 PDFXRegistryName ()13 PDFXTrapped False1313 CreateJDFFile false13 Description ltlt13 ARA 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 BGR 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 CHS ltFEFF4f7f75288fd94e9b8bbe5b9a521b5efa7684002000410064006f006200650020005000440046002065876863900275284e8e9ad88d2891cf76845370524d53705237300260a853ef4ee54f7f75280020004100630072006f0062006100740020548c002000410064006f00620065002000520065006100640065007200200035002e003000204ee553ca66f49ad87248672c676562535f00521b5efa768400200050004400460020658768633002gt13 CHT ltFEFF4f7f752890194e9b8a2d7f6e5efa7acb7684002000410064006f006200650020005000440046002065874ef69069752865bc9ad854c18cea76845370524d5370523786557406300260a853ef4ee54f7f75280020004100630072006f0062006100740020548c002000410064006f00620065002000520065006100640065007200200035002e003000204ee553ca66f49ad87248672c4f86958b555f5df25efa7acb76840020005000440046002065874ef63002gt13 CZE 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 DAN ltFEFF004200720075006700200069006e0064007300740069006c006c0069006e006700650072006e0065002000740069006c0020006100740020006f007000720065007400740065002000410064006f006200650020005000440046002d0064006f006b0075006d0065006e007400650072002c0020006400650072002000620065006400730074002000650067006e006500720020007300690067002000740069006c002000700072006500700072006500730073002d007500640073006b007200690076006e0069006e00670020006100660020006800f8006a0020006b00760061006c0069007400650074002e0020004400650020006f007000720065007400740065006400650020005000440046002d0064006f006b0075006d0065006e0074006500720020006b0061006e002000e50062006e00650073002000690020004100630072006f00620061007400200065006c006c006500720020004100630072006f006200610074002000520065006100640065007200200035002e00300020006f00670020006e0079006500720065002egt13 DEU 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 ESP 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 ETI 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 FRA 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 GRE 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 HEB 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 HRV (Za stvaranje Adobe PDF dokumenata najpogodnijih za visokokvalitetni ispis prije tiskanja koristite ove postavke Stvoreni PDF dokumenti mogu se otvoriti Acrobat i Adobe Reader 50 i kasnijim verzijama)13 HUN 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 ITA 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 JPN ltFEFF9ad854c18cea306a30d730ea30d730ec30b951fa529b7528002000410064006f0062006500200050004400460020658766f8306e4f5c6210306b4f7f75283057307e305930023053306e8a2d5b9a30674f5c62103055308c305f0020005000440046002030d530a130a430eb306f3001004100630072006f0062006100740020304a30883073002000410064006f00620065002000520065006100640065007200200035002e003000204ee5964d3067958b304f30533068304c3067304d307e305930023053306e8a2d5b9a306b306f30d530a930f330c8306e57cb30818fbc307f304c5fc59808306730593002gt13 KOR ltFEFFc7740020c124c815c7440020c0acc6a9d558c5ec0020ace0d488c9c80020c2dcd5d80020c778c1c4c5d00020ac00c7a50020c801d569d55c002000410064006f0062006500200050004400460020bb38c11cb97c0020c791c131d569b2c8b2e4002e0020c774b807ac8c0020c791c131b41c00200050004400460020bb38c11cb2940020004100630072006f0062006100740020bc0f002000410064006f00620065002000520065006100640065007200200035002e00300020c774c0c1c5d0c11c0020c5f40020c2180020c788c2b5b2c8b2e4002egt13 LTH 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 LVI 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 NLD (Gebruik deze instellingen om Adobe PDF-documenten te maken die zijn geoptimaliseerd voor prepress-afdrukken van hoge kwaliteit De gemaakte PDF-documenten kunnen worden geopend met Acrobat en Adobe Reader 50 en hoger)13 NOR 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 POL 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 PTB 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 RUM ltFEFF005500740069006c0069007a00610163006900200061006300650073007400650020007300650074010300720069002000700065006e007400720075002000610020006300720065006100200064006f00630075006d0065006e00740065002000410064006f006200650020005000440046002000610064006500630076006100740065002000700065006e0074007200750020007400690070010300720069007200650061002000700072006500700072006500730073002000640065002000630061006c006900740061007400650020007300750070006500720069006f006100720103002e002000200044006f00630075006d0065006e00740065006c00650020005000440046002000630072006500610074006500200070006f00740020006600690020006400650073006300680069007300650020006300750020004100630072006f006200610074002c002000410064006f00620065002000520065006100640065007200200035002e00300020015f00690020007600650072007300690075006e0069006c006500200075006c0074006500720069006f006100720065002egt13 RUS 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 SKY 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 SLV ltFEFF005400650020006e006100730074006100760069007400760065002000750070006f0072006100620069007400650020007a00610020007500730074007600610072006a0061006e006a006500200064006f006b0075006d0065006e0074006f0076002000410064006f006200650020005000440046002c0020006b006900200073006f0020006e0061006a007000720069006d00650072006e0065006a016100690020007a00610020006b0061006b006f0076006f00730074006e006f0020007400690073006b0061006e006a00650020007300200070007200690070007200610076006f0020006e00610020007400690073006b002e00200020005500730074007600610072006a0065006e006500200064006f006b0075006d0065006e0074006500200050004400460020006a00650020006d006f0067006f010d00650020006f0064007000720065007400690020007a0020004100630072006f00620061007400200069006e002000410064006f00620065002000520065006100640065007200200035002e003000200069006e0020006e006f00760065006a01610069006d002egt13 SUO 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 SVE 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 TUR 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 UKR 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 ENU (Use these settings to create Adobe PDF documents best suited for high-quality prepress printing Created PDF documents can be opened with Acrobat and Adobe Reader 50 and later)13 gtgt13 Namespace [13 (Adobe)13 (Common)13 (10)13 ]13 OtherNamespaces [13 ltlt13 AsReaderSpreads false13 CropImagesToFrames true13 ErrorControl WarnAndContinue13 FlattenerIgnoreSpreadOverrides false13 IncludeGuidesGrids false13 IncludeNonPrinting false13 IncludeSlug false13 Namespace [13 (Adobe)13 (InDesign)13 (40)13 ]13 OmitPlacedBitmaps false13 OmitPlacedEPS false13 OmitPlacedPDF false13 SimulateOverprint Legacy13 gtgt13 ltlt13 AddBleedMarks false13 AddColorBars false13 AddCropMarks false13 AddPageInfo false13 AddRegMarks false13 ConvertColors ConvertToCMYK13 DestinationProfileName ()13 DestinationProfileSelector DocumentCMYK13 Downsample16BitImages true13 FlattenerPreset ltlt13 PresetSelector MediumResolution13 gtgt13 FormElements false13 GenerateStructure false13 IncludeBookmarks false13 IncludeHyperlinks false13 IncludeInteractive false13 IncludeLayers false13 IncludeProfiles false13 MultimediaHandling UseObjectSettings13 Namespace [13 (Adobe)13 (CreativeSuite)13 (20)13 ]13 PDFXOutputIntentProfileSelector DocumentCMYK13 PreserveEditing true13 UntaggedCMYKHandling LeaveUntagged13 UntaggedRGBHandling UseDocumentProfile13 UseDocumentBleed false13 gtgt13 ]13gtgt setdistillerparams13ltlt13 HWResolution [2400 2400]13 PageSize [612000 792000]13gtgt setpagedevice13

Page 10: July 2014 MEE Questions and Analyses - NCBE...This publication includes the questions and analyses from the July 2014 MEE. (In the actual test, the questions are simply numbered rather

FAMILY LAW QUESTION

In 1994 a man and a woman were married in State A

In 1998 their daughter was born in State A

In 2010 the family moved to State B

In 2012 the husband and wife divorced in State B Under the terms of the divorce decree

(a) the husband and wife share legal and physical custody of their daughter (b) the husband must pay the wife $1000 per month in child support until their daughter reaches age 18 (c) the marital residence was awarded to the wife with the proviso that if it is sold before the daughter reaches age 18 the husband will receive 25 of the net sale proceeds remaining after satisfaction of the mortgage on the residence and (d) the remaining marital assets were divided between the husband and the wife equally

Six months ago the husband was offered a job in State A that pays significantly less than his job in State B but provides him with more responsibilities and much better promotion opportunities The husband accepted the job in State A and moved from State B back to State A

Since returning to State A the husband has not paid child support because due to his lower salary he has had insufficient funds to meet all his obligations

One month ago the wife sold the marital home netting $10000 after paying off the mortgage She then moved to a smaller residence The husband believes that he should receive more than 25 of the net sale proceeds given his financial difficulties

Last week when the wife brought the daughter to the husbandrsquos State A home for a weekend visit the husband served the wife with a summons in a State A action to modify the support and marital-residence-sale-proceeds provisions of the State B divorce decree The husband brought the action in the State A court that adjudicates all domestic relations issues

1 Does the State A court have jurisdiction to modify (a) the child support provision of the State B divorce decree Explain (b) the marital-residence-sale-proceeds provision of the State B divorce decree

Explain

2 On the merits could the husband obtain (a) retroactive modification of his child support obligation to the daughter Explain (b) prospective modification of his child support obligation to the daughter Explain (c) modification of the marital-residence-sale-proceeds provision of the State B

divorce decree Explain

6

FEDERAL CIVIL PROCEDURE QUESTION

The United States Forest Service (USFS) manages public lands in national forests including the Scenic National Forest Without conducting an environmental evaluation or preparing an environmental impact statement the USFS approved a development project in the Scenic National Forest that required the clearing of 5000 acres of old-growth forest The trees in the forest are hundreds of years old and the forest is home to a higher concentration of wildlife than can be found anywhere else in the western United States

The USFS solicited bids from logging companies to harvest the trees on the 5000 acres of forest targeted for clearing and it ultimately awarded the logging contract to the company that had submitted the highest bid for the trees However the USFS has not yet issued the company a logging permit Once it does so the company intends to begin cutting down trees immediately

A nonprofit organization whose mission is the preservation of natural resources has filed suit in federal district court against the USFS The nonprofit alleges that the USFS violated the National Environmental Policy Act (NEPA) by failing to prepare an environmental impact statement for the proposed logging project Among other remedies the nonprofit seeks a permanent injunction barring the USFS from issuing a logging permit to the logging company until an adequate environmental impact statement is completed The nonprofit believes that the logging project would destroy important wildlife habitat and thereby cause serious harm to wildlife in the Scenic National Forest including some endangered species

Assume that federal subject-matter jurisdiction is available that the nonprofit has standing to bring this action and that venue is proper

1 If the logging company seeks to join the litigation as a party must the federal district court allow it to do so as a matter of right Explain

2 What types of relief could the nonprofit seek to stop the USFS from issuing a logging permit during the pendency of the action what must the nonprofit demonstrate to obtain that relief and is the federal district court likely to grant that relief Explain

7

EVIDENCE QUESTION

A prison inmate has filed a civil rights lawsuit against a guard at the prison alleging that the guard violated the inmatersquos constitutional rights during an altercation The inmate and the guard are the only witnesses to this altercation They have provided contradictory reports about what occurred

The trial will be before a jury The inmate plans to testify at trial The guardrsquos counsel has moved for leave to impeach the inmate with the following

(a) Twelve years ago the inmate was convicted of felony distribution of marijuana He served a three-year prison sentence which began immediately after he was convicted He served his full sentence and was released from prison nine years ago (b) Eight years ago the inmate pleaded guilty to perjury a misdemeanor punishable by up to one year in jail He paid a $5000 fine (c) Seven years ago the inmate was convicted of felony sexual assault of a child and is currently serving a 10-year prison sentence for the crime The victim was the inmatersquos daughter who was 13 years old at the time of the assault

The inmatersquos counsel objects to the admission of any evidence related to these three convictions and to any cross-examination based on this evidence

The guard also plans to testify at trial The inmatersquos counsel has moved for leave to impeach the guard with the following

Last year the guard applied for a promotion to prison supervisor The guard submitted a reacutesumeacute to the state that indicated that he had been awarded a BA in Criminal Justice from a local college An official copy of the guardrsquos academic transcript from that college indicates that the guard dropped out after his first semester and did not receive a degree

The guardrsquos counsel objects to the admission of this evidence and to any cross-examination based on this evidence

The transcript and the reacutesumeacute have been properly authenticated The trial will be held in a jurisdiction that has adopted all of the Federal Rules of Evidence

1 What evidence if any proffered by the guard to impeach the inmate should be admitted Explain

2 What evidence if any proffered by the inmate to impeach the guard should be admitted Explain

8

CORPORATIONS QUESTION

Mega Inc is a publicly traded corporation incorporated in a state whose corporate statute is modeled on the Model Business Corporation Act (MBCA) Megarsquos articles of incorporation do not address the election of directors or amendment of the bylaws by shareholders

Well within the deadline for the submission of shareholder proposals for the upcoming annual shareholdersrsquo meeting an investor who was a large and long-standing shareholder of Mega submitted a proposed amendment to Megarsquos bylaws The proposal which the investor asked to be included in the corporationrsquos proxy materials and voted on at the upcoming shareholdersrsquo meeting read as follows

Section 20 The Corporation shall include in its proxy materials (including the proxy ballot) for a shareholdersrsquo meeting at which directors are to be elected the name of a person nominated for election to the Board of Directors by a shareholder or group of shareholders that beneficially have owned 3 or more of the Corporationrsquos outstanding common stock for at least one year

This Section shall supersede any inconsistent provision in these Bylaws and may not be amended or repealed by the Board of Directors without shareholder approval

Megarsquos management decided to exclude the investorrsquos proposal from the corporationrsquos proxy materials and explained its reasons in a letter to the investor

The investorrsquos proposed bylaw provision would be inconsistent with relevant state law because the Board of Directors has the authority to manage the business and affairs of the Corporation Generally shareholders lack the authority to interfere with corporate management by seeking to create a method for the nomination and election of directors inconsistent with the method chosen by the Board of Directors

Furthermore at its most recent meeting the Board of Directors unanimously approved an amendment to the Corporationrsquos bylaws that provides for proxy access for director nominations by a shareholder or a group of shareholders holding at least 10 of the Corporationrsquos voting shares for at least three years This procedure takes precedence over any nomination methods that might be sought or approved by shareholders

The investor is considering bringing a suit challenging managementrsquos refusal to include the investorrsquos proposed bylaw provision and challenging the boardrsquos amendment of the bylaws at its recent meeting

1 Is the investorrsquos proposed bylaw provision inconsistent with state law Explain

2 If the investorrsquos proposed bylaw provision were approved by the shareholders would the bylaw amendment previously approved by the board take precedence over the investorrsquos proposed bylaw provision Explain

3 Must the investor make a demand on Megarsquos board of directors before bringing suit Explain

9

July 2014 MEE

ANALYSES

Contracts Family Law

Criminal Law and Procedure

Federal Civil Procedure Evidence

Corporations

CRIMINAL LAW AND PROCEDURE ANALYSIS (Criminal Law and Procedure VA B D)

ANALYSIS

Legal Problems

(1) Did the detective violate the suspectrsquos Sixth Amendment right to counsel when he questioned the suspect about the burglaries without the lawyer present given that the lawyer represented the suspect in an unrelated criminal matter

(2) Under Miranda did the suspect effectively invoke his right to counsel when he said ldquoI think I want my lawyer here before I talk to yourdquo

(3) Was the suspectrsquos waiver of his right to remain silent under Miranda valid

DISCUSSION

Summary

The Sixth Amendment right to counsel as applied to states through the Fourteenth Amendment is offense-specific Although the suspect had an attorney representing him on his pending assault charge he had no Sixth Amendment right to the assistance of counsel with respect to the five uncharged burglaries because formal adversarial proceedings had not yet commenced on those charges The suspectrsquos Sixth Amendment right to counsel was not violated by the detectiversquos failure to inform him that the lawyer was present or of the lawyerrsquos demands

However a person undergoing custodial interrogation also has an independent constitutional right to counsel during custodial interrogation under Miranda When a suspect invokes his right to counsel under Miranda custodial interrogation must immediately cease for a period of at least 14 days However the invocation of the right to counsel must be unambiguous and clearly convey that the suspect has requested counsel Here because the suspectrsquos statement ldquoI think I want my lawyer here before I talk to yourdquo was ambiguous he did not invoke his Miranda right to counsel

A waiver of rights must be knowing intelligent and voluntary Here the suspect waived his right to remain silent under Miranda when he signed the waiver form The fact that the detective did not correct the suspectrsquos assumption that the lawyer would need to drive to the jailmdashby telling him that the lawyer was in the waiting room and was demanding to see himmdashdid not affect the validity of the suspectrsquos waiver

Point One (35) The suspectrsquos Sixth Amendment right to counsel was not violated because the right does not attach on new charges until formal adversarial judicial proceedings have commenced on those charges

The Sixth Amendment as applied to the states through the Fourteenth Amendment provides that ldquo[i]n all criminal prosecutions the accused shall enjoy the right to have the Assistance of Counsel for his defenserdquo The right to counsel does not attach with respect to particular charges until formal adversarial judicial proceedings have commenced (ie ldquoat or after the initiation of

13

Criminal Law and Procedure Analysis

adversary judicial criminal proceedingsmdashwhether by way of formal charge preliminary hearing indictment information or arraignment [or in some states arrest warrant]rdquo McNeil v Wisconsin 501 US 171 175 (1991) (internal quotations omitted)) Once a suspectrsquos Sixth Amendment right to counsel has attached any attempts to ldquodeliberately elicitrdquo statements from him in the absence of his attorney violate the Sixth Amendment See Massiah v United States 377 US 201 (1964) Brewer v Williams 430 US 387 (1977)

The Sixth Amendment right to counsel is charge- or offense -specific Representation by counsel in one prosecution does not in itself guarantee counsel for uncharged offenses See McNeil 501 US at 175 Texas v Cobb 532 US 162 (2001) Here the suspectrsquos Sixth Amendment right to counsel had attached only for the pending aggravated assault charge The suspectrsquos right to counsel for the aggravated assault case did not guarantee counsel for the five unrelated and uncharged burglaries that were the subject of the detectiversquos interrogation Thus because formal adversarial judicial proceedings against the suspect for the uncharged burglaries had not begun he had no Sixth Amendment right to counsel

Finally the detectiversquos failure to inform the suspect of the lawyerrsquos presence and demands to speak with him does not implicate the suspectrsquos Sixth Amendment right to counsel which had not yet attached See id Moran v Burbine 475 US 412 428ndash31 (1986)

Point Two (30) The suspect did not effectively invoke his right to counsel under Miranda because his statement was not unambiguous

A suspect subject to custodial interrogation has a right to consult with counsel and to have an attorney present during questioning Miranda v Arizona 384 US 436 (1966) When a suspect invokes his right to counsel during an interrogation law enforcement must immediately cease all questioning See Edwards v Arizona 451 US 477 484ndash85 (1981) Custodial interrogation cannot be reinitiated unless and until the suspect has been re-advised of his Miranda rights has provided a knowing and voluntary waiver and (1) counsel is present and (2) the suspect himself initiated further communication with the police see id at 484 or (3) (if the suspect was released from custody after the initial interrogation) at least 14 days have passed Maryland v Shatzer 559 US 98 110 (2010)

To invoke the right to counsel a suspectrsquos request must be ldquounambiguousrdquo This means that the suspect must articulate the desire for counsel sufficiently clearly that a reasonable officer would understand the statement to be a request for counsel Davis v United States 512 US 452 459 (1994) If the request is ambiguous the police are not required to stop the interrogation

In this case the suspectrsquos statement ldquoI think I want my lawyer here before I talk to yourdquo was not an unambiguous request for counsel The most reasonable interpretation of this statement is that the suspect might be invoking his right to counsel Id at 461 (ldquomaybe I should talk to a lawyerrdquo is not an unequivocal request for counsel) See also Burket v Angelone 208 F3d 172 197ndash98 (4th Cir 2000) (ldquoI think I need a lawyerrdquo is not an unambiguous request for an attorney) Soffar v Cockrell 300 F3d 588 594ndash95 (5th Cir 2002) (discussion of various statements that did not constitute unequivocal requests for counsel)

Under these circumstances the detective was not required to cease the custodial interrogation of the suspect Nor was the detective required to clarify or ask follow-up questions to determine whether the suspect in fact wanted an attorney Davis 512 US at 459ndash60

14

Criminal Law and Procedure Analysis

Point Three (35) The suspectrsquos waiver of his Miranda rights was knowing intelligent and voluntary despite the fact that he was never told of the lawyerrsquos presence in the jail or of the lawyerrsquos demands

A valid waiver of Miranda rights must be ldquovoluntaryrdquomdashie the product of a free or deliberate choice rather than intimidation coercion or deception Berghuis v Thompkins 560 US 370 382ndash83 (2010) In addition the waiver must be knowing and intelligent That is it ldquomust have been made with a full awareness of both the nature of the right being abandoned and the consequences of the decision to abandon itrdquo Moran v Burbine 475 US 421 (1986)

In this case the suspect signed a Miranda waiver form after receiving proper warnings There is no evidence ldquothat the police resorted to physical or psychological pressure to elicit the statementsrdquo Id The entire interview lasted only 45 minutes The only issue is whether the suspect knowingly and intelligently waived his Miranda rights despite the fact that the detective did not tell the suspect about the lawyerrsquos presence and her demands

The Supreme Court has said that ldquo[e]vents occurring outside of the presence of the suspect and entirely unknown to him surely can have no bearing on the capacity to comprehend and knowingly relinquish a constitutional rightrdquo Id at 422 If the suspect ldquoknew that he could stand mute and request a lawyer and was aware of the Statersquos intention to use his statements to secure a convictionrdquo then the waiver is valid regardless of the information withheld Id at 422ndash23

Here the suspect was correctly informed of his rights Miranda v Arizona 384 US at 467ndash73 His comments demonstrate that he understood that he could have a lawyer present if he desired (ie wondering whether he should call his attorney) and that he understood that there might be consequences to speaking with the detective (ldquoI probably should keep my mouth shut but Irsquom willing to talk to you for a whilerdquo) His comment ldquo[L]etrsquos not waste any time waiting for someone to call my attorney and having her drive hererdquo along with his signature on the Miranda waiver form show that his waiver was valid under the constitutional standard

The fact that the detective did not tell the suspect about the lawyerrsquos presence and demands has no bearing on the validity of the suspectrsquos waiver because ldquosuch conduct is only relevant to the constitutional validity of a waiver if it deprives a defendant of knowledge essential to his ability to understand the nature of his rights and the consequences of abandoning themrdquo Moran at 424 The Supreme Court has specifically declined to adopt a rule requiring that law enforcement tell a suspect of an attorneyrsquos efforts to contact him id at 425 (ldquoNor are we prepared to adopt a rule requiring that the police inform a suspect of an attorneyrsquos efforts to reach himrdquo)

[NOTE An examinee might also recognize that this general rule is further supported by the Supreme Courtrsquos decision in Florida v Powell 559 US 50 (2010) approving state Miranda warnings that do not explicitly warn suspects that they have a right to have counsel present during custodial interrogation]

15

CONTRACTS ANALYSIS (Contracts IB2 IIB IVA3 amp A5)

ANALYSIS

Legal Problems

(1) In the case of a service contract (governed by the common law of contracts) is a modification enforceable when a party agrees to pay more for the same performance than was originally promised

(2) In the case of a contract for the sale of goods (governed by Article 2 of the UCC) is a modification enforceable when a party agrees to pay more for the same goods than was originally promised

(3) May a party avoid an agreement on the basis of economic duress

DISCUSSION

Summary

There are two arguments that the conservatory can make to support the claim that it is not bound to pay the higher prices lack of consideration and economic duress

The organ repair contract is governed by the common law of contracts Under the common law the business would have difficulty recovering the additional $60000 for the organ repair because under the ldquopreexisting duty rulerdquo the agreement of the conservatory to pay the extra price was not supported by consideration However the business might argue that the modification is enforceable under an exception to the preexisting duty rule for fair and equitable modifications made in light of unanticipated circumstances

The organ sale contract is governed by Article 2 of the Uniform Commercial Code The business would likely recover the additional amount under that contract because Article 2 provides that consideration is not required for a modification to be binding

In both cases the conservatory could seek to avoid its agreement on the grounds of economic duress but that argument is not likely to succeed

Point One (45) The business probably cannot recover the additional $60000 for the organ repair because the conservatoryrsquos promise to pay more money was not supported by consideration

The general rule is that to be enforceable a promise must be supported by consideration Under RESTATEMENT (SECOND) OF CONTRACTS sect 71 a promise is supported by consideration if it is bargained for in exchange for a return promise or performance However under the ldquopreexisting duty rulerdquo (exemplified in RESTATEMENT (SECOND) OF CONTRACTS sect 73 and Alaska Packersrsquo Assrsquon v Domenico 117 F 99 (9th Cir 1902)) promise of performance of a legal duty already owed to a promisor which is neither doubtful nor the subject of honest dispute is not consideration

If the business had promised the conservatory anything new or different in exchange for the agreement to pay the additional $60000 (such as for example repairing the pipe organ more

16

Contracts Analysis

quickly or using better parts) that would constitute consideration especially in light of the principle that courts do not inquire into the adequacy of consideration Here however the business already had a legal duty under the original contract and did not agree to do anything else in exchange for the conservatoryrsquos promise to pay $60000 more

However an exception to the preexisting duty rule is sometimes applied in situations of unanticipated changed circumstances Under RESTATEMENT (SECOND) OF CONTRACTS sect 89 followed in many jurisdictions a promise modifying a duty under a contract not fully performed on either side is binding even if not supported by consideration if the modification is fair and equitable in view of circumstances not anticipated by the parties when the contract was made

If a court applies the rule in Restatement sect 89 the critical issues will be whether the modification was in fact ldquofair and equitablerdquo and whether it can be justified in light of unanticipated circumstances In many cases in which modifications have been upheld a party encountered difficulties or burdens in performing far beyond what was knowingly bargained for in the original contract with the result bordering on impracticability such as having to excavate solid rock instead of soft dirt or having to remove garbage far in excess of the amounts contemplated The conservatory would argue that the businessrsquos performance difficulties were not of this sort at allmdashnothing about repairing the pipe organ itself was any different from or more difficult than originally contemplated except that the business itself encountered financial distress unrelated to its burdens in performing its obligations under these contracts

Even if the business satisfies that element of the rule in Restatement sect 89 the business must also demonstrate that the circumstances that gave rise to the need to modify the contract were ldquounanticipatedrdquo at the time the original contract was made Here the facts suggest that when the business entered into the original contract it expected that the price paid by the conservatory would enable it to perform However any evidence that the business knew or had reason to know at the time of execution that it would need more money from the conservatory to be able to perform would mean that the request to modify was not ldquounanticipatedrdquo

[NOTE Some cases such as Schwartzreich v Bauman-Basch Inc 231 NY 196 131 NE 887 (1921) find that if the parties mutually agreed to rescind the original contract and then after rescission entered into an entirely new contract for a higher price the new contract is supported by consideration There is no evidence that such a rescission followed by a new contract took place here]

Point Two (45) The business can recover the additional $40000 for the new organ because no consideration is required under Article 2 of the UCC for good-faith contract modifications

The contract to buy a new organ is a contract for the sale of goods and therefore is governed by Article 2 of the Uniform Commercial Code UCC sect 2-102 Under Article 2 unlike the common law an agreement modifying a contract needs no consideration to be binding UCC sect 2-209(1) Section 2-209(1) thus obviates the preexisting duty rule entirely in contracts for the sale of goods

Even though consideration is not required modifications governed by sect 2-209 must satisfy the obligation of good faith imposed by the UCC UCC sect 1-304 See also Official Comment 2 to UCC sect 2-209 Good faith means ldquohonesty in fact and the observance of reasonable commercial standards of fair dealingrdquo UCC sect 1-201(b)(20) In this context the obligation of good faith means that ldquo[t]he effective use of bad faith to escape performance on the original contract terms is barred and the extortion of a lsquomodificationrsquo without legitimate commercial reason is ineffective as a violation of the duty of good faithrdquo Official Comment 2 to

17

Contracts Analysis

UCC sect 2-209 Here because the businessrsquos financial reversals were serious and apparently unanticipated at the time that the business entered into the contract with the conservatory and commitment of the extra money was needed to enable the business to perform a court would likely find that the business acted in good faith Thus a court would likely uphold the enforceability of the conservatoryrsquos promise to pay the additional $40000

Point Three (10) The conservatory is unlikely to be able to defend against enforcement of its promises to pay additional money under the theory of economic duress because the business probably did not make an improper threat

Under the common law of contracts parties may raise the defense of duress This common law defense also applies to contracts governed by UCC Article 2 See UCC sect 1-103(b)

A contract is voidable on the ground of economic duress by threat when it is established that a partyrsquos manifestation of assent is induced by an improper threat that leaves the party no reasonable alternative See RESTATEMENT (SECOND) OF CONTRACTS sect 175 See also eg Austin Instrument Inc v Loral Corp 272 NE2d 533 (NY 1971) (a threat to withhold essential goods can constitute duress) In order to void its agreement to pay the additional sum because of economic duress the conservatory must demonstrate that (1) the business made a threat to the conservatory (2) the threat was ldquoimproperrdquo or ldquowrongfulrdquo (3) the threat induced the conservatoryrsquos manifestation of assent to the modification and (4) the threat was sufficiently grave to justify the conservatoryrsquos assent

Here it appears that three of the four elements are likely satisfied The business plainly made a threat Moreover the threat induced the conservatoryrsquos assent to the modification and the threat was sufficiently grave to justify that assent If the conservatory had not agreed to pay the business the extra amounts the conservatory would have lost its entire $325000 investment In light of this potential loss a court could easily conclude that the conservatory had no reasonable alternative

However the business has a strong argument that its threat (indicating that it would breach the contracts unless the prices were increased) was not wrongful or improper but was instead nothing more than a communication of the reality of its own perilous situation to the conservatory

A mere threat to breach a contract is not in and of itself improper so as to support an action of economic duress or business compulsion Something more is required such as a breach of the duty of good faith and fair dealing as was present in Austin Instrument Inc supra Because the business could not perform the original contract without the requested modification the economic duress claim for the conservatory would likely fail for much the same reason that the business would be able to enforce the modification At the time the modification was requested the business was not trying to extort a price increase because of the conservatoryrsquos vulnerability but instead was simply stating the reality that the business could not perform without more money

18

FAMILY LAW ANALYSIS (Family Law IIIB D amp G)

ANALYSIS

Legal Problems

(1)(a) Does the State A court have jurisdiction to modify the State B child support order

(1)(b) Does the State A court have jurisdiction to modify the marital-residence-saleshyproceeds provision of the State B property-division decree

(2)(a) May a child support order be modified retroactively

(2)(b) May a child support order be modified prospectively based on a change of employment with a lower salary

(2)(c) May a property-division order be modified after entry of a divorce decree

DISCUSSION

Summary

The State A court may exercise personal jurisdiction over the wife because she was personally served in State A However subject-matter jurisdiction over the interstate modification of child support is governed by the Uniform Interstate Family Support Act (UIFSA) Under UIFSA State A does not have jurisdiction to modify the order for the daughterrsquos support because the wife is still a resident of State B UIFSA on the other hand does not govern property distributions and thus a State A court is not precluded from hearing the husbandrsquos petition to modify the marital-residence-sale-proceeds provision of the divorce decree

A child support order may not be modified retroactively A child support order may be modified prospectively based on a substantial change in circumstances Courts agree that a significant decrease in income is a substantial change in circumstances All states treat voluntary income reductions differently than involuntary reductions but employ different approaches for evaluating the impact of a voluntary reduction Whether the husband could obtain prospective modification of the child support order depends on which approach is applied

A property-division order is not subject to post-divorce modification based on a change in circumstances Thus the husband may in some states obtain prospective modification of the order for the daughterrsquos support but he may not obtain modification of the marital-residenceshysale-proceeds provision

Point One(a) (25) Personal jurisdiction over a nonresident respondent does not confer subject-matter jurisdiction over child support modification Under UIFSA a State A court may not modify a child support order issued by a State B court when as here the child or either parent continues to reside in State B the jurisdiction that issued the child support order

The State A court may exercise personal jurisdiction over the wife The wife was personally served in State A and a state may exercise jurisdiction based on in-state personal service See

19

Family Law Analysis

Burnham v Superior Court 495 US 604 (1990) But personal jurisdiction over the wife is not enough to give a State A court jurisdiction to modify the State B support order

The interstate enforcement and modification of child support is governed by the Uniform Interstate Family Support Act (UIFSA) which has been adopted by all states Under UIFSA the state that originally issued a child support order (here State B) has continuing exclusive jurisdiction to modify the order if that state remains the residence of the obligee the child or the obligor and all parties do not consent to the jurisdiction of another forum See UIFSA sect 205 See also UIFSA sect 603 (ldquoA tribunal of this State shall recognize and enforce but may not modify a registered order if the issuing tribunal had jurisdictionrdquo) The wife and daughter continue to reside in State B and the wife has not consented to the jurisdiction of another forum Thus a State A court does not have jurisdiction to modify the State B child support order

[NOTE Examinees who do not discuss personal jurisdiction but fully discuss UIFSA may receive full credit]

Point One(b) (15) UIFSA does not apply to disputes over property division Thus the State A court may exercise jurisdiction over the husbandrsquos petition to modify the marital-residence-sale-proceeds provision of the State B divorce decree because it has personal jurisdiction over the wife

The State A court in which the husband brought his action has jurisdiction to adjudicate domestic relations issues The husbandrsquos petition to modify the property settlement is a domestic relations issue The courts of State A may exercise personal jurisdiction over the wife because she was personally served in State A See Burnham v Superior Court 495 US 604 (1990) see Point One(a)

UIFSA does not apply to divorce property-division disputes Thus although a State A court may not adjudicate the husbandrsquos petition to modify his child support obligations it may adjudicate his property-division claims (Even though the court has jurisdiction it may not modify the property-division award on the merits See Point Two(c))

Point Two(a) (20) A child support order may not be modified retroactively

State courts have long held that obligations to pay child support ordinarily may not be modified retroactively ldquoIf the hardship is particularly severe the courts sometimes devised a way to protect the obligor but in most instances the courts hold that retroactive modification of this kind is beyond their power and indeed the governing statute may so providerdquo HOMER H CLARK THE LAW OF DOMESTIC RELATIONSHIPS IN THE UNITED STATES 725 (2d ed 1987)

Federal law now goes further and requires the states as a condition of federal child-support funding to adopt rules that absolutely forbid retroactive modification of the support obligation See 42 USC sect 666(a)(9)(C) The states have adopted rules consistent with the federal requirements

Point Two(b) (25) It is unclear whether the husband could obtain prospective downward modification of his child support based on his voluntary acceptance of a job with a lower salary

Prospective modification of a child support order is typically available only when the petitioner can show a substantial change in circumstances See ROBERT E OLIPHANT amp NANCY VER

20

Family Law Analysis

STEEGH FAMILY LAW 213ndash15 (3d ed 2010) A significant decrease in income is typically viewed as a substantial change

However when a parent seeks to modify a child support obligation because he has voluntarily reduced his income a court will not modify the obligation based solely on the income loss Some courts refuse to modify whenever the income shift was voluntary See eg Aguiar v Aguiar 127 P3d 234 (Idaho Ct App 2005) Others look primarily to the petitionerrsquos intentions and permit downward modification if he has acted in good faith See eg In re Marriage of Horn 650 NE2d 1103 (Ill App Ct 1995) Many courts use a multifactor approach See OLIPHANT amp VER STEEGH supra 217ndash18

Here there is no question that the husbandrsquos loss of income was voluntary In a jurisdiction in which voluntary income reduction bars support modification the husbandrsquos petition would be denied

In a jurisdiction employing a good-faith or multifactor approach it is possible but not certain that the husband could obtain downward modification The evidence supports the husbandrsquos good faith his change in employment appears to be based on his new jobrsquos greater responsibilities and better promotion possibilities In a jurisdiction using a multifactor approach the court would likely also consider the impact of such a shift on the daughter the likely duration of the husbandrsquos income loss and the likelihood of a promotion that would ultimately inure to the daughterrsquos benefit Thus on these facts it is possible but by no means certain that the husband could prospectively obtain downward modification of his child support obligation to his daughter

Point Two(c)(15) A divorce property-division award is not subject to modification

A support order is aimed at meeting the post-divorce needs of the supported individual Because the future is unpredictable courts are empowered to modify a support award to take account of changed circumstances that may occur during the period in which support is paid

By contrast a property-distribution award divides assets of the marriage based on the equities at the time of divorce Because the past can be ascertained a property-division award is not subject to post-divorce modification See HARRY A KRAUSE ET AL FAMILY LAW CASES COMMENTS AND QUESTIONS 691 (6th ed 2007)

Here the husband is seeking modification of a property-division award with respect to an asset owned by the parties at the time of divorce Thus the husband may not obtain a modification of the marital-residence-sale-proceeds provision of the divorce decree based on his reduced income

21

FEDERAL CIVIL PROCEDURE ANALYSIS (Federal Civil Procedure III IVC)

ANALYSIS

Legal Problems

(1) Is the logging company entitled to join this action as a matter of right

(2)(a) May the nonprofit organization obtain a temporary restraining order to stop the USFS from issuing a logging permit

(2)(b) May the nonprofit organization obtain a preliminary injunction to stop the USFS from issuing a logging permit during the pendency of the action

DISCUSSION

Summary

The logging company is entitled to intervene in this action as a matter of right because it has an interest in the property or transaction that is the subject of the action and is so situated that its interest may be impaired or impeded as a practical matter if the action goes forward without it The logging companyrsquos interest is not adequately represented by the USFSrsquos presence in the lawsuit

The nonprofit organization may seek a temporary restraining order (TRO) followed by a preliminary injunction to prevent the USFS from issuing a logging permit pending the outcome of the action The nonprofit is likely to obtain a TRO if it can demonstrate a risk of immediate and irreparable injury The nonprofit is also likely to obtain a preliminary injunction if it can demonstrate a significant threat of irreparable harm and a likelihood of success on the merits of its National Environmental Policy Act (NEPA) claim

Point One (50) Rule 24(a) of the Federal Rules of Civil Procedure requires federal courts to allow a person to intervene in an action as a matter of right if the person a) is interested in the property or transaction that is the subject of the action b) is so situated that its interest may be impaired or impeded if the litigation goes forward without it and c) is not adequately represented by existing parties Here the logging company likely meets all three requirements and should be allowed to intervene as a matter of right

Rule 24 of the Federal Rules of Civil Procedure governs intervention the process by which a non-party to an action may join the litigation Under Rule 24(a) (intervention of right) a person must be permitted to intervene if three conditions are met (1) the movant ldquoclaims an interest relating to the property or transaction that is the subject of the actionrdquo (2) the movant ldquois so situated that disposition of the action may as a practical matter impair or impede the movantrsquos ability to protect its interestrdquo and (3) ldquoexisting partiesrdquo do not ldquoadequately represent [the movantrsquos] interestrdquo FED R CIV P 24(a) The three requirements for intervention of right are often ldquovery interrelatedrdquo 7C CHARLES ALAN WRIGHT ET AL FEDERAL PRACTICE AND PROCEDURE sect 1908 at 297 (2007 amp 2011 Supp)

22

Federal Civil Procedure Analysis

Here the court should find that the logging company meets this test First the logging company has a strong interest in the property or transaction that is the subject of this action The USFS has accepted the logging companyrsquos bid and the logging company is merely awaiting issuance of a logging permit to begin logging The nonprofit organization is seeking to prevent this logging The logging company therefore has a strong direct and substantial interest in the subject matter of the lawsuit and in having its winning bid honored and a logging permit issued See eg Kleissler v US Forest Serv 157 F3d 964 972 (3d Cir 1998) (stating that ldquo[t]imber companies have direct and substantial interests in a lawsuit aimed at halting loggingrdquo) see also Natural Resources Defense Council v US Nuclear Regulatory Commrsquon 578 F2d 1341 1343ndash 44 (10th Cir 1978) (holding that applicants whose license renewals were pending had Rule 24(a)(2) interests where the lawsuit sought to halt the license-issuing process pending preparation of environmental impact statements) See generally 7C WRIGHT ET AL supra sect 19081 at 309 (ldquoIf there is a direct substantial legally protectable interest in the proceedings it is clear that this requirement of the rule is satisfiedrdquo) Second the logging companyrsquos interest in receiving a logging permit may well be impaired as a practical matter by the outcome of the lawsuit If the USFS loses the lawsuit it will have to prepare an environmental impact statement before issuing the logging companyrsquos permit This will at a minimum delay the logging companyrsquos ability to exercise its rights and may in the long r un mean that no logging permit is ever issued Intervention of right is not limited to those that would be legally bound as a matter of preclusion doctrine Id sect 19082 at 368 Rather ldquo[t]he rule is satisfied whenever disposition of the present action would put the movant at a practical disadvantage in protecting its interestrdquo Id sect 19082 at 369 Here that condition is easily satisfied See Kleissler 157 F3d at 972 (ldquoTimber companies have direct and substantial interests in a lawsuit aimed at halting logging rdquo)

Given that the logging company has an interest that may be impaired by disposition of the action it should be allowed to intervene unless the court is persuaded that the USFS adequately represents the logging companyrsquos interest See Rule 24(a)(2) 7C WRIGHT ET AL supra sect 1909 Here it could be argued that the USFS adequately represents the logging companyrsquos interest because the USFS presumably wants the court to uphold its development plan and allow it to proceed with issuance of the logging permit which is the same relief that the logging company would seek However whether representation is truly adequate depends upon ldquo[a] discriminating appraisal of the circumstancesrdquo 7C WRIGHT ET AL supra sect 1909 at 440 Although both the government and the logging company wish to avoid the preparation of an environmental impact statement their interests are distinct The USFSrsquos interest is proper management of the national forest system while the logging companyrsquos interest is making a profit from logging the 5000-acre tract The USFSrsquos handling of the litigation is likely to be affected by a variety of policy concerns and political considerations that have nothing to do with the logging companyrsquos purely economic interest in securing the right to cut trees in the Scenic National Forest See eg Kleissler 157 F3d at 973ndash74 (ldquo[T]he government represents numerous complex and conflicting interests in matters of this nature The straightforward business interests asserted by intervenors here may become lost in the thicket of sometimes inconsistent governmental policiesrdquo)

[NOTES (1) Examinees who mistakenly analyze the logging companyrsquos case for joinder under the related but incorrect Rule 19 ldquoRequired Joinder of Partiesrdquo may receive credit Rule 19 allows existing parties to demand joinder of non-parties (or seek dismissal of the case if they canrsquot get it) There is a close relationship between Rule 24 and Rule 19 and both contain a similar standard for determining when ldquointerestedrdquo third parties are ldquoentitledrdquo or ldquorequiredrdquo to be in the lawsuit Indeed the two prongs of the Rule 24 intervention test that are discussed above

23

Federal Civil Procedure Analysis

are nearly identical to the two prongs of the Rule 19(a) required joinder test Examinees who discuss and apply the test should receive credit even if they cite Rule 19 rather than Rule 24

(2) Examinees may discuss permissive joinder Although permissive joinder is a possibility here the question asks only whether the logging company can join the action as a matter of right and a permissive joinder analysis is not responsive to the question To the extent an examinee discusses permissive joinder the analysis will focus on whether the logging company ldquohas a claim or defense that shares with the main action a common question of law or factrdquo FED R CIV P 24(b)(1)(B) The district court also ldquomust consider whether the intervention will unduly delay or prejudice the adjudication of the original partiesrsquo rightsrdquo FED R CIV P 24(b)(3) On our facts the logging companyrsquos claim for the issuance of a logging permit would certainly share common questions of law and fact with the USFSrsquos defense against the nonprofitrsquos claim There are no facts suggesting that the logging companyrsquos presence would unduly delay or otherwise prejudice adjudication of the original action Thus the district court would have discretion to permit the logging company to intervene even if it denied intervention of right]

Point Two(a) (25) The nonprofit organization could seek and would likely obtain a temporary restraining order to stop the USFS from issuing a logging permit pending a hearing on an application for a preliminary injunction

The first type of interim relief the nonprofit could seek to stop the USFS from issuing a logging permit to the logging company is a temporary restraining order (TRO) prohibiting the USFS from issuing the logging permit A TRO can be issued without notice to the adverse party but only in limited circumstances and only for a limited time FED R CIV P 65(b) To secure a TRO without notice the nonprofit would need to submit an affidavit containing specific facts that demonstrate a risk of ldquoimmediate and irreparable injuryrdquo if a permit is issued FED R CIV P 65(b)(1) In deciding whether to grant a TRO courts will also consider the same factors that are relevant in deciding whether to grant a preliminary injunction (eg the moving partyrsquos likelihood of success on the merits the balance of hardships and the public interest) See Point Two(b) infra The TRO would last only long enough for the court to consider and resolve a request by the nonprofit for a preliminary injunction but no longer than 14 days (unless the court extends it for good cause or the adverse party consents to an extension) In addition bond is required

Here the court is likely to grant the nonprofitrsquos request The nonprofit could plausibly claim that cutting down 5000 acres of old-growth forest in an area that is home to the highest concentration of wildlife in the western United States would have ldquoan immediate and irreparablerdquo adverse impact on the environment and cause irreparable harm to the nonprofitrsquos interest in preserving and protecting natural resources including wildlife habitat

Point Two(b) (25) The nonprofit could also seek and would likely obtain a preliminary injunction to stop the USFS which is likely to be granted if the nonprofitrsquos claim that the USFS violated NEPA has a strong basis in fact and law

Because the TRO would be temporary the nonprofit would need to move for a preliminary injunction to prevent the USFS from issuing a logging permit throughout the pendency of the litigation Preliminary injunctions are injunctions that seek to ldquoprotect [the] plaintiff from

24

Federal Civil Procedure Analysis

irreparable injury and to preserve the courtrsquos power to render a meaningful decision after a trial on the meritsrdquo 11A CHARLES ALAN WRIGHT ET AL FEDERAL PRACTICE AND PROCEDURE sect 2947 at 112 (2013) Rule 65 of the Federal Rules of Civil Procedure sets out the procedural requirements for preliminary injunctions Preliminary injunctions may be granted only upon notice to the adverse party FED R CIV P 65(a)(1) and only if the movant ldquogives security in an amount that the court considers proper to pay the costs and damages sustained by any party found to have been wrongfully enjoined or restrainedrdquo FED R CIV P 65(c)

While Rule 65 sets out the procedural requirements for preliminary injunctive relief it does not specify the substantive grounds upon which it may be granted The courtrsquos discretion in ruling upon a motion for a preliminary injunction ldquois exercised in conformity with historic federal equity practicerdquo 11A WRIGHT ET AL supra sect 2947 at 114 The court typically considers four factors

(1) the significance of the threat of irreparable harm to the plaintiff if the injunction is not granted (2) the balance between this harm and the injury that granting the injunction would inflict on the defendant (3) the probability that the plaintiff will succeed on the merits and (4) the public interest

Id sect 2948 at 122ndash24 accord Habitat Educ Center v Bosworth 363 F Supp 2d 1070 1088 (ED Wis 2005) The most important of these factors is the risk of irreparable harm to the plaintiff 11A WRIGHT ET AL supra sect 29481 at 129 If the plaintiff has an adequate remedy at law (eg if money damages can compensate the plaintiff for its loss) then a preliminary injunction will be denied Id sect 29481

Here a court would likely conclude that the potential for environmental damage to the forest creates a significant threat of irreparable harm ldquo[E]nvironmental injury is often irreparable Courts have recognized that logging such as would occur [here] can have longshyterm environmental consequences and thus satisfy the irreparable injury criterionrdquo Habitat Educ Center 363 F Supp 2d at 1089 (citing Idaho Sporting Congress Inc v Alexander 222 F3d 562 569 (9th Cir 2000) (noting that the imminent and continuing logging activities presented ldquoevidence of environmental harm sufficient to tip the balance in favor of injunctive reliefrdquo)) Neighbors of Cuddy Mountain v US Forest Service 137 F3d 1372 1382 (9th Cir 1998) (stating that ldquo[t]he old growth forests plaintiffs seek to protect would if cut take hundreds of years to reproducerdquo) (internal citation omitted)) see also 11C WRIGHT ET AL supra sect 29481 at 151 (noting that ldquoa preliminary injunction has been issued to prevent harm to the environmentrdquo)

The second factor the balance between the harm to the plaintiff and the harm the defendant will suffer if the injunction is issued also appears to support issuance of a preliminary injunction here The USFS will have to wait before it can develop the Scenic National Forest and the logging company may lose money if the delay is prolonged These economic harms could be compensated monetarily if an injunction is issued inappropriately Where ldquoan injunction bond can compensate [the] defendant for any harm the injunction is likely to inflict the balance should be struck in favor of [the] plaintiffrdquo Id sect 29482 at 192 See also Habitat Educ Center 363 F Supp 2d at 1089 (stating that ldquothe relative absence of harmful effects on the Forest Service weighs in favor of granting the injunctionrdquo)

The third factor is the likelihood that the plaintiff will prevail on the merits Although there is limited information concerning the merits of the action the nonprofit alleges that the federal statute (NEPA) requires an environmental impact statement and further states that the USFS created no environmental impact analysis or statement at all Assuming that those

25

Federal Civil Procedure Analysis

allegations are correct it seems plausible to conclude that the nonprofit will be able to show a likelihood of success on the merits

Finally courts deciding whether or not to issue preliminary injunctive relief are to consider the public interest ldquoFocusing on this factor is another way of inquiring whether there are policy considerations that bear on whether the order should issuerdquo 11C WRIGHT ET AL supra sect 29484 at 214 If the court concludes that the nonprofit is likely to succeed on its NEPA claim because the USFS wrongfully failed to conduct an environmental impact assessment it is likely to find that the public interest would be served by restraining the USFS from proceeding with logging in a national forest See Heartwood Inc v US Forest Service 73 F Supp 2d 962 979 (SD Ill 1999) affrsquod on other grounds 230 F3d 947 (7th Cir 2000) (ldquoviolations by federal agencies of NEPArsquos provisions as established by Congress harm the public as well as the environmentrdquo)

Thus a court is very likely to grant a preliminary injunction if it concludes that the nonprofit has a significant likelihood of success on the merits

26

EVIDENCE ANALYSIS (Evidence ID IIA amp C)

ANALYSIS

Legal Problems

(1) Under what circumstances can evidence of prior convictions be used to impeach a witnessrsquos credibility in a civil case

(1)(a) May the inmatersquos credibility be impeached by evidence of a 12-year-old felony drug conviction if he was released from prison 9 years ago

(1)(b) May the inmatersquos credibility be impeached by evidence of an 8-year-old misdemeanor perjury conviction that was punishable by 1 year in jail if he pleaded guilty and was sentenced only to pay a $5000 fine

(1)(c) May the inmatersquos credibility be impeached by evidence of a 7-year-old sexual assault conviction if the inmate is still serving a 10-year prison sentence and the victim was his 13-year-old daughter

(2)(a) May the guardrsquos credibility be impeached by cross-examination regarding specific instances of misconduct (ie lying on his reacutesumeacute) relevant to credibility

(2)(b) May the guardrsquos credibility be impeached by admission of extrinsic evidence (his reacutesumeacute and academic transcript) offered to prove specific instances of misconduct relevant to credibility

DISCUSSION

Summary

Under the Federal Rules of Evidence witnesses can be impeached with evidence of prior convictions andor specific instances of misconduct Whether evidence of prior convictions should be admitted to impeach generally depends on the nature of the crime the amount of time that has passed and (only in criminal cases) whether the ldquowitnessrdquo is the defendant FED R EVID 609(a)

In this civil case evidence of the inmatersquos conviction for distribution of marijuana should be admitted to impeach the inmate because he was convicted of a felony and was released from prison fewer than 10 years ago FED R EVID 609(a)(1) Credibility is critically important in this case because the jury will hear conflicting testimony from the two disputing parties and there were no other eyewitnesses to the altercation Under Rule 609(a)(1) the inmatersquos conviction should be admitted because it has some bearing on his credibility and its probative value is not substantially outweighed by concerns of unfair prejudice confusion or delay Id

Evidence of the inmatersquos misdemeanor conviction for perjury must be admitted because the crime ldquorequired provingmdashor the witnessrsquos admittingmdasha dishonest act or false statementrdquo by the inmate FED R EVID 609(a)(2)

27

Evidence Analysis

Evidence of the inmatersquos felony conviction for sexual assault should be excluded because its probative value is substantially outweighed by the danger of unfair prejudice to the inmate based on the heinous nature of the crime FED R EVID 609(a)(1) In the alternative the judge could limit the evidence relating to this conviction by excluding details of the inmatersquos crime

In all civil (and criminal) cases witnesses can also be impeached with evidence of specific instances of prior misconduct that did not result in a conviction FED R EVID 608(b) Pursuant to Rule 608(b) misconduct probative of untruthfulness can be inquired into on cross-examination but cannot be proved through extrinsic evidence Id Thus the inmatersquos counsel should be permitted to cross-examine the guard regarding the false statement in the guardrsquos reacutesumeacute However extrinsic evidence of the guardrsquos misconduct (ie the guardrsquos authenticated reacutesumeacute and transcript from the local college) should not be admitted even if the guard denies wrongdoing or refuses to answer cross-examination questions about these matters Id

Point One (10) The Federal Rules of Evidence permit impeachment of witnesses with evidence of prior convictions

Whether convictions should be admitted to impeach generally depends on the nature of the crime the amount of time that has passed and (only in criminal cases) whether the ldquowitnessrdquo is the defendant FED R EVID 609(a) Under Rule 609(a) evidence of prior convictions may be admitted for the purpose of ldquoattacking a witnessrsquos character for truthfulnessrdquo Id

There are two basic types of convictions that can be admitted for the purpose of impeachment

(1) convictions for crimes ldquopunishable by death or by imprisonment for more than one yearrdquo (which generally correlates to ldquofeloniesrdquo) FED R EVID 609(a)(1) and (2) convictions ldquofor any crimes regardless of the punishment if the court can readily determine that establishing the elements of the crime required provingmdashor the witnessrsquos admittingmdasha dishonest act or false statementrdquo FED R EVID 609(a)(2)

Pursuant to Rule 609(a)(1) in civil cases the admission of evidence of a felony conviction is ldquosubject to Rule 403 [which says that a court may exclude relevant evidence if its probative value is substantially outweighed by other factors]rdquo FED R EVID 609(a)(1) However Rule 403 does not protect the witness against admission of prior convictions involving dishonestymdashwhich must be admitted by the court FED R EVID 609(a)(2)

Finally Federal Rule of Evidence 609(b) contains the presumption that a conviction that is more than 10 years old or where more than 10 years has passed since the witnessrsquos release from confinement (whichever is later) should not be admitted unless ldquoits probative value supported by specific facts and circumstances substantially outweighs its prejudicial effectrdquo and the proponent has provided the adverse party with reasonable written notice FED R EVID 609(b)

Point One(a) (25) The court should admit evidence of the inmatersquos 12-year-old felony marijuana distribution conviction

The inmatersquos conviction for marijuana distribution was for a felony punishable by imprisonment for more than one year See FED R EVID 609(a)(1) Moreover although the conviction was 12 years ago the 10-year time limit of Rule 609(b) is not exceeded because that time limit runs

28

Evidence Analysis

from the date of either ldquothe witnessrsquos conviction or release from confinement for it whichever is laterrdquo FED R EVID 609(b) Because the inmate served three years in prison he was released from confinement nine years ago

However pursuant to Rule 609(a)(1) the admission of felony convictions to impeach a witness in a civil case is ldquosubject to Rule 403rdquo FED R EVID 609(a)(1) Neither Rule 609(a) nor the advisory committee notes specify which factors courts should consider when balancing the probative value of a conviction against the dangers identified in Rule 403 (which include (1) unfair prejudice (2) confusion of the issues (3) misleading the jury (4) waste of time or undue delay and (5) needless presentation of cumulative evidence) FED R EVID 403

In this case credibility is very important because the evidence consists primarily of the testimony of the disputing parties and there were no other eyewitnesses to the altercation This enhances the probative value of any evidence bearing on the inmatersquos credibility A court is likely to conclude that the inmatersquos prior felony drug conviction is relevant to his credibility See eg United States v Brito 427 F3d 53 64 (1st Cir 2005) (ldquoPrior drug-trafficking crimes are generally viewed as having some bearing on veracityrdquo) Although the probative value of any conviction diminishes with age see eg United States v Brewer 451 F Supp 50 53 (ED Tenn 1978) the inmatersquos ongoing problems with the law suggest that he has continued (and even escalated) his criminal behavior over the past nine years The court should admit this evidence because its probative value is not substantially outweighed by any Rule 403 concerns Specifically any prejudice to the inmate would be slight because the conviction is unrelated to the altercation at issue and the conviction was not for a heinous crime that might inflame the jury

[NOTE Whether an examinee identifies the jury instruction as containing a ldquoconclusiverdquo or ldquomandatoryrdquo presumption is less important than the examineersquos analysis of the constitutional infirmities]

Point One(b) (15) The court must admit evidence of the inmatersquos eight-year-old misdemeanor conviction because perjury is a crime of dishonesty

Rule 609(a)(2) provides that evidence of a criminal conviction ldquomust be admitted if the court can readily determine that establishing the elements of the crime required provingmdashor the witnessrsquos admittingmdasha dishonest act or false statementrdquo FED R EVID 609(a)(2) The inmatersquos conviction for perjury would have necessarily required proving that the inmate engaged in an act of dishonesty This conviction occurred within the past 10 years so it ldquomust be admittedrdquo because in contrast to Rule 609(a)(1) (discussed in Point One(a)) admission under Rule 609(a)(2) is mandatory and not subject to Rule 403

Point One(c) (20) The court should exclude evidence of the inmatersquos seven-year-old felony sexual assault conviction because the probative value of this evidence is substantially outweighed by the danger of unfair prejudice In the alternative the details of the prior conviction could be excluded

The inmatersquos conviction for felony sexual assault was seven years ago and he has not yet been released from incarceration so Rule 609(a) but not 609(b) is applicable here FED R EVID 609(a) This conviction is therefore admissible to impeach the inmate unless its probative value is substantially outweighed by the danger of unfair prejudice or any other Rule 403 concern Id

29

Evidence Analysis

Sex crimes are generally not considered relevant to credibility see Hopkins v State 639 So 2d 1247 1254 (Miss 1993) so the probative value of this conviction is relatively low Moreover the heinous nature of the inmatersquos crime (sexual assault on his daughter) makes the danger of unfair prejudice to the inmate very high Thus the court should exclude evidence of the conviction because it was for a heinous offense that is likely to inflame the jury and it has little bearing on credibility See eg United States v Beahm 664 F2d 414 419 (4th Cir 1981)

As an alternative to excluding this evidence the judge could minimize the unfair prejudice to the inmate by permitting limited cross-examination but refusing to allow specific questions about the nature of the inmatersquos conviction For example a court could limit cross-examination to the fact that the inmate was convicted of a ldquofelonyrdquo or perhaps that he was convicted of a ldquosexual assaultrdquo without identifying the victim However because evidence of the inmatersquos prior convictions can be admitted solely for the purpose of enabling the jury to assess his credibility and because his two earlier convictions should have already been admitted the court should exclude all evidence of the felony sexual assault conviction

Point Two(a) (15) The court should permit the inmatersquos counsel to cross-examine the guard regarding the false statement in his reacutesumeacute because the guardrsquos misconduct bears on his truthfulness

The inmate wishes to cross-examine the guard about his prior dishonest behaviormdashlying on his reacutesumeacutemdashthat did not involve a criminal conviction Rule 608(b) allows witnesses to be cross-examined about specific instances of prior non-conviction misconduct probative of untruthfulness ldquoin order to attack the witnessrsquos character for truthfulnessrdquo FED R EVID 608(b)

The courtrsquos decision to allow cross-examination about the guardrsquos prior dishonest behavior depends on the probative value of such evidence balanced against the danger of unfair prejudice to the guard or any other Rule 403 concern FED R EVID 403 Here the guardrsquos false statement on his reacutesumeacute that he obtained a degree in Criminal Justice is highly probative of his untruthfulness because it grossly misrepresents his actual academic record was made recently and was made with the intent to deceive Because the probative value of this evidence is very strong and is not substantially outweighed by any Rule 403 concerns cross-examination of the guard on this topic should be permitted The court may also consider it fair to permit this cross-examination of the guard on these matters assuming that one or more of the inmatersquos prior convictions have been admitted to impeach his credibility

Point Two(b) (15) The court should exclude extrinsic evidence of the guardrsquos non-conviction misconduct even if the guard denies wrongdoing or refuses to answer questions about the matter

Although Rule 608(b) allows cross-examination about specific instances of prior misconduct probative of untruthfulness ldquoextrinsic evidencerdquo offered to prove such misconduct is not admissible FED R EVID 608(b) The rationale for this rule is that allowing the introduction of extrinsic evidence of prior misconduct by witnesses when these acts are relevant only to the witnessesrsquo truthfulness and not to the main issues in the case would create too great a risk of confusing the jury and unduly delaying the trial The court does not have discretion to admit this extrinsic evidence See eg United States v Elliot 89 F3d 1360 1368 (8th Cir 1996)

30

Evidence Analysis

Here the inmatersquos counsel may cross-examine the guard about the false statement on his reacutesumeacute However the inmatersquos counsel must accept the guardrsquos response Even if the guard denies wrongdoing or refuses to answer questions about the matter the inmatersquos counsel cannot introduce the guardrsquos reacutesumeacute or the transcript from the local college to prove the guardrsquos misconduct

31

CORPORATIONS ANALYSIS (Corporations VA2 IX)

ANALYSIS

Legal Problems

(1) Do shareholders have the authority to amend a corporationrsquos bylaws with respect to director nominations

(2) Do board-approved bylaws on a particular subject here nomination of directors preempt subsequent conflicting bylaw amendments by shareholders

(3) Is a suit challenging both managementrsquos refusal to include the proposed bylaw amendment in Megarsquos proxy statement and the boardrsquos amendment of the bylaws dealing with nomination of directors a direct or derivative suit

DISCUSSION

Summary

The voting and litigation rights of the shareholders of Mega are subject to the provisions of the Model Business Corporations Act (MBCA)

The investorrsquos proposed bylaw provision is not inconsistent with state law Under the MBCA shareholders may amend the bylaws when the amendment deals with a proper matter for the corporationrsquos bylaws such as procedures for nominating directors

The Mega boardrsquos bylaw amendment does not preempt the investorrsquos proposed bylaw provision or the Mega shareholdersrsquo power to approve it While shareholders can limit the boardrsquos power to amend or repeal the bylaws the board cannot limit the shareholdersrsquo power

Whether the investor must make a demand on Megarsquos board depends on how the investor frames its claim If the investor claims a violation of shareholder voting rights the claim is direct and pre-suit demand on the board is not required If on the other hand the investor claims that the directors violated their fiduciary duties by amending the bylaws to entrench themselves the claim is derivative and a pre-suit demand is required

Point One (30) Shareholders may amend the corporationrsquos bylaws where the proposed bylaw provision relates to procedural matters typically included in the bylaws such as the nomination of directors

Internal affairs of the corporation such as the conduct of shareholder meetings and election of directors are subject to the corporate law of the state of incorporation See McDermott Inc v Lewis 531 A2d 206 (Del 1987) (applying law of jurisdiction where corporation was incorporated in case involving voting rights) This statersquos corporate statute is modeled on the MBCA

Under the MBCA ldquoshareholders may amend the corporationrsquos bylawsrdquo MBCA sect 1020(a) Thus the only question is whether the bylaws can specify the procedures for shareholder nomination of directors

32

Corporations Analysis

The MBCA states that the bylaws ldquomay contain any provision that is not inconsistent with law or the articles of incorporationrdquo MBCA sect 206(b) In addition the MBCA was revised in 2009 to address shareholder nomination of directors in public corporations (known as ldquoproxy accessrdquo) and specifies that the bylaws ldquomay contain a requirement that the corporation include in its [proxy materials] one or more individuals nominated by a shareholderrdquo MBCA sect 206(c)(1) see Committee on Corporate Laws ABA Section of Business Law Report on the Roles of Boards of Directors and Shareholders of Publicly Owned Corporations and Changes to the Model Business Corporations ActmdashAdoption of Shareholder Proxy Access Amendments to Chapters 2 and 10 65 BUS LAWYER 1105 (2010)

The inclusion of director-nomination procedures in the bylaws is consistent with practice and is recognized by the Delaware courts whose views on corporate law carry significant weight Typically the procedures for nomination of directors are found in the bylaws See 1 COX amp HAZEN TREATISE ON THE LAW OF CORPORATIONS sect 312 (3d ed 2011) see also 4 FLETCHER CORP FORMS ANN PART III ch 21 (2013) (including sample bylaws that permit nomination of directors by shareholders) The Delaware Supreme Court has confirmed that the bylaws may ldquodefine the process and proceduresrdquo for director elections See CA Inc v AFSCME Employees Pension Plan 953 A2d 227 (Del 2008) (concluding that bylaw amendment requiring reimbursement of election expenses to certain successful shareholder nominators is ldquoproper subjectrdquo under Delaware law)

[NOTE The question of the proper scope of the bylaws can be answered using the more general MBCA sect 206(b) or the 2009 MBCA revision adding sect 206(c)(1) (adopted in CT ME VA) In addition some examinees might raise the point that shareholder proposals may not compel the board to take action such as by including shareholder nominations in the companyrsquos proxy materials on the theory that the ldquobusiness and affairsrdquo of the corporation are to be managed by the board See MBCA sect 801(b) Although shareholders are generally limited to adopting precatory resolutions that recommend or encourage board action this limitation does not apply when shareholders have specific authority to take binding action on their ownmdashsuch as to amend the bylaws]

Point Two (30) Shareholders can amend (or repeal) board-approved bylaws Further shareholders can limit the boardrsquos power to later amend and repeal a shareholder-approved bylaw

Under the MBCA shareholders have the power to amend the bylaws See Point One The board shares this power with the shareholders unless (1) the corporationrsquos articles ldquoreserve that power exclusively to the shareholdersrdquo or (2) ldquothe shareholders in amending repealing or adopting a bylaw expressly provide that the board of directors may not amend repeal or reinstate that bylawrdquo See MBCA sect 1020(b)

Shareholder-approved bylaw provisions can amend or repeal existing bylaw provisions whether originally approved by the board or by shareholders See ALAN R PALMITER CORPORATIONS EXAMPLES AND EXPLANATIONS sect 713 (7th ed 2012) Thus the Mega boardrsquos bylaw amendmentmdashwhich set more demanding thresholds for shareholder nomination of directors than the investorrsquos proposed bylaw provisionmdashwould be superseded (repealed) if Megarsquos shareholders were to approve the investorrsquos proposal

Further a shareholder-approved bylaw generally can limit the power of the board to later amend or repeal it See MBCA sect 1020(b)(2) Thus if Megarsquos shareholders approved the bylaw

33

Corporations Analysis

provision proposed by the investor Megarsquos board could not repeal the provision because it includes a ldquono board repealrdquo clause

The revision to the MBCA in 2009 dealing with shareholder proxy access does not change this conclusion That revision specifies that a shareholder-approved bylaw dealing with director nominations may not limit the boardrsquos power to amend add or repeal ldquoany procedure or condition to such a bylaw in order to provide for a reasonable practicable and orderly processrdquo MBCA sect 206(d) Thus according to the revision if shareholders approve a bylaw amendment that limits further board changes the board would nonetheless retain the power to ldquotinkerrdquo with the bylaw to safeguard the voting process but could not repeal the shareholder-approved bylaw The Official Comment to MBCA sect 206(d) makes clear that the revision is ldquonot intended to allow the board of directors to frustrate the purpose of the shareholder-adopted proxy access provisionrdquo Thus if Megarsquos shareholders were to approve the bylaw provision proposed by the investor Megarsquos board could only amend the provision regarding its procedures or conditions in a manner consistent with its purpose of permitting proxy access for Megarsquos shareholders

[NOTE The boardrsquos attempted interference with a shareholder voting initiative may also have been a violation of the directorsrsquo fiduciary duties See Blasius Indus Inc v Atlas Corp 564 A2d 651 (Del Ch 1988) (finding that directors breached their fiduciary duties by amending bylaws and expanding size of board to thwart insurgentrsquos plan to amend bylaws and seat a majority of new directors) The call however asks examinees to consider whether shareholders or the board have ldquoprecedencerdquo over amending the corporate bylaws Thus an examineersquos answer should be framed in terms of ldquopowerrdquo and not ldquodutyrdquo]

Point Three (40) The investor need not make a demand on the board if the investor states a direct claim such as an allegation that the board interfered with the investorrsquos right to amend the bylaws But the investor must make a demand on the board if the investor states a derivative claim (on behalf of the corporation) such as an allegation that the directors sought to entrench themselves by interfering with the proposed proxy access

The MBCA generally requires that shareholders make a demand on the board of directors before initiation of a derivative suit MBCA sect 742 (shareholder may not bring derivative proceeding until written demand has been made on corporation and 90 days have expired) A derivative suit is essentially two suits in one where the plaintiff-shareholder seeks to bring on behalf of the corporation a claim that vindicates corporate rights usually based on violation of fiduciary duties PALMITER supra sect 1811 (6th ed 2009) The demand permits the board to investigate the situation identified by the shareholder and take suitable action No demand on the board is required however if the shareholder brings a direct suit to vindicate the shareholderrsquos own rights not those of the corporation

Is the suit brought by the investor derivative or direct The MBCA defines a ldquoderivative proceedingrdquo as one brought ldquoin the right of a domestic corporationrdquo MBCA sect 740(1) Thus the answer to how the investorrsquos suit should be characterized turns on what rights the investor seeks to vindicate If the investor frames its claim as one of fiduciary breach by directorsmdashfor example for failing to become adequately informed about voting procedures or for seeking to entrench themselves in office by manipulating the voting structure to avoid a shareholder insurgencymdashthen the suit is ldquoderivativerdquo and the investor must make a demand on the board See MBCA Ch 7 Subch D Introductory Comment (ldquothe derivative suit has historically been the principal method of challenging allegedly illegal action by managementrdquo)

34

Corporations Analysis

If however the investor frames its claim as one to vindicate shareholder rights the suit is direct and no demand is required For many courts the direct-derivative question turns on who is injured and who is to receive the relief sought by the plaintiff-shareholders See Tooley v Donaldson Lufkin amp Jenrette Inc 845 A2d 1031 (Del 2004) (characterizing a merger-delay claim as direct because delay of merger only harmed shareholders not corporation) Thus if the investor claims that managementrsquos refusal to include its proposed bylaw amendment in the corporationrsquos proxy materials violates its shareholder rights to initiate corporate governance reforms the suit will be direct Courts have not questioned the ability of shareholders to bring direct suits challenging board action to exclude their proposed bylaw amendments from the corporationrsquos proxy materials See JANA Master Fund Ltd v CNET Networks Inc 954 A2d 335 (Del Ch 2008) (upholding shareholderrsquos direct challenge to boardrsquos interpretation of advance-notice bylaw) Chesapeake Corp v Shore 771 A2d 293 (Del Ch 2000) (upholding shareholderrsquos direct challenge to actions by board that effectively prevented it from proposing bylaw amendments in contest for control)

Is the way that the investor frames its claim conclusive Courts have permitted shareholder-plaintiffs to challenge a transaction in a direct suit even though the same transaction could also be challenged as a fiduciary breach See Eisenberg v Flying Tiger Line Inc 451 F2d 267 (2d Cir 1971) (permitting direct suit challenging a corporate reorganization as a dilution of shareholder voting power even though reorganization may have involved conflicts of interest and thus constituted a fiduciary breach) Thus the investorrsquos choice to pursue a claim challenging the legality of managementrsquos decision to exclude the investorrsquos proposal from the corporationrsquos proxy materialsmdashrather than a possible breach of fiduciary dutymdashis likely to be respected See 3 COX amp HAZEN supra sect 153 (describing situations in which a claim can be framed as derivative or direct)

[NOTE Some issues under Delaware corporate law regarding pre-suit demand are not relevant here For example whether the Mega directors are independent and disinterested is not relevant to the MBCA requirement of a pre-suit demand As the Official Comment to MBCA sect 742 points out the MBCArsquos requirement of ldquouniversal demandrdquo gives the board ldquothe opportunity to reexamine the act complained of in the light of a potential lawsuit and take corrective actionrdquo even when the directors might be non-independent or have conflicts of interest

Nor is it relevant to the MBCA pre-suit demand requirement that the statutory 90-day waiting period may be onerous The first paragraph of MBCA sect 742 requires a pre-suit demand without exception the second paragraph of the section imposes a 90-day waiting period before a derivative suit may be brought which can be shortened if the board rejects the demand or ldquoirreparable injury to the corporation would result by waiting for the expiration of the 90-day periodrdquo The call as written asks only whether a pre-suit demand should be made and does not ask examinees to address whether the post-demand waiting period should be shortened under the ldquoirreparable injuryrdquo standard]

35

National Conference of Bar Examiners 302 South Bedford Street | Madison WI 53703-3622 Phone 608-280-8550 | Fax 608-280-8552 | TDD 608-661-1275

wwwncbexorg e-mail contactncbexorg

  • Preface
  • Description of the MEE
  • Instructions
  • July 2014 Questions
    • CRIMINAL LAW AND PROCEDURE QUESTION
    • CONTRACTS QUESTION
    • FAMILY LAW QUESTION
    • FEDERAL CIVIL PROCEDURE QUESTION
    • EVIDENCE QUESTION
    • CORPORATIONS QUESTION
      • July 2014 Analyses
        • CRIMINAL LAW AND PROCEDURE ANALYSIS
        • CONTRACTS ANALYSIS
        • FAMILY LAW ANALYSIS
        • FEDERAL CIVIL PROCEDURE ANALYSIS
        • EVIDENCE ANALYSIS
        • CORPORATIONS ANALYSIS
            • ltlt13 ASCII85EncodePages false13 AllowTransparency false13 AutoPositionEPSFiles true13 AutoRotatePages None13 Binding Left13 CalGrayProfile (Dot Gain 20)13 CalRGBProfile (sRGB IEC61966-21)13 CalCMYKProfile (US Web Coated 050SWOP051 v2)13 sRGBProfile (sRGB IEC61966-21)13 CannotEmbedFontPolicy Error13 CompatibilityLevel 1413 CompressObjects Tags13 CompressPages true13 ConvertImagesToIndexed true13 PassThroughJPEGImages true13 CreateJobTicket false13 DefaultRenderingIntent Default13 DetectBlends true13 DetectCurves 0000013 ColorConversionStrategy CMYK13 DoThumbnails false13 EmbedAllFonts true13 EmbedOpenType false13 ParseICCProfilesInComments true13 EmbedJobOptions true13 DSCReportingLevel 013 EmitDSCWarnings false13 EndPage -113 ImageMemory 104857613 LockDistillerParams false13 MaxSubsetPct 10013 Optimize true13 OPM 113 ParseDSCComments true13 ParseDSCCommentsForDocInfo true13 PreserveCopyPage true13 PreserveDICMYKValues true13 PreserveEPSInfo true13 PreserveFlatness true13 PreserveHalftoneInfo false13 PreserveOPIComments true13 PreserveOverprintSettings true13 StartPage 113 SubsetFonts true13 TransferFunctionInfo Apply13 UCRandBGInfo Preserve13 UsePrologue false13 ColorSettingsFile ()13 AlwaysEmbed [ true13 ]13 NeverEmbed [ true13 ]13 AntiAliasColorImages false13 CropColorImages true13 ColorImageMinResolution 30013 ColorImageMinResolutionPolicy OK13 DownsampleColorImages true13 ColorImageDownsampleType Bicubic13 ColorImageResolution 30013 ColorImageDepth -113 ColorImageMinDownsampleDepth 113 ColorImageDownsampleThreshold 15000013 EncodeColorImages true13 ColorImageFilter DCTEncode13 AutoFilterColorImages true13 ColorImageAutoFilterStrategy JPEG13 ColorACSImageDict ltlt13 QFactor 01513 HSamples [1 1 1 1] VSamples [1 1 1 1]13 gtgt13 ColorImageDict ltlt13 QFactor 01513 HSamples [1 1 1 1] VSamples [1 1 1 1]13 gtgt13 JPEG2000ColorACSImageDict ltlt13 TileWidth 25613 TileHeight 25613 Quality 3013 gtgt13 JPEG2000ColorImageDict ltlt13 TileWidth 25613 TileHeight 25613 Quality 3013 gtgt13 AntiAliasGrayImages false13 CropGrayImages true13 GrayImageMinResolution 30013 GrayImageMinResolutionPolicy OK13 DownsampleGrayImages true13 GrayImageDownsampleType Bicubic13 GrayImageResolution 30013 GrayImageDepth -113 GrayImageMinDownsampleDepth 213 GrayImageDownsampleThreshold 15000013 EncodeGrayImages true13 GrayImageFilter DCTEncode13 AutoFilterGrayImages true13 GrayImageAutoFilterStrategy JPEG13 GrayACSImageDict ltlt13 QFactor 01513 HSamples [1 1 1 1] VSamples [1 1 1 1]13 gtgt13 GrayImageDict ltlt13 QFactor 01513 HSamples [1 1 1 1] VSamples [1 1 1 1]13 gtgt13 JPEG2000GrayACSImageDict ltlt13 TileWidth 25613 TileHeight 25613 Quality 3013 gtgt13 JPEG2000GrayImageDict ltlt13 TileWidth 25613 TileHeight 25613 Quality 3013 gtgt13 AntiAliasMonoImages false13 CropMonoImages true13 MonoImageMinResolution 120013 MonoImageMinResolutionPolicy OK13 DownsampleMonoImages true13 MonoImageDownsampleType Bicubic13 MonoImageResolution 120013 MonoImageDepth -113 MonoImageDownsampleThreshold 15000013 EncodeMonoImages true13 MonoImageFilter CCITTFaxEncode13 MonoImageDict ltlt13 K -113 gtgt13 AllowPSXObjects false13 CheckCompliance [13 None13 ]13 PDFX1aCheck false13 PDFX3Check false13 PDFXCompliantPDFOnly false13 PDFXNoTrimBoxError true13 PDFXTrimBoxToMediaBoxOffset [13 00000013 00000013 00000013 00000013 ]13 PDFXSetBleedBoxToMediaBox true13 PDFXBleedBoxToTrimBoxOffset [13 00000013 00000013 00000013 00000013 ]13 PDFXOutputIntentProfile ()13 PDFXOutputConditionIdentifier ()13 PDFXOutputCondition ()13 PDFXRegistryName ()13 PDFXTrapped False1313 CreateJDFFile false13 Description ltlt13 ARA ltFEFF06270633062A062E062F0645002006470630064700200627064406250639062F0627062F0627062A002006440625064606340627062100200648062B062706260642002000410064006F00620065002000500044004600200645062A064806270641064206290020064406440637062806270639062900200641064A00200627064406450637062706280639002006300627062A0020062F0631062C0627062A002006270644062C0648062F0629002006270644063906270644064A0629061B0020064A06450643064600200641062A062D00200648062B0627062606420020005000440046002006270644064506460634062306290020062806270633062A062E062F062706450020004100630072006F0062006100740020064800410064006F006200650020005200650061006400650072002006250635062F0627063100200035002E0030002006480627064406250635062F062706310627062A0020062706440623062D062F062B002E0635062F0627063100200035002E0030002006480627064406250635062F062706310627062A0020062706440623062D062F062B002Egt13 BGR 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 CHS ltFEFF4f7f75288fd94e9b8bbe5b9a521b5efa7684002000410064006f006200650020005000440046002065876863900275284e8e9ad88d2891cf76845370524d53705237300260a853ef4ee54f7f75280020004100630072006f0062006100740020548c002000410064006f00620065002000520065006100640065007200200035002e003000204ee553ca66f49ad87248672c676562535f00521b5efa768400200050004400460020658768633002gt13 CHT ltFEFF4f7f752890194e9b8a2d7f6e5efa7acb7684002000410064006f006200650020005000440046002065874ef69069752865bc9ad854c18cea76845370524d5370523786557406300260a853ef4ee54f7f75280020004100630072006f0062006100740020548c002000410064006f00620065002000520065006100640065007200200035002e003000204ee553ca66f49ad87248672c4f86958b555f5df25efa7acb76840020005000440046002065874ef63002gt13 CZE 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 DAN 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 DEU ltFEFF00560065007200770065006e00640065006e0020005300690065002000640069006500730065002000450069006e007300740065006c006c0075006e00670065006e0020007a0075006d002000450072007300740065006c006c0065006e00200076006f006e002000410064006f006200650020005000440046002d0044006f006b0075006d0065006e00740065006e002c00200076006f006e002000640065006e0065006e002000530069006500200068006f006300680077006500720074006900670065002000500072006500700072006500730073002d0044007200750063006b0065002000650072007a0065007500670065006e0020006d00f60063006800740065006e002e002000450072007300740065006c006c007400650020005000440046002d0044006f006b0075006d0065006e007400650020006b00f6006e006e0065006e0020006d006900740020004100630072006f00620061007400200075006e0064002000410064006f00620065002000520065006100640065007200200035002e00300020006f0064006500720020006800f600680065007200200067006500f600660066006e00650074002000770065007200640065006e002egt13 ESP 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 ETI 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 FRA 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 GRE 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 HEB ltFEFF05D405E905EA05DE05E905D5002005D105D405D205D305E805D505EA002005D005DC05D4002005DB05D305D9002005DC05D905E605D505E8002005DE05E105DE05DB05D9002000410064006F006200650020005000440046002005D405DE05D505EA05D005DE05D905DD002005DC05D405D305E405E105EA002005E705D305DD002D05D305E405D505E1002005D005D905DB05D505EA05D905EA002E002005DE05E105DE05DB05D90020005000440046002005E905E005D505E605E805D5002005E005D905EA05E005D905DD002005DC05E405EA05D905D705D4002005D105D005DE05E605E205D505EA0020004100630072006F006200610074002005D5002D00410064006F00620065002000520065006100640065007200200035002E0030002005D505D205E805E105D005D505EA002005DE05EA05E705D305DE05D505EA002005D905D505EA05E8002E05D005DE05D905DD002005DC002D005000440046002F0058002D0033002C002005E205D905D905E005D5002005D105DE05D305E805D905DA002005DC05DE05E905EA05DE05E9002005E905DC0020004100630072006F006200610074002E002005DE05E105DE05DB05D90020005000440046002005E905E005D505E605E805D5002005E005D905EA05E005D905DD002005DC05E405EA05D905D705D4002005D105D005DE05E605E205D505EA0020004100630072006F006200610074002005D5002D00410064006F00620065002000520065006100640065007200200035002E0030002005D505D205E805E105D005D505EA002005DE05EA05E705D305DE05D505EA002005D905D505EA05E8002Egt13 HRV (Za stvaranje Adobe PDF dokumenata najpogodnijih za visokokvalitetni ispis prije tiskanja koristite ove postavke Stvoreni PDF dokumenti mogu se otvoriti Acrobat i Adobe Reader 50 i kasnijim verzijama)13 HUN 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 ITA 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 JPN ltFEFF9ad854c18cea306a30d730ea30d730ec30b951fa529b7528002000410064006f0062006500200050004400460020658766f8306e4f5c6210306b4f7f75283057307e305930023053306e8a2d5b9a30674f5c62103055308c305f0020005000440046002030d530a130a430eb306f3001004100630072006f0062006100740020304a30883073002000410064006f00620065002000520065006100640065007200200035002e003000204ee5964d3067958b304f30533068304c3067304d307e305930023053306e8a2d5b9a306b306f30d530a930f330c8306e57cb30818fbc307f304c5fc59808306730593002gt13 KOR ltFEFFc7740020c124c815c7440020c0acc6a9d558c5ec0020ace0d488c9c80020c2dcd5d80020c778c1c4c5d00020ac00c7a50020c801d569d55c002000410064006f0062006500200050004400460020bb38c11cb97c0020c791c131d569b2c8b2e4002e0020c774b807ac8c0020c791c131b41c00200050004400460020bb38c11cb2940020004100630072006f0062006100740020bc0f002000410064006f00620065002000520065006100640065007200200035002e00300020c774c0c1c5d0c11c0020c5f40020c2180020c788c2b5b2c8b2e4002egt13 LTH 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 LVI 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 NLD (Gebruik deze instellingen om Adobe PDF-documenten te maken die zijn geoptimaliseerd voor prepress-afdrukken van hoge kwaliteit De gemaakte PDF-documenten kunnen worden geopend met Acrobat en Adobe Reader 50 en hoger)13 NOR 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 POL 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 PTB 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 RUM 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 RUS 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 SKY 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 SLV 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 SUO 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 SVE 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 TUR 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 UKR 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 ENU (Use these settings to create Adobe PDF documents best suited for high-quality prepress printing Created PDF documents can be opened with Acrobat and Adobe Reader 50 and later)13 gtgt13 Namespace [13 (Adobe)13 (Common)13 (10)13 ]13 OtherNamespaces [13 ltlt13 AsReaderSpreads false13 CropImagesToFrames true13 ErrorControl WarnAndContinue13 FlattenerIgnoreSpreadOverrides false13 IncludeGuidesGrids false13 IncludeNonPrinting false13 IncludeSlug false13 Namespace [13 (Adobe)13 (InDesign)13 (40)13 ]13 OmitPlacedBitmaps false13 OmitPlacedEPS false13 OmitPlacedPDF false13 SimulateOverprint Legacy13 gtgt13 ltlt13 AddBleedMarks false13 AddColorBars false13 AddCropMarks false13 AddPageInfo false13 AddRegMarks false13 ConvertColors ConvertToCMYK13 DestinationProfileName ()13 DestinationProfileSelector DocumentCMYK13 Downsample16BitImages true13 FlattenerPreset ltlt13 PresetSelector MediumResolution13 gtgt13 FormElements false13 GenerateStructure false13 IncludeBookmarks false13 IncludeHyperlinks false13 IncludeInteractive false13 IncludeLayers false13 IncludeProfiles false13 MultimediaHandling UseObjectSettings13 Namespace [13 (Adobe)13 (CreativeSuite)13 (20)13 ]13 PDFXOutputIntentProfileSelector DocumentCMYK13 PreserveEditing true13 UntaggedCMYKHandling LeaveUntagged13 UntaggedRGBHandling UseDocumentProfile13 UseDocumentBleed false13 gtgt13 ]13gtgt setdistillerparams13ltlt13 HWResolution [2400 2400]13 PageSize [612000 792000]13gtgt setpagedevice13

Page 11: July 2014 MEE Questions and Analyses - NCBE...This publication includes the questions and analyses from the July 2014 MEE. (In the actual test, the questions are simply numbered rather

FEDERAL CIVIL PROCEDURE QUESTION

The United States Forest Service (USFS) manages public lands in national forests including the Scenic National Forest Without conducting an environmental evaluation or preparing an environmental impact statement the USFS approved a development project in the Scenic National Forest that required the clearing of 5000 acres of old-growth forest The trees in the forest are hundreds of years old and the forest is home to a higher concentration of wildlife than can be found anywhere else in the western United States

The USFS solicited bids from logging companies to harvest the trees on the 5000 acres of forest targeted for clearing and it ultimately awarded the logging contract to the company that had submitted the highest bid for the trees However the USFS has not yet issued the company a logging permit Once it does so the company intends to begin cutting down trees immediately

A nonprofit organization whose mission is the preservation of natural resources has filed suit in federal district court against the USFS The nonprofit alleges that the USFS violated the National Environmental Policy Act (NEPA) by failing to prepare an environmental impact statement for the proposed logging project Among other remedies the nonprofit seeks a permanent injunction barring the USFS from issuing a logging permit to the logging company until an adequate environmental impact statement is completed The nonprofit believes that the logging project would destroy important wildlife habitat and thereby cause serious harm to wildlife in the Scenic National Forest including some endangered species

Assume that federal subject-matter jurisdiction is available that the nonprofit has standing to bring this action and that venue is proper

1 If the logging company seeks to join the litigation as a party must the federal district court allow it to do so as a matter of right Explain

2 What types of relief could the nonprofit seek to stop the USFS from issuing a logging permit during the pendency of the action what must the nonprofit demonstrate to obtain that relief and is the federal district court likely to grant that relief Explain

7

EVIDENCE QUESTION

A prison inmate has filed a civil rights lawsuit against a guard at the prison alleging that the guard violated the inmatersquos constitutional rights during an altercation The inmate and the guard are the only witnesses to this altercation They have provided contradictory reports about what occurred

The trial will be before a jury The inmate plans to testify at trial The guardrsquos counsel has moved for leave to impeach the inmate with the following

(a) Twelve years ago the inmate was convicted of felony distribution of marijuana He served a three-year prison sentence which began immediately after he was convicted He served his full sentence and was released from prison nine years ago (b) Eight years ago the inmate pleaded guilty to perjury a misdemeanor punishable by up to one year in jail He paid a $5000 fine (c) Seven years ago the inmate was convicted of felony sexual assault of a child and is currently serving a 10-year prison sentence for the crime The victim was the inmatersquos daughter who was 13 years old at the time of the assault

The inmatersquos counsel objects to the admission of any evidence related to these three convictions and to any cross-examination based on this evidence

The guard also plans to testify at trial The inmatersquos counsel has moved for leave to impeach the guard with the following

Last year the guard applied for a promotion to prison supervisor The guard submitted a reacutesumeacute to the state that indicated that he had been awarded a BA in Criminal Justice from a local college An official copy of the guardrsquos academic transcript from that college indicates that the guard dropped out after his first semester and did not receive a degree

The guardrsquos counsel objects to the admission of this evidence and to any cross-examination based on this evidence

The transcript and the reacutesumeacute have been properly authenticated The trial will be held in a jurisdiction that has adopted all of the Federal Rules of Evidence

1 What evidence if any proffered by the guard to impeach the inmate should be admitted Explain

2 What evidence if any proffered by the inmate to impeach the guard should be admitted Explain

8

CORPORATIONS QUESTION

Mega Inc is a publicly traded corporation incorporated in a state whose corporate statute is modeled on the Model Business Corporation Act (MBCA) Megarsquos articles of incorporation do not address the election of directors or amendment of the bylaws by shareholders

Well within the deadline for the submission of shareholder proposals for the upcoming annual shareholdersrsquo meeting an investor who was a large and long-standing shareholder of Mega submitted a proposed amendment to Megarsquos bylaws The proposal which the investor asked to be included in the corporationrsquos proxy materials and voted on at the upcoming shareholdersrsquo meeting read as follows

Section 20 The Corporation shall include in its proxy materials (including the proxy ballot) for a shareholdersrsquo meeting at which directors are to be elected the name of a person nominated for election to the Board of Directors by a shareholder or group of shareholders that beneficially have owned 3 or more of the Corporationrsquos outstanding common stock for at least one year

This Section shall supersede any inconsistent provision in these Bylaws and may not be amended or repealed by the Board of Directors without shareholder approval

Megarsquos management decided to exclude the investorrsquos proposal from the corporationrsquos proxy materials and explained its reasons in a letter to the investor

The investorrsquos proposed bylaw provision would be inconsistent with relevant state law because the Board of Directors has the authority to manage the business and affairs of the Corporation Generally shareholders lack the authority to interfere with corporate management by seeking to create a method for the nomination and election of directors inconsistent with the method chosen by the Board of Directors

Furthermore at its most recent meeting the Board of Directors unanimously approved an amendment to the Corporationrsquos bylaws that provides for proxy access for director nominations by a shareholder or a group of shareholders holding at least 10 of the Corporationrsquos voting shares for at least three years This procedure takes precedence over any nomination methods that might be sought or approved by shareholders

The investor is considering bringing a suit challenging managementrsquos refusal to include the investorrsquos proposed bylaw provision and challenging the boardrsquos amendment of the bylaws at its recent meeting

1 Is the investorrsquos proposed bylaw provision inconsistent with state law Explain

2 If the investorrsquos proposed bylaw provision were approved by the shareholders would the bylaw amendment previously approved by the board take precedence over the investorrsquos proposed bylaw provision Explain

3 Must the investor make a demand on Megarsquos board of directors before bringing suit Explain

9

July 2014 MEE

ANALYSES

Contracts Family Law

Criminal Law and Procedure

Federal Civil Procedure Evidence

Corporations

CRIMINAL LAW AND PROCEDURE ANALYSIS (Criminal Law and Procedure VA B D)

ANALYSIS

Legal Problems

(1) Did the detective violate the suspectrsquos Sixth Amendment right to counsel when he questioned the suspect about the burglaries without the lawyer present given that the lawyer represented the suspect in an unrelated criminal matter

(2) Under Miranda did the suspect effectively invoke his right to counsel when he said ldquoI think I want my lawyer here before I talk to yourdquo

(3) Was the suspectrsquos waiver of his right to remain silent under Miranda valid

DISCUSSION

Summary

The Sixth Amendment right to counsel as applied to states through the Fourteenth Amendment is offense-specific Although the suspect had an attorney representing him on his pending assault charge he had no Sixth Amendment right to the assistance of counsel with respect to the five uncharged burglaries because formal adversarial proceedings had not yet commenced on those charges The suspectrsquos Sixth Amendment right to counsel was not violated by the detectiversquos failure to inform him that the lawyer was present or of the lawyerrsquos demands

However a person undergoing custodial interrogation also has an independent constitutional right to counsel during custodial interrogation under Miranda When a suspect invokes his right to counsel under Miranda custodial interrogation must immediately cease for a period of at least 14 days However the invocation of the right to counsel must be unambiguous and clearly convey that the suspect has requested counsel Here because the suspectrsquos statement ldquoI think I want my lawyer here before I talk to yourdquo was ambiguous he did not invoke his Miranda right to counsel

A waiver of rights must be knowing intelligent and voluntary Here the suspect waived his right to remain silent under Miranda when he signed the waiver form The fact that the detective did not correct the suspectrsquos assumption that the lawyer would need to drive to the jailmdashby telling him that the lawyer was in the waiting room and was demanding to see himmdashdid not affect the validity of the suspectrsquos waiver

Point One (35) The suspectrsquos Sixth Amendment right to counsel was not violated because the right does not attach on new charges until formal adversarial judicial proceedings have commenced on those charges

The Sixth Amendment as applied to the states through the Fourteenth Amendment provides that ldquo[i]n all criminal prosecutions the accused shall enjoy the right to have the Assistance of Counsel for his defenserdquo The right to counsel does not attach with respect to particular charges until formal adversarial judicial proceedings have commenced (ie ldquoat or after the initiation of

13

Criminal Law and Procedure Analysis

adversary judicial criminal proceedingsmdashwhether by way of formal charge preliminary hearing indictment information or arraignment [or in some states arrest warrant]rdquo McNeil v Wisconsin 501 US 171 175 (1991) (internal quotations omitted)) Once a suspectrsquos Sixth Amendment right to counsel has attached any attempts to ldquodeliberately elicitrdquo statements from him in the absence of his attorney violate the Sixth Amendment See Massiah v United States 377 US 201 (1964) Brewer v Williams 430 US 387 (1977)

The Sixth Amendment right to counsel is charge- or offense -specific Representation by counsel in one prosecution does not in itself guarantee counsel for uncharged offenses See McNeil 501 US at 175 Texas v Cobb 532 US 162 (2001) Here the suspectrsquos Sixth Amendment right to counsel had attached only for the pending aggravated assault charge The suspectrsquos right to counsel for the aggravated assault case did not guarantee counsel for the five unrelated and uncharged burglaries that were the subject of the detectiversquos interrogation Thus because formal adversarial judicial proceedings against the suspect for the uncharged burglaries had not begun he had no Sixth Amendment right to counsel

Finally the detectiversquos failure to inform the suspect of the lawyerrsquos presence and demands to speak with him does not implicate the suspectrsquos Sixth Amendment right to counsel which had not yet attached See id Moran v Burbine 475 US 412 428ndash31 (1986)

Point Two (30) The suspect did not effectively invoke his right to counsel under Miranda because his statement was not unambiguous

A suspect subject to custodial interrogation has a right to consult with counsel and to have an attorney present during questioning Miranda v Arizona 384 US 436 (1966) When a suspect invokes his right to counsel during an interrogation law enforcement must immediately cease all questioning See Edwards v Arizona 451 US 477 484ndash85 (1981) Custodial interrogation cannot be reinitiated unless and until the suspect has been re-advised of his Miranda rights has provided a knowing and voluntary waiver and (1) counsel is present and (2) the suspect himself initiated further communication with the police see id at 484 or (3) (if the suspect was released from custody after the initial interrogation) at least 14 days have passed Maryland v Shatzer 559 US 98 110 (2010)

To invoke the right to counsel a suspectrsquos request must be ldquounambiguousrdquo This means that the suspect must articulate the desire for counsel sufficiently clearly that a reasonable officer would understand the statement to be a request for counsel Davis v United States 512 US 452 459 (1994) If the request is ambiguous the police are not required to stop the interrogation

In this case the suspectrsquos statement ldquoI think I want my lawyer here before I talk to yourdquo was not an unambiguous request for counsel The most reasonable interpretation of this statement is that the suspect might be invoking his right to counsel Id at 461 (ldquomaybe I should talk to a lawyerrdquo is not an unequivocal request for counsel) See also Burket v Angelone 208 F3d 172 197ndash98 (4th Cir 2000) (ldquoI think I need a lawyerrdquo is not an unambiguous request for an attorney) Soffar v Cockrell 300 F3d 588 594ndash95 (5th Cir 2002) (discussion of various statements that did not constitute unequivocal requests for counsel)

Under these circumstances the detective was not required to cease the custodial interrogation of the suspect Nor was the detective required to clarify or ask follow-up questions to determine whether the suspect in fact wanted an attorney Davis 512 US at 459ndash60

14

Criminal Law and Procedure Analysis

Point Three (35) The suspectrsquos waiver of his Miranda rights was knowing intelligent and voluntary despite the fact that he was never told of the lawyerrsquos presence in the jail or of the lawyerrsquos demands

A valid waiver of Miranda rights must be ldquovoluntaryrdquomdashie the product of a free or deliberate choice rather than intimidation coercion or deception Berghuis v Thompkins 560 US 370 382ndash83 (2010) In addition the waiver must be knowing and intelligent That is it ldquomust have been made with a full awareness of both the nature of the right being abandoned and the consequences of the decision to abandon itrdquo Moran v Burbine 475 US 421 (1986)

In this case the suspect signed a Miranda waiver form after receiving proper warnings There is no evidence ldquothat the police resorted to physical or psychological pressure to elicit the statementsrdquo Id The entire interview lasted only 45 minutes The only issue is whether the suspect knowingly and intelligently waived his Miranda rights despite the fact that the detective did not tell the suspect about the lawyerrsquos presence and her demands

The Supreme Court has said that ldquo[e]vents occurring outside of the presence of the suspect and entirely unknown to him surely can have no bearing on the capacity to comprehend and knowingly relinquish a constitutional rightrdquo Id at 422 If the suspect ldquoknew that he could stand mute and request a lawyer and was aware of the Statersquos intention to use his statements to secure a convictionrdquo then the waiver is valid regardless of the information withheld Id at 422ndash23

Here the suspect was correctly informed of his rights Miranda v Arizona 384 US at 467ndash73 His comments demonstrate that he understood that he could have a lawyer present if he desired (ie wondering whether he should call his attorney) and that he understood that there might be consequences to speaking with the detective (ldquoI probably should keep my mouth shut but Irsquom willing to talk to you for a whilerdquo) His comment ldquo[L]etrsquos not waste any time waiting for someone to call my attorney and having her drive hererdquo along with his signature on the Miranda waiver form show that his waiver was valid under the constitutional standard

The fact that the detective did not tell the suspect about the lawyerrsquos presence and demands has no bearing on the validity of the suspectrsquos waiver because ldquosuch conduct is only relevant to the constitutional validity of a waiver if it deprives a defendant of knowledge essential to his ability to understand the nature of his rights and the consequences of abandoning themrdquo Moran at 424 The Supreme Court has specifically declined to adopt a rule requiring that law enforcement tell a suspect of an attorneyrsquos efforts to contact him id at 425 (ldquoNor are we prepared to adopt a rule requiring that the police inform a suspect of an attorneyrsquos efforts to reach himrdquo)

[NOTE An examinee might also recognize that this general rule is further supported by the Supreme Courtrsquos decision in Florida v Powell 559 US 50 (2010) approving state Miranda warnings that do not explicitly warn suspects that they have a right to have counsel present during custodial interrogation]

15

CONTRACTS ANALYSIS (Contracts IB2 IIB IVA3 amp A5)

ANALYSIS

Legal Problems

(1) In the case of a service contract (governed by the common law of contracts) is a modification enforceable when a party agrees to pay more for the same performance than was originally promised

(2) In the case of a contract for the sale of goods (governed by Article 2 of the UCC) is a modification enforceable when a party agrees to pay more for the same goods than was originally promised

(3) May a party avoid an agreement on the basis of economic duress

DISCUSSION

Summary

There are two arguments that the conservatory can make to support the claim that it is not bound to pay the higher prices lack of consideration and economic duress

The organ repair contract is governed by the common law of contracts Under the common law the business would have difficulty recovering the additional $60000 for the organ repair because under the ldquopreexisting duty rulerdquo the agreement of the conservatory to pay the extra price was not supported by consideration However the business might argue that the modification is enforceable under an exception to the preexisting duty rule for fair and equitable modifications made in light of unanticipated circumstances

The organ sale contract is governed by Article 2 of the Uniform Commercial Code The business would likely recover the additional amount under that contract because Article 2 provides that consideration is not required for a modification to be binding

In both cases the conservatory could seek to avoid its agreement on the grounds of economic duress but that argument is not likely to succeed

Point One (45) The business probably cannot recover the additional $60000 for the organ repair because the conservatoryrsquos promise to pay more money was not supported by consideration

The general rule is that to be enforceable a promise must be supported by consideration Under RESTATEMENT (SECOND) OF CONTRACTS sect 71 a promise is supported by consideration if it is bargained for in exchange for a return promise or performance However under the ldquopreexisting duty rulerdquo (exemplified in RESTATEMENT (SECOND) OF CONTRACTS sect 73 and Alaska Packersrsquo Assrsquon v Domenico 117 F 99 (9th Cir 1902)) promise of performance of a legal duty already owed to a promisor which is neither doubtful nor the subject of honest dispute is not consideration

If the business had promised the conservatory anything new or different in exchange for the agreement to pay the additional $60000 (such as for example repairing the pipe organ more

16

Contracts Analysis

quickly or using better parts) that would constitute consideration especially in light of the principle that courts do not inquire into the adequacy of consideration Here however the business already had a legal duty under the original contract and did not agree to do anything else in exchange for the conservatoryrsquos promise to pay $60000 more

However an exception to the preexisting duty rule is sometimes applied in situations of unanticipated changed circumstances Under RESTATEMENT (SECOND) OF CONTRACTS sect 89 followed in many jurisdictions a promise modifying a duty under a contract not fully performed on either side is binding even if not supported by consideration if the modification is fair and equitable in view of circumstances not anticipated by the parties when the contract was made

If a court applies the rule in Restatement sect 89 the critical issues will be whether the modification was in fact ldquofair and equitablerdquo and whether it can be justified in light of unanticipated circumstances In many cases in which modifications have been upheld a party encountered difficulties or burdens in performing far beyond what was knowingly bargained for in the original contract with the result bordering on impracticability such as having to excavate solid rock instead of soft dirt or having to remove garbage far in excess of the amounts contemplated The conservatory would argue that the businessrsquos performance difficulties were not of this sort at allmdashnothing about repairing the pipe organ itself was any different from or more difficult than originally contemplated except that the business itself encountered financial distress unrelated to its burdens in performing its obligations under these contracts

Even if the business satisfies that element of the rule in Restatement sect 89 the business must also demonstrate that the circumstances that gave rise to the need to modify the contract were ldquounanticipatedrdquo at the time the original contract was made Here the facts suggest that when the business entered into the original contract it expected that the price paid by the conservatory would enable it to perform However any evidence that the business knew or had reason to know at the time of execution that it would need more money from the conservatory to be able to perform would mean that the request to modify was not ldquounanticipatedrdquo

[NOTE Some cases such as Schwartzreich v Bauman-Basch Inc 231 NY 196 131 NE 887 (1921) find that if the parties mutually agreed to rescind the original contract and then after rescission entered into an entirely new contract for a higher price the new contract is supported by consideration There is no evidence that such a rescission followed by a new contract took place here]

Point Two (45) The business can recover the additional $40000 for the new organ because no consideration is required under Article 2 of the UCC for good-faith contract modifications

The contract to buy a new organ is a contract for the sale of goods and therefore is governed by Article 2 of the Uniform Commercial Code UCC sect 2-102 Under Article 2 unlike the common law an agreement modifying a contract needs no consideration to be binding UCC sect 2-209(1) Section 2-209(1) thus obviates the preexisting duty rule entirely in contracts for the sale of goods

Even though consideration is not required modifications governed by sect 2-209 must satisfy the obligation of good faith imposed by the UCC UCC sect 1-304 See also Official Comment 2 to UCC sect 2-209 Good faith means ldquohonesty in fact and the observance of reasonable commercial standards of fair dealingrdquo UCC sect 1-201(b)(20) In this context the obligation of good faith means that ldquo[t]he effective use of bad faith to escape performance on the original contract terms is barred and the extortion of a lsquomodificationrsquo without legitimate commercial reason is ineffective as a violation of the duty of good faithrdquo Official Comment 2 to

17

Contracts Analysis

UCC sect 2-209 Here because the businessrsquos financial reversals were serious and apparently unanticipated at the time that the business entered into the contract with the conservatory and commitment of the extra money was needed to enable the business to perform a court would likely find that the business acted in good faith Thus a court would likely uphold the enforceability of the conservatoryrsquos promise to pay the additional $40000

Point Three (10) The conservatory is unlikely to be able to defend against enforcement of its promises to pay additional money under the theory of economic duress because the business probably did not make an improper threat

Under the common law of contracts parties may raise the defense of duress This common law defense also applies to contracts governed by UCC Article 2 See UCC sect 1-103(b)

A contract is voidable on the ground of economic duress by threat when it is established that a partyrsquos manifestation of assent is induced by an improper threat that leaves the party no reasonable alternative See RESTATEMENT (SECOND) OF CONTRACTS sect 175 See also eg Austin Instrument Inc v Loral Corp 272 NE2d 533 (NY 1971) (a threat to withhold essential goods can constitute duress) In order to void its agreement to pay the additional sum because of economic duress the conservatory must demonstrate that (1) the business made a threat to the conservatory (2) the threat was ldquoimproperrdquo or ldquowrongfulrdquo (3) the threat induced the conservatoryrsquos manifestation of assent to the modification and (4) the threat was sufficiently grave to justify the conservatoryrsquos assent

Here it appears that three of the four elements are likely satisfied The business plainly made a threat Moreover the threat induced the conservatoryrsquos assent to the modification and the threat was sufficiently grave to justify that assent If the conservatory had not agreed to pay the business the extra amounts the conservatory would have lost its entire $325000 investment In light of this potential loss a court could easily conclude that the conservatory had no reasonable alternative

However the business has a strong argument that its threat (indicating that it would breach the contracts unless the prices were increased) was not wrongful or improper but was instead nothing more than a communication of the reality of its own perilous situation to the conservatory

A mere threat to breach a contract is not in and of itself improper so as to support an action of economic duress or business compulsion Something more is required such as a breach of the duty of good faith and fair dealing as was present in Austin Instrument Inc supra Because the business could not perform the original contract without the requested modification the economic duress claim for the conservatory would likely fail for much the same reason that the business would be able to enforce the modification At the time the modification was requested the business was not trying to extort a price increase because of the conservatoryrsquos vulnerability but instead was simply stating the reality that the business could not perform without more money

18

FAMILY LAW ANALYSIS (Family Law IIIB D amp G)

ANALYSIS

Legal Problems

(1)(a) Does the State A court have jurisdiction to modify the State B child support order

(1)(b) Does the State A court have jurisdiction to modify the marital-residence-saleshyproceeds provision of the State B property-division decree

(2)(a) May a child support order be modified retroactively

(2)(b) May a child support order be modified prospectively based on a change of employment with a lower salary

(2)(c) May a property-division order be modified after entry of a divorce decree

DISCUSSION

Summary

The State A court may exercise personal jurisdiction over the wife because she was personally served in State A However subject-matter jurisdiction over the interstate modification of child support is governed by the Uniform Interstate Family Support Act (UIFSA) Under UIFSA State A does not have jurisdiction to modify the order for the daughterrsquos support because the wife is still a resident of State B UIFSA on the other hand does not govern property distributions and thus a State A court is not precluded from hearing the husbandrsquos petition to modify the marital-residence-sale-proceeds provision of the divorce decree

A child support order may not be modified retroactively A child support order may be modified prospectively based on a substantial change in circumstances Courts agree that a significant decrease in income is a substantial change in circumstances All states treat voluntary income reductions differently than involuntary reductions but employ different approaches for evaluating the impact of a voluntary reduction Whether the husband could obtain prospective modification of the child support order depends on which approach is applied

A property-division order is not subject to post-divorce modification based on a change in circumstances Thus the husband may in some states obtain prospective modification of the order for the daughterrsquos support but he may not obtain modification of the marital-residenceshysale-proceeds provision

Point One(a) (25) Personal jurisdiction over a nonresident respondent does not confer subject-matter jurisdiction over child support modification Under UIFSA a State A court may not modify a child support order issued by a State B court when as here the child or either parent continues to reside in State B the jurisdiction that issued the child support order

The State A court may exercise personal jurisdiction over the wife The wife was personally served in State A and a state may exercise jurisdiction based on in-state personal service See

19

Family Law Analysis

Burnham v Superior Court 495 US 604 (1990) But personal jurisdiction over the wife is not enough to give a State A court jurisdiction to modify the State B support order

The interstate enforcement and modification of child support is governed by the Uniform Interstate Family Support Act (UIFSA) which has been adopted by all states Under UIFSA the state that originally issued a child support order (here State B) has continuing exclusive jurisdiction to modify the order if that state remains the residence of the obligee the child or the obligor and all parties do not consent to the jurisdiction of another forum See UIFSA sect 205 See also UIFSA sect 603 (ldquoA tribunal of this State shall recognize and enforce but may not modify a registered order if the issuing tribunal had jurisdictionrdquo) The wife and daughter continue to reside in State B and the wife has not consented to the jurisdiction of another forum Thus a State A court does not have jurisdiction to modify the State B child support order

[NOTE Examinees who do not discuss personal jurisdiction but fully discuss UIFSA may receive full credit]

Point One(b) (15) UIFSA does not apply to disputes over property division Thus the State A court may exercise jurisdiction over the husbandrsquos petition to modify the marital-residence-sale-proceeds provision of the State B divorce decree because it has personal jurisdiction over the wife

The State A court in which the husband brought his action has jurisdiction to adjudicate domestic relations issues The husbandrsquos petition to modify the property settlement is a domestic relations issue The courts of State A may exercise personal jurisdiction over the wife because she was personally served in State A See Burnham v Superior Court 495 US 604 (1990) see Point One(a)

UIFSA does not apply to divorce property-division disputes Thus although a State A court may not adjudicate the husbandrsquos petition to modify his child support obligations it may adjudicate his property-division claims (Even though the court has jurisdiction it may not modify the property-division award on the merits See Point Two(c))

Point Two(a) (20) A child support order may not be modified retroactively

State courts have long held that obligations to pay child support ordinarily may not be modified retroactively ldquoIf the hardship is particularly severe the courts sometimes devised a way to protect the obligor but in most instances the courts hold that retroactive modification of this kind is beyond their power and indeed the governing statute may so providerdquo HOMER H CLARK THE LAW OF DOMESTIC RELATIONSHIPS IN THE UNITED STATES 725 (2d ed 1987)

Federal law now goes further and requires the states as a condition of federal child-support funding to adopt rules that absolutely forbid retroactive modification of the support obligation See 42 USC sect 666(a)(9)(C) The states have adopted rules consistent with the federal requirements

Point Two(b) (25) It is unclear whether the husband could obtain prospective downward modification of his child support based on his voluntary acceptance of a job with a lower salary

Prospective modification of a child support order is typically available only when the petitioner can show a substantial change in circumstances See ROBERT E OLIPHANT amp NANCY VER

20

Family Law Analysis

STEEGH FAMILY LAW 213ndash15 (3d ed 2010) A significant decrease in income is typically viewed as a substantial change

However when a parent seeks to modify a child support obligation because he has voluntarily reduced his income a court will not modify the obligation based solely on the income loss Some courts refuse to modify whenever the income shift was voluntary See eg Aguiar v Aguiar 127 P3d 234 (Idaho Ct App 2005) Others look primarily to the petitionerrsquos intentions and permit downward modification if he has acted in good faith See eg In re Marriage of Horn 650 NE2d 1103 (Ill App Ct 1995) Many courts use a multifactor approach See OLIPHANT amp VER STEEGH supra 217ndash18

Here there is no question that the husbandrsquos loss of income was voluntary In a jurisdiction in which voluntary income reduction bars support modification the husbandrsquos petition would be denied

In a jurisdiction employing a good-faith or multifactor approach it is possible but not certain that the husband could obtain downward modification The evidence supports the husbandrsquos good faith his change in employment appears to be based on his new jobrsquos greater responsibilities and better promotion possibilities In a jurisdiction using a multifactor approach the court would likely also consider the impact of such a shift on the daughter the likely duration of the husbandrsquos income loss and the likelihood of a promotion that would ultimately inure to the daughterrsquos benefit Thus on these facts it is possible but by no means certain that the husband could prospectively obtain downward modification of his child support obligation to his daughter

Point Two(c)(15) A divorce property-division award is not subject to modification

A support order is aimed at meeting the post-divorce needs of the supported individual Because the future is unpredictable courts are empowered to modify a support award to take account of changed circumstances that may occur during the period in which support is paid

By contrast a property-distribution award divides assets of the marriage based on the equities at the time of divorce Because the past can be ascertained a property-division award is not subject to post-divorce modification See HARRY A KRAUSE ET AL FAMILY LAW CASES COMMENTS AND QUESTIONS 691 (6th ed 2007)

Here the husband is seeking modification of a property-division award with respect to an asset owned by the parties at the time of divorce Thus the husband may not obtain a modification of the marital-residence-sale-proceeds provision of the divorce decree based on his reduced income

21

FEDERAL CIVIL PROCEDURE ANALYSIS (Federal Civil Procedure III IVC)

ANALYSIS

Legal Problems

(1) Is the logging company entitled to join this action as a matter of right

(2)(a) May the nonprofit organization obtain a temporary restraining order to stop the USFS from issuing a logging permit

(2)(b) May the nonprofit organization obtain a preliminary injunction to stop the USFS from issuing a logging permit during the pendency of the action

DISCUSSION

Summary

The logging company is entitled to intervene in this action as a matter of right because it has an interest in the property or transaction that is the subject of the action and is so situated that its interest may be impaired or impeded as a practical matter if the action goes forward without it The logging companyrsquos interest is not adequately represented by the USFSrsquos presence in the lawsuit

The nonprofit organization may seek a temporary restraining order (TRO) followed by a preliminary injunction to prevent the USFS from issuing a logging permit pending the outcome of the action The nonprofit is likely to obtain a TRO if it can demonstrate a risk of immediate and irreparable injury The nonprofit is also likely to obtain a preliminary injunction if it can demonstrate a significant threat of irreparable harm and a likelihood of success on the merits of its National Environmental Policy Act (NEPA) claim

Point One (50) Rule 24(a) of the Federal Rules of Civil Procedure requires federal courts to allow a person to intervene in an action as a matter of right if the person a) is interested in the property or transaction that is the subject of the action b) is so situated that its interest may be impaired or impeded if the litigation goes forward without it and c) is not adequately represented by existing parties Here the logging company likely meets all three requirements and should be allowed to intervene as a matter of right

Rule 24 of the Federal Rules of Civil Procedure governs intervention the process by which a non-party to an action may join the litigation Under Rule 24(a) (intervention of right) a person must be permitted to intervene if three conditions are met (1) the movant ldquoclaims an interest relating to the property or transaction that is the subject of the actionrdquo (2) the movant ldquois so situated that disposition of the action may as a practical matter impair or impede the movantrsquos ability to protect its interestrdquo and (3) ldquoexisting partiesrdquo do not ldquoadequately represent [the movantrsquos] interestrdquo FED R CIV P 24(a) The three requirements for intervention of right are often ldquovery interrelatedrdquo 7C CHARLES ALAN WRIGHT ET AL FEDERAL PRACTICE AND PROCEDURE sect 1908 at 297 (2007 amp 2011 Supp)

22

Federal Civil Procedure Analysis

Here the court should find that the logging company meets this test First the logging company has a strong interest in the property or transaction that is the subject of this action The USFS has accepted the logging companyrsquos bid and the logging company is merely awaiting issuance of a logging permit to begin logging The nonprofit organization is seeking to prevent this logging The logging company therefore has a strong direct and substantial interest in the subject matter of the lawsuit and in having its winning bid honored and a logging permit issued See eg Kleissler v US Forest Serv 157 F3d 964 972 (3d Cir 1998) (stating that ldquo[t]imber companies have direct and substantial interests in a lawsuit aimed at halting loggingrdquo) see also Natural Resources Defense Council v US Nuclear Regulatory Commrsquon 578 F2d 1341 1343ndash 44 (10th Cir 1978) (holding that applicants whose license renewals were pending had Rule 24(a)(2) interests where the lawsuit sought to halt the license-issuing process pending preparation of environmental impact statements) See generally 7C WRIGHT ET AL supra sect 19081 at 309 (ldquoIf there is a direct substantial legally protectable interest in the proceedings it is clear that this requirement of the rule is satisfiedrdquo) Second the logging companyrsquos interest in receiving a logging permit may well be impaired as a practical matter by the outcome of the lawsuit If the USFS loses the lawsuit it will have to prepare an environmental impact statement before issuing the logging companyrsquos permit This will at a minimum delay the logging companyrsquos ability to exercise its rights and may in the long r un mean that no logging permit is ever issued Intervention of right is not limited to those that would be legally bound as a matter of preclusion doctrine Id sect 19082 at 368 Rather ldquo[t]he rule is satisfied whenever disposition of the present action would put the movant at a practical disadvantage in protecting its interestrdquo Id sect 19082 at 369 Here that condition is easily satisfied See Kleissler 157 F3d at 972 (ldquoTimber companies have direct and substantial interests in a lawsuit aimed at halting logging rdquo)

Given that the logging company has an interest that may be impaired by disposition of the action it should be allowed to intervene unless the court is persuaded that the USFS adequately represents the logging companyrsquos interest See Rule 24(a)(2) 7C WRIGHT ET AL supra sect 1909 Here it could be argued that the USFS adequately represents the logging companyrsquos interest because the USFS presumably wants the court to uphold its development plan and allow it to proceed with issuance of the logging permit which is the same relief that the logging company would seek However whether representation is truly adequate depends upon ldquo[a] discriminating appraisal of the circumstancesrdquo 7C WRIGHT ET AL supra sect 1909 at 440 Although both the government and the logging company wish to avoid the preparation of an environmental impact statement their interests are distinct The USFSrsquos interest is proper management of the national forest system while the logging companyrsquos interest is making a profit from logging the 5000-acre tract The USFSrsquos handling of the litigation is likely to be affected by a variety of policy concerns and political considerations that have nothing to do with the logging companyrsquos purely economic interest in securing the right to cut trees in the Scenic National Forest See eg Kleissler 157 F3d at 973ndash74 (ldquo[T]he government represents numerous complex and conflicting interests in matters of this nature The straightforward business interests asserted by intervenors here may become lost in the thicket of sometimes inconsistent governmental policiesrdquo)

[NOTES (1) Examinees who mistakenly analyze the logging companyrsquos case for joinder under the related but incorrect Rule 19 ldquoRequired Joinder of Partiesrdquo may receive credit Rule 19 allows existing parties to demand joinder of non-parties (or seek dismissal of the case if they canrsquot get it) There is a close relationship between Rule 24 and Rule 19 and both contain a similar standard for determining when ldquointerestedrdquo third parties are ldquoentitledrdquo or ldquorequiredrdquo to be in the lawsuit Indeed the two prongs of the Rule 24 intervention test that are discussed above

23

Federal Civil Procedure Analysis

are nearly identical to the two prongs of the Rule 19(a) required joinder test Examinees who discuss and apply the test should receive credit even if they cite Rule 19 rather than Rule 24

(2) Examinees may discuss permissive joinder Although permissive joinder is a possibility here the question asks only whether the logging company can join the action as a matter of right and a permissive joinder analysis is not responsive to the question To the extent an examinee discusses permissive joinder the analysis will focus on whether the logging company ldquohas a claim or defense that shares with the main action a common question of law or factrdquo FED R CIV P 24(b)(1)(B) The district court also ldquomust consider whether the intervention will unduly delay or prejudice the adjudication of the original partiesrsquo rightsrdquo FED R CIV P 24(b)(3) On our facts the logging companyrsquos claim for the issuance of a logging permit would certainly share common questions of law and fact with the USFSrsquos defense against the nonprofitrsquos claim There are no facts suggesting that the logging companyrsquos presence would unduly delay or otherwise prejudice adjudication of the original action Thus the district court would have discretion to permit the logging company to intervene even if it denied intervention of right]

Point Two(a) (25) The nonprofit organization could seek and would likely obtain a temporary restraining order to stop the USFS from issuing a logging permit pending a hearing on an application for a preliminary injunction

The first type of interim relief the nonprofit could seek to stop the USFS from issuing a logging permit to the logging company is a temporary restraining order (TRO) prohibiting the USFS from issuing the logging permit A TRO can be issued without notice to the adverse party but only in limited circumstances and only for a limited time FED R CIV P 65(b) To secure a TRO without notice the nonprofit would need to submit an affidavit containing specific facts that demonstrate a risk of ldquoimmediate and irreparable injuryrdquo if a permit is issued FED R CIV P 65(b)(1) In deciding whether to grant a TRO courts will also consider the same factors that are relevant in deciding whether to grant a preliminary injunction (eg the moving partyrsquos likelihood of success on the merits the balance of hardships and the public interest) See Point Two(b) infra The TRO would last only long enough for the court to consider and resolve a request by the nonprofit for a preliminary injunction but no longer than 14 days (unless the court extends it for good cause or the adverse party consents to an extension) In addition bond is required

Here the court is likely to grant the nonprofitrsquos request The nonprofit could plausibly claim that cutting down 5000 acres of old-growth forest in an area that is home to the highest concentration of wildlife in the western United States would have ldquoan immediate and irreparablerdquo adverse impact on the environment and cause irreparable harm to the nonprofitrsquos interest in preserving and protecting natural resources including wildlife habitat

Point Two(b) (25) The nonprofit could also seek and would likely obtain a preliminary injunction to stop the USFS which is likely to be granted if the nonprofitrsquos claim that the USFS violated NEPA has a strong basis in fact and law

Because the TRO would be temporary the nonprofit would need to move for a preliminary injunction to prevent the USFS from issuing a logging permit throughout the pendency of the litigation Preliminary injunctions are injunctions that seek to ldquoprotect [the] plaintiff from

24

Federal Civil Procedure Analysis

irreparable injury and to preserve the courtrsquos power to render a meaningful decision after a trial on the meritsrdquo 11A CHARLES ALAN WRIGHT ET AL FEDERAL PRACTICE AND PROCEDURE sect 2947 at 112 (2013) Rule 65 of the Federal Rules of Civil Procedure sets out the procedural requirements for preliminary injunctions Preliminary injunctions may be granted only upon notice to the adverse party FED R CIV P 65(a)(1) and only if the movant ldquogives security in an amount that the court considers proper to pay the costs and damages sustained by any party found to have been wrongfully enjoined or restrainedrdquo FED R CIV P 65(c)

While Rule 65 sets out the procedural requirements for preliminary injunctive relief it does not specify the substantive grounds upon which it may be granted The courtrsquos discretion in ruling upon a motion for a preliminary injunction ldquois exercised in conformity with historic federal equity practicerdquo 11A WRIGHT ET AL supra sect 2947 at 114 The court typically considers four factors

(1) the significance of the threat of irreparable harm to the plaintiff if the injunction is not granted (2) the balance between this harm and the injury that granting the injunction would inflict on the defendant (3) the probability that the plaintiff will succeed on the merits and (4) the public interest

Id sect 2948 at 122ndash24 accord Habitat Educ Center v Bosworth 363 F Supp 2d 1070 1088 (ED Wis 2005) The most important of these factors is the risk of irreparable harm to the plaintiff 11A WRIGHT ET AL supra sect 29481 at 129 If the plaintiff has an adequate remedy at law (eg if money damages can compensate the plaintiff for its loss) then a preliminary injunction will be denied Id sect 29481

Here a court would likely conclude that the potential for environmental damage to the forest creates a significant threat of irreparable harm ldquo[E]nvironmental injury is often irreparable Courts have recognized that logging such as would occur [here] can have longshyterm environmental consequences and thus satisfy the irreparable injury criterionrdquo Habitat Educ Center 363 F Supp 2d at 1089 (citing Idaho Sporting Congress Inc v Alexander 222 F3d 562 569 (9th Cir 2000) (noting that the imminent and continuing logging activities presented ldquoevidence of environmental harm sufficient to tip the balance in favor of injunctive reliefrdquo)) Neighbors of Cuddy Mountain v US Forest Service 137 F3d 1372 1382 (9th Cir 1998) (stating that ldquo[t]he old growth forests plaintiffs seek to protect would if cut take hundreds of years to reproducerdquo) (internal citation omitted)) see also 11C WRIGHT ET AL supra sect 29481 at 151 (noting that ldquoa preliminary injunction has been issued to prevent harm to the environmentrdquo)

The second factor the balance between the harm to the plaintiff and the harm the defendant will suffer if the injunction is issued also appears to support issuance of a preliminary injunction here The USFS will have to wait before it can develop the Scenic National Forest and the logging company may lose money if the delay is prolonged These economic harms could be compensated monetarily if an injunction is issued inappropriately Where ldquoan injunction bond can compensate [the] defendant for any harm the injunction is likely to inflict the balance should be struck in favor of [the] plaintiffrdquo Id sect 29482 at 192 See also Habitat Educ Center 363 F Supp 2d at 1089 (stating that ldquothe relative absence of harmful effects on the Forest Service weighs in favor of granting the injunctionrdquo)

The third factor is the likelihood that the plaintiff will prevail on the merits Although there is limited information concerning the merits of the action the nonprofit alleges that the federal statute (NEPA) requires an environmental impact statement and further states that the USFS created no environmental impact analysis or statement at all Assuming that those

25

Federal Civil Procedure Analysis

allegations are correct it seems plausible to conclude that the nonprofit will be able to show a likelihood of success on the merits

Finally courts deciding whether or not to issue preliminary injunctive relief are to consider the public interest ldquoFocusing on this factor is another way of inquiring whether there are policy considerations that bear on whether the order should issuerdquo 11C WRIGHT ET AL supra sect 29484 at 214 If the court concludes that the nonprofit is likely to succeed on its NEPA claim because the USFS wrongfully failed to conduct an environmental impact assessment it is likely to find that the public interest would be served by restraining the USFS from proceeding with logging in a national forest See Heartwood Inc v US Forest Service 73 F Supp 2d 962 979 (SD Ill 1999) affrsquod on other grounds 230 F3d 947 (7th Cir 2000) (ldquoviolations by federal agencies of NEPArsquos provisions as established by Congress harm the public as well as the environmentrdquo)

Thus a court is very likely to grant a preliminary injunction if it concludes that the nonprofit has a significant likelihood of success on the merits

26

EVIDENCE ANALYSIS (Evidence ID IIA amp C)

ANALYSIS

Legal Problems

(1) Under what circumstances can evidence of prior convictions be used to impeach a witnessrsquos credibility in a civil case

(1)(a) May the inmatersquos credibility be impeached by evidence of a 12-year-old felony drug conviction if he was released from prison 9 years ago

(1)(b) May the inmatersquos credibility be impeached by evidence of an 8-year-old misdemeanor perjury conviction that was punishable by 1 year in jail if he pleaded guilty and was sentenced only to pay a $5000 fine

(1)(c) May the inmatersquos credibility be impeached by evidence of a 7-year-old sexual assault conviction if the inmate is still serving a 10-year prison sentence and the victim was his 13-year-old daughter

(2)(a) May the guardrsquos credibility be impeached by cross-examination regarding specific instances of misconduct (ie lying on his reacutesumeacute) relevant to credibility

(2)(b) May the guardrsquos credibility be impeached by admission of extrinsic evidence (his reacutesumeacute and academic transcript) offered to prove specific instances of misconduct relevant to credibility

DISCUSSION

Summary

Under the Federal Rules of Evidence witnesses can be impeached with evidence of prior convictions andor specific instances of misconduct Whether evidence of prior convictions should be admitted to impeach generally depends on the nature of the crime the amount of time that has passed and (only in criminal cases) whether the ldquowitnessrdquo is the defendant FED R EVID 609(a)

In this civil case evidence of the inmatersquos conviction for distribution of marijuana should be admitted to impeach the inmate because he was convicted of a felony and was released from prison fewer than 10 years ago FED R EVID 609(a)(1) Credibility is critically important in this case because the jury will hear conflicting testimony from the two disputing parties and there were no other eyewitnesses to the altercation Under Rule 609(a)(1) the inmatersquos conviction should be admitted because it has some bearing on his credibility and its probative value is not substantially outweighed by concerns of unfair prejudice confusion or delay Id

Evidence of the inmatersquos misdemeanor conviction for perjury must be admitted because the crime ldquorequired provingmdashor the witnessrsquos admittingmdasha dishonest act or false statementrdquo by the inmate FED R EVID 609(a)(2)

27

Evidence Analysis

Evidence of the inmatersquos felony conviction for sexual assault should be excluded because its probative value is substantially outweighed by the danger of unfair prejudice to the inmate based on the heinous nature of the crime FED R EVID 609(a)(1) In the alternative the judge could limit the evidence relating to this conviction by excluding details of the inmatersquos crime

In all civil (and criminal) cases witnesses can also be impeached with evidence of specific instances of prior misconduct that did not result in a conviction FED R EVID 608(b) Pursuant to Rule 608(b) misconduct probative of untruthfulness can be inquired into on cross-examination but cannot be proved through extrinsic evidence Id Thus the inmatersquos counsel should be permitted to cross-examine the guard regarding the false statement in the guardrsquos reacutesumeacute However extrinsic evidence of the guardrsquos misconduct (ie the guardrsquos authenticated reacutesumeacute and transcript from the local college) should not be admitted even if the guard denies wrongdoing or refuses to answer cross-examination questions about these matters Id

Point One (10) The Federal Rules of Evidence permit impeachment of witnesses with evidence of prior convictions

Whether convictions should be admitted to impeach generally depends on the nature of the crime the amount of time that has passed and (only in criminal cases) whether the ldquowitnessrdquo is the defendant FED R EVID 609(a) Under Rule 609(a) evidence of prior convictions may be admitted for the purpose of ldquoattacking a witnessrsquos character for truthfulnessrdquo Id

There are two basic types of convictions that can be admitted for the purpose of impeachment

(1) convictions for crimes ldquopunishable by death or by imprisonment for more than one yearrdquo (which generally correlates to ldquofeloniesrdquo) FED R EVID 609(a)(1) and (2) convictions ldquofor any crimes regardless of the punishment if the court can readily determine that establishing the elements of the crime required provingmdashor the witnessrsquos admittingmdasha dishonest act or false statementrdquo FED R EVID 609(a)(2)

Pursuant to Rule 609(a)(1) in civil cases the admission of evidence of a felony conviction is ldquosubject to Rule 403 [which says that a court may exclude relevant evidence if its probative value is substantially outweighed by other factors]rdquo FED R EVID 609(a)(1) However Rule 403 does not protect the witness against admission of prior convictions involving dishonestymdashwhich must be admitted by the court FED R EVID 609(a)(2)

Finally Federal Rule of Evidence 609(b) contains the presumption that a conviction that is more than 10 years old or where more than 10 years has passed since the witnessrsquos release from confinement (whichever is later) should not be admitted unless ldquoits probative value supported by specific facts and circumstances substantially outweighs its prejudicial effectrdquo and the proponent has provided the adverse party with reasonable written notice FED R EVID 609(b)

Point One(a) (25) The court should admit evidence of the inmatersquos 12-year-old felony marijuana distribution conviction

The inmatersquos conviction for marijuana distribution was for a felony punishable by imprisonment for more than one year See FED R EVID 609(a)(1) Moreover although the conviction was 12 years ago the 10-year time limit of Rule 609(b) is not exceeded because that time limit runs

28

Evidence Analysis

from the date of either ldquothe witnessrsquos conviction or release from confinement for it whichever is laterrdquo FED R EVID 609(b) Because the inmate served three years in prison he was released from confinement nine years ago

However pursuant to Rule 609(a)(1) the admission of felony convictions to impeach a witness in a civil case is ldquosubject to Rule 403rdquo FED R EVID 609(a)(1) Neither Rule 609(a) nor the advisory committee notes specify which factors courts should consider when balancing the probative value of a conviction against the dangers identified in Rule 403 (which include (1) unfair prejudice (2) confusion of the issues (3) misleading the jury (4) waste of time or undue delay and (5) needless presentation of cumulative evidence) FED R EVID 403

In this case credibility is very important because the evidence consists primarily of the testimony of the disputing parties and there were no other eyewitnesses to the altercation This enhances the probative value of any evidence bearing on the inmatersquos credibility A court is likely to conclude that the inmatersquos prior felony drug conviction is relevant to his credibility See eg United States v Brito 427 F3d 53 64 (1st Cir 2005) (ldquoPrior drug-trafficking crimes are generally viewed as having some bearing on veracityrdquo) Although the probative value of any conviction diminishes with age see eg United States v Brewer 451 F Supp 50 53 (ED Tenn 1978) the inmatersquos ongoing problems with the law suggest that he has continued (and even escalated) his criminal behavior over the past nine years The court should admit this evidence because its probative value is not substantially outweighed by any Rule 403 concerns Specifically any prejudice to the inmate would be slight because the conviction is unrelated to the altercation at issue and the conviction was not for a heinous crime that might inflame the jury

[NOTE Whether an examinee identifies the jury instruction as containing a ldquoconclusiverdquo or ldquomandatoryrdquo presumption is less important than the examineersquos analysis of the constitutional infirmities]

Point One(b) (15) The court must admit evidence of the inmatersquos eight-year-old misdemeanor conviction because perjury is a crime of dishonesty

Rule 609(a)(2) provides that evidence of a criminal conviction ldquomust be admitted if the court can readily determine that establishing the elements of the crime required provingmdashor the witnessrsquos admittingmdasha dishonest act or false statementrdquo FED R EVID 609(a)(2) The inmatersquos conviction for perjury would have necessarily required proving that the inmate engaged in an act of dishonesty This conviction occurred within the past 10 years so it ldquomust be admittedrdquo because in contrast to Rule 609(a)(1) (discussed in Point One(a)) admission under Rule 609(a)(2) is mandatory and not subject to Rule 403

Point One(c) (20) The court should exclude evidence of the inmatersquos seven-year-old felony sexual assault conviction because the probative value of this evidence is substantially outweighed by the danger of unfair prejudice In the alternative the details of the prior conviction could be excluded

The inmatersquos conviction for felony sexual assault was seven years ago and he has not yet been released from incarceration so Rule 609(a) but not 609(b) is applicable here FED R EVID 609(a) This conviction is therefore admissible to impeach the inmate unless its probative value is substantially outweighed by the danger of unfair prejudice or any other Rule 403 concern Id

29

Evidence Analysis

Sex crimes are generally not considered relevant to credibility see Hopkins v State 639 So 2d 1247 1254 (Miss 1993) so the probative value of this conviction is relatively low Moreover the heinous nature of the inmatersquos crime (sexual assault on his daughter) makes the danger of unfair prejudice to the inmate very high Thus the court should exclude evidence of the conviction because it was for a heinous offense that is likely to inflame the jury and it has little bearing on credibility See eg United States v Beahm 664 F2d 414 419 (4th Cir 1981)

As an alternative to excluding this evidence the judge could minimize the unfair prejudice to the inmate by permitting limited cross-examination but refusing to allow specific questions about the nature of the inmatersquos conviction For example a court could limit cross-examination to the fact that the inmate was convicted of a ldquofelonyrdquo or perhaps that he was convicted of a ldquosexual assaultrdquo without identifying the victim However because evidence of the inmatersquos prior convictions can be admitted solely for the purpose of enabling the jury to assess his credibility and because his two earlier convictions should have already been admitted the court should exclude all evidence of the felony sexual assault conviction

Point Two(a) (15) The court should permit the inmatersquos counsel to cross-examine the guard regarding the false statement in his reacutesumeacute because the guardrsquos misconduct bears on his truthfulness

The inmate wishes to cross-examine the guard about his prior dishonest behaviormdashlying on his reacutesumeacutemdashthat did not involve a criminal conviction Rule 608(b) allows witnesses to be cross-examined about specific instances of prior non-conviction misconduct probative of untruthfulness ldquoin order to attack the witnessrsquos character for truthfulnessrdquo FED R EVID 608(b)

The courtrsquos decision to allow cross-examination about the guardrsquos prior dishonest behavior depends on the probative value of such evidence balanced against the danger of unfair prejudice to the guard or any other Rule 403 concern FED R EVID 403 Here the guardrsquos false statement on his reacutesumeacute that he obtained a degree in Criminal Justice is highly probative of his untruthfulness because it grossly misrepresents his actual academic record was made recently and was made with the intent to deceive Because the probative value of this evidence is very strong and is not substantially outweighed by any Rule 403 concerns cross-examination of the guard on this topic should be permitted The court may also consider it fair to permit this cross-examination of the guard on these matters assuming that one or more of the inmatersquos prior convictions have been admitted to impeach his credibility

Point Two(b) (15) The court should exclude extrinsic evidence of the guardrsquos non-conviction misconduct even if the guard denies wrongdoing or refuses to answer questions about the matter

Although Rule 608(b) allows cross-examination about specific instances of prior misconduct probative of untruthfulness ldquoextrinsic evidencerdquo offered to prove such misconduct is not admissible FED R EVID 608(b) The rationale for this rule is that allowing the introduction of extrinsic evidence of prior misconduct by witnesses when these acts are relevant only to the witnessesrsquo truthfulness and not to the main issues in the case would create too great a risk of confusing the jury and unduly delaying the trial The court does not have discretion to admit this extrinsic evidence See eg United States v Elliot 89 F3d 1360 1368 (8th Cir 1996)

30

Evidence Analysis

Here the inmatersquos counsel may cross-examine the guard about the false statement on his reacutesumeacute However the inmatersquos counsel must accept the guardrsquos response Even if the guard denies wrongdoing or refuses to answer questions about the matter the inmatersquos counsel cannot introduce the guardrsquos reacutesumeacute or the transcript from the local college to prove the guardrsquos misconduct

31

CORPORATIONS ANALYSIS (Corporations VA2 IX)

ANALYSIS

Legal Problems

(1) Do shareholders have the authority to amend a corporationrsquos bylaws with respect to director nominations

(2) Do board-approved bylaws on a particular subject here nomination of directors preempt subsequent conflicting bylaw amendments by shareholders

(3) Is a suit challenging both managementrsquos refusal to include the proposed bylaw amendment in Megarsquos proxy statement and the boardrsquos amendment of the bylaws dealing with nomination of directors a direct or derivative suit

DISCUSSION

Summary

The voting and litigation rights of the shareholders of Mega are subject to the provisions of the Model Business Corporations Act (MBCA)

The investorrsquos proposed bylaw provision is not inconsistent with state law Under the MBCA shareholders may amend the bylaws when the amendment deals with a proper matter for the corporationrsquos bylaws such as procedures for nominating directors

The Mega boardrsquos bylaw amendment does not preempt the investorrsquos proposed bylaw provision or the Mega shareholdersrsquo power to approve it While shareholders can limit the boardrsquos power to amend or repeal the bylaws the board cannot limit the shareholdersrsquo power

Whether the investor must make a demand on Megarsquos board depends on how the investor frames its claim If the investor claims a violation of shareholder voting rights the claim is direct and pre-suit demand on the board is not required If on the other hand the investor claims that the directors violated their fiduciary duties by amending the bylaws to entrench themselves the claim is derivative and a pre-suit demand is required

Point One (30) Shareholders may amend the corporationrsquos bylaws where the proposed bylaw provision relates to procedural matters typically included in the bylaws such as the nomination of directors

Internal affairs of the corporation such as the conduct of shareholder meetings and election of directors are subject to the corporate law of the state of incorporation See McDermott Inc v Lewis 531 A2d 206 (Del 1987) (applying law of jurisdiction where corporation was incorporated in case involving voting rights) This statersquos corporate statute is modeled on the MBCA

Under the MBCA ldquoshareholders may amend the corporationrsquos bylawsrdquo MBCA sect 1020(a) Thus the only question is whether the bylaws can specify the procedures for shareholder nomination of directors

32

Corporations Analysis

The MBCA states that the bylaws ldquomay contain any provision that is not inconsistent with law or the articles of incorporationrdquo MBCA sect 206(b) In addition the MBCA was revised in 2009 to address shareholder nomination of directors in public corporations (known as ldquoproxy accessrdquo) and specifies that the bylaws ldquomay contain a requirement that the corporation include in its [proxy materials] one or more individuals nominated by a shareholderrdquo MBCA sect 206(c)(1) see Committee on Corporate Laws ABA Section of Business Law Report on the Roles of Boards of Directors and Shareholders of Publicly Owned Corporations and Changes to the Model Business Corporations ActmdashAdoption of Shareholder Proxy Access Amendments to Chapters 2 and 10 65 BUS LAWYER 1105 (2010)

The inclusion of director-nomination procedures in the bylaws is consistent with practice and is recognized by the Delaware courts whose views on corporate law carry significant weight Typically the procedures for nomination of directors are found in the bylaws See 1 COX amp HAZEN TREATISE ON THE LAW OF CORPORATIONS sect 312 (3d ed 2011) see also 4 FLETCHER CORP FORMS ANN PART III ch 21 (2013) (including sample bylaws that permit nomination of directors by shareholders) The Delaware Supreme Court has confirmed that the bylaws may ldquodefine the process and proceduresrdquo for director elections See CA Inc v AFSCME Employees Pension Plan 953 A2d 227 (Del 2008) (concluding that bylaw amendment requiring reimbursement of election expenses to certain successful shareholder nominators is ldquoproper subjectrdquo under Delaware law)

[NOTE The question of the proper scope of the bylaws can be answered using the more general MBCA sect 206(b) or the 2009 MBCA revision adding sect 206(c)(1) (adopted in CT ME VA) In addition some examinees might raise the point that shareholder proposals may not compel the board to take action such as by including shareholder nominations in the companyrsquos proxy materials on the theory that the ldquobusiness and affairsrdquo of the corporation are to be managed by the board See MBCA sect 801(b) Although shareholders are generally limited to adopting precatory resolutions that recommend or encourage board action this limitation does not apply when shareholders have specific authority to take binding action on their ownmdashsuch as to amend the bylaws]

Point Two (30) Shareholders can amend (or repeal) board-approved bylaws Further shareholders can limit the boardrsquos power to later amend and repeal a shareholder-approved bylaw

Under the MBCA shareholders have the power to amend the bylaws See Point One The board shares this power with the shareholders unless (1) the corporationrsquos articles ldquoreserve that power exclusively to the shareholdersrdquo or (2) ldquothe shareholders in amending repealing or adopting a bylaw expressly provide that the board of directors may not amend repeal or reinstate that bylawrdquo See MBCA sect 1020(b)

Shareholder-approved bylaw provisions can amend or repeal existing bylaw provisions whether originally approved by the board or by shareholders See ALAN R PALMITER CORPORATIONS EXAMPLES AND EXPLANATIONS sect 713 (7th ed 2012) Thus the Mega boardrsquos bylaw amendmentmdashwhich set more demanding thresholds for shareholder nomination of directors than the investorrsquos proposed bylaw provisionmdashwould be superseded (repealed) if Megarsquos shareholders were to approve the investorrsquos proposal

Further a shareholder-approved bylaw generally can limit the power of the board to later amend or repeal it See MBCA sect 1020(b)(2) Thus if Megarsquos shareholders approved the bylaw

33

Corporations Analysis

provision proposed by the investor Megarsquos board could not repeal the provision because it includes a ldquono board repealrdquo clause

The revision to the MBCA in 2009 dealing with shareholder proxy access does not change this conclusion That revision specifies that a shareholder-approved bylaw dealing with director nominations may not limit the boardrsquos power to amend add or repeal ldquoany procedure or condition to such a bylaw in order to provide for a reasonable practicable and orderly processrdquo MBCA sect 206(d) Thus according to the revision if shareholders approve a bylaw amendment that limits further board changes the board would nonetheless retain the power to ldquotinkerrdquo with the bylaw to safeguard the voting process but could not repeal the shareholder-approved bylaw The Official Comment to MBCA sect 206(d) makes clear that the revision is ldquonot intended to allow the board of directors to frustrate the purpose of the shareholder-adopted proxy access provisionrdquo Thus if Megarsquos shareholders were to approve the bylaw provision proposed by the investor Megarsquos board could only amend the provision regarding its procedures or conditions in a manner consistent with its purpose of permitting proxy access for Megarsquos shareholders

[NOTE The boardrsquos attempted interference with a shareholder voting initiative may also have been a violation of the directorsrsquo fiduciary duties See Blasius Indus Inc v Atlas Corp 564 A2d 651 (Del Ch 1988) (finding that directors breached their fiduciary duties by amending bylaws and expanding size of board to thwart insurgentrsquos plan to amend bylaws and seat a majority of new directors) The call however asks examinees to consider whether shareholders or the board have ldquoprecedencerdquo over amending the corporate bylaws Thus an examineersquos answer should be framed in terms of ldquopowerrdquo and not ldquodutyrdquo]

Point Three (40) The investor need not make a demand on the board if the investor states a direct claim such as an allegation that the board interfered with the investorrsquos right to amend the bylaws But the investor must make a demand on the board if the investor states a derivative claim (on behalf of the corporation) such as an allegation that the directors sought to entrench themselves by interfering with the proposed proxy access

The MBCA generally requires that shareholders make a demand on the board of directors before initiation of a derivative suit MBCA sect 742 (shareholder may not bring derivative proceeding until written demand has been made on corporation and 90 days have expired) A derivative suit is essentially two suits in one where the plaintiff-shareholder seeks to bring on behalf of the corporation a claim that vindicates corporate rights usually based on violation of fiduciary duties PALMITER supra sect 1811 (6th ed 2009) The demand permits the board to investigate the situation identified by the shareholder and take suitable action No demand on the board is required however if the shareholder brings a direct suit to vindicate the shareholderrsquos own rights not those of the corporation

Is the suit brought by the investor derivative or direct The MBCA defines a ldquoderivative proceedingrdquo as one brought ldquoin the right of a domestic corporationrdquo MBCA sect 740(1) Thus the answer to how the investorrsquos suit should be characterized turns on what rights the investor seeks to vindicate If the investor frames its claim as one of fiduciary breach by directorsmdashfor example for failing to become adequately informed about voting procedures or for seeking to entrench themselves in office by manipulating the voting structure to avoid a shareholder insurgencymdashthen the suit is ldquoderivativerdquo and the investor must make a demand on the board See MBCA Ch 7 Subch D Introductory Comment (ldquothe derivative suit has historically been the principal method of challenging allegedly illegal action by managementrdquo)

34

Corporations Analysis

If however the investor frames its claim as one to vindicate shareholder rights the suit is direct and no demand is required For many courts the direct-derivative question turns on who is injured and who is to receive the relief sought by the plaintiff-shareholders See Tooley v Donaldson Lufkin amp Jenrette Inc 845 A2d 1031 (Del 2004) (characterizing a merger-delay claim as direct because delay of merger only harmed shareholders not corporation) Thus if the investor claims that managementrsquos refusal to include its proposed bylaw amendment in the corporationrsquos proxy materials violates its shareholder rights to initiate corporate governance reforms the suit will be direct Courts have not questioned the ability of shareholders to bring direct suits challenging board action to exclude their proposed bylaw amendments from the corporationrsquos proxy materials See JANA Master Fund Ltd v CNET Networks Inc 954 A2d 335 (Del Ch 2008) (upholding shareholderrsquos direct challenge to boardrsquos interpretation of advance-notice bylaw) Chesapeake Corp v Shore 771 A2d 293 (Del Ch 2000) (upholding shareholderrsquos direct challenge to actions by board that effectively prevented it from proposing bylaw amendments in contest for control)

Is the way that the investor frames its claim conclusive Courts have permitted shareholder-plaintiffs to challenge a transaction in a direct suit even though the same transaction could also be challenged as a fiduciary breach See Eisenberg v Flying Tiger Line Inc 451 F2d 267 (2d Cir 1971) (permitting direct suit challenging a corporate reorganization as a dilution of shareholder voting power even though reorganization may have involved conflicts of interest and thus constituted a fiduciary breach) Thus the investorrsquos choice to pursue a claim challenging the legality of managementrsquos decision to exclude the investorrsquos proposal from the corporationrsquos proxy materialsmdashrather than a possible breach of fiduciary dutymdashis likely to be respected See 3 COX amp HAZEN supra sect 153 (describing situations in which a claim can be framed as derivative or direct)

[NOTE Some issues under Delaware corporate law regarding pre-suit demand are not relevant here For example whether the Mega directors are independent and disinterested is not relevant to the MBCA requirement of a pre-suit demand As the Official Comment to MBCA sect 742 points out the MBCArsquos requirement of ldquouniversal demandrdquo gives the board ldquothe opportunity to reexamine the act complained of in the light of a potential lawsuit and take corrective actionrdquo even when the directors might be non-independent or have conflicts of interest

Nor is it relevant to the MBCA pre-suit demand requirement that the statutory 90-day waiting period may be onerous The first paragraph of MBCA sect 742 requires a pre-suit demand without exception the second paragraph of the section imposes a 90-day waiting period before a derivative suit may be brought which can be shortened if the board rejects the demand or ldquoirreparable injury to the corporation would result by waiting for the expiration of the 90-day periodrdquo The call as written asks only whether a pre-suit demand should be made and does not ask examinees to address whether the post-demand waiting period should be shortened under the ldquoirreparable injuryrdquo standard]

35

National Conference of Bar Examiners 302 South Bedford Street | Madison WI 53703-3622 Phone 608-280-8550 | Fax 608-280-8552 | TDD 608-661-1275

wwwncbexorg e-mail contactncbexorg

  • Preface
  • Description of the MEE
  • Instructions
  • July 2014 Questions
    • CRIMINAL LAW AND PROCEDURE QUESTION
    • CONTRACTS QUESTION
    • FAMILY LAW QUESTION
    • FEDERAL CIVIL PROCEDURE QUESTION
    • EVIDENCE QUESTION
    • CORPORATIONS QUESTION
      • July 2014 Analyses
        • CRIMINAL LAW AND PROCEDURE ANALYSIS
        • CONTRACTS ANALYSIS
        • FAMILY LAW ANALYSIS
        • FEDERAL CIVIL PROCEDURE ANALYSIS
        • EVIDENCE ANALYSIS
        • CORPORATIONS ANALYSIS
            • ltlt13 ASCII85EncodePages false13 AllowTransparency false13 AutoPositionEPSFiles true13 AutoRotatePages None13 Binding Left13 CalGrayProfile (Dot Gain 20)13 CalRGBProfile (sRGB IEC61966-21)13 CalCMYKProfile (US Web Coated 050SWOP051 v2)13 sRGBProfile (sRGB IEC61966-21)13 CannotEmbedFontPolicy Error13 CompatibilityLevel 1413 CompressObjects Tags13 CompressPages true13 ConvertImagesToIndexed true13 PassThroughJPEGImages true13 CreateJobTicket false13 DefaultRenderingIntent Default13 DetectBlends true13 DetectCurves 0000013 ColorConversionStrategy CMYK13 DoThumbnails false13 EmbedAllFonts true13 EmbedOpenType false13 ParseICCProfilesInComments true13 EmbedJobOptions true13 DSCReportingLevel 013 EmitDSCWarnings false13 EndPage -113 ImageMemory 104857613 LockDistillerParams false13 MaxSubsetPct 10013 Optimize true13 OPM 113 ParseDSCComments true13 ParseDSCCommentsForDocInfo true13 PreserveCopyPage true13 PreserveDICMYKValues true13 PreserveEPSInfo true13 PreserveFlatness true13 PreserveHalftoneInfo false13 PreserveOPIComments true13 PreserveOverprintSettings true13 StartPage 113 SubsetFonts true13 TransferFunctionInfo Apply13 UCRandBGInfo Preserve13 UsePrologue false13 ColorSettingsFile ()13 AlwaysEmbed [ true13 ]13 NeverEmbed [ true13 ]13 AntiAliasColorImages false13 CropColorImages true13 ColorImageMinResolution 30013 ColorImageMinResolutionPolicy OK13 DownsampleColorImages true13 ColorImageDownsampleType Bicubic13 ColorImageResolution 30013 ColorImageDepth -113 ColorImageMinDownsampleDepth 113 ColorImageDownsampleThreshold 15000013 EncodeColorImages true13 ColorImageFilter DCTEncode13 AutoFilterColorImages true13 ColorImageAutoFilterStrategy JPEG13 ColorACSImageDict ltlt13 QFactor 01513 HSamples [1 1 1 1] VSamples [1 1 1 1]13 gtgt13 ColorImageDict ltlt13 QFactor 01513 HSamples [1 1 1 1] VSamples [1 1 1 1]13 gtgt13 JPEG2000ColorACSImageDict ltlt13 TileWidth 25613 TileHeight 25613 Quality 3013 gtgt13 JPEG2000ColorImageDict ltlt13 TileWidth 25613 TileHeight 25613 Quality 3013 gtgt13 AntiAliasGrayImages false13 CropGrayImages true13 GrayImageMinResolution 30013 GrayImageMinResolutionPolicy OK13 DownsampleGrayImages true13 GrayImageDownsampleType Bicubic13 GrayImageResolution 30013 GrayImageDepth -113 GrayImageMinDownsampleDepth 213 GrayImageDownsampleThreshold 15000013 EncodeGrayImages true13 GrayImageFilter DCTEncode13 AutoFilterGrayImages true13 GrayImageAutoFilterStrategy JPEG13 GrayACSImageDict ltlt13 QFactor 01513 HSamples [1 1 1 1] VSamples [1 1 1 1]13 gtgt13 GrayImageDict ltlt13 QFactor 01513 HSamples [1 1 1 1] VSamples [1 1 1 1]13 gtgt13 JPEG2000GrayACSImageDict ltlt13 TileWidth 25613 TileHeight 25613 Quality 3013 gtgt13 JPEG2000GrayImageDict ltlt13 TileWidth 25613 TileHeight 25613 Quality 3013 gtgt13 AntiAliasMonoImages false13 CropMonoImages true13 MonoImageMinResolution 120013 MonoImageMinResolutionPolicy OK13 DownsampleMonoImages true13 MonoImageDownsampleType Bicubic13 MonoImageResolution 120013 MonoImageDepth -113 MonoImageDownsampleThreshold 15000013 EncodeMonoImages true13 MonoImageFilter CCITTFaxEncode13 MonoImageDict ltlt13 K -113 gtgt13 AllowPSXObjects false13 CheckCompliance [13 None13 ]13 PDFX1aCheck false13 PDFX3Check false13 PDFXCompliantPDFOnly false13 PDFXNoTrimBoxError true13 PDFXTrimBoxToMediaBoxOffset [13 00000013 00000013 00000013 00000013 ]13 PDFXSetBleedBoxToMediaBox true13 PDFXBleedBoxToTrimBoxOffset [13 00000013 00000013 00000013 00000013 ]13 PDFXOutputIntentProfile ()13 PDFXOutputConditionIdentifier ()13 PDFXOutputCondition ()13 PDFXRegistryName ()13 PDFXTrapped False1313 CreateJDFFile false13 Description ltlt13 ARA ltFEFF06270633062A062E062F0645002006470630064700200627064406250639062F0627062F0627062A002006440625064606340627062100200648062B062706260642002000410064006F00620065002000500044004600200645062A064806270641064206290020064406440637062806270639062900200641064A00200627064406450637062706280639002006300627062A0020062F0631062C0627062A002006270644062C0648062F0629002006270644063906270644064A0629061B0020064A06450643064600200641062A062D00200648062B0627062606420020005000440046002006270644064506460634062306290020062806270633062A062E062F062706450020004100630072006F0062006100740020064800410064006F006200650020005200650061006400650072002006250635062F0627063100200035002E0030002006480627064406250635062F062706310627062A0020062706440623062D062F062B002E0635062F0627063100200035002E0030002006480627064406250635062F062706310627062A0020062706440623062D062F062B002Egt13 BGR 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 CHS ltFEFF4f7f75288fd94e9b8bbe5b9a521b5efa7684002000410064006f006200650020005000440046002065876863900275284e8e9ad88d2891cf76845370524d53705237300260a853ef4ee54f7f75280020004100630072006f0062006100740020548c002000410064006f00620065002000520065006100640065007200200035002e003000204ee553ca66f49ad87248672c676562535f00521b5efa768400200050004400460020658768633002gt13 CHT ltFEFF4f7f752890194e9b8a2d7f6e5efa7acb7684002000410064006f006200650020005000440046002065874ef69069752865bc9ad854c18cea76845370524d5370523786557406300260a853ef4ee54f7f75280020004100630072006f0062006100740020548c002000410064006f00620065002000520065006100640065007200200035002e003000204ee553ca66f49ad87248672c4f86958b555f5df25efa7acb76840020005000440046002065874ef63002gt13 CZE 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 DAN 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 DEU ltFEFF00560065007200770065006e00640065006e0020005300690065002000640069006500730065002000450069006e007300740065006c006c0075006e00670065006e0020007a0075006d002000450072007300740065006c006c0065006e00200076006f006e002000410064006f006200650020005000440046002d0044006f006b0075006d0065006e00740065006e002c00200076006f006e002000640065006e0065006e002000530069006500200068006f006300680077006500720074006900670065002000500072006500700072006500730073002d0044007200750063006b0065002000650072007a0065007500670065006e0020006d00f60063006800740065006e002e002000450072007300740065006c006c007400650020005000440046002d0044006f006b0075006d0065006e007400650020006b00f6006e006e0065006e0020006d006900740020004100630072006f00620061007400200075006e0064002000410064006f00620065002000520065006100640065007200200035002e00300020006f0064006500720020006800f600680065007200200067006500f600660066006e00650074002000770065007200640065006e002egt13 ESP 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 ETI 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 FRA 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 GRE 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 HEB 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 HRV (Za stvaranje Adobe PDF dokumenata najpogodnijih za visokokvalitetni ispis prije tiskanja koristite ove postavke Stvoreni PDF dokumenti mogu se otvoriti Acrobat i Adobe Reader 50 i kasnijim verzijama)13 HUN 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 ITA 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 JPN ltFEFF9ad854c18cea306a30d730ea30d730ec30b951fa529b7528002000410064006f0062006500200050004400460020658766f8306e4f5c6210306b4f7f75283057307e305930023053306e8a2d5b9a30674f5c62103055308c305f0020005000440046002030d530a130a430eb306f3001004100630072006f0062006100740020304a30883073002000410064006f00620065002000520065006100640065007200200035002e003000204ee5964d3067958b304f30533068304c3067304d307e305930023053306e8a2d5b9a306b306f30d530a930f330c8306e57cb30818fbc307f304c5fc59808306730593002gt13 KOR ltFEFFc7740020c124c815c7440020c0acc6a9d558c5ec0020ace0d488c9c80020c2dcd5d80020c778c1c4c5d00020ac00c7a50020c801d569d55c002000410064006f0062006500200050004400460020bb38c11cb97c0020c791c131d569b2c8b2e4002e0020c774b807ac8c0020c791c131b41c00200050004400460020bb38c11cb2940020004100630072006f0062006100740020bc0f002000410064006f00620065002000520065006100640065007200200035002e00300020c774c0c1c5d0c11c0020c5f40020c2180020c788c2b5b2c8b2e4002egt13 LTH 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 LVI 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 NLD (Gebruik deze instellingen om Adobe PDF-documenten te maken die zijn geoptimaliseerd voor prepress-afdrukken van hoge kwaliteit De gemaakte PDF-documenten kunnen worden geopend met Acrobat en Adobe Reader 50 en hoger)13 NOR ltFEFF004200720075006b00200064006900730073006500200069006e006e007300740069006c006c0069006e00670065006e0065002000740069006c002000e50020006f0070007000720065007400740065002000410064006f006200650020005000440046002d0064006f006b0075006d0065006e00740065007200200073006f006d00200065007200200062006500730074002000650067006e0065007400200066006f00720020006600f80072007400720079006b006b0073007500740073006b00720069006600740020006100760020006800f800790020006b00760061006c0069007400650074002e0020005000440046002d0064006f006b0075006d0065006e00740065006e00650020006b0061006e002000e50070006e00650073002000690020004100630072006f00620061007400200065006c006c00650072002000410064006f00620065002000520065006100640065007200200035002e003000200065006c006c00650072002000730065006e006500720065002egt13 POL 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 PTB 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 RUM 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 RUS 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 SKY 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 SLV 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 SUO 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 SVE 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 TUR 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 UKR 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 ENU (Use these settings to create Adobe PDF documents best suited for high-quality prepress printing Created PDF documents can be opened with Acrobat and Adobe Reader 50 and later)13 gtgt13 Namespace [13 (Adobe)13 (Common)13 (10)13 ]13 OtherNamespaces [13 ltlt13 AsReaderSpreads false13 CropImagesToFrames true13 ErrorControl WarnAndContinue13 FlattenerIgnoreSpreadOverrides false13 IncludeGuidesGrids false13 IncludeNonPrinting false13 IncludeSlug false13 Namespace [13 (Adobe)13 (InDesign)13 (40)13 ]13 OmitPlacedBitmaps false13 OmitPlacedEPS false13 OmitPlacedPDF false13 SimulateOverprint Legacy13 gtgt13 ltlt13 AddBleedMarks false13 AddColorBars false13 AddCropMarks false13 AddPageInfo false13 AddRegMarks false13 ConvertColors ConvertToCMYK13 DestinationProfileName ()13 DestinationProfileSelector DocumentCMYK13 Downsample16BitImages true13 FlattenerPreset ltlt13 PresetSelector MediumResolution13 gtgt13 FormElements false13 GenerateStructure false13 IncludeBookmarks false13 IncludeHyperlinks false13 IncludeInteractive false13 IncludeLayers false13 IncludeProfiles false13 MultimediaHandling UseObjectSettings13 Namespace [13 (Adobe)13 (CreativeSuite)13 (20)13 ]13 PDFXOutputIntentProfileSelector DocumentCMYK13 PreserveEditing true13 UntaggedCMYKHandling LeaveUntagged13 UntaggedRGBHandling UseDocumentProfile13 UseDocumentBleed false13 gtgt13 ]13gtgt setdistillerparams13ltlt13 HWResolution [2400 2400]13 PageSize [612000 792000]13gtgt setpagedevice13

Page 12: July 2014 MEE Questions and Analyses - NCBE...This publication includes the questions and analyses from the July 2014 MEE. (In the actual test, the questions are simply numbered rather

EVIDENCE QUESTION

A prison inmate has filed a civil rights lawsuit against a guard at the prison alleging that the guard violated the inmatersquos constitutional rights during an altercation The inmate and the guard are the only witnesses to this altercation They have provided contradictory reports about what occurred

The trial will be before a jury The inmate plans to testify at trial The guardrsquos counsel has moved for leave to impeach the inmate with the following

(a) Twelve years ago the inmate was convicted of felony distribution of marijuana He served a three-year prison sentence which began immediately after he was convicted He served his full sentence and was released from prison nine years ago (b) Eight years ago the inmate pleaded guilty to perjury a misdemeanor punishable by up to one year in jail He paid a $5000 fine (c) Seven years ago the inmate was convicted of felony sexual assault of a child and is currently serving a 10-year prison sentence for the crime The victim was the inmatersquos daughter who was 13 years old at the time of the assault

The inmatersquos counsel objects to the admission of any evidence related to these three convictions and to any cross-examination based on this evidence

The guard also plans to testify at trial The inmatersquos counsel has moved for leave to impeach the guard with the following

Last year the guard applied for a promotion to prison supervisor The guard submitted a reacutesumeacute to the state that indicated that he had been awarded a BA in Criminal Justice from a local college An official copy of the guardrsquos academic transcript from that college indicates that the guard dropped out after his first semester and did not receive a degree

The guardrsquos counsel objects to the admission of this evidence and to any cross-examination based on this evidence

The transcript and the reacutesumeacute have been properly authenticated The trial will be held in a jurisdiction that has adopted all of the Federal Rules of Evidence

1 What evidence if any proffered by the guard to impeach the inmate should be admitted Explain

2 What evidence if any proffered by the inmate to impeach the guard should be admitted Explain

8

CORPORATIONS QUESTION

Mega Inc is a publicly traded corporation incorporated in a state whose corporate statute is modeled on the Model Business Corporation Act (MBCA) Megarsquos articles of incorporation do not address the election of directors or amendment of the bylaws by shareholders

Well within the deadline for the submission of shareholder proposals for the upcoming annual shareholdersrsquo meeting an investor who was a large and long-standing shareholder of Mega submitted a proposed amendment to Megarsquos bylaws The proposal which the investor asked to be included in the corporationrsquos proxy materials and voted on at the upcoming shareholdersrsquo meeting read as follows

Section 20 The Corporation shall include in its proxy materials (including the proxy ballot) for a shareholdersrsquo meeting at which directors are to be elected the name of a person nominated for election to the Board of Directors by a shareholder or group of shareholders that beneficially have owned 3 or more of the Corporationrsquos outstanding common stock for at least one year

This Section shall supersede any inconsistent provision in these Bylaws and may not be amended or repealed by the Board of Directors without shareholder approval

Megarsquos management decided to exclude the investorrsquos proposal from the corporationrsquos proxy materials and explained its reasons in a letter to the investor

The investorrsquos proposed bylaw provision would be inconsistent with relevant state law because the Board of Directors has the authority to manage the business and affairs of the Corporation Generally shareholders lack the authority to interfere with corporate management by seeking to create a method for the nomination and election of directors inconsistent with the method chosen by the Board of Directors

Furthermore at its most recent meeting the Board of Directors unanimously approved an amendment to the Corporationrsquos bylaws that provides for proxy access for director nominations by a shareholder or a group of shareholders holding at least 10 of the Corporationrsquos voting shares for at least three years This procedure takes precedence over any nomination methods that might be sought or approved by shareholders

The investor is considering bringing a suit challenging managementrsquos refusal to include the investorrsquos proposed bylaw provision and challenging the boardrsquos amendment of the bylaws at its recent meeting

1 Is the investorrsquos proposed bylaw provision inconsistent with state law Explain

2 If the investorrsquos proposed bylaw provision were approved by the shareholders would the bylaw amendment previously approved by the board take precedence over the investorrsquos proposed bylaw provision Explain

3 Must the investor make a demand on Megarsquos board of directors before bringing suit Explain

9

July 2014 MEE

ANALYSES

Contracts Family Law

Criminal Law and Procedure

Federal Civil Procedure Evidence

Corporations

CRIMINAL LAW AND PROCEDURE ANALYSIS (Criminal Law and Procedure VA B D)

ANALYSIS

Legal Problems

(1) Did the detective violate the suspectrsquos Sixth Amendment right to counsel when he questioned the suspect about the burglaries without the lawyer present given that the lawyer represented the suspect in an unrelated criminal matter

(2) Under Miranda did the suspect effectively invoke his right to counsel when he said ldquoI think I want my lawyer here before I talk to yourdquo

(3) Was the suspectrsquos waiver of his right to remain silent under Miranda valid

DISCUSSION

Summary

The Sixth Amendment right to counsel as applied to states through the Fourteenth Amendment is offense-specific Although the suspect had an attorney representing him on his pending assault charge he had no Sixth Amendment right to the assistance of counsel with respect to the five uncharged burglaries because formal adversarial proceedings had not yet commenced on those charges The suspectrsquos Sixth Amendment right to counsel was not violated by the detectiversquos failure to inform him that the lawyer was present or of the lawyerrsquos demands

However a person undergoing custodial interrogation also has an independent constitutional right to counsel during custodial interrogation under Miranda When a suspect invokes his right to counsel under Miranda custodial interrogation must immediately cease for a period of at least 14 days However the invocation of the right to counsel must be unambiguous and clearly convey that the suspect has requested counsel Here because the suspectrsquos statement ldquoI think I want my lawyer here before I talk to yourdquo was ambiguous he did not invoke his Miranda right to counsel

A waiver of rights must be knowing intelligent and voluntary Here the suspect waived his right to remain silent under Miranda when he signed the waiver form The fact that the detective did not correct the suspectrsquos assumption that the lawyer would need to drive to the jailmdashby telling him that the lawyer was in the waiting room and was demanding to see himmdashdid not affect the validity of the suspectrsquos waiver

Point One (35) The suspectrsquos Sixth Amendment right to counsel was not violated because the right does not attach on new charges until formal adversarial judicial proceedings have commenced on those charges

The Sixth Amendment as applied to the states through the Fourteenth Amendment provides that ldquo[i]n all criminal prosecutions the accused shall enjoy the right to have the Assistance of Counsel for his defenserdquo The right to counsel does not attach with respect to particular charges until formal adversarial judicial proceedings have commenced (ie ldquoat or after the initiation of

13

Criminal Law and Procedure Analysis

adversary judicial criminal proceedingsmdashwhether by way of formal charge preliminary hearing indictment information or arraignment [or in some states arrest warrant]rdquo McNeil v Wisconsin 501 US 171 175 (1991) (internal quotations omitted)) Once a suspectrsquos Sixth Amendment right to counsel has attached any attempts to ldquodeliberately elicitrdquo statements from him in the absence of his attorney violate the Sixth Amendment See Massiah v United States 377 US 201 (1964) Brewer v Williams 430 US 387 (1977)

The Sixth Amendment right to counsel is charge- or offense -specific Representation by counsel in one prosecution does not in itself guarantee counsel for uncharged offenses See McNeil 501 US at 175 Texas v Cobb 532 US 162 (2001) Here the suspectrsquos Sixth Amendment right to counsel had attached only for the pending aggravated assault charge The suspectrsquos right to counsel for the aggravated assault case did not guarantee counsel for the five unrelated and uncharged burglaries that were the subject of the detectiversquos interrogation Thus because formal adversarial judicial proceedings against the suspect for the uncharged burglaries had not begun he had no Sixth Amendment right to counsel

Finally the detectiversquos failure to inform the suspect of the lawyerrsquos presence and demands to speak with him does not implicate the suspectrsquos Sixth Amendment right to counsel which had not yet attached See id Moran v Burbine 475 US 412 428ndash31 (1986)

Point Two (30) The suspect did not effectively invoke his right to counsel under Miranda because his statement was not unambiguous

A suspect subject to custodial interrogation has a right to consult with counsel and to have an attorney present during questioning Miranda v Arizona 384 US 436 (1966) When a suspect invokes his right to counsel during an interrogation law enforcement must immediately cease all questioning See Edwards v Arizona 451 US 477 484ndash85 (1981) Custodial interrogation cannot be reinitiated unless and until the suspect has been re-advised of his Miranda rights has provided a knowing and voluntary waiver and (1) counsel is present and (2) the suspect himself initiated further communication with the police see id at 484 or (3) (if the suspect was released from custody after the initial interrogation) at least 14 days have passed Maryland v Shatzer 559 US 98 110 (2010)

To invoke the right to counsel a suspectrsquos request must be ldquounambiguousrdquo This means that the suspect must articulate the desire for counsel sufficiently clearly that a reasonable officer would understand the statement to be a request for counsel Davis v United States 512 US 452 459 (1994) If the request is ambiguous the police are not required to stop the interrogation

In this case the suspectrsquos statement ldquoI think I want my lawyer here before I talk to yourdquo was not an unambiguous request for counsel The most reasonable interpretation of this statement is that the suspect might be invoking his right to counsel Id at 461 (ldquomaybe I should talk to a lawyerrdquo is not an unequivocal request for counsel) See also Burket v Angelone 208 F3d 172 197ndash98 (4th Cir 2000) (ldquoI think I need a lawyerrdquo is not an unambiguous request for an attorney) Soffar v Cockrell 300 F3d 588 594ndash95 (5th Cir 2002) (discussion of various statements that did not constitute unequivocal requests for counsel)

Under these circumstances the detective was not required to cease the custodial interrogation of the suspect Nor was the detective required to clarify or ask follow-up questions to determine whether the suspect in fact wanted an attorney Davis 512 US at 459ndash60

14

Criminal Law and Procedure Analysis

Point Three (35) The suspectrsquos waiver of his Miranda rights was knowing intelligent and voluntary despite the fact that he was never told of the lawyerrsquos presence in the jail or of the lawyerrsquos demands

A valid waiver of Miranda rights must be ldquovoluntaryrdquomdashie the product of a free or deliberate choice rather than intimidation coercion or deception Berghuis v Thompkins 560 US 370 382ndash83 (2010) In addition the waiver must be knowing and intelligent That is it ldquomust have been made with a full awareness of both the nature of the right being abandoned and the consequences of the decision to abandon itrdquo Moran v Burbine 475 US 421 (1986)

In this case the suspect signed a Miranda waiver form after receiving proper warnings There is no evidence ldquothat the police resorted to physical or psychological pressure to elicit the statementsrdquo Id The entire interview lasted only 45 minutes The only issue is whether the suspect knowingly and intelligently waived his Miranda rights despite the fact that the detective did not tell the suspect about the lawyerrsquos presence and her demands

The Supreme Court has said that ldquo[e]vents occurring outside of the presence of the suspect and entirely unknown to him surely can have no bearing on the capacity to comprehend and knowingly relinquish a constitutional rightrdquo Id at 422 If the suspect ldquoknew that he could stand mute and request a lawyer and was aware of the Statersquos intention to use his statements to secure a convictionrdquo then the waiver is valid regardless of the information withheld Id at 422ndash23

Here the suspect was correctly informed of his rights Miranda v Arizona 384 US at 467ndash73 His comments demonstrate that he understood that he could have a lawyer present if he desired (ie wondering whether he should call his attorney) and that he understood that there might be consequences to speaking with the detective (ldquoI probably should keep my mouth shut but Irsquom willing to talk to you for a whilerdquo) His comment ldquo[L]etrsquos not waste any time waiting for someone to call my attorney and having her drive hererdquo along with his signature on the Miranda waiver form show that his waiver was valid under the constitutional standard

The fact that the detective did not tell the suspect about the lawyerrsquos presence and demands has no bearing on the validity of the suspectrsquos waiver because ldquosuch conduct is only relevant to the constitutional validity of a waiver if it deprives a defendant of knowledge essential to his ability to understand the nature of his rights and the consequences of abandoning themrdquo Moran at 424 The Supreme Court has specifically declined to adopt a rule requiring that law enforcement tell a suspect of an attorneyrsquos efforts to contact him id at 425 (ldquoNor are we prepared to adopt a rule requiring that the police inform a suspect of an attorneyrsquos efforts to reach himrdquo)

[NOTE An examinee might also recognize that this general rule is further supported by the Supreme Courtrsquos decision in Florida v Powell 559 US 50 (2010) approving state Miranda warnings that do not explicitly warn suspects that they have a right to have counsel present during custodial interrogation]

15

CONTRACTS ANALYSIS (Contracts IB2 IIB IVA3 amp A5)

ANALYSIS

Legal Problems

(1) In the case of a service contract (governed by the common law of contracts) is a modification enforceable when a party agrees to pay more for the same performance than was originally promised

(2) In the case of a contract for the sale of goods (governed by Article 2 of the UCC) is a modification enforceable when a party agrees to pay more for the same goods than was originally promised

(3) May a party avoid an agreement on the basis of economic duress

DISCUSSION

Summary

There are two arguments that the conservatory can make to support the claim that it is not bound to pay the higher prices lack of consideration and economic duress

The organ repair contract is governed by the common law of contracts Under the common law the business would have difficulty recovering the additional $60000 for the organ repair because under the ldquopreexisting duty rulerdquo the agreement of the conservatory to pay the extra price was not supported by consideration However the business might argue that the modification is enforceable under an exception to the preexisting duty rule for fair and equitable modifications made in light of unanticipated circumstances

The organ sale contract is governed by Article 2 of the Uniform Commercial Code The business would likely recover the additional amount under that contract because Article 2 provides that consideration is not required for a modification to be binding

In both cases the conservatory could seek to avoid its agreement on the grounds of economic duress but that argument is not likely to succeed

Point One (45) The business probably cannot recover the additional $60000 for the organ repair because the conservatoryrsquos promise to pay more money was not supported by consideration

The general rule is that to be enforceable a promise must be supported by consideration Under RESTATEMENT (SECOND) OF CONTRACTS sect 71 a promise is supported by consideration if it is bargained for in exchange for a return promise or performance However under the ldquopreexisting duty rulerdquo (exemplified in RESTATEMENT (SECOND) OF CONTRACTS sect 73 and Alaska Packersrsquo Assrsquon v Domenico 117 F 99 (9th Cir 1902)) promise of performance of a legal duty already owed to a promisor which is neither doubtful nor the subject of honest dispute is not consideration

If the business had promised the conservatory anything new or different in exchange for the agreement to pay the additional $60000 (such as for example repairing the pipe organ more

16

Contracts Analysis

quickly or using better parts) that would constitute consideration especially in light of the principle that courts do not inquire into the adequacy of consideration Here however the business already had a legal duty under the original contract and did not agree to do anything else in exchange for the conservatoryrsquos promise to pay $60000 more

However an exception to the preexisting duty rule is sometimes applied in situations of unanticipated changed circumstances Under RESTATEMENT (SECOND) OF CONTRACTS sect 89 followed in many jurisdictions a promise modifying a duty under a contract not fully performed on either side is binding even if not supported by consideration if the modification is fair and equitable in view of circumstances not anticipated by the parties when the contract was made

If a court applies the rule in Restatement sect 89 the critical issues will be whether the modification was in fact ldquofair and equitablerdquo and whether it can be justified in light of unanticipated circumstances In many cases in which modifications have been upheld a party encountered difficulties or burdens in performing far beyond what was knowingly bargained for in the original contract with the result bordering on impracticability such as having to excavate solid rock instead of soft dirt or having to remove garbage far in excess of the amounts contemplated The conservatory would argue that the businessrsquos performance difficulties were not of this sort at allmdashnothing about repairing the pipe organ itself was any different from or more difficult than originally contemplated except that the business itself encountered financial distress unrelated to its burdens in performing its obligations under these contracts

Even if the business satisfies that element of the rule in Restatement sect 89 the business must also demonstrate that the circumstances that gave rise to the need to modify the contract were ldquounanticipatedrdquo at the time the original contract was made Here the facts suggest that when the business entered into the original contract it expected that the price paid by the conservatory would enable it to perform However any evidence that the business knew or had reason to know at the time of execution that it would need more money from the conservatory to be able to perform would mean that the request to modify was not ldquounanticipatedrdquo

[NOTE Some cases such as Schwartzreich v Bauman-Basch Inc 231 NY 196 131 NE 887 (1921) find that if the parties mutually agreed to rescind the original contract and then after rescission entered into an entirely new contract for a higher price the new contract is supported by consideration There is no evidence that such a rescission followed by a new contract took place here]

Point Two (45) The business can recover the additional $40000 for the new organ because no consideration is required under Article 2 of the UCC for good-faith contract modifications

The contract to buy a new organ is a contract for the sale of goods and therefore is governed by Article 2 of the Uniform Commercial Code UCC sect 2-102 Under Article 2 unlike the common law an agreement modifying a contract needs no consideration to be binding UCC sect 2-209(1) Section 2-209(1) thus obviates the preexisting duty rule entirely in contracts for the sale of goods

Even though consideration is not required modifications governed by sect 2-209 must satisfy the obligation of good faith imposed by the UCC UCC sect 1-304 See also Official Comment 2 to UCC sect 2-209 Good faith means ldquohonesty in fact and the observance of reasonable commercial standards of fair dealingrdquo UCC sect 1-201(b)(20) In this context the obligation of good faith means that ldquo[t]he effective use of bad faith to escape performance on the original contract terms is barred and the extortion of a lsquomodificationrsquo without legitimate commercial reason is ineffective as a violation of the duty of good faithrdquo Official Comment 2 to

17

Contracts Analysis

UCC sect 2-209 Here because the businessrsquos financial reversals were serious and apparently unanticipated at the time that the business entered into the contract with the conservatory and commitment of the extra money was needed to enable the business to perform a court would likely find that the business acted in good faith Thus a court would likely uphold the enforceability of the conservatoryrsquos promise to pay the additional $40000

Point Three (10) The conservatory is unlikely to be able to defend against enforcement of its promises to pay additional money under the theory of economic duress because the business probably did not make an improper threat

Under the common law of contracts parties may raise the defense of duress This common law defense also applies to contracts governed by UCC Article 2 See UCC sect 1-103(b)

A contract is voidable on the ground of economic duress by threat when it is established that a partyrsquos manifestation of assent is induced by an improper threat that leaves the party no reasonable alternative See RESTATEMENT (SECOND) OF CONTRACTS sect 175 See also eg Austin Instrument Inc v Loral Corp 272 NE2d 533 (NY 1971) (a threat to withhold essential goods can constitute duress) In order to void its agreement to pay the additional sum because of economic duress the conservatory must demonstrate that (1) the business made a threat to the conservatory (2) the threat was ldquoimproperrdquo or ldquowrongfulrdquo (3) the threat induced the conservatoryrsquos manifestation of assent to the modification and (4) the threat was sufficiently grave to justify the conservatoryrsquos assent

Here it appears that three of the four elements are likely satisfied The business plainly made a threat Moreover the threat induced the conservatoryrsquos assent to the modification and the threat was sufficiently grave to justify that assent If the conservatory had not agreed to pay the business the extra amounts the conservatory would have lost its entire $325000 investment In light of this potential loss a court could easily conclude that the conservatory had no reasonable alternative

However the business has a strong argument that its threat (indicating that it would breach the contracts unless the prices were increased) was not wrongful or improper but was instead nothing more than a communication of the reality of its own perilous situation to the conservatory

A mere threat to breach a contract is not in and of itself improper so as to support an action of economic duress or business compulsion Something more is required such as a breach of the duty of good faith and fair dealing as was present in Austin Instrument Inc supra Because the business could not perform the original contract without the requested modification the economic duress claim for the conservatory would likely fail for much the same reason that the business would be able to enforce the modification At the time the modification was requested the business was not trying to extort a price increase because of the conservatoryrsquos vulnerability but instead was simply stating the reality that the business could not perform without more money

18

FAMILY LAW ANALYSIS (Family Law IIIB D amp G)

ANALYSIS

Legal Problems

(1)(a) Does the State A court have jurisdiction to modify the State B child support order

(1)(b) Does the State A court have jurisdiction to modify the marital-residence-saleshyproceeds provision of the State B property-division decree

(2)(a) May a child support order be modified retroactively

(2)(b) May a child support order be modified prospectively based on a change of employment with a lower salary

(2)(c) May a property-division order be modified after entry of a divorce decree

DISCUSSION

Summary

The State A court may exercise personal jurisdiction over the wife because she was personally served in State A However subject-matter jurisdiction over the interstate modification of child support is governed by the Uniform Interstate Family Support Act (UIFSA) Under UIFSA State A does not have jurisdiction to modify the order for the daughterrsquos support because the wife is still a resident of State B UIFSA on the other hand does not govern property distributions and thus a State A court is not precluded from hearing the husbandrsquos petition to modify the marital-residence-sale-proceeds provision of the divorce decree

A child support order may not be modified retroactively A child support order may be modified prospectively based on a substantial change in circumstances Courts agree that a significant decrease in income is a substantial change in circumstances All states treat voluntary income reductions differently than involuntary reductions but employ different approaches for evaluating the impact of a voluntary reduction Whether the husband could obtain prospective modification of the child support order depends on which approach is applied

A property-division order is not subject to post-divorce modification based on a change in circumstances Thus the husband may in some states obtain prospective modification of the order for the daughterrsquos support but he may not obtain modification of the marital-residenceshysale-proceeds provision

Point One(a) (25) Personal jurisdiction over a nonresident respondent does not confer subject-matter jurisdiction over child support modification Under UIFSA a State A court may not modify a child support order issued by a State B court when as here the child or either parent continues to reside in State B the jurisdiction that issued the child support order

The State A court may exercise personal jurisdiction over the wife The wife was personally served in State A and a state may exercise jurisdiction based on in-state personal service See

19

Family Law Analysis

Burnham v Superior Court 495 US 604 (1990) But personal jurisdiction over the wife is not enough to give a State A court jurisdiction to modify the State B support order

The interstate enforcement and modification of child support is governed by the Uniform Interstate Family Support Act (UIFSA) which has been adopted by all states Under UIFSA the state that originally issued a child support order (here State B) has continuing exclusive jurisdiction to modify the order if that state remains the residence of the obligee the child or the obligor and all parties do not consent to the jurisdiction of another forum See UIFSA sect 205 See also UIFSA sect 603 (ldquoA tribunal of this State shall recognize and enforce but may not modify a registered order if the issuing tribunal had jurisdictionrdquo) The wife and daughter continue to reside in State B and the wife has not consented to the jurisdiction of another forum Thus a State A court does not have jurisdiction to modify the State B child support order

[NOTE Examinees who do not discuss personal jurisdiction but fully discuss UIFSA may receive full credit]

Point One(b) (15) UIFSA does not apply to disputes over property division Thus the State A court may exercise jurisdiction over the husbandrsquos petition to modify the marital-residence-sale-proceeds provision of the State B divorce decree because it has personal jurisdiction over the wife

The State A court in which the husband brought his action has jurisdiction to adjudicate domestic relations issues The husbandrsquos petition to modify the property settlement is a domestic relations issue The courts of State A may exercise personal jurisdiction over the wife because she was personally served in State A See Burnham v Superior Court 495 US 604 (1990) see Point One(a)

UIFSA does not apply to divorce property-division disputes Thus although a State A court may not adjudicate the husbandrsquos petition to modify his child support obligations it may adjudicate his property-division claims (Even though the court has jurisdiction it may not modify the property-division award on the merits See Point Two(c))

Point Two(a) (20) A child support order may not be modified retroactively

State courts have long held that obligations to pay child support ordinarily may not be modified retroactively ldquoIf the hardship is particularly severe the courts sometimes devised a way to protect the obligor but in most instances the courts hold that retroactive modification of this kind is beyond their power and indeed the governing statute may so providerdquo HOMER H CLARK THE LAW OF DOMESTIC RELATIONSHIPS IN THE UNITED STATES 725 (2d ed 1987)

Federal law now goes further and requires the states as a condition of federal child-support funding to adopt rules that absolutely forbid retroactive modification of the support obligation See 42 USC sect 666(a)(9)(C) The states have adopted rules consistent with the federal requirements

Point Two(b) (25) It is unclear whether the husband could obtain prospective downward modification of his child support based on his voluntary acceptance of a job with a lower salary

Prospective modification of a child support order is typically available only when the petitioner can show a substantial change in circumstances See ROBERT E OLIPHANT amp NANCY VER

20

Family Law Analysis

STEEGH FAMILY LAW 213ndash15 (3d ed 2010) A significant decrease in income is typically viewed as a substantial change

However when a parent seeks to modify a child support obligation because he has voluntarily reduced his income a court will not modify the obligation based solely on the income loss Some courts refuse to modify whenever the income shift was voluntary See eg Aguiar v Aguiar 127 P3d 234 (Idaho Ct App 2005) Others look primarily to the petitionerrsquos intentions and permit downward modification if he has acted in good faith See eg In re Marriage of Horn 650 NE2d 1103 (Ill App Ct 1995) Many courts use a multifactor approach See OLIPHANT amp VER STEEGH supra 217ndash18

Here there is no question that the husbandrsquos loss of income was voluntary In a jurisdiction in which voluntary income reduction bars support modification the husbandrsquos petition would be denied

In a jurisdiction employing a good-faith or multifactor approach it is possible but not certain that the husband could obtain downward modification The evidence supports the husbandrsquos good faith his change in employment appears to be based on his new jobrsquos greater responsibilities and better promotion possibilities In a jurisdiction using a multifactor approach the court would likely also consider the impact of such a shift on the daughter the likely duration of the husbandrsquos income loss and the likelihood of a promotion that would ultimately inure to the daughterrsquos benefit Thus on these facts it is possible but by no means certain that the husband could prospectively obtain downward modification of his child support obligation to his daughter

Point Two(c)(15) A divorce property-division award is not subject to modification

A support order is aimed at meeting the post-divorce needs of the supported individual Because the future is unpredictable courts are empowered to modify a support award to take account of changed circumstances that may occur during the period in which support is paid

By contrast a property-distribution award divides assets of the marriage based on the equities at the time of divorce Because the past can be ascertained a property-division award is not subject to post-divorce modification See HARRY A KRAUSE ET AL FAMILY LAW CASES COMMENTS AND QUESTIONS 691 (6th ed 2007)

Here the husband is seeking modification of a property-division award with respect to an asset owned by the parties at the time of divorce Thus the husband may not obtain a modification of the marital-residence-sale-proceeds provision of the divorce decree based on his reduced income

21

FEDERAL CIVIL PROCEDURE ANALYSIS (Federal Civil Procedure III IVC)

ANALYSIS

Legal Problems

(1) Is the logging company entitled to join this action as a matter of right

(2)(a) May the nonprofit organization obtain a temporary restraining order to stop the USFS from issuing a logging permit

(2)(b) May the nonprofit organization obtain a preliminary injunction to stop the USFS from issuing a logging permit during the pendency of the action

DISCUSSION

Summary

The logging company is entitled to intervene in this action as a matter of right because it has an interest in the property or transaction that is the subject of the action and is so situated that its interest may be impaired or impeded as a practical matter if the action goes forward without it The logging companyrsquos interest is not adequately represented by the USFSrsquos presence in the lawsuit

The nonprofit organization may seek a temporary restraining order (TRO) followed by a preliminary injunction to prevent the USFS from issuing a logging permit pending the outcome of the action The nonprofit is likely to obtain a TRO if it can demonstrate a risk of immediate and irreparable injury The nonprofit is also likely to obtain a preliminary injunction if it can demonstrate a significant threat of irreparable harm and a likelihood of success on the merits of its National Environmental Policy Act (NEPA) claim

Point One (50) Rule 24(a) of the Federal Rules of Civil Procedure requires federal courts to allow a person to intervene in an action as a matter of right if the person a) is interested in the property or transaction that is the subject of the action b) is so situated that its interest may be impaired or impeded if the litigation goes forward without it and c) is not adequately represented by existing parties Here the logging company likely meets all three requirements and should be allowed to intervene as a matter of right

Rule 24 of the Federal Rules of Civil Procedure governs intervention the process by which a non-party to an action may join the litigation Under Rule 24(a) (intervention of right) a person must be permitted to intervene if three conditions are met (1) the movant ldquoclaims an interest relating to the property or transaction that is the subject of the actionrdquo (2) the movant ldquois so situated that disposition of the action may as a practical matter impair or impede the movantrsquos ability to protect its interestrdquo and (3) ldquoexisting partiesrdquo do not ldquoadequately represent [the movantrsquos] interestrdquo FED R CIV P 24(a) The three requirements for intervention of right are often ldquovery interrelatedrdquo 7C CHARLES ALAN WRIGHT ET AL FEDERAL PRACTICE AND PROCEDURE sect 1908 at 297 (2007 amp 2011 Supp)

22

Federal Civil Procedure Analysis

Here the court should find that the logging company meets this test First the logging company has a strong interest in the property or transaction that is the subject of this action The USFS has accepted the logging companyrsquos bid and the logging company is merely awaiting issuance of a logging permit to begin logging The nonprofit organization is seeking to prevent this logging The logging company therefore has a strong direct and substantial interest in the subject matter of the lawsuit and in having its winning bid honored and a logging permit issued See eg Kleissler v US Forest Serv 157 F3d 964 972 (3d Cir 1998) (stating that ldquo[t]imber companies have direct and substantial interests in a lawsuit aimed at halting loggingrdquo) see also Natural Resources Defense Council v US Nuclear Regulatory Commrsquon 578 F2d 1341 1343ndash 44 (10th Cir 1978) (holding that applicants whose license renewals were pending had Rule 24(a)(2) interests where the lawsuit sought to halt the license-issuing process pending preparation of environmental impact statements) See generally 7C WRIGHT ET AL supra sect 19081 at 309 (ldquoIf there is a direct substantial legally protectable interest in the proceedings it is clear that this requirement of the rule is satisfiedrdquo) Second the logging companyrsquos interest in receiving a logging permit may well be impaired as a practical matter by the outcome of the lawsuit If the USFS loses the lawsuit it will have to prepare an environmental impact statement before issuing the logging companyrsquos permit This will at a minimum delay the logging companyrsquos ability to exercise its rights and may in the long r un mean that no logging permit is ever issued Intervention of right is not limited to those that would be legally bound as a matter of preclusion doctrine Id sect 19082 at 368 Rather ldquo[t]he rule is satisfied whenever disposition of the present action would put the movant at a practical disadvantage in protecting its interestrdquo Id sect 19082 at 369 Here that condition is easily satisfied See Kleissler 157 F3d at 972 (ldquoTimber companies have direct and substantial interests in a lawsuit aimed at halting logging rdquo)

Given that the logging company has an interest that may be impaired by disposition of the action it should be allowed to intervene unless the court is persuaded that the USFS adequately represents the logging companyrsquos interest See Rule 24(a)(2) 7C WRIGHT ET AL supra sect 1909 Here it could be argued that the USFS adequately represents the logging companyrsquos interest because the USFS presumably wants the court to uphold its development plan and allow it to proceed with issuance of the logging permit which is the same relief that the logging company would seek However whether representation is truly adequate depends upon ldquo[a] discriminating appraisal of the circumstancesrdquo 7C WRIGHT ET AL supra sect 1909 at 440 Although both the government and the logging company wish to avoid the preparation of an environmental impact statement their interests are distinct The USFSrsquos interest is proper management of the national forest system while the logging companyrsquos interest is making a profit from logging the 5000-acre tract The USFSrsquos handling of the litigation is likely to be affected by a variety of policy concerns and political considerations that have nothing to do with the logging companyrsquos purely economic interest in securing the right to cut trees in the Scenic National Forest See eg Kleissler 157 F3d at 973ndash74 (ldquo[T]he government represents numerous complex and conflicting interests in matters of this nature The straightforward business interests asserted by intervenors here may become lost in the thicket of sometimes inconsistent governmental policiesrdquo)

[NOTES (1) Examinees who mistakenly analyze the logging companyrsquos case for joinder under the related but incorrect Rule 19 ldquoRequired Joinder of Partiesrdquo may receive credit Rule 19 allows existing parties to demand joinder of non-parties (or seek dismissal of the case if they canrsquot get it) There is a close relationship between Rule 24 and Rule 19 and both contain a similar standard for determining when ldquointerestedrdquo third parties are ldquoentitledrdquo or ldquorequiredrdquo to be in the lawsuit Indeed the two prongs of the Rule 24 intervention test that are discussed above

23

Federal Civil Procedure Analysis

are nearly identical to the two prongs of the Rule 19(a) required joinder test Examinees who discuss and apply the test should receive credit even if they cite Rule 19 rather than Rule 24

(2) Examinees may discuss permissive joinder Although permissive joinder is a possibility here the question asks only whether the logging company can join the action as a matter of right and a permissive joinder analysis is not responsive to the question To the extent an examinee discusses permissive joinder the analysis will focus on whether the logging company ldquohas a claim or defense that shares with the main action a common question of law or factrdquo FED R CIV P 24(b)(1)(B) The district court also ldquomust consider whether the intervention will unduly delay or prejudice the adjudication of the original partiesrsquo rightsrdquo FED R CIV P 24(b)(3) On our facts the logging companyrsquos claim for the issuance of a logging permit would certainly share common questions of law and fact with the USFSrsquos defense against the nonprofitrsquos claim There are no facts suggesting that the logging companyrsquos presence would unduly delay or otherwise prejudice adjudication of the original action Thus the district court would have discretion to permit the logging company to intervene even if it denied intervention of right]

Point Two(a) (25) The nonprofit organization could seek and would likely obtain a temporary restraining order to stop the USFS from issuing a logging permit pending a hearing on an application for a preliminary injunction

The first type of interim relief the nonprofit could seek to stop the USFS from issuing a logging permit to the logging company is a temporary restraining order (TRO) prohibiting the USFS from issuing the logging permit A TRO can be issued without notice to the adverse party but only in limited circumstances and only for a limited time FED R CIV P 65(b) To secure a TRO without notice the nonprofit would need to submit an affidavit containing specific facts that demonstrate a risk of ldquoimmediate and irreparable injuryrdquo if a permit is issued FED R CIV P 65(b)(1) In deciding whether to grant a TRO courts will also consider the same factors that are relevant in deciding whether to grant a preliminary injunction (eg the moving partyrsquos likelihood of success on the merits the balance of hardships and the public interest) See Point Two(b) infra The TRO would last only long enough for the court to consider and resolve a request by the nonprofit for a preliminary injunction but no longer than 14 days (unless the court extends it for good cause or the adverse party consents to an extension) In addition bond is required

Here the court is likely to grant the nonprofitrsquos request The nonprofit could plausibly claim that cutting down 5000 acres of old-growth forest in an area that is home to the highest concentration of wildlife in the western United States would have ldquoan immediate and irreparablerdquo adverse impact on the environment and cause irreparable harm to the nonprofitrsquos interest in preserving and protecting natural resources including wildlife habitat

Point Two(b) (25) The nonprofit could also seek and would likely obtain a preliminary injunction to stop the USFS which is likely to be granted if the nonprofitrsquos claim that the USFS violated NEPA has a strong basis in fact and law

Because the TRO would be temporary the nonprofit would need to move for a preliminary injunction to prevent the USFS from issuing a logging permit throughout the pendency of the litigation Preliminary injunctions are injunctions that seek to ldquoprotect [the] plaintiff from

24

Federal Civil Procedure Analysis

irreparable injury and to preserve the courtrsquos power to render a meaningful decision after a trial on the meritsrdquo 11A CHARLES ALAN WRIGHT ET AL FEDERAL PRACTICE AND PROCEDURE sect 2947 at 112 (2013) Rule 65 of the Federal Rules of Civil Procedure sets out the procedural requirements for preliminary injunctions Preliminary injunctions may be granted only upon notice to the adverse party FED R CIV P 65(a)(1) and only if the movant ldquogives security in an amount that the court considers proper to pay the costs and damages sustained by any party found to have been wrongfully enjoined or restrainedrdquo FED R CIV P 65(c)

While Rule 65 sets out the procedural requirements for preliminary injunctive relief it does not specify the substantive grounds upon which it may be granted The courtrsquos discretion in ruling upon a motion for a preliminary injunction ldquois exercised in conformity with historic federal equity practicerdquo 11A WRIGHT ET AL supra sect 2947 at 114 The court typically considers four factors

(1) the significance of the threat of irreparable harm to the plaintiff if the injunction is not granted (2) the balance between this harm and the injury that granting the injunction would inflict on the defendant (3) the probability that the plaintiff will succeed on the merits and (4) the public interest

Id sect 2948 at 122ndash24 accord Habitat Educ Center v Bosworth 363 F Supp 2d 1070 1088 (ED Wis 2005) The most important of these factors is the risk of irreparable harm to the plaintiff 11A WRIGHT ET AL supra sect 29481 at 129 If the plaintiff has an adequate remedy at law (eg if money damages can compensate the plaintiff for its loss) then a preliminary injunction will be denied Id sect 29481

Here a court would likely conclude that the potential for environmental damage to the forest creates a significant threat of irreparable harm ldquo[E]nvironmental injury is often irreparable Courts have recognized that logging such as would occur [here] can have longshyterm environmental consequences and thus satisfy the irreparable injury criterionrdquo Habitat Educ Center 363 F Supp 2d at 1089 (citing Idaho Sporting Congress Inc v Alexander 222 F3d 562 569 (9th Cir 2000) (noting that the imminent and continuing logging activities presented ldquoevidence of environmental harm sufficient to tip the balance in favor of injunctive reliefrdquo)) Neighbors of Cuddy Mountain v US Forest Service 137 F3d 1372 1382 (9th Cir 1998) (stating that ldquo[t]he old growth forests plaintiffs seek to protect would if cut take hundreds of years to reproducerdquo) (internal citation omitted)) see also 11C WRIGHT ET AL supra sect 29481 at 151 (noting that ldquoa preliminary injunction has been issued to prevent harm to the environmentrdquo)

The second factor the balance between the harm to the plaintiff and the harm the defendant will suffer if the injunction is issued also appears to support issuance of a preliminary injunction here The USFS will have to wait before it can develop the Scenic National Forest and the logging company may lose money if the delay is prolonged These economic harms could be compensated monetarily if an injunction is issued inappropriately Where ldquoan injunction bond can compensate [the] defendant for any harm the injunction is likely to inflict the balance should be struck in favor of [the] plaintiffrdquo Id sect 29482 at 192 See also Habitat Educ Center 363 F Supp 2d at 1089 (stating that ldquothe relative absence of harmful effects on the Forest Service weighs in favor of granting the injunctionrdquo)

The third factor is the likelihood that the plaintiff will prevail on the merits Although there is limited information concerning the merits of the action the nonprofit alleges that the federal statute (NEPA) requires an environmental impact statement and further states that the USFS created no environmental impact analysis or statement at all Assuming that those

25

Federal Civil Procedure Analysis

allegations are correct it seems plausible to conclude that the nonprofit will be able to show a likelihood of success on the merits

Finally courts deciding whether or not to issue preliminary injunctive relief are to consider the public interest ldquoFocusing on this factor is another way of inquiring whether there are policy considerations that bear on whether the order should issuerdquo 11C WRIGHT ET AL supra sect 29484 at 214 If the court concludes that the nonprofit is likely to succeed on its NEPA claim because the USFS wrongfully failed to conduct an environmental impact assessment it is likely to find that the public interest would be served by restraining the USFS from proceeding with logging in a national forest See Heartwood Inc v US Forest Service 73 F Supp 2d 962 979 (SD Ill 1999) affrsquod on other grounds 230 F3d 947 (7th Cir 2000) (ldquoviolations by federal agencies of NEPArsquos provisions as established by Congress harm the public as well as the environmentrdquo)

Thus a court is very likely to grant a preliminary injunction if it concludes that the nonprofit has a significant likelihood of success on the merits

26

EVIDENCE ANALYSIS (Evidence ID IIA amp C)

ANALYSIS

Legal Problems

(1) Under what circumstances can evidence of prior convictions be used to impeach a witnessrsquos credibility in a civil case

(1)(a) May the inmatersquos credibility be impeached by evidence of a 12-year-old felony drug conviction if he was released from prison 9 years ago

(1)(b) May the inmatersquos credibility be impeached by evidence of an 8-year-old misdemeanor perjury conviction that was punishable by 1 year in jail if he pleaded guilty and was sentenced only to pay a $5000 fine

(1)(c) May the inmatersquos credibility be impeached by evidence of a 7-year-old sexual assault conviction if the inmate is still serving a 10-year prison sentence and the victim was his 13-year-old daughter

(2)(a) May the guardrsquos credibility be impeached by cross-examination regarding specific instances of misconduct (ie lying on his reacutesumeacute) relevant to credibility

(2)(b) May the guardrsquos credibility be impeached by admission of extrinsic evidence (his reacutesumeacute and academic transcript) offered to prove specific instances of misconduct relevant to credibility

DISCUSSION

Summary

Under the Federal Rules of Evidence witnesses can be impeached with evidence of prior convictions andor specific instances of misconduct Whether evidence of prior convictions should be admitted to impeach generally depends on the nature of the crime the amount of time that has passed and (only in criminal cases) whether the ldquowitnessrdquo is the defendant FED R EVID 609(a)

In this civil case evidence of the inmatersquos conviction for distribution of marijuana should be admitted to impeach the inmate because he was convicted of a felony and was released from prison fewer than 10 years ago FED R EVID 609(a)(1) Credibility is critically important in this case because the jury will hear conflicting testimony from the two disputing parties and there were no other eyewitnesses to the altercation Under Rule 609(a)(1) the inmatersquos conviction should be admitted because it has some bearing on his credibility and its probative value is not substantially outweighed by concerns of unfair prejudice confusion or delay Id

Evidence of the inmatersquos misdemeanor conviction for perjury must be admitted because the crime ldquorequired provingmdashor the witnessrsquos admittingmdasha dishonest act or false statementrdquo by the inmate FED R EVID 609(a)(2)

27

Evidence Analysis

Evidence of the inmatersquos felony conviction for sexual assault should be excluded because its probative value is substantially outweighed by the danger of unfair prejudice to the inmate based on the heinous nature of the crime FED R EVID 609(a)(1) In the alternative the judge could limit the evidence relating to this conviction by excluding details of the inmatersquos crime

In all civil (and criminal) cases witnesses can also be impeached with evidence of specific instances of prior misconduct that did not result in a conviction FED R EVID 608(b) Pursuant to Rule 608(b) misconduct probative of untruthfulness can be inquired into on cross-examination but cannot be proved through extrinsic evidence Id Thus the inmatersquos counsel should be permitted to cross-examine the guard regarding the false statement in the guardrsquos reacutesumeacute However extrinsic evidence of the guardrsquos misconduct (ie the guardrsquos authenticated reacutesumeacute and transcript from the local college) should not be admitted even if the guard denies wrongdoing or refuses to answer cross-examination questions about these matters Id

Point One (10) The Federal Rules of Evidence permit impeachment of witnesses with evidence of prior convictions

Whether convictions should be admitted to impeach generally depends on the nature of the crime the amount of time that has passed and (only in criminal cases) whether the ldquowitnessrdquo is the defendant FED R EVID 609(a) Under Rule 609(a) evidence of prior convictions may be admitted for the purpose of ldquoattacking a witnessrsquos character for truthfulnessrdquo Id

There are two basic types of convictions that can be admitted for the purpose of impeachment

(1) convictions for crimes ldquopunishable by death or by imprisonment for more than one yearrdquo (which generally correlates to ldquofeloniesrdquo) FED R EVID 609(a)(1) and (2) convictions ldquofor any crimes regardless of the punishment if the court can readily determine that establishing the elements of the crime required provingmdashor the witnessrsquos admittingmdasha dishonest act or false statementrdquo FED R EVID 609(a)(2)

Pursuant to Rule 609(a)(1) in civil cases the admission of evidence of a felony conviction is ldquosubject to Rule 403 [which says that a court may exclude relevant evidence if its probative value is substantially outweighed by other factors]rdquo FED R EVID 609(a)(1) However Rule 403 does not protect the witness against admission of prior convictions involving dishonestymdashwhich must be admitted by the court FED R EVID 609(a)(2)

Finally Federal Rule of Evidence 609(b) contains the presumption that a conviction that is more than 10 years old or where more than 10 years has passed since the witnessrsquos release from confinement (whichever is later) should not be admitted unless ldquoits probative value supported by specific facts and circumstances substantially outweighs its prejudicial effectrdquo and the proponent has provided the adverse party with reasonable written notice FED R EVID 609(b)

Point One(a) (25) The court should admit evidence of the inmatersquos 12-year-old felony marijuana distribution conviction

The inmatersquos conviction for marijuana distribution was for a felony punishable by imprisonment for more than one year See FED R EVID 609(a)(1) Moreover although the conviction was 12 years ago the 10-year time limit of Rule 609(b) is not exceeded because that time limit runs

28

Evidence Analysis

from the date of either ldquothe witnessrsquos conviction or release from confinement for it whichever is laterrdquo FED R EVID 609(b) Because the inmate served three years in prison he was released from confinement nine years ago

However pursuant to Rule 609(a)(1) the admission of felony convictions to impeach a witness in a civil case is ldquosubject to Rule 403rdquo FED R EVID 609(a)(1) Neither Rule 609(a) nor the advisory committee notes specify which factors courts should consider when balancing the probative value of a conviction against the dangers identified in Rule 403 (which include (1) unfair prejudice (2) confusion of the issues (3) misleading the jury (4) waste of time or undue delay and (5) needless presentation of cumulative evidence) FED R EVID 403

In this case credibility is very important because the evidence consists primarily of the testimony of the disputing parties and there were no other eyewitnesses to the altercation This enhances the probative value of any evidence bearing on the inmatersquos credibility A court is likely to conclude that the inmatersquos prior felony drug conviction is relevant to his credibility See eg United States v Brito 427 F3d 53 64 (1st Cir 2005) (ldquoPrior drug-trafficking crimes are generally viewed as having some bearing on veracityrdquo) Although the probative value of any conviction diminishes with age see eg United States v Brewer 451 F Supp 50 53 (ED Tenn 1978) the inmatersquos ongoing problems with the law suggest that he has continued (and even escalated) his criminal behavior over the past nine years The court should admit this evidence because its probative value is not substantially outweighed by any Rule 403 concerns Specifically any prejudice to the inmate would be slight because the conviction is unrelated to the altercation at issue and the conviction was not for a heinous crime that might inflame the jury

[NOTE Whether an examinee identifies the jury instruction as containing a ldquoconclusiverdquo or ldquomandatoryrdquo presumption is less important than the examineersquos analysis of the constitutional infirmities]

Point One(b) (15) The court must admit evidence of the inmatersquos eight-year-old misdemeanor conviction because perjury is a crime of dishonesty

Rule 609(a)(2) provides that evidence of a criminal conviction ldquomust be admitted if the court can readily determine that establishing the elements of the crime required provingmdashor the witnessrsquos admittingmdasha dishonest act or false statementrdquo FED R EVID 609(a)(2) The inmatersquos conviction for perjury would have necessarily required proving that the inmate engaged in an act of dishonesty This conviction occurred within the past 10 years so it ldquomust be admittedrdquo because in contrast to Rule 609(a)(1) (discussed in Point One(a)) admission under Rule 609(a)(2) is mandatory and not subject to Rule 403

Point One(c) (20) The court should exclude evidence of the inmatersquos seven-year-old felony sexual assault conviction because the probative value of this evidence is substantially outweighed by the danger of unfair prejudice In the alternative the details of the prior conviction could be excluded

The inmatersquos conviction for felony sexual assault was seven years ago and he has not yet been released from incarceration so Rule 609(a) but not 609(b) is applicable here FED R EVID 609(a) This conviction is therefore admissible to impeach the inmate unless its probative value is substantially outweighed by the danger of unfair prejudice or any other Rule 403 concern Id

29

Evidence Analysis

Sex crimes are generally not considered relevant to credibility see Hopkins v State 639 So 2d 1247 1254 (Miss 1993) so the probative value of this conviction is relatively low Moreover the heinous nature of the inmatersquos crime (sexual assault on his daughter) makes the danger of unfair prejudice to the inmate very high Thus the court should exclude evidence of the conviction because it was for a heinous offense that is likely to inflame the jury and it has little bearing on credibility See eg United States v Beahm 664 F2d 414 419 (4th Cir 1981)

As an alternative to excluding this evidence the judge could minimize the unfair prejudice to the inmate by permitting limited cross-examination but refusing to allow specific questions about the nature of the inmatersquos conviction For example a court could limit cross-examination to the fact that the inmate was convicted of a ldquofelonyrdquo or perhaps that he was convicted of a ldquosexual assaultrdquo without identifying the victim However because evidence of the inmatersquos prior convictions can be admitted solely for the purpose of enabling the jury to assess his credibility and because his two earlier convictions should have already been admitted the court should exclude all evidence of the felony sexual assault conviction

Point Two(a) (15) The court should permit the inmatersquos counsel to cross-examine the guard regarding the false statement in his reacutesumeacute because the guardrsquos misconduct bears on his truthfulness

The inmate wishes to cross-examine the guard about his prior dishonest behaviormdashlying on his reacutesumeacutemdashthat did not involve a criminal conviction Rule 608(b) allows witnesses to be cross-examined about specific instances of prior non-conviction misconduct probative of untruthfulness ldquoin order to attack the witnessrsquos character for truthfulnessrdquo FED R EVID 608(b)

The courtrsquos decision to allow cross-examination about the guardrsquos prior dishonest behavior depends on the probative value of such evidence balanced against the danger of unfair prejudice to the guard or any other Rule 403 concern FED R EVID 403 Here the guardrsquos false statement on his reacutesumeacute that he obtained a degree in Criminal Justice is highly probative of his untruthfulness because it grossly misrepresents his actual academic record was made recently and was made with the intent to deceive Because the probative value of this evidence is very strong and is not substantially outweighed by any Rule 403 concerns cross-examination of the guard on this topic should be permitted The court may also consider it fair to permit this cross-examination of the guard on these matters assuming that one or more of the inmatersquos prior convictions have been admitted to impeach his credibility

Point Two(b) (15) The court should exclude extrinsic evidence of the guardrsquos non-conviction misconduct even if the guard denies wrongdoing or refuses to answer questions about the matter

Although Rule 608(b) allows cross-examination about specific instances of prior misconduct probative of untruthfulness ldquoextrinsic evidencerdquo offered to prove such misconduct is not admissible FED R EVID 608(b) The rationale for this rule is that allowing the introduction of extrinsic evidence of prior misconduct by witnesses when these acts are relevant only to the witnessesrsquo truthfulness and not to the main issues in the case would create too great a risk of confusing the jury and unduly delaying the trial The court does not have discretion to admit this extrinsic evidence See eg United States v Elliot 89 F3d 1360 1368 (8th Cir 1996)

30

Evidence Analysis

Here the inmatersquos counsel may cross-examine the guard about the false statement on his reacutesumeacute However the inmatersquos counsel must accept the guardrsquos response Even if the guard denies wrongdoing or refuses to answer questions about the matter the inmatersquos counsel cannot introduce the guardrsquos reacutesumeacute or the transcript from the local college to prove the guardrsquos misconduct

31

CORPORATIONS ANALYSIS (Corporations VA2 IX)

ANALYSIS

Legal Problems

(1) Do shareholders have the authority to amend a corporationrsquos bylaws with respect to director nominations

(2) Do board-approved bylaws on a particular subject here nomination of directors preempt subsequent conflicting bylaw amendments by shareholders

(3) Is a suit challenging both managementrsquos refusal to include the proposed bylaw amendment in Megarsquos proxy statement and the boardrsquos amendment of the bylaws dealing with nomination of directors a direct or derivative suit

DISCUSSION

Summary

The voting and litigation rights of the shareholders of Mega are subject to the provisions of the Model Business Corporations Act (MBCA)

The investorrsquos proposed bylaw provision is not inconsistent with state law Under the MBCA shareholders may amend the bylaws when the amendment deals with a proper matter for the corporationrsquos bylaws such as procedures for nominating directors

The Mega boardrsquos bylaw amendment does not preempt the investorrsquos proposed bylaw provision or the Mega shareholdersrsquo power to approve it While shareholders can limit the boardrsquos power to amend or repeal the bylaws the board cannot limit the shareholdersrsquo power

Whether the investor must make a demand on Megarsquos board depends on how the investor frames its claim If the investor claims a violation of shareholder voting rights the claim is direct and pre-suit demand on the board is not required If on the other hand the investor claims that the directors violated their fiduciary duties by amending the bylaws to entrench themselves the claim is derivative and a pre-suit demand is required

Point One (30) Shareholders may amend the corporationrsquos bylaws where the proposed bylaw provision relates to procedural matters typically included in the bylaws such as the nomination of directors

Internal affairs of the corporation such as the conduct of shareholder meetings and election of directors are subject to the corporate law of the state of incorporation See McDermott Inc v Lewis 531 A2d 206 (Del 1987) (applying law of jurisdiction where corporation was incorporated in case involving voting rights) This statersquos corporate statute is modeled on the MBCA

Under the MBCA ldquoshareholders may amend the corporationrsquos bylawsrdquo MBCA sect 1020(a) Thus the only question is whether the bylaws can specify the procedures for shareholder nomination of directors

32

Corporations Analysis

The MBCA states that the bylaws ldquomay contain any provision that is not inconsistent with law or the articles of incorporationrdquo MBCA sect 206(b) In addition the MBCA was revised in 2009 to address shareholder nomination of directors in public corporations (known as ldquoproxy accessrdquo) and specifies that the bylaws ldquomay contain a requirement that the corporation include in its [proxy materials] one or more individuals nominated by a shareholderrdquo MBCA sect 206(c)(1) see Committee on Corporate Laws ABA Section of Business Law Report on the Roles of Boards of Directors and Shareholders of Publicly Owned Corporations and Changes to the Model Business Corporations ActmdashAdoption of Shareholder Proxy Access Amendments to Chapters 2 and 10 65 BUS LAWYER 1105 (2010)

The inclusion of director-nomination procedures in the bylaws is consistent with practice and is recognized by the Delaware courts whose views on corporate law carry significant weight Typically the procedures for nomination of directors are found in the bylaws See 1 COX amp HAZEN TREATISE ON THE LAW OF CORPORATIONS sect 312 (3d ed 2011) see also 4 FLETCHER CORP FORMS ANN PART III ch 21 (2013) (including sample bylaws that permit nomination of directors by shareholders) The Delaware Supreme Court has confirmed that the bylaws may ldquodefine the process and proceduresrdquo for director elections See CA Inc v AFSCME Employees Pension Plan 953 A2d 227 (Del 2008) (concluding that bylaw amendment requiring reimbursement of election expenses to certain successful shareholder nominators is ldquoproper subjectrdquo under Delaware law)

[NOTE The question of the proper scope of the bylaws can be answered using the more general MBCA sect 206(b) or the 2009 MBCA revision adding sect 206(c)(1) (adopted in CT ME VA) In addition some examinees might raise the point that shareholder proposals may not compel the board to take action such as by including shareholder nominations in the companyrsquos proxy materials on the theory that the ldquobusiness and affairsrdquo of the corporation are to be managed by the board See MBCA sect 801(b) Although shareholders are generally limited to adopting precatory resolutions that recommend or encourage board action this limitation does not apply when shareholders have specific authority to take binding action on their ownmdashsuch as to amend the bylaws]

Point Two (30) Shareholders can amend (or repeal) board-approved bylaws Further shareholders can limit the boardrsquos power to later amend and repeal a shareholder-approved bylaw

Under the MBCA shareholders have the power to amend the bylaws See Point One The board shares this power with the shareholders unless (1) the corporationrsquos articles ldquoreserve that power exclusively to the shareholdersrdquo or (2) ldquothe shareholders in amending repealing or adopting a bylaw expressly provide that the board of directors may not amend repeal or reinstate that bylawrdquo See MBCA sect 1020(b)

Shareholder-approved bylaw provisions can amend or repeal existing bylaw provisions whether originally approved by the board or by shareholders See ALAN R PALMITER CORPORATIONS EXAMPLES AND EXPLANATIONS sect 713 (7th ed 2012) Thus the Mega boardrsquos bylaw amendmentmdashwhich set more demanding thresholds for shareholder nomination of directors than the investorrsquos proposed bylaw provisionmdashwould be superseded (repealed) if Megarsquos shareholders were to approve the investorrsquos proposal

Further a shareholder-approved bylaw generally can limit the power of the board to later amend or repeal it See MBCA sect 1020(b)(2) Thus if Megarsquos shareholders approved the bylaw

33

Corporations Analysis

provision proposed by the investor Megarsquos board could not repeal the provision because it includes a ldquono board repealrdquo clause

The revision to the MBCA in 2009 dealing with shareholder proxy access does not change this conclusion That revision specifies that a shareholder-approved bylaw dealing with director nominations may not limit the boardrsquos power to amend add or repeal ldquoany procedure or condition to such a bylaw in order to provide for a reasonable practicable and orderly processrdquo MBCA sect 206(d) Thus according to the revision if shareholders approve a bylaw amendment that limits further board changes the board would nonetheless retain the power to ldquotinkerrdquo with the bylaw to safeguard the voting process but could not repeal the shareholder-approved bylaw The Official Comment to MBCA sect 206(d) makes clear that the revision is ldquonot intended to allow the board of directors to frustrate the purpose of the shareholder-adopted proxy access provisionrdquo Thus if Megarsquos shareholders were to approve the bylaw provision proposed by the investor Megarsquos board could only amend the provision regarding its procedures or conditions in a manner consistent with its purpose of permitting proxy access for Megarsquos shareholders

[NOTE The boardrsquos attempted interference with a shareholder voting initiative may also have been a violation of the directorsrsquo fiduciary duties See Blasius Indus Inc v Atlas Corp 564 A2d 651 (Del Ch 1988) (finding that directors breached their fiduciary duties by amending bylaws and expanding size of board to thwart insurgentrsquos plan to amend bylaws and seat a majority of new directors) The call however asks examinees to consider whether shareholders or the board have ldquoprecedencerdquo over amending the corporate bylaws Thus an examineersquos answer should be framed in terms of ldquopowerrdquo and not ldquodutyrdquo]

Point Three (40) The investor need not make a demand on the board if the investor states a direct claim such as an allegation that the board interfered with the investorrsquos right to amend the bylaws But the investor must make a demand on the board if the investor states a derivative claim (on behalf of the corporation) such as an allegation that the directors sought to entrench themselves by interfering with the proposed proxy access

The MBCA generally requires that shareholders make a demand on the board of directors before initiation of a derivative suit MBCA sect 742 (shareholder may not bring derivative proceeding until written demand has been made on corporation and 90 days have expired) A derivative suit is essentially two suits in one where the plaintiff-shareholder seeks to bring on behalf of the corporation a claim that vindicates corporate rights usually based on violation of fiduciary duties PALMITER supra sect 1811 (6th ed 2009) The demand permits the board to investigate the situation identified by the shareholder and take suitable action No demand on the board is required however if the shareholder brings a direct suit to vindicate the shareholderrsquos own rights not those of the corporation

Is the suit brought by the investor derivative or direct The MBCA defines a ldquoderivative proceedingrdquo as one brought ldquoin the right of a domestic corporationrdquo MBCA sect 740(1) Thus the answer to how the investorrsquos suit should be characterized turns on what rights the investor seeks to vindicate If the investor frames its claim as one of fiduciary breach by directorsmdashfor example for failing to become adequately informed about voting procedures or for seeking to entrench themselves in office by manipulating the voting structure to avoid a shareholder insurgencymdashthen the suit is ldquoderivativerdquo and the investor must make a demand on the board See MBCA Ch 7 Subch D Introductory Comment (ldquothe derivative suit has historically been the principal method of challenging allegedly illegal action by managementrdquo)

34

Corporations Analysis

If however the investor frames its claim as one to vindicate shareholder rights the suit is direct and no demand is required For many courts the direct-derivative question turns on who is injured and who is to receive the relief sought by the plaintiff-shareholders See Tooley v Donaldson Lufkin amp Jenrette Inc 845 A2d 1031 (Del 2004) (characterizing a merger-delay claim as direct because delay of merger only harmed shareholders not corporation) Thus if the investor claims that managementrsquos refusal to include its proposed bylaw amendment in the corporationrsquos proxy materials violates its shareholder rights to initiate corporate governance reforms the suit will be direct Courts have not questioned the ability of shareholders to bring direct suits challenging board action to exclude their proposed bylaw amendments from the corporationrsquos proxy materials See JANA Master Fund Ltd v CNET Networks Inc 954 A2d 335 (Del Ch 2008) (upholding shareholderrsquos direct challenge to boardrsquos interpretation of advance-notice bylaw) Chesapeake Corp v Shore 771 A2d 293 (Del Ch 2000) (upholding shareholderrsquos direct challenge to actions by board that effectively prevented it from proposing bylaw amendments in contest for control)

Is the way that the investor frames its claim conclusive Courts have permitted shareholder-plaintiffs to challenge a transaction in a direct suit even though the same transaction could also be challenged as a fiduciary breach See Eisenberg v Flying Tiger Line Inc 451 F2d 267 (2d Cir 1971) (permitting direct suit challenging a corporate reorganization as a dilution of shareholder voting power even though reorganization may have involved conflicts of interest and thus constituted a fiduciary breach) Thus the investorrsquos choice to pursue a claim challenging the legality of managementrsquos decision to exclude the investorrsquos proposal from the corporationrsquos proxy materialsmdashrather than a possible breach of fiduciary dutymdashis likely to be respected See 3 COX amp HAZEN supra sect 153 (describing situations in which a claim can be framed as derivative or direct)

[NOTE Some issues under Delaware corporate law regarding pre-suit demand are not relevant here For example whether the Mega directors are independent and disinterested is not relevant to the MBCA requirement of a pre-suit demand As the Official Comment to MBCA sect 742 points out the MBCArsquos requirement of ldquouniversal demandrdquo gives the board ldquothe opportunity to reexamine the act complained of in the light of a potential lawsuit and take corrective actionrdquo even when the directors might be non-independent or have conflicts of interest

Nor is it relevant to the MBCA pre-suit demand requirement that the statutory 90-day waiting period may be onerous The first paragraph of MBCA sect 742 requires a pre-suit demand without exception the second paragraph of the section imposes a 90-day waiting period before a derivative suit may be brought which can be shortened if the board rejects the demand or ldquoirreparable injury to the corporation would result by waiting for the expiration of the 90-day periodrdquo The call as written asks only whether a pre-suit demand should be made and does not ask examinees to address whether the post-demand waiting period should be shortened under the ldquoirreparable injuryrdquo standard]

35

National Conference of Bar Examiners 302 South Bedford Street | Madison WI 53703-3622 Phone 608-280-8550 | Fax 608-280-8552 | TDD 608-661-1275

wwwncbexorg e-mail contactncbexorg

  • Preface
  • Description of the MEE
  • Instructions
  • July 2014 Questions
    • CRIMINAL LAW AND PROCEDURE QUESTION
    • CONTRACTS QUESTION
    • FAMILY LAW QUESTION
    • FEDERAL CIVIL PROCEDURE QUESTION
    • EVIDENCE QUESTION
    • CORPORATIONS QUESTION
      • July 2014 Analyses
        • CRIMINAL LAW AND PROCEDURE ANALYSIS
        • CONTRACTS ANALYSIS
        • FAMILY LAW ANALYSIS
        • FEDERAL CIVIL PROCEDURE ANALYSIS
        • EVIDENCE ANALYSIS
        • CORPORATIONS ANALYSIS
            • ltlt13 ASCII85EncodePages false13 AllowTransparency false13 AutoPositionEPSFiles true13 AutoRotatePages None13 Binding Left13 CalGrayProfile (Dot Gain 20)13 CalRGBProfile (sRGB IEC61966-21)13 CalCMYKProfile (US Web Coated 050SWOP051 v2)13 sRGBProfile (sRGB IEC61966-21)13 CannotEmbedFontPolicy Error13 CompatibilityLevel 1413 CompressObjects Tags13 CompressPages true13 ConvertImagesToIndexed true13 PassThroughJPEGImages true13 CreateJobTicket false13 DefaultRenderingIntent Default13 DetectBlends true13 DetectCurves 0000013 ColorConversionStrategy CMYK13 DoThumbnails false13 EmbedAllFonts true13 EmbedOpenType false13 ParseICCProfilesInComments true13 EmbedJobOptions true13 DSCReportingLevel 013 EmitDSCWarnings false13 EndPage -113 ImageMemory 104857613 LockDistillerParams false13 MaxSubsetPct 10013 Optimize true13 OPM 113 ParseDSCComments true13 ParseDSCCommentsForDocInfo true13 PreserveCopyPage true13 PreserveDICMYKValues true13 PreserveEPSInfo true13 PreserveFlatness true13 PreserveHalftoneInfo false13 PreserveOPIComments true13 PreserveOverprintSettings true13 StartPage 113 SubsetFonts true13 TransferFunctionInfo Apply13 UCRandBGInfo Preserve13 UsePrologue false13 ColorSettingsFile ()13 AlwaysEmbed [ true13 ]13 NeverEmbed [ true13 ]13 AntiAliasColorImages false13 CropColorImages true13 ColorImageMinResolution 30013 ColorImageMinResolutionPolicy OK13 DownsampleColorImages true13 ColorImageDownsampleType Bicubic13 ColorImageResolution 30013 ColorImageDepth -113 ColorImageMinDownsampleDepth 113 ColorImageDownsampleThreshold 15000013 EncodeColorImages true13 ColorImageFilter DCTEncode13 AutoFilterColorImages true13 ColorImageAutoFilterStrategy JPEG13 ColorACSImageDict ltlt13 QFactor 01513 HSamples [1 1 1 1] VSamples [1 1 1 1]13 gtgt13 ColorImageDict ltlt13 QFactor 01513 HSamples [1 1 1 1] VSamples [1 1 1 1]13 gtgt13 JPEG2000ColorACSImageDict ltlt13 TileWidth 25613 TileHeight 25613 Quality 3013 gtgt13 JPEG2000ColorImageDict ltlt13 TileWidth 25613 TileHeight 25613 Quality 3013 gtgt13 AntiAliasGrayImages false13 CropGrayImages true13 GrayImageMinResolution 30013 GrayImageMinResolutionPolicy OK13 DownsampleGrayImages true13 GrayImageDownsampleType Bicubic13 GrayImageResolution 30013 GrayImageDepth -113 GrayImageMinDownsampleDepth 213 GrayImageDownsampleThreshold 15000013 EncodeGrayImages true13 GrayImageFilter DCTEncode13 AutoFilterGrayImages true13 GrayImageAutoFilterStrategy JPEG13 GrayACSImageDict ltlt13 QFactor 01513 HSamples [1 1 1 1] VSamples [1 1 1 1]13 gtgt13 GrayImageDict ltlt13 QFactor 01513 HSamples [1 1 1 1] VSamples [1 1 1 1]13 gtgt13 JPEG2000GrayACSImageDict ltlt13 TileWidth 25613 TileHeight 25613 Quality 3013 gtgt13 JPEG2000GrayImageDict ltlt13 TileWidth 25613 TileHeight 25613 Quality 3013 gtgt13 AntiAliasMonoImages false13 CropMonoImages true13 MonoImageMinResolution 120013 MonoImageMinResolutionPolicy OK13 DownsampleMonoImages true13 MonoImageDownsampleType Bicubic13 MonoImageResolution 120013 MonoImageDepth -113 MonoImageDownsampleThreshold 15000013 EncodeMonoImages true13 MonoImageFilter CCITTFaxEncode13 MonoImageDict ltlt13 K -113 gtgt13 AllowPSXObjects false13 CheckCompliance [13 None13 ]13 PDFX1aCheck false13 PDFX3Check false13 PDFXCompliantPDFOnly false13 PDFXNoTrimBoxError true13 PDFXTrimBoxToMediaBoxOffset [13 00000013 00000013 00000013 00000013 ]13 PDFXSetBleedBoxToMediaBox true13 PDFXBleedBoxToTrimBoxOffset [13 00000013 00000013 00000013 00000013 ]13 PDFXOutputIntentProfile ()13 PDFXOutputConditionIdentifier ()13 PDFXOutputCondition ()13 PDFXRegistryName ()13 PDFXTrapped False1313 CreateJDFFile false13 Description ltlt13 ARA 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 BGR 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 CHS ltFEFF4f7f75288fd94e9b8bbe5b9a521b5efa7684002000410064006f006200650020005000440046002065876863900275284e8e9ad88d2891cf76845370524d53705237300260a853ef4ee54f7f75280020004100630072006f0062006100740020548c002000410064006f00620065002000520065006100640065007200200035002e003000204ee553ca66f49ad87248672c676562535f00521b5efa768400200050004400460020658768633002gt13 CHT ltFEFF4f7f752890194e9b8a2d7f6e5efa7acb7684002000410064006f006200650020005000440046002065874ef69069752865bc9ad854c18cea76845370524d5370523786557406300260a853ef4ee54f7f75280020004100630072006f0062006100740020548c002000410064006f00620065002000520065006100640065007200200035002e003000204ee553ca66f49ad87248672c4f86958b555f5df25efa7acb76840020005000440046002065874ef63002gt13 CZE 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 DAN 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 DEU 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 ESP ltFEFF005500740069006c0069006300650020006500730074006100200063006f006e0066006900670075007200610063006900f3006e0020007000610072006100200063007200650061007200200064006f00630075006d0065006e0074006f00730020005000440046002000640065002000410064006f0062006500200061006400650063007500610064006f00730020007000610072006100200069006d0070007200650073006900f3006e0020007000720065002d0065006400690074006f007200690061006c00200064006500200061006c00740061002000630061006c0069006400610064002e002000530065002000700075006500640065006e00200061006200720069007200200064006f00630075006d0065006e0074006f00730020005000440046002000630072006500610064006f007300200063006f006e0020004100630072006f006200610074002c002000410064006f00620065002000520065006100640065007200200035002e003000200079002000760065007200730069006f006e0065007300200070006f00730074006500720069006f007200650073002egt13 ETI 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 FRA 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 GRE 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 HEB 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 HRV (Za stvaranje Adobe PDF dokumenata najpogodnijih za visokokvalitetni ispis prije tiskanja koristite ove postavke Stvoreni PDF dokumenti mogu se otvoriti Acrobat i Adobe Reader 50 i kasnijim verzijama)13 HUN 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 ITA 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 JPN ltFEFF9ad854c18cea306a30d730ea30d730ec30b951fa529b7528002000410064006f0062006500200050004400460020658766f8306e4f5c6210306b4f7f75283057307e305930023053306e8a2d5b9a30674f5c62103055308c305f0020005000440046002030d530a130a430eb306f3001004100630072006f0062006100740020304a30883073002000410064006f00620065002000520065006100640065007200200035002e003000204ee5964d3067958b304f30533068304c3067304d307e305930023053306e8a2d5b9a306b306f30d530a930f330c8306e57cb30818fbc307f304c5fc59808306730593002gt13 KOR ltFEFFc7740020c124c815c7440020c0acc6a9d558c5ec0020ace0d488c9c80020c2dcd5d80020c778c1c4c5d00020ac00c7a50020c801d569d55c002000410064006f0062006500200050004400460020bb38c11cb97c0020c791c131d569b2c8b2e4002e0020c774b807ac8c0020c791c131b41c00200050004400460020bb38c11cb2940020004100630072006f0062006100740020bc0f002000410064006f00620065002000520065006100640065007200200035002e00300020c774c0c1c5d0c11c0020c5f40020c2180020c788c2b5b2c8b2e4002egt13 LTH 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 LVI 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 NLD (Gebruik deze instellingen om Adobe PDF-documenten te maken die zijn geoptimaliseerd voor prepress-afdrukken van hoge kwaliteit De gemaakte PDF-documenten kunnen worden geopend met Acrobat en Adobe Reader 50 en hoger)13 NOR 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 POL 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 PTB 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 RUM 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 RUS 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 SKY ltFEFF0054006900650074006f0020006e006100730074006100760065006e0069006100200070006f0075017e0069007400650020006e00610020007600790074007600e100720061006e0069006500200064006f006b0075006d0065006e0074006f0076002000410064006f006200650020005000440046002c0020006b0074006f007200e90020007300610020006e0061006a006c0065007001610069006500200068006f0064006900610020006e00610020006b00760061006c00690074006e00fa00200074006c0061010d00200061002000700072006500700072006500730073002e00200056007900740076006f00720065006e00e900200064006f006b0075006d0065006e007400790020005000440046002000620075006400650020006d006f017e006e00e90020006f00740076006f00720069016500200076002000700072006f006700720061006d006f006300680020004100630072006f00620061007400200061002000410064006f00620065002000520065006100640065007200200035002e0030002000610020006e006f0076016100ed00630068002egt13 SLV ltFEFF005400650020006e006100730074006100760069007400760065002000750070006f0072006100620069007400650020007a00610020007500730074007600610072006a0061006e006a006500200064006f006b0075006d0065006e0074006f0076002000410064006f006200650020005000440046002c0020006b006900200073006f0020006e0061006a007000720069006d00650072006e0065006a016100690020007a00610020006b0061006b006f0076006f00730074006e006f0020007400690073006b0061006e006a00650020007300200070007200690070007200610076006f0020006e00610020007400690073006b002e00200020005500730074007600610072006a0065006e006500200064006f006b0075006d0065006e0074006500200050004400460020006a00650020006d006f0067006f010d00650020006f0064007000720065007400690020007a0020004100630072006f00620061007400200069006e002000410064006f00620065002000520065006100640065007200200035002e003000200069006e0020006e006f00760065006a01610069006d002egt13 SUO 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 SVE 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 TUR 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 UKR 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 ENU (Use these settings to create Adobe PDF documents best suited for high-quality prepress printing Created PDF documents can be opened with Acrobat and Adobe Reader 50 and later)13 gtgt13 Namespace [13 (Adobe)13 (Common)13 (10)13 ]13 OtherNamespaces [13 ltlt13 AsReaderSpreads false13 CropImagesToFrames true13 ErrorControl WarnAndContinue13 FlattenerIgnoreSpreadOverrides false13 IncludeGuidesGrids false13 IncludeNonPrinting false13 IncludeSlug false13 Namespace [13 (Adobe)13 (InDesign)13 (40)13 ]13 OmitPlacedBitmaps false13 OmitPlacedEPS false13 OmitPlacedPDF false13 SimulateOverprint Legacy13 gtgt13 ltlt13 AddBleedMarks false13 AddColorBars false13 AddCropMarks false13 AddPageInfo false13 AddRegMarks false13 ConvertColors ConvertToCMYK13 DestinationProfileName ()13 DestinationProfileSelector DocumentCMYK13 Downsample16BitImages true13 FlattenerPreset ltlt13 PresetSelector MediumResolution13 gtgt13 FormElements false13 GenerateStructure false13 IncludeBookmarks false13 IncludeHyperlinks false13 IncludeInteractive false13 IncludeLayers false13 IncludeProfiles false13 MultimediaHandling UseObjectSettings13 Namespace [13 (Adobe)13 (CreativeSuite)13 (20)13 ]13 PDFXOutputIntentProfileSelector DocumentCMYK13 PreserveEditing true13 UntaggedCMYKHandling LeaveUntagged13 UntaggedRGBHandling UseDocumentProfile13 UseDocumentBleed false13 gtgt13 ]13gtgt setdistillerparams13ltlt13 HWResolution [2400 2400]13 PageSize [612000 792000]13gtgt setpagedevice13

Page 13: July 2014 MEE Questions and Analyses - NCBE...This publication includes the questions and analyses from the July 2014 MEE. (In the actual test, the questions are simply numbered rather

CORPORATIONS QUESTION

Mega Inc is a publicly traded corporation incorporated in a state whose corporate statute is modeled on the Model Business Corporation Act (MBCA) Megarsquos articles of incorporation do not address the election of directors or amendment of the bylaws by shareholders

Well within the deadline for the submission of shareholder proposals for the upcoming annual shareholdersrsquo meeting an investor who was a large and long-standing shareholder of Mega submitted a proposed amendment to Megarsquos bylaws The proposal which the investor asked to be included in the corporationrsquos proxy materials and voted on at the upcoming shareholdersrsquo meeting read as follows

Section 20 The Corporation shall include in its proxy materials (including the proxy ballot) for a shareholdersrsquo meeting at which directors are to be elected the name of a person nominated for election to the Board of Directors by a shareholder or group of shareholders that beneficially have owned 3 or more of the Corporationrsquos outstanding common stock for at least one year

This Section shall supersede any inconsistent provision in these Bylaws and may not be amended or repealed by the Board of Directors without shareholder approval

Megarsquos management decided to exclude the investorrsquos proposal from the corporationrsquos proxy materials and explained its reasons in a letter to the investor

The investorrsquos proposed bylaw provision would be inconsistent with relevant state law because the Board of Directors has the authority to manage the business and affairs of the Corporation Generally shareholders lack the authority to interfere with corporate management by seeking to create a method for the nomination and election of directors inconsistent with the method chosen by the Board of Directors

Furthermore at its most recent meeting the Board of Directors unanimously approved an amendment to the Corporationrsquos bylaws that provides for proxy access for director nominations by a shareholder or a group of shareholders holding at least 10 of the Corporationrsquos voting shares for at least three years This procedure takes precedence over any nomination methods that might be sought or approved by shareholders

The investor is considering bringing a suit challenging managementrsquos refusal to include the investorrsquos proposed bylaw provision and challenging the boardrsquos amendment of the bylaws at its recent meeting

1 Is the investorrsquos proposed bylaw provision inconsistent with state law Explain

2 If the investorrsquos proposed bylaw provision were approved by the shareholders would the bylaw amendment previously approved by the board take precedence over the investorrsquos proposed bylaw provision Explain

3 Must the investor make a demand on Megarsquos board of directors before bringing suit Explain

9

July 2014 MEE

ANALYSES

Contracts Family Law

Criminal Law and Procedure

Federal Civil Procedure Evidence

Corporations

CRIMINAL LAW AND PROCEDURE ANALYSIS (Criminal Law and Procedure VA B D)

ANALYSIS

Legal Problems

(1) Did the detective violate the suspectrsquos Sixth Amendment right to counsel when he questioned the suspect about the burglaries without the lawyer present given that the lawyer represented the suspect in an unrelated criminal matter

(2) Under Miranda did the suspect effectively invoke his right to counsel when he said ldquoI think I want my lawyer here before I talk to yourdquo

(3) Was the suspectrsquos waiver of his right to remain silent under Miranda valid

DISCUSSION

Summary

The Sixth Amendment right to counsel as applied to states through the Fourteenth Amendment is offense-specific Although the suspect had an attorney representing him on his pending assault charge he had no Sixth Amendment right to the assistance of counsel with respect to the five uncharged burglaries because formal adversarial proceedings had not yet commenced on those charges The suspectrsquos Sixth Amendment right to counsel was not violated by the detectiversquos failure to inform him that the lawyer was present or of the lawyerrsquos demands

However a person undergoing custodial interrogation also has an independent constitutional right to counsel during custodial interrogation under Miranda When a suspect invokes his right to counsel under Miranda custodial interrogation must immediately cease for a period of at least 14 days However the invocation of the right to counsel must be unambiguous and clearly convey that the suspect has requested counsel Here because the suspectrsquos statement ldquoI think I want my lawyer here before I talk to yourdquo was ambiguous he did not invoke his Miranda right to counsel

A waiver of rights must be knowing intelligent and voluntary Here the suspect waived his right to remain silent under Miranda when he signed the waiver form The fact that the detective did not correct the suspectrsquos assumption that the lawyer would need to drive to the jailmdashby telling him that the lawyer was in the waiting room and was demanding to see himmdashdid not affect the validity of the suspectrsquos waiver

Point One (35) The suspectrsquos Sixth Amendment right to counsel was not violated because the right does not attach on new charges until formal adversarial judicial proceedings have commenced on those charges

The Sixth Amendment as applied to the states through the Fourteenth Amendment provides that ldquo[i]n all criminal prosecutions the accused shall enjoy the right to have the Assistance of Counsel for his defenserdquo The right to counsel does not attach with respect to particular charges until formal adversarial judicial proceedings have commenced (ie ldquoat or after the initiation of

13

Criminal Law and Procedure Analysis

adversary judicial criminal proceedingsmdashwhether by way of formal charge preliminary hearing indictment information or arraignment [or in some states arrest warrant]rdquo McNeil v Wisconsin 501 US 171 175 (1991) (internal quotations omitted)) Once a suspectrsquos Sixth Amendment right to counsel has attached any attempts to ldquodeliberately elicitrdquo statements from him in the absence of his attorney violate the Sixth Amendment See Massiah v United States 377 US 201 (1964) Brewer v Williams 430 US 387 (1977)

The Sixth Amendment right to counsel is charge- or offense -specific Representation by counsel in one prosecution does not in itself guarantee counsel for uncharged offenses See McNeil 501 US at 175 Texas v Cobb 532 US 162 (2001) Here the suspectrsquos Sixth Amendment right to counsel had attached only for the pending aggravated assault charge The suspectrsquos right to counsel for the aggravated assault case did not guarantee counsel for the five unrelated and uncharged burglaries that were the subject of the detectiversquos interrogation Thus because formal adversarial judicial proceedings against the suspect for the uncharged burglaries had not begun he had no Sixth Amendment right to counsel

Finally the detectiversquos failure to inform the suspect of the lawyerrsquos presence and demands to speak with him does not implicate the suspectrsquos Sixth Amendment right to counsel which had not yet attached See id Moran v Burbine 475 US 412 428ndash31 (1986)

Point Two (30) The suspect did not effectively invoke his right to counsel under Miranda because his statement was not unambiguous

A suspect subject to custodial interrogation has a right to consult with counsel and to have an attorney present during questioning Miranda v Arizona 384 US 436 (1966) When a suspect invokes his right to counsel during an interrogation law enforcement must immediately cease all questioning See Edwards v Arizona 451 US 477 484ndash85 (1981) Custodial interrogation cannot be reinitiated unless and until the suspect has been re-advised of his Miranda rights has provided a knowing and voluntary waiver and (1) counsel is present and (2) the suspect himself initiated further communication with the police see id at 484 or (3) (if the suspect was released from custody after the initial interrogation) at least 14 days have passed Maryland v Shatzer 559 US 98 110 (2010)

To invoke the right to counsel a suspectrsquos request must be ldquounambiguousrdquo This means that the suspect must articulate the desire for counsel sufficiently clearly that a reasonable officer would understand the statement to be a request for counsel Davis v United States 512 US 452 459 (1994) If the request is ambiguous the police are not required to stop the interrogation

In this case the suspectrsquos statement ldquoI think I want my lawyer here before I talk to yourdquo was not an unambiguous request for counsel The most reasonable interpretation of this statement is that the suspect might be invoking his right to counsel Id at 461 (ldquomaybe I should talk to a lawyerrdquo is not an unequivocal request for counsel) See also Burket v Angelone 208 F3d 172 197ndash98 (4th Cir 2000) (ldquoI think I need a lawyerrdquo is not an unambiguous request for an attorney) Soffar v Cockrell 300 F3d 588 594ndash95 (5th Cir 2002) (discussion of various statements that did not constitute unequivocal requests for counsel)

Under these circumstances the detective was not required to cease the custodial interrogation of the suspect Nor was the detective required to clarify or ask follow-up questions to determine whether the suspect in fact wanted an attorney Davis 512 US at 459ndash60

14

Criminal Law and Procedure Analysis

Point Three (35) The suspectrsquos waiver of his Miranda rights was knowing intelligent and voluntary despite the fact that he was never told of the lawyerrsquos presence in the jail or of the lawyerrsquos demands

A valid waiver of Miranda rights must be ldquovoluntaryrdquomdashie the product of a free or deliberate choice rather than intimidation coercion or deception Berghuis v Thompkins 560 US 370 382ndash83 (2010) In addition the waiver must be knowing and intelligent That is it ldquomust have been made with a full awareness of both the nature of the right being abandoned and the consequences of the decision to abandon itrdquo Moran v Burbine 475 US 421 (1986)

In this case the suspect signed a Miranda waiver form after receiving proper warnings There is no evidence ldquothat the police resorted to physical or psychological pressure to elicit the statementsrdquo Id The entire interview lasted only 45 minutes The only issue is whether the suspect knowingly and intelligently waived his Miranda rights despite the fact that the detective did not tell the suspect about the lawyerrsquos presence and her demands

The Supreme Court has said that ldquo[e]vents occurring outside of the presence of the suspect and entirely unknown to him surely can have no bearing on the capacity to comprehend and knowingly relinquish a constitutional rightrdquo Id at 422 If the suspect ldquoknew that he could stand mute and request a lawyer and was aware of the Statersquos intention to use his statements to secure a convictionrdquo then the waiver is valid regardless of the information withheld Id at 422ndash23

Here the suspect was correctly informed of his rights Miranda v Arizona 384 US at 467ndash73 His comments demonstrate that he understood that he could have a lawyer present if he desired (ie wondering whether he should call his attorney) and that he understood that there might be consequences to speaking with the detective (ldquoI probably should keep my mouth shut but Irsquom willing to talk to you for a whilerdquo) His comment ldquo[L]etrsquos not waste any time waiting for someone to call my attorney and having her drive hererdquo along with his signature on the Miranda waiver form show that his waiver was valid under the constitutional standard

The fact that the detective did not tell the suspect about the lawyerrsquos presence and demands has no bearing on the validity of the suspectrsquos waiver because ldquosuch conduct is only relevant to the constitutional validity of a waiver if it deprives a defendant of knowledge essential to his ability to understand the nature of his rights and the consequences of abandoning themrdquo Moran at 424 The Supreme Court has specifically declined to adopt a rule requiring that law enforcement tell a suspect of an attorneyrsquos efforts to contact him id at 425 (ldquoNor are we prepared to adopt a rule requiring that the police inform a suspect of an attorneyrsquos efforts to reach himrdquo)

[NOTE An examinee might also recognize that this general rule is further supported by the Supreme Courtrsquos decision in Florida v Powell 559 US 50 (2010) approving state Miranda warnings that do not explicitly warn suspects that they have a right to have counsel present during custodial interrogation]

15

CONTRACTS ANALYSIS (Contracts IB2 IIB IVA3 amp A5)

ANALYSIS

Legal Problems

(1) In the case of a service contract (governed by the common law of contracts) is a modification enforceable when a party agrees to pay more for the same performance than was originally promised

(2) In the case of a contract for the sale of goods (governed by Article 2 of the UCC) is a modification enforceable when a party agrees to pay more for the same goods than was originally promised

(3) May a party avoid an agreement on the basis of economic duress

DISCUSSION

Summary

There are two arguments that the conservatory can make to support the claim that it is not bound to pay the higher prices lack of consideration and economic duress

The organ repair contract is governed by the common law of contracts Under the common law the business would have difficulty recovering the additional $60000 for the organ repair because under the ldquopreexisting duty rulerdquo the agreement of the conservatory to pay the extra price was not supported by consideration However the business might argue that the modification is enforceable under an exception to the preexisting duty rule for fair and equitable modifications made in light of unanticipated circumstances

The organ sale contract is governed by Article 2 of the Uniform Commercial Code The business would likely recover the additional amount under that contract because Article 2 provides that consideration is not required for a modification to be binding

In both cases the conservatory could seek to avoid its agreement on the grounds of economic duress but that argument is not likely to succeed

Point One (45) The business probably cannot recover the additional $60000 for the organ repair because the conservatoryrsquos promise to pay more money was not supported by consideration

The general rule is that to be enforceable a promise must be supported by consideration Under RESTATEMENT (SECOND) OF CONTRACTS sect 71 a promise is supported by consideration if it is bargained for in exchange for a return promise or performance However under the ldquopreexisting duty rulerdquo (exemplified in RESTATEMENT (SECOND) OF CONTRACTS sect 73 and Alaska Packersrsquo Assrsquon v Domenico 117 F 99 (9th Cir 1902)) promise of performance of a legal duty already owed to a promisor which is neither doubtful nor the subject of honest dispute is not consideration

If the business had promised the conservatory anything new or different in exchange for the agreement to pay the additional $60000 (such as for example repairing the pipe organ more

16

Contracts Analysis

quickly or using better parts) that would constitute consideration especially in light of the principle that courts do not inquire into the adequacy of consideration Here however the business already had a legal duty under the original contract and did not agree to do anything else in exchange for the conservatoryrsquos promise to pay $60000 more

However an exception to the preexisting duty rule is sometimes applied in situations of unanticipated changed circumstances Under RESTATEMENT (SECOND) OF CONTRACTS sect 89 followed in many jurisdictions a promise modifying a duty under a contract not fully performed on either side is binding even if not supported by consideration if the modification is fair and equitable in view of circumstances not anticipated by the parties when the contract was made

If a court applies the rule in Restatement sect 89 the critical issues will be whether the modification was in fact ldquofair and equitablerdquo and whether it can be justified in light of unanticipated circumstances In many cases in which modifications have been upheld a party encountered difficulties or burdens in performing far beyond what was knowingly bargained for in the original contract with the result bordering on impracticability such as having to excavate solid rock instead of soft dirt or having to remove garbage far in excess of the amounts contemplated The conservatory would argue that the businessrsquos performance difficulties were not of this sort at allmdashnothing about repairing the pipe organ itself was any different from or more difficult than originally contemplated except that the business itself encountered financial distress unrelated to its burdens in performing its obligations under these contracts

Even if the business satisfies that element of the rule in Restatement sect 89 the business must also demonstrate that the circumstances that gave rise to the need to modify the contract were ldquounanticipatedrdquo at the time the original contract was made Here the facts suggest that when the business entered into the original contract it expected that the price paid by the conservatory would enable it to perform However any evidence that the business knew or had reason to know at the time of execution that it would need more money from the conservatory to be able to perform would mean that the request to modify was not ldquounanticipatedrdquo

[NOTE Some cases such as Schwartzreich v Bauman-Basch Inc 231 NY 196 131 NE 887 (1921) find that if the parties mutually agreed to rescind the original contract and then after rescission entered into an entirely new contract for a higher price the new contract is supported by consideration There is no evidence that such a rescission followed by a new contract took place here]

Point Two (45) The business can recover the additional $40000 for the new organ because no consideration is required under Article 2 of the UCC for good-faith contract modifications

The contract to buy a new organ is a contract for the sale of goods and therefore is governed by Article 2 of the Uniform Commercial Code UCC sect 2-102 Under Article 2 unlike the common law an agreement modifying a contract needs no consideration to be binding UCC sect 2-209(1) Section 2-209(1) thus obviates the preexisting duty rule entirely in contracts for the sale of goods

Even though consideration is not required modifications governed by sect 2-209 must satisfy the obligation of good faith imposed by the UCC UCC sect 1-304 See also Official Comment 2 to UCC sect 2-209 Good faith means ldquohonesty in fact and the observance of reasonable commercial standards of fair dealingrdquo UCC sect 1-201(b)(20) In this context the obligation of good faith means that ldquo[t]he effective use of bad faith to escape performance on the original contract terms is barred and the extortion of a lsquomodificationrsquo without legitimate commercial reason is ineffective as a violation of the duty of good faithrdquo Official Comment 2 to

17

Contracts Analysis

UCC sect 2-209 Here because the businessrsquos financial reversals were serious and apparently unanticipated at the time that the business entered into the contract with the conservatory and commitment of the extra money was needed to enable the business to perform a court would likely find that the business acted in good faith Thus a court would likely uphold the enforceability of the conservatoryrsquos promise to pay the additional $40000

Point Three (10) The conservatory is unlikely to be able to defend against enforcement of its promises to pay additional money under the theory of economic duress because the business probably did not make an improper threat

Under the common law of contracts parties may raise the defense of duress This common law defense also applies to contracts governed by UCC Article 2 See UCC sect 1-103(b)

A contract is voidable on the ground of economic duress by threat when it is established that a partyrsquos manifestation of assent is induced by an improper threat that leaves the party no reasonable alternative See RESTATEMENT (SECOND) OF CONTRACTS sect 175 See also eg Austin Instrument Inc v Loral Corp 272 NE2d 533 (NY 1971) (a threat to withhold essential goods can constitute duress) In order to void its agreement to pay the additional sum because of economic duress the conservatory must demonstrate that (1) the business made a threat to the conservatory (2) the threat was ldquoimproperrdquo or ldquowrongfulrdquo (3) the threat induced the conservatoryrsquos manifestation of assent to the modification and (4) the threat was sufficiently grave to justify the conservatoryrsquos assent

Here it appears that three of the four elements are likely satisfied The business plainly made a threat Moreover the threat induced the conservatoryrsquos assent to the modification and the threat was sufficiently grave to justify that assent If the conservatory had not agreed to pay the business the extra amounts the conservatory would have lost its entire $325000 investment In light of this potential loss a court could easily conclude that the conservatory had no reasonable alternative

However the business has a strong argument that its threat (indicating that it would breach the contracts unless the prices were increased) was not wrongful or improper but was instead nothing more than a communication of the reality of its own perilous situation to the conservatory

A mere threat to breach a contract is not in and of itself improper so as to support an action of economic duress or business compulsion Something more is required such as a breach of the duty of good faith and fair dealing as was present in Austin Instrument Inc supra Because the business could not perform the original contract without the requested modification the economic duress claim for the conservatory would likely fail for much the same reason that the business would be able to enforce the modification At the time the modification was requested the business was not trying to extort a price increase because of the conservatoryrsquos vulnerability but instead was simply stating the reality that the business could not perform without more money

18

FAMILY LAW ANALYSIS (Family Law IIIB D amp G)

ANALYSIS

Legal Problems

(1)(a) Does the State A court have jurisdiction to modify the State B child support order

(1)(b) Does the State A court have jurisdiction to modify the marital-residence-saleshyproceeds provision of the State B property-division decree

(2)(a) May a child support order be modified retroactively

(2)(b) May a child support order be modified prospectively based on a change of employment with a lower salary

(2)(c) May a property-division order be modified after entry of a divorce decree

DISCUSSION

Summary

The State A court may exercise personal jurisdiction over the wife because she was personally served in State A However subject-matter jurisdiction over the interstate modification of child support is governed by the Uniform Interstate Family Support Act (UIFSA) Under UIFSA State A does not have jurisdiction to modify the order for the daughterrsquos support because the wife is still a resident of State B UIFSA on the other hand does not govern property distributions and thus a State A court is not precluded from hearing the husbandrsquos petition to modify the marital-residence-sale-proceeds provision of the divorce decree

A child support order may not be modified retroactively A child support order may be modified prospectively based on a substantial change in circumstances Courts agree that a significant decrease in income is a substantial change in circumstances All states treat voluntary income reductions differently than involuntary reductions but employ different approaches for evaluating the impact of a voluntary reduction Whether the husband could obtain prospective modification of the child support order depends on which approach is applied

A property-division order is not subject to post-divorce modification based on a change in circumstances Thus the husband may in some states obtain prospective modification of the order for the daughterrsquos support but he may not obtain modification of the marital-residenceshysale-proceeds provision

Point One(a) (25) Personal jurisdiction over a nonresident respondent does not confer subject-matter jurisdiction over child support modification Under UIFSA a State A court may not modify a child support order issued by a State B court when as here the child or either parent continues to reside in State B the jurisdiction that issued the child support order

The State A court may exercise personal jurisdiction over the wife The wife was personally served in State A and a state may exercise jurisdiction based on in-state personal service See

19

Family Law Analysis

Burnham v Superior Court 495 US 604 (1990) But personal jurisdiction over the wife is not enough to give a State A court jurisdiction to modify the State B support order

The interstate enforcement and modification of child support is governed by the Uniform Interstate Family Support Act (UIFSA) which has been adopted by all states Under UIFSA the state that originally issued a child support order (here State B) has continuing exclusive jurisdiction to modify the order if that state remains the residence of the obligee the child or the obligor and all parties do not consent to the jurisdiction of another forum See UIFSA sect 205 See also UIFSA sect 603 (ldquoA tribunal of this State shall recognize and enforce but may not modify a registered order if the issuing tribunal had jurisdictionrdquo) The wife and daughter continue to reside in State B and the wife has not consented to the jurisdiction of another forum Thus a State A court does not have jurisdiction to modify the State B child support order

[NOTE Examinees who do not discuss personal jurisdiction but fully discuss UIFSA may receive full credit]

Point One(b) (15) UIFSA does not apply to disputes over property division Thus the State A court may exercise jurisdiction over the husbandrsquos petition to modify the marital-residence-sale-proceeds provision of the State B divorce decree because it has personal jurisdiction over the wife

The State A court in which the husband brought his action has jurisdiction to adjudicate domestic relations issues The husbandrsquos petition to modify the property settlement is a domestic relations issue The courts of State A may exercise personal jurisdiction over the wife because she was personally served in State A See Burnham v Superior Court 495 US 604 (1990) see Point One(a)

UIFSA does not apply to divorce property-division disputes Thus although a State A court may not adjudicate the husbandrsquos petition to modify his child support obligations it may adjudicate his property-division claims (Even though the court has jurisdiction it may not modify the property-division award on the merits See Point Two(c))

Point Two(a) (20) A child support order may not be modified retroactively

State courts have long held that obligations to pay child support ordinarily may not be modified retroactively ldquoIf the hardship is particularly severe the courts sometimes devised a way to protect the obligor but in most instances the courts hold that retroactive modification of this kind is beyond their power and indeed the governing statute may so providerdquo HOMER H CLARK THE LAW OF DOMESTIC RELATIONSHIPS IN THE UNITED STATES 725 (2d ed 1987)

Federal law now goes further and requires the states as a condition of federal child-support funding to adopt rules that absolutely forbid retroactive modification of the support obligation See 42 USC sect 666(a)(9)(C) The states have adopted rules consistent with the federal requirements

Point Two(b) (25) It is unclear whether the husband could obtain prospective downward modification of his child support based on his voluntary acceptance of a job with a lower salary

Prospective modification of a child support order is typically available only when the petitioner can show a substantial change in circumstances See ROBERT E OLIPHANT amp NANCY VER

20

Family Law Analysis

STEEGH FAMILY LAW 213ndash15 (3d ed 2010) A significant decrease in income is typically viewed as a substantial change

However when a parent seeks to modify a child support obligation because he has voluntarily reduced his income a court will not modify the obligation based solely on the income loss Some courts refuse to modify whenever the income shift was voluntary See eg Aguiar v Aguiar 127 P3d 234 (Idaho Ct App 2005) Others look primarily to the petitionerrsquos intentions and permit downward modification if he has acted in good faith See eg In re Marriage of Horn 650 NE2d 1103 (Ill App Ct 1995) Many courts use a multifactor approach See OLIPHANT amp VER STEEGH supra 217ndash18

Here there is no question that the husbandrsquos loss of income was voluntary In a jurisdiction in which voluntary income reduction bars support modification the husbandrsquos petition would be denied

In a jurisdiction employing a good-faith or multifactor approach it is possible but not certain that the husband could obtain downward modification The evidence supports the husbandrsquos good faith his change in employment appears to be based on his new jobrsquos greater responsibilities and better promotion possibilities In a jurisdiction using a multifactor approach the court would likely also consider the impact of such a shift on the daughter the likely duration of the husbandrsquos income loss and the likelihood of a promotion that would ultimately inure to the daughterrsquos benefit Thus on these facts it is possible but by no means certain that the husband could prospectively obtain downward modification of his child support obligation to his daughter

Point Two(c)(15) A divorce property-division award is not subject to modification

A support order is aimed at meeting the post-divorce needs of the supported individual Because the future is unpredictable courts are empowered to modify a support award to take account of changed circumstances that may occur during the period in which support is paid

By contrast a property-distribution award divides assets of the marriage based on the equities at the time of divorce Because the past can be ascertained a property-division award is not subject to post-divorce modification See HARRY A KRAUSE ET AL FAMILY LAW CASES COMMENTS AND QUESTIONS 691 (6th ed 2007)

Here the husband is seeking modification of a property-division award with respect to an asset owned by the parties at the time of divorce Thus the husband may not obtain a modification of the marital-residence-sale-proceeds provision of the divorce decree based on his reduced income

21

FEDERAL CIVIL PROCEDURE ANALYSIS (Federal Civil Procedure III IVC)

ANALYSIS

Legal Problems

(1) Is the logging company entitled to join this action as a matter of right

(2)(a) May the nonprofit organization obtain a temporary restraining order to stop the USFS from issuing a logging permit

(2)(b) May the nonprofit organization obtain a preliminary injunction to stop the USFS from issuing a logging permit during the pendency of the action

DISCUSSION

Summary

The logging company is entitled to intervene in this action as a matter of right because it has an interest in the property or transaction that is the subject of the action and is so situated that its interest may be impaired or impeded as a practical matter if the action goes forward without it The logging companyrsquos interest is not adequately represented by the USFSrsquos presence in the lawsuit

The nonprofit organization may seek a temporary restraining order (TRO) followed by a preliminary injunction to prevent the USFS from issuing a logging permit pending the outcome of the action The nonprofit is likely to obtain a TRO if it can demonstrate a risk of immediate and irreparable injury The nonprofit is also likely to obtain a preliminary injunction if it can demonstrate a significant threat of irreparable harm and a likelihood of success on the merits of its National Environmental Policy Act (NEPA) claim

Point One (50) Rule 24(a) of the Federal Rules of Civil Procedure requires federal courts to allow a person to intervene in an action as a matter of right if the person a) is interested in the property or transaction that is the subject of the action b) is so situated that its interest may be impaired or impeded if the litigation goes forward without it and c) is not adequately represented by existing parties Here the logging company likely meets all three requirements and should be allowed to intervene as a matter of right

Rule 24 of the Federal Rules of Civil Procedure governs intervention the process by which a non-party to an action may join the litigation Under Rule 24(a) (intervention of right) a person must be permitted to intervene if three conditions are met (1) the movant ldquoclaims an interest relating to the property or transaction that is the subject of the actionrdquo (2) the movant ldquois so situated that disposition of the action may as a practical matter impair or impede the movantrsquos ability to protect its interestrdquo and (3) ldquoexisting partiesrdquo do not ldquoadequately represent [the movantrsquos] interestrdquo FED R CIV P 24(a) The three requirements for intervention of right are often ldquovery interrelatedrdquo 7C CHARLES ALAN WRIGHT ET AL FEDERAL PRACTICE AND PROCEDURE sect 1908 at 297 (2007 amp 2011 Supp)

22

Federal Civil Procedure Analysis

Here the court should find that the logging company meets this test First the logging company has a strong interest in the property or transaction that is the subject of this action The USFS has accepted the logging companyrsquos bid and the logging company is merely awaiting issuance of a logging permit to begin logging The nonprofit organization is seeking to prevent this logging The logging company therefore has a strong direct and substantial interest in the subject matter of the lawsuit and in having its winning bid honored and a logging permit issued See eg Kleissler v US Forest Serv 157 F3d 964 972 (3d Cir 1998) (stating that ldquo[t]imber companies have direct and substantial interests in a lawsuit aimed at halting loggingrdquo) see also Natural Resources Defense Council v US Nuclear Regulatory Commrsquon 578 F2d 1341 1343ndash 44 (10th Cir 1978) (holding that applicants whose license renewals were pending had Rule 24(a)(2) interests where the lawsuit sought to halt the license-issuing process pending preparation of environmental impact statements) See generally 7C WRIGHT ET AL supra sect 19081 at 309 (ldquoIf there is a direct substantial legally protectable interest in the proceedings it is clear that this requirement of the rule is satisfiedrdquo) Second the logging companyrsquos interest in receiving a logging permit may well be impaired as a practical matter by the outcome of the lawsuit If the USFS loses the lawsuit it will have to prepare an environmental impact statement before issuing the logging companyrsquos permit This will at a minimum delay the logging companyrsquos ability to exercise its rights and may in the long r un mean that no logging permit is ever issued Intervention of right is not limited to those that would be legally bound as a matter of preclusion doctrine Id sect 19082 at 368 Rather ldquo[t]he rule is satisfied whenever disposition of the present action would put the movant at a practical disadvantage in protecting its interestrdquo Id sect 19082 at 369 Here that condition is easily satisfied See Kleissler 157 F3d at 972 (ldquoTimber companies have direct and substantial interests in a lawsuit aimed at halting logging rdquo)

Given that the logging company has an interest that may be impaired by disposition of the action it should be allowed to intervene unless the court is persuaded that the USFS adequately represents the logging companyrsquos interest See Rule 24(a)(2) 7C WRIGHT ET AL supra sect 1909 Here it could be argued that the USFS adequately represents the logging companyrsquos interest because the USFS presumably wants the court to uphold its development plan and allow it to proceed with issuance of the logging permit which is the same relief that the logging company would seek However whether representation is truly adequate depends upon ldquo[a] discriminating appraisal of the circumstancesrdquo 7C WRIGHT ET AL supra sect 1909 at 440 Although both the government and the logging company wish to avoid the preparation of an environmental impact statement their interests are distinct The USFSrsquos interest is proper management of the national forest system while the logging companyrsquos interest is making a profit from logging the 5000-acre tract The USFSrsquos handling of the litigation is likely to be affected by a variety of policy concerns and political considerations that have nothing to do with the logging companyrsquos purely economic interest in securing the right to cut trees in the Scenic National Forest See eg Kleissler 157 F3d at 973ndash74 (ldquo[T]he government represents numerous complex and conflicting interests in matters of this nature The straightforward business interests asserted by intervenors here may become lost in the thicket of sometimes inconsistent governmental policiesrdquo)

[NOTES (1) Examinees who mistakenly analyze the logging companyrsquos case for joinder under the related but incorrect Rule 19 ldquoRequired Joinder of Partiesrdquo may receive credit Rule 19 allows existing parties to demand joinder of non-parties (or seek dismissal of the case if they canrsquot get it) There is a close relationship between Rule 24 and Rule 19 and both contain a similar standard for determining when ldquointerestedrdquo third parties are ldquoentitledrdquo or ldquorequiredrdquo to be in the lawsuit Indeed the two prongs of the Rule 24 intervention test that are discussed above

23

Federal Civil Procedure Analysis

are nearly identical to the two prongs of the Rule 19(a) required joinder test Examinees who discuss and apply the test should receive credit even if they cite Rule 19 rather than Rule 24

(2) Examinees may discuss permissive joinder Although permissive joinder is a possibility here the question asks only whether the logging company can join the action as a matter of right and a permissive joinder analysis is not responsive to the question To the extent an examinee discusses permissive joinder the analysis will focus on whether the logging company ldquohas a claim or defense that shares with the main action a common question of law or factrdquo FED R CIV P 24(b)(1)(B) The district court also ldquomust consider whether the intervention will unduly delay or prejudice the adjudication of the original partiesrsquo rightsrdquo FED R CIV P 24(b)(3) On our facts the logging companyrsquos claim for the issuance of a logging permit would certainly share common questions of law and fact with the USFSrsquos defense against the nonprofitrsquos claim There are no facts suggesting that the logging companyrsquos presence would unduly delay or otherwise prejudice adjudication of the original action Thus the district court would have discretion to permit the logging company to intervene even if it denied intervention of right]

Point Two(a) (25) The nonprofit organization could seek and would likely obtain a temporary restraining order to stop the USFS from issuing a logging permit pending a hearing on an application for a preliminary injunction

The first type of interim relief the nonprofit could seek to stop the USFS from issuing a logging permit to the logging company is a temporary restraining order (TRO) prohibiting the USFS from issuing the logging permit A TRO can be issued without notice to the adverse party but only in limited circumstances and only for a limited time FED R CIV P 65(b) To secure a TRO without notice the nonprofit would need to submit an affidavit containing specific facts that demonstrate a risk of ldquoimmediate and irreparable injuryrdquo if a permit is issued FED R CIV P 65(b)(1) In deciding whether to grant a TRO courts will also consider the same factors that are relevant in deciding whether to grant a preliminary injunction (eg the moving partyrsquos likelihood of success on the merits the balance of hardships and the public interest) See Point Two(b) infra The TRO would last only long enough for the court to consider and resolve a request by the nonprofit for a preliminary injunction but no longer than 14 days (unless the court extends it for good cause or the adverse party consents to an extension) In addition bond is required

Here the court is likely to grant the nonprofitrsquos request The nonprofit could plausibly claim that cutting down 5000 acres of old-growth forest in an area that is home to the highest concentration of wildlife in the western United States would have ldquoan immediate and irreparablerdquo adverse impact on the environment and cause irreparable harm to the nonprofitrsquos interest in preserving and protecting natural resources including wildlife habitat

Point Two(b) (25) The nonprofit could also seek and would likely obtain a preliminary injunction to stop the USFS which is likely to be granted if the nonprofitrsquos claim that the USFS violated NEPA has a strong basis in fact and law

Because the TRO would be temporary the nonprofit would need to move for a preliminary injunction to prevent the USFS from issuing a logging permit throughout the pendency of the litigation Preliminary injunctions are injunctions that seek to ldquoprotect [the] plaintiff from

24

Federal Civil Procedure Analysis

irreparable injury and to preserve the courtrsquos power to render a meaningful decision after a trial on the meritsrdquo 11A CHARLES ALAN WRIGHT ET AL FEDERAL PRACTICE AND PROCEDURE sect 2947 at 112 (2013) Rule 65 of the Federal Rules of Civil Procedure sets out the procedural requirements for preliminary injunctions Preliminary injunctions may be granted only upon notice to the adverse party FED R CIV P 65(a)(1) and only if the movant ldquogives security in an amount that the court considers proper to pay the costs and damages sustained by any party found to have been wrongfully enjoined or restrainedrdquo FED R CIV P 65(c)

While Rule 65 sets out the procedural requirements for preliminary injunctive relief it does not specify the substantive grounds upon which it may be granted The courtrsquos discretion in ruling upon a motion for a preliminary injunction ldquois exercised in conformity with historic federal equity practicerdquo 11A WRIGHT ET AL supra sect 2947 at 114 The court typically considers four factors

(1) the significance of the threat of irreparable harm to the plaintiff if the injunction is not granted (2) the balance between this harm and the injury that granting the injunction would inflict on the defendant (3) the probability that the plaintiff will succeed on the merits and (4) the public interest

Id sect 2948 at 122ndash24 accord Habitat Educ Center v Bosworth 363 F Supp 2d 1070 1088 (ED Wis 2005) The most important of these factors is the risk of irreparable harm to the plaintiff 11A WRIGHT ET AL supra sect 29481 at 129 If the plaintiff has an adequate remedy at law (eg if money damages can compensate the plaintiff for its loss) then a preliminary injunction will be denied Id sect 29481

Here a court would likely conclude that the potential for environmental damage to the forest creates a significant threat of irreparable harm ldquo[E]nvironmental injury is often irreparable Courts have recognized that logging such as would occur [here] can have longshyterm environmental consequences and thus satisfy the irreparable injury criterionrdquo Habitat Educ Center 363 F Supp 2d at 1089 (citing Idaho Sporting Congress Inc v Alexander 222 F3d 562 569 (9th Cir 2000) (noting that the imminent and continuing logging activities presented ldquoevidence of environmental harm sufficient to tip the balance in favor of injunctive reliefrdquo)) Neighbors of Cuddy Mountain v US Forest Service 137 F3d 1372 1382 (9th Cir 1998) (stating that ldquo[t]he old growth forests plaintiffs seek to protect would if cut take hundreds of years to reproducerdquo) (internal citation omitted)) see also 11C WRIGHT ET AL supra sect 29481 at 151 (noting that ldquoa preliminary injunction has been issued to prevent harm to the environmentrdquo)

The second factor the balance between the harm to the plaintiff and the harm the defendant will suffer if the injunction is issued also appears to support issuance of a preliminary injunction here The USFS will have to wait before it can develop the Scenic National Forest and the logging company may lose money if the delay is prolonged These economic harms could be compensated monetarily if an injunction is issued inappropriately Where ldquoan injunction bond can compensate [the] defendant for any harm the injunction is likely to inflict the balance should be struck in favor of [the] plaintiffrdquo Id sect 29482 at 192 See also Habitat Educ Center 363 F Supp 2d at 1089 (stating that ldquothe relative absence of harmful effects on the Forest Service weighs in favor of granting the injunctionrdquo)

The third factor is the likelihood that the plaintiff will prevail on the merits Although there is limited information concerning the merits of the action the nonprofit alleges that the federal statute (NEPA) requires an environmental impact statement and further states that the USFS created no environmental impact analysis or statement at all Assuming that those

25

Federal Civil Procedure Analysis

allegations are correct it seems plausible to conclude that the nonprofit will be able to show a likelihood of success on the merits

Finally courts deciding whether or not to issue preliminary injunctive relief are to consider the public interest ldquoFocusing on this factor is another way of inquiring whether there are policy considerations that bear on whether the order should issuerdquo 11C WRIGHT ET AL supra sect 29484 at 214 If the court concludes that the nonprofit is likely to succeed on its NEPA claim because the USFS wrongfully failed to conduct an environmental impact assessment it is likely to find that the public interest would be served by restraining the USFS from proceeding with logging in a national forest See Heartwood Inc v US Forest Service 73 F Supp 2d 962 979 (SD Ill 1999) affrsquod on other grounds 230 F3d 947 (7th Cir 2000) (ldquoviolations by federal agencies of NEPArsquos provisions as established by Congress harm the public as well as the environmentrdquo)

Thus a court is very likely to grant a preliminary injunction if it concludes that the nonprofit has a significant likelihood of success on the merits

26

EVIDENCE ANALYSIS (Evidence ID IIA amp C)

ANALYSIS

Legal Problems

(1) Under what circumstances can evidence of prior convictions be used to impeach a witnessrsquos credibility in a civil case

(1)(a) May the inmatersquos credibility be impeached by evidence of a 12-year-old felony drug conviction if he was released from prison 9 years ago

(1)(b) May the inmatersquos credibility be impeached by evidence of an 8-year-old misdemeanor perjury conviction that was punishable by 1 year in jail if he pleaded guilty and was sentenced only to pay a $5000 fine

(1)(c) May the inmatersquos credibility be impeached by evidence of a 7-year-old sexual assault conviction if the inmate is still serving a 10-year prison sentence and the victim was his 13-year-old daughter

(2)(a) May the guardrsquos credibility be impeached by cross-examination regarding specific instances of misconduct (ie lying on his reacutesumeacute) relevant to credibility

(2)(b) May the guardrsquos credibility be impeached by admission of extrinsic evidence (his reacutesumeacute and academic transcript) offered to prove specific instances of misconduct relevant to credibility

DISCUSSION

Summary

Under the Federal Rules of Evidence witnesses can be impeached with evidence of prior convictions andor specific instances of misconduct Whether evidence of prior convictions should be admitted to impeach generally depends on the nature of the crime the amount of time that has passed and (only in criminal cases) whether the ldquowitnessrdquo is the defendant FED R EVID 609(a)

In this civil case evidence of the inmatersquos conviction for distribution of marijuana should be admitted to impeach the inmate because he was convicted of a felony and was released from prison fewer than 10 years ago FED R EVID 609(a)(1) Credibility is critically important in this case because the jury will hear conflicting testimony from the two disputing parties and there were no other eyewitnesses to the altercation Under Rule 609(a)(1) the inmatersquos conviction should be admitted because it has some bearing on his credibility and its probative value is not substantially outweighed by concerns of unfair prejudice confusion or delay Id

Evidence of the inmatersquos misdemeanor conviction for perjury must be admitted because the crime ldquorequired provingmdashor the witnessrsquos admittingmdasha dishonest act or false statementrdquo by the inmate FED R EVID 609(a)(2)

27

Evidence Analysis

Evidence of the inmatersquos felony conviction for sexual assault should be excluded because its probative value is substantially outweighed by the danger of unfair prejudice to the inmate based on the heinous nature of the crime FED R EVID 609(a)(1) In the alternative the judge could limit the evidence relating to this conviction by excluding details of the inmatersquos crime

In all civil (and criminal) cases witnesses can also be impeached with evidence of specific instances of prior misconduct that did not result in a conviction FED R EVID 608(b) Pursuant to Rule 608(b) misconduct probative of untruthfulness can be inquired into on cross-examination but cannot be proved through extrinsic evidence Id Thus the inmatersquos counsel should be permitted to cross-examine the guard regarding the false statement in the guardrsquos reacutesumeacute However extrinsic evidence of the guardrsquos misconduct (ie the guardrsquos authenticated reacutesumeacute and transcript from the local college) should not be admitted even if the guard denies wrongdoing or refuses to answer cross-examination questions about these matters Id

Point One (10) The Federal Rules of Evidence permit impeachment of witnesses with evidence of prior convictions

Whether convictions should be admitted to impeach generally depends on the nature of the crime the amount of time that has passed and (only in criminal cases) whether the ldquowitnessrdquo is the defendant FED R EVID 609(a) Under Rule 609(a) evidence of prior convictions may be admitted for the purpose of ldquoattacking a witnessrsquos character for truthfulnessrdquo Id

There are two basic types of convictions that can be admitted for the purpose of impeachment

(1) convictions for crimes ldquopunishable by death or by imprisonment for more than one yearrdquo (which generally correlates to ldquofeloniesrdquo) FED R EVID 609(a)(1) and (2) convictions ldquofor any crimes regardless of the punishment if the court can readily determine that establishing the elements of the crime required provingmdashor the witnessrsquos admittingmdasha dishonest act or false statementrdquo FED R EVID 609(a)(2)

Pursuant to Rule 609(a)(1) in civil cases the admission of evidence of a felony conviction is ldquosubject to Rule 403 [which says that a court may exclude relevant evidence if its probative value is substantially outweighed by other factors]rdquo FED R EVID 609(a)(1) However Rule 403 does not protect the witness against admission of prior convictions involving dishonestymdashwhich must be admitted by the court FED R EVID 609(a)(2)

Finally Federal Rule of Evidence 609(b) contains the presumption that a conviction that is more than 10 years old or where more than 10 years has passed since the witnessrsquos release from confinement (whichever is later) should not be admitted unless ldquoits probative value supported by specific facts and circumstances substantially outweighs its prejudicial effectrdquo and the proponent has provided the adverse party with reasonable written notice FED R EVID 609(b)

Point One(a) (25) The court should admit evidence of the inmatersquos 12-year-old felony marijuana distribution conviction

The inmatersquos conviction for marijuana distribution was for a felony punishable by imprisonment for more than one year See FED R EVID 609(a)(1) Moreover although the conviction was 12 years ago the 10-year time limit of Rule 609(b) is not exceeded because that time limit runs

28

Evidence Analysis

from the date of either ldquothe witnessrsquos conviction or release from confinement for it whichever is laterrdquo FED R EVID 609(b) Because the inmate served three years in prison he was released from confinement nine years ago

However pursuant to Rule 609(a)(1) the admission of felony convictions to impeach a witness in a civil case is ldquosubject to Rule 403rdquo FED R EVID 609(a)(1) Neither Rule 609(a) nor the advisory committee notes specify which factors courts should consider when balancing the probative value of a conviction against the dangers identified in Rule 403 (which include (1) unfair prejudice (2) confusion of the issues (3) misleading the jury (4) waste of time or undue delay and (5) needless presentation of cumulative evidence) FED R EVID 403

In this case credibility is very important because the evidence consists primarily of the testimony of the disputing parties and there were no other eyewitnesses to the altercation This enhances the probative value of any evidence bearing on the inmatersquos credibility A court is likely to conclude that the inmatersquos prior felony drug conviction is relevant to his credibility See eg United States v Brito 427 F3d 53 64 (1st Cir 2005) (ldquoPrior drug-trafficking crimes are generally viewed as having some bearing on veracityrdquo) Although the probative value of any conviction diminishes with age see eg United States v Brewer 451 F Supp 50 53 (ED Tenn 1978) the inmatersquos ongoing problems with the law suggest that he has continued (and even escalated) his criminal behavior over the past nine years The court should admit this evidence because its probative value is not substantially outweighed by any Rule 403 concerns Specifically any prejudice to the inmate would be slight because the conviction is unrelated to the altercation at issue and the conviction was not for a heinous crime that might inflame the jury

[NOTE Whether an examinee identifies the jury instruction as containing a ldquoconclusiverdquo or ldquomandatoryrdquo presumption is less important than the examineersquos analysis of the constitutional infirmities]

Point One(b) (15) The court must admit evidence of the inmatersquos eight-year-old misdemeanor conviction because perjury is a crime of dishonesty

Rule 609(a)(2) provides that evidence of a criminal conviction ldquomust be admitted if the court can readily determine that establishing the elements of the crime required provingmdashor the witnessrsquos admittingmdasha dishonest act or false statementrdquo FED R EVID 609(a)(2) The inmatersquos conviction for perjury would have necessarily required proving that the inmate engaged in an act of dishonesty This conviction occurred within the past 10 years so it ldquomust be admittedrdquo because in contrast to Rule 609(a)(1) (discussed in Point One(a)) admission under Rule 609(a)(2) is mandatory and not subject to Rule 403

Point One(c) (20) The court should exclude evidence of the inmatersquos seven-year-old felony sexual assault conviction because the probative value of this evidence is substantially outweighed by the danger of unfair prejudice In the alternative the details of the prior conviction could be excluded

The inmatersquos conviction for felony sexual assault was seven years ago and he has not yet been released from incarceration so Rule 609(a) but not 609(b) is applicable here FED R EVID 609(a) This conviction is therefore admissible to impeach the inmate unless its probative value is substantially outweighed by the danger of unfair prejudice or any other Rule 403 concern Id

29

Evidence Analysis

Sex crimes are generally not considered relevant to credibility see Hopkins v State 639 So 2d 1247 1254 (Miss 1993) so the probative value of this conviction is relatively low Moreover the heinous nature of the inmatersquos crime (sexual assault on his daughter) makes the danger of unfair prejudice to the inmate very high Thus the court should exclude evidence of the conviction because it was for a heinous offense that is likely to inflame the jury and it has little bearing on credibility See eg United States v Beahm 664 F2d 414 419 (4th Cir 1981)

As an alternative to excluding this evidence the judge could minimize the unfair prejudice to the inmate by permitting limited cross-examination but refusing to allow specific questions about the nature of the inmatersquos conviction For example a court could limit cross-examination to the fact that the inmate was convicted of a ldquofelonyrdquo or perhaps that he was convicted of a ldquosexual assaultrdquo without identifying the victim However because evidence of the inmatersquos prior convictions can be admitted solely for the purpose of enabling the jury to assess his credibility and because his two earlier convictions should have already been admitted the court should exclude all evidence of the felony sexual assault conviction

Point Two(a) (15) The court should permit the inmatersquos counsel to cross-examine the guard regarding the false statement in his reacutesumeacute because the guardrsquos misconduct bears on his truthfulness

The inmate wishes to cross-examine the guard about his prior dishonest behaviormdashlying on his reacutesumeacutemdashthat did not involve a criminal conviction Rule 608(b) allows witnesses to be cross-examined about specific instances of prior non-conviction misconduct probative of untruthfulness ldquoin order to attack the witnessrsquos character for truthfulnessrdquo FED R EVID 608(b)

The courtrsquos decision to allow cross-examination about the guardrsquos prior dishonest behavior depends on the probative value of such evidence balanced against the danger of unfair prejudice to the guard or any other Rule 403 concern FED R EVID 403 Here the guardrsquos false statement on his reacutesumeacute that he obtained a degree in Criminal Justice is highly probative of his untruthfulness because it grossly misrepresents his actual academic record was made recently and was made with the intent to deceive Because the probative value of this evidence is very strong and is not substantially outweighed by any Rule 403 concerns cross-examination of the guard on this topic should be permitted The court may also consider it fair to permit this cross-examination of the guard on these matters assuming that one or more of the inmatersquos prior convictions have been admitted to impeach his credibility

Point Two(b) (15) The court should exclude extrinsic evidence of the guardrsquos non-conviction misconduct even if the guard denies wrongdoing or refuses to answer questions about the matter

Although Rule 608(b) allows cross-examination about specific instances of prior misconduct probative of untruthfulness ldquoextrinsic evidencerdquo offered to prove such misconduct is not admissible FED R EVID 608(b) The rationale for this rule is that allowing the introduction of extrinsic evidence of prior misconduct by witnesses when these acts are relevant only to the witnessesrsquo truthfulness and not to the main issues in the case would create too great a risk of confusing the jury and unduly delaying the trial The court does not have discretion to admit this extrinsic evidence See eg United States v Elliot 89 F3d 1360 1368 (8th Cir 1996)

30

Evidence Analysis

Here the inmatersquos counsel may cross-examine the guard about the false statement on his reacutesumeacute However the inmatersquos counsel must accept the guardrsquos response Even if the guard denies wrongdoing or refuses to answer questions about the matter the inmatersquos counsel cannot introduce the guardrsquos reacutesumeacute or the transcript from the local college to prove the guardrsquos misconduct

31

CORPORATIONS ANALYSIS (Corporations VA2 IX)

ANALYSIS

Legal Problems

(1) Do shareholders have the authority to amend a corporationrsquos bylaws with respect to director nominations

(2) Do board-approved bylaws on a particular subject here nomination of directors preempt subsequent conflicting bylaw amendments by shareholders

(3) Is a suit challenging both managementrsquos refusal to include the proposed bylaw amendment in Megarsquos proxy statement and the boardrsquos amendment of the bylaws dealing with nomination of directors a direct or derivative suit

DISCUSSION

Summary

The voting and litigation rights of the shareholders of Mega are subject to the provisions of the Model Business Corporations Act (MBCA)

The investorrsquos proposed bylaw provision is not inconsistent with state law Under the MBCA shareholders may amend the bylaws when the amendment deals with a proper matter for the corporationrsquos bylaws such as procedures for nominating directors

The Mega boardrsquos bylaw amendment does not preempt the investorrsquos proposed bylaw provision or the Mega shareholdersrsquo power to approve it While shareholders can limit the boardrsquos power to amend or repeal the bylaws the board cannot limit the shareholdersrsquo power

Whether the investor must make a demand on Megarsquos board depends on how the investor frames its claim If the investor claims a violation of shareholder voting rights the claim is direct and pre-suit demand on the board is not required If on the other hand the investor claims that the directors violated their fiduciary duties by amending the bylaws to entrench themselves the claim is derivative and a pre-suit demand is required

Point One (30) Shareholders may amend the corporationrsquos bylaws where the proposed bylaw provision relates to procedural matters typically included in the bylaws such as the nomination of directors

Internal affairs of the corporation such as the conduct of shareholder meetings and election of directors are subject to the corporate law of the state of incorporation See McDermott Inc v Lewis 531 A2d 206 (Del 1987) (applying law of jurisdiction where corporation was incorporated in case involving voting rights) This statersquos corporate statute is modeled on the MBCA

Under the MBCA ldquoshareholders may amend the corporationrsquos bylawsrdquo MBCA sect 1020(a) Thus the only question is whether the bylaws can specify the procedures for shareholder nomination of directors

32

Corporations Analysis

The MBCA states that the bylaws ldquomay contain any provision that is not inconsistent with law or the articles of incorporationrdquo MBCA sect 206(b) In addition the MBCA was revised in 2009 to address shareholder nomination of directors in public corporations (known as ldquoproxy accessrdquo) and specifies that the bylaws ldquomay contain a requirement that the corporation include in its [proxy materials] one or more individuals nominated by a shareholderrdquo MBCA sect 206(c)(1) see Committee on Corporate Laws ABA Section of Business Law Report on the Roles of Boards of Directors and Shareholders of Publicly Owned Corporations and Changes to the Model Business Corporations ActmdashAdoption of Shareholder Proxy Access Amendments to Chapters 2 and 10 65 BUS LAWYER 1105 (2010)

The inclusion of director-nomination procedures in the bylaws is consistent with practice and is recognized by the Delaware courts whose views on corporate law carry significant weight Typically the procedures for nomination of directors are found in the bylaws See 1 COX amp HAZEN TREATISE ON THE LAW OF CORPORATIONS sect 312 (3d ed 2011) see also 4 FLETCHER CORP FORMS ANN PART III ch 21 (2013) (including sample bylaws that permit nomination of directors by shareholders) The Delaware Supreme Court has confirmed that the bylaws may ldquodefine the process and proceduresrdquo for director elections See CA Inc v AFSCME Employees Pension Plan 953 A2d 227 (Del 2008) (concluding that bylaw amendment requiring reimbursement of election expenses to certain successful shareholder nominators is ldquoproper subjectrdquo under Delaware law)

[NOTE The question of the proper scope of the bylaws can be answered using the more general MBCA sect 206(b) or the 2009 MBCA revision adding sect 206(c)(1) (adopted in CT ME VA) In addition some examinees might raise the point that shareholder proposals may not compel the board to take action such as by including shareholder nominations in the companyrsquos proxy materials on the theory that the ldquobusiness and affairsrdquo of the corporation are to be managed by the board See MBCA sect 801(b) Although shareholders are generally limited to adopting precatory resolutions that recommend or encourage board action this limitation does not apply when shareholders have specific authority to take binding action on their ownmdashsuch as to amend the bylaws]

Point Two (30) Shareholders can amend (or repeal) board-approved bylaws Further shareholders can limit the boardrsquos power to later amend and repeal a shareholder-approved bylaw

Under the MBCA shareholders have the power to amend the bylaws See Point One The board shares this power with the shareholders unless (1) the corporationrsquos articles ldquoreserve that power exclusively to the shareholdersrdquo or (2) ldquothe shareholders in amending repealing or adopting a bylaw expressly provide that the board of directors may not amend repeal or reinstate that bylawrdquo See MBCA sect 1020(b)

Shareholder-approved bylaw provisions can amend or repeal existing bylaw provisions whether originally approved by the board or by shareholders See ALAN R PALMITER CORPORATIONS EXAMPLES AND EXPLANATIONS sect 713 (7th ed 2012) Thus the Mega boardrsquos bylaw amendmentmdashwhich set more demanding thresholds for shareholder nomination of directors than the investorrsquos proposed bylaw provisionmdashwould be superseded (repealed) if Megarsquos shareholders were to approve the investorrsquos proposal

Further a shareholder-approved bylaw generally can limit the power of the board to later amend or repeal it See MBCA sect 1020(b)(2) Thus if Megarsquos shareholders approved the bylaw

33

Corporations Analysis

provision proposed by the investor Megarsquos board could not repeal the provision because it includes a ldquono board repealrdquo clause

The revision to the MBCA in 2009 dealing with shareholder proxy access does not change this conclusion That revision specifies that a shareholder-approved bylaw dealing with director nominations may not limit the boardrsquos power to amend add or repeal ldquoany procedure or condition to such a bylaw in order to provide for a reasonable practicable and orderly processrdquo MBCA sect 206(d) Thus according to the revision if shareholders approve a bylaw amendment that limits further board changes the board would nonetheless retain the power to ldquotinkerrdquo with the bylaw to safeguard the voting process but could not repeal the shareholder-approved bylaw The Official Comment to MBCA sect 206(d) makes clear that the revision is ldquonot intended to allow the board of directors to frustrate the purpose of the shareholder-adopted proxy access provisionrdquo Thus if Megarsquos shareholders were to approve the bylaw provision proposed by the investor Megarsquos board could only amend the provision regarding its procedures or conditions in a manner consistent with its purpose of permitting proxy access for Megarsquos shareholders

[NOTE The boardrsquos attempted interference with a shareholder voting initiative may also have been a violation of the directorsrsquo fiduciary duties See Blasius Indus Inc v Atlas Corp 564 A2d 651 (Del Ch 1988) (finding that directors breached their fiduciary duties by amending bylaws and expanding size of board to thwart insurgentrsquos plan to amend bylaws and seat a majority of new directors) The call however asks examinees to consider whether shareholders or the board have ldquoprecedencerdquo over amending the corporate bylaws Thus an examineersquos answer should be framed in terms of ldquopowerrdquo and not ldquodutyrdquo]

Point Three (40) The investor need not make a demand on the board if the investor states a direct claim such as an allegation that the board interfered with the investorrsquos right to amend the bylaws But the investor must make a demand on the board if the investor states a derivative claim (on behalf of the corporation) such as an allegation that the directors sought to entrench themselves by interfering with the proposed proxy access

The MBCA generally requires that shareholders make a demand on the board of directors before initiation of a derivative suit MBCA sect 742 (shareholder may not bring derivative proceeding until written demand has been made on corporation and 90 days have expired) A derivative suit is essentially two suits in one where the plaintiff-shareholder seeks to bring on behalf of the corporation a claim that vindicates corporate rights usually based on violation of fiduciary duties PALMITER supra sect 1811 (6th ed 2009) The demand permits the board to investigate the situation identified by the shareholder and take suitable action No demand on the board is required however if the shareholder brings a direct suit to vindicate the shareholderrsquos own rights not those of the corporation

Is the suit brought by the investor derivative or direct The MBCA defines a ldquoderivative proceedingrdquo as one brought ldquoin the right of a domestic corporationrdquo MBCA sect 740(1) Thus the answer to how the investorrsquos suit should be characterized turns on what rights the investor seeks to vindicate If the investor frames its claim as one of fiduciary breach by directorsmdashfor example for failing to become adequately informed about voting procedures or for seeking to entrench themselves in office by manipulating the voting structure to avoid a shareholder insurgencymdashthen the suit is ldquoderivativerdquo and the investor must make a demand on the board See MBCA Ch 7 Subch D Introductory Comment (ldquothe derivative suit has historically been the principal method of challenging allegedly illegal action by managementrdquo)

34

Corporations Analysis

If however the investor frames its claim as one to vindicate shareholder rights the suit is direct and no demand is required For many courts the direct-derivative question turns on who is injured and who is to receive the relief sought by the plaintiff-shareholders See Tooley v Donaldson Lufkin amp Jenrette Inc 845 A2d 1031 (Del 2004) (characterizing a merger-delay claim as direct because delay of merger only harmed shareholders not corporation) Thus if the investor claims that managementrsquos refusal to include its proposed bylaw amendment in the corporationrsquos proxy materials violates its shareholder rights to initiate corporate governance reforms the suit will be direct Courts have not questioned the ability of shareholders to bring direct suits challenging board action to exclude their proposed bylaw amendments from the corporationrsquos proxy materials See JANA Master Fund Ltd v CNET Networks Inc 954 A2d 335 (Del Ch 2008) (upholding shareholderrsquos direct challenge to boardrsquos interpretation of advance-notice bylaw) Chesapeake Corp v Shore 771 A2d 293 (Del Ch 2000) (upholding shareholderrsquos direct challenge to actions by board that effectively prevented it from proposing bylaw amendments in contest for control)

Is the way that the investor frames its claim conclusive Courts have permitted shareholder-plaintiffs to challenge a transaction in a direct suit even though the same transaction could also be challenged as a fiduciary breach See Eisenberg v Flying Tiger Line Inc 451 F2d 267 (2d Cir 1971) (permitting direct suit challenging a corporate reorganization as a dilution of shareholder voting power even though reorganization may have involved conflicts of interest and thus constituted a fiduciary breach) Thus the investorrsquos choice to pursue a claim challenging the legality of managementrsquos decision to exclude the investorrsquos proposal from the corporationrsquos proxy materialsmdashrather than a possible breach of fiduciary dutymdashis likely to be respected See 3 COX amp HAZEN supra sect 153 (describing situations in which a claim can be framed as derivative or direct)

[NOTE Some issues under Delaware corporate law regarding pre-suit demand are not relevant here For example whether the Mega directors are independent and disinterested is not relevant to the MBCA requirement of a pre-suit demand As the Official Comment to MBCA sect 742 points out the MBCArsquos requirement of ldquouniversal demandrdquo gives the board ldquothe opportunity to reexamine the act complained of in the light of a potential lawsuit and take corrective actionrdquo even when the directors might be non-independent or have conflicts of interest

Nor is it relevant to the MBCA pre-suit demand requirement that the statutory 90-day waiting period may be onerous The first paragraph of MBCA sect 742 requires a pre-suit demand without exception the second paragraph of the section imposes a 90-day waiting period before a derivative suit may be brought which can be shortened if the board rejects the demand or ldquoirreparable injury to the corporation would result by waiting for the expiration of the 90-day periodrdquo The call as written asks only whether a pre-suit demand should be made and does not ask examinees to address whether the post-demand waiting period should be shortened under the ldquoirreparable injuryrdquo standard]

35

National Conference of Bar Examiners 302 South Bedford Street | Madison WI 53703-3622 Phone 608-280-8550 | Fax 608-280-8552 | TDD 608-661-1275

wwwncbexorg e-mail contactncbexorg

  • Preface
  • Description of the MEE
  • Instructions
  • July 2014 Questions
    • CRIMINAL LAW AND PROCEDURE QUESTION
    • CONTRACTS QUESTION
    • FAMILY LAW QUESTION
    • FEDERAL CIVIL PROCEDURE QUESTION
    • EVIDENCE QUESTION
    • CORPORATIONS QUESTION
      • July 2014 Analyses
        • CRIMINAL LAW AND PROCEDURE ANALYSIS
        • CONTRACTS ANALYSIS
        • FAMILY LAW ANALYSIS
        • FEDERAL CIVIL PROCEDURE ANALYSIS
        • EVIDENCE ANALYSIS
        • CORPORATIONS ANALYSIS
            • ltlt13 ASCII85EncodePages false13 AllowTransparency false13 AutoPositionEPSFiles true13 AutoRotatePages None13 Binding Left13 CalGrayProfile (Dot Gain 20)13 CalRGBProfile (sRGB IEC61966-21)13 CalCMYKProfile (US Web Coated 050SWOP051 v2)13 sRGBProfile (sRGB IEC61966-21)13 CannotEmbedFontPolicy Error13 CompatibilityLevel 1413 CompressObjects Tags13 CompressPages true13 ConvertImagesToIndexed true13 PassThroughJPEGImages true13 CreateJobTicket false13 DefaultRenderingIntent Default13 DetectBlends true13 DetectCurves 0000013 ColorConversionStrategy CMYK13 DoThumbnails false13 EmbedAllFonts true13 EmbedOpenType false13 ParseICCProfilesInComments true13 EmbedJobOptions true13 DSCReportingLevel 013 EmitDSCWarnings false13 EndPage -113 ImageMemory 104857613 LockDistillerParams false13 MaxSubsetPct 10013 Optimize true13 OPM 113 ParseDSCComments true13 ParseDSCCommentsForDocInfo true13 PreserveCopyPage true13 PreserveDICMYKValues true13 PreserveEPSInfo true13 PreserveFlatness true13 PreserveHalftoneInfo false13 PreserveOPIComments true13 PreserveOverprintSettings true13 StartPage 113 SubsetFonts true13 TransferFunctionInfo Apply13 UCRandBGInfo Preserve13 UsePrologue false13 ColorSettingsFile ()13 AlwaysEmbed [ true13 ]13 NeverEmbed [ true13 ]13 AntiAliasColorImages false13 CropColorImages true13 ColorImageMinResolution 30013 ColorImageMinResolutionPolicy OK13 DownsampleColorImages true13 ColorImageDownsampleType Bicubic13 ColorImageResolution 30013 ColorImageDepth -113 ColorImageMinDownsampleDepth 113 ColorImageDownsampleThreshold 15000013 EncodeColorImages true13 ColorImageFilter DCTEncode13 AutoFilterColorImages true13 ColorImageAutoFilterStrategy JPEG13 ColorACSImageDict ltlt13 QFactor 01513 HSamples [1 1 1 1] VSamples [1 1 1 1]13 gtgt13 ColorImageDict ltlt13 QFactor 01513 HSamples [1 1 1 1] VSamples [1 1 1 1]13 gtgt13 JPEG2000ColorACSImageDict ltlt13 TileWidth 25613 TileHeight 25613 Quality 3013 gtgt13 JPEG2000ColorImageDict ltlt13 TileWidth 25613 TileHeight 25613 Quality 3013 gtgt13 AntiAliasGrayImages false13 CropGrayImages true13 GrayImageMinResolution 30013 GrayImageMinResolutionPolicy OK13 DownsampleGrayImages true13 GrayImageDownsampleType Bicubic13 GrayImageResolution 30013 GrayImageDepth -113 GrayImageMinDownsampleDepth 213 GrayImageDownsampleThreshold 15000013 EncodeGrayImages true13 GrayImageFilter DCTEncode13 AutoFilterGrayImages true13 GrayImageAutoFilterStrategy JPEG13 GrayACSImageDict ltlt13 QFactor 01513 HSamples [1 1 1 1] VSamples [1 1 1 1]13 gtgt13 GrayImageDict ltlt13 QFactor 01513 HSamples [1 1 1 1] VSamples [1 1 1 1]13 gtgt13 JPEG2000GrayACSImageDict ltlt13 TileWidth 25613 TileHeight 25613 Quality 3013 gtgt13 JPEG2000GrayImageDict ltlt13 TileWidth 25613 TileHeight 25613 Quality 3013 gtgt13 AntiAliasMonoImages false13 CropMonoImages true13 MonoImageMinResolution 120013 MonoImageMinResolutionPolicy OK13 DownsampleMonoImages true13 MonoImageDownsampleType Bicubic13 MonoImageResolution 120013 MonoImageDepth -113 MonoImageDownsampleThreshold 15000013 EncodeMonoImages true13 MonoImageFilter CCITTFaxEncode13 MonoImageDict ltlt13 K -113 gtgt13 AllowPSXObjects false13 CheckCompliance [13 None13 ]13 PDFX1aCheck false13 PDFX3Check false13 PDFXCompliantPDFOnly false13 PDFXNoTrimBoxError true13 PDFXTrimBoxToMediaBoxOffset [13 00000013 00000013 00000013 00000013 ]13 PDFXSetBleedBoxToMediaBox true13 PDFXBleedBoxToTrimBoxOffset [13 00000013 00000013 00000013 00000013 ]13 PDFXOutputIntentProfile ()13 PDFXOutputConditionIdentifier ()13 PDFXOutputCondition ()13 PDFXRegistryName ()13 PDFXTrapped False1313 CreateJDFFile false13 Description ltlt13 ARA 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 BGR ltFEFF04180437043f043e043b043704320430043904420435002004420435043704380020043d0430044104420440043e0439043a0438002c00200437043000200434043000200441044a0437043404300432043004420435002000410064006f00620065002000500044004600200434043e043a0443043c0435043d04420438002c0020043c0430043a04410438043c0430043b043d043e0020043f044004380433043e04340435043d04380020043704300020043204380441043e043a043e043a0430044704350441044204320435043d0020043f04350447043004420020043704300020043f044004350434043f0435044704300442043d04300020043f043e04340433043e0442043e0432043a0430002e002000200421044a04370434043004340435043d043804420435002000500044004600200434043e043a0443043c0435043d044204380020043c043e0433043004420020043404300020044104350020043e0442043204300440044f0442002004410020004100630072006f00620061007400200438002000410064006f00620065002000520065006100640065007200200035002e00300020043800200441043b0435043404320430044904380020043204350440044104380438002egt13 CHS ltFEFF4f7f75288fd94e9b8bbe5b9a521b5efa7684002000410064006f006200650020005000440046002065876863900275284e8e9ad88d2891cf76845370524d53705237300260a853ef4ee54f7f75280020004100630072006f0062006100740020548c002000410064006f00620065002000520065006100640065007200200035002e003000204ee553ca66f49ad87248672c676562535f00521b5efa768400200050004400460020658768633002gt13 CHT ltFEFF4f7f752890194e9b8a2d7f6e5efa7acb7684002000410064006f006200650020005000440046002065874ef69069752865bc9ad854c18cea76845370524d5370523786557406300260a853ef4ee54f7f75280020004100630072006f0062006100740020548c002000410064006f00620065002000520065006100640065007200200035002e003000204ee553ca66f49ad87248672c4f86958b555f5df25efa7acb76840020005000440046002065874ef63002gt13 CZE 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 DAN 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 DEU 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 ESP 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 ETI 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 FRA 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 GRE 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 HEB ltFEFF05D405E905EA05DE05E905D5002005D105D405D205D305E805D505EA002005D005DC05D4002005DB05D305D9002005DC05D905E605D505E8002005DE05E105DE05DB05D9002000410064006F006200650020005000440046002005D405DE05D505EA05D005DE05D905DD002005DC05D405D305E405E105EA002005E705D305DD002D05D305E405D505E1002005D005D905DB05D505EA05D905EA002E002005DE05E105DE05DB05D90020005000440046002005E905E005D505E605E805D5002005E005D905EA05E005D905DD002005DC05E405EA05D905D705D4002005D105D005DE05E605E205D505EA0020004100630072006F006200610074002005D5002D00410064006F00620065002000520065006100640065007200200035002E0030002005D505D205E805E105D005D505EA002005DE05EA05E705D305DE05D505EA002005D905D505EA05E8002E05D005DE05D905DD002005DC002D005000440046002F0058002D0033002C002005E205D905D905E005D5002005D105DE05D305E805D905DA002005DC05DE05E905EA05DE05E9002005E905DC0020004100630072006F006200610074002E002005DE05E105DE05DB05D90020005000440046002005E905E005D505E605E805D5002005E005D905EA05E005D905DD002005DC05E405EA05D905D705D4002005D105D005DE05E605E205D505EA0020004100630072006F006200610074002005D5002D00410064006F00620065002000520065006100640065007200200035002E0030002005D505D205E805E105D005D505EA002005DE05EA05E705D305DE05D505EA002005D905D505EA05E8002Egt13 HRV (Za stvaranje Adobe PDF dokumenata najpogodnijih za visokokvalitetni ispis prije tiskanja koristite ove postavke Stvoreni PDF dokumenti mogu se otvoriti Acrobat i Adobe Reader 50 i kasnijim verzijama)13 HUN 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 ITA ltFEFF005500740069006c0069007a007a006100720065002000710075006500730074006500200069006d0070006f007300740061007a0069006f006e00690020007000650072002000630072006500610072006500200064006f00630075006d0065006e00740069002000410064006f00620065002000500044004600200070006900f900200061006400610074007400690020006100200075006e00610020007000720065007300740061006d0070006100200064006900200061006c007400610020007100750061006c0069007400e0002e0020004900200064006f00630075006d0065006e007400690020005000440046002000630072006500610074006900200070006f00730073006f006e006f0020006500730073006500720065002000610070006500720074006900200063006f006e0020004100630072006f00620061007400200065002000410064006f00620065002000520065006100640065007200200035002e003000200065002000760065007200730069006f006e006900200073007500630063006500730073006900760065002egt13 JPN ltFEFF9ad854c18cea306a30d730ea30d730ec30b951fa529b7528002000410064006f0062006500200050004400460020658766f8306e4f5c6210306b4f7f75283057307e305930023053306e8a2d5b9a30674f5c62103055308c305f0020005000440046002030d530a130a430eb306f3001004100630072006f0062006100740020304a30883073002000410064006f00620065002000520065006100640065007200200035002e003000204ee5964d3067958b304f30533068304c3067304d307e305930023053306e8a2d5b9a306b306f30d530a930f330c8306e57cb30818fbc307f304c5fc59808306730593002gt13 KOR ltFEFFc7740020c124c815c7440020c0acc6a9d558c5ec0020ace0d488c9c80020c2dcd5d80020c778c1c4c5d00020ac00c7a50020c801d569d55c002000410064006f0062006500200050004400460020bb38c11cb97c0020c791c131d569b2c8b2e4002e0020c774b807ac8c0020c791c131b41c00200050004400460020bb38c11cb2940020004100630072006f0062006100740020bc0f002000410064006f00620065002000520065006100640065007200200035002e00300020c774c0c1c5d0c11c0020c5f40020c2180020c788c2b5b2c8b2e4002egt13 LTH 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 LVI 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 NLD (Gebruik deze instellingen om Adobe PDF-documenten te maken die zijn geoptimaliseerd voor prepress-afdrukken van hoge kwaliteit De gemaakte PDF-documenten kunnen worden geopend met Acrobat en Adobe Reader 50 en hoger)13 NOR 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 POL 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 PTB ltFEFF005500740069006c0069007a006500200065007300730061007300200063006f006e00660069006700750072006100e700f50065007300200064006500200066006f0072006d00610020006100200063007200690061007200200064006f00630075006d0065006e0074006f0073002000410064006f0062006500200050004400460020006d00610069007300200061006400650071007500610064006f00730020007000610072006100200070007200e9002d0069006d0070007200650073007300f50065007300200064006500200061006c007400610020007100750061006c00690064006100640065002e0020004f007300200064006f00630075006d0065006e0074006f00730020005000440046002000630072006900610064006f007300200070006f00640065006d0020007300650072002000610062006500720074006f007300200063006f006d0020006f0020004100630072006f006200610074002000650020006f002000410064006f00620065002000520065006100640065007200200035002e0030002000650020007600650072007300f50065007300200070006f00730074006500720069006f007200650073002egt13 RUM 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 RUS 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 SKY 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 SLV 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 SUO 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 SVE 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 TUR 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 UKR ltFEFF04120438043a043e0440043804410442043e043204430439044204350020044604560020043f043004400430043c043504420440043800200434043b044f0020044104420432043e04400435043d043d044f00200434043e043a0443043c0435043d044204560432002000410064006f006200650020005000440046002c0020044f043a04560020043d04300439043a04400430044904350020043f045604340445043e0434044f0442044c00200434043b044f0020043204380441043e043a043e044f043a04560441043d043e0433043e0020043f0435044004350434043404400443043a043e0432043e0433043e0020043404400443043a0443002e00200020042104420432043e04400435043d045600200434043e043a0443043c0435043d0442043800200050004400460020043c043e0436043d04300020043204560434043a0440043804420438002004430020004100630072006f006200610074002004420430002000410064006f00620065002000520065006100640065007200200035002e0030002004300431043e0020043f04560437043d04560448043e04570020043204350440044104560457002egt13 ENU (Use these settings to create Adobe PDF documents best suited for high-quality prepress printing Created PDF documents can be opened with Acrobat and Adobe Reader 50 and later)13 gtgt13 Namespace [13 (Adobe)13 (Common)13 (10)13 ]13 OtherNamespaces [13 ltlt13 AsReaderSpreads false13 CropImagesToFrames true13 ErrorControl WarnAndContinue13 FlattenerIgnoreSpreadOverrides false13 IncludeGuidesGrids false13 IncludeNonPrinting false13 IncludeSlug false13 Namespace [13 (Adobe)13 (InDesign)13 (40)13 ]13 OmitPlacedBitmaps false13 OmitPlacedEPS false13 OmitPlacedPDF false13 SimulateOverprint Legacy13 gtgt13 ltlt13 AddBleedMarks false13 AddColorBars false13 AddCropMarks false13 AddPageInfo false13 AddRegMarks false13 ConvertColors ConvertToCMYK13 DestinationProfileName ()13 DestinationProfileSelector DocumentCMYK13 Downsample16BitImages true13 FlattenerPreset ltlt13 PresetSelector MediumResolution13 gtgt13 FormElements false13 GenerateStructure false13 IncludeBookmarks false13 IncludeHyperlinks false13 IncludeInteractive false13 IncludeLayers false13 IncludeProfiles false13 MultimediaHandling UseObjectSettings13 Namespace [13 (Adobe)13 (CreativeSuite)13 (20)13 ]13 PDFXOutputIntentProfileSelector DocumentCMYK13 PreserveEditing true13 UntaggedCMYKHandling LeaveUntagged13 UntaggedRGBHandling UseDocumentProfile13 UseDocumentBleed false13 gtgt13 ]13gtgt setdistillerparams13ltlt13 HWResolution [2400 2400]13 PageSize [612000 792000]13gtgt setpagedevice13

Page 14: July 2014 MEE Questions and Analyses - NCBE...This publication includes the questions and analyses from the July 2014 MEE. (In the actual test, the questions are simply numbered rather

July 2014 MEE

ANALYSES

Contracts Family Law

Criminal Law and Procedure

Federal Civil Procedure Evidence

Corporations

CRIMINAL LAW AND PROCEDURE ANALYSIS (Criminal Law and Procedure VA B D)

ANALYSIS

Legal Problems

(1) Did the detective violate the suspectrsquos Sixth Amendment right to counsel when he questioned the suspect about the burglaries without the lawyer present given that the lawyer represented the suspect in an unrelated criminal matter

(2) Under Miranda did the suspect effectively invoke his right to counsel when he said ldquoI think I want my lawyer here before I talk to yourdquo

(3) Was the suspectrsquos waiver of his right to remain silent under Miranda valid

DISCUSSION

Summary

The Sixth Amendment right to counsel as applied to states through the Fourteenth Amendment is offense-specific Although the suspect had an attorney representing him on his pending assault charge he had no Sixth Amendment right to the assistance of counsel with respect to the five uncharged burglaries because formal adversarial proceedings had not yet commenced on those charges The suspectrsquos Sixth Amendment right to counsel was not violated by the detectiversquos failure to inform him that the lawyer was present or of the lawyerrsquos demands

However a person undergoing custodial interrogation also has an independent constitutional right to counsel during custodial interrogation under Miranda When a suspect invokes his right to counsel under Miranda custodial interrogation must immediately cease for a period of at least 14 days However the invocation of the right to counsel must be unambiguous and clearly convey that the suspect has requested counsel Here because the suspectrsquos statement ldquoI think I want my lawyer here before I talk to yourdquo was ambiguous he did not invoke his Miranda right to counsel

A waiver of rights must be knowing intelligent and voluntary Here the suspect waived his right to remain silent under Miranda when he signed the waiver form The fact that the detective did not correct the suspectrsquos assumption that the lawyer would need to drive to the jailmdashby telling him that the lawyer was in the waiting room and was demanding to see himmdashdid not affect the validity of the suspectrsquos waiver

Point One (35) The suspectrsquos Sixth Amendment right to counsel was not violated because the right does not attach on new charges until formal adversarial judicial proceedings have commenced on those charges

The Sixth Amendment as applied to the states through the Fourteenth Amendment provides that ldquo[i]n all criminal prosecutions the accused shall enjoy the right to have the Assistance of Counsel for his defenserdquo The right to counsel does not attach with respect to particular charges until formal adversarial judicial proceedings have commenced (ie ldquoat or after the initiation of

13

Criminal Law and Procedure Analysis

adversary judicial criminal proceedingsmdashwhether by way of formal charge preliminary hearing indictment information or arraignment [or in some states arrest warrant]rdquo McNeil v Wisconsin 501 US 171 175 (1991) (internal quotations omitted)) Once a suspectrsquos Sixth Amendment right to counsel has attached any attempts to ldquodeliberately elicitrdquo statements from him in the absence of his attorney violate the Sixth Amendment See Massiah v United States 377 US 201 (1964) Brewer v Williams 430 US 387 (1977)

The Sixth Amendment right to counsel is charge- or offense -specific Representation by counsel in one prosecution does not in itself guarantee counsel for uncharged offenses See McNeil 501 US at 175 Texas v Cobb 532 US 162 (2001) Here the suspectrsquos Sixth Amendment right to counsel had attached only for the pending aggravated assault charge The suspectrsquos right to counsel for the aggravated assault case did not guarantee counsel for the five unrelated and uncharged burglaries that were the subject of the detectiversquos interrogation Thus because formal adversarial judicial proceedings against the suspect for the uncharged burglaries had not begun he had no Sixth Amendment right to counsel

Finally the detectiversquos failure to inform the suspect of the lawyerrsquos presence and demands to speak with him does not implicate the suspectrsquos Sixth Amendment right to counsel which had not yet attached See id Moran v Burbine 475 US 412 428ndash31 (1986)

Point Two (30) The suspect did not effectively invoke his right to counsel under Miranda because his statement was not unambiguous

A suspect subject to custodial interrogation has a right to consult with counsel and to have an attorney present during questioning Miranda v Arizona 384 US 436 (1966) When a suspect invokes his right to counsel during an interrogation law enforcement must immediately cease all questioning See Edwards v Arizona 451 US 477 484ndash85 (1981) Custodial interrogation cannot be reinitiated unless and until the suspect has been re-advised of his Miranda rights has provided a knowing and voluntary waiver and (1) counsel is present and (2) the suspect himself initiated further communication with the police see id at 484 or (3) (if the suspect was released from custody after the initial interrogation) at least 14 days have passed Maryland v Shatzer 559 US 98 110 (2010)

To invoke the right to counsel a suspectrsquos request must be ldquounambiguousrdquo This means that the suspect must articulate the desire for counsel sufficiently clearly that a reasonable officer would understand the statement to be a request for counsel Davis v United States 512 US 452 459 (1994) If the request is ambiguous the police are not required to stop the interrogation

In this case the suspectrsquos statement ldquoI think I want my lawyer here before I talk to yourdquo was not an unambiguous request for counsel The most reasonable interpretation of this statement is that the suspect might be invoking his right to counsel Id at 461 (ldquomaybe I should talk to a lawyerrdquo is not an unequivocal request for counsel) See also Burket v Angelone 208 F3d 172 197ndash98 (4th Cir 2000) (ldquoI think I need a lawyerrdquo is not an unambiguous request for an attorney) Soffar v Cockrell 300 F3d 588 594ndash95 (5th Cir 2002) (discussion of various statements that did not constitute unequivocal requests for counsel)

Under these circumstances the detective was not required to cease the custodial interrogation of the suspect Nor was the detective required to clarify or ask follow-up questions to determine whether the suspect in fact wanted an attorney Davis 512 US at 459ndash60

14

Criminal Law and Procedure Analysis

Point Three (35) The suspectrsquos waiver of his Miranda rights was knowing intelligent and voluntary despite the fact that he was never told of the lawyerrsquos presence in the jail or of the lawyerrsquos demands

A valid waiver of Miranda rights must be ldquovoluntaryrdquomdashie the product of a free or deliberate choice rather than intimidation coercion or deception Berghuis v Thompkins 560 US 370 382ndash83 (2010) In addition the waiver must be knowing and intelligent That is it ldquomust have been made with a full awareness of both the nature of the right being abandoned and the consequences of the decision to abandon itrdquo Moran v Burbine 475 US 421 (1986)

In this case the suspect signed a Miranda waiver form after receiving proper warnings There is no evidence ldquothat the police resorted to physical or psychological pressure to elicit the statementsrdquo Id The entire interview lasted only 45 minutes The only issue is whether the suspect knowingly and intelligently waived his Miranda rights despite the fact that the detective did not tell the suspect about the lawyerrsquos presence and her demands

The Supreme Court has said that ldquo[e]vents occurring outside of the presence of the suspect and entirely unknown to him surely can have no bearing on the capacity to comprehend and knowingly relinquish a constitutional rightrdquo Id at 422 If the suspect ldquoknew that he could stand mute and request a lawyer and was aware of the Statersquos intention to use his statements to secure a convictionrdquo then the waiver is valid regardless of the information withheld Id at 422ndash23

Here the suspect was correctly informed of his rights Miranda v Arizona 384 US at 467ndash73 His comments demonstrate that he understood that he could have a lawyer present if he desired (ie wondering whether he should call his attorney) and that he understood that there might be consequences to speaking with the detective (ldquoI probably should keep my mouth shut but Irsquom willing to talk to you for a whilerdquo) His comment ldquo[L]etrsquos not waste any time waiting for someone to call my attorney and having her drive hererdquo along with his signature on the Miranda waiver form show that his waiver was valid under the constitutional standard

The fact that the detective did not tell the suspect about the lawyerrsquos presence and demands has no bearing on the validity of the suspectrsquos waiver because ldquosuch conduct is only relevant to the constitutional validity of a waiver if it deprives a defendant of knowledge essential to his ability to understand the nature of his rights and the consequences of abandoning themrdquo Moran at 424 The Supreme Court has specifically declined to adopt a rule requiring that law enforcement tell a suspect of an attorneyrsquos efforts to contact him id at 425 (ldquoNor are we prepared to adopt a rule requiring that the police inform a suspect of an attorneyrsquos efforts to reach himrdquo)

[NOTE An examinee might also recognize that this general rule is further supported by the Supreme Courtrsquos decision in Florida v Powell 559 US 50 (2010) approving state Miranda warnings that do not explicitly warn suspects that they have a right to have counsel present during custodial interrogation]

15

CONTRACTS ANALYSIS (Contracts IB2 IIB IVA3 amp A5)

ANALYSIS

Legal Problems

(1) In the case of a service contract (governed by the common law of contracts) is a modification enforceable when a party agrees to pay more for the same performance than was originally promised

(2) In the case of a contract for the sale of goods (governed by Article 2 of the UCC) is a modification enforceable when a party agrees to pay more for the same goods than was originally promised

(3) May a party avoid an agreement on the basis of economic duress

DISCUSSION

Summary

There are two arguments that the conservatory can make to support the claim that it is not bound to pay the higher prices lack of consideration and economic duress

The organ repair contract is governed by the common law of contracts Under the common law the business would have difficulty recovering the additional $60000 for the organ repair because under the ldquopreexisting duty rulerdquo the agreement of the conservatory to pay the extra price was not supported by consideration However the business might argue that the modification is enforceable under an exception to the preexisting duty rule for fair and equitable modifications made in light of unanticipated circumstances

The organ sale contract is governed by Article 2 of the Uniform Commercial Code The business would likely recover the additional amount under that contract because Article 2 provides that consideration is not required for a modification to be binding

In both cases the conservatory could seek to avoid its agreement on the grounds of economic duress but that argument is not likely to succeed

Point One (45) The business probably cannot recover the additional $60000 for the organ repair because the conservatoryrsquos promise to pay more money was not supported by consideration

The general rule is that to be enforceable a promise must be supported by consideration Under RESTATEMENT (SECOND) OF CONTRACTS sect 71 a promise is supported by consideration if it is bargained for in exchange for a return promise or performance However under the ldquopreexisting duty rulerdquo (exemplified in RESTATEMENT (SECOND) OF CONTRACTS sect 73 and Alaska Packersrsquo Assrsquon v Domenico 117 F 99 (9th Cir 1902)) promise of performance of a legal duty already owed to a promisor which is neither doubtful nor the subject of honest dispute is not consideration

If the business had promised the conservatory anything new or different in exchange for the agreement to pay the additional $60000 (such as for example repairing the pipe organ more

16

Contracts Analysis

quickly or using better parts) that would constitute consideration especially in light of the principle that courts do not inquire into the adequacy of consideration Here however the business already had a legal duty under the original contract and did not agree to do anything else in exchange for the conservatoryrsquos promise to pay $60000 more

However an exception to the preexisting duty rule is sometimes applied in situations of unanticipated changed circumstances Under RESTATEMENT (SECOND) OF CONTRACTS sect 89 followed in many jurisdictions a promise modifying a duty under a contract not fully performed on either side is binding even if not supported by consideration if the modification is fair and equitable in view of circumstances not anticipated by the parties when the contract was made

If a court applies the rule in Restatement sect 89 the critical issues will be whether the modification was in fact ldquofair and equitablerdquo and whether it can be justified in light of unanticipated circumstances In many cases in which modifications have been upheld a party encountered difficulties or burdens in performing far beyond what was knowingly bargained for in the original contract with the result bordering on impracticability such as having to excavate solid rock instead of soft dirt or having to remove garbage far in excess of the amounts contemplated The conservatory would argue that the businessrsquos performance difficulties were not of this sort at allmdashnothing about repairing the pipe organ itself was any different from or more difficult than originally contemplated except that the business itself encountered financial distress unrelated to its burdens in performing its obligations under these contracts

Even if the business satisfies that element of the rule in Restatement sect 89 the business must also demonstrate that the circumstances that gave rise to the need to modify the contract were ldquounanticipatedrdquo at the time the original contract was made Here the facts suggest that when the business entered into the original contract it expected that the price paid by the conservatory would enable it to perform However any evidence that the business knew or had reason to know at the time of execution that it would need more money from the conservatory to be able to perform would mean that the request to modify was not ldquounanticipatedrdquo

[NOTE Some cases such as Schwartzreich v Bauman-Basch Inc 231 NY 196 131 NE 887 (1921) find that if the parties mutually agreed to rescind the original contract and then after rescission entered into an entirely new contract for a higher price the new contract is supported by consideration There is no evidence that such a rescission followed by a new contract took place here]

Point Two (45) The business can recover the additional $40000 for the new organ because no consideration is required under Article 2 of the UCC for good-faith contract modifications

The contract to buy a new organ is a contract for the sale of goods and therefore is governed by Article 2 of the Uniform Commercial Code UCC sect 2-102 Under Article 2 unlike the common law an agreement modifying a contract needs no consideration to be binding UCC sect 2-209(1) Section 2-209(1) thus obviates the preexisting duty rule entirely in contracts for the sale of goods

Even though consideration is not required modifications governed by sect 2-209 must satisfy the obligation of good faith imposed by the UCC UCC sect 1-304 See also Official Comment 2 to UCC sect 2-209 Good faith means ldquohonesty in fact and the observance of reasonable commercial standards of fair dealingrdquo UCC sect 1-201(b)(20) In this context the obligation of good faith means that ldquo[t]he effective use of bad faith to escape performance on the original contract terms is barred and the extortion of a lsquomodificationrsquo without legitimate commercial reason is ineffective as a violation of the duty of good faithrdquo Official Comment 2 to

17

Contracts Analysis

UCC sect 2-209 Here because the businessrsquos financial reversals were serious and apparently unanticipated at the time that the business entered into the contract with the conservatory and commitment of the extra money was needed to enable the business to perform a court would likely find that the business acted in good faith Thus a court would likely uphold the enforceability of the conservatoryrsquos promise to pay the additional $40000

Point Three (10) The conservatory is unlikely to be able to defend against enforcement of its promises to pay additional money under the theory of economic duress because the business probably did not make an improper threat

Under the common law of contracts parties may raise the defense of duress This common law defense also applies to contracts governed by UCC Article 2 See UCC sect 1-103(b)

A contract is voidable on the ground of economic duress by threat when it is established that a partyrsquos manifestation of assent is induced by an improper threat that leaves the party no reasonable alternative See RESTATEMENT (SECOND) OF CONTRACTS sect 175 See also eg Austin Instrument Inc v Loral Corp 272 NE2d 533 (NY 1971) (a threat to withhold essential goods can constitute duress) In order to void its agreement to pay the additional sum because of economic duress the conservatory must demonstrate that (1) the business made a threat to the conservatory (2) the threat was ldquoimproperrdquo or ldquowrongfulrdquo (3) the threat induced the conservatoryrsquos manifestation of assent to the modification and (4) the threat was sufficiently grave to justify the conservatoryrsquos assent

Here it appears that three of the four elements are likely satisfied The business plainly made a threat Moreover the threat induced the conservatoryrsquos assent to the modification and the threat was sufficiently grave to justify that assent If the conservatory had not agreed to pay the business the extra amounts the conservatory would have lost its entire $325000 investment In light of this potential loss a court could easily conclude that the conservatory had no reasonable alternative

However the business has a strong argument that its threat (indicating that it would breach the contracts unless the prices were increased) was not wrongful or improper but was instead nothing more than a communication of the reality of its own perilous situation to the conservatory

A mere threat to breach a contract is not in and of itself improper so as to support an action of economic duress or business compulsion Something more is required such as a breach of the duty of good faith and fair dealing as was present in Austin Instrument Inc supra Because the business could not perform the original contract without the requested modification the economic duress claim for the conservatory would likely fail for much the same reason that the business would be able to enforce the modification At the time the modification was requested the business was not trying to extort a price increase because of the conservatoryrsquos vulnerability but instead was simply stating the reality that the business could not perform without more money

18

FAMILY LAW ANALYSIS (Family Law IIIB D amp G)

ANALYSIS

Legal Problems

(1)(a) Does the State A court have jurisdiction to modify the State B child support order

(1)(b) Does the State A court have jurisdiction to modify the marital-residence-saleshyproceeds provision of the State B property-division decree

(2)(a) May a child support order be modified retroactively

(2)(b) May a child support order be modified prospectively based on a change of employment with a lower salary

(2)(c) May a property-division order be modified after entry of a divorce decree

DISCUSSION

Summary

The State A court may exercise personal jurisdiction over the wife because she was personally served in State A However subject-matter jurisdiction over the interstate modification of child support is governed by the Uniform Interstate Family Support Act (UIFSA) Under UIFSA State A does not have jurisdiction to modify the order for the daughterrsquos support because the wife is still a resident of State B UIFSA on the other hand does not govern property distributions and thus a State A court is not precluded from hearing the husbandrsquos petition to modify the marital-residence-sale-proceeds provision of the divorce decree

A child support order may not be modified retroactively A child support order may be modified prospectively based on a substantial change in circumstances Courts agree that a significant decrease in income is a substantial change in circumstances All states treat voluntary income reductions differently than involuntary reductions but employ different approaches for evaluating the impact of a voluntary reduction Whether the husband could obtain prospective modification of the child support order depends on which approach is applied

A property-division order is not subject to post-divorce modification based on a change in circumstances Thus the husband may in some states obtain prospective modification of the order for the daughterrsquos support but he may not obtain modification of the marital-residenceshysale-proceeds provision

Point One(a) (25) Personal jurisdiction over a nonresident respondent does not confer subject-matter jurisdiction over child support modification Under UIFSA a State A court may not modify a child support order issued by a State B court when as here the child or either parent continues to reside in State B the jurisdiction that issued the child support order

The State A court may exercise personal jurisdiction over the wife The wife was personally served in State A and a state may exercise jurisdiction based on in-state personal service See

19

Family Law Analysis

Burnham v Superior Court 495 US 604 (1990) But personal jurisdiction over the wife is not enough to give a State A court jurisdiction to modify the State B support order

The interstate enforcement and modification of child support is governed by the Uniform Interstate Family Support Act (UIFSA) which has been adopted by all states Under UIFSA the state that originally issued a child support order (here State B) has continuing exclusive jurisdiction to modify the order if that state remains the residence of the obligee the child or the obligor and all parties do not consent to the jurisdiction of another forum See UIFSA sect 205 See also UIFSA sect 603 (ldquoA tribunal of this State shall recognize and enforce but may not modify a registered order if the issuing tribunal had jurisdictionrdquo) The wife and daughter continue to reside in State B and the wife has not consented to the jurisdiction of another forum Thus a State A court does not have jurisdiction to modify the State B child support order

[NOTE Examinees who do not discuss personal jurisdiction but fully discuss UIFSA may receive full credit]

Point One(b) (15) UIFSA does not apply to disputes over property division Thus the State A court may exercise jurisdiction over the husbandrsquos petition to modify the marital-residence-sale-proceeds provision of the State B divorce decree because it has personal jurisdiction over the wife

The State A court in which the husband brought his action has jurisdiction to adjudicate domestic relations issues The husbandrsquos petition to modify the property settlement is a domestic relations issue The courts of State A may exercise personal jurisdiction over the wife because she was personally served in State A See Burnham v Superior Court 495 US 604 (1990) see Point One(a)

UIFSA does not apply to divorce property-division disputes Thus although a State A court may not adjudicate the husbandrsquos petition to modify his child support obligations it may adjudicate his property-division claims (Even though the court has jurisdiction it may not modify the property-division award on the merits See Point Two(c))

Point Two(a) (20) A child support order may not be modified retroactively

State courts have long held that obligations to pay child support ordinarily may not be modified retroactively ldquoIf the hardship is particularly severe the courts sometimes devised a way to protect the obligor but in most instances the courts hold that retroactive modification of this kind is beyond their power and indeed the governing statute may so providerdquo HOMER H CLARK THE LAW OF DOMESTIC RELATIONSHIPS IN THE UNITED STATES 725 (2d ed 1987)

Federal law now goes further and requires the states as a condition of federal child-support funding to adopt rules that absolutely forbid retroactive modification of the support obligation See 42 USC sect 666(a)(9)(C) The states have adopted rules consistent with the federal requirements

Point Two(b) (25) It is unclear whether the husband could obtain prospective downward modification of his child support based on his voluntary acceptance of a job with a lower salary

Prospective modification of a child support order is typically available only when the petitioner can show a substantial change in circumstances See ROBERT E OLIPHANT amp NANCY VER

20

Family Law Analysis

STEEGH FAMILY LAW 213ndash15 (3d ed 2010) A significant decrease in income is typically viewed as a substantial change

However when a parent seeks to modify a child support obligation because he has voluntarily reduced his income a court will not modify the obligation based solely on the income loss Some courts refuse to modify whenever the income shift was voluntary See eg Aguiar v Aguiar 127 P3d 234 (Idaho Ct App 2005) Others look primarily to the petitionerrsquos intentions and permit downward modification if he has acted in good faith See eg In re Marriage of Horn 650 NE2d 1103 (Ill App Ct 1995) Many courts use a multifactor approach See OLIPHANT amp VER STEEGH supra 217ndash18

Here there is no question that the husbandrsquos loss of income was voluntary In a jurisdiction in which voluntary income reduction bars support modification the husbandrsquos petition would be denied

In a jurisdiction employing a good-faith or multifactor approach it is possible but not certain that the husband could obtain downward modification The evidence supports the husbandrsquos good faith his change in employment appears to be based on his new jobrsquos greater responsibilities and better promotion possibilities In a jurisdiction using a multifactor approach the court would likely also consider the impact of such a shift on the daughter the likely duration of the husbandrsquos income loss and the likelihood of a promotion that would ultimately inure to the daughterrsquos benefit Thus on these facts it is possible but by no means certain that the husband could prospectively obtain downward modification of his child support obligation to his daughter

Point Two(c)(15) A divorce property-division award is not subject to modification

A support order is aimed at meeting the post-divorce needs of the supported individual Because the future is unpredictable courts are empowered to modify a support award to take account of changed circumstances that may occur during the period in which support is paid

By contrast a property-distribution award divides assets of the marriage based on the equities at the time of divorce Because the past can be ascertained a property-division award is not subject to post-divorce modification See HARRY A KRAUSE ET AL FAMILY LAW CASES COMMENTS AND QUESTIONS 691 (6th ed 2007)

Here the husband is seeking modification of a property-division award with respect to an asset owned by the parties at the time of divorce Thus the husband may not obtain a modification of the marital-residence-sale-proceeds provision of the divorce decree based on his reduced income

21

FEDERAL CIVIL PROCEDURE ANALYSIS (Federal Civil Procedure III IVC)

ANALYSIS

Legal Problems

(1) Is the logging company entitled to join this action as a matter of right

(2)(a) May the nonprofit organization obtain a temporary restraining order to stop the USFS from issuing a logging permit

(2)(b) May the nonprofit organization obtain a preliminary injunction to stop the USFS from issuing a logging permit during the pendency of the action

DISCUSSION

Summary

The logging company is entitled to intervene in this action as a matter of right because it has an interest in the property or transaction that is the subject of the action and is so situated that its interest may be impaired or impeded as a practical matter if the action goes forward without it The logging companyrsquos interest is not adequately represented by the USFSrsquos presence in the lawsuit

The nonprofit organization may seek a temporary restraining order (TRO) followed by a preliminary injunction to prevent the USFS from issuing a logging permit pending the outcome of the action The nonprofit is likely to obtain a TRO if it can demonstrate a risk of immediate and irreparable injury The nonprofit is also likely to obtain a preliminary injunction if it can demonstrate a significant threat of irreparable harm and a likelihood of success on the merits of its National Environmental Policy Act (NEPA) claim

Point One (50) Rule 24(a) of the Federal Rules of Civil Procedure requires federal courts to allow a person to intervene in an action as a matter of right if the person a) is interested in the property or transaction that is the subject of the action b) is so situated that its interest may be impaired or impeded if the litigation goes forward without it and c) is not adequately represented by existing parties Here the logging company likely meets all three requirements and should be allowed to intervene as a matter of right

Rule 24 of the Federal Rules of Civil Procedure governs intervention the process by which a non-party to an action may join the litigation Under Rule 24(a) (intervention of right) a person must be permitted to intervene if three conditions are met (1) the movant ldquoclaims an interest relating to the property or transaction that is the subject of the actionrdquo (2) the movant ldquois so situated that disposition of the action may as a practical matter impair or impede the movantrsquos ability to protect its interestrdquo and (3) ldquoexisting partiesrdquo do not ldquoadequately represent [the movantrsquos] interestrdquo FED R CIV P 24(a) The three requirements for intervention of right are often ldquovery interrelatedrdquo 7C CHARLES ALAN WRIGHT ET AL FEDERAL PRACTICE AND PROCEDURE sect 1908 at 297 (2007 amp 2011 Supp)

22

Federal Civil Procedure Analysis

Here the court should find that the logging company meets this test First the logging company has a strong interest in the property or transaction that is the subject of this action The USFS has accepted the logging companyrsquos bid and the logging company is merely awaiting issuance of a logging permit to begin logging The nonprofit organization is seeking to prevent this logging The logging company therefore has a strong direct and substantial interest in the subject matter of the lawsuit and in having its winning bid honored and a logging permit issued See eg Kleissler v US Forest Serv 157 F3d 964 972 (3d Cir 1998) (stating that ldquo[t]imber companies have direct and substantial interests in a lawsuit aimed at halting loggingrdquo) see also Natural Resources Defense Council v US Nuclear Regulatory Commrsquon 578 F2d 1341 1343ndash 44 (10th Cir 1978) (holding that applicants whose license renewals were pending had Rule 24(a)(2) interests where the lawsuit sought to halt the license-issuing process pending preparation of environmental impact statements) See generally 7C WRIGHT ET AL supra sect 19081 at 309 (ldquoIf there is a direct substantial legally protectable interest in the proceedings it is clear that this requirement of the rule is satisfiedrdquo) Second the logging companyrsquos interest in receiving a logging permit may well be impaired as a practical matter by the outcome of the lawsuit If the USFS loses the lawsuit it will have to prepare an environmental impact statement before issuing the logging companyrsquos permit This will at a minimum delay the logging companyrsquos ability to exercise its rights and may in the long r un mean that no logging permit is ever issued Intervention of right is not limited to those that would be legally bound as a matter of preclusion doctrine Id sect 19082 at 368 Rather ldquo[t]he rule is satisfied whenever disposition of the present action would put the movant at a practical disadvantage in protecting its interestrdquo Id sect 19082 at 369 Here that condition is easily satisfied See Kleissler 157 F3d at 972 (ldquoTimber companies have direct and substantial interests in a lawsuit aimed at halting logging rdquo)

Given that the logging company has an interest that may be impaired by disposition of the action it should be allowed to intervene unless the court is persuaded that the USFS adequately represents the logging companyrsquos interest See Rule 24(a)(2) 7C WRIGHT ET AL supra sect 1909 Here it could be argued that the USFS adequately represents the logging companyrsquos interest because the USFS presumably wants the court to uphold its development plan and allow it to proceed with issuance of the logging permit which is the same relief that the logging company would seek However whether representation is truly adequate depends upon ldquo[a] discriminating appraisal of the circumstancesrdquo 7C WRIGHT ET AL supra sect 1909 at 440 Although both the government and the logging company wish to avoid the preparation of an environmental impact statement their interests are distinct The USFSrsquos interest is proper management of the national forest system while the logging companyrsquos interest is making a profit from logging the 5000-acre tract The USFSrsquos handling of the litigation is likely to be affected by a variety of policy concerns and political considerations that have nothing to do with the logging companyrsquos purely economic interest in securing the right to cut trees in the Scenic National Forest See eg Kleissler 157 F3d at 973ndash74 (ldquo[T]he government represents numerous complex and conflicting interests in matters of this nature The straightforward business interests asserted by intervenors here may become lost in the thicket of sometimes inconsistent governmental policiesrdquo)

[NOTES (1) Examinees who mistakenly analyze the logging companyrsquos case for joinder under the related but incorrect Rule 19 ldquoRequired Joinder of Partiesrdquo may receive credit Rule 19 allows existing parties to demand joinder of non-parties (or seek dismissal of the case if they canrsquot get it) There is a close relationship between Rule 24 and Rule 19 and both contain a similar standard for determining when ldquointerestedrdquo third parties are ldquoentitledrdquo or ldquorequiredrdquo to be in the lawsuit Indeed the two prongs of the Rule 24 intervention test that are discussed above

23

Federal Civil Procedure Analysis

are nearly identical to the two prongs of the Rule 19(a) required joinder test Examinees who discuss and apply the test should receive credit even if they cite Rule 19 rather than Rule 24

(2) Examinees may discuss permissive joinder Although permissive joinder is a possibility here the question asks only whether the logging company can join the action as a matter of right and a permissive joinder analysis is not responsive to the question To the extent an examinee discusses permissive joinder the analysis will focus on whether the logging company ldquohas a claim or defense that shares with the main action a common question of law or factrdquo FED R CIV P 24(b)(1)(B) The district court also ldquomust consider whether the intervention will unduly delay or prejudice the adjudication of the original partiesrsquo rightsrdquo FED R CIV P 24(b)(3) On our facts the logging companyrsquos claim for the issuance of a logging permit would certainly share common questions of law and fact with the USFSrsquos defense against the nonprofitrsquos claim There are no facts suggesting that the logging companyrsquos presence would unduly delay or otherwise prejudice adjudication of the original action Thus the district court would have discretion to permit the logging company to intervene even if it denied intervention of right]

Point Two(a) (25) The nonprofit organization could seek and would likely obtain a temporary restraining order to stop the USFS from issuing a logging permit pending a hearing on an application for a preliminary injunction

The first type of interim relief the nonprofit could seek to stop the USFS from issuing a logging permit to the logging company is a temporary restraining order (TRO) prohibiting the USFS from issuing the logging permit A TRO can be issued without notice to the adverse party but only in limited circumstances and only for a limited time FED R CIV P 65(b) To secure a TRO without notice the nonprofit would need to submit an affidavit containing specific facts that demonstrate a risk of ldquoimmediate and irreparable injuryrdquo if a permit is issued FED R CIV P 65(b)(1) In deciding whether to grant a TRO courts will also consider the same factors that are relevant in deciding whether to grant a preliminary injunction (eg the moving partyrsquos likelihood of success on the merits the balance of hardships and the public interest) See Point Two(b) infra The TRO would last only long enough for the court to consider and resolve a request by the nonprofit for a preliminary injunction but no longer than 14 days (unless the court extends it for good cause or the adverse party consents to an extension) In addition bond is required

Here the court is likely to grant the nonprofitrsquos request The nonprofit could plausibly claim that cutting down 5000 acres of old-growth forest in an area that is home to the highest concentration of wildlife in the western United States would have ldquoan immediate and irreparablerdquo adverse impact on the environment and cause irreparable harm to the nonprofitrsquos interest in preserving and protecting natural resources including wildlife habitat

Point Two(b) (25) The nonprofit could also seek and would likely obtain a preliminary injunction to stop the USFS which is likely to be granted if the nonprofitrsquos claim that the USFS violated NEPA has a strong basis in fact and law

Because the TRO would be temporary the nonprofit would need to move for a preliminary injunction to prevent the USFS from issuing a logging permit throughout the pendency of the litigation Preliminary injunctions are injunctions that seek to ldquoprotect [the] plaintiff from

24

Federal Civil Procedure Analysis

irreparable injury and to preserve the courtrsquos power to render a meaningful decision after a trial on the meritsrdquo 11A CHARLES ALAN WRIGHT ET AL FEDERAL PRACTICE AND PROCEDURE sect 2947 at 112 (2013) Rule 65 of the Federal Rules of Civil Procedure sets out the procedural requirements for preliminary injunctions Preliminary injunctions may be granted only upon notice to the adverse party FED R CIV P 65(a)(1) and only if the movant ldquogives security in an amount that the court considers proper to pay the costs and damages sustained by any party found to have been wrongfully enjoined or restrainedrdquo FED R CIV P 65(c)

While Rule 65 sets out the procedural requirements for preliminary injunctive relief it does not specify the substantive grounds upon which it may be granted The courtrsquos discretion in ruling upon a motion for a preliminary injunction ldquois exercised in conformity with historic federal equity practicerdquo 11A WRIGHT ET AL supra sect 2947 at 114 The court typically considers four factors

(1) the significance of the threat of irreparable harm to the plaintiff if the injunction is not granted (2) the balance between this harm and the injury that granting the injunction would inflict on the defendant (3) the probability that the plaintiff will succeed on the merits and (4) the public interest

Id sect 2948 at 122ndash24 accord Habitat Educ Center v Bosworth 363 F Supp 2d 1070 1088 (ED Wis 2005) The most important of these factors is the risk of irreparable harm to the plaintiff 11A WRIGHT ET AL supra sect 29481 at 129 If the plaintiff has an adequate remedy at law (eg if money damages can compensate the plaintiff for its loss) then a preliminary injunction will be denied Id sect 29481

Here a court would likely conclude that the potential for environmental damage to the forest creates a significant threat of irreparable harm ldquo[E]nvironmental injury is often irreparable Courts have recognized that logging such as would occur [here] can have longshyterm environmental consequences and thus satisfy the irreparable injury criterionrdquo Habitat Educ Center 363 F Supp 2d at 1089 (citing Idaho Sporting Congress Inc v Alexander 222 F3d 562 569 (9th Cir 2000) (noting that the imminent and continuing logging activities presented ldquoevidence of environmental harm sufficient to tip the balance in favor of injunctive reliefrdquo)) Neighbors of Cuddy Mountain v US Forest Service 137 F3d 1372 1382 (9th Cir 1998) (stating that ldquo[t]he old growth forests plaintiffs seek to protect would if cut take hundreds of years to reproducerdquo) (internal citation omitted)) see also 11C WRIGHT ET AL supra sect 29481 at 151 (noting that ldquoa preliminary injunction has been issued to prevent harm to the environmentrdquo)

The second factor the balance between the harm to the plaintiff and the harm the defendant will suffer if the injunction is issued also appears to support issuance of a preliminary injunction here The USFS will have to wait before it can develop the Scenic National Forest and the logging company may lose money if the delay is prolonged These economic harms could be compensated monetarily if an injunction is issued inappropriately Where ldquoan injunction bond can compensate [the] defendant for any harm the injunction is likely to inflict the balance should be struck in favor of [the] plaintiffrdquo Id sect 29482 at 192 See also Habitat Educ Center 363 F Supp 2d at 1089 (stating that ldquothe relative absence of harmful effects on the Forest Service weighs in favor of granting the injunctionrdquo)

The third factor is the likelihood that the plaintiff will prevail on the merits Although there is limited information concerning the merits of the action the nonprofit alleges that the federal statute (NEPA) requires an environmental impact statement and further states that the USFS created no environmental impact analysis or statement at all Assuming that those

25

Federal Civil Procedure Analysis

allegations are correct it seems plausible to conclude that the nonprofit will be able to show a likelihood of success on the merits

Finally courts deciding whether or not to issue preliminary injunctive relief are to consider the public interest ldquoFocusing on this factor is another way of inquiring whether there are policy considerations that bear on whether the order should issuerdquo 11C WRIGHT ET AL supra sect 29484 at 214 If the court concludes that the nonprofit is likely to succeed on its NEPA claim because the USFS wrongfully failed to conduct an environmental impact assessment it is likely to find that the public interest would be served by restraining the USFS from proceeding with logging in a national forest See Heartwood Inc v US Forest Service 73 F Supp 2d 962 979 (SD Ill 1999) affrsquod on other grounds 230 F3d 947 (7th Cir 2000) (ldquoviolations by federal agencies of NEPArsquos provisions as established by Congress harm the public as well as the environmentrdquo)

Thus a court is very likely to grant a preliminary injunction if it concludes that the nonprofit has a significant likelihood of success on the merits

26

EVIDENCE ANALYSIS (Evidence ID IIA amp C)

ANALYSIS

Legal Problems

(1) Under what circumstances can evidence of prior convictions be used to impeach a witnessrsquos credibility in a civil case

(1)(a) May the inmatersquos credibility be impeached by evidence of a 12-year-old felony drug conviction if he was released from prison 9 years ago

(1)(b) May the inmatersquos credibility be impeached by evidence of an 8-year-old misdemeanor perjury conviction that was punishable by 1 year in jail if he pleaded guilty and was sentenced only to pay a $5000 fine

(1)(c) May the inmatersquos credibility be impeached by evidence of a 7-year-old sexual assault conviction if the inmate is still serving a 10-year prison sentence and the victim was his 13-year-old daughter

(2)(a) May the guardrsquos credibility be impeached by cross-examination regarding specific instances of misconduct (ie lying on his reacutesumeacute) relevant to credibility

(2)(b) May the guardrsquos credibility be impeached by admission of extrinsic evidence (his reacutesumeacute and academic transcript) offered to prove specific instances of misconduct relevant to credibility

DISCUSSION

Summary

Under the Federal Rules of Evidence witnesses can be impeached with evidence of prior convictions andor specific instances of misconduct Whether evidence of prior convictions should be admitted to impeach generally depends on the nature of the crime the amount of time that has passed and (only in criminal cases) whether the ldquowitnessrdquo is the defendant FED R EVID 609(a)

In this civil case evidence of the inmatersquos conviction for distribution of marijuana should be admitted to impeach the inmate because he was convicted of a felony and was released from prison fewer than 10 years ago FED R EVID 609(a)(1) Credibility is critically important in this case because the jury will hear conflicting testimony from the two disputing parties and there were no other eyewitnesses to the altercation Under Rule 609(a)(1) the inmatersquos conviction should be admitted because it has some bearing on his credibility and its probative value is not substantially outweighed by concerns of unfair prejudice confusion or delay Id

Evidence of the inmatersquos misdemeanor conviction for perjury must be admitted because the crime ldquorequired provingmdashor the witnessrsquos admittingmdasha dishonest act or false statementrdquo by the inmate FED R EVID 609(a)(2)

27

Evidence Analysis

Evidence of the inmatersquos felony conviction for sexual assault should be excluded because its probative value is substantially outweighed by the danger of unfair prejudice to the inmate based on the heinous nature of the crime FED R EVID 609(a)(1) In the alternative the judge could limit the evidence relating to this conviction by excluding details of the inmatersquos crime

In all civil (and criminal) cases witnesses can also be impeached with evidence of specific instances of prior misconduct that did not result in a conviction FED R EVID 608(b) Pursuant to Rule 608(b) misconduct probative of untruthfulness can be inquired into on cross-examination but cannot be proved through extrinsic evidence Id Thus the inmatersquos counsel should be permitted to cross-examine the guard regarding the false statement in the guardrsquos reacutesumeacute However extrinsic evidence of the guardrsquos misconduct (ie the guardrsquos authenticated reacutesumeacute and transcript from the local college) should not be admitted even if the guard denies wrongdoing or refuses to answer cross-examination questions about these matters Id

Point One (10) The Federal Rules of Evidence permit impeachment of witnesses with evidence of prior convictions

Whether convictions should be admitted to impeach generally depends on the nature of the crime the amount of time that has passed and (only in criminal cases) whether the ldquowitnessrdquo is the defendant FED R EVID 609(a) Under Rule 609(a) evidence of prior convictions may be admitted for the purpose of ldquoattacking a witnessrsquos character for truthfulnessrdquo Id

There are two basic types of convictions that can be admitted for the purpose of impeachment

(1) convictions for crimes ldquopunishable by death or by imprisonment for more than one yearrdquo (which generally correlates to ldquofeloniesrdquo) FED R EVID 609(a)(1) and (2) convictions ldquofor any crimes regardless of the punishment if the court can readily determine that establishing the elements of the crime required provingmdashor the witnessrsquos admittingmdasha dishonest act or false statementrdquo FED R EVID 609(a)(2)

Pursuant to Rule 609(a)(1) in civil cases the admission of evidence of a felony conviction is ldquosubject to Rule 403 [which says that a court may exclude relevant evidence if its probative value is substantially outweighed by other factors]rdquo FED R EVID 609(a)(1) However Rule 403 does not protect the witness against admission of prior convictions involving dishonestymdashwhich must be admitted by the court FED R EVID 609(a)(2)

Finally Federal Rule of Evidence 609(b) contains the presumption that a conviction that is more than 10 years old or where more than 10 years has passed since the witnessrsquos release from confinement (whichever is later) should not be admitted unless ldquoits probative value supported by specific facts and circumstances substantially outweighs its prejudicial effectrdquo and the proponent has provided the adverse party with reasonable written notice FED R EVID 609(b)

Point One(a) (25) The court should admit evidence of the inmatersquos 12-year-old felony marijuana distribution conviction

The inmatersquos conviction for marijuana distribution was for a felony punishable by imprisonment for more than one year See FED R EVID 609(a)(1) Moreover although the conviction was 12 years ago the 10-year time limit of Rule 609(b) is not exceeded because that time limit runs

28

Evidence Analysis

from the date of either ldquothe witnessrsquos conviction or release from confinement for it whichever is laterrdquo FED R EVID 609(b) Because the inmate served three years in prison he was released from confinement nine years ago

However pursuant to Rule 609(a)(1) the admission of felony convictions to impeach a witness in a civil case is ldquosubject to Rule 403rdquo FED R EVID 609(a)(1) Neither Rule 609(a) nor the advisory committee notes specify which factors courts should consider when balancing the probative value of a conviction against the dangers identified in Rule 403 (which include (1) unfair prejudice (2) confusion of the issues (3) misleading the jury (4) waste of time or undue delay and (5) needless presentation of cumulative evidence) FED R EVID 403

In this case credibility is very important because the evidence consists primarily of the testimony of the disputing parties and there were no other eyewitnesses to the altercation This enhances the probative value of any evidence bearing on the inmatersquos credibility A court is likely to conclude that the inmatersquos prior felony drug conviction is relevant to his credibility See eg United States v Brito 427 F3d 53 64 (1st Cir 2005) (ldquoPrior drug-trafficking crimes are generally viewed as having some bearing on veracityrdquo) Although the probative value of any conviction diminishes with age see eg United States v Brewer 451 F Supp 50 53 (ED Tenn 1978) the inmatersquos ongoing problems with the law suggest that he has continued (and even escalated) his criminal behavior over the past nine years The court should admit this evidence because its probative value is not substantially outweighed by any Rule 403 concerns Specifically any prejudice to the inmate would be slight because the conviction is unrelated to the altercation at issue and the conviction was not for a heinous crime that might inflame the jury

[NOTE Whether an examinee identifies the jury instruction as containing a ldquoconclusiverdquo or ldquomandatoryrdquo presumption is less important than the examineersquos analysis of the constitutional infirmities]

Point One(b) (15) The court must admit evidence of the inmatersquos eight-year-old misdemeanor conviction because perjury is a crime of dishonesty

Rule 609(a)(2) provides that evidence of a criminal conviction ldquomust be admitted if the court can readily determine that establishing the elements of the crime required provingmdashor the witnessrsquos admittingmdasha dishonest act or false statementrdquo FED R EVID 609(a)(2) The inmatersquos conviction for perjury would have necessarily required proving that the inmate engaged in an act of dishonesty This conviction occurred within the past 10 years so it ldquomust be admittedrdquo because in contrast to Rule 609(a)(1) (discussed in Point One(a)) admission under Rule 609(a)(2) is mandatory and not subject to Rule 403

Point One(c) (20) The court should exclude evidence of the inmatersquos seven-year-old felony sexual assault conviction because the probative value of this evidence is substantially outweighed by the danger of unfair prejudice In the alternative the details of the prior conviction could be excluded

The inmatersquos conviction for felony sexual assault was seven years ago and he has not yet been released from incarceration so Rule 609(a) but not 609(b) is applicable here FED R EVID 609(a) This conviction is therefore admissible to impeach the inmate unless its probative value is substantially outweighed by the danger of unfair prejudice or any other Rule 403 concern Id

29

Evidence Analysis

Sex crimes are generally not considered relevant to credibility see Hopkins v State 639 So 2d 1247 1254 (Miss 1993) so the probative value of this conviction is relatively low Moreover the heinous nature of the inmatersquos crime (sexual assault on his daughter) makes the danger of unfair prejudice to the inmate very high Thus the court should exclude evidence of the conviction because it was for a heinous offense that is likely to inflame the jury and it has little bearing on credibility See eg United States v Beahm 664 F2d 414 419 (4th Cir 1981)

As an alternative to excluding this evidence the judge could minimize the unfair prejudice to the inmate by permitting limited cross-examination but refusing to allow specific questions about the nature of the inmatersquos conviction For example a court could limit cross-examination to the fact that the inmate was convicted of a ldquofelonyrdquo or perhaps that he was convicted of a ldquosexual assaultrdquo without identifying the victim However because evidence of the inmatersquos prior convictions can be admitted solely for the purpose of enabling the jury to assess his credibility and because his two earlier convictions should have already been admitted the court should exclude all evidence of the felony sexual assault conviction

Point Two(a) (15) The court should permit the inmatersquos counsel to cross-examine the guard regarding the false statement in his reacutesumeacute because the guardrsquos misconduct bears on his truthfulness

The inmate wishes to cross-examine the guard about his prior dishonest behaviormdashlying on his reacutesumeacutemdashthat did not involve a criminal conviction Rule 608(b) allows witnesses to be cross-examined about specific instances of prior non-conviction misconduct probative of untruthfulness ldquoin order to attack the witnessrsquos character for truthfulnessrdquo FED R EVID 608(b)

The courtrsquos decision to allow cross-examination about the guardrsquos prior dishonest behavior depends on the probative value of such evidence balanced against the danger of unfair prejudice to the guard or any other Rule 403 concern FED R EVID 403 Here the guardrsquos false statement on his reacutesumeacute that he obtained a degree in Criminal Justice is highly probative of his untruthfulness because it grossly misrepresents his actual academic record was made recently and was made with the intent to deceive Because the probative value of this evidence is very strong and is not substantially outweighed by any Rule 403 concerns cross-examination of the guard on this topic should be permitted The court may also consider it fair to permit this cross-examination of the guard on these matters assuming that one or more of the inmatersquos prior convictions have been admitted to impeach his credibility

Point Two(b) (15) The court should exclude extrinsic evidence of the guardrsquos non-conviction misconduct even if the guard denies wrongdoing or refuses to answer questions about the matter

Although Rule 608(b) allows cross-examination about specific instances of prior misconduct probative of untruthfulness ldquoextrinsic evidencerdquo offered to prove such misconduct is not admissible FED R EVID 608(b) The rationale for this rule is that allowing the introduction of extrinsic evidence of prior misconduct by witnesses when these acts are relevant only to the witnessesrsquo truthfulness and not to the main issues in the case would create too great a risk of confusing the jury and unduly delaying the trial The court does not have discretion to admit this extrinsic evidence See eg United States v Elliot 89 F3d 1360 1368 (8th Cir 1996)

30

Evidence Analysis

Here the inmatersquos counsel may cross-examine the guard about the false statement on his reacutesumeacute However the inmatersquos counsel must accept the guardrsquos response Even if the guard denies wrongdoing or refuses to answer questions about the matter the inmatersquos counsel cannot introduce the guardrsquos reacutesumeacute or the transcript from the local college to prove the guardrsquos misconduct

31

CORPORATIONS ANALYSIS (Corporations VA2 IX)

ANALYSIS

Legal Problems

(1) Do shareholders have the authority to amend a corporationrsquos bylaws with respect to director nominations

(2) Do board-approved bylaws on a particular subject here nomination of directors preempt subsequent conflicting bylaw amendments by shareholders

(3) Is a suit challenging both managementrsquos refusal to include the proposed bylaw amendment in Megarsquos proxy statement and the boardrsquos amendment of the bylaws dealing with nomination of directors a direct or derivative suit

DISCUSSION

Summary

The voting and litigation rights of the shareholders of Mega are subject to the provisions of the Model Business Corporations Act (MBCA)

The investorrsquos proposed bylaw provision is not inconsistent with state law Under the MBCA shareholders may amend the bylaws when the amendment deals with a proper matter for the corporationrsquos bylaws such as procedures for nominating directors

The Mega boardrsquos bylaw amendment does not preempt the investorrsquos proposed bylaw provision or the Mega shareholdersrsquo power to approve it While shareholders can limit the boardrsquos power to amend or repeal the bylaws the board cannot limit the shareholdersrsquo power

Whether the investor must make a demand on Megarsquos board depends on how the investor frames its claim If the investor claims a violation of shareholder voting rights the claim is direct and pre-suit demand on the board is not required If on the other hand the investor claims that the directors violated their fiduciary duties by amending the bylaws to entrench themselves the claim is derivative and a pre-suit demand is required

Point One (30) Shareholders may amend the corporationrsquos bylaws where the proposed bylaw provision relates to procedural matters typically included in the bylaws such as the nomination of directors

Internal affairs of the corporation such as the conduct of shareholder meetings and election of directors are subject to the corporate law of the state of incorporation See McDermott Inc v Lewis 531 A2d 206 (Del 1987) (applying law of jurisdiction where corporation was incorporated in case involving voting rights) This statersquos corporate statute is modeled on the MBCA

Under the MBCA ldquoshareholders may amend the corporationrsquos bylawsrdquo MBCA sect 1020(a) Thus the only question is whether the bylaws can specify the procedures for shareholder nomination of directors

32

Corporations Analysis

The MBCA states that the bylaws ldquomay contain any provision that is not inconsistent with law or the articles of incorporationrdquo MBCA sect 206(b) In addition the MBCA was revised in 2009 to address shareholder nomination of directors in public corporations (known as ldquoproxy accessrdquo) and specifies that the bylaws ldquomay contain a requirement that the corporation include in its [proxy materials] one or more individuals nominated by a shareholderrdquo MBCA sect 206(c)(1) see Committee on Corporate Laws ABA Section of Business Law Report on the Roles of Boards of Directors and Shareholders of Publicly Owned Corporations and Changes to the Model Business Corporations ActmdashAdoption of Shareholder Proxy Access Amendments to Chapters 2 and 10 65 BUS LAWYER 1105 (2010)

The inclusion of director-nomination procedures in the bylaws is consistent with practice and is recognized by the Delaware courts whose views on corporate law carry significant weight Typically the procedures for nomination of directors are found in the bylaws See 1 COX amp HAZEN TREATISE ON THE LAW OF CORPORATIONS sect 312 (3d ed 2011) see also 4 FLETCHER CORP FORMS ANN PART III ch 21 (2013) (including sample bylaws that permit nomination of directors by shareholders) The Delaware Supreme Court has confirmed that the bylaws may ldquodefine the process and proceduresrdquo for director elections See CA Inc v AFSCME Employees Pension Plan 953 A2d 227 (Del 2008) (concluding that bylaw amendment requiring reimbursement of election expenses to certain successful shareholder nominators is ldquoproper subjectrdquo under Delaware law)

[NOTE The question of the proper scope of the bylaws can be answered using the more general MBCA sect 206(b) or the 2009 MBCA revision adding sect 206(c)(1) (adopted in CT ME VA) In addition some examinees might raise the point that shareholder proposals may not compel the board to take action such as by including shareholder nominations in the companyrsquos proxy materials on the theory that the ldquobusiness and affairsrdquo of the corporation are to be managed by the board See MBCA sect 801(b) Although shareholders are generally limited to adopting precatory resolutions that recommend or encourage board action this limitation does not apply when shareholders have specific authority to take binding action on their ownmdashsuch as to amend the bylaws]

Point Two (30) Shareholders can amend (or repeal) board-approved bylaws Further shareholders can limit the boardrsquos power to later amend and repeal a shareholder-approved bylaw

Under the MBCA shareholders have the power to amend the bylaws See Point One The board shares this power with the shareholders unless (1) the corporationrsquos articles ldquoreserve that power exclusively to the shareholdersrdquo or (2) ldquothe shareholders in amending repealing or adopting a bylaw expressly provide that the board of directors may not amend repeal or reinstate that bylawrdquo See MBCA sect 1020(b)

Shareholder-approved bylaw provisions can amend or repeal existing bylaw provisions whether originally approved by the board or by shareholders See ALAN R PALMITER CORPORATIONS EXAMPLES AND EXPLANATIONS sect 713 (7th ed 2012) Thus the Mega boardrsquos bylaw amendmentmdashwhich set more demanding thresholds for shareholder nomination of directors than the investorrsquos proposed bylaw provisionmdashwould be superseded (repealed) if Megarsquos shareholders were to approve the investorrsquos proposal

Further a shareholder-approved bylaw generally can limit the power of the board to later amend or repeal it See MBCA sect 1020(b)(2) Thus if Megarsquos shareholders approved the bylaw

33

Corporations Analysis

provision proposed by the investor Megarsquos board could not repeal the provision because it includes a ldquono board repealrdquo clause

The revision to the MBCA in 2009 dealing with shareholder proxy access does not change this conclusion That revision specifies that a shareholder-approved bylaw dealing with director nominations may not limit the boardrsquos power to amend add or repeal ldquoany procedure or condition to such a bylaw in order to provide for a reasonable practicable and orderly processrdquo MBCA sect 206(d) Thus according to the revision if shareholders approve a bylaw amendment that limits further board changes the board would nonetheless retain the power to ldquotinkerrdquo with the bylaw to safeguard the voting process but could not repeal the shareholder-approved bylaw The Official Comment to MBCA sect 206(d) makes clear that the revision is ldquonot intended to allow the board of directors to frustrate the purpose of the shareholder-adopted proxy access provisionrdquo Thus if Megarsquos shareholders were to approve the bylaw provision proposed by the investor Megarsquos board could only amend the provision regarding its procedures or conditions in a manner consistent with its purpose of permitting proxy access for Megarsquos shareholders

[NOTE The boardrsquos attempted interference with a shareholder voting initiative may also have been a violation of the directorsrsquo fiduciary duties See Blasius Indus Inc v Atlas Corp 564 A2d 651 (Del Ch 1988) (finding that directors breached their fiduciary duties by amending bylaws and expanding size of board to thwart insurgentrsquos plan to amend bylaws and seat a majority of new directors) The call however asks examinees to consider whether shareholders or the board have ldquoprecedencerdquo over amending the corporate bylaws Thus an examineersquos answer should be framed in terms of ldquopowerrdquo and not ldquodutyrdquo]

Point Three (40) The investor need not make a demand on the board if the investor states a direct claim such as an allegation that the board interfered with the investorrsquos right to amend the bylaws But the investor must make a demand on the board if the investor states a derivative claim (on behalf of the corporation) such as an allegation that the directors sought to entrench themselves by interfering with the proposed proxy access

The MBCA generally requires that shareholders make a demand on the board of directors before initiation of a derivative suit MBCA sect 742 (shareholder may not bring derivative proceeding until written demand has been made on corporation and 90 days have expired) A derivative suit is essentially two suits in one where the plaintiff-shareholder seeks to bring on behalf of the corporation a claim that vindicates corporate rights usually based on violation of fiduciary duties PALMITER supra sect 1811 (6th ed 2009) The demand permits the board to investigate the situation identified by the shareholder and take suitable action No demand on the board is required however if the shareholder brings a direct suit to vindicate the shareholderrsquos own rights not those of the corporation

Is the suit brought by the investor derivative or direct The MBCA defines a ldquoderivative proceedingrdquo as one brought ldquoin the right of a domestic corporationrdquo MBCA sect 740(1) Thus the answer to how the investorrsquos suit should be characterized turns on what rights the investor seeks to vindicate If the investor frames its claim as one of fiduciary breach by directorsmdashfor example for failing to become adequately informed about voting procedures or for seeking to entrench themselves in office by manipulating the voting structure to avoid a shareholder insurgencymdashthen the suit is ldquoderivativerdquo and the investor must make a demand on the board See MBCA Ch 7 Subch D Introductory Comment (ldquothe derivative suit has historically been the principal method of challenging allegedly illegal action by managementrdquo)

34

Corporations Analysis

If however the investor frames its claim as one to vindicate shareholder rights the suit is direct and no demand is required For many courts the direct-derivative question turns on who is injured and who is to receive the relief sought by the plaintiff-shareholders See Tooley v Donaldson Lufkin amp Jenrette Inc 845 A2d 1031 (Del 2004) (characterizing a merger-delay claim as direct because delay of merger only harmed shareholders not corporation) Thus if the investor claims that managementrsquos refusal to include its proposed bylaw amendment in the corporationrsquos proxy materials violates its shareholder rights to initiate corporate governance reforms the suit will be direct Courts have not questioned the ability of shareholders to bring direct suits challenging board action to exclude their proposed bylaw amendments from the corporationrsquos proxy materials See JANA Master Fund Ltd v CNET Networks Inc 954 A2d 335 (Del Ch 2008) (upholding shareholderrsquos direct challenge to boardrsquos interpretation of advance-notice bylaw) Chesapeake Corp v Shore 771 A2d 293 (Del Ch 2000) (upholding shareholderrsquos direct challenge to actions by board that effectively prevented it from proposing bylaw amendments in contest for control)

Is the way that the investor frames its claim conclusive Courts have permitted shareholder-plaintiffs to challenge a transaction in a direct suit even though the same transaction could also be challenged as a fiduciary breach See Eisenberg v Flying Tiger Line Inc 451 F2d 267 (2d Cir 1971) (permitting direct suit challenging a corporate reorganization as a dilution of shareholder voting power even though reorganization may have involved conflicts of interest and thus constituted a fiduciary breach) Thus the investorrsquos choice to pursue a claim challenging the legality of managementrsquos decision to exclude the investorrsquos proposal from the corporationrsquos proxy materialsmdashrather than a possible breach of fiduciary dutymdashis likely to be respected See 3 COX amp HAZEN supra sect 153 (describing situations in which a claim can be framed as derivative or direct)

[NOTE Some issues under Delaware corporate law regarding pre-suit demand are not relevant here For example whether the Mega directors are independent and disinterested is not relevant to the MBCA requirement of a pre-suit demand As the Official Comment to MBCA sect 742 points out the MBCArsquos requirement of ldquouniversal demandrdquo gives the board ldquothe opportunity to reexamine the act complained of in the light of a potential lawsuit and take corrective actionrdquo even when the directors might be non-independent or have conflicts of interest

Nor is it relevant to the MBCA pre-suit demand requirement that the statutory 90-day waiting period may be onerous The first paragraph of MBCA sect 742 requires a pre-suit demand without exception the second paragraph of the section imposes a 90-day waiting period before a derivative suit may be brought which can be shortened if the board rejects the demand or ldquoirreparable injury to the corporation would result by waiting for the expiration of the 90-day periodrdquo The call as written asks only whether a pre-suit demand should be made and does not ask examinees to address whether the post-demand waiting period should be shortened under the ldquoirreparable injuryrdquo standard]

35

National Conference of Bar Examiners 302 South Bedford Street | Madison WI 53703-3622 Phone 608-280-8550 | Fax 608-280-8552 | TDD 608-661-1275

wwwncbexorg e-mail contactncbexorg

  • Preface
  • Description of the MEE
  • Instructions
  • July 2014 Questions
    • CRIMINAL LAW AND PROCEDURE QUESTION
    • CONTRACTS QUESTION
    • FAMILY LAW QUESTION
    • FEDERAL CIVIL PROCEDURE QUESTION
    • EVIDENCE QUESTION
    • CORPORATIONS QUESTION
      • July 2014 Analyses
        • CRIMINAL LAW AND PROCEDURE ANALYSIS
        • CONTRACTS ANALYSIS
        • FAMILY LAW ANALYSIS
        • FEDERAL CIVIL PROCEDURE ANALYSIS
        • EVIDENCE ANALYSIS
        • CORPORATIONS ANALYSIS
            • ltlt13 ASCII85EncodePages false13 AllowTransparency false13 AutoPositionEPSFiles true13 AutoRotatePages None13 Binding Left13 CalGrayProfile (Dot Gain 20)13 CalRGBProfile (sRGB IEC61966-21)13 CalCMYKProfile (US Web Coated 050SWOP051 v2)13 sRGBProfile (sRGB IEC61966-21)13 CannotEmbedFontPolicy Error13 CompatibilityLevel 1413 CompressObjects Tags13 CompressPages true13 ConvertImagesToIndexed true13 PassThroughJPEGImages true13 CreateJobTicket false13 DefaultRenderingIntent Default13 DetectBlends true13 DetectCurves 0000013 ColorConversionStrategy CMYK13 DoThumbnails false13 EmbedAllFonts true13 EmbedOpenType false13 ParseICCProfilesInComments true13 EmbedJobOptions true13 DSCReportingLevel 013 EmitDSCWarnings false13 EndPage -113 ImageMemory 104857613 LockDistillerParams false13 MaxSubsetPct 10013 Optimize true13 OPM 113 ParseDSCComments true13 ParseDSCCommentsForDocInfo true13 PreserveCopyPage true13 PreserveDICMYKValues true13 PreserveEPSInfo true13 PreserveFlatness true13 PreserveHalftoneInfo false13 PreserveOPIComments true13 PreserveOverprintSettings true13 StartPage 113 SubsetFonts true13 TransferFunctionInfo Apply13 UCRandBGInfo Preserve13 UsePrologue false13 ColorSettingsFile ()13 AlwaysEmbed [ true13 ]13 NeverEmbed [ true13 ]13 AntiAliasColorImages false13 CropColorImages true13 ColorImageMinResolution 30013 ColorImageMinResolutionPolicy OK13 DownsampleColorImages true13 ColorImageDownsampleType Bicubic13 ColorImageResolution 30013 ColorImageDepth -113 ColorImageMinDownsampleDepth 113 ColorImageDownsampleThreshold 15000013 EncodeColorImages true13 ColorImageFilter DCTEncode13 AutoFilterColorImages true13 ColorImageAutoFilterStrategy JPEG13 ColorACSImageDict ltlt13 QFactor 01513 HSamples [1 1 1 1] VSamples [1 1 1 1]13 gtgt13 ColorImageDict ltlt13 QFactor 01513 HSamples [1 1 1 1] VSamples [1 1 1 1]13 gtgt13 JPEG2000ColorACSImageDict ltlt13 TileWidth 25613 TileHeight 25613 Quality 3013 gtgt13 JPEG2000ColorImageDict ltlt13 TileWidth 25613 TileHeight 25613 Quality 3013 gtgt13 AntiAliasGrayImages false13 CropGrayImages true13 GrayImageMinResolution 30013 GrayImageMinResolutionPolicy OK13 DownsampleGrayImages true13 GrayImageDownsampleType Bicubic13 GrayImageResolution 30013 GrayImageDepth -113 GrayImageMinDownsampleDepth 213 GrayImageDownsampleThreshold 15000013 EncodeGrayImages true13 GrayImageFilter DCTEncode13 AutoFilterGrayImages true13 GrayImageAutoFilterStrategy JPEG13 GrayACSImageDict ltlt13 QFactor 01513 HSamples [1 1 1 1] VSamples [1 1 1 1]13 gtgt13 GrayImageDict ltlt13 QFactor 01513 HSamples [1 1 1 1] VSamples [1 1 1 1]13 gtgt13 JPEG2000GrayACSImageDict ltlt13 TileWidth 25613 TileHeight 25613 Quality 3013 gtgt13 JPEG2000GrayImageDict ltlt13 TileWidth 25613 TileHeight 25613 Quality 3013 gtgt13 AntiAliasMonoImages false13 CropMonoImages true13 MonoImageMinResolution 120013 MonoImageMinResolutionPolicy OK13 DownsampleMonoImages true13 MonoImageDownsampleType Bicubic13 MonoImageResolution 120013 MonoImageDepth -113 MonoImageDownsampleThreshold 15000013 EncodeMonoImages true13 MonoImageFilter CCITTFaxEncode13 MonoImageDict ltlt13 K -113 gtgt13 AllowPSXObjects false13 CheckCompliance [13 None13 ]13 PDFX1aCheck false13 PDFX3Check false13 PDFXCompliantPDFOnly false13 PDFXNoTrimBoxError true13 PDFXTrimBoxToMediaBoxOffset [13 00000013 00000013 00000013 00000013 ]13 PDFXSetBleedBoxToMediaBox true13 PDFXBleedBoxToTrimBoxOffset [13 00000013 00000013 00000013 00000013 ]13 PDFXOutputIntentProfile ()13 PDFXOutputConditionIdentifier ()13 PDFXOutputCondition ()13 PDFXRegistryName ()13 PDFXTrapped False1313 CreateJDFFile false13 Description ltlt13 ARA 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 BGR ltFEFF04180437043f043e043b043704320430043904420435002004420435043704380020043d0430044104420440043e0439043a0438002c00200437043000200434043000200441044a0437043404300432043004420435002000410064006f00620065002000500044004600200434043e043a0443043c0435043d04420438002c0020043c0430043a04410438043c0430043b043d043e0020043f044004380433043e04340435043d04380020043704300020043204380441043e043a043e043a0430044704350441044204320435043d0020043f04350447043004420020043704300020043f044004350434043f0435044704300442043d04300020043f043e04340433043e0442043e0432043a0430002e002000200421044a04370434043004340435043d043804420435002000500044004600200434043e043a0443043c0435043d044204380020043c043e0433043004420020043404300020044104350020043e0442043204300440044f0442002004410020004100630072006f00620061007400200438002000410064006f00620065002000520065006100640065007200200035002e00300020043800200441043b0435043404320430044904380020043204350440044104380438002egt13 CHS ltFEFF4f7f75288fd94e9b8bbe5b9a521b5efa7684002000410064006f006200650020005000440046002065876863900275284e8e9ad88d2891cf76845370524d53705237300260a853ef4ee54f7f75280020004100630072006f0062006100740020548c002000410064006f00620065002000520065006100640065007200200035002e003000204ee553ca66f49ad87248672c676562535f00521b5efa768400200050004400460020658768633002gt13 CHT ltFEFF4f7f752890194e9b8a2d7f6e5efa7acb7684002000410064006f006200650020005000440046002065874ef69069752865bc9ad854c18cea76845370524d5370523786557406300260a853ef4ee54f7f75280020004100630072006f0062006100740020548c002000410064006f00620065002000520065006100640065007200200035002e003000204ee553ca66f49ad87248672c4f86958b555f5df25efa7acb76840020005000440046002065874ef63002gt13 CZE 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 DAN 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 DEU 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 ESP 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 ETI 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 FRA 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 GRE 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 HEB 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 HRV (Za stvaranje Adobe PDF dokumenata najpogodnijih za visokokvalitetni ispis prije tiskanja koristite ove postavke Stvoreni PDF dokumenti mogu se otvoriti Acrobat i Adobe Reader 50 i kasnijim verzijama)13 HUN 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 ITA 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 JPN ltFEFF9ad854c18cea306a30d730ea30d730ec30b951fa529b7528002000410064006f0062006500200050004400460020658766f8306e4f5c6210306b4f7f75283057307e305930023053306e8a2d5b9a30674f5c62103055308c305f0020005000440046002030d530a130a430eb306f3001004100630072006f0062006100740020304a30883073002000410064006f00620065002000520065006100640065007200200035002e003000204ee5964d3067958b304f30533068304c3067304d307e305930023053306e8a2d5b9a306b306f30d530a930f330c8306e57cb30818fbc307f304c5fc59808306730593002gt13 KOR ltFEFFc7740020c124c815c7440020c0acc6a9d558c5ec0020ace0d488c9c80020c2dcd5d80020c778c1c4c5d00020ac00c7a50020c801d569d55c002000410064006f0062006500200050004400460020bb38c11cb97c0020c791c131d569b2c8b2e4002e0020c774b807ac8c0020c791c131b41c00200050004400460020bb38c11cb2940020004100630072006f0062006100740020bc0f002000410064006f00620065002000520065006100640065007200200035002e00300020c774c0c1c5d0c11c0020c5f40020c2180020c788c2b5b2c8b2e4002egt13 LTH 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 LVI ltFEFF0049007a006d0061006e0074006f006a00690065007400200161006f00730020006900650073007400610074012b006a0075006d00750073002c0020006c0061006900200076006500690064006f00740075002000410064006f00620065002000500044004600200064006f006b0075006d0065006e007400750073002c0020006b006100730020006900720020012b00700061016100690020007000690065006d01130072006f00740069002000610075006700730074006100730020006b00760061006c0069007401010074006500730020007000690072006d007300690065007300700069006501610061006e006100730020006400720075006b00610069002e00200049007a0076006500690064006f006a006900650074002000500044004600200064006f006b0075006d0065006e007400750073002c0020006b006f002000760061007200200061007400760113007200740020006100720020004100630072006f00620061007400200075006e002000410064006f00620065002000520065006100640065007200200035002e0030002c0020006b0101002000610072012b00200074006f0020006a00610075006e0101006b0101006d002000760065007200730069006a0101006d002egt13 NLD (Gebruik deze instellingen om Adobe PDF-documenten te maken die zijn geoptimaliseerd voor prepress-afdrukken van hoge kwaliteit De gemaakte PDF-documenten kunnen worden geopend met Acrobat en Adobe Reader 50 en hoger)13 NOR 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 POL ltFEFF0055007300740061007700690065006e0069006100200064006f002000740077006f0072007a0065006e0069006100200064006f006b0075006d0065006e007400f300770020005000440046002000700072007a0065007a006e00610063007a006f006e00790063006800200064006f002000770079006400720075006b00f30077002000770020007700790073006f006b00690065006a0020006a0061006b006f015b00630069002e002000200044006f006b0075006d0065006e0074007900200050004400460020006d006f017c006e00610020006f007400770069006500720061010700200077002000700072006f006700720061006d006900650020004100630072006f00620061007400200069002000410064006f00620065002000520065006100640065007200200035002e0030002000690020006e006f00770073007a0079006d002egt13 PTB 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 RUM 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 RUS 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 SKY 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 SLV 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 SUO 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 SVE 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 TUR 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 UKR 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 ENU (Use these settings to create Adobe PDF documents best suited for high-quality prepress printing Created PDF documents can be opened with Acrobat and Adobe Reader 50 and later)13 gtgt13 Namespace [13 (Adobe)13 (Common)13 (10)13 ]13 OtherNamespaces [13 ltlt13 AsReaderSpreads false13 CropImagesToFrames true13 ErrorControl WarnAndContinue13 FlattenerIgnoreSpreadOverrides false13 IncludeGuidesGrids false13 IncludeNonPrinting false13 IncludeSlug false13 Namespace [13 (Adobe)13 (InDesign)13 (40)13 ]13 OmitPlacedBitmaps false13 OmitPlacedEPS false13 OmitPlacedPDF false13 SimulateOverprint Legacy13 gtgt13 ltlt13 AddBleedMarks false13 AddColorBars false13 AddCropMarks false13 AddPageInfo false13 AddRegMarks false13 ConvertColors ConvertToCMYK13 DestinationProfileName ()13 DestinationProfileSelector DocumentCMYK13 Downsample16BitImages true13 FlattenerPreset ltlt13 PresetSelector MediumResolution13 gtgt13 FormElements false13 GenerateStructure false13 IncludeBookmarks false13 IncludeHyperlinks false13 IncludeInteractive false13 IncludeLayers false13 IncludeProfiles false13 MultimediaHandling UseObjectSettings13 Namespace [13 (Adobe)13 (CreativeSuite)13 (20)13 ]13 PDFXOutputIntentProfileSelector DocumentCMYK13 PreserveEditing true13 UntaggedCMYKHandling LeaveUntagged13 UntaggedRGBHandling UseDocumentProfile13 UseDocumentBleed false13 gtgt13 ]13gtgt setdistillerparams13ltlt13 HWResolution [2400 2400]13 PageSize [612000 792000]13gtgt setpagedevice13

Page 15: July 2014 MEE Questions and Analyses - NCBE...This publication includes the questions and analyses from the July 2014 MEE. (In the actual test, the questions are simply numbered rather

CRIMINAL LAW AND PROCEDURE ANALYSIS (Criminal Law and Procedure VA B D)

ANALYSIS

Legal Problems

(1) Did the detective violate the suspectrsquos Sixth Amendment right to counsel when he questioned the suspect about the burglaries without the lawyer present given that the lawyer represented the suspect in an unrelated criminal matter

(2) Under Miranda did the suspect effectively invoke his right to counsel when he said ldquoI think I want my lawyer here before I talk to yourdquo

(3) Was the suspectrsquos waiver of his right to remain silent under Miranda valid

DISCUSSION

Summary

The Sixth Amendment right to counsel as applied to states through the Fourteenth Amendment is offense-specific Although the suspect had an attorney representing him on his pending assault charge he had no Sixth Amendment right to the assistance of counsel with respect to the five uncharged burglaries because formal adversarial proceedings had not yet commenced on those charges The suspectrsquos Sixth Amendment right to counsel was not violated by the detectiversquos failure to inform him that the lawyer was present or of the lawyerrsquos demands

However a person undergoing custodial interrogation also has an independent constitutional right to counsel during custodial interrogation under Miranda When a suspect invokes his right to counsel under Miranda custodial interrogation must immediately cease for a period of at least 14 days However the invocation of the right to counsel must be unambiguous and clearly convey that the suspect has requested counsel Here because the suspectrsquos statement ldquoI think I want my lawyer here before I talk to yourdquo was ambiguous he did not invoke his Miranda right to counsel

A waiver of rights must be knowing intelligent and voluntary Here the suspect waived his right to remain silent under Miranda when he signed the waiver form The fact that the detective did not correct the suspectrsquos assumption that the lawyer would need to drive to the jailmdashby telling him that the lawyer was in the waiting room and was demanding to see himmdashdid not affect the validity of the suspectrsquos waiver

Point One (35) The suspectrsquos Sixth Amendment right to counsel was not violated because the right does not attach on new charges until formal adversarial judicial proceedings have commenced on those charges

The Sixth Amendment as applied to the states through the Fourteenth Amendment provides that ldquo[i]n all criminal prosecutions the accused shall enjoy the right to have the Assistance of Counsel for his defenserdquo The right to counsel does not attach with respect to particular charges until formal adversarial judicial proceedings have commenced (ie ldquoat or after the initiation of

13

Criminal Law and Procedure Analysis

adversary judicial criminal proceedingsmdashwhether by way of formal charge preliminary hearing indictment information or arraignment [or in some states arrest warrant]rdquo McNeil v Wisconsin 501 US 171 175 (1991) (internal quotations omitted)) Once a suspectrsquos Sixth Amendment right to counsel has attached any attempts to ldquodeliberately elicitrdquo statements from him in the absence of his attorney violate the Sixth Amendment See Massiah v United States 377 US 201 (1964) Brewer v Williams 430 US 387 (1977)

The Sixth Amendment right to counsel is charge- or offense -specific Representation by counsel in one prosecution does not in itself guarantee counsel for uncharged offenses See McNeil 501 US at 175 Texas v Cobb 532 US 162 (2001) Here the suspectrsquos Sixth Amendment right to counsel had attached only for the pending aggravated assault charge The suspectrsquos right to counsel for the aggravated assault case did not guarantee counsel for the five unrelated and uncharged burglaries that were the subject of the detectiversquos interrogation Thus because formal adversarial judicial proceedings against the suspect for the uncharged burglaries had not begun he had no Sixth Amendment right to counsel

Finally the detectiversquos failure to inform the suspect of the lawyerrsquos presence and demands to speak with him does not implicate the suspectrsquos Sixth Amendment right to counsel which had not yet attached See id Moran v Burbine 475 US 412 428ndash31 (1986)

Point Two (30) The suspect did not effectively invoke his right to counsel under Miranda because his statement was not unambiguous

A suspect subject to custodial interrogation has a right to consult with counsel and to have an attorney present during questioning Miranda v Arizona 384 US 436 (1966) When a suspect invokes his right to counsel during an interrogation law enforcement must immediately cease all questioning See Edwards v Arizona 451 US 477 484ndash85 (1981) Custodial interrogation cannot be reinitiated unless and until the suspect has been re-advised of his Miranda rights has provided a knowing and voluntary waiver and (1) counsel is present and (2) the suspect himself initiated further communication with the police see id at 484 or (3) (if the suspect was released from custody after the initial interrogation) at least 14 days have passed Maryland v Shatzer 559 US 98 110 (2010)

To invoke the right to counsel a suspectrsquos request must be ldquounambiguousrdquo This means that the suspect must articulate the desire for counsel sufficiently clearly that a reasonable officer would understand the statement to be a request for counsel Davis v United States 512 US 452 459 (1994) If the request is ambiguous the police are not required to stop the interrogation

In this case the suspectrsquos statement ldquoI think I want my lawyer here before I talk to yourdquo was not an unambiguous request for counsel The most reasonable interpretation of this statement is that the suspect might be invoking his right to counsel Id at 461 (ldquomaybe I should talk to a lawyerrdquo is not an unequivocal request for counsel) See also Burket v Angelone 208 F3d 172 197ndash98 (4th Cir 2000) (ldquoI think I need a lawyerrdquo is not an unambiguous request for an attorney) Soffar v Cockrell 300 F3d 588 594ndash95 (5th Cir 2002) (discussion of various statements that did not constitute unequivocal requests for counsel)

Under these circumstances the detective was not required to cease the custodial interrogation of the suspect Nor was the detective required to clarify or ask follow-up questions to determine whether the suspect in fact wanted an attorney Davis 512 US at 459ndash60

14

Criminal Law and Procedure Analysis

Point Three (35) The suspectrsquos waiver of his Miranda rights was knowing intelligent and voluntary despite the fact that he was never told of the lawyerrsquos presence in the jail or of the lawyerrsquos demands

A valid waiver of Miranda rights must be ldquovoluntaryrdquomdashie the product of a free or deliberate choice rather than intimidation coercion or deception Berghuis v Thompkins 560 US 370 382ndash83 (2010) In addition the waiver must be knowing and intelligent That is it ldquomust have been made with a full awareness of both the nature of the right being abandoned and the consequences of the decision to abandon itrdquo Moran v Burbine 475 US 421 (1986)

In this case the suspect signed a Miranda waiver form after receiving proper warnings There is no evidence ldquothat the police resorted to physical or psychological pressure to elicit the statementsrdquo Id The entire interview lasted only 45 minutes The only issue is whether the suspect knowingly and intelligently waived his Miranda rights despite the fact that the detective did not tell the suspect about the lawyerrsquos presence and her demands

The Supreme Court has said that ldquo[e]vents occurring outside of the presence of the suspect and entirely unknown to him surely can have no bearing on the capacity to comprehend and knowingly relinquish a constitutional rightrdquo Id at 422 If the suspect ldquoknew that he could stand mute and request a lawyer and was aware of the Statersquos intention to use his statements to secure a convictionrdquo then the waiver is valid regardless of the information withheld Id at 422ndash23

Here the suspect was correctly informed of his rights Miranda v Arizona 384 US at 467ndash73 His comments demonstrate that he understood that he could have a lawyer present if he desired (ie wondering whether he should call his attorney) and that he understood that there might be consequences to speaking with the detective (ldquoI probably should keep my mouth shut but Irsquom willing to talk to you for a whilerdquo) His comment ldquo[L]etrsquos not waste any time waiting for someone to call my attorney and having her drive hererdquo along with his signature on the Miranda waiver form show that his waiver was valid under the constitutional standard

The fact that the detective did not tell the suspect about the lawyerrsquos presence and demands has no bearing on the validity of the suspectrsquos waiver because ldquosuch conduct is only relevant to the constitutional validity of a waiver if it deprives a defendant of knowledge essential to his ability to understand the nature of his rights and the consequences of abandoning themrdquo Moran at 424 The Supreme Court has specifically declined to adopt a rule requiring that law enforcement tell a suspect of an attorneyrsquos efforts to contact him id at 425 (ldquoNor are we prepared to adopt a rule requiring that the police inform a suspect of an attorneyrsquos efforts to reach himrdquo)

[NOTE An examinee might also recognize that this general rule is further supported by the Supreme Courtrsquos decision in Florida v Powell 559 US 50 (2010) approving state Miranda warnings that do not explicitly warn suspects that they have a right to have counsel present during custodial interrogation]

15

CONTRACTS ANALYSIS (Contracts IB2 IIB IVA3 amp A5)

ANALYSIS

Legal Problems

(1) In the case of a service contract (governed by the common law of contracts) is a modification enforceable when a party agrees to pay more for the same performance than was originally promised

(2) In the case of a contract for the sale of goods (governed by Article 2 of the UCC) is a modification enforceable when a party agrees to pay more for the same goods than was originally promised

(3) May a party avoid an agreement on the basis of economic duress

DISCUSSION

Summary

There are two arguments that the conservatory can make to support the claim that it is not bound to pay the higher prices lack of consideration and economic duress

The organ repair contract is governed by the common law of contracts Under the common law the business would have difficulty recovering the additional $60000 for the organ repair because under the ldquopreexisting duty rulerdquo the agreement of the conservatory to pay the extra price was not supported by consideration However the business might argue that the modification is enforceable under an exception to the preexisting duty rule for fair and equitable modifications made in light of unanticipated circumstances

The organ sale contract is governed by Article 2 of the Uniform Commercial Code The business would likely recover the additional amount under that contract because Article 2 provides that consideration is not required for a modification to be binding

In both cases the conservatory could seek to avoid its agreement on the grounds of economic duress but that argument is not likely to succeed

Point One (45) The business probably cannot recover the additional $60000 for the organ repair because the conservatoryrsquos promise to pay more money was not supported by consideration

The general rule is that to be enforceable a promise must be supported by consideration Under RESTATEMENT (SECOND) OF CONTRACTS sect 71 a promise is supported by consideration if it is bargained for in exchange for a return promise or performance However under the ldquopreexisting duty rulerdquo (exemplified in RESTATEMENT (SECOND) OF CONTRACTS sect 73 and Alaska Packersrsquo Assrsquon v Domenico 117 F 99 (9th Cir 1902)) promise of performance of a legal duty already owed to a promisor which is neither doubtful nor the subject of honest dispute is not consideration

If the business had promised the conservatory anything new or different in exchange for the agreement to pay the additional $60000 (such as for example repairing the pipe organ more

16

Contracts Analysis

quickly or using better parts) that would constitute consideration especially in light of the principle that courts do not inquire into the adequacy of consideration Here however the business already had a legal duty under the original contract and did not agree to do anything else in exchange for the conservatoryrsquos promise to pay $60000 more

However an exception to the preexisting duty rule is sometimes applied in situations of unanticipated changed circumstances Under RESTATEMENT (SECOND) OF CONTRACTS sect 89 followed in many jurisdictions a promise modifying a duty under a contract not fully performed on either side is binding even if not supported by consideration if the modification is fair and equitable in view of circumstances not anticipated by the parties when the contract was made

If a court applies the rule in Restatement sect 89 the critical issues will be whether the modification was in fact ldquofair and equitablerdquo and whether it can be justified in light of unanticipated circumstances In many cases in which modifications have been upheld a party encountered difficulties or burdens in performing far beyond what was knowingly bargained for in the original contract with the result bordering on impracticability such as having to excavate solid rock instead of soft dirt or having to remove garbage far in excess of the amounts contemplated The conservatory would argue that the businessrsquos performance difficulties were not of this sort at allmdashnothing about repairing the pipe organ itself was any different from or more difficult than originally contemplated except that the business itself encountered financial distress unrelated to its burdens in performing its obligations under these contracts

Even if the business satisfies that element of the rule in Restatement sect 89 the business must also demonstrate that the circumstances that gave rise to the need to modify the contract were ldquounanticipatedrdquo at the time the original contract was made Here the facts suggest that when the business entered into the original contract it expected that the price paid by the conservatory would enable it to perform However any evidence that the business knew or had reason to know at the time of execution that it would need more money from the conservatory to be able to perform would mean that the request to modify was not ldquounanticipatedrdquo

[NOTE Some cases such as Schwartzreich v Bauman-Basch Inc 231 NY 196 131 NE 887 (1921) find that if the parties mutually agreed to rescind the original contract and then after rescission entered into an entirely new contract for a higher price the new contract is supported by consideration There is no evidence that such a rescission followed by a new contract took place here]

Point Two (45) The business can recover the additional $40000 for the new organ because no consideration is required under Article 2 of the UCC for good-faith contract modifications

The contract to buy a new organ is a contract for the sale of goods and therefore is governed by Article 2 of the Uniform Commercial Code UCC sect 2-102 Under Article 2 unlike the common law an agreement modifying a contract needs no consideration to be binding UCC sect 2-209(1) Section 2-209(1) thus obviates the preexisting duty rule entirely in contracts for the sale of goods

Even though consideration is not required modifications governed by sect 2-209 must satisfy the obligation of good faith imposed by the UCC UCC sect 1-304 See also Official Comment 2 to UCC sect 2-209 Good faith means ldquohonesty in fact and the observance of reasonable commercial standards of fair dealingrdquo UCC sect 1-201(b)(20) In this context the obligation of good faith means that ldquo[t]he effective use of bad faith to escape performance on the original contract terms is barred and the extortion of a lsquomodificationrsquo without legitimate commercial reason is ineffective as a violation of the duty of good faithrdquo Official Comment 2 to

17

Contracts Analysis

UCC sect 2-209 Here because the businessrsquos financial reversals were serious and apparently unanticipated at the time that the business entered into the contract with the conservatory and commitment of the extra money was needed to enable the business to perform a court would likely find that the business acted in good faith Thus a court would likely uphold the enforceability of the conservatoryrsquos promise to pay the additional $40000

Point Three (10) The conservatory is unlikely to be able to defend against enforcement of its promises to pay additional money under the theory of economic duress because the business probably did not make an improper threat

Under the common law of contracts parties may raise the defense of duress This common law defense also applies to contracts governed by UCC Article 2 See UCC sect 1-103(b)

A contract is voidable on the ground of economic duress by threat when it is established that a partyrsquos manifestation of assent is induced by an improper threat that leaves the party no reasonable alternative See RESTATEMENT (SECOND) OF CONTRACTS sect 175 See also eg Austin Instrument Inc v Loral Corp 272 NE2d 533 (NY 1971) (a threat to withhold essential goods can constitute duress) In order to void its agreement to pay the additional sum because of economic duress the conservatory must demonstrate that (1) the business made a threat to the conservatory (2) the threat was ldquoimproperrdquo or ldquowrongfulrdquo (3) the threat induced the conservatoryrsquos manifestation of assent to the modification and (4) the threat was sufficiently grave to justify the conservatoryrsquos assent

Here it appears that three of the four elements are likely satisfied The business plainly made a threat Moreover the threat induced the conservatoryrsquos assent to the modification and the threat was sufficiently grave to justify that assent If the conservatory had not agreed to pay the business the extra amounts the conservatory would have lost its entire $325000 investment In light of this potential loss a court could easily conclude that the conservatory had no reasonable alternative

However the business has a strong argument that its threat (indicating that it would breach the contracts unless the prices were increased) was not wrongful or improper but was instead nothing more than a communication of the reality of its own perilous situation to the conservatory

A mere threat to breach a contract is not in and of itself improper so as to support an action of economic duress or business compulsion Something more is required such as a breach of the duty of good faith and fair dealing as was present in Austin Instrument Inc supra Because the business could not perform the original contract without the requested modification the economic duress claim for the conservatory would likely fail for much the same reason that the business would be able to enforce the modification At the time the modification was requested the business was not trying to extort a price increase because of the conservatoryrsquos vulnerability but instead was simply stating the reality that the business could not perform without more money

18

FAMILY LAW ANALYSIS (Family Law IIIB D amp G)

ANALYSIS

Legal Problems

(1)(a) Does the State A court have jurisdiction to modify the State B child support order

(1)(b) Does the State A court have jurisdiction to modify the marital-residence-saleshyproceeds provision of the State B property-division decree

(2)(a) May a child support order be modified retroactively

(2)(b) May a child support order be modified prospectively based on a change of employment with a lower salary

(2)(c) May a property-division order be modified after entry of a divorce decree

DISCUSSION

Summary

The State A court may exercise personal jurisdiction over the wife because she was personally served in State A However subject-matter jurisdiction over the interstate modification of child support is governed by the Uniform Interstate Family Support Act (UIFSA) Under UIFSA State A does not have jurisdiction to modify the order for the daughterrsquos support because the wife is still a resident of State B UIFSA on the other hand does not govern property distributions and thus a State A court is not precluded from hearing the husbandrsquos petition to modify the marital-residence-sale-proceeds provision of the divorce decree

A child support order may not be modified retroactively A child support order may be modified prospectively based on a substantial change in circumstances Courts agree that a significant decrease in income is a substantial change in circumstances All states treat voluntary income reductions differently than involuntary reductions but employ different approaches for evaluating the impact of a voluntary reduction Whether the husband could obtain prospective modification of the child support order depends on which approach is applied

A property-division order is not subject to post-divorce modification based on a change in circumstances Thus the husband may in some states obtain prospective modification of the order for the daughterrsquos support but he may not obtain modification of the marital-residenceshysale-proceeds provision

Point One(a) (25) Personal jurisdiction over a nonresident respondent does not confer subject-matter jurisdiction over child support modification Under UIFSA a State A court may not modify a child support order issued by a State B court when as here the child or either parent continues to reside in State B the jurisdiction that issued the child support order

The State A court may exercise personal jurisdiction over the wife The wife was personally served in State A and a state may exercise jurisdiction based on in-state personal service See

19

Family Law Analysis

Burnham v Superior Court 495 US 604 (1990) But personal jurisdiction over the wife is not enough to give a State A court jurisdiction to modify the State B support order

The interstate enforcement and modification of child support is governed by the Uniform Interstate Family Support Act (UIFSA) which has been adopted by all states Under UIFSA the state that originally issued a child support order (here State B) has continuing exclusive jurisdiction to modify the order if that state remains the residence of the obligee the child or the obligor and all parties do not consent to the jurisdiction of another forum See UIFSA sect 205 See also UIFSA sect 603 (ldquoA tribunal of this State shall recognize and enforce but may not modify a registered order if the issuing tribunal had jurisdictionrdquo) The wife and daughter continue to reside in State B and the wife has not consented to the jurisdiction of another forum Thus a State A court does not have jurisdiction to modify the State B child support order

[NOTE Examinees who do not discuss personal jurisdiction but fully discuss UIFSA may receive full credit]

Point One(b) (15) UIFSA does not apply to disputes over property division Thus the State A court may exercise jurisdiction over the husbandrsquos petition to modify the marital-residence-sale-proceeds provision of the State B divorce decree because it has personal jurisdiction over the wife

The State A court in which the husband brought his action has jurisdiction to adjudicate domestic relations issues The husbandrsquos petition to modify the property settlement is a domestic relations issue The courts of State A may exercise personal jurisdiction over the wife because she was personally served in State A See Burnham v Superior Court 495 US 604 (1990) see Point One(a)

UIFSA does not apply to divorce property-division disputes Thus although a State A court may not adjudicate the husbandrsquos petition to modify his child support obligations it may adjudicate his property-division claims (Even though the court has jurisdiction it may not modify the property-division award on the merits See Point Two(c))

Point Two(a) (20) A child support order may not be modified retroactively

State courts have long held that obligations to pay child support ordinarily may not be modified retroactively ldquoIf the hardship is particularly severe the courts sometimes devised a way to protect the obligor but in most instances the courts hold that retroactive modification of this kind is beyond their power and indeed the governing statute may so providerdquo HOMER H CLARK THE LAW OF DOMESTIC RELATIONSHIPS IN THE UNITED STATES 725 (2d ed 1987)

Federal law now goes further and requires the states as a condition of federal child-support funding to adopt rules that absolutely forbid retroactive modification of the support obligation See 42 USC sect 666(a)(9)(C) The states have adopted rules consistent with the federal requirements

Point Two(b) (25) It is unclear whether the husband could obtain prospective downward modification of his child support based on his voluntary acceptance of a job with a lower salary

Prospective modification of a child support order is typically available only when the petitioner can show a substantial change in circumstances See ROBERT E OLIPHANT amp NANCY VER

20

Family Law Analysis

STEEGH FAMILY LAW 213ndash15 (3d ed 2010) A significant decrease in income is typically viewed as a substantial change

However when a parent seeks to modify a child support obligation because he has voluntarily reduced his income a court will not modify the obligation based solely on the income loss Some courts refuse to modify whenever the income shift was voluntary See eg Aguiar v Aguiar 127 P3d 234 (Idaho Ct App 2005) Others look primarily to the petitionerrsquos intentions and permit downward modification if he has acted in good faith See eg In re Marriage of Horn 650 NE2d 1103 (Ill App Ct 1995) Many courts use a multifactor approach See OLIPHANT amp VER STEEGH supra 217ndash18

Here there is no question that the husbandrsquos loss of income was voluntary In a jurisdiction in which voluntary income reduction bars support modification the husbandrsquos petition would be denied

In a jurisdiction employing a good-faith or multifactor approach it is possible but not certain that the husband could obtain downward modification The evidence supports the husbandrsquos good faith his change in employment appears to be based on his new jobrsquos greater responsibilities and better promotion possibilities In a jurisdiction using a multifactor approach the court would likely also consider the impact of such a shift on the daughter the likely duration of the husbandrsquos income loss and the likelihood of a promotion that would ultimately inure to the daughterrsquos benefit Thus on these facts it is possible but by no means certain that the husband could prospectively obtain downward modification of his child support obligation to his daughter

Point Two(c)(15) A divorce property-division award is not subject to modification

A support order is aimed at meeting the post-divorce needs of the supported individual Because the future is unpredictable courts are empowered to modify a support award to take account of changed circumstances that may occur during the period in which support is paid

By contrast a property-distribution award divides assets of the marriage based on the equities at the time of divorce Because the past can be ascertained a property-division award is not subject to post-divorce modification See HARRY A KRAUSE ET AL FAMILY LAW CASES COMMENTS AND QUESTIONS 691 (6th ed 2007)

Here the husband is seeking modification of a property-division award with respect to an asset owned by the parties at the time of divorce Thus the husband may not obtain a modification of the marital-residence-sale-proceeds provision of the divorce decree based on his reduced income

21

FEDERAL CIVIL PROCEDURE ANALYSIS (Federal Civil Procedure III IVC)

ANALYSIS

Legal Problems

(1) Is the logging company entitled to join this action as a matter of right

(2)(a) May the nonprofit organization obtain a temporary restraining order to stop the USFS from issuing a logging permit

(2)(b) May the nonprofit organization obtain a preliminary injunction to stop the USFS from issuing a logging permit during the pendency of the action

DISCUSSION

Summary

The logging company is entitled to intervene in this action as a matter of right because it has an interest in the property or transaction that is the subject of the action and is so situated that its interest may be impaired or impeded as a practical matter if the action goes forward without it The logging companyrsquos interest is not adequately represented by the USFSrsquos presence in the lawsuit

The nonprofit organization may seek a temporary restraining order (TRO) followed by a preliminary injunction to prevent the USFS from issuing a logging permit pending the outcome of the action The nonprofit is likely to obtain a TRO if it can demonstrate a risk of immediate and irreparable injury The nonprofit is also likely to obtain a preliminary injunction if it can demonstrate a significant threat of irreparable harm and a likelihood of success on the merits of its National Environmental Policy Act (NEPA) claim

Point One (50) Rule 24(a) of the Federal Rules of Civil Procedure requires federal courts to allow a person to intervene in an action as a matter of right if the person a) is interested in the property or transaction that is the subject of the action b) is so situated that its interest may be impaired or impeded if the litigation goes forward without it and c) is not adequately represented by existing parties Here the logging company likely meets all three requirements and should be allowed to intervene as a matter of right

Rule 24 of the Federal Rules of Civil Procedure governs intervention the process by which a non-party to an action may join the litigation Under Rule 24(a) (intervention of right) a person must be permitted to intervene if three conditions are met (1) the movant ldquoclaims an interest relating to the property or transaction that is the subject of the actionrdquo (2) the movant ldquois so situated that disposition of the action may as a practical matter impair or impede the movantrsquos ability to protect its interestrdquo and (3) ldquoexisting partiesrdquo do not ldquoadequately represent [the movantrsquos] interestrdquo FED R CIV P 24(a) The three requirements for intervention of right are often ldquovery interrelatedrdquo 7C CHARLES ALAN WRIGHT ET AL FEDERAL PRACTICE AND PROCEDURE sect 1908 at 297 (2007 amp 2011 Supp)

22

Federal Civil Procedure Analysis

Here the court should find that the logging company meets this test First the logging company has a strong interest in the property or transaction that is the subject of this action The USFS has accepted the logging companyrsquos bid and the logging company is merely awaiting issuance of a logging permit to begin logging The nonprofit organization is seeking to prevent this logging The logging company therefore has a strong direct and substantial interest in the subject matter of the lawsuit and in having its winning bid honored and a logging permit issued See eg Kleissler v US Forest Serv 157 F3d 964 972 (3d Cir 1998) (stating that ldquo[t]imber companies have direct and substantial interests in a lawsuit aimed at halting loggingrdquo) see also Natural Resources Defense Council v US Nuclear Regulatory Commrsquon 578 F2d 1341 1343ndash 44 (10th Cir 1978) (holding that applicants whose license renewals were pending had Rule 24(a)(2) interests where the lawsuit sought to halt the license-issuing process pending preparation of environmental impact statements) See generally 7C WRIGHT ET AL supra sect 19081 at 309 (ldquoIf there is a direct substantial legally protectable interest in the proceedings it is clear that this requirement of the rule is satisfiedrdquo) Second the logging companyrsquos interest in receiving a logging permit may well be impaired as a practical matter by the outcome of the lawsuit If the USFS loses the lawsuit it will have to prepare an environmental impact statement before issuing the logging companyrsquos permit This will at a minimum delay the logging companyrsquos ability to exercise its rights and may in the long r un mean that no logging permit is ever issued Intervention of right is not limited to those that would be legally bound as a matter of preclusion doctrine Id sect 19082 at 368 Rather ldquo[t]he rule is satisfied whenever disposition of the present action would put the movant at a practical disadvantage in protecting its interestrdquo Id sect 19082 at 369 Here that condition is easily satisfied See Kleissler 157 F3d at 972 (ldquoTimber companies have direct and substantial interests in a lawsuit aimed at halting logging rdquo)

Given that the logging company has an interest that may be impaired by disposition of the action it should be allowed to intervene unless the court is persuaded that the USFS adequately represents the logging companyrsquos interest See Rule 24(a)(2) 7C WRIGHT ET AL supra sect 1909 Here it could be argued that the USFS adequately represents the logging companyrsquos interest because the USFS presumably wants the court to uphold its development plan and allow it to proceed with issuance of the logging permit which is the same relief that the logging company would seek However whether representation is truly adequate depends upon ldquo[a] discriminating appraisal of the circumstancesrdquo 7C WRIGHT ET AL supra sect 1909 at 440 Although both the government and the logging company wish to avoid the preparation of an environmental impact statement their interests are distinct The USFSrsquos interest is proper management of the national forest system while the logging companyrsquos interest is making a profit from logging the 5000-acre tract The USFSrsquos handling of the litigation is likely to be affected by a variety of policy concerns and political considerations that have nothing to do with the logging companyrsquos purely economic interest in securing the right to cut trees in the Scenic National Forest See eg Kleissler 157 F3d at 973ndash74 (ldquo[T]he government represents numerous complex and conflicting interests in matters of this nature The straightforward business interests asserted by intervenors here may become lost in the thicket of sometimes inconsistent governmental policiesrdquo)

[NOTES (1) Examinees who mistakenly analyze the logging companyrsquos case for joinder under the related but incorrect Rule 19 ldquoRequired Joinder of Partiesrdquo may receive credit Rule 19 allows existing parties to demand joinder of non-parties (or seek dismissal of the case if they canrsquot get it) There is a close relationship between Rule 24 and Rule 19 and both contain a similar standard for determining when ldquointerestedrdquo third parties are ldquoentitledrdquo or ldquorequiredrdquo to be in the lawsuit Indeed the two prongs of the Rule 24 intervention test that are discussed above

23

Federal Civil Procedure Analysis

are nearly identical to the two prongs of the Rule 19(a) required joinder test Examinees who discuss and apply the test should receive credit even if they cite Rule 19 rather than Rule 24

(2) Examinees may discuss permissive joinder Although permissive joinder is a possibility here the question asks only whether the logging company can join the action as a matter of right and a permissive joinder analysis is not responsive to the question To the extent an examinee discusses permissive joinder the analysis will focus on whether the logging company ldquohas a claim or defense that shares with the main action a common question of law or factrdquo FED R CIV P 24(b)(1)(B) The district court also ldquomust consider whether the intervention will unduly delay or prejudice the adjudication of the original partiesrsquo rightsrdquo FED R CIV P 24(b)(3) On our facts the logging companyrsquos claim for the issuance of a logging permit would certainly share common questions of law and fact with the USFSrsquos defense against the nonprofitrsquos claim There are no facts suggesting that the logging companyrsquos presence would unduly delay or otherwise prejudice adjudication of the original action Thus the district court would have discretion to permit the logging company to intervene even if it denied intervention of right]

Point Two(a) (25) The nonprofit organization could seek and would likely obtain a temporary restraining order to stop the USFS from issuing a logging permit pending a hearing on an application for a preliminary injunction

The first type of interim relief the nonprofit could seek to stop the USFS from issuing a logging permit to the logging company is a temporary restraining order (TRO) prohibiting the USFS from issuing the logging permit A TRO can be issued without notice to the adverse party but only in limited circumstances and only for a limited time FED R CIV P 65(b) To secure a TRO without notice the nonprofit would need to submit an affidavit containing specific facts that demonstrate a risk of ldquoimmediate and irreparable injuryrdquo if a permit is issued FED R CIV P 65(b)(1) In deciding whether to grant a TRO courts will also consider the same factors that are relevant in deciding whether to grant a preliminary injunction (eg the moving partyrsquos likelihood of success on the merits the balance of hardships and the public interest) See Point Two(b) infra The TRO would last only long enough for the court to consider and resolve a request by the nonprofit for a preliminary injunction but no longer than 14 days (unless the court extends it for good cause or the adverse party consents to an extension) In addition bond is required

Here the court is likely to grant the nonprofitrsquos request The nonprofit could plausibly claim that cutting down 5000 acres of old-growth forest in an area that is home to the highest concentration of wildlife in the western United States would have ldquoan immediate and irreparablerdquo adverse impact on the environment and cause irreparable harm to the nonprofitrsquos interest in preserving and protecting natural resources including wildlife habitat

Point Two(b) (25) The nonprofit could also seek and would likely obtain a preliminary injunction to stop the USFS which is likely to be granted if the nonprofitrsquos claim that the USFS violated NEPA has a strong basis in fact and law

Because the TRO would be temporary the nonprofit would need to move for a preliminary injunction to prevent the USFS from issuing a logging permit throughout the pendency of the litigation Preliminary injunctions are injunctions that seek to ldquoprotect [the] plaintiff from

24

Federal Civil Procedure Analysis

irreparable injury and to preserve the courtrsquos power to render a meaningful decision after a trial on the meritsrdquo 11A CHARLES ALAN WRIGHT ET AL FEDERAL PRACTICE AND PROCEDURE sect 2947 at 112 (2013) Rule 65 of the Federal Rules of Civil Procedure sets out the procedural requirements for preliminary injunctions Preliminary injunctions may be granted only upon notice to the adverse party FED R CIV P 65(a)(1) and only if the movant ldquogives security in an amount that the court considers proper to pay the costs and damages sustained by any party found to have been wrongfully enjoined or restrainedrdquo FED R CIV P 65(c)

While Rule 65 sets out the procedural requirements for preliminary injunctive relief it does not specify the substantive grounds upon which it may be granted The courtrsquos discretion in ruling upon a motion for a preliminary injunction ldquois exercised in conformity with historic federal equity practicerdquo 11A WRIGHT ET AL supra sect 2947 at 114 The court typically considers four factors

(1) the significance of the threat of irreparable harm to the plaintiff if the injunction is not granted (2) the balance between this harm and the injury that granting the injunction would inflict on the defendant (3) the probability that the plaintiff will succeed on the merits and (4) the public interest

Id sect 2948 at 122ndash24 accord Habitat Educ Center v Bosworth 363 F Supp 2d 1070 1088 (ED Wis 2005) The most important of these factors is the risk of irreparable harm to the plaintiff 11A WRIGHT ET AL supra sect 29481 at 129 If the plaintiff has an adequate remedy at law (eg if money damages can compensate the plaintiff for its loss) then a preliminary injunction will be denied Id sect 29481

Here a court would likely conclude that the potential for environmental damage to the forest creates a significant threat of irreparable harm ldquo[E]nvironmental injury is often irreparable Courts have recognized that logging such as would occur [here] can have longshyterm environmental consequences and thus satisfy the irreparable injury criterionrdquo Habitat Educ Center 363 F Supp 2d at 1089 (citing Idaho Sporting Congress Inc v Alexander 222 F3d 562 569 (9th Cir 2000) (noting that the imminent and continuing logging activities presented ldquoevidence of environmental harm sufficient to tip the balance in favor of injunctive reliefrdquo)) Neighbors of Cuddy Mountain v US Forest Service 137 F3d 1372 1382 (9th Cir 1998) (stating that ldquo[t]he old growth forests plaintiffs seek to protect would if cut take hundreds of years to reproducerdquo) (internal citation omitted)) see also 11C WRIGHT ET AL supra sect 29481 at 151 (noting that ldquoa preliminary injunction has been issued to prevent harm to the environmentrdquo)

The second factor the balance between the harm to the plaintiff and the harm the defendant will suffer if the injunction is issued also appears to support issuance of a preliminary injunction here The USFS will have to wait before it can develop the Scenic National Forest and the logging company may lose money if the delay is prolonged These economic harms could be compensated monetarily if an injunction is issued inappropriately Where ldquoan injunction bond can compensate [the] defendant for any harm the injunction is likely to inflict the balance should be struck in favor of [the] plaintiffrdquo Id sect 29482 at 192 See also Habitat Educ Center 363 F Supp 2d at 1089 (stating that ldquothe relative absence of harmful effects on the Forest Service weighs in favor of granting the injunctionrdquo)

The third factor is the likelihood that the plaintiff will prevail on the merits Although there is limited information concerning the merits of the action the nonprofit alleges that the federal statute (NEPA) requires an environmental impact statement and further states that the USFS created no environmental impact analysis or statement at all Assuming that those

25

Federal Civil Procedure Analysis

allegations are correct it seems plausible to conclude that the nonprofit will be able to show a likelihood of success on the merits

Finally courts deciding whether or not to issue preliminary injunctive relief are to consider the public interest ldquoFocusing on this factor is another way of inquiring whether there are policy considerations that bear on whether the order should issuerdquo 11C WRIGHT ET AL supra sect 29484 at 214 If the court concludes that the nonprofit is likely to succeed on its NEPA claim because the USFS wrongfully failed to conduct an environmental impact assessment it is likely to find that the public interest would be served by restraining the USFS from proceeding with logging in a national forest See Heartwood Inc v US Forest Service 73 F Supp 2d 962 979 (SD Ill 1999) affrsquod on other grounds 230 F3d 947 (7th Cir 2000) (ldquoviolations by federal agencies of NEPArsquos provisions as established by Congress harm the public as well as the environmentrdquo)

Thus a court is very likely to grant a preliminary injunction if it concludes that the nonprofit has a significant likelihood of success on the merits

26

EVIDENCE ANALYSIS (Evidence ID IIA amp C)

ANALYSIS

Legal Problems

(1) Under what circumstances can evidence of prior convictions be used to impeach a witnessrsquos credibility in a civil case

(1)(a) May the inmatersquos credibility be impeached by evidence of a 12-year-old felony drug conviction if he was released from prison 9 years ago

(1)(b) May the inmatersquos credibility be impeached by evidence of an 8-year-old misdemeanor perjury conviction that was punishable by 1 year in jail if he pleaded guilty and was sentenced only to pay a $5000 fine

(1)(c) May the inmatersquos credibility be impeached by evidence of a 7-year-old sexual assault conviction if the inmate is still serving a 10-year prison sentence and the victim was his 13-year-old daughter

(2)(a) May the guardrsquos credibility be impeached by cross-examination regarding specific instances of misconduct (ie lying on his reacutesumeacute) relevant to credibility

(2)(b) May the guardrsquos credibility be impeached by admission of extrinsic evidence (his reacutesumeacute and academic transcript) offered to prove specific instances of misconduct relevant to credibility

DISCUSSION

Summary

Under the Federal Rules of Evidence witnesses can be impeached with evidence of prior convictions andor specific instances of misconduct Whether evidence of prior convictions should be admitted to impeach generally depends on the nature of the crime the amount of time that has passed and (only in criminal cases) whether the ldquowitnessrdquo is the defendant FED R EVID 609(a)

In this civil case evidence of the inmatersquos conviction for distribution of marijuana should be admitted to impeach the inmate because he was convicted of a felony and was released from prison fewer than 10 years ago FED R EVID 609(a)(1) Credibility is critically important in this case because the jury will hear conflicting testimony from the two disputing parties and there were no other eyewitnesses to the altercation Under Rule 609(a)(1) the inmatersquos conviction should be admitted because it has some bearing on his credibility and its probative value is not substantially outweighed by concerns of unfair prejudice confusion or delay Id

Evidence of the inmatersquos misdemeanor conviction for perjury must be admitted because the crime ldquorequired provingmdashor the witnessrsquos admittingmdasha dishonest act or false statementrdquo by the inmate FED R EVID 609(a)(2)

27

Evidence Analysis

Evidence of the inmatersquos felony conviction for sexual assault should be excluded because its probative value is substantially outweighed by the danger of unfair prejudice to the inmate based on the heinous nature of the crime FED R EVID 609(a)(1) In the alternative the judge could limit the evidence relating to this conviction by excluding details of the inmatersquos crime

In all civil (and criminal) cases witnesses can also be impeached with evidence of specific instances of prior misconduct that did not result in a conviction FED R EVID 608(b) Pursuant to Rule 608(b) misconduct probative of untruthfulness can be inquired into on cross-examination but cannot be proved through extrinsic evidence Id Thus the inmatersquos counsel should be permitted to cross-examine the guard regarding the false statement in the guardrsquos reacutesumeacute However extrinsic evidence of the guardrsquos misconduct (ie the guardrsquos authenticated reacutesumeacute and transcript from the local college) should not be admitted even if the guard denies wrongdoing or refuses to answer cross-examination questions about these matters Id

Point One (10) The Federal Rules of Evidence permit impeachment of witnesses with evidence of prior convictions

Whether convictions should be admitted to impeach generally depends on the nature of the crime the amount of time that has passed and (only in criminal cases) whether the ldquowitnessrdquo is the defendant FED R EVID 609(a) Under Rule 609(a) evidence of prior convictions may be admitted for the purpose of ldquoattacking a witnessrsquos character for truthfulnessrdquo Id

There are two basic types of convictions that can be admitted for the purpose of impeachment

(1) convictions for crimes ldquopunishable by death or by imprisonment for more than one yearrdquo (which generally correlates to ldquofeloniesrdquo) FED R EVID 609(a)(1) and (2) convictions ldquofor any crimes regardless of the punishment if the court can readily determine that establishing the elements of the crime required provingmdashor the witnessrsquos admittingmdasha dishonest act or false statementrdquo FED R EVID 609(a)(2)

Pursuant to Rule 609(a)(1) in civil cases the admission of evidence of a felony conviction is ldquosubject to Rule 403 [which says that a court may exclude relevant evidence if its probative value is substantially outweighed by other factors]rdquo FED R EVID 609(a)(1) However Rule 403 does not protect the witness against admission of prior convictions involving dishonestymdashwhich must be admitted by the court FED R EVID 609(a)(2)

Finally Federal Rule of Evidence 609(b) contains the presumption that a conviction that is more than 10 years old or where more than 10 years has passed since the witnessrsquos release from confinement (whichever is later) should not be admitted unless ldquoits probative value supported by specific facts and circumstances substantially outweighs its prejudicial effectrdquo and the proponent has provided the adverse party with reasonable written notice FED R EVID 609(b)

Point One(a) (25) The court should admit evidence of the inmatersquos 12-year-old felony marijuana distribution conviction

The inmatersquos conviction for marijuana distribution was for a felony punishable by imprisonment for more than one year See FED R EVID 609(a)(1) Moreover although the conviction was 12 years ago the 10-year time limit of Rule 609(b) is not exceeded because that time limit runs

28

Evidence Analysis

from the date of either ldquothe witnessrsquos conviction or release from confinement for it whichever is laterrdquo FED R EVID 609(b) Because the inmate served three years in prison he was released from confinement nine years ago

However pursuant to Rule 609(a)(1) the admission of felony convictions to impeach a witness in a civil case is ldquosubject to Rule 403rdquo FED R EVID 609(a)(1) Neither Rule 609(a) nor the advisory committee notes specify which factors courts should consider when balancing the probative value of a conviction against the dangers identified in Rule 403 (which include (1) unfair prejudice (2) confusion of the issues (3) misleading the jury (4) waste of time or undue delay and (5) needless presentation of cumulative evidence) FED R EVID 403

In this case credibility is very important because the evidence consists primarily of the testimony of the disputing parties and there were no other eyewitnesses to the altercation This enhances the probative value of any evidence bearing on the inmatersquos credibility A court is likely to conclude that the inmatersquos prior felony drug conviction is relevant to his credibility See eg United States v Brito 427 F3d 53 64 (1st Cir 2005) (ldquoPrior drug-trafficking crimes are generally viewed as having some bearing on veracityrdquo) Although the probative value of any conviction diminishes with age see eg United States v Brewer 451 F Supp 50 53 (ED Tenn 1978) the inmatersquos ongoing problems with the law suggest that he has continued (and even escalated) his criminal behavior over the past nine years The court should admit this evidence because its probative value is not substantially outweighed by any Rule 403 concerns Specifically any prejudice to the inmate would be slight because the conviction is unrelated to the altercation at issue and the conviction was not for a heinous crime that might inflame the jury

[NOTE Whether an examinee identifies the jury instruction as containing a ldquoconclusiverdquo or ldquomandatoryrdquo presumption is less important than the examineersquos analysis of the constitutional infirmities]

Point One(b) (15) The court must admit evidence of the inmatersquos eight-year-old misdemeanor conviction because perjury is a crime of dishonesty

Rule 609(a)(2) provides that evidence of a criminal conviction ldquomust be admitted if the court can readily determine that establishing the elements of the crime required provingmdashor the witnessrsquos admittingmdasha dishonest act or false statementrdquo FED R EVID 609(a)(2) The inmatersquos conviction for perjury would have necessarily required proving that the inmate engaged in an act of dishonesty This conviction occurred within the past 10 years so it ldquomust be admittedrdquo because in contrast to Rule 609(a)(1) (discussed in Point One(a)) admission under Rule 609(a)(2) is mandatory and not subject to Rule 403

Point One(c) (20) The court should exclude evidence of the inmatersquos seven-year-old felony sexual assault conviction because the probative value of this evidence is substantially outweighed by the danger of unfair prejudice In the alternative the details of the prior conviction could be excluded

The inmatersquos conviction for felony sexual assault was seven years ago and he has not yet been released from incarceration so Rule 609(a) but not 609(b) is applicable here FED R EVID 609(a) This conviction is therefore admissible to impeach the inmate unless its probative value is substantially outweighed by the danger of unfair prejudice or any other Rule 403 concern Id

29

Evidence Analysis

Sex crimes are generally not considered relevant to credibility see Hopkins v State 639 So 2d 1247 1254 (Miss 1993) so the probative value of this conviction is relatively low Moreover the heinous nature of the inmatersquos crime (sexual assault on his daughter) makes the danger of unfair prejudice to the inmate very high Thus the court should exclude evidence of the conviction because it was for a heinous offense that is likely to inflame the jury and it has little bearing on credibility See eg United States v Beahm 664 F2d 414 419 (4th Cir 1981)

As an alternative to excluding this evidence the judge could minimize the unfair prejudice to the inmate by permitting limited cross-examination but refusing to allow specific questions about the nature of the inmatersquos conviction For example a court could limit cross-examination to the fact that the inmate was convicted of a ldquofelonyrdquo or perhaps that he was convicted of a ldquosexual assaultrdquo without identifying the victim However because evidence of the inmatersquos prior convictions can be admitted solely for the purpose of enabling the jury to assess his credibility and because his two earlier convictions should have already been admitted the court should exclude all evidence of the felony sexual assault conviction

Point Two(a) (15) The court should permit the inmatersquos counsel to cross-examine the guard regarding the false statement in his reacutesumeacute because the guardrsquos misconduct bears on his truthfulness

The inmate wishes to cross-examine the guard about his prior dishonest behaviormdashlying on his reacutesumeacutemdashthat did not involve a criminal conviction Rule 608(b) allows witnesses to be cross-examined about specific instances of prior non-conviction misconduct probative of untruthfulness ldquoin order to attack the witnessrsquos character for truthfulnessrdquo FED R EVID 608(b)

The courtrsquos decision to allow cross-examination about the guardrsquos prior dishonest behavior depends on the probative value of such evidence balanced against the danger of unfair prejudice to the guard or any other Rule 403 concern FED R EVID 403 Here the guardrsquos false statement on his reacutesumeacute that he obtained a degree in Criminal Justice is highly probative of his untruthfulness because it grossly misrepresents his actual academic record was made recently and was made with the intent to deceive Because the probative value of this evidence is very strong and is not substantially outweighed by any Rule 403 concerns cross-examination of the guard on this topic should be permitted The court may also consider it fair to permit this cross-examination of the guard on these matters assuming that one or more of the inmatersquos prior convictions have been admitted to impeach his credibility

Point Two(b) (15) The court should exclude extrinsic evidence of the guardrsquos non-conviction misconduct even if the guard denies wrongdoing or refuses to answer questions about the matter

Although Rule 608(b) allows cross-examination about specific instances of prior misconduct probative of untruthfulness ldquoextrinsic evidencerdquo offered to prove such misconduct is not admissible FED R EVID 608(b) The rationale for this rule is that allowing the introduction of extrinsic evidence of prior misconduct by witnesses when these acts are relevant only to the witnessesrsquo truthfulness and not to the main issues in the case would create too great a risk of confusing the jury and unduly delaying the trial The court does not have discretion to admit this extrinsic evidence See eg United States v Elliot 89 F3d 1360 1368 (8th Cir 1996)

30

Evidence Analysis

Here the inmatersquos counsel may cross-examine the guard about the false statement on his reacutesumeacute However the inmatersquos counsel must accept the guardrsquos response Even if the guard denies wrongdoing or refuses to answer questions about the matter the inmatersquos counsel cannot introduce the guardrsquos reacutesumeacute or the transcript from the local college to prove the guardrsquos misconduct

31

CORPORATIONS ANALYSIS (Corporations VA2 IX)

ANALYSIS

Legal Problems

(1) Do shareholders have the authority to amend a corporationrsquos bylaws with respect to director nominations

(2) Do board-approved bylaws on a particular subject here nomination of directors preempt subsequent conflicting bylaw amendments by shareholders

(3) Is a suit challenging both managementrsquos refusal to include the proposed bylaw amendment in Megarsquos proxy statement and the boardrsquos amendment of the bylaws dealing with nomination of directors a direct or derivative suit

DISCUSSION

Summary

The voting and litigation rights of the shareholders of Mega are subject to the provisions of the Model Business Corporations Act (MBCA)

The investorrsquos proposed bylaw provision is not inconsistent with state law Under the MBCA shareholders may amend the bylaws when the amendment deals with a proper matter for the corporationrsquos bylaws such as procedures for nominating directors

The Mega boardrsquos bylaw amendment does not preempt the investorrsquos proposed bylaw provision or the Mega shareholdersrsquo power to approve it While shareholders can limit the boardrsquos power to amend or repeal the bylaws the board cannot limit the shareholdersrsquo power

Whether the investor must make a demand on Megarsquos board depends on how the investor frames its claim If the investor claims a violation of shareholder voting rights the claim is direct and pre-suit demand on the board is not required If on the other hand the investor claims that the directors violated their fiduciary duties by amending the bylaws to entrench themselves the claim is derivative and a pre-suit demand is required

Point One (30) Shareholders may amend the corporationrsquos bylaws where the proposed bylaw provision relates to procedural matters typically included in the bylaws such as the nomination of directors

Internal affairs of the corporation such as the conduct of shareholder meetings and election of directors are subject to the corporate law of the state of incorporation See McDermott Inc v Lewis 531 A2d 206 (Del 1987) (applying law of jurisdiction where corporation was incorporated in case involving voting rights) This statersquos corporate statute is modeled on the MBCA

Under the MBCA ldquoshareholders may amend the corporationrsquos bylawsrdquo MBCA sect 1020(a) Thus the only question is whether the bylaws can specify the procedures for shareholder nomination of directors

32

Corporations Analysis

The MBCA states that the bylaws ldquomay contain any provision that is not inconsistent with law or the articles of incorporationrdquo MBCA sect 206(b) In addition the MBCA was revised in 2009 to address shareholder nomination of directors in public corporations (known as ldquoproxy accessrdquo) and specifies that the bylaws ldquomay contain a requirement that the corporation include in its [proxy materials] one or more individuals nominated by a shareholderrdquo MBCA sect 206(c)(1) see Committee on Corporate Laws ABA Section of Business Law Report on the Roles of Boards of Directors and Shareholders of Publicly Owned Corporations and Changes to the Model Business Corporations ActmdashAdoption of Shareholder Proxy Access Amendments to Chapters 2 and 10 65 BUS LAWYER 1105 (2010)

The inclusion of director-nomination procedures in the bylaws is consistent with practice and is recognized by the Delaware courts whose views on corporate law carry significant weight Typically the procedures for nomination of directors are found in the bylaws See 1 COX amp HAZEN TREATISE ON THE LAW OF CORPORATIONS sect 312 (3d ed 2011) see also 4 FLETCHER CORP FORMS ANN PART III ch 21 (2013) (including sample bylaws that permit nomination of directors by shareholders) The Delaware Supreme Court has confirmed that the bylaws may ldquodefine the process and proceduresrdquo for director elections See CA Inc v AFSCME Employees Pension Plan 953 A2d 227 (Del 2008) (concluding that bylaw amendment requiring reimbursement of election expenses to certain successful shareholder nominators is ldquoproper subjectrdquo under Delaware law)

[NOTE The question of the proper scope of the bylaws can be answered using the more general MBCA sect 206(b) or the 2009 MBCA revision adding sect 206(c)(1) (adopted in CT ME VA) In addition some examinees might raise the point that shareholder proposals may not compel the board to take action such as by including shareholder nominations in the companyrsquos proxy materials on the theory that the ldquobusiness and affairsrdquo of the corporation are to be managed by the board See MBCA sect 801(b) Although shareholders are generally limited to adopting precatory resolutions that recommend or encourage board action this limitation does not apply when shareholders have specific authority to take binding action on their ownmdashsuch as to amend the bylaws]

Point Two (30) Shareholders can amend (or repeal) board-approved bylaws Further shareholders can limit the boardrsquos power to later amend and repeal a shareholder-approved bylaw

Under the MBCA shareholders have the power to amend the bylaws See Point One The board shares this power with the shareholders unless (1) the corporationrsquos articles ldquoreserve that power exclusively to the shareholdersrdquo or (2) ldquothe shareholders in amending repealing or adopting a bylaw expressly provide that the board of directors may not amend repeal or reinstate that bylawrdquo See MBCA sect 1020(b)

Shareholder-approved bylaw provisions can amend or repeal existing bylaw provisions whether originally approved by the board or by shareholders See ALAN R PALMITER CORPORATIONS EXAMPLES AND EXPLANATIONS sect 713 (7th ed 2012) Thus the Mega boardrsquos bylaw amendmentmdashwhich set more demanding thresholds for shareholder nomination of directors than the investorrsquos proposed bylaw provisionmdashwould be superseded (repealed) if Megarsquos shareholders were to approve the investorrsquos proposal

Further a shareholder-approved bylaw generally can limit the power of the board to later amend or repeal it See MBCA sect 1020(b)(2) Thus if Megarsquos shareholders approved the bylaw

33

Corporations Analysis

provision proposed by the investor Megarsquos board could not repeal the provision because it includes a ldquono board repealrdquo clause

The revision to the MBCA in 2009 dealing with shareholder proxy access does not change this conclusion That revision specifies that a shareholder-approved bylaw dealing with director nominations may not limit the boardrsquos power to amend add or repeal ldquoany procedure or condition to such a bylaw in order to provide for a reasonable practicable and orderly processrdquo MBCA sect 206(d) Thus according to the revision if shareholders approve a bylaw amendment that limits further board changes the board would nonetheless retain the power to ldquotinkerrdquo with the bylaw to safeguard the voting process but could not repeal the shareholder-approved bylaw The Official Comment to MBCA sect 206(d) makes clear that the revision is ldquonot intended to allow the board of directors to frustrate the purpose of the shareholder-adopted proxy access provisionrdquo Thus if Megarsquos shareholders were to approve the bylaw provision proposed by the investor Megarsquos board could only amend the provision regarding its procedures or conditions in a manner consistent with its purpose of permitting proxy access for Megarsquos shareholders

[NOTE The boardrsquos attempted interference with a shareholder voting initiative may also have been a violation of the directorsrsquo fiduciary duties See Blasius Indus Inc v Atlas Corp 564 A2d 651 (Del Ch 1988) (finding that directors breached their fiduciary duties by amending bylaws and expanding size of board to thwart insurgentrsquos plan to amend bylaws and seat a majority of new directors) The call however asks examinees to consider whether shareholders or the board have ldquoprecedencerdquo over amending the corporate bylaws Thus an examineersquos answer should be framed in terms of ldquopowerrdquo and not ldquodutyrdquo]

Point Three (40) The investor need not make a demand on the board if the investor states a direct claim such as an allegation that the board interfered with the investorrsquos right to amend the bylaws But the investor must make a demand on the board if the investor states a derivative claim (on behalf of the corporation) such as an allegation that the directors sought to entrench themselves by interfering with the proposed proxy access

The MBCA generally requires that shareholders make a demand on the board of directors before initiation of a derivative suit MBCA sect 742 (shareholder may not bring derivative proceeding until written demand has been made on corporation and 90 days have expired) A derivative suit is essentially two suits in one where the plaintiff-shareholder seeks to bring on behalf of the corporation a claim that vindicates corporate rights usually based on violation of fiduciary duties PALMITER supra sect 1811 (6th ed 2009) The demand permits the board to investigate the situation identified by the shareholder and take suitable action No demand on the board is required however if the shareholder brings a direct suit to vindicate the shareholderrsquos own rights not those of the corporation

Is the suit brought by the investor derivative or direct The MBCA defines a ldquoderivative proceedingrdquo as one brought ldquoin the right of a domestic corporationrdquo MBCA sect 740(1) Thus the answer to how the investorrsquos suit should be characterized turns on what rights the investor seeks to vindicate If the investor frames its claim as one of fiduciary breach by directorsmdashfor example for failing to become adequately informed about voting procedures or for seeking to entrench themselves in office by manipulating the voting structure to avoid a shareholder insurgencymdashthen the suit is ldquoderivativerdquo and the investor must make a demand on the board See MBCA Ch 7 Subch D Introductory Comment (ldquothe derivative suit has historically been the principal method of challenging allegedly illegal action by managementrdquo)

34

Corporations Analysis

If however the investor frames its claim as one to vindicate shareholder rights the suit is direct and no demand is required For many courts the direct-derivative question turns on who is injured and who is to receive the relief sought by the plaintiff-shareholders See Tooley v Donaldson Lufkin amp Jenrette Inc 845 A2d 1031 (Del 2004) (characterizing a merger-delay claim as direct because delay of merger only harmed shareholders not corporation) Thus if the investor claims that managementrsquos refusal to include its proposed bylaw amendment in the corporationrsquos proxy materials violates its shareholder rights to initiate corporate governance reforms the suit will be direct Courts have not questioned the ability of shareholders to bring direct suits challenging board action to exclude their proposed bylaw amendments from the corporationrsquos proxy materials See JANA Master Fund Ltd v CNET Networks Inc 954 A2d 335 (Del Ch 2008) (upholding shareholderrsquos direct challenge to boardrsquos interpretation of advance-notice bylaw) Chesapeake Corp v Shore 771 A2d 293 (Del Ch 2000) (upholding shareholderrsquos direct challenge to actions by board that effectively prevented it from proposing bylaw amendments in contest for control)

Is the way that the investor frames its claim conclusive Courts have permitted shareholder-plaintiffs to challenge a transaction in a direct suit even though the same transaction could also be challenged as a fiduciary breach See Eisenberg v Flying Tiger Line Inc 451 F2d 267 (2d Cir 1971) (permitting direct suit challenging a corporate reorganization as a dilution of shareholder voting power even though reorganization may have involved conflicts of interest and thus constituted a fiduciary breach) Thus the investorrsquos choice to pursue a claim challenging the legality of managementrsquos decision to exclude the investorrsquos proposal from the corporationrsquos proxy materialsmdashrather than a possible breach of fiduciary dutymdashis likely to be respected See 3 COX amp HAZEN supra sect 153 (describing situations in which a claim can be framed as derivative or direct)

[NOTE Some issues under Delaware corporate law regarding pre-suit demand are not relevant here For example whether the Mega directors are independent and disinterested is not relevant to the MBCA requirement of a pre-suit demand As the Official Comment to MBCA sect 742 points out the MBCArsquos requirement of ldquouniversal demandrdquo gives the board ldquothe opportunity to reexamine the act complained of in the light of a potential lawsuit and take corrective actionrdquo even when the directors might be non-independent or have conflicts of interest

Nor is it relevant to the MBCA pre-suit demand requirement that the statutory 90-day waiting period may be onerous The first paragraph of MBCA sect 742 requires a pre-suit demand without exception the second paragraph of the section imposes a 90-day waiting period before a derivative suit may be brought which can be shortened if the board rejects the demand or ldquoirreparable injury to the corporation would result by waiting for the expiration of the 90-day periodrdquo The call as written asks only whether a pre-suit demand should be made and does not ask examinees to address whether the post-demand waiting period should be shortened under the ldquoirreparable injuryrdquo standard]

35

National Conference of Bar Examiners 302 South Bedford Street | Madison WI 53703-3622 Phone 608-280-8550 | Fax 608-280-8552 | TDD 608-661-1275

wwwncbexorg e-mail contactncbexorg

  • Preface
  • Description of the MEE
  • Instructions
  • July 2014 Questions
    • CRIMINAL LAW AND PROCEDURE QUESTION
    • CONTRACTS QUESTION
    • FAMILY LAW QUESTION
    • FEDERAL CIVIL PROCEDURE QUESTION
    • EVIDENCE QUESTION
    • CORPORATIONS QUESTION
      • July 2014 Analyses
        • CRIMINAL LAW AND PROCEDURE ANALYSIS
        • CONTRACTS ANALYSIS
        • FAMILY LAW ANALYSIS
        • FEDERAL CIVIL PROCEDURE ANALYSIS
        • EVIDENCE ANALYSIS
        • CORPORATIONS ANALYSIS
            • ltlt13 ASCII85EncodePages false13 AllowTransparency false13 AutoPositionEPSFiles true13 AutoRotatePages None13 Binding Left13 CalGrayProfile (Dot Gain 20)13 CalRGBProfile (sRGB IEC61966-21)13 CalCMYKProfile (US Web Coated 050SWOP051 v2)13 sRGBProfile (sRGB IEC61966-21)13 CannotEmbedFontPolicy Error13 CompatibilityLevel 1413 CompressObjects Tags13 CompressPages true13 ConvertImagesToIndexed true13 PassThroughJPEGImages true13 CreateJobTicket false13 DefaultRenderingIntent Default13 DetectBlends true13 DetectCurves 0000013 ColorConversionStrategy CMYK13 DoThumbnails false13 EmbedAllFonts true13 EmbedOpenType false13 ParseICCProfilesInComments true13 EmbedJobOptions true13 DSCReportingLevel 013 EmitDSCWarnings false13 EndPage -113 ImageMemory 104857613 LockDistillerParams false13 MaxSubsetPct 10013 Optimize true13 OPM 113 ParseDSCComments true13 ParseDSCCommentsForDocInfo true13 PreserveCopyPage true13 PreserveDICMYKValues true13 PreserveEPSInfo true13 PreserveFlatness true13 PreserveHalftoneInfo false13 PreserveOPIComments true13 PreserveOverprintSettings true13 StartPage 113 SubsetFonts true13 TransferFunctionInfo Apply13 UCRandBGInfo Preserve13 UsePrologue false13 ColorSettingsFile ()13 AlwaysEmbed [ true13 ]13 NeverEmbed [ true13 ]13 AntiAliasColorImages false13 CropColorImages true13 ColorImageMinResolution 30013 ColorImageMinResolutionPolicy OK13 DownsampleColorImages true13 ColorImageDownsampleType Bicubic13 ColorImageResolution 30013 ColorImageDepth -113 ColorImageMinDownsampleDepth 113 ColorImageDownsampleThreshold 15000013 EncodeColorImages true13 ColorImageFilter DCTEncode13 AutoFilterColorImages true13 ColorImageAutoFilterStrategy JPEG13 ColorACSImageDict ltlt13 QFactor 01513 HSamples [1 1 1 1] VSamples [1 1 1 1]13 gtgt13 ColorImageDict ltlt13 QFactor 01513 HSamples [1 1 1 1] VSamples [1 1 1 1]13 gtgt13 JPEG2000ColorACSImageDict ltlt13 TileWidth 25613 TileHeight 25613 Quality 3013 gtgt13 JPEG2000ColorImageDict ltlt13 TileWidth 25613 TileHeight 25613 Quality 3013 gtgt13 AntiAliasGrayImages false13 CropGrayImages true13 GrayImageMinResolution 30013 GrayImageMinResolutionPolicy OK13 DownsampleGrayImages true13 GrayImageDownsampleType Bicubic13 GrayImageResolution 30013 GrayImageDepth -113 GrayImageMinDownsampleDepth 213 GrayImageDownsampleThreshold 15000013 EncodeGrayImages true13 GrayImageFilter DCTEncode13 AutoFilterGrayImages true13 GrayImageAutoFilterStrategy JPEG13 GrayACSImageDict ltlt13 QFactor 01513 HSamples [1 1 1 1] VSamples [1 1 1 1]13 gtgt13 GrayImageDict ltlt13 QFactor 01513 HSamples [1 1 1 1] VSamples [1 1 1 1]13 gtgt13 JPEG2000GrayACSImageDict ltlt13 TileWidth 25613 TileHeight 25613 Quality 3013 gtgt13 JPEG2000GrayImageDict ltlt13 TileWidth 25613 TileHeight 25613 Quality 3013 gtgt13 AntiAliasMonoImages false13 CropMonoImages true13 MonoImageMinResolution 120013 MonoImageMinResolutionPolicy OK13 DownsampleMonoImages true13 MonoImageDownsampleType Bicubic13 MonoImageResolution 120013 MonoImageDepth -113 MonoImageDownsampleThreshold 15000013 EncodeMonoImages true13 MonoImageFilter CCITTFaxEncode13 MonoImageDict ltlt13 K -113 gtgt13 AllowPSXObjects false13 CheckCompliance [13 None13 ]13 PDFX1aCheck false13 PDFX3Check false13 PDFXCompliantPDFOnly false13 PDFXNoTrimBoxError true13 PDFXTrimBoxToMediaBoxOffset [13 00000013 00000013 00000013 00000013 ]13 PDFXSetBleedBoxToMediaBox true13 PDFXBleedBoxToTrimBoxOffset [13 00000013 00000013 00000013 00000013 ]13 PDFXOutputIntentProfile ()13 PDFXOutputConditionIdentifier ()13 PDFXOutputCondition ()13 PDFXRegistryName ()13 PDFXTrapped False1313 CreateJDFFile false13 Description ltlt13 ARA 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 BGR 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 CHS ltFEFF4f7f75288fd94e9b8bbe5b9a521b5efa7684002000410064006f006200650020005000440046002065876863900275284e8e9ad88d2891cf76845370524d53705237300260a853ef4ee54f7f75280020004100630072006f0062006100740020548c002000410064006f00620065002000520065006100640065007200200035002e003000204ee553ca66f49ad87248672c676562535f00521b5efa768400200050004400460020658768633002gt13 CHT ltFEFF4f7f752890194e9b8a2d7f6e5efa7acb7684002000410064006f006200650020005000440046002065874ef69069752865bc9ad854c18cea76845370524d5370523786557406300260a853ef4ee54f7f75280020004100630072006f0062006100740020548c002000410064006f00620065002000520065006100640065007200200035002e003000204ee553ca66f49ad87248672c4f86958b555f5df25efa7acb76840020005000440046002065874ef63002gt13 CZE 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 DAN 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 DEU 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 ESP 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 ETI 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 FRA 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 GRE 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 HEB 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 HRV (Za stvaranje Adobe PDF dokumenata najpogodnijih za visokokvalitetni ispis prije tiskanja koristite ove postavke Stvoreni PDF dokumenti mogu se otvoriti Acrobat i Adobe Reader 50 i kasnijim verzijama)13 HUN 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 ITA 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 JPN ltFEFF9ad854c18cea306a30d730ea30d730ec30b951fa529b7528002000410064006f0062006500200050004400460020658766f8306e4f5c6210306b4f7f75283057307e305930023053306e8a2d5b9a30674f5c62103055308c305f0020005000440046002030d530a130a430eb306f3001004100630072006f0062006100740020304a30883073002000410064006f00620065002000520065006100640065007200200035002e003000204ee5964d3067958b304f30533068304c3067304d307e305930023053306e8a2d5b9a306b306f30d530a930f330c8306e57cb30818fbc307f304c5fc59808306730593002gt13 KOR ltFEFFc7740020c124c815c7440020c0acc6a9d558c5ec0020ace0d488c9c80020c2dcd5d80020c778c1c4c5d00020ac00c7a50020c801d569d55c002000410064006f0062006500200050004400460020bb38c11cb97c0020c791c131d569b2c8b2e4002e0020c774b807ac8c0020c791c131b41c00200050004400460020bb38c11cb2940020004100630072006f0062006100740020bc0f002000410064006f00620065002000520065006100640065007200200035002e00300020c774c0c1c5d0c11c0020c5f40020c2180020c788c2b5b2c8b2e4002egt13 LTH 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 LVI 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 NLD (Gebruik deze instellingen om Adobe PDF-documenten te maken die zijn geoptimaliseerd voor prepress-afdrukken van hoge kwaliteit De gemaakte PDF-documenten kunnen worden geopend met Acrobat en Adobe Reader 50 en hoger)13 NOR 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 POL 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 PTB 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 RUM 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 RUS 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 SKY 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 SLV 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 SUO 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 SVE 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 TUR ltFEFF005900fc006b00730065006b0020006b0061006c006900740065006c0069002000f6006e002000790061007a006401310072006d00610020006200610073006b013100730131006e006100200065006e0020006900790069002000750079006100620069006c006500630065006b002000410064006f006200650020005000440046002000620065006c00670065006c0065007200690020006f006c0075015f007400750072006d0061006b0020006900e70069006e00200062007500200061007900610072006c0061007201310020006b0075006c006c0061006e0131006e002e00200020004f006c0075015f0074007500720075006c0061006e0020005000440046002000620065006c00670065006c0065007200690020004100630072006f006200610074002000760065002000410064006f00620065002000520065006100640065007200200035002e003000200076006500200073006f006e0072006100730131006e00640061006b00690020007300fc007200fc006d006c00650072006c00650020006100e70131006c006100620069006c00690072002egt13 UKR 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 ENU (Use these settings to create Adobe PDF documents best suited for high-quality prepress printing Created PDF documents can be opened with Acrobat and Adobe Reader 50 and later)13 gtgt13 Namespace [13 (Adobe)13 (Common)13 (10)13 ]13 OtherNamespaces [13 ltlt13 AsReaderSpreads false13 CropImagesToFrames true13 ErrorControl WarnAndContinue13 FlattenerIgnoreSpreadOverrides false13 IncludeGuidesGrids false13 IncludeNonPrinting false13 IncludeSlug false13 Namespace [13 (Adobe)13 (InDesign)13 (40)13 ]13 OmitPlacedBitmaps false13 OmitPlacedEPS false13 OmitPlacedPDF false13 SimulateOverprint Legacy13 gtgt13 ltlt13 AddBleedMarks false13 AddColorBars false13 AddCropMarks false13 AddPageInfo false13 AddRegMarks false13 ConvertColors ConvertToCMYK13 DestinationProfileName ()13 DestinationProfileSelector DocumentCMYK13 Downsample16BitImages true13 FlattenerPreset ltlt13 PresetSelector MediumResolution13 gtgt13 FormElements false13 GenerateStructure false13 IncludeBookmarks false13 IncludeHyperlinks false13 IncludeInteractive false13 IncludeLayers false13 IncludeProfiles false13 MultimediaHandling UseObjectSettings13 Namespace [13 (Adobe)13 (CreativeSuite)13 (20)13 ]13 PDFXOutputIntentProfileSelector DocumentCMYK13 PreserveEditing true13 UntaggedCMYKHandling LeaveUntagged13 UntaggedRGBHandling UseDocumentProfile13 UseDocumentBleed false13 gtgt13 ]13gtgt setdistillerparams13ltlt13 HWResolution [2400 2400]13 PageSize [612000 792000]13gtgt setpagedevice13

Page 16: July 2014 MEE Questions and Analyses - NCBE...This publication includes the questions and analyses from the July 2014 MEE. (In the actual test, the questions are simply numbered rather

Criminal Law and Procedure Analysis

adversary judicial criminal proceedingsmdashwhether by way of formal charge preliminary hearing indictment information or arraignment [or in some states arrest warrant]rdquo McNeil v Wisconsin 501 US 171 175 (1991) (internal quotations omitted)) Once a suspectrsquos Sixth Amendment right to counsel has attached any attempts to ldquodeliberately elicitrdquo statements from him in the absence of his attorney violate the Sixth Amendment See Massiah v United States 377 US 201 (1964) Brewer v Williams 430 US 387 (1977)

The Sixth Amendment right to counsel is charge- or offense -specific Representation by counsel in one prosecution does not in itself guarantee counsel for uncharged offenses See McNeil 501 US at 175 Texas v Cobb 532 US 162 (2001) Here the suspectrsquos Sixth Amendment right to counsel had attached only for the pending aggravated assault charge The suspectrsquos right to counsel for the aggravated assault case did not guarantee counsel for the five unrelated and uncharged burglaries that were the subject of the detectiversquos interrogation Thus because formal adversarial judicial proceedings against the suspect for the uncharged burglaries had not begun he had no Sixth Amendment right to counsel

Finally the detectiversquos failure to inform the suspect of the lawyerrsquos presence and demands to speak with him does not implicate the suspectrsquos Sixth Amendment right to counsel which had not yet attached See id Moran v Burbine 475 US 412 428ndash31 (1986)

Point Two (30) The suspect did not effectively invoke his right to counsel under Miranda because his statement was not unambiguous

A suspect subject to custodial interrogation has a right to consult with counsel and to have an attorney present during questioning Miranda v Arizona 384 US 436 (1966) When a suspect invokes his right to counsel during an interrogation law enforcement must immediately cease all questioning See Edwards v Arizona 451 US 477 484ndash85 (1981) Custodial interrogation cannot be reinitiated unless and until the suspect has been re-advised of his Miranda rights has provided a knowing and voluntary waiver and (1) counsel is present and (2) the suspect himself initiated further communication with the police see id at 484 or (3) (if the suspect was released from custody after the initial interrogation) at least 14 days have passed Maryland v Shatzer 559 US 98 110 (2010)

To invoke the right to counsel a suspectrsquos request must be ldquounambiguousrdquo This means that the suspect must articulate the desire for counsel sufficiently clearly that a reasonable officer would understand the statement to be a request for counsel Davis v United States 512 US 452 459 (1994) If the request is ambiguous the police are not required to stop the interrogation

In this case the suspectrsquos statement ldquoI think I want my lawyer here before I talk to yourdquo was not an unambiguous request for counsel The most reasonable interpretation of this statement is that the suspect might be invoking his right to counsel Id at 461 (ldquomaybe I should talk to a lawyerrdquo is not an unequivocal request for counsel) See also Burket v Angelone 208 F3d 172 197ndash98 (4th Cir 2000) (ldquoI think I need a lawyerrdquo is not an unambiguous request for an attorney) Soffar v Cockrell 300 F3d 588 594ndash95 (5th Cir 2002) (discussion of various statements that did not constitute unequivocal requests for counsel)

Under these circumstances the detective was not required to cease the custodial interrogation of the suspect Nor was the detective required to clarify or ask follow-up questions to determine whether the suspect in fact wanted an attorney Davis 512 US at 459ndash60

14

Criminal Law and Procedure Analysis

Point Three (35) The suspectrsquos waiver of his Miranda rights was knowing intelligent and voluntary despite the fact that he was never told of the lawyerrsquos presence in the jail or of the lawyerrsquos demands

A valid waiver of Miranda rights must be ldquovoluntaryrdquomdashie the product of a free or deliberate choice rather than intimidation coercion or deception Berghuis v Thompkins 560 US 370 382ndash83 (2010) In addition the waiver must be knowing and intelligent That is it ldquomust have been made with a full awareness of both the nature of the right being abandoned and the consequences of the decision to abandon itrdquo Moran v Burbine 475 US 421 (1986)

In this case the suspect signed a Miranda waiver form after receiving proper warnings There is no evidence ldquothat the police resorted to physical or psychological pressure to elicit the statementsrdquo Id The entire interview lasted only 45 minutes The only issue is whether the suspect knowingly and intelligently waived his Miranda rights despite the fact that the detective did not tell the suspect about the lawyerrsquos presence and her demands

The Supreme Court has said that ldquo[e]vents occurring outside of the presence of the suspect and entirely unknown to him surely can have no bearing on the capacity to comprehend and knowingly relinquish a constitutional rightrdquo Id at 422 If the suspect ldquoknew that he could stand mute and request a lawyer and was aware of the Statersquos intention to use his statements to secure a convictionrdquo then the waiver is valid regardless of the information withheld Id at 422ndash23

Here the suspect was correctly informed of his rights Miranda v Arizona 384 US at 467ndash73 His comments demonstrate that he understood that he could have a lawyer present if he desired (ie wondering whether he should call his attorney) and that he understood that there might be consequences to speaking with the detective (ldquoI probably should keep my mouth shut but Irsquom willing to talk to you for a whilerdquo) His comment ldquo[L]etrsquos not waste any time waiting for someone to call my attorney and having her drive hererdquo along with his signature on the Miranda waiver form show that his waiver was valid under the constitutional standard

The fact that the detective did not tell the suspect about the lawyerrsquos presence and demands has no bearing on the validity of the suspectrsquos waiver because ldquosuch conduct is only relevant to the constitutional validity of a waiver if it deprives a defendant of knowledge essential to his ability to understand the nature of his rights and the consequences of abandoning themrdquo Moran at 424 The Supreme Court has specifically declined to adopt a rule requiring that law enforcement tell a suspect of an attorneyrsquos efforts to contact him id at 425 (ldquoNor are we prepared to adopt a rule requiring that the police inform a suspect of an attorneyrsquos efforts to reach himrdquo)

[NOTE An examinee might also recognize that this general rule is further supported by the Supreme Courtrsquos decision in Florida v Powell 559 US 50 (2010) approving state Miranda warnings that do not explicitly warn suspects that they have a right to have counsel present during custodial interrogation]

15

CONTRACTS ANALYSIS (Contracts IB2 IIB IVA3 amp A5)

ANALYSIS

Legal Problems

(1) In the case of a service contract (governed by the common law of contracts) is a modification enforceable when a party agrees to pay more for the same performance than was originally promised

(2) In the case of a contract for the sale of goods (governed by Article 2 of the UCC) is a modification enforceable when a party agrees to pay more for the same goods than was originally promised

(3) May a party avoid an agreement on the basis of economic duress

DISCUSSION

Summary

There are two arguments that the conservatory can make to support the claim that it is not bound to pay the higher prices lack of consideration and economic duress

The organ repair contract is governed by the common law of contracts Under the common law the business would have difficulty recovering the additional $60000 for the organ repair because under the ldquopreexisting duty rulerdquo the agreement of the conservatory to pay the extra price was not supported by consideration However the business might argue that the modification is enforceable under an exception to the preexisting duty rule for fair and equitable modifications made in light of unanticipated circumstances

The organ sale contract is governed by Article 2 of the Uniform Commercial Code The business would likely recover the additional amount under that contract because Article 2 provides that consideration is not required for a modification to be binding

In both cases the conservatory could seek to avoid its agreement on the grounds of economic duress but that argument is not likely to succeed

Point One (45) The business probably cannot recover the additional $60000 for the organ repair because the conservatoryrsquos promise to pay more money was not supported by consideration

The general rule is that to be enforceable a promise must be supported by consideration Under RESTATEMENT (SECOND) OF CONTRACTS sect 71 a promise is supported by consideration if it is bargained for in exchange for a return promise or performance However under the ldquopreexisting duty rulerdquo (exemplified in RESTATEMENT (SECOND) OF CONTRACTS sect 73 and Alaska Packersrsquo Assrsquon v Domenico 117 F 99 (9th Cir 1902)) promise of performance of a legal duty already owed to a promisor which is neither doubtful nor the subject of honest dispute is not consideration

If the business had promised the conservatory anything new or different in exchange for the agreement to pay the additional $60000 (such as for example repairing the pipe organ more

16

Contracts Analysis

quickly or using better parts) that would constitute consideration especially in light of the principle that courts do not inquire into the adequacy of consideration Here however the business already had a legal duty under the original contract and did not agree to do anything else in exchange for the conservatoryrsquos promise to pay $60000 more

However an exception to the preexisting duty rule is sometimes applied in situations of unanticipated changed circumstances Under RESTATEMENT (SECOND) OF CONTRACTS sect 89 followed in many jurisdictions a promise modifying a duty under a contract not fully performed on either side is binding even if not supported by consideration if the modification is fair and equitable in view of circumstances not anticipated by the parties when the contract was made

If a court applies the rule in Restatement sect 89 the critical issues will be whether the modification was in fact ldquofair and equitablerdquo and whether it can be justified in light of unanticipated circumstances In many cases in which modifications have been upheld a party encountered difficulties or burdens in performing far beyond what was knowingly bargained for in the original contract with the result bordering on impracticability such as having to excavate solid rock instead of soft dirt or having to remove garbage far in excess of the amounts contemplated The conservatory would argue that the businessrsquos performance difficulties were not of this sort at allmdashnothing about repairing the pipe organ itself was any different from or more difficult than originally contemplated except that the business itself encountered financial distress unrelated to its burdens in performing its obligations under these contracts

Even if the business satisfies that element of the rule in Restatement sect 89 the business must also demonstrate that the circumstances that gave rise to the need to modify the contract were ldquounanticipatedrdquo at the time the original contract was made Here the facts suggest that when the business entered into the original contract it expected that the price paid by the conservatory would enable it to perform However any evidence that the business knew or had reason to know at the time of execution that it would need more money from the conservatory to be able to perform would mean that the request to modify was not ldquounanticipatedrdquo

[NOTE Some cases such as Schwartzreich v Bauman-Basch Inc 231 NY 196 131 NE 887 (1921) find that if the parties mutually agreed to rescind the original contract and then after rescission entered into an entirely new contract for a higher price the new contract is supported by consideration There is no evidence that such a rescission followed by a new contract took place here]

Point Two (45) The business can recover the additional $40000 for the new organ because no consideration is required under Article 2 of the UCC for good-faith contract modifications

The contract to buy a new organ is a contract for the sale of goods and therefore is governed by Article 2 of the Uniform Commercial Code UCC sect 2-102 Under Article 2 unlike the common law an agreement modifying a contract needs no consideration to be binding UCC sect 2-209(1) Section 2-209(1) thus obviates the preexisting duty rule entirely in contracts for the sale of goods

Even though consideration is not required modifications governed by sect 2-209 must satisfy the obligation of good faith imposed by the UCC UCC sect 1-304 See also Official Comment 2 to UCC sect 2-209 Good faith means ldquohonesty in fact and the observance of reasonable commercial standards of fair dealingrdquo UCC sect 1-201(b)(20) In this context the obligation of good faith means that ldquo[t]he effective use of bad faith to escape performance on the original contract terms is barred and the extortion of a lsquomodificationrsquo without legitimate commercial reason is ineffective as a violation of the duty of good faithrdquo Official Comment 2 to

17

Contracts Analysis

UCC sect 2-209 Here because the businessrsquos financial reversals were serious and apparently unanticipated at the time that the business entered into the contract with the conservatory and commitment of the extra money was needed to enable the business to perform a court would likely find that the business acted in good faith Thus a court would likely uphold the enforceability of the conservatoryrsquos promise to pay the additional $40000

Point Three (10) The conservatory is unlikely to be able to defend against enforcement of its promises to pay additional money under the theory of economic duress because the business probably did not make an improper threat

Under the common law of contracts parties may raise the defense of duress This common law defense also applies to contracts governed by UCC Article 2 See UCC sect 1-103(b)

A contract is voidable on the ground of economic duress by threat when it is established that a partyrsquos manifestation of assent is induced by an improper threat that leaves the party no reasonable alternative See RESTATEMENT (SECOND) OF CONTRACTS sect 175 See also eg Austin Instrument Inc v Loral Corp 272 NE2d 533 (NY 1971) (a threat to withhold essential goods can constitute duress) In order to void its agreement to pay the additional sum because of economic duress the conservatory must demonstrate that (1) the business made a threat to the conservatory (2) the threat was ldquoimproperrdquo or ldquowrongfulrdquo (3) the threat induced the conservatoryrsquos manifestation of assent to the modification and (4) the threat was sufficiently grave to justify the conservatoryrsquos assent

Here it appears that three of the four elements are likely satisfied The business plainly made a threat Moreover the threat induced the conservatoryrsquos assent to the modification and the threat was sufficiently grave to justify that assent If the conservatory had not agreed to pay the business the extra amounts the conservatory would have lost its entire $325000 investment In light of this potential loss a court could easily conclude that the conservatory had no reasonable alternative

However the business has a strong argument that its threat (indicating that it would breach the contracts unless the prices were increased) was not wrongful or improper but was instead nothing more than a communication of the reality of its own perilous situation to the conservatory

A mere threat to breach a contract is not in and of itself improper so as to support an action of economic duress or business compulsion Something more is required such as a breach of the duty of good faith and fair dealing as was present in Austin Instrument Inc supra Because the business could not perform the original contract without the requested modification the economic duress claim for the conservatory would likely fail for much the same reason that the business would be able to enforce the modification At the time the modification was requested the business was not trying to extort a price increase because of the conservatoryrsquos vulnerability but instead was simply stating the reality that the business could not perform without more money

18

FAMILY LAW ANALYSIS (Family Law IIIB D amp G)

ANALYSIS

Legal Problems

(1)(a) Does the State A court have jurisdiction to modify the State B child support order

(1)(b) Does the State A court have jurisdiction to modify the marital-residence-saleshyproceeds provision of the State B property-division decree

(2)(a) May a child support order be modified retroactively

(2)(b) May a child support order be modified prospectively based on a change of employment with a lower salary

(2)(c) May a property-division order be modified after entry of a divorce decree

DISCUSSION

Summary

The State A court may exercise personal jurisdiction over the wife because she was personally served in State A However subject-matter jurisdiction over the interstate modification of child support is governed by the Uniform Interstate Family Support Act (UIFSA) Under UIFSA State A does not have jurisdiction to modify the order for the daughterrsquos support because the wife is still a resident of State B UIFSA on the other hand does not govern property distributions and thus a State A court is not precluded from hearing the husbandrsquos petition to modify the marital-residence-sale-proceeds provision of the divorce decree

A child support order may not be modified retroactively A child support order may be modified prospectively based on a substantial change in circumstances Courts agree that a significant decrease in income is a substantial change in circumstances All states treat voluntary income reductions differently than involuntary reductions but employ different approaches for evaluating the impact of a voluntary reduction Whether the husband could obtain prospective modification of the child support order depends on which approach is applied

A property-division order is not subject to post-divorce modification based on a change in circumstances Thus the husband may in some states obtain prospective modification of the order for the daughterrsquos support but he may not obtain modification of the marital-residenceshysale-proceeds provision

Point One(a) (25) Personal jurisdiction over a nonresident respondent does not confer subject-matter jurisdiction over child support modification Under UIFSA a State A court may not modify a child support order issued by a State B court when as here the child or either parent continues to reside in State B the jurisdiction that issued the child support order

The State A court may exercise personal jurisdiction over the wife The wife was personally served in State A and a state may exercise jurisdiction based on in-state personal service See

19

Family Law Analysis

Burnham v Superior Court 495 US 604 (1990) But personal jurisdiction over the wife is not enough to give a State A court jurisdiction to modify the State B support order

The interstate enforcement and modification of child support is governed by the Uniform Interstate Family Support Act (UIFSA) which has been adopted by all states Under UIFSA the state that originally issued a child support order (here State B) has continuing exclusive jurisdiction to modify the order if that state remains the residence of the obligee the child or the obligor and all parties do not consent to the jurisdiction of another forum See UIFSA sect 205 See also UIFSA sect 603 (ldquoA tribunal of this State shall recognize and enforce but may not modify a registered order if the issuing tribunal had jurisdictionrdquo) The wife and daughter continue to reside in State B and the wife has not consented to the jurisdiction of another forum Thus a State A court does not have jurisdiction to modify the State B child support order

[NOTE Examinees who do not discuss personal jurisdiction but fully discuss UIFSA may receive full credit]

Point One(b) (15) UIFSA does not apply to disputes over property division Thus the State A court may exercise jurisdiction over the husbandrsquos petition to modify the marital-residence-sale-proceeds provision of the State B divorce decree because it has personal jurisdiction over the wife

The State A court in which the husband brought his action has jurisdiction to adjudicate domestic relations issues The husbandrsquos petition to modify the property settlement is a domestic relations issue The courts of State A may exercise personal jurisdiction over the wife because she was personally served in State A See Burnham v Superior Court 495 US 604 (1990) see Point One(a)

UIFSA does not apply to divorce property-division disputes Thus although a State A court may not adjudicate the husbandrsquos petition to modify his child support obligations it may adjudicate his property-division claims (Even though the court has jurisdiction it may not modify the property-division award on the merits See Point Two(c))

Point Two(a) (20) A child support order may not be modified retroactively

State courts have long held that obligations to pay child support ordinarily may not be modified retroactively ldquoIf the hardship is particularly severe the courts sometimes devised a way to protect the obligor but in most instances the courts hold that retroactive modification of this kind is beyond their power and indeed the governing statute may so providerdquo HOMER H CLARK THE LAW OF DOMESTIC RELATIONSHIPS IN THE UNITED STATES 725 (2d ed 1987)

Federal law now goes further and requires the states as a condition of federal child-support funding to adopt rules that absolutely forbid retroactive modification of the support obligation See 42 USC sect 666(a)(9)(C) The states have adopted rules consistent with the federal requirements

Point Two(b) (25) It is unclear whether the husband could obtain prospective downward modification of his child support based on his voluntary acceptance of a job with a lower salary

Prospective modification of a child support order is typically available only when the petitioner can show a substantial change in circumstances See ROBERT E OLIPHANT amp NANCY VER

20

Family Law Analysis

STEEGH FAMILY LAW 213ndash15 (3d ed 2010) A significant decrease in income is typically viewed as a substantial change

However when a parent seeks to modify a child support obligation because he has voluntarily reduced his income a court will not modify the obligation based solely on the income loss Some courts refuse to modify whenever the income shift was voluntary See eg Aguiar v Aguiar 127 P3d 234 (Idaho Ct App 2005) Others look primarily to the petitionerrsquos intentions and permit downward modification if he has acted in good faith See eg In re Marriage of Horn 650 NE2d 1103 (Ill App Ct 1995) Many courts use a multifactor approach See OLIPHANT amp VER STEEGH supra 217ndash18

Here there is no question that the husbandrsquos loss of income was voluntary In a jurisdiction in which voluntary income reduction bars support modification the husbandrsquos petition would be denied

In a jurisdiction employing a good-faith or multifactor approach it is possible but not certain that the husband could obtain downward modification The evidence supports the husbandrsquos good faith his change in employment appears to be based on his new jobrsquos greater responsibilities and better promotion possibilities In a jurisdiction using a multifactor approach the court would likely also consider the impact of such a shift on the daughter the likely duration of the husbandrsquos income loss and the likelihood of a promotion that would ultimately inure to the daughterrsquos benefit Thus on these facts it is possible but by no means certain that the husband could prospectively obtain downward modification of his child support obligation to his daughter

Point Two(c)(15) A divorce property-division award is not subject to modification

A support order is aimed at meeting the post-divorce needs of the supported individual Because the future is unpredictable courts are empowered to modify a support award to take account of changed circumstances that may occur during the period in which support is paid

By contrast a property-distribution award divides assets of the marriage based on the equities at the time of divorce Because the past can be ascertained a property-division award is not subject to post-divorce modification See HARRY A KRAUSE ET AL FAMILY LAW CASES COMMENTS AND QUESTIONS 691 (6th ed 2007)

Here the husband is seeking modification of a property-division award with respect to an asset owned by the parties at the time of divorce Thus the husband may not obtain a modification of the marital-residence-sale-proceeds provision of the divorce decree based on his reduced income

21

FEDERAL CIVIL PROCEDURE ANALYSIS (Federal Civil Procedure III IVC)

ANALYSIS

Legal Problems

(1) Is the logging company entitled to join this action as a matter of right

(2)(a) May the nonprofit organization obtain a temporary restraining order to stop the USFS from issuing a logging permit

(2)(b) May the nonprofit organization obtain a preliminary injunction to stop the USFS from issuing a logging permit during the pendency of the action

DISCUSSION

Summary

The logging company is entitled to intervene in this action as a matter of right because it has an interest in the property or transaction that is the subject of the action and is so situated that its interest may be impaired or impeded as a practical matter if the action goes forward without it The logging companyrsquos interest is not adequately represented by the USFSrsquos presence in the lawsuit

The nonprofit organization may seek a temporary restraining order (TRO) followed by a preliminary injunction to prevent the USFS from issuing a logging permit pending the outcome of the action The nonprofit is likely to obtain a TRO if it can demonstrate a risk of immediate and irreparable injury The nonprofit is also likely to obtain a preliminary injunction if it can demonstrate a significant threat of irreparable harm and a likelihood of success on the merits of its National Environmental Policy Act (NEPA) claim

Point One (50) Rule 24(a) of the Federal Rules of Civil Procedure requires federal courts to allow a person to intervene in an action as a matter of right if the person a) is interested in the property or transaction that is the subject of the action b) is so situated that its interest may be impaired or impeded if the litigation goes forward without it and c) is not adequately represented by existing parties Here the logging company likely meets all three requirements and should be allowed to intervene as a matter of right

Rule 24 of the Federal Rules of Civil Procedure governs intervention the process by which a non-party to an action may join the litigation Under Rule 24(a) (intervention of right) a person must be permitted to intervene if three conditions are met (1) the movant ldquoclaims an interest relating to the property or transaction that is the subject of the actionrdquo (2) the movant ldquois so situated that disposition of the action may as a practical matter impair or impede the movantrsquos ability to protect its interestrdquo and (3) ldquoexisting partiesrdquo do not ldquoadequately represent [the movantrsquos] interestrdquo FED R CIV P 24(a) The three requirements for intervention of right are often ldquovery interrelatedrdquo 7C CHARLES ALAN WRIGHT ET AL FEDERAL PRACTICE AND PROCEDURE sect 1908 at 297 (2007 amp 2011 Supp)

22

Federal Civil Procedure Analysis

Here the court should find that the logging company meets this test First the logging company has a strong interest in the property or transaction that is the subject of this action The USFS has accepted the logging companyrsquos bid and the logging company is merely awaiting issuance of a logging permit to begin logging The nonprofit organization is seeking to prevent this logging The logging company therefore has a strong direct and substantial interest in the subject matter of the lawsuit and in having its winning bid honored and a logging permit issued See eg Kleissler v US Forest Serv 157 F3d 964 972 (3d Cir 1998) (stating that ldquo[t]imber companies have direct and substantial interests in a lawsuit aimed at halting loggingrdquo) see also Natural Resources Defense Council v US Nuclear Regulatory Commrsquon 578 F2d 1341 1343ndash 44 (10th Cir 1978) (holding that applicants whose license renewals were pending had Rule 24(a)(2) interests where the lawsuit sought to halt the license-issuing process pending preparation of environmental impact statements) See generally 7C WRIGHT ET AL supra sect 19081 at 309 (ldquoIf there is a direct substantial legally protectable interest in the proceedings it is clear that this requirement of the rule is satisfiedrdquo) Second the logging companyrsquos interest in receiving a logging permit may well be impaired as a practical matter by the outcome of the lawsuit If the USFS loses the lawsuit it will have to prepare an environmental impact statement before issuing the logging companyrsquos permit This will at a minimum delay the logging companyrsquos ability to exercise its rights and may in the long r un mean that no logging permit is ever issued Intervention of right is not limited to those that would be legally bound as a matter of preclusion doctrine Id sect 19082 at 368 Rather ldquo[t]he rule is satisfied whenever disposition of the present action would put the movant at a practical disadvantage in protecting its interestrdquo Id sect 19082 at 369 Here that condition is easily satisfied See Kleissler 157 F3d at 972 (ldquoTimber companies have direct and substantial interests in a lawsuit aimed at halting logging rdquo)

Given that the logging company has an interest that may be impaired by disposition of the action it should be allowed to intervene unless the court is persuaded that the USFS adequately represents the logging companyrsquos interest See Rule 24(a)(2) 7C WRIGHT ET AL supra sect 1909 Here it could be argued that the USFS adequately represents the logging companyrsquos interest because the USFS presumably wants the court to uphold its development plan and allow it to proceed with issuance of the logging permit which is the same relief that the logging company would seek However whether representation is truly adequate depends upon ldquo[a] discriminating appraisal of the circumstancesrdquo 7C WRIGHT ET AL supra sect 1909 at 440 Although both the government and the logging company wish to avoid the preparation of an environmental impact statement their interests are distinct The USFSrsquos interest is proper management of the national forest system while the logging companyrsquos interest is making a profit from logging the 5000-acre tract The USFSrsquos handling of the litigation is likely to be affected by a variety of policy concerns and political considerations that have nothing to do with the logging companyrsquos purely economic interest in securing the right to cut trees in the Scenic National Forest See eg Kleissler 157 F3d at 973ndash74 (ldquo[T]he government represents numerous complex and conflicting interests in matters of this nature The straightforward business interests asserted by intervenors here may become lost in the thicket of sometimes inconsistent governmental policiesrdquo)

[NOTES (1) Examinees who mistakenly analyze the logging companyrsquos case for joinder under the related but incorrect Rule 19 ldquoRequired Joinder of Partiesrdquo may receive credit Rule 19 allows existing parties to demand joinder of non-parties (or seek dismissal of the case if they canrsquot get it) There is a close relationship between Rule 24 and Rule 19 and both contain a similar standard for determining when ldquointerestedrdquo third parties are ldquoentitledrdquo or ldquorequiredrdquo to be in the lawsuit Indeed the two prongs of the Rule 24 intervention test that are discussed above

23

Federal Civil Procedure Analysis

are nearly identical to the two prongs of the Rule 19(a) required joinder test Examinees who discuss and apply the test should receive credit even if they cite Rule 19 rather than Rule 24

(2) Examinees may discuss permissive joinder Although permissive joinder is a possibility here the question asks only whether the logging company can join the action as a matter of right and a permissive joinder analysis is not responsive to the question To the extent an examinee discusses permissive joinder the analysis will focus on whether the logging company ldquohas a claim or defense that shares with the main action a common question of law or factrdquo FED R CIV P 24(b)(1)(B) The district court also ldquomust consider whether the intervention will unduly delay or prejudice the adjudication of the original partiesrsquo rightsrdquo FED R CIV P 24(b)(3) On our facts the logging companyrsquos claim for the issuance of a logging permit would certainly share common questions of law and fact with the USFSrsquos defense against the nonprofitrsquos claim There are no facts suggesting that the logging companyrsquos presence would unduly delay or otherwise prejudice adjudication of the original action Thus the district court would have discretion to permit the logging company to intervene even if it denied intervention of right]

Point Two(a) (25) The nonprofit organization could seek and would likely obtain a temporary restraining order to stop the USFS from issuing a logging permit pending a hearing on an application for a preliminary injunction

The first type of interim relief the nonprofit could seek to stop the USFS from issuing a logging permit to the logging company is a temporary restraining order (TRO) prohibiting the USFS from issuing the logging permit A TRO can be issued without notice to the adverse party but only in limited circumstances and only for a limited time FED R CIV P 65(b) To secure a TRO without notice the nonprofit would need to submit an affidavit containing specific facts that demonstrate a risk of ldquoimmediate and irreparable injuryrdquo if a permit is issued FED R CIV P 65(b)(1) In deciding whether to grant a TRO courts will also consider the same factors that are relevant in deciding whether to grant a preliminary injunction (eg the moving partyrsquos likelihood of success on the merits the balance of hardships and the public interest) See Point Two(b) infra The TRO would last only long enough for the court to consider and resolve a request by the nonprofit for a preliminary injunction but no longer than 14 days (unless the court extends it for good cause or the adverse party consents to an extension) In addition bond is required

Here the court is likely to grant the nonprofitrsquos request The nonprofit could plausibly claim that cutting down 5000 acres of old-growth forest in an area that is home to the highest concentration of wildlife in the western United States would have ldquoan immediate and irreparablerdquo adverse impact on the environment and cause irreparable harm to the nonprofitrsquos interest in preserving and protecting natural resources including wildlife habitat

Point Two(b) (25) The nonprofit could also seek and would likely obtain a preliminary injunction to stop the USFS which is likely to be granted if the nonprofitrsquos claim that the USFS violated NEPA has a strong basis in fact and law

Because the TRO would be temporary the nonprofit would need to move for a preliminary injunction to prevent the USFS from issuing a logging permit throughout the pendency of the litigation Preliminary injunctions are injunctions that seek to ldquoprotect [the] plaintiff from

24

Federal Civil Procedure Analysis

irreparable injury and to preserve the courtrsquos power to render a meaningful decision after a trial on the meritsrdquo 11A CHARLES ALAN WRIGHT ET AL FEDERAL PRACTICE AND PROCEDURE sect 2947 at 112 (2013) Rule 65 of the Federal Rules of Civil Procedure sets out the procedural requirements for preliminary injunctions Preliminary injunctions may be granted only upon notice to the adverse party FED R CIV P 65(a)(1) and only if the movant ldquogives security in an amount that the court considers proper to pay the costs and damages sustained by any party found to have been wrongfully enjoined or restrainedrdquo FED R CIV P 65(c)

While Rule 65 sets out the procedural requirements for preliminary injunctive relief it does not specify the substantive grounds upon which it may be granted The courtrsquos discretion in ruling upon a motion for a preliminary injunction ldquois exercised in conformity with historic federal equity practicerdquo 11A WRIGHT ET AL supra sect 2947 at 114 The court typically considers four factors

(1) the significance of the threat of irreparable harm to the plaintiff if the injunction is not granted (2) the balance between this harm and the injury that granting the injunction would inflict on the defendant (3) the probability that the plaintiff will succeed on the merits and (4) the public interest

Id sect 2948 at 122ndash24 accord Habitat Educ Center v Bosworth 363 F Supp 2d 1070 1088 (ED Wis 2005) The most important of these factors is the risk of irreparable harm to the plaintiff 11A WRIGHT ET AL supra sect 29481 at 129 If the plaintiff has an adequate remedy at law (eg if money damages can compensate the plaintiff for its loss) then a preliminary injunction will be denied Id sect 29481

Here a court would likely conclude that the potential for environmental damage to the forest creates a significant threat of irreparable harm ldquo[E]nvironmental injury is often irreparable Courts have recognized that logging such as would occur [here] can have longshyterm environmental consequences and thus satisfy the irreparable injury criterionrdquo Habitat Educ Center 363 F Supp 2d at 1089 (citing Idaho Sporting Congress Inc v Alexander 222 F3d 562 569 (9th Cir 2000) (noting that the imminent and continuing logging activities presented ldquoevidence of environmental harm sufficient to tip the balance in favor of injunctive reliefrdquo)) Neighbors of Cuddy Mountain v US Forest Service 137 F3d 1372 1382 (9th Cir 1998) (stating that ldquo[t]he old growth forests plaintiffs seek to protect would if cut take hundreds of years to reproducerdquo) (internal citation omitted)) see also 11C WRIGHT ET AL supra sect 29481 at 151 (noting that ldquoa preliminary injunction has been issued to prevent harm to the environmentrdquo)

The second factor the balance between the harm to the plaintiff and the harm the defendant will suffer if the injunction is issued also appears to support issuance of a preliminary injunction here The USFS will have to wait before it can develop the Scenic National Forest and the logging company may lose money if the delay is prolonged These economic harms could be compensated monetarily if an injunction is issued inappropriately Where ldquoan injunction bond can compensate [the] defendant for any harm the injunction is likely to inflict the balance should be struck in favor of [the] plaintiffrdquo Id sect 29482 at 192 See also Habitat Educ Center 363 F Supp 2d at 1089 (stating that ldquothe relative absence of harmful effects on the Forest Service weighs in favor of granting the injunctionrdquo)

The third factor is the likelihood that the plaintiff will prevail on the merits Although there is limited information concerning the merits of the action the nonprofit alleges that the federal statute (NEPA) requires an environmental impact statement and further states that the USFS created no environmental impact analysis or statement at all Assuming that those

25

Federal Civil Procedure Analysis

allegations are correct it seems plausible to conclude that the nonprofit will be able to show a likelihood of success on the merits

Finally courts deciding whether or not to issue preliminary injunctive relief are to consider the public interest ldquoFocusing on this factor is another way of inquiring whether there are policy considerations that bear on whether the order should issuerdquo 11C WRIGHT ET AL supra sect 29484 at 214 If the court concludes that the nonprofit is likely to succeed on its NEPA claim because the USFS wrongfully failed to conduct an environmental impact assessment it is likely to find that the public interest would be served by restraining the USFS from proceeding with logging in a national forest See Heartwood Inc v US Forest Service 73 F Supp 2d 962 979 (SD Ill 1999) affrsquod on other grounds 230 F3d 947 (7th Cir 2000) (ldquoviolations by federal agencies of NEPArsquos provisions as established by Congress harm the public as well as the environmentrdquo)

Thus a court is very likely to grant a preliminary injunction if it concludes that the nonprofit has a significant likelihood of success on the merits

26

EVIDENCE ANALYSIS (Evidence ID IIA amp C)

ANALYSIS

Legal Problems

(1) Under what circumstances can evidence of prior convictions be used to impeach a witnessrsquos credibility in a civil case

(1)(a) May the inmatersquos credibility be impeached by evidence of a 12-year-old felony drug conviction if he was released from prison 9 years ago

(1)(b) May the inmatersquos credibility be impeached by evidence of an 8-year-old misdemeanor perjury conviction that was punishable by 1 year in jail if he pleaded guilty and was sentenced only to pay a $5000 fine

(1)(c) May the inmatersquos credibility be impeached by evidence of a 7-year-old sexual assault conviction if the inmate is still serving a 10-year prison sentence and the victim was his 13-year-old daughter

(2)(a) May the guardrsquos credibility be impeached by cross-examination regarding specific instances of misconduct (ie lying on his reacutesumeacute) relevant to credibility

(2)(b) May the guardrsquos credibility be impeached by admission of extrinsic evidence (his reacutesumeacute and academic transcript) offered to prove specific instances of misconduct relevant to credibility

DISCUSSION

Summary

Under the Federal Rules of Evidence witnesses can be impeached with evidence of prior convictions andor specific instances of misconduct Whether evidence of prior convictions should be admitted to impeach generally depends on the nature of the crime the amount of time that has passed and (only in criminal cases) whether the ldquowitnessrdquo is the defendant FED R EVID 609(a)

In this civil case evidence of the inmatersquos conviction for distribution of marijuana should be admitted to impeach the inmate because he was convicted of a felony and was released from prison fewer than 10 years ago FED R EVID 609(a)(1) Credibility is critically important in this case because the jury will hear conflicting testimony from the two disputing parties and there were no other eyewitnesses to the altercation Under Rule 609(a)(1) the inmatersquos conviction should be admitted because it has some bearing on his credibility and its probative value is not substantially outweighed by concerns of unfair prejudice confusion or delay Id

Evidence of the inmatersquos misdemeanor conviction for perjury must be admitted because the crime ldquorequired provingmdashor the witnessrsquos admittingmdasha dishonest act or false statementrdquo by the inmate FED R EVID 609(a)(2)

27

Evidence Analysis

Evidence of the inmatersquos felony conviction for sexual assault should be excluded because its probative value is substantially outweighed by the danger of unfair prejudice to the inmate based on the heinous nature of the crime FED R EVID 609(a)(1) In the alternative the judge could limit the evidence relating to this conviction by excluding details of the inmatersquos crime

In all civil (and criminal) cases witnesses can also be impeached with evidence of specific instances of prior misconduct that did not result in a conviction FED R EVID 608(b) Pursuant to Rule 608(b) misconduct probative of untruthfulness can be inquired into on cross-examination but cannot be proved through extrinsic evidence Id Thus the inmatersquos counsel should be permitted to cross-examine the guard regarding the false statement in the guardrsquos reacutesumeacute However extrinsic evidence of the guardrsquos misconduct (ie the guardrsquos authenticated reacutesumeacute and transcript from the local college) should not be admitted even if the guard denies wrongdoing or refuses to answer cross-examination questions about these matters Id

Point One (10) The Federal Rules of Evidence permit impeachment of witnesses with evidence of prior convictions

Whether convictions should be admitted to impeach generally depends on the nature of the crime the amount of time that has passed and (only in criminal cases) whether the ldquowitnessrdquo is the defendant FED R EVID 609(a) Under Rule 609(a) evidence of prior convictions may be admitted for the purpose of ldquoattacking a witnessrsquos character for truthfulnessrdquo Id

There are two basic types of convictions that can be admitted for the purpose of impeachment

(1) convictions for crimes ldquopunishable by death or by imprisonment for more than one yearrdquo (which generally correlates to ldquofeloniesrdquo) FED R EVID 609(a)(1) and (2) convictions ldquofor any crimes regardless of the punishment if the court can readily determine that establishing the elements of the crime required provingmdashor the witnessrsquos admittingmdasha dishonest act or false statementrdquo FED R EVID 609(a)(2)

Pursuant to Rule 609(a)(1) in civil cases the admission of evidence of a felony conviction is ldquosubject to Rule 403 [which says that a court may exclude relevant evidence if its probative value is substantially outweighed by other factors]rdquo FED R EVID 609(a)(1) However Rule 403 does not protect the witness against admission of prior convictions involving dishonestymdashwhich must be admitted by the court FED R EVID 609(a)(2)

Finally Federal Rule of Evidence 609(b) contains the presumption that a conviction that is more than 10 years old or where more than 10 years has passed since the witnessrsquos release from confinement (whichever is later) should not be admitted unless ldquoits probative value supported by specific facts and circumstances substantially outweighs its prejudicial effectrdquo and the proponent has provided the adverse party with reasonable written notice FED R EVID 609(b)

Point One(a) (25) The court should admit evidence of the inmatersquos 12-year-old felony marijuana distribution conviction

The inmatersquos conviction for marijuana distribution was for a felony punishable by imprisonment for more than one year See FED R EVID 609(a)(1) Moreover although the conviction was 12 years ago the 10-year time limit of Rule 609(b) is not exceeded because that time limit runs

28

Evidence Analysis

from the date of either ldquothe witnessrsquos conviction or release from confinement for it whichever is laterrdquo FED R EVID 609(b) Because the inmate served three years in prison he was released from confinement nine years ago

However pursuant to Rule 609(a)(1) the admission of felony convictions to impeach a witness in a civil case is ldquosubject to Rule 403rdquo FED R EVID 609(a)(1) Neither Rule 609(a) nor the advisory committee notes specify which factors courts should consider when balancing the probative value of a conviction against the dangers identified in Rule 403 (which include (1) unfair prejudice (2) confusion of the issues (3) misleading the jury (4) waste of time or undue delay and (5) needless presentation of cumulative evidence) FED R EVID 403

In this case credibility is very important because the evidence consists primarily of the testimony of the disputing parties and there were no other eyewitnesses to the altercation This enhances the probative value of any evidence bearing on the inmatersquos credibility A court is likely to conclude that the inmatersquos prior felony drug conviction is relevant to his credibility See eg United States v Brito 427 F3d 53 64 (1st Cir 2005) (ldquoPrior drug-trafficking crimes are generally viewed as having some bearing on veracityrdquo) Although the probative value of any conviction diminishes with age see eg United States v Brewer 451 F Supp 50 53 (ED Tenn 1978) the inmatersquos ongoing problems with the law suggest that he has continued (and even escalated) his criminal behavior over the past nine years The court should admit this evidence because its probative value is not substantially outweighed by any Rule 403 concerns Specifically any prejudice to the inmate would be slight because the conviction is unrelated to the altercation at issue and the conviction was not for a heinous crime that might inflame the jury

[NOTE Whether an examinee identifies the jury instruction as containing a ldquoconclusiverdquo or ldquomandatoryrdquo presumption is less important than the examineersquos analysis of the constitutional infirmities]

Point One(b) (15) The court must admit evidence of the inmatersquos eight-year-old misdemeanor conviction because perjury is a crime of dishonesty

Rule 609(a)(2) provides that evidence of a criminal conviction ldquomust be admitted if the court can readily determine that establishing the elements of the crime required provingmdashor the witnessrsquos admittingmdasha dishonest act or false statementrdquo FED R EVID 609(a)(2) The inmatersquos conviction for perjury would have necessarily required proving that the inmate engaged in an act of dishonesty This conviction occurred within the past 10 years so it ldquomust be admittedrdquo because in contrast to Rule 609(a)(1) (discussed in Point One(a)) admission under Rule 609(a)(2) is mandatory and not subject to Rule 403

Point One(c) (20) The court should exclude evidence of the inmatersquos seven-year-old felony sexual assault conviction because the probative value of this evidence is substantially outweighed by the danger of unfair prejudice In the alternative the details of the prior conviction could be excluded

The inmatersquos conviction for felony sexual assault was seven years ago and he has not yet been released from incarceration so Rule 609(a) but not 609(b) is applicable here FED R EVID 609(a) This conviction is therefore admissible to impeach the inmate unless its probative value is substantially outweighed by the danger of unfair prejudice or any other Rule 403 concern Id

29

Evidence Analysis

Sex crimes are generally not considered relevant to credibility see Hopkins v State 639 So 2d 1247 1254 (Miss 1993) so the probative value of this conviction is relatively low Moreover the heinous nature of the inmatersquos crime (sexual assault on his daughter) makes the danger of unfair prejudice to the inmate very high Thus the court should exclude evidence of the conviction because it was for a heinous offense that is likely to inflame the jury and it has little bearing on credibility See eg United States v Beahm 664 F2d 414 419 (4th Cir 1981)

As an alternative to excluding this evidence the judge could minimize the unfair prejudice to the inmate by permitting limited cross-examination but refusing to allow specific questions about the nature of the inmatersquos conviction For example a court could limit cross-examination to the fact that the inmate was convicted of a ldquofelonyrdquo or perhaps that he was convicted of a ldquosexual assaultrdquo without identifying the victim However because evidence of the inmatersquos prior convictions can be admitted solely for the purpose of enabling the jury to assess his credibility and because his two earlier convictions should have already been admitted the court should exclude all evidence of the felony sexual assault conviction

Point Two(a) (15) The court should permit the inmatersquos counsel to cross-examine the guard regarding the false statement in his reacutesumeacute because the guardrsquos misconduct bears on his truthfulness

The inmate wishes to cross-examine the guard about his prior dishonest behaviormdashlying on his reacutesumeacutemdashthat did not involve a criminal conviction Rule 608(b) allows witnesses to be cross-examined about specific instances of prior non-conviction misconduct probative of untruthfulness ldquoin order to attack the witnessrsquos character for truthfulnessrdquo FED R EVID 608(b)

The courtrsquos decision to allow cross-examination about the guardrsquos prior dishonest behavior depends on the probative value of such evidence balanced against the danger of unfair prejudice to the guard or any other Rule 403 concern FED R EVID 403 Here the guardrsquos false statement on his reacutesumeacute that he obtained a degree in Criminal Justice is highly probative of his untruthfulness because it grossly misrepresents his actual academic record was made recently and was made with the intent to deceive Because the probative value of this evidence is very strong and is not substantially outweighed by any Rule 403 concerns cross-examination of the guard on this topic should be permitted The court may also consider it fair to permit this cross-examination of the guard on these matters assuming that one or more of the inmatersquos prior convictions have been admitted to impeach his credibility

Point Two(b) (15) The court should exclude extrinsic evidence of the guardrsquos non-conviction misconduct even if the guard denies wrongdoing or refuses to answer questions about the matter

Although Rule 608(b) allows cross-examination about specific instances of prior misconduct probative of untruthfulness ldquoextrinsic evidencerdquo offered to prove such misconduct is not admissible FED R EVID 608(b) The rationale for this rule is that allowing the introduction of extrinsic evidence of prior misconduct by witnesses when these acts are relevant only to the witnessesrsquo truthfulness and not to the main issues in the case would create too great a risk of confusing the jury and unduly delaying the trial The court does not have discretion to admit this extrinsic evidence See eg United States v Elliot 89 F3d 1360 1368 (8th Cir 1996)

30

Evidence Analysis

Here the inmatersquos counsel may cross-examine the guard about the false statement on his reacutesumeacute However the inmatersquos counsel must accept the guardrsquos response Even if the guard denies wrongdoing or refuses to answer questions about the matter the inmatersquos counsel cannot introduce the guardrsquos reacutesumeacute or the transcript from the local college to prove the guardrsquos misconduct

31

CORPORATIONS ANALYSIS (Corporations VA2 IX)

ANALYSIS

Legal Problems

(1) Do shareholders have the authority to amend a corporationrsquos bylaws with respect to director nominations

(2) Do board-approved bylaws on a particular subject here nomination of directors preempt subsequent conflicting bylaw amendments by shareholders

(3) Is a suit challenging both managementrsquos refusal to include the proposed bylaw amendment in Megarsquos proxy statement and the boardrsquos amendment of the bylaws dealing with nomination of directors a direct or derivative suit

DISCUSSION

Summary

The voting and litigation rights of the shareholders of Mega are subject to the provisions of the Model Business Corporations Act (MBCA)

The investorrsquos proposed bylaw provision is not inconsistent with state law Under the MBCA shareholders may amend the bylaws when the amendment deals with a proper matter for the corporationrsquos bylaws such as procedures for nominating directors

The Mega boardrsquos bylaw amendment does not preempt the investorrsquos proposed bylaw provision or the Mega shareholdersrsquo power to approve it While shareholders can limit the boardrsquos power to amend or repeal the bylaws the board cannot limit the shareholdersrsquo power

Whether the investor must make a demand on Megarsquos board depends on how the investor frames its claim If the investor claims a violation of shareholder voting rights the claim is direct and pre-suit demand on the board is not required If on the other hand the investor claims that the directors violated their fiduciary duties by amending the bylaws to entrench themselves the claim is derivative and a pre-suit demand is required

Point One (30) Shareholders may amend the corporationrsquos bylaws where the proposed bylaw provision relates to procedural matters typically included in the bylaws such as the nomination of directors

Internal affairs of the corporation such as the conduct of shareholder meetings and election of directors are subject to the corporate law of the state of incorporation See McDermott Inc v Lewis 531 A2d 206 (Del 1987) (applying law of jurisdiction where corporation was incorporated in case involving voting rights) This statersquos corporate statute is modeled on the MBCA

Under the MBCA ldquoshareholders may amend the corporationrsquos bylawsrdquo MBCA sect 1020(a) Thus the only question is whether the bylaws can specify the procedures for shareholder nomination of directors

32

Corporations Analysis

The MBCA states that the bylaws ldquomay contain any provision that is not inconsistent with law or the articles of incorporationrdquo MBCA sect 206(b) In addition the MBCA was revised in 2009 to address shareholder nomination of directors in public corporations (known as ldquoproxy accessrdquo) and specifies that the bylaws ldquomay contain a requirement that the corporation include in its [proxy materials] one or more individuals nominated by a shareholderrdquo MBCA sect 206(c)(1) see Committee on Corporate Laws ABA Section of Business Law Report on the Roles of Boards of Directors and Shareholders of Publicly Owned Corporations and Changes to the Model Business Corporations ActmdashAdoption of Shareholder Proxy Access Amendments to Chapters 2 and 10 65 BUS LAWYER 1105 (2010)

The inclusion of director-nomination procedures in the bylaws is consistent with practice and is recognized by the Delaware courts whose views on corporate law carry significant weight Typically the procedures for nomination of directors are found in the bylaws See 1 COX amp HAZEN TREATISE ON THE LAW OF CORPORATIONS sect 312 (3d ed 2011) see also 4 FLETCHER CORP FORMS ANN PART III ch 21 (2013) (including sample bylaws that permit nomination of directors by shareholders) The Delaware Supreme Court has confirmed that the bylaws may ldquodefine the process and proceduresrdquo for director elections See CA Inc v AFSCME Employees Pension Plan 953 A2d 227 (Del 2008) (concluding that bylaw amendment requiring reimbursement of election expenses to certain successful shareholder nominators is ldquoproper subjectrdquo under Delaware law)

[NOTE The question of the proper scope of the bylaws can be answered using the more general MBCA sect 206(b) or the 2009 MBCA revision adding sect 206(c)(1) (adopted in CT ME VA) In addition some examinees might raise the point that shareholder proposals may not compel the board to take action such as by including shareholder nominations in the companyrsquos proxy materials on the theory that the ldquobusiness and affairsrdquo of the corporation are to be managed by the board See MBCA sect 801(b) Although shareholders are generally limited to adopting precatory resolutions that recommend or encourage board action this limitation does not apply when shareholders have specific authority to take binding action on their ownmdashsuch as to amend the bylaws]

Point Two (30) Shareholders can amend (or repeal) board-approved bylaws Further shareholders can limit the boardrsquos power to later amend and repeal a shareholder-approved bylaw

Under the MBCA shareholders have the power to amend the bylaws See Point One The board shares this power with the shareholders unless (1) the corporationrsquos articles ldquoreserve that power exclusively to the shareholdersrdquo or (2) ldquothe shareholders in amending repealing or adopting a bylaw expressly provide that the board of directors may not amend repeal or reinstate that bylawrdquo See MBCA sect 1020(b)

Shareholder-approved bylaw provisions can amend or repeal existing bylaw provisions whether originally approved by the board or by shareholders See ALAN R PALMITER CORPORATIONS EXAMPLES AND EXPLANATIONS sect 713 (7th ed 2012) Thus the Mega boardrsquos bylaw amendmentmdashwhich set more demanding thresholds for shareholder nomination of directors than the investorrsquos proposed bylaw provisionmdashwould be superseded (repealed) if Megarsquos shareholders were to approve the investorrsquos proposal

Further a shareholder-approved bylaw generally can limit the power of the board to later amend or repeal it See MBCA sect 1020(b)(2) Thus if Megarsquos shareholders approved the bylaw

33

Corporations Analysis

provision proposed by the investor Megarsquos board could not repeal the provision because it includes a ldquono board repealrdquo clause

The revision to the MBCA in 2009 dealing with shareholder proxy access does not change this conclusion That revision specifies that a shareholder-approved bylaw dealing with director nominations may not limit the boardrsquos power to amend add or repeal ldquoany procedure or condition to such a bylaw in order to provide for a reasonable practicable and orderly processrdquo MBCA sect 206(d) Thus according to the revision if shareholders approve a bylaw amendment that limits further board changes the board would nonetheless retain the power to ldquotinkerrdquo with the bylaw to safeguard the voting process but could not repeal the shareholder-approved bylaw The Official Comment to MBCA sect 206(d) makes clear that the revision is ldquonot intended to allow the board of directors to frustrate the purpose of the shareholder-adopted proxy access provisionrdquo Thus if Megarsquos shareholders were to approve the bylaw provision proposed by the investor Megarsquos board could only amend the provision regarding its procedures or conditions in a manner consistent with its purpose of permitting proxy access for Megarsquos shareholders

[NOTE The boardrsquos attempted interference with a shareholder voting initiative may also have been a violation of the directorsrsquo fiduciary duties See Blasius Indus Inc v Atlas Corp 564 A2d 651 (Del Ch 1988) (finding that directors breached their fiduciary duties by amending bylaws and expanding size of board to thwart insurgentrsquos plan to amend bylaws and seat a majority of new directors) The call however asks examinees to consider whether shareholders or the board have ldquoprecedencerdquo over amending the corporate bylaws Thus an examineersquos answer should be framed in terms of ldquopowerrdquo and not ldquodutyrdquo]

Point Three (40) The investor need not make a demand on the board if the investor states a direct claim such as an allegation that the board interfered with the investorrsquos right to amend the bylaws But the investor must make a demand on the board if the investor states a derivative claim (on behalf of the corporation) such as an allegation that the directors sought to entrench themselves by interfering with the proposed proxy access

The MBCA generally requires that shareholders make a demand on the board of directors before initiation of a derivative suit MBCA sect 742 (shareholder may not bring derivative proceeding until written demand has been made on corporation and 90 days have expired) A derivative suit is essentially two suits in one where the plaintiff-shareholder seeks to bring on behalf of the corporation a claim that vindicates corporate rights usually based on violation of fiduciary duties PALMITER supra sect 1811 (6th ed 2009) The demand permits the board to investigate the situation identified by the shareholder and take suitable action No demand on the board is required however if the shareholder brings a direct suit to vindicate the shareholderrsquos own rights not those of the corporation

Is the suit brought by the investor derivative or direct The MBCA defines a ldquoderivative proceedingrdquo as one brought ldquoin the right of a domestic corporationrdquo MBCA sect 740(1) Thus the answer to how the investorrsquos suit should be characterized turns on what rights the investor seeks to vindicate If the investor frames its claim as one of fiduciary breach by directorsmdashfor example for failing to become adequately informed about voting procedures or for seeking to entrench themselves in office by manipulating the voting structure to avoid a shareholder insurgencymdashthen the suit is ldquoderivativerdquo and the investor must make a demand on the board See MBCA Ch 7 Subch D Introductory Comment (ldquothe derivative suit has historically been the principal method of challenging allegedly illegal action by managementrdquo)

34

Corporations Analysis

If however the investor frames its claim as one to vindicate shareholder rights the suit is direct and no demand is required For many courts the direct-derivative question turns on who is injured and who is to receive the relief sought by the plaintiff-shareholders See Tooley v Donaldson Lufkin amp Jenrette Inc 845 A2d 1031 (Del 2004) (characterizing a merger-delay claim as direct because delay of merger only harmed shareholders not corporation) Thus if the investor claims that managementrsquos refusal to include its proposed bylaw amendment in the corporationrsquos proxy materials violates its shareholder rights to initiate corporate governance reforms the suit will be direct Courts have not questioned the ability of shareholders to bring direct suits challenging board action to exclude their proposed bylaw amendments from the corporationrsquos proxy materials See JANA Master Fund Ltd v CNET Networks Inc 954 A2d 335 (Del Ch 2008) (upholding shareholderrsquos direct challenge to boardrsquos interpretation of advance-notice bylaw) Chesapeake Corp v Shore 771 A2d 293 (Del Ch 2000) (upholding shareholderrsquos direct challenge to actions by board that effectively prevented it from proposing bylaw amendments in contest for control)

Is the way that the investor frames its claim conclusive Courts have permitted shareholder-plaintiffs to challenge a transaction in a direct suit even though the same transaction could also be challenged as a fiduciary breach See Eisenberg v Flying Tiger Line Inc 451 F2d 267 (2d Cir 1971) (permitting direct suit challenging a corporate reorganization as a dilution of shareholder voting power even though reorganization may have involved conflicts of interest and thus constituted a fiduciary breach) Thus the investorrsquos choice to pursue a claim challenging the legality of managementrsquos decision to exclude the investorrsquos proposal from the corporationrsquos proxy materialsmdashrather than a possible breach of fiduciary dutymdashis likely to be respected See 3 COX amp HAZEN supra sect 153 (describing situations in which a claim can be framed as derivative or direct)

[NOTE Some issues under Delaware corporate law regarding pre-suit demand are not relevant here For example whether the Mega directors are independent and disinterested is not relevant to the MBCA requirement of a pre-suit demand As the Official Comment to MBCA sect 742 points out the MBCArsquos requirement of ldquouniversal demandrdquo gives the board ldquothe opportunity to reexamine the act complained of in the light of a potential lawsuit and take corrective actionrdquo even when the directors might be non-independent or have conflicts of interest

Nor is it relevant to the MBCA pre-suit demand requirement that the statutory 90-day waiting period may be onerous The first paragraph of MBCA sect 742 requires a pre-suit demand without exception the second paragraph of the section imposes a 90-day waiting period before a derivative suit may be brought which can be shortened if the board rejects the demand or ldquoirreparable injury to the corporation would result by waiting for the expiration of the 90-day periodrdquo The call as written asks only whether a pre-suit demand should be made and does not ask examinees to address whether the post-demand waiting period should be shortened under the ldquoirreparable injuryrdquo standard]

35

National Conference of Bar Examiners 302 South Bedford Street | Madison WI 53703-3622 Phone 608-280-8550 | Fax 608-280-8552 | TDD 608-661-1275

wwwncbexorg e-mail contactncbexorg

  • Preface
  • Description of the MEE
  • Instructions
  • July 2014 Questions
    • CRIMINAL LAW AND PROCEDURE QUESTION
    • CONTRACTS QUESTION
    • FAMILY LAW QUESTION
    • FEDERAL CIVIL PROCEDURE QUESTION
    • EVIDENCE QUESTION
    • CORPORATIONS QUESTION
      • July 2014 Analyses
        • CRIMINAL LAW AND PROCEDURE ANALYSIS
        • CONTRACTS ANALYSIS
        • FAMILY LAW ANALYSIS
        • FEDERAL CIVIL PROCEDURE ANALYSIS
        • EVIDENCE ANALYSIS
        • CORPORATIONS ANALYSIS
            • ltlt13 ASCII85EncodePages false13 AllowTransparency false13 AutoPositionEPSFiles true13 AutoRotatePages None13 Binding Left13 CalGrayProfile (Dot Gain 20)13 CalRGBProfile (sRGB IEC61966-21)13 CalCMYKProfile (US Web Coated 050SWOP051 v2)13 sRGBProfile (sRGB IEC61966-21)13 CannotEmbedFontPolicy Error13 CompatibilityLevel 1413 CompressObjects Tags13 CompressPages true13 ConvertImagesToIndexed true13 PassThroughJPEGImages true13 CreateJobTicket false13 DefaultRenderingIntent Default13 DetectBlends true13 DetectCurves 0000013 ColorConversionStrategy CMYK13 DoThumbnails false13 EmbedAllFonts true13 EmbedOpenType false13 ParseICCProfilesInComments true13 EmbedJobOptions true13 DSCReportingLevel 013 EmitDSCWarnings false13 EndPage -113 ImageMemory 104857613 LockDistillerParams false13 MaxSubsetPct 10013 Optimize true13 OPM 113 ParseDSCComments true13 ParseDSCCommentsForDocInfo true13 PreserveCopyPage true13 PreserveDICMYKValues true13 PreserveEPSInfo true13 PreserveFlatness true13 PreserveHalftoneInfo false13 PreserveOPIComments true13 PreserveOverprintSettings true13 StartPage 113 SubsetFonts true13 TransferFunctionInfo Apply13 UCRandBGInfo Preserve13 UsePrologue false13 ColorSettingsFile ()13 AlwaysEmbed [ true13 ]13 NeverEmbed [ true13 ]13 AntiAliasColorImages false13 CropColorImages true13 ColorImageMinResolution 30013 ColorImageMinResolutionPolicy OK13 DownsampleColorImages true13 ColorImageDownsampleType Bicubic13 ColorImageResolution 30013 ColorImageDepth -113 ColorImageMinDownsampleDepth 113 ColorImageDownsampleThreshold 15000013 EncodeColorImages true13 ColorImageFilter DCTEncode13 AutoFilterColorImages true13 ColorImageAutoFilterStrategy JPEG13 ColorACSImageDict ltlt13 QFactor 01513 HSamples [1 1 1 1] VSamples [1 1 1 1]13 gtgt13 ColorImageDict ltlt13 QFactor 01513 HSamples [1 1 1 1] VSamples [1 1 1 1]13 gtgt13 JPEG2000ColorACSImageDict ltlt13 TileWidth 25613 TileHeight 25613 Quality 3013 gtgt13 JPEG2000ColorImageDict ltlt13 TileWidth 25613 TileHeight 25613 Quality 3013 gtgt13 AntiAliasGrayImages false13 CropGrayImages true13 GrayImageMinResolution 30013 GrayImageMinResolutionPolicy OK13 DownsampleGrayImages true13 GrayImageDownsampleType Bicubic13 GrayImageResolution 30013 GrayImageDepth -113 GrayImageMinDownsampleDepth 213 GrayImageDownsampleThreshold 15000013 EncodeGrayImages true13 GrayImageFilter DCTEncode13 AutoFilterGrayImages true13 GrayImageAutoFilterStrategy JPEG13 GrayACSImageDict ltlt13 QFactor 01513 HSamples [1 1 1 1] VSamples [1 1 1 1]13 gtgt13 GrayImageDict ltlt13 QFactor 01513 HSamples [1 1 1 1] VSamples [1 1 1 1]13 gtgt13 JPEG2000GrayACSImageDict ltlt13 TileWidth 25613 TileHeight 25613 Quality 3013 gtgt13 JPEG2000GrayImageDict ltlt13 TileWidth 25613 TileHeight 25613 Quality 3013 gtgt13 AntiAliasMonoImages false13 CropMonoImages true13 MonoImageMinResolution 120013 MonoImageMinResolutionPolicy OK13 DownsampleMonoImages true13 MonoImageDownsampleType Bicubic13 MonoImageResolution 120013 MonoImageDepth -113 MonoImageDownsampleThreshold 15000013 EncodeMonoImages true13 MonoImageFilter CCITTFaxEncode13 MonoImageDict ltlt13 K -113 gtgt13 AllowPSXObjects false13 CheckCompliance [13 None13 ]13 PDFX1aCheck false13 PDFX3Check false13 PDFXCompliantPDFOnly false13 PDFXNoTrimBoxError true13 PDFXTrimBoxToMediaBoxOffset [13 00000013 00000013 00000013 00000013 ]13 PDFXSetBleedBoxToMediaBox true13 PDFXBleedBoxToTrimBoxOffset [13 00000013 00000013 00000013 00000013 ]13 PDFXOutputIntentProfile ()13 PDFXOutputConditionIdentifier ()13 PDFXOutputCondition ()13 PDFXRegistryName ()13 PDFXTrapped False1313 CreateJDFFile false13 Description ltlt13 ARA 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 BGR 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 CHS ltFEFF4f7f75288fd94e9b8bbe5b9a521b5efa7684002000410064006f006200650020005000440046002065876863900275284e8e9ad88d2891cf76845370524d53705237300260a853ef4ee54f7f75280020004100630072006f0062006100740020548c002000410064006f00620065002000520065006100640065007200200035002e003000204ee553ca66f49ad87248672c676562535f00521b5efa768400200050004400460020658768633002gt13 CHT ltFEFF4f7f752890194e9b8a2d7f6e5efa7acb7684002000410064006f006200650020005000440046002065874ef69069752865bc9ad854c18cea76845370524d5370523786557406300260a853ef4ee54f7f75280020004100630072006f0062006100740020548c002000410064006f00620065002000520065006100640065007200200035002e003000204ee553ca66f49ad87248672c4f86958b555f5df25efa7acb76840020005000440046002065874ef63002gt13 CZE 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 DAN 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 DEU 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 ESP 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 ETI 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 FRA 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 GRE ltFEFF03a703c103b703c303b903bc03bf03c003bf03b903ae03c303c403b5002003b103c503c403ad03c2002003c403b903c2002003c103c503b803bc03af03c303b503b903c2002003b303b903b1002003bd03b1002003b403b703bc03b903bf03c503c103b303ae03c303b503c403b5002003ad03b303b303c103b103c603b1002000410064006f006200650020005000440046002003c003bf03c5002003b503af03bd03b103b9002003ba03b103c42019002003b503be03bf03c703ae03bd002003ba03b103c403ac03bb03bb03b703bb03b1002003b303b903b1002003c003c103bf002d03b503ba03c403c503c003c903c403b903ba03ad03c2002003b503c103b303b103c303af03b503c2002003c503c803b703bb03ae03c2002003c003bf03b903cc03c403b703c403b103c2002e0020002003a403b10020005000440046002003ad03b303b303c103b103c603b1002003c003bf03c5002003ad03c703b503c403b5002003b403b703bc03b903bf03c503c103b303ae03c303b503b9002003bc03c003bf03c103bf03cd03bd002003bd03b1002003b103bd03bf03b903c703c403bf03cd03bd002003bc03b5002003c403bf0020004100630072006f006200610074002c002003c403bf002000410064006f00620065002000520065006100640065007200200035002e0030002003ba03b103b9002003bc03b503c403b103b303b503bd03ad03c303c403b503c103b503c2002003b503ba03b403cc03c303b503b903c2002egt13 HEB ltFEFF05D405E905EA05DE05E905D5002005D105D405D205D305E805D505EA002005D005DC05D4002005DB05D305D9002005DC05D905E605D505E8002005DE05E105DE05DB05D9002000410064006F006200650020005000440046002005D405DE05D505EA05D005DE05D905DD002005DC05D405D305E405E105EA002005E705D305DD002D05D305E405D505E1002005D005D905DB05D505EA05D905EA002E002005DE05E105DE05DB05D90020005000440046002005E905E005D505E605E805D5002005E005D905EA05E005D905DD002005DC05E405EA05D905D705D4002005D105D005DE05E605E205D505EA0020004100630072006F006200610074002005D5002D00410064006F00620065002000520065006100640065007200200035002E0030002005D505D205E805E105D005D505EA002005DE05EA05E705D305DE05D505EA002005D905D505EA05E8002E05D005DE05D905DD002005DC002D005000440046002F0058002D0033002C002005E205D905D905E005D5002005D105DE05D305E805D905DA002005DC05DE05E905EA05DE05E9002005E905DC0020004100630072006F006200610074002E002005DE05E105DE05DB05D90020005000440046002005E905E005D505E605E805D5002005E005D905EA05E005D905DD002005DC05E405EA05D905D705D4002005D105D005DE05E605E205D505EA0020004100630072006F006200610074002005D5002D00410064006F00620065002000520065006100640065007200200035002E0030002005D505D205E805E105D005D505EA002005DE05EA05E705D305DE05D505EA002005D905D505EA05E8002Egt13 HRV (Za stvaranje Adobe PDF dokumenata najpogodnijih za visokokvalitetni ispis prije tiskanja koristite ove postavke Stvoreni PDF dokumenti mogu se otvoriti Acrobat i Adobe Reader 50 i kasnijim verzijama)13 HUN 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 ITA ltFEFF005500740069006c0069007a007a006100720065002000710075006500730074006500200069006d0070006f007300740061007a0069006f006e00690020007000650072002000630072006500610072006500200064006f00630075006d0065006e00740069002000410064006f00620065002000500044004600200070006900f900200061006400610074007400690020006100200075006e00610020007000720065007300740061006d0070006100200064006900200061006c007400610020007100750061006c0069007400e0002e0020004900200064006f00630075006d0065006e007400690020005000440046002000630072006500610074006900200070006f00730073006f006e006f0020006500730073006500720065002000610070006500720074006900200063006f006e0020004100630072006f00620061007400200065002000410064006f00620065002000520065006100640065007200200035002e003000200065002000760065007200730069006f006e006900200073007500630063006500730073006900760065002egt13 JPN ltFEFF9ad854c18cea306a30d730ea30d730ec30b951fa529b7528002000410064006f0062006500200050004400460020658766f8306e4f5c6210306b4f7f75283057307e305930023053306e8a2d5b9a30674f5c62103055308c305f0020005000440046002030d530a130a430eb306f3001004100630072006f0062006100740020304a30883073002000410064006f00620065002000520065006100640065007200200035002e003000204ee5964d3067958b304f30533068304c3067304d307e305930023053306e8a2d5b9a306b306f30d530a930f330c8306e57cb30818fbc307f304c5fc59808306730593002gt13 KOR ltFEFFc7740020c124c815c7440020c0acc6a9d558c5ec0020ace0d488c9c80020c2dcd5d80020c778c1c4c5d00020ac00c7a50020c801d569d55c002000410064006f0062006500200050004400460020bb38c11cb97c0020c791c131d569b2c8b2e4002e0020c774b807ac8c0020c791c131b41c00200050004400460020bb38c11cb2940020004100630072006f0062006100740020bc0f002000410064006f00620065002000520065006100640065007200200035002e00300020c774c0c1c5d0c11c0020c5f40020c2180020c788c2b5b2c8b2e4002egt13 LTH 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 LVI 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 NLD (Gebruik deze instellingen om Adobe PDF-documenten te maken die zijn geoptimaliseerd voor prepress-afdrukken van hoge kwaliteit De gemaakte PDF-documenten kunnen worden geopend met Acrobat en Adobe Reader 50 en hoger)13 NOR 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 POL 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 PTB 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 RUM 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 RUS ltFEFF04180441043f043e043b044c04370443043904420435002004340430043d043d044b04350020043d0430044104420440043e0439043a043800200434043b044f00200441043e043704340430043d0438044f00200434043e043a0443043c0435043d0442043e0432002000410064006f006200650020005000440046002c0020043c0430043a04410438043c0430043b044c043d043e0020043f043e04340445043e0434044f04490438044500200434043b044f00200432044b0441043e043a043e043a0430044704350441044204320435043d043d043e0433043e00200434043e043f0435044704300442043d043e0433043e00200432044b0432043e04340430002e002000200421043e043704340430043d043d044b04350020005000440046002d0434043e043a0443043c0435043d0442044b0020043c043e0436043d043e0020043e0442043a0440044b043204300442044c002004410020043f043e043c043e0449044c044e0020004100630072006f00620061007400200438002000410064006f00620065002000520065006100640065007200200035002e00300020043800200431043e043b043504350020043f043e04370434043d043804450020043204350440044104380439002egt13 SKY 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 SLV 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 SUO ltFEFF004b00e40079007400e40020006e00e40069007400e4002000610073006500740075006b007300690061002c0020006b0075006e0020006c0075006f00740020006c00e400680069006e006e00e4002000760061006100740069007600610061006e0020007000610069006e006100740075006b00730065006e002000760061006c006d0069007300740065006c00750074007900f6006800f6006e00200073006f00700069007600690061002000410064006f0062006500200050004400460020002d0064006f006b0075006d0065006e007400740065006a0061002e0020004c0075006f0064007500740020005000440046002d0064006f006b0075006d0065006e00740069007400200076006f0069006400610061006e0020006100760061007400610020004100630072006f0062006100740069006c006c00610020006a0061002000410064006f00620065002000520065006100640065007200200035002e0030003a006c006c00610020006a006100200075007500640065006d006d0069006c006c0061002egt13 SVE 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 TUR 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 UKR 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 ENU (Use these settings to create Adobe PDF documents best suited for high-quality prepress printing Created PDF documents can be opened with Acrobat and Adobe Reader 50 and later)13 gtgt13 Namespace [13 (Adobe)13 (Common)13 (10)13 ]13 OtherNamespaces [13 ltlt13 AsReaderSpreads false13 CropImagesToFrames true13 ErrorControl WarnAndContinue13 FlattenerIgnoreSpreadOverrides false13 IncludeGuidesGrids false13 IncludeNonPrinting false13 IncludeSlug false13 Namespace [13 (Adobe)13 (InDesign)13 (40)13 ]13 OmitPlacedBitmaps false13 OmitPlacedEPS false13 OmitPlacedPDF false13 SimulateOverprint Legacy13 gtgt13 ltlt13 AddBleedMarks false13 AddColorBars false13 AddCropMarks false13 AddPageInfo false13 AddRegMarks false13 ConvertColors ConvertToCMYK13 DestinationProfileName ()13 DestinationProfileSelector DocumentCMYK13 Downsample16BitImages true13 FlattenerPreset ltlt13 PresetSelector MediumResolution13 gtgt13 FormElements false13 GenerateStructure false13 IncludeBookmarks false13 IncludeHyperlinks false13 IncludeInteractive false13 IncludeLayers false13 IncludeProfiles false13 MultimediaHandling UseObjectSettings13 Namespace [13 (Adobe)13 (CreativeSuite)13 (20)13 ]13 PDFXOutputIntentProfileSelector DocumentCMYK13 PreserveEditing true13 UntaggedCMYKHandling LeaveUntagged13 UntaggedRGBHandling UseDocumentProfile13 UseDocumentBleed false13 gtgt13 ]13gtgt setdistillerparams13ltlt13 HWResolution [2400 2400]13 PageSize [612000 792000]13gtgt setpagedevice13

Page 17: July 2014 MEE Questions and Analyses - NCBE...This publication includes the questions and analyses from the July 2014 MEE. (In the actual test, the questions are simply numbered rather

Criminal Law and Procedure Analysis

Point Three (35) The suspectrsquos waiver of his Miranda rights was knowing intelligent and voluntary despite the fact that he was never told of the lawyerrsquos presence in the jail or of the lawyerrsquos demands

A valid waiver of Miranda rights must be ldquovoluntaryrdquomdashie the product of a free or deliberate choice rather than intimidation coercion or deception Berghuis v Thompkins 560 US 370 382ndash83 (2010) In addition the waiver must be knowing and intelligent That is it ldquomust have been made with a full awareness of both the nature of the right being abandoned and the consequences of the decision to abandon itrdquo Moran v Burbine 475 US 421 (1986)

In this case the suspect signed a Miranda waiver form after receiving proper warnings There is no evidence ldquothat the police resorted to physical or psychological pressure to elicit the statementsrdquo Id The entire interview lasted only 45 minutes The only issue is whether the suspect knowingly and intelligently waived his Miranda rights despite the fact that the detective did not tell the suspect about the lawyerrsquos presence and her demands

The Supreme Court has said that ldquo[e]vents occurring outside of the presence of the suspect and entirely unknown to him surely can have no bearing on the capacity to comprehend and knowingly relinquish a constitutional rightrdquo Id at 422 If the suspect ldquoknew that he could stand mute and request a lawyer and was aware of the Statersquos intention to use his statements to secure a convictionrdquo then the waiver is valid regardless of the information withheld Id at 422ndash23

Here the suspect was correctly informed of his rights Miranda v Arizona 384 US at 467ndash73 His comments demonstrate that he understood that he could have a lawyer present if he desired (ie wondering whether he should call his attorney) and that he understood that there might be consequences to speaking with the detective (ldquoI probably should keep my mouth shut but Irsquom willing to talk to you for a whilerdquo) His comment ldquo[L]etrsquos not waste any time waiting for someone to call my attorney and having her drive hererdquo along with his signature on the Miranda waiver form show that his waiver was valid under the constitutional standard

The fact that the detective did not tell the suspect about the lawyerrsquos presence and demands has no bearing on the validity of the suspectrsquos waiver because ldquosuch conduct is only relevant to the constitutional validity of a waiver if it deprives a defendant of knowledge essential to his ability to understand the nature of his rights and the consequences of abandoning themrdquo Moran at 424 The Supreme Court has specifically declined to adopt a rule requiring that law enforcement tell a suspect of an attorneyrsquos efforts to contact him id at 425 (ldquoNor are we prepared to adopt a rule requiring that the police inform a suspect of an attorneyrsquos efforts to reach himrdquo)

[NOTE An examinee might also recognize that this general rule is further supported by the Supreme Courtrsquos decision in Florida v Powell 559 US 50 (2010) approving state Miranda warnings that do not explicitly warn suspects that they have a right to have counsel present during custodial interrogation]

15

CONTRACTS ANALYSIS (Contracts IB2 IIB IVA3 amp A5)

ANALYSIS

Legal Problems

(1) In the case of a service contract (governed by the common law of contracts) is a modification enforceable when a party agrees to pay more for the same performance than was originally promised

(2) In the case of a contract for the sale of goods (governed by Article 2 of the UCC) is a modification enforceable when a party agrees to pay more for the same goods than was originally promised

(3) May a party avoid an agreement on the basis of economic duress

DISCUSSION

Summary

There are two arguments that the conservatory can make to support the claim that it is not bound to pay the higher prices lack of consideration and economic duress

The organ repair contract is governed by the common law of contracts Under the common law the business would have difficulty recovering the additional $60000 for the organ repair because under the ldquopreexisting duty rulerdquo the agreement of the conservatory to pay the extra price was not supported by consideration However the business might argue that the modification is enforceable under an exception to the preexisting duty rule for fair and equitable modifications made in light of unanticipated circumstances

The organ sale contract is governed by Article 2 of the Uniform Commercial Code The business would likely recover the additional amount under that contract because Article 2 provides that consideration is not required for a modification to be binding

In both cases the conservatory could seek to avoid its agreement on the grounds of economic duress but that argument is not likely to succeed

Point One (45) The business probably cannot recover the additional $60000 for the organ repair because the conservatoryrsquos promise to pay more money was not supported by consideration

The general rule is that to be enforceable a promise must be supported by consideration Under RESTATEMENT (SECOND) OF CONTRACTS sect 71 a promise is supported by consideration if it is bargained for in exchange for a return promise or performance However under the ldquopreexisting duty rulerdquo (exemplified in RESTATEMENT (SECOND) OF CONTRACTS sect 73 and Alaska Packersrsquo Assrsquon v Domenico 117 F 99 (9th Cir 1902)) promise of performance of a legal duty already owed to a promisor which is neither doubtful nor the subject of honest dispute is not consideration

If the business had promised the conservatory anything new or different in exchange for the agreement to pay the additional $60000 (such as for example repairing the pipe organ more

16

Contracts Analysis

quickly or using better parts) that would constitute consideration especially in light of the principle that courts do not inquire into the adequacy of consideration Here however the business already had a legal duty under the original contract and did not agree to do anything else in exchange for the conservatoryrsquos promise to pay $60000 more

However an exception to the preexisting duty rule is sometimes applied in situations of unanticipated changed circumstances Under RESTATEMENT (SECOND) OF CONTRACTS sect 89 followed in many jurisdictions a promise modifying a duty under a contract not fully performed on either side is binding even if not supported by consideration if the modification is fair and equitable in view of circumstances not anticipated by the parties when the contract was made

If a court applies the rule in Restatement sect 89 the critical issues will be whether the modification was in fact ldquofair and equitablerdquo and whether it can be justified in light of unanticipated circumstances In many cases in which modifications have been upheld a party encountered difficulties or burdens in performing far beyond what was knowingly bargained for in the original contract with the result bordering on impracticability such as having to excavate solid rock instead of soft dirt or having to remove garbage far in excess of the amounts contemplated The conservatory would argue that the businessrsquos performance difficulties were not of this sort at allmdashnothing about repairing the pipe organ itself was any different from or more difficult than originally contemplated except that the business itself encountered financial distress unrelated to its burdens in performing its obligations under these contracts

Even if the business satisfies that element of the rule in Restatement sect 89 the business must also demonstrate that the circumstances that gave rise to the need to modify the contract were ldquounanticipatedrdquo at the time the original contract was made Here the facts suggest that when the business entered into the original contract it expected that the price paid by the conservatory would enable it to perform However any evidence that the business knew or had reason to know at the time of execution that it would need more money from the conservatory to be able to perform would mean that the request to modify was not ldquounanticipatedrdquo

[NOTE Some cases such as Schwartzreich v Bauman-Basch Inc 231 NY 196 131 NE 887 (1921) find that if the parties mutually agreed to rescind the original contract and then after rescission entered into an entirely new contract for a higher price the new contract is supported by consideration There is no evidence that such a rescission followed by a new contract took place here]

Point Two (45) The business can recover the additional $40000 for the new organ because no consideration is required under Article 2 of the UCC for good-faith contract modifications

The contract to buy a new organ is a contract for the sale of goods and therefore is governed by Article 2 of the Uniform Commercial Code UCC sect 2-102 Under Article 2 unlike the common law an agreement modifying a contract needs no consideration to be binding UCC sect 2-209(1) Section 2-209(1) thus obviates the preexisting duty rule entirely in contracts for the sale of goods

Even though consideration is not required modifications governed by sect 2-209 must satisfy the obligation of good faith imposed by the UCC UCC sect 1-304 See also Official Comment 2 to UCC sect 2-209 Good faith means ldquohonesty in fact and the observance of reasonable commercial standards of fair dealingrdquo UCC sect 1-201(b)(20) In this context the obligation of good faith means that ldquo[t]he effective use of bad faith to escape performance on the original contract terms is barred and the extortion of a lsquomodificationrsquo without legitimate commercial reason is ineffective as a violation of the duty of good faithrdquo Official Comment 2 to

17

Contracts Analysis

UCC sect 2-209 Here because the businessrsquos financial reversals were serious and apparently unanticipated at the time that the business entered into the contract with the conservatory and commitment of the extra money was needed to enable the business to perform a court would likely find that the business acted in good faith Thus a court would likely uphold the enforceability of the conservatoryrsquos promise to pay the additional $40000

Point Three (10) The conservatory is unlikely to be able to defend against enforcement of its promises to pay additional money under the theory of economic duress because the business probably did not make an improper threat

Under the common law of contracts parties may raise the defense of duress This common law defense also applies to contracts governed by UCC Article 2 See UCC sect 1-103(b)

A contract is voidable on the ground of economic duress by threat when it is established that a partyrsquos manifestation of assent is induced by an improper threat that leaves the party no reasonable alternative See RESTATEMENT (SECOND) OF CONTRACTS sect 175 See also eg Austin Instrument Inc v Loral Corp 272 NE2d 533 (NY 1971) (a threat to withhold essential goods can constitute duress) In order to void its agreement to pay the additional sum because of economic duress the conservatory must demonstrate that (1) the business made a threat to the conservatory (2) the threat was ldquoimproperrdquo or ldquowrongfulrdquo (3) the threat induced the conservatoryrsquos manifestation of assent to the modification and (4) the threat was sufficiently grave to justify the conservatoryrsquos assent

Here it appears that three of the four elements are likely satisfied The business plainly made a threat Moreover the threat induced the conservatoryrsquos assent to the modification and the threat was sufficiently grave to justify that assent If the conservatory had not agreed to pay the business the extra amounts the conservatory would have lost its entire $325000 investment In light of this potential loss a court could easily conclude that the conservatory had no reasonable alternative

However the business has a strong argument that its threat (indicating that it would breach the contracts unless the prices were increased) was not wrongful or improper but was instead nothing more than a communication of the reality of its own perilous situation to the conservatory

A mere threat to breach a contract is not in and of itself improper so as to support an action of economic duress or business compulsion Something more is required such as a breach of the duty of good faith and fair dealing as was present in Austin Instrument Inc supra Because the business could not perform the original contract without the requested modification the economic duress claim for the conservatory would likely fail for much the same reason that the business would be able to enforce the modification At the time the modification was requested the business was not trying to extort a price increase because of the conservatoryrsquos vulnerability but instead was simply stating the reality that the business could not perform without more money

18

FAMILY LAW ANALYSIS (Family Law IIIB D amp G)

ANALYSIS

Legal Problems

(1)(a) Does the State A court have jurisdiction to modify the State B child support order

(1)(b) Does the State A court have jurisdiction to modify the marital-residence-saleshyproceeds provision of the State B property-division decree

(2)(a) May a child support order be modified retroactively

(2)(b) May a child support order be modified prospectively based on a change of employment with a lower salary

(2)(c) May a property-division order be modified after entry of a divorce decree

DISCUSSION

Summary

The State A court may exercise personal jurisdiction over the wife because she was personally served in State A However subject-matter jurisdiction over the interstate modification of child support is governed by the Uniform Interstate Family Support Act (UIFSA) Under UIFSA State A does not have jurisdiction to modify the order for the daughterrsquos support because the wife is still a resident of State B UIFSA on the other hand does not govern property distributions and thus a State A court is not precluded from hearing the husbandrsquos petition to modify the marital-residence-sale-proceeds provision of the divorce decree

A child support order may not be modified retroactively A child support order may be modified prospectively based on a substantial change in circumstances Courts agree that a significant decrease in income is a substantial change in circumstances All states treat voluntary income reductions differently than involuntary reductions but employ different approaches for evaluating the impact of a voluntary reduction Whether the husband could obtain prospective modification of the child support order depends on which approach is applied

A property-division order is not subject to post-divorce modification based on a change in circumstances Thus the husband may in some states obtain prospective modification of the order for the daughterrsquos support but he may not obtain modification of the marital-residenceshysale-proceeds provision

Point One(a) (25) Personal jurisdiction over a nonresident respondent does not confer subject-matter jurisdiction over child support modification Under UIFSA a State A court may not modify a child support order issued by a State B court when as here the child or either parent continues to reside in State B the jurisdiction that issued the child support order

The State A court may exercise personal jurisdiction over the wife The wife was personally served in State A and a state may exercise jurisdiction based on in-state personal service See

19

Family Law Analysis

Burnham v Superior Court 495 US 604 (1990) But personal jurisdiction over the wife is not enough to give a State A court jurisdiction to modify the State B support order

The interstate enforcement and modification of child support is governed by the Uniform Interstate Family Support Act (UIFSA) which has been adopted by all states Under UIFSA the state that originally issued a child support order (here State B) has continuing exclusive jurisdiction to modify the order if that state remains the residence of the obligee the child or the obligor and all parties do not consent to the jurisdiction of another forum See UIFSA sect 205 See also UIFSA sect 603 (ldquoA tribunal of this State shall recognize and enforce but may not modify a registered order if the issuing tribunal had jurisdictionrdquo) The wife and daughter continue to reside in State B and the wife has not consented to the jurisdiction of another forum Thus a State A court does not have jurisdiction to modify the State B child support order

[NOTE Examinees who do not discuss personal jurisdiction but fully discuss UIFSA may receive full credit]

Point One(b) (15) UIFSA does not apply to disputes over property division Thus the State A court may exercise jurisdiction over the husbandrsquos petition to modify the marital-residence-sale-proceeds provision of the State B divorce decree because it has personal jurisdiction over the wife

The State A court in which the husband brought his action has jurisdiction to adjudicate domestic relations issues The husbandrsquos petition to modify the property settlement is a domestic relations issue The courts of State A may exercise personal jurisdiction over the wife because she was personally served in State A See Burnham v Superior Court 495 US 604 (1990) see Point One(a)

UIFSA does not apply to divorce property-division disputes Thus although a State A court may not adjudicate the husbandrsquos petition to modify his child support obligations it may adjudicate his property-division claims (Even though the court has jurisdiction it may not modify the property-division award on the merits See Point Two(c))

Point Two(a) (20) A child support order may not be modified retroactively

State courts have long held that obligations to pay child support ordinarily may not be modified retroactively ldquoIf the hardship is particularly severe the courts sometimes devised a way to protect the obligor but in most instances the courts hold that retroactive modification of this kind is beyond their power and indeed the governing statute may so providerdquo HOMER H CLARK THE LAW OF DOMESTIC RELATIONSHIPS IN THE UNITED STATES 725 (2d ed 1987)

Federal law now goes further and requires the states as a condition of federal child-support funding to adopt rules that absolutely forbid retroactive modification of the support obligation See 42 USC sect 666(a)(9)(C) The states have adopted rules consistent with the federal requirements

Point Two(b) (25) It is unclear whether the husband could obtain prospective downward modification of his child support based on his voluntary acceptance of a job with a lower salary

Prospective modification of a child support order is typically available only when the petitioner can show a substantial change in circumstances See ROBERT E OLIPHANT amp NANCY VER

20

Family Law Analysis

STEEGH FAMILY LAW 213ndash15 (3d ed 2010) A significant decrease in income is typically viewed as a substantial change

However when a parent seeks to modify a child support obligation because he has voluntarily reduced his income a court will not modify the obligation based solely on the income loss Some courts refuse to modify whenever the income shift was voluntary See eg Aguiar v Aguiar 127 P3d 234 (Idaho Ct App 2005) Others look primarily to the petitionerrsquos intentions and permit downward modification if he has acted in good faith See eg In re Marriage of Horn 650 NE2d 1103 (Ill App Ct 1995) Many courts use a multifactor approach See OLIPHANT amp VER STEEGH supra 217ndash18

Here there is no question that the husbandrsquos loss of income was voluntary In a jurisdiction in which voluntary income reduction bars support modification the husbandrsquos petition would be denied

In a jurisdiction employing a good-faith or multifactor approach it is possible but not certain that the husband could obtain downward modification The evidence supports the husbandrsquos good faith his change in employment appears to be based on his new jobrsquos greater responsibilities and better promotion possibilities In a jurisdiction using a multifactor approach the court would likely also consider the impact of such a shift on the daughter the likely duration of the husbandrsquos income loss and the likelihood of a promotion that would ultimately inure to the daughterrsquos benefit Thus on these facts it is possible but by no means certain that the husband could prospectively obtain downward modification of his child support obligation to his daughter

Point Two(c)(15) A divorce property-division award is not subject to modification

A support order is aimed at meeting the post-divorce needs of the supported individual Because the future is unpredictable courts are empowered to modify a support award to take account of changed circumstances that may occur during the period in which support is paid

By contrast a property-distribution award divides assets of the marriage based on the equities at the time of divorce Because the past can be ascertained a property-division award is not subject to post-divorce modification See HARRY A KRAUSE ET AL FAMILY LAW CASES COMMENTS AND QUESTIONS 691 (6th ed 2007)

Here the husband is seeking modification of a property-division award with respect to an asset owned by the parties at the time of divorce Thus the husband may not obtain a modification of the marital-residence-sale-proceeds provision of the divorce decree based on his reduced income

21

FEDERAL CIVIL PROCEDURE ANALYSIS (Federal Civil Procedure III IVC)

ANALYSIS

Legal Problems

(1) Is the logging company entitled to join this action as a matter of right

(2)(a) May the nonprofit organization obtain a temporary restraining order to stop the USFS from issuing a logging permit

(2)(b) May the nonprofit organization obtain a preliminary injunction to stop the USFS from issuing a logging permit during the pendency of the action

DISCUSSION

Summary

The logging company is entitled to intervene in this action as a matter of right because it has an interest in the property or transaction that is the subject of the action and is so situated that its interest may be impaired or impeded as a practical matter if the action goes forward without it The logging companyrsquos interest is not adequately represented by the USFSrsquos presence in the lawsuit

The nonprofit organization may seek a temporary restraining order (TRO) followed by a preliminary injunction to prevent the USFS from issuing a logging permit pending the outcome of the action The nonprofit is likely to obtain a TRO if it can demonstrate a risk of immediate and irreparable injury The nonprofit is also likely to obtain a preliminary injunction if it can demonstrate a significant threat of irreparable harm and a likelihood of success on the merits of its National Environmental Policy Act (NEPA) claim

Point One (50) Rule 24(a) of the Federal Rules of Civil Procedure requires federal courts to allow a person to intervene in an action as a matter of right if the person a) is interested in the property or transaction that is the subject of the action b) is so situated that its interest may be impaired or impeded if the litigation goes forward without it and c) is not adequately represented by existing parties Here the logging company likely meets all three requirements and should be allowed to intervene as a matter of right

Rule 24 of the Federal Rules of Civil Procedure governs intervention the process by which a non-party to an action may join the litigation Under Rule 24(a) (intervention of right) a person must be permitted to intervene if three conditions are met (1) the movant ldquoclaims an interest relating to the property or transaction that is the subject of the actionrdquo (2) the movant ldquois so situated that disposition of the action may as a practical matter impair or impede the movantrsquos ability to protect its interestrdquo and (3) ldquoexisting partiesrdquo do not ldquoadequately represent [the movantrsquos] interestrdquo FED R CIV P 24(a) The three requirements for intervention of right are often ldquovery interrelatedrdquo 7C CHARLES ALAN WRIGHT ET AL FEDERAL PRACTICE AND PROCEDURE sect 1908 at 297 (2007 amp 2011 Supp)

22

Federal Civil Procedure Analysis

Here the court should find that the logging company meets this test First the logging company has a strong interest in the property or transaction that is the subject of this action The USFS has accepted the logging companyrsquos bid and the logging company is merely awaiting issuance of a logging permit to begin logging The nonprofit organization is seeking to prevent this logging The logging company therefore has a strong direct and substantial interest in the subject matter of the lawsuit and in having its winning bid honored and a logging permit issued See eg Kleissler v US Forest Serv 157 F3d 964 972 (3d Cir 1998) (stating that ldquo[t]imber companies have direct and substantial interests in a lawsuit aimed at halting loggingrdquo) see also Natural Resources Defense Council v US Nuclear Regulatory Commrsquon 578 F2d 1341 1343ndash 44 (10th Cir 1978) (holding that applicants whose license renewals were pending had Rule 24(a)(2) interests where the lawsuit sought to halt the license-issuing process pending preparation of environmental impact statements) See generally 7C WRIGHT ET AL supra sect 19081 at 309 (ldquoIf there is a direct substantial legally protectable interest in the proceedings it is clear that this requirement of the rule is satisfiedrdquo) Second the logging companyrsquos interest in receiving a logging permit may well be impaired as a practical matter by the outcome of the lawsuit If the USFS loses the lawsuit it will have to prepare an environmental impact statement before issuing the logging companyrsquos permit This will at a minimum delay the logging companyrsquos ability to exercise its rights and may in the long r un mean that no logging permit is ever issued Intervention of right is not limited to those that would be legally bound as a matter of preclusion doctrine Id sect 19082 at 368 Rather ldquo[t]he rule is satisfied whenever disposition of the present action would put the movant at a practical disadvantage in protecting its interestrdquo Id sect 19082 at 369 Here that condition is easily satisfied See Kleissler 157 F3d at 972 (ldquoTimber companies have direct and substantial interests in a lawsuit aimed at halting logging rdquo)

Given that the logging company has an interest that may be impaired by disposition of the action it should be allowed to intervene unless the court is persuaded that the USFS adequately represents the logging companyrsquos interest See Rule 24(a)(2) 7C WRIGHT ET AL supra sect 1909 Here it could be argued that the USFS adequately represents the logging companyrsquos interest because the USFS presumably wants the court to uphold its development plan and allow it to proceed with issuance of the logging permit which is the same relief that the logging company would seek However whether representation is truly adequate depends upon ldquo[a] discriminating appraisal of the circumstancesrdquo 7C WRIGHT ET AL supra sect 1909 at 440 Although both the government and the logging company wish to avoid the preparation of an environmental impact statement their interests are distinct The USFSrsquos interest is proper management of the national forest system while the logging companyrsquos interest is making a profit from logging the 5000-acre tract The USFSrsquos handling of the litigation is likely to be affected by a variety of policy concerns and political considerations that have nothing to do with the logging companyrsquos purely economic interest in securing the right to cut trees in the Scenic National Forest See eg Kleissler 157 F3d at 973ndash74 (ldquo[T]he government represents numerous complex and conflicting interests in matters of this nature The straightforward business interests asserted by intervenors here may become lost in the thicket of sometimes inconsistent governmental policiesrdquo)

[NOTES (1) Examinees who mistakenly analyze the logging companyrsquos case for joinder under the related but incorrect Rule 19 ldquoRequired Joinder of Partiesrdquo may receive credit Rule 19 allows existing parties to demand joinder of non-parties (or seek dismissal of the case if they canrsquot get it) There is a close relationship between Rule 24 and Rule 19 and both contain a similar standard for determining when ldquointerestedrdquo third parties are ldquoentitledrdquo or ldquorequiredrdquo to be in the lawsuit Indeed the two prongs of the Rule 24 intervention test that are discussed above

23

Federal Civil Procedure Analysis

are nearly identical to the two prongs of the Rule 19(a) required joinder test Examinees who discuss and apply the test should receive credit even if they cite Rule 19 rather than Rule 24

(2) Examinees may discuss permissive joinder Although permissive joinder is a possibility here the question asks only whether the logging company can join the action as a matter of right and a permissive joinder analysis is not responsive to the question To the extent an examinee discusses permissive joinder the analysis will focus on whether the logging company ldquohas a claim or defense that shares with the main action a common question of law or factrdquo FED R CIV P 24(b)(1)(B) The district court also ldquomust consider whether the intervention will unduly delay or prejudice the adjudication of the original partiesrsquo rightsrdquo FED R CIV P 24(b)(3) On our facts the logging companyrsquos claim for the issuance of a logging permit would certainly share common questions of law and fact with the USFSrsquos defense against the nonprofitrsquos claim There are no facts suggesting that the logging companyrsquos presence would unduly delay or otherwise prejudice adjudication of the original action Thus the district court would have discretion to permit the logging company to intervene even if it denied intervention of right]

Point Two(a) (25) The nonprofit organization could seek and would likely obtain a temporary restraining order to stop the USFS from issuing a logging permit pending a hearing on an application for a preliminary injunction

The first type of interim relief the nonprofit could seek to stop the USFS from issuing a logging permit to the logging company is a temporary restraining order (TRO) prohibiting the USFS from issuing the logging permit A TRO can be issued without notice to the adverse party but only in limited circumstances and only for a limited time FED R CIV P 65(b) To secure a TRO without notice the nonprofit would need to submit an affidavit containing specific facts that demonstrate a risk of ldquoimmediate and irreparable injuryrdquo if a permit is issued FED R CIV P 65(b)(1) In deciding whether to grant a TRO courts will also consider the same factors that are relevant in deciding whether to grant a preliminary injunction (eg the moving partyrsquos likelihood of success on the merits the balance of hardships and the public interest) See Point Two(b) infra The TRO would last only long enough for the court to consider and resolve a request by the nonprofit for a preliminary injunction but no longer than 14 days (unless the court extends it for good cause or the adverse party consents to an extension) In addition bond is required

Here the court is likely to grant the nonprofitrsquos request The nonprofit could plausibly claim that cutting down 5000 acres of old-growth forest in an area that is home to the highest concentration of wildlife in the western United States would have ldquoan immediate and irreparablerdquo adverse impact on the environment and cause irreparable harm to the nonprofitrsquos interest in preserving and protecting natural resources including wildlife habitat

Point Two(b) (25) The nonprofit could also seek and would likely obtain a preliminary injunction to stop the USFS which is likely to be granted if the nonprofitrsquos claim that the USFS violated NEPA has a strong basis in fact and law

Because the TRO would be temporary the nonprofit would need to move for a preliminary injunction to prevent the USFS from issuing a logging permit throughout the pendency of the litigation Preliminary injunctions are injunctions that seek to ldquoprotect [the] plaintiff from

24

Federal Civil Procedure Analysis

irreparable injury and to preserve the courtrsquos power to render a meaningful decision after a trial on the meritsrdquo 11A CHARLES ALAN WRIGHT ET AL FEDERAL PRACTICE AND PROCEDURE sect 2947 at 112 (2013) Rule 65 of the Federal Rules of Civil Procedure sets out the procedural requirements for preliminary injunctions Preliminary injunctions may be granted only upon notice to the adverse party FED R CIV P 65(a)(1) and only if the movant ldquogives security in an amount that the court considers proper to pay the costs and damages sustained by any party found to have been wrongfully enjoined or restrainedrdquo FED R CIV P 65(c)

While Rule 65 sets out the procedural requirements for preliminary injunctive relief it does not specify the substantive grounds upon which it may be granted The courtrsquos discretion in ruling upon a motion for a preliminary injunction ldquois exercised in conformity with historic federal equity practicerdquo 11A WRIGHT ET AL supra sect 2947 at 114 The court typically considers four factors

(1) the significance of the threat of irreparable harm to the plaintiff if the injunction is not granted (2) the balance between this harm and the injury that granting the injunction would inflict on the defendant (3) the probability that the plaintiff will succeed on the merits and (4) the public interest

Id sect 2948 at 122ndash24 accord Habitat Educ Center v Bosworth 363 F Supp 2d 1070 1088 (ED Wis 2005) The most important of these factors is the risk of irreparable harm to the plaintiff 11A WRIGHT ET AL supra sect 29481 at 129 If the plaintiff has an adequate remedy at law (eg if money damages can compensate the plaintiff for its loss) then a preliminary injunction will be denied Id sect 29481

Here a court would likely conclude that the potential for environmental damage to the forest creates a significant threat of irreparable harm ldquo[E]nvironmental injury is often irreparable Courts have recognized that logging such as would occur [here] can have longshyterm environmental consequences and thus satisfy the irreparable injury criterionrdquo Habitat Educ Center 363 F Supp 2d at 1089 (citing Idaho Sporting Congress Inc v Alexander 222 F3d 562 569 (9th Cir 2000) (noting that the imminent and continuing logging activities presented ldquoevidence of environmental harm sufficient to tip the balance in favor of injunctive reliefrdquo)) Neighbors of Cuddy Mountain v US Forest Service 137 F3d 1372 1382 (9th Cir 1998) (stating that ldquo[t]he old growth forests plaintiffs seek to protect would if cut take hundreds of years to reproducerdquo) (internal citation omitted)) see also 11C WRIGHT ET AL supra sect 29481 at 151 (noting that ldquoa preliminary injunction has been issued to prevent harm to the environmentrdquo)

The second factor the balance between the harm to the plaintiff and the harm the defendant will suffer if the injunction is issued also appears to support issuance of a preliminary injunction here The USFS will have to wait before it can develop the Scenic National Forest and the logging company may lose money if the delay is prolonged These economic harms could be compensated monetarily if an injunction is issued inappropriately Where ldquoan injunction bond can compensate [the] defendant for any harm the injunction is likely to inflict the balance should be struck in favor of [the] plaintiffrdquo Id sect 29482 at 192 See also Habitat Educ Center 363 F Supp 2d at 1089 (stating that ldquothe relative absence of harmful effects on the Forest Service weighs in favor of granting the injunctionrdquo)

The third factor is the likelihood that the plaintiff will prevail on the merits Although there is limited information concerning the merits of the action the nonprofit alleges that the federal statute (NEPA) requires an environmental impact statement and further states that the USFS created no environmental impact analysis or statement at all Assuming that those

25

Federal Civil Procedure Analysis

allegations are correct it seems plausible to conclude that the nonprofit will be able to show a likelihood of success on the merits

Finally courts deciding whether or not to issue preliminary injunctive relief are to consider the public interest ldquoFocusing on this factor is another way of inquiring whether there are policy considerations that bear on whether the order should issuerdquo 11C WRIGHT ET AL supra sect 29484 at 214 If the court concludes that the nonprofit is likely to succeed on its NEPA claim because the USFS wrongfully failed to conduct an environmental impact assessment it is likely to find that the public interest would be served by restraining the USFS from proceeding with logging in a national forest See Heartwood Inc v US Forest Service 73 F Supp 2d 962 979 (SD Ill 1999) affrsquod on other grounds 230 F3d 947 (7th Cir 2000) (ldquoviolations by federal agencies of NEPArsquos provisions as established by Congress harm the public as well as the environmentrdquo)

Thus a court is very likely to grant a preliminary injunction if it concludes that the nonprofit has a significant likelihood of success on the merits

26

EVIDENCE ANALYSIS (Evidence ID IIA amp C)

ANALYSIS

Legal Problems

(1) Under what circumstances can evidence of prior convictions be used to impeach a witnessrsquos credibility in a civil case

(1)(a) May the inmatersquos credibility be impeached by evidence of a 12-year-old felony drug conviction if he was released from prison 9 years ago

(1)(b) May the inmatersquos credibility be impeached by evidence of an 8-year-old misdemeanor perjury conviction that was punishable by 1 year in jail if he pleaded guilty and was sentenced only to pay a $5000 fine

(1)(c) May the inmatersquos credibility be impeached by evidence of a 7-year-old sexual assault conviction if the inmate is still serving a 10-year prison sentence and the victim was his 13-year-old daughter

(2)(a) May the guardrsquos credibility be impeached by cross-examination regarding specific instances of misconduct (ie lying on his reacutesumeacute) relevant to credibility

(2)(b) May the guardrsquos credibility be impeached by admission of extrinsic evidence (his reacutesumeacute and academic transcript) offered to prove specific instances of misconduct relevant to credibility

DISCUSSION

Summary

Under the Federal Rules of Evidence witnesses can be impeached with evidence of prior convictions andor specific instances of misconduct Whether evidence of prior convictions should be admitted to impeach generally depends on the nature of the crime the amount of time that has passed and (only in criminal cases) whether the ldquowitnessrdquo is the defendant FED R EVID 609(a)

In this civil case evidence of the inmatersquos conviction for distribution of marijuana should be admitted to impeach the inmate because he was convicted of a felony and was released from prison fewer than 10 years ago FED R EVID 609(a)(1) Credibility is critically important in this case because the jury will hear conflicting testimony from the two disputing parties and there were no other eyewitnesses to the altercation Under Rule 609(a)(1) the inmatersquos conviction should be admitted because it has some bearing on his credibility and its probative value is not substantially outweighed by concerns of unfair prejudice confusion or delay Id

Evidence of the inmatersquos misdemeanor conviction for perjury must be admitted because the crime ldquorequired provingmdashor the witnessrsquos admittingmdasha dishonest act or false statementrdquo by the inmate FED R EVID 609(a)(2)

27

Evidence Analysis

Evidence of the inmatersquos felony conviction for sexual assault should be excluded because its probative value is substantially outweighed by the danger of unfair prejudice to the inmate based on the heinous nature of the crime FED R EVID 609(a)(1) In the alternative the judge could limit the evidence relating to this conviction by excluding details of the inmatersquos crime

In all civil (and criminal) cases witnesses can also be impeached with evidence of specific instances of prior misconduct that did not result in a conviction FED R EVID 608(b) Pursuant to Rule 608(b) misconduct probative of untruthfulness can be inquired into on cross-examination but cannot be proved through extrinsic evidence Id Thus the inmatersquos counsel should be permitted to cross-examine the guard regarding the false statement in the guardrsquos reacutesumeacute However extrinsic evidence of the guardrsquos misconduct (ie the guardrsquos authenticated reacutesumeacute and transcript from the local college) should not be admitted even if the guard denies wrongdoing or refuses to answer cross-examination questions about these matters Id

Point One (10) The Federal Rules of Evidence permit impeachment of witnesses with evidence of prior convictions

Whether convictions should be admitted to impeach generally depends on the nature of the crime the amount of time that has passed and (only in criminal cases) whether the ldquowitnessrdquo is the defendant FED R EVID 609(a) Under Rule 609(a) evidence of prior convictions may be admitted for the purpose of ldquoattacking a witnessrsquos character for truthfulnessrdquo Id

There are two basic types of convictions that can be admitted for the purpose of impeachment

(1) convictions for crimes ldquopunishable by death or by imprisonment for more than one yearrdquo (which generally correlates to ldquofeloniesrdquo) FED R EVID 609(a)(1) and (2) convictions ldquofor any crimes regardless of the punishment if the court can readily determine that establishing the elements of the crime required provingmdashor the witnessrsquos admittingmdasha dishonest act or false statementrdquo FED R EVID 609(a)(2)

Pursuant to Rule 609(a)(1) in civil cases the admission of evidence of a felony conviction is ldquosubject to Rule 403 [which says that a court may exclude relevant evidence if its probative value is substantially outweighed by other factors]rdquo FED R EVID 609(a)(1) However Rule 403 does not protect the witness against admission of prior convictions involving dishonestymdashwhich must be admitted by the court FED R EVID 609(a)(2)

Finally Federal Rule of Evidence 609(b) contains the presumption that a conviction that is more than 10 years old or where more than 10 years has passed since the witnessrsquos release from confinement (whichever is later) should not be admitted unless ldquoits probative value supported by specific facts and circumstances substantially outweighs its prejudicial effectrdquo and the proponent has provided the adverse party with reasonable written notice FED R EVID 609(b)

Point One(a) (25) The court should admit evidence of the inmatersquos 12-year-old felony marijuana distribution conviction

The inmatersquos conviction for marijuana distribution was for a felony punishable by imprisonment for more than one year See FED R EVID 609(a)(1) Moreover although the conviction was 12 years ago the 10-year time limit of Rule 609(b) is not exceeded because that time limit runs

28

Evidence Analysis

from the date of either ldquothe witnessrsquos conviction or release from confinement for it whichever is laterrdquo FED R EVID 609(b) Because the inmate served three years in prison he was released from confinement nine years ago

However pursuant to Rule 609(a)(1) the admission of felony convictions to impeach a witness in a civil case is ldquosubject to Rule 403rdquo FED R EVID 609(a)(1) Neither Rule 609(a) nor the advisory committee notes specify which factors courts should consider when balancing the probative value of a conviction against the dangers identified in Rule 403 (which include (1) unfair prejudice (2) confusion of the issues (3) misleading the jury (4) waste of time or undue delay and (5) needless presentation of cumulative evidence) FED R EVID 403

In this case credibility is very important because the evidence consists primarily of the testimony of the disputing parties and there were no other eyewitnesses to the altercation This enhances the probative value of any evidence bearing on the inmatersquos credibility A court is likely to conclude that the inmatersquos prior felony drug conviction is relevant to his credibility See eg United States v Brito 427 F3d 53 64 (1st Cir 2005) (ldquoPrior drug-trafficking crimes are generally viewed as having some bearing on veracityrdquo) Although the probative value of any conviction diminishes with age see eg United States v Brewer 451 F Supp 50 53 (ED Tenn 1978) the inmatersquos ongoing problems with the law suggest that he has continued (and even escalated) his criminal behavior over the past nine years The court should admit this evidence because its probative value is not substantially outweighed by any Rule 403 concerns Specifically any prejudice to the inmate would be slight because the conviction is unrelated to the altercation at issue and the conviction was not for a heinous crime that might inflame the jury

[NOTE Whether an examinee identifies the jury instruction as containing a ldquoconclusiverdquo or ldquomandatoryrdquo presumption is less important than the examineersquos analysis of the constitutional infirmities]

Point One(b) (15) The court must admit evidence of the inmatersquos eight-year-old misdemeanor conviction because perjury is a crime of dishonesty

Rule 609(a)(2) provides that evidence of a criminal conviction ldquomust be admitted if the court can readily determine that establishing the elements of the crime required provingmdashor the witnessrsquos admittingmdasha dishonest act or false statementrdquo FED R EVID 609(a)(2) The inmatersquos conviction for perjury would have necessarily required proving that the inmate engaged in an act of dishonesty This conviction occurred within the past 10 years so it ldquomust be admittedrdquo because in contrast to Rule 609(a)(1) (discussed in Point One(a)) admission under Rule 609(a)(2) is mandatory and not subject to Rule 403

Point One(c) (20) The court should exclude evidence of the inmatersquos seven-year-old felony sexual assault conviction because the probative value of this evidence is substantially outweighed by the danger of unfair prejudice In the alternative the details of the prior conviction could be excluded

The inmatersquos conviction for felony sexual assault was seven years ago and he has not yet been released from incarceration so Rule 609(a) but not 609(b) is applicable here FED R EVID 609(a) This conviction is therefore admissible to impeach the inmate unless its probative value is substantially outweighed by the danger of unfair prejudice or any other Rule 403 concern Id

29

Evidence Analysis

Sex crimes are generally not considered relevant to credibility see Hopkins v State 639 So 2d 1247 1254 (Miss 1993) so the probative value of this conviction is relatively low Moreover the heinous nature of the inmatersquos crime (sexual assault on his daughter) makes the danger of unfair prejudice to the inmate very high Thus the court should exclude evidence of the conviction because it was for a heinous offense that is likely to inflame the jury and it has little bearing on credibility See eg United States v Beahm 664 F2d 414 419 (4th Cir 1981)

As an alternative to excluding this evidence the judge could minimize the unfair prejudice to the inmate by permitting limited cross-examination but refusing to allow specific questions about the nature of the inmatersquos conviction For example a court could limit cross-examination to the fact that the inmate was convicted of a ldquofelonyrdquo or perhaps that he was convicted of a ldquosexual assaultrdquo without identifying the victim However because evidence of the inmatersquos prior convictions can be admitted solely for the purpose of enabling the jury to assess his credibility and because his two earlier convictions should have already been admitted the court should exclude all evidence of the felony sexual assault conviction

Point Two(a) (15) The court should permit the inmatersquos counsel to cross-examine the guard regarding the false statement in his reacutesumeacute because the guardrsquos misconduct bears on his truthfulness

The inmate wishes to cross-examine the guard about his prior dishonest behaviormdashlying on his reacutesumeacutemdashthat did not involve a criminal conviction Rule 608(b) allows witnesses to be cross-examined about specific instances of prior non-conviction misconduct probative of untruthfulness ldquoin order to attack the witnessrsquos character for truthfulnessrdquo FED R EVID 608(b)

The courtrsquos decision to allow cross-examination about the guardrsquos prior dishonest behavior depends on the probative value of such evidence balanced against the danger of unfair prejudice to the guard or any other Rule 403 concern FED R EVID 403 Here the guardrsquos false statement on his reacutesumeacute that he obtained a degree in Criminal Justice is highly probative of his untruthfulness because it grossly misrepresents his actual academic record was made recently and was made with the intent to deceive Because the probative value of this evidence is very strong and is not substantially outweighed by any Rule 403 concerns cross-examination of the guard on this topic should be permitted The court may also consider it fair to permit this cross-examination of the guard on these matters assuming that one or more of the inmatersquos prior convictions have been admitted to impeach his credibility

Point Two(b) (15) The court should exclude extrinsic evidence of the guardrsquos non-conviction misconduct even if the guard denies wrongdoing or refuses to answer questions about the matter

Although Rule 608(b) allows cross-examination about specific instances of prior misconduct probative of untruthfulness ldquoextrinsic evidencerdquo offered to prove such misconduct is not admissible FED R EVID 608(b) The rationale for this rule is that allowing the introduction of extrinsic evidence of prior misconduct by witnesses when these acts are relevant only to the witnessesrsquo truthfulness and not to the main issues in the case would create too great a risk of confusing the jury and unduly delaying the trial The court does not have discretion to admit this extrinsic evidence See eg United States v Elliot 89 F3d 1360 1368 (8th Cir 1996)

30

Evidence Analysis

Here the inmatersquos counsel may cross-examine the guard about the false statement on his reacutesumeacute However the inmatersquos counsel must accept the guardrsquos response Even if the guard denies wrongdoing or refuses to answer questions about the matter the inmatersquos counsel cannot introduce the guardrsquos reacutesumeacute or the transcript from the local college to prove the guardrsquos misconduct

31

CORPORATIONS ANALYSIS (Corporations VA2 IX)

ANALYSIS

Legal Problems

(1) Do shareholders have the authority to amend a corporationrsquos bylaws with respect to director nominations

(2) Do board-approved bylaws on a particular subject here nomination of directors preempt subsequent conflicting bylaw amendments by shareholders

(3) Is a suit challenging both managementrsquos refusal to include the proposed bylaw amendment in Megarsquos proxy statement and the boardrsquos amendment of the bylaws dealing with nomination of directors a direct or derivative suit

DISCUSSION

Summary

The voting and litigation rights of the shareholders of Mega are subject to the provisions of the Model Business Corporations Act (MBCA)

The investorrsquos proposed bylaw provision is not inconsistent with state law Under the MBCA shareholders may amend the bylaws when the amendment deals with a proper matter for the corporationrsquos bylaws such as procedures for nominating directors

The Mega boardrsquos bylaw amendment does not preempt the investorrsquos proposed bylaw provision or the Mega shareholdersrsquo power to approve it While shareholders can limit the boardrsquos power to amend or repeal the bylaws the board cannot limit the shareholdersrsquo power

Whether the investor must make a demand on Megarsquos board depends on how the investor frames its claim If the investor claims a violation of shareholder voting rights the claim is direct and pre-suit demand on the board is not required If on the other hand the investor claims that the directors violated their fiduciary duties by amending the bylaws to entrench themselves the claim is derivative and a pre-suit demand is required

Point One (30) Shareholders may amend the corporationrsquos bylaws where the proposed bylaw provision relates to procedural matters typically included in the bylaws such as the nomination of directors

Internal affairs of the corporation such as the conduct of shareholder meetings and election of directors are subject to the corporate law of the state of incorporation See McDermott Inc v Lewis 531 A2d 206 (Del 1987) (applying law of jurisdiction where corporation was incorporated in case involving voting rights) This statersquos corporate statute is modeled on the MBCA

Under the MBCA ldquoshareholders may amend the corporationrsquos bylawsrdquo MBCA sect 1020(a) Thus the only question is whether the bylaws can specify the procedures for shareholder nomination of directors

32

Corporations Analysis

The MBCA states that the bylaws ldquomay contain any provision that is not inconsistent with law or the articles of incorporationrdquo MBCA sect 206(b) In addition the MBCA was revised in 2009 to address shareholder nomination of directors in public corporations (known as ldquoproxy accessrdquo) and specifies that the bylaws ldquomay contain a requirement that the corporation include in its [proxy materials] one or more individuals nominated by a shareholderrdquo MBCA sect 206(c)(1) see Committee on Corporate Laws ABA Section of Business Law Report on the Roles of Boards of Directors and Shareholders of Publicly Owned Corporations and Changes to the Model Business Corporations ActmdashAdoption of Shareholder Proxy Access Amendments to Chapters 2 and 10 65 BUS LAWYER 1105 (2010)

The inclusion of director-nomination procedures in the bylaws is consistent with practice and is recognized by the Delaware courts whose views on corporate law carry significant weight Typically the procedures for nomination of directors are found in the bylaws See 1 COX amp HAZEN TREATISE ON THE LAW OF CORPORATIONS sect 312 (3d ed 2011) see also 4 FLETCHER CORP FORMS ANN PART III ch 21 (2013) (including sample bylaws that permit nomination of directors by shareholders) The Delaware Supreme Court has confirmed that the bylaws may ldquodefine the process and proceduresrdquo for director elections See CA Inc v AFSCME Employees Pension Plan 953 A2d 227 (Del 2008) (concluding that bylaw amendment requiring reimbursement of election expenses to certain successful shareholder nominators is ldquoproper subjectrdquo under Delaware law)

[NOTE The question of the proper scope of the bylaws can be answered using the more general MBCA sect 206(b) or the 2009 MBCA revision adding sect 206(c)(1) (adopted in CT ME VA) In addition some examinees might raise the point that shareholder proposals may not compel the board to take action such as by including shareholder nominations in the companyrsquos proxy materials on the theory that the ldquobusiness and affairsrdquo of the corporation are to be managed by the board See MBCA sect 801(b) Although shareholders are generally limited to adopting precatory resolutions that recommend or encourage board action this limitation does not apply when shareholders have specific authority to take binding action on their ownmdashsuch as to amend the bylaws]

Point Two (30) Shareholders can amend (or repeal) board-approved bylaws Further shareholders can limit the boardrsquos power to later amend and repeal a shareholder-approved bylaw

Under the MBCA shareholders have the power to amend the bylaws See Point One The board shares this power with the shareholders unless (1) the corporationrsquos articles ldquoreserve that power exclusively to the shareholdersrdquo or (2) ldquothe shareholders in amending repealing or adopting a bylaw expressly provide that the board of directors may not amend repeal or reinstate that bylawrdquo See MBCA sect 1020(b)

Shareholder-approved bylaw provisions can amend or repeal existing bylaw provisions whether originally approved by the board or by shareholders See ALAN R PALMITER CORPORATIONS EXAMPLES AND EXPLANATIONS sect 713 (7th ed 2012) Thus the Mega boardrsquos bylaw amendmentmdashwhich set more demanding thresholds for shareholder nomination of directors than the investorrsquos proposed bylaw provisionmdashwould be superseded (repealed) if Megarsquos shareholders were to approve the investorrsquos proposal

Further a shareholder-approved bylaw generally can limit the power of the board to later amend or repeal it See MBCA sect 1020(b)(2) Thus if Megarsquos shareholders approved the bylaw

33

Corporations Analysis

provision proposed by the investor Megarsquos board could not repeal the provision because it includes a ldquono board repealrdquo clause

The revision to the MBCA in 2009 dealing with shareholder proxy access does not change this conclusion That revision specifies that a shareholder-approved bylaw dealing with director nominations may not limit the boardrsquos power to amend add or repeal ldquoany procedure or condition to such a bylaw in order to provide for a reasonable practicable and orderly processrdquo MBCA sect 206(d) Thus according to the revision if shareholders approve a bylaw amendment that limits further board changes the board would nonetheless retain the power to ldquotinkerrdquo with the bylaw to safeguard the voting process but could not repeal the shareholder-approved bylaw The Official Comment to MBCA sect 206(d) makes clear that the revision is ldquonot intended to allow the board of directors to frustrate the purpose of the shareholder-adopted proxy access provisionrdquo Thus if Megarsquos shareholders were to approve the bylaw provision proposed by the investor Megarsquos board could only amend the provision regarding its procedures or conditions in a manner consistent with its purpose of permitting proxy access for Megarsquos shareholders

[NOTE The boardrsquos attempted interference with a shareholder voting initiative may also have been a violation of the directorsrsquo fiduciary duties See Blasius Indus Inc v Atlas Corp 564 A2d 651 (Del Ch 1988) (finding that directors breached their fiduciary duties by amending bylaws and expanding size of board to thwart insurgentrsquos plan to amend bylaws and seat a majority of new directors) The call however asks examinees to consider whether shareholders or the board have ldquoprecedencerdquo over amending the corporate bylaws Thus an examineersquos answer should be framed in terms of ldquopowerrdquo and not ldquodutyrdquo]

Point Three (40) The investor need not make a demand on the board if the investor states a direct claim such as an allegation that the board interfered with the investorrsquos right to amend the bylaws But the investor must make a demand on the board if the investor states a derivative claim (on behalf of the corporation) such as an allegation that the directors sought to entrench themselves by interfering with the proposed proxy access

The MBCA generally requires that shareholders make a demand on the board of directors before initiation of a derivative suit MBCA sect 742 (shareholder may not bring derivative proceeding until written demand has been made on corporation and 90 days have expired) A derivative suit is essentially two suits in one where the plaintiff-shareholder seeks to bring on behalf of the corporation a claim that vindicates corporate rights usually based on violation of fiduciary duties PALMITER supra sect 1811 (6th ed 2009) The demand permits the board to investigate the situation identified by the shareholder and take suitable action No demand on the board is required however if the shareholder brings a direct suit to vindicate the shareholderrsquos own rights not those of the corporation

Is the suit brought by the investor derivative or direct The MBCA defines a ldquoderivative proceedingrdquo as one brought ldquoin the right of a domestic corporationrdquo MBCA sect 740(1) Thus the answer to how the investorrsquos suit should be characterized turns on what rights the investor seeks to vindicate If the investor frames its claim as one of fiduciary breach by directorsmdashfor example for failing to become adequately informed about voting procedures or for seeking to entrench themselves in office by manipulating the voting structure to avoid a shareholder insurgencymdashthen the suit is ldquoderivativerdquo and the investor must make a demand on the board See MBCA Ch 7 Subch D Introductory Comment (ldquothe derivative suit has historically been the principal method of challenging allegedly illegal action by managementrdquo)

34

Corporations Analysis

If however the investor frames its claim as one to vindicate shareholder rights the suit is direct and no demand is required For many courts the direct-derivative question turns on who is injured and who is to receive the relief sought by the plaintiff-shareholders See Tooley v Donaldson Lufkin amp Jenrette Inc 845 A2d 1031 (Del 2004) (characterizing a merger-delay claim as direct because delay of merger only harmed shareholders not corporation) Thus if the investor claims that managementrsquos refusal to include its proposed bylaw amendment in the corporationrsquos proxy materials violates its shareholder rights to initiate corporate governance reforms the suit will be direct Courts have not questioned the ability of shareholders to bring direct suits challenging board action to exclude their proposed bylaw amendments from the corporationrsquos proxy materials See JANA Master Fund Ltd v CNET Networks Inc 954 A2d 335 (Del Ch 2008) (upholding shareholderrsquos direct challenge to boardrsquos interpretation of advance-notice bylaw) Chesapeake Corp v Shore 771 A2d 293 (Del Ch 2000) (upholding shareholderrsquos direct challenge to actions by board that effectively prevented it from proposing bylaw amendments in contest for control)

Is the way that the investor frames its claim conclusive Courts have permitted shareholder-plaintiffs to challenge a transaction in a direct suit even though the same transaction could also be challenged as a fiduciary breach See Eisenberg v Flying Tiger Line Inc 451 F2d 267 (2d Cir 1971) (permitting direct suit challenging a corporate reorganization as a dilution of shareholder voting power even though reorganization may have involved conflicts of interest and thus constituted a fiduciary breach) Thus the investorrsquos choice to pursue a claim challenging the legality of managementrsquos decision to exclude the investorrsquos proposal from the corporationrsquos proxy materialsmdashrather than a possible breach of fiduciary dutymdashis likely to be respected See 3 COX amp HAZEN supra sect 153 (describing situations in which a claim can be framed as derivative or direct)

[NOTE Some issues under Delaware corporate law regarding pre-suit demand are not relevant here For example whether the Mega directors are independent and disinterested is not relevant to the MBCA requirement of a pre-suit demand As the Official Comment to MBCA sect 742 points out the MBCArsquos requirement of ldquouniversal demandrdquo gives the board ldquothe opportunity to reexamine the act complained of in the light of a potential lawsuit and take corrective actionrdquo even when the directors might be non-independent or have conflicts of interest

Nor is it relevant to the MBCA pre-suit demand requirement that the statutory 90-day waiting period may be onerous The first paragraph of MBCA sect 742 requires a pre-suit demand without exception the second paragraph of the section imposes a 90-day waiting period before a derivative suit may be brought which can be shortened if the board rejects the demand or ldquoirreparable injury to the corporation would result by waiting for the expiration of the 90-day periodrdquo The call as written asks only whether a pre-suit demand should be made and does not ask examinees to address whether the post-demand waiting period should be shortened under the ldquoirreparable injuryrdquo standard]

35

National Conference of Bar Examiners 302 South Bedford Street | Madison WI 53703-3622 Phone 608-280-8550 | Fax 608-280-8552 | TDD 608-661-1275

wwwncbexorg e-mail contactncbexorg

  • Preface
  • Description of the MEE
  • Instructions
  • July 2014 Questions
    • CRIMINAL LAW AND PROCEDURE QUESTION
    • CONTRACTS QUESTION
    • FAMILY LAW QUESTION
    • FEDERAL CIVIL PROCEDURE QUESTION
    • EVIDENCE QUESTION
    • CORPORATIONS QUESTION
      • July 2014 Analyses
        • CRIMINAL LAW AND PROCEDURE ANALYSIS
        • CONTRACTS ANALYSIS
        • FAMILY LAW ANALYSIS
        • FEDERAL CIVIL PROCEDURE ANALYSIS
        • EVIDENCE ANALYSIS
        • CORPORATIONS ANALYSIS
            • ltlt13 ASCII85EncodePages false13 AllowTransparency false13 AutoPositionEPSFiles true13 AutoRotatePages None13 Binding Left13 CalGrayProfile (Dot Gain 20)13 CalRGBProfile (sRGB IEC61966-21)13 CalCMYKProfile (US Web Coated 050SWOP051 v2)13 sRGBProfile (sRGB IEC61966-21)13 CannotEmbedFontPolicy Error13 CompatibilityLevel 1413 CompressObjects Tags13 CompressPages true13 ConvertImagesToIndexed true13 PassThroughJPEGImages true13 CreateJobTicket false13 DefaultRenderingIntent Default13 DetectBlends true13 DetectCurves 0000013 ColorConversionStrategy CMYK13 DoThumbnails false13 EmbedAllFonts true13 EmbedOpenType false13 ParseICCProfilesInComments true13 EmbedJobOptions true13 DSCReportingLevel 013 EmitDSCWarnings false13 EndPage -113 ImageMemory 104857613 LockDistillerParams false13 MaxSubsetPct 10013 Optimize true13 OPM 113 ParseDSCComments true13 ParseDSCCommentsForDocInfo true13 PreserveCopyPage true13 PreserveDICMYKValues true13 PreserveEPSInfo true13 PreserveFlatness true13 PreserveHalftoneInfo false13 PreserveOPIComments true13 PreserveOverprintSettings true13 StartPage 113 SubsetFonts true13 TransferFunctionInfo Apply13 UCRandBGInfo Preserve13 UsePrologue false13 ColorSettingsFile ()13 AlwaysEmbed [ true13 ]13 NeverEmbed [ true13 ]13 AntiAliasColorImages false13 CropColorImages true13 ColorImageMinResolution 30013 ColorImageMinResolutionPolicy OK13 DownsampleColorImages true13 ColorImageDownsampleType Bicubic13 ColorImageResolution 30013 ColorImageDepth -113 ColorImageMinDownsampleDepth 113 ColorImageDownsampleThreshold 15000013 EncodeColorImages true13 ColorImageFilter DCTEncode13 AutoFilterColorImages true13 ColorImageAutoFilterStrategy JPEG13 ColorACSImageDict ltlt13 QFactor 01513 HSamples [1 1 1 1] VSamples [1 1 1 1]13 gtgt13 ColorImageDict ltlt13 QFactor 01513 HSamples [1 1 1 1] VSamples [1 1 1 1]13 gtgt13 JPEG2000ColorACSImageDict ltlt13 TileWidth 25613 TileHeight 25613 Quality 3013 gtgt13 JPEG2000ColorImageDict ltlt13 TileWidth 25613 TileHeight 25613 Quality 3013 gtgt13 AntiAliasGrayImages false13 CropGrayImages true13 GrayImageMinResolution 30013 GrayImageMinResolutionPolicy OK13 DownsampleGrayImages true13 GrayImageDownsampleType Bicubic13 GrayImageResolution 30013 GrayImageDepth -113 GrayImageMinDownsampleDepth 213 GrayImageDownsampleThreshold 15000013 EncodeGrayImages true13 GrayImageFilter DCTEncode13 AutoFilterGrayImages true13 GrayImageAutoFilterStrategy JPEG13 GrayACSImageDict ltlt13 QFactor 01513 HSamples [1 1 1 1] VSamples [1 1 1 1]13 gtgt13 GrayImageDict ltlt13 QFactor 01513 HSamples [1 1 1 1] VSamples [1 1 1 1]13 gtgt13 JPEG2000GrayACSImageDict ltlt13 TileWidth 25613 TileHeight 25613 Quality 3013 gtgt13 JPEG2000GrayImageDict ltlt13 TileWidth 25613 TileHeight 25613 Quality 3013 gtgt13 AntiAliasMonoImages false13 CropMonoImages true13 MonoImageMinResolution 120013 MonoImageMinResolutionPolicy OK13 DownsampleMonoImages true13 MonoImageDownsampleType Bicubic13 MonoImageResolution 120013 MonoImageDepth -113 MonoImageDownsampleThreshold 15000013 EncodeMonoImages true13 MonoImageFilter CCITTFaxEncode13 MonoImageDict ltlt13 K -113 gtgt13 AllowPSXObjects false13 CheckCompliance [13 None13 ]13 PDFX1aCheck false13 PDFX3Check false13 PDFXCompliantPDFOnly false13 PDFXNoTrimBoxError true13 PDFXTrimBoxToMediaBoxOffset [13 00000013 00000013 00000013 00000013 ]13 PDFXSetBleedBoxToMediaBox true13 PDFXBleedBoxToTrimBoxOffset [13 00000013 00000013 00000013 00000013 ]13 PDFXOutputIntentProfile ()13 PDFXOutputConditionIdentifier ()13 PDFXOutputCondition ()13 PDFXRegistryName ()13 PDFXTrapped False1313 CreateJDFFile false13 Description ltlt13 ARA 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 BGR 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 CHS ltFEFF4f7f75288fd94e9b8bbe5b9a521b5efa7684002000410064006f006200650020005000440046002065876863900275284e8e9ad88d2891cf76845370524d53705237300260a853ef4ee54f7f75280020004100630072006f0062006100740020548c002000410064006f00620065002000520065006100640065007200200035002e003000204ee553ca66f49ad87248672c676562535f00521b5efa768400200050004400460020658768633002gt13 CHT ltFEFF4f7f752890194e9b8a2d7f6e5efa7acb7684002000410064006f006200650020005000440046002065874ef69069752865bc9ad854c18cea76845370524d5370523786557406300260a853ef4ee54f7f75280020004100630072006f0062006100740020548c002000410064006f00620065002000520065006100640065007200200035002e003000204ee553ca66f49ad87248672c4f86958b555f5df25efa7acb76840020005000440046002065874ef63002gt13 CZE 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 DAN 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 DEU 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 ESP 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 ETI 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 FRA 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 GRE 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 HEB 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 HRV (Za stvaranje Adobe PDF dokumenata najpogodnijih za visokokvalitetni ispis prije tiskanja koristite ove postavke Stvoreni PDF dokumenti mogu se otvoriti Acrobat i Adobe Reader 50 i kasnijim verzijama)13 HUN 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 ITA 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 JPN ltFEFF9ad854c18cea306a30d730ea30d730ec30b951fa529b7528002000410064006f0062006500200050004400460020658766f8306e4f5c6210306b4f7f75283057307e305930023053306e8a2d5b9a30674f5c62103055308c305f0020005000440046002030d530a130a430eb306f3001004100630072006f0062006100740020304a30883073002000410064006f00620065002000520065006100640065007200200035002e003000204ee5964d3067958b304f30533068304c3067304d307e305930023053306e8a2d5b9a306b306f30d530a930f330c8306e57cb30818fbc307f304c5fc59808306730593002gt13 KOR ltFEFFc7740020c124c815c7440020c0acc6a9d558c5ec0020ace0d488c9c80020c2dcd5d80020c778c1c4c5d00020ac00c7a50020c801d569d55c002000410064006f0062006500200050004400460020bb38c11cb97c0020c791c131d569b2c8b2e4002e0020c774b807ac8c0020c791c131b41c00200050004400460020bb38c11cb2940020004100630072006f0062006100740020bc0f002000410064006f00620065002000520065006100640065007200200035002e00300020c774c0c1c5d0c11c0020c5f40020c2180020c788c2b5b2c8b2e4002egt13 LTH 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 LVI 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 NLD (Gebruik deze instellingen om Adobe PDF-documenten te maken die zijn geoptimaliseerd voor prepress-afdrukken van hoge kwaliteit De gemaakte PDF-documenten kunnen worden geopend met Acrobat en Adobe Reader 50 en hoger)13 NOR 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 POL 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 PTB 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 RUM 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 RUS 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 SKY ltFEFF0054006900650074006f0020006e006100730074006100760065006e0069006100200070006f0075017e0069007400650020006e00610020007600790074007600e100720061006e0069006500200064006f006b0075006d0065006e0074006f0076002000410064006f006200650020005000440046002c0020006b0074006f007200e90020007300610020006e0061006a006c0065007001610069006500200068006f0064006900610020006e00610020006b00760061006c00690074006e00fa00200074006c0061010d00200061002000700072006500700072006500730073002e00200056007900740076006f00720065006e00e900200064006f006b0075006d0065006e007400790020005000440046002000620075006400650020006d006f017e006e00e90020006f00740076006f00720069016500200076002000700072006f006700720061006d006f006300680020004100630072006f00620061007400200061002000410064006f00620065002000520065006100640065007200200035002e0030002000610020006e006f0076016100ed00630068002egt13 SLV 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 SUO 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 SVE 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 TUR ltFEFF005900fc006b00730065006b0020006b0061006c006900740065006c0069002000f6006e002000790061007a006401310072006d00610020006200610073006b013100730131006e006100200065006e0020006900790069002000750079006100620069006c006500630065006b002000410064006f006200650020005000440046002000620065006c00670065006c0065007200690020006f006c0075015f007400750072006d0061006b0020006900e70069006e00200062007500200061007900610072006c0061007201310020006b0075006c006c0061006e0131006e002e00200020004f006c0075015f0074007500720075006c0061006e0020005000440046002000620065006c00670065006c0065007200690020004100630072006f006200610074002000760065002000410064006f00620065002000520065006100640065007200200035002e003000200076006500200073006f006e0072006100730131006e00640061006b00690020007300fc007200fc006d006c00650072006c00650020006100e70131006c006100620069006c00690072002egt13 UKR 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 ENU (Use these settings to create Adobe PDF documents best suited for high-quality prepress printing Created PDF documents can be opened with Acrobat and Adobe Reader 50 and later)13 gtgt13 Namespace [13 (Adobe)13 (Common)13 (10)13 ]13 OtherNamespaces [13 ltlt13 AsReaderSpreads false13 CropImagesToFrames true13 ErrorControl WarnAndContinue13 FlattenerIgnoreSpreadOverrides false13 IncludeGuidesGrids false13 IncludeNonPrinting false13 IncludeSlug false13 Namespace [13 (Adobe)13 (InDesign)13 (40)13 ]13 OmitPlacedBitmaps false13 OmitPlacedEPS false13 OmitPlacedPDF false13 SimulateOverprint Legacy13 gtgt13 ltlt13 AddBleedMarks false13 AddColorBars false13 AddCropMarks false13 AddPageInfo false13 AddRegMarks false13 ConvertColors ConvertToCMYK13 DestinationProfileName ()13 DestinationProfileSelector DocumentCMYK13 Downsample16BitImages true13 FlattenerPreset ltlt13 PresetSelector MediumResolution13 gtgt13 FormElements false13 GenerateStructure false13 IncludeBookmarks false13 IncludeHyperlinks false13 IncludeInteractive false13 IncludeLayers false13 IncludeProfiles false13 MultimediaHandling UseObjectSettings13 Namespace [13 (Adobe)13 (CreativeSuite)13 (20)13 ]13 PDFXOutputIntentProfileSelector DocumentCMYK13 PreserveEditing true13 UntaggedCMYKHandling LeaveUntagged13 UntaggedRGBHandling UseDocumentProfile13 UseDocumentBleed false13 gtgt13 ]13gtgt setdistillerparams13ltlt13 HWResolution [2400 2400]13 PageSize [612000 792000]13gtgt setpagedevice13

Page 18: July 2014 MEE Questions and Analyses - NCBE...This publication includes the questions and analyses from the July 2014 MEE. (In the actual test, the questions are simply numbered rather

CONTRACTS ANALYSIS (Contracts IB2 IIB IVA3 amp A5)

ANALYSIS

Legal Problems

(1) In the case of a service contract (governed by the common law of contracts) is a modification enforceable when a party agrees to pay more for the same performance than was originally promised

(2) In the case of a contract for the sale of goods (governed by Article 2 of the UCC) is a modification enforceable when a party agrees to pay more for the same goods than was originally promised

(3) May a party avoid an agreement on the basis of economic duress

DISCUSSION

Summary

There are two arguments that the conservatory can make to support the claim that it is not bound to pay the higher prices lack of consideration and economic duress

The organ repair contract is governed by the common law of contracts Under the common law the business would have difficulty recovering the additional $60000 for the organ repair because under the ldquopreexisting duty rulerdquo the agreement of the conservatory to pay the extra price was not supported by consideration However the business might argue that the modification is enforceable under an exception to the preexisting duty rule for fair and equitable modifications made in light of unanticipated circumstances

The organ sale contract is governed by Article 2 of the Uniform Commercial Code The business would likely recover the additional amount under that contract because Article 2 provides that consideration is not required for a modification to be binding

In both cases the conservatory could seek to avoid its agreement on the grounds of economic duress but that argument is not likely to succeed

Point One (45) The business probably cannot recover the additional $60000 for the organ repair because the conservatoryrsquos promise to pay more money was not supported by consideration

The general rule is that to be enforceable a promise must be supported by consideration Under RESTATEMENT (SECOND) OF CONTRACTS sect 71 a promise is supported by consideration if it is bargained for in exchange for a return promise or performance However under the ldquopreexisting duty rulerdquo (exemplified in RESTATEMENT (SECOND) OF CONTRACTS sect 73 and Alaska Packersrsquo Assrsquon v Domenico 117 F 99 (9th Cir 1902)) promise of performance of a legal duty already owed to a promisor which is neither doubtful nor the subject of honest dispute is not consideration

If the business had promised the conservatory anything new or different in exchange for the agreement to pay the additional $60000 (such as for example repairing the pipe organ more

16

Contracts Analysis

quickly or using better parts) that would constitute consideration especially in light of the principle that courts do not inquire into the adequacy of consideration Here however the business already had a legal duty under the original contract and did not agree to do anything else in exchange for the conservatoryrsquos promise to pay $60000 more

However an exception to the preexisting duty rule is sometimes applied in situations of unanticipated changed circumstances Under RESTATEMENT (SECOND) OF CONTRACTS sect 89 followed in many jurisdictions a promise modifying a duty under a contract not fully performed on either side is binding even if not supported by consideration if the modification is fair and equitable in view of circumstances not anticipated by the parties when the contract was made

If a court applies the rule in Restatement sect 89 the critical issues will be whether the modification was in fact ldquofair and equitablerdquo and whether it can be justified in light of unanticipated circumstances In many cases in which modifications have been upheld a party encountered difficulties or burdens in performing far beyond what was knowingly bargained for in the original contract with the result bordering on impracticability such as having to excavate solid rock instead of soft dirt or having to remove garbage far in excess of the amounts contemplated The conservatory would argue that the businessrsquos performance difficulties were not of this sort at allmdashnothing about repairing the pipe organ itself was any different from or more difficult than originally contemplated except that the business itself encountered financial distress unrelated to its burdens in performing its obligations under these contracts

Even if the business satisfies that element of the rule in Restatement sect 89 the business must also demonstrate that the circumstances that gave rise to the need to modify the contract were ldquounanticipatedrdquo at the time the original contract was made Here the facts suggest that when the business entered into the original contract it expected that the price paid by the conservatory would enable it to perform However any evidence that the business knew or had reason to know at the time of execution that it would need more money from the conservatory to be able to perform would mean that the request to modify was not ldquounanticipatedrdquo

[NOTE Some cases such as Schwartzreich v Bauman-Basch Inc 231 NY 196 131 NE 887 (1921) find that if the parties mutually agreed to rescind the original contract and then after rescission entered into an entirely new contract for a higher price the new contract is supported by consideration There is no evidence that such a rescission followed by a new contract took place here]

Point Two (45) The business can recover the additional $40000 for the new organ because no consideration is required under Article 2 of the UCC for good-faith contract modifications

The contract to buy a new organ is a contract for the sale of goods and therefore is governed by Article 2 of the Uniform Commercial Code UCC sect 2-102 Under Article 2 unlike the common law an agreement modifying a contract needs no consideration to be binding UCC sect 2-209(1) Section 2-209(1) thus obviates the preexisting duty rule entirely in contracts for the sale of goods

Even though consideration is not required modifications governed by sect 2-209 must satisfy the obligation of good faith imposed by the UCC UCC sect 1-304 See also Official Comment 2 to UCC sect 2-209 Good faith means ldquohonesty in fact and the observance of reasonable commercial standards of fair dealingrdquo UCC sect 1-201(b)(20) In this context the obligation of good faith means that ldquo[t]he effective use of bad faith to escape performance on the original contract terms is barred and the extortion of a lsquomodificationrsquo without legitimate commercial reason is ineffective as a violation of the duty of good faithrdquo Official Comment 2 to

17

Contracts Analysis

UCC sect 2-209 Here because the businessrsquos financial reversals were serious and apparently unanticipated at the time that the business entered into the contract with the conservatory and commitment of the extra money was needed to enable the business to perform a court would likely find that the business acted in good faith Thus a court would likely uphold the enforceability of the conservatoryrsquos promise to pay the additional $40000

Point Three (10) The conservatory is unlikely to be able to defend against enforcement of its promises to pay additional money under the theory of economic duress because the business probably did not make an improper threat

Under the common law of contracts parties may raise the defense of duress This common law defense also applies to contracts governed by UCC Article 2 See UCC sect 1-103(b)

A contract is voidable on the ground of economic duress by threat when it is established that a partyrsquos manifestation of assent is induced by an improper threat that leaves the party no reasonable alternative See RESTATEMENT (SECOND) OF CONTRACTS sect 175 See also eg Austin Instrument Inc v Loral Corp 272 NE2d 533 (NY 1971) (a threat to withhold essential goods can constitute duress) In order to void its agreement to pay the additional sum because of economic duress the conservatory must demonstrate that (1) the business made a threat to the conservatory (2) the threat was ldquoimproperrdquo or ldquowrongfulrdquo (3) the threat induced the conservatoryrsquos manifestation of assent to the modification and (4) the threat was sufficiently grave to justify the conservatoryrsquos assent

Here it appears that three of the four elements are likely satisfied The business plainly made a threat Moreover the threat induced the conservatoryrsquos assent to the modification and the threat was sufficiently grave to justify that assent If the conservatory had not agreed to pay the business the extra amounts the conservatory would have lost its entire $325000 investment In light of this potential loss a court could easily conclude that the conservatory had no reasonable alternative

However the business has a strong argument that its threat (indicating that it would breach the contracts unless the prices were increased) was not wrongful or improper but was instead nothing more than a communication of the reality of its own perilous situation to the conservatory

A mere threat to breach a contract is not in and of itself improper so as to support an action of economic duress or business compulsion Something more is required such as a breach of the duty of good faith and fair dealing as was present in Austin Instrument Inc supra Because the business could not perform the original contract without the requested modification the economic duress claim for the conservatory would likely fail for much the same reason that the business would be able to enforce the modification At the time the modification was requested the business was not trying to extort a price increase because of the conservatoryrsquos vulnerability but instead was simply stating the reality that the business could not perform without more money

18

FAMILY LAW ANALYSIS (Family Law IIIB D amp G)

ANALYSIS

Legal Problems

(1)(a) Does the State A court have jurisdiction to modify the State B child support order

(1)(b) Does the State A court have jurisdiction to modify the marital-residence-saleshyproceeds provision of the State B property-division decree

(2)(a) May a child support order be modified retroactively

(2)(b) May a child support order be modified prospectively based on a change of employment with a lower salary

(2)(c) May a property-division order be modified after entry of a divorce decree

DISCUSSION

Summary

The State A court may exercise personal jurisdiction over the wife because she was personally served in State A However subject-matter jurisdiction over the interstate modification of child support is governed by the Uniform Interstate Family Support Act (UIFSA) Under UIFSA State A does not have jurisdiction to modify the order for the daughterrsquos support because the wife is still a resident of State B UIFSA on the other hand does not govern property distributions and thus a State A court is not precluded from hearing the husbandrsquos petition to modify the marital-residence-sale-proceeds provision of the divorce decree

A child support order may not be modified retroactively A child support order may be modified prospectively based on a substantial change in circumstances Courts agree that a significant decrease in income is a substantial change in circumstances All states treat voluntary income reductions differently than involuntary reductions but employ different approaches for evaluating the impact of a voluntary reduction Whether the husband could obtain prospective modification of the child support order depends on which approach is applied

A property-division order is not subject to post-divorce modification based on a change in circumstances Thus the husband may in some states obtain prospective modification of the order for the daughterrsquos support but he may not obtain modification of the marital-residenceshysale-proceeds provision

Point One(a) (25) Personal jurisdiction over a nonresident respondent does not confer subject-matter jurisdiction over child support modification Under UIFSA a State A court may not modify a child support order issued by a State B court when as here the child or either parent continues to reside in State B the jurisdiction that issued the child support order

The State A court may exercise personal jurisdiction over the wife The wife was personally served in State A and a state may exercise jurisdiction based on in-state personal service See

19

Family Law Analysis

Burnham v Superior Court 495 US 604 (1990) But personal jurisdiction over the wife is not enough to give a State A court jurisdiction to modify the State B support order

The interstate enforcement and modification of child support is governed by the Uniform Interstate Family Support Act (UIFSA) which has been adopted by all states Under UIFSA the state that originally issued a child support order (here State B) has continuing exclusive jurisdiction to modify the order if that state remains the residence of the obligee the child or the obligor and all parties do not consent to the jurisdiction of another forum See UIFSA sect 205 See also UIFSA sect 603 (ldquoA tribunal of this State shall recognize and enforce but may not modify a registered order if the issuing tribunal had jurisdictionrdquo) The wife and daughter continue to reside in State B and the wife has not consented to the jurisdiction of another forum Thus a State A court does not have jurisdiction to modify the State B child support order

[NOTE Examinees who do not discuss personal jurisdiction but fully discuss UIFSA may receive full credit]

Point One(b) (15) UIFSA does not apply to disputes over property division Thus the State A court may exercise jurisdiction over the husbandrsquos petition to modify the marital-residence-sale-proceeds provision of the State B divorce decree because it has personal jurisdiction over the wife

The State A court in which the husband brought his action has jurisdiction to adjudicate domestic relations issues The husbandrsquos petition to modify the property settlement is a domestic relations issue The courts of State A may exercise personal jurisdiction over the wife because she was personally served in State A See Burnham v Superior Court 495 US 604 (1990) see Point One(a)

UIFSA does not apply to divorce property-division disputes Thus although a State A court may not adjudicate the husbandrsquos petition to modify his child support obligations it may adjudicate his property-division claims (Even though the court has jurisdiction it may not modify the property-division award on the merits See Point Two(c))

Point Two(a) (20) A child support order may not be modified retroactively

State courts have long held that obligations to pay child support ordinarily may not be modified retroactively ldquoIf the hardship is particularly severe the courts sometimes devised a way to protect the obligor but in most instances the courts hold that retroactive modification of this kind is beyond their power and indeed the governing statute may so providerdquo HOMER H CLARK THE LAW OF DOMESTIC RELATIONSHIPS IN THE UNITED STATES 725 (2d ed 1987)

Federal law now goes further and requires the states as a condition of federal child-support funding to adopt rules that absolutely forbid retroactive modification of the support obligation See 42 USC sect 666(a)(9)(C) The states have adopted rules consistent with the federal requirements

Point Two(b) (25) It is unclear whether the husband could obtain prospective downward modification of his child support based on his voluntary acceptance of a job with a lower salary

Prospective modification of a child support order is typically available only when the petitioner can show a substantial change in circumstances See ROBERT E OLIPHANT amp NANCY VER

20

Family Law Analysis

STEEGH FAMILY LAW 213ndash15 (3d ed 2010) A significant decrease in income is typically viewed as a substantial change

However when a parent seeks to modify a child support obligation because he has voluntarily reduced his income a court will not modify the obligation based solely on the income loss Some courts refuse to modify whenever the income shift was voluntary See eg Aguiar v Aguiar 127 P3d 234 (Idaho Ct App 2005) Others look primarily to the petitionerrsquos intentions and permit downward modification if he has acted in good faith See eg In re Marriage of Horn 650 NE2d 1103 (Ill App Ct 1995) Many courts use a multifactor approach See OLIPHANT amp VER STEEGH supra 217ndash18

Here there is no question that the husbandrsquos loss of income was voluntary In a jurisdiction in which voluntary income reduction bars support modification the husbandrsquos petition would be denied

In a jurisdiction employing a good-faith or multifactor approach it is possible but not certain that the husband could obtain downward modification The evidence supports the husbandrsquos good faith his change in employment appears to be based on his new jobrsquos greater responsibilities and better promotion possibilities In a jurisdiction using a multifactor approach the court would likely also consider the impact of such a shift on the daughter the likely duration of the husbandrsquos income loss and the likelihood of a promotion that would ultimately inure to the daughterrsquos benefit Thus on these facts it is possible but by no means certain that the husband could prospectively obtain downward modification of his child support obligation to his daughter

Point Two(c)(15) A divorce property-division award is not subject to modification

A support order is aimed at meeting the post-divorce needs of the supported individual Because the future is unpredictable courts are empowered to modify a support award to take account of changed circumstances that may occur during the period in which support is paid

By contrast a property-distribution award divides assets of the marriage based on the equities at the time of divorce Because the past can be ascertained a property-division award is not subject to post-divorce modification See HARRY A KRAUSE ET AL FAMILY LAW CASES COMMENTS AND QUESTIONS 691 (6th ed 2007)

Here the husband is seeking modification of a property-division award with respect to an asset owned by the parties at the time of divorce Thus the husband may not obtain a modification of the marital-residence-sale-proceeds provision of the divorce decree based on his reduced income

21

FEDERAL CIVIL PROCEDURE ANALYSIS (Federal Civil Procedure III IVC)

ANALYSIS

Legal Problems

(1) Is the logging company entitled to join this action as a matter of right

(2)(a) May the nonprofit organization obtain a temporary restraining order to stop the USFS from issuing a logging permit

(2)(b) May the nonprofit organization obtain a preliminary injunction to stop the USFS from issuing a logging permit during the pendency of the action

DISCUSSION

Summary

The logging company is entitled to intervene in this action as a matter of right because it has an interest in the property or transaction that is the subject of the action and is so situated that its interest may be impaired or impeded as a practical matter if the action goes forward without it The logging companyrsquos interest is not adequately represented by the USFSrsquos presence in the lawsuit

The nonprofit organization may seek a temporary restraining order (TRO) followed by a preliminary injunction to prevent the USFS from issuing a logging permit pending the outcome of the action The nonprofit is likely to obtain a TRO if it can demonstrate a risk of immediate and irreparable injury The nonprofit is also likely to obtain a preliminary injunction if it can demonstrate a significant threat of irreparable harm and a likelihood of success on the merits of its National Environmental Policy Act (NEPA) claim

Point One (50) Rule 24(a) of the Federal Rules of Civil Procedure requires federal courts to allow a person to intervene in an action as a matter of right if the person a) is interested in the property or transaction that is the subject of the action b) is so situated that its interest may be impaired or impeded if the litigation goes forward without it and c) is not adequately represented by existing parties Here the logging company likely meets all three requirements and should be allowed to intervene as a matter of right

Rule 24 of the Federal Rules of Civil Procedure governs intervention the process by which a non-party to an action may join the litigation Under Rule 24(a) (intervention of right) a person must be permitted to intervene if three conditions are met (1) the movant ldquoclaims an interest relating to the property or transaction that is the subject of the actionrdquo (2) the movant ldquois so situated that disposition of the action may as a practical matter impair or impede the movantrsquos ability to protect its interestrdquo and (3) ldquoexisting partiesrdquo do not ldquoadequately represent [the movantrsquos] interestrdquo FED R CIV P 24(a) The three requirements for intervention of right are often ldquovery interrelatedrdquo 7C CHARLES ALAN WRIGHT ET AL FEDERAL PRACTICE AND PROCEDURE sect 1908 at 297 (2007 amp 2011 Supp)

22

Federal Civil Procedure Analysis

Here the court should find that the logging company meets this test First the logging company has a strong interest in the property or transaction that is the subject of this action The USFS has accepted the logging companyrsquos bid and the logging company is merely awaiting issuance of a logging permit to begin logging The nonprofit organization is seeking to prevent this logging The logging company therefore has a strong direct and substantial interest in the subject matter of the lawsuit and in having its winning bid honored and a logging permit issued See eg Kleissler v US Forest Serv 157 F3d 964 972 (3d Cir 1998) (stating that ldquo[t]imber companies have direct and substantial interests in a lawsuit aimed at halting loggingrdquo) see also Natural Resources Defense Council v US Nuclear Regulatory Commrsquon 578 F2d 1341 1343ndash 44 (10th Cir 1978) (holding that applicants whose license renewals were pending had Rule 24(a)(2) interests where the lawsuit sought to halt the license-issuing process pending preparation of environmental impact statements) See generally 7C WRIGHT ET AL supra sect 19081 at 309 (ldquoIf there is a direct substantial legally protectable interest in the proceedings it is clear that this requirement of the rule is satisfiedrdquo) Second the logging companyrsquos interest in receiving a logging permit may well be impaired as a practical matter by the outcome of the lawsuit If the USFS loses the lawsuit it will have to prepare an environmental impact statement before issuing the logging companyrsquos permit This will at a minimum delay the logging companyrsquos ability to exercise its rights and may in the long r un mean that no logging permit is ever issued Intervention of right is not limited to those that would be legally bound as a matter of preclusion doctrine Id sect 19082 at 368 Rather ldquo[t]he rule is satisfied whenever disposition of the present action would put the movant at a practical disadvantage in protecting its interestrdquo Id sect 19082 at 369 Here that condition is easily satisfied See Kleissler 157 F3d at 972 (ldquoTimber companies have direct and substantial interests in a lawsuit aimed at halting logging rdquo)

Given that the logging company has an interest that may be impaired by disposition of the action it should be allowed to intervene unless the court is persuaded that the USFS adequately represents the logging companyrsquos interest See Rule 24(a)(2) 7C WRIGHT ET AL supra sect 1909 Here it could be argued that the USFS adequately represents the logging companyrsquos interest because the USFS presumably wants the court to uphold its development plan and allow it to proceed with issuance of the logging permit which is the same relief that the logging company would seek However whether representation is truly adequate depends upon ldquo[a] discriminating appraisal of the circumstancesrdquo 7C WRIGHT ET AL supra sect 1909 at 440 Although both the government and the logging company wish to avoid the preparation of an environmental impact statement their interests are distinct The USFSrsquos interest is proper management of the national forest system while the logging companyrsquos interest is making a profit from logging the 5000-acre tract The USFSrsquos handling of the litigation is likely to be affected by a variety of policy concerns and political considerations that have nothing to do with the logging companyrsquos purely economic interest in securing the right to cut trees in the Scenic National Forest See eg Kleissler 157 F3d at 973ndash74 (ldquo[T]he government represents numerous complex and conflicting interests in matters of this nature The straightforward business interests asserted by intervenors here may become lost in the thicket of sometimes inconsistent governmental policiesrdquo)

[NOTES (1) Examinees who mistakenly analyze the logging companyrsquos case for joinder under the related but incorrect Rule 19 ldquoRequired Joinder of Partiesrdquo may receive credit Rule 19 allows existing parties to demand joinder of non-parties (or seek dismissal of the case if they canrsquot get it) There is a close relationship between Rule 24 and Rule 19 and both contain a similar standard for determining when ldquointerestedrdquo third parties are ldquoentitledrdquo or ldquorequiredrdquo to be in the lawsuit Indeed the two prongs of the Rule 24 intervention test that are discussed above

23

Federal Civil Procedure Analysis

are nearly identical to the two prongs of the Rule 19(a) required joinder test Examinees who discuss and apply the test should receive credit even if they cite Rule 19 rather than Rule 24

(2) Examinees may discuss permissive joinder Although permissive joinder is a possibility here the question asks only whether the logging company can join the action as a matter of right and a permissive joinder analysis is not responsive to the question To the extent an examinee discusses permissive joinder the analysis will focus on whether the logging company ldquohas a claim or defense that shares with the main action a common question of law or factrdquo FED R CIV P 24(b)(1)(B) The district court also ldquomust consider whether the intervention will unduly delay or prejudice the adjudication of the original partiesrsquo rightsrdquo FED R CIV P 24(b)(3) On our facts the logging companyrsquos claim for the issuance of a logging permit would certainly share common questions of law and fact with the USFSrsquos defense against the nonprofitrsquos claim There are no facts suggesting that the logging companyrsquos presence would unduly delay or otherwise prejudice adjudication of the original action Thus the district court would have discretion to permit the logging company to intervene even if it denied intervention of right]

Point Two(a) (25) The nonprofit organization could seek and would likely obtain a temporary restraining order to stop the USFS from issuing a logging permit pending a hearing on an application for a preliminary injunction

The first type of interim relief the nonprofit could seek to stop the USFS from issuing a logging permit to the logging company is a temporary restraining order (TRO) prohibiting the USFS from issuing the logging permit A TRO can be issued without notice to the adverse party but only in limited circumstances and only for a limited time FED R CIV P 65(b) To secure a TRO without notice the nonprofit would need to submit an affidavit containing specific facts that demonstrate a risk of ldquoimmediate and irreparable injuryrdquo if a permit is issued FED R CIV P 65(b)(1) In deciding whether to grant a TRO courts will also consider the same factors that are relevant in deciding whether to grant a preliminary injunction (eg the moving partyrsquos likelihood of success on the merits the balance of hardships and the public interest) See Point Two(b) infra The TRO would last only long enough for the court to consider and resolve a request by the nonprofit for a preliminary injunction but no longer than 14 days (unless the court extends it for good cause or the adverse party consents to an extension) In addition bond is required

Here the court is likely to grant the nonprofitrsquos request The nonprofit could plausibly claim that cutting down 5000 acres of old-growth forest in an area that is home to the highest concentration of wildlife in the western United States would have ldquoan immediate and irreparablerdquo adverse impact on the environment and cause irreparable harm to the nonprofitrsquos interest in preserving and protecting natural resources including wildlife habitat

Point Two(b) (25) The nonprofit could also seek and would likely obtain a preliminary injunction to stop the USFS which is likely to be granted if the nonprofitrsquos claim that the USFS violated NEPA has a strong basis in fact and law

Because the TRO would be temporary the nonprofit would need to move for a preliminary injunction to prevent the USFS from issuing a logging permit throughout the pendency of the litigation Preliminary injunctions are injunctions that seek to ldquoprotect [the] plaintiff from

24

Federal Civil Procedure Analysis

irreparable injury and to preserve the courtrsquos power to render a meaningful decision after a trial on the meritsrdquo 11A CHARLES ALAN WRIGHT ET AL FEDERAL PRACTICE AND PROCEDURE sect 2947 at 112 (2013) Rule 65 of the Federal Rules of Civil Procedure sets out the procedural requirements for preliminary injunctions Preliminary injunctions may be granted only upon notice to the adverse party FED R CIV P 65(a)(1) and only if the movant ldquogives security in an amount that the court considers proper to pay the costs and damages sustained by any party found to have been wrongfully enjoined or restrainedrdquo FED R CIV P 65(c)

While Rule 65 sets out the procedural requirements for preliminary injunctive relief it does not specify the substantive grounds upon which it may be granted The courtrsquos discretion in ruling upon a motion for a preliminary injunction ldquois exercised in conformity with historic federal equity practicerdquo 11A WRIGHT ET AL supra sect 2947 at 114 The court typically considers four factors

(1) the significance of the threat of irreparable harm to the plaintiff if the injunction is not granted (2) the balance between this harm and the injury that granting the injunction would inflict on the defendant (3) the probability that the plaintiff will succeed on the merits and (4) the public interest

Id sect 2948 at 122ndash24 accord Habitat Educ Center v Bosworth 363 F Supp 2d 1070 1088 (ED Wis 2005) The most important of these factors is the risk of irreparable harm to the plaintiff 11A WRIGHT ET AL supra sect 29481 at 129 If the plaintiff has an adequate remedy at law (eg if money damages can compensate the plaintiff for its loss) then a preliminary injunction will be denied Id sect 29481

Here a court would likely conclude that the potential for environmental damage to the forest creates a significant threat of irreparable harm ldquo[E]nvironmental injury is often irreparable Courts have recognized that logging such as would occur [here] can have longshyterm environmental consequences and thus satisfy the irreparable injury criterionrdquo Habitat Educ Center 363 F Supp 2d at 1089 (citing Idaho Sporting Congress Inc v Alexander 222 F3d 562 569 (9th Cir 2000) (noting that the imminent and continuing logging activities presented ldquoevidence of environmental harm sufficient to tip the balance in favor of injunctive reliefrdquo)) Neighbors of Cuddy Mountain v US Forest Service 137 F3d 1372 1382 (9th Cir 1998) (stating that ldquo[t]he old growth forests plaintiffs seek to protect would if cut take hundreds of years to reproducerdquo) (internal citation omitted)) see also 11C WRIGHT ET AL supra sect 29481 at 151 (noting that ldquoa preliminary injunction has been issued to prevent harm to the environmentrdquo)

The second factor the balance between the harm to the plaintiff and the harm the defendant will suffer if the injunction is issued also appears to support issuance of a preliminary injunction here The USFS will have to wait before it can develop the Scenic National Forest and the logging company may lose money if the delay is prolonged These economic harms could be compensated monetarily if an injunction is issued inappropriately Where ldquoan injunction bond can compensate [the] defendant for any harm the injunction is likely to inflict the balance should be struck in favor of [the] plaintiffrdquo Id sect 29482 at 192 See also Habitat Educ Center 363 F Supp 2d at 1089 (stating that ldquothe relative absence of harmful effects on the Forest Service weighs in favor of granting the injunctionrdquo)

The third factor is the likelihood that the plaintiff will prevail on the merits Although there is limited information concerning the merits of the action the nonprofit alleges that the federal statute (NEPA) requires an environmental impact statement and further states that the USFS created no environmental impact analysis or statement at all Assuming that those

25

Federal Civil Procedure Analysis

allegations are correct it seems plausible to conclude that the nonprofit will be able to show a likelihood of success on the merits

Finally courts deciding whether or not to issue preliminary injunctive relief are to consider the public interest ldquoFocusing on this factor is another way of inquiring whether there are policy considerations that bear on whether the order should issuerdquo 11C WRIGHT ET AL supra sect 29484 at 214 If the court concludes that the nonprofit is likely to succeed on its NEPA claim because the USFS wrongfully failed to conduct an environmental impact assessment it is likely to find that the public interest would be served by restraining the USFS from proceeding with logging in a national forest See Heartwood Inc v US Forest Service 73 F Supp 2d 962 979 (SD Ill 1999) affrsquod on other grounds 230 F3d 947 (7th Cir 2000) (ldquoviolations by federal agencies of NEPArsquos provisions as established by Congress harm the public as well as the environmentrdquo)

Thus a court is very likely to grant a preliminary injunction if it concludes that the nonprofit has a significant likelihood of success on the merits

26

EVIDENCE ANALYSIS (Evidence ID IIA amp C)

ANALYSIS

Legal Problems

(1) Under what circumstances can evidence of prior convictions be used to impeach a witnessrsquos credibility in a civil case

(1)(a) May the inmatersquos credibility be impeached by evidence of a 12-year-old felony drug conviction if he was released from prison 9 years ago

(1)(b) May the inmatersquos credibility be impeached by evidence of an 8-year-old misdemeanor perjury conviction that was punishable by 1 year in jail if he pleaded guilty and was sentenced only to pay a $5000 fine

(1)(c) May the inmatersquos credibility be impeached by evidence of a 7-year-old sexual assault conviction if the inmate is still serving a 10-year prison sentence and the victim was his 13-year-old daughter

(2)(a) May the guardrsquos credibility be impeached by cross-examination regarding specific instances of misconduct (ie lying on his reacutesumeacute) relevant to credibility

(2)(b) May the guardrsquos credibility be impeached by admission of extrinsic evidence (his reacutesumeacute and academic transcript) offered to prove specific instances of misconduct relevant to credibility

DISCUSSION

Summary

Under the Federal Rules of Evidence witnesses can be impeached with evidence of prior convictions andor specific instances of misconduct Whether evidence of prior convictions should be admitted to impeach generally depends on the nature of the crime the amount of time that has passed and (only in criminal cases) whether the ldquowitnessrdquo is the defendant FED R EVID 609(a)

In this civil case evidence of the inmatersquos conviction for distribution of marijuana should be admitted to impeach the inmate because he was convicted of a felony and was released from prison fewer than 10 years ago FED R EVID 609(a)(1) Credibility is critically important in this case because the jury will hear conflicting testimony from the two disputing parties and there were no other eyewitnesses to the altercation Under Rule 609(a)(1) the inmatersquos conviction should be admitted because it has some bearing on his credibility and its probative value is not substantially outweighed by concerns of unfair prejudice confusion or delay Id

Evidence of the inmatersquos misdemeanor conviction for perjury must be admitted because the crime ldquorequired provingmdashor the witnessrsquos admittingmdasha dishonest act or false statementrdquo by the inmate FED R EVID 609(a)(2)

27

Evidence Analysis

Evidence of the inmatersquos felony conviction for sexual assault should be excluded because its probative value is substantially outweighed by the danger of unfair prejudice to the inmate based on the heinous nature of the crime FED R EVID 609(a)(1) In the alternative the judge could limit the evidence relating to this conviction by excluding details of the inmatersquos crime

In all civil (and criminal) cases witnesses can also be impeached with evidence of specific instances of prior misconduct that did not result in a conviction FED R EVID 608(b) Pursuant to Rule 608(b) misconduct probative of untruthfulness can be inquired into on cross-examination but cannot be proved through extrinsic evidence Id Thus the inmatersquos counsel should be permitted to cross-examine the guard regarding the false statement in the guardrsquos reacutesumeacute However extrinsic evidence of the guardrsquos misconduct (ie the guardrsquos authenticated reacutesumeacute and transcript from the local college) should not be admitted even if the guard denies wrongdoing or refuses to answer cross-examination questions about these matters Id

Point One (10) The Federal Rules of Evidence permit impeachment of witnesses with evidence of prior convictions

Whether convictions should be admitted to impeach generally depends on the nature of the crime the amount of time that has passed and (only in criminal cases) whether the ldquowitnessrdquo is the defendant FED R EVID 609(a) Under Rule 609(a) evidence of prior convictions may be admitted for the purpose of ldquoattacking a witnessrsquos character for truthfulnessrdquo Id

There are two basic types of convictions that can be admitted for the purpose of impeachment

(1) convictions for crimes ldquopunishable by death or by imprisonment for more than one yearrdquo (which generally correlates to ldquofeloniesrdquo) FED R EVID 609(a)(1) and (2) convictions ldquofor any crimes regardless of the punishment if the court can readily determine that establishing the elements of the crime required provingmdashor the witnessrsquos admittingmdasha dishonest act or false statementrdquo FED R EVID 609(a)(2)

Pursuant to Rule 609(a)(1) in civil cases the admission of evidence of a felony conviction is ldquosubject to Rule 403 [which says that a court may exclude relevant evidence if its probative value is substantially outweighed by other factors]rdquo FED R EVID 609(a)(1) However Rule 403 does not protect the witness against admission of prior convictions involving dishonestymdashwhich must be admitted by the court FED R EVID 609(a)(2)

Finally Federal Rule of Evidence 609(b) contains the presumption that a conviction that is more than 10 years old or where more than 10 years has passed since the witnessrsquos release from confinement (whichever is later) should not be admitted unless ldquoits probative value supported by specific facts and circumstances substantially outweighs its prejudicial effectrdquo and the proponent has provided the adverse party with reasonable written notice FED R EVID 609(b)

Point One(a) (25) The court should admit evidence of the inmatersquos 12-year-old felony marijuana distribution conviction

The inmatersquos conviction for marijuana distribution was for a felony punishable by imprisonment for more than one year See FED R EVID 609(a)(1) Moreover although the conviction was 12 years ago the 10-year time limit of Rule 609(b) is not exceeded because that time limit runs

28

Evidence Analysis

from the date of either ldquothe witnessrsquos conviction or release from confinement for it whichever is laterrdquo FED R EVID 609(b) Because the inmate served three years in prison he was released from confinement nine years ago

However pursuant to Rule 609(a)(1) the admission of felony convictions to impeach a witness in a civil case is ldquosubject to Rule 403rdquo FED R EVID 609(a)(1) Neither Rule 609(a) nor the advisory committee notes specify which factors courts should consider when balancing the probative value of a conviction against the dangers identified in Rule 403 (which include (1) unfair prejudice (2) confusion of the issues (3) misleading the jury (4) waste of time or undue delay and (5) needless presentation of cumulative evidence) FED R EVID 403

In this case credibility is very important because the evidence consists primarily of the testimony of the disputing parties and there were no other eyewitnesses to the altercation This enhances the probative value of any evidence bearing on the inmatersquos credibility A court is likely to conclude that the inmatersquos prior felony drug conviction is relevant to his credibility See eg United States v Brito 427 F3d 53 64 (1st Cir 2005) (ldquoPrior drug-trafficking crimes are generally viewed as having some bearing on veracityrdquo) Although the probative value of any conviction diminishes with age see eg United States v Brewer 451 F Supp 50 53 (ED Tenn 1978) the inmatersquos ongoing problems with the law suggest that he has continued (and even escalated) his criminal behavior over the past nine years The court should admit this evidence because its probative value is not substantially outweighed by any Rule 403 concerns Specifically any prejudice to the inmate would be slight because the conviction is unrelated to the altercation at issue and the conviction was not for a heinous crime that might inflame the jury

[NOTE Whether an examinee identifies the jury instruction as containing a ldquoconclusiverdquo or ldquomandatoryrdquo presumption is less important than the examineersquos analysis of the constitutional infirmities]

Point One(b) (15) The court must admit evidence of the inmatersquos eight-year-old misdemeanor conviction because perjury is a crime of dishonesty

Rule 609(a)(2) provides that evidence of a criminal conviction ldquomust be admitted if the court can readily determine that establishing the elements of the crime required provingmdashor the witnessrsquos admittingmdasha dishonest act or false statementrdquo FED R EVID 609(a)(2) The inmatersquos conviction for perjury would have necessarily required proving that the inmate engaged in an act of dishonesty This conviction occurred within the past 10 years so it ldquomust be admittedrdquo because in contrast to Rule 609(a)(1) (discussed in Point One(a)) admission under Rule 609(a)(2) is mandatory and not subject to Rule 403

Point One(c) (20) The court should exclude evidence of the inmatersquos seven-year-old felony sexual assault conviction because the probative value of this evidence is substantially outweighed by the danger of unfair prejudice In the alternative the details of the prior conviction could be excluded

The inmatersquos conviction for felony sexual assault was seven years ago and he has not yet been released from incarceration so Rule 609(a) but not 609(b) is applicable here FED R EVID 609(a) This conviction is therefore admissible to impeach the inmate unless its probative value is substantially outweighed by the danger of unfair prejudice or any other Rule 403 concern Id

29

Evidence Analysis

Sex crimes are generally not considered relevant to credibility see Hopkins v State 639 So 2d 1247 1254 (Miss 1993) so the probative value of this conviction is relatively low Moreover the heinous nature of the inmatersquos crime (sexual assault on his daughter) makes the danger of unfair prejudice to the inmate very high Thus the court should exclude evidence of the conviction because it was for a heinous offense that is likely to inflame the jury and it has little bearing on credibility See eg United States v Beahm 664 F2d 414 419 (4th Cir 1981)

As an alternative to excluding this evidence the judge could minimize the unfair prejudice to the inmate by permitting limited cross-examination but refusing to allow specific questions about the nature of the inmatersquos conviction For example a court could limit cross-examination to the fact that the inmate was convicted of a ldquofelonyrdquo or perhaps that he was convicted of a ldquosexual assaultrdquo without identifying the victim However because evidence of the inmatersquos prior convictions can be admitted solely for the purpose of enabling the jury to assess his credibility and because his two earlier convictions should have already been admitted the court should exclude all evidence of the felony sexual assault conviction

Point Two(a) (15) The court should permit the inmatersquos counsel to cross-examine the guard regarding the false statement in his reacutesumeacute because the guardrsquos misconduct bears on his truthfulness

The inmate wishes to cross-examine the guard about his prior dishonest behaviormdashlying on his reacutesumeacutemdashthat did not involve a criminal conviction Rule 608(b) allows witnesses to be cross-examined about specific instances of prior non-conviction misconduct probative of untruthfulness ldquoin order to attack the witnessrsquos character for truthfulnessrdquo FED R EVID 608(b)

The courtrsquos decision to allow cross-examination about the guardrsquos prior dishonest behavior depends on the probative value of such evidence balanced against the danger of unfair prejudice to the guard or any other Rule 403 concern FED R EVID 403 Here the guardrsquos false statement on his reacutesumeacute that he obtained a degree in Criminal Justice is highly probative of his untruthfulness because it grossly misrepresents his actual academic record was made recently and was made with the intent to deceive Because the probative value of this evidence is very strong and is not substantially outweighed by any Rule 403 concerns cross-examination of the guard on this topic should be permitted The court may also consider it fair to permit this cross-examination of the guard on these matters assuming that one or more of the inmatersquos prior convictions have been admitted to impeach his credibility

Point Two(b) (15) The court should exclude extrinsic evidence of the guardrsquos non-conviction misconduct even if the guard denies wrongdoing or refuses to answer questions about the matter

Although Rule 608(b) allows cross-examination about specific instances of prior misconduct probative of untruthfulness ldquoextrinsic evidencerdquo offered to prove such misconduct is not admissible FED R EVID 608(b) The rationale for this rule is that allowing the introduction of extrinsic evidence of prior misconduct by witnesses when these acts are relevant only to the witnessesrsquo truthfulness and not to the main issues in the case would create too great a risk of confusing the jury and unduly delaying the trial The court does not have discretion to admit this extrinsic evidence See eg United States v Elliot 89 F3d 1360 1368 (8th Cir 1996)

30

Evidence Analysis

Here the inmatersquos counsel may cross-examine the guard about the false statement on his reacutesumeacute However the inmatersquos counsel must accept the guardrsquos response Even if the guard denies wrongdoing or refuses to answer questions about the matter the inmatersquos counsel cannot introduce the guardrsquos reacutesumeacute or the transcript from the local college to prove the guardrsquos misconduct

31

CORPORATIONS ANALYSIS (Corporations VA2 IX)

ANALYSIS

Legal Problems

(1) Do shareholders have the authority to amend a corporationrsquos bylaws with respect to director nominations

(2) Do board-approved bylaws on a particular subject here nomination of directors preempt subsequent conflicting bylaw amendments by shareholders

(3) Is a suit challenging both managementrsquos refusal to include the proposed bylaw amendment in Megarsquos proxy statement and the boardrsquos amendment of the bylaws dealing with nomination of directors a direct or derivative suit

DISCUSSION

Summary

The voting and litigation rights of the shareholders of Mega are subject to the provisions of the Model Business Corporations Act (MBCA)

The investorrsquos proposed bylaw provision is not inconsistent with state law Under the MBCA shareholders may amend the bylaws when the amendment deals with a proper matter for the corporationrsquos bylaws such as procedures for nominating directors

The Mega boardrsquos bylaw amendment does not preempt the investorrsquos proposed bylaw provision or the Mega shareholdersrsquo power to approve it While shareholders can limit the boardrsquos power to amend or repeal the bylaws the board cannot limit the shareholdersrsquo power

Whether the investor must make a demand on Megarsquos board depends on how the investor frames its claim If the investor claims a violation of shareholder voting rights the claim is direct and pre-suit demand on the board is not required If on the other hand the investor claims that the directors violated their fiduciary duties by amending the bylaws to entrench themselves the claim is derivative and a pre-suit demand is required

Point One (30) Shareholders may amend the corporationrsquos bylaws where the proposed bylaw provision relates to procedural matters typically included in the bylaws such as the nomination of directors

Internal affairs of the corporation such as the conduct of shareholder meetings and election of directors are subject to the corporate law of the state of incorporation See McDermott Inc v Lewis 531 A2d 206 (Del 1987) (applying law of jurisdiction where corporation was incorporated in case involving voting rights) This statersquos corporate statute is modeled on the MBCA

Under the MBCA ldquoshareholders may amend the corporationrsquos bylawsrdquo MBCA sect 1020(a) Thus the only question is whether the bylaws can specify the procedures for shareholder nomination of directors

32

Corporations Analysis

The MBCA states that the bylaws ldquomay contain any provision that is not inconsistent with law or the articles of incorporationrdquo MBCA sect 206(b) In addition the MBCA was revised in 2009 to address shareholder nomination of directors in public corporations (known as ldquoproxy accessrdquo) and specifies that the bylaws ldquomay contain a requirement that the corporation include in its [proxy materials] one or more individuals nominated by a shareholderrdquo MBCA sect 206(c)(1) see Committee on Corporate Laws ABA Section of Business Law Report on the Roles of Boards of Directors and Shareholders of Publicly Owned Corporations and Changes to the Model Business Corporations ActmdashAdoption of Shareholder Proxy Access Amendments to Chapters 2 and 10 65 BUS LAWYER 1105 (2010)

The inclusion of director-nomination procedures in the bylaws is consistent with practice and is recognized by the Delaware courts whose views on corporate law carry significant weight Typically the procedures for nomination of directors are found in the bylaws See 1 COX amp HAZEN TREATISE ON THE LAW OF CORPORATIONS sect 312 (3d ed 2011) see also 4 FLETCHER CORP FORMS ANN PART III ch 21 (2013) (including sample bylaws that permit nomination of directors by shareholders) The Delaware Supreme Court has confirmed that the bylaws may ldquodefine the process and proceduresrdquo for director elections See CA Inc v AFSCME Employees Pension Plan 953 A2d 227 (Del 2008) (concluding that bylaw amendment requiring reimbursement of election expenses to certain successful shareholder nominators is ldquoproper subjectrdquo under Delaware law)

[NOTE The question of the proper scope of the bylaws can be answered using the more general MBCA sect 206(b) or the 2009 MBCA revision adding sect 206(c)(1) (adopted in CT ME VA) In addition some examinees might raise the point that shareholder proposals may not compel the board to take action such as by including shareholder nominations in the companyrsquos proxy materials on the theory that the ldquobusiness and affairsrdquo of the corporation are to be managed by the board See MBCA sect 801(b) Although shareholders are generally limited to adopting precatory resolutions that recommend or encourage board action this limitation does not apply when shareholders have specific authority to take binding action on their ownmdashsuch as to amend the bylaws]

Point Two (30) Shareholders can amend (or repeal) board-approved bylaws Further shareholders can limit the boardrsquos power to later amend and repeal a shareholder-approved bylaw

Under the MBCA shareholders have the power to amend the bylaws See Point One The board shares this power with the shareholders unless (1) the corporationrsquos articles ldquoreserve that power exclusively to the shareholdersrdquo or (2) ldquothe shareholders in amending repealing or adopting a bylaw expressly provide that the board of directors may not amend repeal or reinstate that bylawrdquo See MBCA sect 1020(b)

Shareholder-approved bylaw provisions can amend or repeal existing bylaw provisions whether originally approved by the board or by shareholders See ALAN R PALMITER CORPORATIONS EXAMPLES AND EXPLANATIONS sect 713 (7th ed 2012) Thus the Mega boardrsquos bylaw amendmentmdashwhich set more demanding thresholds for shareholder nomination of directors than the investorrsquos proposed bylaw provisionmdashwould be superseded (repealed) if Megarsquos shareholders were to approve the investorrsquos proposal

Further a shareholder-approved bylaw generally can limit the power of the board to later amend or repeal it See MBCA sect 1020(b)(2) Thus if Megarsquos shareholders approved the bylaw

33

Corporations Analysis

provision proposed by the investor Megarsquos board could not repeal the provision because it includes a ldquono board repealrdquo clause

The revision to the MBCA in 2009 dealing with shareholder proxy access does not change this conclusion That revision specifies that a shareholder-approved bylaw dealing with director nominations may not limit the boardrsquos power to amend add or repeal ldquoany procedure or condition to such a bylaw in order to provide for a reasonable practicable and orderly processrdquo MBCA sect 206(d) Thus according to the revision if shareholders approve a bylaw amendment that limits further board changes the board would nonetheless retain the power to ldquotinkerrdquo with the bylaw to safeguard the voting process but could not repeal the shareholder-approved bylaw The Official Comment to MBCA sect 206(d) makes clear that the revision is ldquonot intended to allow the board of directors to frustrate the purpose of the shareholder-adopted proxy access provisionrdquo Thus if Megarsquos shareholders were to approve the bylaw provision proposed by the investor Megarsquos board could only amend the provision regarding its procedures or conditions in a manner consistent with its purpose of permitting proxy access for Megarsquos shareholders

[NOTE The boardrsquos attempted interference with a shareholder voting initiative may also have been a violation of the directorsrsquo fiduciary duties See Blasius Indus Inc v Atlas Corp 564 A2d 651 (Del Ch 1988) (finding that directors breached their fiduciary duties by amending bylaws and expanding size of board to thwart insurgentrsquos plan to amend bylaws and seat a majority of new directors) The call however asks examinees to consider whether shareholders or the board have ldquoprecedencerdquo over amending the corporate bylaws Thus an examineersquos answer should be framed in terms of ldquopowerrdquo and not ldquodutyrdquo]

Point Three (40) The investor need not make a demand on the board if the investor states a direct claim such as an allegation that the board interfered with the investorrsquos right to amend the bylaws But the investor must make a demand on the board if the investor states a derivative claim (on behalf of the corporation) such as an allegation that the directors sought to entrench themselves by interfering with the proposed proxy access

The MBCA generally requires that shareholders make a demand on the board of directors before initiation of a derivative suit MBCA sect 742 (shareholder may not bring derivative proceeding until written demand has been made on corporation and 90 days have expired) A derivative suit is essentially two suits in one where the plaintiff-shareholder seeks to bring on behalf of the corporation a claim that vindicates corporate rights usually based on violation of fiduciary duties PALMITER supra sect 1811 (6th ed 2009) The demand permits the board to investigate the situation identified by the shareholder and take suitable action No demand on the board is required however if the shareholder brings a direct suit to vindicate the shareholderrsquos own rights not those of the corporation

Is the suit brought by the investor derivative or direct The MBCA defines a ldquoderivative proceedingrdquo as one brought ldquoin the right of a domestic corporationrdquo MBCA sect 740(1) Thus the answer to how the investorrsquos suit should be characterized turns on what rights the investor seeks to vindicate If the investor frames its claim as one of fiduciary breach by directorsmdashfor example for failing to become adequately informed about voting procedures or for seeking to entrench themselves in office by manipulating the voting structure to avoid a shareholder insurgencymdashthen the suit is ldquoderivativerdquo and the investor must make a demand on the board See MBCA Ch 7 Subch D Introductory Comment (ldquothe derivative suit has historically been the principal method of challenging allegedly illegal action by managementrdquo)

34

Corporations Analysis

If however the investor frames its claim as one to vindicate shareholder rights the suit is direct and no demand is required For many courts the direct-derivative question turns on who is injured and who is to receive the relief sought by the plaintiff-shareholders See Tooley v Donaldson Lufkin amp Jenrette Inc 845 A2d 1031 (Del 2004) (characterizing a merger-delay claim as direct because delay of merger only harmed shareholders not corporation) Thus if the investor claims that managementrsquos refusal to include its proposed bylaw amendment in the corporationrsquos proxy materials violates its shareholder rights to initiate corporate governance reforms the suit will be direct Courts have not questioned the ability of shareholders to bring direct suits challenging board action to exclude their proposed bylaw amendments from the corporationrsquos proxy materials See JANA Master Fund Ltd v CNET Networks Inc 954 A2d 335 (Del Ch 2008) (upholding shareholderrsquos direct challenge to boardrsquos interpretation of advance-notice bylaw) Chesapeake Corp v Shore 771 A2d 293 (Del Ch 2000) (upholding shareholderrsquos direct challenge to actions by board that effectively prevented it from proposing bylaw amendments in contest for control)

Is the way that the investor frames its claim conclusive Courts have permitted shareholder-plaintiffs to challenge a transaction in a direct suit even though the same transaction could also be challenged as a fiduciary breach See Eisenberg v Flying Tiger Line Inc 451 F2d 267 (2d Cir 1971) (permitting direct suit challenging a corporate reorganization as a dilution of shareholder voting power even though reorganization may have involved conflicts of interest and thus constituted a fiduciary breach) Thus the investorrsquos choice to pursue a claim challenging the legality of managementrsquos decision to exclude the investorrsquos proposal from the corporationrsquos proxy materialsmdashrather than a possible breach of fiduciary dutymdashis likely to be respected See 3 COX amp HAZEN supra sect 153 (describing situations in which a claim can be framed as derivative or direct)

[NOTE Some issues under Delaware corporate law regarding pre-suit demand are not relevant here For example whether the Mega directors are independent and disinterested is not relevant to the MBCA requirement of a pre-suit demand As the Official Comment to MBCA sect 742 points out the MBCArsquos requirement of ldquouniversal demandrdquo gives the board ldquothe opportunity to reexamine the act complained of in the light of a potential lawsuit and take corrective actionrdquo even when the directors might be non-independent or have conflicts of interest

Nor is it relevant to the MBCA pre-suit demand requirement that the statutory 90-day waiting period may be onerous The first paragraph of MBCA sect 742 requires a pre-suit demand without exception the second paragraph of the section imposes a 90-day waiting period before a derivative suit may be brought which can be shortened if the board rejects the demand or ldquoirreparable injury to the corporation would result by waiting for the expiration of the 90-day periodrdquo The call as written asks only whether a pre-suit demand should be made and does not ask examinees to address whether the post-demand waiting period should be shortened under the ldquoirreparable injuryrdquo standard]

35

National Conference of Bar Examiners 302 South Bedford Street | Madison WI 53703-3622 Phone 608-280-8550 | Fax 608-280-8552 | TDD 608-661-1275

wwwncbexorg e-mail contactncbexorg

  • Preface
  • Description of the MEE
  • Instructions
  • July 2014 Questions
    • CRIMINAL LAW AND PROCEDURE QUESTION
    • CONTRACTS QUESTION
    • FAMILY LAW QUESTION
    • FEDERAL CIVIL PROCEDURE QUESTION
    • EVIDENCE QUESTION
    • CORPORATIONS QUESTION
      • July 2014 Analyses
        • CRIMINAL LAW AND PROCEDURE ANALYSIS
        • CONTRACTS ANALYSIS
        • FAMILY LAW ANALYSIS
        • FEDERAL CIVIL PROCEDURE ANALYSIS
        • EVIDENCE ANALYSIS
        • CORPORATIONS ANALYSIS
            • ltlt13 ASCII85EncodePages false13 AllowTransparency false13 AutoPositionEPSFiles true13 AutoRotatePages None13 Binding Left13 CalGrayProfile (Dot Gain 20)13 CalRGBProfile (sRGB IEC61966-21)13 CalCMYKProfile (US Web Coated 050SWOP051 v2)13 sRGBProfile (sRGB IEC61966-21)13 CannotEmbedFontPolicy Error13 CompatibilityLevel 1413 CompressObjects Tags13 CompressPages true13 ConvertImagesToIndexed true13 PassThroughJPEGImages true13 CreateJobTicket false13 DefaultRenderingIntent Default13 DetectBlends true13 DetectCurves 0000013 ColorConversionStrategy CMYK13 DoThumbnails false13 EmbedAllFonts true13 EmbedOpenType false13 ParseICCProfilesInComments true13 EmbedJobOptions true13 DSCReportingLevel 013 EmitDSCWarnings false13 EndPage -113 ImageMemory 104857613 LockDistillerParams false13 MaxSubsetPct 10013 Optimize true13 OPM 113 ParseDSCComments true13 ParseDSCCommentsForDocInfo true13 PreserveCopyPage true13 PreserveDICMYKValues true13 PreserveEPSInfo true13 PreserveFlatness true13 PreserveHalftoneInfo false13 PreserveOPIComments true13 PreserveOverprintSettings true13 StartPage 113 SubsetFonts true13 TransferFunctionInfo Apply13 UCRandBGInfo Preserve13 UsePrologue false13 ColorSettingsFile ()13 AlwaysEmbed [ true13 ]13 NeverEmbed [ true13 ]13 AntiAliasColorImages false13 CropColorImages true13 ColorImageMinResolution 30013 ColorImageMinResolutionPolicy OK13 DownsampleColorImages true13 ColorImageDownsampleType Bicubic13 ColorImageResolution 30013 ColorImageDepth -113 ColorImageMinDownsampleDepth 113 ColorImageDownsampleThreshold 15000013 EncodeColorImages true13 ColorImageFilter DCTEncode13 AutoFilterColorImages true13 ColorImageAutoFilterStrategy JPEG13 ColorACSImageDict ltlt13 QFactor 01513 HSamples [1 1 1 1] VSamples [1 1 1 1]13 gtgt13 ColorImageDict ltlt13 QFactor 01513 HSamples [1 1 1 1] VSamples [1 1 1 1]13 gtgt13 JPEG2000ColorACSImageDict ltlt13 TileWidth 25613 TileHeight 25613 Quality 3013 gtgt13 JPEG2000ColorImageDict ltlt13 TileWidth 25613 TileHeight 25613 Quality 3013 gtgt13 AntiAliasGrayImages false13 CropGrayImages true13 GrayImageMinResolution 30013 GrayImageMinResolutionPolicy OK13 DownsampleGrayImages true13 GrayImageDownsampleType Bicubic13 GrayImageResolution 30013 GrayImageDepth -113 GrayImageMinDownsampleDepth 213 GrayImageDownsampleThreshold 15000013 EncodeGrayImages true13 GrayImageFilter DCTEncode13 AutoFilterGrayImages true13 GrayImageAutoFilterStrategy JPEG13 GrayACSImageDict ltlt13 QFactor 01513 HSamples [1 1 1 1] VSamples [1 1 1 1]13 gtgt13 GrayImageDict ltlt13 QFactor 01513 HSamples [1 1 1 1] VSamples [1 1 1 1]13 gtgt13 JPEG2000GrayACSImageDict ltlt13 TileWidth 25613 TileHeight 25613 Quality 3013 gtgt13 JPEG2000GrayImageDict ltlt13 TileWidth 25613 TileHeight 25613 Quality 3013 gtgt13 AntiAliasMonoImages false13 CropMonoImages true13 MonoImageMinResolution 120013 MonoImageMinResolutionPolicy OK13 DownsampleMonoImages true13 MonoImageDownsampleType Bicubic13 MonoImageResolution 120013 MonoImageDepth -113 MonoImageDownsampleThreshold 15000013 EncodeMonoImages true13 MonoImageFilter CCITTFaxEncode13 MonoImageDict ltlt13 K -113 gtgt13 AllowPSXObjects false13 CheckCompliance [13 None13 ]13 PDFX1aCheck false13 PDFX3Check false13 PDFXCompliantPDFOnly false13 PDFXNoTrimBoxError true13 PDFXTrimBoxToMediaBoxOffset [13 00000013 00000013 00000013 00000013 ]13 PDFXSetBleedBoxToMediaBox true13 PDFXBleedBoxToTrimBoxOffset [13 00000013 00000013 00000013 00000013 ]13 PDFXOutputIntentProfile ()13 PDFXOutputConditionIdentifier ()13 PDFXOutputCondition ()13 PDFXRegistryName ()13 PDFXTrapped False1313 CreateJDFFile false13 Description ltlt13 ARA 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 BGR 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 CHS ltFEFF4f7f75288fd94e9b8bbe5b9a521b5efa7684002000410064006f006200650020005000440046002065876863900275284e8e9ad88d2891cf76845370524d53705237300260a853ef4ee54f7f75280020004100630072006f0062006100740020548c002000410064006f00620065002000520065006100640065007200200035002e003000204ee553ca66f49ad87248672c676562535f00521b5efa768400200050004400460020658768633002gt13 CHT ltFEFF4f7f752890194e9b8a2d7f6e5efa7acb7684002000410064006f006200650020005000440046002065874ef69069752865bc9ad854c18cea76845370524d5370523786557406300260a853ef4ee54f7f75280020004100630072006f0062006100740020548c002000410064006f00620065002000520065006100640065007200200035002e003000204ee553ca66f49ad87248672c4f86958b555f5df25efa7acb76840020005000440046002065874ef63002gt13 CZE 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 DAN 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 DEU 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 ESP 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 ETI 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 FRA 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 GRE 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 HEB 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 HRV (Za stvaranje Adobe PDF dokumenata najpogodnijih za visokokvalitetni ispis prije tiskanja koristite ove postavke Stvoreni PDF dokumenti mogu se otvoriti Acrobat i Adobe Reader 50 i kasnijim verzijama)13 HUN 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 ITA 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 JPN ltFEFF9ad854c18cea306a30d730ea30d730ec30b951fa529b7528002000410064006f0062006500200050004400460020658766f8306e4f5c6210306b4f7f75283057307e305930023053306e8a2d5b9a30674f5c62103055308c305f0020005000440046002030d530a130a430eb306f3001004100630072006f0062006100740020304a30883073002000410064006f00620065002000520065006100640065007200200035002e003000204ee5964d3067958b304f30533068304c3067304d307e305930023053306e8a2d5b9a306b306f30d530a930f330c8306e57cb30818fbc307f304c5fc59808306730593002gt13 KOR ltFEFFc7740020c124c815c7440020c0acc6a9d558c5ec0020ace0d488c9c80020c2dcd5d80020c778c1c4c5d00020ac00c7a50020c801d569d55c002000410064006f0062006500200050004400460020bb38c11cb97c0020c791c131d569b2c8b2e4002e0020c774b807ac8c0020c791c131b41c00200050004400460020bb38c11cb2940020004100630072006f0062006100740020bc0f002000410064006f00620065002000520065006100640065007200200035002e00300020c774c0c1c5d0c11c0020c5f40020c2180020c788c2b5b2c8b2e4002egt13 LTH 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 LVI 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 NLD (Gebruik deze instellingen om Adobe PDF-documenten te maken die zijn geoptimaliseerd voor prepress-afdrukken van hoge kwaliteit De gemaakte PDF-documenten kunnen worden geopend met Acrobat en Adobe Reader 50 en hoger)13 NOR 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 POL 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 PTB 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 RUM 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 RUS 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 SKY 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 SLV 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 SUO 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 SVE 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 TUR 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 UKR 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 ENU (Use these settings to create Adobe PDF documents best suited for high-quality prepress printing Created PDF documents can be opened with Acrobat and Adobe Reader 50 and later)13 gtgt13 Namespace [13 (Adobe)13 (Common)13 (10)13 ]13 OtherNamespaces [13 ltlt13 AsReaderSpreads false13 CropImagesToFrames true13 ErrorControl WarnAndContinue13 FlattenerIgnoreSpreadOverrides false13 IncludeGuidesGrids false13 IncludeNonPrinting false13 IncludeSlug false13 Namespace [13 (Adobe)13 (InDesign)13 (40)13 ]13 OmitPlacedBitmaps false13 OmitPlacedEPS false13 OmitPlacedPDF false13 SimulateOverprint Legacy13 gtgt13 ltlt13 AddBleedMarks false13 AddColorBars false13 AddCropMarks false13 AddPageInfo false13 AddRegMarks false13 ConvertColors ConvertToCMYK13 DestinationProfileName ()13 DestinationProfileSelector DocumentCMYK13 Downsample16BitImages true13 FlattenerPreset ltlt13 PresetSelector MediumResolution13 gtgt13 FormElements false13 GenerateStructure false13 IncludeBookmarks false13 IncludeHyperlinks false13 IncludeInteractive false13 IncludeLayers false13 IncludeProfiles false13 MultimediaHandling UseObjectSettings13 Namespace [13 (Adobe)13 (CreativeSuite)13 (20)13 ]13 PDFXOutputIntentProfileSelector DocumentCMYK13 PreserveEditing true13 UntaggedCMYKHandling LeaveUntagged13 UntaggedRGBHandling UseDocumentProfile13 UseDocumentBleed false13 gtgt13 ]13gtgt setdistillerparams13ltlt13 HWResolution [2400 2400]13 PageSize [612000 792000]13gtgt setpagedevice13

Page 19: July 2014 MEE Questions and Analyses - NCBE...This publication includes the questions and analyses from the July 2014 MEE. (In the actual test, the questions are simply numbered rather

Contracts Analysis

quickly or using better parts) that would constitute consideration especially in light of the principle that courts do not inquire into the adequacy of consideration Here however the business already had a legal duty under the original contract and did not agree to do anything else in exchange for the conservatoryrsquos promise to pay $60000 more

However an exception to the preexisting duty rule is sometimes applied in situations of unanticipated changed circumstances Under RESTATEMENT (SECOND) OF CONTRACTS sect 89 followed in many jurisdictions a promise modifying a duty under a contract not fully performed on either side is binding even if not supported by consideration if the modification is fair and equitable in view of circumstances not anticipated by the parties when the contract was made

If a court applies the rule in Restatement sect 89 the critical issues will be whether the modification was in fact ldquofair and equitablerdquo and whether it can be justified in light of unanticipated circumstances In many cases in which modifications have been upheld a party encountered difficulties or burdens in performing far beyond what was knowingly bargained for in the original contract with the result bordering on impracticability such as having to excavate solid rock instead of soft dirt or having to remove garbage far in excess of the amounts contemplated The conservatory would argue that the businessrsquos performance difficulties were not of this sort at allmdashnothing about repairing the pipe organ itself was any different from or more difficult than originally contemplated except that the business itself encountered financial distress unrelated to its burdens in performing its obligations under these contracts

Even if the business satisfies that element of the rule in Restatement sect 89 the business must also demonstrate that the circumstances that gave rise to the need to modify the contract were ldquounanticipatedrdquo at the time the original contract was made Here the facts suggest that when the business entered into the original contract it expected that the price paid by the conservatory would enable it to perform However any evidence that the business knew or had reason to know at the time of execution that it would need more money from the conservatory to be able to perform would mean that the request to modify was not ldquounanticipatedrdquo

[NOTE Some cases such as Schwartzreich v Bauman-Basch Inc 231 NY 196 131 NE 887 (1921) find that if the parties mutually agreed to rescind the original contract and then after rescission entered into an entirely new contract for a higher price the new contract is supported by consideration There is no evidence that such a rescission followed by a new contract took place here]

Point Two (45) The business can recover the additional $40000 for the new organ because no consideration is required under Article 2 of the UCC for good-faith contract modifications

The contract to buy a new organ is a contract for the sale of goods and therefore is governed by Article 2 of the Uniform Commercial Code UCC sect 2-102 Under Article 2 unlike the common law an agreement modifying a contract needs no consideration to be binding UCC sect 2-209(1) Section 2-209(1) thus obviates the preexisting duty rule entirely in contracts for the sale of goods

Even though consideration is not required modifications governed by sect 2-209 must satisfy the obligation of good faith imposed by the UCC UCC sect 1-304 See also Official Comment 2 to UCC sect 2-209 Good faith means ldquohonesty in fact and the observance of reasonable commercial standards of fair dealingrdquo UCC sect 1-201(b)(20) In this context the obligation of good faith means that ldquo[t]he effective use of bad faith to escape performance on the original contract terms is barred and the extortion of a lsquomodificationrsquo without legitimate commercial reason is ineffective as a violation of the duty of good faithrdquo Official Comment 2 to

17

Contracts Analysis

UCC sect 2-209 Here because the businessrsquos financial reversals were serious and apparently unanticipated at the time that the business entered into the contract with the conservatory and commitment of the extra money was needed to enable the business to perform a court would likely find that the business acted in good faith Thus a court would likely uphold the enforceability of the conservatoryrsquos promise to pay the additional $40000

Point Three (10) The conservatory is unlikely to be able to defend against enforcement of its promises to pay additional money under the theory of economic duress because the business probably did not make an improper threat

Under the common law of contracts parties may raise the defense of duress This common law defense also applies to contracts governed by UCC Article 2 See UCC sect 1-103(b)

A contract is voidable on the ground of economic duress by threat when it is established that a partyrsquos manifestation of assent is induced by an improper threat that leaves the party no reasonable alternative See RESTATEMENT (SECOND) OF CONTRACTS sect 175 See also eg Austin Instrument Inc v Loral Corp 272 NE2d 533 (NY 1971) (a threat to withhold essential goods can constitute duress) In order to void its agreement to pay the additional sum because of economic duress the conservatory must demonstrate that (1) the business made a threat to the conservatory (2) the threat was ldquoimproperrdquo or ldquowrongfulrdquo (3) the threat induced the conservatoryrsquos manifestation of assent to the modification and (4) the threat was sufficiently grave to justify the conservatoryrsquos assent

Here it appears that three of the four elements are likely satisfied The business plainly made a threat Moreover the threat induced the conservatoryrsquos assent to the modification and the threat was sufficiently grave to justify that assent If the conservatory had not agreed to pay the business the extra amounts the conservatory would have lost its entire $325000 investment In light of this potential loss a court could easily conclude that the conservatory had no reasonable alternative

However the business has a strong argument that its threat (indicating that it would breach the contracts unless the prices were increased) was not wrongful or improper but was instead nothing more than a communication of the reality of its own perilous situation to the conservatory

A mere threat to breach a contract is not in and of itself improper so as to support an action of economic duress or business compulsion Something more is required such as a breach of the duty of good faith and fair dealing as was present in Austin Instrument Inc supra Because the business could not perform the original contract without the requested modification the economic duress claim for the conservatory would likely fail for much the same reason that the business would be able to enforce the modification At the time the modification was requested the business was not trying to extort a price increase because of the conservatoryrsquos vulnerability but instead was simply stating the reality that the business could not perform without more money

18

FAMILY LAW ANALYSIS (Family Law IIIB D amp G)

ANALYSIS

Legal Problems

(1)(a) Does the State A court have jurisdiction to modify the State B child support order

(1)(b) Does the State A court have jurisdiction to modify the marital-residence-saleshyproceeds provision of the State B property-division decree

(2)(a) May a child support order be modified retroactively

(2)(b) May a child support order be modified prospectively based on a change of employment with a lower salary

(2)(c) May a property-division order be modified after entry of a divorce decree

DISCUSSION

Summary

The State A court may exercise personal jurisdiction over the wife because she was personally served in State A However subject-matter jurisdiction over the interstate modification of child support is governed by the Uniform Interstate Family Support Act (UIFSA) Under UIFSA State A does not have jurisdiction to modify the order for the daughterrsquos support because the wife is still a resident of State B UIFSA on the other hand does not govern property distributions and thus a State A court is not precluded from hearing the husbandrsquos petition to modify the marital-residence-sale-proceeds provision of the divorce decree

A child support order may not be modified retroactively A child support order may be modified prospectively based on a substantial change in circumstances Courts agree that a significant decrease in income is a substantial change in circumstances All states treat voluntary income reductions differently than involuntary reductions but employ different approaches for evaluating the impact of a voluntary reduction Whether the husband could obtain prospective modification of the child support order depends on which approach is applied

A property-division order is not subject to post-divorce modification based on a change in circumstances Thus the husband may in some states obtain prospective modification of the order for the daughterrsquos support but he may not obtain modification of the marital-residenceshysale-proceeds provision

Point One(a) (25) Personal jurisdiction over a nonresident respondent does not confer subject-matter jurisdiction over child support modification Under UIFSA a State A court may not modify a child support order issued by a State B court when as here the child or either parent continues to reside in State B the jurisdiction that issued the child support order

The State A court may exercise personal jurisdiction over the wife The wife was personally served in State A and a state may exercise jurisdiction based on in-state personal service See

19

Family Law Analysis

Burnham v Superior Court 495 US 604 (1990) But personal jurisdiction over the wife is not enough to give a State A court jurisdiction to modify the State B support order

The interstate enforcement and modification of child support is governed by the Uniform Interstate Family Support Act (UIFSA) which has been adopted by all states Under UIFSA the state that originally issued a child support order (here State B) has continuing exclusive jurisdiction to modify the order if that state remains the residence of the obligee the child or the obligor and all parties do not consent to the jurisdiction of another forum See UIFSA sect 205 See also UIFSA sect 603 (ldquoA tribunal of this State shall recognize and enforce but may not modify a registered order if the issuing tribunal had jurisdictionrdquo) The wife and daughter continue to reside in State B and the wife has not consented to the jurisdiction of another forum Thus a State A court does not have jurisdiction to modify the State B child support order

[NOTE Examinees who do not discuss personal jurisdiction but fully discuss UIFSA may receive full credit]

Point One(b) (15) UIFSA does not apply to disputes over property division Thus the State A court may exercise jurisdiction over the husbandrsquos petition to modify the marital-residence-sale-proceeds provision of the State B divorce decree because it has personal jurisdiction over the wife

The State A court in which the husband brought his action has jurisdiction to adjudicate domestic relations issues The husbandrsquos petition to modify the property settlement is a domestic relations issue The courts of State A may exercise personal jurisdiction over the wife because she was personally served in State A See Burnham v Superior Court 495 US 604 (1990) see Point One(a)

UIFSA does not apply to divorce property-division disputes Thus although a State A court may not adjudicate the husbandrsquos petition to modify his child support obligations it may adjudicate his property-division claims (Even though the court has jurisdiction it may not modify the property-division award on the merits See Point Two(c))

Point Two(a) (20) A child support order may not be modified retroactively

State courts have long held that obligations to pay child support ordinarily may not be modified retroactively ldquoIf the hardship is particularly severe the courts sometimes devised a way to protect the obligor but in most instances the courts hold that retroactive modification of this kind is beyond their power and indeed the governing statute may so providerdquo HOMER H CLARK THE LAW OF DOMESTIC RELATIONSHIPS IN THE UNITED STATES 725 (2d ed 1987)

Federal law now goes further and requires the states as a condition of federal child-support funding to adopt rules that absolutely forbid retroactive modification of the support obligation See 42 USC sect 666(a)(9)(C) The states have adopted rules consistent with the federal requirements

Point Two(b) (25) It is unclear whether the husband could obtain prospective downward modification of his child support based on his voluntary acceptance of a job with a lower salary

Prospective modification of a child support order is typically available only when the petitioner can show a substantial change in circumstances See ROBERT E OLIPHANT amp NANCY VER

20

Family Law Analysis

STEEGH FAMILY LAW 213ndash15 (3d ed 2010) A significant decrease in income is typically viewed as a substantial change

However when a parent seeks to modify a child support obligation because he has voluntarily reduced his income a court will not modify the obligation based solely on the income loss Some courts refuse to modify whenever the income shift was voluntary See eg Aguiar v Aguiar 127 P3d 234 (Idaho Ct App 2005) Others look primarily to the petitionerrsquos intentions and permit downward modification if he has acted in good faith See eg In re Marriage of Horn 650 NE2d 1103 (Ill App Ct 1995) Many courts use a multifactor approach See OLIPHANT amp VER STEEGH supra 217ndash18

Here there is no question that the husbandrsquos loss of income was voluntary In a jurisdiction in which voluntary income reduction bars support modification the husbandrsquos petition would be denied

In a jurisdiction employing a good-faith or multifactor approach it is possible but not certain that the husband could obtain downward modification The evidence supports the husbandrsquos good faith his change in employment appears to be based on his new jobrsquos greater responsibilities and better promotion possibilities In a jurisdiction using a multifactor approach the court would likely also consider the impact of such a shift on the daughter the likely duration of the husbandrsquos income loss and the likelihood of a promotion that would ultimately inure to the daughterrsquos benefit Thus on these facts it is possible but by no means certain that the husband could prospectively obtain downward modification of his child support obligation to his daughter

Point Two(c)(15) A divorce property-division award is not subject to modification

A support order is aimed at meeting the post-divorce needs of the supported individual Because the future is unpredictable courts are empowered to modify a support award to take account of changed circumstances that may occur during the period in which support is paid

By contrast a property-distribution award divides assets of the marriage based on the equities at the time of divorce Because the past can be ascertained a property-division award is not subject to post-divorce modification See HARRY A KRAUSE ET AL FAMILY LAW CASES COMMENTS AND QUESTIONS 691 (6th ed 2007)

Here the husband is seeking modification of a property-division award with respect to an asset owned by the parties at the time of divorce Thus the husband may not obtain a modification of the marital-residence-sale-proceeds provision of the divorce decree based on his reduced income

21

FEDERAL CIVIL PROCEDURE ANALYSIS (Federal Civil Procedure III IVC)

ANALYSIS

Legal Problems

(1) Is the logging company entitled to join this action as a matter of right

(2)(a) May the nonprofit organization obtain a temporary restraining order to stop the USFS from issuing a logging permit

(2)(b) May the nonprofit organization obtain a preliminary injunction to stop the USFS from issuing a logging permit during the pendency of the action

DISCUSSION

Summary

The logging company is entitled to intervene in this action as a matter of right because it has an interest in the property or transaction that is the subject of the action and is so situated that its interest may be impaired or impeded as a practical matter if the action goes forward without it The logging companyrsquos interest is not adequately represented by the USFSrsquos presence in the lawsuit

The nonprofit organization may seek a temporary restraining order (TRO) followed by a preliminary injunction to prevent the USFS from issuing a logging permit pending the outcome of the action The nonprofit is likely to obtain a TRO if it can demonstrate a risk of immediate and irreparable injury The nonprofit is also likely to obtain a preliminary injunction if it can demonstrate a significant threat of irreparable harm and a likelihood of success on the merits of its National Environmental Policy Act (NEPA) claim

Point One (50) Rule 24(a) of the Federal Rules of Civil Procedure requires federal courts to allow a person to intervene in an action as a matter of right if the person a) is interested in the property or transaction that is the subject of the action b) is so situated that its interest may be impaired or impeded if the litigation goes forward without it and c) is not adequately represented by existing parties Here the logging company likely meets all three requirements and should be allowed to intervene as a matter of right

Rule 24 of the Federal Rules of Civil Procedure governs intervention the process by which a non-party to an action may join the litigation Under Rule 24(a) (intervention of right) a person must be permitted to intervene if three conditions are met (1) the movant ldquoclaims an interest relating to the property or transaction that is the subject of the actionrdquo (2) the movant ldquois so situated that disposition of the action may as a practical matter impair or impede the movantrsquos ability to protect its interestrdquo and (3) ldquoexisting partiesrdquo do not ldquoadequately represent [the movantrsquos] interestrdquo FED R CIV P 24(a) The three requirements for intervention of right are often ldquovery interrelatedrdquo 7C CHARLES ALAN WRIGHT ET AL FEDERAL PRACTICE AND PROCEDURE sect 1908 at 297 (2007 amp 2011 Supp)

22

Federal Civil Procedure Analysis

Here the court should find that the logging company meets this test First the logging company has a strong interest in the property or transaction that is the subject of this action The USFS has accepted the logging companyrsquos bid and the logging company is merely awaiting issuance of a logging permit to begin logging The nonprofit organization is seeking to prevent this logging The logging company therefore has a strong direct and substantial interest in the subject matter of the lawsuit and in having its winning bid honored and a logging permit issued See eg Kleissler v US Forest Serv 157 F3d 964 972 (3d Cir 1998) (stating that ldquo[t]imber companies have direct and substantial interests in a lawsuit aimed at halting loggingrdquo) see also Natural Resources Defense Council v US Nuclear Regulatory Commrsquon 578 F2d 1341 1343ndash 44 (10th Cir 1978) (holding that applicants whose license renewals were pending had Rule 24(a)(2) interests where the lawsuit sought to halt the license-issuing process pending preparation of environmental impact statements) See generally 7C WRIGHT ET AL supra sect 19081 at 309 (ldquoIf there is a direct substantial legally protectable interest in the proceedings it is clear that this requirement of the rule is satisfiedrdquo) Second the logging companyrsquos interest in receiving a logging permit may well be impaired as a practical matter by the outcome of the lawsuit If the USFS loses the lawsuit it will have to prepare an environmental impact statement before issuing the logging companyrsquos permit This will at a minimum delay the logging companyrsquos ability to exercise its rights and may in the long r un mean that no logging permit is ever issued Intervention of right is not limited to those that would be legally bound as a matter of preclusion doctrine Id sect 19082 at 368 Rather ldquo[t]he rule is satisfied whenever disposition of the present action would put the movant at a practical disadvantage in protecting its interestrdquo Id sect 19082 at 369 Here that condition is easily satisfied See Kleissler 157 F3d at 972 (ldquoTimber companies have direct and substantial interests in a lawsuit aimed at halting logging rdquo)

Given that the logging company has an interest that may be impaired by disposition of the action it should be allowed to intervene unless the court is persuaded that the USFS adequately represents the logging companyrsquos interest See Rule 24(a)(2) 7C WRIGHT ET AL supra sect 1909 Here it could be argued that the USFS adequately represents the logging companyrsquos interest because the USFS presumably wants the court to uphold its development plan and allow it to proceed with issuance of the logging permit which is the same relief that the logging company would seek However whether representation is truly adequate depends upon ldquo[a] discriminating appraisal of the circumstancesrdquo 7C WRIGHT ET AL supra sect 1909 at 440 Although both the government and the logging company wish to avoid the preparation of an environmental impact statement their interests are distinct The USFSrsquos interest is proper management of the national forest system while the logging companyrsquos interest is making a profit from logging the 5000-acre tract The USFSrsquos handling of the litigation is likely to be affected by a variety of policy concerns and political considerations that have nothing to do with the logging companyrsquos purely economic interest in securing the right to cut trees in the Scenic National Forest See eg Kleissler 157 F3d at 973ndash74 (ldquo[T]he government represents numerous complex and conflicting interests in matters of this nature The straightforward business interests asserted by intervenors here may become lost in the thicket of sometimes inconsistent governmental policiesrdquo)

[NOTES (1) Examinees who mistakenly analyze the logging companyrsquos case for joinder under the related but incorrect Rule 19 ldquoRequired Joinder of Partiesrdquo may receive credit Rule 19 allows existing parties to demand joinder of non-parties (or seek dismissal of the case if they canrsquot get it) There is a close relationship between Rule 24 and Rule 19 and both contain a similar standard for determining when ldquointerestedrdquo third parties are ldquoentitledrdquo or ldquorequiredrdquo to be in the lawsuit Indeed the two prongs of the Rule 24 intervention test that are discussed above

23

Federal Civil Procedure Analysis

are nearly identical to the two prongs of the Rule 19(a) required joinder test Examinees who discuss and apply the test should receive credit even if they cite Rule 19 rather than Rule 24

(2) Examinees may discuss permissive joinder Although permissive joinder is a possibility here the question asks only whether the logging company can join the action as a matter of right and a permissive joinder analysis is not responsive to the question To the extent an examinee discusses permissive joinder the analysis will focus on whether the logging company ldquohas a claim or defense that shares with the main action a common question of law or factrdquo FED R CIV P 24(b)(1)(B) The district court also ldquomust consider whether the intervention will unduly delay or prejudice the adjudication of the original partiesrsquo rightsrdquo FED R CIV P 24(b)(3) On our facts the logging companyrsquos claim for the issuance of a logging permit would certainly share common questions of law and fact with the USFSrsquos defense against the nonprofitrsquos claim There are no facts suggesting that the logging companyrsquos presence would unduly delay or otherwise prejudice adjudication of the original action Thus the district court would have discretion to permit the logging company to intervene even if it denied intervention of right]

Point Two(a) (25) The nonprofit organization could seek and would likely obtain a temporary restraining order to stop the USFS from issuing a logging permit pending a hearing on an application for a preliminary injunction

The first type of interim relief the nonprofit could seek to stop the USFS from issuing a logging permit to the logging company is a temporary restraining order (TRO) prohibiting the USFS from issuing the logging permit A TRO can be issued without notice to the adverse party but only in limited circumstances and only for a limited time FED R CIV P 65(b) To secure a TRO without notice the nonprofit would need to submit an affidavit containing specific facts that demonstrate a risk of ldquoimmediate and irreparable injuryrdquo if a permit is issued FED R CIV P 65(b)(1) In deciding whether to grant a TRO courts will also consider the same factors that are relevant in deciding whether to grant a preliminary injunction (eg the moving partyrsquos likelihood of success on the merits the balance of hardships and the public interest) See Point Two(b) infra The TRO would last only long enough for the court to consider and resolve a request by the nonprofit for a preliminary injunction but no longer than 14 days (unless the court extends it for good cause or the adverse party consents to an extension) In addition bond is required

Here the court is likely to grant the nonprofitrsquos request The nonprofit could plausibly claim that cutting down 5000 acres of old-growth forest in an area that is home to the highest concentration of wildlife in the western United States would have ldquoan immediate and irreparablerdquo adverse impact on the environment and cause irreparable harm to the nonprofitrsquos interest in preserving and protecting natural resources including wildlife habitat

Point Two(b) (25) The nonprofit could also seek and would likely obtain a preliminary injunction to stop the USFS which is likely to be granted if the nonprofitrsquos claim that the USFS violated NEPA has a strong basis in fact and law

Because the TRO would be temporary the nonprofit would need to move for a preliminary injunction to prevent the USFS from issuing a logging permit throughout the pendency of the litigation Preliminary injunctions are injunctions that seek to ldquoprotect [the] plaintiff from

24

Federal Civil Procedure Analysis

irreparable injury and to preserve the courtrsquos power to render a meaningful decision after a trial on the meritsrdquo 11A CHARLES ALAN WRIGHT ET AL FEDERAL PRACTICE AND PROCEDURE sect 2947 at 112 (2013) Rule 65 of the Federal Rules of Civil Procedure sets out the procedural requirements for preliminary injunctions Preliminary injunctions may be granted only upon notice to the adverse party FED R CIV P 65(a)(1) and only if the movant ldquogives security in an amount that the court considers proper to pay the costs and damages sustained by any party found to have been wrongfully enjoined or restrainedrdquo FED R CIV P 65(c)

While Rule 65 sets out the procedural requirements for preliminary injunctive relief it does not specify the substantive grounds upon which it may be granted The courtrsquos discretion in ruling upon a motion for a preliminary injunction ldquois exercised in conformity with historic federal equity practicerdquo 11A WRIGHT ET AL supra sect 2947 at 114 The court typically considers four factors

(1) the significance of the threat of irreparable harm to the plaintiff if the injunction is not granted (2) the balance between this harm and the injury that granting the injunction would inflict on the defendant (3) the probability that the plaintiff will succeed on the merits and (4) the public interest

Id sect 2948 at 122ndash24 accord Habitat Educ Center v Bosworth 363 F Supp 2d 1070 1088 (ED Wis 2005) The most important of these factors is the risk of irreparable harm to the plaintiff 11A WRIGHT ET AL supra sect 29481 at 129 If the plaintiff has an adequate remedy at law (eg if money damages can compensate the plaintiff for its loss) then a preliminary injunction will be denied Id sect 29481

Here a court would likely conclude that the potential for environmental damage to the forest creates a significant threat of irreparable harm ldquo[E]nvironmental injury is often irreparable Courts have recognized that logging such as would occur [here] can have longshyterm environmental consequences and thus satisfy the irreparable injury criterionrdquo Habitat Educ Center 363 F Supp 2d at 1089 (citing Idaho Sporting Congress Inc v Alexander 222 F3d 562 569 (9th Cir 2000) (noting that the imminent and continuing logging activities presented ldquoevidence of environmental harm sufficient to tip the balance in favor of injunctive reliefrdquo)) Neighbors of Cuddy Mountain v US Forest Service 137 F3d 1372 1382 (9th Cir 1998) (stating that ldquo[t]he old growth forests plaintiffs seek to protect would if cut take hundreds of years to reproducerdquo) (internal citation omitted)) see also 11C WRIGHT ET AL supra sect 29481 at 151 (noting that ldquoa preliminary injunction has been issued to prevent harm to the environmentrdquo)

The second factor the balance between the harm to the plaintiff and the harm the defendant will suffer if the injunction is issued also appears to support issuance of a preliminary injunction here The USFS will have to wait before it can develop the Scenic National Forest and the logging company may lose money if the delay is prolonged These economic harms could be compensated monetarily if an injunction is issued inappropriately Where ldquoan injunction bond can compensate [the] defendant for any harm the injunction is likely to inflict the balance should be struck in favor of [the] plaintiffrdquo Id sect 29482 at 192 See also Habitat Educ Center 363 F Supp 2d at 1089 (stating that ldquothe relative absence of harmful effects on the Forest Service weighs in favor of granting the injunctionrdquo)

The third factor is the likelihood that the plaintiff will prevail on the merits Although there is limited information concerning the merits of the action the nonprofit alleges that the federal statute (NEPA) requires an environmental impact statement and further states that the USFS created no environmental impact analysis or statement at all Assuming that those

25

Federal Civil Procedure Analysis

allegations are correct it seems plausible to conclude that the nonprofit will be able to show a likelihood of success on the merits

Finally courts deciding whether or not to issue preliminary injunctive relief are to consider the public interest ldquoFocusing on this factor is another way of inquiring whether there are policy considerations that bear on whether the order should issuerdquo 11C WRIGHT ET AL supra sect 29484 at 214 If the court concludes that the nonprofit is likely to succeed on its NEPA claim because the USFS wrongfully failed to conduct an environmental impact assessment it is likely to find that the public interest would be served by restraining the USFS from proceeding with logging in a national forest See Heartwood Inc v US Forest Service 73 F Supp 2d 962 979 (SD Ill 1999) affrsquod on other grounds 230 F3d 947 (7th Cir 2000) (ldquoviolations by federal agencies of NEPArsquos provisions as established by Congress harm the public as well as the environmentrdquo)

Thus a court is very likely to grant a preliminary injunction if it concludes that the nonprofit has a significant likelihood of success on the merits

26

EVIDENCE ANALYSIS (Evidence ID IIA amp C)

ANALYSIS

Legal Problems

(1) Under what circumstances can evidence of prior convictions be used to impeach a witnessrsquos credibility in a civil case

(1)(a) May the inmatersquos credibility be impeached by evidence of a 12-year-old felony drug conviction if he was released from prison 9 years ago

(1)(b) May the inmatersquos credibility be impeached by evidence of an 8-year-old misdemeanor perjury conviction that was punishable by 1 year in jail if he pleaded guilty and was sentenced only to pay a $5000 fine

(1)(c) May the inmatersquos credibility be impeached by evidence of a 7-year-old sexual assault conviction if the inmate is still serving a 10-year prison sentence and the victim was his 13-year-old daughter

(2)(a) May the guardrsquos credibility be impeached by cross-examination regarding specific instances of misconduct (ie lying on his reacutesumeacute) relevant to credibility

(2)(b) May the guardrsquos credibility be impeached by admission of extrinsic evidence (his reacutesumeacute and academic transcript) offered to prove specific instances of misconduct relevant to credibility

DISCUSSION

Summary

Under the Federal Rules of Evidence witnesses can be impeached with evidence of prior convictions andor specific instances of misconduct Whether evidence of prior convictions should be admitted to impeach generally depends on the nature of the crime the amount of time that has passed and (only in criminal cases) whether the ldquowitnessrdquo is the defendant FED R EVID 609(a)

In this civil case evidence of the inmatersquos conviction for distribution of marijuana should be admitted to impeach the inmate because he was convicted of a felony and was released from prison fewer than 10 years ago FED R EVID 609(a)(1) Credibility is critically important in this case because the jury will hear conflicting testimony from the two disputing parties and there were no other eyewitnesses to the altercation Under Rule 609(a)(1) the inmatersquos conviction should be admitted because it has some bearing on his credibility and its probative value is not substantially outweighed by concerns of unfair prejudice confusion or delay Id

Evidence of the inmatersquos misdemeanor conviction for perjury must be admitted because the crime ldquorequired provingmdashor the witnessrsquos admittingmdasha dishonest act or false statementrdquo by the inmate FED R EVID 609(a)(2)

27

Evidence Analysis

Evidence of the inmatersquos felony conviction for sexual assault should be excluded because its probative value is substantially outweighed by the danger of unfair prejudice to the inmate based on the heinous nature of the crime FED R EVID 609(a)(1) In the alternative the judge could limit the evidence relating to this conviction by excluding details of the inmatersquos crime

In all civil (and criminal) cases witnesses can also be impeached with evidence of specific instances of prior misconduct that did not result in a conviction FED R EVID 608(b) Pursuant to Rule 608(b) misconduct probative of untruthfulness can be inquired into on cross-examination but cannot be proved through extrinsic evidence Id Thus the inmatersquos counsel should be permitted to cross-examine the guard regarding the false statement in the guardrsquos reacutesumeacute However extrinsic evidence of the guardrsquos misconduct (ie the guardrsquos authenticated reacutesumeacute and transcript from the local college) should not be admitted even if the guard denies wrongdoing or refuses to answer cross-examination questions about these matters Id

Point One (10) The Federal Rules of Evidence permit impeachment of witnesses with evidence of prior convictions

Whether convictions should be admitted to impeach generally depends on the nature of the crime the amount of time that has passed and (only in criminal cases) whether the ldquowitnessrdquo is the defendant FED R EVID 609(a) Under Rule 609(a) evidence of prior convictions may be admitted for the purpose of ldquoattacking a witnessrsquos character for truthfulnessrdquo Id

There are two basic types of convictions that can be admitted for the purpose of impeachment

(1) convictions for crimes ldquopunishable by death or by imprisonment for more than one yearrdquo (which generally correlates to ldquofeloniesrdquo) FED R EVID 609(a)(1) and (2) convictions ldquofor any crimes regardless of the punishment if the court can readily determine that establishing the elements of the crime required provingmdashor the witnessrsquos admittingmdasha dishonest act or false statementrdquo FED R EVID 609(a)(2)

Pursuant to Rule 609(a)(1) in civil cases the admission of evidence of a felony conviction is ldquosubject to Rule 403 [which says that a court may exclude relevant evidence if its probative value is substantially outweighed by other factors]rdquo FED R EVID 609(a)(1) However Rule 403 does not protect the witness against admission of prior convictions involving dishonestymdashwhich must be admitted by the court FED R EVID 609(a)(2)

Finally Federal Rule of Evidence 609(b) contains the presumption that a conviction that is more than 10 years old or where more than 10 years has passed since the witnessrsquos release from confinement (whichever is later) should not be admitted unless ldquoits probative value supported by specific facts and circumstances substantially outweighs its prejudicial effectrdquo and the proponent has provided the adverse party with reasonable written notice FED R EVID 609(b)

Point One(a) (25) The court should admit evidence of the inmatersquos 12-year-old felony marijuana distribution conviction

The inmatersquos conviction for marijuana distribution was for a felony punishable by imprisonment for more than one year See FED R EVID 609(a)(1) Moreover although the conviction was 12 years ago the 10-year time limit of Rule 609(b) is not exceeded because that time limit runs

28

Evidence Analysis

from the date of either ldquothe witnessrsquos conviction or release from confinement for it whichever is laterrdquo FED R EVID 609(b) Because the inmate served three years in prison he was released from confinement nine years ago

However pursuant to Rule 609(a)(1) the admission of felony convictions to impeach a witness in a civil case is ldquosubject to Rule 403rdquo FED R EVID 609(a)(1) Neither Rule 609(a) nor the advisory committee notes specify which factors courts should consider when balancing the probative value of a conviction against the dangers identified in Rule 403 (which include (1) unfair prejudice (2) confusion of the issues (3) misleading the jury (4) waste of time or undue delay and (5) needless presentation of cumulative evidence) FED R EVID 403

In this case credibility is very important because the evidence consists primarily of the testimony of the disputing parties and there were no other eyewitnesses to the altercation This enhances the probative value of any evidence bearing on the inmatersquos credibility A court is likely to conclude that the inmatersquos prior felony drug conviction is relevant to his credibility See eg United States v Brito 427 F3d 53 64 (1st Cir 2005) (ldquoPrior drug-trafficking crimes are generally viewed as having some bearing on veracityrdquo) Although the probative value of any conviction diminishes with age see eg United States v Brewer 451 F Supp 50 53 (ED Tenn 1978) the inmatersquos ongoing problems with the law suggest that he has continued (and even escalated) his criminal behavior over the past nine years The court should admit this evidence because its probative value is not substantially outweighed by any Rule 403 concerns Specifically any prejudice to the inmate would be slight because the conviction is unrelated to the altercation at issue and the conviction was not for a heinous crime that might inflame the jury

[NOTE Whether an examinee identifies the jury instruction as containing a ldquoconclusiverdquo or ldquomandatoryrdquo presumption is less important than the examineersquos analysis of the constitutional infirmities]

Point One(b) (15) The court must admit evidence of the inmatersquos eight-year-old misdemeanor conviction because perjury is a crime of dishonesty

Rule 609(a)(2) provides that evidence of a criminal conviction ldquomust be admitted if the court can readily determine that establishing the elements of the crime required provingmdashor the witnessrsquos admittingmdasha dishonest act or false statementrdquo FED R EVID 609(a)(2) The inmatersquos conviction for perjury would have necessarily required proving that the inmate engaged in an act of dishonesty This conviction occurred within the past 10 years so it ldquomust be admittedrdquo because in contrast to Rule 609(a)(1) (discussed in Point One(a)) admission under Rule 609(a)(2) is mandatory and not subject to Rule 403

Point One(c) (20) The court should exclude evidence of the inmatersquos seven-year-old felony sexual assault conviction because the probative value of this evidence is substantially outweighed by the danger of unfair prejudice In the alternative the details of the prior conviction could be excluded

The inmatersquos conviction for felony sexual assault was seven years ago and he has not yet been released from incarceration so Rule 609(a) but not 609(b) is applicable here FED R EVID 609(a) This conviction is therefore admissible to impeach the inmate unless its probative value is substantially outweighed by the danger of unfair prejudice or any other Rule 403 concern Id

29

Evidence Analysis

Sex crimes are generally not considered relevant to credibility see Hopkins v State 639 So 2d 1247 1254 (Miss 1993) so the probative value of this conviction is relatively low Moreover the heinous nature of the inmatersquos crime (sexual assault on his daughter) makes the danger of unfair prejudice to the inmate very high Thus the court should exclude evidence of the conviction because it was for a heinous offense that is likely to inflame the jury and it has little bearing on credibility See eg United States v Beahm 664 F2d 414 419 (4th Cir 1981)

As an alternative to excluding this evidence the judge could minimize the unfair prejudice to the inmate by permitting limited cross-examination but refusing to allow specific questions about the nature of the inmatersquos conviction For example a court could limit cross-examination to the fact that the inmate was convicted of a ldquofelonyrdquo or perhaps that he was convicted of a ldquosexual assaultrdquo without identifying the victim However because evidence of the inmatersquos prior convictions can be admitted solely for the purpose of enabling the jury to assess his credibility and because his two earlier convictions should have already been admitted the court should exclude all evidence of the felony sexual assault conviction

Point Two(a) (15) The court should permit the inmatersquos counsel to cross-examine the guard regarding the false statement in his reacutesumeacute because the guardrsquos misconduct bears on his truthfulness

The inmate wishes to cross-examine the guard about his prior dishonest behaviormdashlying on his reacutesumeacutemdashthat did not involve a criminal conviction Rule 608(b) allows witnesses to be cross-examined about specific instances of prior non-conviction misconduct probative of untruthfulness ldquoin order to attack the witnessrsquos character for truthfulnessrdquo FED R EVID 608(b)

The courtrsquos decision to allow cross-examination about the guardrsquos prior dishonest behavior depends on the probative value of such evidence balanced against the danger of unfair prejudice to the guard or any other Rule 403 concern FED R EVID 403 Here the guardrsquos false statement on his reacutesumeacute that he obtained a degree in Criminal Justice is highly probative of his untruthfulness because it grossly misrepresents his actual academic record was made recently and was made with the intent to deceive Because the probative value of this evidence is very strong and is not substantially outweighed by any Rule 403 concerns cross-examination of the guard on this topic should be permitted The court may also consider it fair to permit this cross-examination of the guard on these matters assuming that one or more of the inmatersquos prior convictions have been admitted to impeach his credibility

Point Two(b) (15) The court should exclude extrinsic evidence of the guardrsquos non-conviction misconduct even if the guard denies wrongdoing or refuses to answer questions about the matter

Although Rule 608(b) allows cross-examination about specific instances of prior misconduct probative of untruthfulness ldquoextrinsic evidencerdquo offered to prove such misconduct is not admissible FED R EVID 608(b) The rationale for this rule is that allowing the introduction of extrinsic evidence of prior misconduct by witnesses when these acts are relevant only to the witnessesrsquo truthfulness and not to the main issues in the case would create too great a risk of confusing the jury and unduly delaying the trial The court does not have discretion to admit this extrinsic evidence See eg United States v Elliot 89 F3d 1360 1368 (8th Cir 1996)

30

Evidence Analysis

Here the inmatersquos counsel may cross-examine the guard about the false statement on his reacutesumeacute However the inmatersquos counsel must accept the guardrsquos response Even if the guard denies wrongdoing or refuses to answer questions about the matter the inmatersquos counsel cannot introduce the guardrsquos reacutesumeacute or the transcript from the local college to prove the guardrsquos misconduct

31

CORPORATIONS ANALYSIS (Corporations VA2 IX)

ANALYSIS

Legal Problems

(1) Do shareholders have the authority to amend a corporationrsquos bylaws with respect to director nominations

(2) Do board-approved bylaws on a particular subject here nomination of directors preempt subsequent conflicting bylaw amendments by shareholders

(3) Is a suit challenging both managementrsquos refusal to include the proposed bylaw amendment in Megarsquos proxy statement and the boardrsquos amendment of the bylaws dealing with nomination of directors a direct or derivative suit

DISCUSSION

Summary

The voting and litigation rights of the shareholders of Mega are subject to the provisions of the Model Business Corporations Act (MBCA)

The investorrsquos proposed bylaw provision is not inconsistent with state law Under the MBCA shareholders may amend the bylaws when the amendment deals with a proper matter for the corporationrsquos bylaws such as procedures for nominating directors

The Mega boardrsquos bylaw amendment does not preempt the investorrsquos proposed bylaw provision or the Mega shareholdersrsquo power to approve it While shareholders can limit the boardrsquos power to amend or repeal the bylaws the board cannot limit the shareholdersrsquo power

Whether the investor must make a demand on Megarsquos board depends on how the investor frames its claim If the investor claims a violation of shareholder voting rights the claim is direct and pre-suit demand on the board is not required If on the other hand the investor claims that the directors violated their fiduciary duties by amending the bylaws to entrench themselves the claim is derivative and a pre-suit demand is required

Point One (30) Shareholders may amend the corporationrsquos bylaws where the proposed bylaw provision relates to procedural matters typically included in the bylaws such as the nomination of directors

Internal affairs of the corporation such as the conduct of shareholder meetings and election of directors are subject to the corporate law of the state of incorporation See McDermott Inc v Lewis 531 A2d 206 (Del 1987) (applying law of jurisdiction where corporation was incorporated in case involving voting rights) This statersquos corporate statute is modeled on the MBCA

Under the MBCA ldquoshareholders may amend the corporationrsquos bylawsrdquo MBCA sect 1020(a) Thus the only question is whether the bylaws can specify the procedures for shareholder nomination of directors

32

Corporations Analysis

The MBCA states that the bylaws ldquomay contain any provision that is not inconsistent with law or the articles of incorporationrdquo MBCA sect 206(b) In addition the MBCA was revised in 2009 to address shareholder nomination of directors in public corporations (known as ldquoproxy accessrdquo) and specifies that the bylaws ldquomay contain a requirement that the corporation include in its [proxy materials] one or more individuals nominated by a shareholderrdquo MBCA sect 206(c)(1) see Committee on Corporate Laws ABA Section of Business Law Report on the Roles of Boards of Directors and Shareholders of Publicly Owned Corporations and Changes to the Model Business Corporations ActmdashAdoption of Shareholder Proxy Access Amendments to Chapters 2 and 10 65 BUS LAWYER 1105 (2010)

The inclusion of director-nomination procedures in the bylaws is consistent with practice and is recognized by the Delaware courts whose views on corporate law carry significant weight Typically the procedures for nomination of directors are found in the bylaws See 1 COX amp HAZEN TREATISE ON THE LAW OF CORPORATIONS sect 312 (3d ed 2011) see also 4 FLETCHER CORP FORMS ANN PART III ch 21 (2013) (including sample bylaws that permit nomination of directors by shareholders) The Delaware Supreme Court has confirmed that the bylaws may ldquodefine the process and proceduresrdquo for director elections See CA Inc v AFSCME Employees Pension Plan 953 A2d 227 (Del 2008) (concluding that bylaw amendment requiring reimbursement of election expenses to certain successful shareholder nominators is ldquoproper subjectrdquo under Delaware law)

[NOTE The question of the proper scope of the bylaws can be answered using the more general MBCA sect 206(b) or the 2009 MBCA revision adding sect 206(c)(1) (adopted in CT ME VA) In addition some examinees might raise the point that shareholder proposals may not compel the board to take action such as by including shareholder nominations in the companyrsquos proxy materials on the theory that the ldquobusiness and affairsrdquo of the corporation are to be managed by the board See MBCA sect 801(b) Although shareholders are generally limited to adopting precatory resolutions that recommend or encourage board action this limitation does not apply when shareholders have specific authority to take binding action on their ownmdashsuch as to amend the bylaws]

Point Two (30) Shareholders can amend (or repeal) board-approved bylaws Further shareholders can limit the boardrsquos power to later amend and repeal a shareholder-approved bylaw

Under the MBCA shareholders have the power to amend the bylaws See Point One The board shares this power with the shareholders unless (1) the corporationrsquos articles ldquoreserve that power exclusively to the shareholdersrdquo or (2) ldquothe shareholders in amending repealing or adopting a bylaw expressly provide that the board of directors may not amend repeal or reinstate that bylawrdquo See MBCA sect 1020(b)

Shareholder-approved bylaw provisions can amend or repeal existing bylaw provisions whether originally approved by the board or by shareholders See ALAN R PALMITER CORPORATIONS EXAMPLES AND EXPLANATIONS sect 713 (7th ed 2012) Thus the Mega boardrsquos bylaw amendmentmdashwhich set more demanding thresholds for shareholder nomination of directors than the investorrsquos proposed bylaw provisionmdashwould be superseded (repealed) if Megarsquos shareholders were to approve the investorrsquos proposal

Further a shareholder-approved bylaw generally can limit the power of the board to later amend or repeal it See MBCA sect 1020(b)(2) Thus if Megarsquos shareholders approved the bylaw

33

Corporations Analysis

provision proposed by the investor Megarsquos board could not repeal the provision because it includes a ldquono board repealrdquo clause

The revision to the MBCA in 2009 dealing with shareholder proxy access does not change this conclusion That revision specifies that a shareholder-approved bylaw dealing with director nominations may not limit the boardrsquos power to amend add or repeal ldquoany procedure or condition to such a bylaw in order to provide for a reasonable practicable and orderly processrdquo MBCA sect 206(d) Thus according to the revision if shareholders approve a bylaw amendment that limits further board changes the board would nonetheless retain the power to ldquotinkerrdquo with the bylaw to safeguard the voting process but could not repeal the shareholder-approved bylaw The Official Comment to MBCA sect 206(d) makes clear that the revision is ldquonot intended to allow the board of directors to frustrate the purpose of the shareholder-adopted proxy access provisionrdquo Thus if Megarsquos shareholders were to approve the bylaw provision proposed by the investor Megarsquos board could only amend the provision regarding its procedures or conditions in a manner consistent with its purpose of permitting proxy access for Megarsquos shareholders

[NOTE The boardrsquos attempted interference with a shareholder voting initiative may also have been a violation of the directorsrsquo fiduciary duties See Blasius Indus Inc v Atlas Corp 564 A2d 651 (Del Ch 1988) (finding that directors breached their fiduciary duties by amending bylaws and expanding size of board to thwart insurgentrsquos plan to amend bylaws and seat a majority of new directors) The call however asks examinees to consider whether shareholders or the board have ldquoprecedencerdquo over amending the corporate bylaws Thus an examineersquos answer should be framed in terms of ldquopowerrdquo and not ldquodutyrdquo]

Point Three (40) The investor need not make a demand on the board if the investor states a direct claim such as an allegation that the board interfered with the investorrsquos right to amend the bylaws But the investor must make a demand on the board if the investor states a derivative claim (on behalf of the corporation) such as an allegation that the directors sought to entrench themselves by interfering with the proposed proxy access

The MBCA generally requires that shareholders make a demand on the board of directors before initiation of a derivative suit MBCA sect 742 (shareholder may not bring derivative proceeding until written demand has been made on corporation and 90 days have expired) A derivative suit is essentially two suits in one where the plaintiff-shareholder seeks to bring on behalf of the corporation a claim that vindicates corporate rights usually based on violation of fiduciary duties PALMITER supra sect 1811 (6th ed 2009) The demand permits the board to investigate the situation identified by the shareholder and take suitable action No demand on the board is required however if the shareholder brings a direct suit to vindicate the shareholderrsquos own rights not those of the corporation

Is the suit brought by the investor derivative or direct The MBCA defines a ldquoderivative proceedingrdquo as one brought ldquoin the right of a domestic corporationrdquo MBCA sect 740(1) Thus the answer to how the investorrsquos suit should be characterized turns on what rights the investor seeks to vindicate If the investor frames its claim as one of fiduciary breach by directorsmdashfor example for failing to become adequately informed about voting procedures or for seeking to entrench themselves in office by manipulating the voting structure to avoid a shareholder insurgencymdashthen the suit is ldquoderivativerdquo and the investor must make a demand on the board See MBCA Ch 7 Subch D Introductory Comment (ldquothe derivative suit has historically been the principal method of challenging allegedly illegal action by managementrdquo)

34

Corporations Analysis

If however the investor frames its claim as one to vindicate shareholder rights the suit is direct and no demand is required For many courts the direct-derivative question turns on who is injured and who is to receive the relief sought by the plaintiff-shareholders See Tooley v Donaldson Lufkin amp Jenrette Inc 845 A2d 1031 (Del 2004) (characterizing a merger-delay claim as direct because delay of merger only harmed shareholders not corporation) Thus if the investor claims that managementrsquos refusal to include its proposed bylaw amendment in the corporationrsquos proxy materials violates its shareholder rights to initiate corporate governance reforms the suit will be direct Courts have not questioned the ability of shareholders to bring direct suits challenging board action to exclude their proposed bylaw amendments from the corporationrsquos proxy materials See JANA Master Fund Ltd v CNET Networks Inc 954 A2d 335 (Del Ch 2008) (upholding shareholderrsquos direct challenge to boardrsquos interpretation of advance-notice bylaw) Chesapeake Corp v Shore 771 A2d 293 (Del Ch 2000) (upholding shareholderrsquos direct challenge to actions by board that effectively prevented it from proposing bylaw amendments in contest for control)

Is the way that the investor frames its claim conclusive Courts have permitted shareholder-plaintiffs to challenge a transaction in a direct suit even though the same transaction could also be challenged as a fiduciary breach See Eisenberg v Flying Tiger Line Inc 451 F2d 267 (2d Cir 1971) (permitting direct suit challenging a corporate reorganization as a dilution of shareholder voting power even though reorganization may have involved conflicts of interest and thus constituted a fiduciary breach) Thus the investorrsquos choice to pursue a claim challenging the legality of managementrsquos decision to exclude the investorrsquos proposal from the corporationrsquos proxy materialsmdashrather than a possible breach of fiduciary dutymdashis likely to be respected See 3 COX amp HAZEN supra sect 153 (describing situations in which a claim can be framed as derivative or direct)

[NOTE Some issues under Delaware corporate law regarding pre-suit demand are not relevant here For example whether the Mega directors are independent and disinterested is not relevant to the MBCA requirement of a pre-suit demand As the Official Comment to MBCA sect 742 points out the MBCArsquos requirement of ldquouniversal demandrdquo gives the board ldquothe opportunity to reexamine the act complained of in the light of a potential lawsuit and take corrective actionrdquo even when the directors might be non-independent or have conflicts of interest

Nor is it relevant to the MBCA pre-suit demand requirement that the statutory 90-day waiting period may be onerous The first paragraph of MBCA sect 742 requires a pre-suit demand without exception the second paragraph of the section imposes a 90-day waiting period before a derivative suit may be brought which can be shortened if the board rejects the demand or ldquoirreparable injury to the corporation would result by waiting for the expiration of the 90-day periodrdquo The call as written asks only whether a pre-suit demand should be made and does not ask examinees to address whether the post-demand waiting period should be shortened under the ldquoirreparable injuryrdquo standard]

35

National Conference of Bar Examiners 302 South Bedford Street | Madison WI 53703-3622 Phone 608-280-8550 | Fax 608-280-8552 | TDD 608-661-1275

wwwncbexorg e-mail contactncbexorg

  • Preface
  • Description of the MEE
  • Instructions
  • July 2014 Questions
    • CRIMINAL LAW AND PROCEDURE QUESTION
    • CONTRACTS QUESTION
    • FAMILY LAW QUESTION
    • FEDERAL CIVIL PROCEDURE QUESTION
    • EVIDENCE QUESTION
    • CORPORATIONS QUESTION
      • July 2014 Analyses
        • CRIMINAL LAW AND PROCEDURE ANALYSIS
        • CONTRACTS ANALYSIS
        • FAMILY LAW ANALYSIS
        • FEDERAL CIVIL PROCEDURE ANALYSIS
        • EVIDENCE ANALYSIS
        • CORPORATIONS ANALYSIS
            • ltlt13 ASCII85EncodePages false13 AllowTransparency false13 AutoPositionEPSFiles true13 AutoRotatePages None13 Binding Left13 CalGrayProfile (Dot Gain 20)13 CalRGBProfile (sRGB IEC61966-21)13 CalCMYKProfile (US Web Coated 050SWOP051 v2)13 sRGBProfile (sRGB IEC61966-21)13 CannotEmbedFontPolicy Error13 CompatibilityLevel 1413 CompressObjects Tags13 CompressPages true13 ConvertImagesToIndexed true13 PassThroughJPEGImages true13 CreateJobTicket false13 DefaultRenderingIntent Default13 DetectBlends true13 DetectCurves 0000013 ColorConversionStrategy CMYK13 DoThumbnails false13 EmbedAllFonts true13 EmbedOpenType false13 ParseICCProfilesInComments true13 EmbedJobOptions true13 DSCReportingLevel 013 EmitDSCWarnings false13 EndPage -113 ImageMemory 104857613 LockDistillerParams false13 MaxSubsetPct 10013 Optimize true13 OPM 113 ParseDSCComments true13 ParseDSCCommentsForDocInfo true13 PreserveCopyPage true13 PreserveDICMYKValues true13 PreserveEPSInfo true13 PreserveFlatness true13 PreserveHalftoneInfo false13 PreserveOPIComments true13 PreserveOverprintSettings true13 StartPage 113 SubsetFonts true13 TransferFunctionInfo Apply13 UCRandBGInfo Preserve13 UsePrologue false13 ColorSettingsFile ()13 AlwaysEmbed [ true13 ]13 NeverEmbed [ true13 ]13 AntiAliasColorImages false13 CropColorImages true13 ColorImageMinResolution 30013 ColorImageMinResolutionPolicy OK13 DownsampleColorImages true13 ColorImageDownsampleType Bicubic13 ColorImageResolution 30013 ColorImageDepth -113 ColorImageMinDownsampleDepth 113 ColorImageDownsampleThreshold 15000013 EncodeColorImages true13 ColorImageFilter DCTEncode13 AutoFilterColorImages true13 ColorImageAutoFilterStrategy JPEG13 ColorACSImageDict ltlt13 QFactor 01513 HSamples [1 1 1 1] VSamples [1 1 1 1]13 gtgt13 ColorImageDict ltlt13 QFactor 01513 HSamples [1 1 1 1] VSamples [1 1 1 1]13 gtgt13 JPEG2000ColorACSImageDict ltlt13 TileWidth 25613 TileHeight 25613 Quality 3013 gtgt13 JPEG2000ColorImageDict ltlt13 TileWidth 25613 TileHeight 25613 Quality 3013 gtgt13 AntiAliasGrayImages false13 CropGrayImages true13 GrayImageMinResolution 30013 GrayImageMinResolutionPolicy OK13 DownsampleGrayImages true13 GrayImageDownsampleType Bicubic13 GrayImageResolution 30013 GrayImageDepth -113 GrayImageMinDownsampleDepth 213 GrayImageDownsampleThreshold 15000013 EncodeGrayImages true13 GrayImageFilter DCTEncode13 AutoFilterGrayImages true13 GrayImageAutoFilterStrategy JPEG13 GrayACSImageDict ltlt13 QFactor 01513 HSamples [1 1 1 1] VSamples [1 1 1 1]13 gtgt13 GrayImageDict ltlt13 QFactor 01513 HSamples [1 1 1 1] VSamples [1 1 1 1]13 gtgt13 JPEG2000GrayACSImageDict ltlt13 TileWidth 25613 TileHeight 25613 Quality 3013 gtgt13 JPEG2000GrayImageDict ltlt13 TileWidth 25613 TileHeight 25613 Quality 3013 gtgt13 AntiAliasMonoImages false13 CropMonoImages true13 MonoImageMinResolution 120013 MonoImageMinResolutionPolicy OK13 DownsampleMonoImages true13 MonoImageDownsampleType Bicubic13 MonoImageResolution 120013 MonoImageDepth -113 MonoImageDownsampleThreshold 15000013 EncodeMonoImages true13 MonoImageFilter CCITTFaxEncode13 MonoImageDict ltlt13 K -113 gtgt13 AllowPSXObjects false13 CheckCompliance [13 None13 ]13 PDFX1aCheck false13 PDFX3Check false13 PDFXCompliantPDFOnly false13 PDFXNoTrimBoxError true13 PDFXTrimBoxToMediaBoxOffset [13 00000013 00000013 00000013 00000013 ]13 PDFXSetBleedBoxToMediaBox true13 PDFXBleedBoxToTrimBoxOffset [13 00000013 00000013 00000013 00000013 ]13 PDFXOutputIntentProfile ()13 PDFXOutputConditionIdentifier ()13 PDFXOutputCondition ()13 PDFXRegistryName ()13 PDFXTrapped False1313 CreateJDFFile false13 Description ltlt13 ARA ltFEFF06270633062A062E062F0645002006470630064700200627064406250639062F0627062F0627062A002006440625064606340627062100200648062B062706260642002000410064006F00620065002000500044004600200645062A064806270641064206290020064406440637062806270639062900200641064A00200627064406450637062706280639002006300627062A0020062F0631062C0627062A002006270644062C0648062F0629002006270644063906270644064A0629061B0020064A06450643064600200641062A062D00200648062B0627062606420020005000440046002006270644064506460634062306290020062806270633062A062E062F062706450020004100630072006F0062006100740020064800410064006F006200650020005200650061006400650072002006250635062F0627063100200035002E0030002006480627064406250635062F062706310627062A0020062706440623062D062F062B002E0635062F0627063100200035002E0030002006480627064406250635062F062706310627062A0020062706440623062D062F062B002Egt13 BGR ltFEFF04180437043f043e043b043704320430043904420435002004420435043704380020043d0430044104420440043e0439043a0438002c00200437043000200434043000200441044a0437043404300432043004420435002000410064006f00620065002000500044004600200434043e043a0443043c0435043d04420438002c0020043c0430043a04410438043c0430043b043d043e0020043f044004380433043e04340435043d04380020043704300020043204380441043e043a043e043a0430044704350441044204320435043d0020043f04350447043004420020043704300020043f044004350434043f0435044704300442043d04300020043f043e04340433043e0442043e0432043a0430002e002000200421044a04370434043004340435043d043804420435002000500044004600200434043e043a0443043c0435043d044204380020043c043e0433043004420020043404300020044104350020043e0442043204300440044f0442002004410020004100630072006f00620061007400200438002000410064006f00620065002000520065006100640065007200200035002e00300020043800200441043b0435043404320430044904380020043204350440044104380438002egt13 CHS ltFEFF4f7f75288fd94e9b8bbe5b9a521b5efa7684002000410064006f006200650020005000440046002065876863900275284e8e9ad88d2891cf76845370524d53705237300260a853ef4ee54f7f75280020004100630072006f0062006100740020548c002000410064006f00620065002000520065006100640065007200200035002e003000204ee553ca66f49ad87248672c676562535f00521b5efa768400200050004400460020658768633002gt13 CHT ltFEFF4f7f752890194e9b8a2d7f6e5efa7acb7684002000410064006f006200650020005000440046002065874ef69069752865bc9ad854c18cea76845370524d5370523786557406300260a853ef4ee54f7f75280020004100630072006f0062006100740020548c002000410064006f00620065002000520065006100640065007200200035002e003000204ee553ca66f49ad87248672c4f86958b555f5df25efa7acb76840020005000440046002065874ef63002gt13 CZE 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 DAN ltFEFF004200720075006700200069006e0064007300740069006c006c0069006e006700650072006e0065002000740069006c0020006100740020006f007000720065007400740065002000410064006f006200650020005000440046002d0064006f006b0075006d0065006e007400650072002c0020006400650072002000620065006400730074002000650067006e006500720020007300690067002000740069006c002000700072006500700072006500730073002d007500640073006b007200690076006e0069006e00670020006100660020006800f8006a0020006b00760061006c0069007400650074002e0020004400650020006f007000720065007400740065006400650020005000440046002d0064006f006b0075006d0065006e0074006500720020006b0061006e002000e50062006e00650073002000690020004100630072006f00620061007400200065006c006c006500720020004100630072006f006200610074002000520065006100640065007200200035002e00300020006f00670020006e0079006500720065002egt13 DEU 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 ESP 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 ETI 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 FRA 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 GRE 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 HEB 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 HRV (Za stvaranje Adobe PDF dokumenata najpogodnijih za visokokvalitetni ispis prije tiskanja koristite ove postavke Stvoreni PDF dokumenti mogu se otvoriti Acrobat i Adobe Reader 50 i kasnijim verzijama)13 HUN ltFEFF004b0069007600e1006c00f30020006d0069006e0151007300e9006701710020006e0079006f006d00640061006900200065006c0151006b00e90073007a00ed007401510020006e0079006f006d00740061007400e100730068006f007a0020006c006500670069006e006b00e1006200620020006d0065006700660065006c0065006c0151002000410064006f00620065002000500044004600200064006f006b0075006d0065006e00740075006d006f006b0061007400200065007a0065006b006b0065006c0020006100200062006500e1006c006c00ed007400e10073006f006b006b0061006c0020006b00e90073007a00ed0074006800650074002e0020002000410020006c00e90074007200650068006f007a006f00740074002000500044004600200064006f006b0075006d0065006e00740075006d006f006b00200061007a0020004100630072006f006200610074002000e9007300200061007a002000410064006f00620065002000520065006100640065007200200035002e0030002c0020007600610067007900200061007a002000610074007400f3006c0020006b00e9007301510062006200690020007600650072007a006900f3006b006b0061006c0020006e00790069007400680061007400f3006b0020006d00650067002egt13 ITA ltFEFF005500740069006c0069007a007a006100720065002000710075006500730074006500200069006d0070006f007300740061007a0069006f006e00690020007000650072002000630072006500610072006500200064006f00630075006d0065006e00740069002000410064006f00620065002000500044004600200070006900f900200061006400610074007400690020006100200075006e00610020007000720065007300740061006d0070006100200064006900200061006c007400610020007100750061006c0069007400e0002e0020004900200064006f00630075006d0065006e007400690020005000440046002000630072006500610074006900200070006f00730073006f006e006f0020006500730073006500720065002000610070006500720074006900200063006f006e0020004100630072006f00620061007400200065002000410064006f00620065002000520065006100640065007200200035002e003000200065002000760065007200730069006f006e006900200073007500630063006500730073006900760065002egt13 JPN ltFEFF9ad854c18cea306a30d730ea30d730ec30b951fa529b7528002000410064006f0062006500200050004400460020658766f8306e4f5c6210306b4f7f75283057307e305930023053306e8a2d5b9a30674f5c62103055308c305f0020005000440046002030d530a130a430eb306f3001004100630072006f0062006100740020304a30883073002000410064006f00620065002000520065006100640065007200200035002e003000204ee5964d3067958b304f30533068304c3067304d307e305930023053306e8a2d5b9a306b306f30d530a930f330c8306e57cb30818fbc307f304c5fc59808306730593002gt13 KOR ltFEFFc7740020c124c815c7440020c0acc6a9d558c5ec0020ace0d488c9c80020c2dcd5d80020c778c1c4c5d00020ac00c7a50020c801d569d55c002000410064006f0062006500200050004400460020bb38c11cb97c0020c791c131d569b2c8b2e4002e0020c774b807ac8c0020c791c131b41c00200050004400460020bb38c11cb2940020004100630072006f0062006100740020bc0f002000410064006f00620065002000520065006100640065007200200035002e00300020c774c0c1c5d0c11c0020c5f40020c2180020c788c2b5b2c8b2e4002egt13 LTH 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 LVI 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 NLD (Gebruik deze instellingen om Adobe PDF-documenten te maken die zijn geoptimaliseerd voor prepress-afdrukken van hoge kwaliteit De gemaakte PDF-documenten kunnen worden geopend met Acrobat en Adobe Reader 50 en hoger)13 NOR 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 POL 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 PTB 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 RUM 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 RUS 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 SKY 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 SLV 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 SUO 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 SVE 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 TUR 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 UKR ltFEFF04120438043a043e0440043804410442043e043204430439044204350020044604560020043f043004400430043c043504420440043800200434043b044f0020044104420432043e04400435043d043d044f00200434043e043a0443043c0435043d044204560432002000410064006f006200650020005000440046002c0020044f043a04560020043d04300439043a04400430044904350020043f045604340445043e0434044f0442044c00200434043b044f0020043204380441043e043a043e044f043a04560441043d043e0433043e0020043f0435044004350434043404400443043a043e0432043e0433043e0020043404400443043a0443002e00200020042104420432043e04400435043d045600200434043e043a0443043c0435043d0442043800200050004400460020043c043e0436043d04300020043204560434043a0440043804420438002004430020004100630072006f006200610074002004420430002000410064006f00620065002000520065006100640065007200200035002e0030002004300431043e0020043f04560437043d04560448043e04570020043204350440044104560457002egt13 ENU (Use these settings to create Adobe PDF documents best suited for high-quality prepress printing Created PDF documents can be opened with Acrobat and Adobe Reader 50 and later)13 gtgt13 Namespace [13 (Adobe)13 (Common)13 (10)13 ]13 OtherNamespaces [13 ltlt13 AsReaderSpreads false13 CropImagesToFrames true13 ErrorControl WarnAndContinue13 FlattenerIgnoreSpreadOverrides false13 IncludeGuidesGrids false13 IncludeNonPrinting false13 IncludeSlug false13 Namespace [13 (Adobe)13 (InDesign)13 (40)13 ]13 OmitPlacedBitmaps false13 OmitPlacedEPS false13 OmitPlacedPDF false13 SimulateOverprint Legacy13 gtgt13 ltlt13 AddBleedMarks false13 AddColorBars false13 AddCropMarks false13 AddPageInfo false13 AddRegMarks false13 ConvertColors ConvertToCMYK13 DestinationProfileName ()13 DestinationProfileSelector DocumentCMYK13 Downsample16BitImages true13 FlattenerPreset ltlt13 PresetSelector MediumResolution13 gtgt13 FormElements false13 GenerateStructure false13 IncludeBookmarks false13 IncludeHyperlinks false13 IncludeInteractive false13 IncludeLayers false13 IncludeProfiles false13 MultimediaHandling UseObjectSettings13 Namespace [13 (Adobe)13 (CreativeSuite)13 (20)13 ]13 PDFXOutputIntentProfileSelector DocumentCMYK13 PreserveEditing true13 UntaggedCMYKHandling LeaveUntagged13 UntaggedRGBHandling UseDocumentProfile13 UseDocumentBleed false13 gtgt13 ]13gtgt setdistillerparams13ltlt13 HWResolution [2400 2400]13 PageSize [612000 792000]13gtgt setpagedevice13

Page 20: July 2014 MEE Questions and Analyses - NCBE...This publication includes the questions and analyses from the July 2014 MEE. (In the actual test, the questions are simply numbered rather

Contracts Analysis

UCC sect 2-209 Here because the businessrsquos financial reversals were serious and apparently unanticipated at the time that the business entered into the contract with the conservatory and commitment of the extra money was needed to enable the business to perform a court would likely find that the business acted in good faith Thus a court would likely uphold the enforceability of the conservatoryrsquos promise to pay the additional $40000

Point Three (10) The conservatory is unlikely to be able to defend against enforcement of its promises to pay additional money under the theory of economic duress because the business probably did not make an improper threat

Under the common law of contracts parties may raise the defense of duress This common law defense also applies to contracts governed by UCC Article 2 See UCC sect 1-103(b)

A contract is voidable on the ground of economic duress by threat when it is established that a partyrsquos manifestation of assent is induced by an improper threat that leaves the party no reasonable alternative See RESTATEMENT (SECOND) OF CONTRACTS sect 175 See also eg Austin Instrument Inc v Loral Corp 272 NE2d 533 (NY 1971) (a threat to withhold essential goods can constitute duress) In order to void its agreement to pay the additional sum because of economic duress the conservatory must demonstrate that (1) the business made a threat to the conservatory (2) the threat was ldquoimproperrdquo or ldquowrongfulrdquo (3) the threat induced the conservatoryrsquos manifestation of assent to the modification and (4) the threat was sufficiently grave to justify the conservatoryrsquos assent

Here it appears that three of the four elements are likely satisfied The business plainly made a threat Moreover the threat induced the conservatoryrsquos assent to the modification and the threat was sufficiently grave to justify that assent If the conservatory had not agreed to pay the business the extra amounts the conservatory would have lost its entire $325000 investment In light of this potential loss a court could easily conclude that the conservatory had no reasonable alternative

However the business has a strong argument that its threat (indicating that it would breach the contracts unless the prices were increased) was not wrongful or improper but was instead nothing more than a communication of the reality of its own perilous situation to the conservatory

A mere threat to breach a contract is not in and of itself improper so as to support an action of economic duress or business compulsion Something more is required such as a breach of the duty of good faith and fair dealing as was present in Austin Instrument Inc supra Because the business could not perform the original contract without the requested modification the economic duress claim for the conservatory would likely fail for much the same reason that the business would be able to enforce the modification At the time the modification was requested the business was not trying to extort a price increase because of the conservatoryrsquos vulnerability but instead was simply stating the reality that the business could not perform without more money

18

FAMILY LAW ANALYSIS (Family Law IIIB D amp G)

ANALYSIS

Legal Problems

(1)(a) Does the State A court have jurisdiction to modify the State B child support order

(1)(b) Does the State A court have jurisdiction to modify the marital-residence-saleshyproceeds provision of the State B property-division decree

(2)(a) May a child support order be modified retroactively

(2)(b) May a child support order be modified prospectively based on a change of employment with a lower salary

(2)(c) May a property-division order be modified after entry of a divorce decree

DISCUSSION

Summary

The State A court may exercise personal jurisdiction over the wife because she was personally served in State A However subject-matter jurisdiction over the interstate modification of child support is governed by the Uniform Interstate Family Support Act (UIFSA) Under UIFSA State A does not have jurisdiction to modify the order for the daughterrsquos support because the wife is still a resident of State B UIFSA on the other hand does not govern property distributions and thus a State A court is not precluded from hearing the husbandrsquos petition to modify the marital-residence-sale-proceeds provision of the divorce decree

A child support order may not be modified retroactively A child support order may be modified prospectively based on a substantial change in circumstances Courts agree that a significant decrease in income is a substantial change in circumstances All states treat voluntary income reductions differently than involuntary reductions but employ different approaches for evaluating the impact of a voluntary reduction Whether the husband could obtain prospective modification of the child support order depends on which approach is applied

A property-division order is not subject to post-divorce modification based on a change in circumstances Thus the husband may in some states obtain prospective modification of the order for the daughterrsquos support but he may not obtain modification of the marital-residenceshysale-proceeds provision

Point One(a) (25) Personal jurisdiction over a nonresident respondent does not confer subject-matter jurisdiction over child support modification Under UIFSA a State A court may not modify a child support order issued by a State B court when as here the child or either parent continues to reside in State B the jurisdiction that issued the child support order

The State A court may exercise personal jurisdiction over the wife The wife was personally served in State A and a state may exercise jurisdiction based on in-state personal service See

19

Family Law Analysis

Burnham v Superior Court 495 US 604 (1990) But personal jurisdiction over the wife is not enough to give a State A court jurisdiction to modify the State B support order

The interstate enforcement and modification of child support is governed by the Uniform Interstate Family Support Act (UIFSA) which has been adopted by all states Under UIFSA the state that originally issued a child support order (here State B) has continuing exclusive jurisdiction to modify the order if that state remains the residence of the obligee the child or the obligor and all parties do not consent to the jurisdiction of another forum See UIFSA sect 205 See also UIFSA sect 603 (ldquoA tribunal of this State shall recognize and enforce but may not modify a registered order if the issuing tribunal had jurisdictionrdquo) The wife and daughter continue to reside in State B and the wife has not consented to the jurisdiction of another forum Thus a State A court does not have jurisdiction to modify the State B child support order

[NOTE Examinees who do not discuss personal jurisdiction but fully discuss UIFSA may receive full credit]

Point One(b) (15) UIFSA does not apply to disputes over property division Thus the State A court may exercise jurisdiction over the husbandrsquos petition to modify the marital-residence-sale-proceeds provision of the State B divorce decree because it has personal jurisdiction over the wife

The State A court in which the husband brought his action has jurisdiction to adjudicate domestic relations issues The husbandrsquos petition to modify the property settlement is a domestic relations issue The courts of State A may exercise personal jurisdiction over the wife because she was personally served in State A See Burnham v Superior Court 495 US 604 (1990) see Point One(a)

UIFSA does not apply to divorce property-division disputes Thus although a State A court may not adjudicate the husbandrsquos petition to modify his child support obligations it may adjudicate his property-division claims (Even though the court has jurisdiction it may not modify the property-division award on the merits See Point Two(c))

Point Two(a) (20) A child support order may not be modified retroactively

State courts have long held that obligations to pay child support ordinarily may not be modified retroactively ldquoIf the hardship is particularly severe the courts sometimes devised a way to protect the obligor but in most instances the courts hold that retroactive modification of this kind is beyond their power and indeed the governing statute may so providerdquo HOMER H CLARK THE LAW OF DOMESTIC RELATIONSHIPS IN THE UNITED STATES 725 (2d ed 1987)

Federal law now goes further and requires the states as a condition of federal child-support funding to adopt rules that absolutely forbid retroactive modification of the support obligation See 42 USC sect 666(a)(9)(C) The states have adopted rules consistent with the federal requirements

Point Two(b) (25) It is unclear whether the husband could obtain prospective downward modification of his child support based on his voluntary acceptance of a job with a lower salary

Prospective modification of a child support order is typically available only when the petitioner can show a substantial change in circumstances See ROBERT E OLIPHANT amp NANCY VER

20

Family Law Analysis

STEEGH FAMILY LAW 213ndash15 (3d ed 2010) A significant decrease in income is typically viewed as a substantial change

However when a parent seeks to modify a child support obligation because he has voluntarily reduced his income a court will not modify the obligation based solely on the income loss Some courts refuse to modify whenever the income shift was voluntary See eg Aguiar v Aguiar 127 P3d 234 (Idaho Ct App 2005) Others look primarily to the petitionerrsquos intentions and permit downward modification if he has acted in good faith See eg In re Marriage of Horn 650 NE2d 1103 (Ill App Ct 1995) Many courts use a multifactor approach See OLIPHANT amp VER STEEGH supra 217ndash18

Here there is no question that the husbandrsquos loss of income was voluntary In a jurisdiction in which voluntary income reduction bars support modification the husbandrsquos petition would be denied

In a jurisdiction employing a good-faith or multifactor approach it is possible but not certain that the husband could obtain downward modification The evidence supports the husbandrsquos good faith his change in employment appears to be based on his new jobrsquos greater responsibilities and better promotion possibilities In a jurisdiction using a multifactor approach the court would likely also consider the impact of such a shift on the daughter the likely duration of the husbandrsquos income loss and the likelihood of a promotion that would ultimately inure to the daughterrsquos benefit Thus on these facts it is possible but by no means certain that the husband could prospectively obtain downward modification of his child support obligation to his daughter

Point Two(c)(15) A divorce property-division award is not subject to modification

A support order is aimed at meeting the post-divorce needs of the supported individual Because the future is unpredictable courts are empowered to modify a support award to take account of changed circumstances that may occur during the period in which support is paid

By contrast a property-distribution award divides assets of the marriage based on the equities at the time of divorce Because the past can be ascertained a property-division award is not subject to post-divorce modification See HARRY A KRAUSE ET AL FAMILY LAW CASES COMMENTS AND QUESTIONS 691 (6th ed 2007)

Here the husband is seeking modification of a property-division award with respect to an asset owned by the parties at the time of divorce Thus the husband may not obtain a modification of the marital-residence-sale-proceeds provision of the divorce decree based on his reduced income

21

FEDERAL CIVIL PROCEDURE ANALYSIS (Federal Civil Procedure III IVC)

ANALYSIS

Legal Problems

(1) Is the logging company entitled to join this action as a matter of right

(2)(a) May the nonprofit organization obtain a temporary restraining order to stop the USFS from issuing a logging permit

(2)(b) May the nonprofit organization obtain a preliminary injunction to stop the USFS from issuing a logging permit during the pendency of the action

DISCUSSION

Summary

The logging company is entitled to intervene in this action as a matter of right because it has an interest in the property or transaction that is the subject of the action and is so situated that its interest may be impaired or impeded as a practical matter if the action goes forward without it The logging companyrsquos interest is not adequately represented by the USFSrsquos presence in the lawsuit

The nonprofit organization may seek a temporary restraining order (TRO) followed by a preliminary injunction to prevent the USFS from issuing a logging permit pending the outcome of the action The nonprofit is likely to obtain a TRO if it can demonstrate a risk of immediate and irreparable injury The nonprofit is also likely to obtain a preliminary injunction if it can demonstrate a significant threat of irreparable harm and a likelihood of success on the merits of its National Environmental Policy Act (NEPA) claim

Point One (50) Rule 24(a) of the Federal Rules of Civil Procedure requires federal courts to allow a person to intervene in an action as a matter of right if the person a) is interested in the property or transaction that is the subject of the action b) is so situated that its interest may be impaired or impeded if the litigation goes forward without it and c) is not adequately represented by existing parties Here the logging company likely meets all three requirements and should be allowed to intervene as a matter of right

Rule 24 of the Federal Rules of Civil Procedure governs intervention the process by which a non-party to an action may join the litigation Under Rule 24(a) (intervention of right) a person must be permitted to intervene if three conditions are met (1) the movant ldquoclaims an interest relating to the property or transaction that is the subject of the actionrdquo (2) the movant ldquois so situated that disposition of the action may as a practical matter impair or impede the movantrsquos ability to protect its interestrdquo and (3) ldquoexisting partiesrdquo do not ldquoadequately represent [the movantrsquos] interestrdquo FED R CIV P 24(a) The three requirements for intervention of right are often ldquovery interrelatedrdquo 7C CHARLES ALAN WRIGHT ET AL FEDERAL PRACTICE AND PROCEDURE sect 1908 at 297 (2007 amp 2011 Supp)

22

Federal Civil Procedure Analysis

Here the court should find that the logging company meets this test First the logging company has a strong interest in the property or transaction that is the subject of this action The USFS has accepted the logging companyrsquos bid and the logging company is merely awaiting issuance of a logging permit to begin logging The nonprofit organization is seeking to prevent this logging The logging company therefore has a strong direct and substantial interest in the subject matter of the lawsuit and in having its winning bid honored and a logging permit issued See eg Kleissler v US Forest Serv 157 F3d 964 972 (3d Cir 1998) (stating that ldquo[t]imber companies have direct and substantial interests in a lawsuit aimed at halting loggingrdquo) see also Natural Resources Defense Council v US Nuclear Regulatory Commrsquon 578 F2d 1341 1343ndash 44 (10th Cir 1978) (holding that applicants whose license renewals were pending had Rule 24(a)(2) interests where the lawsuit sought to halt the license-issuing process pending preparation of environmental impact statements) See generally 7C WRIGHT ET AL supra sect 19081 at 309 (ldquoIf there is a direct substantial legally protectable interest in the proceedings it is clear that this requirement of the rule is satisfiedrdquo) Second the logging companyrsquos interest in receiving a logging permit may well be impaired as a practical matter by the outcome of the lawsuit If the USFS loses the lawsuit it will have to prepare an environmental impact statement before issuing the logging companyrsquos permit This will at a minimum delay the logging companyrsquos ability to exercise its rights and may in the long r un mean that no logging permit is ever issued Intervention of right is not limited to those that would be legally bound as a matter of preclusion doctrine Id sect 19082 at 368 Rather ldquo[t]he rule is satisfied whenever disposition of the present action would put the movant at a practical disadvantage in protecting its interestrdquo Id sect 19082 at 369 Here that condition is easily satisfied See Kleissler 157 F3d at 972 (ldquoTimber companies have direct and substantial interests in a lawsuit aimed at halting logging rdquo)

Given that the logging company has an interest that may be impaired by disposition of the action it should be allowed to intervene unless the court is persuaded that the USFS adequately represents the logging companyrsquos interest See Rule 24(a)(2) 7C WRIGHT ET AL supra sect 1909 Here it could be argued that the USFS adequately represents the logging companyrsquos interest because the USFS presumably wants the court to uphold its development plan and allow it to proceed with issuance of the logging permit which is the same relief that the logging company would seek However whether representation is truly adequate depends upon ldquo[a] discriminating appraisal of the circumstancesrdquo 7C WRIGHT ET AL supra sect 1909 at 440 Although both the government and the logging company wish to avoid the preparation of an environmental impact statement their interests are distinct The USFSrsquos interest is proper management of the national forest system while the logging companyrsquos interest is making a profit from logging the 5000-acre tract The USFSrsquos handling of the litigation is likely to be affected by a variety of policy concerns and political considerations that have nothing to do with the logging companyrsquos purely economic interest in securing the right to cut trees in the Scenic National Forest See eg Kleissler 157 F3d at 973ndash74 (ldquo[T]he government represents numerous complex and conflicting interests in matters of this nature The straightforward business interests asserted by intervenors here may become lost in the thicket of sometimes inconsistent governmental policiesrdquo)

[NOTES (1) Examinees who mistakenly analyze the logging companyrsquos case for joinder under the related but incorrect Rule 19 ldquoRequired Joinder of Partiesrdquo may receive credit Rule 19 allows existing parties to demand joinder of non-parties (or seek dismissal of the case if they canrsquot get it) There is a close relationship between Rule 24 and Rule 19 and both contain a similar standard for determining when ldquointerestedrdquo third parties are ldquoentitledrdquo or ldquorequiredrdquo to be in the lawsuit Indeed the two prongs of the Rule 24 intervention test that are discussed above

23

Federal Civil Procedure Analysis

are nearly identical to the two prongs of the Rule 19(a) required joinder test Examinees who discuss and apply the test should receive credit even if they cite Rule 19 rather than Rule 24

(2) Examinees may discuss permissive joinder Although permissive joinder is a possibility here the question asks only whether the logging company can join the action as a matter of right and a permissive joinder analysis is not responsive to the question To the extent an examinee discusses permissive joinder the analysis will focus on whether the logging company ldquohas a claim or defense that shares with the main action a common question of law or factrdquo FED R CIV P 24(b)(1)(B) The district court also ldquomust consider whether the intervention will unduly delay or prejudice the adjudication of the original partiesrsquo rightsrdquo FED R CIV P 24(b)(3) On our facts the logging companyrsquos claim for the issuance of a logging permit would certainly share common questions of law and fact with the USFSrsquos defense against the nonprofitrsquos claim There are no facts suggesting that the logging companyrsquos presence would unduly delay or otherwise prejudice adjudication of the original action Thus the district court would have discretion to permit the logging company to intervene even if it denied intervention of right]

Point Two(a) (25) The nonprofit organization could seek and would likely obtain a temporary restraining order to stop the USFS from issuing a logging permit pending a hearing on an application for a preliminary injunction

The first type of interim relief the nonprofit could seek to stop the USFS from issuing a logging permit to the logging company is a temporary restraining order (TRO) prohibiting the USFS from issuing the logging permit A TRO can be issued without notice to the adverse party but only in limited circumstances and only for a limited time FED R CIV P 65(b) To secure a TRO without notice the nonprofit would need to submit an affidavit containing specific facts that demonstrate a risk of ldquoimmediate and irreparable injuryrdquo if a permit is issued FED R CIV P 65(b)(1) In deciding whether to grant a TRO courts will also consider the same factors that are relevant in deciding whether to grant a preliminary injunction (eg the moving partyrsquos likelihood of success on the merits the balance of hardships and the public interest) See Point Two(b) infra The TRO would last only long enough for the court to consider and resolve a request by the nonprofit for a preliminary injunction but no longer than 14 days (unless the court extends it for good cause or the adverse party consents to an extension) In addition bond is required

Here the court is likely to grant the nonprofitrsquos request The nonprofit could plausibly claim that cutting down 5000 acres of old-growth forest in an area that is home to the highest concentration of wildlife in the western United States would have ldquoan immediate and irreparablerdquo adverse impact on the environment and cause irreparable harm to the nonprofitrsquos interest in preserving and protecting natural resources including wildlife habitat

Point Two(b) (25) The nonprofit could also seek and would likely obtain a preliminary injunction to stop the USFS which is likely to be granted if the nonprofitrsquos claim that the USFS violated NEPA has a strong basis in fact and law

Because the TRO would be temporary the nonprofit would need to move for a preliminary injunction to prevent the USFS from issuing a logging permit throughout the pendency of the litigation Preliminary injunctions are injunctions that seek to ldquoprotect [the] plaintiff from

24

Federal Civil Procedure Analysis

irreparable injury and to preserve the courtrsquos power to render a meaningful decision after a trial on the meritsrdquo 11A CHARLES ALAN WRIGHT ET AL FEDERAL PRACTICE AND PROCEDURE sect 2947 at 112 (2013) Rule 65 of the Federal Rules of Civil Procedure sets out the procedural requirements for preliminary injunctions Preliminary injunctions may be granted only upon notice to the adverse party FED R CIV P 65(a)(1) and only if the movant ldquogives security in an amount that the court considers proper to pay the costs and damages sustained by any party found to have been wrongfully enjoined or restrainedrdquo FED R CIV P 65(c)

While Rule 65 sets out the procedural requirements for preliminary injunctive relief it does not specify the substantive grounds upon which it may be granted The courtrsquos discretion in ruling upon a motion for a preliminary injunction ldquois exercised in conformity with historic federal equity practicerdquo 11A WRIGHT ET AL supra sect 2947 at 114 The court typically considers four factors

(1) the significance of the threat of irreparable harm to the plaintiff if the injunction is not granted (2) the balance between this harm and the injury that granting the injunction would inflict on the defendant (3) the probability that the plaintiff will succeed on the merits and (4) the public interest

Id sect 2948 at 122ndash24 accord Habitat Educ Center v Bosworth 363 F Supp 2d 1070 1088 (ED Wis 2005) The most important of these factors is the risk of irreparable harm to the plaintiff 11A WRIGHT ET AL supra sect 29481 at 129 If the plaintiff has an adequate remedy at law (eg if money damages can compensate the plaintiff for its loss) then a preliminary injunction will be denied Id sect 29481

Here a court would likely conclude that the potential for environmental damage to the forest creates a significant threat of irreparable harm ldquo[E]nvironmental injury is often irreparable Courts have recognized that logging such as would occur [here] can have longshyterm environmental consequences and thus satisfy the irreparable injury criterionrdquo Habitat Educ Center 363 F Supp 2d at 1089 (citing Idaho Sporting Congress Inc v Alexander 222 F3d 562 569 (9th Cir 2000) (noting that the imminent and continuing logging activities presented ldquoevidence of environmental harm sufficient to tip the balance in favor of injunctive reliefrdquo)) Neighbors of Cuddy Mountain v US Forest Service 137 F3d 1372 1382 (9th Cir 1998) (stating that ldquo[t]he old growth forests plaintiffs seek to protect would if cut take hundreds of years to reproducerdquo) (internal citation omitted)) see also 11C WRIGHT ET AL supra sect 29481 at 151 (noting that ldquoa preliminary injunction has been issued to prevent harm to the environmentrdquo)

The second factor the balance between the harm to the plaintiff and the harm the defendant will suffer if the injunction is issued also appears to support issuance of a preliminary injunction here The USFS will have to wait before it can develop the Scenic National Forest and the logging company may lose money if the delay is prolonged These economic harms could be compensated monetarily if an injunction is issued inappropriately Where ldquoan injunction bond can compensate [the] defendant for any harm the injunction is likely to inflict the balance should be struck in favor of [the] plaintiffrdquo Id sect 29482 at 192 See also Habitat Educ Center 363 F Supp 2d at 1089 (stating that ldquothe relative absence of harmful effects on the Forest Service weighs in favor of granting the injunctionrdquo)

The third factor is the likelihood that the plaintiff will prevail on the merits Although there is limited information concerning the merits of the action the nonprofit alleges that the federal statute (NEPA) requires an environmental impact statement and further states that the USFS created no environmental impact analysis or statement at all Assuming that those

25

Federal Civil Procedure Analysis

allegations are correct it seems plausible to conclude that the nonprofit will be able to show a likelihood of success on the merits

Finally courts deciding whether or not to issue preliminary injunctive relief are to consider the public interest ldquoFocusing on this factor is another way of inquiring whether there are policy considerations that bear on whether the order should issuerdquo 11C WRIGHT ET AL supra sect 29484 at 214 If the court concludes that the nonprofit is likely to succeed on its NEPA claim because the USFS wrongfully failed to conduct an environmental impact assessment it is likely to find that the public interest would be served by restraining the USFS from proceeding with logging in a national forest See Heartwood Inc v US Forest Service 73 F Supp 2d 962 979 (SD Ill 1999) affrsquod on other grounds 230 F3d 947 (7th Cir 2000) (ldquoviolations by federal agencies of NEPArsquos provisions as established by Congress harm the public as well as the environmentrdquo)

Thus a court is very likely to grant a preliminary injunction if it concludes that the nonprofit has a significant likelihood of success on the merits

26

EVIDENCE ANALYSIS (Evidence ID IIA amp C)

ANALYSIS

Legal Problems

(1) Under what circumstances can evidence of prior convictions be used to impeach a witnessrsquos credibility in a civil case

(1)(a) May the inmatersquos credibility be impeached by evidence of a 12-year-old felony drug conviction if he was released from prison 9 years ago

(1)(b) May the inmatersquos credibility be impeached by evidence of an 8-year-old misdemeanor perjury conviction that was punishable by 1 year in jail if he pleaded guilty and was sentenced only to pay a $5000 fine

(1)(c) May the inmatersquos credibility be impeached by evidence of a 7-year-old sexual assault conviction if the inmate is still serving a 10-year prison sentence and the victim was his 13-year-old daughter

(2)(a) May the guardrsquos credibility be impeached by cross-examination regarding specific instances of misconduct (ie lying on his reacutesumeacute) relevant to credibility

(2)(b) May the guardrsquos credibility be impeached by admission of extrinsic evidence (his reacutesumeacute and academic transcript) offered to prove specific instances of misconduct relevant to credibility

DISCUSSION

Summary

Under the Federal Rules of Evidence witnesses can be impeached with evidence of prior convictions andor specific instances of misconduct Whether evidence of prior convictions should be admitted to impeach generally depends on the nature of the crime the amount of time that has passed and (only in criminal cases) whether the ldquowitnessrdquo is the defendant FED R EVID 609(a)

In this civil case evidence of the inmatersquos conviction for distribution of marijuana should be admitted to impeach the inmate because he was convicted of a felony and was released from prison fewer than 10 years ago FED R EVID 609(a)(1) Credibility is critically important in this case because the jury will hear conflicting testimony from the two disputing parties and there were no other eyewitnesses to the altercation Under Rule 609(a)(1) the inmatersquos conviction should be admitted because it has some bearing on his credibility and its probative value is not substantially outweighed by concerns of unfair prejudice confusion or delay Id

Evidence of the inmatersquos misdemeanor conviction for perjury must be admitted because the crime ldquorequired provingmdashor the witnessrsquos admittingmdasha dishonest act or false statementrdquo by the inmate FED R EVID 609(a)(2)

27

Evidence Analysis

Evidence of the inmatersquos felony conviction for sexual assault should be excluded because its probative value is substantially outweighed by the danger of unfair prejudice to the inmate based on the heinous nature of the crime FED R EVID 609(a)(1) In the alternative the judge could limit the evidence relating to this conviction by excluding details of the inmatersquos crime

In all civil (and criminal) cases witnesses can also be impeached with evidence of specific instances of prior misconduct that did not result in a conviction FED R EVID 608(b) Pursuant to Rule 608(b) misconduct probative of untruthfulness can be inquired into on cross-examination but cannot be proved through extrinsic evidence Id Thus the inmatersquos counsel should be permitted to cross-examine the guard regarding the false statement in the guardrsquos reacutesumeacute However extrinsic evidence of the guardrsquos misconduct (ie the guardrsquos authenticated reacutesumeacute and transcript from the local college) should not be admitted even if the guard denies wrongdoing or refuses to answer cross-examination questions about these matters Id

Point One (10) The Federal Rules of Evidence permit impeachment of witnesses with evidence of prior convictions

Whether convictions should be admitted to impeach generally depends on the nature of the crime the amount of time that has passed and (only in criminal cases) whether the ldquowitnessrdquo is the defendant FED R EVID 609(a) Under Rule 609(a) evidence of prior convictions may be admitted for the purpose of ldquoattacking a witnessrsquos character for truthfulnessrdquo Id

There are two basic types of convictions that can be admitted for the purpose of impeachment

(1) convictions for crimes ldquopunishable by death or by imprisonment for more than one yearrdquo (which generally correlates to ldquofeloniesrdquo) FED R EVID 609(a)(1) and (2) convictions ldquofor any crimes regardless of the punishment if the court can readily determine that establishing the elements of the crime required provingmdashor the witnessrsquos admittingmdasha dishonest act or false statementrdquo FED R EVID 609(a)(2)

Pursuant to Rule 609(a)(1) in civil cases the admission of evidence of a felony conviction is ldquosubject to Rule 403 [which says that a court may exclude relevant evidence if its probative value is substantially outweighed by other factors]rdquo FED R EVID 609(a)(1) However Rule 403 does not protect the witness against admission of prior convictions involving dishonestymdashwhich must be admitted by the court FED R EVID 609(a)(2)

Finally Federal Rule of Evidence 609(b) contains the presumption that a conviction that is more than 10 years old or where more than 10 years has passed since the witnessrsquos release from confinement (whichever is later) should not be admitted unless ldquoits probative value supported by specific facts and circumstances substantially outweighs its prejudicial effectrdquo and the proponent has provided the adverse party with reasonable written notice FED R EVID 609(b)

Point One(a) (25) The court should admit evidence of the inmatersquos 12-year-old felony marijuana distribution conviction

The inmatersquos conviction for marijuana distribution was for a felony punishable by imprisonment for more than one year See FED R EVID 609(a)(1) Moreover although the conviction was 12 years ago the 10-year time limit of Rule 609(b) is not exceeded because that time limit runs

28

Evidence Analysis

from the date of either ldquothe witnessrsquos conviction or release from confinement for it whichever is laterrdquo FED R EVID 609(b) Because the inmate served three years in prison he was released from confinement nine years ago

However pursuant to Rule 609(a)(1) the admission of felony convictions to impeach a witness in a civil case is ldquosubject to Rule 403rdquo FED R EVID 609(a)(1) Neither Rule 609(a) nor the advisory committee notes specify which factors courts should consider when balancing the probative value of a conviction against the dangers identified in Rule 403 (which include (1) unfair prejudice (2) confusion of the issues (3) misleading the jury (4) waste of time or undue delay and (5) needless presentation of cumulative evidence) FED R EVID 403

In this case credibility is very important because the evidence consists primarily of the testimony of the disputing parties and there were no other eyewitnesses to the altercation This enhances the probative value of any evidence bearing on the inmatersquos credibility A court is likely to conclude that the inmatersquos prior felony drug conviction is relevant to his credibility See eg United States v Brito 427 F3d 53 64 (1st Cir 2005) (ldquoPrior drug-trafficking crimes are generally viewed as having some bearing on veracityrdquo) Although the probative value of any conviction diminishes with age see eg United States v Brewer 451 F Supp 50 53 (ED Tenn 1978) the inmatersquos ongoing problems with the law suggest that he has continued (and even escalated) his criminal behavior over the past nine years The court should admit this evidence because its probative value is not substantially outweighed by any Rule 403 concerns Specifically any prejudice to the inmate would be slight because the conviction is unrelated to the altercation at issue and the conviction was not for a heinous crime that might inflame the jury

[NOTE Whether an examinee identifies the jury instruction as containing a ldquoconclusiverdquo or ldquomandatoryrdquo presumption is less important than the examineersquos analysis of the constitutional infirmities]

Point One(b) (15) The court must admit evidence of the inmatersquos eight-year-old misdemeanor conviction because perjury is a crime of dishonesty

Rule 609(a)(2) provides that evidence of a criminal conviction ldquomust be admitted if the court can readily determine that establishing the elements of the crime required provingmdashor the witnessrsquos admittingmdasha dishonest act or false statementrdquo FED R EVID 609(a)(2) The inmatersquos conviction for perjury would have necessarily required proving that the inmate engaged in an act of dishonesty This conviction occurred within the past 10 years so it ldquomust be admittedrdquo because in contrast to Rule 609(a)(1) (discussed in Point One(a)) admission under Rule 609(a)(2) is mandatory and not subject to Rule 403

Point One(c) (20) The court should exclude evidence of the inmatersquos seven-year-old felony sexual assault conviction because the probative value of this evidence is substantially outweighed by the danger of unfair prejudice In the alternative the details of the prior conviction could be excluded

The inmatersquos conviction for felony sexual assault was seven years ago and he has not yet been released from incarceration so Rule 609(a) but not 609(b) is applicable here FED R EVID 609(a) This conviction is therefore admissible to impeach the inmate unless its probative value is substantially outweighed by the danger of unfair prejudice or any other Rule 403 concern Id

29

Evidence Analysis

Sex crimes are generally not considered relevant to credibility see Hopkins v State 639 So 2d 1247 1254 (Miss 1993) so the probative value of this conviction is relatively low Moreover the heinous nature of the inmatersquos crime (sexual assault on his daughter) makes the danger of unfair prejudice to the inmate very high Thus the court should exclude evidence of the conviction because it was for a heinous offense that is likely to inflame the jury and it has little bearing on credibility See eg United States v Beahm 664 F2d 414 419 (4th Cir 1981)

As an alternative to excluding this evidence the judge could minimize the unfair prejudice to the inmate by permitting limited cross-examination but refusing to allow specific questions about the nature of the inmatersquos conviction For example a court could limit cross-examination to the fact that the inmate was convicted of a ldquofelonyrdquo or perhaps that he was convicted of a ldquosexual assaultrdquo without identifying the victim However because evidence of the inmatersquos prior convictions can be admitted solely for the purpose of enabling the jury to assess his credibility and because his two earlier convictions should have already been admitted the court should exclude all evidence of the felony sexual assault conviction

Point Two(a) (15) The court should permit the inmatersquos counsel to cross-examine the guard regarding the false statement in his reacutesumeacute because the guardrsquos misconduct bears on his truthfulness

The inmate wishes to cross-examine the guard about his prior dishonest behaviormdashlying on his reacutesumeacutemdashthat did not involve a criminal conviction Rule 608(b) allows witnesses to be cross-examined about specific instances of prior non-conviction misconduct probative of untruthfulness ldquoin order to attack the witnessrsquos character for truthfulnessrdquo FED R EVID 608(b)

The courtrsquos decision to allow cross-examination about the guardrsquos prior dishonest behavior depends on the probative value of such evidence balanced against the danger of unfair prejudice to the guard or any other Rule 403 concern FED R EVID 403 Here the guardrsquos false statement on his reacutesumeacute that he obtained a degree in Criminal Justice is highly probative of his untruthfulness because it grossly misrepresents his actual academic record was made recently and was made with the intent to deceive Because the probative value of this evidence is very strong and is not substantially outweighed by any Rule 403 concerns cross-examination of the guard on this topic should be permitted The court may also consider it fair to permit this cross-examination of the guard on these matters assuming that one or more of the inmatersquos prior convictions have been admitted to impeach his credibility

Point Two(b) (15) The court should exclude extrinsic evidence of the guardrsquos non-conviction misconduct even if the guard denies wrongdoing or refuses to answer questions about the matter

Although Rule 608(b) allows cross-examination about specific instances of prior misconduct probative of untruthfulness ldquoextrinsic evidencerdquo offered to prove such misconduct is not admissible FED R EVID 608(b) The rationale for this rule is that allowing the introduction of extrinsic evidence of prior misconduct by witnesses when these acts are relevant only to the witnessesrsquo truthfulness and not to the main issues in the case would create too great a risk of confusing the jury and unduly delaying the trial The court does not have discretion to admit this extrinsic evidence See eg United States v Elliot 89 F3d 1360 1368 (8th Cir 1996)

30

Evidence Analysis

Here the inmatersquos counsel may cross-examine the guard about the false statement on his reacutesumeacute However the inmatersquos counsel must accept the guardrsquos response Even if the guard denies wrongdoing or refuses to answer questions about the matter the inmatersquos counsel cannot introduce the guardrsquos reacutesumeacute or the transcript from the local college to prove the guardrsquos misconduct

31

CORPORATIONS ANALYSIS (Corporations VA2 IX)

ANALYSIS

Legal Problems

(1) Do shareholders have the authority to amend a corporationrsquos bylaws with respect to director nominations

(2) Do board-approved bylaws on a particular subject here nomination of directors preempt subsequent conflicting bylaw amendments by shareholders

(3) Is a suit challenging both managementrsquos refusal to include the proposed bylaw amendment in Megarsquos proxy statement and the boardrsquos amendment of the bylaws dealing with nomination of directors a direct or derivative suit

DISCUSSION

Summary

The voting and litigation rights of the shareholders of Mega are subject to the provisions of the Model Business Corporations Act (MBCA)

The investorrsquos proposed bylaw provision is not inconsistent with state law Under the MBCA shareholders may amend the bylaws when the amendment deals with a proper matter for the corporationrsquos bylaws such as procedures for nominating directors

The Mega boardrsquos bylaw amendment does not preempt the investorrsquos proposed bylaw provision or the Mega shareholdersrsquo power to approve it While shareholders can limit the boardrsquos power to amend or repeal the bylaws the board cannot limit the shareholdersrsquo power

Whether the investor must make a demand on Megarsquos board depends on how the investor frames its claim If the investor claims a violation of shareholder voting rights the claim is direct and pre-suit demand on the board is not required If on the other hand the investor claims that the directors violated their fiduciary duties by amending the bylaws to entrench themselves the claim is derivative and a pre-suit demand is required

Point One (30) Shareholders may amend the corporationrsquos bylaws where the proposed bylaw provision relates to procedural matters typically included in the bylaws such as the nomination of directors

Internal affairs of the corporation such as the conduct of shareholder meetings and election of directors are subject to the corporate law of the state of incorporation See McDermott Inc v Lewis 531 A2d 206 (Del 1987) (applying law of jurisdiction where corporation was incorporated in case involving voting rights) This statersquos corporate statute is modeled on the MBCA

Under the MBCA ldquoshareholders may amend the corporationrsquos bylawsrdquo MBCA sect 1020(a) Thus the only question is whether the bylaws can specify the procedures for shareholder nomination of directors

32

Corporations Analysis

The MBCA states that the bylaws ldquomay contain any provision that is not inconsistent with law or the articles of incorporationrdquo MBCA sect 206(b) In addition the MBCA was revised in 2009 to address shareholder nomination of directors in public corporations (known as ldquoproxy accessrdquo) and specifies that the bylaws ldquomay contain a requirement that the corporation include in its [proxy materials] one or more individuals nominated by a shareholderrdquo MBCA sect 206(c)(1) see Committee on Corporate Laws ABA Section of Business Law Report on the Roles of Boards of Directors and Shareholders of Publicly Owned Corporations and Changes to the Model Business Corporations ActmdashAdoption of Shareholder Proxy Access Amendments to Chapters 2 and 10 65 BUS LAWYER 1105 (2010)

The inclusion of director-nomination procedures in the bylaws is consistent with practice and is recognized by the Delaware courts whose views on corporate law carry significant weight Typically the procedures for nomination of directors are found in the bylaws See 1 COX amp HAZEN TREATISE ON THE LAW OF CORPORATIONS sect 312 (3d ed 2011) see also 4 FLETCHER CORP FORMS ANN PART III ch 21 (2013) (including sample bylaws that permit nomination of directors by shareholders) The Delaware Supreme Court has confirmed that the bylaws may ldquodefine the process and proceduresrdquo for director elections See CA Inc v AFSCME Employees Pension Plan 953 A2d 227 (Del 2008) (concluding that bylaw amendment requiring reimbursement of election expenses to certain successful shareholder nominators is ldquoproper subjectrdquo under Delaware law)

[NOTE The question of the proper scope of the bylaws can be answered using the more general MBCA sect 206(b) or the 2009 MBCA revision adding sect 206(c)(1) (adopted in CT ME VA) In addition some examinees might raise the point that shareholder proposals may not compel the board to take action such as by including shareholder nominations in the companyrsquos proxy materials on the theory that the ldquobusiness and affairsrdquo of the corporation are to be managed by the board See MBCA sect 801(b) Although shareholders are generally limited to adopting precatory resolutions that recommend or encourage board action this limitation does not apply when shareholders have specific authority to take binding action on their ownmdashsuch as to amend the bylaws]

Point Two (30) Shareholders can amend (or repeal) board-approved bylaws Further shareholders can limit the boardrsquos power to later amend and repeal a shareholder-approved bylaw

Under the MBCA shareholders have the power to amend the bylaws See Point One The board shares this power with the shareholders unless (1) the corporationrsquos articles ldquoreserve that power exclusively to the shareholdersrdquo or (2) ldquothe shareholders in amending repealing or adopting a bylaw expressly provide that the board of directors may not amend repeal or reinstate that bylawrdquo See MBCA sect 1020(b)

Shareholder-approved bylaw provisions can amend or repeal existing bylaw provisions whether originally approved by the board or by shareholders See ALAN R PALMITER CORPORATIONS EXAMPLES AND EXPLANATIONS sect 713 (7th ed 2012) Thus the Mega boardrsquos bylaw amendmentmdashwhich set more demanding thresholds for shareholder nomination of directors than the investorrsquos proposed bylaw provisionmdashwould be superseded (repealed) if Megarsquos shareholders were to approve the investorrsquos proposal

Further a shareholder-approved bylaw generally can limit the power of the board to later amend or repeal it See MBCA sect 1020(b)(2) Thus if Megarsquos shareholders approved the bylaw

33

Corporations Analysis

provision proposed by the investor Megarsquos board could not repeal the provision because it includes a ldquono board repealrdquo clause

The revision to the MBCA in 2009 dealing with shareholder proxy access does not change this conclusion That revision specifies that a shareholder-approved bylaw dealing with director nominations may not limit the boardrsquos power to amend add or repeal ldquoany procedure or condition to such a bylaw in order to provide for a reasonable practicable and orderly processrdquo MBCA sect 206(d) Thus according to the revision if shareholders approve a bylaw amendment that limits further board changes the board would nonetheless retain the power to ldquotinkerrdquo with the bylaw to safeguard the voting process but could not repeal the shareholder-approved bylaw The Official Comment to MBCA sect 206(d) makes clear that the revision is ldquonot intended to allow the board of directors to frustrate the purpose of the shareholder-adopted proxy access provisionrdquo Thus if Megarsquos shareholders were to approve the bylaw provision proposed by the investor Megarsquos board could only amend the provision regarding its procedures or conditions in a manner consistent with its purpose of permitting proxy access for Megarsquos shareholders

[NOTE The boardrsquos attempted interference with a shareholder voting initiative may also have been a violation of the directorsrsquo fiduciary duties See Blasius Indus Inc v Atlas Corp 564 A2d 651 (Del Ch 1988) (finding that directors breached their fiduciary duties by amending bylaws and expanding size of board to thwart insurgentrsquos plan to amend bylaws and seat a majority of new directors) The call however asks examinees to consider whether shareholders or the board have ldquoprecedencerdquo over amending the corporate bylaws Thus an examineersquos answer should be framed in terms of ldquopowerrdquo and not ldquodutyrdquo]

Point Three (40) The investor need not make a demand on the board if the investor states a direct claim such as an allegation that the board interfered with the investorrsquos right to amend the bylaws But the investor must make a demand on the board if the investor states a derivative claim (on behalf of the corporation) such as an allegation that the directors sought to entrench themselves by interfering with the proposed proxy access

The MBCA generally requires that shareholders make a demand on the board of directors before initiation of a derivative suit MBCA sect 742 (shareholder may not bring derivative proceeding until written demand has been made on corporation and 90 days have expired) A derivative suit is essentially two suits in one where the plaintiff-shareholder seeks to bring on behalf of the corporation a claim that vindicates corporate rights usually based on violation of fiduciary duties PALMITER supra sect 1811 (6th ed 2009) The demand permits the board to investigate the situation identified by the shareholder and take suitable action No demand on the board is required however if the shareholder brings a direct suit to vindicate the shareholderrsquos own rights not those of the corporation

Is the suit brought by the investor derivative or direct The MBCA defines a ldquoderivative proceedingrdquo as one brought ldquoin the right of a domestic corporationrdquo MBCA sect 740(1) Thus the answer to how the investorrsquos suit should be characterized turns on what rights the investor seeks to vindicate If the investor frames its claim as one of fiduciary breach by directorsmdashfor example for failing to become adequately informed about voting procedures or for seeking to entrench themselves in office by manipulating the voting structure to avoid a shareholder insurgencymdashthen the suit is ldquoderivativerdquo and the investor must make a demand on the board See MBCA Ch 7 Subch D Introductory Comment (ldquothe derivative suit has historically been the principal method of challenging allegedly illegal action by managementrdquo)

34

Corporations Analysis

If however the investor frames its claim as one to vindicate shareholder rights the suit is direct and no demand is required For many courts the direct-derivative question turns on who is injured and who is to receive the relief sought by the plaintiff-shareholders See Tooley v Donaldson Lufkin amp Jenrette Inc 845 A2d 1031 (Del 2004) (characterizing a merger-delay claim as direct because delay of merger only harmed shareholders not corporation) Thus if the investor claims that managementrsquos refusal to include its proposed bylaw amendment in the corporationrsquos proxy materials violates its shareholder rights to initiate corporate governance reforms the suit will be direct Courts have not questioned the ability of shareholders to bring direct suits challenging board action to exclude their proposed bylaw amendments from the corporationrsquos proxy materials See JANA Master Fund Ltd v CNET Networks Inc 954 A2d 335 (Del Ch 2008) (upholding shareholderrsquos direct challenge to boardrsquos interpretation of advance-notice bylaw) Chesapeake Corp v Shore 771 A2d 293 (Del Ch 2000) (upholding shareholderrsquos direct challenge to actions by board that effectively prevented it from proposing bylaw amendments in contest for control)

Is the way that the investor frames its claim conclusive Courts have permitted shareholder-plaintiffs to challenge a transaction in a direct suit even though the same transaction could also be challenged as a fiduciary breach See Eisenberg v Flying Tiger Line Inc 451 F2d 267 (2d Cir 1971) (permitting direct suit challenging a corporate reorganization as a dilution of shareholder voting power even though reorganization may have involved conflicts of interest and thus constituted a fiduciary breach) Thus the investorrsquos choice to pursue a claim challenging the legality of managementrsquos decision to exclude the investorrsquos proposal from the corporationrsquos proxy materialsmdashrather than a possible breach of fiduciary dutymdashis likely to be respected See 3 COX amp HAZEN supra sect 153 (describing situations in which a claim can be framed as derivative or direct)

[NOTE Some issues under Delaware corporate law regarding pre-suit demand are not relevant here For example whether the Mega directors are independent and disinterested is not relevant to the MBCA requirement of a pre-suit demand As the Official Comment to MBCA sect 742 points out the MBCArsquos requirement of ldquouniversal demandrdquo gives the board ldquothe opportunity to reexamine the act complained of in the light of a potential lawsuit and take corrective actionrdquo even when the directors might be non-independent or have conflicts of interest

Nor is it relevant to the MBCA pre-suit demand requirement that the statutory 90-day waiting period may be onerous The first paragraph of MBCA sect 742 requires a pre-suit demand without exception the second paragraph of the section imposes a 90-day waiting period before a derivative suit may be brought which can be shortened if the board rejects the demand or ldquoirreparable injury to the corporation would result by waiting for the expiration of the 90-day periodrdquo The call as written asks only whether a pre-suit demand should be made and does not ask examinees to address whether the post-demand waiting period should be shortened under the ldquoirreparable injuryrdquo standard]

35

National Conference of Bar Examiners 302 South Bedford Street | Madison WI 53703-3622 Phone 608-280-8550 | Fax 608-280-8552 | TDD 608-661-1275

wwwncbexorg e-mail contactncbexorg

  • Preface
  • Description of the MEE
  • Instructions
  • July 2014 Questions
    • CRIMINAL LAW AND PROCEDURE QUESTION
    • CONTRACTS QUESTION
    • FAMILY LAW QUESTION
    • FEDERAL CIVIL PROCEDURE QUESTION
    • EVIDENCE QUESTION
    • CORPORATIONS QUESTION
      • July 2014 Analyses
        • CRIMINAL LAW AND PROCEDURE ANALYSIS
        • CONTRACTS ANALYSIS
        • FAMILY LAW ANALYSIS
        • FEDERAL CIVIL PROCEDURE ANALYSIS
        • EVIDENCE ANALYSIS
        • CORPORATIONS ANALYSIS
            • ltlt13 ASCII85EncodePages false13 AllowTransparency false13 AutoPositionEPSFiles true13 AutoRotatePages None13 Binding Left13 CalGrayProfile (Dot Gain 20)13 CalRGBProfile (sRGB IEC61966-21)13 CalCMYKProfile (US Web Coated 050SWOP051 v2)13 sRGBProfile (sRGB IEC61966-21)13 CannotEmbedFontPolicy Error13 CompatibilityLevel 1413 CompressObjects Tags13 CompressPages true13 ConvertImagesToIndexed true13 PassThroughJPEGImages true13 CreateJobTicket false13 DefaultRenderingIntent Default13 DetectBlends true13 DetectCurves 0000013 ColorConversionStrategy CMYK13 DoThumbnails false13 EmbedAllFonts true13 EmbedOpenType false13 ParseICCProfilesInComments true13 EmbedJobOptions true13 DSCReportingLevel 013 EmitDSCWarnings false13 EndPage -113 ImageMemory 104857613 LockDistillerParams false13 MaxSubsetPct 10013 Optimize true13 OPM 113 ParseDSCComments true13 ParseDSCCommentsForDocInfo true13 PreserveCopyPage true13 PreserveDICMYKValues true13 PreserveEPSInfo true13 PreserveFlatness true13 PreserveHalftoneInfo false13 PreserveOPIComments true13 PreserveOverprintSettings true13 StartPage 113 SubsetFonts true13 TransferFunctionInfo Apply13 UCRandBGInfo Preserve13 UsePrologue false13 ColorSettingsFile ()13 AlwaysEmbed [ true13 ]13 NeverEmbed [ true13 ]13 AntiAliasColorImages false13 CropColorImages true13 ColorImageMinResolution 30013 ColorImageMinResolutionPolicy OK13 DownsampleColorImages true13 ColorImageDownsampleType Bicubic13 ColorImageResolution 30013 ColorImageDepth -113 ColorImageMinDownsampleDepth 113 ColorImageDownsampleThreshold 15000013 EncodeColorImages true13 ColorImageFilter DCTEncode13 AutoFilterColorImages true13 ColorImageAutoFilterStrategy JPEG13 ColorACSImageDict ltlt13 QFactor 01513 HSamples [1 1 1 1] VSamples [1 1 1 1]13 gtgt13 ColorImageDict ltlt13 QFactor 01513 HSamples [1 1 1 1] VSamples [1 1 1 1]13 gtgt13 JPEG2000ColorACSImageDict ltlt13 TileWidth 25613 TileHeight 25613 Quality 3013 gtgt13 JPEG2000ColorImageDict ltlt13 TileWidth 25613 TileHeight 25613 Quality 3013 gtgt13 AntiAliasGrayImages false13 CropGrayImages true13 GrayImageMinResolution 30013 GrayImageMinResolutionPolicy OK13 DownsampleGrayImages true13 GrayImageDownsampleType Bicubic13 GrayImageResolution 30013 GrayImageDepth -113 GrayImageMinDownsampleDepth 213 GrayImageDownsampleThreshold 15000013 EncodeGrayImages true13 GrayImageFilter DCTEncode13 AutoFilterGrayImages true13 GrayImageAutoFilterStrategy JPEG13 GrayACSImageDict ltlt13 QFactor 01513 HSamples [1 1 1 1] VSamples [1 1 1 1]13 gtgt13 GrayImageDict ltlt13 QFactor 01513 HSamples [1 1 1 1] VSamples [1 1 1 1]13 gtgt13 JPEG2000GrayACSImageDict ltlt13 TileWidth 25613 TileHeight 25613 Quality 3013 gtgt13 JPEG2000GrayImageDict ltlt13 TileWidth 25613 TileHeight 25613 Quality 3013 gtgt13 AntiAliasMonoImages false13 CropMonoImages true13 MonoImageMinResolution 120013 MonoImageMinResolutionPolicy OK13 DownsampleMonoImages true13 MonoImageDownsampleType Bicubic13 MonoImageResolution 120013 MonoImageDepth -113 MonoImageDownsampleThreshold 15000013 EncodeMonoImages true13 MonoImageFilter CCITTFaxEncode13 MonoImageDict ltlt13 K -113 gtgt13 AllowPSXObjects false13 CheckCompliance [13 None13 ]13 PDFX1aCheck false13 PDFX3Check false13 PDFXCompliantPDFOnly false13 PDFXNoTrimBoxError true13 PDFXTrimBoxToMediaBoxOffset [13 00000013 00000013 00000013 00000013 ]13 PDFXSetBleedBoxToMediaBox true13 PDFXBleedBoxToTrimBoxOffset [13 00000013 00000013 00000013 00000013 ]13 PDFXOutputIntentProfile ()13 PDFXOutputConditionIdentifier ()13 PDFXOutputCondition ()13 PDFXRegistryName ()13 PDFXTrapped False1313 CreateJDFFile false13 Description ltlt13 ARA 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 BGR ltFEFF04180437043f043e043b043704320430043904420435002004420435043704380020043d0430044104420440043e0439043a0438002c00200437043000200434043000200441044a0437043404300432043004420435002000410064006f00620065002000500044004600200434043e043a0443043c0435043d04420438002c0020043c0430043a04410438043c0430043b043d043e0020043f044004380433043e04340435043d04380020043704300020043204380441043e043a043e043a0430044704350441044204320435043d0020043f04350447043004420020043704300020043f044004350434043f0435044704300442043d04300020043f043e04340433043e0442043e0432043a0430002e002000200421044a04370434043004340435043d043804420435002000500044004600200434043e043a0443043c0435043d044204380020043c043e0433043004420020043404300020044104350020043e0442043204300440044f0442002004410020004100630072006f00620061007400200438002000410064006f00620065002000520065006100640065007200200035002e00300020043800200441043b0435043404320430044904380020043204350440044104380438002egt13 CHS ltFEFF4f7f75288fd94e9b8bbe5b9a521b5efa7684002000410064006f006200650020005000440046002065876863900275284e8e9ad88d2891cf76845370524d53705237300260a853ef4ee54f7f75280020004100630072006f0062006100740020548c002000410064006f00620065002000520065006100640065007200200035002e003000204ee553ca66f49ad87248672c676562535f00521b5efa768400200050004400460020658768633002gt13 CHT ltFEFF4f7f752890194e9b8a2d7f6e5efa7acb7684002000410064006f006200650020005000440046002065874ef69069752865bc9ad854c18cea76845370524d5370523786557406300260a853ef4ee54f7f75280020004100630072006f0062006100740020548c002000410064006f00620065002000520065006100640065007200200035002e003000204ee553ca66f49ad87248672c4f86958b555f5df25efa7acb76840020005000440046002065874ef63002gt13 CZE 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 DAN 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 DEU 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 ESP ltFEFF005500740069006c0069006300650020006500730074006100200063006f006e0066006900670075007200610063006900f3006e0020007000610072006100200063007200650061007200200064006f00630075006d0065006e0074006f00730020005000440046002000640065002000410064006f0062006500200061006400650063007500610064006f00730020007000610072006100200069006d0070007200650073006900f3006e0020007000720065002d0065006400690074006f007200690061006c00200064006500200061006c00740061002000630061006c0069006400610064002e002000530065002000700075006500640065006e00200061006200720069007200200064006f00630075006d0065006e0074006f00730020005000440046002000630072006500610064006f007300200063006f006e0020004100630072006f006200610074002c002000410064006f00620065002000520065006100640065007200200035002e003000200079002000760065007200730069006f006e0065007300200070006f00730074006500720069006f007200650073002egt13 ETI 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 FRA 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 GRE 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 HEB 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 HRV (Za stvaranje Adobe PDF dokumenata najpogodnijih za visokokvalitetni ispis prije tiskanja koristite ove postavke Stvoreni PDF dokumenti mogu se otvoriti Acrobat i Adobe Reader 50 i kasnijim verzijama)13 HUN 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 ITA 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 JPN ltFEFF9ad854c18cea306a30d730ea30d730ec30b951fa529b7528002000410064006f0062006500200050004400460020658766f8306e4f5c6210306b4f7f75283057307e305930023053306e8a2d5b9a30674f5c62103055308c305f0020005000440046002030d530a130a430eb306f3001004100630072006f0062006100740020304a30883073002000410064006f00620065002000520065006100640065007200200035002e003000204ee5964d3067958b304f30533068304c3067304d307e305930023053306e8a2d5b9a306b306f30d530a930f330c8306e57cb30818fbc307f304c5fc59808306730593002gt13 KOR ltFEFFc7740020c124c815c7440020c0acc6a9d558c5ec0020ace0d488c9c80020c2dcd5d80020c778c1c4c5d00020ac00c7a50020c801d569d55c002000410064006f0062006500200050004400460020bb38c11cb97c0020c791c131d569b2c8b2e4002e0020c774b807ac8c0020c791c131b41c00200050004400460020bb38c11cb2940020004100630072006f0062006100740020bc0f002000410064006f00620065002000520065006100640065007200200035002e00300020c774c0c1c5d0c11c0020c5f40020c2180020c788c2b5b2c8b2e4002egt13 LTH 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 LVI 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 NLD (Gebruik deze instellingen om Adobe PDF-documenten te maken die zijn geoptimaliseerd voor prepress-afdrukken van hoge kwaliteit De gemaakte PDF-documenten kunnen worden geopend met Acrobat en Adobe Reader 50 en hoger)13 NOR 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 POL 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 PTB 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 RUM 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 RUS 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 SKY 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 SLV 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 SUO 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 SVE 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 TUR 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 UKR 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 ENU (Use these settings to create Adobe PDF documents best suited for high-quality prepress printing Created PDF documents can be opened with Acrobat and Adobe Reader 50 and later)13 gtgt13 Namespace [13 (Adobe)13 (Common)13 (10)13 ]13 OtherNamespaces [13 ltlt13 AsReaderSpreads false13 CropImagesToFrames true13 ErrorControl WarnAndContinue13 FlattenerIgnoreSpreadOverrides false13 IncludeGuidesGrids false13 IncludeNonPrinting false13 IncludeSlug false13 Namespace [13 (Adobe)13 (InDesign)13 (40)13 ]13 OmitPlacedBitmaps false13 OmitPlacedEPS false13 OmitPlacedPDF false13 SimulateOverprint Legacy13 gtgt13 ltlt13 AddBleedMarks false13 AddColorBars false13 AddCropMarks false13 AddPageInfo false13 AddRegMarks false13 ConvertColors ConvertToCMYK13 DestinationProfileName ()13 DestinationProfileSelector DocumentCMYK13 Downsample16BitImages true13 FlattenerPreset ltlt13 PresetSelector MediumResolution13 gtgt13 FormElements false13 GenerateStructure false13 IncludeBookmarks false13 IncludeHyperlinks false13 IncludeInteractive false13 IncludeLayers false13 IncludeProfiles false13 MultimediaHandling UseObjectSettings13 Namespace [13 (Adobe)13 (CreativeSuite)13 (20)13 ]13 PDFXOutputIntentProfileSelector DocumentCMYK13 PreserveEditing true13 UntaggedCMYKHandling LeaveUntagged13 UntaggedRGBHandling UseDocumentProfile13 UseDocumentBleed false13 gtgt13 ]13gtgt setdistillerparams13ltlt13 HWResolution [2400 2400]13 PageSize [612000 792000]13gtgt setpagedevice13

Page 21: July 2014 MEE Questions and Analyses - NCBE...This publication includes the questions and analyses from the July 2014 MEE. (In the actual test, the questions are simply numbered rather

FAMILY LAW ANALYSIS (Family Law IIIB D amp G)

ANALYSIS

Legal Problems

(1)(a) Does the State A court have jurisdiction to modify the State B child support order

(1)(b) Does the State A court have jurisdiction to modify the marital-residence-saleshyproceeds provision of the State B property-division decree

(2)(a) May a child support order be modified retroactively

(2)(b) May a child support order be modified prospectively based on a change of employment with a lower salary

(2)(c) May a property-division order be modified after entry of a divorce decree

DISCUSSION

Summary

The State A court may exercise personal jurisdiction over the wife because she was personally served in State A However subject-matter jurisdiction over the interstate modification of child support is governed by the Uniform Interstate Family Support Act (UIFSA) Under UIFSA State A does not have jurisdiction to modify the order for the daughterrsquos support because the wife is still a resident of State B UIFSA on the other hand does not govern property distributions and thus a State A court is not precluded from hearing the husbandrsquos petition to modify the marital-residence-sale-proceeds provision of the divorce decree

A child support order may not be modified retroactively A child support order may be modified prospectively based on a substantial change in circumstances Courts agree that a significant decrease in income is a substantial change in circumstances All states treat voluntary income reductions differently than involuntary reductions but employ different approaches for evaluating the impact of a voluntary reduction Whether the husband could obtain prospective modification of the child support order depends on which approach is applied

A property-division order is not subject to post-divorce modification based on a change in circumstances Thus the husband may in some states obtain prospective modification of the order for the daughterrsquos support but he may not obtain modification of the marital-residenceshysale-proceeds provision

Point One(a) (25) Personal jurisdiction over a nonresident respondent does not confer subject-matter jurisdiction over child support modification Under UIFSA a State A court may not modify a child support order issued by a State B court when as here the child or either parent continues to reside in State B the jurisdiction that issued the child support order

The State A court may exercise personal jurisdiction over the wife The wife was personally served in State A and a state may exercise jurisdiction based on in-state personal service See

19

Family Law Analysis

Burnham v Superior Court 495 US 604 (1990) But personal jurisdiction over the wife is not enough to give a State A court jurisdiction to modify the State B support order

The interstate enforcement and modification of child support is governed by the Uniform Interstate Family Support Act (UIFSA) which has been adopted by all states Under UIFSA the state that originally issued a child support order (here State B) has continuing exclusive jurisdiction to modify the order if that state remains the residence of the obligee the child or the obligor and all parties do not consent to the jurisdiction of another forum See UIFSA sect 205 See also UIFSA sect 603 (ldquoA tribunal of this State shall recognize and enforce but may not modify a registered order if the issuing tribunal had jurisdictionrdquo) The wife and daughter continue to reside in State B and the wife has not consented to the jurisdiction of another forum Thus a State A court does not have jurisdiction to modify the State B child support order

[NOTE Examinees who do not discuss personal jurisdiction but fully discuss UIFSA may receive full credit]

Point One(b) (15) UIFSA does not apply to disputes over property division Thus the State A court may exercise jurisdiction over the husbandrsquos petition to modify the marital-residence-sale-proceeds provision of the State B divorce decree because it has personal jurisdiction over the wife

The State A court in which the husband brought his action has jurisdiction to adjudicate domestic relations issues The husbandrsquos petition to modify the property settlement is a domestic relations issue The courts of State A may exercise personal jurisdiction over the wife because she was personally served in State A See Burnham v Superior Court 495 US 604 (1990) see Point One(a)

UIFSA does not apply to divorce property-division disputes Thus although a State A court may not adjudicate the husbandrsquos petition to modify his child support obligations it may adjudicate his property-division claims (Even though the court has jurisdiction it may not modify the property-division award on the merits See Point Two(c))

Point Two(a) (20) A child support order may not be modified retroactively

State courts have long held that obligations to pay child support ordinarily may not be modified retroactively ldquoIf the hardship is particularly severe the courts sometimes devised a way to protect the obligor but in most instances the courts hold that retroactive modification of this kind is beyond their power and indeed the governing statute may so providerdquo HOMER H CLARK THE LAW OF DOMESTIC RELATIONSHIPS IN THE UNITED STATES 725 (2d ed 1987)

Federal law now goes further and requires the states as a condition of federal child-support funding to adopt rules that absolutely forbid retroactive modification of the support obligation See 42 USC sect 666(a)(9)(C) The states have adopted rules consistent with the federal requirements

Point Two(b) (25) It is unclear whether the husband could obtain prospective downward modification of his child support based on his voluntary acceptance of a job with a lower salary

Prospective modification of a child support order is typically available only when the petitioner can show a substantial change in circumstances See ROBERT E OLIPHANT amp NANCY VER

20

Family Law Analysis

STEEGH FAMILY LAW 213ndash15 (3d ed 2010) A significant decrease in income is typically viewed as a substantial change

However when a parent seeks to modify a child support obligation because he has voluntarily reduced his income a court will not modify the obligation based solely on the income loss Some courts refuse to modify whenever the income shift was voluntary See eg Aguiar v Aguiar 127 P3d 234 (Idaho Ct App 2005) Others look primarily to the petitionerrsquos intentions and permit downward modification if he has acted in good faith See eg In re Marriage of Horn 650 NE2d 1103 (Ill App Ct 1995) Many courts use a multifactor approach See OLIPHANT amp VER STEEGH supra 217ndash18

Here there is no question that the husbandrsquos loss of income was voluntary In a jurisdiction in which voluntary income reduction bars support modification the husbandrsquos petition would be denied

In a jurisdiction employing a good-faith or multifactor approach it is possible but not certain that the husband could obtain downward modification The evidence supports the husbandrsquos good faith his change in employment appears to be based on his new jobrsquos greater responsibilities and better promotion possibilities In a jurisdiction using a multifactor approach the court would likely also consider the impact of such a shift on the daughter the likely duration of the husbandrsquos income loss and the likelihood of a promotion that would ultimately inure to the daughterrsquos benefit Thus on these facts it is possible but by no means certain that the husband could prospectively obtain downward modification of his child support obligation to his daughter

Point Two(c)(15) A divorce property-division award is not subject to modification

A support order is aimed at meeting the post-divorce needs of the supported individual Because the future is unpredictable courts are empowered to modify a support award to take account of changed circumstances that may occur during the period in which support is paid

By contrast a property-distribution award divides assets of the marriage based on the equities at the time of divorce Because the past can be ascertained a property-division award is not subject to post-divorce modification See HARRY A KRAUSE ET AL FAMILY LAW CASES COMMENTS AND QUESTIONS 691 (6th ed 2007)

Here the husband is seeking modification of a property-division award with respect to an asset owned by the parties at the time of divorce Thus the husband may not obtain a modification of the marital-residence-sale-proceeds provision of the divorce decree based on his reduced income

21

FEDERAL CIVIL PROCEDURE ANALYSIS (Federal Civil Procedure III IVC)

ANALYSIS

Legal Problems

(1) Is the logging company entitled to join this action as a matter of right

(2)(a) May the nonprofit organization obtain a temporary restraining order to stop the USFS from issuing a logging permit

(2)(b) May the nonprofit organization obtain a preliminary injunction to stop the USFS from issuing a logging permit during the pendency of the action

DISCUSSION

Summary

The logging company is entitled to intervene in this action as a matter of right because it has an interest in the property or transaction that is the subject of the action and is so situated that its interest may be impaired or impeded as a practical matter if the action goes forward without it The logging companyrsquos interest is not adequately represented by the USFSrsquos presence in the lawsuit

The nonprofit organization may seek a temporary restraining order (TRO) followed by a preliminary injunction to prevent the USFS from issuing a logging permit pending the outcome of the action The nonprofit is likely to obtain a TRO if it can demonstrate a risk of immediate and irreparable injury The nonprofit is also likely to obtain a preliminary injunction if it can demonstrate a significant threat of irreparable harm and a likelihood of success on the merits of its National Environmental Policy Act (NEPA) claim

Point One (50) Rule 24(a) of the Federal Rules of Civil Procedure requires federal courts to allow a person to intervene in an action as a matter of right if the person a) is interested in the property or transaction that is the subject of the action b) is so situated that its interest may be impaired or impeded if the litigation goes forward without it and c) is not adequately represented by existing parties Here the logging company likely meets all three requirements and should be allowed to intervene as a matter of right

Rule 24 of the Federal Rules of Civil Procedure governs intervention the process by which a non-party to an action may join the litigation Under Rule 24(a) (intervention of right) a person must be permitted to intervene if three conditions are met (1) the movant ldquoclaims an interest relating to the property or transaction that is the subject of the actionrdquo (2) the movant ldquois so situated that disposition of the action may as a practical matter impair or impede the movantrsquos ability to protect its interestrdquo and (3) ldquoexisting partiesrdquo do not ldquoadequately represent [the movantrsquos] interestrdquo FED R CIV P 24(a) The three requirements for intervention of right are often ldquovery interrelatedrdquo 7C CHARLES ALAN WRIGHT ET AL FEDERAL PRACTICE AND PROCEDURE sect 1908 at 297 (2007 amp 2011 Supp)

22

Federal Civil Procedure Analysis

Here the court should find that the logging company meets this test First the logging company has a strong interest in the property or transaction that is the subject of this action The USFS has accepted the logging companyrsquos bid and the logging company is merely awaiting issuance of a logging permit to begin logging The nonprofit organization is seeking to prevent this logging The logging company therefore has a strong direct and substantial interest in the subject matter of the lawsuit and in having its winning bid honored and a logging permit issued See eg Kleissler v US Forest Serv 157 F3d 964 972 (3d Cir 1998) (stating that ldquo[t]imber companies have direct and substantial interests in a lawsuit aimed at halting loggingrdquo) see also Natural Resources Defense Council v US Nuclear Regulatory Commrsquon 578 F2d 1341 1343ndash 44 (10th Cir 1978) (holding that applicants whose license renewals were pending had Rule 24(a)(2) interests where the lawsuit sought to halt the license-issuing process pending preparation of environmental impact statements) See generally 7C WRIGHT ET AL supra sect 19081 at 309 (ldquoIf there is a direct substantial legally protectable interest in the proceedings it is clear that this requirement of the rule is satisfiedrdquo) Second the logging companyrsquos interest in receiving a logging permit may well be impaired as a practical matter by the outcome of the lawsuit If the USFS loses the lawsuit it will have to prepare an environmental impact statement before issuing the logging companyrsquos permit This will at a minimum delay the logging companyrsquos ability to exercise its rights and may in the long r un mean that no logging permit is ever issued Intervention of right is not limited to those that would be legally bound as a matter of preclusion doctrine Id sect 19082 at 368 Rather ldquo[t]he rule is satisfied whenever disposition of the present action would put the movant at a practical disadvantage in protecting its interestrdquo Id sect 19082 at 369 Here that condition is easily satisfied See Kleissler 157 F3d at 972 (ldquoTimber companies have direct and substantial interests in a lawsuit aimed at halting logging rdquo)

Given that the logging company has an interest that may be impaired by disposition of the action it should be allowed to intervene unless the court is persuaded that the USFS adequately represents the logging companyrsquos interest See Rule 24(a)(2) 7C WRIGHT ET AL supra sect 1909 Here it could be argued that the USFS adequately represents the logging companyrsquos interest because the USFS presumably wants the court to uphold its development plan and allow it to proceed with issuance of the logging permit which is the same relief that the logging company would seek However whether representation is truly adequate depends upon ldquo[a] discriminating appraisal of the circumstancesrdquo 7C WRIGHT ET AL supra sect 1909 at 440 Although both the government and the logging company wish to avoid the preparation of an environmental impact statement their interests are distinct The USFSrsquos interest is proper management of the national forest system while the logging companyrsquos interest is making a profit from logging the 5000-acre tract The USFSrsquos handling of the litigation is likely to be affected by a variety of policy concerns and political considerations that have nothing to do with the logging companyrsquos purely economic interest in securing the right to cut trees in the Scenic National Forest See eg Kleissler 157 F3d at 973ndash74 (ldquo[T]he government represents numerous complex and conflicting interests in matters of this nature The straightforward business interests asserted by intervenors here may become lost in the thicket of sometimes inconsistent governmental policiesrdquo)

[NOTES (1) Examinees who mistakenly analyze the logging companyrsquos case for joinder under the related but incorrect Rule 19 ldquoRequired Joinder of Partiesrdquo may receive credit Rule 19 allows existing parties to demand joinder of non-parties (or seek dismissal of the case if they canrsquot get it) There is a close relationship between Rule 24 and Rule 19 and both contain a similar standard for determining when ldquointerestedrdquo third parties are ldquoentitledrdquo or ldquorequiredrdquo to be in the lawsuit Indeed the two prongs of the Rule 24 intervention test that are discussed above

23

Federal Civil Procedure Analysis

are nearly identical to the two prongs of the Rule 19(a) required joinder test Examinees who discuss and apply the test should receive credit even if they cite Rule 19 rather than Rule 24

(2) Examinees may discuss permissive joinder Although permissive joinder is a possibility here the question asks only whether the logging company can join the action as a matter of right and a permissive joinder analysis is not responsive to the question To the extent an examinee discusses permissive joinder the analysis will focus on whether the logging company ldquohas a claim or defense that shares with the main action a common question of law or factrdquo FED R CIV P 24(b)(1)(B) The district court also ldquomust consider whether the intervention will unduly delay or prejudice the adjudication of the original partiesrsquo rightsrdquo FED R CIV P 24(b)(3) On our facts the logging companyrsquos claim for the issuance of a logging permit would certainly share common questions of law and fact with the USFSrsquos defense against the nonprofitrsquos claim There are no facts suggesting that the logging companyrsquos presence would unduly delay or otherwise prejudice adjudication of the original action Thus the district court would have discretion to permit the logging company to intervene even if it denied intervention of right]

Point Two(a) (25) The nonprofit organization could seek and would likely obtain a temporary restraining order to stop the USFS from issuing a logging permit pending a hearing on an application for a preliminary injunction

The first type of interim relief the nonprofit could seek to stop the USFS from issuing a logging permit to the logging company is a temporary restraining order (TRO) prohibiting the USFS from issuing the logging permit A TRO can be issued without notice to the adverse party but only in limited circumstances and only for a limited time FED R CIV P 65(b) To secure a TRO without notice the nonprofit would need to submit an affidavit containing specific facts that demonstrate a risk of ldquoimmediate and irreparable injuryrdquo if a permit is issued FED R CIV P 65(b)(1) In deciding whether to grant a TRO courts will also consider the same factors that are relevant in deciding whether to grant a preliminary injunction (eg the moving partyrsquos likelihood of success on the merits the balance of hardships and the public interest) See Point Two(b) infra The TRO would last only long enough for the court to consider and resolve a request by the nonprofit for a preliminary injunction but no longer than 14 days (unless the court extends it for good cause or the adverse party consents to an extension) In addition bond is required

Here the court is likely to grant the nonprofitrsquos request The nonprofit could plausibly claim that cutting down 5000 acres of old-growth forest in an area that is home to the highest concentration of wildlife in the western United States would have ldquoan immediate and irreparablerdquo adverse impact on the environment and cause irreparable harm to the nonprofitrsquos interest in preserving and protecting natural resources including wildlife habitat

Point Two(b) (25) The nonprofit could also seek and would likely obtain a preliminary injunction to stop the USFS which is likely to be granted if the nonprofitrsquos claim that the USFS violated NEPA has a strong basis in fact and law

Because the TRO would be temporary the nonprofit would need to move for a preliminary injunction to prevent the USFS from issuing a logging permit throughout the pendency of the litigation Preliminary injunctions are injunctions that seek to ldquoprotect [the] plaintiff from

24

Federal Civil Procedure Analysis

irreparable injury and to preserve the courtrsquos power to render a meaningful decision after a trial on the meritsrdquo 11A CHARLES ALAN WRIGHT ET AL FEDERAL PRACTICE AND PROCEDURE sect 2947 at 112 (2013) Rule 65 of the Federal Rules of Civil Procedure sets out the procedural requirements for preliminary injunctions Preliminary injunctions may be granted only upon notice to the adverse party FED R CIV P 65(a)(1) and only if the movant ldquogives security in an amount that the court considers proper to pay the costs and damages sustained by any party found to have been wrongfully enjoined or restrainedrdquo FED R CIV P 65(c)

While Rule 65 sets out the procedural requirements for preliminary injunctive relief it does not specify the substantive grounds upon which it may be granted The courtrsquos discretion in ruling upon a motion for a preliminary injunction ldquois exercised in conformity with historic federal equity practicerdquo 11A WRIGHT ET AL supra sect 2947 at 114 The court typically considers four factors

(1) the significance of the threat of irreparable harm to the plaintiff if the injunction is not granted (2) the balance between this harm and the injury that granting the injunction would inflict on the defendant (3) the probability that the plaintiff will succeed on the merits and (4) the public interest

Id sect 2948 at 122ndash24 accord Habitat Educ Center v Bosworth 363 F Supp 2d 1070 1088 (ED Wis 2005) The most important of these factors is the risk of irreparable harm to the plaintiff 11A WRIGHT ET AL supra sect 29481 at 129 If the plaintiff has an adequate remedy at law (eg if money damages can compensate the plaintiff for its loss) then a preliminary injunction will be denied Id sect 29481

Here a court would likely conclude that the potential for environmental damage to the forest creates a significant threat of irreparable harm ldquo[E]nvironmental injury is often irreparable Courts have recognized that logging such as would occur [here] can have longshyterm environmental consequences and thus satisfy the irreparable injury criterionrdquo Habitat Educ Center 363 F Supp 2d at 1089 (citing Idaho Sporting Congress Inc v Alexander 222 F3d 562 569 (9th Cir 2000) (noting that the imminent and continuing logging activities presented ldquoevidence of environmental harm sufficient to tip the balance in favor of injunctive reliefrdquo)) Neighbors of Cuddy Mountain v US Forest Service 137 F3d 1372 1382 (9th Cir 1998) (stating that ldquo[t]he old growth forests plaintiffs seek to protect would if cut take hundreds of years to reproducerdquo) (internal citation omitted)) see also 11C WRIGHT ET AL supra sect 29481 at 151 (noting that ldquoa preliminary injunction has been issued to prevent harm to the environmentrdquo)

The second factor the balance between the harm to the plaintiff and the harm the defendant will suffer if the injunction is issued also appears to support issuance of a preliminary injunction here The USFS will have to wait before it can develop the Scenic National Forest and the logging company may lose money if the delay is prolonged These economic harms could be compensated monetarily if an injunction is issued inappropriately Where ldquoan injunction bond can compensate [the] defendant for any harm the injunction is likely to inflict the balance should be struck in favor of [the] plaintiffrdquo Id sect 29482 at 192 See also Habitat Educ Center 363 F Supp 2d at 1089 (stating that ldquothe relative absence of harmful effects on the Forest Service weighs in favor of granting the injunctionrdquo)

The third factor is the likelihood that the plaintiff will prevail on the merits Although there is limited information concerning the merits of the action the nonprofit alleges that the federal statute (NEPA) requires an environmental impact statement and further states that the USFS created no environmental impact analysis or statement at all Assuming that those

25

Federal Civil Procedure Analysis

allegations are correct it seems plausible to conclude that the nonprofit will be able to show a likelihood of success on the merits

Finally courts deciding whether or not to issue preliminary injunctive relief are to consider the public interest ldquoFocusing on this factor is another way of inquiring whether there are policy considerations that bear on whether the order should issuerdquo 11C WRIGHT ET AL supra sect 29484 at 214 If the court concludes that the nonprofit is likely to succeed on its NEPA claim because the USFS wrongfully failed to conduct an environmental impact assessment it is likely to find that the public interest would be served by restraining the USFS from proceeding with logging in a national forest See Heartwood Inc v US Forest Service 73 F Supp 2d 962 979 (SD Ill 1999) affrsquod on other grounds 230 F3d 947 (7th Cir 2000) (ldquoviolations by federal agencies of NEPArsquos provisions as established by Congress harm the public as well as the environmentrdquo)

Thus a court is very likely to grant a preliminary injunction if it concludes that the nonprofit has a significant likelihood of success on the merits

26

EVIDENCE ANALYSIS (Evidence ID IIA amp C)

ANALYSIS

Legal Problems

(1) Under what circumstances can evidence of prior convictions be used to impeach a witnessrsquos credibility in a civil case

(1)(a) May the inmatersquos credibility be impeached by evidence of a 12-year-old felony drug conviction if he was released from prison 9 years ago

(1)(b) May the inmatersquos credibility be impeached by evidence of an 8-year-old misdemeanor perjury conviction that was punishable by 1 year in jail if he pleaded guilty and was sentenced only to pay a $5000 fine

(1)(c) May the inmatersquos credibility be impeached by evidence of a 7-year-old sexual assault conviction if the inmate is still serving a 10-year prison sentence and the victim was his 13-year-old daughter

(2)(a) May the guardrsquos credibility be impeached by cross-examination regarding specific instances of misconduct (ie lying on his reacutesumeacute) relevant to credibility

(2)(b) May the guardrsquos credibility be impeached by admission of extrinsic evidence (his reacutesumeacute and academic transcript) offered to prove specific instances of misconduct relevant to credibility

DISCUSSION

Summary

Under the Federal Rules of Evidence witnesses can be impeached with evidence of prior convictions andor specific instances of misconduct Whether evidence of prior convictions should be admitted to impeach generally depends on the nature of the crime the amount of time that has passed and (only in criminal cases) whether the ldquowitnessrdquo is the defendant FED R EVID 609(a)

In this civil case evidence of the inmatersquos conviction for distribution of marijuana should be admitted to impeach the inmate because he was convicted of a felony and was released from prison fewer than 10 years ago FED R EVID 609(a)(1) Credibility is critically important in this case because the jury will hear conflicting testimony from the two disputing parties and there were no other eyewitnesses to the altercation Under Rule 609(a)(1) the inmatersquos conviction should be admitted because it has some bearing on his credibility and its probative value is not substantially outweighed by concerns of unfair prejudice confusion or delay Id

Evidence of the inmatersquos misdemeanor conviction for perjury must be admitted because the crime ldquorequired provingmdashor the witnessrsquos admittingmdasha dishonest act or false statementrdquo by the inmate FED R EVID 609(a)(2)

27

Evidence Analysis

Evidence of the inmatersquos felony conviction for sexual assault should be excluded because its probative value is substantially outweighed by the danger of unfair prejudice to the inmate based on the heinous nature of the crime FED R EVID 609(a)(1) In the alternative the judge could limit the evidence relating to this conviction by excluding details of the inmatersquos crime

In all civil (and criminal) cases witnesses can also be impeached with evidence of specific instances of prior misconduct that did not result in a conviction FED R EVID 608(b) Pursuant to Rule 608(b) misconduct probative of untruthfulness can be inquired into on cross-examination but cannot be proved through extrinsic evidence Id Thus the inmatersquos counsel should be permitted to cross-examine the guard regarding the false statement in the guardrsquos reacutesumeacute However extrinsic evidence of the guardrsquos misconduct (ie the guardrsquos authenticated reacutesumeacute and transcript from the local college) should not be admitted even if the guard denies wrongdoing or refuses to answer cross-examination questions about these matters Id

Point One (10) The Federal Rules of Evidence permit impeachment of witnesses with evidence of prior convictions

Whether convictions should be admitted to impeach generally depends on the nature of the crime the amount of time that has passed and (only in criminal cases) whether the ldquowitnessrdquo is the defendant FED R EVID 609(a) Under Rule 609(a) evidence of prior convictions may be admitted for the purpose of ldquoattacking a witnessrsquos character for truthfulnessrdquo Id

There are two basic types of convictions that can be admitted for the purpose of impeachment

(1) convictions for crimes ldquopunishable by death or by imprisonment for more than one yearrdquo (which generally correlates to ldquofeloniesrdquo) FED R EVID 609(a)(1) and (2) convictions ldquofor any crimes regardless of the punishment if the court can readily determine that establishing the elements of the crime required provingmdashor the witnessrsquos admittingmdasha dishonest act or false statementrdquo FED R EVID 609(a)(2)

Pursuant to Rule 609(a)(1) in civil cases the admission of evidence of a felony conviction is ldquosubject to Rule 403 [which says that a court may exclude relevant evidence if its probative value is substantially outweighed by other factors]rdquo FED R EVID 609(a)(1) However Rule 403 does not protect the witness against admission of prior convictions involving dishonestymdashwhich must be admitted by the court FED R EVID 609(a)(2)

Finally Federal Rule of Evidence 609(b) contains the presumption that a conviction that is more than 10 years old or where more than 10 years has passed since the witnessrsquos release from confinement (whichever is later) should not be admitted unless ldquoits probative value supported by specific facts and circumstances substantially outweighs its prejudicial effectrdquo and the proponent has provided the adverse party with reasonable written notice FED R EVID 609(b)

Point One(a) (25) The court should admit evidence of the inmatersquos 12-year-old felony marijuana distribution conviction

The inmatersquos conviction for marijuana distribution was for a felony punishable by imprisonment for more than one year See FED R EVID 609(a)(1) Moreover although the conviction was 12 years ago the 10-year time limit of Rule 609(b) is not exceeded because that time limit runs

28

Evidence Analysis

from the date of either ldquothe witnessrsquos conviction or release from confinement for it whichever is laterrdquo FED R EVID 609(b) Because the inmate served three years in prison he was released from confinement nine years ago

However pursuant to Rule 609(a)(1) the admission of felony convictions to impeach a witness in a civil case is ldquosubject to Rule 403rdquo FED R EVID 609(a)(1) Neither Rule 609(a) nor the advisory committee notes specify which factors courts should consider when balancing the probative value of a conviction against the dangers identified in Rule 403 (which include (1) unfair prejudice (2) confusion of the issues (3) misleading the jury (4) waste of time or undue delay and (5) needless presentation of cumulative evidence) FED R EVID 403

In this case credibility is very important because the evidence consists primarily of the testimony of the disputing parties and there were no other eyewitnesses to the altercation This enhances the probative value of any evidence bearing on the inmatersquos credibility A court is likely to conclude that the inmatersquos prior felony drug conviction is relevant to his credibility See eg United States v Brito 427 F3d 53 64 (1st Cir 2005) (ldquoPrior drug-trafficking crimes are generally viewed as having some bearing on veracityrdquo) Although the probative value of any conviction diminishes with age see eg United States v Brewer 451 F Supp 50 53 (ED Tenn 1978) the inmatersquos ongoing problems with the law suggest that he has continued (and even escalated) his criminal behavior over the past nine years The court should admit this evidence because its probative value is not substantially outweighed by any Rule 403 concerns Specifically any prejudice to the inmate would be slight because the conviction is unrelated to the altercation at issue and the conviction was not for a heinous crime that might inflame the jury

[NOTE Whether an examinee identifies the jury instruction as containing a ldquoconclusiverdquo or ldquomandatoryrdquo presumption is less important than the examineersquos analysis of the constitutional infirmities]

Point One(b) (15) The court must admit evidence of the inmatersquos eight-year-old misdemeanor conviction because perjury is a crime of dishonesty

Rule 609(a)(2) provides that evidence of a criminal conviction ldquomust be admitted if the court can readily determine that establishing the elements of the crime required provingmdashor the witnessrsquos admittingmdasha dishonest act or false statementrdquo FED R EVID 609(a)(2) The inmatersquos conviction for perjury would have necessarily required proving that the inmate engaged in an act of dishonesty This conviction occurred within the past 10 years so it ldquomust be admittedrdquo because in contrast to Rule 609(a)(1) (discussed in Point One(a)) admission under Rule 609(a)(2) is mandatory and not subject to Rule 403

Point One(c) (20) The court should exclude evidence of the inmatersquos seven-year-old felony sexual assault conviction because the probative value of this evidence is substantially outweighed by the danger of unfair prejudice In the alternative the details of the prior conviction could be excluded

The inmatersquos conviction for felony sexual assault was seven years ago and he has not yet been released from incarceration so Rule 609(a) but not 609(b) is applicable here FED R EVID 609(a) This conviction is therefore admissible to impeach the inmate unless its probative value is substantially outweighed by the danger of unfair prejudice or any other Rule 403 concern Id

29

Evidence Analysis

Sex crimes are generally not considered relevant to credibility see Hopkins v State 639 So 2d 1247 1254 (Miss 1993) so the probative value of this conviction is relatively low Moreover the heinous nature of the inmatersquos crime (sexual assault on his daughter) makes the danger of unfair prejudice to the inmate very high Thus the court should exclude evidence of the conviction because it was for a heinous offense that is likely to inflame the jury and it has little bearing on credibility See eg United States v Beahm 664 F2d 414 419 (4th Cir 1981)

As an alternative to excluding this evidence the judge could minimize the unfair prejudice to the inmate by permitting limited cross-examination but refusing to allow specific questions about the nature of the inmatersquos conviction For example a court could limit cross-examination to the fact that the inmate was convicted of a ldquofelonyrdquo or perhaps that he was convicted of a ldquosexual assaultrdquo without identifying the victim However because evidence of the inmatersquos prior convictions can be admitted solely for the purpose of enabling the jury to assess his credibility and because his two earlier convictions should have already been admitted the court should exclude all evidence of the felony sexual assault conviction

Point Two(a) (15) The court should permit the inmatersquos counsel to cross-examine the guard regarding the false statement in his reacutesumeacute because the guardrsquos misconduct bears on his truthfulness

The inmate wishes to cross-examine the guard about his prior dishonest behaviormdashlying on his reacutesumeacutemdashthat did not involve a criminal conviction Rule 608(b) allows witnesses to be cross-examined about specific instances of prior non-conviction misconduct probative of untruthfulness ldquoin order to attack the witnessrsquos character for truthfulnessrdquo FED R EVID 608(b)

The courtrsquos decision to allow cross-examination about the guardrsquos prior dishonest behavior depends on the probative value of such evidence balanced against the danger of unfair prejudice to the guard or any other Rule 403 concern FED R EVID 403 Here the guardrsquos false statement on his reacutesumeacute that he obtained a degree in Criminal Justice is highly probative of his untruthfulness because it grossly misrepresents his actual academic record was made recently and was made with the intent to deceive Because the probative value of this evidence is very strong and is not substantially outweighed by any Rule 403 concerns cross-examination of the guard on this topic should be permitted The court may also consider it fair to permit this cross-examination of the guard on these matters assuming that one or more of the inmatersquos prior convictions have been admitted to impeach his credibility

Point Two(b) (15) The court should exclude extrinsic evidence of the guardrsquos non-conviction misconduct even if the guard denies wrongdoing or refuses to answer questions about the matter

Although Rule 608(b) allows cross-examination about specific instances of prior misconduct probative of untruthfulness ldquoextrinsic evidencerdquo offered to prove such misconduct is not admissible FED R EVID 608(b) The rationale for this rule is that allowing the introduction of extrinsic evidence of prior misconduct by witnesses when these acts are relevant only to the witnessesrsquo truthfulness and not to the main issues in the case would create too great a risk of confusing the jury and unduly delaying the trial The court does not have discretion to admit this extrinsic evidence See eg United States v Elliot 89 F3d 1360 1368 (8th Cir 1996)

30

Evidence Analysis

Here the inmatersquos counsel may cross-examine the guard about the false statement on his reacutesumeacute However the inmatersquos counsel must accept the guardrsquos response Even if the guard denies wrongdoing or refuses to answer questions about the matter the inmatersquos counsel cannot introduce the guardrsquos reacutesumeacute or the transcript from the local college to prove the guardrsquos misconduct

31

CORPORATIONS ANALYSIS (Corporations VA2 IX)

ANALYSIS

Legal Problems

(1) Do shareholders have the authority to amend a corporationrsquos bylaws with respect to director nominations

(2) Do board-approved bylaws on a particular subject here nomination of directors preempt subsequent conflicting bylaw amendments by shareholders

(3) Is a suit challenging both managementrsquos refusal to include the proposed bylaw amendment in Megarsquos proxy statement and the boardrsquos amendment of the bylaws dealing with nomination of directors a direct or derivative suit

DISCUSSION

Summary

The voting and litigation rights of the shareholders of Mega are subject to the provisions of the Model Business Corporations Act (MBCA)

The investorrsquos proposed bylaw provision is not inconsistent with state law Under the MBCA shareholders may amend the bylaws when the amendment deals with a proper matter for the corporationrsquos bylaws such as procedures for nominating directors

The Mega boardrsquos bylaw amendment does not preempt the investorrsquos proposed bylaw provision or the Mega shareholdersrsquo power to approve it While shareholders can limit the boardrsquos power to amend or repeal the bylaws the board cannot limit the shareholdersrsquo power

Whether the investor must make a demand on Megarsquos board depends on how the investor frames its claim If the investor claims a violation of shareholder voting rights the claim is direct and pre-suit demand on the board is not required If on the other hand the investor claims that the directors violated their fiduciary duties by amending the bylaws to entrench themselves the claim is derivative and a pre-suit demand is required

Point One (30) Shareholders may amend the corporationrsquos bylaws where the proposed bylaw provision relates to procedural matters typically included in the bylaws such as the nomination of directors

Internal affairs of the corporation such as the conduct of shareholder meetings and election of directors are subject to the corporate law of the state of incorporation See McDermott Inc v Lewis 531 A2d 206 (Del 1987) (applying law of jurisdiction where corporation was incorporated in case involving voting rights) This statersquos corporate statute is modeled on the MBCA

Under the MBCA ldquoshareholders may amend the corporationrsquos bylawsrdquo MBCA sect 1020(a) Thus the only question is whether the bylaws can specify the procedures for shareholder nomination of directors

32

Corporations Analysis

The MBCA states that the bylaws ldquomay contain any provision that is not inconsistent with law or the articles of incorporationrdquo MBCA sect 206(b) In addition the MBCA was revised in 2009 to address shareholder nomination of directors in public corporations (known as ldquoproxy accessrdquo) and specifies that the bylaws ldquomay contain a requirement that the corporation include in its [proxy materials] one or more individuals nominated by a shareholderrdquo MBCA sect 206(c)(1) see Committee on Corporate Laws ABA Section of Business Law Report on the Roles of Boards of Directors and Shareholders of Publicly Owned Corporations and Changes to the Model Business Corporations ActmdashAdoption of Shareholder Proxy Access Amendments to Chapters 2 and 10 65 BUS LAWYER 1105 (2010)

The inclusion of director-nomination procedures in the bylaws is consistent with practice and is recognized by the Delaware courts whose views on corporate law carry significant weight Typically the procedures for nomination of directors are found in the bylaws See 1 COX amp HAZEN TREATISE ON THE LAW OF CORPORATIONS sect 312 (3d ed 2011) see also 4 FLETCHER CORP FORMS ANN PART III ch 21 (2013) (including sample bylaws that permit nomination of directors by shareholders) The Delaware Supreme Court has confirmed that the bylaws may ldquodefine the process and proceduresrdquo for director elections See CA Inc v AFSCME Employees Pension Plan 953 A2d 227 (Del 2008) (concluding that bylaw amendment requiring reimbursement of election expenses to certain successful shareholder nominators is ldquoproper subjectrdquo under Delaware law)

[NOTE The question of the proper scope of the bylaws can be answered using the more general MBCA sect 206(b) or the 2009 MBCA revision adding sect 206(c)(1) (adopted in CT ME VA) In addition some examinees might raise the point that shareholder proposals may not compel the board to take action such as by including shareholder nominations in the companyrsquos proxy materials on the theory that the ldquobusiness and affairsrdquo of the corporation are to be managed by the board See MBCA sect 801(b) Although shareholders are generally limited to adopting precatory resolutions that recommend or encourage board action this limitation does not apply when shareholders have specific authority to take binding action on their ownmdashsuch as to amend the bylaws]

Point Two (30) Shareholders can amend (or repeal) board-approved bylaws Further shareholders can limit the boardrsquos power to later amend and repeal a shareholder-approved bylaw

Under the MBCA shareholders have the power to amend the bylaws See Point One The board shares this power with the shareholders unless (1) the corporationrsquos articles ldquoreserve that power exclusively to the shareholdersrdquo or (2) ldquothe shareholders in amending repealing or adopting a bylaw expressly provide that the board of directors may not amend repeal or reinstate that bylawrdquo See MBCA sect 1020(b)

Shareholder-approved bylaw provisions can amend or repeal existing bylaw provisions whether originally approved by the board or by shareholders See ALAN R PALMITER CORPORATIONS EXAMPLES AND EXPLANATIONS sect 713 (7th ed 2012) Thus the Mega boardrsquos bylaw amendmentmdashwhich set more demanding thresholds for shareholder nomination of directors than the investorrsquos proposed bylaw provisionmdashwould be superseded (repealed) if Megarsquos shareholders were to approve the investorrsquos proposal

Further a shareholder-approved bylaw generally can limit the power of the board to later amend or repeal it See MBCA sect 1020(b)(2) Thus if Megarsquos shareholders approved the bylaw

33

Corporations Analysis

provision proposed by the investor Megarsquos board could not repeal the provision because it includes a ldquono board repealrdquo clause

The revision to the MBCA in 2009 dealing with shareholder proxy access does not change this conclusion That revision specifies that a shareholder-approved bylaw dealing with director nominations may not limit the boardrsquos power to amend add or repeal ldquoany procedure or condition to such a bylaw in order to provide for a reasonable practicable and orderly processrdquo MBCA sect 206(d) Thus according to the revision if shareholders approve a bylaw amendment that limits further board changes the board would nonetheless retain the power to ldquotinkerrdquo with the bylaw to safeguard the voting process but could not repeal the shareholder-approved bylaw The Official Comment to MBCA sect 206(d) makes clear that the revision is ldquonot intended to allow the board of directors to frustrate the purpose of the shareholder-adopted proxy access provisionrdquo Thus if Megarsquos shareholders were to approve the bylaw provision proposed by the investor Megarsquos board could only amend the provision regarding its procedures or conditions in a manner consistent with its purpose of permitting proxy access for Megarsquos shareholders

[NOTE The boardrsquos attempted interference with a shareholder voting initiative may also have been a violation of the directorsrsquo fiduciary duties See Blasius Indus Inc v Atlas Corp 564 A2d 651 (Del Ch 1988) (finding that directors breached their fiduciary duties by amending bylaws and expanding size of board to thwart insurgentrsquos plan to amend bylaws and seat a majority of new directors) The call however asks examinees to consider whether shareholders or the board have ldquoprecedencerdquo over amending the corporate bylaws Thus an examineersquos answer should be framed in terms of ldquopowerrdquo and not ldquodutyrdquo]

Point Three (40) The investor need not make a demand on the board if the investor states a direct claim such as an allegation that the board interfered with the investorrsquos right to amend the bylaws But the investor must make a demand on the board if the investor states a derivative claim (on behalf of the corporation) such as an allegation that the directors sought to entrench themselves by interfering with the proposed proxy access

The MBCA generally requires that shareholders make a demand on the board of directors before initiation of a derivative suit MBCA sect 742 (shareholder may not bring derivative proceeding until written demand has been made on corporation and 90 days have expired) A derivative suit is essentially two suits in one where the plaintiff-shareholder seeks to bring on behalf of the corporation a claim that vindicates corporate rights usually based on violation of fiduciary duties PALMITER supra sect 1811 (6th ed 2009) The demand permits the board to investigate the situation identified by the shareholder and take suitable action No demand on the board is required however if the shareholder brings a direct suit to vindicate the shareholderrsquos own rights not those of the corporation

Is the suit brought by the investor derivative or direct The MBCA defines a ldquoderivative proceedingrdquo as one brought ldquoin the right of a domestic corporationrdquo MBCA sect 740(1) Thus the answer to how the investorrsquos suit should be characterized turns on what rights the investor seeks to vindicate If the investor frames its claim as one of fiduciary breach by directorsmdashfor example for failing to become adequately informed about voting procedures or for seeking to entrench themselves in office by manipulating the voting structure to avoid a shareholder insurgencymdashthen the suit is ldquoderivativerdquo and the investor must make a demand on the board See MBCA Ch 7 Subch D Introductory Comment (ldquothe derivative suit has historically been the principal method of challenging allegedly illegal action by managementrdquo)

34

Corporations Analysis

If however the investor frames its claim as one to vindicate shareholder rights the suit is direct and no demand is required For many courts the direct-derivative question turns on who is injured and who is to receive the relief sought by the plaintiff-shareholders See Tooley v Donaldson Lufkin amp Jenrette Inc 845 A2d 1031 (Del 2004) (characterizing a merger-delay claim as direct because delay of merger only harmed shareholders not corporation) Thus if the investor claims that managementrsquos refusal to include its proposed bylaw amendment in the corporationrsquos proxy materials violates its shareholder rights to initiate corporate governance reforms the suit will be direct Courts have not questioned the ability of shareholders to bring direct suits challenging board action to exclude their proposed bylaw amendments from the corporationrsquos proxy materials See JANA Master Fund Ltd v CNET Networks Inc 954 A2d 335 (Del Ch 2008) (upholding shareholderrsquos direct challenge to boardrsquos interpretation of advance-notice bylaw) Chesapeake Corp v Shore 771 A2d 293 (Del Ch 2000) (upholding shareholderrsquos direct challenge to actions by board that effectively prevented it from proposing bylaw amendments in contest for control)

Is the way that the investor frames its claim conclusive Courts have permitted shareholder-plaintiffs to challenge a transaction in a direct suit even though the same transaction could also be challenged as a fiduciary breach See Eisenberg v Flying Tiger Line Inc 451 F2d 267 (2d Cir 1971) (permitting direct suit challenging a corporate reorganization as a dilution of shareholder voting power even though reorganization may have involved conflicts of interest and thus constituted a fiduciary breach) Thus the investorrsquos choice to pursue a claim challenging the legality of managementrsquos decision to exclude the investorrsquos proposal from the corporationrsquos proxy materialsmdashrather than a possible breach of fiduciary dutymdashis likely to be respected See 3 COX amp HAZEN supra sect 153 (describing situations in which a claim can be framed as derivative or direct)

[NOTE Some issues under Delaware corporate law regarding pre-suit demand are not relevant here For example whether the Mega directors are independent and disinterested is not relevant to the MBCA requirement of a pre-suit demand As the Official Comment to MBCA sect 742 points out the MBCArsquos requirement of ldquouniversal demandrdquo gives the board ldquothe opportunity to reexamine the act complained of in the light of a potential lawsuit and take corrective actionrdquo even when the directors might be non-independent or have conflicts of interest

Nor is it relevant to the MBCA pre-suit demand requirement that the statutory 90-day waiting period may be onerous The first paragraph of MBCA sect 742 requires a pre-suit demand without exception the second paragraph of the section imposes a 90-day waiting period before a derivative suit may be brought which can be shortened if the board rejects the demand or ldquoirreparable injury to the corporation would result by waiting for the expiration of the 90-day periodrdquo The call as written asks only whether a pre-suit demand should be made and does not ask examinees to address whether the post-demand waiting period should be shortened under the ldquoirreparable injuryrdquo standard]

35

National Conference of Bar Examiners 302 South Bedford Street | Madison WI 53703-3622 Phone 608-280-8550 | Fax 608-280-8552 | TDD 608-661-1275

wwwncbexorg e-mail contactncbexorg

  • Preface
  • Description of the MEE
  • Instructions
  • July 2014 Questions
    • CRIMINAL LAW AND PROCEDURE QUESTION
    • CONTRACTS QUESTION
    • FAMILY LAW QUESTION
    • FEDERAL CIVIL PROCEDURE QUESTION
    • EVIDENCE QUESTION
    • CORPORATIONS QUESTION
      • July 2014 Analyses
        • CRIMINAL LAW AND PROCEDURE ANALYSIS
        • CONTRACTS ANALYSIS
        • FAMILY LAW ANALYSIS
        • FEDERAL CIVIL PROCEDURE ANALYSIS
        • EVIDENCE ANALYSIS
        • CORPORATIONS ANALYSIS
            • ltlt13 ASCII85EncodePages false13 AllowTransparency false13 AutoPositionEPSFiles true13 AutoRotatePages None13 Binding Left13 CalGrayProfile (Dot Gain 20)13 CalRGBProfile (sRGB IEC61966-21)13 CalCMYKProfile (US Web Coated 050SWOP051 v2)13 sRGBProfile (sRGB IEC61966-21)13 CannotEmbedFontPolicy Error13 CompatibilityLevel 1413 CompressObjects Tags13 CompressPages true13 ConvertImagesToIndexed true13 PassThroughJPEGImages true13 CreateJobTicket false13 DefaultRenderingIntent Default13 DetectBlends true13 DetectCurves 0000013 ColorConversionStrategy CMYK13 DoThumbnails false13 EmbedAllFonts true13 EmbedOpenType false13 ParseICCProfilesInComments true13 EmbedJobOptions true13 DSCReportingLevel 013 EmitDSCWarnings false13 EndPage -113 ImageMemory 104857613 LockDistillerParams false13 MaxSubsetPct 10013 Optimize true13 OPM 113 ParseDSCComments true13 ParseDSCCommentsForDocInfo true13 PreserveCopyPage true13 PreserveDICMYKValues true13 PreserveEPSInfo true13 PreserveFlatness true13 PreserveHalftoneInfo false13 PreserveOPIComments true13 PreserveOverprintSettings true13 StartPage 113 SubsetFonts true13 TransferFunctionInfo Apply13 UCRandBGInfo Preserve13 UsePrologue false13 ColorSettingsFile ()13 AlwaysEmbed [ true13 ]13 NeverEmbed [ true13 ]13 AntiAliasColorImages false13 CropColorImages true13 ColorImageMinResolution 30013 ColorImageMinResolutionPolicy OK13 DownsampleColorImages true13 ColorImageDownsampleType Bicubic13 ColorImageResolution 30013 ColorImageDepth -113 ColorImageMinDownsampleDepth 113 ColorImageDownsampleThreshold 15000013 EncodeColorImages true13 ColorImageFilter DCTEncode13 AutoFilterColorImages true13 ColorImageAutoFilterStrategy JPEG13 ColorACSImageDict ltlt13 QFactor 01513 HSamples [1 1 1 1] VSamples [1 1 1 1]13 gtgt13 ColorImageDict ltlt13 QFactor 01513 HSamples [1 1 1 1] VSamples [1 1 1 1]13 gtgt13 JPEG2000ColorACSImageDict ltlt13 TileWidth 25613 TileHeight 25613 Quality 3013 gtgt13 JPEG2000ColorImageDict ltlt13 TileWidth 25613 TileHeight 25613 Quality 3013 gtgt13 AntiAliasGrayImages false13 CropGrayImages true13 GrayImageMinResolution 30013 GrayImageMinResolutionPolicy OK13 DownsampleGrayImages true13 GrayImageDownsampleType Bicubic13 GrayImageResolution 30013 GrayImageDepth -113 GrayImageMinDownsampleDepth 213 GrayImageDownsampleThreshold 15000013 EncodeGrayImages true13 GrayImageFilter DCTEncode13 AutoFilterGrayImages true13 GrayImageAutoFilterStrategy JPEG13 GrayACSImageDict ltlt13 QFactor 01513 HSamples [1 1 1 1] VSamples [1 1 1 1]13 gtgt13 GrayImageDict ltlt13 QFactor 01513 HSamples [1 1 1 1] VSamples [1 1 1 1]13 gtgt13 JPEG2000GrayACSImageDict ltlt13 TileWidth 25613 TileHeight 25613 Quality 3013 gtgt13 JPEG2000GrayImageDict ltlt13 TileWidth 25613 TileHeight 25613 Quality 3013 gtgt13 AntiAliasMonoImages false13 CropMonoImages true13 MonoImageMinResolution 120013 MonoImageMinResolutionPolicy OK13 DownsampleMonoImages true13 MonoImageDownsampleType Bicubic13 MonoImageResolution 120013 MonoImageDepth -113 MonoImageDownsampleThreshold 15000013 EncodeMonoImages true13 MonoImageFilter CCITTFaxEncode13 MonoImageDict ltlt13 K -113 gtgt13 AllowPSXObjects false13 CheckCompliance [13 None13 ]13 PDFX1aCheck false13 PDFX3Check false13 PDFXCompliantPDFOnly false13 PDFXNoTrimBoxError true13 PDFXTrimBoxToMediaBoxOffset [13 00000013 00000013 00000013 00000013 ]13 PDFXSetBleedBoxToMediaBox true13 PDFXBleedBoxToTrimBoxOffset [13 00000013 00000013 00000013 00000013 ]13 PDFXOutputIntentProfile ()13 PDFXOutputConditionIdentifier ()13 PDFXOutputCondition ()13 PDFXRegistryName ()13 PDFXTrapped False1313 CreateJDFFile false13 Description ltlt13 ARA 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 BGR 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 CHS ltFEFF4f7f75288fd94e9b8bbe5b9a521b5efa7684002000410064006f006200650020005000440046002065876863900275284e8e9ad88d2891cf76845370524d53705237300260a853ef4ee54f7f75280020004100630072006f0062006100740020548c002000410064006f00620065002000520065006100640065007200200035002e003000204ee553ca66f49ad87248672c676562535f00521b5efa768400200050004400460020658768633002gt13 CHT ltFEFF4f7f752890194e9b8a2d7f6e5efa7acb7684002000410064006f006200650020005000440046002065874ef69069752865bc9ad854c18cea76845370524d5370523786557406300260a853ef4ee54f7f75280020004100630072006f0062006100740020548c002000410064006f00620065002000520065006100640065007200200035002e003000204ee553ca66f49ad87248672c4f86958b555f5df25efa7acb76840020005000440046002065874ef63002gt13 CZE 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 DAN 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 DEU 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 ESP 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 ETI 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 FRA 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 GRE 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 HEB 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 HRV (Za stvaranje Adobe PDF dokumenata najpogodnijih za visokokvalitetni ispis prije tiskanja koristite ove postavke Stvoreni PDF dokumenti mogu se otvoriti Acrobat i Adobe Reader 50 i kasnijim verzijama)13 HUN 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 ITA 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 JPN ltFEFF9ad854c18cea306a30d730ea30d730ec30b951fa529b7528002000410064006f0062006500200050004400460020658766f8306e4f5c6210306b4f7f75283057307e305930023053306e8a2d5b9a30674f5c62103055308c305f0020005000440046002030d530a130a430eb306f3001004100630072006f0062006100740020304a30883073002000410064006f00620065002000520065006100640065007200200035002e003000204ee5964d3067958b304f30533068304c3067304d307e305930023053306e8a2d5b9a306b306f30d530a930f330c8306e57cb30818fbc307f304c5fc59808306730593002gt13 KOR ltFEFFc7740020c124c815c7440020c0acc6a9d558c5ec0020ace0d488c9c80020c2dcd5d80020c778c1c4c5d00020ac00c7a50020c801d569d55c002000410064006f0062006500200050004400460020bb38c11cb97c0020c791c131d569b2c8b2e4002e0020c774b807ac8c0020c791c131b41c00200050004400460020bb38c11cb2940020004100630072006f0062006100740020bc0f002000410064006f00620065002000520065006100640065007200200035002e00300020c774c0c1c5d0c11c0020c5f40020c2180020c788c2b5b2c8b2e4002egt13 LTH 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 LVI 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 NLD (Gebruik deze instellingen om Adobe PDF-documenten te maken die zijn geoptimaliseerd voor prepress-afdrukken van hoge kwaliteit De gemaakte PDF-documenten kunnen worden geopend met Acrobat en Adobe Reader 50 en hoger)13 NOR ltFEFF004200720075006b00200064006900730073006500200069006e006e007300740069006c006c0069006e00670065006e0065002000740069006c002000e50020006f0070007000720065007400740065002000410064006f006200650020005000440046002d0064006f006b0075006d0065006e00740065007200200073006f006d00200065007200200062006500730074002000650067006e0065007400200066006f00720020006600f80072007400720079006b006b0073007500740073006b00720069006600740020006100760020006800f800790020006b00760061006c0069007400650074002e0020005000440046002d0064006f006b0075006d0065006e00740065006e00650020006b0061006e002000e50070006e00650073002000690020004100630072006f00620061007400200065006c006c00650072002000410064006f00620065002000520065006100640065007200200035002e003000200065006c006c00650072002000730065006e006500720065002egt13 POL 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 PTB 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 RUM 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 RUS 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 SKY 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 SLV 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 SUO 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 SVE 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 TUR ltFEFF005900fc006b00730065006b0020006b0061006c006900740065006c0069002000f6006e002000790061007a006401310072006d00610020006200610073006b013100730131006e006100200065006e0020006900790069002000750079006100620069006c006500630065006b002000410064006f006200650020005000440046002000620065006c00670065006c0065007200690020006f006c0075015f007400750072006d0061006b0020006900e70069006e00200062007500200061007900610072006c0061007201310020006b0075006c006c0061006e0131006e002e00200020004f006c0075015f0074007500720075006c0061006e0020005000440046002000620065006c00670065006c0065007200690020004100630072006f006200610074002000760065002000410064006f00620065002000520065006100640065007200200035002e003000200076006500200073006f006e0072006100730131006e00640061006b00690020007300fc007200fc006d006c00650072006c00650020006100e70131006c006100620069006c00690072002egt13 UKR 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 ENU (Use these settings to create Adobe PDF documents best suited for high-quality prepress printing Created PDF documents can be opened with Acrobat and Adobe Reader 50 and later)13 gtgt13 Namespace [13 (Adobe)13 (Common)13 (10)13 ]13 OtherNamespaces [13 ltlt13 AsReaderSpreads false13 CropImagesToFrames true13 ErrorControl WarnAndContinue13 FlattenerIgnoreSpreadOverrides false13 IncludeGuidesGrids false13 IncludeNonPrinting false13 IncludeSlug false13 Namespace [13 (Adobe)13 (InDesign)13 (40)13 ]13 OmitPlacedBitmaps false13 OmitPlacedEPS false13 OmitPlacedPDF false13 SimulateOverprint Legacy13 gtgt13 ltlt13 AddBleedMarks false13 AddColorBars false13 AddCropMarks false13 AddPageInfo false13 AddRegMarks false13 ConvertColors ConvertToCMYK13 DestinationProfileName ()13 DestinationProfileSelector DocumentCMYK13 Downsample16BitImages true13 FlattenerPreset ltlt13 PresetSelector MediumResolution13 gtgt13 FormElements false13 GenerateStructure false13 IncludeBookmarks false13 IncludeHyperlinks false13 IncludeInteractive false13 IncludeLayers false13 IncludeProfiles false13 MultimediaHandling UseObjectSettings13 Namespace [13 (Adobe)13 (CreativeSuite)13 (20)13 ]13 PDFXOutputIntentProfileSelector DocumentCMYK13 PreserveEditing true13 UntaggedCMYKHandling LeaveUntagged13 UntaggedRGBHandling UseDocumentProfile13 UseDocumentBleed false13 gtgt13 ]13gtgt setdistillerparams13ltlt13 HWResolution [2400 2400]13 PageSize [612000 792000]13gtgt setpagedevice13

Page 22: July 2014 MEE Questions and Analyses - NCBE...This publication includes the questions and analyses from the July 2014 MEE. (In the actual test, the questions are simply numbered rather

Family Law Analysis

Burnham v Superior Court 495 US 604 (1990) But personal jurisdiction over the wife is not enough to give a State A court jurisdiction to modify the State B support order

The interstate enforcement and modification of child support is governed by the Uniform Interstate Family Support Act (UIFSA) which has been adopted by all states Under UIFSA the state that originally issued a child support order (here State B) has continuing exclusive jurisdiction to modify the order if that state remains the residence of the obligee the child or the obligor and all parties do not consent to the jurisdiction of another forum See UIFSA sect 205 See also UIFSA sect 603 (ldquoA tribunal of this State shall recognize and enforce but may not modify a registered order if the issuing tribunal had jurisdictionrdquo) The wife and daughter continue to reside in State B and the wife has not consented to the jurisdiction of another forum Thus a State A court does not have jurisdiction to modify the State B child support order

[NOTE Examinees who do not discuss personal jurisdiction but fully discuss UIFSA may receive full credit]

Point One(b) (15) UIFSA does not apply to disputes over property division Thus the State A court may exercise jurisdiction over the husbandrsquos petition to modify the marital-residence-sale-proceeds provision of the State B divorce decree because it has personal jurisdiction over the wife

The State A court in which the husband brought his action has jurisdiction to adjudicate domestic relations issues The husbandrsquos petition to modify the property settlement is a domestic relations issue The courts of State A may exercise personal jurisdiction over the wife because she was personally served in State A See Burnham v Superior Court 495 US 604 (1990) see Point One(a)

UIFSA does not apply to divorce property-division disputes Thus although a State A court may not adjudicate the husbandrsquos petition to modify his child support obligations it may adjudicate his property-division claims (Even though the court has jurisdiction it may not modify the property-division award on the merits See Point Two(c))

Point Two(a) (20) A child support order may not be modified retroactively

State courts have long held that obligations to pay child support ordinarily may not be modified retroactively ldquoIf the hardship is particularly severe the courts sometimes devised a way to protect the obligor but in most instances the courts hold that retroactive modification of this kind is beyond their power and indeed the governing statute may so providerdquo HOMER H CLARK THE LAW OF DOMESTIC RELATIONSHIPS IN THE UNITED STATES 725 (2d ed 1987)

Federal law now goes further and requires the states as a condition of federal child-support funding to adopt rules that absolutely forbid retroactive modification of the support obligation See 42 USC sect 666(a)(9)(C) The states have adopted rules consistent with the federal requirements

Point Two(b) (25) It is unclear whether the husband could obtain prospective downward modification of his child support based on his voluntary acceptance of a job with a lower salary

Prospective modification of a child support order is typically available only when the petitioner can show a substantial change in circumstances See ROBERT E OLIPHANT amp NANCY VER

20

Family Law Analysis

STEEGH FAMILY LAW 213ndash15 (3d ed 2010) A significant decrease in income is typically viewed as a substantial change

However when a parent seeks to modify a child support obligation because he has voluntarily reduced his income a court will not modify the obligation based solely on the income loss Some courts refuse to modify whenever the income shift was voluntary See eg Aguiar v Aguiar 127 P3d 234 (Idaho Ct App 2005) Others look primarily to the petitionerrsquos intentions and permit downward modification if he has acted in good faith See eg In re Marriage of Horn 650 NE2d 1103 (Ill App Ct 1995) Many courts use a multifactor approach See OLIPHANT amp VER STEEGH supra 217ndash18

Here there is no question that the husbandrsquos loss of income was voluntary In a jurisdiction in which voluntary income reduction bars support modification the husbandrsquos petition would be denied

In a jurisdiction employing a good-faith or multifactor approach it is possible but not certain that the husband could obtain downward modification The evidence supports the husbandrsquos good faith his change in employment appears to be based on his new jobrsquos greater responsibilities and better promotion possibilities In a jurisdiction using a multifactor approach the court would likely also consider the impact of such a shift on the daughter the likely duration of the husbandrsquos income loss and the likelihood of a promotion that would ultimately inure to the daughterrsquos benefit Thus on these facts it is possible but by no means certain that the husband could prospectively obtain downward modification of his child support obligation to his daughter

Point Two(c)(15) A divorce property-division award is not subject to modification

A support order is aimed at meeting the post-divorce needs of the supported individual Because the future is unpredictable courts are empowered to modify a support award to take account of changed circumstances that may occur during the period in which support is paid

By contrast a property-distribution award divides assets of the marriage based on the equities at the time of divorce Because the past can be ascertained a property-division award is not subject to post-divorce modification See HARRY A KRAUSE ET AL FAMILY LAW CASES COMMENTS AND QUESTIONS 691 (6th ed 2007)

Here the husband is seeking modification of a property-division award with respect to an asset owned by the parties at the time of divorce Thus the husband may not obtain a modification of the marital-residence-sale-proceeds provision of the divorce decree based on his reduced income

21

FEDERAL CIVIL PROCEDURE ANALYSIS (Federal Civil Procedure III IVC)

ANALYSIS

Legal Problems

(1) Is the logging company entitled to join this action as a matter of right

(2)(a) May the nonprofit organization obtain a temporary restraining order to stop the USFS from issuing a logging permit

(2)(b) May the nonprofit organization obtain a preliminary injunction to stop the USFS from issuing a logging permit during the pendency of the action

DISCUSSION

Summary

The logging company is entitled to intervene in this action as a matter of right because it has an interest in the property or transaction that is the subject of the action and is so situated that its interest may be impaired or impeded as a practical matter if the action goes forward without it The logging companyrsquos interest is not adequately represented by the USFSrsquos presence in the lawsuit

The nonprofit organization may seek a temporary restraining order (TRO) followed by a preliminary injunction to prevent the USFS from issuing a logging permit pending the outcome of the action The nonprofit is likely to obtain a TRO if it can demonstrate a risk of immediate and irreparable injury The nonprofit is also likely to obtain a preliminary injunction if it can demonstrate a significant threat of irreparable harm and a likelihood of success on the merits of its National Environmental Policy Act (NEPA) claim

Point One (50) Rule 24(a) of the Federal Rules of Civil Procedure requires federal courts to allow a person to intervene in an action as a matter of right if the person a) is interested in the property or transaction that is the subject of the action b) is so situated that its interest may be impaired or impeded if the litigation goes forward without it and c) is not adequately represented by existing parties Here the logging company likely meets all three requirements and should be allowed to intervene as a matter of right

Rule 24 of the Federal Rules of Civil Procedure governs intervention the process by which a non-party to an action may join the litigation Under Rule 24(a) (intervention of right) a person must be permitted to intervene if three conditions are met (1) the movant ldquoclaims an interest relating to the property or transaction that is the subject of the actionrdquo (2) the movant ldquois so situated that disposition of the action may as a practical matter impair or impede the movantrsquos ability to protect its interestrdquo and (3) ldquoexisting partiesrdquo do not ldquoadequately represent [the movantrsquos] interestrdquo FED R CIV P 24(a) The three requirements for intervention of right are often ldquovery interrelatedrdquo 7C CHARLES ALAN WRIGHT ET AL FEDERAL PRACTICE AND PROCEDURE sect 1908 at 297 (2007 amp 2011 Supp)

22

Federal Civil Procedure Analysis

Here the court should find that the logging company meets this test First the logging company has a strong interest in the property or transaction that is the subject of this action The USFS has accepted the logging companyrsquos bid and the logging company is merely awaiting issuance of a logging permit to begin logging The nonprofit organization is seeking to prevent this logging The logging company therefore has a strong direct and substantial interest in the subject matter of the lawsuit and in having its winning bid honored and a logging permit issued See eg Kleissler v US Forest Serv 157 F3d 964 972 (3d Cir 1998) (stating that ldquo[t]imber companies have direct and substantial interests in a lawsuit aimed at halting loggingrdquo) see also Natural Resources Defense Council v US Nuclear Regulatory Commrsquon 578 F2d 1341 1343ndash 44 (10th Cir 1978) (holding that applicants whose license renewals were pending had Rule 24(a)(2) interests where the lawsuit sought to halt the license-issuing process pending preparation of environmental impact statements) See generally 7C WRIGHT ET AL supra sect 19081 at 309 (ldquoIf there is a direct substantial legally protectable interest in the proceedings it is clear that this requirement of the rule is satisfiedrdquo) Second the logging companyrsquos interest in receiving a logging permit may well be impaired as a practical matter by the outcome of the lawsuit If the USFS loses the lawsuit it will have to prepare an environmental impact statement before issuing the logging companyrsquos permit This will at a minimum delay the logging companyrsquos ability to exercise its rights and may in the long r un mean that no logging permit is ever issued Intervention of right is not limited to those that would be legally bound as a matter of preclusion doctrine Id sect 19082 at 368 Rather ldquo[t]he rule is satisfied whenever disposition of the present action would put the movant at a practical disadvantage in protecting its interestrdquo Id sect 19082 at 369 Here that condition is easily satisfied See Kleissler 157 F3d at 972 (ldquoTimber companies have direct and substantial interests in a lawsuit aimed at halting logging rdquo)

Given that the logging company has an interest that may be impaired by disposition of the action it should be allowed to intervene unless the court is persuaded that the USFS adequately represents the logging companyrsquos interest See Rule 24(a)(2) 7C WRIGHT ET AL supra sect 1909 Here it could be argued that the USFS adequately represents the logging companyrsquos interest because the USFS presumably wants the court to uphold its development plan and allow it to proceed with issuance of the logging permit which is the same relief that the logging company would seek However whether representation is truly adequate depends upon ldquo[a] discriminating appraisal of the circumstancesrdquo 7C WRIGHT ET AL supra sect 1909 at 440 Although both the government and the logging company wish to avoid the preparation of an environmental impact statement their interests are distinct The USFSrsquos interest is proper management of the national forest system while the logging companyrsquos interest is making a profit from logging the 5000-acre tract The USFSrsquos handling of the litigation is likely to be affected by a variety of policy concerns and political considerations that have nothing to do with the logging companyrsquos purely economic interest in securing the right to cut trees in the Scenic National Forest See eg Kleissler 157 F3d at 973ndash74 (ldquo[T]he government represents numerous complex and conflicting interests in matters of this nature The straightforward business interests asserted by intervenors here may become lost in the thicket of sometimes inconsistent governmental policiesrdquo)

[NOTES (1) Examinees who mistakenly analyze the logging companyrsquos case for joinder under the related but incorrect Rule 19 ldquoRequired Joinder of Partiesrdquo may receive credit Rule 19 allows existing parties to demand joinder of non-parties (or seek dismissal of the case if they canrsquot get it) There is a close relationship between Rule 24 and Rule 19 and both contain a similar standard for determining when ldquointerestedrdquo third parties are ldquoentitledrdquo or ldquorequiredrdquo to be in the lawsuit Indeed the two prongs of the Rule 24 intervention test that are discussed above

23

Federal Civil Procedure Analysis

are nearly identical to the two prongs of the Rule 19(a) required joinder test Examinees who discuss and apply the test should receive credit even if they cite Rule 19 rather than Rule 24

(2) Examinees may discuss permissive joinder Although permissive joinder is a possibility here the question asks only whether the logging company can join the action as a matter of right and a permissive joinder analysis is not responsive to the question To the extent an examinee discusses permissive joinder the analysis will focus on whether the logging company ldquohas a claim or defense that shares with the main action a common question of law or factrdquo FED R CIV P 24(b)(1)(B) The district court also ldquomust consider whether the intervention will unduly delay or prejudice the adjudication of the original partiesrsquo rightsrdquo FED R CIV P 24(b)(3) On our facts the logging companyrsquos claim for the issuance of a logging permit would certainly share common questions of law and fact with the USFSrsquos defense against the nonprofitrsquos claim There are no facts suggesting that the logging companyrsquos presence would unduly delay or otherwise prejudice adjudication of the original action Thus the district court would have discretion to permit the logging company to intervene even if it denied intervention of right]

Point Two(a) (25) The nonprofit organization could seek and would likely obtain a temporary restraining order to stop the USFS from issuing a logging permit pending a hearing on an application for a preliminary injunction

The first type of interim relief the nonprofit could seek to stop the USFS from issuing a logging permit to the logging company is a temporary restraining order (TRO) prohibiting the USFS from issuing the logging permit A TRO can be issued without notice to the adverse party but only in limited circumstances and only for a limited time FED R CIV P 65(b) To secure a TRO without notice the nonprofit would need to submit an affidavit containing specific facts that demonstrate a risk of ldquoimmediate and irreparable injuryrdquo if a permit is issued FED R CIV P 65(b)(1) In deciding whether to grant a TRO courts will also consider the same factors that are relevant in deciding whether to grant a preliminary injunction (eg the moving partyrsquos likelihood of success on the merits the balance of hardships and the public interest) See Point Two(b) infra The TRO would last only long enough for the court to consider and resolve a request by the nonprofit for a preliminary injunction but no longer than 14 days (unless the court extends it for good cause or the adverse party consents to an extension) In addition bond is required

Here the court is likely to grant the nonprofitrsquos request The nonprofit could plausibly claim that cutting down 5000 acres of old-growth forest in an area that is home to the highest concentration of wildlife in the western United States would have ldquoan immediate and irreparablerdquo adverse impact on the environment and cause irreparable harm to the nonprofitrsquos interest in preserving and protecting natural resources including wildlife habitat

Point Two(b) (25) The nonprofit could also seek and would likely obtain a preliminary injunction to stop the USFS which is likely to be granted if the nonprofitrsquos claim that the USFS violated NEPA has a strong basis in fact and law

Because the TRO would be temporary the nonprofit would need to move for a preliminary injunction to prevent the USFS from issuing a logging permit throughout the pendency of the litigation Preliminary injunctions are injunctions that seek to ldquoprotect [the] plaintiff from

24

Federal Civil Procedure Analysis

irreparable injury and to preserve the courtrsquos power to render a meaningful decision after a trial on the meritsrdquo 11A CHARLES ALAN WRIGHT ET AL FEDERAL PRACTICE AND PROCEDURE sect 2947 at 112 (2013) Rule 65 of the Federal Rules of Civil Procedure sets out the procedural requirements for preliminary injunctions Preliminary injunctions may be granted only upon notice to the adverse party FED R CIV P 65(a)(1) and only if the movant ldquogives security in an amount that the court considers proper to pay the costs and damages sustained by any party found to have been wrongfully enjoined or restrainedrdquo FED R CIV P 65(c)

While Rule 65 sets out the procedural requirements for preliminary injunctive relief it does not specify the substantive grounds upon which it may be granted The courtrsquos discretion in ruling upon a motion for a preliminary injunction ldquois exercised in conformity with historic federal equity practicerdquo 11A WRIGHT ET AL supra sect 2947 at 114 The court typically considers four factors

(1) the significance of the threat of irreparable harm to the plaintiff if the injunction is not granted (2) the balance between this harm and the injury that granting the injunction would inflict on the defendant (3) the probability that the plaintiff will succeed on the merits and (4) the public interest

Id sect 2948 at 122ndash24 accord Habitat Educ Center v Bosworth 363 F Supp 2d 1070 1088 (ED Wis 2005) The most important of these factors is the risk of irreparable harm to the plaintiff 11A WRIGHT ET AL supra sect 29481 at 129 If the plaintiff has an adequate remedy at law (eg if money damages can compensate the plaintiff for its loss) then a preliminary injunction will be denied Id sect 29481

Here a court would likely conclude that the potential for environmental damage to the forest creates a significant threat of irreparable harm ldquo[E]nvironmental injury is often irreparable Courts have recognized that logging such as would occur [here] can have longshyterm environmental consequences and thus satisfy the irreparable injury criterionrdquo Habitat Educ Center 363 F Supp 2d at 1089 (citing Idaho Sporting Congress Inc v Alexander 222 F3d 562 569 (9th Cir 2000) (noting that the imminent and continuing logging activities presented ldquoevidence of environmental harm sufficient to tip the balance in favor of injunctive reliefrdquo)) Neighbors of Cuddy Mountain v US Forest Service 137 F3d 1372 1382 (9th Cir 1998) (stating that ldquo[t]he old growth forests plaintiffs seek to protect would if cut take hundreds of years to reproducerdquo) (internal citation omitted)) see also 11C WRIGHT ET AL supra sect 29481 at 151 (noting that ldquoa preliminary injunction has been issued to prevent harm to the environmentrdquo)

The second factor the balance between the harm to the plaintiff and the harm the defendant will suffer if the injunction is issued also appears to support issuance of a preliminary injunction here The USFS will have to wait before it can develop the Scenic National Forest and the logging company may lose money if the delay is prolonged These economic harms could be compensated monetarily if an injunction is issued inappropriately Where ldquoan injunction bond can compensate [the] defendant for any harm the injunction is likely to inflict the balance should be struck in favor of [the] plaintiffrdquo Id sect 29482 at 192 See also Habitat Educ Center 363 F Supp 2d at 1089 (stating that ldquothe relative absence of harmful effects on the Forest Service weighs in favor of granting the injunctionrdquo)

The third factor is the likelihood that the plaintiff will prevail on the merits Although there is limited information concerning the merits of the action the nonprofit alleges that the federal statute (NEPA) requires an environmental impact statement and further states that the USFS created no environmental impact analysis or statement at all Assuming that those

25

Federal Civil Procedure Analysis

allegations are correct it seems plausible to conclude that the nonprofit will be able to show a likelihood of success on the merits

Finally courts deciding whether or not to issue preliminary injunctive relief are to consider the public interest ldquoFocusing on this factor is another way of inquiring whether there are policy considerations that bear on whether the order should issuerdquo 11C WRIGHT ET AL supra sect 29484 at 214 If the court concludes that the nonprofit is likely to succeed on its NEPA claim because the USFS wrongfully failed to conduct an environmental impact assessment it is likely to find that the public interest would be served by restraining the USFS from proceeding with logging in a national forest See Heartwood Inc v US Forest Service 73 F Supp 2d 962 979 (SD Ill 1999) affrsquod on other grounds 230 F3d 947 (7th Cir 2000) (ldquoviolations by federal agencies of NEPArsquos provisions as established by Congress harm the public as well as the environmentrdquo)

Thus a court is very likely to grant a preliminary injunction if it concludes that the nonprofit has a significant likelihood of success on the merits

26

EVIDENCE ANALYSIS (Evidence ID IIA amp C)

ANALYSIS

Legal Problems

(1) Under what circumstances can evidence of prior convictions be used to impeach a witnessrsquos credibility in a civil case

(1)(a) May the inmatersquos credibility be impeached by evidence of a 12-year-old felony drug conviction if he was released from prison 9 years ago

(1)(b) May the inmatersquos credibility be impeached by evidence of an 8-year-old misdemeanor perjury conviction that was punishable by 1 year in jail if he pleaded guilty and was sentenced only to pay a $5000 fine

(1)(c) May the inmatersquos credibility be impeached by evidence of a 7-year-old sexual assault conviction if the inmate is still serving a 10-year prison sentence and the victim was his 13-year-old daughter

(2)(a) May the guardrsquos credibility be impeached by cross-examination regarding specific instances of misconduct (ie lying on his reacutesumeacute) relevant to credibility

(2)(b) May the guardrsquos credibility be impeached by admission of extrinsic evidence (his reacutesumeacute and academic transcript) offered to prove specific instances of misconduct relevant to credibility

DISCUSSION

Summary

Under the Federal Rules of Evidence witnesses can be impeached with evidence of prior convictions andor specific instances of misconduct Whether evidence of prior convictions should be admitted to impeach generally depends on the nature of the crime the amount of time that has passed and (only in criminal cases) whether the ldquowitnessrdquo is the defendant FED R EVID 609(a)

In this civil case evidence of the inmatersquos conviction for distribution of marijuana should be admitted to impeach the inmate because he was convicted of a felony and was released from prison fewer than 10 years ago FED R EVID 609(a)(1) Credibility is critically important in this case because the jury will hear conflicting testimony from the two disputing parties and there were no other eyewitnesses to the altercation Under Rule 609(a)(1) the inmatersquos conviction should be admitted because it has some bearing on his credibility and its probative value is not substantially outweighed by concerns of unfair prejudice confusion or delay Id

Evidence of the inmatersquos misdemeanor conviction for perjury must be admitted because the crime ldquorequired provingmdashor the witnessrsquos admittingmdasha dishonest act or false statementrdquo by the inmate FED R EVID 609(a)(2)

27

Evidence Analysis

Evidence of the inmatersquos felony conviction for sexual assault should be excluded because its probative value is substantially outweighed by the danger of unfair prejudice to the inmate based on the heinous nature of the crime FED R EVID 609(a)(1) In the alternative the judge could limit the evidence relating to this conviction by excluding details of the inmatersquos crime

In all civil (and criminal) cases witnesses can also be impeached with evidence of specific instances of prior misconduct that did not result in a conviction FED R EVID 608(b) Pursuant to Rule 608(b) misconduct probative of untruthfulness can be inquired into on cross-examination but cannot be proved through extrinsic evidence Id Thus the inmatersquos counsel should be permitted to cross-examine the guard regarding the false statement in the guardrsquos reacutesumeacute However extrinsic evidence of the guardrsquos misconduct (ie the guardrsquos authenticated reacutesumeacute and transcript from the local college) should not be admitted even if the guard denies wrongdoing or refuses to answer cross-examination questions about these matters Id

Point One (10) The Federal Rules of Evidence permit impeachment of witnesses with evidence of prior convictions

Whether convictions should be admitted to impeach generally depends on the nature of the crime the amount of time that has passed and (only in criminal cases) whether the ldquowitnessrdquo is the defendant FED R EVID 609(a) Under Rule 609(a) evidence of prior convictions may be admitted for the purpose of ldquoattacking a witnessrsquos character for truthfulnessrdquo Id

There are two basic types of convictions that can be admitted for the purpose of impeachment

(1) convictions for crimes ldquopunishable by death or by imprisonment for more than one yearrdquo (which generally correlates to ldquofeloniesrdquo) FED R EVID 609(a)(1) and (2) convictions ldquofor any crimes regardless of the punishment if the court can readily determine that establishing the elements of the crime required provingmdashor the witnessrsquos admittingmdasha dishonest act or false statementrdquo FED R EVID 609(a)(2)

Pursuant to Rule 609(a)(1) in civil cases the admission of evidence of a felony conviction is ldquosubject to Rule 403 [which says that a court may exclude relevant evidence if its probative value is substantially outweighed by other factors]rdquo FED R EVID 609(a)(1) However Rule 403 does not protect the witness against admission of prior convictions involving dishonestymdashwhich must be admitted by the court FED R EVID 609(a)(2)

Finally Federal Rule of Evidence 609(b) contains the presumption that a conviction that is more than 10 years old or where more than 10 years has passed since the witnessrsquos release from confinement (whichever is later) should not be admitted unless ldquoits probative value supported by specific facts and circumstances substantially outweighs its prejudicial effectrdquo and the proponent has provided the adverse party with reasonable written notice FED R EVID 609(b)

Point One(a) (25) The court should admit evidence of the inmatersquos 12-year-old felony marijuana distribution conviction

The inmatersquos conviction for marijuana distribution was for a felony punishable by imprisonment for more than one year See FED R EVID 609(a)(1) Moreover although the conviction was 12 years ago the 10-year time limit of Rule 609(b) is not exceeded because that time limit runs

28

Evidence Analysis

from the date of either ldquothe witnessrsquos conviction or release from confinement for it whichever is laterrdquo FED R EVID 609(b) Because the inmate served three years in prison he was released from confinement nine years ago

However pursuant to Rule 609(a)(1) the admission of felony convictions to impeach a witness in a civil case is ldquosubject to Rule 403rdquo FED R EVID 609(a)(1) Neither Rule 609(a) nor the advisory committee notes specify which factors courts should consider when balancing the probative value of a conviction against the dangers identified in Rule 403 (which include (1) unfair prejudice (2) confusion of the issues (3) misleading the jury (4) waste of time or undue delay and (5) needless presentation of cumulative evidence) FED R EVID 403

In this case credibility is very important because the evidence consists primarily of the testimony of the disputing parties and there were no other eyewitnesses to the altercation This enhances the probative value of any evidence bearing on the inmatersquos credibility A court is likely to conclude that the inmatersquos prior felony drug conviction is relevant to his credibility See eg United States v Brito 427 F3d 53 64 (1st Cir 2005) (ldquoPrior drug-trafficking crimes are generally viewed as having some bearing on veracityrdquo) Although the probative value of any conviction diminishes with age see eg United States v Brewer 451 F Supp 50 53 (ED Tenn 1978) the inmatersquos ongoing problems with the law suggest that he has continued (and even escalated) his criminal behavior over the past nine years The court should admit this evidence because its probative value is not substantially outweighed by any Rule 403 concerns Specifically any prejudice to the inmate would be slight because the conviction is unrelated to the altercation at issue and the conviction was not for a heinous crime that might inflame the jury

[NOTE Whether an examinee identifies the jury instruction as containing a ldquoconclusiverdquo or ldquomandatoryrdquo presumption is less important than the examineersquos analysis of the constitutional infirmities]

Point One(b) (15) The court must admit evidence of the inmatersquos eight-year-old misdemeanor conviction because perjury is a crime of dishonesty

Rule 609(a)(2) provides that evidence of a criminal conviction ldquomust be admitted if the court can readily determine that establishing the elements of the crime required provingmdashor the witnessrsquos admittingmdasha dishonest act or false statementrdquo FED R EVID 609(a)(2) The inmatersquos conviction for perjury would have necessarily required proving that the inmate engaged in an act of dishonesty This conviction occurred within the past 10 years so it ldquomust be admittedrdquo because in contrast to Rule 609(a)(1) (discussed in Point One(a)) admission under Rule 609(a)(2) is mandatory and not subject to Rule 403

Point One(c) (20) The court should exclude evidence of the inmatersquos seven-year-old felony sexual assault conviction because the probative value of this evidence is substantially outweighed by the danger of unfair prejudice In the alternative the details of the prior conviction could be excluded

The inmatersquos conviction for felony sexual assault was seven years ago and he has not yet been released from incarceration so Rule 609(a) but not 609(b) is applicable here FED R EVID 609(a) This conviction is therefore admissible to impeach the inmate unless its probative value is substantially outweighed by the danger of unfair prejudice or any other Rule 403 concern Id

29

Evidence Analysis

Sex crimes are generally not considered relevant to credibility see Hopkins v State 639 So 2d 1247 1254 (Miss 1993) so the probative value of this conviction is relatively low Moreover the heinous nature of the inmatersquos crime (sexual assault on his daughter) makes the danger of unfair prejudice to the inmate very high Thus the court should exclude evidence of the conviction because it was for a heinous offense that is likely to inflame the jury and it has little bearing on credibility See eg United States v Beahm 664 F2d 414 419 (4th Cir 1981)

As an alternative to excluding this evidence the judge could minimize the unfair prejudice to the inmate by permitting limited cross-examination but refusing to allow specific questions about the nature of the inmatersquos conviction For example a court could limit cross-examination to the fact that the inmate was convicted of a ldquofelonyrdquo or perhaps that he was convicted of a ldquosexual assaultrdquo without identifying the victim However because evidence of the inmatersquos prior convictions can be admitted solely for the purpose of enabling the jury to assess his credibility and because his two earlier convictions should have already been admitted the court should exclude all evidence of the felony sexual assault conviction

Point Two(a) (15) The court should permit the inmatersquos counsel to cross-examine the guard regarding the false statement in his reacutesumeacute because the guardrsquos misconduct bears on his truthfulness

The inmate wishes to cross-examine the guard about his prior dishonest behaviormdashlying on his reacutesumeacutemdashthat did not involve a criminal conviction Rule 608(b) allows witnesses to be cross-examined about specific instances of prior non-conviction misconduct probative of untruthfulness ldquoin order to attack the witnessrsquos character for truthfulnessrdquo FED R EVID 608(b)

The courtrsquos decision to allow cross-examination about the guardrsquos prior dishonest behavior depends on the probative value of such evidence balanced against the danger of unfair prejudice to the guard or any other Rule 403 concern FED R EVID 403 Here the guardrsquos false statement on his reacutesumeacute that he obtained a degree in Criminal Justice is highly probative of his untruthfulness because it grossly misrepresents his actual academic record was made recently and was made with the intent to deceive Because the probative value of this evidence is very strong and is not substantially outweighed by any Rule 403 concerns cross-examination of the guard on this topic should be permitted The court may also consider it fair to permit this cross-examination of the guard on these matters assuming that one or more of the inmatersquos prior convictions have been admitted to impeach his credibility

Point Two(b) (15) The court should exclude extrinsic evidence of the guardrsquos non-conviction misconduct even if the guard denies wrongdoing or refuses to answer questions about the matter

Although Rule 608(b) allows cross-examination about specific instances of prior misconduct probative of untruthfulness ldquoextrinsic evidencerdquo offered to prove such misconduct is not admissible FED R EVID 608(b) The rationale for this rule is that allowing the introduction of extrinsic evidence of prior misconduct by witnesses when these acts are relevant only to the witnessesrsquo truthfulness and not to the main issues in the case would create too great a risk of confusing the jury and unduly delaying the trial The court does not have discretion to admit this extrinsic evidence See eg United States v Elliot 89 F3d 1360 1368 (8th Cir 1996)

30

Evidence Analysis

Here the inmatersquos counsel may cross-examine the guard about the false statement on his reacutesumeacute However the inmatersquos counsel must accept the guardrsquos response Even if the guard denies wrongdoing or refuses to answer questions about the matter the inmatersquos counsel cannot introduce the guardrsquos reacutesumeacute or the transcript from the local college to prove the guardrsquos misconduct

31

CORPORATIONS ANALYSIS (Corporations VA2 IX)

ANALYSIS

Legal Problems

(1) Do shareholders have the authority to amend a corporationrsquos bylaws with respect to director nominations

(2) Do board-approved bylaws on a particular subject here nomination of directors preempt subsequent conflicting bylaw amendments by shareholders

(3) Is a suit challenging both managementrsquos refusal to include the proposed bylaw amendment in Megarsquos proxy statement and the boardrsquos amendment of the bylaws dealing with nomination of directors a direct or derivative suit

DISCUSSION

Summary

The voting and litigation rights of the shareholders of Mega are subject to the provisions of the Model Business Corporations Act (MBCA)

The investorrsquos proposed bylaw provision is not inconsistent with state law Under the MBCA shareholders may amend the bylaws when the amendment deals with a proper matter for the corporationrsquos bylaws such as procedures for nominating directors

The Mega boardrsquos bylaw amendment does not preempt the investorrsquos proposed bylaw provision or the Mega shareholdersrsquo power to approve it While shareholders can limit the boardrsquos power to amend or repeal the bylaws the board cannot limit the shareholdersrsquo power

Whether the investor must make a demand on Megarsquos board depends on how the investor frames its claim If the investor claims a violation of shareholder voting rights the claim is direct and pre-suit demand on the board is not required If on the other hand the investor claims that the directors violated their fiduciary duties by amending the bylaws to entrench themselves the claim is derivative and a pre-suit demand is required

Point One (30) Shareholders may amend the corporationrsquos bylaws where the proposed bylaw provision relates to procedural matters typically included in the bylaws such as the nomination of directors

Internal affairs of the corporation such as the conduct of shareholder meetings and election of directors are subject to the corporate law of the state of incorporation See McDermott Inc v Lewis 531 A2d 206 (Del 1987) (applying law of jurisdiction where corporation was incorporated in case involving voting rights) This statersquos corporate statute is modeled on the MBCA

Under the MBCA ldquoshareholders may amend the corporationrsquos bylawsrdquo MBCA sect 1020(a) Thus the only question is whether the bylaws can specify the procedures for shareholder nomination of directors

32

Corporations Analysis

The MBCA states that the bylaws ldquomay contain any provision that is not inconsistent with law or the articles of incorporationrdquo MBCA sect 206(b) In addition the MBCA was revised in 2009 to address shareholder nomination of directors in public corporations (known as ldquoproxy accessrdquo) and specifies that the bylaws ldquomay contain a requirement that the corporation include in its [proxy materials] one or more individuals nominated by a shareholderrdquo MBCA sect 206(c)(1) see Committee on Corporate Laws ABA Section of Business Law Report on the Roles of Boards of Directors and Shareholders of Publicly Owned Corporations and Changes to the Model Business Corporations ActmdashAdoption of Shareholder Proxy Access Amendments to Chapters 2 and 10 65 BUS LAWYER 1105 (2010)

The inclusion of director-nomination procedures in the bylaws is consistent with practice and is recognized by the Delaware courts whose views on corporate law carry significant weight Typically the procedures for nomination of directors are found in the bylaws See 1 COX amp HAZEN TREATISE ON THE LAW OF CORPORATIONS sect 312 (3d ed 2011) see also 4 FLETCHER CORP FORMS ANN PART III ch 21 (2013) (including sample bylaws that permit nomination of directors by shareholders) The Delaware Supreme Court has confirmed that the bylaws may ldquodefine the process and proceduresrdquo for director elections See CA Inc v AFSCME Employees Pension Plan 953 A2d 227 (Del 2008) (concluding that bylaw amendment requiring reimbursement of election expenses to certain successful shareholder nominators is ldquoproper subjectrdquo under Delaware law)

[NOTE The question of the proper scope of the bylaws can be answered using the more general MBCA sect 206(b) or the 2009 MBCA revision adding sect 206(c)(1) (adopted in CT ME VA) In addition some examinees might raise the point that shareholder proposals may not compel the board to take action such as by including shareholder nominations in the companyrsquos proxy materials on the theory that the ldquobusiness and affairsrdquo of the corporation are to be managed by the board See MBCA sect 801(b) Although shareholders are generally limited to adopting precatory resolutions that recommend or encourage board action this limitation does not apply when shareholders have specific authority to take binding action on their ownmdashsuch as to amend the bylaws]

Point Two (30) Shareholders can amend (or repeal) board-approved bylaws Further shareholders can limit the boardrsquos power to later amend and repeal a shareholder-approved bylaw

Under the MBCA shareholders have the power to amend the bylaws See Point One The board shares this power with the shareholders unless (1) the corporationrsquos articles ldquoreserve that power exclusively to the shareholdersrdquo or (2) ldquothe shareholders in amending repealing or adopting a bylaw expressly provide that the board of directors may not amend repeal or reinstate that bylawrdquo See MBCA sect 1020(b)

Shareholder-approved bylaw provisions can amend or repeal existing bylaw provisions whether originally approved by the board or by shareholders See ALAN R PALMITER CORPORATIONS EXAMPLES AND EXPLANATIONS sect 713 (7th ed 2012) Thus the Mega boardrsquos bylaw amendmentmdashwhich set more demanding thresholds for shareholder nomination of directors than the investorrsquos proposed bylaw provisionmdashwould be superseded (repealed) if Megarsquos shareholders were to approve the investorrsquos proposal

Further a shareholder-approved bylaw generally can limit the power of the board to later amend or repeal it See MBCA sect 1020(b)(2) Thus if Megarsquos shareholders approved the bylaw

33

Corporations Analysis

provision proposed by the investor Megarsquos board could not repeal the provision because it includes a ldquono board repealrdquo clause

The revision to the MBCA in 2009 dealing with shareholder proxy access does not change this conclusion That revision specifies that a shareholder-approved bylaw dealing with director nominations may not limit the boardrsquos power to amend add or repeal ldquoany procedure or condition to such a bylaw in order to provide for a reasonable practicable and orderly processrdquo MBCA sect 206(d) Thus according to the revision if shareholders approve a bylaw amendment that limits further board changes the board would nonetheless retain the power to ldquotinkerrdquo with the bylaw to safeguard the voting process but could not repeal the shareholder-approved bylaw The Official Comment to MBCA sect 206(d) makes clear that the revision is ldquonot intended to allow the board of directors to frustrate the purpose of the shareholder-adopted proxy access provisionrdquo Thus if Megarsquos shareholders were to approve the bylaw provision proposed by the investor Megarsquos board could only amend the provision regarding its procedures or conditions in a manner consistent with its purpose of permitting proxy access for Megarsquos shareholders

[NOTE The boardrsquos attempted interference with a shareholder voting initiative may also have been a violation of the directorsrsquo fiduciary duties See Blasius Indus Inc v Atlas Corp 564 A2d 651 (Del Ch 1988) (finding that directors breached their fiduciary duties by amending bylaws and expanding size of board to thwart insurgentrsquos plan to amend bylaws and seat a majority of new directors) The call however asks examinees to consider whether shareholders or the board have ldquoprecedencerdquo over amending the corporate bylaws Thus an examineersquos answer should be framed in terms of ldquopowerrdquo and not ldquodutyrdquo]

Point Three (40) The investor need not make a demand on the board if the investor states a direct claim such as an allegation that the board interfered with the investorrsquos right to amend the bylaws But the investor must make a demand on the board if the investor states a derivative claim (on behalf of the corporation) such as an allegation that the directors sought to entrench themselves by interfering with the proposed proxy access

The MBCA generally requires that shareholders make a demand on the board of directors before initiation of a derivative suit MBCA sect 742 (shareholder may not bring derivative proceeding until written demand has been made on corporation and 90 days have expired) A derivative suit is essentially two suits in one where the plaintiff-shareholder seeks to bring on behalf of the corporation a claim that vindicates corporate rights usually based on violation of fiduciary duties PALMITER supra sect 1811 (6th ed 2009) The demand permits the board to investigate the situation identified by the shareholder and take suitable action No demand on the board is required however if the shareholder brings a direct suit to vindicate the shareholderrsquos own rights not those of the corporation

Is the suit brought by the investor derivative or direct The MBCA defines a ldquoderivative proceedingrdquo as one brought ldquoin the right of a domestic corporationrdquo MBCA sect 740(1) Thus the answer to how the investorrsquos suit should be characterized turns on what rights the investor seeks to vindicate If the investor frames its claim as one of fiduciary breach by directorsmdashfor example for failing to become adequately informed about voting procedures or for seeking to entrench themselves in office by manipulating the voting structure to avoid a shareholder insurgencymdashthen the suit is ldquoderivativerdquo and the investor must make a demand on the board See MBCA Ch 7 Subch D Introductory Comment (ldquothe derivative suit has historically been the principal method of challenging allegedly illegal action by managementrdquo)

34

Corporations Analysis

If however the investor frames its claim as one to vindicate shareholder rights the suit is direct and no demand is required For many courts the direct-derivative question turns on who is injured and who is to receive the relief sought by the plaintiff-shareholders See Tooley v Donaldson Lufkin amp Jenrette Inc 845 A2d 1031 (Del 2004) (characterizing a merger-delay claim as direct because delay of merger only harmed shareholders not corporation) Thus if the investor claims that managementrsquos refusal to include its proposed bylaw amendment in the corporationrsquos proxy materials violates its shareholder rights to initiate corporate governance reforms the suit will be direct Courts have not questioned the ability of shareholders to bring direct suits challenging board action to exclude their proposed bylaw amendments from the corporationrsquos proxy materials See JANA Master Fund Ltd v CNET Networks Inc 954 A2d 335 (Del Ch 2008) (upholding shareholderrsquos direct challenge to boardrsquos interpretation of advance-notice bylaw) Chesapeake Corp v Shore 771 A2d 293 (Del Ch 2000) (upholding shareholderrsquos direct challenge to actions by board that effectively prevented it from proposing bylaw amendments in contest for control)

Is the way that the investor frames its claim conclusive Courts have permitted shareholder-plaintiffs to challenge a transaction in a direct suit even though the same transaction could also be challenged as a fiduciary breach See Eisenberg v Flying Tiger Line Inc 451 F2d 267 (2d Cir 1971) (permitting direct suit challenging a corporate reorganization as a dilution of shareholder voting power even though reorganization may have involved conflicts of interest and thus constituted a fiduciary breach) Thus the investorrsquos choice to pursue a claim challenging the legality of managementrsquos decision to exclude the investorrsquos proposal from the corporationrsquos proxy materialsmdashrather than a possible breach of fiduciary dutymdashis likely to be respected See 3 COX amp HAZEN supra sect 153 (describing situations in which a claim can be framed as derivative or direct)

[NOTE Some issues under Delaware corporate law regarding pre-suit demand are not relevant here For example whether the Mega directors are independent and disinterested is not relevant to the MBCA requirement of a pre-suit demand As the Official Comment to MBCA sect 742 points out the MBCArsquos requirement of ldquouniversal demandrdquo gives the board ldquothe opportunity to reexamine the act complained of in the light of a potential lawsuit and take corrective actionrdquo even when the directors might be non-independent or have conflicts of interest

Nor is it relevant to the MBCA pre-suit demand requirement that the statutory 90-day waiting period may be onerous The first paragraph of MBCA sect 742 requires a pre-suit demand without exception the second paragraph of the section imposes a 90-day waiting period before a derivative suit may be brought which can be shortened if the board rejects the demand or ldquoirreparable injury to the corporation would result by waiting for the expiration of the 90-day periodrdquo The call as written asks only whether a pre-suit demand should be made and does not ask examinees to address whether the post-demand waiting period should be shortened under the ldquoirreparable injuryrdquo standard]

35

National Conference of Bar Examiners 302 South Bedford Street | Madison WI 53703-3622 Phone 608-280-8550 | Fax 608-280-8552 | TDD 608-661-1275

wwwncbexorg e-mail contactncbexorg

  • Preface
  • Description of the MEE
  • Instructions
  • July 2014 Questions
    • CRIMINAL LAW AND PROCEDURE QUESTION
    • CONTRACTS QUESTION
    • FAMILY LAW QUESTION
    • FEDERAL CIVIL PROCEDURE QUESTION
    • EVIDENCE QUESTION
    • CORPORATIONS QUESTION
      • July 2014 Analyses
        • CRIMINAL LAW AND PROCEDURE ANALYSIS
        • CONTRACTS ANALYSIS
        • FAMILY LAW ANALYSIS
        • FEDERAL CIVIL PROCEDURE ANALYSIS
        • EVIDENCE ANALYSIS
        • CORPORATIONS ANALYSIS
            • ltlt13 ASCII85EncodePages false13 AllowTransparency false13 AutoPositionEPSFiles true13 AutoRotatePages None13 Binding Left13 CalGrayProfile (Dot Gain 20)13 CalRGBProfile (sRGB IEC61966-21)13 CalCMYKProfile (US Web Coated 050SWOP051 v2)13 sRGBProfile (sRGB IEC61966-21)13 CannotEmbedFontPolicy Error13 CompatibilityLevel 1413 CompressObjects Tags13 CompressPages true13 ConvertImagesToIndexed true13 PassThroughJPEGImages true13 CreateJobTicket false13 DefaultRenderingIntent Default13 DetectBlends true13 DetectCurves 0000013 ColorConversionStrategy CMYK13 DoThumbnails false13 EmbedAllFonts true13 EmbedOpenType false13 ParseICCProfilesInComments true13 EmbedJobOptions true13 DSCReportingLevel 013 EmitDSCWarnings false13 EndPage -113 ImageMemory 104857613 LockDistillerParams false13 MaxSubsetPct 10013 Optimize true13 OPM 113 ParseDSCComments true13 ParseDSCCommentsForDocInfo true13 PreserveCopyPage true13 PreserveDICMYKValues true13 PreserveEPSInfo true13 PreserveFlatness true13 PreserveHalftoneInfo false13 PreserveOPIComments true13 PreserveOverprintSettings true13 StartPage 113 SubsetFonts true13 TransferFunctionInfo Apply13 UCRandBGInfo Preserve13 UsePrologue false13 ColorSettingsFile ()13 AlwaysEmbed [ true13 ]13 NeverEmbed [ true13 ]13 AntiAliasColorImages false13 CropColorImages true13 ColorImageMinResolution 30013 ColorImageMinResolutionPolicy OK13 DownsampleColorImages true13 ColorImageDownsampleType Bicubic13 ColorImageResolution 30013 ColorImageDepth -113 ColorImageMinDownsampleDepth 113 ColorImageDownsampleThreshold 15000013 EncodeColorImages true13 ColorImageFilter DCTEncode13 AutoFilterColorImages true13 ColorImageAutoFilterStrategy JPEG13 ColorACSImageDict ltlt13 QFactor 01513 HSamples [1 1 1 1] VSamples [1 1 1 1]13 gtgt13 ColorImageDict ltlt13 QFactor 01513 HSamples [1 1 1 1] VSamples [1 1 1 1]13 gtgt13 JPEG2000ColorACSImageDict ltlt13 TileWidth 25613 TileHeight 25613 Quality 3013 gtgt13 JPEG2000ColorImageDict ltlt13 TileWidth 25613 TileHeight 25613 Quality 3013 gtgt13 AntiAliasGrayImages false13 CropGrayImages true13 GrayImageMinResolution 30013 GrayImageMinResolutionPolicy OK13 DownsampleGrayImages true13 GrayImageDownsampleType Bicubic13 GrayImageResolution 30013 GrayImageDepth -113 GrayImageMinDownsampleDepth 213 GrayImageDownsampleThreshold 15000013 EncodeGrayImages true13 GrayImageFilter DCTEncode13 AutoFilterGrayImages true13 GrayImageAutoFilterStrategy JPEG13 GrayACSImageDict ltlt13 QFactor 01513 HSamples [1 1 1 1] VSamples [1 1 1 1]13 gtgt13 GrayImageDict ltlt13 QFactor 01513 HSamples [1 1 1 1] VSamples [1 1 1 1]13 gtgt13 JPEG2000GrayACSImageDict ltlt13 TileWidth 25613 TileHeight 25613 Quality 3013 gtgt13 JPEG2000GrayImageDict ltlt13 TileWidth 25613 TileHeight 25613 Quality 3013 gtgt13 AntiAliasMonoImages false13 CropMonoImages true13 MonoImageMinResolution 120013 MonoImageMinResolutionPolicy OK13 DownsampleMonoImages true13 MonoImageDownsampleType Bicubic13 MonoImageResolution 120013 MonoImageDepth -113 MonoImageDownsampleThreshold 15000013 EncodeMonoImages true13 MonoImageFilter CCITTFaxEncode13 MonoImageDict ltlt13 K -113 gtgt13 AllowPSXObjects false13 CheckCompliance [13 None13 ]13 PDFX1aCheck false13 PDFX3Check false13 PDFXCompliantPDFOnly false13 PDFXNoTrimBoxError true13 PDFXTrimBoxToMediaBoxOffset [13 00000013 00000013 00000013 00000013 ]13 PDFXSetBleedBoxToMediaBox true13 PDFXBleedBoxToTrimBoxOffset [13 00000013 00000013 00000013 00000013 ]13 PDFXOutputIntentProfile ()13 PDFXOutputConditionIdentifier ()13 PDFXOutputCondition ()13 PDFXRegistryName ()13 PDFXTrapped False1313 CreateJDFFile false13 Description ltlt13 ARA 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 BGR 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 CHS ltFEFF4f7f75288fd94e9b8bbe5b9a521b5efa7684002000410064006f006200650020005000440046002065876863900275284e8e9ad88d2891cf76845370524d53705237300260a853ef4ee54f7f75280020004100630072006f0062006100740020548c002000410064006f00620065002000520065006100640065007200200035002e003000204ee553ca66f49ad87248672c676562535f00521b5efa768400200050004400460020658768633002gt13 CHT ltFEFF4f7f752890194e9b8a2d7f6e5efa7acb7684002000410064006f006200650020005000440046002065874ef69069752865bc9ad854c18cea76845370524d5370523786557406300260a853ef4ee54f7f75280020004100630072006f0062006100740020548c002000410064006f00620065002000520065006100640065007200200035002e003000204ee553ca66f49ad87248672c4f86958b555f5df25efa7acb76840020005000440046002065874ef63002gt13 CZE 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 DAN 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 DEU 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 ESP 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 ETI 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 FRA 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 GRE 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 HEB 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 HRV (Za stvaranje Adobe PDF dokumenata najpogodnijih za visokokvalitetni ispis prije tiskanja koristite ove postavke Stvoreni PDF dokumenti mogu se otvoriti Acrobat i Adobe Reader 50 i kasnijim verzijama)13 HUN 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 ITA 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 JPN ltFEFF9ad854c18cea306a30d730ea30d730ec30b951fa529b7528002000410064006f0062006500200050004400460020658766f8306e4f5c6210306b4f7f75283057307e305930023053306e8a2d5b9a30674f5c62103055308c305f0020005000440046002030d530a130a430eb306f3001004100630072006f0062006100740020304a30883073002000410064006f00620065002000520065006100640065007200200035002e003000204ee5964d3067958b304f30533068304c3067304d307e305930023053306e8a2d5b9a306b306f30d530a930f330c8306e57cb30818fbc307f304c5fc59808306730593002gt13 KOR ltFEFFc7740020c124c815c7440020c0acc6a9d558c5ec0020ace0d488c9c80020c2dcd5d80020c778c1c4c5d00020ac00c7a50020c801d569d55c002000410064006f0062006500200050004400460020bb38c11cb97c0020c791c131d569b2c8b2e4002e0020c774b807ac8c0020c791c131b41c00200050004400460020bb38c11cb2940020004100630072006f0062006100740020bc0f002000410064006f00620065002000520065006100640065007200200035002e00300020c774c0c1c5d0c11c0020c5f40020c2180020c788c2b5b2c8b2e4002egt13 LTH 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 LVI 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 NLD (Gebruik deze instellingen om Adobe PDF-documenten te maken die zijn geoptimaliseerd voor prepress-afdrukken van hoge kwaliteit De gemaakte PDF-documenten kunnen worden geopend met Acrobat en Adobe Reader 50 en hoger)13 NOR 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 POL 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 PTB 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 RUM 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 RUS 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 SKY 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 SLV 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 SUO 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 SVE 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 TUR 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 UKR ltFEFF04120438043a043e0440043804410442043e043204430439044204350020044604560020043f043004400430043c043504420440043800200434043b044f0020044104420432043e04400435043d043d044f00200434043e043a0443043c0435043d044204560432002000410064006f006200650020005000440046002c0020044f043a04560020043d04300439043a04400430044904350020043f045604340445043e0434044f0442044c00200434043b044f0020043204380441043e043a043e044f043a04560441043d043e0433043e0020043f0435044004350434043404400443043a043e0432043e0433043e0020043404400443043a0443002e00200020042104420432043e04400435043d045600200434043e043a0443043c0435043d0442043800200050004400460020043c043e0436043d04300020043204560434043a0440043804420438002004430020004100630072006f006200610074002004420430002000410064006f00620065002000520065006100640065007200200035002e0030002004300431043e0020043f04560437043d04560448043e04570020043204350440044104560457002egt13 ENU (Use these settings to create Adobe PDF documents best suited for high-quality prepress printing Created PDF documents can be opened with Acrobat and Adobe Reader 50 and later)13 gtgt13 Namespace [13 (Adobe)13 (Common)13 (10)13 ]13 OtherNamespaces [13 ltlt13 AsReaderSpreads false13 CropImagesToFrames true13 ErrorControl WarnAndContinue13 FlattenerIgnoreSpreadOverrides false13 IncludeGuidesGrids false13 IncludeNonPrinting false13 IncludeSlug false13 Namespace [13 (Adobe)13 (InDesign)13 (40)13 ]13 OmitPlacedBitmaps false13 OmitPlacedEPS false13 OmitPlacedPDF false13 SimulateOverprint Legacy13 gtgt13 ltlt13 AddBleedMarks false13 AddColorBars false13 AddCropMarks false13 AddPageInfo false13 AddRegMarks false13 ConvertColors ConvertToCMYK13 DestinationProfileName ()13 DestinationProfileSelector DocumentCMYK13 Downsample16BitImages true13 FlattenerPreset ltlt13 PresetSelector MediumResolution13 gtgt13 FormElements false13 GenerateStructure false13 IncludeBookmarks false13 IncludeHyperlinks false13 IncludeInteractive false13 IncludeLayers false13 IncludeProfiles false13 MultimediaHandling UseObjectSettings13 Namespace [13 (Adobe)13 (CreativeSuite)13 (20)13 ]13 PDFXOutputIntentProfileSelector DocumentCMYK13 PreserveEditing true13 UntaggedCMYKHandling LeaveUntagged13 UntaggedRGBHandling UseDocumentProfile13 UseDocumentBleed false13 gtgt13 ]13gtgt setdistillerparams13ltlt13 HWResolution [2400 2400]13 PageSize [612000 792000]13gtgt setpagedevice13

Page 23: July 2014 MEE Questions and Analyses - NCBE...This publication includes the questions and analyses from the July 2014 MEE. (In the actual test, the questions are simply numbered rather

Family Law Analysis

STEEGH FAMILY LAW 213ndash15 (3d ed 2010) A significant decrease in income is typically viewed as a substantial change

However when a parent seeks to modify a child support obligation because he has voluntarily reduced his income a court will not modify the obligation based solely on the income loss Some courts refuse to modify whenever the income shift was voluntary See eg Aguiar v Aguiar 127 P3d 234 (Idaho Ct App 2005) Others look primarily to the petitionerrsquos intentions and permit downward modification if he has acted in good faith See eg In re Marriage of Horn 650 NE2d 1103 (Ill App Ct 1995) Many courts use a multifactor approach See OLIPHANT amp VER STEEGH supra 217ndash18

Here there is no question that the husbandrsquos loss of income was voluntary In a jurisdiction in which voluntary income reduction bars support modification the husbandrsquos petition would be denied

In a jurisdiction employing a good-faith or multifactor approach it is possible but not certain that the husband could obtain downward modification The evidence supports the husbandrsquos good faith his change in employment appears to be based on his new jobrsquos greater responsibilities and better promotion possibilities In a jurisdiction using a multifactor approach the court would likely also consider the impact of such a shift on the daughter the likely duration of the husbandrsquos income loss and the likelihood of a promotion that would ultimately inure to the daughterrsquos benefit Thus on these facts it is possible but by no means certain that the husband could prospectively obtain downward modification of his child support obligation to his daughter

Point Two(c)(15) A divorce property-division award is not subject to modification

A support order is aimed at meeting the post-divorce needs of the supported individual Because the future is unpredictable courts are empowered to modify a support award to take account of changed circumstances that may occur during the period in which support is paid

By contrast a property-distribution award divides assets of the marriage based on the equities at the time of divorce Because the past can be ascertained a property-division award is not subject to post-divorce modification See HARRY A KRAUSE ET AL FAMILY LAW CASES COMMENTS AND QUESTIONS 691 (6th ed 2007)

Here the husband is seeking modification of a property-division award with respect to an asset owned by the parties at the time of divorce Thus the husband may not obtain a modification of the marital-residence-sale-proceeds provision of the divorce decree based on his reduced income

21

FEDERAL CIVIL PROCEDURE ANALYSIS (Federal Civil Procedure III IVC)

ANALYSIS

Legal Problems

(1) Is the logging company entitled to join this action as a matter of right

(2)(a) May the nonprofit organization obtain a temporary restraining order to stop the USFS from issuing a logging permit

(2)(b) May the nonprofit organization obtain a preliminary injunction to stop the USFS from issuing a logging permit during the pendency of the action

DISCUSSION

Summary

The logging company is entitled to intervene in this action as a matter of right because it has an interest in the property or transaction that is the subject of the action and is so situated that its interest may be impaired or impeded as a practical matter if the action goes forward without it The logging companyrsquos interest is not adequately represented by the USFSrsquos presence in the lawsuit

The nonprofit organization may seek a temporary restraining order (TRO) followed by a preliminary injunction to prevent the USFS from issuing a logging permit pending the outcome of the action The nonprofit is likely to obtain a TRO if it can demonstrate a risk of immediate and irreparable injury The nonprofit is also likely to obtain a preliminary injunction if it can demonstrate a significant threat of irreparable harm and a likelihood of success on the merits of its National Environmental Policy Act (NEPA) claim

Point One (50) Rule 24(a) of the Federal Rules of Civil Procedure requires federal courts to allow a person to intervene in an action as a matter of right if the person a) is interested in the property or transaction that is the subject of the action b) is so situated that its interest may be impaired or impeded if the litigation goes forward without it and c) is not adequately represented by existing parties Here the logging company likely meets all three requirements and should be allowed to intervene as a matter of right

Rule 24 of the Federal Rules of Civil Procedure governs intervention the process by which a non-party to an action may join the litigation Under Rule 24(a) (intervention of right) a person must be permitted to intervene if three conditions are met (1) the movant ldquoclaims an interest relating to the property or transaction that is the subject of the actionrdquo (2) the movant ldquois so situated that disposition of the action may as a practical matter impair or impede the movantrsquos ability to protect its interestrdquo and (3) ldquoexisting partiesrdquo do not ldquoadequately represent [the movantrsquos] interestrdquo FED R CIV P 24(a) The three requirements for intervention of right are often ldquovery interrelatedrdquo 7C CHARLES ALAN WRIGHT ET AL FEDERAL PRACTICE AND PROCEDURE sect 1908 at 297 (2007 amp 2011 Supp)

22

Federal Civil Procedure Analysis

Here the court should find that the logging company meets this test First the logging company has a strong interest in the property or transaction that is the subject of this action The USFS has accepted the logging companyrsquos bid and the logging company is merely awaiting issuance of a logging permit to begin logging The nonprofit organization is seeking to prevent this logging The logging company therefore has a strong direct and substantial interest in the subject matter of the lawsuit and in having its winning bid honored and a logging permit issued See eg Kleissler v US Forest Serv 157 F3d 964 972 (3d Cir 1998) (stating that ldquo[t]imber companies have direct and substantial interests in a lawsuit aimed at halting loggingrdquo) see also Natural Resources Defense Council v US Nuclear Regulatory Commrsquon 578 F2d 1341 1343ndash 44 (10th Cir 1978) (holding that applicants whose license renewals were pending had Rule 24(a)(2) interests where the lawsuit sought to halt the license-issuing process pending preparation of environmental impact statements) See generally 7C WRIGHT ET AL supra sect 19081 at 309 (ldquoIf there is a direct substantial legally protectable interest in the proceedings it is clear that this requirement of the rule is satisfiedrdquo) Second the logging companyrsquos interest in receiving a logging permit may well be impaired as a practical matter by the outcome of the lawsuit If the USFS loses the lawsuit it will have to prepare an environmental impact statement before issuing the logging companyrsquos permit This will at a minimum delay the logging companyrsquos ability to exercise its rights and may in the long r un mean that no logging permit is ever issued Intervention of right is not limited to those that would be legally bound as a matter of preclusion doctrine Id sect 19082 at 368 Rather ldquo[t]he rule is satisfied whenever disposition of the present action would put the movant at a practical disadvantage in protecting its interestrdquo Id sect 19082 at 369 Here that condition is easily satisfied See Kleissler 157 F3d at 972 (ldquoTimber companies have direct and substantial interests in a lawsuit aimed at halting logging rdquo)

Given that the logging company has an interest that may be impaired by disposition of the action it should be allowed to intervene unless the court is persuaded that the USFS adequately represents the logging companyrsquos interest See Rule 24(a)(2) 7C WRIGHT ET AL supra sect 1909 Here it could be argued that the USFS adequately represents the logging companyrsquos interest because the USFS presumably wants the court to uphold its development plan and allow it to proceed with issuance of the logging permit which is the same relief that the logging company would seek However whether representation is truly adequate depends upon ldquo[a] discriminating appraisal of the circumstancesrdquo 7C WRIGHT ET AL supra sect 1909 at 440 Although both the government and the logging company wish to avoid the preparation of an environmental impact statement their interests are distinct The USFSrsquos interest is proper management of the national forest system while the logging companyrsquos interest is making a profit from logging the 5000-acre tract The USFSrsquos handling of the litigation is likely to be affected by a variety of policy concerns and political considerations that have nothing to do with the logging companyrsquos purely economic interest in securing the right to cut trees in the Scenic National Forest See eg Kleissler 157 F3d at 973ndash74 (ldquo[T]he government represents numerous complex and conflicting interests in matters of this nature The straightforward business interests asserted by intervenors here may become lost in the thicket of sometimes inconsistent governmental policiesrdquo)

[NOTES (1) Examinees who mistakenly analyze the logging companyrsquos case for joinder under the related but incorrect Rule 19 ldquoRequired Joinder of Partiesrdquo may receive credit Rule 19 allows existing parties to demand joinder of non-parties (or seek dismissal of the case if they canrsquot get it) There is a close relationship between Rule 24 and Rule 19 and both contain a similar standard for determining when ldquointerestedrdquo third parties are ldquoentitledrdquo or ldquorequiredrdquo to be in the lawsuit Indeed the two prongs of the Rule 24 intervention test that are discussed above

23

Federal Civil Procedure Analysis

are nearly identical to the two prongs of the Rule 19(a) required joinder test Examinees who discuss and apply the test should receive credit even if they cite Rule 19 rather than Rule 24

(2) Examinees may discuss permissive joinder Although permissive joinder is a possibility here the question asks only whether the logging company can join the action as a matter of right and a permissive joinder analysis is not responsive to the question To the extent an examinee discusses permissive joinder the analysis will focus on whether the logging company ldquohas a claim or defense that shares with the main action a common question of law or factrdquo FED R CIV P 24(b)(1)(B) The district court also ldquomust consider whether the intervention will unduly delay or prejudice the adjudication of the original partiesrsquo rightsrdquo FED R CIV P 24(b)(3) On our facts the logging companyrsquos claim for the issuance of a logging permit would certainly share common questions of law and fact with the USFSrsquos defense against the nonprofitrsquos claim There are no facts suggesting that the logging companyrsquos presence would unduly delay or otherwise prejudice adjudication of the original action Thus the district court would have discretion to permit the logging company to intervene even if it denied intervention of right]

Point Two(a) (25) The nonprofit organization could seek and would likely obtain a temporary restraining order to stop the USFS from issuing a logging permit pending a hearing on an application for a preliminary injunction

The first type of interim relief the nonprofit could seek to stop the USFS from issuing a logging permit to the logging company is a temporary restraining order (TRO) prohibiting the USFS from issuing the logging permit A TRO can be issued without notice to the adverse party but only in limited circumstances and only for a limited time FED R CIV P 65(b) To secure a TRO without notice the nonprofit would need to submit an affidavit containing specific facts that demonstrate a risk of ldquoimmediate and irreparable injuryrdquo if a permit is issued FED R CIV P 65(b)(1) In deciding whether to grant a TRO courts will also consider the same factors that are relevant in deciding whether to grant a preliminary injunction (eg the moving partyrsquos likelihood of success on the merits the balance of hardships and the public interest) See Point Two(b) infra The TRO would last only long enough for the court to consider and resolve a request by the nonprofit for a preliminary injunction but no longer than 14 days (unless the court extends it for good cause or the adverse party consents to an extension) In addition bond is required

Here the court is likely to grant the nonprofitrsquos request The nonprofit could plausibly claim that cutting down 5000 acres of old-growth forest in an area that is home to the highest concentration of wildlife in the western United States would have ldquoan immediate and irreparablerdquo adverse impact on the environment and cause irreparable harm to the nonprofitrsquos interest in preserving and protecting natural resources including wildlife habitat

Point Two(b) (25) The nonprofit could also seek and would likely obtain a preliminary injunction to stop the USFS which is likely to be granted if the nonprofitrsquos claim that the USFS violated NEPA has a strong basis in fact and law

Because the TRO would be temporary the nonprofit would need to move for a preliminary injunction to prevent the USFS from issuing a logging permit throughout the pendency of the litigation Preliminary injunctions are injunctions that seek to ldquoprotect [the] plaintiff from

24

Federal Civil Procedure Analysis

irreparable injury and to preserve the courtrsquos power to render a meaningful decision after a trial on the meritsrdquo 11A CHARLES ALAN WRIGHT ET AL FEDERAL PRACTICE AND PROCEDURE sect 2947 at 112 (2013) Rule 65 of the Federal Rules of Civil Procedure sets out the procedural requirements for preliminary injunctions Preliminary injunctions may be granted only upon notice to the adverse party FED R CIV P 65(a)(1) and only if the movant ldquogives security in an amount that the court considers proper to pay the costs and damages sustained by any party found to have been wrongfully enjoined or restrainedrdquo FED R CIV P 65(c)

While Rule 65 sets out the procedural requirements for preliminary injunctive relief it does not specify the substantive grounds upon which it may be granted The courtrsquos discretion in ruling upon a motion for a preliminary injunction ldquois exercised in conformity with historic federal equity practicerdquo 11A WRIGHT ET AL supra sect 2947 at 114 The court typically considers four factors

(1) the significance of the threat of irreparable harm to the plaintiff if the injunction is not granted (2) the balance between this harm and the injury that granting the injunction would inflict on the defendant (3) the probability that the plaintiff will succeed on the merits and (4) the public interest

Id sect 2948 at 122ndash24 accord Habitat Educ Center v Bosworth 363 F Supp 2d 1070 1088 (ED Wis 2005) The most important of these factors is the risk of irreparable harm to the plaintiff 11A WRIGHT ET AL supra sect 29481 at 129 If the plaintiff has an adequate remedy at law (eg if money damages can compensate the plaintiff for its loss) then a preliminary injunction will be denied Id sect 29481

Here a court would likely conclude that the potential for environmental damage to the forest creates a significant threat of irreparable harm ldquo[E]nvironmental injury is often irreparable Courts have recognized that logging such as would occur [here] can have longshyterm environmental consequences and thus satisfy the irreparable injury criterionrdquo Habitat Educ Center 363 F Supp 2d at 1089 (citing Idaho Sporting Congress Inc v Alexander 222 F3d 562 569 (9th Cir 2000) (noting that the imminent and continuing logging activities presented ldquoevidence of environmental harm sufficient to tip the balance in favor of injunctive reliefrdquo)) Neighbors of Cuddy Mountain v US Forest Service 137 F3d 1372 1382 (9th Cir 1998) (stating that ldquo[t]he old growth forests plaintiffs seek to protect would if cut take hundreds of years to reproducerdquo) (internal citation omitted)) see also 11C WRIGHT ET AL supra sect 29481 at 151 (noting that ldquoa preliminary injunction has been issued to prevent harm to the environmentrdquo)

The second factor the balance between the harm to the plaintiff and the harm the defendant will suffer if the injunction is issued also appears to support issuance of a preliminary injunction here The USFS will have to wait before it can develop the Scenic National Forest and the logging company may lose money if the delay is prolonged These economic harms could be compensated monetarily if an injunction is issued inappropriately Where ldquoan injunction bond can compensate [the] defendant for any harm the injunction is likely to inflict the balance should be struck in favor of [the] plaintiffrdquo Id sect 29482 at 192 See also Habitat Educ Center 363 F Supp 2d at 1089 (stating that ldquothe relative absence of harmful effects on the Forest Service weighs in favor of granting the injunctionrdquo)

The third factor is the likelihood that the plaintiff will prevail on the merits Although there is limited information concerning the merits of the action the nonprofit alleges that the federal statute (NEPA) requires an environmental impact statement and further states that the USFS created no environmental impact analysis or statement at all Assuming that those

25

Federal Civil Procedure Analysis

allegations are correct it seems plausible to conclude that the nonprofit will be able to show a likelihood of success on the merits

Finally courts deciding whether or not to issue preliminary injunctive relief are to consider the public interest ldquoFocusing on this factor is another way of inquiring whether there are policy considerations that bear on whether the order should issuerdquo 11C WRIGHT ET AL supra sect 29484 at 214 If the court concludes that the nonprofit is likely to succeed on its NEPA claim because the USFS wrongfully failed to conduct an environmental impact assessment it is likely to find that the public interest would be served by restraining the USFS from proceeding with logging in a national forest See Heartwood Inc v US Forest Service 73 F Supp 2d 962 979 (SD Ill 1999) affrsquod on other grounds 230 F3d 947 (7th Cir 2000) (ldquoviolations by federal agencies of NEPArsquos provisions as established by Congress harm the public as well as the environmentrdquo)

Thus a court is very likely to grant a preliminary injunction if it concludes that the nonprofit has a significant likelihood of success on the merits

26

EVIDENCE ANALYSIS (Evidence ID IIA amp C)

ANALYSIS

Legal Problems

(1) Under what circumstances can evidence of prior convictions be used to impeach a witnessrsquos credibility in a civil case

(1)(a) May the inmatersquos credibility be impeached by evidence of a 12-year-old felony drug conviction if he was released from prison 9 years ago

(1)(b) May the inmatersquos credibility be impeached by evidence of an 8-year-old misdemeanor perjury conviction that was punishable by 1 year in jail if he pleaded guilty and was sentenced only to pay a $5000 fine

(1)(c) May the inmatersquos credibility be impeached by evidence of a 7-year-old sexual assault conviction if the inmate is still serving a 10-year prison sentence and the victim was his 13-year-old daughter

(2)(a) May the guardrsquos credibility be impeached by cross-examination regarding specific instances of misconduct (ie lying on his reacutesumeacute) relevant to credibility

(2)(b) May the guardrsquos credibility be impeached by admission of extrinsic evidence (his reacutesumeacute and academic transcript) offered to prove specific instances of misconduct relevant to credibility

DISCUSSION

Summary

Under the Federal Rules of Evidence witnesses can be impeached with evidence of prior convictions andor specific instances of misconduct Whether evidence of prior convictions should be admitted to impeach generally depends on the nature of the crime the amount of time that has passed and (only in criminal cases) whether the ldquowitnessrdquo is the defendant FED R EVID 609(a)

In this civil case evidence of the inmatersquos conviction for distribution of marijuana should be admitted to impeach the inmate because he was convicted of a felony and was released from prison fewer than 10 years ago FED R EVID 609(a)(1) Credibility is critically important in this case because the jury will hear conflicting testimony from the two disputing parties and there were no other eyewitnesses to the altercation Under Rule 609(a)(1) the inmatersquos conviction should be admitted because it has some bearing on his credibility and its probative value is not substantially outweighed by concerns of unfair prejudice confusion or delay Id

Evidence of the inmatersquos misdemeanor conviction for perjury must be admitted because the crime ldquorequired provingmdashor the witnessrsquos admittingmdasha dishonest act or false statementrdquo by the inmate FED R EVID 609(a)(2)

27

Evidence Analysis

Evidence of the inmatersquos felony conviction for sexual assault should be excluded because its probative value is substantially outweighed by the danger of unfair prejudice to the inmate based on the heinous nature of the crime FED R EVID 609(a)(1) In the alternative the judge could limit the evidence relating to this conviction by excluding details of the inmatersquos crime

In all civil (and criminal) cases witnesses can also be impeached with evidence of specific instances of prior misconduct that did not result in a conviction FED R EVID 608(b) Pursuant to Rule 608(b) misconduct probative of untruthfulness can be inquired into on cross-examination but cannot be proved through extrinsic evidence Id Thus the inmatersquos counsel should be permitted to cross-examine the guard regarding the false statement in the guardrsquos reacutesumeacute However extrinsic evidence of the guardrsquos misconduct (ie the guardrsquos authenticated reacutesumeacute and transcript from the local college) should not be admitted even if the guard denies wrongdoing or refuses to answer cross-examination questions about these matters Id

Point One (10) The Federal Rules of Evidence permit impeachment of witnesses with evidence of prior convictions

Whether convictions should be admitted to impeach generally depends on the nature of the crime the amount of time that has passed and (only in criminal cases) whether the ldquowitnessrdquo is the defendant FED R EVID 609(a) Under Rule 609(a) evidence of prior convictions may be admitted for the purpose of ldquoattacking a witnessrsquos character for truthfulnessrdquo Id

There are two basic types of convictions that can be admitted for the purpose of impeachment

(1) convictions for crimes ldquopunishable by death or by imprisonment for more than one yearrdquo (which generally correlates to ldquofeloniesrdquo) FED R EVID 609(a)(1) and (2) convictions ldquofor any crimes regardless of the punishment if the court can readily determine that establishing the elements of the crime required provingmdashor the witnessrsquos admittingmdasha dishonest act or false statementrdquo FED R EVID 609(a)(2)

Pursuant to Rule 609(a)(1) in civil cases the admission of evidence of a felony conviction is ldquosubject to Rule 403 [which says that a court may exclude relevant evidence if its probative value is substantially outweighed by other factors]rdquo FED R EVID 609(a)(1) However Rule 403 does not protect the witness against admission of prior convictions involving dishonestymdashwhich must be admitted by the court FED R EVID 609(a)(2)

Finally Federal Rule of Evidence 609(b) contains the presumption that a conviction that is more than 10 years old or where more than 10 years has passed since the witnessrsquos release from confinement (whichever is later) should not be admitted unless ldquoits probative value supported by specific facts and circumstances substantially outweighs its prejudicial effectrdquo and the proponent has provided the adverse party with reasonable written notice FED R EVID 609(b)

Point One(a) (25) The court should admit evidence of the inmatersquos 12-year-old felony marijuana distribution conviction

The inmatersquos conviction for marijuana distribution was for a felony punishable by imprisonment for more than one year See FED R EVID 609(a)(1) Moreover although the conviction was 12 years ago the 10-year time limit of Rule 609(b) is not exceeded because that time limit runs

28

Evidence Analysis

from the date of either ldquothe witnessrsquos conviction or release from confinement for it whichever is laterrdquo FED R EVID 609(b) Because the inmate served three years in prison he was released from confinement nine years ago

However pursuant to Rule 609(a)(1) the admission of felony convictions to impeach a witness in a civil case is ldquosubject to Rule 403rdquo FED R EVID 609(a)(1) Neither Rule 609(a) nor the advisory committee notes specify which factors courts should consider when balancing the probative value of a conviction against the dangers identified in Rule 403 (which include (1) unfair prejudice (2) confusion of the issues (3) misleading the jury (4) waste of time or undue delay and (5) needless presentation of cumulative evidence) FED R EVID 403

In this case credibility is very important because the evidence consists primarily of the testimony of the disputing parties and there were no other eyewitnesses to the altercation This enhances the probative value of any evidence bearing on the inmatersquos credibility A court is likely to conclude that the inmatersquos prior felony drug conviction is relevant to his credibility See eg United States v Brito 427 F3d 53 64 (1st Cir 2005) (ldquoPrior drug-trafficking crimes are generally viewed as having some bearing on veracityrdquo) Although the probative value of any conviction diminishes with age see eg United States v Brewer 451 F Supp 50 53 (ED Tenn 1978) the inmatersquos ongoing problems with the law suggest that he has continued (and even escalated) his criminal behavior over the past nine years The court should admit this evidence because its probative value is not substantially outweighed by any Rule 403 concerns Specifically any prejudice to the inmate would be slight because the conviction is unrelated to the altercation at issue and the conviction was not for a heinous crime that might inflame the jury

[NOTE Whether an examinee identifies the jury instruction as containing a ldquoconclusiverdquo or ldquomandatoryrdquo presumption is less important than the examineersquos analysis of the constitutional infirmities]

Point One(b) (15) The court must admit evidence of the inmatersquos eight-year-old misdemeanor conviction because perjury is a crime of dishonesty

Rule 609(a)(2) provides that evidence of a criminal conviction ldquomust be admitted if the court can readily determine that establishing the elements of the crime required provingmdashor the witnessrsquos admittingmdasha dishonest act or false statementrdquo FED R EVID 609(a)(2) The inmatersquos conviction for perjury would have necessarily required proving that the inmate engaged in an act of dishonesty This conviction occurred within the past 10 years so it ldquomust be admittedrdquo because in contrast to Rule 609(a)(1) (discussed in Point One(a)) admission under Rule 609(a)(2) is mandatory and not subject to Rule 403

Point One(c) (20) The court should exclude evidence of the inmatersquos seven-year-old felony sexual assault conviction because the probative value of this evidence is substantially outweighed by the danger of unfair prejudice In the alternative the details of the prior conviction could be excluded

The inmatersquos conviction for felony sexual assault was seven years ago and he has not yet been released from incarceration so Rule 609(a) but not 609(b) is applicable here FED R EVID 609(a) This conviction is therefore admissible to impeach the inmate unless its probative value is substantially outweighed by the danger of unfair prejudice or any other Rule 403 concern Id

29

Evidence Analysis

Sex crimes are generally not considered relevant to credibility see Hopkins v State 639 So 2d 1247 1254 (Miss 1993) so the probative value of this conviction is relatively low Moreover the heinous nature of the inmatersquos crime (sexual assault on his daughter) makes the danger of unfair prejudice to the inmate very high Thus the court should exclude evidence of the conviction because it was for a heinous offense that is likely to inflame the jury and it has little bearing on credibility See eg United States v Beahm 664 F2d 414 419 (4th Cir 1981)

As an alternative to excluding this evidence the judge could minimize the unfair prejudice to the inmate by permitting limited cross-examination but refusing to allow specific questions about the nature of the inmatersquos conviction For example a court could limit cross-examination to the fact that the inmate was convicted of a ldquofelonyrdquo or perhaps that he was convicted of a ldquosexual assaultrdquo without identifying the victim However because evidence of the inmatersquos prior convictions can be admitted solely for the purpose of enabling the jury to assess his credibility and because his two earlier convictions should have already been admitted the court should exclude all evidence of the felony sexual assault conviction

Point Two(a) (15) The court should permit the inmatersquos counsel to cross-examine the guard regarding the false statement in his reacutesumeacute because the guardrsquos misconduct bears on his truthfulness

The inmate wishes to cross-examine the guard about his prior dishonest behaviormdashlying on his reacutesumeacutemdashthat did not involve a criminal conviction Rule 608(b) allows witnesses to be cross-examined about specific instances of prior non-conviction misconduct probative of untruthfulness ldquoin order to attack the witnessrsquos character for truthfulnessrdquo FED R EVID 608(b)

The courtrsquos decision to allow cross-examination about the guardrsquos prior dishonest behavior depends on the probative value of such evidence balanced against the danger of unfair prejudice to the guard or any other Rule 403 concern FED R EVID 403 Here the guardrsquos false statement on his reacutesumeacute that he obtained a degree in Criminal Justice is highly probative of his untruthfulness because it grossly misrepresents his actual academic record was made recently and was made with the intent to deceive Because the probative value of this evidence is very strong and is not substantially outweighed by any Rule 403 concerns cross-examination of the guard on this topic should be permitted The court may also consider it fair to permit this cross-examination of the guard on these matters assuming that one or more of the inmatersquos prior convictions have been admitted to impeach his credibility

Point Two(b) (15) The court should exclude extrinsic evidence of the guardrsquos non-conviction misconduct even if the guard denies wrongdoing or refuses to answer questions about the matter

Although Rule 608(b) allows cross-examination about specific instances of prior misconduct probative of untruthfulness ldquoextrinsic evidencerdquo offered to prove such misconduct is not admissible FED R EVID 608(b) The rationale for this rule is that allowing the introduction of extrinsic evidence of prior misconduct by witnesses when these acts are relevant only to the witnessesrsquo truthfulness and not to the main issues in the case would create too great a risk of confusing the jury and unduly delaying the trial The court does not have discretion to admit this extrinsic evidence See eg United States v Elliot 89 F3d 1360 1368 (8th Cir 1996)

30

Evidence Analysis

Here the inmatersquos counsel may cross-examine the guard about the false statement on his reacutesumeacute However the inmatersquos counsel must accept the guardrsquos response Even if the guard denies wrongdoing or refuses to answer questions about the matter the inmatersquos counsel cannot introduce the guardrsquos reacutesumeacute or the transcript from the local college to prove the guardrsquos misconduct

31

CORPORATIONS ANALYSIS (Corporations VA2 IX)

ANALYSIS

Legal Problems

(1) Do shareholders have the authority to amend a corporationrsquos bylaws with respect to director nominations

(2) Do board-approved bylaws on a particular subject here nomination of directors preempt subsequent conflicting bylaw amendments by shareholders

(3) Is a suit challenging both managementrsquos refusal to include the proposed bylaw amendment in Megarsquos proxy statement and the boardrsquos amendment of the bylaws dealing with nomination of directors a direct or derivative suit

DISCUSSION

Summary

The voting and litigation rights of the shareholders of Mega are subject to the provisions of the Model Business Corporations Act (MBCA)

The investorrsquos proposed bylaw provision is not inconsistent with state law Under the MBCA shareholders may amend the bylaws when the amendment deals with a proper matter for the corporationrsquos bylaws such as procedures for nominating directors

The Mega boardrsquos bylaw amendment does not preempt the investorrsquos proposed bylaw provision or the Mega shareholdersrsquo power to approve it While shareholders can limit the boardrsquos power to amend or repeal the bylaws the board cannot limit the shareholdersrsquo power

Whether the investor must make a demand on Megarsquos board depends on how the investor frames its claim If the investor claims a violation of shareholder voting rights the claim is direct and pre-suit demand on the board is not required If on the other hand the investor claims that the directors violated their fiduciary duties by amending the bylaws to entrench themselves the claim is derivative and a pre-suit demand is required

Point One (30) Shareholders may amend the corporationrsquos bylaws where the proposed bylaw provision relates to procedural matters typically included in the bylaws such as the nomination of directors

Internal affairs of the corporation such as the conduct of shareholder meetings and election of directors are subject to the corporate law of the state of incorporation See McDermott Inc v Lewis 531 A2d 206 (Del 1987) (applying law of jurisdiction where corporation was incorporated in case involving voting rights) This statersquos corporate statute is modeled on the MBCA

Under the MBCA ldquoshareholders may amend the corporationrsquos bylawsrdquo MBCA sect 1020(a) Thus the only question is whether the bylaws can specify the procedures for shareholder nomination of directors

32

Corporations Analysis

The MBCA states that the bylaws ldquomay contain any provision that is not inconsistent with law or the articles of incorporationrdquo MBCA sect 206(b) In addition the MBCA was revised in 2009 to address shareholder nomination of directors in public corporations (known as ldquoproxy accessrdquo) and specifies that the bylaws ldquomay contain a requirement that the corporation include in its [proxy materials] one or more individuals nominated by a shareholderrdquo MBCA sect 206(c)(1) see Committee on Corporate Laws ABA Section of Business Law Report on the Roles of Boards of Directors and Shareholders of Publicly Owned Corporations and Changes to the Model Business Corporations ActmdashAdoption of Shareholder Proxy Access Amendments to Chapters 2 and 10 65 BUS LAWYER 1105 (2010)

The inclusion of director-nomination procedures in the bylaws is consistent with practice and is recognized by the Delaware courts whose views on corporate law carry significant weight Typically the procedures for nomination of directors are found in the bylaws See 1 COX amp HAZEN TREATISE ON THE LAW OF CORPORATIONS sect 312 (3d ed 2011) see also 4 FLETCHER CORP FORMS ANN PART III ch 21 (2013) (including sample bylaws that permit nomination of directors by shareholders) The Delaware Supreme Court has confirmed that the bylaws may ldquodefine the process and proceduresrdquo for director elections See CA Inc v AFSCME Employees Pension Plan 953 A2d 227 (Del 2008) (concluding that bylaw amendment requiring reimbursement of election expenses to certain successful shareholder nominators is ldquoproper subjectrdquo under Delaware law)

[NOTE The question of the proper scope of the bylaws can be answered using the more general MBCA sect 206(b) or the 2009 MBCA revision adding sect 206(c)(1) (adopted in CT ME VA) In addition some examinees might raise the point that shareholder proposals may not compel the board to take action such as by including shareholder nominations in the companyrsquos proxy materials on the theory that the ldquobusiness and affairsrdquo of the corporation are to be managed by the board See MBCA sect 801(b) Although shareholders are generally limited to adopting precatory resolutions that recommend or encourage board action this limitation does not apply when shareholders have specific authority to take binding action on their ownmdashsuch as to amend the bylaws]

Point Two (30) Shareholders can amend (or repeal) board-approved bylaws Further shareholders can limit the boardrsquos power to later amend and repeal a shareholder-approved bylaw

Under the MBCA shareholders have the power to amend the bylaws See Point One The board shares this power with the shareholders unless (1) the corporationrsquos articles ldquoreserve that power exclusively to the shareholdersrdquo or (2) ldquothe shareholders in amending repealing or adopting a bylaw expressly provide that the board of directors may not amend repeal or reinstate that bylawrdquo See MBCA sect 1020(b)

Shareholder-approved bylaw provisions can amend or repeal existing bylaw provisions whether originally approved by the board or by shareholders See ALAN R PALMITER CORPORATIONS EXAMPLES AND EXPLANATIONS sect 713 (7th ed 2012) Thus the Mega boardrsquos bylaw amendmentmdashwhich set more demanding thresholds for shareholder nomination of directors than the investorrsquos proposed bylaw provisionmdashwould be superseded (repealed) if Megarsquos shareholders were to approve the investorrsquos proposal

Further a shareholder-approved bylaw generally can limit the power of the board to later amend or repeal it See MBCA sect 1020(b)(2) Thus if Megarsquos shareholders approved the bylaw

33

Corporations Analysis

provision proposed by the investor Megarsquos board could not repeal the provision because it includes a ldquono board repealrdquo clause

The revision to the MBCA in 2009 dealing with shareholder proxy access does not change this conclusion That revision specifies that a shareholder-approved bylaw dealing with director nominations may not limit the boardrsquos power to amend add or repeal ldquoany procedure or condition to such a bylaw in order to provide for a reasonable practicable and orderly processrdquo MBCA sect 206(d) Thus according to the revision if shareholders approve a bylaw amendment that limits further board changes the board would nonetheless retain the power to ldquotinkerrdquo with the bylaw to safeguard the voting process but could not repeal the shareholder-approved bylaw The Official Comment to MBCA sect 206(d) makes clear that the revision is ldquonot intended to allow the board of directors to frustrate the purpose of the shareholder-adopted proxy access provisionrdquo Thus if Megarsquos shareholders were to approve the bylaw provision proposed by the investor Megarsquos board could only amend the provision regarding its procedures or conditions in a manner consistent with its purpose of permitting proxy access for Megarsquos shareholders

[NOTE The boardrsquos attempted interference with a shareholder voting initiative may also have been a violation of the directorsrsquo fiduciary duties See Blasius Indus Inc v Atlas Corp 564 A2d 651 (Del Ch 1988) (finding that directors breached their fiduciary duties by amending bylaws and expanding size of board to thwart insurgentrsquos plan to amend bylaws and seat a majority of new directors) The call however asks examinees to consider whether shareholders or the board have ldquoprecedencerdquo over amending the corporate bylaws Thus an examineersquos answer should be framed in terms of ldquopowerrdquo and not ldquodutyrdquo]

Point Three (40) The investor need not make a demand on the board if the investor states a direct claim such as an allegation that the board interfered with the investorrsquos right to amend the bylaws But the investor must make a demand on the board if the investor states a derivative claim (on behalf of the corporation) such as an allegation that the directors sought to entrench themselves by interfering with the proposed proxy access

The MBCA generally requires that shareholders make a demand on the board of directors before initiation of a derivative suit MBCA sect 742 (shareholder may not bring derivative proceeding until written demand has been made on corporation and 90 days have expired) A derivative suit is essentially two suits in one where the plaintiff-shareholder seeks to bring on behalf of the corporation a claim that vindicates corporate rights usually based on violation of fiduciary duties PALMITER supra sect 1811 (6th ed 2009) The demand permits the board to investigate the situation identified by the shareholder and take suitable action No demand on the board is required however if the shareholder brings a direct suit to vindicate the shareholderrsquos own rights not those of the corporation

Is the suit brought by the investor derivative or direct The MBCA defines a ldquoderivative proceedingrdquo as one brought ldquoin the right of a domestic corporationrdquo MBCA sect 740(1) Thus the answer to how the investorrsquos suit should be characterized turns on what rights the investor seeks to vindicate If the investor frames its claim as one of fiduciary breach by directorsmdashfor example for failing to become adequately informed about voting procedures or for seeking to entrench themselves in office by manipulating the voting structure to avoid a shareholder insurgencymdashthen the suit is ldquoderivativerdquo and the investor must make a demand on the board See MBCA Ch 7 Subch D Introductory Comment (ldquothe derivative suit has historically been the principal method of challenging allegedly illegal action by managementrdquo)

34

Corporations Analysis

If however the investor frames its claim as one to vindicate shareholder rights the suit is direct and no demand is required For many courts the direct-derivative question turns on who is injured and who is to receive the relief sought by the plaintiff-shareholders See Tooley v Donaldson Lufkin amp Jenrette Inc 845 A2d 1031 (Del 2004) (characterizing a merger-delay claim as direct because delay of merger only harmed shareholders not corporation) Thus if the investor claims that managementrsquos refusal to include its proposed bylaw amendment in the corporationrsquos proxy materials violates its shareholder rights to initiate corporate governance reforms the suit will be direct Courts have not questioned the ability of shareholders to bring direct suits challenging board action to exclude their proposed bylaw amendments from the corporationrsquos proxy materials See JANA Master Fund Ltd v CNET Networks Inc 954 A2d 335 (Del Ch 2008) (upholding shareholderrsquos direct challenge to boardrsquos interpretation of advance-notice bylaw) Chesapeake Corp v Shore 771 A2d 293 (Del Ch 2000) (upholding shareholderrsquos direct challenge to actions by board that effectively prevented it from proposing bylaw amendments in contest for control)

Is the way that the investor frames its claim conclusive Courts have permitted shareholder-plaintiffs to challenge a transaction in a direct suit even though the same transaction could also be challenged as a fiduciary breach See Eisenberg v Flying Tiger Line Inc 451 F2d 267 (2d Cir 1971) (permitting direct suit challenging a corporate reorganization as a dilution of shareholder voting power even though reorganization may have involved conflicts of interest and thus constituted a fiduciary breach) Thus the investorrsquos choice to pursue a claim challenging the legality of managementrsquos decision to exclude the investorrsquos proposal from the corporationrsquos proxy materialsmdashrather than a possible breach of fiduciary dutymdashis likely to be respected See 3 COX amp HAZEN supra sect 153 (describing situations in which a claim can be framed as derivative or direct)

[NOTE Some issues under Delaware corporate law regarding pre-suit demand are not relevant here For example whether the Mega directors are independent and disinterested is not relevant to the MBCA requirement of a pre-suit demand As the Official Comment to MBCA sect 742 points out the MBCArsquos requirement of ldquouniversal demandrdquo gives the board ldquothe opportunity to reexamine the act complained of in the light of a potential lawsuit and take corrective actionrdquo even when the directors might be non-independent or have conflicts of interest

Nor is it relevant to the MBCA pre-suit demand requirement that the statutory 90-day waiting period may be onerous The first paragraph of MBCA sect 742 requires a pre-suit demand without exception the second paragraph of the section imposes a 90-day waiting period before a derivative suit may be brought which can be shortened if the board rejects the demand or ldquoirreparable injury to the corporation would result by waiting for the expiration of the 90-day periodrdquo The call as written asks only whether a pre-suit demand should be made and does not ask examinees to address whether the post-demand waiting period should be shortened under the ldquoirreparable injuryrdquo standard]

35

National Conference of Bar Examiners 302 South Bedford Street | Madison WI 53703-3622 Phone 608-280-8550 | Fax 608-280-8552 | TDD 608-661-1275

wwwncbexorg e-mail contactncbexorg

  • Preface
  • Description of the MEE
  • Instructions
  • July 2014 Questions
    • CRIMINAL LAW AND PROCEDURE QUESTION
    • CONTRACTS QUESTION
    • FAMILY LAW QUESTION
    • FEDERAL CIVIL PROCEDURE QUESTION
    • EVIDENCE QUESTION
    • CORPORATIONS QUESTION
      • July 2014 Analyses
        • CRIMINAL LAW AND PROCEDURE ANALYSIS
        • CONTRACTS ANALYSIS
        • FAMILY LAW ANALYSIS
        • FEDERAL CIVIL PROCEDURE ANALYSIS
        • EVIDENCE ANALYSIS
        • CORPORATIONS ANALYSIS
            • ltlt13 ASCII85EncodePages false13 AllowTransparency false13 AutoPositionEPSFiles true13 AutoRotatePages None13 Binding Left13 CalGrayProfile (Dot Gain 20)13 CalRGBProfile (sRGB IEC61966-21)13 CalCMYKProfile (US Web Coated 050SWOP051 v2)13 sRGBProfile (sRGB IEC61966-21)13 CannotEmbedFontPolicy Error13 CompatibilityLevel 1413 CompressObjects Tags13 CompressPages true13 ConvertImagesToIndexed true13 PassThroughJPEGImages true13 CreateJobTicket false13 DefaultRenderingIntent Default13 DetectBlends true13 DetectCurves 0000013 ColorConversionStrategy CMYK13 DoThumbnails false13 EmbedAllFonts true13 EmbedOpenType false13 ParseICCProfilesInComments true13 EmbedJobOptions true13 DSCReportingLevel 013 EmitDSCWarnings false13 EndPage -113 ImageMemory 104857613 LockDistillerParams false13 MaxSubsetPct 10013 Optimize true13 OPM 113 ParseDSCComments true13 ParseDSCCommentsForDocInfo true13 PreserveCopyPage true13 PreserveDICMYKValues true13 PreserveEPSInfo true13 PreserveFlatness true13 PreserveHalftoneInfo false13 PreserveOPIComments true13 PreserveOverprintSettings true13 StartPage 113 SubsetFonts true13 TransferFunctionInfo Apply13 UCRandBGInfo Preserve13 UsePrologue false13 ColorSettingsFile ()13 AlwaysEmbed [ true13 ]13 NeverEmbed [ true13 ]13 AntiAliasColorImages false13 CropColorImages true13 ColorImageMinResolution 30013 ColorImageMinResolutionPolicy OK13 DownsampleColorImages true13 ColorImageDownsampleType Bicubic13 ColorImageResolution 30013 ColorImageDepth -113 ColorImageMinDownsampleDepth 113 ColorImageDownsampleThreshold 15000013 EncodeColorImages true13 ColorImageFilter DCTEncode13 AutoFilterColorImages true13 ColorImageAutoFilterStrategy JPEG13 ColorACSImageDict ltlt13 QFactor 01513 HSamples [1 1 1 1] VSamples [1 1 1 1]13 gtgt13 ColorImageDict ltlt13 QFactor 01513 HSamples [1 1 1 1] VSamples [1 1 1 1]13 gtgt13 JPEG2000ColorACSImageDict ltlt13 TileWidth 25613 TileHeight 25613 Quality 3013 gtgt13 JPEG2000ColorImageDict ltlt13 TileWidth 25613 TileHeight 25613 Quality 3013 gtgt13 AntiAliasGrayImages false13 CropGrayImages true13 GrayImageMinResolution 30013 GrayImageMinResolutionPolicy OK13 DownsampleGrayImages true13 GrayImageDownsampleType Bicubic13 GrayImageResolution 30013 GrayImageDepth -113 GrayImageMinDownsampleDepth 213 GrayImageDownsampleThreshold 15000013 EncodeGrayImages true13 GrayImageFilter DCTEncode13 AutoFilterGrayImages true13 GrayImageAutoFilterStrategy JPEG13 GrayACSImageDict ltlt13 QFactor 01513 HSamples [1 1 1 1] VSamples [1 1 1 1]13 gtgt13 GrayImageDict ltlt13 QFactor 01513 HSamples [1 1 1 1] VSamples [1 1 1 1]13 gtgt13 JPEG2000GrayACSImageDict ltlt13 TileWidth 25613 TileHeight 25613 Quality 3013 gtgt13 JPEG2000GrayImageDict ltlt13 TileWidth 25613 TileHeight 25613 Quality 3013 gtgt13 AntiAliasMonoImages false13 CropMonoImages true13 MonoImageMinResolution 120013 MonoImageMinResolutionPolicy OK13 DownsampleMonoImages true13 MonoImageDownsampleType Bicubic13 MonoImageResolution 120013 MonoImageDepth -113 MonoImageDownsampleThreshold 15000013 EncodeMonoImages true13 MonoImageFilter CCITTFaxEncode13 MonoImageDict ltlt13 K -113 gtgt13 AllowPSXObjects false13 CheckCompliance [13 None13 ]13 PDFX1aCheck false13 PDFX3Check false13 PDFXCompliantPDFOnly false13 PDFXNoTrimBoxError true13 PDFXTrimBoxToMediaBoxOffset [13 00000013 00000013 00000013 00000013 ]13 PDFXSetBleedBoxToMediaBox true13 PDFXBleedBoxToTrimBoxOffset [13 00000013 00000013 00000013 00000013 ]13 PDFXOutputIntentProfile ()13 PDFXOutputConditionIdentifier ()13 PDFXOutputCondition ()13 PDFXRegistryName ()13 PDFXTrapped False1313 CreateJDFFile false13 Description ltlt13 ARA 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 BGR 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 CHS ltFEFF4f7f75288fd94e9b8bbe5b9a521b5efa7684002000410064006f006200650020005000440046002065876863900275284e8e9ad88d2891cf76845370524d53705237300260a853ef4ee54f7f75280020004100630072006f0062006100740020548c002000410064006f00620065002000520065006100640065007200200035002e003000204ee553ca66f49ad87248672c676562535f00521b5efa768400200050004400460020658768633002gt13 CHT ltFEFF4f7f752890194e9b8a2d7f6e5efa7acb7684002000410064006f006200650020005000440046002065874ef69069752865bc9ad854c18cea76845370524d5370523786557406300260a853ef4ee54f7f75280020004100630072006f0062006100740020548c002000410064006f00620065002000520065006100640065007200200035002e003000204ee553ca66f49ad87248672c4f86958b555f5df25efa7acb76840020005000440046002065874ef63002gt13 CZE 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 DAN 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 DEU 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 ESP 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 ETI 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 FRA 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 GRE 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 HEB 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 HRV (Za stvaranje Adobe PDF dokumenata najpogodnijih za visokokvalitetni ispis prije tiskanja koristite ove postavke Stvoreni PDF dokumenti mogu se otvoriti Acrobat i Adobe Reader 50 i kasnijim verzijama)13 HUN 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 ITA 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 JPN ltFEFF9ad854c18cea306a30d730ea30d730ec30b951fa529b7528002000410064006f0062006500200050004400460020658766f8306e4f5c6210306b4f7f75283057307e305930023053306e8a2d5b9a30674f5c62103055308c305f0020005000440046002030d530a130a430eb306f3001004100630072006f0062006100740020304a30883073002000410064006f00620065002000520065006100640065007200200035002e003000204ee5964d3067958b304f30533068304c3067304d307e305930023053306e8a2d5b9a306b306f30d530a930f330c8306e57cb30818fbc307f304c5fc59808306730593002gt13 KOR ltFEFFc7740020c124c815c7440020c0acc6a9d558c5ec0020ace0d488c9c80020c2dcd5d80020c778c1c4c5d00020ac00c7a50020c801d569d55c002000410064006f0062006500200050004400460020bb38c11cb97c0020c791c131d569b2c8b2e4002e0020c774b807ac8c0020c791c131b41c00200050004400460020bb38c11cb2940020004100630072006f0062006100740020bc0f002000410064006f00620065002000520065006100640065007200200035002e00300020c774c0c1c5d0c11c0020c5f40020c2180020c788c2b5b2c8b2e4002egt13 LTH 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 LVI 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 NLD (Gebruik deze instellingen om Adobe PDF-documenten te maken die zijn geoptimaliseerd voor prepress-afdrukken van hoge kwaliteit De gemaakte PDF-documenten kunnen worden geopend met Acrobat en Adobe Reader 50 en hoger)13 NOR 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 POL ltFEFF0055007300740061007700690065006e0069006100200064006f002000740077006f0072007a0065006e0069006100200064006f006b0075006d0065006e007400f300770020005000440046002000700072007a0065007a006e00610063007a006f006e00790063006800200064006f002000770079006400720075006b00f30077002000770020007700790073006f006b00690065006a0020006a0061006b006f015b00630069002e002000200044006f006b0075006d0065006e0074007900200050004400460020006d006f017c006e00610020006f007400770069006500720061010700200077002000700072006f006700720061006d006900650020004100630072006f00620061007400200069002000410064006f00620065002000520065006100640065007200200035002e0030002000690020006e006f00770073007a0079006d002egt13 PTB 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 RUM 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 RUS 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 SKY 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 SLV 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 SUO ltFEFF004b00e40079007400e40020006e00e40069007400e4002000610073006500740075006b007300690061002c0020006b0075006e0020006c0075006f00740020006c00e400680069006e006e00e4002000760061006100740069007600610061006e0020007000610069006e006100740075006b00730065006e002000760061006c006d0069007300740065006c00750074007900f6006800f6006e00200073006f00700069007600690061002000410064006f0062006500200050004400460020002d0064006f006b0075006d0065006e007400740065006a0061002e0020004c0075006f0064007500740020005000440046002d0064006f006b0075006d0065006e00740069007400200076006f0069006400610061006e0020006100760061007400610020004100630072006f0062006100740069006c006c00610020006a0061002000410064006f00620065002000520065006100640065007200200035002e0030003a006c006c00610020006a006100200075007500640065006d006d0069006c006c0061002egt13 SVE 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 TUR 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 UKR ltFEFF04120438043a043e0440043804410442043e043204430439044204350020044604560020043f043004400430043c043504420440043800200434043b044f0020044104420432043e04400435043d043d044f00200434043e043a0443043c0435043d044204560432002000410064006f006200650020005000440046002c0020044f043a04560020043d04300439043a04400430044904350020043f045604340445043e0434044f0442044c00200434043b044f0020043204380441043e043a043e044f043a04560441043d043e0433043e0020043f0435044004350434043404400443043a043e0432043e0433043e0020043404400443043a0443002e00200020042104420432043e04400435043d045600200434043e043a0443043c0435043d0442043800200050004400460020043c043e0436043d04300020043204560434043a0440043804420438002004430020004100630072006f006200610074002004420430002000410064006f00620065002000520065006100640065007200200035002e0030002004300431043e0020043f04560437043d04560448043e04570020043204350440044104560457002egt13 ENU (Use these settings to create Adobe PDF documents best suited for high-quality prepress printing Created PDF documents can be opened with Acrobat and Adobe Reader 50 and later)13 gtgt13 Namespace [13 (Adobe)13 (Common)13 (10)13 ]13 OtherNamespaces [13 ltlt13 AsReaderSpreads false13 CropImagesToFrames true13 ErrorControl WarnAndContinue13 FlattenerIgnoreSpreadOverrides false13 IncludeGuidesGrids false13 IncludeNonPrinting false13 IncludeSlug false13 Namespace [13 (Adobe)13 (InDesign)13 (40)13 ]13 OmitPlacedBitmaps false13 OmitPlacedEPS false13 OmitPlacedPDF false13 SimulateOverprint Legacy13 gtgt13 ltlt13 AddBleedMarks false13 AddColorBars false13 AddCropMarks false13 AddPageInfo false13 AddRegMarks false13 ConvertColors ConvertToCMYK13 DestinationProfileName ()13 DestinationProfileSelector DocumentCMYK13 Downsample16BitImages true13 FlattenerPreset ltlt13 PresetSelector MediumResolution13 gtgt13 FormElements false13 GenerateStructure false13 IncludeBookmarks false13 IncludeHyperlinks false13 IncludeInteractive false13 IncludeLayers false13 IncludeProfiles false13 MultimediaHandling UseObjectSettings13 Namespace [13 (Adobe)13 (CreativeSuite)13 (20)13 ]13 PDFXOutputIntentProfileSelector DocumentCMYK13 PreserveEditing true13 UntaggedCMYKHandling LeaveUntagged13 UntaggedRGBHandling UseDocumentProfile13 UseDocumentBleed false13 gtgt13 ]13gtgt setdistillerparams13ltlt13 HWResolution [2400 2400]13 PageSize [612000 792000]13gtgt setpagedevice13

Page 24: July 2014 MEE Questions and Analyses - NCBE...This publication includes the questions and analyses from the July 2014 MEE. (In the actual test, the questions are simply numbered rather

FEDERAL CIVIL PROCEDURE ANALYSIS (Federal Civil Procedure III IVC)

ANALYSIS

Legal Problems

(1) Is the logging company entitled to join this action as a matter of right

(2)(a) May the nonprofit organization obtain a temporary restraining order to stop the USFS from issuing a logging permit

(2)(b) May the nonprofit organization obtain a preliminary injunction to stop the USFS from issuing a logging permit during the pendency of the action

DISCUSSION

Summary

The logging company is entitled to intervene in this action as a matter of right because it has an interest in the property or transaction that is the subject of the action and is so situated that its interest may be impaired or impeded as a practical matter if the action goes forward without it The logging companyrsquos interest is not adequately represented by the USFSrsquos presence in the lawsuit

The nonprofit organization may seek a temporary restraining order (TRO) followed by a preliminary injunction to prevent the USFS from issuing a logging permit pending the outcome of the action The nonprofit is likely to obtain a TRO if it can demonstrate a risk of immediate and irreparable injury The nonprofit is also likely to obtain a preliminary injunction if it can demonstrate a significant threat of irreparable harm and a likelihood of success on the merits of its National Environmental Policy Act (NEPA) claim

Point One (50) Rule 24(a) of the Federal Rules of Civil Procedure requires federal courts to allow a person to intervene in an action as a matter of right if the person a) is interested in the property or transaction that is the subject of the action b) is so situated that its interest may be impaired or impeded if the litigation goes forward without it and c) is not adequately represented by existing parties Here the logging company likely meets all three requirements and should be allowed to intervene as a matter of right

Rule 24 of the Federal Rules of Civil Procedure governs intervention the process by which a non-party to an action may join the litigation Under Rule 24(a) (intervention of right) a person must be permitted to intervene if three conditions are met (1) the movant ldquoclaims an interest relating to the property or transaction that is the subject of the actionrdquo (2) the movant ldquois so situated that disposition of the action may as a practical matter impair or impede the movantrsquos ability to protect its interestrdquo and (3) ldquoexisting partiesrdquo do not ldquoadequately represent [the movantrsquos] interestrdquo FED R CIV P 24(a) The three requirements for intervention of right are often ldquovery interrelatedrdquo 7C CHARLES ALAN WRIGHT ET AL FEDERAL PRACTICE AND PROCEDURE sect 1908 at 297 (2007 amp 2011 Supp)

22

Federal Civil Procedure Analysis

Here the court should find that the logging company meets this test First the logging company has a strong interest in the property or transaction that is the subject of this action The USFS has accepted the logging companyrsquos bid and the logging company is merely awaiting issuance of a logging permit to begin logging The nonprofit organization is seeking to prevent this logging The logging company therefore has a strong direct and substantial interest in the subject matter of the lawsuit and in having its winning bid honored and a logging permit issued See eg Kleissler v US Forest Serv 157 F3d 964 972 (3d Cir 1998) (stating that ldquo[t]imber companies have direct and substantial interests in a lawsuit aimed at halting loggingrdquo) see also Natural Resources Defense Council v US Nuclear Regulatory Commrsquon 578 F2d 1341 1343ndash 44 (10th Cir 1978) (holding that applicants whose license renewals were pending had Rule 24(a)(2) interests where the lawsuit sought to halt the license-issuing process pending preparation of environmental impact statements) See generally 7C WRIGHT ET AL supra sect 19081 at 309 (ldquoIf there is a direct substantial legally protectable interest in the proceedings it is clear that this requirement of the rule is satisfiedrdquo) Second the logging companyrsquos interest in receiving a logging permit may well be impaired as a practical matter by the outcome of the lawsuit If the USFS loses the lawsuit it will have to prepare an environmental impact statement before issuing the logging companyrsquos permit This will at a minimum delay the logging companyrsquos ability to exercise its rights and may in the long r un mean that no logging permit is ever issued Intervention of right is not limited to those that would be legally bound as a matter of preclusion doctrine Id sect 19082 at 368 Rather ldquo[t]he rule is satisfied whenever disposition of the present action would put the movant at a practical disadvantage in protecting its interestrdquo Id sect 19082 at 369 Here that condition is easily satisfied See Kleissler 157 F3d at 972 (ldquoTimber companies have direct and substantial interests in a lawsuit aimed at halting logging rdquo)

Given that the logging company has an interest that may be impaired by disposition of the action it should be allowed to intervene unless the court is persuaded that the USFS adequately represents the logging companyrsquos interest See Rule 24(a)(2) 7C WRIGHT ET AL supra sect 1909 Here it could be argued that the USFS adequately represents the logging companyrsquos interest because the USFS presumably wants the court to uphold its development plan and allow it to proceed with issuance of the logging permit which is the same relief that the logging company would seek However whether representation is truly adequate depends upon ldquo[a] discriminating appraisal of the circumstancesrdquo 7C WRIGHT ET AL supra sect 1909 at 440 Although both the government and the logging company wish to avoid the preparation of an environmental impact statement their interests are distinct The USFSrsquos interest is proper management of the national forest system while the logging companyrsquos interest is making a profit from logging the 5000-acre tract The USFSrsquos handling of the litigation is likely to be affected by a variety of policy concerns and political considerations that have nothing to do with the logging companyrsquos purely economic interest in securing the right to cut trees in the Scenic National Forest See eg Kleissler 157 F3d at 973ndash74 (ldquo[T]he government represents numerous complex and conflicting interests in matters of this nature The straightforward business interests asserted by intervenors here may become lost in the thicket of sometimes inconsistent governmental policiesrdquo)

[NOTES (1) Examinees who mistakenly analyze the logging companyrsquos case for joinder under the related but incorrect Rule 19 ldquoRequired Joinder of Partiesrdquo may receive credit Rule 19 allows existing parties to demand joinder of non-parties (or seek dismissal of the case if they canrsquot get it) There is a close relationship between Rule 24 and Rule 19 and both contain a similar standard for determining when ldquointerestedrdquo third parties are ldquoentitledrdquo or ldquorequiredrdquo to be in the lawsuit Indeed the two prongs of the Rule 24 intervention test that are discussed above

23

Federal Civil Procedure Analysis

are nearly identical to the two prongs of the Rule 19(a) required joinder test Examinees who discuss and apply the test should receive credit even if they cite Rule 19 rather than Rule 24

(2) Examinees may discuss permissive joinder Although permissive joinder is a possibility here the question asks only whether the logging company can join the action as a matter of right and a permissive joinder analysis is not responsive to the question To the extent an examinee discusses permissive joinder the analysis will focus on whether the logging company ldquohas a claim or defense that shares with the main action a common question of law or factrdquo FED R CIV P 24(b)(1)(B) The district court also ldquomust consider whether the intervention will unduly delay or prejudice the adjudication of the original partiesrsquo rightsrdquo FED R CIV P 24(b)(3) On our facts the logging companyrsquos claim for the issuance of a logging permit would certainly share common questions of law and fact with the USFSrsquos defense against the nonprofitrsquos claim There are no facts suggesting that the logging companyrsquos presence would unduly delay or otherwise prejudice adjudication of the original action Thus the district court would have discretion to permit the logging company to intervene even if it denied intervention of right]

Point Two(a) (25) The nonprofit organization could seek and would likely obtain a temporary restraining order to stop the USFS from issuing a logging permit pending a hearing on an application for a preliminary injunction

The first type of interim relief the nonprofit could seek to stop the USFS from issuing a logging permit to the logging company is a temporary restraining order (TRO) prohibiting the USFS from issuing the logging permit A TRO can be issued without notice to the adverse party but only in limited circumstances and only for a limited time FED R CIV P 65(b) To secure a TRO without notice the nonprofit would need to submit an affidavit containing specific facts that demonstrate a risk of ldquoimmediate and irreparable injuryrdquo if a permit is issued FED R CIV P 65(b)(1) In deciding whether to grant a TRO courts will also consider the same factors that are relevant in deciding whether to grant a preliminary injunction (eg the moving partyrsquos likelihood of success on the merits the balance of hardships and the public interest) See Point Two(b) infra The TRO would last only long enough for the court to consider and resolve a request by the nonprofit for a preliminary injunction but no longer than 14 days (unless the court extends it for good cause or the adverse party consents to an extension) In addition bond is required

Here the court is likely to grant the nonprofitrsquos request The nonprofit could plausibly claim that cutting down 5000 acres of old-growth forest in an area that is home to the highest concentration of wildlife in the western United States would have ldquoan immediate and irreparablerdquo adverse impact on the environment and cause irreparable harm to the nonprofitrsquos interest in preserving and protecting natural resources including wildlife habitat

Point Two(b) (25) The nonprofit could also seek and would likely obtain a preliminary injunction to stop the USFS which is likely to be granted if the nonprofitrsquos claim that the USFS violated NEPA has a strong basis in fact and law

Because the TRO would be temporary the nonprofit would need to move for a preliminary injunction to prevent the USFS from issuing a logging permit throughout the pendency of the litigation Preliminary injunctions are injunctions that seek to ldquoprotect [the] plaintiff from

24

Federal Civil Procedure Analysis

irreparable injury and to preserve the courtrsquos power to render a meaningful decision after a trial on the meritsrdquo 11A CHARLES ALAN WRIGHT ET AL FEDERAL PRACTICE AND PROCEDURE sect 2947 at 112 (2013) Rule 65 of the Federal Rules of Civil Procedure sets out the procedural requirements for preliminary injunctions Preliminary injunctions may be granted only upon notice to the adverse party FED R CIV P 65(a)(1) and only if the movant ldquogives security in an amount that the court considers proper to pay the costs and damages sustained by any party found to have been wrongfully enjoined or restrainedrdquo FED R CIV P 65(c)

While Rule 65 sets out the procedural requirements for preliminary injunctive relief it does not specify the substantive grounds upon which it may be granted The courtrsquos discretion in ruling upon a motion for a preliminary injunction ldquois exercised in conformity with historic federal equity practicerdquo 11A WRIGHT ET AL supra sect 2947 at 114 The court typically considers four factors

(1) the significance of the threat of irreparable harm to the plaintiff if the injunction is not granted (2) the balance between this harm and the injury that granting the injunction would inflict on the defendant (3) the probability that the plaintiff will succeed on the merits and (4) the public interest

Id sect 2948 at 122ndash24 accord Habitat Educ Center v Bosworth 363 F Supp 2d 1070 1088 (ED Wis 2005) The most important of these factors is the risk of irreparable harm to the plaintiff 11A WRIGHT ET AL supra sect 29481 at 129 If the plaintiff has an adequate remedy at law (eg if money damages can compensate the plaintiff for its loss) then a preliminary injunction will be denied Id sect 29481

Here a court would likely conclude that the potential for environmental damage to the forest creates a significant threat of irreparable harm ldquo[E]nvironmental injury is often irreparable Courts have recognized that logging such as would occur [here] can have longshyterm environmental consequences and thus satisfy the irreparable injury criterionrdquo Habitat Educ Center 363 F Supp 2d at 1089 (citing Idaho Sporting Congress Inc v Alexander 222 F3d 562 569 (9th Cir 2000) (noting that the imminent and continuing logging activities presented ldquoevidence of environmental harm sufficient to tip the balance in favor of injunctive reliefrdquo)) Neighbors of Cuddy Mountain v US Forest Service 137 F3d 1372 1382 (9th Cir 1998) (stating that ldquo[t]he old growth forests plaintiffs seek to protect would if cut take hundreds of years to reproducerdquo) (internal citation omitted)) see also 11C WRIGHT ET AL supra sect 29481 at 151 (noting that ldquoa preliminary injunction has been issued to prevent harm to the environmentrdquo)

The second factor the balance between the harm to the plaintiff and the harm the defendant will suffer if the injunction is issued also appears to support issuance of a preliminary injunction here The USFS will have to wait before it can develop the Scenic National Forest and the logging company may lose money if the delay is prolonged These economic harms could be compensated monetarily if an injunction is issued inappropriately Where ldquoan injunction bond can compensate [the] defendant for any harm the injunction is likely to inflict the balance should be struck in favor of [the] plaintiffrdquo Id sect 29482 at 192 See also Habitat Educ Center 363 F Supp 2d at 1089 (stating that ldquothe relative absence of harmful effects on the Forest Service weighs in favor of granting the injunctionrdquo)

The third factor is the likelihood that the plaintiff will prevail on the merits Although there is limited information concerning the merits of the action the nonprofit alleges that the federal statute (NEPA) requires an environmental impact statement and further states that the USFS created no environmental impact analysis or statement at all Assuming that those

25

Federal Civil Procedure Analysis

allegations are correct it seems plausible to conclude that the nonprofit will be able to show a likelihood of success on the merits

Finally courts deciding whether or not to issue preliminary injunctive relief are to consider the public interest ldquoFocusing on this factor is another way of inquiring whether there are policy considerations that bear on whether the order should issuerdquo 11C WRIGHT ET AL supra sect 29484 at 214 If the court concludes that the nonprofit is likely to succeed on its NEPA claim because the USFS wrongfully failed to conduct an environmental impact assessment it is likely to find that the public interest would be served by restraining the USFS from proceeding with logging in a national forest See Heartwood Inc v US Forest Service 73 F Supp 2d 962 979 (SD Ill 1999) affrsquod on other grounds 230 F3d 947 (7th Cir 2000) (ldquoviolations by federal agencies of NEPArsquos provisions as established by Congress harm the public as well as the environmentrdquo)

Thus a court is very likely to grant a preliminary injunction if it concludes that the nonprofit has a significant likelihood of success on the merits

26

EVIDENCE ANALYSIS (Evidence ID IIA amp C)

ANALYSIS

Legal Problems

(1) Under what circumstances can evidence of prior convictions be used to impeach a witnessrsquos credibility in a civil case

(1)(a) May the inmatersquos credibility be impeached by evidence of a 12-year-old felony drug conviction if he was released from prison 9 years ago

(1)(b) May the inmatersquos credibility be impeached by evidence of an 8-year-old misdemeanor perjury conviction that was punishable by 1 year in jail if he pleaded guilty and was sentenced only to pay a $5000 fine

(1)(c) May the inmatersquos credibility be impeached by evidence of a 7-year-old sexual assault conviction if the inmate is still serving a 10-year prison sentence and the victim was his 13-year-old daughter

(2)(a) May the guardrsquos credibility be impeached by cross-examination regarding specific instances of misconduct (ie lying on his reacutesumeacute) relevant to credibility

(2)(b) May the guardrsquos credibility be impeached by admission of extrinsic evidence (his reacutesumeacute and academic transcript) offered to prove specific instances of misconduct relevant to credibility

DISCUSSION

Summary

Under the Federal Rules of Evidence witnesses can be impeached with evidence of prior convictions andor specific instances of misconduct Whether evidence of prior convictions should be admitted to impeach generally depends on the nature of the crime the amount of time that has passed and (only in criminal cases) whether the ldquowitnessrdquo is the defendant FED R EVID 609(a)

In this civil case evidence of the inmatersquos conviction for distribution of marijuana should be admitted to impeach the inmate because he was convicted of a felony and was released from prison fewer than 10 years ago FED R EVID 609(a)(1) Credibility is critically important in this case because the jury will hear conflicting testimony from the two disputing parties and there were no other eyewitnesses to the altercation Under Rule 609(a)(1) the inmatersquos conviction should be admitted because it has some bearing on his credibility and its probative value is not substantially outweighed by concerns of unfair prejudice confusion or delay Id

Evidence of the inmatersquos misdemeanor conviction for perjury must be admitted because the crime ldquorequired provingmdashor the witnessrsquos admittingmdasha dishonest act or false statementrdquo by the inmate FED R EVID 609(a)(2)

27

Evidence Analysis

Evidence of the inmatersquos felony conviction for sexual assault should be excluded because its probative value is substantially outweighed by the danger of unfair prejudice to the inmate based on the heinous nature of the crime FED R EVID 609(a)(1) In the alternative the judge could limit the evidence relating to this conviction by excluding details of the inmatersquos crime

In all civil (and criminal) cases witnesses can also be impeached with evidence of specific instances of prior misconduct that did not result in a conviction FED R EVID 608(b) Pursuant to Rule 608(b) misconduct probative of untruthfulness can be inquired into on cross-examination but cannot be proved through extrinsic evidence Id Thus the inmatersquos counsel should be permitted to cross-examine the guard regarding the false statement in the guardrsquos reacutesumeacute However extrinsic evidence of the guardrsquos misconduct (ie the guardrsquos authenticated reacutesumeacute and transcript from the local college) should not be admitted even if the guard denies wrongdoing or refuses to answer cross-examination questions about these matters Id

Point One (10) The Federal Rules of Evidence permit impeachment of witnesses with evidence of prior convictions

Whether convictions should be admitted to impeach generally depends on the nature of the crime the amount of time that has passed and (only in criminal cases) whether the ldquowitnessrdquo is the defendant FED R EVID 609(a) Under Rule 609(a) evidence of prior convictions may be admitted for the purpose of ldquoattacking a witnessrsquos character for truthfulnessrdquo Id

There are two basic types of convictions that can be admitted for the purpose of impeachment

(1) convictions for crimes ldquopunishable by death or by imprisonment for more than one yearrdquo (which generally correlates to ldquofeloniesrdquo) FED R EVID 609(a)(1) and (2) convictions ldquofor any crimes regardless of the punishment if the court can readily determine that establishing the elements of the crime required provingmdashor the witnessrsquos admittingmdasha dishonest act or false statementrdquo FED R EVID 609(a)(2)

Pursuant to Rule 609(a)(1) in civil cases the admission of evidence of a felony conviction is ldquosubject to Rule 403 [which says that a court may exclude relevant evidence if its probative value is substantially outweighed by other factors]rdquo FED R EVID 609(a)(1) However Rule 403 does not protect the witness against admission of prior convictions involving dishonestymdashwhich must be admitted by the court FED R EVID 609(a)(2)

Finally Federal Rule of Evidence 609(b) contains the presumption that a conviction that is more than 10 years old or where more than 10 years has passed since the witnessrsquos release from confinement (whichever is later) should not be admitted unless ldquoits probative value supported by specific facts and circumstances substantially outweighs its prejudicial effectrdquo and the proponent has provided the adverse party with reasonable written notice FED R EVID 609(b)

Point One(a) (25) The court should admit evidence of the inmatersquos 12-year-old felony marijuana distribution conviction

The inmatersquos conviction for marijuana distribution was for a felony punishable by imprisonment for more than one year See FED R EVID 609(a)(1) Moreover although the conviction was 12 years ago the 10-year time limit of Rule 609(b) is not exceeded because that time limit runs

28

Evidence Analysis

from the date of either ldquothe witnessrsquos conviction or release from confinement for it whichever is laterrdquo FED R EVID 609(b) Because the inmate served three years in prison he was released from confinement nine years ago

However pursuant to Rule 609(a)(1) the admission of felony convictions to impeach a witness in a civil case is ldquosubject to Rule 403rdquo FED R EVID 609(a)(1) Neither Rule 609(a) nor the advisory committee notes specify which factors courts should consider when balancing the probative value of a conviction against the dangers identified in Rule 403 (which include (1) unfair prejudice (2) confusion of the issues (3) misleading the jury (4) waste of time or undue delay and (5) needless presentation of cumulative evidence) FED R EVID 403

In this case credibility is very important because the evidence consists primarily of the testimony of the disputing parties and there were no other eyewitnesses to the altercation This enhances the probative value of any evidence bearing on the inmatersquos credibility A court is likely to conclude that the inmatersquos prior felony drug conviction is relevant to his credibility See eg United States v Brito 427 F3d 53 64 (1st Cir 2005) (ldquoPrior drug-trafficking crimes are generally viewed as having some bearing on veracityrdquo) Although the probative value of any conviction diminishes with age see eg United States v Brewer 451 F Supp 50 53 (ED Tenn 1978) the inmatersquos ongoing problems with the law suggest that he has continued (and even escalated) his criminal behavior over the past nine years The court should admit this evidence because its probative value is not substantially outweighed by any Rule 403 concerns Specifically any prejudice to the inmate would be slight because the conviction is unrelated to the altercation at issue and the conviction was not for a heinous crime that might inflame the jury

[NOTE Whether an examinee identifies the jury instruction as containing a ldquoconclusiverdquo or ldquomandatoryrdquo presumption is less important than the examineersquos analysis of the constitutional infirmities]

Point One(b) (15) The court must admit evidence of the inmatersquos eight-year-old misdemeanor conviction because perjury is a crime of dishonesty

Rule 609(a)(2) provides that evidence of a criminal conviction ldquomust be admitted if the court can readily determine that establishing the elements of the crime required provingmdashor the witnessrsquos admittingmdasha dishonest act or false statementrdquo FED R EVID 609(a)(2) The inmatersquos conviction for perjury would have necessarily required proving that the inmate engaged in an act of dishonesty This conviction occurred within the past 10 years so it ldquomust be admittedrdquo because in contrast to Rule 609(a)(1) (discussed in Point One(a)) admission under Rule 609(a)(2) is mandatory and not subject to Rule 403

Point One(c) (20) The court should exclude evidence of the inmatersquos seven-year-old felony sexual assault conviction because the probative value of this evidence is substantially outweighed by the danger of unfair prejudice In the alternative the details of the prior conviction could be excluded

The inmatersquos conviction for felony sexual assault was seven years ago and he has not yet been released from incarceration so Rule 609(a) but not 609(b) is applicable here FED R EVID 609(a) This conviction is therefore admissible to impeach the inmate unless its probative value is substantially outweighed by the danger of unfair prejudice or any other Rule 403 concern Id

29

Evidence Analysis

Sex crimes are generally not considered relevant to credibility see Hopkins v State 639 So 2d 1247 1254 (Miss 1993) so the probative value of this conviction is relatively low Moreover the heinous nature of the inmatersquos crime (sexual assault on his daughter) makes the danger of unfair prejudice to the inmate very high Thus the court should exclude evidence of the conviction because it was for a heinous offense that is likely to inflame the jury and it has little bearing on credibility See eg United States v Beahm 664 F2d 414 419 (4th Cir 1981)

As an alternative to excluding this evidence the judge could minimize the unfair prejudice to the inmate by permitting limited cross-examination but refusing to allow specific questions about the nature of the inmatersquos conviction For example a court could limit cross-examination to the fact that the inmate was convicted of a ldquofelonyrdquo or perhaps that he was convicted of a ldquosexual assaultrdquo without identifying the victim However because evidence of the inmatersquos prior convictions can be admitted solely for the purpose of enabling the jury to assess his credibility and because his two earlier convictions should have already been admitted the court should exclude all evidence of the felony sexual assault conviction

Point Two(a) (15) The court should permit the inmatersquos counsel to cross-examine the guard regarding the false statement in his reacutesumeacute because the guardrsquos misconduct bears on his truthfulness

The inmate wishes to cross-examine the guard about his prior dishonest behaviormdashlying on his reacutesumeacutemdashthat did not involve a criminal conviction Rule 608(b) allows witnesses to be cross-examined about specific instances of prior non-conviction misconduct probative of untruthfulness ldquoin order to attack the witnessrsquos character for truthfulnessrdquo FED R EVID 608(b)

The courtrsquos decision to allow cross-examination about the guardrsquos prior dishonest behavior depends on the probative value of such evidence balanced against the danger of unfair prejudice to the guard or any other Rule 403 concern FED R EVID 403 Here the guardrsquos false statement on his reacutesumeacute that he obtained a degree in Criminal Justice is highly probative of his untruthfulness because it grossly misrepresents his actual academic record was made recently and was made with the intent to deceive Because the probative value of this evidence is very strong and is not substantially outweighed by any Rule 403 concerns cross-examination of the guard on this topic should be permitted The court may also consider it fair to permit this cross-examination of the guard on these matters assuming that one or more of the inmatersquos prior convictions have been admitted to impeach his credibility

Point Two(b) (15) The court should exclude extrinsic evidence of the guardrsquos non-conviction misconduct even if the guard denies wrongdoing or refuses to answer questions about the matter

Although Rule 608(b) allows cross-examination about specific instances of prior misconduct probative of untruthfulness ldquoextrinsic evidencerdquo offered to prove such misconduct is not admissible FED R EVID 608(b) The rationale for this rule is that allowing the introduction of extrinsic evidence of prior misconduct by witnesses when these acts are relevant only to the witnessesrsquo truthfulness and not to the main issues in the case would create too great a risk of confusing the jury and unduly delaying the trial The court does not have discretion to admit this extrinsic evidence See eg United States v Elliot 89 F3d 1360 1368 (8th Cir 1996)

30

Evidence Analysis

Here the inmatersquos counsel may cross-examine the guard about the false statement on his reacutesumeacute However the inmatersquos counsel must accept the guardrsquos response Even if the guard denies wrongdoing or refuses to answer questions about the matter the inmatersquos counsel cannot introduce the guardrsquos reacutesumeacute or the transcript from the local college to prove the guardrsquos misconduct

31

CORPORATIONS ANALYSIS (Corporations VA2 IX)

ANALYSIS

Legal Problems

(1) Do shareholders have the authority to amend a corporationrsquos bylaws with respect to director nominations

(2) Do board-approved bylaws on a particular subject here nomination of directors preempt subsequent conflicting bylaw amendments by shareholders

(3) Is a suit challenging both managementrsquos refusal to include the proposed bylaw amendment in Megarsquos proxy statement and the boardrsquos amendment of the bylaws dealing with nomination of directors a direct or derivative suit

DISCUSSION

Summary

The voting and litigation rights of the shareholders of Mega are subject to the provisions of the Model Business Corporations Act (MBCA)

The investorrsquos proposed bylaw provision is not inconsistent with state law Under the MBCA shareholders may amend the bylaws when the amendment deals with a proper matter for the corporationrsquos bylaws such as procedures for nominating directors

The Mega boardrsquos bylaw amendment does not preempt the investorrsquos proposed bylaw provision or the Mega shareholdersrsquo power to approve it While shareholders can limit the boardrsquos power to amend or repeal the bylaws the board cannot limit the shareholdersrsquo power

Whether the investor must make a demand on Megarsquos board depends on how the investor frames its claim If the investor claims a violation of shareholder voting rights the claim is direct and pre-suit demand on the board is not required If on the other hand the investor claims that the directors violated their fiduciary duties by amending the bylaws to entrench themselves the claim is derivative and a pre-suit demand is required

Point One (30) Shareholders may amend the corporationrsquos bylaws where the proposed bylaw provision relates to procedural matters typically included in the bylaws such as the nomination of directors

Internal affairs of the corporation such as the conduct of shareholder meetings and election of directors are subject to the corporate law of the state of incorporation See McDermott Inc v Lewis 531 A2d 206 (Del 1987) (applying law of jurisdiction where corporation was incorporated in case involving voting rights) This statersquos corporate statute is modeled on the MBCA

Under the MBCA ldquoshareholders may amend the corporationrsquos bylawsrdquo MBCA sect 1020(a) Thus the only question is whether the bylaws can specify the procedures for shareholder nomination of directors

32

Corporations Analysis

The MBCA states that the bylaws ldquomay contain any provision that is not inconsistent with law or the articles of incorporationrdquo MBCA sect 206(b) In addition the MBCA was revised in 2009 to address shareholder nomination of directors in public corporations (known as ldquoproxy accessrdquo) and specifies that the bylaws ldquomay contain a requirement that the corporation include in its [proxy materials] one or more individuals nominated by a shareholderrdquo MBCA sect 206(c)(1) see Committee on Corporate Laws ABA Section of Business Law Report on the Roles of Boards of Directors and Shareholders of Publicly Owned Corporations and Changes to the Model Business Corporations ActmdashAdoption of Shareholder Proxy Access Amendments to Chapters 2 and 10 65 BUS LAWYER 1105 (2010)

The inclusion of director-nomination procedures in the bylaws is consistent with practice and is recognized by the Delaware courts whose views on corporate law carry significant weight Typically the procedures for nomination of directors are found in the bylaws See 1 COX amp HAZEN TREATISE ON THE LAW OF CORPORATIONS sect 312 (3d ed 2011) see also 4 FLETCHER CORP FORMS ANN PART III ch 21 (2013) (including sample bylaws that permit nomination of directors by shareholders) The Delaware Supreme Court has confirmed that the bylaws may ldquodefine the process and proceduresrdquo for director elections See CA Inc v AFSCME Employees Pension Plan 953 A2d 227 (Del 2008) (concluding that bylaw amendment requiring reimbursement of election expenses to certain successful shareholder nominators is ldquoproper subjectrdquo under Delaware law)

[NOTE The question of the proper scope of the bylaws can be answered using the more general MBCA sect 206(b) or the 2009 MBCA revision adding sect 206(c)(1) (adopted in CT ME VA) In addition some examinees might raise the point that shareholder proposals may not compel the board to take action such as by including shareholder nominations in the companyrsquos proxy materials on the theory that the ldquobusiness and affairsrdquo of the corporation are to be managed by the board See MBCA sect 801(b) Although shareholders are generally limited to adopting precatory resolutions that recommend or encourage board action this limitation does not apply when shareholders have specific authority to take binding action on their ownmdashsuch as to amend the bylaws]

Point Two (30) Shareholders can amend (or repeal) board-approved bylaws Further shareholders can limit the boardrsquos power to later amend and repeal a shareholder-approved bylaw

Under the MBCA shareholders have the power to amend the bylaws See Point One The board shares this power with the shareholders unless (1) the corporationrsquos articles ldquoreserve that power exclusively to the shareholdersrdquo or (2) ldquothe shareholders in amending repealing or adopting a bylaw expressly provide that the board of directors may not amend repeal or reinstate that bylawrdquo See MBCA sect 1020(b)

Shareholder-approved bylaw provisions can amend or repeal existing bylaw provisions whether originally approved by the board or by shareholders See ALAN R PALMITER CORPORATIONS EXAMPLES AND EXPLANATIONS sect 713 (7th ed 2012) Thus the Mega boardrsquos bylaw amendmentmdashwhich set more demanding thresholds for shareholder nomination of directors than the investorrsquos proposed bylaw provisionmdashwould be superseded (repealed) if Megarsquos shareholders were to approve the investorrsquos proposal

Further a shareholder-approved bylaw generally can limit the power of the board to later amend or repeal it See MBCA sect 1020(b)(2) Thus if Megarsquos shareholders approved the bylaw

33

Corporations Analysis

provision proposed by the investor Megarsquos board could not repeal the provision because it includes a ldquono board repealrdquo clause

The revision to the MBCA in 2009 dealing with shareholder proxy access does not change this conclusion That revision specifies that a shareholder-approved bylaw dealing with director nominations may not limit the boardrsquos power to amend add or repeal ldquoany procedure or condition to such a bylaw in order to provide for a reasonable practicable and orderly processrdquo MBCA sect 206(d) Thus according to the revision if shareholders approve a bylaw amendment that limits further board changes the board would nonetheless retain the power to ldquotinkerrdquo with the bylaw to safeguard the voting process but could not repeal the shareholder-approved bylaw The Official Comment to MBCA sect 206(d) makes clear that the revision is ldquonot intended to allow the board of directors to frustrate the purpose of the shareholder-adopted proxy access provisionrdquo Thus if Megarsquos shareholders were to approve the bylaw provision proposed by the investor Megarsquos board could only amend the provision regarding its procedures or conditions in a manner consistent with its purpose of permitting proxy access for Megarsquos shareholders

[NOTE The boardrsquos attempted interference with a shareholder voting initiative may also have been a violation of the directorsrsquo fiduciary duties See Blasius Indus Inc v Atlas Corp 564 A2d 651 (Del Ch 1988) (finding that directors breached their fiduciary duties by amending bylaws and expanding size of board to thwart insurgentrsquos plan to amend bylaws and seat a majority of new directors) The call however asks examinees to consider whether shareholders or the board have ldquoprecedencerdquo over amending the corporate bylaws Thus an examineersquos answer should be framed in terms of ldquopowerrdquo and not ldquodutyrdquo]

Point Three (40) The investor need not make a demand on the board if the investor states a direct claim such as an allegation that the board interfered with the investorrsquos right to amend the bylaws But the investor must make a demand on the board if the investor states a derivative claim (on behalf of the corporation) such as an allegation that the directors sought to entrench themselves by interfering with the proposed proxy access

The MBCA generally requires that shareholders make a demand on the board of directors before initiation of a derivative suit MBCA sect 742 (shareholder may not bring derivative proceeding until written demand has been made on corporation and 90 days have expired) A derivative suit is essentially two suits in one where the plaintiff-shareholder seeks to bring on behalf of the corporation a claim that vindicates corporate rights usually based on violation of fiduciary duties PALMITER supra sect 1811 (6th ed 2009) The demand permits the board to investigate the situation identified by the shareholder and take suitable action No demand on the board is required however if the shareholder brings a direct suit to vindicate the shareholderrsquos own rights not those of the corporation

Is the suit brought by the investor derivative or direct The MBCA defines a ldquoderivative proceedingrdquo as one brought ldquoin the right of a domestic corporationrdquo MBCA sect 740(1) Thus the answer to how the investorrsquos suit should be characterized turns on what rights the investor seeks to vindicate If the investor frames its claim as one of fiduciary breach by directorsmdashfor example for failing to become adequately informed about voting procedures or for seeking to entrench themselves in office by manipulating the voting structure to avoid a shareholder insurgencymdashthen the suit is ldquoderivativerdquo and the investor must make a demand on the board See MBCA Ch 7 Subch D Introductory Comment (ldquothe derivative suit has historically been the principal method of challenging allegedly illegal action by managementrdquo)

34

Corporations Analysis

If however the investor frames its claim as one to vindicate shareholder rights the suit is direct and no demand is required For many courts the direct-derivative question turns on who is injured and who is to receive the relief sought by the plaintiff-shareholders See Tooley v Donaldson Lufkin amp Jenrette Inc 845 A2d 1031 (Del 2004) (characterizing a merger-delay claim as direct because delay of merger only harmed shareholders not corporation) Thus if the investor claims that managementrsquos refusal to include its proposed bylaw amendment in the corporationrsquos proxy materials violates its shareholder rights to initiate corporate governance reforms the suit will be direct Courts have not questioned the ability of shareholders to bring direct suits challenging board action to exclude their proposed bylaw amendments from the corporationrsquos proxy materials See JANA Master Fund Ltd v CNET Networks Inc 954 A2d 335 (Del Ch 2008) (upholding shareholderrsquos direct challenge to boardrsquos interpretation of advance-notice bylaw) Chesapeake Corp v Shore 771 A2d 293 (Del Ch 2000) (upholding shareholderrsquos direct challenge to actions by board that effectively prevented it from proposing bylaw amendments in contest for control)

Is the way that the investor frames its claim conclusive Courts have permitted shareholder-plaintiffs to challenge a transaction in a direct suit even though the same transaction could also be challenged as a fiduciary breach See Eisenberg v Flying Tiger Line Inc 451 F2d 267 (2d Cir 1971) (permitting direct suit challenging a corporate reorganization as a dilution of shareholder voting power even though reorganization may have involved conflicts of interest and thus constituted a fiduciary breach) Thus the investorrsquos choice to pursue a claim challenging the legality of managementrsquos decision to exclude the investorrsquos proposal from the corporationrsquos proxy materialsmdashrather than a possible breach of fiduciary dutymdashis likely to be respected See 3 COX amp HAZEN supra sect 153 (describing situations in which a claim can be framed as derivative or direct)

[NOTE Some issues under Delaware corporate law regarding pre-suit demand are not relevant here For example whether the Mega directors are independent and disinterested is not relevant to the MBCA requirement of a pre-suit demand As the Official Comment to MBCA sect 742 points out the MBCArsquos requirement of ldquouniversal demandrdquo gives the board ldquothe opportunity to reexamine the act complained of in the light of a potential lawsuit and take corrective actionrdquo even when the directors might be non-independent or have conflicts of interest

Nor is it relevant to the MBCA pre-suit demand requirement that the statutory 90-day waiting period may be onerous The first paragraph of MBCA sect 742 requires a pre-suit demand without exception the second paragraph of the section imposes a 90-day waiting period before a derivative suit may be brought which can be shortened if the board rejects the demand or ldquoirreparable injury to the corporation would result by waiting for the expiration of the 90-day periodrdquo The call as written asks only whether a pre-suit demand should be made and does not ask examinees to address whether the post-demand waiting period should be shortened under the ldquoirreparable injuryrdquo standard]

35

National Conference of Bar Examiners 302 South Bedford Street | Madison WI 53703-3622 Phone 608-280-8550 | Fax 608-280-8552 | TDD 608-661-1275

wwwncbexorg e-mail contactncbexorg

  • Preface
  • Description of the MEE
  • Instructions
  • July 2014 Questions
    • CRIMINAL LAW AND PROCEDURE QUESTION
    • CONTRACTS QUESTION
    • FAMILY LAW QUESTION
    • FEDERAL CIVIL PROCEDURE QUESTION
    • EVIDENCE QUESTION
    • CORPORATIONS QUESTION
      • July 2014 Analyses
        • CRIMINAL LAW AND PROCEDURE ANALYSIS
        • CONTRACTS ANALYSIS
        • FAMILY LAW ANALYSIS
        • FEDERAL CIVIL PROCEDURE ANALYSIS
        • EVIDENCE ANALYSIS
        • CORPORATIONS ANALYSIS
            • ltlt13 ASCII85EncodePages false13 AllowTransparency false13 AutoPositionEPSFiles true13 AutoRotatePages None13 Binding Left13 CalGrayProfile (Dot Gain 20)13 CalRGBProfile (sRGB IEC61966-21)13 CalCMYKProfile (US Web Coated 050SWOP051 v2)13 sRGBProfile (sRGB IEC61966-21)13 CannotEmbedFontPolicy Error13 CompatibilityLevel 1413 CompressObjects Tags13 CompressPages true13 ConvertImagesToIndexed true13 PassThroughJPEGImages true13 CreateJobTicket false13 DefaultRenderingIntent Default13 DetectBlends true13 DetectCurves 0000013 ColorConversionStrategy CMYK13 DoThumbnails false13 EmbedAllFonts true13 EmbedOpenType false13 ParseICCProfilesInComments true13 EmbedJobOptions true13 DSCReportingLevel 013 EmitDSCWarnings false13 EndPage -113 ImageMemory 104857613 LockDistillerParams false13 MaxSubsetPct 10013 Optimize true13 OPM 113 ParseDSCComments true13 ParseDSCCommentsForDocInfo true13 PreserveCopyPage true13 PreserveDICMYKValues true13 PreserveEPSInfo true13 PreserveFlatness true13 PreserveHalftoneInfo false13 PreserveOPIComments true13 PreserveOverprintSettings true13 StartPage 113 SubsetFonts true13 TransferFunctionInfo Apply13 UCRandBGInfo Preserve13 UsePrologue false13 ColorSettingsFile ()13 AlwaysEmbed [ true13 ]13 NeverEmbed [ true13 ]13 AntiAliasColorImages false13 CropColorImages true13 ColorImageMinResolution 30013 ColorImageMinResolutionPolicy OK13 DownsampleColorImages true13 ColorImageDownsampleType Bicubic13 ColorImageResolution 30013 ColorImageDepth -113 ColorImageMinDownsampleDepth 113 ColorImageDownsampleThreshold 15000013 EncodeColorImages true13 ColorImageFilter DCTEncode13 AutoFilterColorImages true13 ColorImageAutoFilterStrategy JPEG13 ColorACSImageDict ltlt13 QFactor 01513 HSamples [1 1 1 1] VSamples [1 1 1 1]13 gtgt13 ColorImageDict ltlt13 QFactor 01513 HSamples [1 1 1 1] VSamples [1 1 1 1]13 gtgt13 JPEG2000ColorACSImageDict ltlt13 TileWidth 25613 TileHeight 25613 Quality 3013 gtgt13 JPEG2000ColorImageDict ltlt13 TileWidth 25613 TileHeight 25613 Quality 3013 gtgt13 AntiAliasGrayImages false13 CropGrayImages true13 GrayImageMinResolution 30013 GrayImageMinResolutionPolicy OK13 DownsampleGrayImages true13 GrayImageDownsampleType Bicubic13 GrayImageResolution 30013 GrayImageDepth -113 GrayImageMinDownsampleDepth 213 GrayImageDownsampleThreshold 15000013 EncodeGrayImages true13 GrayImageFilter DCTEncode13 AutoFilterGrayImages true13 GrayImageAutoFilterStrategy JPEG13 GrayACSImageDict ltlt13 QFactor 01513 HSamples [1 1 1 1] VSamples [1 1 1 1]13 gtgt13 GrayImageDict ltlt13 QFactor 01513 HSamples [1 1 1 1] VSamples [1 1 1 1]13 gtgt13 JPEG2000GrayACSImageDict ltlt13 TileWidth 25613 TileHeight 25613 Quality 3013 gtgt13 JPEG2000GrayImageDict ltlt13 TileWidth 25613 TileHeight 25613 Quality 3013 gtgt13 AntiAliasMonoImages false13 CropMonoImages true13 MonoImageMinResolution 120013 MonoImageMinResolutionPolicy OK13 DownsampleMonoImages true13 MonoImageDownsampleType Bicubic13 MonoImageResolution 120013 MonoImageDepth -113 MonoImageDownsampleThreshold 15000013 EncodeMonoImages true13 MonoImageFilter CCITTFaxEncode13 MonoImageDict ltlt13 K -113 gtgt13 AllowPSXObjects false13 CheckCompliance [13 None13 ]13 PDFX1aCheck false13 PDFX3Check false13 PDFXCompliantPDFOnly false13 PDFXNoTrimBoxError true13 PDFXTrimBoxToMediaBoxOffset [13 00000013 00000013 00000013 00000013 ]13 PDFXSetBleedBoxToMediaBox true13 PDFXBleedBoxToTrimBoxOffset [13 00000013 00000013 00000013 00000013 ]13 PDFXOutputIntentProfile ()13 PDFXOutputConditionIdentifier ()13 PDFXOutputCondition ()13 PDFXRegistryName ()13 PDFXTrapped False1313 CreateJDFFile false13 Description ltlt13 ARA 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 BGR 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 CHS ltFEFF4f7f75288fd94e9b8bbe5b9a521b5efa7684002000410064006f006200650020005000440046002065876863900275284e8e9ad88d2891cf76845370524d53705237300260a853ef4ee54f7f75280020004100630072006f0062006100740020548c002000410064006f00620065002000520065006100640065007200200035002e003000204ee553ca66f49ad87248672c676562535f00521b5efa768400200050004400460020658768633002gt13 CHT ltFEFF4f7f752890194e9b8a2d7f6e5efa7acb7684002000410064006f006200650020005000440046002065874ef69069752865bc9ad854c18cea76845370524d5370523786557406300260a853ef4ee54f7f75280020004100630072006f0062006100740020548c002000410064006f00620065002000520065006100640065007200200035002e003000204ee553ca66f49ad87248672c4f86958b555f5df25efa7acb76840020005000440046002065874ef63002gt13 CZE 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 DAN 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 DEU ltFEFF00560065007200770065006e00640065006e0020005300690065002000640069006500730065002000450069006e007300740065006c006c0075006e00670065006e0020007a0075006d002000450072007300740065006c006c0065006e00200076006f006e002000410064006f006200650020005000440046002d0044006f006b0075006d0065006e00740065006e002c00200076006f006e002000640065006e0065006e002000530069006500200068006f006300680077006500720074006900670065002000500072006500700072006500730073002d0044007200750063006b0065002000650072007a0065007500670065006e0020006d00f60063006800740065006e002e002000450072007300740065006c006c007400650020005000440046002d0044006f006b0075006d0065006e007400650020006b00f6006e006e0065006e0020006d006900740020004100630072006f00620061007400200075006e0064002000410064006f00620065002000520065006100640065007200200035002e00300020006f0064006500720020006800f600680065007200200067006500f600660066006e00650074002000770065007200640065006e002egt13 ESP 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 ETI ltFEFF004b00610073007500740061006700650020006e0065006900640020007300e4007400740065006900640020006b00760061006c006900740065006500740073006500200074007200fc006b006900650065006c007300650020007000720069006e00740069006d0069007300650020006a0061006f006b007300200073006f00620069006c0069006b0065002000410064006f006200650020005000440046002d0064006f006b0075006d0065006e00740069006400650020006c006f006f006d006900730065006b0073002e00200020004c006f006f0064007500640020005000440046002d0064006f006b0075006d0065006e00740065002000730061006100740065002000610076006100640061002000700072006f006700720061006d006d006900640065006700610020004100630072006f0062006100740020006e0069006e0067002000410064006f00620065002000520065006100640065007200200035002e00300020006a00610020007500750065006d006100740065002000760065007200730069006f006f006e00690064006500670061002e000d000agt13 FRA ltFEFF005500740069006c006900730065007a00200063006500730020006f007000740069006f006e00730020006100660069006e00200064006500200063007200e900650072002000640065007300200064006f00630075006d0065006e00740073002000410064006f00620065002000500044004600200070006f0075007200200075006e00650020007100750061006c0069007400e90020006400270069006d007000720065007300730069006f006e00200070007200e9007000720065007300730065002e0020004c0065007300200064006f00630075006d0065006e00740073002000500044004600200063007200e900e90073002000700065007500760065006e0074002000ea0074007200650020006f007500760065007200740073002000640061006e00730020004100630072006f006200610074002c002000610069006e00730069002000710075002700410064006f00620065002000520065006100640065007200200035002e0030002000650074002000760065007200730069006f006e007300200075006c007400e90072006900650075007200650073002egt13 GRE 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 HEB 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 HRV (Za stvaranje Adobe PDF dokumenata najpogodnijih za visokokvalitetni ispis prije tiskanja koristite ove postavke Stvoreni PDF dokumenti mogu se otvoriti Acrobat i Adobe Reader 50 i kasnijim verzijama)13 HUN 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 ITA 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 JPN ltFEFF9ad854c18cea306a30d730ea30d730ec30b951fa529b7528002000410064006f0062006500200050004400460020658766f8306e4f5c6210306b4f7f75283057307e305930023053306e8a2d5b9a30674f5c62103055308c305f0020005000440046002030d530a130a430eb306f3001004100630072006f0062006100740020304a30883073002000410064006f00620065002000520065006100640065007200200035002e003000204ee5964d3067958b304f30533068304c3067304d307e305930023053306e8a2d5b9a306b306f30d530a930f330c8306e57cb30818fbc307f304c5fc59808306730593002gt13 KOR ltFEFFc7740020c124c815c7440020c0acc6a9d558c5ec0020ace0d488c9c80020c2dcd5d80020c778c1c4c5d00020ac00c7a50020c801d569d55c002000410064006f0062006500200050004400460020bb38c11cb97c0020c791c131d569b2c8b2e4002e0020c774b807ac8c0020c791c131b41c00200050004400460020bb38c11cb2940020004100630072006f0062006100740020bc0f002000410064006f00620065002000520065006100640065007200200035002e00300020c774c0c1c5d0c11c0020c5f40020c2180020c788c2b5b2c8b2e4002egt13 LTH 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 LVI 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 NLD (Gebruik deze instellingen om Adobe PDF-documenten te maken die zijn geoptimaliseerd voor prepress-afdrukken van hoge kwaliteit De gemaakte PDF-documenten kunnen worden geopend met Acrobat en Adobe Reader 50 en hoger)13 NOR ltFEFF004200720075006b00200064006900730073006500200069006e006e007300740069006c006c0069006e00670065006e0065002000740069006c002000e50020006f0070007000720065007400740065002000410064006f006200650020005000440046002d0064006f006b0075006d0065006e00740065007200200073006f006d00200065007200200062006500730074002000650067006e0065007400200066006f00720020006600f80072007400720079006b006b0073007500740073006b00720069006600740020006100760020006800f800790020006b00760061006c0069007400650074002e0020005000440046002d0064006f006b0075006d0065006e00740065006e00650020006b0061006e002000e50070006e00650073002000690020004100630072006f00620061007400200065006c006c00650072002000410064006f00620065002000520065006100640065007200200035002e003000200065006c006c00650072002000730065006e006500720065002egt13 POL 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 PTB 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 RUM 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 RUS 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 SKY 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 SLV 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 SUO 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 SVE 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 TUR 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 UKR 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 ENU (Use these settings to create Adobe PDF documents best suited for high-quality prepress printing Created PDF documents can be opened with Acrobat and Adobe Reader 50 and later)13 gtgt13 Namespace [13 (Adobe)13 (Common)13 (10)13 ]13 OtherNamespaces [13 ltlt13 AsReaderSpreads false13 CropImagesToFrames true13 ErrorControl WarnAndContinue13 FlattenerIgnoreSpreadOverrides false13 IncludeGuidesGrids false13 IncludeNonPrinting false13 IncludeSlug false13 Namespace [13 (Adobe)13 (InDesign)13 (40)13 ]13 OmitPlacedBitmaps false13 OmitPlacedEPS false13 OmitPlacedPDF false13 SimulateOverprint Legacy13 gtgt13 ltlt13 AddBleedMarks false13 AddColorBars false13 AddCropMarks false13 AddPageInfo false13 AddRegMarks false13 ConvertColors ConvertToCMYK13 DestinationProfileName ()13 DestinationProfileSelector DocumentCMYK13 Downsample16BitImages true13 FlattenerPreset ltlt13 PresetSelector MediumResolution13 gtgt13 FormElements false13 GenerateStructure false13 IncludeBookmarks false13 IncludeHyperlinks false13 IncludeInteractive false13 IncludeLayers false13 IncludeProfiles false13 MultimediaHandling UseObjectSettings13 Namespace [13 (Adobe)13 (CreativeSuite)13 (20)13 ]13 PDFXOutputIntentProfileSelector DocumentCMYK13 PreserveEditing true13 UntaggedCMYKHandling LeaveUntagged13 UntaggedRGBHandling UseDocumentProfile13 UseDocumentBleed false13 gtgt13 ]13gtgt setdistillerparams13ltlt13 HWResolution [2400 2400]13 PageSize [612000 792000]13gtgt setpagedevice13

Page 25: July 2014 MEE Questions and Analyses - NCBE...This publication includes the questions and analyses from the July 2014 MEE. (In the actual test, the questions are simply numbered rather

Federal Civil Procedure Analysis

Here the court should find that the logging company meets this test First the logging company has a strong interest in the property or transaction that is the subject of this action The USFS has accepted the logging companyrsquos bid and the logging company is merely awaiting issuance of a logging permit to begin logging The nonprofit organization is seeking to prevent this logging The logging company therefore has a strong direct and substantial interest in the subject matter of the lawsuit and in having its winning bid honored and a logging permit issued See eg Kleissler v US Forest Serv 157 F3d 964 972 (3d Cir 1998) (stating that ldquo[t]imber companies have direct and substantial interests in a lawsuit aimed at halting loggingrdquo) see also Natural Resources Defense Council v US Nuclear Regulatory Commrsquon 578 F2d 1341 1343ndash 44 (10th Cir 1978) (holding that applicants whose license renewals were pending had Rule 24(a)(2) interests where the lawsuit sought to halt the license-issuing process pending preparation of environmental impact statements) See generally 7C WRIGHT ET AL supra sect 19081 at 309 (ldquoIf there is a direct substantial legally protectable interest in the proceedings it is clear that this requirement of the rule is satisfiedrdquo) Second the logging companyrsquos interest in receiving a logging permit may well be impaired as a practical matter by the outcome of the lawsuit If the USFS loses the lawsuit it will have to prepare an environmental impact statement before issuing the logging companyrsquos permit This will at a minimum delay the logging companyrsquos ability to exercise its rights and may in the long r un mean that no logging permit is ever issued Intervention of right is not limited to those that would be legally bound as a matter of preclusion doctrine Id sect 19082 at 368 Rather ldquo[t]he rule is satisfied whenever disposition of the present action would put the movant at a practical disadvantage in protecting its interestrdquo Id sect 19082 at 369 Here that condition is easily satisfied See Kleissler 157 F3d at 972 (ldquoTimber companies have direct and substantial interests in a lawsuit aimed at halting logging rdquo)

Given that the logging company has an interest that may be impaired by disposition of the action it should be allowed to intervene unless the court is persuaded that the USFS adequately represents the logging companyrsquos interest See Rule 24(a)(2) 7C WRIGHT ET AL supra sect 1909 Here it could be argued that the USFS adequately represents the logging companyrsquos interest because the USFS presumably wants the court to uphold its development plan and allow it to proceed with issuance of the logging permit which is the same relief that the logging company would seek However whether representation is truly adequate depends upon ldquo[a] discriminating appraisal of the circumstancesrdquo 7C WRIGHT ET AL supra sect 1909 at 440 Although both the government and the logging company wish to avoid the preparation of an environmental impact statement their interests are distinct The USFSrsquos interest is proper management of the national forest system while the logging companyrsquos interest is making a profit from logging the 5000-acre tract The USFSrsquos handling of the litigation is likely to be affected by a variety of policy concerns and political considerations that have nothing to do with the logging companyrsquos purely economic interest in securing the right to cut trees in the Scenic National Forest See eg Kleissler 157 F3d at 973ndash74 (ldquo[T]he government represents numerous complex and conflicting interests in matters of this nature The straightforward business interests asserted by intervenors here may become lost in the thicket of sometimes inconsistent governmental policiesrdquo)

[NOTES (1) Examinees who mistakenly analyze the logging companyrsquos case for joinder under the related but incorrect Rule 19 ldquoRequired Joinder of Partiesrdquo may receive credit Rule 19 allows existing parties to demand joinder of non-parties (or seek dismissal of the case if they canrsquot get it) There is a close relationship between Rule 24 and Rule 19 and both contain a similar standard for determining when ldquointerestedrdquo third parties are ldquoentitledrdquo or ldquorequiredrdquo to be in the lawsuit Indeed the two prongs of the Rule 24 intervention test that are discussed above

23

Federal Civil Procedure Analysis

are nearly identical to the two prongs of the Rule 19(a) required joinder test Examinees who discuss and apply the test should receive credit even if they cite Rule 19 rather than Rule 24

(2) Examinees may discuss permissive joinder Although permissive joinder is a possibility here the question asks only whether the logging company can join the action as a matter of right and a permissive joinder analysis is not responsive to the question To the extent an examinee discusses permissive joinder the analysis will focus on whether the logging company ldquohas a claim or defense that shares with the main action a common question of law or factrdquo FED R CIV P 24(b)(1)(B) The district court also ldquomust consider whether the intervention will unduly delay or prejudice the adjudication of the original partiesrsquo rightsrdquo FED R CIV P 24(b)(3) On our facts the logging companyrsquos claim for the issuance of a logging permit would certainly share common questions of law and fact with the USFSrsquos defense against the nonprofitrsquos claim There are no facts suggesting that the logging companyrsquos presence would unduly delay or otherwise prejudice adjudication of the original action Thus the district court would have discretion to permit the logging company to intervene even if it denied intervention of right]

Point Two(a) (25) The nonprofit organization could seek and would likely obtain a temporary restraining order to stop the USFS from issuing a logging permit pending a hearing on an application for a preliminary injunction

The first type of interim relief the nonprofit could seek to stop the USFS from issuing a logging permit to the logging company is a temporary restraining order (TRO) prohibiting the USFS from issuing the logging permit A TRO can be issued without notice to the adverse party but only in limited circumstances and only for a limited time FED R CIV P 65(b) To secure a TRO without notice the nonprofit would need to submit an affidavit containing specific facts that demonstrate a risk of ldquoimmediate and irreparable injuryrdquo if a permit is issued FED R CIV P 65(b)(1) In deciding whether to grant a TRO courts will also consider the same factors that are relevant in deciding whether to grant a preliminary injunction (eg the moving partyrsquos likelihood of success on the merits the balance of hardships and the public interest) See Point Two(b) infra The TRO would last only long enough for the court to consider and resolve a request by the nonprofit for a preliminary injunction but no longer than 14 days (unless the court extends it for good cause or the adverse party consents to an extension) In addition bond is required

Here the court is likely to grant the nonprofitrsquos request The nonprofit could plausibly claim that cutting down 5000 acres of old-growth forest in an area that is home to the highest concentration of wildlife in the western United States would have ldquoan immediate and irreparablerdquo adverse impact on the environment and cause irreparable harm to the nonprofitrsquos interest in preserving and protecting natural resources including wildlife habitat

Point Two(b) (25) The nonprofit could also seek and would likely obtain a preliminary injunction to stop the USFS which is likely to be granted if the nonprofitrsquos claim that the USFS violated NEPA has a strong basis in fact and law

Because the TRO would be temporary the nonprofit would need to move for a preliminary injunction to prevent the USFS from issuing a logging permit throughout the pendency of the litigation Preliminary injunctions are injunctions that seek to ldquoprotect [the] plaintiff from

24

Federal Civil Procedure Analysis

irreparable injury and to preserve the courtrsquos power to render a meaningful decision after a trial on the meritsrdquo 11A CHARLES ALAN WRIGHT ET AL FEDERAL PRACTICE AND PROCEDURE sect 2947 at 112 (2013) Rule 65 of the Federal Rules of Civil Procedure sets out the procedural requirements for preliminary injunctions Preliminary injunctions may be granted only upon notice to the adverse party FED R CIV P 65(a)(1) and only if the movant ldquogives security in an amount that the court considers proper to pay the costs and damages sustained by any party found to have been wrongfully enjoined or restrainedrdquo FED R CIV P 65(c)

While Rule 65 sets out the procedural requirements for preliminary injunctive relief it does not specify the substantive grounds upon which it may be granted The courtrsquos discretion in ruling upon a motion for a preliminary injunction ldquois exercised in conformity with historic federal equity practicerdquo 11A WRIGHT ET AL supra sect 2947 at 114 The court typically considers four factors

(1) the significance of the threat of irreparable harm to the plaintiff if the injunction is not granted (2) the balance between this harm and the injury that granting the injunction would inflict on the defendant (3) the probability that the plaintiff will succeed on the merits and (4) the public interest

Id sect 2948 at 122ndash24 accord Habitat Educ Center v Bosworth 363 F Supp 2d 1070 1088 (ED Wis 2005) The most important of these factors is the risk of irreparable harm to the plaintiff 11A WRIGHT ET AL supra sect 29481 at 129 If the plaintiff has an adequate remedy at law (eg if money damages can compensate the plaintiff for its loss) then a preliminary injunction will be denied Id sect 29481

Here a court would likely conclude that the potential for environmental damage to the forest creates a significant threat of irreparable harm ldquo[E]nvironmental injury is often irreparable Courts have recognized that logging such as would occur [here] can have longshyterm environmental consequences and thus satisfy the irreparable injury criterionrdquo Habitat Educ Center 363 F Supp 2d at 1089 (citing Idaho Sporting Congress Inc v Alexander 222 F3d 562 569 (9th Cir 2000) (noting that the imminent and continuing logging activities presented ldquoevidence of environmental harm sufficient to tip the balance in favor of injunctive reliefrdquo)) Neighbors of Cuddy Mountain v US Forest Service 137 F3d 1372 1382 (9th Cir 1998) (stating that ldquo[t]he old growth forests plaintiffs seek to protect would if cut take hundreds of years to reproducerdquo) (internal citation omitted)) see also 11C WRIGHT ET AL supra sect 29481 at 151 (noting that ldquoa preliminary injunction has been issued to prevent harm to the environmentrdquo)

The second factor the balance between the harm to the plaintiff and the harm the defendant will suffer if the injunction is issued also appears to support issuance of a preliminary injunction here The USFS will have to wait before it can develop the Scenic National Forest and the logging company may lose money if the delay is prolonged These economic harms could be compensated monetarily if an injunction is issued inappropriately Where ldquoan injunction bond can compensate [the] defendant for any harm the injunction is likely to inflict the balance should be struck in favor of [the] plaintiffrdquo Id sect 29482 at 192 See also Habitat Educ Center 363 F Supp 2d at 1089 (stating that ldquothe relative absence of harmful effects on the Forest Service weighs in favor of granting the injunctionrdquo)

The third factor is the likelihood that the plaintiff will prevail on the merits Although there is limited information concerning the merits of the action the nonprofit alleges that the federal statute (NEPA) requires an environmental impact statement and further states that the USFS created no environmental impact analysis or statement at all Assuming that those

25

Federal Civil Procedure Analysis

allegations are correct it seems plausible to conclude that the nonprofit will be able to show a likelihood of success on the merits

Finally courts deciding whether or not to issue preliminary injunctive relief are to consider the public interest ldquoFocusing on this factor is another way of inquiring whether there are policy considerations that bear on whether the order should issuerdquo 11C WRIGHT ET AL supra sect 29484 at 214 If the court concludes that the nonprofit is likely to succeed on its NEPA claim because the USFS wrongfully failed to conduct an environmental impact assessment it is likely to find that the public interest would be served by restraining the USFS from proceeding with logging in a national forest See Heartwood Inc v US Forest Service 73 F Supp 2d 962 979 (SD Ill 1999) affrsquod on other grounds 230 F3d 947 (7th Cir 2000) (ldquoviolations by federal agencies of NEPArsquos provisions as established by Congress harm the public as well as the environmentrdquo)

Thus a court is very likely to grant a preliminary injunction if it concludes that the nonprofit has a significant likelihood of success on the merits

26

EVIDENCE ANALYSIS (Evidence ID IIA amp C)

ANALYSIS

Legal Problems

(1) Under what circumstances can evidence of prior convictions be used to impeach a witnessrsquos credibility in a civil case

(1)(a) May the inmatersquos credibility be impeached by evidence of a 12-year-old felony drug conviction if he was released from prison 9 years ago

(1)(b) May the inmatersquos credibility be impeached by evidence of an 8-year-old misdemeanor perjury conviction that was punishable by 1 year in jail if he pleaded guilty and was sentenced only to pay a $5000 fine

(1)(c) May the inmatersquos credibility be impeached by evidence of a 7-year-old sexual assault conviction if the inmate is still serving a 10-year prison sentence and the victim was his 13-year-old daughter

(2)(a) May the guardrsquos credibility be impeached by cross-examination regarding specific instances of misconduct (ie lying on his reacutesumeacute) relevant to credibility

(2)(b) May the guardrsquos credibility be impeached by admission of extrinsic evidence (his reacutesumeacute and academic transcript) offered to prove specific instances of misconduct relevant to credibility

DISCUSSION

Summary

Under the Federal Rules of Evidence witnesses can be impeached with evidence of prior convictions andor specific instances of misconduct Whether evidence of prior convictions should be admitted to impeach generally depends on the nature of the crime the amount of time that has passed and (only in criminal cases) whether the ldquowitnessrdquo is the defendant FED R EVID 609(a)

In this civil case evidence of the inmatersquos conviction for distribution of marijuana should be admitted to impeach the inmate because he was convicted of a felony and was released from prison fewer than 10 years ago FED R EVID 609(a)(1) Credibility is critically important in this case because the jury will hear conflicting testimony from the two disputing parties and there were no other eyewitnesses to the altercation Under Rule 609(a)(1) the inmatersquos conviction should be admitted because it has some bearing on his credibility and its probative value is not substantially outweighed by concerns of unfair prejudice confusion or delay Id

Evidence of the inmatersquos misdemeanor conviction for perjury must be admitted because the crime ldquorequired provingmdashor the witnessrsquos admittingmdasha dishonest act or false statementrdquo by the inmate FED R EVID 609(a)(2)

27

Evidence Analysis

Evidence of the inmatersquos felony conviction for sexual assault should be excluded because its probative value is substantially outweighed by the danger of unfair prejudice to the inmate based on the heinous nature of the crime FED R EVID 609(a)(1) In the alternative the judge could limit the evidence relating to this conviction by excluding details of the inmatersquos crime

In all civil (and criminal) cases witnesses can also be impeached with evidence of specific instances of prior misconduct that did not result in a conviction FED R EVID 608(b) Pursuant to Rule 608(b) misconduct probative of untruthfulness can be inquired into on cross-examination but cannot be proved through extrinsic evidence Id Thus the inmatersquos counsel should be permitted to cross-examine the guard regarding the false statement in the guardrsquos reacutesumeacute However extrinsic evidence of the guardrsquos misconduct (ie the guardrsquos authenticated reacutesumeacute and transcript from the local college) should not be admitted even if the guard denies wrongdoing or refuses to answer cross-examination questions about these matters Id

Point One (10) The Federal Rules of Evidence permit impeachment of witnesses with evidence of prior convictions

Whether convictions should be admitted to impeach generally depends on the nature of the crime the amount of time that has passed and (only in criminal cases) whether the ldquowitnessrdquo is the defendant FED R EVID 609(a) Under Rule 609(a) evidence of prior convictions may be admitted for the purpose of ldquoattacking a witnessrsquos character for truthfulnessrdquo Id

There are two basic types of convictions that can be admitted for the purpose of impeachment

(1) convictions for crimes ldquopunishable by death or by imprisonment for more than one yearrdquo (which generally correlates to ldquofeloniesrdquo) FED R EVID 609(a)(1) and (2) convictions ldquofor any crimes regardless of the punishment if the court can readily determine that establishing the elements of the crime required provingmdashor the witnessrsquos admittingmdasha dishonest act or false statementrdquo FED R EVID 609(a)(2)

Pursuant to Rule 609(a)(1) in civil cases the admission of evidence of a felony conviction is ldquosubject to Rule 403 [which says that a court may exclude relevant evidence if its probative value is substantially outweighed by other factors]rdquo FED R EVID 609(a)(1) However Rule 403 does not protect the witness against admission of prior convictions involving dishonestymdashwhich must be admitted by the court FED R EVID 609(a)(2)

Finally Federal Rule of Evidence 609(b) contains the presumption that a conviction that is more than 10 years old or where more than 10 years has passed since the witnessrsquos release from confinement (whichever is later) should not be admitted unless ldquoits probative value supported by specific facts and circumstances substantially outweighs its prejudicial effectrdquo and the proponent has provided the adverse party with reasonable written notice FED R EVID 609(b)

Point One(a) (25) The court should admit evidence of the inmatersquos 12-year-old felony marijuana distribution conviction

The inmatersquos conviction for marijuana distribution was for a felony punishable by imprisonment for more than one year See FED R EVID 609(a)(1) Moreover although the conviction was 12 years ago the 10-year time limit of Rule 609(b) is not exceeded because that time limit runs

28

Evidence Analysis

from the date of either ldquothe witnessrsquos conviction or release from confinement for it whichever is laterrdquo FED R EVID 609(b) Because the inmate served three years in prison he was released from confinement nine years ago

However pursuant to Rule 609(a)(1) the admission of felony convictions to impeach a witness in a civil case is ldquosubject to Rule 403rdquo FED R EVID 609(a)(1) Neither Rule 609(a) nor the advisory committee notes specify which factors courts should consider when balancing the probative value of a conviction against the dangers identified in Rule 403 (which include (1) unfair prejudice (2) confusion of the issues (3) misleading the jury (4) waste of time or undue delay and (5) needless presentation of cumulative evidence) FED R EVID 403

In this case credibility is very important because the evidence consists primarily of the testimony of the disputing parties and there were no other eyewitnesses to the altercation This enhances the probative value of any evidence bearing on the inmatersquos credibility A court is likely to conclude that the inmatersquos prior felony drug conviction is relevant to his credibility See eg United States v Brito 427 F3d 53 64 (1st Cir 2005) (ldquoPrior drug-trafficking crimes are generally viewed as having some bearing on veracityrdquo) Although the probative value of any conviction diminishes with age see eg United States v Brewer 451 F Supp 50 53 (ED Tenn 1978) the inmatersquos ongoing problems with the law suggest that he has continued (and even escalated) his criminal behavior over the past nine years The court should admit this evidence because its probative value is not substantially outweighed by any Rule 403 concerns Specifically any prejudice to the inmate would be slight because the conviction is unrelated to the altercation at issue and the conviction was not for a heinous crime that might inflame the jury

[NOTE Whether an examinee identifies the jury instruction as containing a ldquoconclusiverdquo or ldquomandatoryrdquo presumption is less important than the examineersquos analysis of the constitutional infirmities]

Point One(b) (15) The court must admit evidence of the inmatersquos eight-year-old misdemeanor conviction because perjury is a crime of dishonesty

Rule 609(a)(2) provides that evidence of a criminal conviction ldquomust be admitted if the court can readily determine that establishing the elements of the crime required provingmdashor the witnessrsquos admittingmdasha dishonest act or false statementrdquo FED R EVID 609(a)(2) The inmatersquos conviction for perjury would have necessarily required proving that the inmate engaged in an act of dishonesty This conviction occurred within the past 10 years so it ldquomust be admittedrdquo because in contrast to Rule 609(a)(1) (discussed in Point One(a)) admission under Rule 609(a)(2) is mandatory and not subject to Rule 403

Point One(c) (20) The court should exclude evidence of the inmatersquos seven-year-old felony sexual assault conviction because the probative value of this evidence is substantially outweighed by the danger of unfair prejudice In the alternative the details of the prior conviction could be excluded

The inmatersquos conviction for felony sexual assault was seven years ago and he has not yet been released from incarceration so Rule 609(a) but not 609(b) is applicable here FED R EVID 609(a) This conviction is therefore admissible to impeach the inmate unless its probative value is substantially outweighed by the danger of unfair prejudice or any other Rule 403 concern Id

29

Evidence Analysis

Sex crimes are generally not considered relevant to credibility see Hopkins v State 639 So 2d 1247 1254 (Miss 1993) so the probative value of this conviction is relatively low Moreover the heinous nature of the inmatersquos crime (sexual assault on his daughter) makes the danger of unfair prejudice to the inmate very high Thus the court should exclude evidence of the conviction because it was for a heinous offense that is likely to inflame the jury and it has little bearing on credibility See eg United States v Beahm 664 F2d 414 419 (4th Cir 1981)

As an alternative to excluding this evidence the judge could minimize the unfair prejudice to the inmate by permitting limited cross-examination but refusing to allow specific questions about the nature of the inmatersquos conviction For example a court could limit cross-examination to the fact that the inmate was convicted of a ldquofelonyrdquo or perhaps that he was convicted of a ldquosexual assaultrdquo without identifying the victim However because evidence of the inmatersquos prior convictions can be admitted solely for the purpose of enabling the jury to assess his credibility and because his two earlier convictions should have already been admitted the court should exclude all evidence of the felony sexual assault conviction

Point Two(a) (15) The court should permit the inmatersquos counsel to cross-examine the guard regarding the false statement in his reacutesumeacute because the guardrsquos misconduct bears on his truthfulness

The inmate wishes to cross-examine the guard about his prior dishonest behaviormdashlying on his reacutesumeacutemdashthat did not involve a criminal conviction Rule 608(b) allows witnesses to be cross-examined about specific instances of prior non-conviction misconduct probative of untruthfulness ldquoin order to attack the witnessrsquos character for truthfulnessrdquo FED R EVID 608(b)

The courtrsquos decision to allow cross-examination about the guardrsquos prior dishonest behavior depends on the probative value of such evidence balanced against the danger of unfair prejudice to the guard or any other Rule 403 concern FED R EVID 403 Here the guardrsquos false statement on his reacutesumeacute that he obtained a degree in Criminal Justice is highly probative of his untruthfulness because it grossly misrepresents his actual academic record was made recently and was made with the intent to deceive Because the probative value of this evidence is very strong and is not substantially outweighed by any Rule 403 concerns cross-examination of the guard on this topic should be permitted The court may also consider it fair to permit this cross-examination of the guard on these matters assuming that one or more of the inmatersquos prior convictions have been admitted to impeach his credibility

Point Two(b) (15) The court should exclude extrinsic evidence of the guardrsquos non-conviction misconduct even if the guard denies wrongdoing or refuses to answer questions about the matter

Although Rule 608(b) allows cross-examination about specific instances of prior misconduct probative of untruthfulness ldquoextrinsic evidencerdquo offered to prove such misconduct is not admissible FED R EVID 608(b) The rationale for this rule is that allowing the introduction of extrinsic evidence of prior misconduct by witnesses when these acts are relevant only to the witnessesrsquo truthfulness and not to the main issues in the case would create too great a risk of confusing the jury and unduly delaying the trial The court does not have discretion to admit this extrinsic evidence See eg United States v Elliot 89 F3d 1360 1368 (8th Cir 1996)

30

Evidence Analysis

Here the inmatersquos counsel may cross-examine the guard about the false statement on his reacutesumeacute However the inmatersquos counsel must accept the guardrsquos response Even if the guard denies wrongdoing or refuses to answer questions about the matter the inmatersquos counsel cannot introduce the guardrsquos reacutesumeacute or the transcript from the local college to prove the guardrsquos misconduct

31

CORPORATIONS ANALYSIS (Corporations VA2 IX)

ANALYSIS

Legal Problems

(1) Do shareholders have the authority to amend a corporationrsquos bylaws with respect to director nominations

(2) Do board-approved bylaws on a particular subject here nomination of directors preempt subsequent conflicting bylaw amendments by shareholders

(3) Is a suit challenging both managementrsquos refusal to include the proposed bylaw amendment in Megarsquos proxy statement and the boardrsquos amendment of the bylaws dealing with nomination of directors a direct or derivative suit

DISCUSSION

Summary

The voting and litigation rights of the shareholders of Mega are subject to the provisions of the Model Business Corporations Act (MBCA)

The investorrsquos proposed bylaw provision is not inconsistent with state law Under the MBCA shareholders may amend the bylaws when the amendment deals with a proper matter for the corporationrsquos bylaws such as procedures for nominating directors

The Mega boardrsquos bylaw amendment does not preempt the investorrsquos proposed bylaw provision or the Mega shareholdersrsquo power to approve it While shareholders can limit the boardrsquos power to amend or repeal the bylaws the board cannot limit the shareholdersrsquo power

Whether the investor must make a demand on Megarsquos board depends on how the investor frames its claim If the investor claims a violation of shareholder voting rights the claim is direct and pre-suit demand on the board is not required If on the other hand the investor claims that the directors violated their fiduciary duties by amending the bylaws to entrench themselves the claim is derivative and a pre-suit demand is required

Point One (30) Shareholders may amend the corporationrsquos bylaws where the proposed bylaw provision relates to procedural matters typically included in the bylaws such as the nomination of directors

Internal affairs of the corporation such as the conduct of shareholder meetings and election of directors are subject to the corporate law of the state of incorporation See McDermott Inc v Lewis 531 A2d 206 (Del 1987) (applying law of jurisdiction where corporation was incorporated in case involving voting rights) This statersquos corporate statute is modeled on the MBCA

Under the MBCA ldquoshareholders may amend the corporationrsquos bylawsrdquo MBCA sect 1020(a) Thus the only question is whether the bylaws can specify the procedures for shareholder nomination of directors

32

Corporations Analysis

The MBCA states that the bylaws ldquomay contain any provision that is not inconsistent with law or the articles of incorporationrdquo MBCA sect 206(b) In addition the MBCA was revised in 2009 to address shareholder nomination of directors in public corporations (known as ldquoproxy accessrdquo) and specifies that the bylaws ldquomay contain a requirement that the corporation include in its [proxy materials] one or more individuals nominated by a shareholderrdquo MBCA sect 206(c)(1) see Committee on Corporate Laws ABA Section of Business Law Report on the Roles of Boards of Directors and Shareholders of Publicly Owned Corporations and Changes to the Model Business Corporations ActmdashAdoption of Shareholder Proxy Access Amendments to Chapters 2 and 10 65 BUS LAWYER 1105 (2010)

The inclusion of director-nomination procedures in the bylaws is consistent with practice and is recognized by the Delaware courts whose views on corporate law carry significant weight Typically the procedures for nomination of directors are found in the bylaws See 1 COX amp HAZEN TREATISE ON THE LAW OF CORPORATIONS sect 312 (3d ed 2011) see also 4 FLETCHER CORP FORMS ANN PART III ch 21 (2013) (including sample bylaws that permit nomination of directors by shareholders) The Delaware Supreme Court has confirmed that the bylaws may ldquodefine the process and proceduresrdquo for director elections See CA Inc v AFSCME Employees Pension Plan 953 A2d 227 (Del 2008) (concluding that bylaw amendment requiring reimbursement of election expenses to certain successful shareholder nominators is ldquoproper subjectrdquo under Delaware law)

[NOTE The question of the proper scope of the bylaws can be answered using the more general MBCA sect 206(b) or the 2009 MBCA revision adding sect 206(c)(1) (adopted in CT ME VA) In addition some examinees might raise the point that shareholder proposals may not compel the board to take action such as by including shareholder nominations in the companyrsquos proxy materials on the theory that the ldquobusiness and affairsrdquo of the corporation are to be managed by the board See MBCA sect 801(b) Although shareholders are generally limited to adopting precatory resolutions that recommend or encourage board action this limitation does not apply when shareholders have specific authority to take binding action on their ownmdashsuch as to amend the bylaws]

Point Two (30) Shareholders can amend (or repeal) board-approved bylaws Further shareholders can limit the boardrsquos power to later amend and repeal a shareholder-approved bylaw

Under the MBCA shareholders have the power to amend the bylaws See Point One The board shares this power with the shareholders unless (1) the corporationrsquos articles ldquoreserve that power exclusively to the shareholdersrdquo or (2) ldquothe shareholders in amending repealing or adopting a bylaw expressly provide that the board of directors may not amend repeal or reinstate that bylawrdquo See MBCA sect 1020(b)

Shareholder-approved bylaw provisions can amend or repeal existing bylaw provisions whether originally approved by the board or by shareholders See ALAN R PALMITER CORPORATIONS EXAMPLES AND EXPLANATIONS sect 713 (7th ed 2012) Thus the Mega boardrsquos bylaw amendmentmdashwhich set more demanding thresholds for shareholder nomination of directors than the investorrsquos proposed bylaw provisionmdashwould be superseded (repealed) if Megarsquos shareholders were to approve the investorrsquos proposal

Further a shareholder-approved bylaw generally can limit the power of the board to later amend or repeal it See MBCA sect 1020(b)(2) Thus if Megarsquos shareholders approved the bylaw

33

Corporations Analysis

provision proposed by the investor Megarsquos board could not repeal the provision because it includes a ldquono board repealrdquo clause

The revision to the MBCA in 2009 dealing with shareholder proxy access does not change this conclusion That revision specifies that a shareholder-approved bylaw dealing with director nominations may not limit the boardrsquos power to amend add or repeal ldquoany procedure or condition to such a bylaw in order to provide for a reasonable practicable and orderly processrdquo MBCA sect 206(d) Thus according to the revision if shareholders approve a bylaw amendment that limits further board changes the board would nonetheless retain the power to ldquotinkerrdquo with the bylaw to safeguard the voting process but could not repeal the shareholder-approved bylaw The Official Comment to MBCA sect 206(d) makes clear that the revision is ldquonot intended to allow the board of directors to frustrate the purpose of the shareholder-adopted proxy access provisionrdquo Thus if Megarsquos shareholders were to approve the bylaw provision proposed by the investor Megarsquos board could only amend the provision regarding its procedures or conditions in a manner consistent with its purpose of permitting proxy access for Megarsquos shareholders

[NOTE The boardrsquos attempted interference with a shareholder voting initiative may also have been a violation of the directorsrsquo fiduciary duties See Blasius Indus Inc v Atlas Corp 564 A2d 651 (Del Ch 1988) (finding that directors breached their fiduciary duties by amending bylaws and expanding size of board to thwart insurgentrsquos plan to amend bylaws and seat a majority of new directors) The call however asks examinees to consider whether shareholders or the board have ldquoprecedencerdquo over amending the corporate bylaws Thus an examineersquos answer should be framed in terms of ldquopowerrdquo and not ldquodutyrdquo]

Point Three (40) The investor need not make a demand on the board if the investor states a direct claim such as an allegation that the board interfered with the investorrsquos right to amend the bylaws But the investor must make a demand on the board if the investor states a derivative claim (on behalf of the corporation) such as an allegation that the directors sought to entrench themselves by interfering with the proposed proxy access

The MBCA generally requires that shareholders make a demand on the board of directors before initiation of a derivative suit MBCA sect 742 (shareholder may not bring derivative proceeding until written demand has been made on corporation and 90 days have expired) A derivative suit is essentially two suits in one where the plaintiff-shareholder seeks to bring on behalf of the corporation a claim that vindicates corporate rights usually based on violation of fiduciary duties PALMITER supra sect 1811 (6th ed 2009) The demand permits the board to investigate the situation identified by the shareholder and take suitable action No demand on the board is required however if the shareholder brings a direct suit to vindicate the shareholderrsquos own rights not those of the corporation

Is the suit brought by the investor derivative or direct The MBCA defines a ldquoderivative proceedingrdquo as one brought ldquoin the right of a domestic corporationrdquo MBCA sect 740(1) Thus the answer to how the investorrsquos suit should be characterized turns on what rights the investor seeks to vindicate If the investor frames its claim as one of fiduciary breach by directorsmdashfor example for failing to become adequately informed about voting procedures or for seeking to entrench themselves in office by manipulating the voting structure to avoid a shareholder insurgencymdashthen the suit is ldquoderivativerdquo and the investor must make a demand on the board See MBCA Ch 7 Subch D Introductory Comment (ldquothe derivative suit has historically been the principal method of challenging allegedly illegal action by managementrdquo)

34

Corporations Analysis

If however the investor frames its claim as one to vindicate shareholder rights the suit is direct and no demand is required For many courts the direct-derivative question turns on who is injured and who is to receive the relief sought by the plaintiff-shareholders See Tooley v Donaldson Lufkin amp Jenrette Inc 845 A2d 1031 (Del 2004) (characterizing a merger-delay claim as direct because delay of merger only harmed shareholders not corporation) Thus if the investor claims that managementrsquos refusal to include its proposed bylaw amendment in the corporationrsquos proxy materials violates its shareholder rights to initiate corporate governance reforms the suit will be direct Courts have not questioned the ability of shareholders to bring direct suits challenging board action to exclude their proposed bylaw amendments from the corporationrsquos proxy materials See JANA Master Fund Ltd v CNET Networks Inc 954 A2d 335 (Del Ch 2008) (upholding shareholderrsquos direct challenge to boardrsquos interpretation of advance-notice bylaw) Chesapeake Corp v Shore 771 A2d 293 (Del Ch 2000) (upholding shareholderrsquos direct challenge to actions by board that effectively prevented it from proposing bylaw amendments in contest for control)

Is the way that the investor frames its claim conclusive Courts have permitted shareholder-plaintiffs to challenge a transaction in a direct suit even though the same transaction could also be challenged as a fiduciary breach See Eisenberg v Flying Tiger Line Inc 451 F2d 267 (2d Cir 1971) (permitting direct suit challenging a corporate reorganization as a dilution of shareholder voting power even though reorganization may have involved conflicts of interest and thus constituted a fiduciary breach) Thus the investorrsquos choice to pursue a claim challenging the legality of managementrsquos decision to exclude the investorrsquos proposal from the corporationrsquos proxy materialsmdashrather than a possible breach of fiduciary dutymdashis likely to be respected See 3 COX amp HAZEN supra sect 153 (describing situations in which a claim can be framed as derivative or direct)

[NOTE Some issues under Delaware corporate law regarding pre-suit demand are not relevant here For example whether the Mega directors are independent and disinterested is not relevant to the MBCA requirement of a pre-suit demand As the Official Comment to MBCA sect 742 points out the MBCArsquos requirement of ldquouniversal demandrdquo gives the board ldquothe opportunity to reexamine the act complained of in the light of a potential lawsuit and take corrective actionrdquo even when the directors might be non-independent or have conflicts of interest

Nor is it relevant to the MBCA pre-suit demand requirement that the statutory 90-day waiting period may be onerous The first paragraph of MBCA sect 742 requires a pre-suit demand without exception the second paragraph of the section imposes a 90-day waiting period before a derivative suit may be brought which can be shortened if the board rejects the demand or ldquoirreparable injury to the corporation would result by waiting for the expiration of the 90-day periodrdquo The call as written asks only whether a pre-suit demand should be made and does not ask examinees to address whether the post-demand waiting period should be shortened under the ldquoirreparable injuryrdquo standard]

35

National Conference of Bar Examiners 302 South Bedford Street | Madison WI 53703-3622 Phone 608-280-8550 | Fax 608-280-8552 | TDD 608-661-1275

wwwncbexorg e-mail contactncbexorg

  • Preface
  • Description of the MEE
  • Instructions
  • July 2014 Questions
    • CRIMINAL LAW AND PROCEDURE QUESTION
    • CONTRACTS QUESTION
    • FAMILY LAW QUESTION
    • FEDERAL CIVIL PROCEDURE QUESTION
    • EVIDENCE QUESTION
    • CORPORATIONS QUESTION
      • July 2014 Analyses
        • CRIMINAL LAW AND PROCEDURE ANALYSIS
        • CONTRACTS ANALYSIS
        • FAMILY LAW ANALYSIS
        • FEDERAL CIVIL PROCEDURE ANALYSIS
        • EVIDENCE ANALYSIS
        • CORPORATIONS ANALYSIS
            • ltlt13 ASCII85EncodePages false13 AllowTransparency false13 AutoPositionEPSFiles true13 AutoRotatePages None13 Binding Left13 CalGrayProfile (Dot Gain 20)13 CalRGBProfile (sRGB IEC61966-21)13 CalCMYKProfile (US Web Coated 050SWOP051 v2)13 sRGBProfile (sRGB IEC61966-21)13 CannotEmbedFontPolicy Error13 CompatibilityLevel 1413 CompressObjects Tags13 CompressPages true13 ConvertImagesToIndexed true13 PassThroughJPEGImages true13 CreateJobTicket false13 DefaultRenderingIntent Default13 DetectBlends true13 DetectCurves 0000013 ColorConversionStrategy CMYK13 DoThumbnails false13 EmbedAllFonts true13 EmbedOpenType false13 ParseICCProfilesInComments true13 EmbedJobOptions true13 DSCReportingLevel 013 EmitDSCWarnings false13 EndPage -113 ImageMemory 104857613 LockDistillerParams false13 MaxSubsetPct 10013 Optimize true13 OPM 113 ParseDSCComments true13 ParseDSCCommentsForDocInfo true13 PreserveCopyPage true13 PreserveDICMYKValues true13 PreserveEPSInfo true13 PreserveFlatness true13 PreserveHalftoneInfo false13 PreserveOPIComments true13 PreserveOverprintSettings true13 StartPage 113 SubsetFonts true13 TransferFunctionInfo Apply13 UCRandBGInfo Preserve13 UsePrologue false13 ColorSettingsFile ()13 AlwaysEmbed [ true13 ]13 NeverEmbed [ true13 ]13 AntiAliasColorImages false13 CropColorImages true13 ColorImageMinResolution 30013 ColorImageMinResolutionPolicy OK13 DownsampleColorImages true13 ColorImageDownsampleType Bicubic13 ColorImageResolution 30013 ColorImageDepth -113 ColorImageMinDownsampleDepth 113 ColorImageDownsampleThreshold 15000013 EncodeColorImages true13 ColorImageFilter DCTEncode13 AutoFilterColorImages true13 ColorImageAutoFilterStrategy JPEG13 ColorACSImageDict ltlt13 QFactor 01513 HSamples [1 1 1 1] VSamples [1 1 1 1]13 gtgt13 ColorImageDict ltlt13 QFactor 01513 HSamples [1 1 1 1] VSamples [1 1 1 1]13 gtgt13 JPEG2000ColorACSImageDict ltlt13 TileWidth 25613 TileHeight 25613 Quality 3013 gtgt13 JPEG2000ColorImageDict ltlt13 TileWidth 25613 TileHeight 25613 Quality 3013 gtgt13 AntiAliasGrayImages false13 CropGrayImages true13 GrayImageMinResolution 30013 GrayImageMinResolutionPolicy OK13 DownsampleGrayImages true13 GrayImageDownsampleType Bicubic13 GrayImageResolution 30013 GrayImageDepth -113 GrayImageMinDownsampleDepth 213 GrayImageDownsampleThreshold 15000013 EncodeGrayImages true13 GrayImageFilter DCTEncode13 AutoFilterGrayImages true13 GrayImageAutoFilterStrategy JPEG13 GrayACSImageDict ltlt13 QFactor 01513 HSamples [1 1 1 1] VSamples [1 1 1 1]13 gtgt13 GrayImageDict ltlt13 QFactor 01513 HSamples [1 1 1 1] VSamples [1 1 1 1]13 gtgt13 JPEG2000GrayACSImageDict ltlt13 TileWidth 25613 TileHeight 25613 Quality 3013 gtgt13 JPEG2000GrayImageDict ltlt13 TileWidth 25613 TileHeight 25613 Quality 3013 gtgt13 AntiAliasMonoImages false13 CropMonoImages true13 MonoImageMinResolution 120013 MonoImageMinResolutionPolicy OK13 DownsampleMonoImages true13 MonoImageDownsampleType Bicubic13 MonoImageResolution 120013 MonoImageDepth -113 MonoImageDownsampleThreshold 15000013 EncodeMonoImages true13 MonoImageFilter CCITTFaxEncode13 MonoImageDict ltlt13 K -113 gtgt13 AllowPSXObjects false13 CheckCompliance [13 None13 ]13 PDFX1aCheck false13 PDFX3Check false13 PDFXCompliantPDFOnly false13 PDFXNoTrimBoxError true13 PDFXTrimBoxToMediaBoxOffset [13 00000013 00000013 00000013 00000013 ]13 PDFXSetBleedBoxToMediaBox true13 PDFXBleedBoxToTrimBoxOffset [13 00000013 00000013 00000013 00000013 ]13 PDFXOutputIntentProfile ()13 PDFXOutputConditionIdentifier ()13 PDFXOutputCondition ()13 PDFXRegistryName ()13 PDFXTrapped False1313 CreateJDFFile false13 Description ltlt13 ARA 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 BGR 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 CHS ltFEFF4f7f75288fd94e9b8bbe5b9a521b5efa7684002000410064006f006200650020005000440046002065876863900275284e8e9ad88d2891cf76845370524d53705237300260a853ef4ee54f7f75280020004100630072006f0062006100740020548c002000410064006f00620065002000520065006100640065007200200035002e003000204ee553ca66f49ad87248672c676562535f00521b5efa768400200050004400460020658768633002gt13 CHT ltFEFF4f7f752890194e9b8a2d7f6e5efa7acb7684002000410064006f006200650020005000440046002065874ef69069752865bc9ad854c18cea76845370524d5370523786557406300260a853ef4ee54f7f75280020004100630072006f0062006100740020548c002000410064006f00620065002000520065006100640065007200200035002e003000204ee553ca66f49ad87248672c4f86958b555f5df25efa7acb76840020005000440046002065874ef63002gt13 CZE 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 DAN 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 DEU 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 ESP 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 ETI 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 FRA 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 GRE ltFEFF03a703c103b703c303b903bc03bf03c003bf03b903ae03c303c403b5002003b103c503c403ad03c2002003c403b903c2002003c103c503b803bc03af03c303b503b903c2002003b303b903b1002003bd03b1002003b403b703bc03b903bf03c503c103b303ae03c303b503c403b5002003ad03b303b303c103b103c603b1002000410064006f006200650020005000440046002003c003bf03c5002003b503af03bd03b103b9002003ba03b103c42019002003b503be03bf03c703ae03bd002003ba03b103c403ac03bb03bb03b703bb03b1002003b303b903b1002003c003c103bf002d03b503ba03c403c503c003c903c403b903ba03ad03c2002003b503c103b303b103c303af03b503c2002003c503c803b703bb03ae03c2002003c003bf03b903cc03c403b703c403b103c2002e0020002003a403b10020005000440046002003ad03b303b303c103b103c603b1002003c003bf03c5002003ad03c703b503c403b5002003b403b703bc03b903bf03c503c103b303ae03c303b503b9002003bc03c003bf03c103bf03cd03bd002003bd03b1002003b103bd03bf03b903c703c403bf03cd03bd002003bc03b5002003c403bf0020004100630072006f006200610074002c002003c403bf002000410064006f00620065002000520065006100640065007200200035002e0030002003ba03b103b9002003bc03b503c403b103b303b503bd03ad03c303c403b503c103b503c2002003b503ba03b403cc03c303b503b903c2002egt13 HEB 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 HRV (Za stvaranje Adobe PDF dokumenata najpogodnijih za visokokvalitetni ispis prije tiskanja koristite ove postavke Stvoreni PDF dokumenti mogu se otvoriti Acrobat i Adobe Reader 50 i kasnijim verzijama)13 HUN 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 ITA 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 JPN ltFEFF9ad854c18cea306a30d730ea30d730ec30b951fa529b7528002000410064006f0062006500200050004400460020658766f8306e4f5c6210306b4f7f75283057307e305930023053306e8a2d5b9a30674f5c62103055308c305f0020005000440046002030d530a130a430eb306f3001004100630072006f0062006100740020304a30883073002000410064006f00620065002000520065006100640065007200200035002e003000204ee5964d3067958b304f30533068304c3067304d307e305930023053306e8a2d5b9a306b306f30d530a930f330c8306e57cb30818fbc307f304c5fc59808306730593002gt13 KOR ltFEFFc7740020c124c815c7440020c0acc6a9d558c5ec0020ace0d488c9c80020c2dcd5d80020c778c1c4c5d00020ac00c7a50020c801d569d55c002000410064006f0062006500200050004400460020bb38c11cb97c0020c791c131d569b2c8b2e4002e0020c774b807ac8c0020c791c131b41c00200050004400460020bb38c11cb2940020004100630072006f0062006100740020bc0f002000410064006f00620065002000520065006100640065007200200035002e00300020c774c0c1c5d0c11c0020c5f40020c2180020c788c2b5b2c8b2e4002egt13 LTH 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 LVI 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 NLD (Gebruik deze instellingen om Adobe PDF-documenten te maken die zijn geoptimaliseerd voor prepress-afdrukken van hoge kwaliteit De gemaakte PDF-documenten kunnen worden geopend met Acrobat en Adobe Reader 50 en hoger)13 NOR 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 POL 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 PTB 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 RUM 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 RUS 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 SKY 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 SLV 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 SUO 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 SVE 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 TUR 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 UKR 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 ENU (Use these settings to create Adobe PDF documents best suited for high-quality prepress printing Created PDF documents can be opened with Acrobat and Adobe Reader 50 and later)13 gtgt13 Namespace [13 (Adobe)13 (Common)13 (10)13 ]13 OtherNamespaces [13 ltlt13 AsReaderSpreads false13 CropImagesToFrames true13 ErrorControl WarnAndContinue13 FlattenerIgnoreSpreadOverrides false13 IncludeGuidesGrids false13 IncludeNonPrinting false13 IncludeSlug false13 Namespace [13 (Adobe)13 (InDesign)13 (40)13 ]13 OmitPlacedBitmaps false13 OmitPlacedEPS false13 OmitPlacedPDF false13 SimulateOverprint Legacy13 gtgt13 ltlt13 AddBleedMarks false13 AddColorBars false13 AddCropMarks false13 AddPageInfo false13 AddRegMarks false13 ConvertColors ConvertToCMYK13 DestinationProfileName ()13 DestinationProfileSelector DocumentCMYK13 Downsample16BitImages true13 FlattenerPreset ltlt13 PresetSelector MediumResolution13 gtgt13 FormElements false13 GenerateStructure false13 IncludeBookmarks false13 IncludeHyperlinks false13 IncludeInteractive false13 IncludeLayers false13 IncludeProfiles false13 MultimediaHandling UseObjectSettings13 Namespace [13 (Adobe)13 (CreativeSuite)13 (20)13 ]13 PDFXOutputIntentProfileSelector DocumentCMYK13 PreserveEditing true13 UntaggedCMYKHandling LeaveUntagged13 UntaggedRGBHandling UseDocumentProfile13 UseDocumentBleed false13 gtgt13 ]13gtgt setdistillerparams13ltlt13 HWResolution [2400 2400]13 PageSize [612000 792000]13gtgt setpagedevice13

Page 26: July 2014 MEE Questions and Analyses - NCBE...This publication includes the questions and analyses from the July 2014 MEE. (In the actual test, the questions are simply numbered rather

Federal Civil Procedure Analysis

are nearly identical to the two prongs of the Rule 19(a) required joinder test Examinees who discuss and apply the test should receive credit even if they cite Rule 19 rather than Rule 24

(2) Examinees may discuss permissive joinder Although permissive joinder is a possibility here the question asks only whether the logging company can join the action as a matter of right and a permissive joinder analysis is not responsive to the question To the extent an examinee discusses permissive joinder the analysis will focus on whether the logging company ldquohas a claim or defense that shares with the main action a common question of law or factrdquo FED R CIV P 24(b)(1)(B) The district court also ldquomust consider whether the intervention will unduly delay or prejudice the adjudication of the original partiesrsquo rightsrdquo FED R CIV P 24(b)(3) On our facts the logging companyrsquos claim for the issuance of a logging permit would certainly share common questions of law and fact with the USFSrsquos defense against the nonprofitrsquos claim There are no facts suggesting that the logging companyrsquos presence would unduly delay or otherwise prejudice adjudication of the original action Thus the district court would have discretion to permit the logging company to intervene even if it denied intervention of right]

Point Two(a) (25) The nonprofit organization could seek and would likely obtain a temporary restraining order to stop the USFS from issuing a logging permit pending a hearing on an application for a preliminary injunction

The first type of interim relief the nonprofit could seek to stop the USFS from issuing a logging permit to the logging company is a temporary restraining order (TRO) prohibiting the USFS from issuing the logging permit A TRO can be issued without notice to the adverse party but only in limited circumstances and only for a limited time FED R CIV P 65(b) To secure a TRO without notice the nonprofit would need to submit an affidavit containing specific facts that demonstrate a risk of ldquoimmediate and irreparable injuryrdquo if a permit is issued FED R CIV P 65(b)(1) In deciding whether to grant a TRO courts will also consider the same factors that are relevant in deciding whether to grant a preliminary injunction (eg the moving partyrsquos likelihood of success on the merits the balance of hardships and the public interest) See Point Two(b) infra The TRO would last only long enough for the court to consider and resolve a request by the nonprofit for a preliminary injunction but no longer than 14 days (unless the court extends it for good cause or the adverse party consents to an extension) In addition bond is required

Here the court is likely to grant the nonprofitrsquos request The nonprofit could plausibly claim that cutting down 5000 acres of old-growth forest in an area that is home to the highest concentration of wildlife in the western United States would have ldquoan immediate and irreparablerdquo adverse impact on the environment and cause irreparable harm to the nonprofitrsquos interest in preserving and protecting natural resources including wildlife habitat

Point Two(b) (25) The nonprofit could also seek and would likely obtain a preliminary injunction to stop the USFS which is likely to be granted if the nonprofitrsquos claim that the USFS violated NEPA has a strong basis in fact and law

Because the TRO would be temporary the nonprofit would need to move for a preliminary injunction to prevent the USFS from issuing a logging permit throughout the pendency of the litigation Preliminary injunctions are injunctions that seek to ldquoprotect [the] plaintiff from

24

Federal Civil Procedure Analysis

irreparable injury and to preserve the courtrsquos power to render a meaningful decision after a trial on the meritsrdquo 11A CHARLES ALAN WRIGHT ET AL FEDERAL PRACTICE AND PROCEDURE sect 2947 at 112 (2013) Rule 65 of the Federal Rules of Civil Procedure sets out the procedural requirements for preliminary injunctions Preliminary injunctions may be granted only upon notice to the adverse party FED R CIV P 65(a)(1) and only if the movant ldquogives security in an amount that the court considers proper to pay the costs and damages sustained by any party found to have been wrongfully enjoined or restrainedrdquo FED R CIV P 65(c)

While Rule 65 sets out the procedural requirements for preliminary injunctive relief it does not specify the substantive grounds upon which it may be granted The courtrsquos discretion in ruling upon a motion for a preliminary injunction ldquois exercised in conformity with historic federal equity practicerdquo 11A WRIGHT ET AL supra sect 2947 at 114 The court typically considers four factors

(1) the significance of the threat of irreparable harm to the plaintiff if the injunction is not granted (2) the balance between this harm and the injury that granting the injunction would inflict on the defendant (3) the probability that the plaintiff will succeed on the merits and (4) the public interest

Id sect 2948 at 122ndash24 accord Habitat Educ Center v Bosworth 363 F Supp 2d 1070 1088 (ED Wis 2005) The most important of these factors is the risk of irreparable harm to the plaintiff 11A WRIGHT ET AL supra sect 29481 at 129 If the plaintiff has an adequate remedy at law (eg if money damages can compensate the plaintiff for its loss) then a preliminary injunction will be denied Id sect 29481

Here a court would likely conclude that the potential for environmental damage to the forest creates a significant threat of irreparable harm ldquo[E]nvironmental injury is often irreparable Courts have recognized that logging such as would occur [here] can have longshyterm environmental consequences and thus satisfy the irreparable injury criterionrdquo Habitat Educ Center 363 F Supp 2d at 1089 (citing Idaho Sporting Congress Inc v Alexander 222 F3d 562 569 (9th Cir 2000) (noting that the imminent and continuing logging activities presented ldquoevidence of environmental harm sufficient to tip the balance in favor of injunctive reliefrdquo)) Neighbors of Cuddy Mountain v US Forest Service 137 F3d 1372 1382 (9th Cir 1998) (stating that ldquo[t]he old growth forests plaintiffs seek to protect would if cut take hundreds of years to reproducerdquo) (internal citation omitted)) see also 11C WRIGHT ET AL supra sect 29481 at 151 (noting that ldquoa preliminary injunction has been issued to prevent harm to the environmentrdquo)

The second factor the balance between the harm to the plaintiff and the harm the defendant will suffer if the injunction is issued also appears to support issuance of a preliminary injunction here The USFS will have to wait before it can develop the Scenic National Forest and the logging company may lose money if the delay is prolonged These economic harms could be compensated monetarily if an injunction is issued inappropriately Where ldquoan injunction bond can compensate [the] defendant for any harm the injunction is likely to inflict the balance should be struck in favor of [the] plaintiffrdquo Id sect 29482 at 192 See also Habitat Educ Center 363 F Supp 2d at 1089 (stating that ldquothe relative absence of harmful effects on the Forest Service weighs in favor of granting the injunctionrdquo)

The third factor is the likelihood that the plaintiff will prevail on the merits Although there is limited information concerning the merits of the action the nonprofit alleges that the federal statute (NEPA) requires an environmental impact statement and further states that the USFS created no environmental impact analysis or statement at all Assuming that those

25

Federal Civil Procedure Analysis

allegations are correct it seems plausible to conclude that the nonprofit will be able to show a likelihood of success on the merits

Finally courts deciding whether or not to issue preliminary injunctive relief are to consider the public interest ldquoFocusing on this factor is another way of inquiring whether there are policy considerations that bear on whether the order should issuerdquo 11C WRIGHT ET AL supra sect 29484 at 214 If the court concludes that the nonprofit is likely to succeed on its NEPA claim because the USFS wrongfully failed to conduct an environmental impact assessment it is likely to find that the public interest would be served by restraining the USFS from proceeding with logging in a national forest See Heartwood Inc v US Forest Service 73 F Supp 2d 962 979 (SD Ill 1999) affrsquod on other grounds 230 F3d 947 (7th Cir 2000) (ldquoviolations by federal agencies of NEPArsquos provisions as established by Congress harm the public as well as the environmentrdquo)

Thus a court is very likely to grant a preliminary injunction if it concludes that the nonprofit has a significant likelihood of success on the merits

26

EVIDENCE ANALYSIS (Evidence ID IIA amp C)

ANALYSIS

Legal Problems

(1) Under what circumstances can evidence of prior convictions be used to impeach a witnessrsquos credibility in a civil case

(1)(a) May the inmatersquos credibility be impeached by evidence of a 12-year-old felony drug conviction if he was released from prison 9 years ago

(1)(b) May the inmatersquos credibility be impeached by evidence of an 8-year-old misdemeanor perjury conviction that was punishable by 1 year in jail if he pleaded guilty and was sentenced only to pay a $5000 fine

(1)(c) May the inmatersquos credibility be impeached by evidence of a 7-year-old sexual assault conviction if the inmate is still serving a 10-year prison sentence and the victim was his 13-year-old daughter

(2)(a) May the guardrsquos credibility be impeached by cross-examination regarding specific instances of misconduct (ie lying on his reacutesumeacute) relevant to credibility

(2)(b) May the guardrsquos credibility be impeached by admission of extrinsic evidence (his reacutesumeacute and academic transcript) offered to prove specific instances of misconduct relevant to credibility

DISCUSSION

Summary

Under the Federal Rules of Evidence witnesses can be impeached with evidence of prior convictions andor specific instances of misconduct Whether evidence of prior convictions should be admitted to impeach generally depends on the nature of the crime the amount of time that has passed and (only in criminal cases) whether the ldquowitnessrdquo is the defendant FED R EVID 609(a)

In this civil case evidence of the inmatersquos conviction for distribution of marijuana should be admitted to impeach the inmate because he was convicted of a felony and was released from prison fewer than 10 years ago FED R EVID 609(a)(1) Credibility is critically important in this case because the jury will hear conflicting testimony from the two disputing parties and there were no other eyewitnesses to the altercation Under Rule 609(a)(1) the inmatersquos conviction should be admitted because it has some bearing on his credibility and its probative value is not substantially outweighed by concerns of unfair prejudice confusion or delay Id

Evidence of the inmatersquos misdemeanor conviction for perjury must be admitted because the crime ldquorequired provingmdashor the witnessrsquos admittingmdasha dishonest act or false statementrdquo by the inmate FED R EVID 609(a)(2)

27

Evidence Analysis

Evidence of the inmatersquos felony conviction for sexual assault should be excluded because its probative value is substantially outweighed by the danger of unfair prejudice to the inmate based on the heinous nature of the crime FED R EVID 609(a)(1) In the alternative the judge could limit the evidence relating to this conviction by excluding details of the inmatersquos crime

In all civil (and criminal) cases witnesses can also be impeached with evidence of specific instances of prior misconduct that did not result in a conviction FED R EVID 608(b) Pursuant to Rule 608(b) misconduct probative of untruthfulness can be inquired into on cross-examination but cannot be proved through extrinsic evidence Id Thus the inmatersquos counsel should be permitted to cross-examine the guard regarding the false statement in the guardrsquos reacutesumeacute However extrinsic evidence of the guardrsquos misconduct (ie the guardrsquos authenticated reacutesumeacute and transcript from the local college) should not be admitted even if the guard denies wrongdoing or refuses to answer cross-examination questions about these matters Id

Point One (10) The Federal Rules of Evidence permit impeachment of witnesses with evidence of prior convictions

Whether convictions should be admitted to impeach generally depends on the nature of the crime the amount of time that has passed and (only in criminal cases) whether the ldquowitnessrdquo is the defendant FED R EVID 609(a) Under Rule 609(a) evidence of prior convictions may be admitted for the purpose of ldquoattacking a witnessrsquos character for truthfulnessrdquo Id

There are two basic types of convictions that can be admitted for the purpose of impeachment

(1) convictions for crimes ldquopunishable by death or by imprisonment for more than one yearrdquo (which generally correlates to ldquofeloniesrdquo) FED R EVID 609(a)(1) and (2) convictions ldquofor any crimes regardless of the punishment if the court can readily determine that establishing the elements of the crime required provingmdashor the witnessrsquos admittingmdasha dishonest act or false statementrdquo FED R EVID 609(a)(2)

Pursuant to Rule 609(a)(1) in civil cases the admission of evidence of a felony conviction is ldquosubject to Rule 403 [which says that a court may exclude relevant evidence if its probative value is substantially outweighed by other factors]rdquo FED R EVID 609(a)(1) However Rule 403 does not protect the witness against admission of prior convictions involving dishonestymdashwhich must be admitted by the court FED R EVID 609(a)(2)

Finally Federal Rule of Evidence 609(b) contains the presumption that a conviction that is more than 10 years old or where more than 10 years has passed since the witnessrsquos release from confinement (whichever is later) should not be admitted unless ldquoits probative value supported by specific facts and circumstances substantially outweighs its prejudicial effectrdquo and the proponent has provided the adverse party with reasonable written notice FED R EVID 609(b)

Point One(a) (25) The court should admit evidence of the inmatersquos 12-year-old felony marijuana distribution conviction

The inmatersquos conviction for marijuana distribution was for a felony punishable by imprisonment for more than one year See FED R EVID 609(a)(1) Moreover although the conviction was 12 years ago the 10-year time limit of Rule 609(b) is not exceeded because that time limit runs

28

Evidence Analysis

from the date of either ldquothe witnessrsquos conviction or release from confinement for it whichever is laterrdquo FED R EVID 609(b) Because the inmate served three years in prison he was released from confinement nine years ago

However pursuant to Rule 609(a)(1) the admission of felony convictions to impeach a witness in a civil case is ldquosubject to Rule 403rdquo FED R EVID 609(a)(1) Neither Rule 609(a) nor the advisory committee notes specify which factors courts should consider when balancing the probative value of a conviction against the dangers identified in Rule 403 (which include (1) unfair prejudice (2) confusion of the issues (3) misleading the jury (4) waste of time or undue delay and (5) needless presentation of cumulative evidence) FED R EVID 403

In this case credibility is very important because the evidence consists primarily of the testimony of the disputing parties and there were no other eyewitnesses to the altercation This enhances the probative value of any evidence bearing on the inmatersquos credibility A court is likely to conclude that the inmatersquos prior felony drug conviction is relevant to his credibility See eg United States v Brito 427 F3d 53 64 (1st Cir 2005) (ldquoPrior drug-trafficking crimes are generally viewed as having some bearing on veracityrdquo) Although the probative value of any conviction diminishes with age see eg United States v Brewer 451 F Supp 50 53 (ED Tenn 1978) the inmatersquos ongoing problems with the law suggest that he has continued (and even escalated) his criminal behavior over the past nine years The court should admit this evidence because its probative value is not substantially outweighed by any Rule 403 concerns Specifically any prejudice to the inmate would be slight because the conviction is unrelated to the altercation at issue and the conviction was not for a heinous crime that might inflame the jury

[NOTE Whether an examinee identifies the jury instruction as containing a ldquoconclusiverdquo or ldquomandatoryrdquo presumption is less important than the examineersquos analysis of the constitutional infirmities]

Point One(b) (15) The court must admit evidence of the inmatersquos eight-year-old misdemeanor conviction because perjury is a crime of dishonesty

Rule 609(a)(2) provides that evidence of a criminal conviction ldquomust be admitted if the court can readily determine that establishing the elements of the crime required provingmdashor the witnessrsquos admittingmdasha dishonest act or false statementrdquo FED R EVID 609(a)(2) The inmatersquos conviction for perjury would have necessarily required proving that the inmate engaged in an act of dishonesty This conviction occurred within the past 10 years so it ldquomust be admittedrdquo because in contrast to Rule 609(a)(1) (discussed in Point One(a)) admission under Rule 609(a)(2) is mandatory and not subject to Rule 403

Point One(c) (20) The court should exclude evidence of the inmatersquos seven-year-old felony sexual assault conviction because the probative value of this evidence is substantially outweighed by the danger of unfair prejudice In the alternative the details of the prior conviction could be excluded

The inmatersquos conviction for felony sexual assault was seven years ago and he has not yet been released from incarceration so Rule 609(a) but not 609(b) is applicable here FED R EVID 609(a) This conviction is therefore admissible to impeach the inmate unless its probative value is substantially outweighed by the danger of unfair prejudice or any other Rule 403 concern Id

29

Evidence Analysis

Sex crimes are generally not considered relevant to credibility see Hopkins v State 639 So 2d 1247 1254 (Miss 1993) so the probative value of this conviction is relatively low Moreover the heinous nature of the inmatersquos crime (sexual assault on his daughter) makes the danger of unfair prejudice to the inmate very high Thus the court should exclude evidence of the conviction because it was for a heinous offense that is likely to inflame the jury and it has little bearing on credibility See eg United States v Beahm 664 F2d 414 419 (4th Cir 1981)

As an alternative to excluding this evidence the judge could minimize the unfair prejudice to the inmate by permitting limited cross-examination but refusing to allow specific questions about the nature of the inmatersquos conviction For example a court could limit cross-examination to the fact that the inmate was convicted of a ldquofelonyrdquo or perhaps that he was convicted of a ldquosexual assaultrdquo without identifying the victim However because evidence of the inmatersquos prior convictions can be admitted solely for the purpose of enabling the jury to assess his credibility and because his two earlier convictions should have already been admitted the court should exclude all evidence of the felony sexual assault conviction

Point Two(a) (15) The court should permit the inmatersquos counsel to cross-examine the guard regarding the false statement in his reacutesumeacute because the guardrsquos misconduct bears on his truthfulness

The inmate wishes to cross-examine the guard about his prior dishonest behaviormdashlying on his reacutesumeacutemdashthat did not involve a criminal conviction Rule 608(b) allows witnesses to be cross-examined about specific instances of prior non-conviction misconduct probative of untruthfulness ldquoin order to attack the witnessrsquos character for truthfulnessrdquo FED R EVID 608(b)

The courtrsquos decision to allow cross-examination about the guardrsquos prior dishonest behavior depends on the probative value of such evidence balanced against the danger of unfair prejudice to the guard or any other Rule 403 concern FED R EVID 403 Here the guardrsquos false statement on his reacutesumeacute that he obtained a degree in Criminal Justice is highly probative of his untruthfulness because it grossly misrepresents his actual academic record was made recently and was made with the intent to deceive Because the probative value of this evidence is very strong and is not substantially outweighed by any Rule 403 concerns cross-examination of the guard on this topic should be permitted The court may also consider it fair to permit this cross-examination of the guard on these matters assuming that one or more of the inmatersquos prior convictions have been admitted to impeach his credibility

Point Two(b) (15) The court should exclude extrinsic evidence of the guardrsquos non-conviction misconduct even if the guard denies wrongdoing or refuses to answer questions about the matter

Although Rule 608(b) allows cross-examination about specific instances of prior misconduct probative of untruthfulness ldquoextrinsic evidencerdquo offered to prove such misconduct is not admissible FED R EVID 608(b) The rationale for this rule is that allowing the introduction of extrinsic evidence of prior misconduct by witnesses when these acts are relevant only to the witnessesrsquo truthfulness and not to the main issues in the case would create too great a risk of confusing the jury and unduly delaying the trial The court does not have discretion to admit this extrinsic evidence See eg United States v Elliot 89 F3d 1360 1368 (8th Cir 1996)

30

Evidence Analysis

Here the inmatersquos counsel may cross-examine the guard about the false statement on his reacutesumeacute However the inmatersquos counsel must accept the guardrsquos response Even if the guard denies wrongdoing or refuses to answer questions about the matter the inmatersquos counsel cannot introduce the guardrsquos reacutesumeacute or the transcript from the local college to prove the guardrsquos misconduct

31

CORPORATIONS ANALYSIS (Corporations VA2 IX)

ANALYSIS

Legal Problems

(1) Do shareholders have the authority to amend a corporationrsquos bylaws with respect to director nominations

(2) Do board-approved bylaws on a particular subject here nomination of directors preempt subsequent conflicting bylaw amendments by shareholders

(3) Is a suit challenging both managementrsquos refusal to include the proposed bylaw amendment in Megarsquos proxy statement and the boardrsquos amendment of the bylaws dealing with nomination of directors a direct or derivative suit

DISCUSSION

Summary

The voting and litigation rights of the shareholders of Mega are subject to the provisions of the Model Business Corporations Act (MBCA)

The investorrsquos proposed bylaw provision is not inconsistent with state law Under the MBCA shareholders may amend the bylaws when the amendment deals with a proper matter for the corporationrsquos bylaws such as procedures for nominating directors

The Mega boardrsquos bylaw amendment does not preempt the investorrsquos proposed bylaw provision or the Mega shareholdersrsquo power to approve it While shareholders can limit the boardrsquos power to amend or repeal the bylaws the board cannot limit the shareholdersrsquo power

Whether the investor must make a demand on Megarsquos board depends on how the investor frames its claim If the investor claims a violation of shareholder voting rights the claim is direct and pre-suit demand on the board is not required If on the other hand the investor claims that the directors violated their fiduciary duties by amending the bylaws to entrench themselves the claim is derivative and a pre-suit demand is required

Point One (30) Shareholders may amend the corporationrsquos bylaws where the proposed bylaw provision relates to procedural matters typically included in the bylaws such as the nomination of directors

Internal affairs of the corporation such as the conduct of shareholder meetings and election of directors are subject to the corporate law of the state of incorporation See McDermott Inc v Lewis 531 A2d 206 (Del 1987) (applying law of jurisdiction where corporation was incorporated in case involving voting rights) This statersquos corporate statute is modeled on the MBCA

Under the MBCA ldquoshareholders may amend the corporationrsquos bylawsrdquo MBCA sect 1020(a) Thus the only question is whether the bylaws can specify the procedures for shareholder nomination of directors

32

Corporations Analysis

The MBCA states that the bylaws ldquomay contain any provision that is not inconsistent with law or the articles of incorporationrdquo MBCA sect 206(b) In addition the MBCA was revised in 2009 to address shareholder nomination of directors in public corporations (known as ldquoproxy accessrdquo) and specifies that the bylaws ldquomay contain a requirement that the corporation include in its [proxy materials] one or more individuals nominated by a shareholderrdquo MBCA sect 206(c)(1) see Committee on Corporate Laws ABA Section of Business Law Report on the Roles of Boards of Directors and Shareholders of Publicly Owned Corporations and Changes to the Model Business Corporations ActmdashAdoption of Shareholder Proxy Access Amendments to Chapters 2 and 10 65 BUS LAWYER 1105 (2010)

The inclusion of director-nomination procedures in the bylaws is consistent with practice and is recognized by the Delaware courts whose views on corporate law carry significant weight Typically the procedures for nomination of directors are found in the bylaws See 1 COX amp HAZEN TREATISE ON THE LAW OF CORPORATIONS sect 312 (3d ed 2011) see also 4 FLETCHER CORP FORMS ANN PART III ch 21 (2013) (including sample bylaws that permit nomination of directors by shareholders) The Delaware Supreme Court has confirmed that the bylaws may ldquodefine the process and proceduresrdquo for director elections See CA Inc v AFSCME Employees Pension Plan 953 A2d 227 (Del 2008) (concluding that bylaw amendment requiring reimbursement of election expenses to certain successful shareholder nominators is ldquoproper subjectrdquo under Delaware law)

[NOTE The question of the proper scope of the bylaws can be answered using the more general MBCA sect 206(b) or the 2009 MBCA revision adding sect 206(c)(1) (adopted in CT ME VA) In addition some examinees might raise the point that shareholder proposals may not compel the board to take action such as by including shareholder nominations in the companyrsquos proxy materials on the theory that the ldquobusiness and affairsrdquo of the corporation are to be managed by the board See MBCA sect 801(b) Although shareholders are generally limited to adopting precatory resolutions that recommend or encourage board action this limitation does not apply when shareholders have specific authority to take binding action on their ownmdashsuch as to amend the bylaws]

Point Two (30) Shareholders can amend (or repeal) board-approved bylaws Further shareholders can limit the boardrsquos power to later amend and repeal a shareholder-approved bylaw

Under the MBCA shareholders have the power to amend the bylaws See Point One The board shares this power with the shareholders unless (1) the corporationrsquos articles ldquoreserve that power exclusively to the shareholdersrdquo or (2) ldquothe shareholders in amending repealing or adopting a bylaw expressly provide that the board of directors may not amend repeal or reinstate that bylawrdquo See MBCA sect 1020(b)

Shareholder-approved bylaw provisions can amend or repeal existing bylaw provisions whether originally approved by the board or by shareholders See ALAN R PALMITER CORPORATIONS EXAMPLES AND EXPLANATIONS sect 713 (7th ed 2012) Thus the Mega boardrsquos bylaw amendmentmdashwhich set more demanding thresholds for shareholder nomination of directors than the investorrsquos proposed bylaw provisionmdashwould be superseded (repealed) if Megarsquos shareholders were to approve the investorrsquos proposal

Further a shareholder-approved bylaw generally can limit the power of the board to later amend or repeal it See MBCA sect 1020(b)(2) Thus if Megarsquos shareholders approved the bylaw

33

Corporations Analysis

provision proposed by the investor Megarsquos board could not repeal the provision because it includes a ldquono board repealrdquo clause

The revision to the MBCA in 2009 dealing with shareholder proxy access does not change this conclusion That revision specifies that a shareholder-approved bylaw dealing with director nominations may not limit the boardrsquos power to amend add or repeal ldquoany procedure or condition to such a bylaw in order to provide for a reasonable practicable and orderly processrdquo MBCA sect 206(d) Thus according to the revision if shareholders approve a bylaw amendment that limits further board changes the board would nonetheless retain the power to ldquotinkerrdquo with the bylaw to safeguard the voting process but could not repeal the shareholder-approved bylaw The Official Comment to MBCA sect 206(d) makes clear that the revision is ldquonot intended to allow the board of directors to frustrate the purpose of the shareholder-adopted proxy access provisionrdquo Thus if Megarsquos shareholders were to approve the bylaw provision proposed by the investor Megarsquos board could only amend the provision regarding its procedures or conditions in a manner consistent with its purpose of permitting proxy access for Megarsquos shareholders

[NOTE The boardrsquos attempted interference with a shareholder voting initiative may also have been a violation of the directorsrsquo fiduciary duties See Blasius Indus Inc v Atlas Corp 564 A2d 651 (Del Ch 1988) (finding that directors breached their fiduciary duties by amending bylaws and expanding size of board to thwart insurgentrsquos plan to amend bylaws and seat a majority of new directors) The call however asks examinees to consider whether shareholders or the board have ldquoprecedencerdquo over amending the corporate bylaws Thus an examineersquos answer should be framed in terms of ldquopowerrdquo and not ldquodutyrdquo]

Point Three (40) The investor need not make a demand on the board if the investor states a direct claim such as an allegation that the board interfered with the investorrsquos right to amend the bylaws But the investor must make a demand on the board if the investor states a derivative claim (on behalf of the corporation) such as an allegation that the directors sought to entrench themselves by interfering with the proposed proxy access

The MBCA generally requires that shareholders make a demand on the board of directors before initiation of a derivative suit MBCA sect 742 (shareholder may not bring derivative proceeding until written demand has been made on corporation and 90 days have expired) A derivative suit is essentially two suits in one where the plaintiff-shareholder seeks to bring on behalf of the corporation a claim that vindicates corporate rights usually based on violation of fiduciary duties PALMITER supra sect 1811 (6th ed 2009) The demand permits the board to investigate the situation identified by the shareholder and take suitable action No demand on the board is required however if the shareholder brings a direct suit to vindicate the shareholderrsquos own rights not those of the corporation

Is the suit brought by the investor derivative or direct The MBCA defines a ldquoderivative proceedingrdquo as one brought ldquoin the right of a domestic corporationrdquo MBCA sect 740(1) Thus the answer to how the investorrsquos suit should be characterized turns on what rights the investor seeks to vindicate If the investor frames its claim as one of fiduciary breach by directorsmdashfor example for failing to become adequately informed about voting procedures or for seeking to entrench themselves in office by manipulating the voting structure to avoid a shareholder insurgencymdashthen the suit is ldquoderivativerdquo and the investor must make a demand on the board See MBCA Ch 7 Subch D Introductory Comment (ldquothe derivative suit has historically been the principal method of challenging allegedly illegal action by managementrdquo)

34

Corporations Analysis

If however the investor frames its claim as one to vindicate shareholder rights the suit is direct and no demand is required For many courts the direct-derivative question turns on who is injured and who is to receive the relief sought by the plaintiff-shareholders See Tooley v Donaldson Lufkin amp Jenrette Inc 845 A2d 1031 (Del 2004) (characterizing a merger-delay claim as direct because delay of merger only harmed shareholders not corporation) Thus if the investor claims that managementrsquos refusal to include its proposed bylaw amendment in the corporationrsquos proxy materials violates its shareholder rights to initiate corporate governance reforms the suit will be direct Courts have not questioned the ability of shareholders to bring direct suits challenging board action to exclude their proposed bylaw amendments from the corporationrsquos proxy materials See JANA Master Fund Ltd v CNET Networks Inc 954 A2d 335 (Del Ch 2008) (upholding shareholderrsquos direct challenge to boardrsquos interpretation of advance-notice bylaw) Chesapeake Corp v Shore 771 A2d 293 (Del Ch 2000) (upholding shareholderrsquos direct challenge to actions by board that effectively prevented it from proposing bylaw amendments in contest for control)

Is the way that the investor frames its claim conclusive Courts have permitted shareholder-plaintiffs to challenge a transaction in a direct suit even though the same transaction could also be challenged as a fiduciary breach See Eisenberg v Flying Tiger Line Inc 451 F2d 267 (2d Cir 1971) (permitting direct suit challenging a corporate reorganization as a dilution of shareholder voting power even though reorganization may have involved conflicts of interest and thus constituted a fiduciary breach) Thus the investorrsquos choice to pursue a claim challenging the legality of managementrsquos decision to exclude the investorrsquos proposal from the corporationrsquos proxy materialsmdashrather than a possible breach of fiduciary dutymdashis likely to be respected See 3 COX amp HAZEN supra sect 153 (describing situations in which a claim can be framed as derivative or direct)

[NOTE Some issues under Delaware corporate law regarding pre-suit demand are not relevant here For example whether the Mega directors are independent and disinterested is not relevant to the MBCA requirement of a pre-suit demand As the Official Comment to MBCA sect 742 points out the MBCArsquos requirement of ldquouniversal demandrdquo gives the board ldquothe opportunity to reexamine the act complained of in the light of a potential lawsuit and take corrective actionrdquo even when the directors might be non-independent or have conflicts of interest

Nor is it relevant to the MBCA pre-suit demand requirement that the statutory 90-day waiting period may be onerous The first paragraph of MBCA sect 742 requires a pre-suit demand without exception the second paragraph of the section imposes a 90-day waiting period before a derivative suit may be brought which can be shortened if the board rejects the demand or ldquoirreparable injury to the corporation would result by waiting for the expiration of the 90-day periodrdquo The call as written asks only whether a pre-suit demand should be made and does not ask examinees to address whether the post-demand waiting period should be shortened under the ldquoirreparable injuryrdquo standard]

35

National Conference of Bar Examiners 302 South Bedford Street | Madison WI 53703-3622 Phone 608-280-8550 | Fax 608-280-8552 | TDD 608-661-1275

wwwncbexorg e-mail contactncbexorg

  • Preface
  • Description of the MEE
  • Instructions
  • July 2014 Questions
    • CRIMINAL LAW AND PROCEDURE QUESTION
    • CONTRACTS QUESTION
    • FAMILY LAW QUESTION
    • FEDERAL CIVIL PROCEDURE QUESTION
    • EVIDENCE QUESTION
    • CORPORATIONS QUESTION
      • July 2014 Analyses
        • CRIMINAL LAW AND PROCEDURE ANALYSIS
        • CONTRACTS ANALYSIS
        • FAMILY LAW ANALYSIS
        • FEDERAL CIVIL PROCEDURE ANALYSIS
        • EVIDENCE ANALYSIS
        • CORPORATIONS ANALYSIS
            • ltlt13 ASCII85EncodePages false13 AllowTransparency false13 AutoPositionEPSFiles true13 AutoRotatePages None13 Binding Left13 CalGrayProfile (Dot Gain 20)13 CalRGBProfile (sRGB IEC61966-21)13 CalCMYKProfile (US Web Coated 050SWOP051 v2)13 sRGBProfile (sRGB IEC61966-21)13 CannotEmbedFontPolicy Error13 CompatibilityLevel 1413 CompressObjects Tags13 CompressPages true13 ConvertImagesToIndexed true13 PassThroughJPEGImages true13 CreateJobTicket false13 DefaultRenderingIntent Default13 DetectBlends true13 DetectCurves 0000013 ColorConversionStrategy CMYK13 DoThumbnails false13 EmbedAllFonts true13 EmbedOpenType false13 ParseICCProfilesInComments true13 EmbedJobOptions true13 DSCReportingLevel 013 EmitDSCWarnings false13 EndPage -113 ImageMemory 104857613 LockDistillerParams false13 MaxSubsetPct 10013 Optimize true13 OPM 113 ParseDSCComments true13 ParseDSCCommentsForDocInfo true13 PreserveCopyPage true13 PreserveDICMYKValues true13 PreserveEPSInfo true13 PreserveFlatness true13 PreserveHalftoneInfo false13 PreserveOPIComments true13 PreserveOverprintSettings true13 StartPage 113 SubsetFonts true13 TransferFunctionInfo Apply13 UCRandBGInfo Preserve13 UsePrologue false13 ColorSettingsFile ()13 AlwaysEmbed [ true13 ]13 NeverEmbed [ true13 ]13 AntiAliasColorImages false13 CropColorImages true13 ColorImageMinResolution 30013 ColorImageMinResolutionPolicy OK13 DownsampleColorImages true13 ColorImageDownsampleType Bicubic13 ColorImageResolution 30013 ColorImageDepth -113 ColorImageMinDownsampleDepth 113 ColorImageDownsampleThreshold 15000013 EncodeColorImages true13 ColorImageFilter DCTEncode13 AutoFilterColorImages true13 ColorImageAutoFilterStrategy JPEG13 ColorACSImageDict ltlt13 QFactor 01513 HSamples [1 1 1 1] VSamples [1 1 1 1]13 gtgt13 ColorImageDict ltlt13 QFactor 01513 HSamples [1 1 1 1] VSamples [1 1 1 1]13 gtgt13 JPEG2000ColorACSImageDict ltlt13 TileWidth 25613 TileHeight 25613 Quality 3013 gtgt13 JPEG2000ColorImageDict ltlt13 TileWidth 25613 TileHeight 25613 Quality 3013 gtgt13 AntiAliasGrayImages false13 CropGrayImages true13 GrayImageMinResolution 30013 GrayImageMinResolutionPolicy OK13 DownsampleGrayImages true13 GrayImageDownsampleType Bicubic13 GrayImageResolution 30013 GrayImageDepth -113 GrayImageMinDownsampleDepth 213 GrayImageDownsampleThreshold 15000013 EncodeGrayImages true13 GrayImageFilter DCTEncode13 AutoFilterGrayImages true13 GrayImageAutoFilterStrategy JPEG13 GrayACSImageDict ltlt13 QFactor 01513 HSamples [1 1 1 1] VSamples [1 1 1 1]13 gtgt13 GrayImageDict ltlt13 QFactor 01513 HSamples [1 1 1 1] VSamples [1 1 1 1]13 gtgt13 JPEG2000GrayACSImageDict ltlt13 TileWidth 25613 TileHeight 25613 Quality 3013 gtgt13 JPEG2000GrayImageDict ltlt13 TileWidth 25613 TileHeight 25613 Quality 3013 gtgt13 AntiAliasMonoImages false13 CropMonoImages true13 MonoImageMinResolution 120013 MonoImageMinResolutionPolicy OK13 DownsampleMonoImages true13 MonoImageDownsampleType Bicubic13 MonoImageResolution 120013 MonoImageDepth -113 MonoImageDownsampleThreshold 15000013 EncodeMonoImages true13 MonoImageFilter CCITTFaxEncode13 MonoImageDict ltlt13 K -113 gtgt13 AllowPSXObjects false13 CheckCompliance [13 None13 ]13 PDFX1aCheck false13 PDFX3Check false13 PDFXCompliantPDFOnly false13 PDFXNoTrimBoxError true13 PDFXTrimBoxToMediaBoxOffset [13 00000013 00000013 00000013 00000013 ]13 PDFXSetBleedBoxToMediaBox true13 PDFXBleedBoxToTrimBoxOffset [13 00000013 00000013 00000013 00000013 ]13 PDFXOutputIntentProfile ()13 PDFXOutputConditionIdentifier ()13 PDFXOutputCondition ()13 PDFXRegistryName ()13 PDFXTrapped False1313 CreateJDFFile false13 Description ltlt13 ARA 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 BGR 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 CHS ltFEFF4f7f75288fd94e9b8bbe5b9a521b5efa7684002000410064006f006200650020005000440046002065876863900275284e8e9ad88d2891cf76845370524d53705237300260a853ef4ee54f7f75280020004100630072006f0062006100740020548c002000410064006f00620065002000520065006100640065007200200035002e003000204ee553ca66f49ad87248672c676562535f00521b5efa768400200050004400460020658768633002gt13 CHT ltFEFF4f7f752890194e9b8a2d7f6e5efa7acb7684002000410064006f006200650020005000440046002065874ef69069752865bc9ad854c18cea76845370524d5370523786557406300260a853ef4ee54f7f75280020004100630072006f0062006100740020548c002000410064006f00620065002000520065006100640065007200200035002e003000204ee553ca66f49ad87248672c4f86958b555f5df25efa7acb76840020005000440046002065874ef63002gt13 CZE 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 DAN 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 DEU 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 ESP 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 ETI 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 FRA ltFEFF005500740069006c006900730065007a00200063006500730020006f007000740069006f006e00730020006100660069006e00200064006500200063007200e900650072002000640065007300200064006f00630075006d0065006e00740073002000410064006f00620065002000500044004600200070006f0075007200200075006e00650020007100750061006c0069007400e90020006400270069006d007000720065007300730069006f006e00200070007200e9007000720065007300730065002e0020004c0065007300200064006f00630075006d0065006e00740073002000500044004600200063007200e900e90073002000700065007500760065006e0074002000ea0074007200650020006f007500760065007200740073002000640061006e00730020004100630072006f006200610074002c002000610069006e00730069002000710075002700410064006f00620065002000520065006100640065007200200035002e0030002000650074002000760065007200730069006f006e007300200075006c007400e90072006900650075007200650073002egt13 GRE ltFEFF03a703c103b703c303b903bc03bf03c003bf03b903ae03c303c403b5002003b103c503c403ad03c2002003c403b903c2002003c103c503b803bc03af03c303b503b903c2002003b303b903b1002003bd03b1002003b403b703bc03b903bf03c503c103b303ae03c303b503c403b5002003ad03b303b303c103b103c603b1002000410064006f006200650020005000440046002003c003bf03c5002003b503af03bd03b103b9002003ba03b103c42019002003b503be03bf03c703ae03bd002003ba03b103c403ac03bb03bb03b703bb03b1002003b303b903b1002003c003c103bf002d03b503ba03c403c503c003c903c403b903ba03ad03c2002003b503c103b303b103c303af03b503c2002003c503c803b703bb03ae03c2002003c003bf03b903cc03c403b703c403b103c2002e0020002003a403b10020005000440046002003ad03b303b303c103b103c603b1002003c003bf03c5002003ad03c703b503c403b5002003b403b703bc03b903bf03c503c103b303ae03c303b503b9002003bc03c003bf03c103bf03cd03bd002003bd03b1002003b103bd03bf03b903c703c403bf03cd03bd002003bc03b5002003c403bf0020004100630072006f006200610074002c002003c403bf002000410064006f00620065002000520065006100640065007200200035002e0030002003ba03b103b9002003bc03b503c403b103b303b503bd03ad03c303c403b503c103b503c2002003b503ba03b403cc03c303b503b903c2002egt13 HEB 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 HRV (Za stvaranje Adobe PDF dokumenata najpogodnijih za visokokvalitetni ispis prije tiskanja koristite ove postavke Stvoreni PDF dokumenti mogu se otvoriti Acrobat i Adobe Reader 50 i kasnijim verzijama)13 HUN 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 ITA ltFEFF005500740069006c0069007a007a006100720065002000710075006500730074006500200069006d0070006f007300740061007a0069006f006e00690020007000650072002000630072006500610072006500200064006f00630075006d0065006e00740069002000410064006f00620065002000500044004600200070006900f900200061006400610074007400690020006100200075006e00610020007000720065007300740061006d0070006100200064006900200061006c007400610020007100750061006c0069007400e0002e0020004900200064006f00630075006d0065006e007400690020005000440046002000630072006500610074006900200070006f00730073006f006e006f0020006500730073006500720065002000610070006500720074006900200063006f006e0020004100630072006f00620061007400200065002000410064006f00620065002000520065006100640065007200200035002e003000200065002000760065007200730069006f006e006900200073007500630063006500730073006900760065002egt13 JPN ltFEFF9ad854c18cea306a30d730ea30d730ec30b951fa529b7528002000410064006f0062006500200050004400460020658766f8306e4f5c6210306b4f7f75283057307e305930023053306e8a2d5b9a30674f5c62103055308c305f0020005000440046002030d530a130a430eb306f3001004100630072006f0062006100740020304a30883073002000410064006f00620065002000520065006100640065007200200035002e003000204ee5964d3067958b304f30533068304c3067304d307e305930023053306e8a2d5b9a306b306f30d530a930f330c8306e57cb30818fbc307f304c5fc59808306730593002gt13 KOR ltFEFFc7740020c124c815c7440020c0acc6a9d558c5ec0020ace0d488c9c80020c2dcd5d80020c778c1c4c5d00020ac00c7a50020c801d569d55c002000410064006f0062006500200050004400460020bb38c11cb97c0020c791c131d569b2c8b2e4002e0020c774b807ac8c0020c791c131b41c00200050004400460020bb38c11cb2940020004100630072006f0062006100740020bc0f002000410064006f00620065002000520065006100640065007200200035002e00300020c774c0c1c5d0c11c0020c5f40020c2180020c788c2b5b2c8b2e4002egt13 LTH ltFEFF004e006100750064006f006b0069007400650020016100690075006f007300200070006100720061006d006500740072007500730020006e006f0072011700640061006d00690020006b0075007200740069002000410064006f00620065002000500044004600200064006f006b0075006d0065006e007400750073002c0020006b00750072006900650020006c0061006200690061007500730069006100690020007000720069007400610069006b007900740069002000610075006b01610074006f00730020006b006f006b007900620117007300200070006100720065006e006700740069006e00690061006d00200073007000610075007300640069006e0069006d00750069002e0020002000530075006b0075007200740069002000500044004600200064006f006b0075006d0065006e007400610069002000670061006c006900200062016b007400690020006100740069006400610072006f006d00690020004100630072006f006200610074002000690072002000410064006f00620065002000520065006100640065007200200035002e0030002000610072002000760117006c00650073006e0117006d00690073002000760065007200730069006a006f006d00690073002egt13 LVI 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 NLD (Gebruik deze instellingen om Adobe PDF-documenten te maken die zijn geoptimaliseerd voor prepress-afdrukken van hoge kwaliteit De gemaakte PDF-documenten kunnen worden geopend met Acrobat en Adobe Reader 50 en hoger)13 NOR 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 POL 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 PTB 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 RUM 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 RUS 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 SKY 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 SLV 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 SUO 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 SVE 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 TUR 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 UKR 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 ENU (Use these settings to create Adobe PDF documents best suited for high-quality prepress printing Created PDF documents can be opened with Acrobat and Adobe Reader 50 and later)13 gtgt13 Namespace [13 (Adobe)13 (Common)13 (10)13 ]13 OtherNamespaces [13 ltlt13 AsReaderSpreads false13 CropImagesToFrames true13 ErrorControl WarnAndContinue13 FlattenerIgnoreSpreadOverrides false13 IncludeGuidesGrids false13 IncludeNonPrinting false13 IncludeSlug false13 Namespace [13 (Adobe)13 (InDesign)13 (40)13 ]13 OmitPlacedBitmaps false13 OmitPlacedEPS false13 OmitPlacedPDF false13 SimulateOverprint Legacy13 gtgt13 ltlt13 AddBleedMarks false13 AddColorBars false13 AddCropMarks false13 AddPageInfo false13 AddRegMarks false13 ConvertColors ConvertToCMYK13 DestinationProfileName ()13 DestinationProfileSelector DocumentCMYK13 Downsample16BitImages true13 FlattenerPreset ltlt13 PresetSelector MediumResolution13 gtgt13 FormElements false13 GenerateStructure false13 IncludeBookmarks false13 IncludeHyperlinks false13 IncludeInteractive false13 IncludeLayers false13 IncludeProfiles false13 MultimediaHandling UseObjectSettings13 Namespace [13 (Adobe)13 (CreativeSuite)13 (20)13 ]13 PDFXOutputIntentProfileSelector DocumentCMYK13 PreserveEditing true13 UntaggedCMYKHandling LeaveUntagged13 UntaggedRGBHandling UseDocumentProfile13 UseDocumentBleed false13 gtgt13 ]13gtgt setdistillerparams13ltlt13 HWResolution [2400 2400]13 PageSize [612000 792000]13gtgt setpagedevice13

Page 27: July 2014 MEE Questions and Analyses - NCBE...This publication includes the questions and analyses from the July 2014 MEE. (In the actual test, the questions are simply numbered rather

Federal Civil Procedure Analysis

irreparable injury and to preserve the courtrsquos power to render a meaningful decision after a trial on the meritsrdquo 11A CHARLES ALAN WRIGHT ET AL FEDERAL PRACTICE AND PROCEDURE sect 2947 at 112 (2013) Rule 65 of the Federal Rules of Civil Procedure sets out the procedural requirements for preliminary injunctions Preliminary injunctions may be granted only upon notice to the adverse party FED R CIV P 65(a)(1) and only if the movant ldquogives security in an amount that the court considers proper to pay the costs and damages sustained by any party found to have been wrongfully enjoined or restrainedrdquo FED R CIV P 65(c)

While Rule 65 sets out the procedural requirements for preliminary injunctive relief it does not specify the substantive grounds upon which it may be granted The courtrsquos discretion in ruling upon a motion for a preliminary injunction ldquois exercised in conformity with historic federal equity practicerdquo 11A WRIGHT ET AL supra sect 2947 at 114 The court typically considers four factors

(1) the significance of the threat of irreparable harm to the plaintiff if the injunction is not granted (2) the balance between this harm and the injury that granting the injunction would inflict on the defendant (3) the probability that the plaintiff will succeed on the merits and (4) the public interest

Id sect 2948 at 122ndash24 accord Habitat Educ Center v Bosworth 363 F Supp 2d 1070 1088 (ED Wis 2005) The most important of these factors is the risk of irreparable harm to the plaintiff 11A WRIGHT ET AL supra sect 29481 at 129 If the plaintiff has an adequate remedy at law (eg if money damages can compensate the plaintiff for its loss) then a preliminary injunction will be denied Id sect 29481

Here a court would likely conclude that the potential for environmental damage to the forest creates a significant threat of irreparable harm ldquo[E]nvironmental injury is often irreparable Courts have recognized that logging such as would occur [here] can have longshyterm environmental consequences and thus satisfy the irreparable injury criterionrdquo Habitat Educ Center 363 F Supp 2d at 1089 (citing Idaho Sporting Congress Inc v Alexander 222 F3d 562 569 (9th Cir 2000) (noting that the imminent and continuing logging activities presented ldquoevidence of environmental harm sufficient to tip the balance in favor of injunctive reliefrdquo)) Neighbors of Cuddy Mountain v US Forest Service 137 F3d 1372 1382 (9th Cir 1998) (stating that ldquo[t]he old growth forests plaintiffs seek to protect would if cut take hundreds of years to reproducerdquo) (internal citation omitted)) see also 11C WRIGHT ET AL supra sect 29481 at 151 (noting that ldquoa preliminary injunction has been issued to prevent harm to the environmentrdquo)

The second factor the balance between the harm to the plaintiff and the harm the defendant will suffer if the injunction is issued also appears to support issuance of a preliminary injunction here The USFS will have to wait before it can develop the Scenic National Forest and the logging company may lose money if the delay is prolonged These economic harms could be compensated monetarily if an injunction is issued inappropriately Where ldquoan injunction bond can compensate [the] defendant for any harm the injunction is likely to inflict the balance should be struck in favor of [the] plaintiffrdquo Id sect 29482 at 192 See also Habitat Educ Center 363 F Supp 2d at 1089 (stating that ldquothe relative absence of harmful effects on the Forest Service weighs in favor of granting the injunctionrdquo)

The third factor is the likelihood that the plaintiff will prevail on the merits Although there is limited information concerning the merits of the action the nonprofit alleges that the federal statute (NEPA) requires an environmental impact statement and further states that the USFS created no environmental impact analysis or statement at all Assuming that those

25

Federal Civil Procedure Analysis

allegations are correct it seems plausible to conclude that the nonprofit will be able to show a likelihood of success on the merits

Finally courts deciding whether or not to issue preliminary injunctive relief are to consider the public interest ldquoFocusing on this factor is another way of inquiring whether there are policy considerations that bear on whether the order should issuerdquo 11C WRIGHT ET AL supra sect 29484 at 214 If the court concludes that the nonprofit is likely to succeed on its NEPA claim because the USFS wrongfully failed to conduct an environmental impact assessment it is likely to find that the public interest would be served by restraining the USFS from proceeding with logging in a national forest See Heartwood Inc v US Forest Service 73 F Supp 2d 962 979 (SD Ill 1999) affrsquod on other grounds 230 F3d 947 (7th Cir 2000) (ldquoviolations by federal agencies of NEPArsquos provisions as established by Congress harm the public as well as the environmentrdquo)

Thus a court is very likely to grant a preliminary injunction if it concludes that the nonprofit has a significant likelihood of success on the merits

26

EVIDENCE ANALYSIS (Evidence ID IIA amp C)

ANALYSIS

Legal Problems

(1) Under what circumstances can evidence of prior convictions be used to impeach a witnessrsquos credibility in a civil case

(1)(a) May the inmatersquos credibility be impeached by evidence of a 12-year-old felony drug conviction if he was released from prison 9 years ago

(1)(b) May the inmatersquos credibility be impeached by evidence of an 8-year-old misdemeanor perjury conviction that was punishable by 1 year in jail if he pleaded guilty and was sentenced only to pay a $5000 fine

(1)(c) May the inmatersquos credibility be impeached by evidence of a 7-year-old sexual assault conviction if the inmate is still serving a 10-year prison sentence and the victim was his 13-year-old daughter

(2)(a) May the guardrsquos credibility be impeached by cross-examination regarding specific instances of misconduct (ie lying on his reacutesumeacute) relevant to credibility

(2)(b) May the guardrsquos credibility be impeached by admission of extrinsic evidence (his reacutesumeacute and academic transcript) offered to prove specific instances of misconduct relevant to credibility

DISCUSSION

Summary

Under the Federal Rules of Evidence witnesses can be impeached with evidence of prior convictions andor specific instances of misconduct Whether evidence of prior convictions should be admitted to impeach generally depends on the nature of the crime the amount of time that has passed and (only in criminal cases) whether the ldquowitnessrdquo is the defendant FED R EVID 609(a)

In this civil case evidence of the inmatersquos conviction for distribution of marijuana should be admitted to impeach the inmate because he was convicted of a felony and was released from prison fewer than 10 years ago FED R EVID 609(a)(1) Credibility is critically important in this case because the jury will hear conflicting testimony from the two disputing parties and there were no other eyewitnesses to the altercation Under Rule 609(a)(1) the inmatersquos conviction should be admitted because it has some bearing on his credibility and its probative value is not substantially outweighed by concerns of unfair prejudice confusion or delay Id

Evidence of the inmatersquos misdemeanor conviction for perjury must be admitted because the crime ldquorequired provingmdashor the witnessrsquos admittingmdasha dishonest act or false statementrdquo by the inmate FED R EVID 609(a)(2)

27

Evidence Analysis

Evidence of the inmatersquos felony conviction for sexual assault should be excluded because its probative value is substantially outweighed by the danger of unfair prejudice to the inmate based on the heinous nature of the crime FED R EVID 609(a)(1) In the alternative the judge could limit the evidence relating to this conviction by excluding details of the inmatersquos crime

In all civil (and criminal) cases witnesses can also be impeached with evidence of specific instances of prior misconduct that did not result in a conviction FED R EVID 608(b) Pursuant to Rule 608(b) misconduct probative of untruthfulness can be inquired into on cross-examination but cannot be proved through extrinsic evidence Id Thus the inmatersquos counsel should be permitted to cross-examine the guard regarding the false statement in the guardrsquos reacutesumeacute However extrinsic evidence of the guardrsquos misconduct (ie the guardrsquos authenticated reacutesumeacute and transcript from the local college) should not be admitted even if the guard denies wrongdoing or refuses to answer cross-examination questions about these matters Id

Point One (10) The Federal Rules of Evidence permit impeachment of witnesses with evidence of prior convictions

Whether convictions should be admitted to impeach generally depends on the nature of the crime the amount of time that has passed and (only in criminal cases) whether the ldquowitnessrdquo is the defendant FED R EVID 609(a) Under Rule 609(a) evidence of prior convictions may be admitted for the purpose of ldquoattacking a witnessrsquos character for truthfulnessrdquo Id

There are two basic types of convictions that can be admitted for the purpose of impeachment

(1) convictions for crimes ldquopunishable by death or by imprisonment for more than one yearrdquo (which generally correlates to ldquofeloniesrdquo) FED R EVID 609(a)(1) and (2) convictions ldquofor any crimes regardless of the punishment if the court can readily determine that establishing the elements of the crime required provingmdashor the witnessrsquos admittingmdasha dishonest act or false statementrdquo FED R EVID 609(a)(2)

Pursuant to Rule 609(a)(1) in civil cases the admission of evidence of a felony conviction is ldquosubject to Rule 403 [which says that a court may exclude relevant evidence if its probative value is substantially outweighed by other factors]rdquo FED R EVID 609(a)(1) However Rule 403 does not protect the witness against admission of prior convictions involving dishonestymdashwhich must be admitted by the court FED R EVID 609(a)(2)

Finally Federal Rule of Evidence 609(b) contains the presumption that a conviction that is more than 10 years old or where more than 10 years has passed since the witnessrsquos release from confinement (whichever is later) should not be admitted unless ldquoits probative value supported by specific facts and circumstances substantially outweighs its prejudicial effectrdquo and the proponent has provided the adverse party with reasonable written notice FED R EVID 609(b)

Point One(a) (25) The court should admit evidence of the inmatersquos 12-year-old felony marijuana distribution conviction

The inmatersquos conviction for marijuana distribution was for a felony punishable by imprisonment for more than one year See FED R EVID 609(a)(1) Moreover although the conviction was 12 years ago the 10-year time limit of Rule 609(b) is not exceeded because that time limit runs

28

Evidence Analysis

from the date of either ldquothe witnessrsquos conviction or release from confinement for it whichever is laterrdquo FED R EVID 609(b) Because the inmate served three years in prison he was released from confinement nine years ago

However pursuant to Rule 609(a)(1) the admission of felony convictions to impeach a witness in a civil case is ldquosubject to Rule 403rdquo FED R EVID 609(a)(1) Neither Rule 609(a) nor the advisory committee notes specify which factors courts should consider when balancing the probative value of a conviction against the dangers identified in Rule 403 (which include (1) unfair prejudice (2) confusion of the issues (3) misleading the jury (4) waste of time or undue delay and (5) needless presentation of cumulative evidence) FED R EVID 403

In this case credibility is very important because the evidence consists primarily of the testimony of the disputing parties and there were no other eyewitnesses to the altercation This enhances the probative value of any evidence bearing on the inmatersquos credibility A court is likely to conclude that the inmatersquos prior felony drug conviction is relevant to his credibility See eg United States v Brito 427 F3d 53 64 (1st Cir 2005) (ldquoPrior drug-trafficking crimes are generally viewed as having some bearing on veracityrdquo) Although the probative value of any conviction diminishes with age see eg United States v Brewer 451 F Supp 50 53 (ED Tenn 1978) the inmatersquos ongoing problems with the law suggest that he has continued (and even escalated) his criminal behavior over the past nine years The court should admit this evidence because its probative value is not substantially outweighed by any Rule 403 concerns Specifically any prejudice to the inmate would be slight because the conviction is unrelated to the altercation at issue and the conviction was not for a heinous crime that might inflame the jury

[NOTE Whether an examinee identifies the jury instruction as containing a ldquoconclusiverdquo or ldquomandatoryrdquo presumption is less important than the examineersquos analysis of the constitutional infirmities]

Point One(b) (15) The court must admit evidence of the inmatersquos eight-year-old misdemeanor conviction because perjury is a crime of dishonesty

Rule 609(a)(2) provides that evidence of a criminal conviction ldquomust be admitted if the court can readily determine that establishing the elements of the crime required provingmdashor the witnessrsquos admittingmdasha dishonest act or false statementrdquo FED R EVID 609(a)(2) The inmatersquos conviction for perjury would have necessarily required proving that the inmate engaged in an act of dishonesty This conviction occurred within the past 10 years so it ldquomust be admittedrdquo because in contrast to Rule 609(a)(1) (discussed in Point One(a)) admission under Rule 609(a)(2) is mandatory and not subject to Rule 403

Point One(c) (20) The court should exclude evidence of the inmatersquos seven-year-old felony sexual assault conviction because the probative value of this evidence is substantially outweighed by the danger of unfair prejudice In the alternative the details of the prior conviction could be excluded

The inmatersquos conviction for felony sexual assault was seven years ago and he has not yet been released from incarceration so Rule 609(a) but not 609(b) is applicable here FED R EVID 609(a) This conviction is therefore admissible to impeach the inmate unless its probative value is substantially outweighed by the danger of unfair prejudice or any other Rule 403 concern Id

29

Evidence Analysis

Sex crimes are generally not considered relevant to credibility see Hopkins v State 639 So 2d 1247 1254 (Miss 1993) so the probative value of this conviction is relatively low Moreover the heinous nature of the inmatersquos crime (sexual assault on his daughter) makes the danger of unfair prejudice to the inmate very high Thus the court should exclude evidence of the conviction because it was for a heinous offense that is likely to inflame the jury and it has little bearing on credibility See eg United States v Beahm 664 F2d 414 419 (4th Cir 1981)

As an alternative to excluding this evidence the judge could minimize the unfair prejudice to the inmate by permitting limited cross-examination but refusing to allow specific questions about the nature of the inmatersquos conviction For example a court could limit cross-examination to the fact that the inmate was convicted of a ldquofelonyrdquo or perhaps that he was convicted of a ldquosexual assaultrdquo without identifying the victim However because evidence of the inmatersquos prior convictions can be admitted solely for the purpose of enabling the jury to assess his credibility and because his two earlier convictions should have already been admitted the court should exclude all evidence of the felony sexual assault conviction

Point Two(a) (15) The court should permit the inmatersquos counsel to cross-examine the guard regarding the false statement in his reacutesumeacute because the guardrsquos misconduct bears on his truthfulness

The inmate wishes to cross-examine the guard about his prior dishonest behaviormdashlying on his reacutesumeacutemdashthat did not involve a criminal conviction Rule 608(b) allows witnesses to be cross-examined about specific instances of prior non-conviction misconduct probative of untruthfulness ldquoin order to attack the witnessrsquos character for truthfulnessrdquo FED R EVID 608(b)

The courtrsquos decision to allow cross-examination about the guardrsquos prior dishonest behavior depends on the probative value of such evidence balanced against the danger of unfair prejudice to the guard or any other Rule 403 concern FED R EVID 403 Here the guardrsquos false statement on his reacutesumeacute that he obtained a degree in Criminal Justice is highly probative of his untruthfulness because it grossly misrepresents his actual academic record was made recently and was made with the intent to deceive Because the probative value of this evidence is very strong and is not substantially outweighed by any Rule 403 concerns cross-examination of the guard on this topic should be permitted The court may also consider it fair to permit this cross-examination of the guard on these matters assuming that one or more of the inmatersquos prior convictions have been admitted to impeach his credibility

Point Two(b) (15) The court should exclude extrinsic evidence of the guardrsquos non-conviction misconduct even if the guard denies wrongdoing or refuses to answer questions about the matter

Although Rule 608(b) allows cross-examination about specific instances of prior misconduct probative of untruthfulness ldquoextrinsic evidencerdquo offered to prove such misconduct is not admissible FED R EVID 608(b) The rationale for this rule is that allowing the introduction of extrinsic evidence of prior misconduct by witnesses when these acts are relevant only to the witnessesrsquo truthfulness and not to the main issues in the case would create too great a risk of confusing the jury and unduly delaying the trial The court does not have discretion to admit this extrinsic evidence See eg United States v Elliot 89 F3d 1360 1368 (8th Cir 1996)

30

Evidence Analysis

Here the inmatersquos counsel may cross-examine the guard about the false statement on his reacutesumeacute However the inmatersquos counsel must accept the guardrsquos response Even if the guard denies wrongdoing or refuses to answer questions about the matter the inmatersquos counsel cannot introduce the guardrsquos reacutesumeacute or the transcript from the local college to prove the guardrsquos misconduct

31

CORPORATIONS ANALYSIS (Corporations VA2 IX)

ANALYSIS

Legal Problems

(1) Do shareholders have the authority to amend a corporationrsquos bylaws with respect to director nominations

(2) Do board-approved bylaws on a particular subject here nomination of directors preempt subsequent conflicting bylaw amendments by shareholders

(3) Is a suit challenging both managementrsquos refusal to include the proposed bylaw amendment in Megarsquos proxy statement and the boardrsquos amendment of the bylaws dealing with nomination of directors a direct or derivative suit

DISCUSSION

Summary

The voting and litigation rights of the shareholders of Mega are subject to the provisions of the Model Business Corporations Act (MBCA)

The investorrsquos proposed bylaw provision is not inconsistent with state law Under the MBCA shareholders may amend the bylaws when the amendment deals with a proper matter for the corporationrsquos bylaws such as procedures for nominating directors

The Mega boardrsquos bylaw amendment does not preempt the investorrsquos proposed bylaw provision or the Mega shareholdersrsquo power to approve it While shareholders can limit the boardrsquos power to amend or repeal the bylaws the board cannot limit the shareholdersrsquo power

Whether the investor must make a demand on Megarsquos board depends on how the investor frames its claim If the investor claims a violation of shareholder voting rights the claim is direct and pre-suit demand on the board is not required If on the other hand the investor claims that the directors violated their fiduciary duties by amending the bylaws to entrench themselves the claim is derivative and a pre-suit demand is required

Point One (30) Shareholders may amend the corporationrsquos bylaws where the proposed bylaw provision relates to procedural matters typically included in the bylaws such as the nomination of directors

Internal affairs of the corporation such as the conduct of shareholder meetings and election of directors are subject to the corporate law of the state of incorporation See McDermott Inc v Lewis 531 A2d 206 (Del 1987) (applying law of jurisdiction where corporation was incorporated in case involving voting rights) This statersquos corporate statute is modeled on the MBCA

Under the MBCA ldquoshareholders may amend the corporationrsquos bylawsrdquo MBCA sect 1020(a) Thus the only question is whether the bylaws can specify the procedures for shareholder nomination of directors

32

Corporations Analysis

The MBCA states that the bylaws ldquomay contain any provision that is not inconsistent with law or the articles of incorporationrdquo MBCA sect 206(b) In addition the MBCA was revised in 2009 to address shareholder nomination of directors in public corporations (known as ldquoproxy accessrdquo) and specifies that the bylaws ldquomay contain a requirement that the corporation include in its [proxy materials] one or more individuals nominated by a shareholderrdquo MBCA sect 206(c)(1) see Committee on Corporate Laws ABA Section of Business Law Report on the Roles of Boards of Directors and Shareholders of Publicly Owned Corporations and Changes to the Model Business Corporations ActmdashAdoption of Shareholder Proxy Access Amendments to Chapters 2 and 10 65 BUS LAWYER 1105 (2010)

The inclusion of director-nomination procedures in the bylaws is consistent with practice and is recognized by the Delaware courts whose views on corporate law carry significant weight Typically the procedures for nomination of directors are found in the bylaws See 1 COX amp HAZEN TREATISE ON THE LAW OF CORPORATIONS sect 312 (3d ed 2011) see also 4 FLETCHER CORP FORMS ANN PART III ch 21 (2013) (including sample bylaws that permit nomination of directors by shareholders) The Delaware Supreme Court has confirmed that the bylaws may ldquodefine the process and proceduresrdquo for director elections See CA Inc v AFSCME Employees Pension Plan 953 A2d 227 (Del 2008) (concluding that bylaw amendment requiring reimbursement of election expenses to certain successful shareholder nominators is ldquoproper subjectrdquo under Delaware law)

[NOTE The question of the proper scope of the bylaws can be answered using the more general MBCA sect 206(b) or the 2009 MBCA revision adding sect 206(c)(1) (adopted in CT ME VA) In addition some examinees might raise the point that shareholder proposals may not compel the board to take action such as by including shareholder nominations in the companyrsquos proxy materials on the theory that the ldquobusiness and affairsrdquo of the corporation are to be managed by the board See MBCA sect 801(b) Although shareholders are generally limited to adopting precatory resolutions that recommend or encourage board action this limitation does not apply when shareholders have specific authority to take binding action on their ownmdashsuch as to amend the bylaws]

Point Two (30) Shareholders can amend (or repeal) board-approved bylaws Further shareholders can limit the boardrsquos power to later amend and repeal a shareholder-approved bylaw

Under the MBCA shareholders have the power to amend the bylaws See Point One The board shares this power with the shareholders unless (1) the corporationrsquos articles ldquoreserve that power exclusively to the shareholdersrdquo or (2) ldquothe shareholders in amending repealing or adopting a bylaw expressly provide that the board of directors may not amend repeal or reinstate that bylawrdquo See MBCA sect 1020(b)

Shareholder-approved bylaw provisions can amend or repeal existing bylaw provisions whether originally approved by the board or by shareholders See ALAN R PALMITER CORPORATIONS EXAMPLES AND EXPLANATIONS sect 713 (7th ed 2012) Thus the Mega boardrsquos bylaw amendmentmdashwhich set more demanding thresholds for shareholder nomination of directors than the investorrsquos proposed bylaw provisionmdashwould be superseded (repealed) if Megarsquos shareholders were to approve the investorrsquos proposal

Further a shareholder-approved bylaw generally can limit the power of the board to later amend or repeal it See MBCA sect 1020(b)(2) Thus if Megarsquos shareholders approved the bylaw

33

Corporations Analysis

provision proposed by the investor Megarsquos board could not repeal the provision because it includes a ldquono board repealrdquo clause

The revision to the MBCA in 2009 dealing with shareholder proxy access does not change this conclusion That revision specifies that a shareholder-approved bylaw dealing with director nominations may not limit the boardrsquos power to amend add or repeal ldquoany procedure or condition to such a bylaw in order to provide for a reasonable practicable and orderly processrdquo MBCA sect 206(d) Thus according to the revision if shareholders approve a bylaw amendment that limits further board changes the board would nonetheless retain the power to ldquotinkerrdquo with the bylaw to safeguard the voting process but could not repeal the shareholder-approved bylaw The Official Comment to MBCA sect 206(d) makes clear that the revision is ldquonot intended to allow the board of directors to frustrate the purpose of the shareholder-adopted proxy access provisionrdquo Thus if Megarsquos shareholders were to approve the bylaw provision proposed by the investor Megarsquos board could only amend the provision regarding its procedures or conditions in a manner consistent with its purpose of permitting proxy access for Megarsquos shareholders

[NOTE The boardrsquos attempted interference with a shareholder voting initiative may also have been a violation of the directorsrsquo fiduciary duties See Blasius Indus Inc v Atlas Corp 564 A2d 651 (Del Ch 1988) (finding that directors breached their fiduciary duties by amending bylaws and expanding size of board to thwart insurgentrsquos plan to amend bylaws and seat a majority of new directors) The call however asks examinees to consider whether shareholders or the board have ldquoprecedencerdquo over amending the corporate bylaws Thus an examineersquos answer should be framed in terms of ldquopowerrdquo and not ldquodutyrdquo]

Point Three (40) The investor need not make a demand on the board if the investor states a direct claim such as an allegation that the board interfered with the investorrsquos right to amend the bylaws But the investor must make a demand on the board if the investor states a derivative claim (on behalf of the corporation) such as an allegation that the directors sought to entrench themselves by interfering with the proposed proxy access

The MBCA generally requires that shareholders make a demand on the board of directors before initiation of a derivative suit MBCA sect 742 (shareholder may not bring derivative proceeding until written demand has been made on corporation and 90 days have expired) A derivative suit is essentially two suits in one where the plaintiff-shareholder seeks to bring on behalf of the corporation a claim that vindicates corporate rights usually based on violation of fiduciary duties PALMITER supra sect 1811 (6th ed 2009) The demand permits the board to investigate the situation identified by the shareholder and take suitable action No demand on the board is required however if the shareholder brings a direct suit to vindicate the shareholderrsquos own rights not those of the corporation

Is the suit brought by the investor derivative or direct The MBCA defines a ldquoderivative proceedingrdquo as one brought ldquoin the right of a domestic corporationrdquo MBCA sect 740(1) Thus the answer to how the investorrsquos suit should be characterized turns on what rights the investor seeks to vindicate If the investor frames its claim as one of fiduciary breach by directorsmdashfor example for failing to become adequately informed about voting procedures or for seeking to entrench themselves in office by manipulating the voting structure to avoid a shareholder insurgencymdashthen the suit is ldquoderivativerdquo and the investor must make a demand on the board See MBCA Ch 7 Subch D Introductory Comment (ldquothe derivative suit has historically been the principal method of challenging allegedly illegal action by managementrdquo)

34

Corporations Analysis

If however the investor frames its claim as one to vindicate shareholder rights the suit is direct and no demand is required For many courts the direct-derivative question turns on who is injured and who is to receive the relief sought by the plaintiff-shareholders See Tooley v Donaldson Lufkin amp Jenrette Inc 845 A2d 1031 (Del 2004) (characterizing a merger-delay claim as direct because delay of merger only harmed shareholders not corporation) Thus if the investor claims that managementrsquos refusal to include its proposed bylaw amendment in the corporationrsquos proxy materials violates its shareholder rights to initiate corporate governance reforms the suit will be direct Courts have not questioned the ability of shareholders to bring direct suits challenging board action to exclude their proposed bylaw amendments from the corporationrsquos proxy materials See JANA Master Fund Ltd v CNET Networks Inc 954 A2d 335 (Del Ch 2008) (upholding shareholderrsquos direct challenge to boardrsquos interpretation of advance-notice bylaw) Chesapeake Corp v Shore 771 A2d 293 (Del Ch 2000) (upholding shareholderrsquos direct challenge to actions by board that effectively prevented it from proposing bylaw amendments in contest for control)

Is the way that the investor frames its claim conclusive Courts have permitted shareholder-plaintiffs to challenge a transaction in a direct suit even though the same transaction could also be challenged as a fiduciary breach See Eisenberg v Flying Tiger Line Inc 451 F2d 267 (2d Cir 1971) (permitting direct suit challenging a corporate reorganization as a dilution of shareholder voting power even though reorganization may have involved conflicts of interest and thus constituted a fiduciary breach) Thus the investorrsquos choice to pursue a claim challenging the legality of managementrsquos decision to exclude the investorrsquos proposal from the corporationrsquos proxy materialsmdashrather than a possible breach of fiduciary dutymdashis likely to be respected See 3 COX amp HAZEN supra sect 153 (describing situations in which a claim can be framed as derivative or direct)

[NOTE Some issues under Delaware corporate law regarding pre-suit demand are not relevant here For example whether the Mega directors are independent and disinterested is not relevant to the MBCA requirement of a pre-suit demand As the Official Comment to MBCA sect 742 points out the MBCArsquos requirement of ldquouniversal demandrdquo gives the board ldquothe opportunity to reexamine the act complained of in the light of a potential lawsuit and take corrective actionrdquo even when the directors might be non-independent or have conflicts of interest

Nor is it relevant to the MBCA pre-suit demand requirement that the statutory 90-day waiting period may be onerous The first paragraph of MBCA sect 742 requires a pre-suit demand without exception the second paragraph of the section imposes a 90-day waiting period before a derivative suit may be brought which can be shortened if the board rejects the demand or ldquoirreparable injury to the corporation would result by waiting for the expiration of the 90-day periodrdquo The call as written asks only whether a pre-suit demand should be made and does not ask examinees to address whether the post-demand waiting period should be shortened under the ldquoirreparable injuryrdquo standard]

35

National Conference of Bar Examiners 302 South Bedford Street | Madison WI 53703-3622 Phone 608-280-8550 | Fax 608-280-8552 | TDD 608-661-1275

wwwncbexorg e-mail contactncbexorg

  • Preface
  • Description of the MEE
  • Instructions
  • July 2014 Questions
    • CRIMINAL LAW AND PROCEDURE QUESTION
    • CONTRACTS QUESTION
    • FAMILY LAW QUESTION
    • FEDERAL CIVIL PROCEDURE QUESTION
    • EVIDENCE QUESTION
    • CORPORATIONS QUESTION
      • July 2014 Analyses
        • CRIMINAL LAW AND PROCEDURE ANALYSIS
        • CONTRACTS ANALYSIS
        • FAMILY LAW ANALYSIS
        • FEDERAL CIVIL PROCEDURE ANALYSIS
        • EVIDENCE ANALYSIS
        • CORPORATIONS ANALYSIS
            • ltlt13 ASCII85EncodePages false13 AllowTransparency false13 AutoPositionEPSFiles true13 AutoRotatePages None13 Binding Left13 CalGrayProfile (Dot Gain 20)13 CalRGBProfile (sRGB IEC61966-21)13 CalCMYKProfile (US Web Coated 050SWOP051 v2)13 sRGBProfile (sRGB IEC61966-21)13 CannotEmbedFontPolicy Error13 CompatibilityLevel 1413 CompressObjects Tags13 CompressPages true13 ConvertImagesToIndexed true13 PassThroughJPEGImages true13 CreateJobTicket false13 DefaultRenderingIntent Default13 DetectBlends true13 DetectCurves 0000013 ColorConversionStrategy CMYK13 DoThumbnails false13 EmbedAllFonts true13 EmbedOpenType false13 ParseICCProfilesInComments true13 EmbedJobOptions true13 DSCReportingLevel 013 EmitDSCWarnings false13 EndPage -113 ImageMemory 104857613 LockDistillerParams false13 MaxSubsetPct 10013 Optimize true13 OPM 113 ParseDSCComments true13 ParseDSCCommentsForDocInfo true13 PreserveCopyPage true13 PreserveDICMYKValues true13 PreserveEPSInfo true13 PreserveFlatness true13 PreserveHalftoneInfo false13 PreserveOPIComments true13 PreserveOverprintSettings true13 StartPage 113 SubsetFonts true13 TransferFunctionInfo Apply13 UCRandBGInfo Preserve13 UsePrologue false13 ColorSettingsFile ()13 AlwaysEmbed [ true13 ]13 NeverEmbed [ true13 ]13 AntiAliasColorImages false13 CropColorImages true13 ColorImageMinResolution 30013 ColorImageMinResolutionPolicy OK13 DownsampleColorImages true13 ColorImageDownsampleType Bicubic13 ColorImageResolution 30013 ColorImageDepth -113 ColorImageMinDownsampleDepth 113 ColorImageDownsampleThreshold 15000013 EncodeColorImages true13 ColorImageFilter DCTEncode13 AutoFilterColorImages true13 ColorImageAutoFilterStrategy JPEG13 ColorACSImageDict ltlt13 QFactor 01513 HSamples [1 1 1 1] VSamples [1 1 1 1]13 gtgt13 ColorImageDict ltlt13 QFactor 01513 HSamples [1 1 1 1] VSamples [1 1 1 1]13 gtgt13 JPEG2000ColorACSImageDict ltlt13 TileWidth 25613 TileHeight 25613 Quality 3013 gtgt13 JPEG2000ColorImageDict ltlt13 TileWidth 25613 TileHeight 25613 Quality 3013 gtgt13 AntiAliasGrayImages false13 CropGrayImages true13 GrayImageMinResolution 30013 GrayImageMinResolutionPolicy OK13 DownsampleGrayImages true13 GrayImageDownsampleType Bicubic13 GrayImageResolution 30013 GrayImageDepth -113 GrayImageMinDownsampleDepth 213 GrayImageDownsampleThreshold 15000013 EncodeGrayImages true13 GrayImageFilter DCTEncode13 AutoFilterGrayImages true13 GrayImageAutoFilterStrategy JPEG13 GrayACSImageDict ltlt13 QFactor 01513 HSamples [1 1 1 1] VSamples [1 1 1 1]13 gtgt13 GrayImageDict ltlt13 QFactor 01513 HSamples [1 1 1 1] VSamples [1 1 1 1]13 gtgt13 JPEG2000GrayACSImageDict ltlt13 TileWidth 25613 TileHeight 25613 Quality 3013 gtgt13 JPEG2000GrayImageDict ltlt13 TileWidth 25613 TileHeight 25613 Quality 3013 gtgt13 AntiAliasMonoImages false13 CropMonoImages true13 MonoImageMinResolution 120013 MonoImageMinResolutionPolicy OK13 DownsampleMonoImages true13 MonoImageDownsampleType Bicubic13 MonoImageResolution 120013 MonoImageDepth -113 MonoImageDownsampleThreshold 15000013 EncodeMonoImages true13 MonoImageFilter CCITTFaxEncode13 MonoImageDict ltlt13 K -113 gtgt13 AllowPSXObjects false13 CheckCompliance [13 None13 ]13 PDFX1aCheck false13 PDFX3Check false13 PDFXCompliantPDFOnly false13 PDFXNoTrimBoxError true13 PDFXTrimBoxToMediaBoxOffset [13 00000013 00000013 00000013 00000013 ]13 PDFXSetBleedBoxToMediaBox true13 PDFXBleedBoxToTrimBoxOffset [13 00000013 00000013 00000013 00000013 ]13 PDFXOutputIntentProfile ()13 PDFXOutputConditionIdentifier ()13 PDFXOutputCondition ()13 PDFXRegistryName ()13 PDFXTrapped False1313 CreateJDFFile false13 Description ltlt13 ARA 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 BGR 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 CHS ltFEFF4f7f75288fd94e9b8bbe5b9a521b5efa7684002000410064006f006200650020005000440046002065876863900275284e8e9ad88d2891cf76845370524d53705237300260a853ef4ee54f7f75280020004100630072006f0062006100740020548c002000410064006f00620065002000520065006100640065007200200035002e003000204ee553ca66f49ad87248672c676562535f00521b5efa768400200050004400460020658768633002gt13 CHT ltFEFF4f7f752890194e9b8a2d7f6e5efa7acb7684002000410064006f006200650020005000440046002065874ef69069752865bc9ad854c18cea76845370524d5370523786557406300260a853ef4ee54f7f75280020004100630072006f0062006100740020548c002000410064006f00620065002000520065006100640065007200200035002e003000204ee553ca66f49ad87248672c4f86958b555f5df25efa7acb76840020005000440046002065874ef63002gt13 CZE 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 DAN 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 DEU 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 ESP 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 ETI ltFEFF004b00610073007500740061006700650020006e0065006900640020007300e4007400740065006900640020006b00760061006c006900740065006500740073006500200074007200fc006b006900650065006c007300650020007000720069006e00740069006d0069007300650020006a0061006f006b007300200073006f00620069006c0069006b0065002000410064006f006200650020005000440046002d0064006f006b0075006d0065006e00740069006400650020006c006f006f006d006900730065006b0073002e00200020004c006f006f0064007500640020005000440046002d0064006f006b0075006d0065006e00740065002000730061006100740065002000610076006100640061002000700072006f006700720061006d006d006900640065006700610020004100630072006f0062006100740020006e0069006e0067002000410064006f00620065002000520065006100640065007200200035002e00300020006a00610020007500750065006d006100740065002000760065007200730069006f006f006e00690064006500670061002e000d000agt13 FRA 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 GRE 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 HEB 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 HRV (Za stvaranje Adobe PDF dokumenata najpogodnijih za visokokvalitetni ispis prije tiskanja koristite ove postavke Stvoreni PDF dokumenti mogu se otvoriti Acrobat i Adobe Reader 50 i kasnijim verzijama)13 HUN 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 ITA ltFEFF005500740069006c0069007a007a006100720065002000710075006500730074006500200069006d0070006f007300740061007a0069006f006e00690020007000650072002000630072006500610072006500200064006f00630075006d0065006e00740069002000410064006f00620065002000500044004600200070006900f900200061006400610074007400690020006100200075006e00610020007000720065007300740061006d0070006100200064006900200061006c007400610020007100750061006c0069007400e0002e0020004900200064006f00630075006d0065006e007400690020005000440046002000630072006500610074006900200070006f00730073006f006e006f0020006500730073006500720065002000610070006500720074006900200063006f006e0020004100630072006f00620061007400200065002000410064006f00620065002000520065006100640065007200200035002e003000200065002000760065007200730069006f006e006900200073007500630063006500730073006900760065002egt13 JPN ltFEFF9ad854c18cea306a30d730ea30d730ec30b951fa529b7528002000410064006f0062006500200050004400460020658766f8306e4f5c6210306b4f7f75283057307e305930023053306e8a2d5b9a30674f5c62103055308c305f0020005000440046002030d530a130a430eb306f3001004100630072006f0062006100740020304a30883073002000410064006f00620065002000520065006100640065007200200035002e003000204ee5964d3067958b304f30533068304c3067304d307e305930023053306e8a2d5b9a306b306f30d530a930f330c8306e57cb30818fbc307f304c5fc59808306730593002gt13 KOR ltFEFFc7740020c124c815c7440020c0acc6a9d558c5ec0020ace0d488c9c80020c2dcd5d80020c778c1c4c5d00020ac00c7a50020c801d569d55c002000410064006f0062006500200050004400460020bb38c11cb97c0020c791c131d569b2c8b2e4002e0020c774b807ac8c0020c791c131b41c00200050004400460020bb38c11cb2940020004100630072006f0062006100740020bc0f002000410064006f00620065002000520065006100640065007200200035002e00300020c774c0c1c5d0c11c0020c5f40020c2180020c788c2b5b2c8b2e4002egt13 LTH 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 LVI 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 NLD (Gebruik deze instellingen om Adobe PDF-documenten te maken die zijn geoptimaliseerd voor prepress-afdrukken van hoge kwaliteit De gemaakte PDF-documenten kunnen worden geopend met Acrobat en Adobe Reader 50 en hoger)13 NOR 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 POL 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 PTB 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 RUM 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 RUS ltFEFF04180441043f043e043b044c04370443043904420435002004340430043d043d044b04350020043d0430044104420440043e0439043a043800200434043b044f00200441043e043704340430043d0438044f00200434043e043a0443043c0435043d0442043e0432002000410064006f006200650020005000440046002c0020043c0430043a04410438043c0430043b044c043d043e0020043f043e04340445043e0434044f04490438044500200434043b044f00200432044b0441043e043a043e043a0430044704350441044204320435043d043d043e0433043e00200434043e043f0435044704300442043d043e0433043e00200432044b0432043e04340430002e002000200421043e043704340430043d043d044b04350020005000440046002d0434043e043a0443043c0435043d0442044b0020043c043e0436043d043e0020043e0442043a0440044b043204300442044c002004410020043f043e043c043e0449044c044e0020004100630072006f00620061007400200438002000410064006f00620065002000520065006100640065007200200035002e00300020043800200431043e043b043504350020043f043e04370434043d043804450020043204350440044104380439002egt13 SKY 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 SLV ltFEFF005400650020006e006100730074006100760069007400760065002000750070006f0072006100620069007400650020007a00610020007500730074007600610072006a0061006e006a006500200064006f006b0075006d0065006e0074006f0076002000410064006f006200650020005000440046002c0020006b006900200073006f0020006e0061006a007000720069006d00650072006e0065006a016100690020007a00610020006b0061006b006f0076006f00730074006e006f0020007400690073006b0061006e006a00650020007300200070007200690070007200610076006f0020006e00610020007400690073006b002e00200020005500730074007600610072006a0065006e006500200064006f006b0075006d0065006e0074006500200050004400460020006a00650020006d006f0067006f010d00650020006f0064007000720065007400690020007a0020004100630072006f00620061007400200069006e002000410064006f00620065002000520065006100640065007200200035002e003000200069006e0020006e006f00760065006a01610069006d002egt13 SUO 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 SVE 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 TUR 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 UKR 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 ENU (Use these settings to create Adobe PDF documents best suited for high-quality prepress printing Created PDF documents can be opened with Acrobat and Adobe Reader 50 and later)13 gtgt13 Namespace [13 (Adobe)13 (Common)13 (10)13 ]13 OtherNamespaces [13 ltlt13 AsReaderSpreads false13 CropImagesToFrames true13 ErrorControl WarnAndContinue13 FlattenerIgnoreSpreadOverrides false13 IncludeGuidesGrids false13 IncludeNonPrinting false13 IncludeSlug false13 Namespace [13 (Adobe)13 (InDesign)13 (40)13 ]13 OmitPlacedBitmaps false13 OmitPlacedEPS false13 OmitPlacedPDF false13 SimulateOverprint Legacy13 gtgt13 ltlt13 AddBleedMarks false13 AddColorBars false13 AddCropMarks false13 AddPageInfo false13 AddRegMarks false13 ConvertColors ConvertToCMYK13 DestinationProfileName ()13 DestinationProfileSelector DocumentCMYK13 Downsample16BitImages true13 FlattenerPreset ltlt13 PresetSelector MediumResolution13 gtgt13 FormElements false13 GenerateStructure false13 IncludeBookmarks false13 IncludeHyperlinks false13 IncludeInteractive false13 IncludeLayers false13 IncludeProfiles false13 MultimediaHandling UseObjectSettings13 Namespace [13 (Adobe)13 (CreativeSuite)13 (20)13 ]13 PDFXOutputIntentProfileSelector DocumentCMYK13 PreserveEditing true13 UntaggedCMYKHandling LeaveUntagged13 UntaggedRGBHandling UseDocumentProfile13 UseDocumentBleed false13 gtgt13 ]13gtgt setdistillerparams13ltlt13 HWResolution [2400 2400]13 PageSize [612000 792000]13gtgt setpagedevice13

Page 28: July 2014 MEE Questions and Analyses - NCBE...This publication includes the questions and analyses from the July 2014 MEE. (In the actual test, the questions are simply numbered rather

Federal Civil Procedure Analysis

allegations are correct it seems plausible to conclude that the nonprofit will be able to show a likelihood of success on the merits

Finally courts deciding whether or not to issue preliminary injunctive relief are to consider the public interest ldquoFocusing on this factor is another way of inquiring whether there are policy considerations that bear on whether the order should issuerdquo 11C WRIGHT ET AL supra sect 29484 at 214 If the court concludes that the nonprofit is likely to succeed on its NEPA claim because the USFS wrongfully failed to conduct an environmental impact assessment it is likely to find that the public interest would be served by restraining the USFS from proceeding with logging in a national forest See Heartwood Inc v US Forest Service 73 F Supp 2d 962 979 (SD Ill 1999) affrsquod on other grounds 230 F3d 947 (7th Cir 2000) (ldquoviolations by federal agencies of NEPArsquos provisions as established by Congress harm the public as well as the environmentrdquo)

Thus a court is very likely to grant a preliminary injunction if it concludes that the nonprofit has a significant likelihood of success on the merits

26

EVIDENCE ANALYSIS (Evidence ID IIA amp C)

ANALYSIS

Legal Problems

(1) Under what circumstances can evidence of prior convictions be used to impeach a witnessrsquos credibility in a civil case

(1)(a) May the inmatersquos credibility be impeached by evidence of a 12-year-old felony drug conviction if he was released from prison 9 years ago

(1)(b) May the inmatersquos credibility be impeached by evidence of an 8-year-old misdemeanor perjury conviction that was punishable by 1 year in jail if he pleaded guilty and was sentenced only to pay a $5000 fine

(1)(c) May the inmatersquos credibility be impeached by evidence of a 7-year-old sexual assault conviction if the inmate is still serving a 10-year prison sentence and the victim was his 13-year-old daughter

(2)(a) May the guardrsquos credibility be impeached by cross-examination regarding specific instances of misconduct (ie lying on his reacutesumeacute) relevant to credibility

(2)(b) May the guardrsquos credibility be impeached by admission of extrinsic evidence (his reacutesumeacute and academic transcript) offered to prove specific instances of misconduct relevant to credibility

DISCUSSION

Summary

Under the Federal Rules of Evidence witnesses can be impeached with evidence of prior convictions andor specific instances of misconduct Whether evidence of prior convictions should be admitted to impeach generally depends on the nature of the crime the amount of time that has passed and (only in criminal cases) whether the ldquowitnessrdquo is the defendant FED R EVID 609(a)

In this civil case evidence of the inmatersquos conviction for distribution of marijuana should be admitted to impeach the inmate because he was convicted of a felony and was released from prison fewer than 10 years ago FED R EVID 609(a)(1) Credibility is critically important in this case because the jury will hear conflicting testimony from the two disputing parties and there were no other eyewitnesses to the altercation Under Rule 609(a)(1) the inmatersquos conviction should be admitted because it has some bearing on his credibility and its probative value is not substantially outweighed by concerns of unfair prejudice confusion or delay Id

Evidence of the inmatersquos misdemeanor conviction for perjury must be admitted because the crime ldquorequired provingmdashor the witnessrsquos admittingmdasha dishonest act or false statementrdquo by the inmate FED R EVID 609(a)(2)

27

Evidence Analysis

Evidence of the inmatersquos felony conviction for sexual assault should be excluded because its probative value is substantially outweighed by the danger of unfair prejudice to the inmate based on the heinous nature of the crime FED R EVID 609(a)(1) In the alternative the judge could limit the evidence relating to this conviction by excluding details of the inmatersquos crime

In all civil (and criminal) cases witnesses can also be impeached with evidence of specific instances of prior misconduct that did not result in a conviction FED R EVID 608(b) Pursuant to Rule 608(b) misconduct probative of untruthfulness can be inquired into on cross-examination but cannot be proved through extrinsic evidence Id Thus the inmatersquos counsel should be permitted to cross-examine the guard regarding the false statement in the guardrsquos reacutesumeacute However extrinsic evidence of the guardrsquos misconduct (ie the guardrsquos authenticated reacutesumeacute and transcript from the local college) should not be admitted even if the guard denies wrongdoing or refuses to answer cross-examination questions about these matters Id

Point One (10) The Federal Rules of Evidence permit impeachment of witnesses with evidence of prior convictions

Whether convictions should be admitted to impeach generally depends on the nature of the crime the amount of time that has passed and (only in criminal cases) whether the ldquowitnessrdquo is the defendant FED R EVID 609(a) Under Rule 609(a) evidence of prior convictions may be admitted for the purpose of ldquoattacking a witnessrsquos character for truthfulnessrdquo Id

There are two basic types of convictions that can be admitted for the purpose of impeachment

(1) convictions for crimes ldquopunishable by death or by imprisonment for more than one yearrdquo (which generally correlates to ldquofeloniesrdquo) FED R EVID 609(a)(1) and (2) convictions ldquofor any crimes regardless of the punishment if the court can readily determine that establishing the elements of the crime required provingmdashor the witnessrsquos admittingmdasha dishonest act or false statementrdquo FED R EVID 609(a)(2)

Pursuant to Rule 609(a)(1) in civil cases the admission of evidence of a felony conviction is ldquosubject to Rule 403 [which says that a court may exclude relevant evidence if its probative value is substantially outweighed by other factors]rdquo FED R EVID 609(a)(1) However Rule 403 does not protect the witness against admission of prior convictions involving dishonestymdashwhich must be admitted by the court FED R EVID 609(a)(2)

Finally Federal Rule of Evidence 609(b) contains the presumption that a conviction that is more than 10 years old or where more than 10 years has passed since the witnessrsquos release from confinement (whichever is later) should not be admitted unless ldquoits probative value supported by specific facts and circumstances substantially outweighs its prejudicial effectrdquo and the proponent has provided the adverse party with reasonable written notice FED R EVID 609(b)

Point One(a) (25) The court should admit evidence of the inmatersquos 12-year-old felony marijuana distribution conviction

The inmatersquos conviction for marijuana distribution was for a felony punishable by imprisonment for more than one year See FED R EVID 609(a)(1) Moreover although the conviction was 12 years ago the 10-year time limit of Rule 609(b) is not exceeded because that time limit runs

28

Evidence Analysis

from the date of either ldquothe witnessrsquos conviction or release from confinement for it whichever is laterrdquo FED R EVID 609(b) Because the inmate served three years in prison he was released from confinement nine years ago

However pursuant to Rule 609(a)(1) the admission of felony convictions to impeach a witness in a civil case is ldquosubject to Rule 403rdquo FED R EVID 609(a)(1) Neither Rule 609(a) nor the advisory committee notes specify which factors courts should consider when balancing the probative value of a conviction against the dangers identified in Rule 403 (which include (1) unfair prejudice (2) confusion of the issues (3) misleading the jury (4) waste of time or undue delay and (5) needless presentation of cumulative evidence) FED R EVID 403

In this case credibility is very important because the evidence consists primarily of the testimony of the disputing parties and there were no other eyewitnesses to the altercation This enhances the probative value of any evidence bearing on the inmatersquos credibility A court is likely to conclude that the inmatersquos prior felony drug conviction is relevant to his credibility See eg United States v Brito 427 F3d 53 64 (1st Cir 2005) (ldquoPrior drug-trafficking crimes are generally viewed as having some bearing on veracityrdquo) Although the probative value of any conviction diminishes with age see eg United States v Brewer 451 F Supp 50 53 (ED Tenn 1978) the inmatersquos ongoing problems with the law suggest that he has continued (and even escalated) his criminal behavior over the past nine years The court should admit this evidence because its probative value is not substantially outweighed by any Rule 403 concerns Specifically any prejudice to the inmate would be slight because the conviction is unrelated to the altercation at issue and the conviction was not for a heinous crime that might inflame the jury

[NOTE Whether an examinee identifies the jury instruction as containing a ldquoconclusiverdquo or ldquomandatoryrdquo presumption is less important than the examineersquos analysis of the constitutional infirmities]

Point One(b) (15) The court must admit evidence of the inmatersquos eight-year-old misdemeanor conviction because perjury is a crime of dishonesty

Rule 609(a)(2) provides that evidence of a criminal conviction ldquomust be admitted if the court can readily determine that establishing the elements of the crime required provingmdashor the witnessrsquos admittingmdasha dishonest act or false statementrdquo FED R EVID 609(a)(2) The inmatersquos conviction for perjury would have necessarily required proving that the inmate engaged in an act of dishonesty This conviction occurred within the past 10 years so it ldquomust be admittedrdquo because in contrast to Rule 609(a)(1) (discussed in Point One(a)) admission under Rule 609(a)(2) is mandatory and not subject to Rule 403

Point One(c) (20) The court should exclude evidence of the inmatersquos seven-year-old felony sexual assault conviction because the probative value of this evidence is substantially outweighed by the danger of unfair prejudice In the alternative the details of the prior conviction could be excluded

The inmatersquos conviction for felony sexual assault was seven years ago and he has not yet been released from incarceration so Rule 609(a) but not 609(b) is applicable here FED R EVID 609(a) This conviction is therefore admissible to impeach the inmate unless its probative value is substantially outweighed by the danger of unfair prejudice or any other Rule 403 concern Id

29

Evidence Analysis

Sex crimes are generally not considered relevant to credibility see Hopkins v State 639 So 2d 1247 1254 (Miss 1993) so the probative value of this conviction is relatively low Moreover the heinous nature of the inmatersquos crime (sexual assault on his daughter) makes the danger of unfair prejudice to the inmate very high Thus the court should exclude evidence of the conviction because it was for a heinous offense that is likely to inflame the jury and it has little bearing on credibility See eg United States v Beahm 664 F2d 414 419 (4th Cir 1981)

As an alternative to excluding this evidence the judge could minimize the unfair prejudice to the inmate by permitting limited cross-examination but refusing to allow specific questions about the nature of the inmatersquos conviction For example a court could limit cross-examination to the fact that the inmate was convicted of a ldquofelonyrdquo or perhaps that he was convicted of a ldquosexual assaultrdquo without identifying the victim However because evidence of the inmatersquos prior convictions can be admitted solely for the purpose of enabling the jury to assess his credibility and because his two earlier convictions should have already been admitted the court should exclude all evidence of the felony sexual assault conviction

Point Two(a) (15) The court should permit the inmatersquos counsel to cross-examine the guard regarding the false statement in his reacutesumeacute because the guardrsquos misconduct bears on his truthfulness

The inmate wishes to cross-examine the guard about his prior dishonest behaviormdashlying on his reacutesumeacutemdashthat did not involve a criminal conviction Rule 608(b) allows witnesses to be cross-examined about specific instances of prior non-conviction misconduct probative of untruthfulness ldquoin order to attack the witnessrsquos character for truthfulnessrdquo FED R EVID 608(b)

The courtrsquos decision to allow cross-examination about the guardrsquos prior dishonest behavior depends on the probative value of such evidence balanced against the danger of unfair prejudice to the guard or any other Rule 403 concern FED R EVID 403 Here the guardrsquos false statement on his reacutesumeacute that he obtained a degree in Criminal Justice is highly probative of his untruthfulness because it grossly misrepresents his actual academic record was made recently and was made with the intent to deceive Because the probative value of this evidence is very strong and is not substantially outweighed by any Rule 403 concerns cross-examination of the guard on this topic should be permitted The court may also consider it fair to permit this cross-examination of the guard on these matters assuming that one or more of the inmatersquos prior convictions have been admitted to impeach his credibility

Point Two(b) (15) The court should exclude extrinsic evidence of the guardrsquos non-conviction misconduct even if the guard denies wrongdoing or refuses to answer questions about the matter

Although Rule 608(b) allows cross-examination about specific instances of prior misconduct probative of untruthfulness ldquoextrinsic evidencerdquo offered to prove such misconduct is not admissible FED R EVID 608(b) The rationale for this rule is that allowing the introduction of extrinsic evidence of prior misconduct by witnesses when these acts are relevant only to the witnessesrsquo truthfulness and not to the main issues in the case would create too great a risk of confusing the jury and unduly delaying the trial The court does not have discretion to admit this extrinsic evidence See eg United States v Elliot 89 F3d 1360 1368 (8th Cir 1996)

30

Evidence Analysis

Here the inmatersquos counsel may cross-examine the guard about the false statement on his reacutesumeacute However the inmatersquos counsel must accept the guardrsquos response Even if the guard denies wrongdoing or refuses to answer questions about the matter the inmatersquos counsel cannot introduce the guardrsquos reacutesumeacute or the transcript from the local college to prove the guardrsquos misconduct

31

CORPORATIONS ANALYSIS (Corporations VA2 IX)

ANALYSIS

Legal Problems

(1) Do shareholders have the authority to amend a corporationrsquos bylaws with respect to director nominations

(2) Do board-approved bylaws on a particular subject here nomination of directors preempt subsequent conflicting bylaw amendments by shareholders

(3) Is a suit challenging both managementrsquos refusal to include the proposed bylaw amendment in Megarsquos proxy statement and the boardrsquos amendment of the bylaws dealing with nomination of directors a direct or derivative suit

DISCUSSION

Summary

The voting and litigation rights of the shareholders of Mega are subject to the provisions of the Model Business Corporations Act (MBCA)

The investorrsquos proposed bylaw provision is not inconsistent with state law Under the MBCA shareholders may amend the bylaws when the amendment deals with a proper matter for the corporationrsquos bylaws such as procedures for nominating directors

The Mega boardrsquos bylaw amendment does not preempt the investorrsquos proposed bylaw provision or the Mega shareholdersrsquo power to approve it While shareholders can limit the boardrsquos power to amend or repeal the bylaws the board cannot limit the shareholdersrsquo power

Whether the investor must make a demand on Megarsquos board depends on how the investor frames its claim If the investor claims a violation of shareholder voting rights the claim is direct and pre-suit demand on the board is not required If on the other hand the investor claims that the directors violated their fiduciary duties by amending the bylaws to entrench themselves the claim is derivative and a pre-suit demand is required

Point One (30) Shareholders may amend the corporationrsquos bylaws where the proposed bylaw provision relates to procedural matters typically included in the bylaws such as the nomination of directors

Internal affairs of the corporation such as the conduct of shareholder meetings and election of directors are subject to the corporate law of the state of incorporation See McDermott Inc v Lewis 531 A2d 206 (Del 1987) (applying law of jurisdiction where corporation was incorporated in case involving voting rights) This statersquos corporate statute is modeled on the MBCA

Under the MBCA ldquoshareholders may amend the corporationrsquos bylawsrdquo MBCA sect 1020(a) Thus the only question is whether the bylaws can specify the procedures for shareholder nomination of directors

32

Corporations Analysis

The MBCA states that the bylaws ldquomay contain any provision that is not inconsistent with law or the articles of incorporationrdquo MBCA sect 206(b) In addition the MBCA was revised in 2009 to address shareholder nomination of directors in public corporations (known as ldquoproxy accessrdquo) and specifies that the bylaws ldquomay contain a requirement that the corporation include in its [proxy materials] one or more individuals nominated by a shareholderrdquo MBCA sect 206(c)(1) see Committee on Corporate Laws ABA Section of Business Law Report on the Roles of Boards of Directors and Shareholders of Publicly Owned Corporations and Changes to the Model Business Corporations ActmdashAdoption of Shareholder Proxy Access Amendments to Chapters 2 and 10 65 BUS LAWYER 1105 (2010)

The inclusion of director-nomination procedures in the bylaws is consistent with practice and is recognized by the Delaware courts whose views on corporate law carry significant weight Typically the procedures for nomination of directors are found in the bylaws See 1 COX amp HAZEN TREATISE ON THE LAW OF CORPORATIONS sect 312 (3d ed 2011) see also 4 FLETCHER CORP FORMS ANN PART III ch 21 (2013) (including sample bylaws that permit nomination of directors by shareholders) The Delaware Supreme Court has confirmed that the bylaws may ldquodefine the process and proceduresrdquo for director elections See CA Inc v AFSCME Employees Pension Plan 953 A2d 227 (Del 2008) (concluding that bylaw amendment requiring reimbursement of election expenses to certain successful shareholder nominators is ldquoproper subjectrdquo under Delaware law)

[NOTE The question of the proper scope of the bylaws can be answered using the more general MBCA sect 206(b) or the 2009 MBCA revision adding sect 206(c)(1) (adopted in CT ME VA) In addition some examinees might raise the point that shareholder proposals may not compel the board to take action such as by including shareholder nominations in the companyrsquos proxy materials on the theory that the ldquobusiness and affairsrdquo of the corporation are to be managed by the board See MBCA sect 801(b) Although shareholders are generally limited to adopting precatory resolutions that recommend or encourage board action this limitation does not apply when shareholders have specific authority to take binding action on their ownmdashsuch as to amend the bylaws]

Point Two (30) Shareholders can amend (or repeal) board-approved bylaws Further shareholders can limit the boardrsquos power to later amend and repeal a shareholder-approved bylaw

Under the MBCA shareholders have the power to amend the bylaws See Point One The board shares this power with the shareholders unless (1) the corporationrsquos articles ldquoreserve that power exclusively to the shareholdersrdquo or (2) ldquothe shareholders in amending repealing or adopting a bylaw expressly provide that the board of directors may not amend repeal or reinstate that bylawrdquo See MBCA sect 1020(b)

Shareholder-approved bylaw provisions can amend or repeal existing bylaw provisions whether originally approved by the board or by shareholders See ALAN R PALMITER CORPORATIONS EXAMPLES AND EXPLANATIONS sect 713 (7th ed 2012) Thus the Mega boardrsquos bylaw amendmentmdashwhich set more demanding thresholds for shareholder nomination of directors than the investorrsquos proposed bylaw provisionmdashwould be superseded (repealed) if Megarsquos shareholders were to approve the investorrsquos proposal

Further a shareholder-approved bylaw generally can limit the power of the board to later amend or repeal it See MBCA sect 1020(b)(2) Thus if Megarsquos shareholders approved the bylaw

33

Corporations Analysis

provision proposed by the investor Megarsquos board could not repeal the provision because it includes a ldquono board repealrdquo clause

The revision to the MBCA in 2009 dealing with shareholder proxy access does not change this conclusion That revision specifies that a shareholder-approved bylaw dealing with director nominations may not limit the boardrsquos power to amend add or repeal ldquoany procedure or condition to such a bylaw in order to provide for a reasonable practicable and orderly processrdquo MBCA sect 206(d) Thus according to the revision if shareholders approve a bylaw amendment that limits further board changes the board would nonetheless retain the power to ldquotinkerrdquo with the bylaw to safeguard the voting process but could not repeal the shareholder-approved bylaw The Official Comment to MBCA sect 206(d) makes clear that the revision is ldquonot intended to allow the board of directors to frustrate the purpose of the shareholder-adopted proxy access provisionrdquo Thus if Megarsquos shareholders were to approve the bylaw provision proposed by the investor Megarsquos board could only amend the provision regarding its procedures or conditions in a manner consistent with its purpose of permitting proxy access for Megarsquos shareholders

[NOTE The boardrsquos attempted interference with a shareholder voting initiative may also have been a violation of the directorsrsquo fiduciary duties See Blasius Indus Inc v Atlas Corp 564 A2d 651 (Del Ch 1988) (finding that directors breached their fiduciary duties by amending bylaws and expanding size of board to thwart insurgentrsquos plan to amend bylaws and seat a majority of new directors) The call however asks examinees to consider whether shareholders or the board have ldquoprecedencerdquo over amending the corporate bylaws Thus an examineersquos answer should be framed in terms of ldquopowerrdquo and not ldquodutyrdquo]

Point Three (40) The investor need not make a demand on the board if the investor states a direct claim such as an allegation that the board interfered with the investorrsquos right to amend the bylaws But the investor must make a demand on the board if the investor states a derivative claim (on behalf of the corporation) such as an allegation that the directors sought to entrench themselves by interfering with the proposed proxy access

The MBCA generally requires that shareholders make a demand on the board of directors before initiation of a derivative suit MBCA sect 742 (shareholder may not bring derivative proceeding until written demand has been made on corporation and 90 days have expired) A derivative suit is essentially two suits in one where the plaintiff-shareholder seeks to bring on behalf of the corporation a claim that vindicates corporate rights usually based on violation of fiduciary duties PALMITER supra sect 1811 (6th ed 2009) The demand permits the board to investigate the situation identified by the shareholder and take suitable action No demand on the board is required however if the shareholder brings a direct suit to vindicate the shareholderrsquos own rights not those of the corporation

Is the suit brought by the investor derivative or direct The MBCA defines a ldquoderivative proceedingrdquo as one brought ldquoin the right of a domestic corporationrdquo MBCA sect 740(1) Thus the answer to how the investorrsquos suit should be characterized turns on what rights the investor seeks to vindicate If the investor frames its claim as one of fiduciary breach by directorsmdashfor example for failing to become adequately informed about voting procedures or for seeking to entrench themselves in office by manipulating the voting structure to avoid a shareholder insurgencymdashthen the suit is ldquoderivativerdquo and the investor must make a demand on the board See MBCA Ch 7 Subch D Introductory Comment (ldquothe derivative suit has historically been the principal method of challenging allegedly illegal action by managementrdquo)

34

Corporations Analysis

If however the investor frames its claim as one to vindicate shareholder rights the suit is direct and no demand is required For many courts the direct-derivative question turns on who is injured and who is to receive the relief sought by the plaintiff-shareholders See Tooley v Donaldson Lufkin amp Jenrette Inc 845 A2d 1031 (Del 2004) (characterizing a merger-delay claim as direct because delay of merger only harmed shareholders not corporation) Thus if the investor claims that managementrsquos refusal to include its proposed bylaw amendment in the corporationrsquos proxy materials violates its shareholder rights to initiate corporate governance reforms the suit will be direct Courts have not questioned the ability of shareholders to bring direct suits challenging board action to exclude their proposed bylaw amendments from the corporationrsquos proxy materials See JANA Master Fund Ltd v CNET Networks Inc 954 A2d 335 (Del Ch 2008) (upholding shareholderrsquos direct challenge to boardrsquos interpretation of advance-notice bylaw) Chesapeake Corp v Shore 771 A2d 293 (Del Ch 2000) (upholding shareholderrsquos direct challenge to actions by board that effectively prevented it from proposing bylaw amendments in contest for control)

Is the way that the investor frames its claim conclusive Courts have permitted shareholder-plaintiffs to challenge a transaction in a direct suit even though the same transaction could also be challenged as a fiduciary breach See Eisenberg v Flying Tiger Line Inc 451 F2d 267 (2d Cir 1971) (permitting direct suit challenging a corporate reorganization as a dilution of shareholder voting power even though reorganization may have involved conflicts of interest and thus constituted a fiduciary breach) Thus the investorrsquos choice to pursue a claim challenging the legality of managementrsquos decision to exclude the investorrsquos proposal from the corporationrsquos proxy materialsmdashrather than a possible breach of fiduciary dutymdashis likely to be respected See 3 COX amp HAZEN supra sect 153 (describing situations in which a claim can be framed as derivative or direct)

[NOTE Some issues under Delaware corporate law regarding pre-suit demand are not relevant here For example whether the Mega directors are independent and disinterested is not relevant to the MBCA requirement of a pre-suit demand As the Official Comment to MBCA sect 742 points out the MBCArsquos requirement of ldquouniversal demandrdquo gives the board ldquothe opportunity to reexamine the act complained of in the light of a potential lawsuit and take corrective actionrdquo even when the directors might be non-independent or have conflicts of interest

Nor is it relevant to the MBCA pre-suit demand requirement that the statutory 90-day waiting period may be onerous The first paragraph of MBCA sect 742 requires a pre-suit demand without exception the second paragraph of the section imposes a 90-day waiting period before a derivative suit may be brought which can be shortened if the board rejects the demand or ldquoirreparable injury to the corporation would result by waiting for the expiration of the 90-day periodrdquo The call as written asks only whether a pre-suit demand should be made and does not ask examinees to address whether the post-demand waiting period should be shortened under the ldquoirreparable injuryrdquo standard]

35

National Conference of Bar Examiners 302 South Bedford Street | Madison WI 53703-3622 Phone 608-280-8550 | Fax 608-280-8552 | TDD 608-661-1275

wwwncbexorg e-mail contactncbexorg

  • Preface
  • Description of the MEE
  • Instructions
  • July 2014 Questions
    • CRIMINAL LAW AND PROCEDURE QUESTION
    • CONTRACTS QUESTION
    • FAMILY LAW QUESTION
    • FEDERAL CIVIL PROCEDURE QUESTION
    • EVIDENCE QUESTION
    • CORPORATIONS QUESTION
      • July 2014 Analyses
        • CRIMINAL LAW AND PROCEDURE ANALYSIS
        • CONTRACTS ANALYSIS
        • FAMILY LAW ANALYSIS
        • FEDERAL CIVIL PROCEDURE ANALYSIS
        • EVIDENCE ANALYSIS
        • CORPORATIONS ANALYSIS
            • ltlt13 ASCII85EncodePages false13 AllowTransparency false13 AutoPositionEPSFiles true13 AutoRotatePages None13 Binding Left13 CalGrayProfile (Dot Gain 20)13 CalRGBProfile (sRGB IEC61966-21)13 CalCMYKProfile (US Web Coated 050SWOP051 v2)13 sRGBProfile (sRGB IEC61966-21)13 CannotEmbedFontPolicy Error13 CompatibilityLevel 1413 CompressObjects Tags13 CompressPages true13 ConvertImagesToIndexed true13 PassThroughJPEGImages true13 CreateJobTicket false13 DefaultRenderingIntent Default13 DetectBlends true13 DetectCurves 0000013 ColorConversionStrategy CMYK13 DoThumbnails false13 EmbedAllFonts true13 EmbedOpenType false13 ParseICCProfilesInComments true13 EmbedJobOptions true13 DSCReportingLevel 013 EmitDSCWarnings false13 EndPage -113 ImageMemory 104857613 LockDistillerParams false13 MaxSubsetPct 10013 Optimize true13 OPM 113 ParseDSCComments true13 ParseDSCCommentsForDocInfo true13 PreserveCopyPage true13 PreserveDICMYKValues true13 PreserveEPSInfo true13 PreserveFlatness true13 PreserveHalftoneInfo false13 PreserveOPIComments true13 PreserveOverprintSettings true13 StartPage 113 SubsetFonts true13 TransferFunctionInfo Apply13 UCRandBGInfo Preserve13 UsePrologue false13 ColorSettingsFile ()13 AlwaysEmbed [ true13 ]13 NeverEmbed [ true13 ]13 AntiAliasColorImages false13 CropColorImages true13 ColorImageMinResolution 30013 ColorImageMinResolutionPolicy OK13 DownsampleColorImages true13 ColorImageDownsampleType Bicubic13 ColorImageResolution 30013 ColorImageDepth -113 ColorImageMinDownsampleDepth 113 ColorImageDownsampleThreshold 15000013 EncodeColorImages true13 ColorImageFilter DCTEncode13 AutoFilterColorImages true13 ColorImageAutoFilterStrategy JPEG13 ColorACSImageDict ltlt13 QFactor 01513 HSamples [1 1 1 1] VSamples [1 1 1 1]13 gtgt13 ColorImageDict ltlt13 QFactor 01513 HSamples [1 1 1 1] VSamples [1 1 1 1]13 gtgt13 JPEG2000ColorACSImageDict ltlt13 TileWidth 25613 TileHeight 25613 Quality 3013 gtgt13 JPEG2000ColorImageDict ltlt13 TileWidth 25613 TileHeight 25613 Quality 3013 gtgt13 AntiAliasGrayImages false13 CropGrayImages true13 GrayImageMinResolution 30013 GrayImageMinResolutionPolicy OK13 DownsampleGrayImages true13 GrayImageDownsampleType Bicubic13 GrayImageResolution 30013 GrayImageDepth -113 GrayImageMinDownsampleDepth 213 GrayImageDownsampleThreshold 15000013 EncodeGrayImages true13 GrayImageFilter DCTEncode13 AutoFilterGrayImages true13 GrayImageAutoFilterStrategy JPEG13 GrayACSImageDict ltlt13 QFactor 01513 HSamples [1 1 1 1] VSamples [1 1 1 1]13 gtgt13 GrayImageDict ltlt13 QFactor 01513 HSamples [1 1 1 1] VSamples [1 1 1 1]13 gtgt13 JPEG2000GrayACSImageDict ltlt13 TileWidth 25613 TileHeight 25613 Quality 3013 gtgt13 JPEG2000GrayImageDict ltlt13 TileWidth 25613 TileHeight 25613 Quality 3013 gtgt13 AntiAliasMonoImages false13 CropMonoImages true13 MonoImageMinResolution 120013 MonoImageMinResolutionPolicy OK13 DownsampleMonoImages true13 MonoImageDownsampleType Bicubic13 MonoImageResolution 120013 MonoImageDepth -113 MonoImageDownsampleThreshold 15000013 EncodeMonoImages true13 MonoImageFilter CCITTFaxEncode13 MonoImageDict ltlt13 K -113 gtgt13 AllowPSXObjects false13 CheckCompliance [13 None13 ]13 PDFX1aCheck false13 PDFX3Check false13 PDFXCompliantPDFOnly false13 PDFXNoTrimBoxError true13 PDFXTrimBoxToMediaBoxOffset [13 00000013 00000013 00000013 00000013 ]13 PDFXSetBleedBoxToMediaBox true13 PDFXBleedBoxToTrimBoxOffset [13 00000013 00000013 00000013 00000013 ]13 PDFXOutputIntentProfile ()13 PDFXOutputConditionIdentifier ()13 PDFXOutputCondition ()13 PDFXRegistryName ()13 PDFXTrapped False1313 CreateJDFFile false13 Description ltlt13 ARA 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 BGR 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 CHS ltFEFF4f7f75288fd94e9b8bbe5b9a521b5efa7684002000410064006f006200650020005000440046002065876863900275284e8e9ad88d2891cf76845370524d53705237300260a853ef4ee54f7f75280020004100630072006f0062006100740020548c002000410064006f00620065002000520065006100640065007200200035002e003000204ee553ca66f49ad87248672c676562535f00521b5efa768400200050004400460020658768633002gt13 CHT ltFEFF4f7f752890194e9b8a2d7f6e5efa7acb7684002000410064006f006200650020005000440046002065874ef69069752865bc9ad854c18cea76845370524d5370523786557406300260a853ef4ee54f7f75280020004100630072006f0062006100740020548c002000410064006f00620065002000520065006100640065007200200035002e003000204ee553ca66f49ad87248672c4f86958b555f5df25efa7acb76840020005000440046002065874ef63002gt13 CZE 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 DAN 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 DEU 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 ESP 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 ETI 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 FRA 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 GRE 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 HEB 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 HRV (Za stvaranje Adobe PDF dokumenata najpogodnijih za visokokvalitetni ispis prije tiskanja koristite ove postavke Stvoreni PDF dokumenti mogu se otvoriti Acrobat i Adobe Reader 50 i kasnijim verzijama)13 HUN 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 ITA ltFEFF005500740069006c0069007a007a006100720065002000710075006500730074006500200069006d0070006f007300740061007a0069006f006e00690020007000650072002000630072006500610072006500200064006f00630075006d0065006e00740069002000410064006f00620065002000500044004600200070006900f900200061006400610074007400690020006100200075006e00610020007000720065007300740061006d0070006100200064006900200061006c007400610020007100750061006c0069007400e0002e0020004900200064006f00630075006d0065006e007400690020005000440046002000630072006500610074006900200070006f00730073006f006e006f0020006500730073006500720065002000610070006500720074006900200063006f006e0020004100630072006f00620061007400200065002000410064006f00620065002000520065006100640065007200200035002e003000200065002000760065007200730069006f006e006900200073007500630063006500730073006900760065002egt13 JPN ltFEFF9ad854c18cea306a30d730ea30d730ec30b951fa529b7528002000410064006f0062006500200050004400460020658766f8306e4f5c6210306b4f7f75283057307e305930023053306e8a2d5b9a30674f5c62103055308c305f0020005000440046002030d530a130a430eb306f3001004100630072006f0062006100740020304a30883073002000410064006f00620065002000520065006100640065007200200035002e003000204ee5964d3067958b304f30533068304c3067304d307e305930023053306e8a2d5b9a306b306f30d530a930f330c8306e57cb30818fbc307f304c5fc59808306730593002gt13 KOR ltFEFFc7740020c124c815c7440020c0acc6a9d558c5ec0020ace0d488c9c80020c2dcd5d80020c778c1c4c5d00020ac00c7a50020c801d569d55c002000410064006f0062006500200050004400460020bb38c11cb97c0020c791c131d569b2c8b2e4002e0020c774b807ac8c0020c791c131b41c00200050004400460020bb38c11cb2940020004100630072006f0062006100740020bc0f002000410064006f00620065002000520065006100640065007200200035002e00300020c774c0c1c5d0c11c0020c5f40020c2180020c788c2b5b2c8b2e4002egt13 LTH 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 LVI 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 NLD (Gebruik deze instellingen om Adobe PDF-documenten te maken die zijn geoptimaliseerd voor prepress-afdrukken van hoge kwaliteit De gemaakte PDF-documenten kunnen worden geopend met Acrobat en Adobe Reader 50 en hoger)13 NOR 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 POL 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 PTB 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 RUM ltFEFF005500740069006c0069007a00610163006900200061006300650073007400650020007300650074010300720069002000700065006e007400720075002000610020006300720065006100200064006f00630075006d0065006e00740065002000410064006f006200650020005000440046002000610064006500630076006100740065002000700065006e0074007200750020007400690070010300720069007200650061002000700072006500700072006500730073002000640065002000630061006c006900740061007400650020007300750070006500720069006f006100720103002e002000200044006f00630075006d0065006e00740065006c00650020005000440046002000630072006500610074006500200070006f00740020006600690020006400650073006300680069007300650020006300750020004100630072006f006200610074002c002000410064006f00620065002000520065006100640065007200200035002e00300020015f00690020007600650072007300690075006e0069006c006500200075006c0074006500720069006f006100720065002egt13 RUS ltFEFF04180441043f043e043b044c04370443043904420435002004340430043d043d044b04350020043d0430044104420440043e0439043a043800200434043b044f00200441043e043704340430043d0438044f00200434043e043a0443043c0435043d0442043e0432002000410064006f006200650020005000440046002c0020043c0430043a04410438043c0430043b044c043d043e0020043f043e04340445043e0434044f04490438044500200434043b044f00200432044b0441043e043a043e043a0430044704350441044204320435043d043d043e0433043e00200434043e043f0435044704300442043d043e0433043e00200432044b0432043e04340430002e002000200421043e043704340430043d043d044b04350020005000440046002d0434043e043a0443043c0435043d0442044b0020043c043e0436043d043e0020043e0442043a0440044b043204300442044c002004410020043f043e043c043e0449044c044e0020004100630072006f00620061007400200438002000410064006f00620065002000520065006100640065007200200035002e00300020043800200431043e043b043504350020043f043e04370434043d043804450020043204350440044104380439002egt13 SKY 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 SLV 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 SUO 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 SVE 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 TUR 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 UKR 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 ENU (Use these settings to create Adobe PDF documents best suited for high-quality prepress printing Created PDF documents can be opened with Acrobat and Adobe Reader 50 and later)13 gtgt13 Namespace [13 (Adobe)13 (Common)13 (10)13 ]13 OtherNamespaces [13 ltlt13 AsReaderSpreads false13 CropImagesToFrames true13 ErrorControl WarnAndContinue13 FlattenerIgnoreSpreadOverrides false13 IncludeGuidesGrids false13 IncludeNonPrinting false13 IncludeSlug false13 Namespace [13 (Adobe)13 (InDesign)13 (40)13 ]13 OmitPlacedBitmaps false13 OmitPlacedEPS false13 OmitPlacedPDF false13 SimulateOverprint Legacy13 gtgt13 ltlt13 AddBleedMarks false13 AddColorBars false13 AddCropMarks false13 AddPageInfo false13 AddRegMarks false13 ConvertColors ConvertToCMYK13 DestinationProfileName ()13 DestinationProfileSelector DocumentCMYK13 Downsample16BitImages true13 FlattenerPreset ltlt13 PresetSelector MediumResolution13 gtgt13 FormElements false13 GenerateStructure false13 IncludeBookmarks false13 IncludeHyperlinks false13 IncludeInteractive false13 IncludeLayers false13 IncludeProfiles false13 MultimediaHandling UseObjectSettings13 Namespace [13 (Adobe)13 (CreativeSuite)13 (20)13 ]13 PDFXOutputIntentProfileSelector DocumentCMYK13 PreserveEditing true13 UntaggedCMYKHandling LeaveUntagged13 UntaggedRGBHandling UseDocumentProfile13 UseDocumentBleed false13 gtgt13 ]13gtgt setdistillerparams13ltlt13 HWResolution [2400 2400]13 PageSize [612000 792000]13gtgt setpagedevice13

Page 29: July 2014 MEE Questions and Analyses - NCBE...This publication includes the questions and analyses from the July 2014 MEE. (In the actual test, the questions are simply numbered rather

EVIDENCE ANALYSIS (Evidence ID IIA amp C)

ANALYSIS

Legal Problems

(1) Under what circumstances can evidence of prior convictions be used to impeach a witnessrsquos credibility in a civil case

(1)(a) May the inmatersquos credibility be impeached by evidence of a 12-year-old felony drug conviction if he was released from prison 9 years ago

(1)(b) May the inmatersquos credibility be impeached by evidence of an 8-year-old misdemeanor perjury conviction that was punishable by 1 year in jail if he pleaded guilty and was sentenced only to pay a $5000 fine

(1)(c) May the inmatersquos credibility be impeached by evidence of a 7-year-old sexual assault conviction if the inmate is still serving a 10-year prison sentence and the victim was his 13-year-old daughter

(2)(a) May the guardrsquos credibility be impeached by cross-examination regarding specific instances of misconduct (ie lying on his reacutesumeacute) relevant to credibility

(2)(b) May the guardrsquos credibility be impeached by admission of extrinsic evidence (his reacutesumeacute and academic transcript) offered to prove specific instances of misconduct relevant to credibility

DISCUSSION

Summary

Under the Federal Rules of Evidence witnesses can be impeached with evidence of prior convictions andor specific instances of misconduct Whether evidence of prior convictions should be admitted to impeach generally depends on the nature of the crime the amount of time that has passed and (only in criminal cases) whether the ldquowitnessrdquo is the defendant FED R EVID 609(a)

In this civil case evidence of the inmatersquos conviction for distribution of marijuana should be admitted to impeach the inmate because he was convicted of a felony and was released from prison fewer than 10 years ago FED R EVID 609(a)(1) Credibility is critically important in this case because the jury will hear conflicting testimony from the two disputing parties and there were no other eyewitnesses to the altercation Under Rule 609(a)(1) the inmatersquos conviction should be admitted because it has some bearing on his credibility and its probative value is not substantially outweighed by concerns of unfair prejudice confusion or delay Id

Evidence of the inmatersquos misdemeanor conviction for perjury must be admitted because the crime ldquorequired provingmdashor the witnessrsquos admittingmdasha dishonest act or false statementrdquo by the inmate FED R EVID 609(a)(2)

27

Evidence Analysis

Evidence of the inmatersquos felony conviction for sexual assault should be excluded because its probative value is substantially outweighed by the danger of unfair prejudice to the inmate based on the heinous nature of the crime FED R EVID 609(a)(1) In the alternative the judge could limit the evidence relating to this conviction by excluding details of the inmatersquos crime

In all civil (and criminal) cases witnesses can also be impeached with evidence of specific instances of prior misconduct that did not result in a conviction FED R EVID 608(b) Pursuant to Rule 608(b) misconduct probative of untruthfulness can be inquired into on cross-examination but cannot be proved through extrinsic evidence Id Thus the inmatersquos counsel should be permitted to cross-examine the guard regarding the false statement in the guardrsquos reacutesumeacute However extrinsic evidence of the guardrsquos misconduct (ie the guardrsquos authenticated reacutesumeacute and transcript from the local college) should not be admitted even if the guard denies wrongdoing or refuses to answer cross-examination questions about these matters Id

Point One (10) The Federal Rules of Evidence permit impeachment of witnesses with evidence of prior convictions

Whether convictions should be admitted to impeach generally depends on the nature of the crime the amount of time that has passed and (only in criminal cases) whether the ldquowitnessrdquo is the defendant FED R EVID 609(a) Under Rule 609(a) evidence of prior convictions may be admitted for the purpose of ldquoattacking a witnessrsquos character for truthfulnessrdquo Id

There are two basic types of convictions that can be admitted for the purpose of impeachment

(1) convictions for crimes ldquopunishable by death or by imprisonment for more than one yearrdquo (which generally correlates to ldquofeloniesrdquo) FED R EVID 609(a)(1) and (2) convictions ldquofor any crimes regardless of the punishment if the court can readily determine that establishing the elements of the crime required provingmdashor the witnessrsquos admittingmdasha dishonest act or false statementrdquo FED R EVID 609(a)(2)

Pursuant to Rule 609(a)(1) in civil cases the admission of evidence of a felony conviction is ldquosubject to Rule 403 [which says that a court may exclude relevant evidence if its probative value is substantially outweighed by other factors]rdquo FED R EVID 609(a)(1) However Rule 403 does not protect the witness against admission of prior convictions involving dishonestymdashwhich must be admitted by the court FED R EVID 609(a)(2)

Finally Federal Rule of Evidence 609(b) contains the presumption that a conviction that is more than 10 years old or where more than 10 years has passed since the witnessrsquos release from confinement (whichever is later) should not be admitted unless ldquoits probative value supported by specific facts and circumstances substantially outweighs its prejudicial effectrdquo and the proponent has provided the adverse party with reasonable written notice FED R EVID 609(b)

Point One(a) (25) The court should admit evidence of the inmatersquos 12-year-old felony marijuana distribution conviction

The inmatersquos conviction for marijuana distribution was for a felony punishable by imprisonment for more than one year See FED R EVID 609(a)(1) Moreover although the conviction was 12 years ago the 10-year time limit of Rule 609(b) is not exceeded because that time limit runs

28

Evidence Analysis

from the date of either ldquothe witnessrsquos conviction or release from confinement for it whichever is laterrdquo FED R EVID 609(b) Because the inmate served three years in prison he was released from confinement nine years ago

However pursuant to Rule 609(a)(1) the admission of felony convictions to impeach a witness in a civil case is ldquosubject to Rule 403rdquo FED R EVID 609(a)(1) Neither Rule 609(a) nor the advisory committee notes specify which factors courts should consider when balancing the probative value of a conviction against the dangers identified in Rule 403 (which include (1) unfair prejudice (2) confusion of the issues (3) misleading the jury (4) waste of time or undue delay and (5) needless presentation of cumulative evidence) FED R EVID 403

In this case credibility is very important because the evidence consists primarily of the testimony of the disputing parties and there were no other eyewitnesses to the altercation This enhances the probative value of any evidence bearing on the inmatersquos credibility A court is likely to conclude that the inmatersquos prior felony drug conviction is relevant to his credibility See eg United States v Brito 427 F3d 53 64 (1st Cir 2005) (ldquoPrior drug-trafficking crimes are generally viewed as having some bearing on veracityrdquo) Although the probative value of any conviction diminishes with age see eg United States v Brewer 451 F Supp 50 53 (ED Tenn 1978) the inmatersquos ongoing problems with the law suggest that he has continued (and even escalated) his criminal behavior over the past nine years The court should admit this evidence because its probative value is not substantially outweighed by any Rule 403 concerns Specifically any prejudice to the inmate would be slight because the conviction is unrelated to the altercation at issue and the conviction was not for a heinous crime that might inflame the jury

[NOTE Whether an examinee identifies the jury instruction as containing a ldquoconclusiverdquo or ldquomandatoryrdquo presumption is less important than the examineersquos analysis of the constitutional infirmities]

Point One(b) (15) The court must admit evidence of the inmatersquos eight-year-old misdemeanor conviction because perjury is a crime of dishonesty

Rule 609(a)(2) provides that evidence of a criminal conviction ldquomust be admitted if the court can readily determine that establishing the elements of the crime required provingmdashor the witnessrsquos admittingmdasha dishonest act or false statementrdquo FED R EVID 609(a)(2) The inmatersquos conviction for perjury would have necessarily required proving that the inmate engaged in an act of dishonesty This conviction occurred within the past 10 years so it ldquomust be admittedrdquo because in contrast to Rule 609(a)(1) (discussed in Point One(a)) admission under Rule 609(a)(2) is mandatory and not subject to Rule 403

Point One(c) (20) The court should exclude evidence of the inmatersquos seven-year-old felony sexual assault conviction because the probative value of this evidence is substantially outweighed by the danger of unfair prejudice In the alternative the details of the prior conviction could be excluded

The inmatersquos conviction for felony sexual assault was seven years ago and he has not yet been released from incarceration so Rule 609(a) but not 609(b) is applicable here FED R EVID 609(a) This conviction is therefore admissible to impeach the inmate unless its probative value is substantially outweighed by the danger of unfair prejudice or any other Rule 403 concern Id

29

Evidence Analysis

Sex crimes are generally not considered relevant to credibility see Hopkins v State 639 So 2d 1247 1254 (Miss 1993) so the probative value of this conviction is relatively low Moreover the heinous nature of the inmatersquos crime (sexual assault on his daughter) makes the danger of unfair prejudice to the inmate very high Thus the court should exclude evidence of the conviction because it was for a heinous offense that is likely to inflame the jury and it has little bearing on credibility See eg United States v Beahm 664 F2d 414 419 (4th Cir 1981)

As an alternative to excluding this evidence the judge could minimize the unfair prejudice to the inmate by permitting limited cross-examination but refusing to allow specific questions about the nature of the inmatersquos conviction For example a court could limit cross-examination to the fact that the inmate was convicted of a ldquofelonyrdquo or perhaps that he was convicted of a ldquosexual assaultrdquo without identifying the victim However because evidence of the inmatersquos prior convictions can be admitted solely for the purpose of enabling the jury to assess his credibility and because his two earlier convictions should have already been admitted the court should exclude all evidence of the felony sexual assault conviction

Point Two(a) (15) The court should permit the inmatersquos counsel to cross-examine the guard regarding the false statement in his reacutesumeacute because the guardrsquos misconduct bears on his truthfulness

The inmate wishes to cross-examine the guard about his prior dishonest behaviormdashlying on his reacutesumeacutemdashthat did not involve a criminal conviction Rule 608(b) allows witnesses to be cross-examined about specific instances of prior non-conviction misconduct probative of untruthfulness ldquoin order to attack the witnessrsquos character for truthfulnessrdquo FED R EVID 608(b)

The courtrsquos decision to allow cross-examination about the guardrsquos prior dishonest behavior depends on the probative value of such evidence balanced against the danger of unfair prejudice to the guard or any other Rule 403 concern FED R EVID 403 Here the guardrsquos false statement on his reacutesumeacute that he obtained a degree in Criminal Justice is highly probative of his untruthfulness because it grossly misrepresents his actual academic record was made recently and was made with the intent to deceive Because the probative value of this evidence is very strong and is not substantially outweighed by any Rule 403 concerns cross-examination of the guard on this topic should be permitted The court may also consider it fair to permit this cross-examination of the guard on these matters assuming that one or more of the inmatersquos prior convictions have been admitted to impeach his credibility

Point Two(b) (15) The court should exclude extrinsic evidence of the guardrsquos non-conviction misconduct even if the guard denies wrongdoing or refuses to answer questions about the matter

Although Rule 608(b) allows cross-examination about specific instances of prior misconduct probative of untruthfulness ldquoextrinsic evidencerdquo offered to prove such misconduct is not admissible FED R EVID 608(b) The rationale for this rule is that allowing the introduction of extrinsic evidence of prior misconduct by witnesses when these acts are relevant only to the witnessesrsquo truthfulness and not to the main issues in the case would create too great a risk of confusing the jury and unduly delaying the trial The court does not have discretion to admit this extrinsic evidence See eg United States v Elliot 89 F3d 1360 1368 (8th Cir 1996)

30

Evidence Analysis

Here the inmatersquos counsel may cross-examine the guard about the false statement on his reacutesumeacute However the inmatersquos counsel must accept the guardrsquos response Even if the guard denies wrongdoing or refuses to answer questions about the matter the inmatersquos counsel cannot introduce the guardrsquos reacutesumeacute or the transcript from the local college to prove the guardrsquos misconduct

31

CORPORATIONS ANALYSIS (Corporations VA2 IX)

ANALYSIS

Legal Problems

(1) Do shareholders have the authority to amend a corporationrsquos bylaws with respect to director nominations

(2) Do board-approved bylaws on a particular subject here nomination of directors preempt subsequent conflicting bylaw amendments by shareholders

(3) Is a suit challenging both managementrsquos refusal to include the proposed bylaw amendment in Megarsquos proxy statement and the boardrsquos amendment of the bylaws dealing with nomination of directors a direct or derivative suit

DISCUSSION

Summary

The voting and litigation rights of the shareholders of Mega are subject to the provisions of the Model Business Corporations Act (MBCA)

The investorrsquos proposed bylaw provision is not inconsistent with state law Under the MBCA shareholders may amend the bylaws when the amendment deals with a proper matter for the corporationrsquos bylaws such as procedures for nominating directors

The Mega boardrsquos bylaw amendment does not preempt the investorrsquos proposed bylaw provision or the Mega shareholdersrsquo power to approve it While shareholders can limit the boardrsquos power to amend or repeal the bylaws the board cannot limit the shareholdersrsquo power

Whether the investor must make a demand on Megarsquos board depends on how the investor frames its claim If the investor claims a violation of shareholder voting rights the claim is direct and pre-suit demand on the board is not required If on the other hand the investor claims that the directors violated their fiduciary duties by amending the bylaws to entrench themselves the claim is derivative and a pre-suit demand is required

Point One (30) Shareholders may amend the corporationrsquos bylaws where the proposed bylaw provision relates to procedural matters typically included in the bylaws such as the nomination of directors

Internal affairs of the corporation such as the conduct of shareholder meetings and election of directors are subject to the corporate law of the state of incorporation See McDermott Inc v Lewis 531 A2d 206 (Del 1987) (applying law of jurisdiction where corporation was incorporated in case involving voting rights) This statersquos corporate statute is modeled on the MBCA

Under the MBCA ldquoshareholders may amend the corporationrsquos bylawsrdquo MBCA sect 1020(a) Thus the only question is whether the bylaws can specify the procedures for shareholder nomination of directors

32

Corporations Analysis

The MBCA states that the bylaws ldquomay contain any provision that is not inconsistent with law or the articles of incorporationrdquo MBCA sect 206(b) In addition the MBCA was revised in 2009 to address shareholder nomination of directors in public corporations (known as ldquoproxy accessrdquo) and specifies that the bylaws ldquomay contain a requirement that the corporation include in its [proxy materials] one or more individuals nominated by a shareholderrdquo MBCA sect 206(c)(1) see Committee on Corporate Laws ABA Section of Business Law Report on the Roles of Boards of Directors and Shareholders of Publicly Owned Corporations and Changes to the Model Business Corporations ActmdashAdoption of Shareholder Proxy Access Amendments to Chapters 2 and 10 65 BUS LAWYER 1105 (2010)

The inclusion of director-nomination procedures in the bylaws is consistent with practice and is recognized by the Delaware courts whose views on corporate law carry significant weight Typically the procedures for nomination of directors are found in the bylaws See 1 COX amp HAZEN TREATISE ON THE LAW OF CORPORATIONS sect 312 (3d ed 2011) see also 4 FLETCHER CORP FORMS ANN PART III ch 21 (2013) (including sample bylaws that permit nomination of directors by shareholders) The Delaware Supreme Court has confirmed that the bylaws may ldquodefine the process and proceduresrdquo for director elections See CA Inc v AFSCME Employees Pension Plan 953 A2d 227 (Del 2008) (concluding that bylaw amendment requiring reimbursement of election expenses to certain successful shareholder nominators is ldquoproper subjectrdquo under Delaware law)

[NOTE The question of the proper scope of the bylaws can be answered using the more general MBCA sect 206(b) or the 2009 MBCA revision adding sect 206(c)(1) (adopted in CT ME VA) In addition some examinees might raise the point that shareholder proposals may not compel the board to take action such as by including shareholder nominations in the companyrsquos proxy materials on the theory that the ldquobusiness and affairsrdquo of the corporation are to be managed by the board See MBCA sect 801(b) Although shareholders are generally limited to adopting precatory resolutions that recommend or encourage board action this limitation does not apply when shareholders have specific authority to take binding action on their ownmdashsuch as to amend the bylaws]

Point Two (30) Shareholders can amend (or repeal) board-approved bylaws Further shareholders can limit the boardrsquos power to later amend and repeal a shareholder-approved bylaw

Under the MBCA shareholders have the power to amend the bylaws See Point One The board shares this power with the shareholders unless (1) the corporationrsquos articles ldquoreserve that power exclusively to the shareholdersrdquo or (2) ldquothe shareholders in amending repealing or adopting a bylaw expressly provide that the board of directors may not amend repeal or reinstate that bylawrdquo See MBCA sect 1020(b)

Shareholder-approved bylaw provisions can amend or repeal existing bylaw provisions whether originally approved by the board or by shareholders See ALAN R PALMITER CORPORATIONS EXAMPLES AND EXPLANATIONS sect 713 (7th ed 2012) Thus the Mega boardrsquos bylaw amendmentmdashwhich set more demanding thresholds for shareholder nomination of directors than the investorrsquos proposed bylaw provisionmdashwould be superseded (repealed) if Megarsquos shareholders were to approve the investorrsquos proposal

Further a shareholder-approved bylaw generally can limit the power of the board to later amend or repeal it See MBCA sect 1020(b)(2) Thus if Megarsquos shareholders approved the bylaw

33

Corporations Analysis

provision proposed by the investor Megarsquos board could not repeal the provision because it includes a ldquono board repealrdquo clause

The revision to the MBCA in 2009 dealing with shareholder proxy access does not change this conclusion That revision specifies that a shareholder-approved bylaw dealing with director nominations may not limit the boardrsquos power to amend add or repeal ldquoany procedure or condition to such a bylaw in order to provide for a reasonable practicable and orderly processrdquo MBCA sect 206(d) Thus according to the revision if shareholders approve a bylaw amendment that limits further board changes the board would nonetheless retain the power to ldquotinkerrdquo with the bylaw to safeguard the voting process but could not repeal the shareholder-approved bylaw The Official Comment to MBCA sect 206(d) makes clear that the revision is ldquonot intended to allow the board of directors to frustrate the purpose of the shareholder-adopted proxy access provisionrdquo Thus if Megarsquos shareholders were to approve the bylaw provision proposed by the investor Megarsquos board could only amend the provision regarding its procedures or conditions in a manner consistent with its purpose of permitting proxy access for Megarsquos shareholders

[NOTE The boardrsquos attempted interference with a shareholder voting initiative may also have been a violation of the directorsrsquo fiduciary duties See Blasius Indus Inc v Atlas Corp 564 A2d 651 (Del Ch 1988) (finding that directors breached their fiduciary duties by amending bylaws and expanding size of board to thwart insurgentrsquos plan to amend bylaws and seat a majority of new directors) The call however asks examinees to consider whether shareholders or the board have ldquoprecedencerdquo over amending the corporate bylaws Thus an examineersquos answer should be framed in terms of ldquopowerrdquo and not ldquodutyrdquo]

Point Three (40) The investor need not make a demand on the board if the investor states a direct claim such as an allegation that the board interfered with the investorrsquos right to amend the bylaws But the investor must make a demand on the board if the investor states a derivative claim (on behalf of the corporation) such as an allegation that the directors sought to entrench themselves by interfering with the proposed proxy access

The MBCA generally requires that shareholders make a demand on the board of directors before initiation of a derivative suit MBCA sect 742 (shareholder may not bring derivative proceeding until written demand has been made on corporation and 90 days have expired) A derivative suit is essentially two suits in one where the plaintiff-shareholder seeks to bring on behalf of the corporation a claim that vindicates corporate rights usually based on violation of fiduciary duties PALMITER supra sect 1811 (6th ed 2009) The demand permits the board to investigate the situation identified by the shareholder and take suitable action No demand on the board is required however if the shareholder brings a direct suit to vindicate the shareholderrsquos own rights not those of the corporation

Is the suit brought by the investor derivative or direct The MBCA defines a ldquoderivative proceedingrdquo as one brought ldquoin the right of a domestic corporationrdquo MBCA sect 740(1) Thus the answer to how the investorrsquos suit should be characterized turns on what rights the investor seeks to vindicate If the investor frames its claim as one of fiduciary breach by directorsmdashfor example for failing to become adequately informed about voting procedures or for seeking to entrench themselves in office by manipulating the voting structure to avoid a shareholder insurgencymdashthen the suit is ldquoderivativerdquo and the investor must make a demand on the board See MBCA Ch 7 Subch D Introductory Comment (ldquothe derivative suit has historically been the principal method of challenging allegedly illegal action by managementrdquo)

34

Corporations Analysis

If however the investor frames its claim as one to vindicate shareholder rights the suit is direct and no demand is required For many courts the direct-derivative question turns on who is injured and who is to receive the relief sought by the plaintiff-shareholders See Tooley v Donaldson Lufkin amp Jenrette Inc 845 A2d 1031 (Del 2004) (characterizing a merger-delay claim as direct because delay of merger only harmed shareholders not corporation) Thus if the investor claims that managementrsquos refusal to include its proposed bylaw amendment in the corporationrsquos proxy materials violates its shareholder rights to initiate corporate governance reforms the suit will be direct Courts have not questioned the ability of shareholders to bring direct suits challenging board action to exclude their proposed bylaw amendments from the corporationrsquos proxy materials See JANA Master Fund Ltd v CNET Networks Inc 954 A2d 335 (Del Ch 2008) (upholding shareholderrsquos direct challenge to boardrsquos interpretation of advance-notice bylaw) Chesapeake Corp v Shore 771 A2d 293 (Del Ch 2000) (upholding shareholderrsquos direct challenge to actions by board that effectively prevented it from proposing bylaw amendments in contest for control)

Is the way that the investor frames its claim conclusive Courts have permitted shareholder-plaintiffs to challenge a transaction in a direct suit even though the same transaction could also be challenged as a fiduciary breach See Eisenberg v Flying Tiger Line Inc 451 F2d 267 (2d Cir 1971) (permitting direct suit challenging a corporate reorganization as a dilution of shareholder voting power even though reorganization may have involved conflicts of interest and thus constituted a fiduciary breach) Thus the investorrsquos choice to pursue a claim challenging the legality of managementrsquos decision to exclude the investorrsquos proposal from the corporationrsquos proxy materialsmdashrather than a possible breach of fiduciary dutymdashis likely to be respected See 3 COX amp HAZEN supra sect 153 (describing situations in which a claim can be framed as derivative or direct)

[NOTE Some issues under Delaware corporate law regarding pre-suit demand are not relevant here For example whether the Mega directors are independent and disinterested is not relevant to the MBCA requirement of a pre-suit demand As the Official Comment to MBCA sect 742 points out the MBCArsquos requirement of ldquouniversal demandrdquo gives the board ldquothe opportunity to reexamine the act complained of in the light of a potential lawsuit and take corrective actionrdquo even when the directors might be non-independent or have conflicts of interest

Nor is it relevant to the MBCA pre-suit demand requirement that the statutory 90-day waiting period may be onerous The first paragraph of MBCA sect 742 requires a pre-suit demand without exception the second paragraph of the section imposes a 90-day waiting period before a derivative suit may be brought which can be shortened if the board rejects the demand or ldquoirreparable injury to the corporation would result by waiting for the expiration of the 90-day periodrdquo The call as written asks only whether a pre-suit demand should be made and does not ask examinees to address whether the post-demand waiting period should be shortened under the ldquoirreparable injuryrdquo standard]

35

National Conference of Bar Examiners 302 South Bedford Street | Madison WI 53703-3622 Phone 608-280-8550 | Fax 608-280-8552 | TDD 608-661-1275

wwwncbexorg e-mail contactncbexorg

  • Preface
  • Description of the MEE
  • Instructions
  • July 2014 Questions
    • CRIMINAL LAW AND PROCEDURE QUESTION
    • CONTRACTS QUESTION
    • FAMILY LAW QUESTION
    • FEDERAL CIVIL PROCEDURE QUESTION
    • EVIDENCE QUESTION
    • CORPORATIONS QUESTION
      • July 2014 Analyses
        • CRIMINAL LAW AND PROCEDURE ANALYSIS
        • CONTRACTS ANALYSIS
        • FAMILY LAW ANALYSIS
        • FEDERAL CIVIL PROCEDURE ANALYSIS
        • EVIDENCE ANALYSIS
        • CORPORATIONS ANALYSIS
            • ltlt13 ASCII85EncodePages false13 AllowTransparency false13 AutoPositionEPSFiles true13 AutoRotatePages None13 Binding Left13 CalGrayProfile (Dot Gain 20)13 CalRGBProfile (sRGB IEC61966-21)13 CalCMYKProfile (US Web Coated 050SWOP051 v2)13 sRGBProfile (sRGB IEC61966-21)13 CannotEmbedFontPolicy Error13 CompatibilityLevel 1413 CompressObjects Tags13 CompressPages true13 ConvertImagesToIndexed true13 PassThroughJPEGImages true13 CreateJobTicket false13 DefaultRenderingIntent Default13 DetectBlends true13 DetectCurves 0000013 ColorConversionStrategy CMYK13 DoThumbnails false13 EmbedAllFonts true13 EmbedOpenType false13 ParseICCProfilesInComments true13 EmbedJobOptions true13 DSCReportingLevel 013 EmitDSCWarnings false13 EndPage -113 ImageMemory 104857613 LockDistillerParams false13 MaxSubsetPct 10013 Optimize true13 OPM 113 ParseDSCComments true13 ParseDSCCommentsForDocInfo true13 PreserveCopyPage true13 PreserveDICMYKValues true13 PreserveEPSInfo true13 PreserveFlatness true13 PreserveHalftoneInfo false13 PreserveOPIComments true13 PreserveOverprintSettings true13 StartPage 113 SubsetFonts true13 TransferFunctionInfo Apply13 UCRandBGInfo Preserve13 UsePrologue false13 ColorSettingsFile ()13 AlwaysEmbed [ true13 ]13 NeverEmbed [ true13 ]13 AntiAliasColorImages false13 CropColorImages true13 ColorImageMinResolution 30013 ColorImageMinResolutionPolicy OK13 DownsampleColorImages true13 ColorImageDownsampleType Bicubic13 ColorImageResolution 30013 ColorImageDepth -113 ColorImageMinDownsampleDepth 113 ColorImageDownsampleThreshold 15000013 EncodeColorImages true13 ColorImageFilter DCTEncode13 AutoFilterColorImages true13 ColorImageAutoFilterStrategy JPEG13 ColorACSImageDict ltlt13 QFactor 01513 HSamples [1 1 1 1] VSamples [1 1 1 1]13 gtgt13 ColorImageDict ltlt13 QFactor 01513 HSamples [1 1 1 1] VSamples [1 1 1 1]13 gtgt13 JPEG2000ColorACSImageDict ltlt13 TileWidth 25613 TileHeight 25613 Quality 3013 gtgt13 JPEG2000ColorImageDict ltlt13 TileWidth 25613 TileHeight 25613 Quality 3013 gtgt13 AntiAliasGrayImages false13 CropGrayImages true13 GrayImageMinResolution 30013 GrayImageMinResolutionPolicy OK13 DownsampleGrayImages true13 GrayImageDownsampleType Bicubic13 GrayImageResolution 30013 GrayImageDepth -113 GrayImageMinDownsampleDepth 213 GrayImageDownsampleThreshold 15000013 EncodeGrayImages true13 GrayImageFilter DCTEncode13 AutoFilterGrayImages true13 GrayImageAutoFilterStrategy JPEG13 GrayACSImageDict ltlt13 QFactor 01513 HSamples [1 1 1 1] VSamples [1 1 1 1]13 gtgt13 GrayImageDict ltlt13 QFactor 01513 HSamples [1 1 1 1] VSamples [1 1 1 1]13 gtgt13 JPEG2000GrayACSImageDict ltlt13 TileWidth 25613 TileHeight 25613 Quality 3013 gtgt13 JPEG2000GrayImageDict ltlt13 TileWidth 25613 TileHeight 25613 Quality 3013 gtgt13 AntiAliasMonoImages false13 CropMonoImages true13 MonoImageMinResolution 120013 MonoImageMinResolutionPolicy OK13 DownsampleMonoImages true13 MonoImageDownsampleType Bicubic13 MonoImageResolution 120013 MonoImageDepth -113 MonoImageDownsampleThreshold 15000013 EncodeMonoImages true13 MonoImageFilter CCITTFaxEncode13 MonoImageDict ltlt13 K -113 gtgt13 AllowPSXObjects false13 CheckCompliance [13 None13 ]13 PDFX1aCheck false13 PDFX3Check false13 PDFXCompliantPDFOnly false13 PDFXNoTrimBoxError true13 PDFXTrimBoxToMediaBoxOffset [13 00000013 00000013 00000013 00000013 ]13 PDFXSetBleedBoxToMediaBox true13 PDFXBleedBoxToTrimBoxOffset [13 00000013 00000013 00000013 00000013 ]13 PDFXOutputIntentProfile ()13 PDFXOutputConditionIdentifier ()13 PDFXOutputCondition ()13 PDFXRegistryName ()13 PDFXTrapped False1313 CreateJDFFile false13 Description ltlt13 ARA 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 BGR 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 CHS ltFEFF4f7f75288fd94e9b8bbe5b9a521b5efa7684002000410064006f006200650020005000440046002065876863900275284e8e9ad88d2891cf76845370524d53705237300260a853ef4ee54f7f75280020004100630072006f0062006100740020548c002000410064006f00620065002000520065006100640065007200200035002e003000204ee553ca66f49ad87248672c676562535f00521b5efa768400200050004400460020658768633002gt13 CHT ltFEFF4f7f752890194e9b8a2d7f6e5efa7acb7684002000410064006f006200650020005000440046002065874ef69069752865bc9ad854c18cea76845370524d5370523786557406300260a853ef4ee54f7f75280020004100630072006f0062006100740020548c002000410064006f00620065002000520065006100640065007200200035002e003000204ee553ca66f49ad87248672c4f86958b555f5df25efa7acb76840020005000440046002065874ef63002gt13 CZE 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 DAN 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 DEU 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 ESP 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 ETI 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 FRA 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 GRE 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 HEB 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 HRV (Za stvaranje Adobe PDF dokumenata najpogodnijih za visokokvalitetni ispis prije tiskanja koristite ove postavke Stvoreni PDF dokumenti mogu se otvoriti Acrobat i Adobe Reader 50 i kasnijim verzijama)13 HUN 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 ITA 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 JPN ltFEFF9ad854c18cea306a30d730ea30d730ec30b951fa529b7528002000410064006f0062006500200050004400460020658766f8306e4f5c6210306b4f7f75283057307e305930023053306e8a2d5b9a30674f5c62103055308c305f0020005000440046002030d530a130a430eb306f3001004100630072006f0062006100740020304a30883073002000410064006f00620065002000520065006100640065007200200035002e003000204ee5964d3067958b304f30533068304c3067304d307e305930023053306e8a2d5b9a306b306f30d530a930f330c8306e57cb30818fbc307f304c5fc59808306730593002gt13 KOR ltFEFFc7740020c124c815c7440020c0acc6a9d558c5ec0020ace0d488c9c80020c2dcd5d80020c778c1c4c5d00020ac00c7a50020c801d569d55c002000410064006f0062006500200050004400460020bb38c11cb97c0020c791c131d569b2c8b2e4002e0020c774b807ac8c0020c791c131b41c00200050004400460020bb38c11cb2940020004100630072006f0062006100740020bc0f002000410064006f00620065002000520065006100640065007200200035002e00300020c774c0c1c5d0c11c0020c5f40020c2180020c788c2b5b2c8b2e4002egt13 LTH 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 LVI 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 NLD (Gebruik deze instellingen om Adobe PDF-documenten te maken die zijn geoptimaliseerd voor prepress-afdrukken van hoge kwaliteit De gemaakte PDF-documenten kunnen worden geopend met Acrobat en Adobe Reader 50 en hoger)13 NOR 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 POL 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 PTB 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 RUM ltFEFF005500740069006c0069007a00610163006900200061006300650073007400650020007300650074010300720069002000700065006e007400720075002000610020006300720065006100200064006f00630075006d0065006e00740065002000410064006f006200650020005000440046002000610064006500630076006100740065002000700065006e0074007200750020007400690070010300720069007200650061002000700072006500700072006500730073002000640065002000630061006c006900740061007400650020007300750070006500720069006f006100720103002e002000200044006f00630075006d0065006e00740065006c00650020005000440046002000630072006500610074006500200070006f00740020006600690020006400650073006300680069007300650020006300750020004100630072006f006200610074002c002000410064006f00620065002000520065006100640065007200200035002e00300020015f00690020007600650072007300690075006e0069006c006500200075006c0074006500720069006f006100720065002egt13 RUS 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 SKY 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 SLV 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 SUO 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 SVE ltFEFF0041006e007600e4006e00640020006400650020006800e4007200200069006e0073007400e4006c006c006e0069006e006700610072006e00610020006f006d002000640075002000760069006c006c00200073006b006100700061002000410064006f006200650020005000440046002d0064006f006b0075006d0065006e007400200073006f006d002000e400720020006c00e4006d0070006c0069006700610020006600f60072002000700072006500700072006500730073002d007500740073006b00720069006600740020006d006500640020006800f600670020006b00760061006c0069007400650074002e002000200053006b006100700061006400650020005000440046002d0064006f006b0075006d0065006e00740020006b0061006e002000f600700070006e00610073002000690020004100630072006f0062006100740020006f00630068002000410064006f00620065002000520065006100640065007200200035002e00300020006f00630068002000730065006e006100720065002egt13 TUR ltFEFF005900fc006b00730065006b0020006b0061006c006900740065006c0069002000f6006e002000790061007a006401310072006d00610020006200610073006b013100730131006e006100200065006e0020006900790069002000750079006100620069006c006500630065006b002000410064006f006200650020005000440046002000620065006c00670065006c0065007200690020006f006c0075015f007400750072006d0061006b0020006900e70069006e00200062007500200061007900610072006c0061007201310020006b0075006c006c0061006e0131006e002e00200020004f006c0075015f0074007500720075006c0061006e0020005000440046002000620065006c00670065006c0065007200690020004100630072006f006200610074002000760065002000410064006f00620065002000520065006100640065007200200035002e003000200076006500200073006f006e0072006100730131006e00640061006b00690020007300fc007200fc006d006c00650072006c00650020006100e70131006c006100620069006c00690072002egt13 UKR 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 ENU (Use these settings to create Adobe PDF documents best suited for high-quality prepress printing Created PDF documents can be opened with Acrobat and Adobe Reader 50 and later)13 gtgt13 Namespace [13 (Adobe)13 (Common)13 (10)13 ]13 OtherNamespaces [13 ltlt13 AsReaderSpreads false13 CropImagesToFrames true13 ErrorControl WarnAndContinue13 FlattenerIgnoreSpreadOverrides false13 IncludeGuidesGrids false13 IncludeNonPrinting false13 IncludeSlug false13 Namespace [13 (Adobe)13 (InDesign)13 (40)13 ]13 OmitPlacedBitmaps false13 OmitPlacedEPS false13 OmitPlacedPDF false13 SimulateOverprint Legacy13 gtgt13 ltlt13 AddBleedMarks false13 AddColorBars false13 AddCropMarks false13 AddPageInfo false13 AddRegMarks false13 ConvertColors ConvertToCMYK13 DestinationProfileName ()13 DestinationProfileSelector DocumentCMYK13 Downsample16BitImages true13 FlattenerPreset ltlt13 PresetSelector MediumResolution13 gtgt13 FormElements false13 GenerateStructure false13 IncludeBookmarks false13 IncludeHyperlinks false13 IncludeInteractive false13 IncludeLayers false13 IncludeProfiles false13 MultimediaHandling UseObjectSettings13 Namespace [13 (Adobe)13 (CreativeSuite)13 (20)13 ]13 PDFXOutputIntentProfileSelector DocumentCMYK13 PreserveEditing true13 UntaggedCMYKHandling LeaveUntagged13 UntaggedRGBHandling UseDocumentProfile13 UseDocumentBleed false13 gtgt13 ]13gtgt setdistillerparams13ltlt13 HWResolution [2400 2400]13 PageSize [612000 792000]13gtgt setpagedevice13

Page 30: July 2014 MEE Questions and Analyses - NCBE...This publication includes the questions and analyses from the July 2014 MEE. (In the actual test, the questions are simply numbered rather

Evidence Analysis

Evidence of the inmatersquos felony conviction for sexual assault should be excluded because its probative value is substantially outweighed by the danger of unfair prejudice to the inmate based on the heinous nature of the crime FED R EVID 609(a)(1) In the alternative the judge could limit the evidence relating to this conviction by excluding details of the inmatersquos crime

In all civil (and criminal) cases witnesses can also be impeached with evidence of specific instances of prior misconduct that did not result in a conviction FED R EVID 608(b) Pursuant to Rule 608(b) misconduct probative of untruthfulness can be inquired into on cross-examination but cannot be proved through extrinsic evidence Id Thus the inmatersquos counsel should be permitted to cross-examine the guard regarding the false statement in the guardrsquos reacutesumeacute However extrinsic evidence of the guardrsquos misconduct (ie the guardrsquos authenticated reacutesumeacute and transcript from the local college) should not be admitted even if the guard denies wrongdoing or refuses to answer cross-examination questions about these matters Id

Point One (10) The Federal Rules of Evidence permit impeachment of witnesses with evidence of prior convictions

Whether convictions should be admitted to impeach generally depends on the nature of the crime the amount of time that has passed and (only in criminal cases) whether the ldquowitnessrdquo is the defendant FED R EVID 609(a) Under Rule 609(a) evidence of prior convictions may be admitted for the purpose of ldquoattacking a witnessrsquos character for truthfulnessrdquo Id

There are two basic types of convictions that can be admitted for the purpose of impeachment

(1) convictions for crimes ldquopunishable by death or by imprisonment for more than one yearrdquo (which generally correlates to ldquofeloniesrdquo) FED R EVID 609(a)(1) and (2) convictions ldquofor any crimes regardless of the punishment if the court can readily determine that establishing the elements of the crime required provingmdashor the witnessrsquos admittingmdasha dishonest act or false statementrdquo FED R EVID 609(a)(2)

Pursuant to Rule 609(a)(1) in civil cases the admission of evidence of a felony conviction is ldquosubject to Rule 403 [which says that a court may exclude relevant evidence if its probative value is substantially outweighed by other factors]rdquo FED R EVID 609(a)(1) However Rule 403 does not protect the witness against admission of prior convictions involving dishonestymdashwhich must be admitted by the court FED R EVID 609(a)(2)

Finally Federal Rule of Evidence 609(b) contains the presumption that a conviction that is more than 10 years old or where more than 10 years has passed since the witnessrsquos release from confinement (whichever is later) should not be admitted unless ldquoits probative value supported by specific facts and circumstances substantially outweighs its prejudicial effectrdquo and the proponent has provided the adverse party with reasonable written notice FED R EVID 609(b)

Point One(a) (25) The court should admit evidence of the inmatersquos 12-year-old felony marijuana distribution conviction

The inmatersquos conviction for marijuana distribution was for a felony punishable by imprisonment for more than one year See FED R EVID 609(a)(1) Moreover although the conviction was 12 years ago the 10-year time limit of Rule 609(b) is not exceeded because that time limit runs

28

Evidence Analysis

from the date of either ldquothe witnessrsquos conviction or release from confinement for it whichever is laterrdquo FED R EVID 609(b) Because the inmate served three years in prison he was released from confinement nine years ago

However pursuant to Rule 609(a)(1) the admission of felony convictions to impeach a witness in a civil case is ldquosubject to Rule 403rdquo FED R EVID 609(a)(1) Neither Rule 609(a) nor the advisory committee notes specify which factors courts should consider when balancing the probative value of a conviction against the dangers identified in Rule 403 (which include (1) unfair prejudice (2) confusion of the issues (3) misleading the jury (4) waste of time or undue delay and (5) needless presentation of cumulative evidence) FED R EVID 403

In this case credibility is very important because the evidence consists primarily of the testimony of the disputing parties and there were no other eyewitnesses to the altercation This enhances the probative value of any evidence bearing on the inmatersquos credibility A court is likely to conclude that the inmatersquos prior felony drug conviction is relevant to his credibility See eg United States v Brito 427 F3d 53 64 (1st Cir 2005) (ldquoPrior drug-trafficking crimes are generally viewed as having some bearing on veracityrdquo) Although the probative value of any conviction diminishes with age see eg United States v Brewer 451 F Supp 50 53 (ED Tenn 1978) the inmatersquos ongoing problems with the law suggest that he has continued (and even escalated) his criminal behavior over the past nine years The court should admit this evidence because its probative value is not substantially outweighed by any Rule 403 concerns Specifically any prejudice to the inmate would be slight because the conviction is unrelated to the altercation at issue and the conviction was not for a heinous crime that might inflame the jury

[NOTE Whether an examinee identifies the jury instruction as containing a ldquoconclusiverdquo or ldquomandatoryrdquo presumption is less important than the examineersquos analysis of the constitutional infirmities]

Point One(b) (15) The court must admit evidence of the inmatersquos eight-year-old misdemeanor conviction because perjury is a crime of dishonesty

Rule 609(a)(2) provides that evidence of a criminal conviction ldquomust be admitted if the court can readily determine that establishing the elements of the crime required provingmdashor the witnessrsquos admittingmdasha dishonest act or false statementrdquo FED R EVID 609(a)(2) The inmatersquos conviction for perjury would have necessarily required proving that the inmate engaged in an act of dishonesty This conviction occurred within the past 10 years so it ldquomust be admittedrdquo because in contrast to Rule 609(a)(1) (discussed in Point One(a)) admission under Rule 609(a)(2) is mandatory and not subject to Rule 403

Point One(c) (20) The court should exclude evidence of the inmatersquos seven-year-old felony sexual assault conviction because the probative value of this evidence is substantially outweighed by the danger of unfair prejudice In the alternative the details of the prior conviction could be excluded

The inmatersquos conviction for felony sexual assault was seven years ago and he has not yet been released from incarceration so Rule 609(a) but not 609(b) is applicable here FED R EVID 609(a) This conviction is therefore admissible to impeach the inmate unless its probative value is substantially outweighed by the danger of unfair prejudice or any other Rule 403 concern Id

29

Evidence Analysis

Sex crimes are generally not considered relevant to credibility see Hopkins v State 639 So 2d 1247 1254 (Miss 1993) so the probative value of this conviction is relatively low Moreover the heinous nature of the inmatersquos crime (sexual assault on his daughter) makes the danger of unfair prejudice to the inmate very high Thus the court should exclude evidence of the conviction because it was for a heinous offense that is likely to inflame the jury and it has little bearing on credibility See eg United States v Beahm 664 F2d 414 419 (4th Cir 1981)

As an alternative to excluding this evidence the judge could minimize the unfair prejudice to the inmate by permitting limited cross-examination but refusing to allow specific questions about the nature of the inmatersquos conviction For example a court could limit cross-examination to the fact that the inmate was convicted of a ldquofelonyrdquo or perhaps that he was convicted of a ldquosexual assaultrdquo without identifying the victim However because evidence of the inmatersquos prior convictions can be admitted solely for the purpose of enabling the jury to assess his credibility and because his two earlier convictions should have already been admitted the court should exclude all evidence of the felony sexual assault conviction

Point Two(a) (15) The court should permit the inmatersquos counsel to cross-examine the guard regarding the false statement in his reacutesumeacute because the guardrsquos misconduct bears on his truthfulness

The inmate wishes to cross-examine the guard about his prior dishonest behaviormdashlying on his reacutesumeacutemdashthat did not involve a criminal conviction Rule 608(b) allows witnesses to be cross-examined about specific instances of prior non-conviction misconduct probative of untruthfulness ldquoin order to attack the witnessrsquos character for truthfulnessrdquo FED R EVID 608(b)

The courtrsquos decision to allow cross-examination about the guardrsquos prior dishonest behavior depends on the probative value of such evidence balanced against the danger of unfair prejudice to the guard or any other Rule 403 concern FED R EVID 403 Here the guardrsquos false statement on his reacutesumeacute that he obtained a degree in Criminal Justice is highly probative of his untruthfulness because it grossly misrepresents his actual academic record was made recently and was made with the intent to deceive Because the probative value of this evidence is very strong and is not substantially outweighed by any Rule 403 concerns cross-examination of the guard on this topic should be permitted The court may also consider it fair to permit this cross-examination of the guard on these matters assuming that one or more of the inmatersquos prior convictions have been admitted to impeach his credibility

Point Two(b) (15) The court should exclude extrinsic evidence of the guardrsquos non-conviction misconduct even if the guard denies wrongdoing or refuses to answer questions about the matter

Although Rule 608(b) allows cross-examination about specific instances of prior misconduct probative of untruthfulness ldquoextrinsic evidencerdquo offered to prove such misconduct is not admissible FED R EVID 608(b) The rationale for this rule is that allowing the introduction of extrinsic evidence of prior misconduct by witnesses when these acts are relevant only to the witnessesrsquo truthfulness and not to the main issues in the case would create too great a risk of confusing the jury and unduly delaying the trial The court does not have discretion to admit this extrinsic evidence See eg United States v Elliot 89 F3d 1360 1368 (8th Cir 1996)

30

Evidence Analysis

Here the inmatersquos counsel may cross-examine the guard about the false statement on his reacutesumeacute However the inmatersquos counsel must accept the guardrsquos response Even if the guard denies wrongdoing or refuses to answer questions about the matter the inmatersquos counsel cannot introduce the guardrsquos reacutesumeacute or the transcript from the local college to prove the guardrsquos misconduct

31

CORPORATIONS ANALYSIS (Corporations VA2 IX)

ANALYSIS

Legal Problems

(1) Do shareholders have the authority to amend a corporationrsquos bylaws with respect to director nominations

(2) Do board-approved bylaws on a particular subject here nomination of directors preempt subsequent conflicting bylaw amendments by shareholders

(3) Is a suit challenging both managementrsquos refusal to include the proposed bylaw amendment in Megarsquos proxy statement and the boardrsquos amendment of the bylaws dealing with nomination of directors a direct or derivative suit

DISCUSSION

Summary

The voting and litigation rights of the shareholders of Mega are subject to the provisions of the Model Business Corporations Act (MBCA)

The investorrsquos proposed bylaw provision is not inconsistent with state law Under the MBCA shareholders may amend the bylaws when the amendment deals with a proper matter for the corporationrsquos bylaws such as procedures for nominating directors

The Mega boardrsquos bylaw amendment does not preempt the investorrsquos proposed bylaw provision or the Mega shareholdersrsquo power to approve it While shareholders can limit the boardrsquos power to amend or repeal the bylaws the board cannot limit the shareholdersrsquo power

Whether the investor must make a demand on Megarsquos board depends on how the investor frames its claim If the investor claims a violation of shareholder voting rights the claim is direct and pre-suit demand on the board is not required If on the other hand the investor claims that the directors violated their fiduciary duties by amending the bylaws to entrench themselves the claim is derivative and a pre-suit demand is required

Point One (30) Shareholders may amend the corporationrsquos bylaws where the proposed bylaw provision relates to procedural matters typically included in the bylaws such as the nomination of directors

Internal affairs of the corporation such as the conduct of shareholder meetings and election of directors are subject to the corporate law of the state of incorporation See McDermott Inc v Lewis 531 A2d 206 (Del 1987) (applying law of jurisdiction where corporation was incorporated in case involving voting rights) This statersquos corporate statute is modeled on the MBCA

Under the MBCA ldquoshareholders may amend the corporationrsquos bylawsrdquo MBCA sect 1020(a) Thus the only question is whether the bylaws can specify the procedures for shareholder nomination of directors

32

Corporations Analysis

The MBCA states that the bylaws ldquomay contain any provision that is not inconsistent with law or the articles of incorporationrdquo MBCA sect 206(b) In addition the MBCA was revised in 2009 to address shareholder nomination of directors in public corporations (known as ldquoproxy accessrdquo) and specifies that the bylaws ldquomay contain a requirement that the corporation include in its [proxy materials] one or more individuals nominated by a shareholderrdquo MBCA sect 206(c)(1) see Committee on Corporate Laws ABA Section of Business Law Report on the Roles of Boards of Directors and Shareholders of Publicly Owned Corporations and Changes to the Model Business Corporations ActmdashAdoption of Shareholder Proxy Access Amendments to Chapters 2 and 10 65 BUS LAWYER 1105 (2010)

The inclusion of director-nomination procedures in the bylaws is consistent with practice and is recognized by the Delaware courts whose views on corporate law carry significant weight Typically the procedures for nomination of directors are found in the bylaws See 1 COX amp HAZEN TREATISE ON THE LAW OF CORPORATIONS sect 312 (3d ed 2011) see also 4 FLETCHER CORP FORMS ANN PART III ch 21 (2013) (including sample bylaws that permit nomination of directors by shareholders) The Delaware Supreme Court has confirmed that the bylaws may ldquodefine the process and proceduresrdquo for director elections See CA Inc v AFSCME Employees Pension Plan 953 A2d 227 (Del 2008) (concluding that bylaw amendment requiring reimbursement of election expenses to certain successful shareholder nominators is ldquoproper subjectrdquo under Delaware law)

[NOTE The question of the proper scope of the bylaws can be answered using the more general MBCA sect 206(b) or the 2009 MBCA revision adding sect 206(c)(1) (adopted in CT ME VA) In addition some examinees might raise the point that shareholder proposals may not compel the board to take action such as by including shareholder nominations in the companyrsquos proxy materials on the theory that the ldquobusiness and affairsrdquo of the corporation are to be managed by the board See MBCA sect 801(b) Although shareholders are generally limited to adopting precatory resolutions that recommend or encourage board action this limitation does not apply when shareholders have specific authority to take binding action on their ownmdashsuch as to amend the bylaws]

Point Two (30) Shareholders can amend (or repeal) board-approved bylaws Further shareholders can limit the boardrsquos power to later amend and repeal a shareholder-approved bylaw

Under the MBCA shareholders have the power to amend the bylaws See Point One The board shares this power with the shareholders unless (1) the corporationrsquos articles ldquoreserve that power exclusively to the shareholdersrdquo or (2) ldquothe shareholders in amending repealing or adopting a bylaw expressly provide that the board of directors may not amend repeal or reinstate that bylawrdquo See MBCA sect 1020(b)

Shareholder-approved bylaw provisions can amend or repeal existing bylaw provisions whether originally approved by the board or by shareholders See ALAN R PALMITER CORPORATIONS EXAMPLES AND EXPLANATIONS sect 713 (7th ed 2012) Thus the Mega boardrsquos bylaw amendmentmdashwhich set more demanding thresholds for shareholder nomination of directors than the investorrsquos proposed bylaw provisionmdashwould be superseded (repealed) if Megarsquos shareholders were to approve the investorrsquos proposal

Further a shareholder-approved bylaw generally can limit the power of the board to later amend or repeal it See MBCA sect 1020(b)(2) Thus if Megarsquos shareholders approved the bylaw

33

Corporations Analysis

provision proposed by the investor Megarsquos board could not repeal the provision because it includes a ldquono board repealrdquo clause

The revision to the MBCA in 2009 dealing with shareholder proxy access does not change this conclusion That revision specifies that a shareholder-approved bylaw dealing with director nominations may not limit the boardrsquos power to amend add or repeal ldquoany procedure or condition to such a bylaw in order to provide for a reasonable practicable and orderly processrdquo MBCA sect 206(d) Thus according to the revision if shareholders approve a bylaw amendment that limits further board changes the board would nonetheless retain the power to ldquotinkerrdquo with the bylaw to safeguard the voting process but could not repeal the shareholder-approved bylaw The Official Comment to MBCA sect 206(d) makes clear that the revision is ldquonot intended to allow the board of directors to frustrate the purpose of the shareholder-adopted proxy access provisionrdquo Thus if Megarsquos shareholders were to approve the bylaw provision proposed by the investor Megarsquos board could only amend the provision regarding its procedures or conditions in a manner consistent with its purpose of permitting proxy access for Megarsquos shareholders

[NOTE The boardrsquos attempted interference with a shareholder voting initiative may also have been a violation of the directorsrsquo fiduciary duties See Blasius Indus Inc v Atlas Corp 564 A2d 651 (Del Ch 1988) (finding that directors breached their fiduciary duties by amending bylaws and expanding size of board to thwart insurgentrsquos plan to amend bylaws and seat a majority of new directors) The call however asks examinees to consider whether shareholders or the board have ldquoprecedencerdquo over amending the corporate bylaws Thus an examineersquos answer should be framed in terms of ldquopowerrdquo and not ldquodutyrdquo]

Point Three (40) The investor need not make a demand on the board if the investor states a direct claim such as an allegation that the board interfered with the investorrsquos right to amend the bylaws But the investor must make a demand on the board if the investor states a derivative claim (on behalf of the corporation) such as an allegation that the directors sought to entrench themselves by interfering with the proposed proxy access

The MBCA generally requires that shareholders make a demand on the board of directors before initiation of a derivative suit MBCA sect 742 (shareholder may not bring derivative proceeding until written demand has been made on corporation and 90 days have expired) A derivative suit is essentially two suits in one where the plaintiff-shareholder seeks to bring on behalf of the corporation a claim that vindicates corporate rights usually based on violation of fiduciary duties PALMITER supra sect 1811 (6th ed 2009) The demand permits the board to investigate the situation identified by the shareholder and take suitable action No demand on the board is required however if the shareholder brings a direct suit to vindicate the shareholderrsquos own rights not those of the corporation

Is the suit brought by the investor derivative or direct The MBCA defines a ldquoderivative proceedingrdquo as one brought ldquoin the right of a domestic corporationrdquo MBCA sect 740(1) Thus the answer to how the investorrsquos suit should be characterized turns on what rights the investor seeks to vindicate If the investor frames its claim as one of fiduciary breach by directorsmdashfor example for failing to become adequately informed about voting procedures or for seeking to entrench themselves in office by manipulating the voting structure to avoid a shareholder insurgencymdashthen the suit is ldquoderivativerdquo and the investor must make a demand on the board See MBCA Ch 7 Subch D Introductory Comment (ldquothe derivative suit has historically been the principal method of challenging allegedly illegal action by managementrdquo)

34

Corporations Analysis

If however the investor frames its claim as one to vindicate shareholder rights the suit is direct and no demand is required For many courts the direct-derivative question turns on who is injured and who is to receive the relief sought by the plaintiff-shareholders See Tooley v Donaldson Lufkin amp Jenrette Inc 845 A2d 1031 (Del 2004) (characterizing a merger-delay claim as direct because delay of merger only harmed shareholders not corporation) Thus if the investor claims that managementrsquos refusal to include its proposed bylaw amendment in the corporationrsquos proxy materials violates its shareholder rights to initiate corporate governance reforms the suit will be direct Courts have not questioned the ability of shareholders to bring direct suits challenging board action to exclude their proposed bylaw amendments from the corporationrsquos proxy materials See JANA Master Fund Ltd v CNET Networks Inc 954 A2d 335 (Del Ch 2008) (upholding shareholderrsquos direct challenge to boardrsquos interpretation of advance-notice bylaw) Chesapeake Corp v Shore 771 A2d 293 (Del Ch 2000) (upholding shareholderrsquos direct challenge to actions by board that effectively prevented it from proposing bylaw amendments in contest for control)

Is the way that the investor frames its claim conclusive Courts have permitted shareholder-plaintiffs to challenge a transaction in a direct suit even though the same transaction could also be challenged as a fiduciary breach See Eisenberg v Flying Tiger Line Inc 451 F2d 267 (2d Cir 1971) (permitting direct suit challenging a corporate reorganization as a dilution of shareholder voting power even though reorganization may have involved conflicts of interest and thus constituted a fiduciary breach) Thus the investorrsquos choice to pursue a claim challenging the legality of managementrsquos decision to exclude the investorrsquos proposal from the corporationrsquos proxy materialsmdashrather than a possible breach of fiduciary dutymdashis likely to be respected See 3 COX amp HAZEN supra sect 153 (describing situations in which a claim can be framed as derivative or direct)

[NOTE Some issues under Delaware corporate law regarding pre-suit demand are not relevant here For example whether the Mega directors are independent and disinterested is not relevant to the MBCA requirement of a pre-suit demand As the Official Comment to MBCA sect 742 points out the MBCArsquos requirement of ldquouniversal demandrdquo gives the board ldquothe opportunity to reexamine the act complained of in the light of a potential lawsuit and take corrective actionrdquo even when the directors might be non-independent or have conflicts of interest

Nor is it relevant to the MBCA pre-suit demand requirement that the statutory 90-day waiting period may be onerous The first paragraph of MBCA sect 742 requires a pre-suit demand without exception the second paragraph of the section imposes a 90-day waiting period before a derivative suit may be brought which can be shortened if the board rejects the demand or ldquoirreparable injury to the corporation would result by waiting for the expiration of the 90-day periodrdquo The call as written asks only whether a pre-suit demand should be made and does not ask examinees to address whether the post-demand waiting period should be shortened under the ldquoirreparable injuryrdquo standard]

35

National Conference of Bar Examiners 302 South Bedford Street | Madison WI 53703-3622 Phone 608-280-8550 | Fax 608-280-8552 | TDD 608-661-1275

wwwncbexorg e-mail contactncbexorg

  • Preface
  • Description of the MEE
  • Instructions
  • July 2014 Questions
    • CRIMINAL LAW AND PROCEDURE QUESTION
    • CONTRACTS QUESTION
    • FAMILY LAW QUESTION
    • FEDERAL CIVIL PROCEDURE QUESTION
    • EVIDENCE QUESTION
    • CORPORATIONS QUESTION
      • July 2014 Analyses
        • CRIMINAL LAW AND PROCEDURE ANALYSIS
        • CONTRACTS ANALYSIS
        • FAMILY LAW ANALYSIS
        • FEDERAL CIVIL PROCEDURE ANALYSIS
        • EVIDENCE ANALYSIS
        • CORPORATIONS ANALYSIS
            • ltlt13 ASCII85EncodePages false13 AllowTransparency false13 AutoPositionEPSFiles true13 AutoRotatePages None13 Binding Left13 CalGrayProfile (Dot Gain 20)13 CalRGBProfile (sRGB IEC61966-21)13 CalCMYKProfile (US Web Coated 050SWOP051 v2)13 sRGBProfile (sRGB IEC61966-21)13 CannotEmbedFontPolicy Error13 CompatibilityLevel 1413 CompressObjects Tags13 CompressPages true13 ConvertImagesToIndexed true13 PassThroughJPEGImages true13 CreateJobTicket false13 DefaultRenderingIntent Default13 DetectBlends true13 DetectCurves 0000013 ColorConversionStrategy CMYK13 DoThumbnails false13 EmbedAllFonts true13 EmbedOpenType false13 ParseICCProfilesInComments true13 EmbedJobOptions true13 DSCReportingLevel 013 EmitDSCWarnings false13 EndPage -113 ImageMemory 104857613 LockDistillerParams false13 MaxSubsetPct 10013 Optimize true13 OPM 113 ParseDSCComments true13 ParseDSCCommentsForDocInfo true13 PreserveCopyPage true13 PreserveDICMYKValues true13 PreserveEPSInfo true13 PreserveFlatness true13 PreserveHalftoneInfo false13 PreserveOPIComments true13 PreserveOverprintSettings true13 StartPage 113 SubsetFonts true13 TransferFunctionInfo Apply13 UCRandBGInfo Preserve13 UsePrologue false13 ColorSettingsFile ()13 AlwaysEmbed [ true13 ]13 NeverEmbed [ true13 ]13 AntiAliasColorImages false13 CropColorImages true13 ColorImageMinResolution 30013 ColorImageMinResolutionPolicy OK13 DownsampleColorImages true13 ColorImageDownsampleType Bicubic13 ColorImageResolution 30013 ColorImageDepth -113 ColorImageMinDownsampleDepth 113 ColorImageDownsampleThreshold 15000013 EncodeColorImages true13 ColorImageFilter DCTEncode13 AutoFilterColorImages true13 ColorImageAutoFilterStrategy JPEG13 ColorACSImageDict ltlt13 QFactor 01513 HSamples [1 1 1 1] VSamples [1 1 1 1]13 gtgt13 ColorImageDict ltlt13 QFactor 01513 HSamples [1 1 1 1] VSamples [1 1 1 1]13 gtgt13 JPEG2000ColorACSImageDict ltlt13 TileWidth 25613 TileHeight 25613 Quality 3013 gtgt13 JPEG2000ColorImageDict ltlt13 TileWidth 25613 TileHeight 25613 Quality 3013 gtgt13 AntiAliasGrayImages false13 CropGrayImages true13 GrayImageMinResolution 30013 GrayImageMinResolutionPolicy OK13 DownsampleGrayImages true13 GrayImageDownsampleType Bicubic13 GrayImageResolution 30013 GrayImageDepth -113 GrayImageMinDownsampleDepth 213 GrayImageDownsampleThreshold 15000013 EncodeGrayImages true13 GrayImageFilter DCTEncode13 AutoFilterGrayImages true13 GrayImageAutoFilterStrategy JPEG13 GrayACSImageDict ltlt13 QFactor 01513 HSamples [1 1 1 1] VSamples [1 1 1 1]13 gtgt13 GrayImageDict ltlt13 QFactor 01513 HSamples [1 1 1 1] VSamples [1 1 1 1]13 gtgt13 JPEG2000GrayACSImageDict ltlt13 TileWidth 25613 TileHeight 25613 Quality 3013 gtgt13 JPEG2000GrayImageDict ltlt13 TileWidth 25613 TileHeight 25613 Quality 3013 gtgt13 AntiAliasMonoImages false13 CropMonoImages true13 MonoImageMinResolution 120013 MonoImageMinResolutionPolicy OK13 DownsampleMonoImages true13 MonoImageDownsampleType Bicubic13 MonoImageResolution 120013 MonoImageDepth -113 MonoImageDownsampleThreshold 15000013 EncodeMonoImages true13 MonoImageFilter CCITTFaxEncode13 MonoImageDict ltlt13 K -113 gtgt13 AllowPSXObjects false13 CheckCompliance [13 None13 ]13 PDFX1aCheck false13 PDFX3Check false13 PDFXCompliantPDFOnly false13 PDFXNoTrimBoxError true13 PDFXTrimBoxToMediaBoxOffset [13 00000013 00000013 00000013 00000013 ]13 PDFXSetBleedBoxToMediaBox true13 PDFXBleedBoxToTrimBoxOffset [13 00000013 00000013 00000013 00000013 ]13 PDFXOutputIntentProfile ()13 PDFXOutputConditionIdentifier ()13 PDFXOutputCondition ()13 PDFXRegistryName ()13 PDFXTrapped False1313 CreateJDFFile false13 Description ltlt13 ARA 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 BGR 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 CHS ltFEFF4f7f75288fd94e9b8bbe5b9a521b5efa7684002000410064006f006200650020005000440046002065876863900275284e8e9ad88d2891cf76845370524d53705237300260a853ef4ee54f7f75280020004100630072006f0062006100740020548c002000410064006f00620065002000520065006100640065007200200035002e003000204ee553ca66f49ad87248672c676562535f00521b5efa768400200050004400460020658768633002gt13 CHT ltFEFF4f7f752890194e9b8a2d7f6e5efa7acb7684002000410064006f006200650020005000440046002065874ef69069752865bc9ad854c18cea76845370524d5370523786557406300260a853ef4ee54f7f75280020004100630072006f0062006100740020548c002000410064006f00620065002000520065006100640065007200200035002e003000204ee553ca66f49ad87248672c4f86958b555f5df25efa7acb76840020005000440046002065874ef63002gt13 CZE 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 DAN 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 DEU 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 ESP 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 ETI 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 FRA 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 GRE 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 HEB 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 HRV (Za stvaranje Adobe PDF dokumenata najpogodnijih za visokokvalitetni ispis prije tiskanja koristite ove postavke Stvoreni PDF dokumenti mogu se otvoriti Acrobat i Adobe Reader 50 i kasnijim verzijama)13 HUN 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 ITA 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 JPN ltFEFF9ad854c18cea306a30d730ea30d730ec30b951fa529b7528002000410064006f0062006500200050004400460020658766f8306e4f5c6210306b4f7f75283057307e305930023053306e8a2d5b9a30674f5c62103055308c305f0020005000440046002030d530a130a430eb306f3001004100630072006f0062006100740020304a30883073002000410064006f00620065002000520065006100640065007200200035002e003000204ee5964d3067958b304f30533068304c3067304d307e305930023053306e8a2d5b9a306b306f30d530a930f330c8306e57cb30818fbc307f304c5fc59808306730593002gt13 KOR ltFEFFc7740020c124c815c7440020c0acc6a9d558c5ec0020ace0d488c9c80020c2dcd5d80020c778c1c4c5d00020ac00c7a50020c801d569d55c002000410064006f0062006500200050004400460020bb38c11cb97c0020c791c131d569b2c8b2e4002e0020c774b807ac8c0020c791c131b41c00200050004400460020bb38c11cb2940020004100630072006f0062006100740020bc0f002000410064006f00620065002000520065006100640065007200200035002e00300020c774c0c1c5d0c11c0020c5f40020c2180020c788c2b5b2c8b2e4002egt13 LTH 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 LVI 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 NLD (Gebruik deze instellingen om Adobe PDF-documenten te maken die zijn geoptimaliseerd voor prepress-afdrukken van hoge kwaliteit De gemaakte PDF-documenten kunnen worden geopend met Acrobat en Adobe Reader 50 en hoger)13 NOR 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 POL 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 PTB ltFEFF005500740069006c0069007a006500200065007300730061007300200063006f006e00660069006700750072006100e700f50065007300200064006500200066006f0072006d00610020006100200063007200690061007200200064006f00630075006d0065006e0074006f0073002000410064006f0062006500200050004400460020006d00610069007300200061006400650071007500610064006f00730020007000610072006100200070007200e9002d0069006d0070007200650073007300f50065007300200064006500200061006c007400610020007100750061006c00690064006100640065002e0020004f007300200064006f00630075006d0065006e0074006f00730020005000440046002000630072006900610064006f007300200070006f00640065006d0020007300650072002000610062006500720074006f007300200063006f006d0020006f0020004100630072006f006200610074002000650020006f002000410064006f00620065002000520065006100640065007200200035002e0030002000650020007600650072007300f50065007300200070006f00730074006500720069006f007200650073002egt13 RUM 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 RUS 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 SKY ltFEFF0054006900650074006f0020006e006100730074006100760065006e0069006100200070006f0075017e0069007400650020006e00610020007600790074007600e100720061006e0069006500200064006f006b0075006d0065006e0074006f0076002000410064006f006200650020005000440046002c0020006b0074006f007200e90020007300610020006e0061006a006c0065007001610069006500200068006f0064006900610020006e00610020006b00760061006c00690074006e00fa00200074006c0061010d00200061002000700072006500700072006500730073002e00200056007900740076006f00720065006e00e900200064006f006b0075006d0065006e007400790020005000440046002000620075006400650020006d006f017e006e00e90020006f00740076006f00720069016500200076002000700072006f006700720061006d006f006300680020004100630072006f00620061007400200061002000410064006f00620065002000520065006100640065007200200035002e0030002000610020006e006f0076016100ed00630068002egt13 SLV 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 SUO 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 SVE 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 TUR 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 UKR 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 ENU (Use these settings to create Adobe PDF documents best suited for high-quality prepress printing Created PDF documents can be opened with Acrobat and Adobe Reader 50 and later)13 gtgt13 Namespace [13 (Adobe)13 (Common)13 (10)13 ]13 OtherNamespaces [13 ltlt13 AsReaderSpreads false13 CropImagesToFrames true13 ErrorControl WarnAndContinue13 FlattenerIgnoreSpreadOverrides false13 IncludeGuidesGrids false13 IncludeNonPrinting false13 IncludeSlug false13 Namespace [13 (Adobe)13 (InDesign)13 (40)13 ]13 OmitPlacedBitmaps false13 OmitPlacedEPS false13 OmitPlacedPDF false13 SimulateOverprint Legacy13 gtgt13 ltlt13 AddBleedMarks false13 AddColorBars false13 AddCropMarks false13 AddPageInfo false13 AddRegMarks false13 ConvertColors ConvertToCMYK13 DestinationProfileName ()13 DestinationProfileSelector DocumentCMYK13 Downsample16BitImages true13 FlattenerPreset ltlt13 PresetSelector MediumResolution13 gtgt13 FormElements false13 GenerateStructure false13 IncludeBookmarks false13 IncludeHyperlinks false13 IncludeInteractive false13 IncludeLayers false13 IncludeProfiles false13 MultimediaHandling UseObjectSettings13 Namespace [13 (Adobe)13 (CreativeSuite)13 (20)13 ]13 PDFXOutputIntentProfileSelector DocumentCMYK13 PreserveEditing true13 UntaggedCMYKHandling LeaveUntagged13 UntaggedRGBHandling UseDocumentProfile13 UseDocumentBleed false13 gtgt13 ]13gtgt setdistillerparams13ltlt13 HWResolution [2400 2400]13 PageSize [612000 792000]13gtgt setpagedevice13

Page 31: July 2014 MEE Questions and Analyses - NCBE...This publication includes the questions and analyses from the July 2014 MEE. (In the actual test, the questions are simply numbered rather

Evidence Analysis

from the date of either ldquothe witnessrsquos conviction or release from confinement for it whichever is laterrdquo FED R EVID 609(b) Because the inmate served three years in prison he was released from confinement nine years ago

However pursuant to Rule 609(a)(1) the admission of felony convictions to impeach a witness in a civil case is ldquosubject to Rule 403rdquo FED R EVID 609(a)(1) Neither Rule 609(a) nor the advisory committee notes specify which factors courts should consider when balancing the probative value of a conviction against the dangers identified in Rule 403 (which include (1) unfair prejudice (2) confusion of the issues (3) misleading the jury (4) waste of time or undue delay and (5) needless presentation of cumulative evidence) FED R EVID 403

In this case credibility is very important because the evidence consists primarily of the testimony of the disputing parties and there were no other eyewitnesses to the altercation This enhances the probative value of any evidence bearing on the inmatersquos credibility A court is likely to conclude that the inmatersquos prior felony drug conviction is relevant to his credibility See eg United States v Brito 427 F3d 53 64 (1st Cir 2005) (ldquoPrior drug-trafficking crimes are generally viewed as having some bearing on veracityrdquo) Although the probative value of any conviction diminishes with age see eg United States v Brewer 451 F Supp 50 53 (ED Tenn 1978) the inmatersquos ongoing problems with the law suggest that he has continued (and even escalated) his criminal behavior over the past nine years The court should admit this evidence because its probative value is not substantially outweighed by any Rule 403 concerns Specifically any prejudice to the inmate would be slight because the conviction is unrelated to the altercation at issue and the conviction was not for a heinous crime that might inflame the jury

[NOTE Whether an examinee identifies the jury instruction as containing a ldquoconclusiverdquo or ldquomandatoryrdquo presumption is less important than the examineersquos analysis of the constitutional infirmities]

Point One(b) (15) The court must admit evidence of the inmatersquos eight-year-old misdemeanor conviction because perjury is a crime of dishonesty

Rule 609(a)(2) provides that evidence of a criminal conviction ldquomust be admitted if the court can readily determine that establishing the elements of the crime required provingmdashor the witnessrsquos admittingmdasha dishonest act or false statementrdquo FED R EVID 609(a)(2) The inmatersquos conviction for perjury would have necessarily required proving that the inmate engaged in an act of dishonesty This conviction occurred within the past 10 years so it ldquomust be admittedrdquo because in contrast to Rule 609(a)(1) (discussed in Point One(a)) admission under Rule 609(a)(2) is mandatory and not subject to Rule 403

Point One(c) (20) The court should exclude evidence of the inmatersquos seven-year-old felony sexual assault conviction because the probative value of this evidence is substantially outweighed by the danger of unfair prejudice In the alternative the details of the prior conviction could be excluded

The inmatersquos conviction for felony sexual assault was seven years ago and he has not yet been released from incarceration so Rule 609(a) but not 609(b) is applicable here FED R EVID 609(a) This conviction is therefore admissible to impeach the inmate unless its probative value is substantially outweighed by the danger of unfair prejudice or any other Rule 403 concern Id

29

Evidence Analysis

Sex crimes are generally not considered relevant to credibility see Hopkins v State 639 So 2d 1247 1254 (Miss 1993) so the probative value of this conviction is relatively low Moreover the heinous nature of the inmatersquos crime (sexual assault on his daughter) makes the danger of unfair prejudice to the inmate very high Thus the court should exclude evidence of the conviction because it was for a heinous offense that is likely to inflame the jury and it has little bearing on credibility See eg United States v Beahm 664 F2d 414 419 (4th Cir 1981)

As an alternative to excluding this evidence the judge could minimize the unfair prejudice to the inmate by permitting limited cross-examination but refusing to allow specific questions about the nature of the inmatersquos conviction For example a court could limit cross-examination to the fact that the inmate was convicted of a ldquofelonyrdquo or perhaps that he was convicted of a ldquosexual assaultrdquo without identifying the victim However because evidence of the inmatersquos prior convictions can be admitted solely for the purpose of enabling the jury to assess his credibility and because his two earlier convictions should have already been admitted the court should exclude all evidence of the felony sexual assault conviction

Point Two(a) (15) The court should permit the inmatersquos counsel to cross-examine the guard regarding the false statement in his reacutesumeacute because the guardrsquos misconduct bears on his truthfulness

The inmate wishes to cross-examine the guard about his prior dishonest behaviormdashlying on his reacutesumeacutemdashthat did not involve a criminal conviction Rule 608(b) allows witnesses to be cross-examined about specific instances of prior non-conviction misconduct probative of untruthfulness ldquoin order to attack the witnessrsquos character for truthfulnessrdquo FED R EVID 608(b)

The courtrsquos decision to allow cross-examination about the guardrsquos prior dishonest behavior depends on the probative value of such evidence balanced against the danger of unfair prejudice to the guard or any other Rule 403 concern FED R EVID 403 Here the guardrsquos false statement on his reacutesumeacute that he obtained a degree in Criminal Justice is highly probative of his untruthfulness because it grossly misrepresents his actual academic record was made recently and was made with the intent to deceive Because the probative value of this evidence is very strong and is not substantially outweighed by any Rule 403 concerns cross-examination of the guard on this topic should be permitted The court may also consider it fair to permit this cross-examination of the guard on these matters assuming that one or more of the inmatersquos prior convictions have been admitted to impeach his credibility

Point Two(b) (15) The court should exclude extrinsic evidence of the guardrsquos non-conviction misconduct even if the guard denies wrongdoing or refuses to answer questions about the matter

Although Rule 608(b) allows cross-examination about specific instances of prior misconduct probative of untruthfulness ldquoextrinsic evidencerdquo offered to prove such misconduct is not admissible FED R EVID 608(b) The rationale for this rule is that allowing the introduction of extrinsic evidence of prior misconduct by witnesses when these acts are relevant only to the witnessesrsquo truthfulness and not to the main issues in the case would create too great a risk of confusing the jury and unduly delaying the trial The court does not have discretion to admit this extrinsic evidence See eg United States v Elliot 89 F3d 1360 1368 (8th Cir 1996)

30

Evidence Analysis

Here the inmatersquos counsel may cross-examine the guard about the false statement on his reacutesumeacute However the inmatersquos counsel must accept the guardrsquos response Even if the guard denies wrongdoing or refuses to answer questions about the matter the inmatersquos counsel cannot introduce the guardrsquos reacutesumeacute or the transcript from the local college to prove the guardrsquos misconduct

31

CORPORATIONS ANALYSIS (Corporations VA2 IX)

ANALYSIS

Legal Problems

(1) Do shareholders have the authority to amend a corporationrsquos bylaws with respect to director nominations

(2) Do board-approved bylaws on a particular subject here nomination of directors preempt subsequent conflicting bylaw amendments by shareholders

(3) Is a suit challenging both managementrsquos refusal to include the proposed bylaw amendment in Megarsquos proxy statement and the boardrsquos amendment of the bylaws dealing with nomination of directors a direct or derivative suit

DISCUSSION

Summary

The voting and litigation rights of the shareholders of Mega are subject to the provisions of the Model Business Corporations Act (MBCA)

The investorrsquos proposed bylaw provision is not inconsistent with state law Under the MBCA shareholders may amend the bylaws when the amendment deals with a proper matter for the corporationrsquos bylaws such as procedures for nominating directors

The Mega boardrsquos bylaw amendment does not preempt the investorrsquos proposed bylaw provision or the Mega shareholdersrsquo power to approve it While shareholders can limit the boardrsquos power to amend or repeal the bylaws the board cannot limit the shareholdersrsquo power

Whether the investor must make a demand on Megarsquos board depends on how the investor frames its claim If the investor claims a violation of shareholder voting rights the claim is direct and pre-suit demand on the board is not required If on the other hand the investor claims that the directors violated their fiduciary duties by amending the bylaws to entrench themselves the claim is derivative and a pre-suit demand is required

Point One (30) Shareholders may amend the corporationrsquos bylaws where the proposed bylaw provision relates to procedural matters typically included in the bylaws such as the nomination of directors

Internal affairs of the corporation such as the conduct of shareholder meetings and election of directors are subject to the corporate law of the state of incorporation See McDermott Inc v Lewis 531 A2d 206 (Del 1987) (applying law of jurisdiction where corporation was incorporated in case involving voting rights) This statersquos corporate statute is modeled on the MBCA

Under the MBCA ldquoshareholders may amend the corporationrsquos bylawsrdquo MBCA sect 1020(a) Thus the only question is whether the bylaws can specify the procedures for shareholder nomination of directors

32

Corporations Analysis

The MBCA states that the bylaws ldquomay contain any provision that is not inconsistent with law or the articles of incorporationrdquo MBCA sect 206(b) In addition the MBCA was revised in 2009 to address shareholder nomination of directors in public corporations (known as ldquoproxy accessrdquo) and specifies that the bylaws ldquomay contain a requirement that the corporation include in its [proxy materials] one or more individuals nominated by a shareholderrdquo MBCA sect 206(c)(1) see Committee on Corporate Laws ABA Section of Business Law Report on the Roles of Boards of Directors and Shareholders of Publicly Owned Corporations and Changes to the Model Business Corporations ActmdashAdoption of Shareholder Proxy Access Amendments to Chapters 2 and 10 65 BUS LAWYER 1105 (2010)

The inclusion of director-nomination procedures in the bylaws is consistent with practice and is recognized by the Delaware courts whose views on corporate law carry significant weight Typically the procedures for nomination of directors are found in the bylaws See 1 COX amp HAZEN TREATISE ON THE LAW OF CORPORATIONS sect 312 (3d ed 2011) see also 4 FLETCHER CORP FORMS ANN PART III ch 21 (2013) (including sample bylaws that permit nomination of directors by shareholders) The Delaware Supreme Court has confirmed that the bylaws may ldquodefine the process and proceduresrdquo for director elections See CA Inc v AFSCME Employees Pension Plan 953 A2d 227 (Del 2008) (concluding that bylaw amendment requiring reimbursement of election expenses to certain successful shareholder nominators is ldquoproper subjectrdquo under Delaware law)

[NOTE The question of the proper scope of the bylaws can be answered using the more general MBCA sect 206(b) or the 2009 MBCA revision adding sect 206(c)(1) (adopted in CT ME VA) In addition some examinees might raise the point that shareholder proposals may not compel the board to take action such as by including shareholder nominations in the companyrsquos proxy materials on the theory that the ldquobusiness and affairsrdquo of the corporation are to be managed by the board See MBCA sect 801(b) Although shareholders are generally limited to adopting precatory resolutions that recommend or encourage board action this limitation does not apply when shareholders have specific authority to take binding action on their ownmdashsuch as to amend the bylaws]

Point Two (30) Shareholders can amend (or repeal) board-approved bylaws Further shareholders can limit the boardrsquos power to later amend and repeal a shareholder-approved bylaw

Under the MBCA shareholders have the power to amend the bylaws See Point One The board shares this power with the shareholders unless (1) the corporationrsquos articles ldquoreserve that power exclusively to the shareholdersrdquo or (2) ldquothe shareholders in amending repealing or adopting a bylaw expressly provide that the board of directors may not amend repeal or reinstate that bylawrdquo See MBCA sect 1020(b)

Shareholder-approved bylaw provisions can amend or repeal existing bylaw provisions whether originally approved by the board or by shareholders See ALAN R PALMITER CORPORATIONS EXAMPLES AND EXPLANATIONS sect 713 (7th ed 2012) Thus the Mega boardrsquos bylaw amendmentmdashwhich set more demanding thresholds for shareholder nomination of directors than the investorrsquos proposed bylaw provisionmdashwould be superseded (repealed) if Megarsquos shareholders were to approve the investorrsquos proposal

Further a shareholder-approved bylaw generally can limit the power of the board to later amend or repeal it See MBCA sect 1020(b)(2) Thus if Megarsquos shareholders approved the bylaw

33

Corporations Analysis

provision proposed by the investor Megarsquos board could not repeal the provision because it includes a ldquono board repealrdquo clause

The revision to the MBCA in 2009 dealing with shareholder proxy access does not change this conclusion That revision specifies that a shareholder-approved bylaw dealing with director nominations may not limit the boardrsquos power to amend add or repeal ldquoany procedure or condition to such a bylaw in order to provide for a reasonable practicable and orderly processrdquo MBCA sect 206(d) Thus according to the revision if shareholders approve a bylaw amendment that limits further board changes the board would nonetheless retain the power to ldquotinkerrdquo with the bylaw to safeguard the voting process but could not repeal the shareholder-approved bylaw The Official Comment to MBCA sect 206(d) makes clear that the revision is ldquonot intended to allow the board of directors to frustrate the purpose of the shareholder-adopted proxy access provisionrdquo Thus if Megarsquos shareholders were to approve the bylaw provision proposed by the investor Megarsquos board could only amend the provision regarding its procedures or conditions in a manner consistent with its purpose of permitting proxy access for Megarsquos shareholders

[NOTE The boardrsquos attempted interference with a shareholder voting initiative may also have been a violation of the directorsrsquo fiduciary duties See Blasius Indus Inc v Atlas Corp 564 A2d 651 (Del Ch 1988) (finding that directors breached their fiduciary duties by amending bylaws and expanding size of board to thwart insurgentrsquos plan to amend bylaws and seat a majority of new directors) The call however asks examinees to consider whether shareholders or the board have ldquoprecedencerdquo over amending the corporate bylaws Thus an examineersquos answer should be framed in terms of ldquopowerrdquo and not ldquodutyrdquo]

Point Three (40) The investor need not make a demand on the board if the investor states a direct claim such as an allegation that the board interfered with the investorrsquos right to amend the bylaws But the investor must make a demand on the board if the investor states a derivative claim (on behalf of the corporation) such as an allegation that the directors sought to entrench themselves by interfering with the proposed proxy access

The MBCA generally requires that shareholders make a demand on the board of directors before initiation of a derivative suit MBCA sect 742 (shareholder may not bring derivative proceeding until written demand has been made on corporation and 90 days have expired) A derivative suit is essentially two suits in one where the plaintiff-shareholder seeks to bring on behalf of the corporation a claim that vindicates corporate rights usually based on violation of fiduciary duties PALMITER supra sect 1811 (6th ed 2009) The demand permits the board to investigate the situation identified by the shareholder and take suitable action No demand on the board is required however if the shareholder brings a direct suit to vindicate the shareholderrsquos own rights not those of the corporation

Is the suit brought by the investor derivative or direct The MBCA defines a ldquoderivative proceedingrdquo as one brought ldquoin the right of a domestic corporationrdquo MBCA sect 740(1) Thus the answer to how the investorrsquos suit should be characterized turns on what rights the investor seeks to vindicate If the investor frames its claim as one of fiduciary breach by directorsmdashfor example for failing to become adequately informed about voting procedures or for seeking to entrench themselves in office by manipulating the voting structure to avoid a shareholder insurgencymdashthen the suit is ldquoderivativerdquo and the investor must make a demand on the board See MBCA Ch 7 Subch D Introductory Comment (ldquothe derivative suit has historically been the principal method of challenging allegedly illegal action by managementrdquo)

34

Corporations Analysis

If however the investor frames its claim as one to vindicate shareholder rights the suit is direct and no demand is required For many courts the direct-derivative question turns on who is injured and who is to receive the relief sought by the plaintiff-shareholders See Tooley v Donaldson Lufkin amp Jenrette Inc 845 A2d 1031 (Del 2004) (characterizing a merger-delay claim as direct because delay of merger only harmed shareholders not corporation) Thus if the investor claims that managementrsquos refusal to include its proposed bylaw amendment in the corporationrsquos proxy materials violates its shareholder rights to initiate corporate governance reforms the suit will be direct Courts have not questioned the ability of shareholders to bring direct suits challenging board action to exclude their proposed bylaw amendments from the corporationrsquos proxy materials See JANA Master Fund Ltd v CNET Networks Inc 954 A2d 335 (Del Ch 2008) (upholding shareholderrsquos direct challenge to boardrsquos interpretation of advance-notice bylaw) Chesapeake Corp v Shore 771 A2d 293 (Del Ch 2000) (upholding shareholderrsquos direct challenge to actions by board that effectively prevented it from proposing bylaw amendments in contest for control)

Is the way that the investor frames its claim conclusive Courts have permitted shareholder-plaintiffs to challenge a transaction in a direct suit even though the same transaction could also be challenged as a fiduciary breach See Eisenberg v Flying Tiger Line Inc 451 F2d 267 (2d Cir 1971) (permitting direct suit challenging a corporate reorganization as a dilution of shareholder voting power even though reorganization may have involved conflicts of interest and thus constituted a fiduciary breach) Thus the investorrsquos choice to pursue a claim challenging the legality of managementrsquos decision to exclude the investorrsquos proposal from the corporationrsquos proxy materialsmdashrather than a possible breach of fiduciary dutymdashis likely to be respected See 3 COX amp HAZEN supra sect 153 (describing situations in which a claim can be framed as derivative or direct)

[NOTE Some issues under Delaware corporate law regarding pre-suit demand are not relevant here For example whether the Mega directors are independent and disinterested is not relevant to the MBCA requirement of a pre-suit demand As the Official Comment to MBCA sect 742 points out the MBCArsquos requirement of ldquouniversal demandrdquo gives the board ldquothe opportunity to reexamine the act complained of in the light of a potential lawsuit and take corrective actionrdquo even when the directors might be non-independent or have conflicts of interest

Nor is it relevant to the MBCA pre-suit demand requirement that the statutory 90-day waiting period may be onerous The first paragraph of MBCA sect 742 requires a pre-suit demand without exception the second paragraph of the section imposes a 90-day waiting period before a derivative suit may be brought which can be shortened if the board rejects the demand or ldquoirreparable injury to the corporation would result by waiting for the expiration of the 90-day periodrdquo The call as written asks only whether a pre-suit demand should be made and does not ask examinees to address whether the post-demand waiting period should be shortened under the ldquoirreparable injuryrdquo standard]

35

National Conference of Bar Examiners 302 South Bedford Street | Madison WI 53703-3622 Phone 608-280-8550 | Fax 608-280-8552 | TDD 608-661-1275

wwwncbexorg e-mail contactncbexorg

  • Preface
  • Description of the MEE
  • Instructions
  • July 2014 Questions
    • CRIMINAL LAW AND PROCEDURE QUESTION
    • CONTRACTS QUESTION
    • FAMILY LAW QUESTION
    • FEDERAL CIVIL PROCEDURE QUESTION
    • EVIDENCE QUESTION
    • CORPORATIONS QUESTION
      • July 2014 Analyses
        • CRIMINAL LAW AND PROCEDURE ANALYSIS
        • CONTRACTS ANALYSIS
        • FAMILY LAW ANALYSIS
        • FEDERAL CIVIL PROCEDURE ANALYSIS
        • EVIDENCE ANALYSIS
        • CORPORATIONS ANALYSIS
            • ltlt13 ASCII85EncodePages false13 AllowTransparency false13 AutoPositionEPSFiles true13 AutoRotatePages None13 Binding Left13 CalGrayProfile (Dot Gain 20)13 CalRGBProfile (sRGB IEC61966-21)13 CalCMYKProfile (US Web Coated 050SWOP051 v2)13 sRGBProfile (sRGB IEC61966-21)13 CannotEmbedFontPolicy Error13 CompatibilityLevel 1413 CompressObjects Tags13 CompressPages true13 ConvertImagesToIndexed true13 PassThroughJPEGImages true13 CreateJobTicket false13 DefaultRenderingIntent Default13 DetectBlends true13 DetectCurves 0000013 ColorConversionStrategy CMYK13 DoThumbnails false13 EmbedAllFonts true13 EmbedOpenType false13 ParseICCProfilesInComments true13 EmbedJobOptions true13 DSCReportingLevel 013 EmitDSCWarnings false13 EndPage -113 ImageMemory 104857613 LockDistillerParams false13 MaxSubsetPct 10013 Optimize true13 OPM 113 ParseDSCComments true13 ParseDSCCommentsForDocInfo true13 PreserveCopyPage true13 PreserveDICMYKValues true13 PreserveEPSInfo true13 PreserveFlatness true13 PreserveHalftoneInfo false13 PreserveOPIComments true13 PreserveOverprintSettings true13 StartPage 113 SubsetFonts true13 TransferFunctionInfo Apply13 UCRandBGInfo Preserve13 UsePrologue false13 ColorSettingsFile ()13 AlwaysEmbed [ true13 ]13 NeverEmbed [ true13 ]13 AntiAliasColorImages false13 CropColorImages true13 ColorImageMinResolution 30013 ColorImageMinResolutionPolicy OK13 DownsampleColorImages true13 ColorImageDownsampleType Bicubic13 ColorImageResolution 30013 ColorImageDepth -113 ColorImageMinDownsampleDepth 113 ColorImageDownsampleThreshold 15000013 EncodeColorImages true13 ColorImageFilter DCTEncode13 AutoFilterColorImages true13 ColorImageAutoFilterStrategy JPEG13 ColorACSImageDict ltlt13 QFactor 01513 HSamples [1 1 1 1] VSamples [1 1 1 1]13 gtgt13 ColorImageDict ltlt13 QFactor 01513 HSamples [1 1 1 1] VSamples [1 1 1 1]13 gtgt13 JPEG2000ColorACSImageDict ltlt13 TileWidth 25613 TileHeight 25613 Quality 3013 gtgt13 JPEG2000ColorImageDict ltlt13 TileWidth 25613 TileHeight 25613 Quality 3013 gtgt13 AntiAliasGrayImages false13 CropGrayImages true13 GrayImageMinResolution 30013 GrayImageMinResolutionPolicy OK13 DownsampleGrayImages true13 GrayImageDownsampleType Bicubic13 GrayImageResolution 30013 GrayImageDepth -113 GrayImageMinDownsampleDepth 213 GrayImageDownsampleThreshold 15000013 EncodeGrayImages true13 GrayImageFilter DCTEncode13 AutoFilterGrayImages true13 GrayImageAutoFilterStrategy JPEG13 GrayACSImageDict ltlt13 QFactor 01513 HSamples [1 1 1 1] VSamples [1 1 1 1]13 gtgt13 GrayImageDict ltlt13 QFactor 01513 HSamples [1 1 1 1] VSamples [1 1 1 1]13 gtgt13 JPEG2000GrayACSImageDict ltlt13 TileWidth 25613 TileHeight 25613 Quality 3013 gtgt13 JPEG2000GrayImageDict ltlt13 TileWidth 25613 TileHeight 25613 Quality 3013 gtgt13 AntiAliasMonoImages false13 CropMonoImages true13 MonoImageMinResolution 120013 MonoImageMinResolutionPolicy OK13 DownsampleMonoImages true13 MonoImageDownsampleType Bicubic13 MonoImageResolution 120013 MonoImageDepth -113 MonoImageDownsampleThreshold 15000013 EncodeMonoImages true13 MonoImageFilter CCITTFaxEncode13 MonoImageDict ltlt13 K -113 gtgt13 AllowPSXObjects false13 CheckCompliance [13 None13 ]13 PDFX1aCheck false13 PDFX3Check false13 PDFXCompliantPDFOnly false13 PDFXNoTrimBoxError true13 PDFXTrimBoxToMediaBoxOffset [13 00000013 00000013 00000013 00000013 ]13 PDFXSetBleedBoxToMediaBox true13 PDFXBleedBoxToTrimBoxOffset [13 00000013 00000013 00000013 00000013 ]13 PDFXOutputIntentProfile ()13 PDFXOutputConditionIdentifier ()13 PDFXOutputCondition ()13 PDFXRegistryName ()13 PDFXTrapped False1313 CreateJDFFile false13 Description ltlt13 ARA 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 BGR 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 CHS ltFEFF4f7f75288fd94e9b8bbe5b9a521b5efa7684002000410064006f006200650020005000440046002065876863900275284e8e9ad88d2891cf76845370524d53705237300260a853ef4ee54f7f75280020004100630072006f0062006100740020548c002000410064006f00620065002000520065006100640065007200200035002e003000204ee553ca66f49ad87248672c676562535f00521b5efa768400200050004400460020658768633002gt13 CHT ltFEFF4f7f752890194e9b8a2d7f6e5efa7acb7684002000410064006f006200650020005000440046002065874ef69069752865bc9ad854c18cea76845370524d5370523786557406300260a853ef4ee54f7f75280020004100630072006f0062006100740020548c002000410064006f00620065002000520065006100640065007200200035002e003000204ee553ca66f49ad87248672c4f86958b555f5df25efa7acb76840020005000440046002065874ef63002gt13 CZE 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 DAN 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 DEU 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 ESP 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 ETI 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 FRA 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 GRE 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 HEB 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 HRV (Za stvaranje Adobe PDF dokumenata najpogodnijih za visokokvalitetni ispis prije tiskanja koristite ove postavke Stvoreni PDF dokumenti mogu se otvoriti Acrobat i Adobe Reader 50 i kasnijim verzijama)13 HUN 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 ITA 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 JPN ltFEFF9ad854c18cea306a30d730ea30d730ec30b951fa529b7528002000410064006f0062006500200050004400460020658766f8306e4f5c6210306b4f7f75283057307e305930023053306e8a2d5b9a30674f5c62103055308c305f0020005000440046002030d530a130a430eb306f3001004100630072006f0062006100740020304a30883073002000410064006f00620065002000520065006100640065007200200035002e003000204ee5964d3067958b304f30533068304c3067304d307e305930023053306e8a2d5b9a306b306f30d530a930f330c8306e57cb30818fbc307f304c5fc59808306730593002gt13 KOR ltFEFFc7740020c124c815c7440020c0acc6a9d558c5ec0020ace0d488c9c80020c2dcd5d80020c778c1c4c5d00020ac00c7a50020c801d569d55c002000410064006f0062006500200050004400460020bb38c11cb97c0020c791c131d569b2c8b2e4002e0020c774b807ac8c0020c791c131b41c00200050004400460020bb38c11cb2940020004100630072006f0062006100740020bc0f002000410064006f00620065002000520065006100640065007200200035002e00300020c774c0c1c5d0c11c0020c5f40020c2180020c788c2b5b2c8b2e4002egt13 LTH 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 LVI ltFEFF0049007a006d0061006e0074006f006a00690065007400200161006f00730020006900650073007400610074012b006a0075006d00750073002c0020006c0061006900200076006500690064006f00740075002000410064006f00620065002000500044004600200064006f006b0075006d0065006e007400750073002c0020006b006100730020006900720020012b00700061016100690020007000690065006d01130072006f00740069002000610075006700730074006100730020006b00760061006c0069007401010074006500730020007000690072006d007300690065007300700069006501610061006e006100730020006400720075006b00610069002e00200049007a0076006500690064006f006a006900650074002000500044004600200064006f006b0075006d0065006e007400750073002c0020006b006f002000760061007200200061007400760113007200740020006100720020004100630072006f00620061007400200075006e002000410064006f00620065002000520065006100640065007200200035002e0030002c0020006b0101002000610072012b00200074006f0020006a00610075006e0101006b0101006d002000760065007200730069006a0101006d002egt13 NLD (Gebruik deze instellingen om Adobe PDF-documenten te maken die zijn geoptimaliseerd voor prepress-afdrukken van hoge kwaliteit De gemaakte PDF-documenten kunnen worden geopend met Acrobat en Adobe Reader 50 en hoger)13 NOR ltFEFF004200720075006b00200064006900730073006500200069006e006e007300740069006c006c0069006e00670065006e0065002000740069006c002000e50020006f0070007000720065007400740065002000410064006f006200650020005000440046002d0064006f006b0075006d0065006e00740065007200200073006f006d00200065007200200062006500730074002000650067006e0065007400200066006f00720020006600f80072007400720079006b006b0073007500740073006b00720069006600740020006100760020006800f800790020006b00760061006c0069007400650074002e0020005000440046002d0064006f006b0075006d0065006e00740065006e00650020006b0061006e002000e50070006e00650073002000690020004100630072006f00620061007400200065006c006c00650072002000410064006f00620065002000520065006100640065007200200035002e003000200065006c006c00650072002000730065006e006500720065002egt13 POL 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 PTB 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 RUM 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 RUS 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 SKY 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 SLV 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 SUO 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 SVE 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 TUR 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 UKR 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 ENU (Use these settings to create Adobe PDF documents best suited for high-quality prepress printing Created PDF documents can be opened with Acrobat and Adobe Reader 50 and later)13 gtgt13 Namespace [13 (Adobe)13 (Common)13 (10)13 ]13 OtherNamespaces [13 ltlt13 AsReaderSpreads false13 CropImagesToFrames true13 ErrorControl WarnAndContinue13 FlattenerIgnoreSpreadOverrides false13 IncludeGuidesGrids false13 IncludeNonPrinting false13 IncludeSlug false13 Namespace [13 (Adobe)13 (InDesign)13 (40)13 ]13 OmitPlacedBitmaps false13 OmitPlacedEPS false13 OmitPlacedPDF false13 SimulateOverprint Legacy13 gtgt13 ltlt13 AddBleedMarks false13 AddColorBars false13 AddCropMarks false13 AddPageInfo false13 AddRegMarks false13 ConvertColors ConvertToCMYK13 DestinationProfileName ()13 DestinationProfileSelector DocumentCMYK13 Downsample16BitImages true13 FlattenerPreset ltlt13 PresetSelector MediumResolution13 gtgt13 FormElements false13 GenerateStructure false13 IncludeBookmarks false13 IncludeHyperlinks false13 IncludeInteractive false13 IncludeLayers false13 IncludeProfiles false13 MultimediaHandling UseObjectSettings13 Namespace [13 (Adobe)13 (CreativeSuite)13 (20)13 ]13 PDFXOutputIntentProfileSelector DocumentCMYK13 PreserveEditing true13 UntaggedCMYKHandling LeaveUntagged13 UntaggedRGBHandling UseDocumentProfile13 UseDocumentBleed false13 gtgt13 ]13gtgt setdistillerparams13ltlt13 HWResolution [2400 2400]13 PageSize [612000 792000]13gtgt setpagedevice13

Page 32: July 2014 MEE Questions and Analyses - NCBE...This publication includes the questions and analyses from the July 2014 MEE. (In the actual test, the questions are simply numbered rather

Evidence Analysis

Sex crimes are generally not considered relevant to credibility see Hopkins v State 639 So 2d 1247 1254 (Miss 1993) so the probative value of this conviction is relatively low Moreover the heinous nature of the inmatersquos crime (sexual assault on his daughter) makes the danger of unfair prejudice to the inmate very high Thus the court should exclude evidence of the conviction because it was for a heinous offense that is likely to inflame the jury and it has little bearing on credibility See eg United States v Beahm 664 F2d 414 419 (4th Cir 1981)

As an alternative to excluding this evidence the judge could minimize the unfair prejudice to the inmate by permitting limited cross-examination but refusing to allow specific questions about the nature of the inmatersquos conviction For example a court could limit cross-examination to the fact that the inmate was convicted of a ldquofelonyrdquo or perhaps that he was convicted of a ldquosexual assaultrdquo without identifying the victim However because evidence of the inmatersquos prior convictions can be admitted solely for the purpose of enabling the jury to assess his credibility and because his two earlier convictions should have already been admitted the court should exclude all evidence of the felony sexual assault conviction

Point Two(a) (15) The court should permit the inmatersquos counsel to cross-examine the guard regarding the false statement in his reacutesumeacute because the guardrsquos misconduct bears on his truthfulness

The inmate wishes to cross-examine the guard about his prior dishonest behaviormdashlying on his reacutesumeacutemdashthat did not involve a criminal conviction Rule 608(b) allows witnesses to be cross-examined about specific instances of prior non-conviction misconduct probative of untruthfulness ldquoin order to attack the witnessrsquos character for truthfulnessrdquo FED R EVID 608(b)

The courtrsquos decision to allow cross-examination about the guardrsquos prior dishonest behavior depends on the probative value of such evidence balanced against the danger of unfair prejudice to the guard or any other Rule 403 concern FED R EVID 403 Here the guardrsquos false statement on his reacutesumeacute that he obtained a degree in Criminal Justice is highly probative of his untruthfulness because it grossly misrepresents his actual academic record was made recently and was made with the intent to deceive Because the probative value of this evidence is very strong and is not substantially outweighed by any Rule 403 concerns cross-examination of the guard on this topic should be permitted The court may also consider it fair to permit this cross-examination of the guard on these matters assuming that one or more of the inmatersquos prior convictions have been admitted to impeach his credibility

Point Two(b) (15) The court should exclude extrinsic evidence of the guardrsquos non-conviction misconduct even if the guard denies wrongdoing or refuses to answer questions about the matter

Although Rule 608(b) allows cross-examination about specific instances of prior misconduct probative of untruthfulness ldquoextrinsic evidencerdquo offered to prove such misconduct is not admissible FED R EVID 608(b) The rationale for this rule is that allowing the introduction of extrinsic evidence of prior misconduct by witnesses when these acts are relevant only to the witnessesrsquo truthfulness and not to the main issues in the case would create too great a risk of confusing the jury and unduly delaying the trial The court does not have discretion to admit this extrinsic evidence See eg United States v Elliot 89 F3d 1360 1368 (8th Cir 1996)

30

Evidence Analysis

Here the inmatersquos counsel may cross-examine the guard about the false statement on his reacutesumeacute However the inmatersquos counsel must accept the guardrsquos response Even if the guard denies wrongdoing or refuses to answer questions about the matter the inmatersquos counsel cannot introduce the guardrsquos reacutesumeacute or the transcript from the local college to prove the guardrsquos misconduct

31

CORPORATIONS ANALYSIS (Corporations VA2 IX)

ANALYSIS

Legal Problems

(1) Do shareholders have the authority to amend a corporationrsquos bylaws with respect to director nominations

(2) Do board-approved bylaws on a particular subject here nomination of directors preempt subsequent conflicting bylaw amendments by shareholders

(3) Is a suit challenging both managementrsquos refusal to include the proposed bylaw amendment in Megarsquos proxy statement and the boardrsquos amendment of the bylaws dealing with nomination of directors a direct or derivative suit

DISCUSSION

Summary

The voting and litigation rights of the shareholders of Mega are subject to the provisions of the Model Business Corporations Act (MBCA)

The investorrsquos proposed bylaw provision is not inconsistent with state law Under the MBCA shareholders may amend the bylaws when the amendment deals with a proper matter for the corporationrsquos bylaws such as procedures for nominating directors

The Mega boardrsquos bylaw amendment does not preempt the investorrsquos proposed bylaw provision or the Mega shareholdersrsquo power to approve it While shareholders can limit the boardrsquos power to amend or repeal the bylaws the board cannot limit the shareholdersrsquo power

Whether the investor must make a demand on Megarsquos board depends on how the investor frames its claim If the investor claims a violation of shareholder voting rights the claim is direct and pre-suit demand on the board is not required If on the other hand the investor claims that the directors violated their fiduciary duties by amending the bylaws to entrench themselves the claim is derivative and a pre-suit demand is required

Point One (30) Shareholders may amend the corporationrsquos bylaws where the proposed bylaw provision relates to procedural matters typically included in the bylaws such as the nomination of directors

Internal affairs of the corporation such as the conduct of shareholder meetings and election of directors are subject to the corporate law of the state of incorporation See McDermott Inc v Lewis 531 A2d 206 (Del 1987) (applying law of jurisdiction where corporation was incorporated in case involving voting rights) This statersquos corporate statute is modeled on the MBCA

Under the MBCA ldquoshareholders may amend the corporationrsquos bylawsrdquo MBCA sect 1020(a) Thus the only question is whether the bylaws can specify the procedures for shareholder nomination of directors

32

Corporations Analysis

The MBCA states that the bylaws ldquomay contain any provision that is not inconsistent with law or the articles of incorporationrdquo MBCA sect 206(b) In addition the MBCA was revised in 2009 to address shareholder nomination of directors in public corporations (known as ldquoproxy accessrdquo) and specifies that the bylaws ldquomay contain a requirement that the corporation include in its [proxy materials] one or more individuals nominated by a shareholderrdquo MBCA sect 206(c)(1) see Committee on Corporate Laws ABA Section of Business Law Report on the Roles of Boards of Directors and Shareholders of Publicly Owned Corporations and Changes to the Model Business Corporations ActmdashAdoption of Shareholder Proxy Access Amendments to Chapters 2 and 10 65 BUS LAWYER 1105 (2010)

The inclusion of director-nomination procedures in the bylaws is consistent with practice and is recognized by the Delaware courts whose views on corporate law carry significant weight Typically the procedures for nomination of directors are found in the bylaws See 1 COX amp HAZEN TREATISE ON THE LAW OF CORPORATIONS sect 312 (3d ed 2011) see also 4 FLETCHER CORP FORMS ANN PART III ch 21 (2013) (including sample bylaws that permit nomination of directors by shareholders) The Delaware Supreme Court has confirmed that the bylaws may ldquodefine the process and proceduresrdquo for director elections See CA Inc v AFSCME Employees Pension Plan 953 A2d 227 (Del 2008) (concluding that bylaw amendment requiring reimbursement of election expenses to certain successful shareholder nominators is ldquoproper subjectrdquo under Delaware law)

[NOTE The question of the proper scope of the bylaws can be answered using the more general MBCA sect 206(b) or the 2009 MBCA revision adding sect 206(c)(1) (adopted in CT ME VA) In addition some examinees might raise the point that shareholder proposals may not compel the board to take action such as by including shareholder nominations in the companyrsquos proxy materials on the theory that the ldquobusiness and affairsrdquo of the corporation are to be managed by the board See MBCA sect 801(b) Although shareholders are generally limited to adopting precatory resolutions that recommend or encourage board action this limitation does not apply when shareholders have specific authority to take binding action on their ownmdashsuch as to amend the bylaws]

Point Two (30) Shareholders can amend (or repeal) board-approved bylaws Further shareholders can limit the boardrsquos power to later amend and repeal a shareholder-approved bylaw

Under the MBCA shareholders have the power to amend the bylaws See Point One The board shares this power with the shareholders unless (1) the corporationrsquos articles ldquoreserve that power exclusively to the shareholdersrdquo or (2) ldquothe shareholders in amending repealing or adopting a bylaw expressly provide that the board of directors may not amend repeal or reinstate that bylawrdquo See MBCA sect 1020(b)

Shareholder-approved bylaw provisions can amend or repeal existing bylaw provisions whether originally approved by the board or by shareholders See ALAN R PALMITER CORPORATIONS EXAMPLES AND EXPLANATIONS sect 713 (7th ed 2012) Thus the Mega boardrsquos bylaw amendmentmdashwhich set more demanding thresholds for shareholder nomination of directors than the investorrsquos proposed bylaw provisionmdashwould be superseded (repealed) if Megarsquos shareholders were to approve the investorrsquos proposal

Further a shareholder-approved bylaw generally can limit the power of the board to later amend or repeal it See MBCA sect 1020(b)(2) Thus if Megarsquos shareholders approved the bylaw

33

Corporations Analysis

provision proposed by the investor Megarsquos board could not repeal the provision because it includes a ldquono board repealrdquo clause

The revision to the MBCA in 2009 dealing with shareholder proxy access does not change this conclusion That revision specifies that a shareholder-approved bylaw dealing with director nominations may not limit the boardrsquos power to amend add or repeal ldquoany procedure or condition to such a bylaw in order to provide for a reasonable practicable and orderly processrdquo MBCA sect 206(d) Thus according to the revision if shareholders approve a bylaw amendment that limits further board changes the board would nonetheless retain the power to ldquotinkerrdquo with the bylaw to safeguard the voting process but could not repeal the shareholder-approved bylaw The Official Comment to MBCA sect 206(d) makes clear that the revision is ldquonot intended to allow the board of directors to frustrate the purpose of the shareholder-adopted proxy access provisionrdquo Thus if Megarsquos shareholders were to approve the bylaw provision proposed by the investor Megarsquos board could only amend the provision regarding its procedures or conditions in a manner consistent with its purpose of permitting proxy access for Megarsquos shareholders

[NOTE The boardrsquos attempted interference with a shareholder voting initiative may also have been a violation of the directorsrsquo fiduciary duties See Blasius Indus Inc v Atlas Corp 564 A2d 651 (Del Ch 1988) (finding that directors breached their fiduciary duties by amending bylaws and expanding size of board to thwart insurgentrsquos plan to amend bylaws and seat a majority of new directors) The call however asks examinees to consider whether shareholders or the board have ldquoprecedencerdquo over amending the corporate bylaws Thus an examineersquos answer should be framed in terms of ldquopowerrdquo and not ldquodutyrdquo]

Point Three (40) The investor need not make a demand on the board if the investor states a direct claim such as an allegation that the board interfered with the investorrsquos right to amend the bylaws But the investor must make a demand on the board if the investor states a derivative claim (on behalf of the corporation) such as an allegation that the directors sought to entrench themselves by interfering with the proposed proxy access

The MBCA generally requires that shareholders make a demand on the board of directors before initiation of a derivative suit MBCA sect 742 (shareholder may not bring derivative proceeding until written demand has been made on corporation and 90 days have expired) A derivative suit is essentially two suits in one where the plaintiff-shareholder seeks to bring on behalf of the corporation a claim that vindicates corporate rights usually based on violation of fiduciary duties PALMITER supra sect 1811 (6th ed 2009) The demand permits the board to investigate the situation identified by the shareholder and take suitable action No demand on the board is required however if the shareholder brings a direct suit to vindicate the shareholderrsquos own rights not those of the corporation

Is the suit brought by the investor derivative or direct The MBCA defines a ldquoderivative proceedingrdquo as one brought ldquoin the right of a domestic corporationrdquo MBCA sect 740(1) Thus the answer to how the investorrsquos suit should be characterized turns on what rights the investor seeks to vindicate If the investor frames its claim as one of fiduciary breach by directorsmdashfor example for failing to become adequately informed about voting procedures or for seeking to entrench themselves in office by manipulating the voting structure to avoid a shareholder insurgencymdashthen the suit is ldquoderivativerdquo and the investor must make a demand on the board See MBCA Ch 7 Subch D Introductory Comment (ldquothe derivative suit has historically been the principal method of challenging allegedly illegal action by managementrdquo)

34

Corporations Analysis

If however the investor frames its claim as one to vindicate shareholder rights the suit is direct and no demand is required For many courts the direct-derivative question turns on who is injured and who is to receive the relief sought by the plaintiff-shareholders See Tooley v Donaldson Lufkin amp Jenrette Inc 845 A2d 1031 (Del 2004) (characterizing a merger-delay claim as direct because delay of merger only harmed shareholders not corporation) Thus if the investor claims that managementrsquos refusal to include its proposed bylaw amendment in the corporationrsquos proxy materials violates its shareholder rights to initiate corporate governance reforms the suit will be direct Courts have not questioned the ability of shareholders to bring direct suits challenging board action to exclude their proposed bylaw amendments from the corporationrsquos proxy materials See JANA Master Fund Ltd v CNET Networks Inc 954 A2d 335 (Del Ch 2008) (upholding shareholderrsquos direct challenge to boardrsquos interpretation of advance-notice bylaw) Chesapeake Corp v Shore 771 A2d 293 (Del Ch 2000) (upholding shareholderrsquos direct challenge to actions by board that effectively prevented it from proposing bylaw amendments in contest for control)

Is the way that the investor frames its claim conclusive Courts have permitted shareholder-plaintiffs to challenge a transaction in a direct suit even though the same transaction could also be challenged as a fiduciary breach See Eisenberg v Flying Tiger Line Inc 451 F2d 267 (2d Cir 1971) (permitting direct suit challenging a corporate reorganization as a dilution of shareholder voting power even though reorganization may have involved conflicts of interest and thus constituted a fiduciary breach) Thus the investorrsquos choice to pursue a claim challenging the legality of managementrsquos decision to exclude the investorrsquos proposal from the corporationrsquos proxy materialsmdashrather than a possible breach of fiduciary dutymdashis likely to be respected See 3 COX amp HAZEN supra sect 153 (describing situations in which a claim can be framed as derivative or direct)

[NOTE Some issues under Delaware corporate law regarding pre-suit demand are not relevant here For example whether the Mega directors are independent and disinterested is not relevant to the MBCA requirement of a pre-suit demand As the Official Comment to MBCA sect 742 points out the MBCArsquos requirement of ldquouniversal demandrdquo gives the board ldquothe opportunity to reexamine the act complained of in the light of a potential lawsuit and take corrective actionrdquo even when the directors might be non-independent or have conflicts of interest

Nor is it relevant to the MBCA pre-suit demand requirement that the statutory 90-day waiting period may be onerous The first paragraph of MBCA sect 742 requires a pre-suit demand without exception the second paragraph of the section imposes a 90-day waiting period before a derivative suit may be brought which can be shortened if the board rejects the demand or ldquoirreparable injury to the corporation would result by waiting for the expiration of the 90-day periodrdquo The call as written asks only whether a pre-suit demand should be made and does not ask examinees to address whether the post-demand waiting period should be shortened under the ldquoirreparable injuryrdquo standard]

35

National Conference of Bar Examiners 302 South Bedford Street | Madison WI 53703-3622 Phone 608-280-8550 | Fax 608-280-8552 | TDD 608-661-1275

wwwncbexorg e-mail contactncbexorg

  • Preface
  • Description of the MEE
  • Instructions
  • July 2014 Questions
    • CRIMINAL LAW AND PROCEDURE QUESTION
    • CONTRACTS QUESTION
    • FAMILY LAW QUESTION
    • FEDERAL CIVIL PROCEDURE QUESTION
    • EVIDENCE QUESTION
    • CORPORATIONS QUESTION
      • July 2014 Analyses
        • CRIMINAL LAW AND PROCEDURE ANALYSIS
        • CONTRACTS ANALYSIS
        • FAMILY LAW ANALYSIS
        • FEDERAL CIVIL PROCEDURE ANALYSIS
        • EVIDENCE ANALYSIS
        • CORPORATIONS ANALYSIS
            • ltlt13 ASCII85EncodePages false13 AllowTransparency false13 AutoPositionEPSFiles true13 AutoRotatePages None13 Binding Left13 CalGrayProfile (Dot Gain 20)13 CalRGBProfile (sRGB IEC61966-21)13 CalCMYKProfile (US Web Coated 050SWOP051 v2)13 sRGBProfile (sRGB IEC61966-21)13 CannotEmbedFontPolicy Error13 CompatibilityLevel 1413 CompressObjects Tags13 CompressPages true13 ConvertImagesToIndexed true13 PassThroughJPEGImages true13 CreateJobTicket false13 DefaultRenderingIntent Default13 DetectBlends true13 DetectCurves 0000013 ColorConversionStrategy CMYK13 DoThumbnails false13 EmbedAllFonts true13 EmbedOpenType false13 ParseICCProfilesInComments true13 EmbedJobOptions true13 DSCReportingLevel 013 EmitDSCWarnings false13 EndPage -113 ImageMemory 104857613 LockDistillerParams false13 MaxSubsetPct 10013 Optimize true13 OPM 113 ParseDSCComments true13 ParseDSCCommentsForDocInfo true13 PreserveCopyPage true13 PreserveDICMYKValues true13 PreserveEPSInfo true13 PreserveFlatness true13 PreserveHalftoneInfo false13 PreserveOPIComments true13 PreserveOverprintSettings true13 StartPage 113 SubsetFonts true13 TransferFunctionInfo Apply13 UCRandBGInfo Preserve13 UsePrologue false13 ColorSettingsFile ()13 AlwaysEmbed [ true13 ]13 NeverEmbed [ true13 ]13 AntiAliasColorImages false13 CropColorImages true13 ColorImageMinResolution 30013 ColorImageMinResolutionPolicy OK13 DownsampleColorImages true13 ColorImageDownsampleType Bicubic13 ColorImageResolution 30013 ColorImageDepth -113 ColorImageMinDownsampleDepth 113 ColorImageDownsampleThreshold 15000013 EncodeColorImages true13 ColorImageFilter DCTEncode13 AutoFilterColorImages true13 ColorImageAutoFilterStrategy JPEG13 ColorACSImageDict ltlt13 QFactor 01513 HSamples [1 1 1 1] VSamples [1 1 1 1]13 gtgt13 ColorImageDict ltlt13 QFactor 01513 HSamples [1 1 1 1] VSamples [1 1 1 1]13 gtgt13 JPEG2000ColorACSImageDict ltlt13 TileWidth 25613 TileHeight 25613 Quality 3013 gtgt13 JPEG2000ColorImageDict ltlt13 TileWidth 25613 TileHeight 25613 Quality 3013 gtgt13 AntiAliasGrayImages false13 CropGrayImages true13 GrayImageMinResolution 30013 GrayImageMinResolutionPolicy OK13 DownsampleGrayImages true13 GrayImageDownsampleType Bicubic13 GrayImageResolution 30013 GrayImageDepth -113 GrayImageMinDownsampleDepth 213 GrayImageDownsampleThreshold 15000013 EncodeGrayImages true13 GrayImageFilter DCTEncode13 AutoFilterGrayImages true13 GrayImageAutoFilterStrategy JPEG13 GrayACSImageDict ltlt13 QFactor 01513 HSamples [1 1 1 1] VSamples [1 1 1 1]13 gtgt13 GrayImageDict ltlt13 QFactor 01513 HSamples [1 1 1 1] VSamples [1 1 1 1]13 gtgt13 JPEG2000GrayACSImageDict ltlt13 TileWidth 25613 TileHeight 25613 Quality 3013 gtgt13 JPEG2000GrayImageDict ltlt13 TileWidth 25613 TileHeight 25613 Quality 3013 gtgt13 AntiAliasMonoImages false13 CropMonoImages true13 MonoImageMinResolution 120013 MonoImageMinResolutionPolicy OK13 DownsampleMonoImages true13 MonoImageDownsampleType Bicubic13 MonoImageResolution 120013 MonoImageDepth -113 MonoImageDownsampleThreshold 15000013 EncodeMonoImages true13 MonoImageFilter CCITTFaxEncode13 MonoImageDict ltlt13 K -113 gtgt13 AllowPSXObjects false13 CheckCompliance [13 None13 ]13 PDFX1aCheck false13 PDFX3Check false13 PDFXCompliantPDFOnly false13 PDFXNoTrimBoxError true13 PDFXTrimBoxToMediaBoxOffset [13 00000013 00000013 00000013 00000013 ]13 PDFXSetBleedBoxToMediaBox true13 PDFXBleedBoxToTrimBoxOffset [13 00000013 00000013 00000013 00000013 ]13 PDFXOutputIntentProfile ()13 PDFXOutputConditionIdentifier ()13 PDFXOutputCondition ()13 PDFXRegistryName ()13 PDFXTrapped False1313 CreateJDFFile false13 Description ltlt13 ARA 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 BGR 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 CHS ltFEFF4f7f75288fd94e9b8bbe5b9a521b5efa7684002000410064006f006200650020005000440046002065876863900275284e8e9ad88d2891cf76845370524d53705237300260a853ef4ee54f7f75280020004100630072006f0062006100740020548c002000410064006f00620065002000520065006100640065007200200035002e003000204ee553ca66f49ad87248672c676562535f00521b5efa768400200050004400460020658768633002gt13 CHT ltFEFF4f7f752890194e9b8a2d7f6e5efa7acb7684002000410064006f006200650020005000440046002065874ef69069752865bc9ad854c18cea76845370524d5370523786557406300260a853ef4ee54f7f75280020004100630072006f0062006100740020548c002000410064006f00620065002000520065006100640065007200200035002e003000204ee553ca66f49ad87248672c4f86958b555f5df25efa7acb76840020005000440046002065874ef63002gt13 CZE 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 DAN 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 DEU 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 ESP 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 ETI 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 FRA 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 GRE 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 HEB ltFEFF05D405E905EA05DE05E905D5002005D105D405D205D305E805D505EA002005D005DC05D4002005DB05D305D9002005DC05D905E605D505E8002005DE05E105DE05DB05D9002000410064006F006200650020005000440046002005D405DE05D505EA05D005DE05D905DD002005DC05D405D305E405E105EA002005E705D305DD002D05D305E405D505E1002005D005D905DB05D505EA05D905EA002E002005DE05E105DE05DB05D90020005000440046002005E905E005D505E605E805D5002005E005D905EA05E005D905DD002005DC05E405EA05D905D705D4002005D105D005DE05E605E205D505EA0020004100630072006F006200610074002005D5002D00410064006F00620065002000520065006100640065007200200035002E0030002005D505D205E805E105D005D505EA002005DE05EA05E705D305DE05D505EA002005D905D505EA05E8002E05D005DE05D905DD002005DC002D005000440046002F0058002D0033002C002005E205D905D905E005D5002005D105DE05D305E805D905DA002005DC05DE05E905EA05DE05E9002005E905DC0020004100630072006F006200610074002E002005DE05E105DE05DB05D90020005000440046002005E905E005D505E605E805D5002005E005D905EA05E005D905DD002005DC05E405EA05D905D705D4002005D105D005DE05E605E205D505EA0020004100630072006F006200610074002005D5002D00410064006F00620065002000520065006100640065007200200035002E0030002005D505D205E805E105D005D505EA002005DE05EA05E705D305DE05D505EA002005D905D505EA05E8002Egt13 HRV (Za stvaranje Adobe PDF dokumenata najpogodnijih za visokokvalitetni ispis prije tiskanja koristite ove postavke Stvoreni PDF dokumenti mogu se otvoriti Acrobat i Adobe Reader 50 i kasnijim verzijama)13 HUN 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 ITA 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 JPN ltFEFF9ad854c18cea306a30d730ea30d730ec30b951fa529b7528002000410064006f0062006500200050004400460020658766f8306e4f5c6210306b4f7f75283057307e305930023053306e8a2d5b9a30674f5c62103055308c305f0020005000440046002030d530a130a430eb306f3001004100630072006f0062006100740020304a30883073002000410064006f00620065002000520065006100640065007200200035002e003000204ee5964d3067958b304f30533068304c3067304d307e305930023053306e8a2d5b9a306b306f30d530a930f330c8306e57cb30818fbc307f304c5fc59808306730593002gt13 KOR ltFEFFc7740020c124c815c7440020c0acc6a9d558c5ec0020ace0d488c9c80020c2dcd5d80020c778c1c4c5d00020ac00c7a50020c801d569d55c002000410064006f0062006500200050004400460020bb38c11cb97c0020c791c131d569b2c8b2e4002e0020c774b807ac8c0020c791c131b41c00200050004400460020bb38c11cb2940020004100630072006f0062006100740020bc0f002000410064006f00620065002000520065006100640065007200200035002e00300020c774c0c1c5d0c11c0020c5f40020c2180020c788c2b5b2c8b2e4002egt13 LTH 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 LVI 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 NLD (Gebruik deze instellingen om Adobe PDF-documenten te maken die zijn geoptimaliseerd voor prepress-afdrukken van hoge kwaliteit De gemaakte PDF-documenten kunnen worden geopend met Acrobat en Adobe Reader 50 en hoger)13 NOR 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 POL ltFEFF0055007300740061007700690065006e0069006100200064006f002000740077006f0072007a0065006e0069006100200064006f006b0075006d0065006e007400f300770020005000440046002000700072007a0065007a006e00610063007a006f006e00790063006800200064006f002000770079006400720075006b00f30077002000770020007700790073006f006b00690065006a0020006a0061006b006f015b00630069002e002000200044006f006b0075006d0065006e0074007900200050004400460020006d006f017c006e00610020006f007400770069006500720061010700200077002000700072006f006700720061006d006900650020004100630072006f00620061007400200069002000410064006f00620065002000520065006100640065007200200035002e0030002000690020006e006f00770073007a0079006d002egt13 PTB ltFEFF005500740069006c0069007a006500200065007300730061007300200063006f006e00660069006700750072006100e700f50065007300200064006500200066006f0072006d00610020006100200063007200690061007200200064006f00630075006d0065006e0074006f0073002000410064006f0062006500200050004400460020006d00610069007300200061006400650071007500610064006f00730020007000610072006100200070007200e9002d0069006d0070007200650073007300f50065007300200064006500200061006c007400610020007100750061006c00690064006100640065002e0020004f007300200064006f00630075006d0065006e0074006f00730020005000440046002000630072006900610064006f007300200070006f00640065006d0020007300650072002000610062006500720074006f007300200063006f006d0020006f0020004100630072006f006200610074002000650020006f002000410064006f00620065002000520065006100640065007200200035002e0030002000650020007600650072007300f50065007300200070006f00730074006500720069006f007200650073002egt13 RUM 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 RUS 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 SKY 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 SLV 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 SUO 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 SVE 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 TUR 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 UKR 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 ENU (Use these settings to create Adobe PDF documents best suited for high-quality prepress printing Created PDF documents can be opened with Acrobat and Adobe Reader 50 and later)13 gtgt13 Namespace [13 (Adobe)13 (Common)13 (10)13 ]13 OtherNamespaces [13 ltlt13 AsReaderSpreads false13 CropImagesToFrames true13 ErrorControl WarnAndContinue13 FlattenerIgnoreSpreadOverrides false13 IncludeGuidesGrids false13 IncludeNonPrinting false13 IncludeSlug false13 Namespace [13 (Adobe)13 (InDesign)13 (40)13 ]13 OmitPlacedBitmaps false13 OmitPlacedEPS false13 OmitPlacedPDF false13 SimulateOverprint Legacy13 gtgt13 ltlt13 AddBleedMarks false13 AddColorBars false13 AddCropMarks false13 AddPageInfo false13 AddRegMarks false13 ConvertColors ConvertToCMYK13 DestinationProfileName ()13 DestinationProfileSelector DocumentCMYK13 Downsample16BitImages true13 FlattenerPreset ltlt13 PresetSelector MediumResolution13 gtgt13 FormElements false13 GenerateStructure false13 IncludeBookmarks false13 IncludeHyperlinks false13 IncludeInteractive false13 IncludeLayers false13 IncludeProfiles false13 MultimediaHandling UseObjectSettings13 Namespace [13 (Adobe)13 (CreativeSuite)13 (20)13 ]13 PDFXOutputIntentProfileSelector DocumentCMYK13 PreserveEditing true13 UntaggedCMYKHandling LeaveUntagged13 UntaggedRGBHandling UseDocumentProfile13 UseDocumentBleed false13 gtgt13 ]13gtgt setdistillerparams13ltlt13 HWResolution [2400 2400]13 PageSize [612000 792000]13gtgt setpagedevice13

Page 33: July 2014 MEE Questions and Analyses - NCBE...This publication includes the questions and analyses from the July 2014 MEE. (In the actual test, the questions are simply numbered rather

Evidence Analysis

Here the inmatersquos counsel may cross-examine the guard about the false statement on his reacutesumeacute However the inmatersquos counsel must accept the guardrsquos response Even if the guard denies wrongdoing or refuses to answer questions about the matter the inmatersquos counsel cannot introduce the guardrsquos reacutesumeacute or the transcript from the local college to prove the guardrsquos misconduct

31

CORPORATIONS ANALYSIS (Corporations VA2 IX)

ANALYSIS

Legal Problems

(1) Do shareholders have the authority to amend a corporationrsquos bylaws with respect to director nominations

(2) Do board-approved bylaws on a particular subject here nomination of directors preempt subsequent conflicting bylaw amendments by shareholders

(3) Is a suit challenging both managementrsquos refusal to include the proposed bylaw amendment in Megarsquos proxy statement and the boardrsquos amendment of the bylaws dealing with nomination of directors a direct or derivative suit

DISCUSSION

Summary

The voting and litigation rights of the shareholders of Mega are subject to the provisions of the Model Business Corporations Act (MBCA)

The investorrsquos proposed bylaw provision is not inconsistent with state law Under the MBCA shareholders may amend the bylaws when the amendment deals with a proper matter for the corporationrsquos bylaws such as procedures for nominating directors

The Mega boardrsquos bylaw amendment does not preempt the investorrsquos proposed bylaw provision or the Mega shareholdersrsquo power to approve it While shareholders can limit the boardrsquos power to amend or repeal the bylaws the board cannot limit the shareholdersrsquo power

Whether the investor must make a demand on Megarsquos board depends on how the investor frames its claim If the investor claims a violation of shareholder voting rights the claim is direct and pre-suit demand on the board is not required If on the other hand the investor claims that the directors violated their fiduciary duties by amending the bylaws to entrench themselves the claim is derivative and a pre-suit demand is required

Point One (30) Shareholders may amend the corporationrsquos bylaws where the proposed bylaw provision relates to procedural matters typically included in the bylaws such as the nomination of directors

Internal affairs of the corporation such as the conduct of shareholder meetings and election of directors are subject to the corporate law of the state of incorporation See McDermott Inc v Lewis 531 A2d 206 (Del 1987) (applying law of jurisdiction where corporation was incorporated in case involving voting rights) This statersquos corporate statute is modeled on the MBCA

Under the MBCA ldquoshareholders may amend the corporationrsquos bylawsrdquo MBCA sect 1020(a) Thus the only question is whether the bylaws can specify the procedures for shareholder nomination of directors

32

Corporations Analysis

The MBCA states that the bylaws ldquomay contain any provision that is not inconsistent with law or the articles of incorporationrdquo MBCA sect 206(b) In addition the MBCA was revised in 2009 to address shareholder nomination of directors in public corporations (known as ldquoproxy accessrdquo) and specifies that the bylaws ldquomay contain a requirement that the corporation include in its [proxy materials] one or more individuals nominated by a shareholderrdquo MBCA sect 206(c)(1) see Committee on Corporate Laws ABA Section of Business Law Report on the Roles of Boards of Directors and Shareholders of Publicly Owned Corporations and Changes to the Model Business Corporations ActmdashAdoption of Shareholder Proxy Access Amendments to Chapters 2 and 10 65 BUS LAWYER 1105 (2010)

The inclusion of director-nomination procedures in the bylaws is consistent with practice and is recognized by the Delaware courts whose views on corporate law carry significant weight Typically the procedures for nomination of directors are found in the bylaws See 1 COX amp HAZEN TREATISE ON THE LAW OF CORPORATIONS sect 312 (3d ed 2011) see also 4 FLETCHER CORP FORMS ANN PART III ch 21 (2013) (including sample bylaws that permit nomination of directors by shareholders) The Delaware Supreme Court has confirmed that the bylaws may ldquodefine the process and proceduresrdquo for director elections See CA Inc v AFSCME Employees Pension Plan 953 A2d 227 (Del 2008) (concluding that bylaw amendment requiring reimbursement of election expenses to certain successful shareholder nominators is ldquoproper subjectrdquo under Delaware law)

[NOTE The question of the proper scope of the bylaws can be answered using the more general MBCA sect 206(b) or the 2009 MBCA revision adding sect 206(c)(1) (adopted in CT ME VA) In addition some examinees might raise the point that shareholder proposals may not compel the board to take action such as by including shareholder nominations in the companyrsquos proxy materials on the theory that the ldquobusiness and affairsrdquo of the corporation are to be managed by the board See MBCA sect 801(b) Although shareholders are generally limited to adopting precatory resolutions that recommend or encourage board action this limitation does not apply when shareholders have specific authority to take binding action on their ownmdashsuch as to amend the bylaws]

Point Two (30) Shareholders can amend (or repeal) board-approved bylaws Further shareholders can limit the boardrsquos power to later amend and repeal a shareholder-approved bylaw

Under the MBCA shareholders have the power to amend the bylaws See Point One The board shares this power with the shareholders unless (1) the corporationrsquos articles ldquoreserve that power exclusively to the shareholdersrdquo or (2) ldquothe shareholders in amending repealing or adopting a bylaw expressly provide that the board of directors may not amend repeal or reinstate that bylawrdquo See MBCA sect 1020(b)

Shareholder-approved bylaw provisions can amend or repeal existing bylaw provisions whether originally approved by the board or by shareholders See ALAN R PALMITER CORPORATIONS EXAMPLES AND EXPLANATIONS sect 713 (7th ed 2012) Thus the Mega boardrsquos bylaw amendmentmdashwhich set more demanding thresholds for shareholder nomination of directors than the investorrsquos proposed bylaw provisionmdashwould be superseded (repealed) if Megarsquos shareholders were to approve the investorrsquos proposal

Further a shareholder-approved bylaw generally can limit the power of the board to later amend or repeal it See MBCA sect 1020(b)(2) Thus if Megarsquos shareholders approved the bylaw

33

Corporations Analysis

provision proposed by the investor Megarsquos board could not repeal the provision because it includes a ldquono board repealrdquo clause

The revision to the MBCA in 2009 dealing with shareholder proxy access does not change this conclusion That revision specifies that a shareholder-approved bylaw dealing with director nominations may not limit the boardrsquos power to amend add or repeal ldquoany procedure or condition to such a bylaw in order to provide for a reasonable practicable and orderly processrdquo MBCA sect 206(d) Thus according to the revision if shareholders approve a bylaw amendment that limits further board changes the board would nonetheless retain the power to ldquotinkerrdquo with the bylaw to safeguard the voting process but could not repeal the shareholder-approved bylaw The Official Comment to MBCA sect 206(d) makes clear that the revision is ldquonot intended to allow the board of directors to frustrate the purpose of the shareholder-adopted proxy access provisionrdquo Thus if Megarsquos shareholders were to approve the bylaw provision proposed by the investor Megarsquos board could only amend the provision regarding its procedures or conditions in a manner consistent with its purpose of permitting proxy access for Megarsquos shareholders

[NOTE The boardrsquos attempted interference with a shareholder voting initiative may also have been a violation of the directorsrsquo fiduciary duties See Blasius Indus Inc v Atlas Corp 564 A2d 651 (Del Ch 1988) (finding that directors breached their fiduciary duties by amending bylaws and expanding size of board to thwart insurgentrsquos plan to amend bylaws and seat a majority of new directors) The call however asks examinees to consider whether shareholders or the board have ldquoprecedencerdquo over amending the corporate bylaws Thus an examineersquos answer should be framed in terms of ldquopowerrdquo and not ldquodutyrdquo]

Point Three (40) The investor need not make a demand on the board if the investor states a direct claim such as an allegation that the board interfered with the investorrsquos right to amend the bylaws But the investor must make a demand on the board if the investor states a derivative claim (on behalf of the corporation) such as an allegation that the directors sought to entrench themselves by interfering with the proposed proxy access

The MBCA generally requires that shareholders make a demand on the board of directors before initiation of a derivative suit MBCA sect 742 (shareholder may not bring derivative proceeding until written demand has been made on corporation and 90 days have expired) A derivative suit is essentially two suits in one where the plaintiff-shareholder seeks to bring on behalf of the corporation a claim that vindicates corporate rights usually based on violation of fiduciary duties PALMITER supra sect 1811 (6th ed 2009) The demand permits the board to investigate the situation identified by the shareholder and take suitable action No demand on the board is required however if the shareholder brings a direct suit to vindicate the shareholderrsquos own rights not those of the corporation

Is the suit brought by the investor derivative or direct The MBCA defines a ldquoderivative proceedingrdquo as one brought ldquoin the right of a domestic corporationrdquo MBCA sect 740(1) Thus the answer to how the investorrsquos suit should be characterized turns on what rights the investor seeks to vindicate If the investor frames its claim as one of fiduciary breach by directorsmdashfor example for failing to become adequately informed about voting procedures or for seeking to entrench themselves in office by manipulating the voting structure to avoid a shareholder insurgencymdashthen the suit is ldquoderivativerdquo and the investor must make a demand on the board See MBCA Ch 7 Subch D Introductory Comment (ldquothe derivative suit has historically been the principal method of challenging allegedly illegal action by managementrdquo)

34

Corporations Analysis

If however the investor frames its claim as one to vindicate shareholder rights the suit is direct and no demand is required For many courts the direct-derivative question turns on who is injured and who is to receive the relief sought by the plaintiff-shareholders See Tooley v Donaldson Lufkin amp Jenrette Inc 845 A2d 1031 (Del 2004) (characterizing a merger-delay claim as direct because delay of merger only harmed shareholders not corporation) Thus if the investor claims that managementrsquos refusal to include its proposed bylaw amendment in the corporationrsquos proxy materials violates its shareholder rights to initiate corporate governance reforms the suit will be direct Courts have not questioned the ability of shareholders to bring direct suits challenging board action to exclude their proposed bylaw amendments from the corporationrsquos proxy materials See JANA Master Fund Ltd v CNET Networks Inc 954 A2d 335 (Del Ch 2008) (upholding shareholderrsquos direct challenge to boardrsquos interpretation of advance-notice bylaw) Chesapeake Corp v Shore 771 A2d 293 (Del Ch 2000) (upholding shareholderrsquos direct challenge to actions by board that effectively prevented it from proposing bylaw amendments in contest for control)

Is the way that the investor frames its claim conclusive Courts have permitted shareholder-plaintiffs to challenge a transaction in a direct suit even though the same transaction could also be challenged as a fiduciary breach See Eisenberg v Flying Tiger Line Inc 451 F2d 267 (2d Cir 1971) (permitting direct suit challenging a corporate reorganization as a dilution of shareholder voting power even though reorganization may have involved conflicts of interest and thus constituted a fiduciary breach) Thus the investorrsquos choice to pursue a claim challenging the legality of managementrsquos decision to exclude the investorrsquos proposal from the corporationrsquos proxy materialsmdashrather than a possible breach of fiduciary dutymdashis likely to be respected See 3 COX amp HAZEN supra sect 153 (describing situations in which a claim can be framed as derivative or direct)

[NOTE Some issues under Delaware corporate law regarding pre-suit demand are not relevant here For example whether the Mega directors are independent and disinterested is not relevant to the MBCA requirement of a pre-suit demand As the Official Comment to MBCA sect 742 points out the MBCArsquos requirement of ldquouniversal demandrdquo gives the board ldquothe opportunity to reexamine the act complained of in the light of a potential lawsuit and take corrective actionrdquo even when the directors might be non-independent or have conflicts of interest

Nor is it relevant to the MBCA pre-suit demand requirement that the statutory 90-day waiting period may be onerous The first paragraph of MBCA sect 742 requires a pre-suit demand without exception the second paragraph of the section imposes a 90-day waiting period before a derivative suit may be brought which can be shortened if the board rejects the demand or ldquoirreparable injury to the corporation would result by waiting for the expiration of the 90-day periodrdquo The call as written asks only whether a pre-suit demand should be made and does not ask examinees to address whether the post-demand waiting period should be shortened under the ldquoirreparable injuryrdquo standard]

35

National Conference of Bar Examiners 302 South Bedford Street | Madison WI 53703-3622 Phone 608-280-8550 | Fax 608-280-8552 | TDD 608-661-1275

wwwncbexorg e-mail contactncbexorg

  • Preface
  • Description of the MEE
  • Instructions
  • July 2014 Questions
    • CRIMINAL LAW AND PROCEDURE QUESTION
    • CONTRACTS QUESTION
    • FAMILY LAW QUESTION
    • FEDERAL CIVIL PROCEDURE QUESTION
    • EVIDENCE QUESTION
    • CORPORATIONS QUESTION
      • July 2014 Analyses
        • CRIMINAL LAW AND PROCEDURE ANALYSIS
        • CONTRACTS ANALYSIS
        • FAMILY LAW ANALYSIS
        • FEDERAL CIVIL PROCEDURE ANALYSIS
        • EVIDENCE ANALYSIS
        • CORPORATIONS ANALYSIS
            • ltlt13 ASCII85EncodePages false13 AllowTransparency false13 AutoPositionEPSFiles true13 AutoRotatePages None13 Binding Left13 CalGrayProfile (Dot Gain 20)13 CalRGBProfile (sRGB IEC61966-21)13 CalCMYKProfile (US Web Coated 050SWOP051 v2)13 sRGBProfile (sRGB IEC61966-21)13 CannotEmbedFontPolicy Error13 CompatibilityLevel 1413 CompressObjects Tags13 CompressPages true13 ConvertImagesToIndexed true13 PassThroughJPEGImages true13 CreateJobTicket false13 DefaultRenderingIntent Default13 DetectBlends true13 DetectCurves 0000013 ColorConversionStrategy CMYK13 DoThumbnails false13 EmbedAllFonts true13 EmbedOpenType false13 ParseICCProfilesInComments true13 EmbedJobOptions true13 DSCReportingLevel 013 EmitDSCWarnings false13 EndPage -113 ImageMemory 104857613 LockDistillerParams false13 MaxSubsetPct 10013 Optimize true13 OPM 113 ParseDSCComments true13 ParseDSCCommentsForDocInfo true13 PreserveCopyPage true13 PreserveDICMYKValues true13 PreserveEPSInfo true13 PreserveFlatness true13 PreserveHalftoneInfo false13 PreserveOPIComments true13 PreserveOverprintSettings true13 StartPage 113 SubsetFonts true13 TransferFunctionInfo Apply13 UCRandBGInfo Preserve13 UsePrologue false13 ColorSettingsFile ()13 AlwaysEmbed [ true13 ]13 NeverEmbed [ true13 ]13 AntiAliasColorImages false13 CropColorImages true13 ColorImageMinResolution 30013 ColorImageMinResolutionPolicy OK13 DownsampleColorImages true13 ColorImageDownsampleType Bicubic13 ColorImageResolution 30013 ColorImageDepth -113 ColorImageMinDownsampleDepth 113 ColorImageDownsampleThreshold 15000013 EncodeColorImages true13 ColorImageFilter DCTEncode13 AutoFilterColorImages true13 ColorImageAutoFilterStrategy JPEG13 ColorACSImageDict ltlt13 QFactor 01513 HSamples [1 1 1 1] VSamples [1 1 1 1]13 gtgt13 ColorImageDict ltlt13 QFactor 01513 HSamples [1 1 1 1] VSamples [1 1 1 1]13 gtgt13 JPEG2000ColorACSImageDict ltlt13 TileWidth 25613 TileHeight 25613 Quality 3013 gtgt13 JPEG2000ColorImageDict ltlt13 TileWidth 25613 TileHeight 25613 Quality 3013 gtgt13 AntiAliasGrayImages false13 CropGrayImages true13 GrayImageMinResolution 30013 GrayImageMinResolutionPolicy OK13 DownsampleGrayImages true13 GrayImageDownsampleType Bicubic13 GrayImageResolution 30013 GrayImageDepth -113 GrayImageMinDownsampleDepth 213 GrayImageDownsampleThreshold 15000013 EncodeGrayImages true13 GrayImageFilter DCTEncode13 AutoFilterGrayImages true13 GrayImageAutoFilterStrategy JPEG13 GrayACSImageDict ltlt13 QFactor 01513 HSamples [1 1 1 1] VSamples [1 1 1 1]13 gtgt13 GrayImageDict ltlt13 QFactor 01513 HSamples [1 1 1 1] VSamples [1 1 1 1]13 gtgt13 JPEG2000GrayACSImageDict ltlt13 TileWidth 25613 TileHeight 25613 Quality 3013 gtgt13 JPEG2000GrayImageDict ltlt13 TileWidth 25613 TileHeight 25613 Quality 3013 gtgt13 AntiAliasMonoImages false13 CropMonoImages true13 MonoImageMinResolution 120013 MonoImageMinResolutionPolicy OK13 DownsampleMonoImages true13 MonoImageDownsampleType Bicubic13 MonoImageResolution 120013 MonoImageDepth -113 MonoImageDownsampleThreshold 15000013 EncodeMonoImages true13 MonoImageFilter CCITTFaxEncode13 MonoImageDict ltlt13 K -113 gtgt13 AllowPSXObjects false13 CheckCompliance [13 None13 ]13 PDFX1aCheck false13 PDFX3Check false13 PDFXCompliantPDFOnly false13 PDFXNoTrimBoxError true13 PDFXTrimBoxToMediaBoxOffset [13 00000013 00000013 00000013 00000013 ]13 PDFXSetBleedBoxToMediaBox true13 PDFXBleedBoxToTrimBoxOffset [13 00000013 00000013 00000013 00000013 ]13 PDFXOutputIntentProfile ()13 PDFXOutputConditionIdentifier ()13 PDFXOutputCondition ()13 PDFXRegistryName ()13 PDFXTrapped False1313 CreateJDFFile false13 Description ltlt13 ARA 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 BGR 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 CHS ltFEFF4f7f75288fd94e9b8bbe5b9a521b5efa7684002000410064006f006200650020005000440046002065876863900275284e8e9ad88d2891cf76845370524d53705237300260a853ef4ee54f7f75280020004100630072006f0062006100740020548c002000410064006f00620065002000520065006100640065007200200035002e003000204ee553ca66f49ad87248672c676562535f00521b5efa768400200050004400460020658768633002gt13 CHT ltFEFF4f7f752890194e9b8a2d7f6e5efa7acb7684002000410064006f006200650020005000440046002065874ef69069752865bc9ad854c18cea76845370524d5370523786557406300260a853ef4ee54f7f75280020004100630072006f0062006100740020548c002000410064006f00620065002000520065006100640065007200200035002e003000204ee553ca66f49ad87248672c4f86958b555f5df25efa7acb76840020005000440046002065874ef63002gt13 CZE 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 DAN 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 DEU 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 ESP ltFEFF005500740069006c0069006300650020006500730074006100200063006f006e0066006900670075007200610063006900f3006e0020007000610072006100200063007200650061007200200064006f00630075006d0065006e0074006f00730020005000440046002000640065002000410064006f0062006500200061006400650063007500610064006f00730020007000610072006100200069006d0070007200650073006900f3006e0020007000720065002d0065006400690074006f007200690061006c00200064006500200061006c00740061002000630061006c0069006400610064002e002000530065002000700075006500640065006e00200061006200720069007200200064006f00630075006d0065006e0074006f00730020005000440046002000630072006500610064006f007300200063006f006e0020004100630072006f006200610074002c002000410064006f00620065002000520065006100640065007200200035002e003000200079002000760065007200730069006f006e0065007300200070006f00730074006500720069006f007200650073002egt13 ETI 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 FRA 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 GRE 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 HEB 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 HRV (Za stvaranje Adobe PDF dokumenata najpogodnijih za visokokvalitetni ispis prije tiskanja koristite ove postavke Stvoreni PDF dokumenti mogu se otvoriti Acrobat i Adobe Reader 50 i kasnijim verzijama)13 HUN 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 ITA 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 JPN ltFEFF9ad854c18cea306a30d730ea30d730ec30b951fa529b7528002000410064006f0062006500200050004400460020658766f8306e4f5c6210306b4f7f75283057307e305930023053306e8a2d5b9a30674f5c62103055308c305f0020005000440046002030d530a130a430eb306f3001004100630072006f0062006100740020304a30883073002000410064006f00620065002000520065006100640065007200200035002e003000204ee5964d3067958b304f30533068304c3067304d307e305930023053306e8a2d5b9a306b306f30d530a930f330c8306e57cb30818fbc307f304c5fc59808306730593002gt13 KOR ltFEFFc7740020c124c815c7440020c0acc6a9d558c5ec0020ace0d488c9c80020c2dcd5d80020c778c1c4c5d00020ac00c7a50020c801d569d55c002000410064006f0062006500200050004400460020bb38c11cb97c0020c791c131d569b2c8b2e4002e0020c774b807ac8c0020c791c131b41c00200050004400460020bb38c11cb2940020004100630072006f0062006100740020bc0f002000410064006f00620065002000520065006100640065007200200035002e00300020c774c0c1c5d0c11c0020c5f40020c2180020c788c2b5b2c8b2e4002egt13 LTH ltFEFF004e006100750064006f006b0069007400650020016100690075006f007300200070006100720061006d006500740072007500730020006e006f0072011700640061006d00690020006b0075007200740069002000410064006f00620065002000500044004600200064006f006b0075006d0065006e007400750073002c0020006b00750072006900650020006c0061006200690061007500730069006100690020007000720069007400610069006b007900740069002000610075006b01610074006f00730020006b006f006b007900620117007300200070006100720065006e006700740069006e00690061006d00200073007000610075007300640069006e0069006d00750069002e0020002000530075006b0075007200740069002000500044004600200064006f006b0075006d0065006e007400610069002000670061006c006900200062016b007400690020006100740069006400610072006f006d00690020004100630072006f006200610074002000690072002000410064006f00620065002000520065006100640065007200200035002e0030002000610072002000760117006c00650073006e0117006d00690073002000760065007200730069006a006f006d00690073002egt13 LVI 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 NLD (Gebruik deze instellingen om Adobe PDF-documenten te maken die zijn geoptimaliseerd voor prepress-afdrukken van hoge kwaliteit De gemaakte PDF-documenten kunnen worden geopend met Acrobat en Adobe Reader 50 en hoger)13 NOR 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 POL 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 PTB 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 RUM ltFEFF005500740069006c0069007a00610163006900200061006300650073007400650020007300650074010300720069002000700065006e007400720075002000610020006300720065006100200064006f00630075006d0065006e00740065002000410064006f006200650020005000440046002000610064006500630076006100740065002000700065006e0074007200750020007400690070010300720069007200650061002000700072006500700072006500730073002000640065002000630061006c006900740061007400650020007300750070006500720069006f006100720103002e002000200044006f00630075006d0065006e00740065006c00650020005000440046002000630072006500610074006500200070006f00740020006600690020006400650073006300680069007300650020006300750020004100630072006f006200610074002c002000410064006f00620065002000520065006100640065007200200035002e00300020015f00690020007600650072007300690075006e0069006c006500200075006c0074006500720069006f006100720065002egt13 RUS 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 SKY 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 SLV 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 SUO 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 SVE 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 TUR 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 UKR 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 ENU (Use these settings to create Adobe PDF documents best suited for high-quality prepress printing Created PDF documents can be opened with Acrobat and Adobe Reader 50 and later)13 gtgt13 Namespace [13 (Adobe)13 (Common)13 (10)13 ]13 OtherNamespaces [13 ltlt13 AsReaderSpreads false13 CropImagesToFrames true13 ErrorControl WarnAndContinue13 FlattenerIgnoreSpreadOverrides false13 IncludeGuidesGrids false13 IncludeNonPrinting false13 IncludeSlug false13 Namespace [13 (Adobe)13 (InDesign)13 (40)13 ]13 OmitPlacedBitmaps false13 OmitPlacedEPS false13 OmitPlacedPDF false13 SimulateOverprint Legacy13 gtgt13 ltlt13 AddBleedMarks false13 AddColorBars false13 AddCropMarks false13 AddPageInfo false13 AddRegMarks false13 ConvertColors ConvertToCMYK13 DestinationProfileName ()13 DestinationProfileSelector DocumentCMYK13 Downsample16BitImages true13 FlattenerPreset ltlt13 PresetSelector MediumResolution13 gtgt13 FormElements false13 GenerateStructure false13 IncludeBookmarks false13 IncludeHyperlinks false13 IncludeInteractive false13 IncludeLayers false13 IncludeProfiles false13 MultimediaHandling UseObjectSettings13 Namespace [13 (Adobe)13 (CreativeSuite)13 (20)13 ]13 PDFXOutputIntentProfileSelector DocumentCMYK13 PreserveEditing true13 UntaggedCMYKHandling LeaveUntagged13 UntaggedRGBHandling UseDocumentProfile13 UseDocumentBleed false13 gtgt13 ]13gtgt setdistillerparams13ltlt13 HWResolution [2400 2400]13 PageSize [612000 792000]13gtgt setpagedevice13

Page 34: July 2014 MEE Questions and Analyses - NCBE...This publication includes the questions and analyses from the July 2014 MEE. (In the actual test, the questions are simply numbered rather

CORPORATIONS ANALYSIS (Corporations VA2 IX)

ANALYSIS

Legal Problems

(1) Do shareholders have the authority to amend a corporationrsquos bylaws with respect to director nominations

(2) Do board-approved bylaws on a particular subject here nomination of directors preempt subsequent conflicting bylaw amendments by shareholders

(3) Is a suit challenging both managementrsquos refusal to include the proposed bylaw amendment in Megarsquos proxy statement and the boardrsquos amendment of the bylaws dealing with nomination of directors a direct or derivative suit

DISCUSSION

Summary

The voting and litigation rights of the shareholders of Mega are subject to the provisions of the Model Business Corporations Act (MBCA)

The investorrsquos proposed bylaw provision is not inconsistent with state law Under the MBCA shareholders may amend the bylaws when the amendment deals with a proper matter for the corporationrsquos bylaws such as procedures for nominating directors

The Mega boardrsquos bylaw amendment does not preempt the investorrsquos proposed bylaw provision or the Mega shareholdersrsquo power to approve it While shareholders can limit the boardrsquos power to amend or repeal the bylaws the board cannot limit the shareholdersrsquo power

Whether the investor must make a demand on Megarsquos board depends on how the investor frames its claim If the investor claims a violation of shareholder voting rights the claim is direct and pre-suit demand on the board is not required If on the other hand the investor claims that the directors violated their fiduciary duties by amending the bylaws to entrench themselves the claim is derivative and a pre-suit demand is required

Point One (30) Shareholders may amend the corporationrsquos bylaws where the proposed bylaw provision relates to procedural matters typically included in the bylaws such as the nomination of directors

Internal affairs of the corporation such as the conduct of shareholder meetings and election of directors are subject to the corporate law of the state of incorporation See McDermott Inc v Lewis 531 A2d 206 (Del 1987) (applying law of jurisdiction where corporation was incorporated in case involving voting rights) This statersquos corporate statute is modeled on the MBCA

Under the MBCA ldquoshareholders may amend the corporationrsquos bylawsrdquo MBCA sect 1020(a) Thus the only question is whether the bylaws can specify the procedures for shareholder nomination of directors

32

Corporations Analysis

The MBCA states that the bylaws ldquomay contain any provision that is not inconsistent with law or the articles of incorporationrdquo MBCA sect 206(b) In addition the MBCA was revised in 2009 to address shareholder nomination of directors in public corporations (known as ldquoproxy accessrdquo) and specifies that the bylaws ldquomay contain a requirement that the corporation include in its [proxy materials] one or more individuals nominated by a shareholderrdquo MBCA sect 206(c)(1) see Committee on Corporate Laws ABA Section of Business Law Report on the Roles of Boards of Directors and Shareholders of Publicly Owned Corporations and Changes to the Model Business Corporations ActmdashAdoption of Shareholder Proxy Access Amendments to Chapters 2 and 10 65 BUS LAWYER 1105 (2010)

The inclusion of director-nomination procedures in the bylaws is consistent with practice and is recognized by the Delaware courts whose views on corporate law carry significant weight Typically the procedures for nomination of directors are found in the bylaws See 1 COX amp HAZEN TREATISE ON THE LAW OF CORPORATIONS sect 312 (3d ed 2011) see also 4 FLETCHER CORP FORMS ANN PART III ch 21 (2013) (including sample bylaws that permit nomination of directors by shareholders) The Delaware Supreme Court has confirmed that the bylaws may ldquodefine the process and proceduresrdquo for director elections See CA Inc v AFSCME Employees Pension Plan 953 A2d 227 (Del 2008) (concluding that bylaw amendment requiring reimbursement of election expenses to certain successful shareholder nominators is ldquoproper subjectrdquo under Delaware law)

[NOTE The question of the proper scope of the bylaws can be answered using the more general MBCA sect 206(b) or the 2009 MBCA revision adding sect 206(c)(1) (adopted in CT ME VA) In addition some examinees might raise the point that shareholder proposals may not compel the board to take action such as by including shareholder nominations in the companyrsquos proxy materials on the theory that the ldquobusiness and affairsrdquo of the corporation are to be managed by the board See MBCA sect 801(b) Although shareholders are generally limited to adopting precatory resolutions that recommend or encourage board action this limitation does not apply when shareholders have specific authority to take binding action on their ownmdashsuch as to amend the bylaws]

Point Two (30) Shareholders can amend (or repeal) board-approved bylaws Further shareholders can limit the boardrsquos power to later amend and repeal a shareholder-approved bylaw

Under the MBCA shareholders have the power to amend the bylaws See Point One The board shares this power with the shareholders unless (1) the corporationrsquos articles ldquoreserve that power exclusively to the shareholdersrdquo or (2) ldquothe shareholders in amending repealing or adopting a bylaw expressly provide that the board of directors may not amend repeal or reinstate that bylawrdquo See MBCA sect 1020(b)

Shareholder-approved bylaw provisions can amend or repeal existing bylaw provisions whether originally approved by the board or by shareholders See ALAN R PALMITER CORPORATIONS EXAMPLES AND EXPLANATIONS sect 713 (7th ed 2012) Thus the Mega boardrsquos bylaw amendmentmdashwhich set more demanding thresholds for shareholder nomination of directors than the investorrsquos proposed bylaw provisionmdashwould be superseded (repealed) if Megarsquos shareholders were to approve the investorrsquos proposal

Further a shareholder-approved bylaw generally can limit the power of the board to later amend or repeal it See MBCA sect 1020(b)(2) Thus if Megarsquos shareholders approved the bylaw

33

Corporations Analysis

provision proposed by the investor Megarsquos board could not repeal the provision because it includes a ldquono board repealrdquo clause

The revision to the MBCA in 2009 dealing with shareholder proxy access does not change this conclusion That revision specifies that a shareholder-approved bylaw dealing with director nominations may not limit the boardrsquos power to amend add or repeal ldquoany procedure or condition to such a bylaw in order to provide for a reasonable practicable and orderly processrdquo MBCA sect 206(d) Thus according to the revision if shareholders approve a bylaw amendment that limits further board changes the board would nonetheless retain the power to ldquotinkerrdquo with the bylaw to safeguard the voting process but could not repeal the shareholder-approved bylaw The Official Comment to MBCA sect 206(d) makes clear that the revision is ldquonot intended to allow the board of directors to frustrate the purpose of the shareholder-adopted proxy access provisionrdquo Thus if Megarsquos shareholders were to approve the bylaw provision proposed by the investor Megarsquos board could only amend the provision regarding its procedures or conditions in a manner consistent with its purpose of permitting proxy access for Megarsquos shareholders

[NOTE The boardrsquos attempted interference with a shareholder voting initiative may also have been a violation of the directorsrsquo fiduciary duties See Blasius Indus Inc v Atlas Corp 564 A2d 651 (Del Ch 1988) (finding that directors breached their fiduciary duties by amending bylaws and expanding size of board to thwart insurgentrsquos plan to amend bylaws and seat a majority of new directors) The call however asks examinees to consider whether shareholders or the board have ldquoprecedencerdquo over amending the corporate bylaws Thus an examineersquos answer should be framed in terms of ldquopowerrdquo and not ldquodutyrdquo]

Point Three (40) The investor need not make a demand on the board if the investor states a direct claim such as an allegation that the board interfered with the investorrsquos right to amend the bylaws But the investor must make a demand on the board if the investor states a derivative claim (on behalf of the corporation) such as an allegation that the directors sought to entrench themselves by interfering with the proposed proxy access

The MBCA generally requires that shareholders make a demand on the board of directors before initiation of a derivative suit MBCA sect 742 (shareholder may not bring derivative proceeding until written demand has been made on corporation and 90 days have expired) A derivative suit is essentially two suits in one where the plaintiff-shareholder seeks to bring on behalf of the corporation a claim that vindicates corporate rights usually based on violation of fiduciary duties PALMITER supra sect 1811 (6th ed 2009) The demand permits the board to investigate the situation identified by the shareholder and take suitable action No demand on the board is required however if the shareholder brings a direct suit to vindicate the shareholderrsquos own rights not those of the corporation

Is the suit brought by the investor derivative or direct The MBCA defines a ldquoderivative proceedingrdquo as one brought ldquoin the right of a domestic corporationrdquo MBCA sect 740(1) Thus the answer to how the investorrsquos suit should be characterized turns on what rights the investor seeks to vindicate If the investor frames its claim as one of fiduciary breach by directorsmdashfor example for failing to become adequately informed about voting procedures or for seeking to entrench themselves in office by manipulating the voting structure to avoid a shareholder insurgencymdashthen the suit is ldquoderivativerdquo and the investor must make a demand on the board See MBCA Ch 7 Subch D Introductory Comment (ldquothe derivative suit has historically been the principal method of challenging allegedly illegal action by managementrdquo)

34

Corporations Analysis

If however the investor frames its claim as one to vindicate shareholder rights the suit is direct and no demand is required For many courts the direct-derivative question turns on who is injured and who is to receive the relief sought by the plaintiff-shareholders See Tooley v Donaldson Lufkin amp Jenrette Inc 845 A2d 1031 (Del 2004) (characterizing a merger-delay claim as direct because delay of merger only harmed shareholders not corporation) Thus if the investor claims that managementrsquos refusal to include its proposed bylaw amendment in the corporationrsquos proxy materials violates its shareholder rights to initiate corporate governance reforms the suit will be direct Courts have not questioned the ability of shareholders to bring direct suits challenging board action to exclude their proposed bylaw amendments from the corporationrsquos proxy materials See JANA Master Fund Ltd v CNET Networks Inc 954 A2d 335 (Del Ch 2008) (upholding shareholderrsquos direct challenge to boardrsquos interpretation of advance-notice bylaw) Chesapeake Corp v Shore 771 A2d 293 (Del Ch 2000) (upholding shareholderrsquos direct challenge to actions by board that effectively prevented it from proposing bylaw amendments in contest for control)

Is the way that the investor frames its claim conclusive Courts have permitted shareholder-plaintiffs to challenge a transaction in a direct suit even though the same transaction could also be challenged as a fiduciary breach See Eisenberg v Flying Tiger Line Inc 451 F2d 267 (2d Cir 1971) (permitting direct suit challenging a corporate reorganization as a dilution of shareholder voting power even though reorganization may have involved conflicts of interest and thus constituted a fiduciary breach) Thus the investorrsquos choice to pursue a claim challenging the legality of managementrsquos decision to exclude the investorrsquos proposal from the corporationrsquos proxy materialsmdashrather than a possible breach of fiduciary dutymdashis likely to be respected See 3 COX amp HAZEN supra sect 153 (describing situations in which a claim can be framed as derivative or direct)

[NOTE Some issues under Delaware corporate law regarding pre-suit demand are not relevant here For example whether the Mega directors are independent and disinterested is not relevant to the MBCA requirement of a pre-suit demand As the Official Comment to MBCA sect 742 points out the MBCArsquos requirement of ldquouniversal demandrdquo gives the board ldquothe opportunity to reexamine the act complained of in the light of a potential lawsuit and take corrective actionrdquo even when the directors might be non-independent or have conflicts of interest

Nor is it relevant to the MBCA pre-suit demand requirement that the statutory 90-day waiting period may be onerous The first paragraph of MBCA sect 742 requires a pre-suit demand without exception the second paragraph of the section imposes a 90-day waiting period before a derivative suit may be brought which can be shortened if the board rejects the demand or ldquoirreparable injury to the corporation would result by waiting for the expiration of the 90-day periodrdquo The call as written asks only whether a pre-suit demand should be made and does not ask examinees to address whether the post-demand waiting period should be shortened under the ldquoirreparable injuryrdquo standard]

35

National Conference of Bar Examiners 302 South Bedford Street | Madison WI 53703-3622 Phone 608-280-8550 | Fax 608-280-8552 | TDD 608-661-1275

wwwncbexorg e-mail contactncbexorg

  • Preface
  • Description of the MEE
  • Instructions
  • July 2014 Questions
    • CRIMINAL LAW AND PROCEDURE QUESTION
    • CONTRACTS QUESTION
    • FAMILY LAW QUESTION
    • FEDERAL CIVIL PROCEDURE QUESTION
    • EVIDENCE QUESTION
    • CORPORATIONS QUESTION
      • July 2014 Analyses
        • CRIMINAL LAW AND PROCEDURE ANALYSIS
        • CONTRACTS ANALYSIS
        • FAMILY LAW ANALYSIS
        • FEDERAL CIVIL PROCEDURE ANALYSIS
        • EVIDENCE ANALYSIS
        • CORPORATIONS ANALYSIS
            • ltlt13 ASCII85EncodePages false13 AllowTransparency false13 AutoPositionEPSFiles true13 AutoRotatePages None13 Binding Left13 CalGrayProfile (Dot Gain 20)13 CalRGBProfile (sRGB IEC61966-21)13 CalCMYKProfile (US Web Coated 050SWOP051 v2)13 sRGBProfile (sRGB IEC61966-21)13 CannotEmbedFontPolicy Error13 CompatibilityLevel 1413 CompressObjects Tags13 CompressPages true13 ConvertImagesToIndexed true13 PassThroughJPEGImages true13 CreateJobTicket false13 DefaultRenderingIntent Default13 DetectBlends true13 DetectCurves 0000013 ColorConversionStrategy CMYK13 DoThumbnails false13 EmbedAllFonts true13 EmbedOpenType false13 ParseICCProfilesInComments true13 EmbedJobOptions true13 DSCReportingLevel 013 EmitDSCWarnings false13 EndPage -113 ImageMemory 104857613 LockDistillerParams false13 MaxSubsetPct 10013 Optimize true13 OPM 113 ParseDSCComments true13 ParseDSCCommentsForDocInfo true13 PreserveCopyPage true13 PreserveDICMYKValues true13 PreserveEPSInfo true13 PreserveFlatness true13 PreserveHalftoneInfo false13 PreserveOPIComments true13 PreserveOverprintSettings true13 StartPage 113 SubsetFonts true13 TransferFunctionInfo Apply13 UCRandBGInfo Preserve13 UsePrologue false13 ColorSettingsFile ()13 AlwaysEmbed [ true13 ]13 NeverEmbed [ true13 ]13 AntiAliasColorImages false13 CropColorImages true13 ColorImageMinResolution 30013 ColorImageMinResolutionPolicy OK13 DownsampleColorImages true13 ColorImageDownsampleType Bicubic13 ColorImageResolution 30013 ColorImageDepth -113 ColorImageMinDownsampleDepth 113 ColorImageDownsampleThreshold 15000013 EncodeColorImages true13 ColorImageFilter DCTEncode13 AutoFilterColorImages true13 ColorImageAutoFilterStrategy JPEG13 ColorACSImageDict ltlt13 QFactor 01513 HSamples [1 1 1 1] VSamples [1 1 1 1]13 gtgt13 ColorImageDict ltlt13 QFactor 01513 HSamples [1 1 1 1] VSamples [1 1 1 1]13 gtgt13 JPEG2000ColorACSImageDict ltlt13 TileWidth 25613 TileHeight 25613 Quality 3013 gtgt13 JPEG2000ColorImageDict ltlt13 TileWidth 25613 TileHeight 25613 Quality 3013 gtgt13 AntiAliasGrayImages false13 CropGrayImages true13 GrayImageMinResolution 30013 GrayImageMinResolutionPolicy OK13 DownsampleGrayImages true13 GrayImageDownsampleType Bicubic13 GrayImageResolution 30013 GrayImageDepth -113 GrayImageMinDownsampleDepth 213 GrayImageDownsampleThreshold 15000013 EncodeGrayImages true13 GrayImageFilter DCTEncode13 AutoFilterGrayImages true13 GrayImageAutoFilterStrategy JPEG13 GrayACSImageDict ltlt13 QFactor 01513 HSamples [1 1 1 1] VSamples [1 1 1 1]13 gtgt13 GrayImageDict ltlt13 QFactor 01513 HSamples [1 1 1 1] VSamples [1 1 1 1]13 gtgt13 JPEG2000GrayACSImageDict ltlt13 TileWidth 25613 TileHeight 25613 Quality 3013 gtgt13 JPEG2000GrayImageDict ltlt13 TileWidth 25613 TileHeight 25613 Quality 3013 gtgt13 AntiAliasMonoImages false13 CropMonoImages true13 MonoImageMinResolution 120013 MonoImageMinResolutionPolicy OK13 DownsampleMonoImages true13 MonoImageDownsampleType Bicubic13 MonoImageResolution 120013 MonoImageDepth -113 MonoImageDownsampleThreshold 15000013 EncodeMonoImages true13 MonoImageFilter CCITTFaxEncode13 MonoImageDict ltlt13 K -113 gtgt13 AllowPSXObjects false13 CheckCompliance [13 None13 ]13 PDFX1aCheck false13 PDFX3Check false13 PDFXCompliantPDFOnly false13 PDFXNoTrimBoxError true13 PDFXTrimBoxToMediaBoxOffset [13 00000013 00000013 00000013 00000013 ]13 PDFXSetBleedBoxToMediaBox true13 PDFXBleedBoxToTrimBoxOffset [13 00000013 00000013 00000013 00000013 ]13 PDFXOutputIntentProfile ()13 PDFXOutputConditionIdentifier ()13 PDFXOutputCondition ()13 PDFXRegistryName ()13 PDFXTrapped False1313 CreateJDFFile false13 Description ltlt13 ARA ltFEFF06270633062A062E062F0645002006470630064700200627064406250639062F0627062F0627062A002006440625064606340627062100200648062B062706260642002000410064006F00620065002000500044004600200645062A064806270641064206290020064406440637062806270639062900200641064A00200627064406450637062706280639002006300627062A0020062F0631062C0627062A002006270644062C0648062F0629002006270644063906270644064A0629061B0020064A06450643064600200641062A062D00200648062B0627062606420020005000440046002006270644064506460634062306290020062806270633062A062E062F062706450020004100630072006F0062006100740020064800410064006F006200650020005200650061006400650072002006250635062F0627063100200035002E0030002006480627064406250635062F062706310627062A0020062706440623062D062F062B002E0635062F0627063100200035002E0030002006480627064406250635062F062706310627062A0020062706440623062D062F062B002Egt13 BGR ltFEFF04180437043f043e043b043704320430043904420435002004420435043704380020043d0430044104420440043e0439043a0438002c00200437043000200434043000200441044a0437043404300432043004420435002000410064006f00620065002000500044004600200434043e043a0443043c0435043d04420438002c0020043c0430043a04410438043c0430043b043d043e0020043f044004380433043e04340435043d04380020043704300020043204380441043e043a043e043a0430044704350441044204320435043d0020043f04350447043004420020043704300020043f044004350434043f0435044704300442043d04300020043f043e04340433043e0442043e0432043a0430002e002000200421044a04370434043004340435043d043804420435002000500044004600200434043e043a0443043c0435043d044204380020043c043e0433043004420020043404300020044104350020043e0442043204300440044f0442002004410020004100630072006f00620061007400200438002000410064006f00620065002000520065006100640065007200200035002e00300020043800200441043b0435043404320430044904380020043204350440044104380438002egt13 CHS ltFEFF4f7f75288fd94e9b8bbe5b9a521b5efa7684002000410064006f006200650020005000440046002065876863900275284e8e9ad88d2891cf76845370524d53705237300260a853ef4ee54f7f75280020004100630072006f0062006100740020548c002000410064006f00620065002000520065006100640065007200200035002e003000204ee553ca66f49ad87248672c676562535f00521b5efa768400200050004400460020658768633002gt13 CHT ltFEFF4f7f752890194e9b8a2d7f6e5efa7acb7684002000410064006f006200650020005000440046002065874ef69069752865bc9ad854c18cea76845370524d5370523786557406300260a853ef4ee54f7f75280020004100630072006f0062006100740020548c002000410064006f00620065002000520065006100640065007200200035002e003000204ee553ca66f49ad87248672c4f86958b555f5df25efa7acb76840020005000440046002065874ef63002gt13 CZE 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 DAN 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 DEU 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 ESP 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 ETI 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 FRA 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 GRE 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 HEB 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 HRV (Za stvaranje Adobe PDF dokumenata najpogodnijih za visokokvalitetni ispis prije tiskanja koristite ove postavke Stvoreni PDF dokumenti mogu se otvoriti Acrobat i Adobe Reader 50 i kasnijim verzijama)13 HUN ltFEFF004b0069007600e1006c00f30020006d0069006e0151007300e9006701710020006e0079006f006d00640061006900200065006c0151006b00e90073007a00ed007401510020006e0079006f006d00740061007400e100730068006f007a0020006c006500670069006e006b00e1006200620020006d0065006700660065006c0065006c0151002000410064006f00620065002000500044004600200064006f006b0075006d0065006e00740075006d006f006b0061007400200065007a0065006b006b0065006c0020006100200062006500e1006c006c00ed007400e10073006f006b006b0061006c0020006b00e90073007a00ed0074006800650074002e0020002000410020006c00e90074007200650068006f007a006f00740074002000500044004600200064006f006b0075006d0065006e00740075006d006f006b00200061007a0020004100630072006f006200610074002000e9007300200061007a002000410064006f00620065002000520065006100640065007200200035002e0030002c0020007600610067007900200061007a002000610074007400f3006c0020006b00e9007301510062006200690020007600650072007a006900f3006b006b0061006c0020006e00790069007400680061007400f3006b0020006d00650067002egt13 ITA 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 JPN ltFEFF9ad854c18cea306a30d730ea30d730ec30b951fa529b7528002000410064006f0062006500200050004400460020658766f8306e4f5c6210306b4f7f75283057307e305930023053306e8a2d5b9a30674f5c62103055308c305f0020005000440046002030d530a130a430eb306f3001004100630072006f0062006100740020304a30883073002000410064006f00620065002000520065006100640065007200200035002e003000204ee5964d3067958b304f30533068304c3067304d307e305930023053306e8a2d5b9a306b306f30d530a930f330c8306e57cb30818fbc307f304c5fc59808306730593002gt13 KOR ltFEFFc7740020c124c815c7440020c0acc6a9d558c5ec0020ace0d488c9c80020c2dcd5d80020c778c1c4c5d00020ac00c7a50020c801d569d55c002000410064006f0062006500200050004400460020bb38c11cb97c0020c791c131d569b2c8b2e4002e0020c774b807ac8c0020c791c131b41c00200050004400460020bb38c11cb2940020004100630072006f0062006100740020bc0f002000410064006f00620065002000520065006100640065007200200035002e00300020c774c0c1c5d0c11c0020c5f40020c2180020c788c2b5b2c8b2e4002egt13 LTH 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 LVI 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 NLD (Gebruik deze instellingen om Adobe PDF-documenten te maken die zijn geoptimaliseerd voor prepress-afdrukken van hoge kwaliteit De gemaakte PDF-documenten kunnen worden geopend met Acrobat en Adobe Reader 50 en hoger)13 NOR 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 POL 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 PTB ltFEFF005500740069006c0069007a006500200065007300730061007300200063006f006e00660069006700750072006100e700f50065007300200064006500200066006f0072006d00610020006100200063007200690061007200200064006f00630075006d0065006e0074006f0073002000410064006f0062006500200050004400460020006d00610069007300200061006400650071007500610064006f00730020007000610072006100200070007200e9002d0069006d0070007200650073007300f50065007300200064006500200061006c007400610020007100750061006c00690064006100640065002e0020004f007300200064006f00630075006d0065006e0074006f00730020005000440046002000630072006900610064006f007300200070006f00640065006d0020007300650072002000610062006500720074006f007300200063006f006d0020006f0020004100630072006f006200610074002000650020006f002000410064006f00620065002000520065006100640065007200200035002e0030002000650020007600650072007300f50065007300200070006f00730074006500720069006f007200650073002egt13 RUM 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 RUS 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 SKY ltFEFF0054006900650074006f0020006e006100730074006100760065006e0069006100200070006f0075017e0069007400650020006e00610020007600790074007600e100720061006e0069006500200064006f006b0075006d0065006e0074006f0076002000410064006f006200650020005000440046002c0020006b0074006f007200e90020007300610020006e0061006a006c0065007001610069006500200068006f0064006900610020006e00610020006b00760061006c00690074006e00fa00200074006c0061010d00200061002000700072006500700072006500730073002e00200056007900740076006f00720065006e00e900200064006f006b0075006d0065006e007400790020005000440046002000620075006400650020006d006f017e006e00e90020006f00740076006f00720069016500200076002000700072006f006700720061006d006f006300680020004100630072006f00620061007400200061002000410064006f00620065002000520065006100640065007200200035002e0030002000610020006e006f0076016100ed00630068002egt13 SLV 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 SUO 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 SVE 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 TUR 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 UKR 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 ENU (Use these settings to create Adobe PDF documents best suited for high-quality prepress printing Created PDF documents can be opened with Acrobat and Adobe Reader 50 and later)13 gtgt13 Namespace [13 (Adobe)13 (Common)13 (10)13 ]13 OtherNamespaces [13 ltlt13 AsReaderSpreads false13 CropImagesToFrames true13 ErrorControl WarnAndContinue13 FlattenerIgnoreSpreadOverrides false13 IncludeGuidesGrids false13 IncludeNonPrinting false13 IncludeSlug false13 Namespace [13 (Adobe)13 (InDesign)13 (40)13 ]13 OmitPlacedBitmaps false13 OmitPlacedEPS false13 OmitPlacedPDF false13 SimulateOverprint Legacy13 gtgt13 ltlt13 AddBleedMarks false13 AddColorBars false13 AddCropMarks false13 AddPageInfo false13 AddRegMarks false13 ConvertColors ConvertToCMYK13 DestinationProfileName ()13 DestinationProfileSelector DocumentCMYK13 Downsample16BitImages true13 FlattenerPreset ltlt13 PresetSelector MediumResolution13 gtgt13 FormElements false13 GenerateStructure false13 IncludeBookmarks false13 IncludeHyperlinks false13 IncludeInteractive false13 IncludeLayers false13 IncludeProfiles false13 MultimediaHandling UseObjectSettings13 Namespace [13 (Adobe)13 (CreativeSuite)13 (20)13 ]13 PDFXOutputIntentProfileSelector DocumentCMYK13 PreserveEditing true13 UntaggedCMYKHandling LeaveUntagged13 UntaggedRGBHandling UseDocumentProfile13 UseDocumentBleed false13 gtgt13 ]13gtgt setdistillerparams13ltlt13 HWResolution [2400 2400]13 PageSize [612000 792000]13gtgt setpagedevice13

Page 35: July 2014 MEE Questions and Analyses - NCBE...This publication includes the questions and analyses from the July 2014 MEE. (In the actual test, the questions are simply numbered rather

Corporations Analysis

The MBCA states that the bylaws ldquomay contain any provision that is not inconsistent with law or the articles of incorporationrdquo MBCA sect 206(b) In addition the MBCA was revised in 2009 to address shareholder nomination of directors in public corporations (known as ldquoproxy accessrdquo) and specifies that the bylaws ldquomay contain a requirement that the corporation include in its [proxy materials] one or more individuals nominated by a shareholderrdquo MBCA sect 206(c)(1) see Committee on Corporate Laws ABA Section of Business Law Report on the Roles of Boards of Directors and Shareholders of Publicly Owned Corporations and Changes to the Model Business Corporations ActmdashAdoption of Shareholder Proxy Access Amendments to Chapters 2 and 10 65 BUS LAWYER 1105 (2010)

The inclusion of director-nomination procedures in the bylaws is consistent with practice and is recognized by the Delaware courts whose views on corporate law carry significant weight Typically the procedures for nomination of directors are found in the bylaws See 1 COX amp HAZEN TREATISE ON THE LAW OF CORPORATIONS sect 312 (3d ed 2011) see also 4 FLETCHER CORP FORMS ANN PART III ch 21 (2013) (including sample bylaws that permit nomination of directors by shareholders) The Delaware Supreme Court has confirmed that the bylaws may ldquodefine the process and proceduresrdquo for director elections See CA Inc v AFSCME Employees Pension Plan 953 A2d 227 (Del 2008) (concluding that bylaw amendment requiring reimbursement of election expenses to certain successful shareholder nominators is ldquoproper subjectrdquo under Delaware law)

[NOTE The question of the proper scope of the bylaws can be answered using the more general MBCA sect 206(b) or the 2009 MBCA revision adding sect 206(c)(1) (adopted in CT ME VA) In addition some examinees might raise the point that shareholder proposals may not compel the board to take action such as by including shareholder nominations in the companyrsquos proxy materials on the theory that the ldquobusiness and affairsrdquo of the corporation are to be managed by the board See MBCA sect 801(b) Although shareholders are generally limited to adopting precatory resolutions that recommend or encourage board action this limitation does not apply when shareholders have specific authority to take binding action on their ownmdashsuch as to amend the bylaws]

Point Two (30) Shareholders can amend (or repeal) board-approved bylaws Further shareholders can limit the boardrsquos power to later amend and repeal a shareholder-approved bylaw

Under the MBCA shareholders have the power to amend the bylaws See Point One The board shares this power with the shareholders unless (1) the corporationrsquos articles ldquoreserve that power exclusively to the shareholdersrdquo or (2) ldquothe shareholders in amending repealing or adopting a bylaw expressly provide that the board of directors may not amend repeal or reinstate that bylawrdquo See MBCA sect 1020(b)

Shareholder-approved bylaw provisions can amend or repeal existing bylaw provisions whether originally approved by the board or by shareholders See ALAN R PALMITER CORPORATIONS EXAMPLES AND EXPLANATIONS sect 713 (7th ed 2012) Thus the Mega boardrsquos bylaw amendmentmdashwhich set more demanding thresholds for shareholder nomination of directors than the investorrsquos proposed bylaw provisionmdashwould be superseded (repealed) if Megarsquos shareholders were to approve the investorrsquos proposal

Further a shareholder-approved bylaw generally can limit the power of the board to later amend or repeal it See MBCA sect 1020(b)(2) Thus if Megarsquos shareholders approved the bylaw

33

Corporations Analysis

provision proposed by the investor Megarsquos board could not repeal the provision because it includes a ldquono board repealrdquo clause

The revision to the MBCA in 2009 dealing with shareholder proxy access does not change this conclusion That revision specifies that a shareholder-approved bylaw dealing with director nominations may not limit the boardrsquos power to amend add or repeal ldquoany procedure or condition to such a bylaw in order to provide for a reasonable practicable and orderly processrdquo MBCA sect 206(d) Thus according to the revision if shareholders approve a bylaw amendment that limits further board changes the board would nonetheless retain the power to ldquotinkerrdquo with the bylaw to safeguard the voting process but could not repeal the shareholder-approved bylaw The Official Comment to MBCA sect 206(d) makes clear that the revision is ldquonot intended to allow the board of directors to frustrate the purpose of the shareholder-adopted proxy access provisionrdquo Thus if Megarsquos shareholders were to approve the bylaw provision proposed by the investor Megarsquos board could only amend the provision regarding its procedures or conditions in a manner consistent with its purpose of permitting proxy access for Megarsquos shareholders

[NOTE The boardrsquos attempted interference with a shareholder voting initiative may also have been a violation of the directorsrsquo fiduciary duties See Blasius Indus Inc v Atlas Corp 564 A2d 651 (Del Ch 1988) (finding that directors breached their fiduciary duties by amending bylaws and expanding size of board to thwart insurgentrsquos plan to amend bylaws and seat a majority of new directors) The call however asks examinees to consider whether shareholders or the board have ldquoprecedencerdquo over amending the corporate bylaws Thus an examineersquos answer should be framed in terms of ldquopowerrdquo and not ldquodutyrdquo]

Point Three (40) The investor need not make a demand on the board if the investor states a direct claim such as an allegation that the board interfered with the investorrsquos right to amend the bylaws But the investor must make a demand on the board if the investor states a derivative claim (on behalf of the corporation) such as an allegation that the directors sought to entrench themselves by interfering with the proposed proxy access

The MBCA generally requires that shareholders make a demand on the board of directors before initiation of a derivative suit MBCA sect 742 (shareholder may not bring derivative proceeding until written demand has been made on corporation and 90 days have expired) A derivative suit is essentially two suits in one where the plaintiff-shareholder seeks to bring on behalf of the corporation a claim that vindicates corporate rights usually based on violation of fiduciary duties PALMITER supra sect 1811 (6th ed 2009) The demand permits the board to investigate the situation identified by the shareholder and take suitable action No demand on the board is required however if the shareholder brings a direct suit to vindicate the shareholderrsquos own rights not those of the corporation

Is the suit brought by the investor derivative or direct The MBCA defines a ldquoderivative proceedingrdquo as one brought ldquoin the right of a domestic corporationrdquo MBCA sect 740(1) Thus the answer to how the investorrsquos suit should be characterized turns on what rights the investor seeks to vindicate If the investor frames its claim as one of fiduciary breach by directorsmdashfor example for failing to become adequately informed about voting procedures or for seeking to entrench themselves in office by manipulating the voting structure to avoid a shareholder insurgencymdashthen the suit is ldquoderivativerdquo and the investor must make a demand on the board See MBCA Ch 7 Subch D Introductory Comment (ldquothe derivative suit has historically been the principal method of challenging allegedly illegal action by managementrdquo)

34

Corporations Analysis

If however the investor frames its claim as one to vindicate shareholder rights the suit is direct and no demand is required For many courts the direct-derivative question turns on who is injured and who is to receive the relief sought by the plaintiff-shareholders See Tooley v Donaldson Lufkin amp Jenrette Inc 845 A2d 1031 (Del 2004) (characterizing a merger-delay claim as direct because delay of merger only harmed shareholders not corporation) Thus if the investor claims that managementrsquos refusal to include its proposed bylaw amendment in the corporationrsquos proxy materials violates its shareholder rights to initiate corporate governance reforms the suit will be direct Courts have not questioned the ability of shareholders to bring direct suits challenging board action to exclude their proposed bylaw amendments from the corporationrsquos proxy materials See JANA Master Fund Ltd v CNET Networks Inc 954 A2d 335 (Del Ch 2008) (upholding shareholderrsquos direct challenge to boardrsquos interpretation of advance-notice bylaw) Chesapeake Corp v Shore 771 A2d 293 (Del Ch 2000) (upholding shareholderrsquos direct challenge to actions by board that effectively prevented it from proposing bylaw amendments in contest for control)

Is the way that the investor frames its claim conclusive Courts have permitted shareholder-plaintiffs to challenge a transaction in a direct suit even though the same transaction could also be challenged as a fiduciary breach See Eisenberg v Flying Tiger Line Inc 451 F2d 267 (2d Cir 1971) (permitting direct suit challenging a corporate reorganization as a dilution of shareholder voting power even though reorganization may have involved conflicts of interest and thus constituted a fiduciary breach) Thus the investorrsquos choice to pursue a claim challenging the legality of managementrsquos decision to exclude the investorrsquos proposal from the corporationrsquos proxy materialsmdashrather than a possible breach of fiduciary dutymdashis likely to be respected See 3 COX amp HAZEN supra sect 153 (describing situations in which a claim can be framed as derivative or direct)

[NOTE Some issues under Delaware corporate law regarding pre-suit demand are not relevant here For example whether the Mega directors are independent and disinterested is not relevant to the MBCA requirement of a pre-suit demand As the Official Comment to MBCA sect 742 points out the MBCArsquos requirement of ldquouniversal demandrdquo gives the board ldquothe opportunity to reexamine the act complained of in the light of a potential lawsuit and take corrective actionrdquo even when the directors might be non-independent or have conflicts of interest

Nor is it relevant to the MBCA pre-suit demand requirement that the statutory 90-day waiting period may be onerous The first paragraph of MBCA sect 742 requires a pre-suit demand without exception the second paragraph of the section imposes a 90-day waiting period before a derivative suit may be brought which can be shortened if the board rejects the demand or ldquoirreparable injury to the corporation would result by waiting for the expiration of the 90-day periodrdquo The call as written asks only whether a pre-suit demand should be made and does not ask examinees to address whether the post-demand waiting period should be shortened under the ldquoirreparable injuryrdquo standard]

35

National Conference of Bar Examiners 302 South Bedford Street | Madison WI 53703-3622 Phone 608-280-8550 | Fax 608-280-8552 | TDD 608-661-1275

wwwncbexorg e-mail contactncbexorg

  • Preface
  • Description of the MEE
  • Instructions
  • July 2014 Questions
    • CRIMINAL LAW AND PROCEDURE QUESTION
    • CONTRACTS QUESTION
    • FAMILY LAW QUESTION
    • FEDERAL CIVIL PROCEDURE QUESTION
    • EVIDENCE QUESTION
    • CORPORATIONS QUESTION
      • July 2014 Analyses
        • CRIMINAL LAW AND PROCEDURE ANALYSIS
        • CONTRACTS ANALYSIS
        • FAMILY LAW ANALYSIS
        • FEDERAL CIVIL PROCEDURE ANALYSIS
        • EVIDENCE ANALYSIS
        • CORPORATIONS ANALYSIS
            • ltlt13 ASCII85EncodePages false13 AllowTransparency false13 AutoPositionEPSFiles true13 AutoRotatePages None13 Binding Left13 CalGrayProfile (Dot Gain 20)13 CalRGBProfile (sRGB IEC61966-21)13 CalCMYKProfile (US Web Coated 050SWOP051 v2)13 sRGBProfile (sRGB IEC61966-21)13 CannotEmbedFontPolicy Error13 CompatibilityLevel 1413 CompressObjects Tags13 CompressPages true13 ConvertImagesToIndexed true13 PassThroughJPEGImages true13 CreateJobTicket false13 DefaultRenderingIntent Default13 DetectBlends true13 DetectCurves 0000013 ColorConversionStrategy CMYK13 DoThumbnails false13 EmbedAllFonts true13 EmbedOpenType false13 ParseICCProfilesInComments true13 EmbedJobOptions true13 DSCReportingLevel 013 EmitDSCWarnings false13 EndPage -113 ImageMemory 104857613 LockDistillerParams false13 MaxSubsetPct 10013 Optimize true13 OPM 113 ParseDSCComments true13 ParseDSCCommentsForDocInfo true13 PreserveCopyPage true13 PreserveDICMYKValues true13 PreserveEPSInfo true13 PreserveFlatness true13 PreserveHalftoneInfo false13 PreserveOPIComments true13 PreserveOverprintSettings true13 StartPage 113 SubsetFonts true13 TransferFunctionInfo Apply13 UCRandBGInfo Preserve13 UsePrologue false13 ColorSettingsFile ()13 AlwaysEmbed [ true13 ]13 NeverEmbed [ true13 ]13 AntiAliasColorImages false13 CropColorImages true13 ColorImageMinResolution 30013 ColorImageMinResolutionPolicy OK13 DownsampleColorImages true13 ColorImageDownsampleType Bicubic13 ColorImageResolution 30013 ColorImageDepth -113 ColorImageMinDownsampleDepth 113 ColorImageDownsampleThreshold 15000013 EncodeColorImages true13 ColorImageFilter DCTEncode13 AutoFilterColorImages true13 ColorImageAutoFilterStrategy JPEG13 ColorACSImageDict ltlt13 QFactor 01513 HSamples [1 1 1 1] VSamples [1 1 1 1]13 gtgt13 ColorImageDict ltlt13 QFactor 01513 HSamples [1 1 1 1] VSamples [1 1 1 1]13 gtgt13 JPEG2000ColorACSImageDict ltlt13 TileWidth 25613 TileHeight 25613 Quality 3013 gtgt13 JPEG2000ColorImageDict ltlt13 TileWidth 25613 TileHeight 25613 Quality 3013 gtgt13 AntiAliasGrayImages false13 CropGrayImages true13 GrayImageMinResolution 30013 GrayImageMinResolutionPolicy OK13 DownsampleGrayImages true13 GrayImageDownsampleType Bicubic13 GrayImageResolution 30013 GrayImageDepth -113 GrayImageMinDownsampleDepth 213 GrayImageDownsampleThreshold 15000013 EncodeGrayImages true13 GrayImageFilter DCTEncode13 AutoFilterGrayImages true13 GrayImageAutoFilterStrategy JPEG13 GrayACSImageDict ltlt13 QFactor 01513 HSamples [1 1 1 1] VSamples [1 1 1 1]13 gtgt13 GrayImageDict ltlt13 QFactor 01513 HSamples [1 1 1 1] VSamples [1 1 1 1]13 gtgt13 JPEG2000GrayACSImageDict ltlt13 TileWidth 25613 TileHeight 25613 Quality 3013 gtgt13 JPEG2000GrayImageDict ltlt13 TileWidth 25613 TileHeight 25613 Quality 3013 gtgt13 AntiAliasMonoImages false13 CropMonoImages true13 MonoImageMinResolution 120013 MonoImageMinResolutionPolicy OK13 DownsampleMonoImages true13 MonoImageDownsampleType Bicubic13 MonoImageResolution 120013 MonoImageDepth -113 MonoImageDownsampleThreshold 15000013 EncodeMonoImages true13 MonoImageFilter CCITTFaxEncode13 MonoImageDict ltlt13 K -113 gtgt13 AllowPSXObjects false13 CheckCompliance [13 None13 ]13 PDFX1aCheck false13 PDFX3Check false13 PDFXCompliantPDFOnly false13 PDFXNoTrimBoxError true13 PDFXTrimBoxToMediaBoxOffset [13 00000013 00000013 00000013 00000013 ]13 PDFXSetBleedBoxToMediaBox true13 PDFXBleedBoxToTrimBoxOffset [13 00000013 00000013 00000013 00000013 ]13 PDFXOutputIntentProfile ()13 PDFXOutputConditionIdentifier ()13 PDFXOutputCondition ()13 PDFXRegistryName ()13 PDFXTrapped False1313 CreateJDFFile false13 Description ltlt13 ARA 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 BGR ltFEFF04180437043f043e043b043704320430043904420435002004420435043704380020043d0430044104420440043e0439043a0438002c00200437043000200434043000200441044a0437043404300432043004420435002000410064006f00620065002000500044004600200434043e043a0443043c0435043d04420438002c0020043c0430043a04410438043c0430043b043d043e0020043f044004380433043e04340435043d04380020043704300020043204380441043e043a043e043a0430044704350441044204320435043d0020043f04350447043004420020043704300020043f044004350434043f0435044704300442043d04300020043f043e04340433043e0442043e0432043a0430002e002000200421044a04370434043004340435043d043804420435002000500044004600200434043e043a0443043c0435043d044204380020043c043e0433043004420020043404300020044104350020043e0442043204300440044f0442002004410020004100630072006f00620061007400200438002000410064006f00620065002000520065006100640065007200200035002e00300020043800200441043b0435043404320430044904380020043204350440044104380438002egt13 CHS ltFEFF4f7f75288fd94e9b8bbe5b9a521b5efa7684002000410064006f006200650020005000440046002065876863900275284e8e9ad88d2891cf76845370524d53705237300260a853ef4ee54f7f75280020004100630072006f0062006100740020548c002000410064006f00620065002000520065006100640065007200200035002e003000204ee553ca66f49ad87248672c676562535f00521b5efa768400200050004400460020658768633002gt13 CHT ltFEFF4f7f752890194e9b8a2d7f6e5efa7acb7684002000410064006f006200650020005000440046002065874ef69069752865bc9ad854c18cea76845370524d5370523786557406300260a853ef4ee54f7f75280020004100630072006f0062006100740020548c002000410064006f00620065002000520065006100640065007200200035002e003000204ee553ca66f49ad87248672c4f86958b555f5df25efa7acb76840020005000440046002065874ef63002gt13 CZE 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 DAN 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 DEU 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 ESP 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 ETI 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 FRA 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 GRE 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 HEB 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 HRV (Za stvaranje Adobe PDF dokumenata najpogodnijih za visokokvalitetni ispis prije tiskanja koristite ove postavke Stvoreni PDF dokumenti mogu se otvoriti Acrobat i Adobe Reader 50 i kasnijim verzijama)13 HUN 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 ITA 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 JPN ltFEFF9ad854c18cea306a30d730ea30d730ec30b951fa529b7528002000410064006f0062006500200050004400460020658766f8306e4f5c6210306b4f7f75283057307e305930023053306e8a2d5b9a30674f5c62103055308c305f0020005000440046002030d530a130a430eb306f3001004100630072006f0062006100740020304a30883073002000410064006f00620065002000520065006100640065007200200035002e003000204ee5964d3067958b304f30533068304c3067304d307e305930023053306e8a2d5b9a306b306f30d530a930f330c8306e57cb30818fbc307f304c5fc59808306730593002gt13 KOR ltFEFFc7740020c124c815c7440020c0acc6a9d558c5ec0020ace0d488c9c80020c2dcd5d80020c778c1c4c5d00020ac00c7a50020c801d569d55c002000410064006f0062006500200050004400460020bb38c11cb97c0020c791c131d569b2c8b2e4002e0020c774b807ac8c0020c791c131b41c00200050004400460020bb38c11cb2940020004100630072006f0062006100740020bc0f002000410064006f00620065002000520065006100640065007200200035002e00300020c774c0c1c5d0c11c0020c5f40020c2180020c788c2b5b2c8b2e4002egt13 LTH 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 LVI 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 NLD (Gebruik deze instellingen om Adobe PDF-documenten te maken die zijn geoptimaliseerd voor prepress-afdrukken van hoge kwaliteit De gemaakte PDF-documenten kunnen worden geopend met Acrobat en Adobe Reader 50 en hoger)13 NOR 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 POL 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 PTB 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 RUM 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 RUS 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 SKY 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 SLV 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 SUO 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 SVE ltFEFF0041006e007600e4006e00640020006400650020006800e4007200200069006e0073007400e4006c006c006e0069006e006700610072006e00610020006f006d002000640075002000760069006c006c00200073006b006100700061002000410064006f006200650020005000440046002d0064006f006b0075006d0065006e007400200073006f006d002000e400720020006c00e4006d0070006c0069006700610020006600f60072002000700072006500700072006500730073002d007500740073006b00720069006600740020006d006500640020006800f600670020006b00760061006c0069007400650074002e002000200053006b006100700061006400650020005000440046002d0064006f006b0075006d0065006e00740020006b0061006e002000f600700070006e00610073002000690020004100630072006f0062006100740020006f00630068002000410064006f00620065002000520065006100640065007200200035002e00300020006f00630068002000730065006e006100720065002egt13 TUR 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 UKR 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 ENU (Use these settings to create Adobe PDF documents best suited for high-quality prepress printing Created PDF documents can be opened with Acrobat and Adobe Reader 50 and later)13 gtgt13 Namespace [13 (Adobe)13 (Common)13 (10)13 ]13 OtherNamespaces [13 ltlt13 AsReaderSpreads false13 CropImagesToFrames true13 ErrorControl WarnAndContinue13 FlattenerIgnoreSpreadOverrides false13 IncludeGuidesGrids false13 IncludeNonPrinting false13 IncludeSlug false13 Namespace [13 (Adobe)13 (InDesign)13 (40)13 ]13 OmitPlacedBitmaps false13 OmitPlacedEPS false13 OmitPlacedPDF false13 SimulateOverprint Legacy13 gtgt13 ltlt13 AddBleedMarks false13 AddColorBars false13 AddCropMarks false13 AddPageInfo false13 AddRegMarks false13 ConvertColors ConvertToCMYK13 DestinationProfileName ()13 DestinationProfileSelector DocumentCMYK13 Downsample16BitImages true13 FlattenerPreset ltlt13 PresetSelector MediumResolution13 gtgt13 FormElements false13 GenerateStructure false13 IncludeBookmarks false13 IncludeHyperlinks false13 IncludeInteractive false13 IncludeLayers false13 IncludeProfiles false13 MultimediaHandling UseObjectSettings13 Namespace [13 (Adobe)13 (CreativeSuite)13 (20)13 ]13 PDFXOutputIntentProfileSelector DocumentCMYK13 PreserveEditing true13 UntaggedCMYKHandling LeaveUntagged13 UntaggedRGBHandling UseDocumentProfile13 UseDocumentBleed false13 gtgt13 ]13gtgt setdistillerparams13ltlt13 HWResolution [2400 2400]13 PageSize [612000 792000]13gtgt setpagedevice13

Page 36: July 2014 MEE Questions and Analyses - NCBE...This publication includes the questions and analyses from the July 2014 MEE. (In the actual test, the questions are simply numbered rather

Corporations Analysis

provision proposed by the investor Megarsquos board could not repeal the provision because it includes a ldquono board repealrdquo clause

The revision to the MBCA in 2009 dealing with shareholder proxy access does not change this conclusion That revision specifies that a shareholder-approved bylaw dealing with director nominations may not limit the boardrsquos power to amend add or repeal ldquoany procedure or condition to such a bylaw in order to provide for a reasonable practicable and orderly processrdquo MBCA sect 206(d) Thus according to the revision if shareholders approve a bylaw amendment that limits further board changes the board would nonetheless retain the power to ldquotinkerrdquo with the bylaw to safeguard the voting process but could not repeal the shareholder-approved bylaw The Official Comment to MBCA sect 206(d) makes clear that the revision is ldquonot intended to allow the board of directors to frustrate the purpose of the shareholder-adopted proxy access provisionrdquo Thus if Megarsquos shareholders were to approve the bylaw provision proposed by the investor Megarsquos board could only amend the provision regarding its procedures or conditions in a manner consistent with its purpose of permitting proxy access for Megarsquos shareholders

[NOTE The boardrsquos attempted interference with a shareholder voting initiative may also have been a violation of the directorsrsquo fiduciary duties See Blasius Indus Inc v Atlas Corp 564 A2d 651 (Del Ch 1988) (finding that directors breached their fiduciary duties by amending bylaws and expanding size of board to thwart insurgentrsquos plan to amend bylaws and seat a majority of new directors) The call however asks examinees to consider whether shareholders or the board have ldquoprecedencerdquo over amending the corporate bylaws Thus an examineersquos answer should be framed in terms of ldquopowerrdquo and not ldquodutyrdquo]

Point Three (40) The investor need not make a demand on the board if the investor states a direct claim such as an allegation that the board interfered with the investorrsquos right to amend the bylaws But the investor must make a demand on the board if the investor states a derivative claim (on behalf of the corporation) such as an allegation that the directors sought to entrench themselves by interfering with the proposed proxy access

The MBCA generally requires that shareholders make a demand on the board of directors before initiation of a derivative suit MBCA sect 742 (shareholder may not bring derivative proceeding until written demand has been made on corporation and 90 days have expired) A derivative suit is essentially two suits in one where the plaintiff-shareholder seeks to bring on behalf of the corporation a claim that vindicates corporate rights usually based on violation of fiduciary duties PALMITER supra sect 1811 (6th ed 2009) The demand permits the board to investigate the situation identified by the shareholder and take suitable action No demand on the board is required however if the shareholder brings a direct suit to vindicate the shareholderrsquos own rights not those of the corporation

Is the suit brought by the investor derivative or direct The MBCA defines a ldquoderivative proceedingrdquo as one brought ldquoin the right of a domestic corporationrdquo MBCA sect 740(1) Thus the answer to how the investorrsquos suit should be characterized turns on what rights the investor seeks to vindicate If the investor frames its claim as one of fiduciary breach by directorsmdashfor example for failing to become adequately informed about voting procedures or for seeking to entrench themselves in office by manipulating the voting structure to avoid a shareholder insurgencymdashthen the suit is ldquoderivativerdquo and the investor must make a demand on the board See MBCA Ch 7 Subch D Introductory Comment (ldquothe derivative suit has historically been the principal method of challenging allegedly illegal action by managementrdquo)

34

Corporations Analysis

If however the investor frames its claim as one to vindicate shareholder rights the suit is direct and no demand is required For many courts the direct-derivative question turns on who is injured and who is to receive the relief sought by the plaintiff-shareholders See Tooley v Donaldson Lufkin amp Jenrette Inc 845 A2d 1031 (Del 2004) (characterizing a merger-delay claim as direct because delay of merger only harmed shareholders not corporation) Thus if the investor claims that managementrsquos refusal to include its proposed bylaw amendment in the corporationrsquos proxy materials violates its shareholder rights to initiate corporate governance reforms the suit will be direct Courts have not questioned the ability of shareholders to bring direct suits challenging board action to exclude their proposed bylaw amendments from the corporationrsquos proxy materials See JANA Master Fund Ltd v CNET Networks Inc 954 A2d 335 (Del Ch 2008) (upholding shareholderrsquos direct challenge to boardrsquos interpretation of advance-notice bylaw) Chesapeake Corp v Shore 771 A2d 293 (Del Ch 2000) (upholding shareholderrsquos direct challenge to actions by board that effectively prevented it from proposing bylaw amendments in contest for control)

Is the way that the investor frames its claim conclusive Courts have permitted shareholder-plaintiffs to challenge a transaction in a direct suit even though the same transaction could also be challenged as a fiduciary breach See Eisenberg v Flying Tiger Line Inc 451 F2d 267 (2d Cir 1971) (permitting direct suit challenging a corporate reorganization as a dilution of shareholder voting power even though reorganization may have involved conflicts of interest and thus constituted a fiduciary breach) Thus the investorrsquos choice to pursue a claim challenging the legality of managementrsquos decision to exclude the investorrsquos proposal from the corporationrsquos proxy materialsmdashrather than a possible breach of fiduciary dutymdashis likely to be respected See 3 COX amp HAZEN supra sect 153 (describing situations in which a claim can be framed as derivative or direct)

[NOTE Some issues under Delaware corporate law regarding pre-suit demand are not relevant here For example whether the Mega directors are independent and disinterested is not relevant to the MBCA requirement of a pre-suit demand As the Official Comment to MBCA sect 742 points out the MBCArsquos requirement of ldquouniversal demandrdquo gives the board ldquothe opportunity to reexamine the act complained of in the light of a potential lawsuit and take corrective actionrdquo even when the directors might be non-independent or have conflicts of interest

Nor is it relevant to the MBCA pre-suit demand requirement that the statutory 90-day waiting period may be onerous The first paragraph of MBCA sect 742 requires a pre-suit demand without exception the second paragraph of the section imposes a 90-day waiting period before a derivative suit may be brought which can be shortened if the board rejects the demand or ldquoirreparable injury to the corporation would result by waiting for the expiration of the 90-day periodrdquo The call as written asks only whether a pre-suit demand should be made and does not ask examinees to address whether the post-demand waiting period should be shortened under the ldquoirreparable injuryrdquo standard]

35

National Conference of Bar Examiners 302 South Bedford Street | Madison WI 53703-3622 Phone 608-280-8550 | Fax 608-280-8552 | TDD 608-661-1275

wwwncbexorg e-mail contactncbexorg

  • Preface
  • Description of the MEE
  • Instructions
  • July 2014 Questions
    • CRIMINAL LAW AND PROCEDURE QUESTION
    • CONTRACTS QUESTION
    • FAMILY LAW QUESTION
    • FEDERAL CIVIL PROCEDURE QUESTION
    • EVIDENCE QUESTION
    • CORPORATIONS QUESTION
      • July 2014 Analyses
        • CRIMINAL LAW AND PROCEDURE ANALYSIS
        • CONTRACTS ANALYSIS
        • FAMILY LAW ANALYSIS
        • FEDERAL CIVIL PROCEDURE ANALYSIS
        • EVIDENCE ANALYSIS
        • CORPORATIONS ANALYSIS
            • ltlt13 ASCII85EncodePages false13 AllowTransparency false13 AutoPositionEPSFiles true13 AutoRotatePages None13 Binding Left13 CalGrayProfile (Dot Gain 20)13 CalRGBProfile (sRGB IEC61966-21)13 CalCMYKProfile (US Web Coated 050SWOP051 v2)13 sRGBProfile (sRGB IEC61966-21)13 CannotEmbedFontPolicy Error13 CompatibilityLevel 1413 CompressObjects Tags13 CompressPages true13 ConvertImagesToIndexed true13 PassThroughJPEGImages true13 CreateJobTicket false13 DefaultRenderingIntent Default13 DetectBlends true13 DetectCurves 0000013 ColorConversionStrategy CMYK13 DoThumbnails false13 EmbedAllFonts true13 EmbedOpenType false13 ParseICCProfilesInComments true13 EmbedJobOptions true13 DSCReportingLevel 013 EmitDSCWarnings false13 EndPage -113 ImageMemory 104857613 LockDistillerParams false13 MaxSubsetPct 10013 Optimize true13 OPM 113 ParseDSCComments true13 ParseDSCCommentsForDocInfo true13 PreserveCopyPage true13 PreserveDICMYKValues true13 PreserveEPSInfo true13 PreserveFlatness true13 PreserveHalftoneInfo false13 PreserveOPIComments true13 PreserveOverprintSettings true13 StartPage 113 SubsetFonts true13 TransferFunctionInfo Apply13 UCRandBGInfo Preserve13 UsePrologue false13 ColorSettingsFile ()13 AlwaysEmbed [ true13 ]13 NeverEmbed [ true13 ]13 AntiAliasColorImages false13 CropColorImages true13 ColorImageMinResolution 30013 ColorImageMinResolutionPolicy OK13 DownsampleColorImages true13 ColorImageDownsampleType Bicubic13 ColorImageResolution 30013 ColorImageDepth -113 ColorImageMinDownsampleDepth 113 ColorImageDownsampleThreshold 15000013 EncodeColorImages true13 ColorImageFilter DCTEncode13 AutoFilterColorImages true13 ColorImageAutoFilterStrategy JPEG13 ColorACSImageDict ltlt13 QFactor 01513 HSamples [1 1 1 1] VSamples [1 1 1 1]13 gtgt13 ColorImageDict ltlt13 QFactor 01513 HSamples [1 1 1 1] VSamples [1 1 1 1]13 gtgt13 JPEG2000ColorACSImageDict ltlt13 TileWidth 25613 TileHeight 25613 Quality 3013 gtgt13 JPEG2000ColorImageDict ltlt13 TileWidth 25613 TileHeight 25613 Quality 3013 gtgt13 AntiAliasGrayImages false13 CropGrayImages true13 GrayImageMinResolution 30013 GrayImageMinResolutionPolicy OK13 DownsampleGrayImages true13 GrayImageDownsampleType Bicubic13 GrayImageResolution 30013 GrayImageDepth -113 GrayImageMinDownsampleDepth 213 GrayImageDownsampleThreshold 15000013 EncodeGrayImages true13 GrayImageFilter DCTEncode13 AutoFilterGrayImages true13 GrayImageAutoFilterStrategy JPEG13 GrayACSImageDict ltlt13 QFactor 01513 HSamples [1 1 1 1] VSamples [1 1 1 1]13 gtgt13 GrayImageDict ltlt13 QFactor 01513 HSamples [1 1 1 1] VSamples [1 1 1 1]13 gtgt13 JPEG2000GrayACSImageDict ltlt13 TileWidth 25613 TileHeight 25613 Quality 3013 gtgt13 JPEG2000GrayImageDict ltlt13 TileWidth 25613 TileHeight 25613 Quality 3013 gtgt13 AntiAliasMonoImages false13 CropMonoImages true13 MonoImageMinResolution 120013 MonoImageMinResolutionPolicy OK13 DownsampleMonoImages true13 MonoImageDownsampleType Bicubic13 MonoImageResolution 120013 MonoImageDepth -113 MonoImageDownsampleThreshold 15000013 EncodeMonoImages true13 MonoImageFilter CCITTFaxEncode13 MonoImageDict ltlt13 K -113 gtgt13 AllowPSXObjects false13 CheckCompliance [13 None13 ]13 PDFX1aCheck false13 PDFX3Check false13 PDFXCompliantPDFOnly false13 PDFXNoTrimBoxError true13 PDFXTrimBoxToMediaBoxOffset [13 00000013 00000013 00000013 00000013 ]13 PDFXSetBleedBoxToMediaBox true13 PDFXBleedBoxToTrimBoxOffset [13 00000013 00000013 00000013 00000013 ]13 PDFXOutputIntentProfile ()13 PDFXOutputConditionIdentifier ()13 PDFXOutputCondition ()13 PDFXRegistryName ()13 PDFXTrapped False1313 CreateJDFFile false13 Description ltlt13 ARA 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 BGR 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 CHS ltFEFF4f7f75288fd94e9b8bbe5b9a521b5efa7684002000410064006f006200650020005000440046002065876863900275284e8e9ad88d2891cf76845370524d53705237300260a853ef4ee54f7f75280020004100630072006f0062006100740020548c002000410064006f00620065002000520065006100640065007200200035002e003000204ee553ca66f49ad87248672c676562535f00521b5efa768400200050004400460020658768633002gt13 CHT ltFEFF4f7f752890194e9b8a2d7f6e5efa7acb7684002000410064006f006200650020005000440046002065874ef69069752865bc9ad854c18cea76845370524d5370523786557406300260a853ef4ee54f7f75280020004100630072006f0062006100740020548c002000410064006f00620065002000520065006100640065007200200035002e003000204ee553ca66f49ad87248672c4f86958b555f5df25efa7acb76840020005000440046002065874ef63002gt13 CZE 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 DAN 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 DEU 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 ESP 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 ETI 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 FRA 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 GRE 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 HEB ltFEFF05D405E905EA05DE05E905D5002005D105D405D205D305E805D505EA002005D005DC05D4002005DB05D305D9002005DC05D905E605D505E8002005DE05E105DE05DB05D9002000410064006F006200650020005000440046002005D405DE05D505EA05D005DE05D905DD002005DC05D405D305E405E105EA002005E705D305DD002D05D305E405D505E1002005D005D905DB05D505EA05D905EA002E002005DE05E105DE05DB05D90020005000440046002005E905E005D505E605E805D5002005E005D905EA05E005D905DD002005DC05E405EA05D905D705D4002005D105D005DE05E605E205D505EA0020004100630072006F006200610074002005D5002D00410064006F00620065002000520065006100640065007200200035002E0030002005D505D205E805E105D005D505EA002005DE05EA05E705D305DE05D505EA002005D905D505EA05E8002E05D005DE05D905DD002005DC002D005000440046002F0058002D0033002C002005E205D905D905E005D5002005D105DE05D305E805D905DA002005DC05DE05E905EA05DE05E9002005E905DC0020004100630072006F006200610074002E002005DE05E105DE05DB05D90020005000440046002005E905E005D505E605E805D5002005E005D905EA05E005D905DD002005DC05E405EA05D905D705D4002005D105D005DE05E605E205D505EA0020004100630072006F006200610074002005D5002D00410064006F00620065002000520065006100640065007200200035002E0030002005D505D205E805E105D005D505EA002005DE05EA05E705D305DE05D505EA002005D905D505EA05E8002Egt13 HRV (Za stvaranje Adobe PDF dokumenata najpogodnijih za visokokvalitetni ispis prije tiskanja koristite ove postavke Stvoreni PDF dokumenti mogu se otvoriti Acrobat i Adobe Reader 50 i kasnijim verzijama)13 HUN 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 ITA 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 JPN ltFEFF9ad854c18cea306a30d730ea30d730ec30b951fa529b7528002000410064006f0062006500200050004400460020658766f8306e4f5c6210306b4f7f75283057307e305930023053306e8a2d5b9a30674f5c62103055308c305f0020005000440046002030d530a130a430eb306f3001004100630072006f0062006100740020304a30883073002000410064006f00620065002000520065006100640065007200200035002e003000204ee5964d3067958b304f30533068304c3067304d307e305930023053306e8a2d5b9a306b306f30d530a930f330c8306e57cb30818fbc307f304c5fc59808306730593002gt13 KOR ltFEFFc7740020c124c815c7440020c0acc6a9d558c5ec0020ace0d488c9c80020c2dcd5d80020c778c1c4c5d00020ac00c7a50020c801d569d55c002000410064006f0062006500200050004400460020bb38c11cb97c0020c791c131d569b2c8b2e4002e0020c774b807ac8c0020c791c131b41c00200050004400460020bb38c11cb2940020004100630072006f0062006100740020bc0f002000410064006f00620065002000520065006100640065007200200035002e00300020c774c0c1c5d0c11c0020c5f40020c2180020c788c2b5b2c8b2e4002egt13 LTH 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 LVI 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 NLD (Gebruik deze instellingen om Adobe PDF-documenten te maken die zijn geoptimaliseerd voor prepress-afdrukken van hoge kwaliteit De gemaakte PDF-documenten kunnen worden geopend met Acrobat en Adobe Reader 50 en hoger)13 NOR 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 POL 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 PTB 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 RUM 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 RUS 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 SKY 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 SLV 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 SUO 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 SVE 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 TUR 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 UKR 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 ENU (Use these settings to create Adobe PDF documents best suited for high-quality prepress printing Created PDF documents can be opened with Acrobat and Adobe Reader 50 and later)13 gtgt13 Namespace [13 (Adobe)13 (Common)13 (10)13 ]13 OtherNamespaces [13 ltlt13 AsReaderSpreads false13 CropImagesToFrames true13 ErrorControl WarnAndContinue13 FlattenerIgnoreSpreadOverrides false13 IncludeGuidesGrids false13 IncludeNonPrinting false13 IncludeSlug false13 Namespace [13 (Adobe)13 (InDesign)13 (40)13 ]13 OmitPlacedBitmaps false13 OmitPlacedEPS false13 OmitPlacedPDF false13 SimulateOverprint Legacy13 gtgt13 ltlt13 AddBleedMarks false13 AddColorBars false13 AddCropMarks false13 AddPageInfo false13 AddRegMarks false13 ConvertColors ConvertToCMYK13 DestinationProfileName ()13 DestinationProfileSelector DocumentCMYK13 Downsample16BitImages true13 FlattenerPreset ltlt13 PresetSelector MediumResolution13 gtgt13 FormElements false13 GenerateStructure false13 IncludeBookmarks false13 IncludeHyperlinks false13 IncludeInteractive false13 IncludeLayers false13 IncludeProfiles false13 MultimediaHandling UseObjectSettings13 Namespace [13 (Adobe)13 (CreativeSuite)13 (20)13 ]13 PDFXOutputIntentProfileSelector DocumentCMYK13 PreserveEditing true13 UntaggedCMYKHandling LeaveUntagged13 UntaggedRGBHandling UseDocumentProfile13 UseDocumentBleed false13 gtgt13 ]13gtgt setdistillerparams13ltlt13 HWResolution [2400 2400]13 PageSize [612000 792000]13gtgt setpagedevice13

Page 37: July 2014 MEE Questions and Analyses - NCBE...This publication includes the questions and analyses from the July 2014 MEE. (In the actual test, the questions are simply numbered rather

Corporations Analysis

If however the investor frames its claim as one to vindicate shareholder rights the suit is direct and no demand is required For many courts the direct-derivative question turns on who is injured and who is to receive the relief sought by the plaintiff-shareholders See Tooley v Donaldson Lufkin amp Jenrette Inc 845 A2d 1031 (Del 2004) (characterizing a merger-delay claim as direct because delay of merger only harmed shareholders not corporation) Thus if the investor claims that managementrsquos refusal to include its proposed bylaw amendment in the corporationrsquos proxy materials violates its shareholder rights to initiate corporate governance reforms the suit will be direct Courts have not questioned the ability of shareholders to bring direct suits challenging board action to exclude their proposed bylaw amendments from the corporationrsquos proxy materials See JANA Master Fund Ltd v CNET Networks Inc 954 A2d 335 (Del Ch 2008) (upholding shareholderrsquos direct challenge to boardrsquos interpretation of advance-notice bylaw) Chesapeake Corp v Shore 771 A2d 293 (Del Ch 2000) (upholding shareholderrsquos direct challenge to actions by board that effectively prevented it from proposing bylaw amendments in contest for control)

Is the way that the investor frames its claim conclusive Courts have permitted shareholder-plaintiffs to challenge a transaction in a direct suit even though the same transaction could also be challenged as a fiduciary breach See Eisenberg v Flying Tiger Line Inc 451 F2d 267 (2d Cir 1971) (permitting direct suit challenging a corporate reorganization as a dilution of shareholder voting power even though reorganization may have involved conflicts of interest and thus constituted a fiduciary breach) Thus the investorrsquos choice to pursue a claim challenging the legality of managementrsquos decision to exclude the investorrsquos proposal from the corporationrsquos proxy materialsmdashrather than a possible breach of fiduciary dutymdashis likely to be respected See 3 COX amp HAZEN supra sect 153 (describing situations in which a claim can be framed as derivative or direct)

[NOTE Some issues under Delaware corporate law regarding pre-suit demand are not relevant here For example whether the Mega directors are independent and disinterested is not relevant to the MBCA requirement of a pre-suit demand As the Official Comment to MBCA sect 742 points out the MBCArsquos requirement of ldquouniversal demandrdquo gives the board ldquothe opportunity to reexamine the act complained of in the light of a potential lawsuit and take corrective actionrdquo even when the directors might be non-independent or have conflicts of interest

Nor is it relevant to the MBCA pre-suit demand requirement that the statutory 90-day waiting period may be onerous The first paragraph of MBCA sect 742 requires a pre-suit demand without exception the second paragraph of the section imposes a 90-day waiting period before a derivative suit may be brought which can be shortened if the board rejects the demand or ldquoirreparable injury to the corporation would result by waiting for the expiration of the 90-day periodrdquo The call as written asks only whether a pre-suit demand should be made and does not ask examinees to address whether the post-demand waiting period should be shortened under the ldquoirreparable injuryrdquo standard]

35

National Conference of Bar Examiners 302 South Bedford Street | Madison WI 53703-3622 Phone 608-280-8550 | Fax 608-280-8552 | TDD 608-661-1275

wwwncbexorg e-mail contactncbexorg

  • Preface
  • Description of the MEE
  • Instructions
  • July 2014 Questions
    • CRIMINAL LAW AND PROCEDURE QUESTION
    • CONTRACTS QUESTION
    • FAMILY LAW QUESTION
    • FEDERAL CIVIL PROCEDURE QUESTION
    • EVIDENCE QUESTION
    • CORPORATIONS QUESTION
      • July 2014 Analyses
        • CRIMINAL LAW AND PROCEDURE ANALYSIS
        • CONTRACTS ANALYSIS
        • FAMILY LAW ANALYSIS
        • FEDERAL CIVIL PROCEDURE ANALYSIS
        • EVIDENCE ANALYSIS
        • CORPORATIONS ANALYSIS
            • ltlt13 ASCII85EncodePages false13 AllowTransparency false13 AutoPositionEPSFiles true13 AutoRotatePages None13 Binding Left13 CalGrayProfile (Dot Gain 20)13 CalRGBProfile (sRGB IEC61966-21)13 CalCMYKProfile (US Web Coated 050SWOP051 v2)13 sRGBProfile (sRGB IEC61966-21)13 CannotEmbedFontPolicy Error13 CompatibilityLevel 1413 CompressObjects Tags13 CompressPages true13 ConvertImagesToIndexed true13 PassThroughJPEGImages true13 CreateJobTicket false13 DefaultRenderingIntent Default13 DetectBlends true13 DetectCurves 0000013 ColorConversionStrategy CMYK13 DoThumbnails false13 EmbedAllFonts true13 EmbedOpenType false13 ParseICCProfilesInComments true13 EmbedJobOptions true13 DSCReportingLevel 013 EmitDSCWarnings false13 EndPage -113 ImageMemory 104857613 LockDistillerParams false13 MaxSubsetPct 10013 Optimize true13 OPM 113 ParseDSCComments true13 ParseDSCCommentsForDocInfo true13 PreserveCopyPage true13 PreserveDICMYKValues true13 PreserveEPSInfo true13 PreserveFlatness true13 PreserveHalftoneInfo false13 PreserveOPIComments true13 PreserveOverprintSettings true13 StartPage 113 SubsetFonts true13 TransferFunctionInfo Apply13 UCRandBGInfo Preserve13 UsePrologue false13 ColorSettingsFile ()13 AlwaysEmbed [ true13 ]13 NeverEmbed [ true13 ]13 AntiAliasColorImages false13 CropColorImages true13 ColorImageMinResolution 30013 ColorImageMinResolutionPolicy OK13 DownsampleColorImages true13 ColorImageDownsampleType Bicubic13 ColorImageResolution 30013 ColorImageDepth -113 ColorImageMinDownsampleDepth 113 ColorImageDownsampleThreshold 15000013 EncodeColorImages true13 ColorImageFilter DCTEncode13 AutoFilterColorImages true13 ColorImageAutoFilterStrategy JPEG13 ColorACSImageDict ltlt13 QFactor 01513 HSamples [1 1 1 1] VSamples [1 1 1 1]13 gtgt13 ColorImageDict ltlt13 QFactor 01513 HSamples [1 1 1 1] VSamples [1 1 1 1]13 gtgt13 JPEG2000ColorACSImageDict ltlt13 TileWidth 25613 TileHeight 25613 Quality 3013 gtgt13 JPEG2000ColorImageDict ltlt13 TileWidth 25613 TileHeight 25613 Quality 3013 gtgt13 AntiAliasGrayImages false13 CropGrayImages true13 GrayImageMinResolution 30013 GrayImageMinResolutionPolicy OK13 DownsampleGrayImages true13 GrayImageDownsampleType Bicubic13 GrayImageResolution 30013 GrayImageDepth -113 GrayImageMinDownsampleDepth 213 GrayImageDownsampleThreshold 15000013 EncodeGrayImages true13 GrayImageFilter DCTEncode13 AutoFilterGrayImages true13 GrayImageAutoFilterStrategy JPEG13 GrayACSImageDict ltlt13 QFactor 01513 HSamples [1 1 1 1] VSamples [1 1 1 1]13 gtgt13 GrayImageDict ltlt13 QFactor 01513 HSamples [1 1 1 1] VSamples [1 1 1 1]13 gtgt13 JPEG2000GrayACSImageDict ltlt13 TileWidth 25613 TileHeight 25613 Quality 3013 gtgt13 JPEG2000GrayImageDict ltlt13 TileWidth 25613 TileHeight 25613 Quality 3013 gtgt13 AntiAliasMonoImages false13 CropMonoImages true13 MonoImageMinResolution 120013 MonoImageMinResolutionPolicy OK13 DownsampleMonoImages true13 MonoImageDownsampleType Bicubic13 MonoImageResolution 120013 MonoImageDepth -113 MonoImageDownsampleThreshold 15000013 EncodeMonoImages true13 MonoImageFilter CCITTFaxEncode13 MonoImageDict ltlt13 K -113 gtgt13 AllowPSXObjects false13 CheckCompliance [13 None13 ]13 PDFX1aCheck false13 PDFX3Check false13 PDFXCompliantPDFOnly false13 PDFXNoTrimBoxError true13 PDFXTrimBoxToMediaBoxOffset [13 00000013 00000013 00000013 00000013 ]13 PDFXSetBleedBoxToMediaBox true13 PDFXBleedBoxToTrimBoxOffset [13 00000013 00000013 00000013 00000013 ]13 PDFXOutputIntentProfile ()13 PDFXOutputConditionIdentifier ()13 PDFXOutputCondition ()13 PDFXRegistryName ()13 PDFXTrapped False1313 CreateJDFFile false13 Description ltlt13 ARA 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 BGR ltFEFF04180437043f043e043b043704320430043904420435002004420435043704380020043d0430044104420440043e0439043a0438002c00200437043000200434043000200441044a0437043404300432043004420435002000410064006f00620065002000500044004600200434043e043a0443043c0435043d04420438002c0020043c0430043a04410438043c0430043b043d043e0020043f044004380433043e04340435043d04380020043704300020043204380441043e043a043e043a0430044704350441044204320435043d0020043f04350447043004420020043704300020043f044004350434043f0435044704300442043d04300020043f043e04340433043e0442043e0432043a0430002e002000200421044a04370434043004340435043d043804420435002000500044004600200434043e043a0443043c0435043d044204380020043c043e0433043004420020043404300020044104350020043e0442043204300440044f0442002004410020004100630072006f00620061007400200438002000410064006f00620065002000520065006100640065007200200035002e00300020043800200441043b0435043404320430044904380020043204350440044104380438002egt13 CHS ltFEFF4f7f75288fd94e9b8bbe5b9a521b5efa7684002000410064006f006200650020005000440046002065876863900275284e8e9ad88d2891cf76845370524d53705237300260a853ef4ee54f7f75280020004100630072006f0062006100740020548c002000410064006f00620065002000520065006100640065007200200035002e003000204ee553ca66f49ad87248672c676562535f00521b5efa768400200050004400460020658768633002gt13 CHT ltFEFF4f7f752890194e9b8a2d7f6e5efa7acb7684002000410064006f006200650020005000440046002065874ef69069752865bc9ad854c18cea76845370524d5370523786557406300260a853ef4ee54f7f75280020004100630072006f0062006100740020548c002000410064006f00620065002000520065006100640065007200200035002e003000204ee553ca66f49ad87248672c4f86958b555f5df25efa7acb76840020005000440046002065874ef63002gt13 CZE 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 DAN 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 DEU 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 ESP ltFEFF005500740069006c0069006300650020006500730074006100200063006f006e0066006900670075007200610063006900f3006e0020007000610072006100200063007200650061007200200064006f00630075006d0065006e0074006f00730020005000440046002000640065002000410064006f0062006500200061006400650063007500610064006f00730020007000610072006100200069006d0070007200650073006900f3006e0020007000720065002d0065006400690074006f007200690061006c00200064006500200061006c00740061002000630061006c0069006400610064002e002000530065002000700075006500640065006e00200061006200720069007200200064006f00630075006d0065006e0074006f00730020005000440046002000630072006500610064006f007300200063006f006e0020004100630072006f006200610074002c002000410064006f00620065002000520065006100640065007200200035002e003000200079002000760065007200730069006f006e0065007300200070006f00730074006500720069006f007200650073002egt13 ETI 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 FRA 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 GRE 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 HEB ltFEFF05D405E905EA05DE05E905D5002005D105D405D205D305E805D505EA002005D005DC05D4002005DB05D305D9002005DC05D905E605D505E8002005DE05E105DE05DB05D9002000410064006F006200650020005000440046002005D405DE05D505EA05D005DE05D905DD002005DC05D405D305E405E105EA002005E705D305DD002D05D305E405D505E1002005D005D905DB05D505EA05D905EA002E002005DE05E105DE05DB05D90020005000440046002005E905E005D505E605E805D5002005E005D905EA05E005D905DD002005DC05E405EA05D905D705D4002005D105D005DE05E605E205D505EA0020004100630072006F006200610074002005D5002D00410064006F00620065002000520065006100640065007200200035002E0030002005D505D205E805E105D005D505EA002005DE05EA05E705D305DE05D505EA002005D905D505EA05E8002E05D005DE05D905DD002005DC002D005000440046002F0058002D0033002C002005E205D905D905E005D5002005D105DE05D305E805D905DA002005DC05DE05E905EA05DE05E9002005E905DC0020004100630072006F006200610074002E002005DE05E105DE05DB05D90020005000440046002005E905E005D505E605E805D5002005E005D905EA05E005D905DD002005DC05E405EA05D905D705D4002005D105D005DE05E605E205D505EA0020004100630072006F006200610074002005D5002D00410064006F00620065002000520065006100640065007200200035002E0030002005D505D205E805E105D005D505EA002005DE05EA05E705D305DE05D505EA002005D905D505EA05E8002Egt13 HRV (Za stvaranje Adobe PDF dokumenata najpogodnijih za visokokvalitetni ispis prije tiskanja koristite ove postavke Stvoreni PDF dokumenti mogu se otvoriti Acrobat i Adobe Reader 50 i kasnijim verzijama)13 HUN 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 ITA 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 JPN ltFEFF9ad854c18cea306a30d730ea30d730ec30b951fa529b7528002000410064006f0062006500200050004400460020658766f8306e4f5c6210306b4f7f75283057307e305930023053306e8a2d5b9a30674f5c62103055308c305f0020005000440046002030d530a130a430eb306f3001004100630072006f0062006100740020304a30883073002000410064006f00620065002000520065006100640065007200200035002e003000204ee5964d3067958b304f30533068304c3067304d307e305930023053306e8a2d5b9a306b306f30d530a930f330c8306e57cb30818fbc307f304c5fc59808306730593002gt13 KOR ltFEFFc7740020c124c815c7440020c0acc6a9d558c5ec0020ace0d488c9c80020c2dcd5d80020c778c1c4c5d00020ac00c7a50020c801d569d55c002000410064006f0062006500200050004400460020bb38c11cb97c0020c791c131d569b2c8b2e4002e0020c774b807ac8c0020c791c131b41c00200050004400460020bb38c11cb2940020004100630072006f0062006100740020bc0f002000410064006f00620065002000520065006100640065007200200035002e00300020c774c0c1c5d0c11c0020c5f40020c2180020c788c2b5b2c8b2e4002egt13 LTH 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 LVI 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 NLD (Gebruik deze instellingen om Adobe PDF-documenten te maken die zijn geoptimaliseerd voor prepress-afdrukken van hoge kwaliteit De gemaakte PDF-documenten kunnen worden geopend met Acrobat en Adobe Reader 50 en hoger)13 NOR ltFEFF004200720075006b00200064006900730073006500200069006e006e007300740069006c006c0069006e00670065006e0065002000740069006c002000e50020006f0070007000720065007400740065002000410064006f006200650020005000440046002d0064006f006b0075006d0065006e00740065007200200073006f006d00200065007200200062006500730074002000650067006e0065007400200066006f00720020006600f80072007400720079006b006b0073007500740073006b00720069006600740020006100760020006800f800790020006b00760061006c0069007400650074002e0020005000440046002d0064006f006b0075006d0065006e00740065006e00650020006b0061006e002000e50070006e00650073002000690020004100630072006f00620061007400200065006c006c00650072002000410064006f00620065002000520065006100640065007200200035002e003000200065006c006c00650072002000730065006e006500720065002egt13 POL 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 PTB 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 RUM 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 RUS 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 SKY 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 SLV 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 SUO 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 SVE ltFEFF0041006e007600e4006e00640020006400650020006800e4007200200069006e0073007400e4006c006c006e0069006e006700610072006e00610020006f006d002000640075002000760069006c006c00200073006b006100700061002000410064006f006200650020005000440046002d0064006f006b0075006d0065006e007400200073006f006d002000e400720020006c00e4006d0070006c0069006700610020006600f60072002000700072006500700072006500730073002d007500740073006b00720069006600740020006d006500640020006800f600670020006b00760061006c0069007400650074002e002000200053006b006100700061006400650020005000440046002d0064006f006b0075006d0065006e00740020006b0061006e002000f600700070006e00610073002000690020004100630072006f0062006100740020006f00630068002000410064006f00620065002000520065006100640065007200200035002e00300020006f00630068002000730065006e006100720065002egt13 TUR 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 UKR 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 ENU (Use these settings to create Adobe PDF documents best suited for high-quality prepress printing Created PDF documents can be opened with Acrobat and Adobe Reader 50 and later)13 gtgt13 Namespace [13 (Adobe)13 (Common)13 (10)13 ]13 OtherNamespaces [13 ltlt13 AsReaderSpreads false13 CropImagesToFrames true13 ErrorControl WarnAndContinue13 FlattenerIgnoreSpreadOverrides false13 IncludeGuidesGrids false13 IncludeNonPrinting false13 IncludeSlug false13 Namespace [13 (Adobe)13 (InDesign)13 (40)13 ]13 OmitPlacedBitmaps false13 OmitPlacedEPS false13 OmitPlacedPDF false13 SimulateOverprint Legacy13 gtgt13 ltlt13 AddBleedMarks false13 AddColorBars false13 AddCropMarks false13 AddPageInfo false13 AddRegMarks false13 ConvertColors ConvertToCMYK13 DestinationProfileName ()13 DestinationProfileSelector DocumentCMYK13 Downsample16BitImages true13 FlattenerPreset ltlt13 PresetSelector MediumResolution13 gtgt13 FormElements false13 GenerateStructure false13 IncludeBookmarks false13 IncludeHyperlinks false13 IncludeInteractive false13 IncludeLayers false13 IncludeProfiles false13 MultimediaHandling UseObjectSettings13 Namespace [13 (Adobe)13 (CreativeSuite)13 (20)13 ]13 PDFXOutputIntentProfileSelector DocumentCMYK13 PreserveEditing true13 UntaggedCMYKHandling LeaveUntagged13 UntaggedRGBHandling UseDocumentProfile13 UseDocumentBleed false13 gtgt13 ]13gtgt setdistillerparams13ltlt13 HWResolution [2400 2400]13 PageSize [612000 792000]13gtgt setpagedevice13

Page 38: July 2014 MEE Questions and Analyses - NCBE...This publication includes the questions and analyses from the July 2014 MEE. (In the actual test, the questions are simply numbered rather

National Conference of Bar Examiners 302 South Bedford Street | Madison WI 53703-3622 Phone 608-280-8550 | Fax 608-280-8552 | TDD 608-661-1275

wwwncbexorg e-mail contactncbexorg

  • Preface
  • Description of the MEE
  • Instructions
  • July 2014 Questions
    • CRIMINAL LAW AND PROCEDURE QUESTION
    • CONTRACTS QUESTION
    • FAMILY LAW QUESTION
    • FEDERAL CIVIL PROCEDURE QUESTION
    • EVIDENCE QUESTION
    • CORPORATIONS QUESTION
      • July 2014 Analyses
        • CRIMINAL LAW AND PROCEDURE ANALYSIS
        • CONTRACTS ANALYSIS
        • FAMILY LAW ANALYSIS
        • FEDERAL CIVIL PROCEDURE ANALYSIS
        • EVIDENCE ANALYSIS
        • CORPORATIONS ANALYSIS
            • ltlt13 ASCII85EncodePages false13 AllowTransparency false13 AutoPositionEPSFiles true13 AutoRotatePages None13 Binding Left13 CalGrayProfile (Dot Gain 20)13 CalRGBProfile (sRGB IEC61966-21)13 CalCMYKProfile (US Web Coated 050SWOP051 v2)13 sRGBProfile (sRGB IEC61966-21)13 CannotEmbedFontPolicy Error13 CompatibilityLevel 1413 CompressObjects Tags13 CompressPages true13 ConvertImagesToIndexed true13 PassThroughJPEGImages true13 CreateJobTicket false13 DefaultRenderingIntent Default13 DetectBlends true13 DetectCurves 0000013 ColorConversionStrategy CMYK13 DoThumbnails false13 EmbedAllFonts true13 EmbedOpenType false13 ParseICCProfilesInComments true13 EmbedJobOptions true13 DSCReportingLevel 013 EmitDSCWarnings false13 EndPage -113 ImageMemory 104857613 LockDistillerParams false13 MaxSubsetPct 10013 Optimize true13 OPM 113 ParseDSCComments true13 ParseDSCCommentsForDocInfo true13 PreserveCopyPage true13 PreserveDICMYKValues true13 PreserveEPSInfo true13 PreserveFlatness true13 PreserveHalftoneInfo false13 PreserveOPIComments true13 PreserveOverprintSettings true13 StartPage 113 SubsetFonts true13 TransferFunctionInfo Apply13 UCRandBGInfo Preserve13 UsePrologue false13 ColorSettingsFile ()13 AlwaysEmbed [ true13 ]13 NeverEmbed [ true13 ]13 AntiAliasColorImages false13 CropColorImages true13 ColorImageMinResolution 30013 ColorImageMinResolutionPolicy OK13 DownsampleColorImages true13 ColorImageDownsampleType Bicubic13 ColorImageResolution 30013 ColorImageDepth -113 ColorImageMinDownsampleDepth 113 ColorImageDownsampleThreshold 15000013 EncodeColorImages true13 ColorImageFilter DCTEncode13 AutoFilterColorImages true13 ColorImageAutoFilterStrategy JPEG13 ColorACSImageDict ltlt13 QFactor 01513 HSamples [1 1 1 1] VSamples [1 1 1 1]13 gtgt13 ColorImageDict ltlt13 QFactor 01513 HSamples [1 1 1 1] VSamples [1 1 1 1]13 gtgt13 JPEG2000ColorACSImageDict ltlt13 TileWidth 25613 TileHeight 25613 Quality 3013 gtgt13 JPEG2000ColorImageDict ltlt13 TileWidth 25613 TileHeight 25613 Quality 3013 gtgt13 AntiAliasGrayImages false13 CropGrayImages true13 GrayImageMinResolution 30013 GrayImageMinResolutionPolicy OK13 DownsampleGrayImages true13 GrayImageDownsampleType Bicubic13 GrayImageResolution 30013 GrayImageDepth -113 GrayImageMinDownsampleDepth 213 GrayImageDownsampleThreshold 15000013 EncodeGrayImages true13 GrayImageFilter DCTEncode13 AutoFilterGrayImages true13 GrayImageAutoFilterStrategy JPEG13 GrayACSImageDict ltlt13 QFactor 01513 HSamples [1 1 1 1] VSamples [1 1 1 1]13 gtgt13 GrayImageDict ltlt13 QFactor 01513 HSamples [1 1 1 1] VSamples [1 1 1 1]13 gtgt13 JPEG2000GrayACSImageDict ltlt13 TileWidth 25613 TileHeight 25613 Quality 3013 gtgt13 JPEG2000GrayImageDict ltlt13 TileWidth 25613 TileHeight 25613 Quality 3013 gtgt13 AntiAliasMonoImages false13 CropMonoImages true13 MonoImageMinResolution 120013 MonoImageMinResolutionPolicy OK13 DownsampleMonoImages true13 MonoImageDownsampleType Bicubic13 MonoImageResolution 120013 MonoImageDepth -113 MonoImageDownsampleThreshold 15000013 EncodeMonoImages true13 MonoImageFilter CCITTFaxEncode13 MonoImageDict ltlt13 K -113 gtgt13 AllowPSXObjects false13 CheckCompliance [13 None13 ]13 PDFX1aCheck false13 PDFX3Check false13 PDFXCompliantPDFOnly false13 PDFXNoTrimBoxError true13 PDFXTrimBoxToMediaBoxOffset [13 00000013 00000013 00000013 00000013 ]13 PDFXSetBleedBoxToMediaBox true13 PDFXBleedBoxToTrimBoxOffset [13 00000013 00000013 00000013 00000013 ]13 PDFXOutputIntentProfile ()13 PDFXOutputConditionIdentifier ()13 PDFXOutputCondition ()13 PDFXRegistryName ()13 PDFXTrapped False1313 CreateJDFFile false13 Description ltlt13 ARA 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 BGR ltFEFF04180437043f043e043b043704320430043904420435002004420435043704380020043d0430044104420440043e0439043a0438002c00200437043000200434043000200441044a0437043404300432043004420435002000410064006f00620065002000500044004600200434043e043a0443043c0435043d04420438002c0020043c0430043a04410438043c0430043b043d043e0020043f044004380433043e04340435043d04380020043704300020043204380441043e043a043e043a0430044704350441044204320435043d0020043f04350447043004420020043704300020043f044004350434043f0435044704300442043d04300020043f043e04340433043e0442043e0432043a0430002e002000200421044a04370434043004340435043d043804420435002000500044004600200434043e043a0443043c0435043d044204380020043c043e0433043004420020043404300020044104350020043e0442043204300440044f0442002004410020004100630072006f00620061007400200438002000410064006f00620065002000520065006100640065007200200035002e00300020043800200441043b0435043404320430044904380020043204350440044104380438002egt13 CHS ltFEFF4f7f75288fd94e9b8bbe5b9a521b5efa7684002000410064006f006200650020005000440046002065876863900275284e8e9ad88d2891cf76845370524d53705237300260a853ef4ee54f7f75280020004100630072006f0062006100740020548c002000410064006f00620065002000520065006100640065007200200035002e003000204ee553ca66f49ad87248672c676562535f00521b5efa768400200050004400460020658768633002gt13 CHT ltFEFF4f7f752890194e9b8a2d7f6e5efa7acb7684002000410064006f006200650020005000440046002065874ef69069752865bc9ad854c18cea76845370524d5370523786557406300260a853ef4ee54f7f75280020004100630072006f0062006100740020548c002000410064006f00620065002000520065006100640065007200200035002e003000204ee553ca66f49ad87248672c4f86958b555f5df25efa7acb76840020005000440046002065874ef63002gt13 CZE 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 DAN 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 DEU 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 ESP ltFEFF005500740069006c0069006300650020006500730074006100200063006f006e0066006900670075007200610063006900f3006e0020007000610072006100200063007200650061007200200064006f00630075006d0065006e0074006f00730020005000440046002000640065002000410064006f0062006500200061006400650063007500610064006f00730020007000610072006100200069006d0070007200650073006900f3006e0020007000720065002d0065006400690074006f007200690061006c00200064006500200061006c00740061002000630061006c0069006400610064002e002000530065002000700075006500640065006e00200061006200720069007200200064006f00630075006d0065006e0074006f00730020005000440046002000630072006500610064006f007300200063006f006e0020004100630072006f006200610074002c002000410064006f00620065002000520065006100640065007200200035002e003000200079002000760065007200730069006f006e0065007300200070006f00730074006500720069006f007200650073002egt13 ETI 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 FRA 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 GRE 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 HEB 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 HRV (Za stvaranje Adobe PDF dokumenata najpogodnijih za visokokvalitetni ispis prije tiskanja koristite ove postavke Stvoreni PDF dokumenti mogu se otvoriti Acrobat i Adobe Reader 50 i kasnijim verzijama)13 HUN 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 ITA 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 JPN ltFEFF9ad854c18cea306a30d730ea30d730ec30b951fa529b7528002000410064006f0062006500200050004400460020658766f8306e4f5c6210306b4f7f75283057307e305930023053306e8a2d5b9a30674f5c62103055308c305f0020005000440046002030d530a130a430eb306f3001004100630072006f0062006100740020304a30883073002000410064006f00620065002000520065006100640065007200200035002e003000204ee5964d3067958b304f30533068304c3067304d307e305930023053306e8a2d5b9a306b306f30d530a930f330c8306e57cb30818fbc307f304c5fc59808306730593002gt13 KOR ltFEFFc7740020c124c815c7440020c0acc6a9d558c5ec0020ace0d488c9c80020c2dcd5d80020c778c1c4c5d00020ac00c7a50020c801d569d55c002000410064006f0062006500200050004400460020bb38c11cb97c0020c791c131d569b2c8b2e4002e0020c774b807ac8c0020c791c131b41c00200050004400460020bb38c11cb2940020004100630072006f0062006100740020bc0f002000410064006f00620065002000520065006100640065007200200035002e00300020c774c0c1c5d0c11c0020c5f40020c2180020c788c2b5b2c8b2e4002egt13 LTH 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 LVI 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 NLD (Gebruik deze instellingen om Adobe PDF-documenten te maken die zijn geoptimaliseerd voor prepress-afdrukken van hoge kwaliteit De gemaakte PDF-documenten kunnen worden geopend met Acrobat en Adobe Reader 50 en hoger)13 NOR ltFEFF004200720075006b00200064006900730073006500200069006e006e007300740069006c006c0069006e00670065006e0065002000740069006c002000e50020006f0070007000720065007400740065002000410064006f006200650020005000440046002d0064006f006b0075006d0065006e00740065007200200073006f006d00200065007200200062006500730074002000650067006e0065007400200066006f00720020006600f80072007400720079006b006b0073007500740073006b00720069006600740020006100760020006800f800790020006b00760061006c0069007400650074002e0020005000440046002d0064006f006b0075006d0065006e00740065006e00650020006b0061006e002000e50070006e00650073002000690020004100630072006f00620061007400200065006c006c00650072002000410064006f00620065002000520065006100640065007200200035002e003000200065006c006c00650072002000730065006e006500720065002egt13 POL 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 PTB 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 RUM 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 RUS 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 SKY ltFEFF0054006900650074006f0020006e006100730074006100760065006e0069006100200070006f0075017e0069007400650020006e00610020007600790074007600e100720061006e0069006500200064006f006b0075006d0065006e0074006f0076002000410064006f006200650020005000440046002c0020006b0074006f007200e90020007300610020006e0061006a006c0065007001610069006500200068006f0064006900610020006e00610020006b00760061006c00690074006e00fa00200074006c0061010d00200061002000700072006500700072006500730073002e00200056007900740076006f00720065006e00e900200064006f006b0075006d0065006e007400790020005000440046002000620075006400650020006d006f017e006e00e90020006f00740076006f00720069016500200076002000700072006f006700720061006d006f006300680020004100630072006f00620061007400200061002000410064006f00620065002000520065006100640065007200200035002e0030002000610020006e006f0076016100ed00630068002egt13 SLV 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 SUO 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 SVE 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 TUR ltFEFF005900fc006b00730065006b0020006b0061006c006900740065006c0069002000f6006e002000790061007a006401310072006d00610020006200610073006b013100730131006e006100200065006e0020006900790069002000750079006100620069006c006500630065006b002000410064006f006200650020005000440046002000620065006c00670065006c0065007200690020006f006c0075015f007400750072006d0061006b0020006900e70069006e00200062007500200061007900610072006c0061007201310020006b0075006c006c0061006e0131006e002e00200020004f006c0075015f0074007500720075006c0061006e0020005000440046002000620065006c00670065006c0065007200690020004100630072006f006200610074002000760065002000410064006f00620065002000520065006100640065007200200035002e003000200076006500200073006f006e0072006100730131006e00640061006b00690020007300fc007200fc006d006c00650072006c00650020006100e70131006c006100620069006c00690072002egt13 UKR 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 ENU (Use these settings to create Adobe PDF documents best suited for high-quality prepress printing Created PDF documents can be opened with Acrobat and Adobe Reader 50 and later)13 gtgt13 Namespace [13 (Adobe)13 (Common)13 (10)13 ]13 OtherNamespaces [13 ltlt13 AsReaderSpreads false13 CropImagesToFrames true13 ErrorControl WarnAndContinue13 FlattenerIgnoreSpreadOverrides false13 IncludeGuidesGrids false13 IncludeNonPrinting false13 IncludeSlug false13 Namespace [13 (Adobe)13 (InDesign)13 (40)13 ]13 OmitPlacedBitmaps false13 OmitPlacedEPS false13 OmitPlacedPDF false13 SimulateOverprint Legacy13 gtgt13 ltlt13 AddBleedMarks false13 AddColorBars false13 AddCropMarks false13 AddPageInfo false13 AddRegMarks false13 ConvertColors ConvertToCMYK13 DestinationProfileName ()13 DestinationProfileSelector DocumentCMYK13 Downsample16BitImages true13 FlattenerPreset ltlt13 PresetSelector MediumResolution13 gtgt13 FormElements false13 GenerateStructure false13 IncludeBookmarks false13 IncludeHyperlinks false13 IncludeInteractive false13 IncludeLayers false13 IncludeProfiles false13 MultimediaHandling UseObjectSettings13 Namespace [13 (Adobe)13 (CreativeSuite)13 (20)13 ]13 PDFXOutputIntentProfileSelector DocumentCMYK13 PreserveEditing true13 UntaggedCMYKHandling LeaveUntagged13 UntaggedRGBHandling UseDocumentProfile13 UseDocumentBleed false13 gtgt13 ]13gtgt setdistillerparams13ltlt13 HWResolution [2400 2400]13 PageSize [612000 792000]13gtgt setpagedevice13